You are on page 1of 763

Visit : www.EasyEngineering.

net

ww
w.E
a syE
ngi
n eer
i ng.
net

Visit : www.EasyEngineering.net
Visit : www.EasyEngineering.net

(i)

Upkar’s
COMMON ADMISSION TEST

ww
w .Ea
syE
ngi
nee
By rin
Dharmendra Mittal
g.n
Dean, International Institution of Management
Delhi
e t

UPKAR PRAKASHAN, AGRA-2

Visit : www.EasyEngineering.net
Visit : www.EasyEngineering.net

( ii )

www.upkar.in

© Author

ww
Publishers
UPKAR PRAKASHAN
(An ISO 9001 : 2000 Company)

w
2/11A, Swadeshi Bima Nagar, AGRA–282 002
Phone : 4053333, 2530966, 2531101
Fax : (0562) 4053330, 4031570
E-mail : publisher@upkar.in
Website : www.upkar.in .Ea
syE
4845, Ansari Road, Daryaganj,
New Delhi–110 002
Branch Offices
ngi
1-8-1/B, R.R. Complex (Near Sundaraiah Park,
Adjacent to Manasa Enclave Gate),
Phone : 011–23251844/66
nee
Bagh Lingampally, Hyderabad–500 044 (A.P.)
Phone : 040–66753330

rin

the publishers shall not be responsible for the same.
g.n
The publishers have taken all possible precautions in publishing this book, yet if any mistake has crept in,


 This book or any part thereof may not be reproduced in any form by Photographic, Mechanical, or any
other method, for any use, without written permission from the Publishers.

Only the courts at Agra shall have the jurisdiction for any legal dispute.
e t

ISBN 978-93-5013-198-5
Price : ` 490.00
( Rs. Four Hundred Ninety Only)
Code No. 1501

Printed at : UPKAR PRAKASHAN (Printing Unit) Bye-pass, AGRA

Visit : www.EasyEngineering.net
Visit : www.EasyEngineering.net

( iii )

ww This Book is Dedicated


w .Ea
To my Lovely &
Supportive Family.
syE ☯  ☯

ngi
nee
J.D.K. Mittal, Raj Bala, Sharvan, Vinay,
Renu, Kavita, Rakhee
Vidushi, Mittali, Sonali, Nikunj, Vansh rin
Jai Raj Mittal. g.n
e t

Visit : www.EasyEngineering.net
Visit : www.EasyEngineering.net

( iv )

ww
w.Ea
syE
ngi
nee
rin
g.n
e t

Visit : www.EasyEngineering.net
Visit : www.EasyEngineering.net

(v)

Basic Introduction

● Just in three steps you can crack CAT


● F A Q on CAT
● THIS BOOK BEFORE CAT – Why ?
● How to use PAT Technique To crack CAT ?
● Examination Tips

ww
More than 3000 questions

● w
All you want to know about CAT

.Ea
2 Mock Test on CAT patterns

Dear students,
syE
You are not just appearing for CAT the ultimate destination of tomorrow’s leaders in the field of manage-

ngi
ment. You are about to embark on a journey to cross formidable frontier. The unpredictable and seemingly
unbreakable Common Admission Test (CAT).

nee
Thousands of students are plagued by the haunting question. “How do I tackle CAT?” Which is notorious
for being unpredictable and of a very, very high standard my advice to you is ‘first relax’ CAT is just another

rin
tough exam. To surf through it all you need is to raise the bar of preparation and execute a few well planned
intelligent strategies. You will need the help of PAT to crack CAT.

g.n
PAT is the first teaching technique of it kind on CAT. It will help the students become adept and adroit at
CAT level. Further, plan early with skilled guidance to provide the right approach to meet CAT head on. Equip

be outstanding and the butterflies in your stomach will be on vocation.


Here are some tips to help you overcome CAT anxiety and gain confidence.
e
yourself with the right skills and prowess to excel in your mission of cracking CAT. And I’m sure the results will

t
FA Q ?

Q. How many hours of regular study is necessary in the preparation of CAT ?


Ans. After 2-3 hrs of coaching, 4-5 hrs of self study is necessary. Student should go for the balance study
of all portion(subjects). More time should be devoted to a weak subject. Students can take help of their teacher
in setting a proper daily schedule.
Q. What sort of Study material and Book should a student use in the preparation ?
Ans. Generally, recommended book of previously selected students and teachers are helpful. Apart from
this, coaching institutes provide some sort of study material etc. Generally good coaching institute provide a
book list also.
Q. What is the right time to start the preparing for CAT ?
Ans. When a student starts his/her graduation.

Visit : www.EasyEngineering.net
Visit : www.EasyEngineering.net

( vi )

THIS BOOK BEFORE CAT – Why ?

If you are planning to take the CAT (Common Admission Test), this book will be indispensable for a higher
score.
You are well aware that the CAT is one of the most important examinations that you will ever take. Your
entire future may well depend on your performance on the CAT. The results of this test will determine, in great
measure, whether you will be admitted to the Institute of your choice. There will be many candidates taking the
CAT and not all will score well enough to be accepted by the Institute they choose.
This book is designed to guide you in your study so that you will score high on the CAT. This claim that this
book will help you to achieve a higher ranking has both educational and psychological validity, for these rea-
sons—

ww
1. You will know what to study : A candidate will do better on a test if he or she knows what to study.

w
The questions in this book will show you what is required and therefore help you get the most benefit
from your study time.

.Ea
2. You will spot your weaknesses : Using this book, you will discover where your weaknesses lie. This

syE
self-diagnosis will provide you with guidelines for spending your time where it will do the most good.
3. Exam before Exam : You will get the ‘feel’ of the exam. It is important to get the ‘feel’ of the entire

ngi
examination. Gestalt (meaning configuration or pattern) psychology stresses that true learning results
in a grasp of the entire situation. Gestaltists tell us that we learn by ‘insight’. One of the salient

nee
principles of this kind of learning is that we succeed in ‘seeing through’ a problem as a consequence of
experiencing previous similar situations. This book contains many ‘similar situations’ as you will dis-

rin
cover when you take the actual examination.
4. You will gain confidence : While preparing for the exam you will build up confidence, and you will

g.n
retain this confidence when you enter the examination hall. This feeling of confidence will be a natural
consequence of getting the ‘feel’ of the exam.

e t
5. You will add to your knowledge : In going over the practice questions in this book, you will not if you
use this book properly be satisfied merely with the answer to a particular question. You will want to do
additional research on the other choices for the same question. In this way, you will broaden your;
background to be prepared adequately for the exam to come, since it is quite possible that a question on
the exam which you are going to take may require your knowing the meaning of one of these other
choices.

Author
Dharmendra Mittal
drdimperfection@gmail.com

Visit : www.EasyEngineering.net
Visit : www.EasyEngineering.net

( vii )

How to STUDY the whole YEAR ?

“Direction is more important than speed”


Hard work is the key to success and there is no alternative to it. You should pay more attention towards
study round the year. Do not waste time on gossiping with unreliable sources. Main Aim of your study is to
achieve success in life, therefore, one should not leave any stone unturned that comes in your way to gain more
knowledge and confidence.
You are required to study round the year with dedication and devotion. You must be serious in studies in the

ww
class as well as at home.
Make sure of the following :

w
(because Revolutions are not made, they come.)

.Ea
 Make a timetable for daily routine.
 Get up early in the morning and go for a morning walk and read the newspaper.

syE
 Must be attentive in the class.
 Listen to the teachers carefully.

ngi
 Make a note of all points discussed in your notebook.
 Put as many questions to the teacher again and again if you cannot follow the method or are not clear

nee
about the concept.
 Note down the points discussed by the teacher in the notebook, use pens of many colours, which will
indicate unclear concepts, revision required, important concept etc.

rin
 Practice at home and revise all the problems in detail and make short notes for further study.

g.n
 Even if there are still some points you could not follow, you must put questions to the teacher the next
day.
 Provide time to each subject daily.
 Give extra time to the subject in which you are weak.
e
 You may request the teacher to give extra time for you. He will surely help if you are truthful and
honest.
t
 Discuss your weak subjects with your friends who are good in that subject.
 Enjoy study in the class and do not feel nervous. Rich men have no faults.
 Must help your family members as and when you find free time. This will give you satisfaction. This
will also enhance your confidence because human being is a social animal.
 One who has worked hard round the year keeping all the suggestions given above in mind is very well
on the right track.
 Hopefully, at this stage you will find yourself more confident about the problems discussed in the class.
As such you will find yourself at a stage to face any challenge boldly posed by the so called examina-
tion.
 Read Books beside your syllabus like—Alchemist, Made in China, Think Big, One Minute Manager,
Best quotations of each day, You can win, Puzzle A.
Remember—“Books are for nothing but to inspire”.

Visit : www.EasyEngineering.net
Visit : www.EasyEngineering.net

( viii )

How to Take the CAT ?

TEST-TAKING STRATEGY
● Approach the test confidently. Take it calmly.
● Arrive well before time. No one will be admitted once the test has begun.
● Bring everything you will need including :
Your admit card or Roll number card.
At least two sharpened medium-soft pencils and an eraser.
A watch, if possible, to keep track of the time.
Listen carefully to all directions. If you have any questions, ask them now, before the test begins.

ww

● Read all directions carefully-twice if necessary.



wNote the time allotted for each section and budget your time intelligently.

.Ea
Work quickly but carefully.
Do all the questions that are easy for you first. Then go back and do the more difficult questions.


syE
Read each question carefully. Make sure that you understand the question before you answer.
Stay calm and confident throughout the test. Don’t let anything upset you.

ngi
Use all your time for each section. The supervisor will tell you when to start and when to stop.

nee
Time : A time limit is set for each section on the examination. Therefore, follow the time instructions
carefully. This book tells you how much time is available for each section of the test. You should calculate in

rin
advance about how much time you can afford to spend on each question. Your watch can help you here. Even
if you haven’t finished a section when the time limit is up, you must pass on to the next section.

g.n
Pay Close Attention : Be sure you understand what you’re doing at all times. It is natural to feel stress
when taking an examination, and it is easy to skip a word or jump to a false conclusion, which may cost you

e t
points off your score. Examiners sometimes deliberately insert plausible-appearing false answers, in order to
each the candidate who is not alert.

What To Do Before The Test ?

WHEN THE COUNTDOWN BEGINS


The road to IIMs-CAT is no easy ride. There are forms to fill out, last minute updates, time schedules,
yes the formidable entrance exam itself before you win a ticket and jump onto the bandwagon.

DOING THE TEST

1. BE A WISE TEST-TAKER – A CAT exam is standardised. For every question the points come only
if they are answered correctly regardless of their being particularly difficult. So, there is no point
plodding through a ‘hard’ question to prove your ability, it will fetch the same marks as a soft one.

Visit : www.EasyEngineering.net
Visit : www.EasyEngineering.net

( ix )

2. DO NOT ANSWER QUESTIONS IN A RIGID ORDER – Use PAT Technique.


3. THINK BEFORE ANSWERING – Paper setters love to confuse they provide answers and alter-
natives that look right but are not. If you do not think before answering, for sure you will fall into an
open trap. Beware!
4. FAMILIARITY AIDS SCORING – The best thing a student can do is to get familiar with questions
and practise papers. It makes you go through the sections faster knowing what to expect. Do CAT
Mock .
5. MEMORIZE THE DIRECTIONS FOR EACH TYPE OF QUESTIONS – Directions do not
change, once familiar with what you are expected to do, you gain time.

WHAT TO PACK
Atleast 2 HB pencils, pen, rubber.

ww

● Hall ticket/Admission ticket.


w A watch, which is accurate, has served you well or just plain lucky for you.

.Ea A Day Before Exam


“You cannot teach an old dog new tricks”

syE
You might have covered the whole syllabus by now. It is enough to have studied the whole year leaving no
stone unturned. Yes, you can go through the notes to recall your thoughts and refresh your memory. You should

ngi
enjoy with family members and watch T.V. Serials to remove mental stress. This will give you more energy and
enhance your confidence. Keep the stationery items like pen, HB pencil, rubber, sharpener, scale and wrist

nee
watch ready for the exam. Get your conveyance ready and fit in all respect. Do not keep any thing in mind that
can put a unwarranted stress and bad impression on you. You should go to the bed as early as possible and have
a sound sleep at night.

rin
On The Exam Day
g.n
e
After a very light, leisurely meal, get to the examination room ahead of time, perhaps ten minutes early.
The reason for coming early is to help you get accustomed to the room. It will help you to a better start.
Bring All Necessary Equipment : Three or four sharpened pencils, watch and eraser are needed. No t
pencils or erasers will be provided at the test centre. You may not take any books, dictionaries, notes, etc., into
the examination room. Scores of those individuals taking part in any form of cheating will be cancelled.
Get Settled : Find your seat and stay in it. The test paper will be given by a test supervisor who reads the
directions and otherwise tells you what to do. The people who walk about passing out the test papers and
assisting with the examination are test proctors. If you’re not able to see or hear properly, notify the supervisor
or a proctor. If you have any other difficulties during the examination, like, if it’s too hot or cold or dark or drafty,
let them know. You’re entitled to favourable test conditions, and if you don’t have them you won’t be able to do
your best. Don’t be a crank, but don’t be shy either. An important function of the proctor is to see to it that you
have favourable test conditions.
Relax : Don’t bring on unnecessary tenseness by worrying about the difficulty of the examination. If
necessary, wait a minute before beginning to write. If you’re still tense, take a couple of deep breaths, look over
your test equipment.

Visit : www.EasyEngineering.net
Visit : www.EasyEngineering.net

(x)

Review
Early Rising – Get up early in the morning check the headlines of T.V. channel.
Exercise – Go for a short morning walk or do yoga.
Regards – Wish everybody you meet in the way and feel more energetic than ever before and
create a joyful atmosphere around you.
Prayer – Pay your prayer to God.
Breakfast – Have a light breakfast.
Wearing – Wear comfortable clothes and shoes of your choice.
Conveyance – Ensure that your conveyance is ready and fit in all respect.
Admit Card – Collect your Admit Card.

ww
Stationery
Well wishes
– Collect your stationery items and wrist watch.
– Take the well wishes of elderly people and your parents so that you may feel

w
Punctual
Confidence
heavenly and blessed.

.Ea
– Reach at the exam centre at least 15 minutes before it starts.
– Walk confidently to the information board to know the room and seat number. Go to

syE
the exam room as and when you are allowed to go. Take your seat, organise your-
self, feel comfortable and get ready to take the exam. By that time you will observe

Filling of entries ngi


that the Invigilator is distributing the exam paper.
– Fill up all the entries as directed by the invigilator at the appropriate space.

nee D. Mittal

“The Smile of God is Victory


rin
BEST OF LUCK”
g.n
e t

Visit : www.EasyEngineering.net
Visit : www.EasyEngineering.net

( xi )

How To Crack CAT in the first attempt ?


Tips – To Crack CAT in First Attempt

An important goal of your test preparation is to help you to give the best possible account of yourself by
effectively using your knowledge to answer the examination questions.
First, get rid of any negative attitudes toward the test. Your attitude is negative if you view the test as a
device to ‘trip you up’ rather than an opportunity to show how effectively you have learned.

ww
Approach the Test with Self-Confidence : Working through this book is a difficult job, and after you’ve
done it you will probably be better prepared than 90 per cent of the people taking the CAT. Self-confidence is
one of the biggest strategic assets you can bring to the testing room.

w
Nobody likes tests, but some people actually permit themselves to get upset or angry when they see what

.Ea
they think is an unfair test. This can only hurt your score. Keep calm and move right ahead. After all, everyone
is taking the same test. Anger, resentment, and fear all slow you down and impair your judgment.

syE
Besides, every test you take, including this one, is a valuable experience that improves your skill. Since,
you will undoubtedly be taking other tests in the years to come, it may help you to regard the CAT as training
to perfect your skill.

ngi
Keep calm; there’s no point in panicing. If you’ve done your work, there’s no need for it; if you haven’t, a

nee
cool head is your very first requirement.
At the very least, this book should remove some of the fear and mystery that surrounds examinations. A
certain amount of concern is normal and good, but excessive worry saps your strength and keenness. In other
words, be prepared emotionally.
rin
Pre Test Review
g.n
If you know any others who are taking this test, you’ll probably find it helpful to review the book and your

e
notes with them. The group should be small, certainly not more than twelve. Team study at this stage should

t
seek to review the material in a different way from the way you learned it originally; strive for an exchange of
ideas. Be selective in sticking to important ideas, and stress the vague and the unfamiliar rather then that which
you all know well. End sessions as soon as you get tired.
One of the worst strategies in test-taking is to try to do all your preparation the night before the exam.
Cramming is a very good way to guarantee poor test results. Schedule your study properly so as not to suffer
from the fatigue and emotional disturbance that come from cramming the night before.
However, you would be wise to review your notes in the 48 hours preceding the exam. You shouldn’t have
to spend more than two or three hours in this way. Stick to salient points. The other will fall into place quickly.
Don’t confuse cramming with a final, calm review that helps you focus on the significant areas of this book
and further strengthens your confidence in your ability to handle the test questions. In other words, prepare
yourself factually.
Keep Fit. Mind and body work together. Poor physical condition will lower your mental efficiency. In
preparing for an examination, observe the common sense rules of health. Get sufficient sleep and rest, eat
proper foods, plan recreation and exercise. In relation to health and examinations, two cautions are in order.
Don’t miss your meals prior to an examination in order to get extra time for study. Likewise, don’t miss your

Visit : www.EasyEngineering.net
Visit : www.EasyEngineering.net

( xii )

regular sleep by sitting up late to ‘cram’ for the examination. Cramming is an attempt to learn in a very short
period of time what should have been learned through regular and consistent study. Not only are these two
habits detrimental to health, but seldom do they pay off in terms of effective learning. It is likely that you will be
more confused rather than better prepared on the day of the examination if you have broken into your daily
routine by missing your meals or sleep.
On the night before the examination, go to bed at your regular time and try to get a good
night’s sleep. Don’t go to the movies. Don’t date. In other words, prepare yourself physically.
Review
● AIM
It is the foremost requirement of a student to decide and choose his/her Aim before he/she is going to
start preparation. You must be sincere and honest enough in assessing your suitability for the Aim.
“I awoke one morning and found myself famous”

ww
● GET-SET
For success in achieving your aim, it is required to start preparation right from Graduation Ist year with

w
dedication and devotion sincerely.
PLANNING

.Ea

For a college going student it may be little difficult to go for academic as well as competitive studies
together. If you plan and make a timetable it will be an easy task for you.
● PUNCTUALITY
syE
“Be not slow to visit the sick”

ngi
It is very much important to be punctual while preparing for exam, else you may find that some part of
your syllabus is not covered and may cause lack of confidence.
● IDENTIFY YOUR WEAK TOPICS
nee
You must analyse in which AREAS you are weak and needs to put in more efforts to improve.
● HONEST APPROACH
“All or Nothing” rin
g.n
You must solve the problems honestly at your own. Bring your answer to the question first and then
compare with the given Answer Sheet. Do not depend on readymade solutions only.
● CHOOSING QUALITY MATERIAL AND GUIDANCE
“The Tree is known by its fruit”
e
You must consult good quality reading material/books. Consulting poor quality books may result in
t
danger or may create confusion in your mind. To get confidence, better go for choosing good quality
material, guidance and depend on self study only.
● SERIOUS APPROACH
“We must learn to walk before we can run”
You must take your study as a entertaining instrument. You must enjoy your study. This will create a
conducive atmosphere for study and avoid unwarranted tension.
● MOCK TESTS
You must undergo frequent MOCK TESTS within time frame to know your position about the exami-
nation standard you are appearing.
● CONSULTATION
In case you find yourself stuck in solving problems, you must consult your friends and teachers to save
time or mail me. My email:- drdimperfection@gmail.com.

Visit : www.EasyEngineering.net
Visit : www.EasyEngineering.net

( xiii )

P A T Teaching for CAT

CAT is not just a competition, it is a battle ……..where only the fittest wins. Yes, CAT is dream destination
of future managers of India. Thousand of students just ask one question, “I am studying well but how do I
tackle the Common Admission Test (CAT) of IIM’s which is so well-known to be an unpredictable exam”?
But now, panacea is in front of you in the form of PAT Technique CAT is just another exam. To do well
in it, all one needs is a high level of preparation and a few intelligently planned strategies. PAT Teaching is the
first Technique of its kind on CAT which is wholly dedicated to meeting the needs of the CAT (MBA)
aspirants. There are no shortcuts to success. Therefore, I believe in beginning early to plan, guide and provide
direction to the career path of all our students. So, that they may acquire the necessary skills and vision to excel
in the preparation of their destination.
Flow chart of PAT Technique :-

ww
PAT = PASS – AVERAGE – TOP
Aim : How to leave 35% of Question in minimum time that a student (you) can get maximum time for

w
65% of Questions.

.Ea
Examination Tips : Today many competition exam consist of objective type questions. All questions of
multiple-choice type and there are four or five choices per question. Perhaps you score four marks for every
correct answer and deduct one mark for every incorrect answer. Here we emphasis on strategy to enhance

syE
your score in these tests. Although, PAT is best.
Sound knowledge and deaf Fundamentals : In these types of exams you must have every strong

ngi
concepts because every objective question is based on an particular concepts. Last moment cramming may be
dangerous for you, so begin with good general textbooks. Concurrently you must take practice tests and use
the results to guide your study.
nee
Conscious guessing : In most situations you can resort to guessing where unsure. If you were to simply

rin
put a random answer, you would get approximately 25 percentage right and 75 percentage wrong , that is three
answer for every that is three wrong answer for every right answer. Since, a right answer is worth four marks

g.n
as a wrong answer deducts one mark each from your score, complete random guessing would make no
difference in your score. Since, the odds are that you know something about some aspect about a question, or

e
about one of the choices, talk a guess on questions of which you are completely sure, and you can eliminate one
or two choices as obviously wrong answer.
Time To Jump : Do not waste time on difficult questions. As PAT Technique also suggests. Skip them
and mark your examination book to show that you have done so, and proceed to an easier one. Once you have
finished return and try to attempt the marked questions. Do not rush. Thoroughly re-check.
t

Visit : www.EasyEngineering.net
Visit : www.EasyEngineering.net

( xiv )

INDEX

PART – I : Mathematics

Part - A : Basic Topics of MBA Mathematics

1. Basic Mathematical Operation ...................................................................................... 3–18

ww
2. H.C.F. and L.C.M. of Polynomials ................................................................................
3. Percentage ..................................................................................................................
19–28
29–48

w
4. Profit and Loss ............................................................................................................

.Ea
5. Average ......................................................................................................................
6. Height and Distance .....................................................................................................
49–72
73–78
79–94

syE
7. Ratio and Proportion ....................................................................................................
8. Mixture or Alligations ...................................................................................................
95–107
108–117

ngi
9. Time, Distance and Speed ............................................................................................ 118–136

nee
10. Time and Work ............................................................................................................ 137–157
11. Simple and Compound Interest ...................................................................................... 158–169

rin
12. Plane Geometry ...........................................................................................................
13. Mensuration ................................................................................................................
170–183
184–214

g.n
14. Linear and Quadratic Equation ...................................................................................... 215–238

e
15. Progression and Sequence ............................................................................................
● One Day Capsule of Numerical Aptiude ........................................................................ 263–287
239–262

t
Part - B : Advance Topics of MBA Mathematics

1. Numbers System ......................................................................................................... 288–297


2. Set Theory .................................................................................................................. 298–305
3. Logarithms .................................................................................................................. 306–314
4. Functions and Graphs ................................................................................................... 315–353
5. Inequalities .................................................................................................................. 354–373
6. Permutations and Combinations .................................................................................... 374–392
7. Probability ................................................................................................................... 393–416
8. Co-ordinate Geometry .................................................................................................. 417–426
9. Puzzles, Verbal Problems and Equations ........................................................................ 427–450

Visit : www.EasyEngineering.net
Visit : www.EasyEngineering.net

( xv )

PART – II : VERBAL

Part A : English Verbal

1. Structure of Sentence .................................................................................................. 453–463


2. Parts of Speech .......................................................................................................... 464–484
3. Sentence of Connectors–Linkers ................................................................................. 485–486
4. Subject Verb Agreement ............................................................................................. 487–490
5. Simple, Compound and Complex Sentences .................................................................. 491–494
6. Modals ....................................................................................................................... 495–498

ww
7. Tenses ....................................................................................................................... 499–514

w
8. Para Jumbles ..............................................................................................................

.Ea
Part B : English Reading Comprehensive
515–524

syE
1. Reading Comprehensive ..............................................................................................
2. Synonyms ..................................................................................................................
525–604
605–610

ngi
3. Antonyms .................................................................................................................. 611–617

nee
4. One Word Substitution ................................................................................................
5. How to Use Foreign Expression ..................................................................................
618–625
626–630

rin
6. Drills .......................................................................................................................... 631–635

g.n
7. Test Papers ................................................................................................................ 636–641

PART – III : LOGIC


e
8. Use of Proper Vocabulary ........................................................................................... 642–646

t
ONE DAY CAPSULE OF REASONING

Part - A
Logical Reasoning .......................................................................................................... 649–659
1. Analogy ..................................................................................................................... 649
2. Logical Diagram ......................................................................................................... 651
3. Ranking ..................................................................................................................... 651
4. Calendar .................................................................................................................... 653
5. Blood Relation Test .................................................................................................... 653

Visit : www.EasyEngineering.net
Visit : www.EasyEngineering.net

( xvi )

6. Odd One Out (Classification) ....................................................................................... 654


7. Number and Alphabetical Series ................................................................................... 655
8. Direction Sense ........................................................................................................... 657
9. Coding and Decoding ................................................................................................... 657

Part - B

Data Interpretation .......................................................................................................... 660–714

PART – IV : PRACTICE SETS


Mock CAT – 1 ................................................................................................................ 717–730

ww
Mock CAT – 2 ................................................................................................................ 731–744

w .Ea
syE
ngi
nee
rin
g.n
e t

Visit : www.EasyEngineering.net
Visit : www.EasyEngineering.net

PART–I : MATHEMATICS

ww
w.Ea
syE
ngi
nee
rin
g.n
e t

Visit : www.EasyEngineering.net
Visit : www.EasyEngineering.net

ww
w.Ea
syE
ngi
nee
rin
g.n
e t

Visit : www.EasyEngineering.net
Visit : www.EasyEngineering.net

Part A

BASIC TOPICS OF MBA MATHEMATICS

1 Basic Mathematical Operation


1. Polynomial : Algebraic expressions in which the 14. Geometrical meaning of zeros of a polynomial :
variables involved with only non-negative integral To draw the graph of a polynomial p(x), consider y = p(x)
exponents are called polynomials. and substitute values of x to find out corresponding y to
2. Terms of a polynomial and their coefficients : If get (x,u) i.e., points on the graph.

ww
P(x) = a0 xn + a 1 xn – 1 + a 2 xn – 2 + …… + a n – 1 x + a n is
polynomial in variable x, then a0xn, a1xn – 1,……, an – 1 x,
an are known as the terms of polynomial p(x) and a0 , a1 ,
Zero of a polynomial is precisely the x-co-ordinate of
the point where the graph intersects x-axis.

w
a2 ,……an are known as their coefficients.

.Ea
3. Degree of a polynomial : Highest power of the
variable in a polynomial is called the degree of poly-
(i) A linear polynomial has exactly one zero.
(ii) A quadratic polynomial has at most two zeros.
(iii) A cubic polynomial has at most three zeros.
(iv) A polynomial p(x) of degree n has at most zeros
nomial.

syE
4. Constant polynomial : A polynomial of degree
zero is called a constant polynomial.
i.e., it can cross x-axis n times.
(v) The graph of a quadratic polynomial (ax2 + bx + c)
is ∪- shaped called as parabola.
5. Zero polynomial : The constant polynomial 0 is
called zero polynomial. Degree of zero polynomial is not ngi If a > 0 in a x2 bx + c, the shape of parabola is ∪
(opening upward).
defined.
6. Monomials : Polynomials having one term are
known as monomials.
nee If a < 0 in ax2 + bx + c, the shape of parabola is ∩
(opening downward).

7. Binomials : polynomials having two terms only


are known as binomials. rin
15. Relationship between the zero and the
coefficient of a polynomial : If α and β are the zeros of a

8. Trinomials : Polynomials having three terms only


are known as trinomials. α+β = g.n
quadratic polynomial p(x) = ax2 + bx + c; a ≠ 0, then
–b
a
i.e., the sum of zeroes

9. Linear polynomials : A polynomial of degree 1 is


called linear polynomial, e.g., ax + b.
10. Quadratic polynomial : A polynomial of degree and αβ =
=

c
a
e
– (Coefficient of x)
Coefficient of x2
t
i.e., the product of zeroes
2 is called quadratic polynomial or in other words, any
polynomial of the form ax + bx2 + cx + d, where a, b and c constant term
= =
c are real numbers and a ≠ 0, is called quadratic a coefficient of x 2
polynomial. If α, β, γ are the zeroes of cubic polynomial ax3 + bx2
+ cx + d, a ≠ 0.
11. Cubic polynomial : A polynomial of degree 3 is
–b
called cubic polynomial. In other words, a polynomial of Then, α + β + γ = ,
a
the form ax3 + bx2 + cx + d, where a, b, c and d are real
c –d
numbers and a ≠ 0, is called cubic polynomial. αβ + βγ + γα = , αβγ =
a a
12.The value of polynomial p(x) at k : If p(x) is a 16. Division algorithm : From Euclid division
polynomial in x and if k is any real constant, then real Algorithm we have
value (number) obtained by replacing x by k in p(x), is Divided = Divisor × Quotient + Remainder
called the value of p(x) at k and is represented by p(k). Quotient is always be an integer.
13. Zero of a polynomial : A real number k is called Remainder is always be either equal or greater than
a zero of polynomial p(x) if p(k) = 0. zero or less than the Divisor.

CAT Complete Course | 3

Visit : www.EasyEngineering.net
Visit : www.EasyEngineering.net

Using this concept in polynomials, we can say that Illustration 2. Reduce the following rational expres-
for any given polynomial f(x) and g(x) ≠ 0; there exist sion into lowest form :
unique polynomials, q(x) and r(x) satisfying x4 – 10x2 + 9
·
f (x) = g(x) × q(x) + r (x) x3 – 4x2 + 3x
f (x) r (x) Solution :
⇒ = q (x) +
g (x) g (x) x4 – 10x2 + 9 x4 – 9x2 – x2 + 9
3 2 =
where r(x) = 0 or degree of r(x) will have smaller x – 4x + 3x x (x 2 – 4x + 3)
than g(x) this result is known as “Division Algorithm” for x2 (x2 – 9) – 1 (x 2 – 9)
=
polynomials. x (x 2 – x – 3x + 3)
17. Synthetic division : Synthetic division is a (x2 – 9)(x2 – 1)
=
method of performing polynomial long division without x[x (x – 1) – 3 (x – 1)]
having to maintain long record of the process of long (x + 3)(x – 3)(x + 1)(x – 1)
=
division. However, it only deals with division by monic x (x – 1)(x – 3)
linear polynomials, that is, binomials of the form (x – b). (x + 3)(x + 1)
Change the sign of b when divided by (x + b) i.e., – b. =
x

ww
18. Rational Expressions : If p(x) and q(x) where
q(x) ≠ 0 are two polynomials over integers, then the
p(x) whose
Illustration 3. Express
x+y
x–y
to a rational expression

w
rational expression is said to be ·
q(x)
(i) numerator is x3 + y3,

.Ea
19. Reduction of rational expression to lowest
terms using factorization : A rational expression is said
to be in its lowest if the H.C.F. of its numerator and
(ii) denominator is x2 – y2.
Solution :
denominator is 1.
syE
In other words, we can say that a rational expression
(i)
x+y
x–y
=
(x + y)(x2 – xy + y2)
(x – y)(x2 – xy + y2)
x3 + y3
is in its lowest terms if there is no common factor in the
numerator and denominator of the rational expression.
ngi x+y
=
x3 – 2x2y + 2xy2 – y3
(x + y)(x + y)
If p(x) and q(x) where q(x) ≠ 0 are two polynomials

nee
(ii) =
x–y (x – y)(x + y)
p(x)
over integers then the rational expression is said to x2 + 2xy + y2
q(x) =

rin
be in its lowest terms, if the H.C.F of p(x) and q(x) is 1. x2 – y2
3x + 5 20. Addition and subtraction of rational expres-
For example, the rational expression is in its

g.n
5x + 7 sions : We perform operations of addition subtraction and
lowest terms since the H.C.F. of (3x + 5) and (5x + 7) is 1, multiplication on rational expressions in the same manner
2(x – 1)2 (2x + 5)(x + 2)3 as we do on fractions in number system.
where as the rational expression
4(x – 1)(3x + 7)(x + 2)2
is not in its lowest terms because here the H.C.F. of
numerator and denominator is 2(x – 1)(x + 2)2 which is
not 1.
p(x)
q(x)
and
r(x)
q(x)
e
Thus, the sum of two rational expression (having
same denominators)

is given by
t
Illustration 1. Examine if the rational expression p(x) r(x) p(x) + q(x)
(x – 6)(x + 1) + =
is in lowest terms. If not express it in lowest q(x) q(x) q(x)
(x + 2)(x + 1)
terms. and sum of two rational expression (having different
denominators)
Solution :
p(x) r(x) P(x) s(x) + r(x) q(x)
Here, p(x) = (x – 6) (x + 1) and =
q(x) s(x) q(x) s (x)
q(x) = (x + 2) (x + 1) Note : Like rational numbers, sum of two rational
H.C.F. of p(x) and q(x) = x + 1 expressions is a rational expression. Now, we solve some
example to make the concept clear.
∴ H.C.F. of p(x) and q(x) is not 1.
21. Additive inverse of a rational expression : We
∴ The given rational expression is not in lowest
terms.
3 –3
know that additive inverse of is
4 4
3
, since +
4
–3
4 ( )
p(x) (x – 6)(x + 1) x – 6 3 + (– 3)
Now, = = = = 0.
q(x) (x + 2)(x + 1) x + 2 4

4 | CAT Complete Course

Visit : www.EasyEngineering.net
Visit : www.EasyEngineering.net

Similarly, the additive of the rational expression q(x)


is called the reciprocal (or multiplicative inverse) of
p(x) – p(x) p(x)
is ·
q(x) q(x) p(x)
·
Since,
p(x)
q(x) ( )
+
– p(x)
q(x)
=
p(x) + (– p(x)) 0
q(x)
=
q(x)
=0
q(x)
Note :
P(x)
and
q(x)
are reciprocal of each other. For
– p(x) p(x) q(x) p(x)
∴ is the additive inverse of · 3x + 2 2x2 – 1
q(x) q(x) example 2 is the reciprocal of ·
2x – 3 – (2x – 3) 2x – 1′ 3x + 2
Thus, the additive inverse of 2 is
x –1 x2 + 1 24. Division of rational expressions : We know that
– 2x + 3 to divide a rational number by another rational number,
or 2 ·
x +1 we multiply the first number by the reciprocal of the
p(x) – p(x) second number (if it exists). Similarly, to divide one
Note : and are additive inverses of each
q(x) q(x) rational expression by another rational expression, we
other. multiply the first expression by the reciprocal of the
Now, we define the subtraction of two rational second expression.
expressions as the sum of the first rational expression and p(x) r(x)
the additive inverse of the second rational expression, thus Note : + is a rational expression.
q(x) s(x)

ww
p(x) r(x) p(x) – r(x)  p(x) s(x) – r(x) q(x)
– = +
q(x) s(x) q(x)  s (x) 
=
q(x) s(x)
· 25. Raising Rational Numbers to a power with a
natural exponent. Taking the root : In addition to the
Note : The difference of two rational expressions is a fundamental operations of arithmetic, raising to a power

w
rational expression.

.Ea
Illustration 4. Given that
(or involution) and taking the root (or evolution) are also
introduced for rational numbers.
a2 (b – c) + b2 (c – a) + c2 (a – b) = – (a – b) (b – c) number.
Let a be a rational number and let n be a natural

a2
+
b2
syE
(c – a) simplify the following expression :
+
(a – b)(b – c) (b – c)(b – a) (c – a)(c – b)
c2
Definition : The power of a number a with a natural
exponent n (n ≥ 2) is defined as product of n factors each
of which is equal to a :
Solution : Given expression :
=
–a 2

b 2

(a – b)(c – a) (b – c)(a – b) (c – a)(b – c)
c2

ngi an = a.a…..a (n times)


Besides, by definition, a1 = a.

nee
The number a repeated as a factor is called the base
 a2 (b – c) + b2 (c – a) + c 2 (a – b)  of the power; the number n which indicates how many
= 
 (a – b)(b – c)(c – a) times the base is to be used as a factor is called the
 – (a – b)(b – c)(c – a) 
= 
 (a – b)(b – c)(c – a)  rin
exponent of the power. The result (an ) is called the
power with a natural exponent.
The second power of a number a2 = a.a s also called

= [ a2 (b – c) + b2 (c – a) + c 2 (a – b) = – (a – b)
g.n
the square of the number a (or a squared); the third power
(b – c)(c – a)…Given] a3 = a.a.a is also called the cube of the number a (or a
= 1.
22. Multiplication of rational expressions : We
know that the product of two rational numbers
a c a c a × c ac
and is × = =
cubed).

number
e
It follows from the above definition that :
(1) An even power of a negative number is a positive
: for instance, (– 5)20 > 0;
t
b d b d b × d bd (2) An odd power of a negative number is a negative
Similarly, the product of two rational expressions
2 15
p(x)
q(x)
and
r(x) p(x) r(x) p(x) × r(x)
is × =
s(x) q(x) s(x) q(x) × s(x)
number : for instance, –( ) 3
< 0;

In other words, the product of two or more rational (3) Any power of a positive number is a positive
number: an > 0 if a > 0;
expressions is the product of their numerators divided by
the product of their denominators. When we cancel all (4) The result of raising zero unity to a power with
factors common to the two terms of the new expression, any natural exponent is zero : 0 n = 0;
the resulting expression will be in its lowest terms. (5) The result of raising unity to a power with any
Note : The product of two rational expressions is natural exponent is unity : 1n =1.
rational expression. If an expression contains no parentheses, then the
p(x) operations should be performed in the following order:
23. Reciprocal of a rational expression : Let , first involution, then multiplication and division in the
q(x)
q(x) ≠ 0 be a non zero rational expression. Then exists sequence in which they are indicated, and, finally,
q(x) p(x) q(x) addition and subtraction, again in the sequence in which
another rational expression such that × =1
p(x) q(x) p(x) they are given.

CAT Complete Course | 5

Visit : www.EasyEngineering.net
Visit : www.EasyEngineering.net

For example, It has been proved that approximate values of roots


3 of positive numbers always exist for any rational number
3·5 2 – 16 : 2. (12) + 26 = 3·25 – 16 : 2. 81 + 26 α > 0.
1
= 75 – 8· + 26 = 100.  2 to within 71 ·
Illustration 5. Find √
8

√2 =√
Evolution is defined as an operation inverse to 2·72 98 98 = √
98.
involution. Solution : Note that 2 = = ;
72 49 49 7

Definition : An nth root of a number a(n ≥ 2) is a
Therefore, it suffices to find 
√98 with an accuracy to 1
number which, when takes as a factor n times (raised to
the nth power), produces the number a. and divide the obtained number by 7. Since, √ 98 ≈ 9
√ 2 ≈ 7 (with an accuracy to 71 ).
9
n (with an accuracy to 1), 
The nth root of a number a is symbolized as √
 a. the
2 2
() ( )·
n 9 9 1
notation 
√ a = b means that b n = a. The exponent (the Indeed, <2 < +
natural number n (n ≥ 2) here is the index of the root, and 7 7 7
the number a is termed the integrand. In most case the measure of accuracy of α is taken to

ww
If n = 2, the root is usually called the square root; in
this case the index 2 is usually omitted. For instance,
2
be equal to
1
10m
(m is a natural number), and the
approximate value of the root is taken to be equal to a

w
instead of √
 7 we write√ 7. The third root (n = 3) is called
the cube root.

.Ea
On the set of rational numbers the operation of
taking the root is not always performable. For instance,
decimal fraction with m digits after the decimal point.
Solving a couple of example, let us consider the rule
for extracting the square root of a number.

Let us prove this.


Suppose the contrary : 
syE
there is no rational number equal to the square root of 2.

√ 2 is a rational number :
Illustration 6. Find √

72·6115 correct to 0.01.
Solution : Perform the following operations :

 2 = qp
√ ngi (1) Partition the digits in the radicand by pairs in the
following manner : the integral part from right to left and
the fractional part from left to right : 
√
where p and q are whole numbers and q ≠ 0. The
fraction
p
will be regarded as irreducible (this can nee 72·6115;
(2) Take (correct to 1) the square root of the first pair
of digits, i.e., of number 72 and carry over the second pair

rin
q
always be achieved by applying the basic property of the of digits (61) :
fraction). 

According to the definition of the root, we have
g.n
72′·61′15 = 8……;
– 64
2
(pq) = 2 or p = 2q , i.e., p is an even number : p = 2p
2 2
1,

where p1 is an integer. Then (2p1 )2 = 2q2 or q2 = 2p2 1 , i.e., left :


q is also an even number q = 2q1, where q1 is an integer.
) 861
e
(3)Double the found root and write the result on the
t
p 2p 
√ 72′·61′15 = 8……;
Consequently, the fraction = 1 is reducible, which
q 2q1 – 64
contradicts the hypothesis. 16 ) 861
From the obtained contradiction it follows that  √ 2 is (4) Add the greatest possible digit to the right of the
not a rational number. number 16, so that the product of the obtained three- digit
number by this digit does not exceed 861. In our example
Definition : A rational number b > 0 is termed an such a digit will be 5:165.5 = 825 < 861, we obtain
n
approximate value of the nth root √ a . (a > 0) with deficit 
√ 72′·61′15 = 8·5……;
and with an accuracy to α (α is a positive rational – 64
number) if
165 861
bn < a < (b + α)n. 5 – 825
In this case the number b + α is said to be an 36
n
approximate value of the root √
 a with excess and with an (5) Double the found root, carry over the third pair of
accuracy to α. digits (15) and proceed as in (4) :

6 | CAT Complete Course

Visit : www.EasyEngineering.net
Visit : www.EasyEngineering.net


√ 72′·61′15 = 8·52……; Definition : Two irrational numbers
a = c · b1 b2 ……bn ……and a′ = c′ · b′1 b′2…b′n…
– 64
are regarded to equal if and only if c = c′, b1 = b′ 1 , b2 =
165 861
b′2,….., bn = b′n,……….and so on.
5 – 825
Of two positive numbers the number a is greater than
1702 3615 the number a′ if c > c′ or if c = c′ but b1 > b′1 or if c = c′
2 – 3404 and b1 = b′1 but b2 > b′2 , and so on.
211 (remainder) If a > a′ > 0, then we regard that – a < – a′ and vice-
versa.
The square root  √
72·6115 = 8.52 (with deficit to
Let a = c. b 1 b2 …bn …… > 0. The fractions c.b1 ,
within 0.01), i.e., 8.522 < 72.6115 < (8.52 + 0.01)2 .
c.b1 b2 , etc., are said to be decimal approximations to the
Illustration 7. Find the approximate value of √

113·5 irrational number a with deficit. The decimals c.b1 +1,
with excess to within 0.001. c.b1 (b2+1). etc., obtained by adding a unity to the last
Solution : Let us separate the radicand in pairs of retained decimal digit of number a are called decimal
approximations to the irrational number a with excess.
digits : 
√
113·5 = √ 
1′13′·50′00′00. Then proceed as in

ww
the preceding example :

√ 1′13′·50′00′00 = 10·653……;
In order to compare an irrational number with a
rational, the latter can be represented in the form of a
periodic fraction and then it is possible to compare the

w 20
206
–1
13

.Ea
1350
6 – 1236
decimal approximations to these numbers using the same
rule as in the comparison of two irrational numbers. In
this case a terminating decimal is regarded as a periodic
fraction with period zero.

2135 11400
5 – 10625syE For example, √
1.41 = 1.4100…….
 2 > 1·41, since√
 2 = 1.4142….. and

Irrational numbers can be exemplified by square and


21303 77500
3 – 63909 ngi cube roots of natural numbers which are not squares and
cubes of natural numbers, respectively.

The square root  √


13591
113·5 ≈ 10·653 (with deficit to nee
 2, √
√  3, √
 5, √
 6, √
 7, ……√
3
 2, √
3
 3, √
3
 4, √
3
 5, √
3
 6, √
Irrational numbers are obtained not only in
3
 7,…

within 0.01) : 
0.001).
√
113·5 ≈ 10·654 (with excess to within taking roots.

rin
For instance, the number π = 3.14…. used to denote the
26. Concept of an Irrational number : Let us
1 1
extract the square root of 2 with an accuracy to ‚ 2 , g.n
ratio of the circumference of a circle to its diameter is an
irrational number. The values of logarithms of positive
numbers and trigonometric functions, as a rule, are also
1
10
we shall obtain a non-terminating decimal : √
10 10
…, n and so on. Continuing this process without bound,

2 =
irrational numbers.
e
Irrational numbers are subject to the arithmetic
operations, subtraction and division being defined as the
inverse operations to addition and multiplication. The
t
1.41421… This fraction cannot be periodic, since, as √
2 basic properties of arithmetic operations with rational
is not a rational number, and we know that a periodic numbers hold for irrational numbers as well. A rigorous
fractions represents only a rational number. proof of these operations and their properties is given in
Thus, in the process of taking roots there appears higher mathematics.
non-terminating non-periodic decimals. Fractions of this 27. Real numbers. Arithmetical roots. Rect-
type define new, irrational numbers. angular Co-ordinates in the Plane : The union of the
Definition : Any non-terminating non-periodic deci- sets of rational and irrational numbers forms the set of
mal of the form real numbers. Thus, a real number means either rational
A = c·b1 b2 …bn … or irrational number. Every real number can be approxi-
mately replaced by a terminating decimal.
(c ≥ 0; b 1 , b2 ,……are digits) is called a positive
irrational number. The following arithmetic operations are established
to be performed with real numbers: addition, subtraction,
Every positive irrational number a is associated with multiplication and division. The basic properties of the
an opposite to it negative number arithmetic operation for whole numbers also hold for real
– a = – c·b1 b2 ……bn …… numbers.

CAT Complete Course | 7

Visit : www.EasyEngineering.net
Visit : www.EasyEngineering.net

The sum or difference of a rational or an irrational


number is always an irrational number. This is also true
an = {aaa….a
a
; n times if n ≥ 2‚
; if n =1
for a product and a quotient if only the rational number is n
and √ a = b if bn = a (n ≥ 2).
not equal to zero.
Alongwith powers with natural exponents, we shall
But arithmetic operations on two irrational number
also consider powers with any real exponent.
can lead to rational numbers.
n
For instance, For real numbers taking of the root √  a is always
3 3 feasible except for the case when n is even and a < 0. But
(5 + √
 2) – √ 2 = 5, √ 3 · √ 9 = 3. this operation is not always single valued.
Any real number can be represented in the form of a
For instance, √16 = 4 and √16 = – 4, since 42 = (– 4)2
sum of two addends; this can be in various ways.
= 16, we would have to write √ 16 = ± 4. To avoid the
For example, the number 27.2 can be represented as
two-valued property of the root, the notion of the
the sum of the numbers 10 and 17.2 or 20 and 7.2 or 27
principal or arithmetic root is introduced.
and 0.2 or – 3 and 30.2 etc., we shall represented a real
number in the form of a sum of two such addends, one of Definition : The arithmetic nth of a non-negative
which is the integral part of the given number and the number a is a non-negative number b for which bn = a.

ww
other its fractional part.
Definition : The integral part of a number x is the
In what follows we shall consider only the arithmetic
value of the root, i.e., √
n
 a has sense only for a ≥ 0 and

w
greatest integer not exceeding x. It is denoted by the takes on only non -negative values.
symbol [x].
For instance,
.Ea
[27·2] = 27, [0·54] = 0, [– 3] = – 3, [– 4·5] = – 5
For instance, √
square root 16.
16 = 4 is the arithmetic value of the

The arithmetic value of a root exits for every a ≥ 0.

syE
If x is a whole number (or integer) then [x] = x. If x is
a non-integral number (or no integral), then [x] < x; in this
case the number x is enclosed between two consecutive
Let us now prove its uniqueness.
n
Let √ a = b1 and √
n
 a = b2 , where a ≥ 0 and b1 ≥ 0 and
integers [x] < x < [x] + 1. Thus, for any x the inequality
[x] ≤ x < [x] +1 holds true. ngi n n
b2 ≥ 0, then b1 = b2 = a.
If b 1 ≠ b2 , say b1 < b 2 , then, by the property of
Definition : The fractional part of a number x is the
difference between the number x and its integral part. It is
denoted as {x}. Hence, {x} = x – {x}. nee n n
inequalities, b1 < b2 , which is incorrect. The obtained
contradiction implies the uniqueness of the arithmetical

rin
root, i.e., b1 = b2 .
For instance, {27·2} = 27.2 – [27.2] = 0.2, {0·54}
If x is a real number, then |x| is the modulus (or
= 0·54 – [0·54] = 0·54

{ g
absolute value) of the number x. By definition,
{– 3} = – 3 – [– 3] = 0,
{– 4·5} = – 4·5 – [– 4·5]
.ne
x if x ≥ 0‚
|x | = – x if x < 0.

= – 4·5 – [– 5] = 0·5.
Since, [x] ≤ x < [x] + 1, 0 ≤ x – [x] < 1 i.e., for any x
the inequality 0 ≤ {x} <1 holds. The fractional part of a
number is a non-negative number less than 1.
Like rational numbers, real numbers can be
represented by points on the number or co-ordinate line.
Let x1 and x 2 be real numbers and let M1 (x1 ) and
M2 (x2 ) be the corresponding points on the number line.
t
According to the definition of the fractional part of a The following formula is valid for the distance
number, {x} = x – [x]. Hence, x = [x] + {x}, i.e., any between any two points on the number line :
number can be represented as the sum of its integral and |M1 M2 | = |x2 – x1| , ……………(1)
fractional parts. where |M1 M2 | is the length of the line segment M1
For instance, M2 . This formula is proved in the same way as in the case
of rational numbers x1 and x2 .
27·2 = 27 + 0·2,
Let us pass from the straight line to the plane.
0·54 = 0 + 0.54,
Two mutually perpendicular number axes with a
– 3 = – 3 + 0, – 4·5 = – 5 + 0·5.
common origin O from a rectangular co-ordinate system
Raising to a power with a natural exponent and in the plane. The horizontal axis is called the axis of
taking the root are defined for real numbers just in the abscissa, or the x-axis the vertical axis is termed the axis
same way as for rational numbers. of ordinates, or the y-axis (Fig. 1). The plane on which a
Le a be a real number and let n be natural number. co-ordinate system is chosen is called the co-ordinate
By definition, plane.

8 | CAT Complete Course

Visit : www.EasyEngineering.net
Visit : www.EasyEngineering.net

To prove this, let us consider the right- angled


triangle M1 M2 N (see fig. 1) in which, by formula (1), the
length of the leg M1 N to |x 2 – x 1 | and the length of the leg
M2 N to |y2 – y1|. By the Pythagoras theorem,
|M1 M2 | = √
|M1 N|2 + |M 2 N|2
= √
|x2 – x1|2 + |y2 – y1|2
= √
(x2 – x1)2 + (y2 – y1)2
Formula (2) is also correct for the case when x1= x2
or y1 = y2. Then this formula yields either

Fig. 1 |M1 M2 | = √


(y2 – y1)2 = |y2 – y1|
The co-ordinate plane is divided by the x-axis and |M1 M2 | = √
(x2 – x1)2 = |x2 – x1|.
y-axis into four parts called the co-ordinate quarters, or For instance, let us find the distance between the
quarters, or quadrants. These are known as the first, points M1 (1,3) and M2(– 3, 0) by formula (2),
second, third, and fourth quadrants. Their enumeration is

ww
shown in the figure. Right angles formed by the co-
ordinate axes are called the co-ordinate angles.
|M1 M2 | = 

= √
(– 3 – 1)2 + (0 – 3)2
25 = 5.

w
Let M be an arbitrary point of the co-ordinate plane If a point M has the co-ordinate (x,y), then its distance
and let us project it on the axis of abscissa and on the axis from the x-axis is equal to |y|, the distance from the y-axis

.Ea
of ordinate, i.e., drop perpendicular from this point on to
the co-ordinate axes (see fig. 1).
to |x|, and from the point O – √
|x|, and |OM| = √ 
x2 + y2 .

x2 + y2 : |MP| = |y|, |MQ| =

syE
Definition : The co-ordinate of the projection of the
point M on the x-axis is called the abscissa of the point
M, and the co-ordinate of projection of the point M on the
28. Powers with a Natural Exponent : By the
definition of a power with a natural exponent n,
y-axis the ordinate of the point M. The abscissa and
ordinate of the point M are called the co-ordinates of the
point M. We use the following notation: M (x, y) (the ngi {
an = a
aa·a ; n times if n ≥ 2
; if n = 1
(the base a is any real number). Let us prove the follow-
abscissas is always written in the first place).
Thus, to every point M of the co-ordinate plane there nee
ing properties of raising to a power :
(1) When multiplying powers with equal bases, the

coordinates. numbers).
rin
corresponds an ordered pair of numbers (x, y ) –its exponents are added, i.e., am an = am + n (m, n-natural

g.n
Conversely, to every pair of numbers x and y there Proof :
corresponds a unique point M of the co-ordinate plane am an = (aa…a) × (aa….a) = aa……..a
with the co-ordinates (x, y). Hence, the co-ordinate x and m times × n times (m + n) times

plane.
Indeed, mark a point on the axis of abscissa with co-
an = am + n. e
y determine the position of appoint (or locate a point) in a (by the associative property of multiplication). Hence, am

(2) When dividing powers with equal bases, the


ordinate x and draw through this point a perpendicular to exponents are subtracted, i.e.,
t
this axis: then mark a point on the axis of ordinates with am
= am – n (a ≠ 0, m, n – natural number, m > n).
co-ordinate y and draw through it a perpendicular to the an
axis of ordinate. The intersection of these perpendicular 
am aa…a (m times)
just yields the required point M (Fig. 1 shows the case =
an 
when x > 0 and y > 0). aa…a (n times)
Reducing the fraction, we obtain
If a point lies on the axis of abscissa, then its ordinate 
is equal to zero. If a point lies on the axis of ordinate, am aa……a [(m – n) times] m – n
= a .
then its abscissa is zero. The converses are also true. The an 1
origin has the abscissa and ordinate equal to zero: O 1
Remark. If m < n, then am : an = n – m ; if m = n, then
(0,0). a
m n
Let there be given two points M 1 (x1 ,y1) and M2 (x 2 , a : a = 1.
y2) on a co-ordinate plane. For finding the distance am – n if m > n‚ 
a m 1 if m = n (a ≠ 0)·
Thus, n = 
between them, the following formula holds :
a  1 if m < n‚ 
|M1 M2 | = √
an – m 

(x2 – x1)2 + (y2 – y1)2.

CAT Complete Course | 9

Visit : www.EasyEngineering.net
Visit : www.EasyEngineering.net

(3) When raising a power, the exponents are multi- Illustration 10. Perform the indicated operations :
plied, i.e., (– 2) · (– 3)17 – (– 3)16
(am)n = amn (m.n–natural numbers). 97 · 15
Proof : Solution :
amam…am a (– 2) · (– 3)17 – (– 3)16
=
(– 2) · (– 3)17 – (– 3)16
(am)n = = m + m + m + … +m 97 · 15 97 ·15
n times n times
2·3 – 316 316 (2·3 – 1)
17
(by the property of multiplication of powers with = =
97 · 15 (32)7 · 3·5
equal exponents).
316 (6 – 1) 316 · 5
Hence, (am)n = amn = = 15 = 3.
314 · 3·5 3 ·5
(4) { }
an if n is even
(– a)n = – an if n is odd Illustration 11. Arrange the following numbers in
3 3 2 2
This property follows directly from the definition of
a power with a natural exponent.
( ) ( )
increasing order : –
4
, –
5
, 0·3 2 , (– 1·2) 2 .

(5) When raising a product to a power, each factor is Solution : We find


3 3 3 2
raised to this power, i.e.,
( ) (34) = – 6427 , – (25)
ww
Proof :
(ab)n = an b n (n-natural number).

4
= –

2 42
= ( ) = = 0·16,

w
(ab)n = (ab) (ab) ….(ab) = (aa.aa) (bb….b) 5 25
n times n times n times (0·3)2 = 0·09, (– 1·2)2

.Ea
(by the associative and commutative properties of
multiplication). Hence, (ab) n = an b n
(– 43)
3
= (1·2)2 = 1·44.
2 2
( )
syE
(6) When raising a fraction to a power, both the
numerator and denominator are raised to this power, i.e.,

() a n an
= n (b ≠ 0, n-natural number).
Hence, < 0·3 2 < –
5
< (– 1·2)2.

Illustration 12. Which of the given numbers is

Proof :
b b
ngi greater :
(a) 2 300 or 3200 (b) 544 or 2112;

() a n
b
a a
= · … (n times)
b b
a
b
aa…a (n times) nee (c) (0.4)4 or (0.8)3 ?
Solution : (a) 2300
3200
= (23)100 = 8 100 ,
(32)100 = 9 100 ;

rin
= =
bb…b (n times)
Hence, 3200 > 2300 ;
(according to the rule for multiplication of fraction).
Hence, ()a n an
b
= n·
b
(b) 544
2112
=
=
g.n
(2·27)4 = (2·33)4 = 2 4 ·3 12,
(3·7)12 = 312·7 12
Let us show how this property are applied when
performing computations and identical transformations.
Illustration 8. Compute 2 4 ·
153 · 21 2
35 · 3 (c)
(73) 4·312
Hence, 21 12 > 54 4 .
(0·4)4 =
=

e
(343) 4 · 3 12

4
(25) = (25) · 52 ,
3
t
153 · 21 2 (3·5)3 · (3·7) 2 3 3
= ( ) =( ) ·2
Solution : = 4 2
352 · 3 4 (5·7)2 · 3 4 (0·8)3 3
5 5
33 ·5 5 ·3 2 ·7 2 3
= ( ) · 8.
= = 3·5 = 15. 2
52 ·7 2 ·3 2
5
Illustration 9. Evaluate the expression
Hence, (08)3 > (0·4)4 .
4 3
( )
(– 1·4) 3 . 3
7
. 29. Arithmetical square root
Definition : The arithmetical square root of a non-
Solution :
negative number a is a non-negative number b whose
4 3 27 3
( )
(– 1·4) 3 3
7 ( )
= (– 1·4)3
7 square is equal to a : 
√ a = b.
25 3 For instance, 
√16 = 4, √
 0 = 0. The symbol √ is the
( )
= – 1·4 ·
7 sign of the arithmetical square root, and a is called the
14 25 3
( )
= –
10 7
· radicand. The expression √  a is read: “the arithmetical
square root of the number a” or, simply, “the square root
= – (5)3 = – 125. of a”(a ≥ 0).

10 | CAT Complete Course

Visit : www.EasyEngineering.net
Visit : www.EasyEngineering.net

From the definition of the arithmetical square root it (1)The root taken of the product of non-negative
follows that : factors is equal to the product of the root taken factors,
i.e.,
(1) The expression 
√ a has sense only for a ≥ 0.
ab = √ a · √ b
√ (a ≥ 0, b ≥ 0).
(2) For any number a ≥ 0 the inequality √
 a ≥ 0 is
Proof : We have to established that
fulfilled.
 a · √ b ≥ 0 and (√ a · √ b)2 = ab

(3) For any number a ≥ 0 the equality (√  a)2 = a is
fulfilled. Since, a ≥ 0 and b ≥ 0, by the definition of the
In order to prove that b is an arithmetical square root arithmetical root, we have (√  a)2 = a and ( √  b) 2 = b.
of a number a ≥ 0, check to see that the following two co- Further, the square of a product is equal to the product of
ordinates are fulfilled : (1) b ≥ 0; (2) b2 = a. the squares of its factor. Therefore,
(
√ a·√ b)2 = (√ a)2 . (√ b)2 = ab.
For instance, 
√25 = 5, since 52 = 25 and 5 > 0.
Theorem : Of any real number a ≥ 0 it is possible to Since, 
√ a ≥ 0 and √
 b ≥ 0, we have √ a.√ b ≥ 0
take only one arithmetical square root. The property has been proved.
Hence, it follows that
We are not going to prove that √
 a (a ≥ 0) exists (the

ww
proof of this statement is rather difficult). Let us prove
the uniqueness of the arithmetical square root.
 a.√ b = √ab (a ≥ 0, b ≥ 0)

(2) The root taken of a fraction with a non-negative
numerator and a positive denominator is equal to the root

w
Let √
 a = b1 and √  a = b2 , where a ≥ 0, b1 ‡ 0, and b2 taken of the numerator divided by the root taken of the
≥ 0 then b1 2 = b2 2 . From the obtained contradiction there denominator, i.e.,

.Ea
follows the uniqueness of the arithmetical square root,
i.e., b1=b2 .

√ a √
=
 a (a ≥ 0, b > 0).

syE
b √ b
Theorem : For any real number a
Proof : It is necessary to show that
√a2 = |a|.  √ 
2


√ ( ) 
ngi
Proof : Consider two cases : a a a
≥ 0 and   =b ·
(1) If a ≥ 0, then by the definition of the arithmetical b
√  √
( ) b 
square root, √ a = a.
nee
2
Since, a ≥ 0 and b > 0, by the definition of the
(2) If a < 0, then (– a) > 0. The number (– a) is arithmetical root, we have (√  a) 2 = a and ( √
 b)2 = b ,
positive and (– a) = a . Therefore, √
2 2 a = – a.
2
√ a , we obtain
Thus,
squaring the fraction
b

rin
{ if ≥ 0‚ √ 2 √
g.n
2
√ a a
a2 = – a if a < 0
  a =
( ) a a
 √  2= b·
Or, by the definition of the modulus of a real number,  b ( ) b

√a2 = |a| which was just stated.
For instance,  √ 2
(– 5) = |– 5| = 5
Since, 

The
√ a ≥ and √
property has been
e
 b > 0, we have √ a ≥ 0.
proved.
b

t

√ { a – b if a ≥ 0‚
(a – b) 2 = |a – b| = b – a if a < 0 Hence, it follows that
 a = a (a ≥ 0, b > 0)

The equality √ a2 = |a| is fulfilled for any values of a.
Hence, this equality is an identity on the set of real b
√ 
√ b
numbers. (3) If a >b >0, then 
√ a > √ b.
Substituting the number a = bn (n is natural number) Proof : Let us assume that √
a ≤ √  b. Then, squaring
into the identity √
a2 = |a|, we obtain the identity both members of the inequality, we obtain a ≤ b, which
b2n = |bn |.
√ contradicts the hypothesis. Hence, 
√ a > √ b .
Hence, it follows that
If b ≥ 0, we have 
√b2n = bn If a > 0, b > 0 and √
 a > √ b, then a > b.
For instance, Consider the transformation of square roots.
54 = 52 = 25, √
√ 
(– 3)6 = |(– 3)2 | = 27. 1. Removing a Factor from the Radicand : This is
the term for the transformation of the form
Let us prove the following properties of the
arithmetical square root : 
√ a2 b = a√
 b (a ≥ 0, b ≥ 0)

CAT Complete Course | 11

Visit : www.EasyEngineering.net
Visit : www.EasyEngineering.net


√ a2 b = √a2 · √ b = |a| √ b = a√
 b.
 √ yx ·

Indeed, x2
Illustration 18. Simplify the expression ·
Remark If a < 0 and b ≥ 0, then y




√ a2 b = – a√
 b, x2
Solution : The root has sense only for y > 0
y
Since, |a| = – a for a < 0.

√ y
2. Bringing a Factor in the Radicand : This is how and the root when x and y have the same sign.
x
the transformation of the form
Therefore, for the given expression, x > 0 and y > 0. We
a√
b = √ a2 b (a ≥ 0, b ≥ 0)
 √
√  √
x2 y x2 y
is called. Let us show the application of the properties have · ·= · = √ x, where x > 0 and y > 0.
y x y x
and transformation of arithmetical square roots when 30. Powers with an Integral Exponent : The
performing operations with them. concept and properties of a power with a natural exponent
Illustration 13. Remove a factor from the radicand were considered in previous section. Generalizing the
in the expression  √
4a2 b3, where a < 0 and b > 0. notion of power, we shall introduce here powers with a
Solution : We have zero or an integral negative exponent.
Definition : If a ≠ 0, then a0 = 1.

√ 4a2 b3 = √ 4 · √a2 · √b3

ww = 2|a| 
√b ·√ b = 2|a| |b| · √ b .
2

Since, a < 0 and b > 0, we get |a| = – a and |b| = b.


The expression 00 has no sense.
Definition : If a ≠ 0 and n is a natural number, then
1
a– n = n ·

w
Therefore, 

in the expression 
√
4a2 b3, = – 2ab 

√ .Ea
√ b.
Illustration 14. Remove a factor from the radicand
16a4 b6 c3, where b < 0 and c > 0.
The expression 0-n has no sense.
a

Using the notions of powers with a zero or an


integral negative exponent, the property of a power with a
Solution : We have

√ 16a4 b6 c3 = √
syE
16·√a4 ·√b6 ·√c3
= 4 |a2| |b3| |c| · 
√c
natural exponent

a m
= 
am – n if m > n‚
1 if m = n (a ≠ 0)
 1 if m < n‚
ngi a n
The number a2 is always non-negative; |a2| = a2 since an–m
b < 0 and c > 0, |b3 | = – b3 and |c| = c, therefore  √
16a4 b6 c3 can be written in the form
= – 4a b c 
2 3 √ c.
Illustration 15. Bring a factor in the radicand in the nee am
an
= am–n (a ≠ 0)

expression
√ x , where x ≥ 0 and y < 0.
y
rin
For any natural exponents m and n.
The following properties hold true for any power

g.n
with any integral exponent :
Solution : Since, y < 0,  √y2 = |y| = – y. (1) ap aq = ap + q;
Hence, y = –√ y2. (2) (ap)q = apq;

Therefore,
x = √

y –
x =–
√y2 √ x
y2
·

Illustration 16. Perform the indicated operations :


qp
(3) q = ap – q;
a
(4) (ab)p = ap bp ;
a p ap
e t

√ 343 – √252 – √  7. (5)() b
= p;
b
Solution : Note that 343 = 49.7, 252 = 36.7. 18–3· 37
Illustration19. Compute ·
2–5
Therefore, √ 343 – √252 – √
7 18–3 · 3 7 25 ·3 7
Solution : =
= 7√ 7 – 6√ 7–√  7 = 0. 2 –5 183
25 ·3 7 25 ·3 7
Illustration 17. Compare the numbers 3√  5 and 4√ 3. = =
(2·32 )3 23 ·3 6
Solution : Bringing the factors 3 and 4 in the respec- = 22 ·3 = 12.
tive radicands, we obtain Illustration 20. Find the value of the expression
1·7 – 3 : 5·1-3 × 6–3.
3√ 5 = √9·5 = √ 45, more 4√ 3 Solution :
1·7 × 6 –3
= √
16·3 =  √48.
Since, 45 < 48, by the property of comparison of
1·7 – 3 : 5·1– 3 × 6–3 = ( ) 5·1
6 –3
roots, we obtain that  √45 < √48 or 3√ 5 < 4√ 3. ()
=
3
1
= 2 –3 = ·
8

12 | CAT Complete Course

Visit : www.EasyEngineering.net
Visit : www.EasyEngineering.net

Illustration 21. Write the expression In particular,


(ab) 4 n n n n
more (a ≠ 0, b ≠ 0) 
√ an b = √
an √ b = a√
b (a ≥ 0, b ≥ 0)
(a–2 · b3 )–3
(the rule for removing a factor from the radicand).
in the form ap bq (p, q are integers)
(3) When dividing arithmetical roots with equal
Solution : indices, the radicands are divided and the index of the
(ab) 4 a4 b4 root remains unchanged, i.e.,
= –2 –3 3 –3
(a–2 · b3 ) –3 (a ) (b ) n
a =

√ ab (a ≥ 0, b > 0)
n
a4 · b4
= 6 –9 = a–2b13 · n
a ·b b

31. The Arithmetical nth Root : Let a ≥ 0 be a real The property is proved in the same way as the
number and let n ≥ 2 be a natural number. preceding property, in particular,
Definition : The arithmetical nth root of non-
 √abb

n n n–1
a
negative number a is a non-negative number b if bn = a, = n
b
n
which is written : √ a = b. For n = 2, we have the arith- n

√ ab n – 1
metical square root. The definition of the arithmetical nth = (a ≥ 0, b > 0)

ww
root implies that :

(1) The expression 


n
√ a has sense only for a ≥ 0;
b
(the rule for riding the radicand of the denominator).
(4) When raising an arithmetical root to a power with

n
w
(2) The expression 

√ a ≥ 0;
n
√ a is always non-negative, i.e.,
.Ea
a natural exponent, the radicand is raised to this power
and the index of the root remain unchanged, i.e., radicand
remain unchanged, i.e.,
n n
(
√ a) m = am

(3) The equality (
n

syE
√ a)n = a is true for any a ≥ 0.
Let us prove following basic rules for operations
(a ≥ 0, m – a natural number)
This rule follows from the rule multiplying roots
with arithmetical roots :
(1) The Fundamental Property of the Arithmetical
ngi root.
(5) When taking the root of, the indices of the roots
are multiplied and he radicand remains unchanged, i.e.,

ne√ e
Root : The value of an arithmetical root remains un-
changed if its index is multiplied by any natural number k m
n mn
and the radicand is simultaneously raised to the power k, a = √
√ a
i.e.,
n
a = √

nk
ak (a ≥ 0) rin
(a ≥ 0; m, n – natural numbers, m ≥ 2, n ≥ 2).
Indeed, according to the rule for raising a root to a

g.n
power, we have
n
Indeed, let √ a = b(b ≥ 0;). This means that bn = a.
 √ a =
√  √

m m m
n n n
a=
√ (( √
 a) n)m
Then, by the property of a power,
(bn)k = bnk = ak

Hence it follows that b = √


nk
 Thus, √ a = b = √  ak.
n nk
ak,
Hence,
m


 n 
mn
m
 √ a = √am = a (a ≥ 0).
et
which was just stated. Consequently, by the definition of the arithmetical
(2) When multiplying arithmetical roots with equal
√
m
n mn
indices, the radicands are multiplied and the index of the root, a= √
√  a, which was required to be proved.
root remains unchanged, i.e., Definition : Roots are said to be like (or similar) if
n n n their radicands are equal and their indices are also equal.
 a √ b = √
√ ab (a ≥ 0, b ≥ 0).
In the general case, the sum or the difference of two
Indeed, by the property of a power, we have distinct roots cannot be simplified. Simplifications are
n n n n possible only in the case of like roots.
(
√ a √ b)n = (√ a)n (√ b)n = ab,
For instance,
n n
Since, (√
 a)n = a and (√
 b)n = b for a ≥ 0; and b ≥ 0; (6
√ 2 + 5√ 3) – (6√
 2 + 7√ 3) = 6 √
 2 + 5√
 3 – 6√
2 –
Hence, by the definition of the arithmetical root, it
7√
 3 = – 2√ 3.
follows that
The rule for comparing arithmetical roots is based on
n n n n n n
ab = √an √ b or √
√  a √ b = √ab . n n
the following property: if a > b > 0, then a > √
√  b, and
The statement has been proved. n n
conversely, if √
 a > √ b (a > 0 and b > 0), then a > b.
CAT Complete Course | 13

Visit : www.EasyEngineering.net
Visit : www.EasyEngineering.net

It follows from the properties of inequalities for Illustration 24. Remove a factor from the radicand
3
in the expression 
√
3
instance, √
 3 > √ 2. To prove this, let us first, applying the x8y10z5 , where y > 0 and z > 0.
3 Solution : We have
fundamental property of the root, reduce √
 3 and √
 2 to 3 3
the common index 6 (the least common multiple of the √

x8y10z5 = √

x6x2y9yz3z2
3 6 6 6
indices of the given roots) : 
√ 3 = √32 = √ 9 and √ 2 = √23 3 3 3 3
= x6 √y9 √ z3 √
√ x2yz2
6
= √
 8. We then use the rule for comparing roots and = x2y3z 
√
x2yz2
3

6 6 3
obtain that √
 9 > √ 8 or √ 3 > √ 2. Since, 
3
√x6 = x 2 for any x,√
y9 = y 3 , √
z3 = z for y > 0
3 3

We shall always consider only arithmetical roots. and z > 0.


Therefore, it is necessary to check attentively that both the If we change the conditions and assume that y < 0
radicand and the result of taking the root be non-negative. 3 3 3 3
3
and z > 0, then 
√
x8y10z5 , = 
√x8 √y10 √ z5. It is know that
For instance, we do not consider the product √
 2·√– 3, 3 3 3 3
3 x8 = x2 √
√ x2; for y < 0 the root √
y 10 = √

(– y)9 (– y) =
since √
– 3 is not an arithmetical root. Multiplying √
2 3 3 3 3
(– y)3 
√– y = – y 3 √
– y ; for z > 0 the root √ z5 = z √ z2.
and
6
√72. ww
3

 3, we obtain √
2 ·√
3 =√
 · √
 = √
 =
3 6
23
6
32
6
23 ·32
The arithmetical root 
3
√y10 has sense for any y, while

w
3
In the case of an arithmetical square root the identity the arithmetical root √ z5 only for z ≥ 0. We obtain that
√a = |a| holds for any real number. Analogously, we
2

obtain that

{
if n is an even number‚ n ≥ 2;
.Ea 3

√ x8y10z5 , = – x2y3z √

3
– x2yz2 if y < 0 and z > 0.

Illustration 25. Prove that √


ab = √– a √– b if a ≤ 0
4 4 4

an = |a|
syE
n
√ a if a ≥ 0 and n is an odd number‚ n ≥ 3; and b ≤ 0.
For instance, Solution : If a ≤ 0 and b ≤ 0, then ab ≥ 0. Therefore,

ngi
4
3 4 the root √
ab has sense. Since, ab = (– a) (– b) and both
x3 = x (x ≥ 0), √
√ x4
factors are non-negative for a ≤ 0 and b ≤ 0, by the rule
6
= |x|, √
x6 = |x|;
nee
for multiplying roots, we have
In the transformations 4 4 4
ab = √
√ – a √– b
(a ≤ 0, b ≤ 0)

√√x
3

rin
3 which was required to be proved.
x2 = 
√√x2 = √ |x|,
6 4
x3 =
√ 3 =  x (x ≥ 0), √

Illustration 26. Perform the indicated operations

√x = √ x (x ≥ 0), and so on.



5


 g.n
15 3 5 3
 =

√

x3 3
3
Let us work out some examples illustrating how the 2 2 2.
obtained properties are applied.
Illustration 22. Bring a factor in the radicand in the

expression a


3
1+
1
a3
, where a > 0.
Solution : We first transform 2

Therefore, 2√
We then have :
2=
√ 2 = √24 .

√ √√2
3 e 3
4 =
3

6
√24 .
3

t
3
Solution : Since, a 
√a3 (a > 0),


3

√
√ √2√2
3


√ √
3 3 6
1 3 1 2 2 2 = 4

√ 3
a 1+ 3 = a3 1+ 3
a a
√2 = √2

3
6 18



3 10 10
a (1 + ) = 
1 3 =
= √
a +1 3
3
3
a 9 9
= √25 = √32
Illustration 23. Remove a factor from the radicand
(We have applied the fundamental property of the

√1 + a1 , where a < 0.


6
in the expression root and the rule for extracting the root of the root).
6
32. Powers with a Rational Exponent : The notion
6
Solution : We know that 
√a6 = |a| = – a, since a < 0. and properties of powers with any integral exponent were
Therefore, considered in previous section. Let us now introduce
6 6
powers with rational fractional exponent.
a6 + 1 √ √

√ 

6 6
1 a6 + 1 a6 + 1 Definition : If a > 0 and x is an arbitrary rational
1+ = = =–
a6 a 6
6 a p
a6
√ number represented by the fraction , where p is an
q
14 | CAT Complete Course

Visit : www.EasyEngineering.net
Visit : www.EasyEngineering.net

p q Illustration 28. Find the value of the expression


integer and q ≥ 2 is a natural number, then ax = a q = √
ap ; 1

x
If a = 0 and x > 0, then a = 0.
2 5
(0·04)– 1·5 · (0·125) –
4
3 – ( ) 1
121
2
·
For instance, a5 =√
 for any
a2 Solution :



4 1
3 3 4 3 1 1 1
=
√ –
a ≥ 0, b– 4 = b– 4 b– 3 or b– 4 =
b3
=
4
= 3
b3 b4

(0·04)– 1·5 · (0·125) ( )

4
3 –
1
121
2

3 4 1
for b > 0. – – –
= ( ) ·( ) –(
121)
1 1 1 2 3 2
p 25 8
Remark 1. If is an irreducible fraction, then for
q 3 4 1
– – –
= ( ) ·( ) –( )
pm p 1 1 1 2 3 2
any fraction of the form = (n-natural number), we 5 2 112 3 2
pm q
3 4 1
have = (5–2) – 2
· – (11–2) – 2 (2– 3)– 3
qn q
apn/qn = √
apn = √ap = ap/q 53 ·2 4 – 11 = (5·2)3 ·2 – 11 = 2000 – 11 = 1989
(we have made use here of the fundamental property of Illustration 29. Perform the indicated operations :
1 3 1 3
the arithmetical root).
( ) ()
ww
Remark 2. If a > 0 and x is a whole number,
p
represented by a fraction of the from , where p is an
2·4 –2 + 81– 2 ·
1
125– 3 ·
1
()
–2
+ (√
9
3)0 ·
1 –2
()
w
q 5 2
integer and q ≥ 2 is a natural number, then the equality 1 –3
1 3
( ) ()
ap/q = √
a

.Ea
ap is also true, not by the definition of a power
with a fractional exponent but by the definition of the
Solution : (1) 2·4-2 + 81– 2 ·

= 2· 2 + 
1 1  3
 ·9
9
3

arithmetical root. Indeed, if


p
q
since (ax)q = aqx = ap and ax > 0, hence
q
= x, then 
syE
√ap = ax (a > 0),
=
4
1 1
√
+ · 93
8 93
81

ax = ap/q =  √ap .
The definition of a power with a fractional exponent
q

ngi ( ) 1
= +1= ·
8 8
9

1
implies that a–p/q = p/q , where a > 0 indeed,
a
nee
(2) 125– 3 ·
1 1
5
–2
+ (√
 3)0 ·
1
(12)
–2

rin
p p q
a– q = a– q = √

a– p = · 5 2 + 1·22
3

√ 125
1

q
1

or a– p/q =
=
ap
=
1
·
=
1 2
5
· 5 + 1·22

= 5 + 4 = 9. g.n
Since, for a ≠ 0, = 1 and a0
1
q
√ap
= n (n–natural
a
1
ap/q

a– n
9
8
1
8
1
(3) : 9 = the fractional is equal to ·
8 e
Illustration 30. Simplify the expression t
number), for any rational number x, we have : a– x = x (if 1
·√

3 – 1
a 
√ x–1/2 y–2 · ( x–5/2 y–4 3 ) x–2 y– 3
a > 0).
Solution : We have
The properties of a power with an integral exponent
1
also extend to a power with any rational exponent and a x–1/2 y– 2 = If x > 0, y ≠ 0;
positive base. y√
2 x
Let us give some worked examples illustrating 5 1 1
x– 2 y–4 = = If x > 0, y ≠ 0;
applications of the above considered properties. y4 √
x5 x2y4√ x
2 1 1 2 1
Illustration 27. Compute 8 3 – 16 4 + 92 – 83 – 164 +

√ 1 1
1
1 x–2 y– 3 = = If x ≠ 0; y > 0.


92 · 6
 y |x|√
y
3
Solution : x2√
2 1 1 3 4 The given expression has sense for x > 0 and y > 0,
83 – 164 + 9 2 = 82 – √
√ 16 + √ 9 we obtain



3 4
= √
64 – √
16 + √ 9 3 3
=
1

√ x2y4 √
x× 1
= 4 – 2 + 3 = 5. y2 x
√ x
√y
6

CAT Complete Course | 15

Visit : www.EasyEngineering.net
Visit : www.EasyEngineering.net

2
(pq)


3
x2 y4√ x · 1 = 3 or p2 = 3q2
=
y √
2  x x √6
y Hence, it follows that p is divisible by 3 : p = 3p 1
3 1 where p1 is a whole number. Then (3p1 )2 = 3q2 = or q2 =
= √ 
x2y2 · 3p 2 . Hence, q is also divisible by 3 : q = 3q where q is
1 1 1
6
x
√y p 3p1
a whole number. Consequently the fraction = is
q 3q1


1 6 x4y4 1 6 4 3
= = √
xy reducible, which contradicts the hypothesis. We conclude
x y x
If x > 0 and y > 0. that 
√ 3 is an irrational number.
33. Notion of a Power with an Irrational Expo- Remark. We have used the following statements: if
nent : the square of a whole number is divisible by 3, then the
The Properties of a Power with a Real Exponent number itself is also divisible by 3. The reader is invited
Let a be a positive real number and let x be a positive to prove this statement.
irrational number. As any irrational number, x is a non- p
(2) Let log2 3 be a rational number , where p and q
terminating non-periodic decimal : x = c. b1b2 …..bn …… q
deficit c.b1,c b1 b2 …..bn ,….. and with excess c.b 1 + 1, are whole numbers and q ≠ 0. Then log2 3 = p · By the

ww
c.b (b + 1),…c.b b ….(b + 1), …..
1 2 1 2 n
Let us form two new sequences :
p
q
definition of logarithm, 2q = 3 or (2p/q)q = 3q , we obtain

w ac.b1,ac.b1b 2,……, ac.b1b2…. b n…….. 2p = 3q . But this equality is impossible since 2p is an even
number and 3q is an odd number. Hence, our supposition

.Ea
a c.b + 1 1, a 1(b2+1), ……….ac.b1b2….(b n+1)…..
c.b
is false and log2 3 is an irrational number.
This number is denoted by a x and is called a power
of the number a>0 with an irrational exponent x. Illustration 33. Simplify the expression 3
√ 23
syE
For instance, the power 2√2 is a number for which the
sequences 21.4, 21.41 , 21.414, 21.4142……and 21.5,2 1.42,
21.415, 21.4143,……..are sequences of decimal approxima- –2
√ 32 + √ 6 + √150.
tions with deficit and with excess, respectively (√
1·4142…).
2 =
ng√i √Solution :
2 3
By definition,
1
a– x = x (a > 0),
a
where x is a positive irrational number.
nee√ √
3
3
–2
2
+√
 6 + √
150.

2 3
22 · + √
 6 + √

rin
= 32 · – 25·6
3 2
Theorem : For any real x and y and admissible a
and b the following equalities are valid : = √ 6 – √
 6 + √ 6 + 5√ 6.
(1) ax · ay = ax + y;
(2) (ax)y = axy; g.n = 6√ 6.

e
Illustration 34. For what values of x are the
ax
(3) y = ax – y;

(5)
a
(4) (ab)x = ax· bx;

()a x ax
b
= x·
b
following equalities true :
(a) x + |x| = 0;
x
(c) = – 1;
|x|
(b) x + |x| = 2x;

(d) x
√ 3 = – √ 3x2;
t
34. Worked problems (e) 3x
√ 2 = √
18x2 ?
Illustration 31. Prove that 165 + 215 is divisible by Solution : (a) x + |x| = 0; or |x| = – x. The equality is
33. true for any x ≤ 0.
Solution : 165 + 2 15 = (24 )5 + 215 = 220 + 215 = 215
(2 + 1) = 2 15·33.
5 (b) x + |x| = 2x or |x| = x. The equality is true for any
x ≥ 0.
Illustration 32. Prove that the numbers  √ 3 and log 3
x
are irrational. (c) = – 1 or |x| = – x where x ≠ 0. The equality is
|x|
Solution : (1) Suppose that √  3 is a rational number : true for x < 0.
p
3= q,

(d) x √
3 = – √ 3x2 or x√ 3 = – |x|√ 3, x = – |x|. The
where p and q are integers and q ≠ 0. The fraction
p equality is true for x ≤ 0.
q
will be regarded as irreducible. According to the (e) 3x√2 = √ 
18x2 or 3x√
 2 = 3 |x|  2, x = – |x|. The

definition of the root, we have equality is true for x ≥ 0.

16 | CAT Complete Course

Visit : www.EasyEngineering.net
Visit : www.EasyEngineering.net

is reliable, since it is obtained by adding up two correct


Illustration 35. Simplify the expression  √
x2 – 2x + 1
inequality √18 > √16 and √28 > √27.
–√
x2 + 2x + 1.
Solution : Note that x 2 – 2x + 1 = (x – 1)2 , x2 + 2x + 1 Illustration 38. Arrange the following numbers in
3 4 5
= (x +1)2 . Therefore, increasing order: √
 3, √
 4 and √ 5.


x2 – 2x + 1 – √

x2 + 2x + 1. = √

(x – 1)2 – √

(x + 1)2 4 4
Solution : Note that 
√ 4 = √22 = √ 2. Compare first
= |x – 1| – |x + 1|.
3
By the definition of modulus,  3 and √
√  2. Reduce these two numbers to a common
|x – 1| = {x – 1 if (x – 1) ≥ 0‚
– (x – 1) if (x – 1) < 0} index 6 : √
3
 3 =√
32 = √
6
 9 and √ 2 =
6 6
23 = √

6 3
 8, hence √
3
>√ 2.
|x – 1| = { – (x – 1) if x < 1;}
x – 1 if x ≥ 1‚
Or 5 10 10
Compare now  2 and √
√  5. : √ 2 = √
25 = √
32 and
|x + 1| = { – (x + 1) if (x + 1) < 0. }
x + 1 if (x + 1) ≥ 0‚ 5 10
 5. = √
√ 52
|x + 1| = { – (x + 1) if x < – 1. }
x + 1 if x ≥ – 1‚

ww
10 5
Or = 25. Consequently, √
√  2 > √ 5 .
The points x = – 1 and x =1 divides the number axis 5 4 3
Thus, √
 5. < √ 4 < √ 3.
into three intervals : (– ∞, – 1), (– 1, 1), (1, + ∞).

w
If x ≤ – 1, then |x – 1| – |x + 1|

.Ea
= – (x – 1) – (– (x + 1))
= – x + 1 + x + 1 = 2.
Illustration 39. Evaluate the expression

( a5/12 a–3/8
a7/24 )

4
3
For a = 125.

syE
If – 1 < x < 1, then |x – 1| – |x + 1|
= – (x – 1) – (x + 1)
Solution : We first simplify the given expression,
applying the property of powers with a fractional

ng(i )
exponent :
= – 2x.
4
If x ≥ 1, then |x – 1| – |x + 1| a5/12 a–3/8

3
= (a5/12 a–3/8 a7/24)– 4/3

Thus,


– 2x + 1 – √
= (x – 1) – (x + 1) = – 2.


nee a7/24

(a1/4)– 4/3 = a1/3 


√a
3

x2

= 
x2
+ 2x + 1
– 2 if x ≤ – 1‚ 

For a = 125, we obtain √

125 = 5.
rin 3

– 2x if – 1
– 2 if x ≥ 1.< x < 1‚

Illustration 36. For what values of x are the follow-
Trigonometry
g.n
ing inequalities true :
(a) |– x| ≤ x;
(c) √
x2 ≤ – x;
(b) x |x| ≥ x2;
(d) x
√ 2 > √2x2 ?
Important Point and Formula
1.
AM = x, perpendicular = PM = y and Hypotenuse =
AP = r.
e
In the right triangle AMP right-angled at M, Base =
t
Solution : (a) |– x| = |x|. The inequality |x| ≤ x is
correct for x ≥ 0.
(b) The inequality is not fulfilled for x < 0. For x ≥ 0,
|x| = x, and the slack inequality x2 ≥ x2 is correct.
(c) √
x2 = |x|; |x| ≤ – x for x ≤ 0.
We define the following six trigonometric ratios :
(d) 
√2x2 = |x| √ 2; x √ 2 > |x| √ 2 or x > |x| is incorrect perpendicular y
for any x. (i) sin θ = = , and is written as
hypotenuse r
Illustration 37. Prove that the inequality 3 √
 2 + 2√ 7 sin θ.
> 3
√ 3 + 4 is valid. base x
(ii) Cosine θ = = , as written as cos θ.
hypotenuse r
Solution : Let us bring the factors in the radicands
perpendicular y
and rewrite the given inequality as follows; √ 32 ·2 + √
22 ·7 (iii) Tangent θ = = , and is written as
base x
>√
32 ·3 + √16 or 
√18 + √28 > √27 + √16. This equality tan θ.

CAT Complete Course | 17

Visit : www.EasyEngineering.net
Visit : www.EasyEngineering.net

hypotenuse r 1
(iv) Cosecant θ = = , and is written (x) sec θ + tan θ =
perpendicular y sec θ – tan θ
as cosec θ. 1
hypotenuse r (xi) cosec θ + cot θ =
(v) Secant θ = = , and is written as cosec θ – cot θ
base x 3. Complementary angle are the angles whose sum is
sec θ. 900 .
base x Trigonometric ratios of complementary angles :
(vi) Cotangent θ = = , and is written
perpendicular y (i) sin (900 – θ) = cos θ
as cot θ.
(ii) cos (900 – θ) = sin θ
Note : It should be noted that sin θ is an abbreviation
for “since of angle θ”, it is not the product of sin and (iii) tan (90 0 – θ) = cot θ
θ. Similar is the case for other trigonometric ratios. (iv) cot (900 – θ) = tan θ
2. Some useful trigonometric identities (v) sec (900 – θ) = cosec θ
(i) sin 2 θ + cos2 θ = 1 (vi) cosec (900 – θ) = sec θ
(ii) cos2 θ = 1 – sin2θ Trigonometric Ratios of Particular Angles :

ww
(iii)
(iv)
sin 2 θ = 1 – cos2 θ
1 + tan2 θ = sec2 θ
T-ratio/θ
sin θ 0

1
30° 45°
1
60°

3
√ 1
90°

(v)

w
(vi)
sec2 θ – tan2 θ = 1

(vii) 1+ cot2 θ = cosec2 θ


.Ea
sec2 θ – 1 = tan2 θ cos θ

tan θ
1
2

3

2
2

1
2

1
2
2
0

syE
(viii) cosec2 θ – cot2 θ = 1 0 1 1 3
√ Not defined
(ix) cosec2 θ – 1 = cot2 θ √ 3

ngi ●●●

nee
rin
g.n
e t

18 | CAT Complete Course

Visit : www.EasyEngineering.net
Visit : www.EasyEngineering.net

2 H.C.F. and L.C.M. of Polynomials


We have studied greatest common divisor and least Illustration 4.
common multiple of numbers in our lower classes. Here, Find the G.C.D. of the polynomials 2x 2 – x – 1 and
we shall discuss highest common factor (H.C.F. or 4x2 + 8x + 3.
Greatest Common Divisor—G.C.D.) and least common
Solution :
multiple (L.C.M.) or polynomials with integral coeffi-
cients. Here, 2x2 – x – 1 = 2x2 – 2x + x – 1

Highest Common Factor = 2x (x – 1) + 1 (x – 1)

wwA common factor of two or more polynomials is a


polynomial which divides each of them exactly. For
example, x is a common factor of x 2y and xz; xy or x 2 y
4x 2
= (2x + 1) (x – 1)
+ 8x + 3 = 4x 2 + 2x + 6x + 3
= 2x (2x + 1) + 3 (2x + 1)

w
or xy2 or x2y2 or x2y3 etc., x2y3 are common factors of x3y3

.Ea
and x2y5. Similarly, x – 1 is a common factor of x2 – 1 and
x 3 – 1.
Illustration 1.
= (2x + 1)(2x + 3)
Only common factor is (2x + 1).
G.C.D. = 2x + 1.
Find the H.C.F of
Solution :
a2 b4 c3
syEand a3b2 c5. Illustration 5.
Find the H.C.F. of the polynomials P(x) and Q(x)
H.C.F of
H.C.F. of
a3
b2
and
and
a2
b4
is a2 ,
is b2 and H.C.F. of c5 and c3 is c3 ngiwhere P(x) = 10x 2 y (x 3 + y 3) and Q(x) = 15xy2 (x4 – y4).
Solution :

∴ Required H.C.F. is a2 b2 c3.


Illustration 2. nee P(x) = 10x2y (x 3 + y 3 )
= 10x 2y (x + y) (x2 – xy + y2)

Find the G.C.D. of 24x2 y3z3 , 36x3y5z11 and 18xy2 z7 .


Solution : rin
Q(x) = 15xy2 (x 4 – y4)
= 15xy2 (x 2 + y2)(x2 – y2)
We have
24x 2y 3z3 = 31 × 23 × x 2 y 3z 3 g.n
= 15xy2 (x + y)(x – y)(x2 + y2)
Now, H.C.F. of 10 and 15 is 5.
36x 3y 5z 11 = 32 × 22 × x3 y5z11
18xy2 z7 = 32 × 21 × x × y2 × z7
∴ Required G.C.D. = 31 × 21 × x × y2 × z3
e
Common factors are x, y and x + y.
∴ H.C.F. of P(x) and Q(x) = 5xy (x + y).
Illustration 6. t
= 6xy2 z3 . Find the H.C.F. of a3 + b3 + c3 – 3abc and (a + b + c)3,
given that ab + bc + ca = 0.
Illustration 3.
Solution :
Find the H.C.F. of x2 + 5x + 6 and x2 + x – 6.
Let P(a, b, c) = a3 + b3 + c3 – 3abc
Solution :
and Q(a , b, c) = (a + b + c)3
Here, x2 + 5x + 6 = x2 + 2x + 3x + 6
We know that
= x (x + 2) + 3 (x + 2)
a3 + b3 + c3 – 3abc = (a + b + c) [(a + b + c)2
= (x + 2) (x + 3) – 3(ab + bc + ca)]
x2 + x – 6 = x 2 + 3x – 2x – 6 We are given that ab + bc + ca = 0
= x (x + 3) – 2 (x + 3) ∴ a3 + b3 + c3 – 3abc = (a + b + c) (a + b + c)2
= (x + 3) (x – 2) = (a + b + c)3
Only common factor is x + 3 P(a, b, c) = (a + b + c)3.
∴ H.C.F. = x + 3. Thus, from (i) and (ii), required H.C.F. is (a + b + c)3 .

CAT Complete Course | 19

Visit : www.EasyEngineering.net
Visit : www.EasyEngineering.net

Illustration 7. Lowest Common Multiple


If H.C.F. of the polynomial x 3 – 2x2 + px + 6 and x 3 A common multiple of two or more polynomials is a
– x – 5x + q is x – 3, find the value of 3p + 2q2 .
2
polynomial which is exactly divisible by each of them.
Solution : For example, x 3 is a common multiple of x 2 and x 3 and
xyz is a common multiple of xy and yz.
As x – 3 is the H.C.F. of the given polynomials
Illustration 9.
x – 3 is a factor of
Find the L.C.M. of 12a2 b3 c2 and 18a4b2 c3
x 3 – 2x 2 + px + 6 …(i)
Solution :
and x 3 – x 2 – 5x + q …(ii)
12a2 b3 c2 = 22 × 3 × a2 × b3 × c2
∴ When x – 3 = 0 or x = 3, then the value of and 18a4 b2 c3 = 2 × 32 × a4 × b2 × c3
polynomials (i) and (ii) is zero.
∴ Required L.C.M. = 22 × 32 × a4 × b3 × c3
Putting x = 3 in (i) , we get = 36a4 b3 c3.
(3)3 – 2(3) 2 + p(3) + 6 = 0 Illustration 10.
Or 27 – 18 + 3p + 6 = 0 or 3p = – 15 Find the L.C.M. of the following polynomials :

ww
Or p = –5
Similarly, putting x = 3 in (ii), we get
(x + 3)2 (x + 4), (x – 3) (x + 4)2
Solution :

Or
Or
w(3) 3 – (3) 2 – 5 (3) + q = 0

.Ea
27 – 9 – 15 + q = 0
3 + q = 0 or q = – 3
Let
and
P(x) = (x – 3)2 (x + 4)
Q(x) = (x – 3) (x + 4)2
Now, L.C.M of (x – 3)2 and (x – 3) is (x – 3)2

syE
3p + 2q2 = 3 (– 5) + 2(– 3)2
= – 15 + 18 = 3.
and L.C.M. of (x + 4) and (x + 4)2 is (x + 4)2
Thus, L.C.M. of P(x) and Q(x) is (x – 3)2 (x + 4)2 .

ngi
Illustration 8. Illustration 11.
If x 2 – x – 2 is the G.C.D. of the expressions (x – 2) Find the L.C.M. of x 3 + x 2 + x + 1 and x 3 + 2x 2 +
(2x + ax + 1) and (x + 1) (3x 2 + bx + 2), find the values
2 x + 2.
of a and b.
Solution : nee
Solution :
x 3 + x 2 + x + 1 = x 2 (x + 1) + 1 (x + 1)
Here, G.C.D. = x2 – x – 2
= x2 – 2x + x – 2 and rin = (x2 + 1)(x + 1)
x 3 + 2x 2 + x + 2 = x 3 + x + 2x 2 + 2
= x (x – 2) + 1 (x – 2)
= (x – 2)(x +1) g.n
= x (x 2 + 1) + 2 (x2 + 1)
= (x2 + 1)(x + 2)
We are given that (x – 2)(x +1) is also a factor of
(x – 2)(2x 2 + ax + 1).
∴ (x + 1) is a factor of 2x 2 + ax + 1
degree 1. e
Here, (x2 + 1), (x + 1) and (x + 2) each has highest

L.C.M. = (x2 + 1)(x + 1)(x + 2)


t
∴ When x + 1 = 0 i.e., x = – 1, 2x 2 + ax + 1 = 0 Illustration 12.
i.e., 2(– 1)2 + a (– 1) + 1 = 0 Find the L.C.M of the following polynomials :
or 2 – a + 1 = 0 or a = 3 P(x) = 12(x 4 – 36); Q(x) = 8 (x 4 + 5x2 – 6)
Similarly, as (x – 2)(x + 1) is also a factor of (x + 1) Solution :
(3x 2 + bx + 2). Here, P(x) = 12(x 4 – 36)
∴ (x – 2) is a factor of 3x 2 + bx + 2 = 12 [(x 2 )2 – (6) 2 ]
When x – 2 = 0 or x = 2, 3x 2 + bx + 2 = 0 = 12 (x2 – 6)(x 2 + 6)
i.e., 3(2) 2 + b(2) + 2 = 0 and Q(x) = 8 (x 4 + 5x 2 – 6)
or 12 + 2b + 2 = 0 = 8 (x4 + 6x2 – x2 – 6)
or 2b = – 14 = 8 [x2 (x2 + 6) – 1 (x2 + 6)]
or b = –7 = 8 (x 2 – 1)(x 2 + 6)
Thus, a = 3, b = – 7. Now, L.C.M of 12 and 8 is 24.

20 | CAT Complete Course

Visit : www.EasyEngineering.net
Visit : www.EasyEngineering.net

The factors (x2 – 6), (x2 + 6), (x2 – 1) each has highest (b) Here, f (x) = 4(x2 – 1) = 22 (x + 1)(x – 1)
degree 1. and g (x) = 12(1 – x3 )
∴ L.C.M. of P(x) and Q(x) = 22 × 3(1 – x)(1 + x + x2)
= 24 (x2 – 6)(x2 + 6)(x2 – 1) = – 2 2 × 3(x – 1)(1 + x + x2)
Product of Two Polynomials The least powers of the common factors 2, (x – 1) are
2 and 1 respectively.
Product of two polynomials = The product of ∴ H.C.F. = 22 (x – 1) = 4(x – 1)
their H.C.F. and L.C.M.
The highest powers of all the factors 2, 3, (x – 1),
f (x) × g (x) = H.C.F. × L.C.M. (x + 1) and (1 + x + x2 ) are 2, 1, 1, 1 and 1.
Let α and ∂ be two polynomials and H be their H.C.F. ∴ L.C.M. = 22 × 3(x – 1)(x + 1)(1 + x + x 2 )
and L be their L.C.M. = 12(x + 1)(x3 – 1)
Then, LH = ± α∂ Product of H.C.F. and L.C.M.
α∂ = 4(x – 1) × 12 (x + 1)(x3 – 1)
Or L = ±
H = 48(x2 – 1)(x3 – 1) …(iii)
α∂ and product of f (x) and g (x) = 4(x – 1) × 12 (1 – x 3 )
and H = ±

ww
Notes :
L

1. If f (x), g (x) and one of L.C.M., H.C.F. are given,


= 48(x2 – 1)(x3 – 1)
Comparing (iii) and (iv), we get
H.C.F. × L.C.M. = – f (x) × g (x)
…(iv)

w
the other can be found without ambiguity because L.C.M.

.Ea
and H.C.F., are unique except for a factor of (– 1). The
sign + or – is so chosen that the coefficient of the highest
degree term becomes positive.
which prove the required result.
Illustration 14.
The H.C.F. of two polynomials f (x) = (x – 1)(x2 – x – 6)

syE
2. If L.C.M., H.C.F. and one of the polynomials are
given, then the second polynomial can have both + and –
and g (x) = (x – 2)(x2 – 1) is x – 1. Find their L.C.M.
Solution :
f (x) = (x – 1)(x2 – x – 6)

ngi
signs.
= (x – 1)(x – 3)(x + 2)
3. If both L.C.M. and H.C.F. are to be found, then
we first find H.C.F. and then L.C.M. and g (x) = (x – 2)(x2 – 1)
Illustration 13.
Find the H.C.F and L.C.M of the following given nee = (x – 1)(x + 1)(x – 2)
Given that H.C.F. = x – 1
α∂

rin
polynomials and verify that the product of the L.C.M and Then, L.C.M. = ±
H.C.F differs from the product of the polynomials if at all H
by a factor (– 1).

g.n
(x – 1)(x – 3)(x + 2)(x – 1)(x + 1)(x – 2)
(a) f (x) = (2x + 1)2 (3x – 1), g (x) = (2x + 1)(3x – 1)3 = ±
(x – 1)
(b) f (x) = 4(x2 – 1), g (x) = 12 (1 – x3 ) = ± (x – 3)(x + 2)(x – 1)(x + 1)(x – 2)
Solution :
(a) We are given
f (x) = (2x + 1)2 (3x – 1)
g (x – 1) = (2x + 1)(3x – 1)3
sign
e
= ± (x2 – 1)(x2 – 4)(x – 3)
To make the coefficient of x5 positive, we take + ve
t
∴ L.C.M. = (x2 – 1)(x2 – 4)(x – 3).
Here, common factor are (2x + 1) and (3x – 1) and Illustration 15.
their least powers are 1 and 1 respectively. The G.C.D. of two polynomials α and ∂ is (x2 – 4x – 5)
∴ H.C.F. = (2x +1)(3x – 1) (x – 5x + 6). If α = 5(x + 1)(x2 – 7x + 10)(x2 – x – 6) and
2

The highest powers of all the factors (2x + 1) and ∂ = 9(x – 2)(x2 – 4x – 5)(x2 – 9), then find the L.C.M. of α
(3x – 1) are 2 and 3 respectively. and ∂.
L.C.M. = (2x + 1)2 (3x – 1)3 Solution :
Product of H.C.F and L.C.M = (2x + 1) (3x – 1) Here, α = 5(x + 1)(x2 – 7x + 10)(x2 – x – 6)
(2x + 1)2 (3x – 1)3 = 5(x + 1)(x – 2)(x – 5)(x – 3)(x + 2)
= (2x + 1)3 (3x – 1)4 …(i) and ∂ = 9(x – 2)(x2 – 4x – 5)(x 2 – 9)
And product of f (x) and g (x) = (2x + 1)2 (3x – 1)
= 9(x + 1)(x – 2)(x – 5)(x – 3)(x + 3)
(2x + 1) (3x – 1)3
= (2x + 1)3 (3x – 1)4 …(ii) and G.C.D. of α and ∂
Comparing (i) and (ii), we get = (x2 – 4x – 5)(x 2 – 5x + 6)
H.C.F. × L.C.M. = f (x) × g (x) = (x + 1)(x – 2)(x – 5)(x – 3)

CAT Complete Course | 21

Visit : www.EasyEngineering.net
Visit : www.EasyEngineering.net

α∂ Find the values of a and b so that the polynomials


We know that the L.C.M. = ±
H P(x), Q(x) and have H(x) as their H.C.F. :
= ± 45(x2 + 3x + 2)(x2 – 7x + 10) 16. P(x) = (x2 + 3x + 2) (x2 – 7x + a)
(x2 – 9) Q(x) = (x2 – 2x – 8) (x2 – 2x + b)
We take + ve sign to make the coefficient of the H(x) = (x + 1)(x – 4)
highest degree term positive. 17. P(x) = (x2 – 3x + 2) (x 2 + 7x + a)
Thus, the L.C.M. of α and ∂ is 45(x + 3x + 2)(x – 7x
2 2
Q(x) = (x2 + 5x + 4)(x2 – 5x + b)
+ 10)(x2 – 9).
H(x) = (x – 1)(x + 4)
Exercise A 18. P(x) = (x2 + 3x + 2) (x2 + 2x + a)
1. Find the G.C.D. of the following polynomials— Q(x) = (x2 + 7x + 12) (x2 + 7x + b)
18(x 3 – x 2 + x – 1); 12(x4 – 1) H(x) = (x + 1) (x + 3)
2. Find the G.C.D. of the following polynomials— 19. Find the G.C.D. of 4(x4 – 1) and 6(x 3 – x 2 – x + 1).
2(x4 – y4); 3(x3 + 2x2y – xy2 – 2y3) 20. If the G.C.D. of (x – 5) (x2 – x – a) and (x – 4)
(x2 – 2x – b) is (x – 4), (x – 5). Find the value of a, b.

ww
3. Find the G.C.D. of (x 3 + 2x 2 – 3x) and (2x 3 + 5x 2 –
3x)
4. Find the G.C.D. of the following polynomials—
21. Find the value of k for which the G.C.D. of (x2 – 2x –
24) and (x 2 – kx – 6) is (x – 6).

w
Find the G.C.D. of the following polynomials :
6x3 + 3x2 – 3x, 8x3 + 8x2 – 6x 22. x2 – x – 6; x 3 – 27

.Ea
5. If (x2 – x – 2) is the G.C.D. of the expressions (x – 2) 23. 2x2 – 9x + 4; 8x3 – 1
(2x2 + a x + 1) and (x + 1)(3x 2 + b x + 2). Find the 24. 18(6x4 + x3 – x2); 45(2x 6 + 3x5 + x4)
values of a and b.
2
syE 25. 12(3x 4 – 14x3 – 5x 2); 30(3x 5 + 4x 4 + x 3)
6. If (x – x – 6) is the G.C.D. of the expressions (x + 2) 26. 18x 3 + 45x 2 – 27x; 15x 4 – 135x 2
(2x2 + a x + 3) and (x – 3)(3x 2 + b x + 8). Find the
values of of a and b.
7. (x2 + x – 2) is the G.C.D. of the expressions (x – 1) ngi Exercise B
Find the L.C.M. of the following polynomials :

nee 4 3
(2x2 + ax + 2) and (x + 2) (3x 2 + bx + 1). Find the 1. 18x – 36x + 18x ; 45x – 45x
values of a and b.
2 6

2. 12x 4 + 324x; 36x 3 + 90x 2 – 54x


3

8. The L.C.M. and G.C.D. of the polynomials. P(x) and


Q(x) are 56(x4 + x) and 4(x2 – x + 1) respectively. If
rin
3. The G.C.D. of polynomials P(x) and Q(x) is (x – 3).
If P(x) = (x – 3) (x 2 + x – 2) and Q(x) = x 2 – 5x + 6.
Find the L.C.M. of P(x) and Q(x).
P(x) = 28(x3 + 1). Find Q(x).
9. The L.C.M. and G.C.D. of two polynomials P(x) and g.n
4. The G.C.D. of polynomials G(x) and H(x) is 10(x + 3)
(x – 1). If the polynomial G(x) = 10(x2 – 9)(x2 – 3x + 2)
Q(x) are 2(x4 – 1) and (x + 1) (x2 + 1) respectively. If
3 2
P(x) = x + x + x + 1 find Q(x).
3 2
G(x) and H(x).
e
and H(x) = 10x (x + 3)(x – 1)2 . Find the L.C.M. of

10. If P(x) = (x + 1) (x – 1) and Q(x) = (x – x + 1) 5. The G.C.D. and L.C.M. of the polynomials P(x) and
(x2 – 3x + 2). Find G.C.D. and L.C.M. of P(x) and
t
Q(x) respectively are 5(x + 3)(x – 1) and 20x (x2 – 9)
Q(x). (x 2 – 3x + 2). If P(x) = 10(x 2 – 9)(x – 1). Find Q(x).
11. Find the G.C.D. and L.C.M. of two polynomials P(x) 6. The G.C.D. and L.C.M. of two polynomials P(x) and
3 3 Q(x) are x (x + a) and 12(x + a)(x 2 – a2) respectively.
and Q(x). where P(x) = (x – 27) (x – 3x + 2) and
If P(x) = 4x (x + a), find Q(x).
Q(x) = (x2 + 3x + 9) (x2 – 5x + 6).
7. The G.C.D. and L.C.M. of two polynomials P(x) and
12. (x – k) is the G.C.D. of (x2 + x – 12) and (2x2 – kx – 9). Q(x) are x (x + a) and 12x2 (x – a)(x2 – a 2 ) respec-
Find the value of k. tively. If P(x) = 4x 2 (x + a)2, find Q(x).
13. (x + 1)(x – 4) is the G.C.D. of the polynomials (x – 4) 8. The G.C.D. of two polynomials P(x) = 4x2 (x2 – 3x + 2)
2 2
(2x + x – a) and (x + 1) (2x + bx – 12). Find a and and Q(x) = 12x (x – 2)(x2 – 4) is 4x (x – 2). Find the
b. L.C.M. of the polynomials P(x) and Q(x).
2
14. For what value of k, the G.C.D. of the x + x – (2k + 2) 9. Find the L.C.M. of the following polynomial—
2
and (2x + kx – 12) is (x + 4) ? P(x) = x 4 – 1; Q(x) = x 3 – x.
15. (x – 3) is the G.C.D. of (x 3 – 2x2 + px + 6) and (x2 – 10. Find the L.C.M. of the following polynomial—
5x + q). Find (6p + 5q). P(x) = 12(x4 – 25); Q(x) = (x4 + 4x2 – 5).

22 | CAT Complete Course

Visit : www.EasyEngineering.net
Visit : www.EasyEngineering.net

11. Find the L.C.M. of the following polynomial— = 3x (2x2 + 2x – x – 1)


P(x) = 12(x4 – 36); Q(x) = 8(x4 + 5x2 – 6). = 3x (2x – 1) (x + 1)
12. Find the G.C.D. of P(x) = 8(x4 –16); Q(x) = 12(x3 – 8). and Q(x) = 8x 3 + 8x 2 – 6x
13. Find L.C.M. of the following polynomials— = 2x (4x2 + 4x – 3)
4x 2 (x 2 – a2 ); 9x 2 (x 3 – a3 ) = 2x (4x2 + 6x – 2x – 3)
14. Find the L.C.M. of the following polynomial— = 2x (2x – 1) (2x + 3)
3(x 2 – 7x + 12); 24(x 2 – 9x + 20). G.C.D. = x (2x – 1).
15. The L.C.M. and H.C.F. of two polynomials P(x) and 5. x2 – x – 2 = x2 – 2x + x – 2
Q(x) are 27x 3 (x + a)(x3 – a 3 ) and x2 (x – a) respec- = x (x – 2) + 1 (x – 2)
tively. If P(x) = 3x 2 (x 2 – a2 ), find Q(x).
= (x + 1) (x – 2)
16. Find the G.C.D. of the following polynomials—
x = – 1 or 2
(1 + x + x3 – x 5) and (1 – x 4 – x 6 + x 7)
Let P(x) = (x – 2) (2x 2 + ax + 1)
17. If (x – k) be the H.C.F. of ax2 + bx + c and cx2 + ax + b.
When x = – 1
Prove that a3 +b3 + c3 – 3abc = 0

ww
18. If H (x) and L (x) denote the H.C.F. and L.C.M. of
two polynomials f (x) and g (x) such that H (x) + L (x)
= f (x) + g (x). Then Prove that {H (x)}3 + {L(x)}3 =
P(– 1) = (– 1 – 2) [2 (– 1)2 + a (– 1) + 1]
= – 3 (2 – a + 1) = – 3 (3 – a)
Since, (x + 1) is a factor of P(x)

w
{f (x)}3 + {g (x)}3.

.Ea
19. Find all the common roots of the equations—
x 4 + 5x 3 – x 2 – 5x = 0 and x 3 + 4x 2 – x – 4 = 0
P (– 1) = 0
– 3 (3 – a) = 0
3–a = 0
Answer
syE
Exercise A Let
a = 3
Q(x) = (x + 1) (3x 2 + bx + 2)

ngi
When x = 2
1. Let P(x) = 18 (x 3 – x 2 + x – 1)
Q(2) = (2 + 1) (12 + 2b + 2)
= 18 [x 2 (x – 1) + 1 (x – 1)]

nee
= 3 (14 + 2b)
= 18 (x 2 + 1) (x – 1)
Since, (x – 2) is a factor of Q(x)
and Q(x) = 12 (x 4 – 1)
Q (2) = 0
=
=
12 (x 2 – 1) (x 2 + 1)
12 (x – 1) (x + 1) (x 2 + 1)
3 (14 + 2b) = 0
14 + 2b = 0 rin
2. Let
G.C.D. =
P(x) =
6 (x – 1) (x 2 + 1).
2 (x 4 – y 4)
2b = – 14
b = –7 g.n
and
=
=
Q(x) =
2 (x 2 – y 2) (x 2 + y2)
2 (x – y) (x + y) (x 2 + y2)
3 (x 3 + 2x 2y – xy2 – 2y3)
6.
a = 3, b = – 7.
e
x 2 – x – 6 = x 2 – 3x + 2x – 6
= x (x – 3) + 2 (x – 3)
t
= 3 (x + 2y) (x 2 – y2)
= (x – 3) (x + 2)
= 3 (x + 2y) (x + y) (x – y)
x = 3 or – 2
G.C.D. = (x – y) (x + y) = (x2 – y2).
Let P(x) = (x + 2) (2x 2 + ax + 3)
3. Let P(x) = x3 + 2x 2 – 3x
When x = 3
= x (x 2 + 2x – 3)
P(3) = (3 + 2) × [2 × 9 + 3a + 3]
= x (x 2 + 3x – x – 3)
= (5) (18 + 3a + 3)
= x (x + 3) (x – 1) = 5 (3a + 21)
and Q(x) = 2x 3 + 5x 2 – 3x Since, (x – 3) is a factor of P(x)
= x (2x 2 + 6x – x – 3) P (3) = 0
= x (x + 3) (2x – 1) 5 (3a + 21) = 0
G.C.D. = x (x + 3). 3a + 21 = 0
4. Let P(x) = 6x3 + 3x 2 – 3x 3a = – 21
= 3x (2x 2 + x – 1) a = –7

CAT Complete Course | 23

Visit : www.EasyEngineering.net
Visit : www.EasyEngineering.net

Let Q(x) = (x – 3) (3x 2 + bx + 8) (L.C.M.) × (G.C.D.)


Q(x) =
When x = – 2 P(x)
2(x 4 – 1) × (x + 1) (x2 + 1)
Q(– 2) = (– 2 – 3) × (3 × 4 – 2b + 8) =
(x 2 + 1)(x + 1)
= (– 5) × (12 – 2b + 8) 4
= 2 (x – 1).
= – 5 (20 – 2b) 10. P(x) = (x 3 + 1) (x – 1)
Since, (x + 2) is a factor of Q(x) = (x + 1)(x2 – x + 1)(x – 1)
Q(– 2) = 0 Q(x) = (x 2 – x + 1)(x 2 – 3x + 2)
– 5 (20 – 2b) = 0 = (x 2 – x + 1) [x (x – 2) – 1 (x – 2)]
20 – 2b = 0 = (x 2 – x + 1) (x – 1)(x – 2)
– 2b = – 20 G.C.D. = (x – 1) (x 2 – x + 1)
b = 10 and L.C.M. = (x + 1) (x 2 – x + 1) (x – 1)(x – 2)
a = – 7, b = 10. = (x 3 + 1)(x2 – 3x + 2).
7. 2
x +x–2 = (x + 2) (x – 1) 11. P(x) = (x 3 – 27)(x 2 – 3x + 2)

ww
Let
x
P(x)
=
=
– 2 or 1
(x – 1) (2x 2 + ax + 2)
= (x 3 – 33 )(x 2 – 2x – x + 2)
= (x – 3)(x 2 + 3x + 9)(x – 1)(x – 2)

w
When x = – 2
P (– 2) = (– 2 – 1) × (8 – 2a + 2)

.Ea
= – 3 (10 – 2a)
Since, (x + 2) is a factor of P(x)
Q(x) = (x 2 + 3x + 9)(x2 – 5x + 6)
= (x2 + 3x + 9)(x2 – 3x – 2x + 6)
= (x2 + 3x + 9)(x – 3)(x – 2)
G.C.D. = (x2 + 3x + 9)(x – 2)(x – 3)
P(– 2) = 0
– 3 (10 – 2a) = 0 syE
[Putting the value p (– 2)]
= (x 3 – 27)(x – 2)
and L.C.M. = (x – 3)(x 2 + 3x + 9)(x – 1)(x – 2).
– 30 + 6a = 0
6a = 30 ngi12. Let
Put x = k
P(x) = x 2 + x – 12

Let
a = 5
Q(x) = (x + 2) (3x 2 + bx + 1) nee P(k) = k2 + k – 12
Since, (x – k) is a factor of p(x).
When x = 1
Q(1) = (1 + 2) × (3 + b + 1) = 3 (4 + b) ⇒
rin
P(k) = 0
k2 + k – 12 = 0

Since, (x – 1) is a factor of Q(x)


Q(1) = 0
⇒ (k + 4)(k – 3) = 0
⇒ g.n
k = – 4 or k = 3 …(i)

3 (4 + b)
12 + 3b
3b
=
=
=
0
0
– 12
Let

e
2
Q(x) = 2x – kx – 9
Put x = k, Q(x) = 2k 2 – k 2 – 9
= k2 – 9 t
Since, (x – k) is a factor of Q(x)
b = –4
Q(k) = 0
a = 5, b = – 4.
k2 – 9 = 0
8. P(x) = 28 (x 3 + 1)
k2 = 9
(L.C.M.) × (G.C.D.)
Q(x) = k = ±3 …(ii)
P(x)
From (i) and (ii)
56 (x + x) × 4 (x2 – x + 1)
4
= k = 3 is the only common solution.
28 (x3 + 1)
56x (x 3 + 1) × 4 (x 2 – x + 1) 13. Let P(x) = (x – 4)(2x 2 + x – a)
=
28 (x 3 + 1) Put x = –1
= 8x (x 2 – x + 1). P(– 1) = (– 1 – 4) × (2 – 1 – a)
9. P(x) = x3 + x2 + x + 1 = – 5 (1 – a)
= x2 (x + 1) + 1 (x + 1) Since, (x + 1) is a factor of P(x)
= (x2 + 1) (x + 1) P(– 1) = 0

24 | CAT Complete Course

Visit : www.EasyEngineering.net
Visit : www.EasyEngineering.net

– 5 (1 – a) = 0 a = 12
– 5 + 5a = 0 Q(x) = (x2 – 2x – 8)(x 2 – 2x + b)
a = 1 = (x – 4)(x + 2)(x2 – 2x + b)
Let Q(x) = (x + 1)(2x 2 + bx – 12) (x + 1) = 0 Put x = – 1
Put x = 4 Then , Q(–1) = (–1 –4)(–1 + 2)(1 + 2 + b)
Q(4) = (4 + 1) × (2 × 16 + 4b – 12) = (–5)(1)(3 + b)
= 5 (4b + 20) = – 5 (3 + b)
Since, (x – 4) is a factor of Q(x) Q(–1) = 0
Q(4) = 0 – 5 (3 + b) = 0
5 (4b + 20) = 0 b = – 3.
4b + 20 = 0 17. P(x) = (x 2 – 3x + 2) (x 2 + 7x + a)
b = – 5. = (x – 2)(x – 1)(x 2 + 7x + a)
14. Let P(x) = x 2 + x – (2k + 2) x+4 = 0

ww
Then
x+4 = 0
x = –4
P(– 4) = 16 – 4 – 2k – 2 = 10 – 2k
Put
Then,
x = –4
P(– 4) = (– 6)(– 5)(16 – 28 + a)
= 30 (a – 12)

w
Since, (x + 4) is a factor of P(x)
P(– 4) = 0
10 – 2k = 0 .Ea
Since, (x – 1)(x + 4) as their HCF ……… (Given)
P(– 4) = 0
30 (a – 12) = 0
2k = 10
k = 5. syE a = 12
Q(x) = (x 2 + 5x + 4)(x 2 – 5x + b)

ngi
15. A(x) = x 3 – 2x 2 + px + 6 = (x + 1)(x + 4)(x2 – 5x + b)
A(3) = 27 – 18 + 3p + 6 = 3p + 15 (x – 1) = 0; Put x = 1

nee
Since, (x – 3) is a factor of A(x) Then, Q(1) = (2) (5) (1 – 5 + b)
A(3) = 0 = 10 (b – 4)
3p + 15 = 0 Q(1) = 0
3p = – 15
p = –5 rin
10 (b – 4) = 0
b = 4.
B(x) = x 2 – 5x + q
B(3) = 9 – 15 + q = q – 6
18.
g.n
P(x) = (x2 + 3x + 2)(x 2 + 2x + a)
= (x + 1)(x + 2)(x 2 + 2x + a)
Since, (x – 3) is a factor of B(x).
B(3) = 0
q–6 = 0
Put
Then,
x = –3
e t
P(– 3) = (– 3 + 1)(– 3 + 2)(9 – 6 + a)
= (– 2)(– 1)(3 + a)
q = 6 = 2 (3 + a)
6p + 5q = 6 (–5) + 5 (6) Since, (x + 1) (x + 3) is H.C.F. ……… (Given)
= – 30 + 30 = 0. P(– 3) = 0
16. P(x) = (x2 + 3x + 2) (x 2 – 7x + a) 2 (3 + a) = 0
= (x + 1)(x + 2)(x 2 – 7x + a) 3 + a = 0

x–4 = 0 a = –3

Put x = 4 Q(x) = (x 2 + 7x + 12)(x 2 + 7x + b)


= (x + 3) (x + 4)(x 2 + 7x + b)
Then, P(4) = (4 + 1) (4 + 2) (16 – 28 + a)
Put x = –1
= (5)(6)(– 12 + a) = 30 (a – 12)
Since, (x + 1)(x – 4) as their H.C.F. ……… (Given) Then, Q(–1) = (– 1 + 3)(– 1 + 4)(1 – 7 + b)
P(4) = 0 = (2)(3)(b – 6)
30 (a – 12) = 0 = 6(b – 6)

CAT Complete Course | 25

Visit : www.EasyEngineering.net
Visit : www.EasyEngineering.net

Q(– 1) = 0 25. 12 (3x4 – 14x3 – 5x2)


6 (b – 6) = 0 = 12x 2 (3x 2 – 14x – 5)
b–6 = 0 = 2 × 2 × 3x 2 (x – 5) (3x + 1)
b = 6 30 (3x 5 + 4x 4 + x 3) = 30x3 (3x 2 + 4x + 1)
a = – 3, b = 6. = 2 × 3 × 5x 3 (3x + 1)(x + 1)
19. 4 (x 4 – 1) = 4 (x 2 – 1)(x2 + 1) G.C.D. = 2 × 3x 2 (3x + 1)
= 6x 2 (3x + 1) .
= 4 (x + 1)(x – 1)(x 2 + 1)
26. 18x 3 + 45x 2 – 27x = 9x (2x 2 + 5x – 3)
6 (x 3 – x 2 – x + 1) = 6 (x – 1)2 (x + 1)
= 3 × 3x (x + 3)(2x – 1)
G.C.D. = 2 (x – 1) (x + 1)
15x 4 – 135x2 = 15x2 (x 2 – 9)
= 2 (x 2 – 1).
= 3 × 5x 2 (x – 3)(x + 3)
20. Let P(x) = (x – 5)(x2 – x – a) G.C.D. = 3x (x + 3).
Put x = 4
Exercise B
Then, P(4) = (4 – 5)(16 – 4 – a)
1. 18x 4 – 36x 3 + 18x 2 = 18x 2 (x 2 – 2x + 1)

ww = – 1 (12 – a)
Since, (x – 4) is a factor of P(x)
P(4) = 0
= 18x 2 (x – 1)2
45x 6 – 45x 3 = 45x 3 (x 3 – 1)

Let
w – 1 (12 – a) = 0

.Ea
a = 12
Q(x) = (x – 4)(x 2 – 2x – b)
L.C.M. of 18 and 45 = 90
= 45x 3 (x – 1)(x 2 + x + 1)

L.C.M. = 90x 3 (x – 1)2 (x 2 + x + 1)

Put
Then ,
x = 5
syE
Q(5) = (5 – 4)(25 – 10 – b)
2
= 90x3 (x – 1)(x 3 – 1).
12x 4 + 324x = 12x (x 3 + 27)
= 12x (x + 3)(x 2 – 3x + 9)

ngi
= (15 – b)
36x + 90x – 54x = 18x (2x 2 + 5x – 3)
3 2
Since, (x – 5) is a factor of Q(x)
= 18x (2x – 1)(x + 3)

nee
Q(5) = 0
L.C.M. of 12 and 18 = 36
15 – b = 0
L.C.M. = 36x (x + 3)(x 2 – 3x + 9)
b = 15.
21. Let P(x) = x 2 – kx – 6
x = 6 rin 3
= 36x (x + 27)(2x – 1).
(2x – 1)

P(6) = 36 – 6k – 6 = 30 – 6k
Hence, (x – 6) is the G.C.D. ……… (Given) g.n
3. Factors of x + x – 2 = x2 + 2x – x – 2
2

= x (x + 2) – 1 (x + 2)

∴ (x – 6) is the factor of P(6).


∴ P(6) = 0
30 – 6k = 0
2
e
= (x + 2)(x – 1)
Factors of x – 5x + 6 = x 2 – 3x – 2x + 6
= x (x – 3) – 2 (x – 3) t
– 6k = – 30 = (x – 3)(x – 2)
k = 5. L.C.M. × G.C.D. = P(x) × Q(x)
22. x 2 – x – 6 = (x – 3)(x + 2) P(x) × Q(x)
L.C.M. =
x 3 – 27 = (x – 3)(x 2 + 3x + 9) G.C.D.
(x – 3)(x + 2)(x – 1)(x – 2)(x – 3)
G.C.D. = x – 3. =
(x – 3)
23. 2
2x – 9x + 4 = (2x – 1)(x – 4)
= (x – 2)(x – 3)(x – 1)(x + 2).
8x 3 – 1 = (2x – 1)(4x2 + 2x + 1)
4. G.C.D. × L.C.M. of two polynomials = Product of
G.C.D. = (2x – 1). two polynomials
24. 18 (6x 4 + x 3 – x 2) = 18x 2 (6x 2 + x – 1) 10 (x + 3)(x – 1) × L.C.M. = 10 (x 2 – 9)(x 2 – 3x + 2)
= 18x 2 (2x + 1)(3x – 1) 10x (x + 3)(x – 1)2
45 (2x 6 + 3x 5 + x 4) = 45x 4 (2x 2 + 3x + 1) Factors of x 2 – 3x + 2 = x 2 – 2x – x + 2
= 45x 4 (x + 1)(2x + 1) = x (x – 2) – 1 (x – 2)
G.C.D. = 9x 2 (2x + 1) = (x – 2)(x – 1)

26 | CAT Complete Course

Visit : www.EasyEngineering.net
Visit : www.EasyEngineering.net

Factors of x 2 – 9 = (x – 3)(x + 3) = 12x 2 (x – 2)2 (x – 1)(x + 2)


10 (x 2 – 9)(x 2 – 3x + 2) = 12x 2 (x – 2)(x – 1)(x – 2)(x + 2)
10x (x + 3)(x – 1)2
L.C.M. = = 12x 2 (x 2 – 3x + 2)(x 2 – 4).
10(x + 3)(x – 1)
9. P(x) = (x 4 – 1)
10 (x – 3)(x + 3)(x – 2)(x – 1)
10x (x + 3)(x – 1)2 = (x – 1)(x + 1)(x2 + 1)
=
10 (x + 3)(x – 1) Q(x) = x3 – x
= (x – 3)(x – 2) 10x (x + 3)(x – 1)2 = x (x 2 – 1)
= 10x (x – 2)(x – 3)(x + 3) (x – 1)2 = x (x + 1)(x – 1)
= 10x (x 2 – 9)(x – 2)(x – 1)2 . L.C.M. = x (x – 1)(x + 1)(x 2 + 1)
5. G.C.D. = 5 (x + 3)(x – 1) = x (x 4 – 1).
L.C.M. = 20x (x 2 – 9)(x 2 – 3x + 2) 10. P(x) = 12 (x 4 – 25)
P(x) = 10 (x 2 – 9)(x – 1) = 12 (x 2 – 5)(x 2 + 5)
Q(x) = ? Q(x) = x 4 + 4x2 – 5
Now, P(x) × Q(x) = G.C.D. × L.C.M.

ww
⇒ 10 (x 2 – 9)(x – 1) × Q(x) = 5 (x + 3) (x – 1)
× 20x (x 2 – 9)(x 2 – 3x + 2)
L.C.M.
=
=
=
(x2 – 1)(x2 + 5)
12 (x 2 – 5)(x 2 + 5)(x 2 – 1)
12 (x 4 – 25)(x 2 – 1).

w
⇒ Q(x)

=
.Ea
5 (x + 3)(x – 1) × 20x (x 2 – 9)(x 2 – 3x + 2)
10 (x 2 – 9)(x – 1)
11. P(x)

Q(x)
=
=
=
12 (x 4 – 36)
12 (x 2 – 6)(x 2 + 6)
8x 4 + 5x 2 – 6

6. We know that :
syE
= 10x (x + 3)(x 2 – 3x + 2)
= 10x (x + 3)(x – 1)(x – 2). L.C.M.
=
=
8 (x 2 + 6)(x 2 – 1)
24 (x 2 – 6)(x 2 + 6)(x 2 – 1)

ngi
= 24 (x 4 – 36)(x 2 – 1).
L.C.M. × G.C.D. = P(x) × Q(x) 12. P(x) = 8 (x 4 – 16)
x × (x + a) × 12 (x + a) (x 2 – a2 ) 8 (x 2 + 4)(x – 2)(x + 2)

nee
=
= 4x (x + a) × Q(x) Q(x) = 12(x 3 – 8)
x × (x + a) × 12 (x + a)(x2 – a2 )
Q(x) = = 12 (x – 2) (x2 + 2x + 4)
4x (x + a)
= 3 (x + a)(x – a2 ).
2
13.
G.C.D.
4x 2 (x 2 – a2 )
=
= rin
4 (x – 2).
4x 2 (x + a)(x – a)
7. We know that :
L.C.M. × G.C.D. = P(x) × Q(x)
g.n
9x 2 (x 3 – a3 ) = 9x 2 (x – a)(x2 + xa + a2 )

e
x × (x + a) × 12x 2 (x – a)(x2 – a2 ) L.C.M. of 4 and 9 = 36

Q(x) =
= 4x 2 (x + a)2 × Q(x)
x (x + a) × 12x 2 (x – a)(x 2 – a2 )
4x 2 (x + a)2
3x (x – a)(x2 – a2 )
14.
L.C.M. = 36x 2 (x – a)(x + a)(x2 + xa + a2 )
= 36x 2 (x 3 – a3 )(x + a).
3 (x 2 – 7x + 12) = 3 (x – 4)(x – 3)
t
= 24 (x 2 – 9x + 20) = 24 (x – 4) (x – 5)
(x + a)
L.C.M. of 3 and 24 = 24
3x (x – a)(x + a)(x – a)
= L.C.M. = 24 (x – 4)(x – 3)(x – 5).
(x + a)
= 3x (x – a)2 . 15. G.C.D. × L.C.M. = Product of two polynomials
8. G.C.D. × L.C.M. = Product of the two polynomials 27x 3 (x + a)(x 3 – a3 ) × x 2 (x – a)
4x (x – 2) × L.C.M. = 3x 2 (x2 – a2 ) × Q(x)
= 4x 2 (x 2 – 3x + 2) × 12x (x – 2) (x 2 – 4) 27x 3 (x + a)(x3 – a3 ) × x2 (x – a)
Q(x) =
= 4x 2 (x 2 – 2x – x + 2) × 12x (x – 2) (x – 2)(x + 2) 3x 2 (x 2 – a2 )
= 48x 3 (x – 2)(x – 1)(x – 2)2 (x + 2) = 9x3 (x3 – a3 ).
= 48x 3 (x – 2)3 (x – 1)(x + 2) 16. P(x) = 1 + x + x 3 – x 5
48x 3 (x – 2)3 (x – 1)(x + 2) = (1 + x) + x 3 (1 – x 2)
L.C.M. =
4x (x – 2) = (1 + x) + x 3 (1 – x)(1 + x)

CAT Complete Course | 27

Visit : www.EasyEngineering.net
Visit : www.EasyEngineering.net

= (1 + x) [1 + x 3 (1 – x)] b2 – ac c2 – ab
2 = 2
= (1 + x)(1 + x 3 – x 4) c – ab a – bc
Q(x) = (1 – x 4 – x 6 + x 7) ⇒ (b – ac)(a2 – bc)
2 = (c2 – ab)2
= (1 – x 4 ) – (x 6 – x 7) ⇒ a2 b2 – b3 c – a3c + abc 2 = c4 – 2c2ab + a2 b2
= (1 + x 2) (1 – x 2 ) – x 6 (1 – x) ⇒ a3 c + b3 c + c 4 – 3abc2 = 0
= (1 – x) [(1 + x 2)(1 + x) – x 6] ⇒ c (a3 + b3 + c3 – 3abc) = 0
⇒ a3 + b3 + c3 – 3abc = 0 [Q c = 0]
= (1 – x) [1 + x + x 2 + x 3 – x 6]
= (1 – x) [(1 + x) + x 2 + x 3 (1 – x 3)] 18. We have
= (1 – x) [(1 + x + x 2) + x 3 (1 – x)(1 + x + x 2)] H (x) + L(x) = f (x) + g (x) …(i)
= (1 – x)(1 + x + x2)(1 + x3 – x 4) ⇒ {H (x) + L(x)}3 = {f (x) + g (x)}3
G.C.D. = 1 + x 3 – x 4. ⇒ {H (x)3} + {L(x)}3 + 3H (x). L(x). {H(x) + L(x)}
17. Let p(x) = ax2 + bx + c and q(x) = cx2 + ax + b. = {f (x3 )} + {g (x3 )} + 3 {f (x) g (x)
Since, (x – k) is the H.C.F. of the given polynomials. {f (x) + g (x)}
⇒ {H(x)3 + {L(x)}3 + 3 H (x). L(x) {H(x) + L(x)}

ww
This means k is a zero of both the given polynomials
p(x) and q(x).
Now, k is a zero of ax 2 + bx + c
= {f (x)}3 + {g (x)}3 + 3 H (x) L(x)
{H(x) + L(x}

w ak 2 + bk + c = 0

ck2 + ak + b = 0
2

.Ea
Also, k is a zero of cx + ax + b = 0
…(i)

…(ii)
[Using (i) and f (x). g (x) = H(x). L(x)]
⇒ {H(x)}3 + {L(x)}3 = {f (x)3 + {g (x)}3
19. Given in Ist expression,

2
k2
b – ac
= 2 syE
Solving (i) and (ii), by cross multiplication, we get
k
=
c – ab a2 – bc
1
…(iii)
= x4 + 5x3 – x2 – 5x
= x3 (x + 5) – x (x + 5)
= (x + 5) (x3 – x)
From first two members of (iii), we get

k = 2
b2 – ac ngi
…(iv) and IInd expression,
= x (x 2 – 1) (x + 5)

c – ab
From last two members of (iii), we get
c2 – ab
nee = x3 + 4x2 – x – 4
= x2 (x + 4) – 1 (x + 4)
k = 2

From (iv) and (v), we get


a – bc
…(v)

rin
= (x + 4) (x2 – 1)
So, common of both expression in (x2 – 1).

g.n ●●●

e t

28 | CAT Complete Course

Visit : www.EasyEngineering.net
Visit : www.EasyEngineering.net

3 Percentage

Introduction 12
(b) % = ?
5
The word per cent means per hundred. The sign of
per cent is %. (c)  2% = ?

If a student gets 20% marks, it means that he gets 20 (d) log10 100% = ?
marks out of 100 marks.
1
If my mother gives me Rs. 40 out of Rs. 100. It means (e) % = ?
2– 2
I get 40% of what my mother had.

ww
If a fraction number has denominator 100. It is known
as percentage and its numerator is called as rate per cent.
(f)

(g)
5
5% = ?

32% = ?

w
Some Important Points

.Ea
[A] To find the percentage of a fraction, decimal
or a whole number just multiply these numbers by 100
with sign %—
(h)

(i)
3
64% = ?

1
tan 30°
% = ?

(i)
2
5
1
2
5
1
syE
= × 100% = 40%

= × 100% = 16·66%
(j)
1
 2 (sin 45° + cos 45°)

 1 1/2
% = ?

ngi
(ii)
6 6 (k)  22 + 3 2  % = ?
3 3  
(iii) = × 100% = 75%
 1 1/2
nee
4 4
2·5 = 2·5 × 100% = 250% (l)  52 – 32  % = ?
(iv)  
(v) 0·05 = 0·05 × 100% = 5%

rin
Solution :
(vi) 1·75 = 1·75 × 100% = 175%
4 4 1 1
(vii) 4 = 4 × 100% = 400% (a) % = × = ·

g.n
3 3 100 75
(viii) 12 = 12 × 100% = 1200% 12 1 3
(b) × = ·
(ix) 3 = 3 × 100% = 300% 5 100 125
[B] The percentage of fraction, decimal or a whole
number can be converted as fractional number as
follow—
(c)

(d)
1·41% =
1·41
=
e
100 10000
141

log10 10 2 % = 2 log10 10%.


·
t
2 2 1 2 1
% = × = = 1 1
5 5 100 500 250 = 2×1× =
100 50
3 3 1 3
or % = × = 4 1
4 4 100 400 (e) 22 % = 4% = = ·
100 25
1 25
2·5% = 2·5 × = 5 1
100 1000 (f) = ·
100 20
1 35 1 35
or 0·35% = 0·35 × = × = 5 5
1
100 100 100 10000 (g) 25 % = 25 % = 2% = 50
√ ·
4 3
4% = and 3% = 3 3
100 100 4 1
(h) 43 % = 43 % = 4% = 100
√ = ·
Illustration 1. 25
1
Represent the following percentages as fractional (i) % =  3% = 1·732%

numbers— 1/√
3
1 1·732 1732
(a) 1 % = ? = =
3 100 100000

CAT Complete Course | 29

Visit : www.EasyEngineering.net
Visit : www.EasyEngineering.net

1 Illustration 4.
(j) %
√  2  sin 45°. 1 + cos 45°. 1 
 2√ What per cent of—
 2
√  2
√ (i) 20 is 80 (ii) 125 is 25
1 1 1 (iii) 3 is 18 (iv) 150 is 5
= × % = × 1%
2 sin 90° 2 Solution :
1 1 1
= × = · (i) Let required percentage be a%
2 100 200
Now, 20 × a% = 80
 1  1/2 1
(k)   % = % 20 ×
a
4 + 9  √13 or
100
= 80

13 × 1 = 3‚501 × 1
√ 80 × 100
= ⇒ a =
13 100 13 100 20
3501 ⇒ a = 400
=
1300000 Hence, 80 is 400% of 20.
 1  1/2  1  1/2 1 1 (ii) Required percentage of
 25 – 9  % =  16  % = 4 % = 400 ·
ww
(l)

Illustration 2.
   125 =
25 × 100
125
= 20

w
Hence, 25 is 20% of 125.
Represent the following percentages as decimals—
(iii) Required percentage of
(i)
(ii)
25% = ?
3·5% = ?
.Ea 3 =
18 × 100
3
= 600
(iii)

(iv)
120% = ?
12
5
% = ? syE Hence, 18 is 600% of 3.
(iv) Required percentage of
5 × 100 10
(v)
Solution :
90% = ?
ngi 10
150 =
150
=
3

nee
25 Hence, 5 is % of 150.
(i) = 0·25 3
100 Illustration 5.
35

rin
(ii) = 0·035 Evaluate—
100 × 10
120 (i) 30 % of 900 = ?
(iii) = 1·2

(iv)
12 1
×
5 100
100
=
3
125
= 0·024
(ii)
1

(iii) 42% of 700 = ? g.n


1 % of 250 = ?
2

(v)

Illustration 3.
90
100
= 0·9
(iv) 2·5% of 12500 = ?
Solution :
(i)
30
e
× 900 = 270
t
Represent the following decimals as rate percent— 100
(i) 0·25 = ? 3 1 15
(ii) × × 250 =
2 100 4
(ii) 0·015 = ?
42
(iii) 1·75 = ? (iii) × 700 = 294
100
(iv) 3·33 = ? 25
(v) 0·005 = ? (iv) × 12500 = 312·5
1000
Solution : Illustration 6.
(i) (0·25 × 100)% = 25%. Ram pays 5% Income Tax. If his annual income is
15 3 Rs. 12500. How much does he pay as Income Tax
(ii) (0·015 × 100)% = % = %.
10 2 annually ?
(iii) (1·75 × 100)% = 175%. Solution :
(iv) (3·33 × 100)% = 333%. Income Tax = 5% of Annual Income = 5% × 12500
5 1 5
(v) (0·005 × 100)% = % = %. = × 12500 = Rs. 625
10 2 100

30 | CAT Complete Course

Visit : www.EasyEngineering.net
Visit : www.EasyEngineering.net

Illustration 7. Solution :
My mother has Rs. 5000 and she gives me 20%. Increase in price of potato (F.V. – B.V.) = Rs. 2 / kg,
How much I get ? and Base Value = Rs. 10 / kg
Solution : 2
Percentage Increase = × 100% = 20%.
I get = 20% of 5000 10
20 Illustration 11.
= × 5000 = Rs. 1000.
100 If Rahim’s salary is 40% more than that of Ram’s
Illustration 8. salary, then how much per cent Ram’s salary less than that
The monthly salary of Shyam is Rs. 10,000. He of Rahim ?
spends 25% on the education of his children, 40% on Solution :
food, 10% on house rent, 3% on travels and 5% on Let Ram’s salary = Rs. 100
miscellaneous and rest he save for the future. Find his
savings. Salary of Rahim = Rs. 140
Solution : Clearly Ram’s salary is Rs. 40 less than that of
Rahim.
Total expenditure = 25 + 10 + 40 + 3 + 5 = 83 %

ww
Out of Rs. 100 Shyam expense Rs. 83. So he saves
Rs. 17 out of Rs. 100. less
When Rahim’s salary is 140, then Ram’s salary = 40

w 17 40
Now, His savings = 17% of 10000 = × 10000 Rahim’s salary is Re. 1, then Ram’s salary = less
100 140

.Ea
= Rs. 1700.
Percentage Increase / Percentage Decrease
Rahim’s salary is Rs. 100, then Ram’s salary =
4
14
×

(i) Percentage Change


syE
Final value – Initial value
× 100
100 =
200
7
Illustration 12.
2
% = 28 % less.
7

ngi
% Change =
Initial value If initial cost of pen is increased by 25%, then find
∆ Value the final cost.
= × 100

=
Initial value
Difference
Initial value
× 100 nee
Solution :
Let initial cost of pen = a
Increased cost price = a + a × 25%
(ii) Percentage Increase
Increase value rin = a+
25a

g.n
100
Percentage Increase = × 100
Base value 100 + 25 125
Clearly final cost = ×a= a
(iii) Percentage Decrease 100 100

Percentage Decrease =
Decrease value
Base value
× 100

Percentage increase/decrease is considered with


e
Final cost = Initial price ×

t
 100 + percentage increase 

 100


respect to the Base Value (Initial Value) unless mentioned
otherwise. [C] Two different percentage of a number—
Illustration 9. If a% and b% of a number N is x 1 and x2 respec-
The salary of Ram increases from Rs. 5000 to tively. Find the number N.
Rs. 6000. Find the percentage increase in salary ? Solution :
Solution : Na
x 1 = N × a% = …(1)
100
Percentage Increase in salary
Nb
Final value – Initial value x 2 = N × b% = …(2)
= × 100% 100
Initial value
6000 – 5000 From (1) and (2), we get
= × 100% = 20%.
5000 x + x2
N = 1 × 100
Illustration 10. a+b
The price of potato increases from Rs. 10 / kg to Sum of results
So, Original Number = × 100
Rs. 12/kg. Find percentage increase in the price of potato. Sum of percentage

CAT Complete Course | 31

Visit : www.EasyEngineering.net
Visit : www.EasyEngineering.net

Similarly, Illustration 17.


Difference of results The Sum of two numbers is C. Five times of the
Original Number = × 100
Difference of percentages greater number exceeds thrice the smaller one by zC.
Illustration 13. Find by what per cent the greater number is more than the
smaller one ?
If 15% and 25% of a number is 60 and 100 respec-
tively. Find the number N. Solution :
Solution : Let x and y be the required numbers where x > y
If N be the required number According to question—
60 + 100 x+y = C …(1)
N = × 100
15 + 25 5x – 3y = z C …(2)
160 Putting value of y from (1) in equation (2), we get
= × 100 = 400.
40 5x – 3 (C – x) = z C
Illustration 14. or, 5x – 3C + 3x = z C
If the sum of numbers obtained by taking percentage z C + 3C
or, x =
10% and 30% of a certain number is 200. Find the original 8

ww
number.
Solution :
So, y = C–x=C–
z C + 3C
8

w
5C – z C
Using Formula : =
8
Original Number =
.Ea Sum of results
Sum of percentages
200
× 100
Difference = x – y = 
z + 3 5 – z 
 8

8 

=

= syE
(10 + 30)
200
40
× 100

× 100 = 500. % Difference


2z – 2
C =
8
×C

2(z – 1)
Illustration 15.
The difference between the Number obtained by ngi x
× 100% =
8
C

×
8
× 100%
increasing a certain number by 3% and that obtained by
diminish it by 7% is 81. Find the original number. nee 2(z – 1)
=
(z + 3)
C (z + 3)

× 100%
Solution :
Using formula :
rin =
z–1
z+3
× 200%
Original number =
Difference of result values
Difference of per cents
81
× 100
So, greater number x is
g.n
z–1
z+3
× 200% more than the

Illustration 16.
=

=
[3 – (– 7)]
81
10
× 100

× 100 = 810.
smaller number y.

e
[D] Problems on expenditure —
(I) When expenditure is not constant t
If a be the percentage change in the price of com-
x% of a Number a is equal to y % of a Number b. modity and b be the percentage change in the con-
Find what per cent of a is b ? sumption of the commodity.
Solution : Then net % change in expenditure is given by
x y
×a = ×b  ab 
100 100  a + b + 100  %
=
a y  
⇒ =
b x (II)When the expenditure does not change
x
So, b = a (1) If price increases by a% consumption must
y
 a 
x  decrease by  × 100  %.
b = y a × 100 %  100 + a 
 
(2) If consumption increases by a%, price must
x   a 
Hence, b =  × 100  a%. decrease by  × 100 %.
 y   100 + a 

32 | CAT Complete Course

Visit : www.EasyEngineering.net
Visit : www.EasyEngineering.net

Illustration 18. Total revenue expensed = Rs. A. B


The price of petrol is decreased by 5% and its Now, New price = A – X% of A
consumption increases by 12%. Find the net percentage AX (100 – X)
change in expenditure and ratio of initial and final = A– =A
100 100
expenditure. New consumption = B + Y% of B
Solution : BY (100 + Y)
= B+ =B
By using formula : 100 100
Initial expenditure (100) 2 AB
= New revenue expenses = × (100 – X)
Final expenditure (100 + a)(100 + b) 100 × 100
Initial expenditure 100 × 100 (100 + Y)
=
Final expenditure (100 + 5)(100 + 12) Change in revenue expenses
100 × 100 125
= = AB (100 – X)(100 + Y)
105 × 112 147 = – AB
100 × 100
Net percentage change in expenditure Clearly % effect on revenue
 ab 

ww
XY
=  a + b + 100 % = Y–X–
  100
 [F] Examination problems—
× 12
 – 5 + 12 – 5100
w =

.Ea
=

 60 
 7 – 100  % = 32
  5
%

%
(I) If passing marks in an examination is X% and a
candidate who secures Y marks fails by Z marks, then the
maximum marks,
100(Y + Z)
Illustration 19.
syE
The price of coal gas is increased by 25%. Find how
M =
X
(II) A candidate scoring X% in an examination fails
by ‘a’ marks while another candidate who scores Y%
much per cent its consumption must be decreased if
expenditure remains constant ?
Solution : ngi
marks gets ‘b’ marks more than the minimum required
passing marks. Then maximum marks for that examina-

By using Formula
 a  nee
tion is

M =
100(a + b)
Y–X
Decrease in consumption =  100 + a × 100  %
 
rin
Let the maximum marks for the examination be M
thus, marks obtained by the first candidate = M of x % and
 25
= 
 100 + 25

× 100  %

g.n
marks obtained by second candidate = M of y %
Now, Passing marks for both the candidate is equal so

[E] Problems on revenue—


 25
= 
 125

× 100  % = 20%
 Mx
100
+a =
My
100
–b
e
M of x% + a = M of y % – b

t
(A) If price of a commodity is decreased by X% and 100(a + b)
M =
its consumption is increased by Y%. y–x
(B) Or if the price of a commodity is increased by (III) In a class test x % failed in English, y % failed in
X% and its consumption is decreased by Y%, then effect Hindi. If z% students failed in both English and Hindi,
on revenue. the percentage of students who passed in both the subject
is given by [100 – (x + y – z)]%.
= Increased % – Decreased% value
Proof : % of student who failed in English only =
(Increase % value)(Decrease % value)
– (x – z )%
100
% of student who failed in Hindi only = (y – z)%
XY
Clearly Y – X – …(A) % of students who failed in either subject = x% +
100
(x – z)%+ (y – z)% = (x + y – z)%
XY
X–Y– …(B) So, % of students who passed in both subjects
100
Proof : Let the price of the commodity be Rs. A/unit = 100% – (x + y – z)%
and consumption be B units. = [100 – (x + y – z)]%

CAT Complete Course | 33

Visit : www.EasyEngineering.net
Visit : www.EasyEngineering.net

[G] Formulae for population— Solution :


If the population of a town is p and it increases at the By using formula :
rate of r% per annum, then Population after 3 years
(i) Population after n years  15    40 
 = 25,000  1 +   1 – 25   1 + 100 
r 
n

= p 1 +   100   100   
 100  115 75 140
= 25,000 × × ×
(ii) Population n years ago 100 100 100
p 250 × 23 × 21 120750
= = =
 r 
n 4 4
 1 + 100 
  = 30,187·5 = 30,187.
Illustration 22.
(iii) If r changes every year
Such as r1 in first year The tax on a commodity is diminished by 2.5% and
its consumption increases by 5%. Find the effect on revenue.
r2 in second year and so on.
Solution :

ww
Population after n years

= P 1 +

r1  
 
100  
r 
1+ 2 
100 
tion.
Let A be tax on a commodity and B be the consump-

Now, new revenue = tax × consumption

w .Ea


r3 
 1 + 100
 rn 
 ……  1 + 100
 


=
 2·5  

97·5 105
 
5 
 1 – 100  A.  1 + 100 B

syE
Illustration 20. = × AB
100 100
The population of Delhi is 40,00,000 it increases at
the rate of 10% per annum. What was its population 2 Effect on revenue
years ago and also what will be its population after 2
years ?
Solution : ngi = x+y+
xy
100 × 100
5 × 2·5
Using formula :
P nee = – 2·5 + 5 –

= 2·5 –
5
100 × 100

= 2·5 – 0·00125 = 2·5

rin
Population n years ago = 4000
 r 
n

 1 + 100  Illustration 23.


 
Population 2 years ago =
40‚00‚000
 10 
2
g.n
Gita scores 30% and fails by 20 marks, while Sita
who scores 60% marks gets 40 marks more than the
minimum required marks for the examination. Find the

=
 1 + 100

40‚00‚000
11 × 11


× 100
maximum marks for the examination.
Solution :
By using formula :
e t
= 3305785 100(20 + 40)
Maximum marks =
(60 – 30)
 r 
n

Population after n years = P  1 +  100 × 60


 100  =
30
= 200.

 10 
2
Illustration 24.
= 40,00,000  1 + 
 100  In an examination 75% students failed in Economics,
121 55% failed in Maths and 35% failed in both the subjects
= 40,00,000 × and 500 passed in both the subjects. Find the total number
100
of students.
= 48,40,000.
Solution :
Illustration 21.
By using formula :
The population of a city is 25,000. If increased by
15% during first year. During second year it decreased by % of students who passed in both subjects
25% and increased by 40% during the third year. What is = [100 – (x + y – z)] %
the population after 3 years ? = [100 – ( 75 + 55 – 35)]%

34 | CAT Complete Course

Visit : www.EasyEngineering.net
Visit : www.EasyEngineering.net

= (100 – 95 )% Illustration 27.


= 5% The maximum marks in Civics is 120. A candidates
Since, 5 % students = 500 students scores 60 marks but fails by 12 marks. What is the
⇒ 100% (5 × 20) students percentage pass marks ?
= 500 × 20 Solution :
= 10,000 students. Since candidates get total (60 + 12) marks
Illustration 25. ∴ Candidates get 72 marks out of 120 marks
There are 500 students in an examination 85% i.e., In 120 marks, candidates get 72 marks
students passed in Geography and some of the students 72
passed in Civics while 65% students passed in both the ∴ In 100 marks, candidates get × 100%
120
subjects. If 300 students failed in both the subjects. Find
the % of students who passed in Civics. Hence, percentage pass marks = 60%.
Solution : Illustration 28.
Let the required % of students who passed in Civics Gita and Reeta appears for an examination. Gita
scores 40% and fails by 24 marks. While Reeta scores

ww
is x.
Now, by using Set Theory formula
n (A ∪ B) = n (A) + n (B) – n (A ∩ B)
45% which is 16 marks more than the pass marks. What
are the maximum marks ?

w
No. of students passed in one or both the subjects

.Ea= x % + 85% – 65%


= (x + 20)%
Solution :
Let the maximum marks = M

Gita gets pass marks =


40
100
M + 24

syE
No. of students passed one or both the subjects
= (x + 20) %
While Reeta gets pass marks =
45
100
M – 16

ngi
Now, Number of students failed Since, both are equal,
= [100 – (x + 20)]% 40 45
So, M + 24 = M – 16
100 100

nee
= (80 – x)%
Since, in 100 students, (80 – x) students is failed 5
or 40 = M
80 – x 100
∴ In 500 students, × 500 students is failed

According to questions—
100 ∴
Illustration 29. rinM = 800.

80 – x
100
× 500 = 300
g.n
In a school library, 25% books are in Hindi, 80% of
the remaining are in English and the remaining 9,000 are
∴ 80 – x = 60; x = 80 – 60 = 20
Required % of students who passed in Civics = 20.
Illustration 26.
in English ?
Solution :
e
in various languages. What are the total number of books

t
In an examination a candidates scores 35% but fails Let the total books in Library = 100
by 45 marks. If the passing marks is 65%. What is the
maximum marks ? Now, books in Hindi = 25
Solution : 80
Books in English = 75 × = 60
100
Let the maximum marks = 100
Remaining books = 100 – (25 + 60) = 15
Secured marks = 35
Passing marks = 65 When remaining books are 15, then total books
= 100
Difference = 65 – 35 = 30
∴ If remaining books are 9,000, then total books
When he fails by 30 marks maximum marks = 100
100
∴ He fails by 1 marks, maximum marks =
100 = × 9,000 = 60,000
30 15
∴ He fails by 45 marks maximum marks Hence, the books in English are
100 60
= × 45 = 150. = × 60,000 = 36,000
30 100

CAT Complete Course | 35

Visit : www.EasyEngineering.net
Visit : www.EasyEngineering.net

Illustration 30. decreased by 5% during the second year and again


An Army lost 25% of its men in war, 10% of the increased by 15%during the third year. What is its
remaining due to diseases and 5% of the rest declared war population now ?
disabled. Thus, the strength was reduced to 6,41,250 11. In a factory, the present price of a machine is Rs.
active men. Find the original strength. 75,000. What was its price 2 years ago and will be its
Solution : price 2 year hence. If annual rate of depreciation of
the machine is 20%.
Let the original strength = 100
12. In an election be two candidates Ram and Rahim,
Due to war lost men = 25 Ram got 40% of the votes polled and is defeated by
10 1600 votes. Find the number of votes polled for
Due to diseases lost men = 75 × = 7·5
100 Rahim.
5 13. An ore contains 35% of mass as impurity, whole the
Disabled men = (75 – 7·5) ×
100 metal extracted from this ore contains 5% impurity.
= 3·375 How much metal will 240 tonnes of the ore yield ?
Reduced strength = 100 – (25 + 7·5 + 3·375) 14. In April, a cricket team that played 70 games had
won 40% of its games. After a phenomenal arising

ww
= 64·125
streak this team raised its average to 60%. How
When 64·125 is active then original is 100. many games the team have won in a row to attain
6,41,250 is active men, then original strength = this average ?

w
10,00,000.

1 .Ea
Exercise A
1. If 12 % and 5% of a number is 550 and 220. Find
15. Sheela’s income increases by Rs. 2,000 but the rate
of tax being reduced from 15% to 12%. She pays the
same amount of tax as before. What is her increased
income, if 10%of her income is exempted from tax
2
the number.
syE
2. If the sum of numbers obtained by taking percentage
16.
in both the cases ?
Two numbers are 20% and 25% less than a third
number. How much % is the second number less
of 25% and 10% of a certain number is 350. Find the
certain number ?
ngi 17.
than the first ?
A reduction of Rs. 2/kg enables a man to purchases 8

nee
3. The difference between the numbers obtained by kg. more sugar for Rs. 32. Find the original price of
increasing a certain number by 2% and that obtained rice.
by diminishing it by 1·5% is 350. Find the original
18. A reduction of 60% in the price of chocolates enables
number ?
4. P% of a number C is equal to Q% of a number D.
rin
a person to buy 10 kg. more for Rs. 240. Find the
original price of chocolates per kg.

g.n
Find the per cent of C relative to D ?
19. A two digital number obtained by after interchang-
5. The sum of two numbers is A. Seven times of greater ing the initial number increases by 9. What is percent-
number exceeds five times of the smaller one by kA.
Find what per cent the greater number is more than
the smaller one ?
1
6. A shopkeeper marks prices at 12 % higher than the
2
20.
e
age increase in number, if the sum of numbers is 3 ?
‘A’ spends 75% of his income and saves the rest.
When the cost of living increased, his expenses
1
increased by 25 % and his income also increased by
2
t
original price. Due to increase inflation he further 1
1 12 %. What percentage of his income does he save
increases the price by 6 %. How much % profit will 2
2 now ?
he get ? 21. If the length of a rectangle is increased by 60% and
7. The price of rice is increased by 40%. Find how the breadth is decreased by 40%, then find the %
much per cent its consumption must be decreased if change in area of the rectangle.
expenditure remains constant ? 22. Nitin’s contributions to charity, religious donation
8. The price of wheat decreases by 24·5 %. Find by and community welfare are in the ratios of 10 : 20 :
how much per cent its consumption must be increased 40 respectively. Express the contribution in percen-
if expenditure remains constant ? tage terms.
9. The price of kerosene decreases by 25%. Find how 50 2
23. Which is greatest in % 0·3, ?
much per cent must its consumption be increased if 2 15
expenditure remains constant ? 24. Gita’s salary is 20% less than Sita’s salary but 30%
10. The population of Bombay was 15,00,000, 3 years more than Rita’s salary. If Rita’s salary is Rs. 75 less
ago. If population increased by 10% during first year, than Sita’s salary, find the salary of each.

36 | CAT Complete Course

Visit : www.EasyEngineering.net
Visit : www.EasyEngineering.net

25. Ramesh gets a lumpsum amount of retirement. First of 25 (Paise) coins and 24% of 50 (Paise) coins are
he spends 70% to buy a house 33% of the remained, removed. The percentage of money removed from
he spends on his daughter’s marriage from the the bag is nearly ?
balance he invest 60% in a business and finally he is 37. A reduction of 20% in the price of rice enables a
left with Rs. 2,25,500. How much did he get on person to buy 10 kg more for Rs. 100. What is the
retirement ? reduced price / kg.
26. The price of a saree is reduced by successive 38. A reduction of 30% in the price of wheat enables a
discounts of 10% and 15%. Find the net percentage person to buys 3 kg. more for Rs. 90. Find the
decrease in price. original price of wheat / kg. ?
27. The price of a Maruti car is first increased by 12% 39. A class has girls and boys in the ratio 2 : 3. Among
and later on reduced by 10%. If the original price the girls, the ratio of Mathematics to Physics students
was Rs. 4,00,000. What is the final price and the % is 4 : 5. If the ratio of Mathematics and Physics
change in the price ? students in the entire class is 1 : 2. What percentage
28. In an election between two candidates 60% of the of class comprises girls studying Mathematics ?
votes cast their votes, out of which 5% of the votes 40. If X 2 % of Y = Y2 % of Z. And Z 4 % of X = Y%
were declared invalid. A candidate got 17,100 votes of Y. Find the relation between X and Y.

ww
which were75% of the total valid votes. Find the
total number of votes enrolled in that election.
29. Roma’s Mathematics test had 75 problems, i.e. 10
41. One type of liquid contains 30% of water and the
second type of liquid contains 40% of water. A glass

w
Arithmetic, 30 Algebra and 35 Geometry problems.

.Ea
Although she answered 60% of arithmetic, 50% of
the algebra and 40% of the Geometry problems cor-
rectly. She did not pass the test because she got less 42.
is filled with 10 parts of first liquid and 6 parts of
second liquid.What is the percentage of water in
the new mixture in the glass ?
Due to an increase of 30% in the price of eggs, 3

syE
than 60% of the problems correct. How many more
questions she would have needed to answer correctly
to score a 60% passing grade ?
eggs less are available for Rs. 7·80. What is the
present rate of eggs per dozen ?
Exercise B
30. If 80% of (x – y) = 50% (x + y), then what per cent
of x is y ?
ngi 1. A motorist decreases his distance covered annually
(in km) by 10% when the price of petrol is increased
31. A salesman’s commission is 5% on all sales upto
Rs. 10,000 and 4% on all sales exceeding this. He
remits Rs. 31,100 to his parent company after deduct- nee by 3%. Find the percentage change in petrol bill.
2. The rate of increase of the price of rice is observed to

rin
be 3 per cent more than the inflation rate expressed
ing his commission. Find the total sales of salesman.
in percentage. The price of rice on January 1, 2008 is
32. Radha spends 80% of her income. After a increment, Rs. 25/kg. The inflation rates of years 2008 and
her income is increased by 40% and she also
increased her expenditure by 35%. Find the per-
centage increase in savings. g.n
2009 are expected to be 13% each. The expected
price of rice on January 1, 2010 would be.

33. The weight of the container alone is 25% of the


container filled with a certain fluid when some fluid
is removed, the weight of the container and remaining
fluid is 50% of the original total weight. What frac-
e
3. Ganesh can buy 40 oranges or 50 mangoes. He
retains 10% of the amount for bus fares and buys 20
oranges and of the balance. How many mangoes can
he purchase ? t
tional part of the liquid has been removed ? 4. Raju bought 5 erasers, 4 pencils and 7 pens. Sonali
34. In a competitive examination in stage X, 10% can- bought 9 pens, 10 erasers and 8 pencils for an amount
didates got selection from the total appeared candi- which was half more what Raju had paid. What per
dates. Stage Y had an equal number of candidates cent of the total paid by Raju were paid for the pens ?
appeared and 12% candidates got selected with 80 5. In a survey of children education, 90% of those asked
more candidates got selected than X. What was the are in the favour of at least one of the proposal : I, II
number of candidates appeared from each stages ? and III, 48% of those asked favoured proposal I, 36%
35. The price of maruti is Rs. 10,00,000. It was insured favoured proposal II and 16% favoured, proposal III.
to 90% of its price. The maruti was damaged If 5% of those asked favoured all three of the
completely man’s accident and the insurance com- proposals. What percentage of those asked favoured
pany paid 80% of the insurance. What was the more than one of the three proposals ?
difference between the price of maruti and the 6. Jyoti took five papers in an examination where the
amount received ? full marks were for each paper, her marks in these
36. A bag contains 800 coins of 25 (Paise) denomination papers were in proportion of 2 : 3 : 4 : 5 : 6. In all
and 1500 coins of 50 (Paise) denomination. If 12% papers together, the candidates obtained 50% of the

CAT Complete Course | 37

Visit : www.EasyEngineering.net
Visit : www.EasyEngineering.net

total marks then what is the number of papers in 18. Raj Shekhar bought 3 books, 4 pencils and 5
which she got more than 40% marks ? chocolates while Sonali bought 12 books, 16 pencils
7. One bacteria splits into eight bacteria of the next and 16 chocolates for an amount which was thrice
generation. But due to unfavorable conditions only and half more what Raj Shekhar had paid ? What
25% of one generation can produce the next genera- percentage of the total paid by Raj Shekhar was paid
tion. If the fourth generation number is 64 millions. for chocolates ?
What is the number in first generation ? 19. In a survey of political preferences, 80% of those
8. Sixty per cent of the employees of a TCS company asked were in favour of at least one of the proposals
are men and 40% of them are earning more than I, II and III. 50% of those asked favoured proposal I,
Rs. 40,000 per year. If 40% of the company’s 30% favoured proposal II and 20% favoured proposal
employees earn more than Rs. 40,000 / year. What III. If 4% of those asked favoured all three of the
fraction of the women employed by the company proposals, what percentage of those asked favoured
earn Rs. 40,000 / year ? more than one of the three proposals ?
9. In a factory there are four types of machines x, y, z 20. Riya took five proposals in an examination where the
and A which produces 10%, 20%, 75% and 25% of full marks were for each paper her marks in these
the total products respectively. Machines x, y, z and papers were in proportion of 4 : 3 : 5 : 6 : 7. In all

ww
A produces 1%, 4%, 5% and 6% defective products
respectively. What is the percentage of non-defective
products ?
papers together, the candidates obtained 60% of the
total marks. Then what is the number of papers in
which she got more than 50% marks ?
10.

11.
w
Each edge of a cube is increased by 25%, then the

.Ea
percentage increase in surface area of the cube is ?
The boys and girls in a college are in the ratio 4 : 5.
21. Mr. Ivan Royston is working in the Max New York
Life Insurance (MAX). He was hired on the basis of
commission and he get the bonus only on the first
years commission. He got the policies of Rs. 4 lakh

syE
If 40% of boys and 35% of the girls are adults, the
percentage of students who are not adults is ? having maturity period of 10 years. His commission
in the first, second, third, fourth, and for the rest of
12. Given series is 1 + 4 + 9 + 16 + 25 + 36 + ………. .
the years is 25%, 20%, 10%, 8% and 5% respec-

13.
What is the percentage of 10th term relative to 15th
term ?
A driver reduces his distance covered annually (in ngi tively. The bonus is 30% of the commission. If
annual premium is Rs. 40,000, then what is his total

km) by 25% when the price of petrol is increased by


5%. Find the per cent increase/ decrease in his annual nee commission if the completion of the maturity of all
the policies in mandatory ?
22. A person gives 20% to his wife and 15% of the

rin
petrol bill ?
remaining to a hospital (as a donation) again 10% of
14. In an election 8% of the people in the voter’s list did
the remaining to Prime Minister’s Retired Fund. Then

g.n
not participate and 48 votes were declared invalid.
There are only two candidates Shyam and Krishna. he has only Rs. 8,400 with him. What was the initial
Shyam defeated Krishana by 440 votes. It was found sum of money with that person ?

15.
that 60% of the people listed in the voter’s list voted
for Shyam. Find the total number of votes polled.
The rate of increase of the price of coal is observed
to be five per cent more than the inflation rate
e
23. Find the percentage of the quantity 13 + 23 + 33 + 43

t
+ 53 with respect to 13 + 2 3 + 4 3 + 5 3 + 6 3 + 7 3 .
24. If a cone and a cylinder have the same height and
radius. What is the percentage difference in volume
expressed in %. The price of coal, on March 4, 2008 of cone relative to cylinder ?
is Rs. 100 / tonnes. The inflation rates of years 2008
25. A resistor of length l and cross sectional area A. If it
and 2009 are expected to be 10% each. Find the
is cut in four part across the length. Find the %
expected price of coal on March 4, 2009.
change in resistance of the resistor.
16. The number of votes not cast for the Samata Party
increased by 20% in the general election over those 26. A wire of length l and cross-sectional area A. If its
not cast for it in the previous assembly polls and the length increase by 20% and its cross-sectional area
Samata Party lost by a majority thrice as large as that increases by 10%. Find the percentage change in the
by which it had won the assembly polls. If a total resistance of the wire.
3,60,000 people voted each time, how many voted 27. A box of wood of thickness 2 cm. If outer dimensions
for the Samata Party in the previous assembly polls ? of the box 24 × 20 × 16 cm3 . How much per cent of
17. Gita buys 25 red pens or 50 blue pens. She retains air occupied inside the box ?
20% for autorikshaw as fares and buys 10 red pens 28. Convert the following fraction into percentage
and of the balance. She purchases blue pens. What is log1 + log 2 + log 5
·
the number of blue pens she can purchase ? log1 + log2 + log5 + log10

38 | CAT Complete Course

Visit : www.EasyEngineering.net
Visit : www.EasyEngineering.net

29. A triangle ABC is made inside a semicircle such that 5. Let x and y be the required numbers where x > y
BC is its diameter and point A lies on the circum- x+y = A …(1)
ference. If AB = 3, AC = 4. Find the percentage of
7x – 5y = kA …(2)
area not enclosed by the semi-circle.
From (1) and (2)
30. Given series is 5 + 10 + 20 + 40 + 80 + ……….…
What per cent difference between 11th and 10th term ⇒ 7x – 5(A – x) = kA
relative to 10th term ? or, 12x = A(k + 5)
31. If the temperature of iron rod increased from 80°C to k+5
∴ x = ·A
90°C. Find what is percentage change the Fahrenheit, 12
if the relation between Fahrenheit and degree centi- k+5 7–k
and y = A– ·A= ·A
grade is 12 12
C
=
F – 32
· k + 5 7 – k 
100 180 Difference = x – y =  – . A
 12 12 
32. Each edge of a cube is increased by 20%, then the
2k – 2
percentage increase in surface of the cube is ? = ·A
12

ww Answers
Exercise A
% Difference
x
=
 2(k – 1)A.12
 12(k + 5)A × 100 %


w
1. By using formula :

Original number =
.Ea
Sum of results
Sum of per cents
550 + 220
× 100

6.

k – 1
= 

% profit =  a + b +
k + 5

ab 
%

× 200 %

=
 1
syE 
 122 + 5 
 
× 100 


100 
 1   1 
 12   6 
 1 1  2   2 
=
770 × 2
35
× 100 = 22 × 200
ngi =  12 + 6 +


2 2
25 × 13
100
 %

2. By using formula :
= 4,400.

nee =  19 +

= 19·8%.
400 
% = 317
16
%

rin
Sum of results
Certain number = × 100
Sum of per cents 7. Decrease in consumption

g.n
350 a
= × 100 = × 100%
25 + 10 100 + a
350 40 40

3. By using formula :

Original number =
=
35
× 100 = 1,000.

Difference of results
× 100
=

=
100 + 40
200
7
e
× 100% =

% = 28·57%.

8. Increase in consumption
140
× 100%

t
Difference in per cents
350 350  a 
= × 100 = × 100 =  100 – a × 100  %
2 + 1·5 3·5  
= 1000  24·5 
=  × 100 %
4. According to question,  100 – 24·5 

P
= D×
Q  24·5 
100 100 =  × 100 %
 75·5 
C Q
⇒ =  4900 
D P =   % = 32·45%.
 151 
Q
⇒ C = ×D 9. Increase in consumption
P
Q   a 
⇒ C =  P × 100  % of D. =  100 – a × 100 %
   

CAT Complete Course | 39

Visit : www.EasyEngineering.net
Visit : www.EasyEngineering.net

 25  Now, quantity of pure metal


=  100 – 25 × 100 % 
  13 
=  65 – 4  tonnes
 25  1
=  × 100  % = 33 %
 75  3 13 × 9
= tonnes
4
10. Here, rate is different for each year. So, using formula
for different rates. Clearly, 100 tonnes of ore gives
13 × 9
Present population = tonnes pure metal
4
 10   5  13 × 9
= 15,00,000  1 +   1–  ∴ 1 tonne of ore gives = tonnes pure metal
 100   100  4 × 100
 15  ∴ 240 tonnes of ore gives = 70·2 tonnes pure metal.
 1 + 100  14.
  Let x be the number of games won in a row.
11 19 23 Now, according to question,
= 15,00,000 × × ×
10 20 20 (70 of 40%) + x 60 3
= =

ww = 375 × 209 × 23 = 18,02,625.


11. Price of machine (2 years ago) or,
70 + x
28 + x
70 + x
=
100 5
3
5

w
P 75‚000
= = or, 2x = 210 – 140 = 70
 r 
2
 20  2
 1 – 100

75‚000 .Ea
  1 – 100
 


75‚000 × 25
15.
x = 35.
Since, same percentage of his income is exempted
=
4×4
syE
× 25 =

= Rs.1,17,187·50.
16
from tax in the both cases, this data is not be
considered.
Initial amount of tax = Final amount of tax
Price of machine (after 2 years)

= P 1 –


r 

100 
2
 20 
= 75,000  1 – 
 100 
2
ngi or,
(a – 2000) × 15% = a.12 %

(a – 2000)
15
=
12a

= 75,000 ×
16
25
= 48,000. nee or,

100 100
3a = 2000 × 15
a = 10,000
12. Let Rahim got x votes polled.
Number of votes polled for Ram rin
Hence, Increased income = Rs. 10,000.

= (x – 1600)
Total votes polled = x + x – 1600 = (2x – 1600)
16.

g.n
Let the third number = 100
First number = 100 – 20% of 100

e
20
= 100 – × 100 = 80
According to Questions,


x – 1600 = 40% (2x – 1600)

x – 1600 =
40
100
(2x – 1600)

= 100 –
100
Second number = 100 – 25% × 100
25
100
× 100 = 75
t
x = 8000 – 3200 = 4800. Now, second number is lessen than first number
13. From question it is clear that 100 tonnes of ore gives second number is 5 less than first number.
65 tonnes of metal which contains 5% impurity. 5 25
% less = × 100 = = 6·25%.
Since, 100 tonnes of metal contain 80 4
= 5 tonnes impurity 17. Here, expenditure is fixed Rs. 32.
∴ 1 tonne of metal contain Since, rate decrease while amount of rice increases
5 with the same expenditure.
= tonnes impurity
100 Let the original price = Rs. x / kg.
∴ 65 tonne of metal contain Expenditure × rate change
x (x + rate change) =
5 Change in available quantity
= × 65 tonnes
100 32 × 2
or, x (x – 2) =
13 8
= tonnes impurity
4 or, x 2 – 2x – 8 = 0

40 | CAT Complete Course

Visit : www.EasyEngineering.net
Visit : www.EasyEngineering.net

or, (x – 4)(x + 2) = 0 % saving of income = New saving × 100


∴ x = 4, x ≠ – 2 3
x
16
Original price of rice = Rs. 4 / kg. = × 100
9x
18. Here, expenditure remains constant. 8
Rate change = (60%) of original price 100
60 3x = = 16·66%.
= × x = = 0·6x 6
100 5 xy
21. % change in result = x + y +
where x is the original price 100
Expenditure × rate change Here, x = 60 and y = – 40
x (x + rate change) =
Change in available quantity 60 × 40
240 × 0·6x = 60 – 40 – = 20 – 24 = – 4
or, x (x – 0·6x) = 100
10
Hence, area of the rectangle decrease by 4%.
or, 0·4x = 24 × 0·6;
22. Given ratios are 10 : 20 : 40
x = 36.
Their sum = 10 + 20 + 40 = 70

ww

19. Let
Original price = Rs. 36 / kg.
units digit = x ∴ Charity =
10
70
× 100 =
100
7
2
= 14 %
7

w
10th digit = y
20 200 4
According to question, Religious donations = × 100 = = 28 %

.Ea
70 7 7
Number = 10y + x Community welfare fund
x+y = 3 …(1)

syE
40 400 1
= × 100 = = 57 %
After interchanging digits 70 7 7
new number = 10x + y 50 50
23. I : % = = 0·25
10x + y – (10y + x) = 9
or, 9x – 9y = 9 ngi II :
2 2 × 100
0·3 = 0·3
or, x–y = 1
By equations (1) and (2)
…(2)
neeIII :
2
15
= 0·13

Clearly 0.3 is the greatest number among above.


2x = 4, y = 1; x = 2
Original number = 12
24.
rin
Let Gita’s salary = Rs. G
Since, Gita’s salary is 20% less than Sita’s salary
New number = 21

% increase in number =
Change
× 100 g.n
⇒ G = 0·8 S ; Here S is Sita’s salary
and, Gita’s salary 30% more than Rita’s salary

=
Original number
21 – 12

9
12
× 100 e
⇒ G = 1.3 R; Here R is Rita’s salary
When Gita’s salary is Rs. x, Sita’s salary

= Rs.
8
·x
t
= × 100 = 75%. 10
12
Clearly 0·8 S = 1·3 R
20. Let the income of A is Rs. x.
13
Income Expenditure Saving Sita’s salary = Rita’s salary
8
Before 75 3 1
·x= x
increase x x Difference between Gita’s salary and Sita’s salary
100 4 4
S – R = 75
After  25  3  50 
x 1 +  x 1 +  Rs. y  13 
increase  200  4  200  ⇒  8 – 1  R = Rs. 75
 
9 3 5
x. x· Rita’s salary = Rs. 120
8 4 4
9x 15x ⇒ G = 1·3 R = 1·3 × 20 = Rs 156 is Gita’s salary
Now, New saving = –
8 16 G 156
⇒ S = =
18x – 15x 3 0·8 0·8
= = x
16 16 = Rs. 195 is Sita’s salary.

CAT Complete Course | 41

Visit : www.EasyEngineering.net
Visit : www.EasyEngineering.net

25. Suppose that Ramesh gets a lumpsum amount 29. Numbered questions attempted correctly
= Rs. 100 = 60% of 10 + 50% of 30 + 40% of 35
Now, amount spends in house = 70 60 50 40
= × 10 + × 30 + × 35
Daughter’s marriage (33% of remain amount) 100 100 100
33 = 6 + 15 + 14
= 30 × = 10 = 21 + 14 = 35
100
Amount left After daughter’s marriage Questions to be answered correctly for 60% grade
= 30 – 10 = 20 = 60 % of 75
60
In business, 60% of 20, amount = × 75 = 45
100
60
= 20 × = 12 Required number of questions = 45 – 35 = 10.
100
30. Given 80% (x – y) = 50% (x + y)
Finally left amount = 20 – 12 = 8
4 1
When Rs. 8 left then lumpsum = Rs. 100 Or, (x – y) = (x + y)
5 2
∴ 2,25,500 left when lumpsum Or, 8x – 8y = 5x + 5y

ww =
100
8
× 2,25,500

= Rs. 28,18,750.
Or, 3x = 13y

Required percentage =
y 
 x × 100  %

26. wNet % change =




Here, x = – 10, y = – 15 .Ea
xy 
 x + y + 100 %


3


=  × 100  % = 23·07%.
 13 

 syE
Net % change =  – 10 – 15 +
10 × 15
100 
%
31. Let his total sales be Rs. x.
Now, Total sales – commission = Rs. 31,100
Or, x – [5% of 10,000 + 4% of (x – 10,000)] = 31,100

27. Using formula :


= (– 25 + 1·5)% = – 23·5%

ngi 
Or, x –  500 +

4
100

(x – 10‚000)  = 31,100

nee
100 + a 100 + b
Final Price = Initial Price × ×
100 100 x
Or, x– = 31,200
Here, a = 12 , b = – 10 25

Final Price = 4,00,000 ×


100 + 12 100 – 10
100
×
100
rin
x =
31‚200 × 25
24

Net % change =
= 40 × 112 × 90 = Rs. 4,03,200
 xy 
 x + y + 100 %
32. Let Radha’s income = Rs. 100
g.n
x = Rs. 32,500.



=  12 – 10 –


12 × 10
100 
%
Expenditure = Rs. 80
Savings = Rs. 20
New income = Rs. 140
135
e t
 6 4 New expenditure = × 80 = Rs. 108
=  5 –  % = % = 0·80%. 100
 5 5
Now, savings = 140 – 108 = Rs. 32
28. Let the total number of votes enrolled by x, then 12
number of votes cast = 60 % of x. % increase in savings = × 100 = 60%.
20
95 33. Let the weight of the container = x
Valid votes = of (60% of x)
100
The weight of the fluid = y
= 95% of 0·6x = 0·95 × 0·6x = 0·57x
According to question,
Now, according to question :
x + y = 100%
75% of 0·57x = 17100
Hence, y = 75% = 3x
⇒ 0·75 × 0·57x = 17100
y = 3x …(1)
 17100 × 100 × 100 
⇒ x =   Given, Weight of container + Remaining fluid
 75 × 57  50
= (x + y)
x = 40,000. 100

42 | CAT Complete Course

Visit : www.EasyEngineering.net
Visit : www.EasyEngineering.net

1 37. Given amount = Rs. 100


x + Remaining of Fluid = (x + 3x)
2 20
Reduced amount = 100 × = Rs. 20
Remaining fluid = 2x – x = x 100
Removed fluid = 3x – x = 2x Since Rs. 20 the person buys 10 kg. rice
Fractional part of the liquid removed Price of rice = Rs. 2 / kg.
2x 2 This is the reduced price of rice = Rs. 2 / kg.
= = ·
3x 3 38. Amount given = Rs. 90
34. Since, the same number of candidates appeared from 30
each stages. Reduction in amount = 90 × = Rs. 27
100
So, Let we suppose that 100 candidates appeared From question :
from each stage.
Then Rs. 27, Here 3 kg. wheat
Now, For stage X,
Price of wheat = Rs. 9 / kg.
10
Selected candidates = × 100 = 10 When reduced price is 70, then original price = 100
100
When reduced price is 9, then the original price
For stage Y,

ww Selected candidates =
12
100
× 100 = 12 =
100
70
× 9 = 12·85.

w
Clearly, 12 – 10 = 2 39. Let the girls and boys in the class are 2x and 3x
respectively.
When 2 candidates are selected more from stage Y,

.Ea
then there are 100 candidates in each stage
100
Girls students of Mathematics
4
= × 2x =
8x

syE
When 1 candidates … = 9 9
2
100 Percentage of girls studying Mathematics
When 80 candidates … = × 80
2 8x

35.
= 4,000 candidates in each stage.
Price of maruti = Rs. 10,00,000 ngi =

=
9 × 2x
400
× 100

= 44·44%.
Amount insured = 10,00,000 ×

= Rs. 9,00,000
90
100
nee
40. Given
9
X2 % of Y = Y2 % of Z
X2 Y2
Received amount = 9,00,000 ×
80
100
100
×Y =

rin
100
×Z

X2 = Y. Z

g.n
Or, …(1)
= Rs. 7,20,000
and Z 4% of X = Y% of Y
Now, Required difference
Y2

e
Z4
= Price of maruti – Received amount ×X =
= 10,00,000 – 7,20,000
= Rs. 2,80,000.
 25
 800 × 100 50 

100 100
X × Z4 = Y2
Putting the value of Z in equation (2), we get
t
…(2)

36. Total money = + 1500 ×  X2  4


 100  X ×   = Y2
Y 
= Rs. 950
12 XX8
25 paise coins removed = × 800 = Rs. 96 Or, = Y2
100 Y4
24 X9 = Y6 .
50 paise coins removed = × 1500 = Rs. 360
100 41. Let the amount of first type liquid = X
 25 50 
Money removed =  96 × + 360 ×  The amount of second type of liquid = Y
 100 100 
3
Now, amount of water in first type of liquid = X
= Rs. 204 10
204 4
Required percentage = × 100% Amount of water in second type of liquid = Y
950 10
408 3
= % ≅ 21·5%. Since, X parts of liquid contains = X parts of water
19 10

CAT Complete Course | 43

Visit : www.EasyEngineering.net
Visit : www.EasyEngineering.net

3 3. Let us suppose Ganesh has Rs. 100 with him.


10 parts of liquid contains = × 10 = 3
10 Clearly , price of one orange = Rs. 2·5
Similarly, Price of one mangoe = Rs. 2
6 parts of 2nd type liquid contains Since, Ganesh retains 10% amount for his bus fares.
4×6
= part of water So, left amount = Rs. 90
10
He buys 20 organes, hence, cost of 20 oranges = 20
= 2·4 parts × 2·5 = Rs. 50
Total amount of water = 3 + 2·4 = 5·4 parts Remaining amount = Rs. (90 – 50) = Rs. 40
Total amount of mixture = 10 + 6 = 16 Since, price of one mango = Rs. 2
5·4
% Change = × 100 = 33·75%. Number of mangoes = 20.
16
4. Let the Raju’s amount = Rs. 100; cost of one pen =
42. Price of eggs increases by the amount
Rs. a
30
= 7·8 × = 2·34 cost of one pencil = Rs. b and cost of one eraser =
100
Rs. c

ww
Hence, price of 3 eggs is Rs. 2·34

Price of 1 egg is = Rs.


2·34
3
= 78 paise.
According to question,
7a + 4b + 5c = 100 …(1)

w Exercise B

.Ea
1. Since, petrol bill is directly proportional to the
distance covered and price of petrol.
9a + 8b + 10c = 150 …(2)
Multiplying equation (1) by 2 and subtracting equa-
tion (2), we get
5a = 50
Let petrol bill = a
Distance covered = b
Price of petrol = c
syE ⇒
Total amount paid by Raju
a = 10

Now, a × b and a × c
a = k.b.c ngi = 7a = Rs. 7 × 10 = Rs. 70

∴ Required percentage =
70
× 100 = 70%.
(where k = constant, suppose k = 1)
a = b.c nee
5.
100

New price of petrol =


103
100
·c
X rin c Z I (48)

and New distance covered =

New bill
90
100
·b

103 × c × 90 × b 927 a g.n


W
b

e
= = III (16)
100 × 100 × c × b 1000

t
Original bill
% change in petrol bill II (36)
Y
New bill – Original bill
= × 100
Original bill
According to question,
 927 
=  – 1  × 100 X + Y + Z + a + b + c + w = 90
 1000 
⇒ 48 – (b + c + 2) + 36 – (a + b + 2) + 16 – (c + a
73 + 2) + (a + b + c + 2) = 90
= – = – 7·3%.
10
⇒ 96 – 2 (a + b + c) – 6 + (a + b + c) + 2 = 90
2. Increase in price of rice = (13 + 3)% = 16%
⇒ 100 – (a + b + c ) – 4 = 90
∴ Price of rice on January 1, 2010
⇒ 96 – (a + b + c) = 90
 16 
2

= 25  1 +  ⇒ (a + b + c) = 6
 100  ∴ Percentage of those asked favoured more than
29 × 29 one proposal = 6 + 2 = 8.
= 25 ×
25 × 25 6. Let the marks scored in five subjects be 2x, 3x, 4x,
841 5x, 6x.
= = Rs. 33·64.
25 Total marks in all the five subjects = 20x

44 | CAT Complete Course

Visit : www.EasyEngineering.net
Visit : www.EasyEngineering.net

Maximum marks of the five subject given 50% = 20x 95


Machine z = (0·45 a) ×
100% = 40x 100
94
Maximum marks in each subject Machine A = (0·25 a) ×
100
1
= × 40x = 8x Now, Total non-defective products
5
Hence, percentage in each subject a
= (9·90 + 19·20 + 42·75) + 23·50)
100
2.x
× 100 = 25% a
8.x = × 95·35
100
3.x
× 100 = 37·5% % non-defective products = 95·35%.
8.x
4.x 10. Let side of the cube = a m.
× 100 = 50%
8.x Final / new side of the cube
5.x 25 5
× 100 = 62·5% = a+a× = a
8.x 100 4
6.x Initial surface area = 6a2
× 100 = 75%

ww 8.x
She got more than 40% in three papers.
 5a  2
Final/new surface area = 6  
4 

w
7. Let the number of bacteria in the first generation be x.
∴ Number of bacteria in the second, third and fourth
generation would be
.Ea
 x   2x   4x   8x 
Change in surface area = 6a2 
 25 – 16 
 16
Now, percentage change in surface area

9
 = 6a2 . 16

8
       
syE
 4  , 8  4  , 8  4  , 8  4  = 2x, 4x, 8x, 16x

The above numbers are in G.P. with common ratio 2.


=
Change
6a2
9
× 100

Now, fourth term of these G.P. = 2x. 23 = x. 24


According to question x. 24 = 64 ngi =
6a2
6a2
16
× 100

⇒ x = 4 millions.
8. Let number of men employees = 60 nee =
9
16
× 100 =
225
4
= 56·25%.

11. Let the number of boys = 4x and the number of girls


Number of women employees = 40
Number of men earning more than Rs. 40,000 / year rin
= 5x. Total students = 9x
40 8

= 60 ×
40
100
= 24
g.n
Adults boys = 4x.

Adults girls = 5x.


100 5
35 7
= x

= x
Total number of employees earning more than
Rs. 40,000 / year = 40
Number of women earning more than Rs. 40,000
e 8
Total adult students = x + x

=
5
100 4
7
4
32x + 35x 67
= x
t
= 40 – 24 = 16 20 20
Now, fraction of the women earning Rs. 40,000 or Students who are not adults
40 – 16 24 3 67 113
less = = = = 9x – x = x
40 40 5 20 20
9. Let the total products = a Percentage of students who are not adults
Now, machine x produces products = a × 10% = 0·1a 113x
= × 100 = 62·78%.
20 × 9x
and it produces 1% defective products.
12. Given series is
It means machine x produces 99% non-defective
products. 1 + 4 + 9 + 16 + 25 + 36 + ……
So, amount of products produced by machines = 1 + 22 + 3 2 + 4 2 + 5 2 + 6 2 + ……
99 10th term = 102
x = (0·1a) ×
100 15th term = 152
96 10th term 100
Similarly, machiney = (0·2a) × Now, × 100 = × 100 = 44·44%.
100 15th term 225

CAT Complete Course | 45

Visit : www.EasyEngineering.net
Visit : www.EasyEngineering.net

13. The petrol bill is directly proportional to the distance And votes polled in favour of the party = (3,60,000
covered and price of petrol. – 1·2a)
Let petrol bill = z and Distance covered = 100 and Majority of votes by which party lost in general
Rice of petrol = 100 election = 1·2a – (3,60,000 – 1·2a)
Original petrol bill z = 100 × 100 = 2·4a – 3,60,000
New petrol price = 105 and New distance covered = 75 It is given that ,
New petrol bill = 105 × 75 2.4a – 3,60,000 = 3 (3,60,000 – 2a)
Percentage change in petrol bill 2·4a + 6a = 4 × 3,60,000
New bill – Original bill 4 × 3‚60‚000
= × 100 Or, a =
Original bill 8·4
105 × 75 – 100 × 100 ⇒
= × 100 a = 1,71,428
100 × 100
17. Suppose Gita has Rs. 100 with her.
Percentage change in petrol bill
From question :
 63 
=  80 – 1  × 100% Price of one red pen = Rs. 4

ww = –

17
80

× 100%
and Price of one blue pen = Rs. 2
She retain Rs. 20 for autorikshaw fare

w 85
= – % = – 21·25%.
4

.Ea
14. Let the total number of people in the voter’s list be x.
∴ left amount = Rs. 80
The price of 10 red pens = Rs. 40
Remaining amount = Rs. 40

syE
The number of people who participated in voting = Since, price of one blue pen = Rs. 2
0.92x 40
Number of blue pens = = 20.
Since, 48 votes were declared invalid. 2
So, valid votes polled = (0·92x – 48).
Since, Shyam gets 60% of x. ngi 18. Let the Raj Shekhar’s amount = Rs. 100
If cost of one book = Rs. x
Number of votes gained by Shyam = 0·6x.
Votes obtained by Shyam + Votes obtained by nee Cost of one pencil = Rs. y
Cost of one chocolate = Rs. z

rin
Krishna = (0·92x – 48). According to question,
Or, votes obtained by Krishna = (0·92x – 48 – 0·6x) 3x + 4y + 5z = 100 …(1)
= (0·32x – 48)
Now, According to question; Difference = 440
Or, 0·6x – (0·32x – 48) = 440 g.n
12x + 16y + 16z = 300
Multiplying equation (1) by 4 and subtract with (2),
…(2)



0·28x = 440 – 48
x =
392
0·28
= 1400.
we get


e
4z = 50
z = 12·5 t
15. Increase in price of coal Total amount paid by Raj Shekhar
= (10 + 5)% = 15% = 5 × 12·5 = Rs. 62·5
∴ Price of coal on March 4, 2009 Required percentage = 62.5%
 15  19. [50 – (x + z + w)] + [30 – (x + y + w)] + [20 – (x + z + w)]
= 100  1 + 
 100  = 80
115
= 100 × = Rs. 115 / tonnes.
100
16. Let ‘a’ voters voted against the party in the assembly
poll.
Then votes in favour = (3,60,000 – a)
Majority of votes by which party won in previous
poll = (3,60,000 – a) – a = (3,60,000 – 2a)
Now, votes polled against the party in general
election = 1·2a

46 | CAT Complete Course

Visit : www.EasyEngineering.net
Visit : www.EasyEngineering.net

46 – (x + y) + 26 – (x + z) + 16 – (y + z) + (x + y When rest amount is 61.2, then initial amount 100


+ z + 4) ∴ Rest amount is 8,400, then initial amount should
= 80 100 × 8400
be =
Or, x + y + z = 92 – 80 61·2
700000
⇒ x + y + z = 12 = = Rs. 13,725·5.
51
Percentage of those asked favoured more than one
proposal = 12 + 4 = 16.
23. Required percentage
20. Let the marks scored in five subjects be 4x, 3x, 5x,
6x, 7x.  13 + 2 3 + 3 3 + 4 3 + 5 3 
=  13 + 23 + 33 + 43 + 53 + 63 + 73 × 100 %
Total marks in all the five subjects = 25x
Maximum marks of the five subjects given 60% = 5 5 + 1    
2
25x   2    
100
× 25x =
125x =      × 100%
100% =
60 3 7 7 + 1    
2

  2    
ww
Maximum marks in each subject

=
1  125x  25x

5 3  3
= =
25 36
× × 100% = 28·70%.
49 64

w
Hence, percentage in each subject are

=
.Ea
4x
25x
× 3 × 100 = 48%
24. Let r be the radius and height h volume of cone is
given by
1
Vcone = πr2 h

syE
3x 3
= × 3 × 100 = 36%
25x Volume of cylinder Vcy. = πr2 h
5x Vcone 1
× 3 × 100 = 60%

ngi
= Now, =
25x Vcy 3
6x
= × 3 × 100 = 72% Vcy – Vcone
× 100%

nee
25x % change =
Vcy
7x
= × 3 × 100 = 84% 2
25x = × 100%

rin
3
She got more than 50% in three papers.
= 66·6%.
21.
Year Rate of commission Commission in values
1 25% 0·25 × 40,000 = 10,000
25. We know that R α
g.n
l;Rα
1
A

e
l
30% (bonus) 0·3 × 10,000 = 3,000 R = ρ …(1)
2
3
4
20%
10%
8%
0·2 × 40,000 = 8,000
0·1 × 40,000 = 4,000
0·08 × 40,000 = 32,00
In case of Ist R1 = ρ
A
l
A
l R
t
In 2nd case R2 = ρ = 1
4A 4
5-10 5% 6 × 0·05 × 40,000 = 12,000
3R
Total commission = Rs. 40,200. Now, Change = R2 – R1 = – 1
4
22. Let the person has Rs. 100.
R
His wife gets amount = Rs. 20 3 1
4
Remaining amount = Rs. 80 % Change = – × 100% = – 75%
R1
15
Donation to hospital = 80 × = Rs. 12 Clearly after cutting resistance of the resistor decrease
100 by 75%.
Again rest money = 80 – 12 = Rs. 68 26. Initial length = l ; cross-sectional area = A
Prime Minister’s relief fund amount l
10 Resistance is R = ρ
= 68 × = Rs. 6·8 A
100 12
New length = ·l
The rest amount = 68 – 6·8 = Rs. 61·2 10

CAT Complete Course | 47

Visit : www.EasyEngineering.net
Visit : www.EasyEngineering.net

11 30·50
New cross-sectional area = × A; New resistance is = × 100%
10 25 × 3·14
12 3050
l = % = 38·8%.
10 12 l
R1 = =ρ× × 78·5
11 11 A
·A 30. Given series is
10
12 1 = 5 + 10 + 20 + 40 + 80 + ………
Change = R 1 – R = R × –R= ×R
11 11 = 5 + 5.2 + 5·22 + 5·23 + 5·24 + ………
1 a = 5, r = 2
·R
11
% Change = × 100% = 9·09%. So, 11th term = a. r10 = 5·210
R
10th term = a. r9 = 5·29
27. Original length = 24 cm.;
Difference between 11th term and 10th term
New length = (24 – 4) = 20 cm.
Original height = 16 cm.; = 5·2 10 – 5·2 9 = 5·29
5·2 9
New height = (16 – 4 ) = 12 cm. % Difference = × 100% = 100%
5·2 9
Original breadth = 20 cm;

ww New breadth = (20 – 4) = 16 cm.


Original volume = 24 × 20 × 16 cm3
31. We have the following relationship
C
100
=
F – 32
180

w = 7680 cm3
New volume = 20 × 16 × 12 = 3840 cm3

.Ea
Volume of air inside the box
= 7680 – 3840 = 3840 cm3
Now, C1 = 80°C; C2 = 90°

Now,
F 1 – 32
9
=
80
5

% Volume of air =
syE
3840
7680
× 100% = 50%.
log 1 + log 2 + log 5 When C 2 = 90°
F 1 = 16 × 9 + 32;
F 1 = 144 + 32 = 176°

28. Percentage =
log 1 + log 2 + log 5 + log 10
× 100%ngi F 2 – 32
9
=
90
5
=
log 1.2.5
log 1.2.5.10
log 10
× 100%
nee F 2 = 18 × 9 + 32
F 2 = 162 + 32
=
log 102
1
× 100%

= × 100% = 50%.
Change in Fahrenheit
rin
F 2 = 194°

2
29. Since, ∆ABC is formed inside the circle so it is right
g.n
= F2 – F1
= 194° – 176° = 18°
angled triangle.
BC = 
√ AB2 + AC 2= 
1
√
1
32 + 4 2 = 5 cm.

Area of semi-circle = πr2 = × 3·14 ×  


2 2
 5 2
2 
Now, % change =

=
9
e
18°
176°

88
× 100

× 100 =
225
22
t
= 10·2%.

32. Let Initial edge = a m.


25 × 3·14 2
= cm 12
8 Final edge = ×am
1 10
Area of triangle ABC = × 3 × 4 = 6 cm2
2 Initial volume = a3 m 3
Now, Area not enclosed by semi-circle  12  3 1728 3 3
25 × 3·14  10 × a = 1000 × a m
Final Volume =
= –6=
78·50
–6  
8 8
30·50  1728  3
= Change in Volume =  –1×a
8  1000 
% Area not enclosed by semi-circle 728
= × a3
30·50 1000
8 728
= × 100% % Change in volume = × 100% = 72·8%.
25 × 3·14 1000
8 ●●●

48 | CAT Complete Course

Visit : www.EasyEngineering.net
Visit : www.EasyEngineering.net

4 Profit and Loss


Cost Price (C.P.)—The price at which an article is So, Profit = S.P. – C.P.
bought is called the cost price (C.P.) of the article. 16 – 12
Now, Gain % = × 100
Example—If a customer buys any object for Rs. X 12
from the shopkeeper / retailer / wholesaler. 4
= × 100 = 33·3%.
Then this price is known as C.P. (Cost Price). 12
C. P. = Rs. X Illustration 2.
Selling Price (S.P.)—The price at which an article is Kamala buys a dress for Rs. 72 and after six months

ww
sold is called the selling price (S.P.).
Example—If a person sells an article at any cost to
sells it for Rs. 90. Find her profit per cent.
Solution :

w
the other person. Then that price is called Selling Price Given C.P. = Rs. 72
(S.P.). S.P. = Rs. 90

.Ea
Profit or Gain—When S.P. > C.P.
Profit or Gain = S.P. – C.P.
Here,
Profit takes place
S.P. > C.P.

Loss : When
syE
C.P. > S.P.
Loss = C.P. – S.P.
Profit = S.P. – C.P.
= Rs. (90 – 72) = Rs. 18
Some fundamental formula
% profit =
S.P. – C.P.
× 100 ngi % profit =
Profit
C.P.
× 100%

nee
C.P. 18
= × 100% = 25%
C.P. – S.P. 72
% loss = × 100
C.P. Illustration 3.
In case of profit
 100 + % profit  rin
Reena buys a pizza for Rs. 42 and sells it for Rs. 40.
Find loss or profit per cent.
S.P. =

and in case of loss



 100
 × C.P.
 Solution :
Given g.n
C.P. = Rs. 42

S.P. =
 100 – % loss 
 100

If S.P. and profit are given
 × C.P.
 Since,
So,
e
S.P. = Rs. 40
S.P. < C.P.
loss = C.P. – S.P. t
= 42 – 40 = Rs. 2
 
C.P. =  100 +100  × S.P. Loss
 % profit  % loss =
C.P.
× 100
 100  2
and C.P. =   × S.P. % loss = × 100
 100 – % loss  42
Illustration 1. 100
= = 4·76%.
21
Sohan buys a pen for Rs. 12 and sells it for Rs. 16.
Find his gain per cent. Illustration 4.
Solution : Rajeev sells an article at 10% gain for Rs. 340. What
is its cost price ?
Using formula :
S.P. – C.P. Solution :
Gain % = × 100
C.P. By using formula :
Here, C.P. = Rs. 12, S.P. = Rs. 16 S.P. = 340
S.P. > C.P. and % Profit = 10

CAT Complete Course | 49

Visit : www.EasyEngineering.net
Visit : www.EasyEngineering.net

100 Profit = S.P. – C.P.


C.P. = × S.P.
100 + % profit = 115 – 100 = Rs. 15
100
C.P. = × 340 % profit =
Profit
× 100
100 + 10 C.P.
100
= × 340 = Rs. 309. =
15
× 100 = 15%.
110 100
Illustration 5. Illustration 9.
Rakesh buys an article for Rs. 400 and sells it at 15% Radha buys A tables for Rs. a and sells B tables for
profit. Find its S.P. Rs. b. Find net profit / loss per cent.
Solution : Solution :
We have C.P. = Rs. 400 a
C.P. of one table = Rs.
% Profit = 15 A
100 + % profit b
S.P. = × C.P. S.P. of one table = Rs.
100 B
100 + 15 In the above C.P. and S.P. we do not know which is

ww S.P. =

=
115
100
× 400

× 400 = Rs. 460.


greater than other.
So, we have two cases—

w
100
Case I : If S.P. > C.P.
Illustration 6.

.Ea
Akash buys a table for Rs. 600 and sells it at a profit
of Rs. 120. Find per cent gain.
Then, profit = S.P. – C.P.

=
b a

syE
B A
Solution :
Profit
By using formula : % Profit = × 100
C.P.

ngi
Profit/Gain
% Gain = × 100   b a  a 
C.P. =   B – A  / A  × 100
120
× 100 = Rs. 20.

nee
% Gain =
600 b A 
Illustration 7. % Profit =  B × a – 1  × 100
 
Gita buys 12 pens for Rs. 96 and sells 10 pens for
Rs. 90. Find per cent profit or loss. Case II : If
rin
C.P. > S.P.
Loss = C.P. – S.P.
Solution :

Cost price of one pens =


96
12
= Rs. 8 Loss =
g.n
a b

A B

Selling price of one pens =


90
10
= Rs. 9

Since, selling price is greater than cost price.


% Loss =
e
Loss
C.P.

=
× 100

a b  A
 A – B  × a × 100
t
Profit = 9 – 8 = Re. 1  
Profit
% Profit = × 100  b A
C.P. % Loss =  1 – B × a  × 100
 
1
= × 100 = 12·5%.
8 Problems on Successive Profits and Losses
Illustration 8. Illustration 10.
Leela buys 6 dresses for 600 and sells 5 dresses for Sita buys a cooker for Rs. 500 and sells it to Soni at
Rs. 575. Find per cent loss / profit. 20% profit. Soni sells it to Ritu for 15% profit. Ritu uses
Solution : it for two years and then sells it to Sonali at a loss of
600 12%. For how much does Ritu sell the cooker to Sonali ?
C.P. of one dress = = Rs. 100
6 Solution :
575 C.P. of cooker = Rs. 500
S.P. of one dress = = Rs. 115
5 120
Since, S.P. > C.P. Sita sells it at = Rs. × 500
100

50 | CAT Complete Course

Visit : www.EasyEngineering.net
Visit : www.EasyEngineering.net

120 Problems based on Dishonest Seller/Buyer


Soni buys cooker at Rs. × 500
100 using faulty Measures of Weight/Long
120 115 Difference between True
Now, Soni sells it at = 500 × × measure and false measure
100 100 % Gain = × 100
Amount for which payment made
120
Again, Ritu buys the cooker from Soni at 500 × Illustration 12.
100
115 Sohan sells cloth at cost price but he uses faulty
×
100 metre rod. His metre rod measures 80 cms. only. Find his
gain per cent.
Sonali buys the cooker at
Solution :
120 115  100 – 12 
= 500 × × ×  On selling each metre he gets profit as the price of 20
100 100  100  cm. cloth.
Ritu sells the cooker to the Ritu at C.P. of the cloth sold as one metre = price of 80 cm.
By using formula :
120 115 88
= 500 × × × 20
100 100 100 % Profit = × 100 = 25%,

ww
Clearly :

Final S.P. = C.P. ×  1 +
% profit   profit 
  1 + % 100 
Illustration 13.
80

A trader buys corn from a farmer using faulty weight

w 

.Ea
100  




loss 
 Loss – %100 

at a rate which is same as his selling price in the market.
His 1 kg weights 1080 gms. Find his gain per cent.
Solution :
For every 1 kg that the trader buys, he gets

syE
← Product Factors →
So, In case of successive profits / loses.
Addition = 1080 – 1000 = 80 gms
Profit = price of 80 gms
Final S.P. = Initial C.P. × Product factors.
Illustration 11.
ngi C.P. = price of 1 kg
Because trader pays only for 1000 gms and not for
1080 gms.
X sells an article to Y at a loss of 10%, Y to Z at a
gain of 15%, Z to U at a loss of 5% and U to V at a profit
of 20%. If V had to pay Rs. 450. How much did X pay nee % Profit =
Profit
C.P.
× 100

for it ?
Solution :
rin =
80
1000
× 100 = 8%.

g.n
Illustration 14.
From above formula :
The cost price of 8 pens is equal to the selling price
 10   15   5  of 10 pens. Find the profit per cent.
Product factors =  1 – 100   1 + 100
 
  1 – 100



20 
 1 + 100



Solution :
e
S.P. of 10 pens = C.P. of 8 pens
S.P. of 1 pen = C.P. of pen
4
5
t
90 115 95 120 Let C.P. of one pen = Rs. a
= × × ×
100 100 100 100 4
Now, S.P. of one pen = Rs. · a
S.P. = Rs. 450 5
S.P. C.P. > S.P.
C.P. =
Product Factors So, Loss = C.P. – S.P.
450 4
= = a– ·a
90 115 95 120 5
× × ×
100 100 100 100 1
= ·a
5
450 × 1000000
C.P. = Loss
9 × 115 × 95 × 12 Now, % Loss = × 100
C.P.
500000
= 1
1311 ·a
5
= × 100 = 20%.
= Rs. 381·4 a

CAT Complete Course | 51

Visit : www.EasyEngineering.net
Visit : www.EasyEngineering.net

Illustration 15. Illustration 17.


If the cost price of a articles equals the selling price Raju gains a% when he sells an article for Rs. b.
of b articles. Find the per cent profit / loss. What would be the profit or loss per cent if he sold it for
Solution : Rs. c ?
Let C.P. of one article = Rs. X Solution :
Since, S.P. of b articles = C.P. of a articles When Raju gains a%
a  100 
S.P. of one article = Rs. · X C.P. =  100 + a  × b
b  
If a > b Case I : If C > C.P.
Case I : a > b 100
C– ×b
100 + a
S.P. > C.P. % profit = × 100
100
Profit = S.P. – C.P. ×b
100 + a
Profit  C (100 + a) 
× 100
× b – 1 × 100
% profit =
C.P. % profit =  100 

ww % profit = 
 a X – X
b
X 

 × 100 Case II : If C.P. > C
 C (100 + a) 

w
Case II : .Ea
% profit =

b > a
a 
 b – 1  × 100
  Illustration 18.
% loss =  1 – 100 × b × 100

Raju loses a% when he sells an article for Rs. b.

syE
C.P. > S.P.
Loss = C.P. - S.P.
Loss
What would be the profit or loss per cent if he sold it for
Rs. C ?
Solution :
% loss =
C.P.
a
X– X
× 100

ngi When Raju loses a%


C.P. =
100
×b
% loss =
X
b

 a
× 100

 1 – b  × 100
nee Case I : If C.P. > C
100 – a

Loss = C.P. – C

Illustration 16.
% loss =
 
rin
=
100
100 – a
×b–C

Shyam loses 15% when he sells his camera for


Rs. 500. For how much should he sells it in order to gain
% Loss =
g.n
Loss
C.P.
× 100

 C (100 – a) 
10% ?
Solution :
Since, Shyam sells camera at 85 whose C.P. = Rs. 100 Case II : If
% Loss =
e
C > C.P.
 1 – 100 × b  × 100

Profit = C – C.P.

t
100
∴ Shyam sells camera at 1 whose C.P. = 100
85 = C– ×b
100 – a
∴ Shyam sells camera at Rs. 500 whose C.P.
100 10000  C (100 – a) 
= × 500 = Rs. % profit =  100 × b – 1  × 100
85 17  
If Shyam wants to profit of 10%. Illustration 19.
So, when C.P. is Rs. 100, then S.P. = Rs. 110 Gita sold an article at a profit of X%. If he had
10000 110 10000 bought if for Y% less and sold it for Rs. W more, he
∴ C.P. is Rs. then S.P. = ×
17 100 17 would have made profit of Z%. Find the cost price of the
11 × 1000 article.
=
17 Solution :
11000 Let the original cost price be Rs. K
=
17
 100 + X 
11000 Initial S.P. = Rs.  K
Required S.P. = Rs.
17  100 

52 | CAT Complete Course

Visit : www.EasyEngineering.net
Visit : www.EasyEngineering.net

 100 + Y  5 × 1000
New C.P. = Rs.  K =
1045 – 1020
 100 
5 × 1000
 100 + Z   100 – Y  =
25
New S.P. = Rs.   K
 100   100  Cost of price of magazine = Rs. 200.
Difference between the two sole price equals Rs. Z. Illustration 22.
 100 + Z   100 – Y   +X Ashu sells an article at a loss of 10%. If he had
So,    K –  100 K = W
 100   100   100  bought it at 20% less and sold it for Rs. 90 more, he
would have made a profit of 20%. Find the cost price.
W × 100 × 100
∴K= Solution :
(100 + Z)(100 – Y) – (100 + X) × 100
Illustration 20. Here, we have
Sita buys certain quantity of an article for Rs. z. She X = 10
sells mth part of the stock at a loss of x%. At what per cent Y = 20
gain should she sell the remaining stock, so as to make an W = 90
overall profit of y% on the total transaction ?

ww
Solution :
C.P. of mth part of stock = Rs. (m × p) or Rs. m. z
Cost price of article
Z = 20

W × 100 × 100

wS.P. of m th part of stock =

.Ea
100 – x
100
×z

For y% profit on total transaction, its S.P.


 100 + y 
=

=
(100 + Z)(100 – Y) – (100 – X) × 100
W × 100 × 100
(100 + 20)(100 – 20) – (100 – 10) × 100

syE=  100  × z
 
S.P. of the remaining (1 – m)th part of stock
=

=
W × 100 × 100
=
90 × 100
120 × 80 – 90 × 100 96 – 90
9000
= Rs. 1,500.
=
 100 + y 
 
 – x
 100  × z –  100 ×m×z
 100  ngi Illustration 23.
6

C.P. of the remaining (1 – m)th part of stock


= (1 – m). z nee Sohan invests Rs. Z in shares. He sells mth part of it
at a profit of X% and the remaining at a loss of Y%. Find
his overall % profit or loss.
% profit =
S.P. – C.P.
C.P.
× 100 Solution :
rin
C.P. of mth part = m . Z

g.n
(100 + y) – (100 – x)m – 100(1 – m)
% profit = × 1000
(1 – m)z × 100  100 + X 
S.P. of m th part =  100  m.Z
y + xm  
Required % profit =

Illustration 21.
1–m
%

Krishna sells a magazine at a profit of 2%. If he had


e
C.P. of remaining (1 – m)th part = (1 – m). Z
S.P. of remaining (1 – m)th part
 100 – Y   – Y
t
bought it at 5% less and sold it for Rs. 5 more, he would =   m. Z +  100 
have gained 10%. Find the cost price of the magazine.  100   100 
Solution : (1 – m ). Z
Here, we have Total C. P. = Rs. Z
X = 2 S.P. – C.P.
% profit = × 100
Y = 5 C.P.
W = 5 % profit = (100 – Y)(1 – m) + (100 + X).
Z = 10 m – 100
Using formula : % profit = [m.X – (X – m)Y] %.
Cost price of magazine Illustration 24.
W × 100 × 100 A shopkeeper buys wheat for Rs. 2000. He had to
=
(100 + Z)(100 – Y) – (100 + X) × 100 sell one fifth at a loss of 20%. At what per cent gain he
5 × 100 × 100 should sell the remaining stock, so as to make an overall
=
110 × 95 – 102 × 100 profit of 10% on the total transaction ?

CAT Complete Course | 53

Visit : www.EasyEngineering.net
Visit : www.EasyEngineering.net

Solution : S.P. of remaining (1 – n)th the part


Here, Z = Rs. 2000  100 + Y   100 + X 
= Rs.   .a–  n.a
X = 20  100   100 
1
m = (100 + Y) – (100 + X).n
5 = Rs. ×a
100
Y = 10 S.P. – C.P.
Using formula : % profit = × 100
C.P.
Y + X.m 100 + Y – 100n – Xn
% profit = %
1–m – 100 + 100n
= × 100%
1 100(1 – n) × a
10 + 20 ×
5
% profit = %  Y – Xn 
1 % profit =  1 – n  %.
1–
5  
14 × 5 Illustration 27.
= %
4 A man buys wheat for Rs. 3000. He sells one-third

ww
Illustration 25.
% profit = 17·5%. of it at a profit of 12%. At what per cent gain should he
sell remaining two-third, so as to make an overall profit of
15% on the whole transaction ?

w
A trader buys rice for Rs. 4800. He sells one third at
a loss of 12%. At what per cent gain should he sell the

.Ea
remaining stock, so as to make an overall profit of 10% ?
Solution :
Solution :
1
C.P. of rd =
3
3000
3
= Rs. 1000

Here,

We have, syE
Z = 4800

m =
1
3
1
S.P. of rd =
3
120
100
× 1000 = Rs. 1200

Total C.P. = Rs. 3000


X = 12
Y = 10 ngi Required S.P. =
115
100
× 3000 = Rs. 3450

Using formula :

% profit =
Y + X.m
%
nee 2
C.P. of rest rd = Rs. 2000
3

rin
2
1–m Now, S.P. of rd = 3450 – 1200 = Rs. 2250
3
1
10 + 12 ×

g.n
S.P. – C.P.
=
3
% % profit = × 100
1 C.P.
1–
3 2250 – 2000

% profit = 21%.
14 × 3
2
%
=

= e 250
2000
2000
× 100

× 100 = 12·5%.
t
Illustration 26. Illustration 28.
Ramesh buys certain quantity of an article for Rs. a A hike of X% in the price of an article forces a
he sells n th part of it at a profit of X%. At what per cent person to buys Y kg less for Rs. Z. Find the new and the
profit or loss should be sell the remaining (1 – n)th part of original price per kg of the article.
the stock, so as to make an overall profit of Y% ? Solution :
Solution : The hike in the price of the article
C.P. of nth part = Rs. n.a X
= ×Z
100
 100 + X 
S.P. of nth part = Rs.   n.a This is the cost of Y kg article.
 100 
X.Z
C.P. of total = Rs. a C.P. of Y kg of the article =
100
 100 + Y  X.Z
Total S.P required = Rs.  a Cost of 1 kg of the article =
 100  100Y
C.P. of remaining (1 – n)th the part = Rs. (1 – n).a This is the new price of the article.

54 | CAT Complete Course

Visit : www.EasyEngineering.net
Visit : www.EasyEngineering.net

100 Solution :
Now, Original price = × new price
100 + X By using formula :
100 X.Z We have X = 20
= ×
100 + X 100 × Y
Y = 5
X.Z
New Price = Z = 250
100 × Y
X.Z
100 X.Z New price =
Original Price = × 100 × Y
100 + X 100 × Y
20 × 250
X.Z = = Rs. 10
Original Price = 100 × 5
(100 + X) × Y
X.Z
Illustration 29. Original Price =
(100 + X) × Y
A person sells one-fifth of his stock of good at a 20 × 250
= = Rs. 8·35.
profit of 25%. At what per cent profit or loss should he 120 × 5
sell his remaining stock, so as to make an overall profit of Illustration 31.
12% ?

ww
Solution :
Let C.P. of all the good = Rs. a
A reduction of 5% in the price of rice enables a
1
person to buy kg more for Rs. 190. Find the original
2

w 1 price /kg of rice and also its reduced price.


Now, C.P. of th part of goods
5 Solution :

.Ea = Rs.
a
5
Using formula :
X = 5
1
5
4
syE
S.P. of th part of goods = Rs.
125 a 25a
× =
100 5 100
4
Y = kg.
1
2

ngi
C.P. of th part of goods = Rs. × a Z = 190
5 5 X.Z
New price =
112
×a 100 × Y

nee
S.P. of the whole goods =
100 5 × 190
=
4 1
S.P. of the th part of goods 100 ×

rin
5 2
112 – 25 95 × 2
= ×a = = Rs. 19/kg.
10

g.n
100
X.Z
87 Original Price =
= ×a (100 – X) × Y
100
4 4
Clearly, S.P. of th part > C.P. of th of the goods

So,
5 5
profit = S.P. – C.P. Illustration 32.
=
e 5 × 190
(100 – 5) ×
1
2
× Rs. 20 / kg.

t
112 87 A man purchased m oranges at X a rupee and n
= ×a– ×a
100 100 oranges at Y a rupee. He mixed them together and sold
25 1 them at Z a rupee. What is his per cent loss or gain. What
= ×a= ×a happen when m = n ?
100 4
1 Solution :
·a m
4 5
% Profit = × 100 = × 100 C.P. of m oranges = Rs.
X
4 16
·a
5 n
C.P. of n oranges = Rs.
Y
125
= = 31·25%. m n 
4
Total C.P. = Rs. X + Y 
Illustration 30.  
A 20% hike in the price of sugar forces a person to Now, S.P. of (m + n) oranges
purchase 5 kg less for Rs. 250. Find the new and original m+n
= Rs.
price of sugar. Z

CAT Complete Course | 55

Visit : www.EasyEngineering.net
Visit : www.EasyEngineering.net

S.P. – C.P. Illustration 35.


% profit = × 100%
100 A man buys oranges at 20 a rupee for how many a
m + n  rupee should he sell it so as to gain 10% ?
 Z 
% profit =  – 1 × 100% Solution :
m + n 
X Y  Let C.P. of 20 oranges = Re. 1
110
 (m + n)X.Y  S.P. of 20 oranges = Rs. = Rs. 1·1
100
% profit =  Z(mY + n.X) – 1  × 100%
Since, Cost of 20 oranges is Rs. 1·1
When m = n 20 200
In Rs. 1 oranges is = = = 18·1 ≅ 18.
 2m.X.Y  1·1 11
% profit =  m(X + Y).Z – 1  × 100%
  Illustration 36.
 2XY  A milkman purchases the milk at Rs. 18/litre and
% profit =  – 1  × 100%. sells it at Rs. 20 liter still he mixes 2 litres of water with
 (X + Y).Z  every 8 litres of pure milk. What is per cent profit ?
Illustration 33.

ww
Solution :
A person mixes 30 kg of tea bought at Rs. 120 / kg For Simplicity :
with 25 kg of tea bought at Rs. 100 / kg. He sells the Let pure milk = 8 litres

w
mixture at Rs. 105 kg. Find his total per cent loss / gain ?
Solution :

.Ea
C.P. of 30 kg of tea = 30 × 120 = 3600
C.P. of 25 kg of tea = 25 × 100 = 2500
Now, C.P. of pure milk = 8 × 18 = Rs. 144
Since, he sell ( 8 + 2 ) litres with the cost of 20 /litres
S.P. = 10 × 20 = Rs. 200

syE
Total C.P. = Rs. 6100
S.P. of 55 kg of tea = 55 × 105 = Rs. 5775
Profit = 200 – 144 = Rs. 56

% Profit =
56
144
× 100
Since,
So,
C.P. > S.P.
Loss = C.P. – S.P. ngi =
350
9
= 38·8%.

% Loss =
= 6100 – 5775 = 325
Loss
C.P.
× 100% nee
Illustration 37.
Two articles are sold at the same price. One at a
profit of 50% and another one at a loss of 30%. What is
=
325
6100
× 100%
the overall profit / loss ?
Solution : rin
=
325
61
% = 5·32%. Let
g.n
C.P. of Ist article = X
C.P. of 2nd article = Y
Illustration 34.
Shyam purchase 100 oranges at 4 a rupee and 200
oranges at 2 a rupee. He mixed them and sells at 3 oranges
a rupee. Find his per cent loss or gain.
e
S.P. of Ist article = 1·5X
S.P. of 2nd article = 0·7Y
According to question :
t
Solution : 1·5X = 0·7Y
C.P. of 100 oranges = Rs. 25 X 7
= …(1)
C.P. of 200 oranges = Rs. 100 Y 15
S.P. of 300 oranges = Rs. 100 Total C.P. = X + Y
C.P. of 300 oranges = Rs. 125 7 22
= Y+ ×Y= ×Y
15 15
C.P. > S.P.
Total S.P. = 1·5X + 0·7Y
Loss = C.P. – S.P.
= 125 – 100 = Rs. 25 = 1·4Y
Loss Loss = – S.P. + C.P.
% loss = × 100
C.P. 22
Loss = – 1·4Y + ×Y
25 15
= × 100%
125 – 21Y + 22Y
=
% loss = 20%. 15

56 | CAT Complete Course

Visit : www.EasyEngineering.net
Visit : www.EasyEngineering.net

1 whole she neither loss or gain. What did each table


Loss = ·Y
15 cost ?
1 6. Gopal buys some milk contained in 5 vessels of
·Y
15 equal size. If he sells his milk at Rs. 6 a liter he
% loss = × 100
22 losses Rs. 150 while selling it at Rs. 8 a liter he
·Y
15 would gain Rs. 250 on the whole. Find the number of
100 50 liter contain in each case.
= = = 4·55%.
22 11 7. Ram losses 10% by selling pencils at the rate of 20 a
Illustration 38. rupee. How many for a rupee must he sell them to
The profit percentage on the three articles X, Y and gain 10% ?
Z is 5%, 10% and 20% and the ratio of the cost price is 8. A reduction of 40 per cent in the price of bananas
2 : 3 : 5. Also the ratio of number of article sold of X, Y would enable a man to obtain 32 more for Rs. 40.
and Z is 1 : 2 : 4. Then what is the overall profit What is the reduced price per dozen ?
percentage ?
9. Gita buys watch for Rs. 1000 and sells it to Sita at
Solution : 10% loss. Sita sells it to Sonali at 25% profit and

ww
Let C.P. of one X article = 2a
C.P. of one Y article = 3a
C.P. of one Z article = 5a
Sonali sells it to Pinki at 5% profit. How much did
Pinki pay for the watch ?
10. A shopkeeper uses faulty measure of weight of 900

w Total C.P. = 2a + 2 × 3a + 4 × 5a

.Ea = 2a + 6a + 20a = 28a


S.P. = 1·05 × 2a + 6a (1·1)
gm. For 1 kg weight and sells wheat at Rs. 20/kg in
place of Rs. 24 /kg. Find the per cent change in actual
rate.
11. A salesman first marks the price of an article 25%
+ 2a × 1·2

syE
= (2·10 + 6·6 + 24 )a
= 32·70a
above the cost price but later reduces it by 10% while
selling. Find the net per cent profit.

ngi
12. Soni sells an article at 10% profit. If she sells it for
Profit = S.P. – C.P. Rs. 20 more she will make a profit of 12%. Find the
= (32·70 – 28)a cost price of the article.

% Profit =
= 4·70a
4·7a
× 100% nee
13. Rahim sells an article at a loss of 15%. If he had
bought it for 20% loss and sold it for Rs. 40 more he

rin
28a would have gained 30%. Find the cost price.
470 14. When a man sold an article for Rs. 900 and made a
= % = 16·7%

g.n
28 loss of 10%. At what price should he sell it so as to
incur a loss of only 5% ?
Exercise A
15. Mohit bought 80 kg of tomatoes and plans to sell it at
1. A woman buys 4 tables and 16 chairs for Rs. 3200.
She sells the tables at a profit of 20% and chairs at a
profit of 10% and makes a profit of Rs. 600. At what
price did she buys tables and chairs ?
e
10% profit at Rs. 4·5 / kg 10 kg of tomatoes were
found rotten. At what price should be now sell rest of
the tomatoes in order to make 12% overall profit ? t
2. 5 kg of rice costs as much 10 kg of wheat, 20 kg of 16. A trader buys 10 shirts for payment of 8 shirts at the
wheat costs as much as 2 kg of tea, 4 kg of tea costs marked price and sells them at the marked price. Find
as much as 24 kg of sugar. Find the cost of 5 kg of his per cent profit.
sugar if 3 kg of rice costs Rs 30. 2
17. Saurav invested Rs. 25,000 in sugar. He sold th at a
3. A fan and a watch are sold of the same price of Rs. 5
927. The shopkeeper earns a profit of 10% on the fan loss of 15%. At what per cent gain he should sell the
whereas he incurs a loss of 10% on the watch. Find remaining stock in order to make overall per cent of
his overall per cent gain or loss. 20% ?
4. Rajesh purchased a chair marked at Rs. 800 at two 18. A reduction of 10% in the price of watch enables a
successive discounts of 10% and 15% respectively. Monu to buy 2 more for Rs. 32000. Find the original
He spent Rs. 28 on transportation and sold the chair and the reduced price per watch.
for Rs. 800. What is his gain per cent ? 19. A man buys a certain number of mangoes at 6 a
5. Kavita buys two tables in Rs. 1,350. She sells one so rupee and equal number at 3 a rupee. He mixes them
1 together and sells at a 5 a rupee. Find per cent profit
as to loss 5% and other so as to gain 7 % on the
2 or loss.

CAT Complete Course | 57

Visit : www.EasyEngineering.net
Visit : www.EasyEngineering.net

20. The marked price of an Apple is Rs. 10 the shop- 38400 – 32000 – 148.X
Or, 600 =
keeper allows discount of 8% and still make a profit 10
of 10%. What would be his profit if he did not allow Or, 148.X = 6400 – 600
discount? 5800
X = = Rs. 39·18
21. Jiya started selling vegetables at Rs. 12 /kg but could 148
not find buyers at this rate. So, she reduced the price C.P. of 16 chairs = 16 × 39·18 = Rs. 627
to Rs. 10 kg but uses of a faulty weight of 900 gm.
C.P. of 4 tables = 3200 – 627 = Rs 2573.
For 1 kg. Find the per cent change in actual price or
loss. 2. Cost of 3 kg of rice = Rs. 30
22. Rohani bought 20 kg of rice at the rate of Rs. 5/kg Cost of 5 kg of rice = Rs. 50
and 30 kg of rice at the rate of Rs. 6/kg. She mixed Cost of 10 kg of wheat = Rs. 50
the two and sold the mixture at the rate of Rs. 7·5/kg. Cost of 20 kg of wheat = Rs. 100
What was her loss or gain in the total transaction ?
Cost of 2 kg of tea = Rs. 100
23. Find the difference between a single discount of 60%
Cost of 4 kg of tea = Rs. 200
on Rs. 800 and two successive discounts of 40% and
20% on the same amount. Cost of 24 kg of sugar = Rs. 200

ww
24. Shyam bought 2 dozen apples at Rs. 20 per dozen
and 4 dozen apples at Rs. 15 per dozen. He sold all
∴ Cost of 1 kg of sugar = Rs.
200
24
100

w
of them to earn 28%. A what price per dozen did he ∴ Cost of 5 kg of sugar = Rs. ×5
sell the apples ? 12

.Ea
25. Find the cost price of an article, which on being sold
at a gain of 15% yields Rs. 5 more than when it is
= Rs.
500
12
= Rs. 41·66.

sold at a loss of 12%.

syE
26. 12 kg of potato costs as much as 3 kg of tomato, 9 kg
of tomato costs as much as 60 kg of onion, 15 kg of
3. Since, Cost of a fan = Rs. 927
Cost of a watch = Rs. 927
Total S.P. = Rs. 1854
onion costs as much as 21 kg of cabbage. If 12 kg of
cabbage costs Rs. 108. Find the cost of 18 kg of
potato ? ngi C.P. of a fan =
100
110
× 927 = Rs. 842·72

nee
100
C.P. of a watch = × 927 = Rs. 1030
27. Ravi sells two articles for the same price. On one he 90
incurs 20% loss while on the other he incurs 10% Total C.P. = 1030 + 842·72
loss. Find his overall per cent loss.
28. Avinash buys a cap for Rs. 160 and sells it to Aman Since, rin = Rs. 1872.72
C.P. > S.P.
at 8% loss, Aman sells it to Raman at 10% profit and
Raman sells it to Rahim at 20% profit. How much
did Rahim pay for the cap ? g.n
Loss = C.P. – S.P.
= 1872·72 – 1854

1. Let C.P. of one chair


Answers
e
% loss =
= Rs. 18.72
loss
C.P.
× 100

18·72
t
= Rs. X = × 100% = 1%
1872·72
C.P. of 16 chairs = Rs. 16X 10
4. 1st discount = × 800 = Rs. 80
C.P. of 4 tables = 3200 – 16X 100
120 15
S.P. of 4 tables = × (3200 – 16.X) 2nd discount = × Rest amount
100 100
110 15
S.P. of 16 chairs = × 4X = × 720 = Rs. 108
100 100
10 Rest Amount = 720 – 108 = Rs. 612
Total S.P. = [12(3200 – 16X) + 44X]
100 Total Cost Price = 612 + 28 = Rs. 640
1 S.P. = Rs. 800
= (38400 – 192X + 44X)
10 Profit = S.P. – C.P.
Profit = Total S.P. – Total C.P. Profit = 800 – 640 = Rs. 160
1 160
Profit = (38400 – 148X) – 3200 % Profit = × 100 = 25%
10 640

58 | CAT Complete Course

Visit : www.EasyEngineering.net
Visit : www.EasyEngineering.net

5. Let the cost of Ist table = Rs. X 90


9. Gita buys watch at = Rs. × 1000
The Cost of IInd table = Rs. ( 1350 – X ) 100
95 19 = Rs. 900
S.P. of 1st table = ·X= ·X
100 20 125
Sita buys watch at = × 900 = Rs. 1125
107·5 100
S.P. of 2nd table = × (1350 – X)
100 105
Sonali buys watch at = × 1125
Since, there is no loss or gain. 100
So, C.P. = S.P. = Rs. 1181·25
19 107·5 So, Pinki pays Rs. 1181·25.
1350 = ·X+ (1350 – X)
20 20 10. After reduction,
Or, 135000 = 95.X – 107·5X + 107·5 × 1350 The price of 900 gm wheat = Rs. 20
12·5X = 145125 – 135000 ∴ The price of 1000 gm wheat
12·5X = 10125 20 200
= × 1000 = Rs.
10125 900 9
X =

ww 12·5 200 25
Loss = Rs. 25 – = Rs.
9 9
101250
X = = Rs. 810 25
125

w Cost of 1 st table = Rs. 810

.Ea
Cost of 2 nd table = 1350 – 810 = Rs. 540.
6. Let the vessels contain = X litre of milk
% Loss =
9
25
× 100% =

= 11·11%.
11. Let the actual price of the article = Rs. X
100
9
%

syE
Now, cost price of milk in Ist case
= 6X + 150
Marked price in case 1st =
125
100
X

ngi
In 2nd case cost price of milk After reduction of 10%
125 90
= 8X – 250 Marked price = X×
100 100
Since, C.P. does not change
6X + 150 = 8X – 250 nee 5 9
= × X= X
4 10 8
9

2X = 400
X = 200
rin
Profit = marked price – actual price
9
= X–X=
X

7.
Clearly, 5 vessels contain 200 litres of milk.
S.P. of pencils = Re. 1 % profit =
8

g.n 8
 Profit 
 Actual price  × 100
To get 10% profit
11
C.P. of pencils = Rs.
10 =

X
8
X
e 

× 100 = 12·5%. t
11
If Ram sells at the rate of 20 for rupee. He gains 12. Let the cost price of the article = Rs. X
10
10%. After 10% profit
20 110
So, In one rupee = × 10 S.P. of the article = ×X
11 100
200 11
= ≅ 18 pencils = ×X
11 10
40 When Soni gets 12% profit, then
8. Reduction in price = 40 × = Rs. 16
100 11
S.P. = × X + 20
According to question, 10
Man buys 32 bananas in Rs. 16. According to question,
16 When selling price is 112, then C.P. = 100
∴ Man buys 12 bananas in Rs. × 12 = Rs. 6
32 100
∴ Selling price is 1, then C.P. =
∴ Rs. 6 / dozens 112

CAT Complete Course | 59

Visit : www.EasyEngineering.net
Visit : www.EasyEngineering.net

 11  100 Now, the trader buys 10 shirts at Rs. 8X.


∴ Selling price is  × X + 20  , then C.P. = ×
 10  112 He sells shirts at marked price.
 11  So, S.P. of 10 shirts = Rs. 10X
 10 × X + 20
  Now, profit = 10X – 8X = Rs. 2X
2.X
100  11X  % profit = × 100 = Rs. 25%.
This equal to X = × + 20  8.X
112  10  2
17. Now, S.P. of th stock of sugar
 110  110 5
Or, X 1 –  = 20 × 112
 112 
=
85 2
× 25000 × = Rs. 8500
100 5
X = Rs. 1000.
3
13. Let the C.P. of the article = Rs. X Let S.P. of th stock of sugar = X
5
85
S.P. of the article when Loss is 15% = ·X Now, Total S.P. = 8500 + X
100
C.P. of Total sugar = 25000
80
He bought the article at = Rs. ·X

ww 100 120
Now, Total S.P. = × 25,000 = Rs. 30,000
100
80
Now, Rahim sells the article = · X + 40 According to question,
100

w
According to question,
80
100
·X =
.Ea
 80 
 100 · X + 40  × 100
  113
8500 + X = 30,000
X = 30,000 – 8,500
X = Rs. 21,500

Or,
8
10
8
syE
· X =  · X + 40  ×
 10
 100
 113
18. The reduction in the price

=
10
100
× 32,000 = 3,200

Or,
4
5
 100 
· X 1 –
 113 
100
 = 40 × 113
ngi From the question,
It is clear that cost of 2 watch = Rs. 3200
Or,
4
5
·X×
13
113
=
4000
113
4000 × 5 nee Cost of one watch = Rs. 1600
When C.P. is 90 the original price = Rs. 100

rin
Or, X = C.P. is 1600 the original price
13 × 4
5000 100
X = Rs. · = × 1600

g.n
13 90
14. Given S.P. = Rs. 900 16000
= = 1777·7.
9

e
100
C.P. = × 900 = Rs. 1000

t
90 19. Let the total number of mangoes = 2.X
To get 5% loss the S.P. Now, C.P. of 2.X mangoes
95 X X 3 X
= × 1000 = Rs. 950. = + = ·X=
100 6 3 6 2
15. When Mohit gets 10% profit, then C.P. of tomatoes 2
S.P. of 2.X mangoes = · X
100 5
= × 4·5 = Rs. 5
90 C.P. > S.P.
So, C.P. of 80 kg of tomatoes = 80 × 5 = Rs. 400 So, Loss = C.P. – S.P.
Rest tomatoes = 70 kg. X 2X X
= – =
Now, S.P. of 70 kg of tomatoes 2 5 10
112 Loss
= × 400 = Rs. 448 % Loss = × 100%
100 C.P.
X
448
S.P. of 1 kg of tomatoes = = Rs. 6·4. 10
70 = × 100%
3.X
16. Let C.P. of one shirt = Rs. X 6
Now, C.P. of 8 shirts = Rs. 8X 60
= = 20%.
This is equal to 10 shirts = Rs. 8X 3

60 | CAT Complete Course

Visit : www.EasyEngineering.net
Visit : www.EasyEngineering.net

8 128
20. Discount = 10 × = 0·8 S.P. = × 100 = Rs. 128
100 100
C.P. of Apple = 10 – 0·8 = 9·2 128
Now, S.P. of 12 apples = = Rs. 21·33.
110 6
S.P. = × 9·2 = Rs. 10.12
100 25. Let C.P. of the article = X
If discounts is not allowed, then 115
1st S.P. = ·X
Profit = 10·12 – 10 = 0·12 100
Profit 88
% Profit = × 100% 2nd S.P. = ·X
MP 100
0·12  115 88 
= × 100% = 1·2% Difference =  100 – 100 . X
10  
21. After reduction 27
5 = ·X
The price of 900 gm of vegetables = Rs. 10 100
∴ The price of 1000 gm of vegetables 500
X = Rs.
27

ww
10 100
= × 1000 =
900 9 X = Rs. 18·5.
100 8
Loss = 12 – = 26. Let C.P. of 1 kg of potato = Rs. X

w .Ea
% Loss =
8
9
12
9

× 100%
9
C.P. of 12 kg of potato = Rs. 12.X
C.P. of 3 kg of tomato = Rs. 12.X
C.P. of 9 kg of tomato = Rs. 36.X
=
syE
200
27
% = 7·4%.

22. C.P. of 20 kg of rice = 20 × 5 = Rs. 100


C.P. of 60 kg of onion = Rs. 36.X

C.P. of 15 kg of onion = Rs.


36
· X × 15

ngi
60
C.P. of 30 kg of rice = 30 × 6 = Rs. 180
= Rs. 9.X
S.P. of 50 kg of rice = 50 × 7·5 = Rs. 375
Total C.P. = 280
Total S.P. = 375 nee C.P. of 21 kg of cabbage = Rs. 9.X

C.P. of 12 kg of cabbage = Rs.


9
21
· X × 12
Profit = S.P. – C.P.
= 375 – 280 = Rs. 95 rin =
36
7
·X

% profit =
Profit
C.P.
× 100
g.n
108 =
36
7
·X

=
95
280
950
28
× 100

= 33·92%
e
X =
108 × 7
36
= Rs. 21

Cost of 18 kg of potato = 18.X = 18 (21) t


= Rs. 378.
23. When discount is 60%
27. Let S.P. of each article = Rs. X
60
Then discount = 800 × = Rs. 480 Total S.P. of articles = Rs. 2.X
100
When there are two successive discounts, then 100
C.P. of 1 st article = × 2.X = 2·5X
80
40
Ist discount = × 800 = Rs. 320 100 20
100 C.P. of 2 nd article = × 2.X = · X
90 9
Rest Amount = 800 – 320 = Rs. 480
20
20 Total C.P. = 2·5.X + ·X
and 2nd Amount = 480 × = Rs. 96 9
100
22·5.X + 20.X
Total amount = 320 + 96 = Rs. 416 =
9
Now, Difference = 480 – 416 = Rs. 64. 42·5
= ·X
24. Total C.P. = 2 × 20 + 4 × 15 9
= 40 + 60 = Rs. 100 C.P. > S.P.

CAT Complete Course | 61

Visit : www.EasyEngineering.net
Visit : www.EasyEngineering.net

So, Loss = C.P. – S.P. conditions the selling price can not be increased
42·5 beyond 10%.
= · X – 2.X
9 (i) What is the maximum current gain possible.
24·5 (ii) If the Germany dollar becomes cheap by 10%
= ·X
9 over its original cost and the cost of U.K. mark
Loss increased by 15%. What will be the gain ?
% Loss = × 100%
C.P. 4. A on his dead bed, keeps half his property for his
24·5X wife and divides the rest equally among his three
9 sons; B, C, D. Some year later, B dies leaving half
= × 100% his property to his window and half to his brothers C
42·5X
9 and D sharing equally. When C makes his will, he
keeps half his property for his widow and the rest he
24·5
= × 100% begueaths to his younger brother D. When D dies
42·5
some years later, he keeps half his property for his
= 57·64%. widow and the remaining for his mother. The mother
92 now has Rs. 63,00,000.
28.

ww Aman buys a cap at = Rs.


100
× 160

= Rs. 147·20
5. A milkman purchases the milk at Rs. Y/litre and sells
it at Rs. 1·5Y / litre still he mixes 2·5 litres of water
with every 5 litres of pure milk. What is profit per-

wRaman buys a cap at = Rs.

.Ea
110
100
× 147·20

= Rs. 161·92
120
6.
centage ?
A Radio dealer incurs on expense of Rs. 150 for
producing every radio. He also incurs an additional

syE
Rahim buys a cap at = Rs. × 161·92 expenditure of Rs. 10,000 which is independent of
100 the number of radio produced. If he is able to sell a
= Rs. 194.304 radio during the season, he sells it for Rs. 300. If he

ngi
fails to do so, he has to sell each watch for Rs. 200.
Special
(i) If he is able to sell only 120 out of 150 radios he
1. Ganesh, on his dead bed, keeps half his property for has made in the season, then he has made a
his wife and divides the rest equally among his three
sons; Ram, Shyam, Krishna. Some year later, Ram
dies leaving half his property to his widow and half
nee profit of ?
(ii) If he produces 150 radios, what is the number of

rin
radios that he must sell during the season in
to his brothers. Shyam and Krishna, sharing equally. order to breakeven, given that he is able to sell
When Shyam makes his will, he keeps half his all the radios produced ?
property for his widow and the rest he bequeaths to
his younger brother Krishna when Krishna dies some
years later, he keeps half his property for his widow
7.
g.n
What should be the minimum markup percentage
such that after giving a discount of 22·5% there will
and the remaining for his mother. The mother now
has Rs. 18,30,000.
(i) What was the worth of total property ?
not be a loss ?
Exercise Be
1. A retailer bought 25 kg of tea at a discount of 20%. t
(ii) What was Shyam’s original share ? Besides 1 kg tea was freely offered to him by the
(iii) What was the ratio of the property owned by the wholesaler at the purchase of 25 kg tea. Now, he
window of the three sons, in the end ? sells all the tea at the marked price to a customer.
What is the profit of retailer ?
2. Ashu and Tannu both are dealers of classic scooter
2. Ram bought 15 oranges for a rupee and sold them at
the price of classic scooter is Rs. 35,000. Ashu gives
12 oranges for a rupee. What is the profit percentage ?
a discount of 15% on whole, while Tannu gives a
discount of 20% on the first Rs. 25,000 and 5% on 3. Tarun purchased the books for Rs. 1,20,000. He sold
the rest Rs. 10,000. What is the difference between 50% of it at a profit of 12·5% and rest at a loss. Find
their selling prices ? the loss percentage on the remaining if the overall
loss is 10% ?
3. A company purchases components X and Y from
U.K. and Germany, respectively X and Y from 30% 4. The ratio of cost price and marked price of an articles
and 40% of the total product cost current gain is 25% is 1·5 : 2 and ratio of percentage profit and percentage
due to change in the international scenario, cost of discount is 2 : 3. What is the discount percentage ?
the U.K. mark increased by 30% and that of 5. Shyam sold 10 cameras at a profit of 30% and 6
Germany dollar increased by 20%. Due to market cameras at a profit of 20%. If he had sold all the 16

62 | CAT Complete Course

Visit : www.EasyEngineering.net
Visit : www.EasyEngineering.net

cameras at a profit of 25%, then he would have 15. X, Y and Z invest in the ratio of 1 : 3 : 5 the percent-
gained Rs. 48 more. What is the cost price of each age of return on their investment are in the ratio of
camera ? 5 : 4 : 3. Find the total earnings if Z earns Rs. 510
6. A single discount equivalent to three successive more Y.
discounts of 20%, 40% and 10% ? 16. Ram bought a house in Delhi city, whose sale price
7. The market price of an article is increased by 20% was Rs. 16 lakh. He availed 20% discount as an early
and the selling price is increased by 12·5%, then the bird offer and then 10% discount due to cash
amount of profit doubles. If the original marked price payment. After that he spent 10% of the cost price in
be Rs. 500 which is greater than the corresponding interior decoration and lawn of the house. At what
cost price by 25%. What is the increased selling price should he sell the house to earn a profit of
price ? 25% ?
8. Sprite and coke, there are two companies, selling the 17. A car mechanic purchased four old cars for Rs. 2
packs of cold-drinks. For the same selling price sprite lakh. He spent total 4 lakh in the maintenance and
gives two successive discounts of 5 % and 25%. repairing of four cars. What is the average sale price
While coke sells it by giving two successive dis- of the rest three cars to get 50% total profit if he
counts of 10% and 20%. Which selling price is already sold one of the four car Rs. 3 lakh ?

ww
greatest, if print price of both are the same ?
9. An automobile company launched a scheme for their
18. Cost price of two motorcycles is same. One is sold at
a profit of 12% and the other for Rs. 5,000 more than

w
dealers that if a dealer purchases 5 bikes of Model A, the first. If the net profit is 20%. Find the cost price
two extra bikes will be free and if he purchases 2 of each motorcycle.

.Ea
bikes of Model B, he will get one extra bike. If cost
of 3 bikes of Model A and cost of 2 bikes of Model
19. Profit on selling 10 books equal selling price of 3
pens. While loss on selling 10 pens equal to selling

syE
B are Rs. 1,20,000 and Rs. 1,00,000. and A Dealer
purchases 4 bikes of Model A and 3 bikes of Model
B. At what price these bikes should be sold so that
price of 4 books. Also profit percentage equals to the
loss percentage and cost of books is half of the cost
of a pen. What is the ratio of selling price of books to

ngi
the agency can get overall profit of 12·5% ? the selling price of a pen ?
10. When a electric bike manufacturer reduced its selling 20. The cost of setting up a newspapers Rs. 12,000. The
price by 40%. The number of electric bike sold cost of paper and ink etc. is Rs. 80 per 100 copies
radically increased by 500%. Initially the manu-
facturer was getting only 120% profit. What is the nee and printing cost is Rs. 120 per 100 copies. In the
last month 2500 copies were printed but only 2000

rin
percentage increase of its revenue ? copies could be sold at Rs. 10 each. Total 25% profit
11. The ratio of selling price of 2 articles X and Y is 1 : 2 on the sale price was realized. There is one more
resource of income from the newspaper which is
and the ratio of percentage profit is 1 : 2 respectively.
If the profit percentage of X is 12·5% and the cost
price of Y is Rs. 500. What is overall percentage g.n
advertising. What sum of money was obtained from
the adverting in newspaper ?
gain ?
12. X and Y are two partners and they have invested Rs.
1,20,000 and 1,50,000 in a business. After one year
e
21. A milkman mixes 25% water in pure milk but he is
not content with it so he again mixes 10% more water
in the previous mixture. What is the profit percentage
of milkman if he sells it at cost price ?
t
X received Rs. 2,000 as his share of profit out of total
22. A man sold two flats for 5,00,000 each. On one he
profit of Rs. 5,600 including his certain commission gains 25% while on the other he losses 25%. How
on total profit since he is a working partner and rest much does he gain or loss in the whole transaction ?
profit is received by Y. What is the commission of X
23. Shyam sold three fourth of his articles at a gain of
as a percentage of the total profit ?
40% and the remaining at cost price. Find the gain
13. Gita sold her car to Sita at a profit of 20% and Sita earned by him in the whole transaction.
sold it to Chandani at a profit of 10%. Chandani sold
24. After getting two successive discounts, a shirt with a
it to a mechanic at a loss of 8%. Mechanic spend
list price of Rs. 200 is available at Rs. 160. If the
12% of his purchasing price and then sold it at a
second discount is 4%. Find the first discount.
profit of 20% to Gita once again. What is the loss of
Gita ? 25. Tarun got 20% concession on the labeled price of an
article and sold it for Rs. 9,000 with 25% profit on
14. In a factory the number of workers reduces in
the price he bought. What is the labeled price ?
the ratio of 3 : 2 and the salary increases in the ratio
13 : 15. What is the profit percentage of workers 26. A retailer marks all his good at 50% above the cost
over the previous salary ? price and thinking that he will still make 20% profit,

CAT Complete Course | 63

Visit : www.EasyEngineering.net
Visit : www.EasyEngineering.net

offers a discount of 25% on the marked price. What Answer Special


is his actual profit on the sales ? 1. Let Total property of the Ganesh = Rs. X
27. Ramji purchased 20 dozens of toys at the rate of X
Rs. 375 per dozen. He sold each one of them at the Ganesh’s wife gets = Rs.
2
rate of Rs. 33. What was his percentage profit ?
X  1
28. A cotton shirt is listed in three different department Ram gets = Rs.  ×
stores at Rs. 200, Rs. 400 and Rs. 800 respectively. 2  3
The three stores offers a discount of 10%, 15% and X  1
20% respectively on these prices. Recession force Shyam gets = Rs.   ×
2  3
them to offer further discounts of 5%, 10% and 10%
respectively on the above reduced price. At this stage X  1
Krishna gets = Rs.   ×
the difference between the maximum and minimum 2  3
and maximum and minimum prices at the initial
After Ram death,
stage. Find the difference.
X
29. An alloy is made of three metals X, Y and Z. Metal His widow gets = Rs.
12
Y constitutes 40% of the total weight and rest of the

ww
weight is distributed in the ratio of 2 : 3 between X
and Z. The costprice per unit weight of X, Y and Z
is in the ratio of 5 : 2 : 3. The alloy is sold in the
Shyam gets = Rs.
X
24
X

w
Krishna gets = Rs.
market at 40% profit. Due to change in market 24

.Ea
conditions there is 20% increase in the cost of Y and
25% increases in the cost of Z. The selling price
remains the same. Calculate the per cent profit or loss
According to Shyam’s will,
X X 1
His wife gets = Rs.  + ×
 24 6  2
in the transaction.
syE
30. Ram and Shyam have 20 cows with them. Ram sells
his cows at a different rate than Shyam. They both
1 X X 
Krishna gets = Rs. ×  +  = Rs.
2  6 24 
5X
48
receive the same total sum. If Ram had sold his cows
at Shyam’s price, he would have received Rs. 245. If ngi After Krishna death,
1   5X 1 X  
Shyam sold his cows at Ram’s price. He would have
received Rs. 320. At what price did Shyam sell each
of his cows ? nee His wife gets = Rs. ×  × + 
2   48 2 6  

1  5X X 
31. A trader sells widgets at a certain list price if any
customer buys a second widget during the week he rin
His mother gets = Rs. ×
2  48 6 
+ = Rs.
13X
96
gives 10% discount on the list price for the second
widget and every additional widget that the customer
buys during the week is supplied at a price 10% less Or,
X 13
+ g.n
Total Mother’s money = 1830000

· X = 1830000
than that of the second widget. Further, if a customer
buys 5 or more widgets during a week the trader
gives an addition 2% of the list price as quantity
discount. If a certain customer buys 5 widgets during
Or,
2 96
61
96
e
· X = 1830000

1830000 × 96
t
Or, X =
a week what is the average discount the customer 61
gets per widget ? X = Rs. 2880000
32. In a garden there are only orange and apple trees X X
Shyam’s share = +
fruits available on each tree are equal to the total 6 24
number of trees of the same kind. Also, orange trees 5
are twice in number as compared to the apple trees. = ·X
24
Only 40% of total mangoes and 70% of total apples 5
are in good condition. These have to be stored apples. Shyam’s share = × 2880000
24
Rest were wasted. Selling price per orange is Rs. 10
= 5 × 120000
and per apple is Rs. 5. Investment to store (consider
only this as the cost price) is Rs. 20 per mango and = Rs. 6,00,000
Rs. 10 per apple. In this transaction, the Garden’s X
Ram’s wife gets = Rs.
owner lost Rs. 15,000. What is the total number of 12
trees in the garden ? = Rs. 2,40,000

64 | CAT Complete Course

Visit : www.EasyEngineering.net
Visit : www.EasyEngineering.net

X X 1  10 
Shyam’s wife gets = Rs.  + × New S.P. = 117 ×  1 + 
 24 6  2  100 
1 5 = 117 × 1·1 = 128·7
= Rs. × ·X
2 24 Profit = 128·7 – 125 = 3·7
5 3·7
= Rs. × 2880000 % profit = × 100
2 × 24 125
= Rs. 3,00,000 14·8
= = 2·96%.
5
1  5.X 1 X 
Kirshna’s wife gets =  × +  New cost of component A = 30 × 1·1 = Rs. 33
2  24 2 6 
New cost of component B = Rs. 40 × 0·85 = Rs. 34
1  5.X X 
= Rs.  +  Production cost = 33 + 30 + 34 = 97
2  48 6 
So, gain = 125 – 97 = 28
 1 13.X  18
= Rs.  ×  % gain = × 100% = 22·4%.
 2 48  125

ww = Rs.
1 13
× × 2880000
2 48
4. Let total property = Rs. X

His wife gets = Rs.


X
2

andw = Rs. 3,90,000

.Ea
Their ratio = 8 : 10 : 13
2. In case of Ashu
B gets = Rs.

C gets = Rs.
X
6
X

syE
Discount on Scooter = 35000 ×
15
100
S.P. = 35000 – 5250
= Rs. 5,250
D gets = Rs.
6
X
6

In case of Tannu
= Rs. 29,750
ngi Now, after B’s death,

B’s wife share =


X

nee
20 12
Ist discount = 25000 × = Rs. 5,000 X
100 C’ share = Rs.
5 24
2nd discount = 10000 ×

Total discount = Rs. 5,500


100
= Rs. 500

rin
D’ share = Rs.
X
24

S.P. = 35000 – 5500


= Rs. 29,500
Total C’ share =
g.n
X X 5
+ =
24 6 24
·X

Now, Difference = 29750 – 29500


= Rs. 250
3. Let the Total production cost = Rs. 100
Total D’ share =

After C’s death,

His wife’s share =


e
X X 5
+
6 24 24

5
= ·X

·X= ·X
5
t
24 × 2 48
Cost of component X = Rs. 30
5
Cost of component Y = Rs. 40 D’ share = ·X
48
Remaining production cost 30 expenses on other 5 5 15
things. Total D’share = ·X+ ·X= ·X
24 48 48
Since, profit is 2.5% Now, after D’s death,
∴ S.P. = 125 15
His wife’s share = ·X
New price of component X 96
30 15
= 30 + 30 × = Rs. 39 His mother’s share = ·X
100 96
New price of component Y 15 X 48 + 15
Total mother’s share = ·X+ = ·X
20 96 2 96
= 40 + 40 × = Rs. 48
100 63 21
= ·X= ·X
New production cost = 39 + 48 + 30 = Rs. 117 96 32

CAT Complete Course | 65

Visit : www.EasyEngineering.net
Visit : www.EasyEngineering.net

(i) What was the worth of total property ? (ii) Production cost of 150 radios = 32,500
(ii) What was C’share ? Let he sells X radios during the season.
According to question, Amount received after selling
21 = 300.X + (150 – X) × 200
· X = 63,00,000
32 = 100.X + 30,000
6300000 × 32
X = Now, break-even is achieved if production cost is
21 equal to the selling price.
= Rs. 96,00,000 ∴ 100.X + 30000 = 32500
5
Total C’share = ·X 100.X = 32500 – 30000
24
2500
5 X =
= × 9600000 100
24
X = 25
= Rs. 20,00,000
7. Let C.P. = X
Since, there is no loss.
X.Y  100 + Y 
So

ww C.P. = S.P.

X =
77·5 X (100 + Y)
100
×
100
Mark up price = X +

After giving discount,


100
= X. 
 100 

Or,
Or,
Or,
w 100 × 100 = 7750 + 77·5Y

.Ea
10,000 – 7750 = 77·5Y
2250 = 77·5Y
Discounted price =

Answer B
77·5 X (100 + Y)
100
×
100

syE
Y =

Y =
22500
775
900
1. Let the marked price of 1 kg tea = Rs. X
Let the marked price of 25 kg tea = Rs. 25X
80
31
Y = 29·03%.
ngiNow, C.P. of 25 kg tea = Rs. 25.X ×

S.P. of tea = 26X


100
= 20X

5. Let pure milk is 5 litres (simplicity).


Now, milk + water
5 + 2·5 = 7·5
nee Profit = S.P. – C.P.
= 26X – 20X = 6.X

He sells 2.5 litres water at the cost of milk.


S.P. = 7·5 × 1·5Y = 11·25Y rin
% profit =
Profit
C.P.
× 100

g.n
So, 6.X
= × 100 = 30%.
C.P. = 5 × 1·5 Y = 7·5Y 20.X
1

e
Profit = S.P. – C.P. 2. C.P. of one oranges = Rs.

t
= 11·25Y – 7·50Y = 3·75Y 15
1
3·75 S.P. of one oranges = Rs.
% profit = × 100 12
7·5
Profit = S.P. – C.P.
4 375
= × % 1 1 5–4 1
3 10 = – = =
12 15 3 × 5 × 4 60
2 × 75
% profit = = 50%. 1
3
60
6. In producing radios % profit = × 100
1
Total cost = 150 × 150 + 10000 15
= Rs. 32,500 15
= × 100 = 25%.
Amount received after the sell of radios 60
= 120 × 300 + 30 × 250 3. C.P. of books = Rs. 1,20,000
= 36000 + 7500 C.P. of 50% books = Rs. 60,000
= Rs. 43,500 112·5
S.P. of 50% books = 60000 ×
∴ Profit earned = 43500 – 32500 100
= Rs. 11,000 = Rs. 67,500

66 | CAT Complete Course

Visit : www.EasyEngineering.net
Visit : www.EasyEngineering.net

Since, overall loss is 10%. 80.X 8.X


Remaining amount = =
90 100 10
So, total S.P. = × 120000
100 8.X 40 32.X
2nd discount = × =
= Rs. 1,08,000 10 100 100
Now, S.P. of the remaining 50% books 16.X
=
50
= 108000 – 67500 = 40,500
8.X 16.X
Loss on remaining books Remaining amount = –
10 50
= 60000 – 40500 = 19,500
40X – 16X
19500 =
% loss = × 100 = 32·5%. 50
60000
24
4. Let C.P. of the article = 1·5 X = ·X
50
M.P. of the article = 2X 24 10
3rd discount = ·X×
Percentage profit = 2Y% 50 100
Percentage discount = 3Y% 6

ww
Since,
 Discount 
C.P. = M.P.  1 –
 100 

=
125
If marked price = Rs. 100
·X

Or,
w 3.Ea
 3.Y 
1·5X = 2X  1 –
 100 
= 1–
3.Y

Now, equivalent discount

=
X 16
+
5 50
·X+

25 + 40 + 6
6
125
·X

Y = ×syE 100
1 100 100
4 3
=
12
=
125
= 56·8%.
× 100

Y =
25
3
= 8·33%

Percentage profit = 2 × 8·33 = 16·66%.


ngi 7. Original M.P. = Rs. 500
When M.P. is 125, then C.P. Rs. 100.

5. Let cost price of each camera = Rs. X


C.P. of 16 cameras = Rs. 16 X
nee ∴ 500, then C.P. =
100
125
After increasing M.P. by 20%
× 500 = Rs. 400

S.P. of 10 cameras =
13
10
× 10.X = 13 X
rin 
Final M.P. = 500  1 +

20 

100 
S.P. of 6 cameras =
120
100
× 6.X = 7·2X
g.n 6
= 500 × = Rs. 500
Total S.P. = 13X + 7·2X
= 20·2X
When overall profit is 25%.
…(1) e
If profit = 2.X
Now, initial profit = X
5

Initial S.P. = (400 + X)


t
125
Then, S.P. = × 16.X
100 After increasing 12.5%
5 112·5
= × 16.X S.P. = × (400 + X)
4 100
= 20X …(2) According to question,
Difference (1) and (2) is given by 112·5
× (400 + X) – 400 = 2.X
100
20·2X – 20X = 48
 112·5   112·5 
0·2X = 48 Or,  100 – 1  × 400 =  2 – 100 . X
X = 240
Cost Price of each camera = Rs. 240.  12·5 × 400  87·5
Or,  100  = 100 · X
6. Let the marked price = Rs. X
20.X X 12·5 × 400 400
∴ Ist discount = = X = = Rs.
100 5 87·5 7

CAT Complete Course | 67

Visit : www.EasyEngineering.net
Visit : www.EasyEngineering.net

Increased selling price 11. Let S.P. of X = a


112·5  400  Let S.P. of Y = 2a
=
100   400 +
7  Profit of X = b
112·5 × 400 × 8 Profit of Y = 2b
= = Rs. 514·30
100 × 7 C.P. of X = a – b
8. Let the Print Price of both are the same. C.P. of Y = 2a – 2b
Let C.P. of sprite = Rs. P Since, we get profit of 12·5% in X.
Let C.P. of coke = Rs. Q 12·5 b
∴ = × 100
According the question, 1 a–b
95 75 90 80 25 b
P× × = Q× × Or, = × 100
100 100 100 100 2 a–b
Or, P × 19 × 15 = 4 × 9 × 8 × Q Or, 8b = a – b
P 4 × 9 × 8 3 × 32 a – 9b = 0 …(1)
Or, = =
Q 15 × 19 5 × 19 C.P. of Y is 500

ww
Or,
P
Q
=
96
95
96
Or,
2a – 2b = 500
a – b = 250 …(2)
Or,

w P =

Clearly, P is the greatest.


95

.Ea
·Q

9. Since, 2 bikes of A is free if dealer buys 5 bikes.


By putting the value of a in equation (2), we get

Or,
9b – b = 250
8b = 250
250
price of 3 bikes.
syE
So, If Dealer buys 4 bikes of A, then he pays only

Similarly, after buying 2 bikes of B dealer gets one


b =

a = 9×
8
250

ngi
8
bike extra.
Total C.P. = 3a – 3b
So, he pays only price of 2 bikes B.
= 3 (a – b) = 750
∴ Total C.P. of 4 bikes A + 3 bikes B
= 120000 + 100000 nee Total S.P. = 3a = 3 × 9 ×
250
8
= Rs. 2,20,000

S.P. of bikes = 220000 ×


112·5
rin
Profit = S.P. – C.P. = 27 ×
250
8
– 750

100
= Rs. 2,47,500
g.n
= 3 × 250 ×

Profit
1
8
3 × 250
10. Let C.P. of a bicycles = Rs. 100
Since profit = 120%
S.P. = Rs. 220
% profit =

=
C.P.
100
8
e × 100 =

= 12·50%.
8 × 3 × 250
× 100

t
Now , sale price of new bicycle 12. Ratio of profit X : Y = 12 : 15
60 = 4:5
= Rs. 220 × = Rs. 132
100 Now, share of profit = 5600 – 2000 = Rs. 3600
Profit = 132 – 100 = 32 Now the share of profit of X
192 = 3600 – 1600 = 2000
% profit = × 100
600 ∴ Therefore, required percentage
192 2000 10
= = 32% = × 100% = × 100
6 5600 28
Suppose initially they are selling n bikes after 250 5
= % = 35 %.
discount scheme the no. of sold bike = 6n bike 7 7
Initial Revenue = SP × n = 220 × n = 220n 13. Let cost price of Gita Car = Rs. X
New Revenue = SP × n = 132 × 5n = 660n Sita buys the car at
660 – 220 120
% Increase in Revenue = × 100 = 200% = Rs. ×X
220 100

68 | CAT Complete Course

Visit : www.EasyEngineering.net
Visit : www.EasyEngineering.net

Chandani buys the car at 16. Let marked price of house = Rs. 100
120 110 Now, after 20% and 10% discount
= Rs. ×X×
100 100 C.P. = (100 – 20) – 8 = Rs. 72
Mechanic buys the car at Since, Ram spent 10% in interior decoration
92 110 120
= Rs. × × ×X ∴ C.P. = 72 + 72 ×
10
= 79·2
100 100 100 100
Mechanic spent 12% on the car He want to earn 25% profit.
Now, price of car 125
So, S.P. = 79·2 × = 99
112 92 110 120 120 100
= × × × × ×X
100 100 100 100 100 When marked price is 100 … sale price = 1600000
∴ Mechanic sold the car at 1600000
∴ Marked price is 99 … sale price = × 99
112 92 110 120 120 100
= × × × × ×X
100 100 100 100 100 = Rs. 1,58,400
144 × 112 × 92 × 11
= ×X 17. Total cost of 4 cars = 2 + 4 = 6 Lakh
10000000

ww
Now, at last Gita buys the car
 16321536 
 10000000 – 1  × X
Total S.P. of 4 cars = 6 × 1·5 = Rs. 9 lakh
S.P. of one car = Rs. 3 lakh

w
So, Loss = Now, S.P. of 3 cars = Rs. 6 lakh
 
So, average S.P. of all the 3 cars = Rs. 2 lakh.
Loss =
.Ea
6321536
1000000
×X 18. Let C.P. of the 2 motorcycles = Rs. 2.X
S.P. = 1·2 ( 2.X)

syE
6321536
% loss = % = 63·21%. = 2·4X
100000
14. We know that 112
S.P. of 1st article = · X = 1·12X

ngi
100
Total salary = no. of woker × salary per workers
S.P. of 2nd article = 1‚12 X + 5000
Let initial workers = 3X

nee
According to question,
Let final workers = 2X
Initial salary = 13Y 2.4X = 1.12X + 5000 + 1·12X
Or, 2.4X = 2.24X + 5000

rin
Final salary = 15Y
According to question, Or, 0·16X = 5000
Total salary = 3X × 13Y = 39X.Y
Total salary = 2X × 15Y = 30X.Y.
X =
g.n
5000 500000
0·16
=
16
X = Rs. 31,250.

15.
% profit =

X
39X.Y – 30X.Y

Y
39X.Y
= 23.07 %.
× 100%

Z
19. Let
C.P.
S.P.
Book
a
c
e Pen
b
d
t
Investment a 3a 5a According to question,
5b 4b 3b 2a = b …(1)
Rate of return
100 100 100 Profit on selling books = 10(c – a) = 3d …(2)
5ab 12ab 15ab
Total = + + Loss on selling pens = 10 (b – d) = 4c
100 100 100
5 (b – d) = 2c …(3)
32ab
= 10 (c – a)
100 % profit on books = × 100
a
Z’s earning – Y’s earning = Rs. 510
10 (b – d)
15ab – 12ab 3ab % loss on pens = × 100
= = Rs. 510 b
100 100
32 × ab These are equal.
Total earning =
100 10 (c – a) 10 (b – d)
∴ × 100 = × 100
32 × 17000 10a 10b
= = Rs. 5,440.
100 Or, (c – a)b = a(b – d)

CAT Complete Course | 69

Visit : www.EasyEngineering.net
Visit : www.EasyEngineering.net

Or, putting value of b in above equation 23. Let Cost price = Rs. X
(c – a).2a = a(b – d) 3 3
Now, C.P. of th article = Rs. X
2c – 2a = b – d 4 4
Or, 2c + d = 2b …(4) 1 1
C.P. of th article = Rs. X
4 4
Now, solving equation (2) and (3), we get
5 3 3 140
c
= Now, S.P. of th article = × X
d 6 4 4 100
20. Set up cost = Rs. 12,000 3 7 21
= × X= X
4 5 20
80
Paper cost = Rs. × 2500 1 1
100 S.P. of th article = X
4 4
= Rs. 2,000
21 1
120 ∴ Total S.P. of the article = X+ X
Printing cost = Rs. × 2500 20 4
100
21X + 5X
= Rs. 3,000 S.P. =
20

ww Total cost = Rs. 17,000


Now, Sale price = Rs. 2000 × 10 =
26
20
X+ X
13
10

w
= Rs. 20,000 Profit = S.P. – C.P.
Let the amount obtained from adverstising is X, then

.Ea
∴ (20,000 + X) – 17,000 =
25
100
× 20,000
=
13
10
X–X=

Profit
3
10
X

Or,
syE
3000 + X = 5000
X = Rs. 2,000
% gain =
C.P.
3X
× 100

ngi
21. Let us suppose that pure milk = 100 of milk 10
= × 100 = 30%.
After mixing 25% water X
24. From question it is clear that

nee
Amount of milk = 100 + 25 = 125
10 Market price = Rs. 200
Again Amount of milk = 125 × + 125
100 S.P. of shirt = Rs. 160
= 125 + 12·5 = 137·5
Since, milk is sold at cost price. rin
Let first discount = Rs. X%
X
Let cost price of pure milk = Rs. Y
Since, cost price of 100 pure milk = Rs. Y g.n
Now, Ist discount = 200 ×
100
= 2X

2nd discount = (200 – 2X) ×


4
Now, selling price of 100 pure milk = Rs. Y

∴ Selling price of 137.5 pure milk =


137·5
100
Y =
e 25 

100
 200 – 2X 
 t
37·5
Profit = Y  200 – 2X 
100 Now, S.P. of the shirt = 200 –  2X + 
37·5 100  25 
% profit = Y× %
100 Y  50X + 200 – 2X 
160 = 200 –  
= 37·5 %  25
22. In such case, there is always a loss. 48X + 200
Or, 40 =
The selling price is immaterial. 25
 Common loss and gain  2 48X = 800
∴ % loss =  
 100  50
Or, X = %.
 25  5 
2 2
25 3
=   %=  %= %
 10  2  4 25. Let the labeled price = Rs. X
1 20 4
% loss = 6 % After concession price = X – X. = X
4 100 5

70 | CAT Complete Course

Visit : www.EasyEngineering.net
Visit : www.EasyEngineering.net

Since, Tarun gets 25% profit Now, Price = 180 – 9 = Rs. 171
4 125 When m.p. is 400
So, S.P. of the price = X× =X
5 100 15
Ist discount = 400 × = Rs. 60
X = 9000 100
So, labeled price = Rs. 9,000 2nd discount = 340 – 34 = Rs. 306
26. Let the marked price = Rs. X When m.p. = Rs. 800
When marked price is 150, then C.P. = 100 20
Ist discount = 800 × = Rs. 160
100 100
X marked price is 150, then C.P. = X
150 10
2nd discount = 640 × = Rs. 64
2 100
= X
3 Price = 640 – 64 = Rs. 576
25 X Now Difference = Maximum price
Now, Discount = X – =
100 4 – Mainimum price
X 3
Now, S.P. = X – = X …(1) = 576 – 171 = Rs. 405 …(2)
4 4

ww
When C.P. is X.
3
2
Now, Difference between equation (1) and equation
(2) is
600 – 405 = 195.

w S.P. =

.Ea
120 2
100 3
6 2
= × X= X
5 3
× X

5
4
…(2)
29. Let total weight of the alloy = a

Weight =
X
2

2
Y
2
a
Z
3 2
× a
Now,
syE
3
profit = X – X =
4
Profit
2
3

× 100
X
12 Ratio of C.P. 5
∴ Total C.P.
5 5 5
2
5 5
3

ngi
Now, % profit =
C.P.
2 
X =  5 a × 25 × 5 + 23 a × 2 + 5 6× 5 a × 3
 

nee
12
= × 100
2X 4 4 18 8 18
= a+ a+ a= a+ a
3 5 5 25 5 25

=
1 3
12 2
× × 100 = 12·5%. =
58
25
a
rin
27. Since, cost of one dozen is Rs. 375
cost of 20 dozens is Rs. 375 × 20 = Rs. 7,500
S.P. =
140 58
100 25
g.n
× a= × a
7 58
5 25

e
Now, Selling price of one dozen = Rs. 33 × 12 Due to changed market condition :
Selling price of 20 dozens = 33 × 12 × 20 = Rs. 7,920
Now, profit = 7920 – 7500 = Rs. 420

% profit =
420
7500
× 100
C.P. of Z =
2
C.P. of Y = a × 2 ×
5
18
25

120 4
100 5
125 9
= a
100 11
= a× = a
6 24
5 25
t
4 24 9
420 84 Now, Total C.P. = a + a + a
= = = 5·6% 5 25 10
75 15
40a + 48a + 45a 133
28. In initial stage : = = a
25 × 2 50
Difference = Maxmium – Minimum 406 133 812 – 665
Profit = a– a= a
= 800 – 200 = Rs. 600 125 50 25 × 2 × 5
After providing successive discounts of 10% and 5% 147
= a
on the price Rs. 200. 10 × 25
 Profit
10  % profit = × 100
Now, Discount = 200 –  200 ×  C.P.
 100 
147a
= 200 – 20 = Rs. 180 25 × 10
= × 100
5 133a
2nd Discount = 180 × = Rs. 9
100 50

CAT Complete Course | 71

Visit : www.EasyEngineering.net
Visit : www.EasyEngineering.net

147 21 420 Less 2% quantity discount on 5th widget = 10


= × 100 = × 20 =
5 × 133 19 19 ∴ Total price of 5 widgets = 423
2 423
= 22 %. Average price per widgets = = 84·6
19 5
30. Ram Shyam 32. Let there be a oranges on each tree, so there will be a
320 oranges trees in this garden.
Number of cows X
Y Total number of oranges = a2
245
S.P. / cow Y Total number of apples available = b2
X
According to question,
Since, both Ram and Shyam have the same amount
of money. a = 2b …(1)
Oranges available to store = 0.4a 2
320 245
∴ × = X.Y
Y X Apples available to store = 0·7b2
Or, (X.Y) 2 = 245 × 320 Hence, cost price = 0.4a2 × 20 + 0·7b2 × 10
Or, X.Y = 280 = 8a2 + 7b2 …(2)

ww
Given X+
320
Y
= 20
So, total oranges available for selling
= 0·75 (0·4a2)

Or,
w X+
320
280

X 1 +

X = 20

8
.Ea
 = 20
7
= 0·3a2
Total apples available for selling
6
= × 0·7b2 = 0·6b2
7

X = syE
7 × 20 7 × 4 28
15
=
3
=
3
∴ Selling price = 10 × 0·3a2 + 5 × 0·6b2
= 3a2 + 3b2
Now, Shyam sold each of cow at

=
245 105
= = 26·25 ngi Loss = C.P. – S.P.
= 5a2 + 4b2

31.
28
3
4

Price of 1st widget = 100


nee Now,
15000 = 5a2 + 4b2
2 2
15000 = 5(2b) + 4b [from equation (3)]
15000 = 24b2
…(3)

Price of 2nd widget = 90 b2 =


rin
15000
25
Price of 3rd widget = 90 – 90 ×

Price of 4th widget = 81


10
100
= 81
=
g.n
5000 2500
8
=
4

Price of 5th widget = 81


Now, total = 433
b =
50
2
a = 2b = 50
e
= 25

●●●
t

72 | CAT Complete Course

Visit : www.EasyEngineering.net
Visit : www.EasyEngineering.net

5 Average

Average Solution :
By the above definition average age
The numerical result obtained by dividing the sum of
two or more quantities by the number of quantities is 20 + 22 + 18 + 24 84
= = = 21 years
called Average . 4 4
Therefore, average age of student = 21 years
An arithmetic mean of given observations is called
Average . Weighted Average

ww
Average is defined in so many ways we can say
average mean Usual or Normal kind, amount, quality,
rate, etc.
The concept of weighted Average is used when we
have two or more groups whose individual averages age
are known.

w
Average is a number or value of a set of values

.Ea
carefully defined to typify the set, as a median or mode.
Average refers to the result obtained by dividing a
sum by the number of quantities added. For example,
Suppose in a class, there are 2 student of 20 years,3
of 21 years ,4 of 22 years and 5 of 23 years, then their
average age is given by
(2 × 20) + (3 × 21) + (4 × 22) + (5 × 23)

the average of 15, 12, 27 is


syE
15 + 12 + 27 54
3
=
3
= 18 and in
extended use is applied to the usual or ordinary kind,
=
2
14
2+3+4+5
3 4
× 20 + × 21 + × 22 + × 23
14 14 14
5

ngi
306
instance, etc. = years
14
Average is different from mean and median. 2 3 4 5

nee
Here, , , and are called the weights of
14 14 14 14
The Average of a given set of numbers is a measure each category of students.
of the central tendency of the set. In other words, it is the Illustration 2.
mean value of a set of numbers or values. Therefore,
average of a set of numbers is given by
rin
What is the average concentration of a mixture if 3L
of 36 % sulphuric acid is added to 9L of 24% sulphuric
Average =
(x 1 + x 2 + x 3 + … + x n )
n
acid solution ?
Solution : g.n
Or in other words average of some observations

=
Sum of all observations
Number of observations
Mean commonly designates a figure intermediate
is the weighted average

=
3 
 
e
The average concentration of the combined mixture

9 
 12  × 36 +  12  × 24
 
t
between two extremes. For example, the mean tempera- = 9 + 18 = 27%
ture for a day with a high of 24°C and a low of 28°C is In other words, weights are the fraction of the number
24 + 28 in that category with respect to the total students in that
= 26°C and the median is the middle number or
2 class. This average is also called the weighted average of
point in a series arranged in order of size i.e., the median that class.
grade in the group 50, 55, 85, 88, 92 is 85; the average is Average Speed
74.
If a (body ) certain distance is covered in parts at
Norm implies a standard of average performance for different speeds , the average speed is given by
a given group i.e., a child below the norm for his age in Total distance covered
reading comprehension. Average speed =
Total time taken
Illustration 1. As, if a body travels d1 , d2 , d3 , …… dn distance, with
In a class, the age of four students are 20 years, 22 speed s1 , s2, s3, …… sn in time t1 , t2 , t3 , .… tn respectively,
years, 18 years, and 24 years, then what is the average then the average speed of the body through the total
age of the student of class ? distance is given by

CAT Complete Course | 73

Visit : www.EasyEngineering.net
Visit : www.EasyEngineering.net

Total distance travelled Solutions :


Average speed =
Total time taken Total weight reduced of 6 men = 6 × 3 = 18 kg
Always remember that, Average speed This weight of the group is reduced because the man
Sum of speeds s + s2 + s3 + … + sn weighing 80 kg is replaced by a man who is 18 kg lighter
≠ ≠ 1
Number of different speeds n than him. Therefore, weight of new man = (80 – 18) = 62
d + d2 + … + dn kg.
Average speed = 1
t1 + t2 + … + tn Runs and Average
s t + s2 t2 + … + sn tn Illustration 6.
= 11
t1 + t2 + t3 + … + tn
A cricketer has a certain average of 9 innings. In the
d + d2 + d3 + … + dn tenth inning he scores 100 runs, thereby increasing his
= 1
d1 d2 d3 d average by 8 runs. Calculate his new average.
+ + +…+ n
s1 s2 s3 sn Solution :
Short cut Let the average of 9 innings be x runs, hence new
If you travel equal distance with speeds u and v, then average will be (x + 8) runs.

ww
the average speed over the entire journey is
2uv
(u + v)
·

If a man changes his speed in the ratio m : n, then the


Total runs scored for 9 innings = 9x
Total runs scored for 10 innings = (9x + 100)
Total runs

w
Average for 10 innings =
ratio of times taken becomes n : m. 10
Illustration 3.

.Ea
Let the distance between two points A and B is d and
speed in travelling from point A to B is u km/hr and from


(x + 8) =
(9x + 100)

x = 20
10

point B to A is v km/hr.
Solution : syE Therefore, new average = (20 + 8) = 28 runs
Average of Some Important Series of Num-
bers
Then, average speed =
Total distance
Total time
2d ngi (a) The average of odd numbers from 1 to n is
(n + 1)
=
d d
+
u v
If two speeds are given as u km/hr and v km/hr, then
nee 2
, when n = last odd number
(b) The average of even numbers from 2 to n is
(n + 2)
Average speed (distance being same)
2uv
2
rin
, when n = last even number
(c) The average of square of natural numbers till

Illustration 4.
=
u+v n=
n (n + 1)(n + 1)
6n
g.n
If a person travels two equal distances at 10 km/hr
and 30 km/hr .What is the average speed for the entire
journey ?
Solution : is =

(n + 1)(2n + 1)
6
e
(d) The average of cubes of natural numbers till n
n2 (n + 1)2
4n
t
2 × 30 × 10 n (n + 1)2
Average speed = ⇒
30 + 10 4
600 (e) The average of first n consecutive even num-
= = 15 km/hr.
40 bers is (n +1)
Age and Average (f) The average of first n consecutive odd num-
bers is n.
If the average age of n persons decreases by x years.
(g) The average of squares of first n consecutive
Then, the total age of n persons decreases by (n × x) years.
2(n + 1)(2n + 1)
Also, if the average age of n persons increase by x years. even numbers is
3
Then, the total age of n persons increases by (n × x)
(h) The average of squares of consecutive even
years.
(n + 1)(n + 2)
Illustration 5. numbers till n is
3
The average weight of 6 men decrease by 63 kg (i) The average of squares of consecutive odd
when one of them weighing 80 kg is replaced by a new n (n + 2)
numbers till n is
man. Calculate the weight of the new man. 3

74 | CAT Complete Course

Visit : www.EasyEngineering.net
Visit : www.EasyEngineering.net

Illustration 7. Solution :
What is the average of odd numbers from 1 to 25 ? 12(12 + 2)
Average =
Solution : 3
25 + 1 12 × 14
Average = = 13 = = 56
2 3
Illustration 8. Exercise A
What is the average of even numbers from 1 to 40 ? 1. The average of 5 quantities is 10 and the average of 3
Solution : of them is 9. What is the average of the remaining 2 ?
40 + 2 (A) 11·0 (B) 12·0
Average = = 21
2 (C) 11·5 (D) 12·5
Illustration 9. 2. The average of 5 quantities is 6. The average of 3 of
What is the average of square of natural numbers them is 8. What is the average of the remaining two
from 1 to 20 ? numbers ?
Solution : (A) 6·5 (B) 4

ww Average =

Illustration 10.
(20 + 1)(40 + 1)
6
= 143·5 (C) 3 (D) 3·5
3. The average mark in a group of 25 students on a test

w
is reduced by 2 when a new student replaces the
What is the average of cubes of natural numbers topper who scored 95 marks. How many marks did
from 1 to 5 ?
Solution : .Ea the new student have ?
(A) 90 marks (B) 50 marks
Average =

Illustration 11. syE


5(5 + 1)2
4
= 45 (C) 45 marks (D) 95 marks
4. The average of marks obtained by 120 candidates in
a certain examination is 35. If the average marks of
What is the average of first 49 consecutive even
numbers ? ngi passed candidates is 39 and that failed candidates is
15. What is the number of candidates who passed the
Solution :
Average = 49 + 1
nee examination ?
(A) 65
(C) 100
(B) 40
(D) 35

rin
Illustration 12.
What is the average of first 19 consecutive odd 5. The average of 11 results is 50. If the average of first

g.n
numbers ? six results is 49 and that of last six is 52, find the
Solution : sixth result—
(A) 55 (B) 45

e
Average = 19
Illustration 13.
What is the average of square of first 10 consecutive
even numbers ?
Solution :
(C) 56 (D) 38
6. The average weight of 8 men is increased by 1·5 kg.
when one of the men who weights 65 kg is replaced
by a new man. The weight of the new man is—
t
2(10 + 1)(20 + 1) (A) 76 kg (B) 76·5 kg
Average =
3 (C) 76·7 kg (D) 77 kg
2 × 11 × 21
= = 154 7. The average of marks in Mathematics for 5 students
3
Illustration 14. was found to be 50. Later on it was discovered that in
the case of one student the marks 48 were misread as
What is the average of square of consecutive even 84. The correct average is—
numbers till 10 ?
(A) 40·2 (B) 40·8
Solution :
(C) 42·8 (D) 48·2
(10 + 1)(10 + 2) 11 × 12
Average = = = 44
3 3 8. The average of 7 consecutive numbers is 33. The
Illustration 15. highest of these numbers is—
What is the average of square of consecutive odd (A) 28 (B) 30
numbers till 12? (C) 33 (D) 36

CAT Complete Course | 75

Visit : www.EasyEngineering.net
Visit : www.EasyEngineering.net

9. The average monthly salary paid to 75 employees in weighted average of marks scored by Sohan in these
a company is Rs. 1420. The average salary of 25 of five subjects ?
them is Rs. 1350 and that of 30 others is Rs. 1425. (A) 60 (B) 62
The average salary of the remaining employees is— (C) 72 (D) 74
(A) Rs. 1350 (B) Rs. 1425
5. The average of eight numbers is 50, that of the first
(C) Rs. 1500 (D) Rs. 1475 four is 40 and the next three is 25. The eight number
10. Four ropes of increasing length are provided. Their is less than the ninth by 7, and less than the tenth by
average length is 74 inches, and the difference in 12. Then, tenth number is—
length amongst the first three ropes is 2 inches. The (A) 81 (B) 93
difference between the third and the fourth is 6 (C) 91 (D) 90
inches. Thus, the longest rope is—
6. The average amount of sales in a shop per day for 5
(A) 72 inch (B) 80 inch
days from Monday is Rs1400. The average amount
(C) 74 inch (D) 70 inch of sales per day for 5 days from Tuesday is Rs1490.
11. The average age of a committee of a members is 40 By how much does the sale on Saturday exceed that
years. A member aged 55 years retired and his place on Monday?

ww
was taken by another member aged 39 years. The
average age of the present committee is—
(A) Rs. 400
(C) Rs. 450
(B) Rs. 500
(D) Rs. 600

w
(A) 39 years (B) 38 years
7. The average age of a class of 20 students is 25 years.
(C) 36 years (D) 35 years

.Ea
12. The average of 5 quantities is 10 and the average of
3 of them is 9. What is the average of the remaining
The average increases by 1 when the teacher's age is
also included. What is the teacher's age ?
(A) 40 years (B) 46 years
2?
(A) 11 syE
(B) 12
(C) 20 years (D) 25 years
8. A batsman has a certain average of runs in 15

ngi
(C) 11·5 (D) 12·5 innings. In the 16th inning, he makes a score of 90
Exercise B runs, there by increasing his average by 2. What is
the average after the 16th inning ?
1. The average weight of a class of 33 students is 48.5
kg. If the weight of the teacher is included, then the
average rises by 500 gm. The weight of the teacher
nee (A) 54
(C) 56
(B) 60
(D) None of these
is—
(A) 66·5 kg (B) 65·5 kg rin
9. In ascending order, the correct sequence of mean,
mode and median for 3, 7, 9, 11, 5, 2, 11, 7, 4, 6, 7,
(C) 63·5 kg (D) 69 kg
5, 3, 11, 3, 7 is—
(A) mean, median mode g.n
2. The average age of a couple was 27 years at the time
of their marriage. After 20 years of marriage, the
average age of the family with 2 children became 30
years. What is the average age of the children ?
(B) median, mean, mode
(C) mode, median, mean,
(D) median, mode, mean
e t
(A) 23 years (B) 11 years 10. There are five persons A, B, C, D, and E. A weighs
(C) 13 years (D) 26 years twice as much as B. Weight of B is 50% of the
weight of C. Weight of D is 40% of that of E. Weight
3. The average temperature at Delhi for Monday, of E is 125% of the weight of A. The person with
Tuesday and Wednesday was 27°C while for least weight is—
Tuesday, Wednesday and Thursday, it was 25°C. If
(A) C (B) E
the temperature on Thursday was 26°C, then the
temperature on Monday was— (C) B (D) A
(A) 27°C (B) 21°C 11. The average of N numbers is X. If one of the
numbers A is replaced by another one B, the new
(C) 30°C (D) 32°C
average is Y. The new number B equals—
4. Sohan scores 80, 40, 90, 50 and 60 per cent marks in (A) NY – NX – A
the subject Hindi, English, Maths, History and
(B) Y – X
Chemistry respectively. However, the weights
attached to each of these subjects are 4, 3, 1, 2, and 5 (C) N(Y – X) + A
respectively. Which of the following represents the (D) (Y – X + A)/N

76 | CAT Complete Course

Visit : www.EasyEngineering.net
Visit : www.EasyEngineering.net

Directions—(Q. 12 and 13) Read the given data to 4. (C) Sum of all marks obtained
answer the question that follow. A batsman's average = Average × Number of students
score for a certain number of innings was 22.5 per inning.
= 35 × 120 = 4200
He played 2 innings more and scored 42 and 46 runs
respectively, thus increasing his average by 0.5. Let the number of passed candidates is x.
12. How many innings did he play in all ? Then, Sum of marks obtained by passed candidates
= 39 × x = 39x
(A) 86 (B) 75
Sum of marks obtained by failed candidates
(C) 42 (D) 27
= 15 × (120 – x)
13. What was his total score?
= 1800 – 15x
(A) 966 (B) 1935
[Number of failed candidates = 120 – x]
(C) 1978 (D) None of these
Now obviously, Sum of total marks obtained = Sum
14. What is the average amount of wine in 10 additional of marks obtained by passed candidates + Sum of
casks, which when added to 30 casks with average marks obtained by failed candidates
capacity of 18 litres, increases the average capacity ⇒ 4200 = 39x + 1800 – 15x

ww
to 90 litres ?
⇒ 4200 – 1800 = 39x – 15x
(A) 300 litres (B) 306 litres
⇒ 2400 = 24x
(C) 210 litres (D) None of these

w
15. The average of two numbers increases by 25 when

.Ea
one of the numbers is doubled and increases by 30,
when the other number is doubled. What must be the
original average ?

100 candidates passed.


x =
2400
24
= 100

5. (C) The total sum of 11 result = 11 × 50 = 550


(A) 50
(C) 55·5 syE
(B) 55
(D) None of these
The total sum of first 6 result = 6 × 49 = 294
The total sum of last 6 result = 6 × 52 = 312

Answers
Exercise A ngi 6. (D)
Sixth result = 294 + 312 – 550 = 56.
65 + 1·5 × 8 = 77
50 × 5 – 84 + 48 250 – 84 + 48
1. (C) The average of 5 quantities is 10. Therefore, the
sum of all '5 quantities is 56. nee
7. (C)
5
=

214
5

rin
= = 42·8.
The average of 3 of them is 9. Therefore, the sum of 5
the 3 quantities is 27. Therefore, the sum of the 8 (C) (x – 3) + (x – 2) + (x – 1) + x + (x + 1) + (x + 2)
remaining two quantities = 50 – 27 = 23.
Hence, the average of the 2 quantities = 23/2 = 11·5.
+ (x + 3) = 33 × 7
∴ g.n
7x = 33 × 7
2. (C) The average of 5 quantities is 6.
Therefore, the sum of the 5 quantities is 5 × 6 = 30.,
The average of three of these 5 quantities, is 8.
Therefore, the sum of these three quantities = 3 × 8
9 (C) e
x = 33
1420 × 75 – 25 × 1350 – 30 × 1425
20
= 1500.
t
10. (B) Let the length of the ropes be x, x + 2, x + 4
= 24. (difference in lengths of the first three ropes being 2
The sum of the remaining two quantities = 30 – 24 inches) and x + 10 (difference in lengths of the third
= 6. and the fourth ropes being 6 inches).
Average of these two quantities = 6/2 = 3. Thus, 4x + 16 = 74 × 4
3. (C) Let initial average be x, then the initial total is
⇒ x = 70 inches.
25x
Hence, the length of the longest piece of rope is 80
New average will be x – 2 and the new total will be
inches (x + 10).
(x – 2) × 25 = 25x – 50
40 × 8 – 55 + 39
The reduction of the 50 is created by the replace- 11. (B) = 38.
8
ment.
12. (C) The average of 5 quantities is 10.Therefore, the
Hence, the new student has 50 marks less than the
sum of all 5, quantities is 50.
topper.
The new student's marks = Topper's marks – the The average of. 3 of them is 9.
difference in their marks be 95 – 50 = 45 Therefore, the sum of the 3 quantities is 27.

CAT Complete Course | 77

Visit : www.EasyEngineering.net
Visit : www.EasyEngineering.net

Therefore, the sum of the remaining two quantities = 8. (B) Let the average for 15th inning be X.
50 – 27 = 23. 15X + 90
⇒ = X + 2 (Given).
Hence, the average of the 2 quantities = 23/2 = 11·5. 16
Exercise B ⇒ 15X + 90 – 16X = 32
⇒ – X = – 58
1. (B) Sum of weight of 33 students
⇒ X = 58
= 33 × 48·5 = 1600·5 kg
Requires average = 58 + 2 = 60.
Let the weight of teacher = x kg
3+7+…+3+7
Sum of weight of class (including teacher) 9. (A) Mean = = 6·3125. Mode = 7.
16
1600·5 + x
= Now in ascending order we can write, the given
34
numbers as
⇒ x = 65·5 kg.
2, 3, 3, 3, 4, 5, 5, 6, 7, 7, 7, 7, 9, 11, 11, 11.
2. (C) Sum of the age of couple = 27 × 2 = 54. 8th term + 9th term
Sum of the age of couple = 47 × 2 = 94. ∴ Median = = 6·5.
2
Total ages of all family = 30 × 4 = 120

ww
Sum of ages of children = 26 years
26
10. (C) A's weight is 2 times that of B, so weight of A is
not the least.
B's weight is 50% of that of A, so weight of C is not

w
Average required = = 13 years.
2 the least.

Tuesday and Wednesday


.Ea
3. (D) Sum of temperature at Delhi on Monday,

M + T + W = 3 × 27 = 81 …(1)
D's weight is 40% of that of E, so weight of E is not
the least.
Hence, the only possible answer is B or D. Let a, b,

From equations (1) and (2) syE


T + W + Thus. = 3 × 25 = 75 …(2) c, d and e stand for the weight of A, B, C, D and E
respectively.

ngi
2
M – Thus = 6 Then, d = 40% of e = e. But e = 125% of a
5
M = 6 + 26 = 32° or e = 1·25a
So, temperature on Monday was 32°C.
4. (B) Average requires
(80 × 4) + (40 × 3) + (90 × 1)
nee 2
So, d = × 1·25a = 0·5a. Since, a = 2b, so b =
5
1
2

rin
a = 0·5a.
+ (50 × 2) + (60 × 5) Hence, b = d. So, the least weight is that of B and D.
=
(4 + 3 + 1 + 2 + 5)

=
320 + 120 + 90 + 100 + 300
15 g.n
11. (C) Total of numbers = NX. New Total
= NX – A + B
New Average = Y = New Total ÷ N
=
930
15
= 62 marks.

5. (B) Sum of tenth results = 10 × 50 = 500


⇒ 500 = (40 × 4 + 3 × 25 + X + X + 10 + X + 12)


e
NY = NX – A + B
B = N (Y –X) + A.
12. (A) Let he play n innings
t
⇒ 500 = 257 + 3X 22·5x (n – 2) + 42 + 46
= = 23
n
or X = 81
Solving n = 86.
Required last not = 81 + 12 = 93.
13. (C) Total score = 86 × 23 = 1978.
6. (C) M + T + Wed + Th + F = 1400 × 5 …(1)
14. (B) 10X + 30 × 18 = 40 × 90
T + W + Th + F + S = 1490 × 5 …(2)
10X = 3600 – 540
Solving equations (1) and (2)
10X = 3060 litres.
S – M = 90 × 5 = Rs. 450.
X = 306 litres.
7. (B) Sum of ages of students = 20 × 25 = 500 years
15. (C) When one of the numbers is doubled, the average
Let teacher's age = x.
is increased by 2. So, first number must be 50. (25 ×
500 + x 2 = 50). Similarly, second number must be 60.
= = 26
21 50 + 60
= 500 + x = 546 So, the original average is = = 55·5.
2
= x = 46 years. ●●●

78 | CAT Complete Course

Visit : www.EasyEngineering.net
Visit : www.EasyEngineering.net

6 Height and Distance


(A) Pythagoras Theorem—This theorem is appli- observation at a higher level than the horizontal line OX.
cable for a right angled triangle. In right angled triangle Then, the angle XOA is called the angle of elevation.
there are three basic terms—
(i) Hypotenuse—It is the opposite side of the right
angle in right-angled triangle. It is the longest side of
right-angled triangle.
(ii) Height and

ww
It is called the angle of elevation because the observer
(iii) Base
has to elevate (raise) his line of sight from the horizontal
The other sides of the right line OX to see the object A.
angled triangle are called the

w
legs. One of these is height and

.Ea
the other one base of the triangle.
∴ Hypotensue = AC
(ii) Angle of Depression—If ‘O’ is the observer
‘OX’ is the horizontal line through ‘O’ and ‘A’ is the
object of observation at a lower than the horizontal line,
then the angle XOA is called the angle of depression.
Height = AB
Base = BC
syE
(Hypotensue)2 = (Height)2 + (Base)2
AC2 = AB2 + BC2
(B) Trigonometric Ratios— ngi
The most important properties of
trigonometric is to find the re-
maining sides and angles of a
nee Since, the observer has to depress his line of sight
from the horizontal line OX to see the object, so it is
called the angle of depression.
triangle, when some of its sides
and angles are given. rin
(iii) Angle Subtended by an
Object—If the observer is at ‘C’
Consider a right angle tri-
angle ABC such that g.n
and object is ‘AB’. Now, the angle
subtracted by the object ‘AB’ at
‘C’ i.e., at the observer’s eye, then
AB = P, AC = h, BC = b and ∠B = 90°

sin θ =
Perpendicular
Hypotenuse
=
P
h
e
it is called angle subtracted by the
object. Here, ‘θ’ is the subtracted
angle of an object.
(D) Some Result Useful in Finding Height and
t
Base b
cos θ = = Distance—
Hypotenuse h
Perpendicular P (i) Sine Rule—
tan θ = = a b c
Base b = =
sin A sin B sin C
Base b
cot θ = =
Perpendicular P
Hypotenuse h
sec θ = =
Base b
Hypotenuse h
cosec θ = =
Height P
(C) Important Terms Related to Height and Dis-
tance—
(i) Angle of Elevation—If ‘O’ is the observer, ‘OX’ Here, a = length of side BC which is opposite to
is the horizontal through ‘O’ and ‘A’ is the object of point A and here A, B, C represent angles.

CAT Complete Course | 79

Visit : www.EasyEngineering.net
Visit : www.EasyEngineering.net

(ii) In Any Triangle ABC— 6. From the top and bottom of a building of height h
metres the angles of elevation of the top of a tower
are α and β respectively. Prove that the height of the
h tan β
tower is ·
tan β – tan α
7. The angle of elevation of the top of a tower as seen
from the point A and B situated in the same line and
at a distance p and q respectively from the foot of the
If AD divides the angle A into two parts α and β such tower are complementary. Prove that the height of
that the tower is √pq.
BD m
= 8. From the top of a building 15 m. high, the angle of
DC n
elevation of the top of a tower is found to be 30°.
∠ADC = θ From the bottom of the same building, the angle of
Then, (m + n) cot θ = m cot α – n cot β elevation of the top of the tower is found to be 60°.
(iii) In a right angled triangle XOY if AB || XY, then Find the height of the tower and distance between the
tower and the building.

ww
AB OA OB
= = 9. From the top of a tower 50 m. high the angles of
XY OX OY
depression of the top and bottom of pole are
observed to be 45° and 60° respectively. Find the

w .Ea
height of the pole, and the tower stand in the same
plane.
10. The shadow of a vertical tower on level ground

syE
increases by 10 metres, when the latitude of the sun
changes from angle of elevation 45° to 30°. Find the
Exercise A height of the tower, correct to one place of decimal.
(Take √ 3 = 1·73)
1. A person observed the angle of elevation of the top
of a tower as 30°. He walked 50 m. towards the foot
ngi 11. A round balloon of radius ‘a’ subtends an angle θ at
the eye of observer while the angle of elevation of its

nee
of the tower along level ground and found the angle
of elevation of the top of the tower as 60°. Find the centre is φ Prove that the height of the centre of the
height of the tower. θ
balloon is a sin φ cosec ·

rin
2
2. A ladder rests against a wall at an angle α to the hori-
zontal. Its foot is pulled away from the wall through 12. A pole 5 m. high is fixed on the top of a tower. The

g.n
a distance ‘a’, so that it slides a distance ‘b’ down the angle of elevation of the top of the pole observed
wall making an angle β with the horizontal. Show from a point ‘A’ on the ground is 60° and the angle
a cos α – cos β of depression of the point A from the top of the

e
that =
b sin β – sin α tower is 45°. Find the height of the tower.
3. The angle of elevation of the top of a tower from a
point A on the ground is 30°. On moving a distance
of 20 metres towards the foot of the tower to a point
B, the angle of elevation increases to 60°. Find the
13. An aeroplane, when 3000 m. high, passes vertically
t
above another aeroplane at an instant when the angles
of elevation of the two aeroplane from the same point
on the ground are 60° and 45° respectively. Find the
height of the tower and the distance of the tower vertical distance between the two aeroplanes.
from the point A. 14. From a window (h metres high above the ground) of
4. The angle of elevation of an aeroplane from a point P a house in a street, the angle of elevation and depres-
on the ground is 60°. After a flight of 15 seconds, the sion of the top and the foot of another house on the
angle of elevation changes to 30°. If the aeroplane is opposite side of the street are θ and φ respectively.
Show that the height of the opposite house is h (1 +
flying at a constant height of 1500√  3 m, find the
tan θ cot φ).
speed of the aeroplane.
5. The angle of elevation θ, of an vertical tower from a 15. An aeroplane flying horizontally 1 km above the
5 ground is observed at an elevation of 60°. After 10
point on the ground is such that its tangent is · On seconds, its elevation is observed to be 30°. Find the
12
walking 192 m. towards the tower in the same speed of the aeroplane in km/hr.
straight line, the tangent of the angle of elevation is 16. A man on the deck of a ship is 12 m above water
3 level. He observes that the angle of elevation of the
found to be · Find the height of the tower.
4 top of a cliff is 45° and the angle of depression of the

80 | CAT Complete Course

Visit : www.EasyEngineering.net
Visit : www.EasyEngineering.net

base is 30°. Calculate the distance of the cliff from 27. A vertically straight tree, 15 m high, is broken by
the ship and the height of the cliff. wind in such a way that its top just touches the
17. From the top of a hill, the angles of depression of ground and makes an angle of 60° with ground. At
two consecutive stones, 1 kilometre apart, due east, what length above the ground did the tree break?
are found to be 30° and 45°. Find the height of the (Use √ 3 = 1·73)
hill.
Exercise B
18. The angle of elevation of the top of a tower from a
point on the same level as the foot of the tower is α. 1. We have a right angle triangle ABC whose ∠B = 90°
On advancing ‘p’ metres towards the foot of the and sides are BC = 3, AB = 4 and ∠ACB = θ, then
tower, the angle of elevation becomes β. Show that find AC and sin θ, cos θ, tan θ—
p tan α. tan β 2 4
the height ‘h’ of the tower is given by = · (A) (B)
tan β – tan α 3 3
Also, determine the height of the tower if p = 150 m. 1 3
(C) (D)
α = 30° and β = 60°. 2 4
19. From the top of a building, 60 metres high, the (E) None of these
angles of depression of the top and the bottom of a 2. The string of a kite is 100 metres long and it make

ww
vertical lamp-post are observed to be 30° and 60°
respectively. Find (i) the horizontal distance between
the building and the lamp post, (ii) The difference
angle of 30° with the horizontal. Find the height of
the kite—
(A) 15 metre (B) 25 metre

20.
w
between the height of the building and the lamp post.

.Ea
A fire at a building B is reported on telephone to two
fire stations F 1 and F 2 10 km apart from each other.
(C) 50 metre
(E) 60 metre
(D) 35 metre

F 1 observes that the fire is at angle of 60° from it and 3. From a tower 250 high, the angle of depression of a

syE
bus is 60°. Find how far the bus is from the tower ?
F 2 observes that it is at angle of 45° from it. Which
(A) 125 metre (B) 500 metre
station should send its team and how much distance
250
it has to travel ? (C) 250√  3 metre (D) metre
21. A person standing on the bank of a river observes
that the angle of elevation of the top of a tree
ngi (E) 250 metre
3

nee
standing on the opposite bank is 60°. When he 4. If the angle of elevation of the sun changed from 30°
moves 40 m. away from the bank, he finds the angle to 45°, then find the difference between the lengths
of elevation to be 30°. Find the height of the tree and of shadows of a pole 30 metre high, made at these

22.
the width of the river.
A vertical tower stands on a horizontal plane and is
positions—
(A) 38 metre
rin (B) 12 metre

g.n
surmounted by a vertical flagstaff of height 5 metres. (C) 16 metre (D) 22 metre
At a point on the plane, the angles of the elevation of (E) 30 metre
the bottom and the top of the flagstaff are respec-

23.
tively 30° and 60°. Find the height of the tower.
A tower is 50 m high. Its shadow is x m shorter when
the Sun’s altitude is 45°, then when it is 30°, find x
correct to the nearest cm.
e
5. The angle of elevation of the top of a tower 50 metre
high, from two points on the level ground on its
opposite sides are 60° and 30°. Find the distance
between the two points— t
(A) 400 metre (B) 100 metre
24. From the top of a tower, the angles of depression of
200
two objects on the same side of the tower are found (C) metre (D) 200 metre
to be α and β (α > β). If the distance between the 3

objects is ‘p’ metres, show that the height ‘h’ of the (E) 200√  3 metre
p tan α. tan β
tower is given by h = · 6. A vertical tower stands on a horizontal plane and is
tan α – tan β
surmounted by a vertical flog staff of height 10
25. A man on the deck of a ship is 16 m above water metre. At a point on the plane, the angle of elevation
level. He observes that the angle of elevation of the of the bottom and the top of the flag staff are 45° and
top of a cliff is 45°, and the angle of depression of 60° respectively. Find the height of the tower—
the base is 30°. Calculate the distance of the cliff
10
from the ship and the height of the cliff. (A) metre (
(B) 5 √ )
 3 – 1 metre
26. The angle of elevation of the top Q of a vertical 
√ 3 + 1
tower PQ from a point X on the ground is 60°. At a (
(C) 5 √ )
 3 + 1 metre (D) 10 metre
point Y, 40 m above X, the angle of elevation is 45°.
Find the height of the tower PQ and the distance XQ. (
(E) 10 √ )
 3 + 1 metre

CAT Complete Course | 81

Visit : www.EasyEngineering.net
Visit : www.EasyEngineering.net

7. From an observation tower 50 metre above the level what is the distance of the boat from the foot of the
of river, the angle of depression of a point on the light house ?
near shore is 30° and that of a point directly beyond √
 3 + 1 metre
on the far shore is 60°. Find the width of the river— (A) 60 × 
100  3 – 1 
√
(A) metre (B) 100 metre
3
√ √
 3 – 1 metre
(B) 60 × 
(C) 100√
 3 metre (D) 50 metre  3 + 1
√
(E) 200 metre
(C) 60 × 
1  metre
8. From a balloon rising vertically, a person observes 
√ 3 – 1
two consecutive kilometre stones on the same sides
of a straight road and finds their angles of depressions √ 3 + 1 metre
(D) 60 × 
 3 
to be 30° and 60°. Find the altitude of the balloon—
 √
 3 km

(A) (B)  3 km
√ (E) 60√
 3 metre
2

ww 1
(C) km
2
2
(D) 1 km Answers with Hints
Exercise A
(E)

w3

km

.Ea
9. The angle of elevation of a jet plane from a point on
the ground is 60°. After a flight of 18 seconds, the
1. Suppose height of tower AB = x m.
Distance BC = y m.

syE
angle of elevation changes to 30°. If the jet plane is
flying at a constant height of 2000
speed of the plane in km/hr—
√ 3 metre. Find the

(A) 800 km/hr


(C) 600 km/hr
(B) 700 km/hr
(D) 500 km/hr ngi
(E) 400 km/hr
10. A tower sub stands an angle θ1 at the point A on the nee In rt. ∆ ABC,
AB
= tan 60°
same level, as the foot of the tower, and at a second
point B. A metre vertically above A the depression of
rin
BC
x
= √
3
the foot of the tower is θ 2 . Find the height of the
tower—
(A) a tan θ1 cot θ2 (B) a tan θ2 cot θ1
y

y = g.n
x
…(1)

(C) a

(E) a
tan θ1
cot θ2
(D) a
tan θ2
cot θ1
In rt. ∆ ABD,
AB
DB
3

e
= tan 30° t
11. Two pillars of equal height are on either side of a x 1
=
road, which is 100 metre wide. The angles of eleva- 50 + y 3

tion of the top of the pillars are 60° and 30° at a point
on the road between the pillars. Find the position  3x = 50 + y
√ …(2)
of two pillars and height of each pillar— Putting the value of y in (2), we get

(A) 25√  3 metre (B) 50


1
metre  3x = 50 + x

3
√ 3

 3x = 50√
1
(D) 50 √ 3 metre 3+ x
(C) 25 metre ⇒ √

3 3

(E) 50 metre
⇒ 3x = 50√
3+ x
12. A light house is 60 metres high with its base being at
the sea level, if the angle of depression of a boat in ⇒ 3x – x = 50√
3
the sea from the top of the light house is 15°, then ⇒ 2x = 50 (1·73)

82 | CAT Complete Course

Visit : www.EasyEngineering.net
Visit : www.EasyEngineering.net

⇒ 2x = 86·50 m 3 = h

⇒ x = 43·25 m. 1 x
2. Let the ladder be AC making an angle α with the h = x√3 …(1)
horizontal. On pulling, the ladder comes in the posi-
In rt., ∆ ACD,
tion A′ C′ making an angle β with the horizontal.
h
A′ A′ = a, C′ C = b tan 30° =
x + 20
1 x√3
= [From (1)]
3
√ x + 20
3x = x + 20
2x = 20
a AD A′ D
In ∆A′ AD = = x = BC = 10 cm.
sin (α – β) sin β sin (180° – α)
a sin β Taking the value of x in (1),
Here, AD =
sin (α – β) h = x
√3

ww A′D =
a sin α
sin (α – β)
[Q sin (180° – α) = sin α]
= (10) (1·73) [Q 
= 17·3 m.
√ 3 = 1·73]

w
In ∆ CC′ D
b
sin (α – β)
.Ea
=

=
C′ D
sin (90° – α)
CD
∴ Height of tower = 17·3 m.
and distance of tower from A
= AC = 20 + x

C′ D = syE
sin (90° + β)
b cos α
sin (α – β)
= 20 + 10 = 30 m.
4. Let the initial position of the aeroplane be A and
after 15 seconds it reaches B.
CD =
b cos β
sin (α – β)
ngi Let PM = x m and MN = y m

[Q sin (90° – α) = cos α and sin (90° + β) = cos β]


Length of ladder nee


a sin β
AD + CD = A′ D + C′ D

+
b cos β
=
a sin α
rin
sin (α – Β) sin (α – β) sin (α – β)

+
b cos α
sin (α – β)
In right ∆ AMP,
g.n AM
a sin β + b cos β = a sin α + b cos α
a (sin β – sin β)
a
= b (cos α – cos β)
cos α – cos β
tan 60° =

e
 3 = 1500
√ x
x
√ 3
t
= ·
b sin β – sin α  3x = 1500√
√ 3
3. Let CD be the tower ⇒ x = 1500 m
CD = hm Now, in right ∆ BNP,
BC = xm BN
tan 30° =
x+y
1 1500
√3
⇒ =
3
√ 1500 + y

⇒ 1500 + y = 1500√
 3 × √ 3
⇒ 1500 + y = 4500
⇒ y = 4500 – 1500
In rt., ∆ BCD,
CD ⇒ y = 3000 m
tan 60° =
BC ∴ The aeroplane travels 3000 m in 15 sec.

CAT Complete Course | 83

Visit : www.EasyEngineering.net
Visit : www.EasyEngineering.net

3000 In right ∠d ∆ ANP,


∴ In 1 sec. it will travel m = 200 m
15 y
= cot α
∴ Speed of the aeroplane = 200 m/s x–h
18 ∴ y = (x – h) cot α …(1)
Or, 200 × km/hr = 720 km/hr
5 In right ∠d ∆ BMP,
5
5. Given tan θ = y
= cot β
12 x
3
tan φ = ∴ y = x cot β …(2)
4
From (1) and (2), we get
AB = 192 m
(x – h) cot α =
x cot β
Let height of the tower DC be h m and BC = x m
x–h x
∴ =
tan α tan β
⇒ x tan β – h tan β =
x tan α
⇒ x (tan α – tan α) = h tan β

ww
In ∆ DBC
x =
h tan β
tan β – tan α
h tan β

w
Height of the tower = ·
tan β – tan α
h


3 .Ea
tan φ =

=
x
h
7. Let CD be the tower.
Let CD = h units


4

x =
x

syE
4h
3
…(1)
and
Then,
∠ CAD = θ
∠ CBD = (90° – θ)

In ∆ DAC,

tan θ =
h ngi
5
12
=
x + 192
h
x+ 192 nee
⇒ 12h = 5x + 960
4h In rt. ∆ ACD,
rin


12h = 5 × + 960
3
36h = 20h + 2880
tan θ =
CD
AC
g.n
e
h
⇒ 16h = 2880 tan θ =


h =
2880
16
h = 180 m.
In rt. ∆ BCD,
p
h = p tan θ

CD
t
6. AB = h be the building and x be the height of the tan (90° – θ) =
BC
tower MP S.t. ∠NAP = α, ∠MBP = β h
cot θ = [Q tan (90° – θ) = cot θ]
q
h = q cot θ
Multiplying (1) and (2)
h.h = p tan θ. q cot θ
h2 = pq [Q tan θ. cot θ = 1]

h (height) = + 
√pq unit
[Q height cannot be – ve]
8. Let AD be the building and CE be the tower.
From A, draw AN ⊥ MP Let CE = h m
Let AN = BM = y AB = x m

84 | CAT Complete Course

Visit : www.EasyEngineering.net
Visit : www.EasyEngineering.net

In rt. ∆ ABC
AB
tan 45° =
BC
50 – y
1 =
x
x = (50 – y) …(1)
In rt. ∆ ADE
Then, BC = (h – 15) m
AD
In rt. ∆ ABC, tan 60° =
DE
BC
tan 30° =
AB  3 = 50
√ x
1 h – 15
= 50
3
√ x ⇒ x = …(2)
3

x = √ 3 (h – 15) …(1) 50
= 50 – y [From (1) and (2)]

ww
In rt. ∆ CED,

tan 60° =
CE
DE
3

y = 50 –
50
3


w 3 = h

1 x
 3x = h
√ .Ea[Taking value of x from (1)]
y =
50
√ 3 – 50
3

 3 [√ 3 (h – 15)] = h

3h – 45 = h syE y =
50 (√ 3 – 1 ) × √3
3
√ 3


3h – h = 45
2h = 45
ngi y =
3
(
50 3 – √ 3 )

nee
45
So, h = or 22·5 m. 50 (3 – 1·732)
2 y = [Q 
√ 3 = 1·732]
3
Now, using value of h in (1), 50 × 1·268
x = 3 (h – 15)

 
y =

rin
3
∴ Height of the pole = 21·13 m.
= 21·133

 3  45
= √
 2
– 15 
 10. Let the tower AB = y
g.n
= 1·73 ×

=
25·95
2
15
2

= 12·975 m.
[Q √ 3 = 1·73]
e t
9. Let AD and CE be the tower and pole respectively.

Angles of altitude are 45° and 30°.


∴ m ∠ ACB = 45°
and m ∠ ADB = 30°
DC = 10 m
Let CB = x m
In ∆ ACB,
AB
= tan 45°
BC
Let BC = DE = x m y
⇒ = 1
Let CE = BD = y m x
Then, AB = (50 – y) m ⇒ y = x

CAT Complete Course | 85

Visit : www.EasyEngineering.net
Visit : www.EasyEngineering.net

In ∆ ADB,
AB
= tan 30°
BD
y 1
⇒ =
x + 10 3

x 1
⇒ = [Q y = x]
x + 10 1·73
⇒ 1·73x = x + 10
⇒ 1·73x = x + 10
⇒ 1·73x – x = 10 Then in rt. ∆ ABC,
BC
⇒ 0·73x = 10 tan 45° =
AB
100
⇒ x = 10 × = 13·7 x
73 ⇒ 1 =
y
∴ Height of tower = y = x = 13·7 m.
⇒ y = x …(1)

ww
11. OA and OB are tangents to the spherical ball.
In rt. ∆ ABD,

tan 60° =
BD

w .Ea ⇒



AB

 3 = x +y 5

√ 3y = x + 5

syE ⇒

 3x = x + 5

 3x – x = 5

[From (1)]

∴ AC ⊥ OA
ngi
[Q A tangent to a circle is to the ⇒ (√ 3 – 1 ) x = 5

nee
radius through the point of contact] 5
⇒ x =
and ∠OAC = 90° 3–1

Similarly, CB ⊥ OB √ 3 + 1
∴ ∠OBC = 90°
θ OC

rin x =
5
3–1 √
√ 3+1
×

In rt. ∆ CAO, cosec =


2 AC
OC = AC. cosec
θ ⇒
g.n
x =
5
3–1
(√ 3 + 1)

In rt. ∆ ODC,
OC = a. cosec
θ
2
2
…(1)
⇒ x =

x =
e5 (1·732 + 1)
2
5 (2·732)
2
t
CD
sin φ = ⇒ x = 5 × 1·366 = 6·83
OC
h ∴ Height of the tower = 6·83 m.
⇒ sin φ =
OC 13. Let height of Ist aeroplane = BD
⇒ h = OC. sin φ and height of 2nd aeroplane = BC
θ
⇒ h = a cosec · sin φ
2
∴ Height of the centre of balloon
θ
= a sin φ cosec
2
12. Let BC be the height of the tower and CD be the
height of the pole.
Let BC = x m
and AB = y m

86 | CAT Complete Course

Visit : www.EasyEngineering.net
Visit : www.EasyEngineering.net

Let BC = x m and AB = y m. 15. Let O be the point of observation. A be the position


BC of the aeroplane such that ∠ AOC = 60°, AC = 1 km
In rt. ∆ ABC, tan 45° = and OC = x km.
AB
x
⇒ 1 =
y
⇒ y = x …(1)
In rt. ∆ ABD,
BD
tan 60° =
AB
 3 = 3000
√ After 10 seconds

1 y Let B be the position of the aeroplane.
⇒ √ 3y = 3000 Then, ∠BOD = 30°,
⇒  3x = 3000
√ [From (i)] BD = AC = 1 km
3000 and CD = y km

ww

x =
3

3000
×
√ 3
In ∆ OAC,
OC
AC
= cot 60°

w
x =
3
√ 3
√ 1
⇒ x = …(1)

.Ea
x =
3000
3
√3
In ∆ OBD,
3


⇒ syE
x = 1000 (1·732)
x = 1732 m.

OC
AC
x+y = √
= cot 30°

3 …(2)
∴ Difference in height = 3000 – 1732 = 1268 m.
14. Let W be the window.
ngi Subtracting (2) from (1), we get
y = – 3+ 1

and AB be the house on the opposite side.
Then, WP is the width of the street
nee ⇒ y = 3– 1

3

3

⇒ rin
y =
3–1
=
2

g.n 3

In 10 seconds distance covered =
3

2
3

km

PB
e
In 3600 seconds distance covered =
2
3

= 240
×

√ 3 km
3600
10
t
In ∆ BPW, tan φ =
WP
∴ The speed of the aeroplane = 240√
3
h
= tan φ = 240 × 1·732
WP
= 415·68 km/hr.
WP = h cot φ …(1)
x
Now, In ∆ AWP, 16. = tan 45°
y
AP
tan θ =
WP
h′
= tan θ
WP
⇒ h′ = WP tan θ
⇒ h′ = h cot φ tan θ
∴ Height of house = h + h′ = h + h tan θ cot φ
= h (1 + tan θ cot φ).

CAT Complete Course | 87

Visit : www.EasyEngineering.net
Visit : www.EasyEngineering.net

⇒ x = y [Q tan 45° = 1] AB
In rt. ∆ ABC, tan β =
BC
12
= tan 30° tan β h
y =
1 BC
12
⇒ y = BC. tan β = h
tan 30°
h
⇒ y = 12 ×  √3 BC =
tan β
…(1)
⇒ y = 12 × 1·732 AB
In rt. ∆ ABD, tan α =
⇒ y = 20·784 BD
∴ Distance of ship from cliff = 20·784 tan α h
=
1 BC + p
∴ Required Height = 20·784 + 12 = 32·784 m.
h = tan α. (BC + p)
17. Suppose height of hill DC = x km.
 h 
Distance between two stones AB = 1 km. = tan α  +p
 tan β 
Suppose distance of stone B from hill = y km.
 h 
 Q BC = tan β from (1) 
ww = tan α 
 h + p. tan β 

w .Ea
 tan β 
h tan β = h. tan α + p. tan β. tan α
h tan β – h tan α = p. tan β. tan α
h (tan β – tan α) = p. tan β. tan α
In ∆ BCD,
DC
BC
x
syE
= tan 45°
h =
p. tan α. tan β
tan β – tan α


y
= 1

x = y …(1)ngi p = 150 m, α = 30°, β = 60°


h =
p. tan α. tan β
tan β – tan α
In ∆ ACD,
CD
AC
= tan 30°
nee =
150. tan 30°. tan 60°
tan 60° – tan 30°

150.   √
rin
1 1

x
y+1
= …(2) ( )
3
3
√ √
 3
=
Putting the value of x = y in (2), we get
x
=
1 g.n 3– 1

3



x+1 3

 3x = x + 1

 3x – x = 1

=
e
150
3–1
3

= 150 ×
√ 3
2
t
= 75 × (1·732) [Q 
√ 3 = 1·732]
⇒ (√ 3 – 1)x = 1 = 129·9 m.
⇒ (1·73 – 1)x = 1 19. Let AB be the building, CD the vertical lamp post.
⇒ 0·73x = 1 Let the horizontal distance between the building and
the lamp-post be x m.
1 100
⇒ x = = = 1·37 km. Now, in ∆ ABC,
0·73 73
18. Let AB be the tower = h

88 | CAT Complete Course

Visit : www.EasyEngineering.net
Visit : www.EasyEngineering.net

60 F 1 F 2 = F 1 M + MF2
tan 60° =
x 10 = a cos 60° + a sin 60°
60
⇒ 3 = x
√ 10 = a (0·5 + 0·866)
10 = 1·366a
60
⇒ x = = 20 √
3 10
3
√ a =
1·366
= 7·32 km.
In ∆ AEC, 21. Let height of the tree = y m.
60 – h
tan 30° = Distance of man standing on bank = x m.
x
1 60 – h
=
3
√ 20
√3
60 – h = 20
h = 40 m.
(i) Hence, the horizontal distance between the build-

ww
ing and lamp-post = 20√  3 = 34·64 m.
(ii) Difference between the heights of the building
and the lamp post
In rt. ∆ ABC,

w 60 – h = 60 – 40 = 20 m.

.Ea
20. Let M be the foot of the perpendicular from B on F1 F 2 .

AB
BC
y
x
= tan 60°

= √
3

syE ⇒
In rt. ∆ ABD,
y =  3x
√ …(1)

In rt. ∆ BMF1 ngi ⇒


AB
BD
= tan 30°

BM
F1B
= sin 60°
nee ⇒
y
x + 40
=
1
3

In rt. ∆ BMF2
= BM = F1B sin 60° …(1) ⇒


 3. √
√ rin
 3y = x + 40
 3x = x + 40 [From (1)]
BM
F2B
= sin 45° ⇒
⇒ g.n
3x = x + 40
3x – x = 40
BM = F 2 B sin 45°
From (1) and (2), F1B sin 60° = F 2 B sin 45°
…(2)

Since, sin θ increases as θ increases from 0 to 90°.




2x = 40
x = 20e
Now, putting the value of x = 20 in (1)
t
sin 60° > sin 45°, so that F1B < F2 B
∴ F 1 is nearer to B than F2 y =  3x = 1·732 × 20 = 34·64

∴ The station F1 should send its team. ∴ Height of the tree (y) = 34.64 m.
Let F 1 B = a km. and Width of the river (x) = 20 m.
In rt. ∆ BMF1 22. Let BC = x m.
F1M
= cos 60°
F1B
⇒ F 1 M = F 1 B cos 60°
⇒ F 1 M = a cos 60°
MF2
In rt. ∆ BMF2 · =cot 45° = 1
BM
MF2 = BM = F1 B sin 60° Vertical tower CD = 5 metre in the flagstaff.
= a sin 60° [Using (1)] Let angle of elevations at A be 30° and 60°.

CAT Complete Course | 89

Visit : www.EasyEngineering.net
Visit : www.EasyEngineering.net

BC h
In ∆ ABC, = tan 30° In ∆ ABC, = tan α
AB x
x 1 h
= ⇒ x = …(1)
AB 3
√ tan α
h
AB = √  3x …(1) In ∆ OBC, = tan β
p+x
BD
In ∆ ABD, = tan 60° h
AB = tan β
h
x+5 p+
= √3 tan α
AB
h
x +5 h = p tan β + · tan β
⇒ AB = tan α
3
√  tan β 
h  1 – = p tan β
tan α 
From (1) and (2), we get

 3x = x + 5
√  tan α – tan β 
3
√ h  tan α  = p tan β

ww 3x = x + 5
2x = 5 h =
p tan α tan β
tan α – tan β

w x = 2·5 m.

.Ea
23. Let AB be the tower of height 50 m. p = 50 m., α = 60°, β = 30°

h =
50 tan 60° tan 30°
tan 60° – tan 30°

syE =
50 × (√ 3) ×  1 
√
 3

In rt. ∆ BCD ngi 3– 1



3

BC
CD
= cot 45°

BC = 50 × 1
nee =
50
3 – 1 

√
=
 √
3  3
50
2
= 50 ×
√ 3
2

In rt. ∆ ACD
BC = 50 m.
rin
= 25 × (1·73) = 43·25

AC
CD
= cot 30°
∴ g.n
Height (h) = 43·25 m.
[Q √ 3 = 1·73]
AC = 50 ×  √3
= 50 (1·732)
= 86·600 m
25. Height of deck BC = 16 m.
e t
Required distance, (1)
x = 86·6 – 50
x = 36·6 m
Also, determine the height of the tower if p = 50
metres, α = 60°, β = 30°.
24. Let BC = h = height of tower BC
OA = p, AB = x In rt. ∆ OBC, = tan 30°
OB
16 1
⇒ =
OB 3

⇒ OB = 16√
3m …(1)
In rt. ∆ OBA,
BA
= tan 45°
OB

90 | CAT Complete Course

Visit : www.EasyEngineering.net
Visit : www.EasyEngineering.net

BA ∴ Height (PQ) = 40 + 54·64 = 94·64 metres


⇒ = 1
16
√3 In rt. ∆ QPX,
⇒ BA = 16√3m …(2) PQ
= sin 60°
XQ
∴ Height of the cliff = AB + BC
PQ 3

= 16√
 3 + 16 ⇒ =
XQ 2
= 16 (√ 3 + 1) ⇒ XQ =
2
· PQ
= 16 (1·732 + 1·00) m 3

= 43·712 m 2 √ 3 · PQ
⇒ Distance (XQ) = ×
and distance between cliff and ship. 3 √
√ 3
= OB = 16 √
 3 = 27·71 m. 2√ 3 × 94.64
=
26. QP (Height) = 40 + x 3
2 × 1·732
= × 94·64
3

ww = 109·3 m.
27. Height of the tree AB = 15 m.

w .Ea
syE
In rt. ∆ QPX,
QP
= tan 60° ngi
nee
XP
40 + x
= √
3 …(1) It broke at C. Its top A touches the ground at D.
XP
Now, AC = CD, ∠BDC = 60°
In rt. ∆ QRY,
QR rin
BC = ?

g.n
= tan 45° Let BC = x
YR
AC = 15 – x
QR
⇒ = tan 45° [Q YR = XP] and CD = 15 – x [Q AC = CD]


XP
x
XP
= 1

x = XP …(2)
In rt. ∆ CBD,
BC
CD
e
= sin 60° t
Using the value of x in (1), we get x 3

=
40 + x 15 – x 2
= √3
XP 2x = (15 – x)
√3
⇒ 40 + XP =√ 3XP 2x = 15√
3–√
 3x
⇒ 40 = √ 3XP – XP 2x + √
 3x = 15√
3
⇒ 40 = (√ 3 – 1 ) XP x (2 + √
 3) = 15√
3
40 √ 3 + 1 15√
3
⇒ XP = × x =
3–1 √
√ 3+1 2+√
3
40 (√ 3 + 1) 15
√3 2 –√
3
⇒ x = ×
=
2 2+√
3 2 –√
3
= 20 (1·732 + 1) 30
√ 3 – 15 × 3
x =
= 20 × 2·732 = 54·64 4–3

CAT Complete Course | 91

Visit : www.EasyEngineering.net
Visit : www.EasyEngineering.net

30 × 1·73 – 45 In ∆ ABC
⇒ x =
1 AB
51·9 – 45 tan 30° =
∴ x = BD
1 1 30
⇒ x = 6·9 m =
3
√ BD
∴ The tree broke at 6.9 metres from the ground.
BD = 30√
3 …(2)
Exercise B
Now, CD = BD – BC = 30√
 3 – 30
1. (B) To find the value of sin θ we need hypotenuse. So,
using Pythagoras theorem : = 30 (√ 3 – 1 )
AC2 = AB2 + BC2 = 30 (1·732 – 1) = 30 × 0·732
AC2 = 42 + 3 2 = 16 + 9 = 25 CD = 21·96 metre ≅ 22 metre.
AC = 5
5. (C) OA is the tower.
Height 4
Now, sin θ = = We have to find AC + AB = CB
Hypotenuse 5

ww cos θ =
Base
Hypotenuse 5
Height 4
=
3 In ∆ ACO

tan 30° =
50
AC

w tan θ =
Base 3

.Ea
= ·

2. (C) Let C be the position of kite at a height h and AB


is horizontal line.
1
3

=

AC = 50
50
AC

√3
ABC—
syE
AC represents string. Now, in right angled triangle

We have to find AC, so using formula :


In ∆ AOB

tan 60° =
50
AB
sin 30° =
BC
AC
ngi AB =
3

50

1
2
=
h
AC
AC 100
nee AB + AC = 50  √

 3 + 1  = 200 metre.
 3 √
√ 3
h =
2
=
2
= 50 metre

∴ Height of kite above the ground = 50 metre. rin


6. (C) Let AB is flag staff and AO is the tower now we
have to find AO.
3. (D) Here, we have to find AB.
So using formula :
In ∆ AOC

tan 45° = g.n


AO Height
=

and
cos 60° =

tan 60° =
AB
BO
Height
1 =

AO = CO
CO
AO
CO
e
Base

…(1)
t
Base
250 In ∆ BOC
 3 = Base
√ OB
tan 60° =
OC
250
Base = metre.
3
√ Or,  3 = OB
√ OC
4. (D) First we draw roughly diagram.
√ 3 × OC = OB
Clearly, AB is pole and we have to find CD.
For ∆ ABC  3 × AO = OB
√ [By equation (1)]
30  3 × AO = AB + AO

tan 45° =
BC AB
AO =
30 3–1

BC =
tan 45°
30 10 √ 3 + 1
10
BC = = 30 metre …(1) AO = = ×
1 3–1 √
√ 3–1 √
3+1

92 | CAT Complete Course

Visit : www.EasyEngineering.net
Visit : www.EasyEngineering.net

=
10 (√ 3 + 1) 9. (A) When the plane is at point A it makes an angle of
60° and when at B it makes an angle of 30° at the
3–1
same point where the observer is standing. Clearly
= 5 (√ 3 + 1) metre. from Diagram, Distance AB = CD is covered in 18
seconds.
7. (A) Height of tower = AB = 50 metre
In ∆ BCE
In ∆ ABC
BC
AB 50 tan 30° =
tan 60° = = CE
BC BC
50 1 2000
√3
 3 = BC
√ =
CE
3

50 CE = 6000 metre
BC = …(1)
3
√ In ∆ ADE
In ∆ ABD AD
tan 60° =
AB ED
tan 30° =
BD

ww
Or,
1
3

=
50
BD

 3 = BC
√ ED
2000√
3
[AD = BD]

w
ED = = 2000 metre
BD = 50√ 3 …(2) 3

Now, we have to find CD
∴ CD = BD – BC .Ea ∴ AB = CD = CE – ED
= 6000 – 2000 = 4000 metre

= 50√  3 – 50
3
√ syE Speed =
Distance AB 4000
Time
4000
=
t
=

× 3600
18

ngi
150 – 50 =
= 1000 × 18
3
√ = 800 km/hr.
=
100
3

metre.

8. (A) According to question, C and D are two kilo-


nee
10. (A) Here, CD is a tower whose foot is C.
In ∆ ABC

rin
AB
metres stones tan θ2 =
AC

g.n
CD = 1 km a
Or, AC = …(1)
Now, In ∆ABC, tan θ2
In ∆ ACD

e
AB h
tan 60° = =

t
BC BC CD
tan θ1 =
h AC
BC = …(1) CD = AC tan θ1
3

a
In ∆ ABD, CD = × tan θ1
tan θ2
AB
tan 30° = CD = a tan θ1 cot θ2
BD
11. (A) Let h be the height of the pillar. C be the point on
1 h
= the road whose distance from the pole AB is X.
3
√ BD
In ∆ ABC, we have
BD = h 
√3 …(2) AB h
tan 60° = =
Since, CD = 1 km BC X
h
Or, BD – BC = 1 km X =
tan 60°
 3 – h = 1 km
h√ h = √
 3.X …(1)
3

Now, In ∆ CDE
 3 km.
√ ED h
h = tan 30° = =
2 CD 100 – X

CAT Complete Course | 93

Visit : www.EasyEngineering.net
Visit : www.EasyEngineering.net

1 h AB
Or, = …(2) tan 15° =
3
√ 100 – X BC
60
1  3.X
√ tan (60° – 45°) =
BC
Or, = [From equation (1)]
3
√ 100 – X
60
BC =
Or, 100 – X = 3X tan (60° – 45°)
Or, 4.X = 100 60
=
tan 60° – tan 45°
X = 25 metre
1 + tan 45° tan 60°
h = 25√  3 metre.
60 √
 3 + 1 metre.
12. (A) AB is the light house and C is the boat. We have Or, BC = = 60 × 
to find BC. 3–1
√  3 – 1 
√
In ∆ ABC 1+√ 3.1
●●●

ww
w .Ea
syE
ngi
nee
rin
g.n
e t

94 | CAT Complete Course

Visit : www.EasyEngineering.net
Visit : www.EasyEngineering.net

7 Ratio and Proportion

Ratio Proportion
A ratio is a comparison of two numbers by di- When two ratios are equal, the four quantities com-
vision—Ratio is the relation which one quantity bears to posing them are said to be proportional.
another of the same kind, the comparison being made by The equality of two ratio is called proportion a, b, c,
considering what part, multiple, one quantity is of the d are said to be in proportion if a : b = c : d or a : b :: c :
other. Therefore, every ratio is an abstract quantity. d.
a

ww
If a and b are two quantities of the same kind, then In a proportion, the first and fourth terms are known
b as extremes, while second and third terms are known as
is known as the ratio of a and b. Therefore, the ratio of means. Hence, a and d are extremes and b and c are
two quantities in the same units is a fraction that one

w
means. Hence a and d are extremes and b and c are means
quantity is of the other. of the proportion a : b :: c : d.
a 
Thus, a to b is a ratio 
.Ea
 , written as a : b.
b 
In a proportion we always have :
Product of extremes = product of means

syE
The first term of the ratio is called antecedent, while
the second term is called consequent. Illustration 3.
a×d = b ×c

In the ratio a : b.
Ist term ‘a’ is known as antecedent.
ngi If 0.75 : X : : 5 : 8, then find X.
Solution :

nee
IInd term ‘b’ is known as consequent. Since, these quantities are in proportion.
Ratio between 60 kg and 100 kg is 3 : 5. So, product of means = product of extreme
0.75 5

rin
Illustration 1. Or, =
X 8
Ratio of 50 kg and 60 kg is—
Or, X = 0.15 × 8
Solution :
50
60
5
= =5:6
6
Or,
Illustration 4. g.n
X = 1.2

The multiplication or division of each term of ratio


by a same non-zero number does not affect the ratio.
Hence, 3 : 5 is the same as 6 : 10 or 9 : 15 or 12 : 20 etc.
Solution : e
Find a fourth proportional 6mn2 : 9m3n :: 4mn3 : ?

Let fourth proportional is p; then 6mn2 : 9m3n ::


4mn3 : p
t
Ratio can be expressed as percentages. To express
the value of a ratio as a Percentage, we multiply the ratio ∴ p = 6m3n2
by 100. Continued Proportion
3 Four Quantities—a, b, c, d, are said to be in a con-
Therefore, = 0·6 = 60%.
5 tinued proportion , if
Illustration 2. a b c
a : b = b : c = c : d or = =
Two numbers are in the ratio of 3 : 7. If 4 be added b c d
to each, they are in the ratio of 7 : 11. Find the numbers. Three quantities are said to be in continued propor-
Solution : tion, if
3x + 4 If a : b = b : c
Let the numbers be denoted by 3x and 7x. Then
7x + 4 Or, ac = b2
7
= · Hence, x = 1. In this relationship, b is said to be the mean pro-
11 portional between a and c and c is said to be a third pro-
Numbers are 3 and 7. portional to a and b.

CAT Complete Course | 95

Visit : www.EasyEngineering.net
Visit : www.EasyEngineering.net

Illustration 5. (2) Number of quantities are four


An object is 20 m long casts a shadow 25 m long. At A a B X C α
If = = =
the same time another object kept merely casts a shadow B b C Y D β
30 m. long. Find the length of the second object. Then, A : B : C : D = a X : b X : b Y : b Y β
Solution : Some Results on Ratio and Proportion
Ratio of length of the object to its shadow would be
1. Invertendo – If a : b :: c: d, then b: a :: d : c
the same.
2. Alternendo – If a : b :: c : d, then a : c :: b : d.
20 X
∴ = 3. Componendo – If a : b :: c : d, then
25 30
20 × 30 (a + b ): b :: (c + d): d
X = =4×6
25 4. Dividendo – If a : b :: c : d then
X = 24 m. (a – b) : b :: (c – d) : d
Illustration 6. 5. Componendo – If a: b :: c : d then
An object 1.6 m long casts a shadow 1.4 m long. At and Dividendo (a + b) : (a – b) :: (c + d) : (c – d).

ww
the same time another object kept nearby casts a shadow
6.2 m long. Find the length of the second object.
Solution :
Equating the components of two and more ratios
a 2 b 4
If = , = . Then find a : b : c.
b 3 c 5


w
Ratio of length of the object to its shadow would be
the same.

.Ea
1.6 : 1.4 = x : 6.2
To equate , common component b in the two ratios ,
take the LCM of 3 and 4 which is 12. Hence, the new
ratios are obtained as
a 2×4 8 b 4×3 12
or

Properties of Proportion
syE
x =
1.6 × 6.2
1.4
= 7.08 m = = , =
b 3 × 4 12 c 5 × 3
=
15
Now, since the common component b in the two

ngi
ratio has the equal values, a : b : c = 8 : 12 : 15.
a c a n1 b n
If
b
= , then
d Therefore, if = , = 2
b m1 c m2
(i)
b
a
=
d
c
(ii)
a
c
=
b
d nee ∴
Illustration 7.
a : b : c = n1 × n2 : m1 × n2 : m1 × m2

(iii)
c
a
=
b
d
(iv)
a+b
a
=
c+d
d
a 3 b 4
If = , = , Find a : b : c.
b 5 c 7
rin
(v)
a–b
a
a+b
=
c–d
d
a–b
(vi)
a+b
a–b
=
c+d
c–d
a+c a–c
Solution :
g.n
a : b : c = 3 × 4 : 5 × 4 : 5 × 7 = 12 : 20 : 35

e
a
(vii) = (viii) = = a 2 b 4 c 7

t
c+d c–d b b+d b–d Suppose there are three ratio = = = and =
b 3 c 5 d 15
Relation among More than Two Quantities Now, to find the value of a : b : c : d. First of all we
(1) The number of quantities are three equate the common term b in the two ratio and then the
A a B X same process is repeated to equate the common term c.
If = =
B b C Y LCM of 3 and 4 = 12
Then, A : B : C = Product of all numerators : First a 2×4 8 b 4×3
∴ = = and = two ratio and then the
denominator × Second Numerator : Product of all deno- b 3 × 4 12 c 5×3
minator same process is repeated to equate the common term c.
A:B:C = aX : bX : bY LCM of 3 and 4 = 12
Pictorial representation is a 2×4 8 b 4 × 3 12
∴ = = and = = .
b 3 × 4 12 c 5 × 3 15
A: B = a b
Now, LCM of 15 and 7 = 105
A B C b 12 × 7 84 c 7 × 15 105
∴ = = and = =
c 15 × 7 105 d 15 × 15 225
B :C = X Y Again, we have to equate b for first two ratios
Follow the arrow diagram and multiply to get a 8 × 7 56 b 84 c 105
∴ = = , = , =
A : B : C b 12 × 7 84 c 105 d 225

96 | CAT Complete Course

Visit : www.EasyEngineering.net
Visit : www.EasyEngineering.net

Hence, a : b : c : d = 56 : 84 : 105 : 225 Let x litre of water be added to make the ratio 1 : 2.
a n b n c n 40 1
Therefore, if = 1 , = 2 and = 3 = ⇒ x = 60 litres.
b m1 c m2 d m3 20 + x 2
Then, a : b : c : d = n1 × n2 × n3 : m1 × n2 × n3 : Let x litre of water be added to make the ratio 1 : 2.
m1 × m2 × n3 : m1 × m2 × m3 40 1
= ⇒ x = 60 litres.
Illustration 8. 20 + x 2
a 2 b 4 c 7 Illustration 13.
If = , = and = .
b 3 c 5 d 11 A bag contains rupee, fifty paisa and five pasia coins
Solution : whose values are in the proportion of 2 : 3 : 4. If the total
number of coins are 480, find the value of each coin and
a = 2 × 4 × 7 = 56, b = 3 × 4 × 7 = 84, c = 5 × 3 × 7
the total amount in rupees.
=105 and d =11 × 5 × 3 = 165.
Solution :
∴ a : b : c : d = 56 : 84 : 105 : 165.
Amount in rupees
Illustration 9. Number of coins =
Value of coins in rupees
a 1 b 4 c 7

ww
If = , = , = . Find a : b : c : d.
b 3 c 5 d 9
Solution :
∴ Number of one rupee coin =
2x
1
3x

w
Number of one 50 paisa = = 6x
a = 1 × 4 × 7 = 28, b = 3 × 4 × 7 = 84, c = 5 × 3 × 7 1
= 105, d = 3 × 5 × 9 =135

.Ea
∴ a : b : c : d = 28 : 84 : 105 : 135
and number of 25 paisa coin =
2
4x
1
= 16x

syE
Illustration 10.
2
If 2x + 3y : 3x + 5y = 18 : 19. Find x : y.
Given 2x + 6x + 16x = 480 ⇒ x = 20.
Solution :
2x + 3y
3x + 5y
=
18
19 ngi ∴ Value of one rupee coin = 2x = Rs. 40,
Value of 50 paisa coin = 3x = Rs. 60 and

Or
x 
2 + 3
y 
=
2k + 3 18
=
nee Value of 25 paisa coin = 4x = Rs. 80.
Illustration 14.
x 
3 + 5
y 
2k + 5 19

rin
Two vessel contain mixture of water and milk in the
ratio 1 : 4 and 2 : 5. These mixture of two vessels are

g.n
mixed in the ratio 1 : 4. Find the ratio of water and milk
 x  33 in the resulting mixture.
Solving the equation for k, we get k =  = –
 y  16 Solution.
Illustration 11.
Three numbers are in the ratio 3 : 4 : 5, the sum of
whose squares is 800. Find the numbers.
e
1
3
2
2
In vessel 1 quantity of water = and that of milk = .
3
5
In vessel 2 quantity of water = and that of milk = .
t
7 7
Solution :
1 4
Let the numbers be 3x, 4x and 5x. From vessel 1, is taken and from vessel 2, is
5 5
Then, 9x2 + 16x2 + 25x2 = 800 taken. Therefore, the ratio of water to milk in the new
vessel.
Or, 50x2 = 800 ⇒ x2 = 16 or x = 4.
So, the numbers are 12, 16 and 20. 1 1 2 4  2 1 5 4 
= 3 × 5 + 7 × 5  : 3 × 5 + 7 × 5 
Illustration 12.    
In a mixture of 60 litres, the ratio of milk and water = 31 : 74
is 2 : 1. What amount of water must be added to make the Illustration 15.
ratio 1 : 2? A and B are two alloys of gold and copper prepared
Solution : by mixing metals in proportion 7 : 2 and 7 : 11 respect-
2 ively. If equal quantities of alloys are melted to form a
Quantity of milk = × 60 = 40 litres and that of
3 third alloy c , then find the proportion of gold and copper
water = 20 litres. in C.

CAT Complete Course | 97

Visit : www.EasyEngineering.net
Visit : www.EasyEngineering.net

Solution : If is therefore important to note here that the variat-


ion is direct and proportional. If one quantity is doubled
 7 7  21
In alloy c, Gold =  9 + 18  = 18 the related quantity will also be doubled.
  Other examples of direct proportion are :
 2 11  15 (a) Simple Interest Vs Time (principal and rate being
and Copper =  +  =
 9 18  18 constant)
(b) Density Vs Mass ( volume being constant )
21 15
∴ Ratio of gold and copper = : = 7 : 5. (c) Force Vs Acceleration ( mass being constant )
18 18
Illustration 16. Direct Variation
A and B started a joint firm. A’s investment was If A is said to vary directly as B, then as A increases
thrice the investment of B and period of his investment B also increases but not proportionally. This variation is
was two times the period of investment of B. If B got Rs. denoted by A ∝B or A = KB, where K is a constant.
4000 as profit, find their total profit. For Example, the total cost of production is directly
Solution : related to the number of items being produced.
Ratio of investment of A and B = 3 × 2 : 1 × 1 Here, the variation is direct but not proportional.

ww 1
= 6:1

Share of B = × Total profit = 4000


Inverse Proportion
A is inversely proportional to B means if A increases


w
Illustration 17.
7

.Ea
Total profit = Rs. 28,000
B decreases proportionally. If speed is doubled, time
taken to cover the same distance is reduced to half.
Other examples of inverse proportion are
(a) Density Vs volume (mass being constant)

syE
a b c
If = = , what is the value of each of (b) Number of person Vs time taken to complete the
b+c c+a a+b
the fraction ? work. (work being same )
Inverse Variation
Given that (a, b, c > 0).
Solution :
ngi If A is inversely related to (or) varies inversely as B,
then if B increases as A decreases but not proportionally.
If each of the ratio is equal to K, then a = (b + c) K,
b = (c + a) K and c = (a + b) K.
Hence, a + b + c = (2a + 2b + 2c) K
nee
This relation can be expressed mathematically as A ∝
1
1
B

or, (a + b + c)– 2K(a + b + c) = 0


rin
⇒ A = K × , where K is a constant.
B
Here, the variation is inverse but not proportional.
or,

So,
(a + b + c) (1 – 2K) = 0

K =
1
2
Illustration 19.
g.n
A can do a piece of work in 12 days, B is 60% more

Illustration 18.
If
Find
a : b = c : d and e : f = g : h,
(ae + bf) : (ae – bf).
do the same piece of work.
Solution :
Ratio of efficiencies of
e
efficient than A. Find the number of days that B takes to

t
Solution : A and B = 100 : 160 = 5 : 8
Since, efficiency is inversely proportional to the
a c e g
= and = number of days, hence ratio of days taken to complete the
b d f h
job is 8 : 5.
ae cg 5 1
= So, number of days taken by B = × 12 = 7 days.
bf dh 8 2
ae + bf cg + dh Illustration 20.
=
ae – bf cg – dh The speeds of three cars are in the ratio 2 : 3 : 4.
(Applying componendo and Dividendo) Find the ratio between the times taken by these cars to
travel the same distance.
Direct Proportion Solution :
If A is direct proportional to B, then as A increases B Speed is inversely proportional to time taken.
also increases proportionally. For example the relation 1 1 1
between speed, distance and time, speed is directly pro- Hence, ratio of time taken by these cars = : : =
2 3 4
portional to distance, when time is kept constant. 6 : 4 : 3.

98 | CAT Complete Course

Visit : www.EasyEngineering.net
Visit : www.EasyEngineering.net

Exercise A 9. The monthly incomes of A and B are in the ratio 4 :


1. The ratio of the numbers of gents to ladies in a party 5, their expenses are in the ratio 5 : 6. If 'A’ saves
was 2 : 3. When 20 more gents joined the group, the Rs. 25 per month and 'B' saves Rs. 50 per month,
ratio was reversed. The number of ladies in the party what are their respective incomes ?
was— (A) Rs. 400 and Rs. 500
(A) 16 (B) 24 (B) Rs. 240 and Rs. 300
(C) 30 (D) 36 (C) Rs. 320 and Rs. 400
2. A man ordered 4 pairs of black socks and some pairs (D) Rs. 440 and Rs. 550
of brown socks. The price of a black pair is double 10. Find the ratio of a and b from the equation 12a2 +
that of a brown pair. While preparing the bill, the 35b2 – 43ab = 0—
clerk interchanged the number of black and brown
(A) 7/3;5/4 (B) 7/4;6/5
pairs by mistake which increased the bill by 50%.
The ratio of the number of black and brown pair of (C) 5/2 only (D) None of the above
socks in the original order was— 11. If x and y are connected by the relation x 2 + 4y 2 =
(A) 4 : 1 (B) 2 : 1 4xy, then ratio of x to y is—

ww
(C) 1 : 4 (D) 1 : 2
3. A and B compared their incomes and found that A's
income was to that of B as 7 : 9 and that the third of
(A) 2 : 1
(C) 1 : 1
(B) 1 : 2
(D) 4 : 1

w
12. If a, b > 0, choose the write option—
A's income was Rs. 30 greater than the difference of (A) (a3 + b3 ) : (a2 + b2 ) is greater than (a2 + b2 ) :
two—
.Ea
their incomes. Find the difference of the income of (a + b)
(B) (a3 + b3 ) : (a 2 + b2 ) is smaller than (a2 + b2 ) :

syE
(A) Rs. 180 (B) Rs. 240 (a + b)
(C) Rs. 320 (D) Rs. 160 (C) (a3 + b 3 ) : (a 2 + b 2 ) is equal to (a 2 + b 2 ) :
4. In a large office, 3/4th of the staff can neither type (a + b)
nor take shorthand. However 1/5th can type and 1/3rd
can take shorthand. What proportion of people can
ngi (D) None of the above
13. If a > b > 0, choose the write option—

nee
do both ?
(A) The ratio a2 – b2 : a2 + b2 is greater than a – b :
(A) 13/40 (B) 17/60 a+b
(C) 1/5 (D) 3/40 (B) The ratio a2 – b2 : a2 + b2 is smaller than a – b :
5. What must be added to two numbers that are in the
ratio 3 : 4, so that they come in ratio 4 : 5—
a+b
rin
(C) The ratio a2 – b2 : a2 + b2 is equal to a – b : a + b
(A) 1
(C) 5
(B) 6
(D) 3
(D) None of the above
g.n
14. What number must be added to each term of the ratio
6. A person bought two bikes for Rs. 15,000. He sold
one of them for a profit of 10% and another for a
loss of 5% and on whole he found that he neither
gained nor loss. What is the CP of each bike ?
(A) 13
(C) 15
e
2 : 5 so that it may become equal to 5 : 6 ?
(B) 16
(D) 30 t
15. What quantity must be added to the terms of the ratio
(A) Rs. 5,000, Rs. 10,000 p + q : p – q to make it equal to (p + q)2 : (p – q)2 ?
(B) Rs. 7,500, Rs. 7,500 q2 – p2
(A) p/q (B)
(C) Rs. 8,000, Rs. 7,000 2p
(C) q/p (D) None of these
(D) Rs. 9,000, Rs. 6,000
16. If two numbers are in the ratio 5 : 7 and if 3 is
7. Divide Rs. 1,350 in three parts such that 12 times the
subtracted from each of them, the ratio becomes 2 : 3.
first is equal to 5 times the second and 6 times the
Find the numbers—
third—
(A) 25; 35 (B) 60; 84
(A) 500,100,750 (B) 250,750, 350
(C) 15; 21 (D) 30; 42
(C) 300,600,450 (D) 250, 600,500
3x – 4y 5x – 6y
8. If x : y :: 5 : 2, the value of 8x + 9y : 8x + 2y is— 17. If = , find the value of x : y—
2x – 3y 4x – 5y
(A) 22 : 29 (B) 29 : 22 (A) 5 : 3 (B) 6 : 5
(C) 61 : 26 (D) 26 : 61 (C) 5 : 2 (D) 1 : 1

CAT Complete Course | 99

Visit : www.EasyEngineering.net
Visit : www.EasyEngineering.net

18. If (2x + 1) : (3x + 13) is the sub-duplicate ratio of 9 : 8. If 2X + 3Y : 3X + 5Y = 7 : 8, then find X : Y ?


25, find x— (A) 11 : 7 (B) 8 : 11
(A) 35 (B) 54 (C) 7 : 11 (D) 5 : 11
(C) 52 (D) 34 (E) 11 : 5
19. What is the compounded ratio of the following ratio 9. The ratio between two number is 2 : 5, then each
5 : 14 and 7 : 15 ? number is increased by 2, then the ratio becomes 3 : 2.
(A) 8 : 3 (B) 9 : 5 Find the number?
(C) 1 : 6 (D) 1 : 7 (A) 11 : 7 (B) 2 : 11
(C) 7 : 11 (D) 5 : 11
20. The ages of two persons are in the ratio 5 : 7.
Eighteen years ago their ages were in the ratio 8 : 13. (E) 11 : 5
Find their present ages— 10. The ratio of the number of boys and girls in a school
(A) 53 (B) 10 is 2 : 5. If there are 700 students in the school. Find
the number of girls in the school?
(C) 52 (D) 11
(A) 200 (B) 350

ww Exercise B
1. If a : 5 : : 10 : 25, then find a ?
(C) 500
(E) 50
(D) 650

(A) 2
(C) 4
(E) 6
w .Ea
(B) 3
(D) 5
a 2 b 5 c 6
11. If = , = , = the find a : b : c : d?
b 3 c 4 d 7
(A) 20 : 15 : 12 : 14
(C) 10 : 15 : 15 : 4
(B) 10 : 15 : 12 : 3
(D) 10 : 15 : 12 : 14

syE
2. Find the mean proportion of 0·8 and 0·2 ?
(E) 20 : 15 : 12 : 3
(A) 0·4 (B) 0·6
(C) 0·5 (D) 0·3 12. The sum of numbers are in the ratio 5 : 3 : 4. The
(E) 0·7
3· Find the third proportion to 0·016 and 0·024 ? ngi Sum of whose squares is 800. Find the biggest
numbers ?
(A) 20 (B) 16
(A) 0·036
(C) 0·016
(B) 0·024
(D) 0·020 nee (C) 12
(E) 4
(D) 8

(E) 0·040
4. Find the mean proportional of 9 and 16 ? rin
13. In a mixture of 50 litres, the ratio of milk and water
is 2 : 3. What amount of water must be added to
(A) 10
(C) 12
(B) 11
(D) 13
make the ratio 1 : 3?
(A) 25 litres g.n
(B) 15 litres

(E) 14
5. Find the fourth proportional to 5, 6 and 150 ?
(A) 100 (B) 120
(C) 20 litres
(E) 40 litres e
(D) 30 litres

14. The two vessels contain rupee, fifty paisa, and


twenty five paisa coins, whose values are in the
t
(C) 140 (D) 160 proportion of 1 : 2 : 5. If the total number of coins
are 1000. Find the value of the total amount in
(E) 180
rupees?
6. Given that A : B = 5 : 2 and B : C = 3 : 2. Find (A) Rs. 200 (B) Rs. 400
A:C?
(C) Rs. 320 (D) Rs. 120
(A) 4 : 7 (B) 5 : 3
(E) Rs. 40
(C) 3 : 5 (D) 4 : 5
15. Two vessels contain mixtures of water and milk in
(E) 5 : 4 the ratio 1 : 2 and 2 : 3. These mixtures of two
5 1 vessels are mixed in the ratio 1 : 4. Find the ratio of
7. is what part of ?
48 16 water and milk in the resulting mixture?
(A) 5/3 (B) 1/3 (A) 11 : 76 (B) 75 : 76
(C) 2/3 (D) 4/3 (C) 11 : 75 (D) 1 : 3
(E) 7/3 (E) 2 : 3

100 | CAT Complete Course

Visit : www.EasyEngineering.net
Visit : www.EasyEngineering.net

16. The sum of the present age of A, B and C are 90 (A) 5 : 9 (B) 2 : 5
years. Six years ago, their ages were in the ratio 3 : 2 (C) 1 : 3 (D) 1 : 4
: 1 . What is the present age of C?
(E) 9 : 5
(A) 12 (B) 14
23. In an express train the passengers travelling in A.C.
(C) 16 (D) 20 sleeper class. First class and sleeper class are in
(E) 18 the ratio 2 : 3 : 5 and rate for each class in the ratio
a b c 1 : 2 : 4. If the total income from the train is Rs.
17. If = = · What is the value of each of 56,000. Find the income of Indian Railways from
b+c c+a a+b
the fractions ? (Given a, b, c > 0) A.C. sleeper class?
(A) 1 (B) 1/2 (A) Rs. 4,000 (B) Rs. 5000
(C) 1/3 (D) 1/4 (C) Rs. 40,000 (D) Rs. 50,000
(E) 1/5 (E) Rs. 16,000
18. A bag contains rupees, fifty paisa, twenty five paisa 24. The income of Ram and Raju are in the ratio 3 : 2
and ten paisa coins in the proportion 1 : 3 : 5 : 7. If and expenses are in the ratio 5 : 3. If both save
Rs. 2000. What is the income of Ram ?

ww
the total amount is Rs. 22.25. Find the number of
coins of fifty paisa kind?
(A) 10 (B) 15
(A) Rs. 3,000
(C) Rs. 5,000
(B) Rs. 4,000
(D) Rs. 6,000

w
(C) 20
(E) 35
(D) 25

.Ea
19. When 5 is subtracted from both the numerator and
(E) Rs. 4,500
25. Ram, Shyam and Sohan together earn Rs. 11950 and
spend 70%, 60% and 65% of their salaries

syE
the denominator of a fraction. The fraction reduces respectively. If their saving are 14 : 21 : 15. What
1 is the salary of Shyam?
to . When 2 is add to both the numerator and the
2 (A) Rs. 4,425 (B) Rs. 4,990
2
denominator the fraction reduces to . Find the
fraction ?
3
ngi (C) Rs. 6,500
(E) Rs. 3,625
(D) Rs. 5,400

(A) 4/5
(C) 4/3
(B) 1/5
(D) 2/5 nee Answer with Hints
Exercise A
(E) 3/5
20. Sita distributes her pens among four friends A, B, C
1. (B)
rin
G:L = 2:3
G 2
1 1 1 1
and D in the ratio : : : . What is the minimum
3 4 5 6
number of pens that Sita should have ?

or
G + 20
L
g.n
=

=
3
3
……..(i)

…….(ii)
(A) 57
(C) 60
(E) Cannot be determine
(B) 360
(D) 120
L
Solving (i) and (ii), we get
G = 16
e2

t
L = 24
21. Ramu divides his property so that his son’s share 2. (C)
to his wife’s and the wife’s share to his daughter are
2 : 3 and 4 : 5 respectively. If the daughter gets 3. (A) A:B = 7:9
Rs. 49,000 more than the son. Find the total worth of A
⇒ B = 9
the property ? 7
(A) 4,45,000 (B) 2,00,000 A
Also, = 30 + B – A
3
(C) 2,45,000 (D) 3,00,000
Putting the value of B, we get
(E) 3,50,000
A  9A 
2 = 30 +   – A
22. In what proportion must a number be added, so that
5 3 7 
1 A 
of the first part and of the second part are together
3 ⇒  3  = 30 + 2A
1   7
equal to of the original number ?
4 ⇒ A = 630

CAT Complete Course | 101

Visit : www.EasyEngineering.net
Visit : www.EasyEngineering.net

∴ Difference of income Now, sum of all the ratios


 630  5 + 12 + 10 = 27
=  3  – 30 = 180
  ∴ 1st part =
5
× Rs. 1350
27
3 1 1
4. (B) – = + –X = Rs. 250
4 5 3
7 12
X = ∴ 2nd part = × Rs. 1350 = Rs. 600
60 27
5. (A) Let x be added to them. 10
∴ 3rd part = × Rs. 1350
27
Then, 3 + x : 4 + x : : 4 : 5
= Rs. 500.
⇒ (3 + x) × 5 = (4 × x) × 4
x 5
⇒ 15 + 5x = 16 + 4x 8. (B)
y
=
2
⇒ x = 16 – 15 x
8 +9
= 1 8x + 9y y
=
∴1 must be added. 8x + 2y x

ww
6. (A) According to the question gain on 1st
= loss on 2 nd =
8 +2

29
22
y

w = 10% of CP of the 1st

.Ea
= 5% of CP of the 2nd

=
10
of CP of 1st
9. (A) Let A’s income be = 4x
A’s expenses, therefore = 4x – 25
Let B’s income be = 5x

= syE
100
5
100
of CP of the 2nd
B’s expenses,
Therefore, = 5x – 50

=
CP of Ist
CP of 2 nd
ngi We know that the ratio of their expenses
= 5:6

nee
5 100 5 4x – 25 5
= × = ⇒ =
100 10 10 5x – 50 6
Sum of the ratio = 5 + 10 = 15 ⇒ 24x – 150 = 25x – 250

CP of the 1st bike =


5
15
× 15000 = Rs. 5000
⇒ Therefore,
rin
x = 100
⇒ A’s income 4x = 400 and
and CP of the 2nd bike
10
× 15000 = Rs. 10000
g.n
B’s income = 5x = 500.

e
= 10. (A) 12a2 + 35b2 – 43ab = 0
15
7. (D) We have, 12 × (1st part)
= 5 × (2nd part)
= 6 × (3rd part) = x(say) ⇒
a  2
b
a 
⇒ 12  – 43  + 35 = 0
  b 
12x2 + 43x + 35 = 0
t
x a
⇒ 1st part = where x =
12 b
x
2nd part = , 3 rd part =
x ⇒ (3x – 7)(4x – 5) = 0
5 6 7
∴ The ratio, 1st part : 2nd part : 3rd part ⇒ x =
3
x x x 5
= : : or x =
12 5 6 4
On multiplying all the terms by 60, a 7
⇒ =
1st part : 2nd part : 3rd part b 3
60x 60x 60x a 5
= : : or =
12 5 6 b 4
= 5x; 12x : 10x 11. (A) x2 + 4y2 = 4xy
= 5 : 12 : 10 can be written as

102 | CAT Complete Course

Visit : www.EasyEngineering.net
Visit : www.EasyEngineering.net

x2 – 4xy + 4y2 = 0 ⇒ 6(2 + x) = 5(5 + x)


⇒ (x – 2y)2 = 0 ⇒ x = 13
Therefore, x = 2y, 15. (B) Let the required quantity be x. Then,
x/y = 2, p+q+x (p + q)2
=
Hence, the ratio of x to y is 2 : 1. p–q+x (p – q) 2
12. (A) Let A = a3 + b3 ⇒ (p + q + x)(p – q) = (p – q – x)(p + q)2
2

B = a2 + b2 ⇒ (p + q) (p – q)2 + (p – q)2x
C = a2 + b2 = (p – q)(p + q)2 + x(p + q)2
and D = a+b ⇒ x[(p + q)2(p + q)2] + (p – q) (p – q)2
We know that – (p + q) (p + q)2
A:B > C:D ⇒ x[– 4pq) = (p + q) {(p + q) – (p – q)}
iff AD > BC ⇒ – 4xpq = (p + q)(p – q) (2q)
Now, AD = (a3 + b3) (a + b) ⇒ – 4xpq = (p2 – q2 )2q
⇒ – 4xpq = (p2 – q2 )2q

ww = a4 + b4 + ab (a2 + b2 )
and BC = (a2 + b2 ) (a2 + b2 )
= a4 + b4 + 2a2 b2
⇒ x =
(p2 – q2 )2q
– 4pq

w
AD – BC = {a4 + b4 + ab(a2 + b2 )}

.Ea – {a4 + b2 + 2a2 b2 }


= ab(a2 + b2) – 2a2b2 – ab(a2 + b2 – 2ab)
=

=
p2 – q2
– 2p
q2 – p2
2p
= ab(a – b) 2 > 0
syE
[Q a, b, 0 ∴ ab > 0 and (a + b)2 > 0]
16. (C) Let

Then,
the two numbers
5x – 3
7x – 3
=
be
2
3
5x and 7x.

⇒ AD > BC
⇒ A:B > C:D ngi ⇒

15x – 9 = 14x – 6
x = 3

13. (A) Let


3 3 2

2
A = a –b ,
2

2
2 2
Hence, (a + b ) : (a + b ) > (a + b ) : (a + b)
nee
Hence, the numbers are 15 and 21.

17. (D)
3x – 4y
=
5x – 6y
2
B = a +b ,
C = a–b
2


2x –

rin
3y
3(x/y) – 4
=
4x – 5y
5(x/y) – 6

and
We know that
D = a+b
2(x/y) – 3

g.n 4(x/y) – 5
[Dividing numerator and denominator by y]

e
3a – 4 5a – 6
⇒ =
A:B > C:D
iff AD > BC
2 2
We have AD = (a – b ) (a + b)
= (a + b) (a + b)2
where


2a – 3
x
y
= a
4a – 5

(3a – 4)(4a – 5) = (2a – 3)(5a – 6)


t
⇒ 12a 2 – 31a + 20 = 10a2 – 27a + 18
and BC = (a2 + b2 ) (a + b)
⇒ 2a2 – 4a + 2 = 0
∴ AD – BA = (a + b)(a + b)2 – (a2 + b2 ) (a – b)
⇒ a2 – 2a + 1 = 0
= (a + b) {(a + b)2 – (a – b)}
⇒ (a – 1) 2 = 0
= (a - b) (2ab) ⇒ a = 1
= 2ab (a – b) > 0 [Q a > b > 0] x 1
⇒ =
⇒ AD > BC y 1
⇒ A:B > C:D ⇒ x:y = 1:1
⇒ (a2 – b2 ) : (a2 + b2 ) > (a – b) : (a + b) 18. (D) Since, (2x + 1) : (3x + 13) is the sub-duplicate
ratio of 9 : 25, therefore
14. (A) Let the number to be added be x.
2+x 5 2x + 1 √ 9
Then, = =
5+x 6 3x + 13 25

CAT Complete Course | 103

Visit : www.EasyEngineering.net
Visit : www.EasyEngineering.net

2x + 1 3 2X + 3Y 7
⇒ = 8. (E) Given =
3x + 13 5 3X + 5Y 8
⇒ 10x + 5 = 9x + 39 X
2× +3
⇒ x = 34 Or,
Y
=
7
X 8
5 7  3× +3
19. (C) The required ratio =  14 × 15  Y
  X X
1 Or, 16 × + 24 = 21 × + 35
= =1:6 Y Y
6 X
20. (B) Let the present ages be 5x and 7x years. Or, 5 × = – 11
Y
5x – 18 8 X 11
Then, = Or, = –
7x – 18 13 Y 5
⇒ x = 10 X : Y = – 11 : 5
Exercise B 9. (B) Let ratio between two numbers is 2 : 5.
a 10 ∴ Numerator = 2 X
1. (A) =

2.
ww 5 25
a = 2
(A) If X be the required mean proportion, then
Denominator = 5 X
According to question,

w (0·8)

.Ea
X
=
X
(0·2)
X2 = 0·8 × 0·2
Or,
Or,
2X + 2
5X + 2
=
3
2
4X + 4 = 15 X + 6
11X = – 2

3.
syE
X = 0·4
(A) Let a be the third proportion.
Now, 0·016 : 0·024 : : 0·024 : a
X = –
2
11

Or, a =
0·024 × 0·024
0·016 ngi10. (C) Let number of boys = 2X
Number of girls = 5 X

4.
a = 0·036
(C) Let the mean quantity = X nee According to question,
2X + 5X = 700
Now, 9 : X : : X : 16
X2 = 9 × 16
rin
X = 100
Number of girls = 5 × 100

5.
X = 12
(E) Let the fourth proportional be a.
11. (D) First Method : g.n
= 500

Now, 5 : 6 : : 150 : a

Or,
5
6
=
150

a = 180
a
Given : a:b = 2:3
b:c = 5:4
c:d
e
= 6:7 t
A 5 a b
6. (B) Since, = =
B 2 2 3
B 3 b c
and = =
C 2 5 4
A B 5 2
× = × c
=
d
B C 2 3 6 7
5
= On multiplying increase by 5, 3 & 2.
3
a b
∴ A:C = 5:3 We get, =
10 15
5 1
7. (A) Required part will be the ratio of and · b c
48 16 =
15 12
5
48 5 c d
∴ Part = = 12
=
14
1 3
16 ∴ a : b : c : d = 10 : 15 : 12 : 14

104 | CAT Complete Course

Visit : www.EasyEngineering.net
Visit : www.EasyEngineering.net

Second Method : In Second vessel :


a = Product of numerators 2
Quantity of water =
a = 2 × 5 × 6 = 60 5
3
b = Denominator of Ist ratio × product and milk =
5
of numerator of rest ratios
In resultant vessel :
= 3 × 5 × 6 = 90
1
c = 4 × 3 × 6 = 4 × 18 part of mixture of First vessel is taken and
5
d = 3×4×7 4
part of mixture of Second vessel is taken
a : b : c : d = 10 : 15 : 12 : 14 5
12. (A) Let the numbers be 5X, 3X, 4X. So,the ratio of water to milk in the new vessel
Then, (5X)2 + (3X) 2 + (4X) 2 = 800 1 1 2 1  2 4 3 4 
∴ × + × :  × + × 
50X2 = 800 3 5 5 5  3 5 5 5 
X = 4  1 2   8 12 
=  15 + 25  :  15 + 25 
13.
ww
So, the numbers are 12, 16 and 20.
(D) In the first mixture
2 =
  
5 + 6 40 + 36
75
:
75

w Amount of milk = × 50 = 20 liters

.Ea
5
Amount of water = 30 litres
If X litres of water is added to the mixture, then
=
11 76
:
75 75
= 11 : 76

syE
20
30 + X
=
1
3
60 = 30 + X
16. (E) Let six years ago their ages were 3X, 2X and X
respectively.
Now, according to question,

14. (C) Number of coins =


X = 30 litres
Amount in rupees ngi Sum of present ages = 90
Or, (3X + 6) + (2X + 6) + (X + 6) = 90
Value of coins in rupees
Number of one rupee coin = X
2X
nee Or,
Or,
6X + 18 = 90
6X = 72
Number of 50 paisa coin =
1
2
= 4X

rin X = 12
C’s present age = 12 + 6

Number of twenty five paisa coin


5X 17. (B) Let
a
b+c g.n
=
b
c+a
= 18 years

e
= = 20X
1

t
c
4 = =k
a+b
Given : X + 4X + 20X = 1000 ∴ a = (b + c)·k ……(1)
25X = 1000 b = (c + a)·k ……(2)
X = 40 c = (a + b)·k ……(3)
Number of rupee coin = 40 Now, Adding equation (1), (2) and (3), we get
Number of fifty paisa coin = 160 a + b + c = (2a + 2b + 2c)·k
Number of twenty five paisa coin 1
k =
= 800 2
Total value of coins 18. (B) Let the number of coins be X, 3X, 5X and 7X.
1 1 Now, X coin of one rupees = X rupees
= 1 × 40 + × 160 + × 800
2 4 3X
= 40 + 80 + 200 3X coins of fifty paise = rupees
2
= Rs. 320 5X
5X coins of twenty paise = rupees
15. (A) In First vessel : 4
1 2 7X
Quantity of water = and milk = 7X coins of ten paise = rupees
3 3 10

CAT Complete Course | 105

Visit : www.EasyEngineering.net
Visit : www.EasyEngineering.net

According to question, Now, Son’s share = 8·X


3X 5X 7X Wife’s share = 12·X
X+ + + = 22·25
2 4 10 Daughter’s share = 15·X
89X
Or, = 22·25 Now according to question,
20
15·X – 8·X = 49,000
Or, X = 5
Or, 7·X = 49,000
∴ Number of rupee = 5
X = 7,000
∴ Number of 50 paise coin = 15
Total worth of property
∴ Number of 25 paise coin = 25
= 8X + 12X + 15X
∴ Number of 10 paise coin = 35
= 35X
19. (A) Let numerator = X
= 35 × 7000 = 2,45,000
Denominator = Y
22. (A) Always remember, when a number is to be
X–5 1
= ……(1) divided in the proportion assume the number as First
Y–5 2
divided in X : 1.

ww
and

From equation (1)


X+2
Y+2
=
2
3
……(2)
First part =
X
X+1

w 2X – Y = 5

.Ea
From equation (3)—equation (2), we get
X = 10 + 2 = 12
……(3)

Given :
Second part =

2  X  1
 +
5 X + 1  3
1
X+1
 1 
X + 1 
 

Fractional number = syE


Y = 20 – 5 = 15
12 4
=
15 5
1
=
4

20. (A) L.C.M. of 3, 4, 5 and 6 is 60


ngi
So, the pens are distributed among A, B, C and D in
Or,
6·X + 5
15(X + 1)
=
1
4

the ratio
1 1 1 1
× 60 : × 60 : × 60 : × 60
nee Or,
Or,
24·X + 20 = 15·X + 15
24·X – 15·X = 15 – 20

rin
3 4 5 6 Or, 9·X = – 5
That is 20 : 15 : 12 : 10 5
X = –

g.n
9
Total number of pens
Required proportion = X : 1
= 20·X + 15·X + 12·X + 10·X
= 57·X
For minimum number of pens,
X = 1
e
= 5:9
23. (A) Number of passengers care 2·X, 3·X and 5·X
Rate = X, 2X·4X t
So, Sita should have at least 57 pens. Since, income = Number of passenger × rate
21. (C) According to question, = income in the ratio
Son’s Share 2 = 2 : 6 : 20
=
Wife’s Share 3 Income from A. C. sleeper class
Wife’s Share 4 2
= = × 56000
Daughter Share 5 28
Son’s Share Wife’s Share = Rs. 4,000
Now, =
2 3
24. (D) Income of Ram = 3·X
Wife’s Share Daughter Share
= Income of Raju = 2·X
4 5
Expense of Ram = 5·Y
Son’s Share Wife’s Share
= Expense of Raju = 3·Y
8 12
Daughter Share Now, saving amount of Ram = 3X – 5Y
=
15 Saving amount of Raju = 2X – 3Y

106 | CAT Complete Course

Visit : www.EasyEngineering.net
Visit : www.EasyEngineering.net

⇒ (3X – 5Y) + (2X – 3Y) = 2000  14·X 21·X 15·X 


Now, 100 ×  + + 
⇒ 5X – 8Y = 2000 ……(1)  30 40 35 
Since saving of Ram = Saving of Raju = 11,950
3X – 5Y = 2X – 3Y  14 21 15 
or  + +  100X
X = 2Y ……(2)  5 × 6 5 × 8 5 × 7
Putting the value of X in equation (1), we get  14 21 15  1
5 × 2Y – 8Y = 2000 =  + +  × 100X
6 8 7 5
2Y = 2000
 7 21 15 
Y = 1000 =  + +  × 20X
3 8 7 
So, Ram’s income = 3·X
 392 + 441 + 360 
= Rs. 6,000 =  
 168 
25. (A) Let the saving amount of Ram = 14·X
1193 × 20
Let the saving amount of Shyam = 21·X = 1260 = X
168

ww Let the saving amount of Sohan = 15·X = 900


Since, Rs. 30 is saved when income Or, X =
= 11950
11950 × 168

w .Ea
∴ Rs. 14·X is saved when income =
= 100
100
30
× 14·X
X ≈ 84
1193 × 20

Salary or income of Shyam

syE 21·X 100


Shyam’s income = 100 × ≈ × 21 × 84
40 40
15·X ≈ Rs. 4,410
Raju’s income = 100 ×
35

ngi = Rs. 4,425


●●●

nee
rin
g.n
e t

CAT Complete Course | 107

Visit : www.EasyEngineering.net
Visit : www.EasyEngineering.net

8 Mixture or Alligations
(A) Mixture—A mixture of two or more different Solution :
ingredients is formed when they mix in any ratio. Using formula :
Alligation rule—Alligation means “linking”. C2 – Cm
Ratio =
When different quantities of some or different Cm – C1
ingredients, of different cost are mixed together to 0·25 1
= =
produce a mixture of a mean cost, the ratio of their 0·75 3
quantities are inversely proportional to the difference in (C) When two mixtures of same ingredients mixed in

ww
their cost from the mean cost.
Quantity of smaller cost ingredient
Quantity of larger cost ingredient
different ratios—
Mixture has ingredients (A, B) in X : Y
Mixture has ingredients (A, B) in a : b

Now,
w m–C
d–m .Ea
=

= 2
x
x1
Larger cost – Mean cost
Mean cost – Smaller cost The First mixture ratio of quantities =

Second mixture has ratio of quantities =


X
Y
a

syE
b
C.P. of a unit Quantity of cheaper (C) Now, M unit of mixture first and N unit of second
C.P. of unit Quantity of dearer (d) mixture are mixed to form a resultant mixture with

ngi
ingredients (A and B) in AA : AB .
Mean price = m
Case I : When AA and AB are to be found out—
(B) Price / Value of the mixture—
Quantity of ingredient A A
Case I : When two ingredients X and Y are mixed.
A1 and A2 be the amounts of ingredients X and Y
respectively.
nee Quantity of ingredient B
 x 
M×

= A
AB

 + N ×  a +a b 
C 1 and C2 be their cost price. =
 X
rin
+
 Y 
Y  



 + N ×  a +a b 
Now,Cost price mixture is given by
C A + C2 A 2
Cm = 1 1
A1 + A 2
M×
X + Y 
g.n  
Amount of ingredient A in the resultant mixture
Case II : When more than two ingredients are mixed.
C A + C2A2 + C3A3 + …… + Cn An
Then , C m = 1 1
A1 + A2 + A3 + …… + An
=
AA
AA + AB e
× (M + N)

Amount of ingredient B in the resultant mixturet


Illustration 1. AB
= × (M + N)
AA + AB
In what proportion should one variety of oil at Rs. 8/
kg be mixed with another of Rs. 10 /kg to get a mixture Illustration 3.
worth Rs. 9 /kg ? Ramu lent out Rs. 12,00 in two parts one at 8% and
Solution : the other at 10% interest. The yearly comes out to be
9·5%. Find the amount lent in two parts ?
Using formula :
Solution :
10 – 9 1
Ratio = = =1:1 0·5 1
9–8 1 So, Ratio of quantity = =
1·5 3
C – C m 10 – 9 1
Ratio = 2 = = =1:1 ∴ Quantity of money lent at 8%
Cm – C1 9 – 8 1
1
Illustration 2. = × 1200 = Rs. 300
4
Alcohol cost Rs. 3·5 /litre and kerosene oil cost Rs. ∴ Quantity of money lent at 10%
2·5 / litre. In what proportion should these be mixed, so 3
that the resulting mixture may be 2·75 / litre ? = × 1200 = Rs. 900
4

108 | CAT Complete Course

Visit : www.EasyEngineering.net
Visit : www.EasyEngineering.net

Case II : When M and N are to be found Illustration 6.


X Two equal glasses filled with mixtures of alcohol and
Quantity of ingredient A in mixture-I =
X+y water in the proportion of 2 : 3 and 2 : 1 respectively
a were emptied into a third glass. What is the proportion of
Quantity of ingredient A in mixture-II =
a+b alcohol and water in the third glass ?
AA Solution :
In the resultant mixture =
AA + AB Using formula :
Now,
Quantity of mixture-I
=
C  X   a 
Quantity of mixture-II D M×  +N× 
AA X + Y  a + b 
a AA =
– AB  Y   b 
a + b AA + AB M×  +N× 
=
AA X X + Y  a + b 

AA + AB X + Y When since glasses are equal :
∴ Amount of mixture-I in resultant mixture ∴ M = N
 C  X 2

ww =  C + D  × (M + N)
 
Amount of mixture-II in resultant mixture
Given

and
Y
a
b
=

=
3
2
1

w
Illustration 4. .Ea
=
 D 
 C + D  × (M + N)
  ∴
AA
AB
=
2 2
+
5 3 16 8
=
3 1 10 5
+
=

syE
5 3
Three litres of water is added to a certain quantity of
So, in third glass, alcohol and water are in proportion
pure milk costing Rs. 6 /litre. If by selling the mixture at
of 8 : 5.
the same price as before, a profit of 20% is made. What is
the amount of pure milk in the mixture ?
Solution : ngi Removal and Replacement by Equal Amount
(i) Suppose a container contains M unit of mixture
Since, selling price of mixture = cost price of mixture

Or,
120
100
× C.P. of mixture = 6 nee
of ‘A’ and ‘B’. From this R unit of mixture is taken out
and replaced by an equal amount of ingredient B only this
process is repeated ‘n’ times, then after n operations.

Or, C.P. of mixture =


6 × 10
12
=5 ∴
rin
Amount of ‘A’ Left  R
= 1 – 
Amount of ‘A’ originally present  M 
n

Ratio milk and water is given by


Milk
=
5–0
=5 g.n
Amount of B left = M – Amount of A left

Since,
Water
C.P. of water = 0
∴ For 3 litres of water.
6–5

Quantity of milk = 5 × 3 = 15 litres.


e
(ii) Consider a container containing only ingredient

by and equal amount of ingredient B. This process is


repeated n times, then after n operations. t
‘A’ of X0 unit. From this Xr unit is taken out and replaced

n
Amount of ‘A’ Left  X
Amount of ‘A’ originally present  X0
Illustration 5. = 1– r
How many kg of sugar at 34 paise/kg must Raju n
mix with 25 kg of salt at 20 paise/kg. So, that he may,  X
So, Amount of ‘A’ left = X0 ×  1 – r 
on selling the mixture at 40 paise/kg. gain 20% on the  X0 
outlay ? n
 Xr 
Solution : 1 – X 
Using formula : Amount of A left  0
So, = n
100 Amount of B left  Xr 
Cost price of mixture = 40 × = 32 paise 1 – 1 – 
120  X0 
Using formula : Illustration 7.
A 32 – 20 12
Ratio of quantity = 34 = = =6 In two alloys, copper and zinc are related in the
A20 34 – 32 2 ratios 2 : 1 and 2 : 3 respectively. After alloying together
So, A34 = 6 × A20 12 kg of the first alloy, 10 kg of the second and a certain
= 6 × 25 = 150 kg. amount of zinc and alloy is obtained in which copper and

CAT Complete Course | 109

Visit : www.EasyEngineering.net
Visit : www.EasyEngineering.net

zinc are in equal proportions. Find the weight of pure zinc Illustration 9.
added ? Six kilograms of sugar at Rs. 15 /kg and 5 kgs of
Solution : sugar at Rs. 20 /kg are mixed together and the mixture is
Using formula : sold at a 11% profit. What is the selling price per kg of
the mixture ?
2 2
12 × + 10 × Solution :
Amount of Copper 3 5
=
Amount of Zinc 1 3 Cost price per kg of mixture is given by
12 × + 10 × + x
3 5 6 × 15 + 5 × 20
=
8+4 6+5
=
4+6+x 90 + 100 190
= =
Where x is the amount of pure zinc. 11 11
Amount of Copper 1 Now, Selling price / kg of mixture
So, Given =
Amount of Zinc 1 110 190
= × = Rs. 19 / kg.
10 + x 100 11
Or = 1
12 Illustration 10.

ww
Or,
Illustration 8.
x = 2 kg. Two liquids are mixed in the proportion of 2 : 1 and
the mixture is sold at Rs. 12 per litre at a 20% profit. If

w
the First liquid costs Rs. 2 more per litre than the second.
In two alloys, the ratio of copper and zinc are 5 : 4 What does it cost / litre ?

.Ea
and 1 : 2. How many kg of the first alloy and of the
second alloy should be melted together to obtain 24 kg of
a new alloy with equal contents of copper and zinc ?
Solution :
Since, mixture is sold at 20% profit.
Solution :
syE
Here, two alloys having same ingredients are mixed
So, cost price of the mixture =
100
120
× 12 = Rs. 10

Let the price of First liquid = Rs. x /litre


to obtain a new alloy.
Amount of each alloy is to be found out
ngi The cost price of Second liquid = Rs. (x + 2) / litre
2x + 1 (x + 2)

nee
individually. Now, Cost price of mixture =
X 5 3
So, In First mixture copper = =
X+Y 9 3x + 2
10 =
Copper in Second mixture =
a
a+b 3
=
1
rin 28 = 3x
3

In the resultant mixture =


AA
AA + AB
= copper
g.n x = Rs.
28
3
/ litre.

C
=

X
1
2


AA
X + Y AA + AB
Illustration 11.

e
Sea water contains 10% salt by water. How many
t
litre fresh water must be added to 80 litre of sea water for
the content of salt in solution to be made 4% ?
So, now =
D AA a Solution :

AA + AB a + b Amount of salt present in 80 litre of sea water
5 1 10
– = 80 × = 8 kg
9 2 100
=
1 1
– After adding x litre of fresh water mixture = 80 + x
2 3
8 4
C 3 1 Now, =
= = 80 + x 100
D 9 3
Or, 200 = 80 + x
C
So, Amount of First alloy = × 24 Or, x = 120 kg.
C+D
Illustration 12.
1
= × 24 = 6 kg A sum of Rs. 6.25 is made up of so coins which are
4
either 10 paise or 5 paise coins. How many of it are of 5
Amount of Second alloy = 24 – 6 = 18 kg. paise coins ?

110 | CAT Complete Course

Visit : www.EasyEngineering.net
Visit : www.EasyEngineering.net

Solution : Or, 10Y – 20 = 3Y + 12


Let, In the mixture : 7Y = 32
Number of coins = m 32
Y =
6·25 625 7
Price of mixture = = paise
80 80 Illustration 15.
6·25 Three equal glasses are filled with a mixture of spirit
10 –
5 – paise coins 80 and water. The proportion of spirit to water in each glass
Now, =
10 – paise coins 6·25 is 1 : 2, 2 : 3 and 3 : 1 respectively. The contents of three
–5
80 glasses are emptied into a single vessel. What is the
800 – 625 175 7 proportion of spirit and water in the vessel ?
= = = ·
625 – 400 225 9 Solution :
Illustration 13. 1 2 3
+ +
A mixture contain milk and water in ratio of 4 : X. Amount of Spirit 3 5 4
Now, =
When 30 litres of the mixture and 6 litre of water are Amount of Water 2 3 1
+ +
mixed the ratio of milk and water becomes 2 : 1. Find the 3 5 4

ww
values of x ?
Solution :
Exercise A
=
20 + 24 + 45 89
=
40 + 36 + 15 91

w
Amount of mixture = 36 litres
2 1. In what proportion must tea at Rs. 72/kg be mixed

.Ea
Amount of milk = × 36 = 24 litre
3
1
Amount of water = × 36 = 12 litre
with tea at Rs. 90/kg in order to obtain the mixture
worth Rs. 85 / kg ?

On mixing,
Amount of milk
syE 3

4
(A)
5
18
3
(B)
7
3
13

ngi
= (C) (D)
Amount of water X 7 5
24 4 5
Or, = (E)

Or,
Or,
6
X×6 = 6
X

X = 1 litre.
nee 13
2. In what proportion water is mixed with pure milk in
order to make a profit of 25% by selling it at cost price?
Illustration 14. (A) 3 : 4
rin (B) 4 : 5

g.n
2 (C) 1 : 6 (d) 1 : 5
A dishonest milkman fills up his bucket which is th
3 (E) 1 : 4
full of milk, with water. He again removes 2 litres of his
mixture from the bucket and adds an equal quantity of
water. If milk is now 60% of the mixture. What is the
capacity of the bucket in litres ?
Solution :
e
3. Two vessels A and B contain milk and water in the
ratio 7: 5 and 17 : 4 respectively. In what ratio
mixtures from two vessels should be mixed to get
a new mixture containing milk and water in the ratio
3:2?
t
Let Y be the capacity of bucket. 22 7
(A) (B)
Y = original amount of mixture 105 12
Amount of mixture removed = 2 litre 88 17
(C) (D)
2 7 21
So, Milk = (Y – 2) ×
3 3
(E)
1 5
Water = × (Y – 2) + 2
3 4. There are n vessels of sizes, C1 , C2 , C3 , ……… Cn
2 containing mixtures of milk and water in the ratio
× (Y – 2)
3 60 3 a1 : b 1 , a2 : b 2 ………… a n : b n respectively. The
Now, = =
1 100 5 contents are emptied into a single large vessel. Find
× (Y – 2) + 2
3 the ratio of milk to water in the resulting mixture ?
2 × (Y – 2) 3 5. Three glasses of sizes 3 litres, 4 litres and 5 litres
Or, =
Y–2+6 5 contain mixture of milk and water in the ratio of 2 : 3,
Or, (2Y – 4) × 5 = 3 × (Y + 4) 3 : 7 and 4 : 11 respectively. The contents of all the

CAT Complete Course | 111

Visit : www.EasyEngineering.net
Visit : www.EasyEngineering.net

three glasses are poured into single vessel. Find the (A) 1 : 4 : 4 : 5 (B) 1 : 2 : 4 : 4
ratio of milk to water in the resultant mixture ? (C) 3 : 2 : 4 : 5 (D) 5 : 2 : 4 : 5
(A) 1 : 1 (B) 14 : 31 (E) 1 : 2 : 4 : 5
(C) 56 : 15 (D) 124 : 15
13. Ramu has 60 kgs of rice. He sells a part of it at 20%
(E) 9 : 21 profit and the rest at 30% profit. If he gain 25% on
6. Four vessels of equal sizes contain mixture of sprit the whole, find the quantity of each part ?
and water. The concentration of spirit in four vessel (A) Quantity sold at 30% profit = 20 kg; Quantity
are 50%, 60%, 75% and 90% respectively. If all the sold at 20% profit = 40 kg
four mixtures are mixed. Find in the resultant (B) Quantity sold at 30% profit = 10 kg; Quantity
mixture the ratio of spirit to water ? sold at 20% profit = 50 kg
(A) 11 : 5 (B) 13 : 16 (C) Quantity sold at 30% profit = 25 kg; Quantity
(C) 15 : 14 (D) 5 : 4 sold at 20% profit = 35 kg
(E) 25 : 12 (D) Quantity sold at 30% profit = 30 kg; Quantity
7. Three glasses of capacity 2 litres, 3 litres and 5 litres sold at 20% profit = 30 kg

ww
contain mixture of milk and water with concentration
60%, 80% and 90% respectively. The content of three
glasses is emptied into a large vessel. Find the milk
(E) Quantity sold at 30% profit = 35 kg; Quantity
sold at 20% profit = 25 kg
14. Rajesh bought two tables for Rs. 2000. He sells one

w
concentration and ratio of milk to water in the
resultant mixture ?
(A) 81 : 19
(C) 45 : 14 .Ea
(B) 1 : 1
(D) 17 : 19
at 10% loss and the other at 10% profit and thus on
the whole. He had no gain no lose in whole transac-
tion. Find the cost price of each ?
(A) Rs. 1000 for Loss Rs. 1000 for gain
(E) 7 : 9
syE
8. 80 litres of milk has 75% milk concentration. How
(B) Rs. 950 for Loss Rs. 1050 for gain
(C) Rs. 800 for Loss Rs. 1200 for gain

ngi
much water should be added to make its concentra- (D) Rs. 1200 for Loss Rs. 800 for gain
tion 60% ?
(E) Cannot be determine
(A) 14 litres (B) 21 litres
(C) 24 litres
(E) 16 litres
(D) 18 litres
nee
15. Sita bought a certain quantity of sugar for Rs. 100.
She sells one fourth of it at 5% loss. At what per cent
profit should he sell the remainder stock so as to
9. 42 litre of a mixture has wine and water in the ratio
4 : 5. How much water must be added to get wine to
rin
make an overall profit of 20% ?
85

g.n
water of 5 : 7 in the resultant mixture ? (A) % profit (B) 85% profit
3
(A) 84 litres (B) 62 litres 65
(C) 65% profit (D) % profit

e
(C) 57 litres (D) 46 litres 3
(E) 74 litres
10. In what proportion may three kinds to tea prices @
Rs. 80, Rs. 70 and Rs. 50/kg be mixed to produce a
mixture worth Rs. 65 /kg ?
(E)
35
3
% profit

16. In a courtyard there are many chickens and goats. If


heads are counted it comes to 80 but when legs are
t
(a) 3 : 2 : 5 (B) 2 : 3 : 5 counted it comes 220. Find the number of chickens
(C) 1 : 2 : 3 (D) 3 : 3 : 4 and goats in the courtyard ?
(E) 1 : 1 : 1 (A) 45 : 35 (B) 55 : 25
11. In what proportion may three kinds of wheat bought (C) 50 : 30 (D) 60 : 20
@ Rs. 8, Rs. 10 and Rs. 14 be mixed to produce a (E) 40 : 40
mixture. Which would earn 25% on selling it at Rs.
15 / kg. ? Exercise B
(A) 1 : 1 : 3 (B) 2 : 2 : 3 1. Shyam buys spirit at Rs. 75 /litre adds water to it and
(C) 3 : 1 : 3 (D) 1 : 2 : 3 then sells it at 91 /litre. What is the ratio of spirit to
water if his profit in deal is 30%?
(E) 3 : 3 : 4
(A) 75 : 91 (B) 91 : 130
12. Find the proportion in which 4 types of tea priced @
Rs. 20, Rs. 30, Rs. 50. Rs. 80 be mixed so as to obtain (C) 71 : 20 (D) 75 : 16
a mixture worth Rs. 70 / kg ? (E) 70 : 30

112 | CAT Complete Course

Visit : www.EasyEngineering.net
Visit : www.EasyEngineering.net

2. There are two containers; the first contains 600 ml.  b  n  bn 


of alcohol, while the second contains 600 ml. of (A)  a  (B)  a 
water. Three cups of alcohol from the first container n
is taken out and is mixed well in second container. a  b
(C) (D)  1 – 
Then three cups of this mixture is taken out and is nb  a
mixed first container. Let X denote the proportion of
water in the first container and Y denote the propor- (E) Cannot be determine
tion of alcohol in the second container. Find the 9. From a cask of wine containing 20 litres, 5 litres are
relation between X and Y ? with drawn and the cask is refilled with water. The
[Capacity of each cup 100 mL.] process is repeated a second and then a third time.
Find the ratio of wine to water in the resulting
3. Sita has Rs. 2000. She invests a part of it at 2% per
mixture?
annum and the remainder at 5% per annum simple
interest. Her total income in 2 years is Rs. 500. Find (A) 1 : 5 (B) 13 : 15
the sum invested at higher rate of interest ? (C) 64 : 61 (D) 61 : 64
(A) Rs. 1200 (B) Rs. 1166·67 (E) 4 : 5

ww
(C) Rs. 833·33
(E) Rs. 1400
(D) Rs. 900 10. A vessel contains mixture of liquids X and Y in the
ratio 3 : 2. When 20 litres of the mixture is taken out
and replaced by 20 litres of liquid Y, the ratio

w
4. Raju covers a distance of 200 km. In 20 hours partly
changes to 1 : 4. How many litres of liquid X was
by walking at 10 km/hr and rest by running at 22

(A) 8·67 km .Ea


km/hr. Find the distance covered by higher speed ?
(B) 115 km
there initially present in the vessel ?
(A) 8 litres (B) 28 litres

(C) 19·67 km
(E) 191·67 km syE
(D) 86·67
(C) 18 litres
(E) 12 litres
(D) 22 litres

11. A piece of an alloy of two metals ( X and Y) weighs


5. The expenditure and saving of an employee are in
the ratio 3 : 2. His income increases by 15% but at
the same time his expenditure also increases by 20%. ngi 12 gm and costs Rs. 180. If the weights of two
metals be interchanged the new alloy would be worth
Find increase or decrease in his savings ?
(A) Decrease 7·5% (B) Decrease 5% nee Rs. 120. If the price of metal X is Rs. 15 /gm. Find
the weight of the other metal in the original piece of
alloy ?
(C) Remain same
(E) Increase 5%
(D) Increase 7·5% (A) 4·8 gm
(C) 4 gm rin (B) 1·2 gm
(D) 5 gm
6. A sum of Rs. 250 is divided among 10 students. Each
boy gets Rs. 20 where as a girl gets Rs. 30. Calculate
(E) 2 gm
g.n
the number of boys in class ?
(A) 5
(C) 4
(B) 6
(D) 3
e
12. A heard of 2 legged and 4 legged animals give a
head count as H. When legs are counted, it comes to
L numbers. Find the number of 4 legged animals in
terms of H and L ? t
L L+H
(E) 7 (A) + H (B)
2 2
7. A sum of Rs. 21 is made up of 60 coins which consist L–H
of either 50 paise or 25 paise. How many are there of (C) L – H (D)
2
25 paise coins ?
L
(A) 24 (B) 36 (E) – H
2
(C) 40 (D) 20 13. There were two different iron alloys of total weight
(E) 30 50 kg. The first contains 40% less iron than the
8. A vessel contains ‘a’ litres of liquid A and ‘b’ litres second. Determine the percentage of iron in the first
alloys, if it is known that there were 6 kg of iron in
be with drawn and replaced by liquid B, then ‘b’ litres
the first alloy and 12 kg in the second ?
of mixture be with drawn and replaced by liquid B
and the operation is repeated ‘n’ times in all. Find the (A) 15% (B) 35%
ratio of liquid A left after nth operation to the whole (C) 20% (D) 55%
quantity of liquid A initially present in the vessel ? (E) 60%

CAT Complete Course | 113

Visit : www.EasyEngineering.net
Visit : www.EasyEngineering.net

14. Four glasses of sizes 3 litres, 4 litres, 6 litres and In the resulting mixture :
7 litres contain mixture of milk and water in the ratio Milk : Water
of 2 : 1, 5 : 3, 6 : 3 and 9 : 5 respectively. Find the  a1 c1 a2 c2 an cn 
ratio of milk to water if the contents of all the four =  a1 + b1 + a2 + b2 + …… + an + bn  :
glasses are poured into one large vessel ?
(A) 2 : 5 (B) 8 : 7  b1 c1 b2 c2 bn cn 
(C) 13 : 7 (d) 5 : 7  a1 + b1 + a2 + b2 + …… + an + bn 
(E) 1 : 7 5. (B) Using previous formula :
15. An insert has 6 legs while another insect has 4 legs. Milk : Water
In a group with both types present the total number  2 × 3 4 × 3 5 × 4 
of heads is 80 and total number of legs is 420. Find =  2 + 5 + 3 + 7 + 4 + 11  :
the number of 4 legged insects ?
(A) 10 (B) 30  3 × 3 7 × 4 11 × 5 
 3 + 2 + 3 + 7 + 4 + 11 
(C) 20 (D) 40
(E) 23  6 12 20   9 28 55 

ww Answers with Hints


=  + + :  +
 5 10 15   5 10 15 
 56   124 
+ 

w Exercise A

.Ea
1. (E) Using allegation rule :
Quantity of cheaper tea
Quantity of dearer tea
=
90 – 85 5
=
85 – 72 13
=  : 
 15   15 

= 56 : 124 = 14 : 31.
6. (A) Here, the given per cent figures indicate the frac-

Now, syE
2. (D) Let cost price of pure milk be Re. 1 /litre
S.P. of mixture = Rs. 1 /litre
tion of spirit in the mixture.
Concentration of water in four vessel are 50%, 40%,
25% and 10% respectively.
Profit = 25%
So, C.P. of 1 /litre of mixture
ngi In the resultant mixture :
Spirit : Water = (0·5 + 0·6 + 0·75 + 0·9)
=
100
125
×1=
4
5
We assume the C.P. of 1 litre of water is zero. nee : (0·5 + 0·4 + 0·25 + 0·1)
= 2·75 : 1·25 = 1·1 : 0·05

Quantity of water
1
5 1 rin
= 11 : 5
7. (A) The total amount of milk

g.n
= =
Quantity of milk 4 4 = 2 × 0·6 + 3 × 0·8 + 5 × 0·9
5
= 1·2 + 2·4 + 4·5 = 8·1 litres
Ratio of water to pure milk in mixture 1 : 5.

3. (C) The amount of milk in First mixture =

The amount of milk in Second mixture =


7
12
17
= e
Milk concentration in the resultant mixture
8·1
2+3+5
× 100
t
8·1
21 = × 100 = 81%
10
3
In combination, the amount of milk = Water concentration in the resultant mixture
5
Now, Using allegation rule : = 100 – 81 = 19
22 1 ∴ Milk : Water = 81 : 19
Required ratio = :
105 60 8. (E) Given milk has 75% concentration water which
22 × 60 88 is to be added has 0% milk concentration.
= =
105 7 ∴ Find concentration of solution is 60%
4. Amount of milk in different vessels By allegation rule :
a1 c1 a c a c Water should be added to the given milk in the ratio
= , 2 2 ,……, n n
a1 + b1 a2 + b2 an + bn 15 : 60 = 1 : 4
Amount of water in different vessels ∴ Quantity of water to be added
b1 c1 b c b c 1
= , 2 2 ,……, n n = × 80 = 16 litres.
a1 + b1 a2 + b2 an + bn 5

114 | CAT Complete Course

Visit : www.EasyEngineering.net
Visit : www.EasyEngineering.net

4 14. (A) C.P. of table sold at 10% loss


9. (B) Fraction of water in the mixture =
9 1
= × 2000 = Rs. 1000
For water to be added fraction = 1 2
5 C.P. of the table sold at 10% gain
Fraction of water in the resultant mixture =
12 1
= × 2000 = Rs. 1000
So, water must be added in the ratio 2
31 7 31 15. (A) Let the remainder stock be sold at X% profit.
= : =
36 12 21 1
So, Quantity of water to be added X – 20 4
Now, =
31 25 3
= × 42 = 62 litres.
21 4
10. (D) We write the prices according order as 1
Or, X – 20 = 25 ×
3
Make pairs by choosing one from each side of the
mean price and apply allegation rule then add the Or, 3X = 25 + 60 = 85
quantity obtained under each price. 85
Or, X =
3

ww
This will give the ratio in which the ingredient should
be mixed.
Required ratio = 15 : 15 : 20
X =
85
3
% profit.

11. (C)w = 3 : 3 : 4.

.Ea
S.P. of mixture = Rs. 15 / kg
Profit = 25%
16. (C) Let the number of goats be X.
∴ Number of chickens = 80 – X

Now,
Total legs = 2 × (80 – X) + 4X
2X + 160 = 220

syE
C.P. of mixture = 15 ×

Now, Required ratio = 6 : 2 : 6


100
125
= Rs. 12 / kg 2X = 60
X = 30

ngi
So, Number of goats = 30
= 3 : 1 : 3. Number of chickens = 50
12. (E) Using allegation rule :
Exercise B
Required ratio = 10 : 20 : 40 : 50
= 1 : 2 : 4 : 5. nee
1. (C) S.P. of the mixture =
91
130
× 100 = Rs. 70
13. (D) First Method :
Let X kg of rice is sold at a profit of 20%. rin
Assuming cost of water is Rs. 0.
Now, using allegation rule, we get Required ratio of
Now, (60 – X) kg rice is sold at 30% profit.
Let the cost price of rice be Rs. 100 / kg.
spirit is 71 : 20.
g.n
2. Given Capacity of each cup be 100 mL.

Total C.P. = Rs. 60 × 100 = Rs. 6000


Gains 25% on the whole, therefore,

Total S.P. =
125
× 6000 = 7500
alcohol and 600 mL water. e
After first operation, first container will have 300 mL
of alcohol and second container will have 300 mL

Ratio water to alcohol in the second container


t
100 = 9:3=3 : 1
Also, Total S.P. = 120 × X + 130 × (60 – X) After second operation, the quantity of water and
3
7500 = 7800 – 10X alcohol left would be 300 × = 225 mL
4
300
X = = 30 1
10 and 300 × = 75 mL
4
X = 30 kg. Now, Quantity of water in first container = 225 mL
So, he sells 30 kg at 20% profit and rest 30 kg at Alcohol = 75 + 300 = 375 mL
30% profit.
So, ratio of water and alcohol
Second Method : 225 3
Ratio = 5 : 5 = 1 : 1 = =
375 5
1 So, clearly there are different ratios
Quantity sold at 20% profit = × 60 = 30 kg
2
Second ratio > First ratio
Quantity sold at 30% profit = 30 kg Y > X

CAT Complete Course | 115

Visit : www.EasyEngineering.net
Visit : www.EasyEngineering.net

3. (B) Average rate of interest 2


Number of 50 paise = × 60 = 24
500 × 100 50 5
= = %
2000 × 2 4 Number of 25 paise coins
25 = 60 – 24 = 36
= % = 12·5%.
2 b b
Investment at 2% per annum 8. (D) b litres = of litres = of the whole quantity of
a a
7·5 2·5 liquid A after first operation
= × 2000 = × 2000
18 6
 b
25
× 2000 = Rs.
2500 =  1 – a  of whole
=
60 3  
10·5 Quantity of liquid after second operation
Investment at 5% per annum = × 2000
18  b 2
3·5 =  1 –  of whole
= × 2000  a
6
35 Quantity of liquid A left after second operation
= × 100 Quantity of liquid A initially present
3

ww = Rs.
3500
3

= 1 –

b

a
2

w
= Rs. 1166·67 From n times :
200 Quantity of liquid A left after nth operation

.Ea
4. (E) Average speed = = 40 km/hr
5 Whole quantity of liquid A initially present
Ratio of time taken at 10 km/hr to 10 km/hr
 b
n

= 1 – 
syE
= 2 : 10 = 1 : 5
Time taken at 10 km/hr  a
1 5 5 1
= × 5 = hr 9. (C) = th part of cask

ngi
6 6 20 4
Distance covered at 10 km/hr Using the formula :
10 25 Quantity of wine left third with drawn
×5=

nee
=
6 3  1  3 4  3
Distance covered at 22 km/hr =  1 – 5  =  5  of the whole
   

rin
25 575
= 200 – = = 191·67 km. 64
3 3 = × 20 litres
125
5. (D) Here, expenditure and saving are two ingredients
of income. Therefore we can write as under, assum-
ing x % increases in savings. = 20 – g.n
So, Quantity of water left after third withdraw
64
× 20

Or,
Or,
15 – x
5
=
30 – 2x = 15
2x = 15
3
2 =
125
Final ratio of wine to water
e
125
125 – 64
× 20 =
61
125
t
× 20 litre

15 64
x = = 7·5%. × 20
2 125 64
= =
6. (A) Average money per student 61 61
× 20
250 125
= = Rs. 25
10 Wine : Water = 64 : 61.
Number of Girls 30 – 25 1 10. (C) % of liquid Y initially present in the vessel
Now, = =
Number of boys 25 – 20 1 2
= × 100 = 40%
1 3+2
So, number of boys = × 10 = 5.
2 % of liquid Y finally present in the vessel
7. (B) Average value of 60 coins 4
= × 100 = 80%
21 × 100 1+4
=
60 First solution in which the percentage of liquid Y is
7 × 100 40%.
= = 35 paise
20 The second solution is liquid Y which is being
10 : 15 = 2 : 3 mixed and it has 100% liquid Y.

116 | CAT Complete Course

Visit : www.EasyEngineering.net
Visit : www.EasyEngineering.net

So, 80% of liquid Y present in the resultant mixture Quantity of copper in the first alloy
may be taken as average percentage so using rule of x – 40
= ×a
allegation on liquid Y present we can write— 100
The ratio of liquid left in the vessel to liquid Y being xb
= 12
mixed = 1 : 2. 100
Since, the quantity of liquid Y being mixed is 20 litres, 1200
b =
the quantity of liquid left in the vessel in 10 litres. x
Therefore, the total quantity of liquid initially present x – 40
and ×a = 6
in the vessel 100
600
= 10 + 20 = 30 litres a =
x – 40
3
Quantity of liquid X = × 30 = 18 litres. 600 1200
2+3 a+b = +
x – 40 x
11. (A) If the two alloys are mixed, the mixture would
600x + 1200x – 48000
contain 12 gm of each metal and it would cost Rs. =
x (x – 40)
(180 + 120) = Rs. 300
1800x – 48000
Cost of (12 gm of metal X + 12 gm of metal Y) = Rs. =
x 2 – 40x

ww
300
Cost of (1 gm of metal X + 1 gm of metal Y) =
300 So,
a – b = 50
1800x – 48000
= 50

w
Rs. = Rs. 25 x 2 – 40x
12
Or, 1800x – 48000 = 50x 2 – 2000x

.Ea
Cost of 1 gm of metal Y = Rs. (25 – 15) = Rs. 10
Average cost of original piece of alloy
180
Or,
Or,
50x 2 – 3800x + 48000 = 0
x 2 – 76x + 960 = 0

syE
= = Rs. 12 / gm Or, 2
x – 60x – 16x + 960 = 0
15
Quantity of metal X 12 – 10 2 Or, x (x – 60) – 16 (x – 60) = 0
So, = =
Quantity of metal Y 15 – 12 3 (x – 60)(x – 16) = 0
2
Hence, the Quantity of metal X = × 12 = 4·8 gm.
5
ngi x = 16, 60
Since, percentage iron content in an alloy cannot be

nee
12. (E) Let the number of 2 legged animals be Y, then the negative, we discard x = 16.
Number of 4 legged animals be H – Y. Percentage iron content in the first alloy = 20%
Total number of legs = 4 (H – Y) + 2Y % iron content in the second alloy = 60%
Given 4 (H – Y) + 2Y = L
4H – 2Y = L
rin
14. (C) Ratio of milk to water is
 2 × 3 5 × 4 6 × 6 9 × 7
 3 + 8 + 9 + 14 
2H – Y =
L
2
L
=

g.n 
 1 × 3 3 × 4 3 × 6 5 × 7
Y = 2H –
and number of 2 legged animals
= 2H –
2

L
2
 5
:
 3
e
9 
+
8

3
+

= 2 + + 4 +  : 1 + + 2 + 
 2 2  2
9

5
2
+ 
14 

t
Number of 4 legged animals  12 + 14   6 + 8 
 L =  :  
= H –  2H –   2   2 
 2
26 14
L L = : = 13 : 7.
= –H+ = –H 2 2
2 2
15. (B) Let number of 6 legged insects = X
13. (C) Let the weight of two alloys be a and b respec-
Number of 4 legged insects = 80 – X
tively.
According to question ,
a + b = 50
420 = 6 X + 4 × ( 80 – X)
Let the percentage iron content in the second alloy
be x %. 420 = 2X + 320
Then, the percentage iron content in the first alloy 100
X = = 50
2
= (x – 40)%
Quantity of iron in the second alloy So, 6 legged insects = 50
x 4 legged insects = 80 – 50 = 30.
= ×a ●●●
100

CAT Complete Course | 117

Visit : www.EasyEngineering.net
Visit : www.EasyEngineering.net

9 Time, Distance and Speed


The speed of a body is given by (B) To convert V m/sec in km/hr or km/minutes.
Distance Solution :
Speed =
Time If speed is given in V m/sec, then
Distance = Speed × Time Speed = V m/sec
The distance covered by the body is equal to the 1
km
multiplication of speed and time. 1000
= V×
Some Fundamental Rules 1

ww
1. For Constant Speed—
Distance ∝ Time Speed (m/sec) = V ×
3600
18
hr

km/hr

w
5
It means more distance is covered in more time for a Again,
constant speed.
.Ea
2. For the Same Distance—
Speed (m/sec) = V ×
60
1000

Speed ∝
1
syE
Time
Speed (m/sec) = V ×
3
50
km/minutes

ngi
More speed is acquired in less time for same distance. Illustration 1.
3. For the Same Time— What is the speed of a maruti car if it travels 132 km
in 132 minutes ?
Speed ∝ Distance
More speed is covered more distance in same time.
Conversion of One Unit System to Other
nee
Solution :
Distance covered = 132 km

(A) If speed is given in km/hr, then to change it in


rin
Time taken = 132 min.
= 132/60 hrs.
m/sec or m/minutes.
Solution : Now, Speed =
g.nDistance
Time
Now, speed =

=
km
hr
1000 metre
3600 sec
Illustration 2.
=
e
132 × 60
132
= 60 km/hr.

If Ram’s car speed is 60 km/hr, then find the time


t
5 taken by the car to travel a distance of 1200 km.
Speed 1 km/hr = m/sec
18 Solution :
If we have v km/hr Distance
Since, Time =
Then in m/sec it is given by Speed
5 1200
V km/hr = V m/sec = = 20 hours.
18 60
Similarly, if speed is V km/hr Illustration 3.
km Find the distance if Ramesh’s bike’s speed is 80
Speed = V. m/sec and it takes time 60 minutes.
hr
V × 1000 metre Solution :
=
60 minutes Distance = Speed × Time
50 = 80 m/sec × 60 min
Speed (km/hr) = V × metre/minutes
3 = 288000 m = 288 km

118 | CAT Complete Course

Visit : www.EasyEngineering.net
Visit : www.EasyEngineering.net

Average Speed km
= 18 ×
3600 sec
Total Distance Covered
Average Speed = 1
Total Time Taken = km/sec = 0·005 km/sec
200
d + d2 + d3 + …… + dn
Average Speed = 1 1 km
t1 + t2 + t3 + …… + tn = ×
200 1
s1 t1 + s2 t2 + s3 t3 + …… + sn tn minutes
60
=
t1 + t2 + t3 + …… + tn 60
= = 0·3 km/minutes
d1 + d2 + d3 + …… + dn 200
=
d1 d2 d3 d Illustration 7.
+ + + …… + n
s1 s2 s3 tn Sita covers a distance d1 km at V1 km/hr and, then d2
s1 + s2 + s3 + …… + sn km at V2 km/hr. Find his average speed during the whole

n journey.
Illustration 4. Solution :
If Gita’s car covers 73 km in 68 sec., 135 km in 82 Time taken to travel d1 km at V1 km/hr is

ww
sec., 89 km in 76 sec. and 63 km in 44 sec., respectively.
Then, find the average speed of the Gita’s car. t1 =
d1
V1
…(1)

w
Solution :
Time taken to travel d2 km at V2 km/hr
From formula :

Average Speed =
.Ea
Total Distance Covered
Total Time Taken
∴ t2 =
d2
V2
…(2)

Average Speed =
syE
d1 + d2 + d3 + d4
t1 + t2 + t3 + t4
Average Speed =
Total Distance Covered
Total Time Taken
d + d2
Average Speed = 1

ngi
73 + 135 + 89 + 63
Now, Average speed = × 60 t1 + t2
68 + 82 + 76 + 44
360 × 60 d1 + d2
=

Illustration 5.
=
270
= 80 km/hr.

nee d1 d2
+
V1 V2
 d1 + d2 
If Sonali’s car speed is 36 km/hr, then find car’s is
speed in metre/second or metre/minutes.

rin  d .V + d .V 
Average speed = V1 × V2 ×
 1 2 2 1
Solution :
Sonali’s car speed = 36 km/hr g.n
Average Speed for Same Distance
If distances are same, then d1 = d2
5
= 36 × = 10 m/sec
18
Again Sonali’s car speed = 10 m/sec
10 m
Then, Average speed = 2 ×
e
Average speed = 2 ×
 V1.V2 
V + V 
 1 2

Product of speeds
t
= Sum of speeds
1
minutes Illustration 8.
60
= 600 metre/minutes. A train covers a distance between A and B. If it
travels from A to B with speed 40 km/hr and from B to A
Illustration 6. with speed 60 km/hr, then find the average speed of the
Ram’s bike speed is 5 m/sec, then find bike speed in train.
km/hr or km/sec or km/minutes.
Solution :
Solution :
Since, distance between A and B is fix.
Now, Bike speed = 5 m/sec
 V1.V2 
1 So, Average speed = 2 ×  
5× km  V1 + V 2 
1000 36 18
= =5× =5×
1 10 5 40 × 60
hr Average speed = 2 ×
3600 40 + 60
Bike speed = 18 km/hr = 48 km/hr.

CAT Complete Course | 119

Visit : www.EasyEngineering.net
Visit : www.EasyEngineering.net

Illustration 9. 20 × 30 600
= × (1 + 2) = ×3
Sonu covers 50 km of his journey at 20 km/hr and 30 – 20 10
the remaining distance at 30 km/hr. If the total journey is d = 180 km.
of 110 km. What is his average speed for the whole Illustration 12.
journey ? Ram covers his onward journey at speed V1 km/hr
Solution : and covers the return journey of equal distance at speed
Since, distances are different. V2 km/hr. If the total time taken by Ram is T hours for
So time taken to cover 50 km t1 whole journey. What is the one way journey distance ?
50 Solution :
t1 = = 2·5 hr
20 Let the one way journey distance = d
Time taken to cover 60 km t2 d
Time taken during onward journey = t1 =
60 V1
t2 = = 2 hr d
30 Time taken during return journey = t2 =
Total Distance Covered V2
Now, Average speed =
Total Time Taken Now, according to question,

ww =
110
=
2·5 + 2 4·5
110 T = t1 + t2

T =
d
+
V1 V2
d

w
1100 220
= = km/hr.
45 9 V1.V2
Illustration 10.
.Ea
A train goes from A to B at speed V1 km/hr and is late Illustration 13.
d =
V1 + V 2
×T

syE
by t 1 hour. If it goes at V2 km/hr it reaches t2 hours early.
Now, find the distance taken between A to B.
Solution :
An old man walks to his office at 4 km/hr and returns
to his house at 2 km/hr. If he spends total 5 hours on his
to and for walking. What is the distance between his
Let the distance between A to B = d kms
d ngi house and office ?
Solution :

nee
Time taken to reach with speed V1 = According to question,
V1
d V1 = 4 km/hr
and time taken to reach with speed V2 = V2 = 2 km/hr
V2
According to question,
rin
T = 5 hours = t1 + t2
V1.V2
d
–t =
V1 1
1 1 
d
+t
V2 2
Now, d =

g.n
V1 + V 2
4×2
×T

8 20
× 5 = × 5 = km.

e
=
Or, d  V – V  = t1 + t2 4+2 6 3

Now,

Illustration 11.
 1 2
d =
V1.V2
V2 – V1
× (t1 + t2)
Relative Speed
Case I : When two bodies move with speed V1 t
km/hr and V 2 km/hr respectively in the same direction.

Monu goes to school from his house at speed 20 A V1
km/hr and is late by one hour. If he goes at 30 km/hr he
reaches 2 hours early. Find the distance between his

house and school. B V2
Solution : A’s speed in relative with B or B’s Speed in relative
Given Speeds are V1 = 20 km/hr with A is called Relative Speed.
V2 = 30 km/hr Relative Speed = V1 – V2 (If V1 > V2 )
Case II : When two bodies move with speed V1 km/hr
t1 = 1 hour
and V2 km/hr respectively in the opposite direction.
t2 = 2 hours →
From the formula : A V1
V1.V2
d = × (t1 + t2) ←
V2 – V1 V2 B

120 | CAT Complete Course

Visit : www.EasyEngineering.net
Visit : www.EasyEngineering.net

A’s speed in relative with B or B’s Speed in relative 7 3


= 7+ =7+1
with A is called Relative Speed. 4 4
Relative Speed = V1 + V2 = 8 hrs 45 minutes.
Illustration 14. Illustration 17.
Two cars A and B start from the same point at speeds When a person covers the distance between his house
60 km/hr and 40 km/hr in the same direction. Find the and office at 60 km/hr. He is late by 15 minutes. But
distance between them after 5 hours. when he travels at 80 km/hr he reaches 5 minutes early.
Solution : What is the distance between his house and office ?
Since, both the cars move in the same direction. Solution :
∴ Relative Speed = V1 – V2 We assign,
= 60 – 40 = 20 km/hr Late time as positive and early time as –ve.
Distance between A and B after 5 hours So, T1 = 15 minute
= 20 × 5 = 100 km. T2 = – 5 minute
Illustration 15. Given speeds are V1 = 60 km/hr

ww
Two cars A and B start from the same point but move
in opposite directions at speeds of 55 km/hr and 45 km/hr
respectively. Find (i) their relative speed and (ii) their
V2 = 80 km/hr
According to formula :

w
separation after 3 hours.
Solution :
.Ea
A’s speed = 55 km/hr
Distance between his house and office

=
V1 V2
(T – T2 )
V2 – V1 1
80 × 60

syE
B’s speed = 45 km/hr
(i) Relative speed = 55 + 45 = 100 km/hr
(ii) Now, separation after 3 hours
=

=
80 – 60
[(15 + 5) minute]

80 × 60  20 
×  hrs.  = 80 km.
= Relative speed × time
= 100 × 3 = 300 km. ngiIInd Method
20  60 

Illustration 16.
Car A starts from X at 5 a.m. and reaches Y at 10 nee T+
15
60
=
d
60
…(1)

rin
a.m. Another car B starts from Y at 7 a.m. and reaches X 5 d
T– = …(2)
at 2 p.m. At what time do the two cars meet on their way ? 60 80
Solution :

A’s speed =
Distance
By equations (1) and (2)
15 5
+
60 60
= g.n
d

d
60 80

e
Time

and B’s speed =


=

d
7
d
=
10 – 5 5
d
Illustration 18.
d = 80 km.

Two persons X and Y start their journey at the same


time in opposite directions from two points and after
t
In such case we use the formula, passing each other they complete their remaining journey
They will meet at in T1 and T2 hours respectively. Then, find the ratio of
= Second’s starting time speed of X and Y.
(First’s arrival time – Second’s starting time) Solution :
(Time taken by second)
+ A Z B
Sum of time taken by both
T1 = 5 am; T2 = 10 am; T3 = 7 am; T4 = 2 pm (14) V1 V2
(T2 – T3 )(T4 – T3 )
Required time = T3 + Let the total distance between the points is d km.
(T2 – T1 ) + (T4 – T3 )
X’s speed = V1
(10 – 7)(14 – 7)
∴ Required time = 7 + Y’s speed = V2
(10 – 5) + (14 – 7)
Since, X and Y move in opposite direction.
3×7
= 7+ So, Relative speed = V1 + V2
12

CAT Complete Course | 121

Visit : www.EasyEngineering.net
Visit : www.EasyEngineering.net

d Ram’s speed
Time taken when they meet = ∴ = 3

V1 + V 2 Shyam’s speed

Time taken to move from A to Z =


d 21√
3
V1 + V 2 Or, Shyam’s speed =
3

— d
Distance traveled by X = AZ = V1 × = 21 km/hr.
V1 + V 2
Illustration 20.
Similarly,
A train leaves Delhi at 6 a.m. and reaches Kanpur at
— d
Distance travelled by Y = BZ = V2 × 11 a.m. Another train leaves Kanpur at 7 a.m. and reaches
V1 + V 2 Delhi at 1 p.m. At what time do the two trains meet ?
Remaining distance travelled by X Solution :
d
= V2 × Let the distance between Delhi and Kanpur by d
V1 + V 2 km.
Remaining distance travelled by Y Let the speed of train leaving from Delhi is X
d and Let the speed of train leaving from Kanpur is Y.
= V1 ×

ww
V1 + V 2 d d
Now, X’s speed = = km/hr
Now, Time taken by X to travel 11 – 6 5
  d
 V2 × V d+ V 
w
Y’s speed = km/hr
6
—  1 2

.Ea
BZ (T1 ) =

Time taken by Y to travel


V1
…(1) Let the both the trains meet at t hour after 6 a.m.
Now, Distance travelled by A in t hour
d


AZ (T2 ) =
syE
 1

 V1 × V d+ V 
2
…(2)
= ×t
5
Distance travelled by B in (t – 1) hour

ngi
V1 d
= × (t – 1)
According to question, 6
Ratios of X and Y = T1 : T2 Now, Total distance = d
 V2 
V 
 1 T1 nee d
5
d
× t + × (t – 1) = d
6
t t–1
=
 V1 T2
V 
 2
Or,

rin
5
+
6
6t + 5 (t – 1)
= 1

 V22
 V  = T1
 1
T
Or,

Or,
30
g.n = 1

11t – 5 = 30

e
2
Or, 11t = 35


 V2 
V  =
 1
V1 : V2 =


T1
T2

T2 : √T1 .

2
11
t =
35
11
hours

t = 3 hours or at 9·11 am they will meet.


t
Illustration 19. Some Basic Formula For Trains
Ram starts his journey from Patna to Delhi and (A) If a trains crosses a stationary person or pole or
Simultaneously Shyam starts from Delhi to Patna. After signal post, then time taken by the train is given below :
crossing each other they finish their remaining distance in Length of the train
1 Time taken =
2 and 7 hours respectively. What is Shyam’s speed if Train’s speed
3
(B) If train cross a platform, bridge, tunnel or stand-
Ram’s speed is 21√ 3 km/hr ? ing train of length l km, then time taken is given by
Solution : If length of the Train = X km
Ram’s speed : Shyam’s speed = √√213
7:
Time taken =
Its speed = V km/hr
Length train + Length of tunnel/bridge
7 :√73
Speed of Train
= √
X+l
Time taken =
= 3:1
√ V

122 | CAT Complete Course

Visit : www.EasyEngineering.net
Visit : www.EasyEngineering.net

(C) If a train A of length l1 moving at speed V passes Illustration 23.


on object B (or train B) of length l2 whose speed is U in A train is moving with speed 36 km/hr. How much
the same direction. Then, time it will take to cross a standing pole, the length of the
Relative speed between train A and object B or train B train is 90 m ?
= V–U Solution :
Now, If V > U Speed of train, V = 36 km/hr
Time taken by train A to cross the object B 5
= 36 ×
18
l +l
T = 1 2 = 10 m/sec
V–U
Illustration 21. Distance l
Now, Length of the train = =
Time V
The distance between two stations P and Q is 480
90
km. Two trains that starts simultaneously from P and Q = = 9 metres.
10
and meet after 5 hours. If the difference in their speed is 8
km/hr. Find their speeds. Illustration 24.

ww
Solution :
According to above formula :
A train of length 120 metres is moving with speed 54
km/hr. How much it will take to cross a plateform of
length 120 meters ?

w a = 8 km/hr

.Ea
T = 5 hours
S = 480 km
Solution :
Given Speed of train, V = 54 km/hr = 54 ×

= 15 m/sec
5
18

=
1 S
 +a
2 T
syE
Now, Speed of the faster train



Length of the train, l1 = 120 m
Length of the platforms l2 = 120 m

1  480
= 
2 5
 1
+ 8  = (96 + 8)
 2 ngi From formula, Time taken =
l1 + l2 120 + 120
V
=
15

=
1
2
× 104 = 52 km/hr nee =
240 80
15
=
5
= 16 seconds.
Speed of the slower train
1 S   
Illustration 25.
rin
=  – a  = 12  480
2 T   5
–8

g.n
Two trains X and Y start simultaneously from
stations A and B and move towards each other at speed
V1 and V2 km/hr respectively. At the point where they

Illustration 22.
=
1
2
(96 – 8) = 44 km/hr.

Ram traveling at 60 km/hr reach his school from his


e
meet, one train has covered a km more than the other.
Now, find the distance between A and B.
Solution : t
Let V1 > V2
house 40 minutes earlier. If he had traveled at 50 km/hr
he would have reached 10 minute late. How far is the Let us suppose that they meet at time t hr.
school ? Now, Distance covered by the train X = V1 × t
Solution : Distance covered by the train Y = V2 × t
According to question, According to question,
d 40 d 10 V1 × t – V2 × t = a
+ = –
60 60 50 60
a
Or, t =
1 1  2 1 4+1 V1 – V2
Or, d –  = + =
 50 60  3 6 6 Total Distance = V1 × t + V2 × t
 6–5  5 = t (V1 + V2 )
Or, d  5 × 6 × 10 = 6
   V1 + V 2 
Total Distance = a ×   km.
d = 250 km.  V1 – V2

CAT Complete Course | 123

Visit : www.EasyEngineering.net
Visit : www.EasyEngineering.net

Illustration 26. If the both the trains are equal.


Let two trains start from Delhi and Bombay and move l1 = l2 = l
towards with speed 60 km/hr and 50 km/hr respectively.  T1 + T2
At the meeting point it is found that train from Delhi has V1 = l ×   m/sec
traveled 20 km more than the train coming from Bombay.  T1 . T2 
Find the distance between Delhi and Bombay ?  T – T2 
V2 = l ×  1  m/sec.
Solution :  T1 . T2 
In our question, Illustration 28.
V1 = 60 km/hr Two trains of lengths 1 km and 2 km run on parallel
V2 = 50 km/hr tracks. When running in the same direction the faster
a = 20 km train crosses the slower one in 20 seconds. When running
in opposite directions at speed same as their earlier
 V1 + V 2 
Then, Distance = a ×   speeds, they pass each other completely in 10 seconds.
 V1 – V2 Find the speed of each train.
 60 + 50  Solution :
= 20 ×  
ww = 20 ×
 60 – 50 
110
= 220 km.
l1 = 1 km
l2 = 2 km

w
Illustration 27.

.Ea
10

Two trains of lengths l1 and l2 metres run on parallel


tracks. When moving in the same direction, the faster
T1 = 20 sec
T2 = 10 sec
Now, according to above formula,
 l1 + l2   T1 + T2

syE
train passes the slower one in T1 seconds. But when they
are moving in opposite directions at same speeds as
earlier, they cross each other completely in T2 seconds.
V1 =  2   T . T  m/sec
  1 2
(1 + 2)1000  20 + 10 
Find the speeds of the trains.
Solution : ngi =

3000
2
30
× 
 20 × 10 
Let the speed of faster train = V1 km/hr
Let the speed of slower train = V2 km/hr
nee =

=
2
900
×
20 × 10

= 225 m/sec

rin
When they are running in the same direction. 4
Relative Speed = (V 1 – V2) km/hr  l1 + l2   T1 – T2 
 2   T . T  m/sec

g.n
Then, T 1 = time taken to cross each other in the same V2 =
  1 2
direction
l +l 3000  20 – 10 

Or,
= 1 2
V1 – V2
l +l
V1 – V2 = 1 2
T2
…(1)
=

e
2

= 1500 ×
×
 20 × 10
10
200

t
When they are running in opposite direction = 75 m/sec.
Relative Speed = V1 + V2 Illustration 29.
T2 = Time taken to cross each other in opposite A train running at 54 km/hr passes a tunnel com-
direction pletely in 5 minutes. While inside the tunnel it meets
l +l another train of three-fourth of its length traveling at 72
= 1 2
V1 – V2 km/hr and passes it completely in 8 seconds. Find the
l +l length of the trains and of the tunnel.
V1 + V 2 = 1 2 …(2)
T2 Solution :
Solving equation (1) and equation (2), we get Speed of the first train = 54 km/hr
 l1 + l2   T1 + T2 5
= 54 × = 15 m/sec
V1 =  2   T . T  m/sec 18
  1 2
Since, it passes tunnel completely in 5 minutes.
 l1 + l2   T1 – T2 
V2 =  2   T . T  m/sec If l be the length of train and d be the length of
  1 2 tunnel.

124 | CAT Complete Course

Visit : www.EasyEngineering.net
Visit : www.EasyEngineering.net

Distance In this case—


Then, Time required =
Speed Down-stream Speed = (V + U) m/sec. …(2)
l+d
5 × 60 sec = From equation (1) and equation (2)
15
Speed of boat in still water
Or, l + d = 75 × 60 m = 4500 m
1
Now, relative speed of the trains = (Down-stream speed + Up-stream speed)
2
= 15 + 20 = 35 m/sec.
Speed of the stream
Total distance covered in crossing the trains
1
3 7 = (Down-stream speed – Up-stream speed)
= l+ l= l 2
4 4
Illustration 31.
= Relative speed × Time
A swimmer covers a certain distance down-stream in
= 35 × 8 = 280 m 90 minutes but takes 100 minutes to return up-stream the
7 same distance. If he can swims in still water with the
Or, l = 280
4 speed of 60 metre per second. Find the speed of the

ww l = 160 m
Length of tunnel = 4500 – 160 = 4340 m.
current.
Solution :

w
Illustration 30. Using the formula,
Two trains C and D starts simultaneously from  T1 + T2

.Ea
stations A and B, S km a part and move toward each
other. They meet after time T hours. If the difference in
V = W× 
 T2 – T1 

syE
speeds of the two trains is a km/hr. Find the speeds of the Here, T1 = 90 min
two trains. T2 = 100 min
Solution : W = 60 m/sec
Let the speed of trains C and D be V1 and V2 km/hr
respectively and V1 > V2 .
ngi Now, V = W×
 T1 + T2

 T2 – T1 
Now, Given

So,
V 1 – V2 = a
Since, both the trains move in opposite direction.
Total distance = Distance covered by C
nee = 60 × 
 90 + 100 

 100 – 90 
+ Distance covered by D
S = V 1 × T + V2 × T rin
= 60 ×
190
10

V1 + V 2 =
S
T
…(1)
Illustration 32. g.n
V = 1140 m/sec.

V1 – V2 = a
Solving equation (1) and equation (2), we get
1 S 
…(2)
e t
A man can run a ship in still water at V km/hr. In a
stream flowing at W km/hr. If it takes T hours to run to a
point and come back. Find the distance between the two
∴ Speed of faster train = V1 =  + a  points.
2 T  Solution :
1 S  Down-stream speed = (V + W) km/hr
Speed of slower train = V2 =  –a
2 T  Up-stream speed = (V – W) km/hr
Boats and Streams Let the distance between the two points = d km
Up-Stream—The motion of boats or ships against Sum of time taken down-stream and up-stream
the stream is called up-stream. equals total time.
Let the speed of boat is V m/sec. d d
∴ + = T
While stream flows with U m/sec. V+W V–W
Now, Speed of the boat against the stream T (W + V)(V – W)
Or, d =
2.V
= (V – U) m/sec. …(1)
 V 2 – W2 
Down-Stream—The motion of boat and ship along d = T×  km.
the direction of stream is called down-stream motion.  2.V 

CAT Complete Course | 125

Visit : www.EasyEngineering.net
Visit : www.EasyEngineering.net

Illustration 33. (A) 12 km/hr (B) 15 km/hr


Ramu can swim down-stream 30 km in 3 hours and (C) 22 km/hr (D) 25 km/hr
up-stream 24 km in 3 hours. Find his speed in still water (E) 30 km/hr
and also the speed of the current ? 2. A man can row 20 km up-stream and 25 km down-
Solution : stream in 8 hours. Also he can row 30 km up-stream
30 km and 45 km down-stream in 10 hours. Find the speed
Down-stream = = 10 km/hr
3 hr of the man in still water ?
24 km 75 75
Up-stream = = 8 km/hr (A) km/hr (B) km/hr
3 hr 16 8
1 75 75
Speed in still water = (10 + 8) = 9 km/hr (C) km/hr (D) km/hr
2 4 2
1 (E) 75 km/hr
Speed of the stream = (10 – 8) = 1 km/hr.
2 3. A boat covers a certain distance down-stream in 2
Illustration 34. 5
hours, while it comes back in hours. If the speed of
Gita can Swim down-stream d1 km in T1 hours and 2

ww
up-stream d2 in T2 hours. Find her speed in still water and
speed of current.
Solution :
stream be 4 km/hr. What is the speed of the boat in
still water ?
(A) 45 km/hr (B) 224 km/hr

w .Ea
Now, down-stream speed = 1 km/hr

d
d
T1

Up-stream speed = 2 km/hr


(C) 36 km/hr
(E) 25 km/hr
(D) 18 km/hr

4. A train running at 54 km/hr takes 15 seconds to pass

syE
a platform. Next it takes 12 seconds to pass a man
T2 walking at 6 km/hr in the same direction in which the
1  d1 d2  train is going. Find the ratio between the length of
2  T1 T2 
Speed in still water = + km/hr the train and the length of the platform ?

Speed of the current =


1  d1 d2 
2  T1 T2 
– km/hr. ngi (A) 43 : 34
(C) 32 : 13
(B) 28 : 23
(D) 14 : 11

Illustration 35. nee (E) 7 : 5


5. A train A starts from Patna at 6 p.m. and reaches
Reena swims a certain distance down-stream in T1
hours and returns the same distance up-stream in T2
rin
Banaras at 7 p.m. while another train B starts from
Banaras at 6 p.m. and reaches Patna at 7:30 p.m. The

g.n
hours. If the speed of the stream be W km/hr. Find the two trains will cross each other. Find the time when
Reena’s speed in still water. they will crossing each other ?
Solution : (A) 6 : 24 pm (B) 6 : 36 pm
Let the speed of Reena in still water = V km/hr
Now, down-stream speed = (V + W) km/hr
Up-stream speed = (V – W) km/hr
(C) 6 : 48 pm
(E) 6 : 18 pm e
(D) 6 : 54 pm

6. Two trains, one from Delhi to Patna and the other t


from Patna to Delhi, start simultaneously. After they
According to question, meet, the trains reach their destination after 16 hours
The distance covered down-stream and up-stream and 64 hours respectively. What is the ratio of their
are equal. speeds ?
So, (V + W) × T 1 = (V – W) × T2 (A) 2 : 1 (B) 3 : 2
(C) 4 : 3 (D) 5 : 3
 T1 + T2 
V = W×  (E) 4 : 1
 – T 1 + T2
7. Two train 4001 and 1432 are 120 km apart. The train
 T + T2 4001 starts from A at 6 a.m. and travels towards B at
V = W× 1  km/hr.
 T2 – T1  45 km/hr another train starts from B at 7 a.m. and
travels towards A at a speed of 25 km/hr. At what
Exercise A time they meet ?
1. In a stream running at 2 km/hr a boat goes 6 km. up- (A) 8 : 04 am (B) 7 : 36 am
stream and back again to be the starting point is 33 (C) 7 : 12 am (D) 8 : 44 am
minutes. Find the speed of the boat in still water ? (E) 9 : 12 am

126 | CAT Complete Course

Visit : www.EasyEngineering.net
Visit : www.EasyEngineering.net

8. Two train start from A and B respectively and travel km/hr than the speed of Mumbai–Chennai Express.
towards each other at a speed of 54 km/hr and 36 Both trains meet at Wadi junction, which is exactly
km/hr respectively. By the time they meet, the first at the centre between Mumbai and Chennai. At what
train has traveled 150 m more than the second. Find time do both the trains meet ?
the distance between A and B ? (A) 11 : 30 pm same day
(A) 350 m (B) 450 m (B) 5 : 20 am next day
(C) 540 m (D) 300 m (C) 5 : 20 pm next day
(E) 500 m (D) Sharp 12 same day (mid night)
(E) Sharp 12 next day (mid day)
9. X is twice as fast as fast Y and Y thrice as fast as Z.
The journey covered by Z in 60 minutes then. Find 15. The distance between Patna and Delhi is 300 km two
time required by X ? trains simultaneously leave from Patna and Delhi.
(A) 8 minutes (B) 6 minutes After they meet, the train traveling towards Delhi
reaches there after 9 hours, while the train traveling
(C) 10 minutes (D) 12 minutes towards Delhi reaches there after 4 hours. Find the
(E) 15 minutes speed of each train in km/hr ?

ww
10. Walking at
4
5
of its usual speed, a train is 10 minutes
late. Find the usual time to cover the journey ?
(A) 30; 20
(C) 40; 30
(E) 60; 35
(B) 35; 25
(D) 25; 12

w
(A) 20 minutes
(C) 30 minutes
(E) 40 minutes
(B) 25 minutes

.Ea
(D) 35 minutes
16. Two trains 100 km a part towards each other on the
same track. One train travels at 55 km/hr, other
travels at 45 km/hr. A bird starts flying at a speed of
80 km/hr at the location of the faster train. When it

syE
11. A thief is spotted by a policeman from a distance of
1200 m. When the policeman starts the chase, the
thief also starts running. If the speed of the thief be
reaches the slower train, it turns around and flies in
the opposite direction at the same speed. When it
reaches the faster train again it turns around and so
12 km/hr and that of the policeman 16 km/hr, how
far the thief will have run before he is catches ?
(A) 1212 metres (B) 1220 metres ngi on. When the trains collide, how far has the bird
flown ?

(C) 1236 metres


(E) 1272 metres
(D) 1260 metres
nee (A) 60 km
(C) 80 km
(E) 100 km
(B) 70 km
(D) 90 km

12. A car driver travels from the plains to the hill stations,
which are 108 km apart at an average speed of 18 rin
17. Two trains 100 m and 120 m long are running in the
same direction with speeds of 72 km/hr and 54
km/hr. In the return trip, he covers the same distance
at average speed of 36 km/hr. Find the average speed g.n
km/hr. In how much time will the first train cross the
second ?
of the car cover the entire distance of 500 km. ?
(A) 12 km/hr
(C) 24 km/hr
(E) 36 km/hr
(B) 18 km/hr
(D) 45 km/hr
(A) 35 sec
(C) 58 sec
(E) 90 sec
e
(B) 44 sec
(D) 75 sec
t
18. A Train 4321 running at 81 km/hr takes 180 seconds
13. The average speed of a train in the onward journey is to pass a platform. Next it takes 75 seconds to pass a
50% more than that in the return journey. The train man walking at 9 km/hr in the same direction in
halts for one hour on reaching the destination. The which the train is going. Find the ratio length of the
total distance taken for the complete to and for jour- train and the length of the platform ?
ney is 16 hours, covering a distance of 1000 km. (A) 1 : 2 (B) 4 : 5
What is the speed of the train in the onward journey ? (C) 3 : 2 (D) 2 : 3
(A) 55·55 km/hr (B) 33·33 km/hr (E) 1 : 3
(C) 66·66 km/hr (D) Cannot be determine 19. Two trains running in opposite directions cross a
(E) Data insufficient man standing on the platform in 30 seconds and 20
14. The distance between Mumbai and Chennai is 1020 seconds respectively and they cross each other in 25
km Mumbai–Channai expresses leaves Mumbai CST seconds. Find the ratio of their speeds ?
at 6·00 p.m. towards Chennai on the same day, (A) 2 : 3 (B) 1 : 1
Chennai–Mumabi Express leaves Chennai for (C) 2 : 5 (D) 3 : 5
Mumbai at 8.00 p.m. and its speed higher by 10 (E) 3 : 2

CAT Complete Course | 127

Visit : www.EasyEngineering.net
Visit : www.EasyEngineering.net

20. There are two different roads between two cities. The 2. Ramesh arrives at his office 30 minutes late every-
first is 20 km longer than the second A car travels day. On a particular day, he reduces his speed by
along the second road and covers the distance 25% and hence arrived 50 min. late instead. Find
1 how much speed he should increase so that he will be
between the cities in 3 hours. Another car travels
2 on time on a particular day ?
1
along the second road and covers the distance in 2 (A) 50% (B) 75%
2
hours. If the speed of the first car is 40 km/hr less (C) 100% (D) 125%
than that of the second car, then what is the speed of (E) 150%
faster car ? 3. A car traveled 30% of time at a speed of 20 km/hr,
(A) 60 (B) 80 40% of time at a speed of 30 km/hr and rest of the
(C) 120 (D) 160 journey at a speed at 40 km/hr. What is the average
speed of the car for the entire journey ?
(E) 200
(A) 25 km/hr (B) 45 km/hr
21. Taruna can row a boat d1 km up-stream and d2 km
down-stream in T1 hours. He can row d1 ′ km up- (C) 30 km/hr (D) 60 km/hr
stream and d2 ′ km down-stream in T2 hours. Then (E) 15 km/hr

ww
find the down-stream speeds ?
d d ′ – d1′d2
(A) 1 2
d d ′ – d1′d2
(B) 1 2
4. A thief steals a car at 10.30 a.m. and drives it at 60
km/hr. The theft is discovered at 11·00 a.m. and the

w d1 T2 – T1 d2 ′
d d ′ – d1′d2
(C) 1 2
d1 T1 – T1 d1 ′
d d ′ – d2′d2 .Ea
d2 T2 – T1 d1 ′
d d ′ – d1′d2
(D) 1 2
d1 T2 – T2 d1 ′
owner sets off in another car at 75 km/hr when will
he overtake the thief ?
(A) 12·30 (B) 1·00

syE
(E) 1 1 (C) 1·15 (D) 1·45
d1 T2 – T1 d1 ′
(E) 1·30
22. A pedestrian and a cyclist start simultaneously
5. A man travels 600 km partly by train and partly by
towards each other from towns X and Y. Which are
80 km apart and meet two hours from start, then they
continue their journey and the cyclist arrive at X 20 ngi car. If he covers 400 km by train and the rest by car,
it takes him 6 hours and 30 minutes. But if he travels
hours earlier than the pedestrian arrive at Y. Find the
ratio of their speeds ?
nee 200 km by train and rest by car, he takes half an hour
longer. Find the speed of the car ?
(A) 50 km/hr (B) 60 km/hr

rin
(A) 1 : 2 (B) 1 : 3
(C) 1 : 4 (D) 1 : 9 (C) 65 km/hr (D) 75 km/hr

g.n
(E) 1 : 8 (E) 80 km/hr

23. Sita and Soni start simultaneously from point P 6. A cart race track has a circumference of 3000 m the
length of the race is 9000 m. The fastest and the
towards Q, 60 kms. apart. Sita’s spee is 4 km/hr less
than Soni’s speed. Sita after reaching Soni turns back
and meets Sita at 12 km from Soni. Find Sita’s
speed ?
e
slowest cart meet for the first time after the start of

t
the race at the end of the length in 10 minute. All the
carts start at the same point and fastest cart moves at
twice the speed of the slowest cart, what is the time
(A) 20 km/hr (B) 16 km/hr taken by the fastest cart to finish the race ?
(C) 24 km/hr (D) 12 km/hr
(A) 30 minutes (B) 25 minutes
(E) 9 km/hr
(C) 20 minute (D) 40 minutes
Exercise B (E) 45 minutes
1. Sita sees his friends standing at a distance of 240 7. The local shuttle service trains which travel at a
metres from his position. She increases her speed by uniform speed run at regular intervals, Sunita,
50% and hence takes 20 seconds now to reach her. walking down along the railway track at uniform
(i) If she travels at the original speed, how much speed, found that every 8 minutes there is a local
time will she take? train coming in opposite direction and every 24
(ii) What was her original speed ? minutes, there is a local train overtaking her from
behind. What is the time gap between one local train
(A) 30 sec; 8 m/sec (B) 40 sec; 6 m/sec
passing a stationary point on the railway route and
(C) 24 sec; 10 m/sec (D) 36sec. the immediately next local train in the same direction
(E) None of these passing the same point ?

128 | CAT Complete Course

Visit : www.EasyEngineering.net
Visit : www.EasyEngineering.net

(A) 12 minutes (B) 15 minutes 12. What is the speed of train ?


(C) 18 minutes (D) 20 minutes (i) The train crosses 300 metres long platform in 20
(E) 24 minutes seconds.
(ii) The train crosses another stationary train of
8. Two trains are moving towards each other at speed
equal length in 15 seconds.
of 60 m/sec and 90 m/sec. At the time, the two trains
are 150 m apart, a fly starts moving alongside one of (iii) The train crosses a signal pole in 10 seconds.
the trains at 5 m/sec the fly changes its course when (A) (i) and (ii) only
the two trains collide and flies a distance of 10 m to
(B) (i) and either (ii) or (iii) only
the East, then 25 m to the North and then 55 m to the
South. At this point it spots two cars moving head (C) (ii) and either (i) or (ii) only
long towards each other at speed 20 m/sec and 30 (D) (iii) and either (i) or (ii) only
m/sec the fly immediately changes its course again
(E) None of these
and flies at thrice its original speed. The distance
bet-ween the two cars is equal to the distance 13. What is the speed of stream ?
covered by the fly until it spots the cars. How many (i) The boat covers 20 km in 5 hours moving up
metres will the fly travel before the cars collide ?

ww
(A) 128 m
(C) 122 m
(B) 126 m
(D) 134 m
stream.
(ii) The boat covers 20 km in 4 hours moving down
stream.

w
(E) 114 m

.Ea
9. Ramesh goes from X to Y via U, V and W. The
distance between in the ratio of 2 : 3 : 2 : 3. The time
taken by the man to cover these distances in the ratio
(iii) The ratio between the speed of boat and stream
is 2 : 1 respectively.
(A) Any two of the three

syE
of 5 : 3 : 2 : 5. If the man takes 5 hours to go from X
to Y, then what is the differences between the time
taken by the man to go from U to V and that from W
(B) (i) and (ii) only
(C) (ii) and (iii) only
(D) (i) and (iii) only
to Y ?
(A) 30 minutes (B) 40 minutes ngi (E) All (i), (ii) and (iii)

(C) 60 minutes
(E) None of these
(D) 80 minutes
nee
14. A tank is fitted with two inlet pipes A and B both the
pipes are kept open for 10 minutes. So, that the tank
is two-thirds full and then pipe A is closed. How
Question based on data sufficient (Q. 10 to 16)—
10. A train running at a certain speed crosses another
the tank ?
rin
much time will B take to fill the remaining part of

train running in the opposite direction in 5 seconds.


To find out the speed of the first train, which of the
g.n
(i) Pipe A is thrice as fast as pipe B.
(ii) Pipe B alone can fill the tank in 60 minutes.
following information X and Y is sufficient ?
X = the length of the first train
Y = the length of the second train
(A) only X is sufficient
(A) Only (i)
(B) Only (ii)
(C) Any one is sufficient
e t
(D) All (i) and (ii)
(B) only Y is sufficient
(E) None (i) & (ii)
(C) Either X or Y is sufficient
(D) Both X and Y are sufficient 15. A tank is fitted with two taps A and B. In how much
time will the tank be full if both the taps are opened
(E) Both X and Y are not sufficient.
together ?
11. What is the speed of the train ? (i) A is 50% more efficient than B
(i) The train crosses a tree in 13 seconds.
(ii) A alone takes 16 hours to fill the tank
(ii) The train crosses a platform of length 250 metres
(iii) B alone takes 24 hours to fill the tank.
in 27 seconds.
(iii) The train crosses another train running in the (A) (ii) and (iii) only
same direction in 32 seconds. (B) All (i), (ii) and (iii)
(A) (i) and (ii) only (B) (ii) and (iii) only (C) (i) and (ii) only
(C) (i) and (iii) only (D) Any two of the three (D) (i) and (iii) only
(E) None of these (E) Any two of the three

CAT Complete Course | 129

Visit : www.EasyEngineering.net
Visit : www.EasyEngineering.net

16. A train running at a certain speed crosses a stationary Now, subtract equation (4) – equation (3), we get
engine in 20 seconds. To find out the speed of the 75
train, which of the following information is neces- 2.V =
4
sary ? 75
V = km/hr.
(A) Only the length of the train 8
(B) Only the length of engine 3. (C) Now, Let the speed of the boat in still water = W
(C) Either the length of the train or the length of the km/hr
engine Now, according to question,
(D) Both the length of the train and the length of the Since, Distance is fixed / constant.
engine Distance in down-stream motion
(E) None of these = 2 (4 + W)
Answers with Hints Distance in up-stream motion
Exercise A 5
= (W – 4)
2
1. (C) Let speed of boat in still water = V km/hr

ww
Now, down-stream speed = V + 2
Up-stream speed = V – 2
Now,

Or,
5
2 (4 + W) = (W – 4)
2
16 + 4W = 5W – 20

w
According to question,

.Ea
Total time taken =
6
+
6
=
V + 2 V – 2 60
33
Or, W = 36 km/hr.
4. (C) Let length of the platform and train are l1 and l2
respectively.

syE
Or, 11.V2 – 240.V – 44 = 0 5
Now, 54 km/hr = 54 × = 15 m/sec
Or, (V – 22)(11.V + 2) = 0 18
l1 + l2
V = 22, Or, = 15
V ≠ –
2
11 ngi 15
l1 + l2 = 225 m …(1)

∴ Speed of boat in still water = 22 km/hr.


2. (B) Let the man can row in still water in V km/hr and nee In second case man and train both move in the same
direction. So,
5

rin
Relative Speed = (54 – 6) ×
speed of stream U km/hr.
18
According to question,
48 × 5 8 × 5 40
20
+
V–U V+U
25
= 8 …(1)

Now,
l2
=

g.n
= 12
18
=
3
=
3
m/sec

e
30 45
+ = 10 …(2) 40
V–U V+U
Now, multiplying equation (1) with 3 and equation
(2) with 2, then subtracting, we get
75 90
3
l2 = 160 m
Putting the value of l3 in equation (1), we get
t
…(2)

∴ – = 24 – 20 l1 = 225 – 160 = 65 m
V+U V+U
∴ Length of the train = 160 m
15
– = 4 ∴ Length of the platform = 65 m
V+U
15 Hence, required ration = 160 : 65 = 32:13
V+U = – …(3)
4 5. (B) Since, the distance between Patna and Banaras is
Now, putting the value of (V + U) equation (1), we constant which is X m.
get X
Now, Speed of train A = m/min.
20 25 60
+ = 8 X
V – U 15 Speed of train B = m/min.
4 90
20 20 4 Let they meet at time Y min.
= 8– =
V–U 3 3 Now, Total Distance = Distance Covered by
V – U = 15 …(4) the both the trains

130 | CAT Complete Course

Visit : www.EasyEngineering.net
Visit : www.EasyEngineering.net

X X 5
Or, X = ·Y+ ·Y New time taken of the usual time.
60 90 4
Y Y (3 + 2)Y According to question,
Or, 1 = + =
60 90 180 5
of the usual time – usual time = 10
∴ 5.Y = 180 4
180 1
Or, Y = = 36 minutes Or, of the usual time = 10
5 4
Clearly, they cross each other at 6:36 pm. Usual time = 10 × 4 = 40 minutes.
6. (A) According to the formula, 11. (D) Since, policeman and thief move in the same
direction.
The ratio of their speed =√T2 : √T1
So, Relative speed = 16 – 12 = 4 km/hr
= √64 : √16 Since, Speed of the policeman is greater than the
= 8:4=2:1 thief.
7. (A) Now, Let they meet after t hours of 6 am. So, Time taken by the policeman to cover 1200 m
Now, Total distance remains fix. 1200 metre 1200

ww

Or,
120 = 45.t + 25 (t – 1)
120 = 70.t – 25
(t > 1 hour)
=
4 km/hr
=

5
18

Or,

Or,
w t =

t =
145
70

70.Ea
145
hour

× 60
= 300 ×
18
5
= 60 × 18 = 1080 seconds
1080

syE
= 124.3 minutes = 2hrs 4 minutes
Hence, they will meet at 8:04 am.
=
60
minutes = 18 minutes

In 18 minutes the distance covered by the thief

ngi
5 5
8. (D) Speed of the first train = 54 × = 15m/sec = 12 × × 18 = 60 metres
18 18
5 The total distance covered by the thief before he is
Speed of the second train = 36 × = 10m/sec

If the distance between A and B = X m


18
nee catched = 60 metres.
12. (C) According to question,
Now, when they meet then time is fix.

So,
X
=
X + 150
= 300 m.
Distance is fixed.
rin
So, Average speed is given by
10
9. (C) Let speed of Z = a m/min
15
=
g.n
2V1V2
V1 + V 2
=2×
18 × 36
18 + 36
Now, Speed of Y = 3.a m/min
Speed of X = 6.a m/min
Ratio of speeds X : Y : Z = 6a : 3a : a
= 6:3:1
=
e
2 × 18 × 36
18 + 36
= 24 km/hr.

13. (A) Let the speed of the train in the onward = V


km/hr.
t
Since, Distance remains constant. Now, the speed of the train in return
1 1 150 3
So, Ratio of time = : : 1 = · V = · V km/hr
6 3 100 2
= 1:2:6 According to question,
Clearly, when Z takes 6 min. then X takes 1 min. if Z Total time taken = 16 – 1 = 15 hours
takes 60 min.
Total Distance
1 Average speed =
X takes = × 60 = 10 minutes. Total Time
6
2V1V2
10. (E) Since, Distance is constant. = (As distance equal)
V1 + V 2
1
∴ Speed ∝ 3
2×V× V
Time 2 1000
⇒ =
4 3 15
New Speed = of the usual speed V+ V
5 2

CAT Complete Course | 131

Visit : www.EasyEngineering.net
Visit : www.EasyEngineering.net

6V 1000 a
⇒ = Its Speed = km/hr
5 15 4
1000 × 2·5 But it has traveled (300 – a) km to reach the meeting
⇒ V = = 55·55 km/hr.
18 point.
1020 ∴ Time taken to reach the meeting points is
14. (B) Each train has to cover a distance of km =
2
300 – a  300 – a 
510 km = =4× 
a  a 
Let the speed of Mumbai–Chennai Express = x 4
km/hr
9a  300 – a 
So, time taken by it to reach Wadi-junction is =
510 ∴ = 4 × 
x 300 – a  a 
hr Or, 9a2 = 4 × (300 – a)2
Now, speed of Chennai–Mumbai Express = (x + 10) = {2 × (300 – a)}2
km/hr
∴ 3a = 2 × (300 – a)
510
Now, Time taken to reach Wadi-junction = Or, 5a = 2 × 300

ww
x + 10
∴ a = 120 km.
According to question ,
a 120
510 510 Speed of train moving towards Patna is = = 30

w
+2 = 4 4
x + 10 x
km/hr and Speed of train moving towards Delhi is
Or, 510 ×

Or,
10
x (x + 10)
.Ea
= 2

510 × 5 = x 2 + 10x
300 – 120
9
= 20 km/hr.

Or, x 2 + 10x – 2550 = 0


syE
– 10 ± √

102 + 4 × 2550
16. (C) Since, the trains are 100 km apart.
Since, trains are moving in opposite direction.
So, Relative speed = 55 km/hr + 45 km/hr

ngi
Or, x =
2 = 100 km/hr
– 10 ± 10√
1 + 102 So, time taken to collide the train
=

Or, x = – 5 ± 5√

103
2

nee =
Distance
Relative speed

rin
100
Or, x = – 5 + 5
√
103 km/hr =
100
= 1 hour
x ≅ 45 km/hr

g.n
So, the distance covered by bird in 1 hour
Now, Both the trains meet after = 80 × 1 = 80 km.
510 102 34 17. (B) Since, the trains are running in the same direction.
=
45
1
=
9
=
3

= 11 = 11 hr 20 min.
3
e
So, Relative speed = 72 – 54 = 18 km/hr
5
= 18 × = 5 m/sec
18 t
Hence, meeting time 6 pm + 11 hr 20 min = 5·20 am Time taken by the trains to cover each other
next day.
= Time taken to cover
15. (A) Suppose both the trains meet at a point which is
(100 + 120) m at 5 m/sec
a km from Patna.
220
The train traveling towards Delhi travels for (300 – = = 44 sec.
5
a) km after the meeting point in 9 hours.
18. (B) Let the length of the train 4321 is a m. and length
300 – a of the platform = b m.
Its speed = km/hr
9
Since, the train and the man is in the same direction.
But it has travelled a km from Patna to reach the
meeting point. ∴ Relative speed = 81 – 9 = 72 km/hr
9a 5
∴ Time required to travel a km = hr = 72 × = 20 m/sec
300 – a 18

The train traveling towards Patna travels a km in 4 Now, the train crosses the man in 75 seconds.
hours. So, Length of the train = 20 × 75 = 1500 m

132 | CAT Complete Course

Visit : www.EasyEngineering.net
Visit : www.EasyEngineering.net

Now, speed of the train = 81 km/hr 1


Now, (d d ′ – d1′ d2 ) = T1 d2 ′ – T 2 d2
5 45 V 1 2
= 81 × = m/sec
18 2 d d ′ – d1′d2
V = 1 2 km/hr
a+b T1 d2 ′ – T 2 d2
Now, = 20
45 d2 T d ′ – T 2 d2
2 = T1 – d1 × 1 2
U d1 d2 ′ – d1′d2
a + b = 450 m T1 d1 ′d2 – d1 T1 d2 ′ – d1T1 d2 ′ + d1 T2 d2
=
Since, a = 200 m d1 d2 ′ – d1′d2
b = 450 – 200 = 250 m. d2 (d1T2 – T1 d1 ′)
=
Hence, required ratio is 200 : 250 : 4 : 5. d1 d2 ′ – d1′d2
19. (B) Let the speed of first train = x m/sec d1 d2 ′ – d1′d2
∴ U = km/hr.
and the speed of the second train = y m/sec. d1 T2 – T1 d1 ′
So, length of the first train = 30x 22. (D) The pedestrian and the cyclist both are moving in
opposite direction.
Length of the second train = 20y

ww
Let the speed of pedestrian = a km/hr
∴ They cross each other in 25 seconds
The speed of cyclist = b km/hr
30x + 20y
So, = 25 Now, relative speed = (a + b) km/hr
x+y


w 5y = 5x
x
y
.Ea
=
1
1
Now,
Or,
Distance = Speed × Time
80 = 2 (a + b)
a + b = 40

syE
The ratio of speeds are 1 : 1.
20 (D) Let the length of first road be a km and so the
length of the second road will be (a – 20) km.

4
80 80
b
+

4
a
= 20

ngi
Or, – = 1
Speed of car I = x km/hr a 40 – a
Speed of car II = (x + 40) km/hr 4(40 – 2a)
Or, = 1

nee
(40 – a)a
According to question ,
Or, 160 – 8a = 40a – a2
a 7
= …(1) Or, a2 – 48a + 160 = 0

rin
x 2

and
a – 20
=
5
…(2) 48 ± √

(48) 2 – 4 × 160 × 1
Or, a =

g.n
x + 40 2 2×1
Now, putting the value of a in equation (2), we get 48 ± √

1664
Or, a =

e
7 2
x – 20

Or,
2
x + 40
7x – 40
2 (x + 40)
=

=
5
2
5
2
Or,

Or,
a =
48 ± 40

a = 44; 4
2

Q a + b = 40 and b cannot be in negative. So, a = 44


t
Or, 7x – 40 = 5x + 200
not possible and by a = 4 ; b = 36.
Or, 2x = 240 So, the required ratio is 4: 36 = 1 : 9
Or, x = 120 km/hr 23. (B) Let Speed of Soni = V km/hr
∴ x + 40 = 120 + 40 = 160 km/hr Let Speed of Sita = (V – 4 ) km/hr
Hence, the speed of fastest car is 160 km/hr. Now, Time taken by Sita and Soni as equal.
21. (B) Let the up-stream = V km/hr 60 – 12 48
So, Time taken by Sita = =
Let the down-stream = U km/hr V–4 V–4
According to question, 60 + 12 72
Time taken by Soni = =
d1 d2 V+4 V+4
+ = T1 …(1) 48 72
V U Now, =
d1 ′ d2 ′ V–4 V+4
+ = T2 …(2) Or, 4 (V + 4) = 6 (V – 4)
V U

CAT Complete Course | 133

Visit : www.EasyEngineering.net
Visit : www.EasyEngineering.net

2 (V + 4) = 3 (V – 4) Now, the distance between owner and thief is 30 km.


V = 8 + 12 = 20 Since, the owner and the thief move in the same
V = 20 km/hr direction.
Soni’s speed = 20 km/hr So, relative speed = 75 – 60 = 15 km/hr
Sita’s speed = 16 km/hr. From Time taken by the owner in overtaking the thief
30
Exercise B =
15
= 2 hr
1. (A) (i) Let the Sita’s original speed = V At 1.00 pm he will catch the thief.
3 5. (E) Let the speed of train be V km/hr
Now, after increasing speed the final speed = V
2
and the speed of car = U km/hr
2
Now, she will take T time to travel the same In Ist Case
3
distance. 400 200 1 13
+ = 6 = …(1)
V U 2 2
∴ According to question,
2 In 2nd Case

ww 3
T = 20

T = 30 sec.
200 400
V
+
U
= 7

Now, equation (1) – equation (2), we get


…(2)

w
Now, her original time to reach her friend = 30 sec.

(ii) Original speed =

.Ea
=
Distance
Original Time
240
= 8 m/sec.
800 200
V

V
600
= 13 – 7

= 6

syE
V
30
V = 100 km/hr
2. (C) If the initial speed = V
3 From putting the value of V in equation (2), we get
New speed = V
4
4
New time = T (Where T = original time) ngi 200 400
+
100 U
= 7

3
Now, According to question,
4
nee Or,
400
U
= 7–2=5

U = 80 km/hr.

Hence,
3
T – T = 50 – 30 = 20

T = 60 minutes rin
6. (A) Let the speed of the slowest cart = x m/min

Mean if he will covers his journey in 30 minutes


(Half of initial time) he will be on time. g.n
The speed of the fastest cart = 2x m/min
Thus, when they meet for the first time, the fastest
cart takes one around more than the slowest cart.
So, Desired Speed = double of initial speed (as we
know that Distance is constant).
3. (C) Let time taken = T hr
e
speed of (2x – x) m/min in 10 minutes.
3000
2x – x
= 10
t
That is the fastest cart move 3000 more with relative

Now, the distance traveled in 30% of time of


20 km/hr = 0·3 T × 20 = 6 T km x = 300 m/min
The Distance traveled in 40% of time of So, time taken by the fastest cart to complete the race
9000
30 km/hr = 0·4 T × 30 = 12 T km is = = 30 minutes
300
The distance traveled in 30% of time in 40 km/hr
7. (A) The time internal between the train = t
= 0·3 T × 40 = 12T
The distance between any two consecutive trains
Total Distance
Now, Average Speed = coming in the same direction as Sunita at
Total Time
6T + 12T + 12T where a = speed of train
=
T b = speed of Sunita
= 30 km/hr. at
Now, = 8
4. (B) In an half hour the distance traveled by the thief a+b
1 at
= 60 × = 30 km and = 24 min
2 a–b

134 | CAT Complete Course

Visit : www.EasyEngineering.net
Visit : www.EasyEngineering.net

a+b a–b 1 1 4 So, length of trains are not sufficient.


+ = + =
at at 8 24 24 Correct answer is (E).
2a 1
= = 11. (A) Let the speed of the train be a metres/sec.
at 6
Length of the train
t = 12 minutes. Time taken to cross a tree =
Speed of the train
8. (E) In the Ist case, and time taken to cross a platform
When the fly is moving alongside the train. Length of train + Length of platform
=
Relative speed of the trains Speed of the train
l
= 60 + 90 = 150 m/sec So, First gives 13 =
a
Time taken for the two trains to collide
l = 13 a
150 m
= = 1 second l + 250
150 Second gives 27 =
a
The speed of the fly = 5 m/sec 125
a = m/sec.
In the IInd case, 7

ww
Now, Distance covered by the fly = 5 m
After this it covers a distance of 10 + 25 + 55 = 90 m
Thus, First and Second give the speed of the train.
12. (B) Let a be the speed of the train and l be length of
the train.

w
Total distance covered by the fly = 90 + 5 = 95 m
Now, From (i)
Now, this is the distance between two cars.

.Ea
Now, Relative speed of the cars
= 20 m/sec + 30 m/sec
20 =
l + 300

2l
a
…(1)

syE
= 50 m/sec
Time taken by the cars to collide
From (ii)

From (iii)
15 =

10 =
a
l
a
…(2)

…(3)
=
95 19
=
50 10
sec

Now, speed of the fly = 2 × 5 = 10 m/sec ngi Clearly, (i) and (ii) gives us answer
and (i) and (iii) also provides answer.
Distance traveled by the fly
19
= 10 × = 19 m
nee
13. (A) From (i)
Speed of up-stream =
20
= 4 km/hr

rin
5
10
From (ii)
Total Travel by the fly = 95 + 19 = 114 m.

g.n
20
9. (B) Time taken by the man is in the ratio of 5 : 3 : 2 : 5 Down-stream speed = = 5 km/hr
4
from X to U, U to V, V to W and W to Y respec- From (iii)
tively.
Let this time will be 5a, 3a, 2a, 5a.
So, total time taken = 5a + 3a + 2a + 5a = 5 hours
1
e
Speed of boat = 2a km/hr
Speed of stream = a km/hr
Now, From (i) and (ii) t
Or, a = hour 5–4 1
3 Speed of stream = = km/hr
2 2
Time From U to V = 3a = 1 hour = 60 minutes From (ii) and (iii), we get
1 a = 5 km/hr
Time From W to Y = 5a = 5 × = 100 minutes
3
So, speed of boat = 10 km/hr
Now, Required difference
Clearly, any two of the three will give the answer.
= 100 – 60 = 40 minutes.
14. (C) In First Case,
10. (E) Let length of the trains are a metre and b metre
1
respectively and V and U are their speed respectively. In one minute let the pipe B can fill part of the tank
x
When they move in opposite direction. 3
∴ In one minute pipe A can fill part of the tank
Time taken to cross each other x
a+b In one minute (A + B) can fill
= =5
V+U 1 3 4
= + = part of the tank
To find V we need a, b and U. x x x

CAT Complete Course | 135

Visit : www.EasyEngineering.net
Visit : www.EasyEngineering.net

40 So, B can fill the tank in 24 hours.


∴In 10 minutes can fill = tank
x 1 1 
According to question, In one hour (A + B) can fill  +  part of the
 16 24 
40 2
= tank.
x 3
5
⇒ x = 60 In one hour (A + B) can fill = part of the tank
48
So, B can fill the tank in 60 minutes.
Thus, (1) and (2) give the answer.
A can fill the tank in 20 minutes.
From (ii)
In Second Case,
1
B can fill the tank in 60 minutes. In one hour B can fill part of the tank
24
So A can fill the tank in 20 minutes.
Now, From (ii) and (iii), we get the same answer
15. (E) From (ii)
1 and From (i) and (iii), we also get the same answer.
In one hour A can fill part of the tank.
16 16. (D) We know that the time taken by the train to cross
a stationary engine

ww
1
Let us suppose in one hour B can fill part of the
x Length of train + Length of engine
= = 20
tank. Speed of the train

w
Now, From (i)
1
16

.Ea
=

x = 24
150 1 3
× =
100 x 2x
Clearly, to find the speed of the train, the length of
the train and the length of the engine both must be
known in the above statements.
●●●

syE
ngi
nee
rin
g.n
e t

136 | CAT Complete Course

Visit : www.EasyEngineering.net
Visit : www.EasyEngineering.net

10 Time and Work


Generally, in our daily life we do everything’s are 1
(iv) In how many days he can dig th of the trench ?
known as work. For doing these work we require time. 20
So, time and work are related to each other. (v) In how many days he can dig 5 such trench?
Since, a person performs / complete work and it Solution :
takes time to do any work. (i) A farmer in 100 days can dig a trench.
So, number of person also affects the work and the 1
∴ A farmer in one day can dig th trench
time. 100

ww
Here, we have some important relevant relationship
between person, work and time.
(ii) A farmer in one day can dig
1
100
th trench

w
4
1. More work requires more persons. ∴ A farmer in 4 days can dig th trench
100

.Ea
If time remains constant it means for doing any work
time is fixed. Then, Person is directly proportional to the
work.
=
1
25
trench

So, syE
If we have to do a lot of work it need a lot of person.
Person ∝ Work …(1)
(iii) Since, the farmer can dig a trench in 100 days
3
∴ The farmer can dig th trench in × 100
4
3
4
2. If the work to be done kept constant, more persons
will take less time to complete it and vice-versa.
ngi 1
= 75 days

(iv) The farmer can dig th trench in


100
days
Clearly,

Person ∝
1
Time nee 20
= 5 days
20

rin
(v) Since, the farmer can dig one trench in 100 days
1
Time ∝ …(2) ∴ The farmer can dig 5 trench in 500 days.
Person
3. If to do any work, the number of person is fixed
then, time depends on work and vice-versa.
Illustration 2.

g.n
7 persons can do a certain piece of work in 21 days.
Clearly, more work requires more time.
Time ∝ Work
Work ∝ Time …(3)
work in 14 days ?
e
(i) How many persons required to do the same

(ii) In how many days can 14 persons complete the


same work ?
t
Now, combining (1), (2) and (3) Solution :
We have elegant equation, Since, in 21 days, 7 persons can do a certain work.
Person × Time
= Constant So, in 1 day 7 × 21 persons can do a certain work.
Work
7 × 21
P 1 × T1 P × T2 in 14 days persons can do a certain work
Or = 2 14
W1 W2
21
Illustration 1. = persons
2
A farmer can dig a trench in 100 days (ii) Since, 7 persons can do a certain work in 21 days
(i) How much he can dig in 1 day? ∴ 1 persons can do a certain work in 21 × 7 days
(ii) How much he can dig in 4 days? 21 × 7
∴ 14 persons can do a certain work in days
3 14
(iii) How much days he will take to dig th of the
4 21
trench ? = days.
2

CAT Complete Course | 137

Visit : www.EasyEngineering.net
Visit : www.EasyEngineering.net

Illustration 3. Now, when C and D work together they can do in


80 persons can do a certain job in 5 days. How many X + Y 
one day   work
persons are required to do the same job in 20 days ?  XY 
Solution : X + Y 
Here, we have W1 = W2 =  XY 
 
P 1 × T1 P × T2
Given = 2 X + Y 
W1 W2 Since,   work is done by A and B together in
P 1 × T1 = P 2 × T2  XY 
one day
T × P1
Or, P2 = 1 ∴ 1 work is done by A and B together in
T2
Here, P1 = 80, T1 = 5 days, T 2 = 20 days 1 XY
=
X+Y X+Y
80 × 5 = P 2 × 20
XY
P 2 = 20 persons.
Clearly, C and D work together the same work in
Illustration 4. XY

ww
10 persons can make 40 toys in 5 hours. How many
toys can 8 persons make in 10 hours ?
X+Y
days.

Illustration 7.

w
Solution :
Given that P 1 = 10, P 2 = 8, T1 = 5 hours, T2 = 10 hours,
W1 = 40 days, W2 = ?
.Ea
P 1 × T1
= 2
P × T2
C can do a work in 5 days and D in 25 days. How
many Hours will they take to do the same work, if both
work together and in a day their work hour is 6 ?
Solution :

Or,
W1
10 × 5
40
syE
=
W2
8 × 10
W2
C and D work together the same work in

25 × 5 125 25
XY
X+Y
days

W2 =
40 × 8
5 ngi ∴ = =
5 + 25 30
25
=
6

Illustration 5.
W2 = 64 toys.
nee
Illustration 8.
=
6
× 6 hours = 25 hours.

25 men can cut 10 trees in 5 hours. In how many


hours 20 men can cut 15 trees ?
rin
A and B can together does a piece of work in 20
days, B alone can do it in 5 days. In how many days can
Solution :
Given that P1 = 25, P 2 = 20, W1 = 10 trees, W2 = 15
trees, T1 = 5 hours, T2 = ?
A alone do it ?
Solution :
g.n
Or,
P 1 × T1
W1
25 × 5
=
P × T2
= 2
W2
20 × T2
days.
According to question,
e
A and B can together do a piece of work in
XY

t
X+Y

10 15 5X
20 =
25 × 75 75 X+5
T2 = = hours.
200 8 Or, 20X + 100 = 5X
Illustration 6. 100
X = days.
A man C can do a work in X days and another man 3
D can do the same work in Y days, then show that C and Illustration 9.
XY
D together can do the some work in days ? Ram, Shyam, Rahim and Abdul can do a piece of
X+Y
work in X, Y, Z and W days respectively. In how many
Solution : days they can do this work if they work together ?
Man C can do a work in X days Solution :
1 1
∴ man C can do in one day work In one day Ram can do work
X X
1 1
Similarly, in one day man D can do work In one day Shyam can do work
Y Y

138 | CAT Complete Course

Visit : www.EasyEngineering.net
Visit : www.EasyEngineering.net

1 Similarly,
In one day Rahim can do work
Z 2abc
X alone can do the work =
1 a.b + b.c – c.a
In one day Abdul can do work
W 2abc
Y alone can do the work = ·
Now, in one day all can do together can do a.c + c.b – b.a
1 1 1 1 Illustration 12.
+ + + work
X Y Z W X and Y can do work in 10 days, Y and Z in 20 days,
Then, they can together do the same work in Z and X in 30 days. In how many days can they complete
1 it, if they work together ?
= days
1 1 1 1 Solution :
+ + +
X Y Z W 1
X.Y.Z.W (X + Y)’s one day work =
= days 10
Y.Z.W + X.Z.W + X.Y.W + X.Y.Z 1
(Y + Z)’s one day work =
Illustration 10. 20
A, B, C and D can do a piece of work in 3, 6, 9 and 1

ww
(X + Z)’s one day work =
12 days respectively. In how many days they can do this 30
work if they work together ? 2 (X + Y + Z)’s one day work
Solution :

w
1 1 1
= + +
According to formula, 10 20 30

= .Ea
A, B, C and D can do a piece of work in
3 × 6 × 9 × 12
=
1 

10 
1 1
1+ + 
2 3

=
36
25
days. syE
6 × 9 × 12 + 3 × 9 × 12 + 3 × 6 × 9 + 6 × 9 × 12
=
1 6 + 3 + 2 
×
10  6


Illustration 11.
X and Y can do a piece of work in a days Y and Z in ngi =
1 11
×
10 6

nee
b days, Z and X in c days. How long would each take to 11
So (X + Y + Z)’s one day work =
do the same work separately ? 120
120
Solution : So (X + Y + Z)’s can work = days
1
In one day X and Y can do together work
a
…(1)
rin 11
≅ 11 days.
1
In one day Y and Z can do together work
b
…(2)
Illustration 13.
g.n
X and Y together can do a piece of work in 24 days,
1
In one day Z and X can do together work
c
∴ In one day X, Y and Z can do together
…(3)
1
2
e
Y and Z together can do it in 12 days. X starts the work

t
and works on it for 8 days, then Y takes it up and works
for 10 days. Finally Z finishes the work in 16 days. In
how many days can each do the work when doing it
1 1 1  separately ?
 a + b + c  work
  Solution :
1 1 1 1  1 1
Now, Z’s one day work =  + + – (X + Y)’s one day work = …(1)
2 a b c  a 24
1
1 1 1 1  (Y + Z)’s one day work = …(2)
=  + –  12
2 b c a 
Now, according to question,
1 1 1 1 
Z’s one day work is  + –  8 2 14
2 b c a  1 = +
24 12 d
+
1 8 + 4 14
Clearly, Z alone can do the work in 1 = +
1 1 1 1  24 d
 + – 
2 b c a  1 14
1 = +
2abc 2 d
= days
c.a + a.b – b.c d = 28

CAT Complete Course | 139

Visit : www.EasyEngineering.net
Visit : www.EasyEngineering.net

Hence, Z can do it in 28 days individually . Illustration 15.


1 1 28 – 12 A tank can be filled in 40 minutes but there is a
Y individual = – =
12 28 28 × 12 leakage in it which can empty the full tank in 80 minutes.
16 1 In how many minutes it can be filled ?
= =
28 × 12 21 Solution :
Hence, Y can do it in 21 days individually . In 40 minutes one tank is filled.
1 1 1
X individual = – In one minute part of tank is filled.
21 24 40
8–7 1 In 80 minutes one tank is made empty.
= =
3 × 7 × 8 168 1
Hence, X can do it in 168 days individually. In one minute part of tank is made empty
80
Illustration 14. 1 1 
X and Y can do a work in 12 days while Y and Z can Now, In one minute  –  part of tank is filled
 40 80 
2
do it in 6 days. After X had worked on it for 3 days and 2–1 1
3 ∴ In one minute = part of tank is filled

ww
Y for 4 days, Z finished the work in 7 days. In how many
days could each do the work separately ?
Solution :
80 80
∴ One tank is filled in 80 minutes.

w
Let the daily work of X, Y and Z be a, b, c respec-
tively, then we can write
.Ea
a+b =
1
Concept of Efficiency
A man has efficiency 50 per day it means he can do
1
2

syE
12 half work in a day.
1 Clearly, efficiency indicates or measure time in which
a = –b …(1)
12 the whole work is done.
b+c =
3
20
3 ngi Efficiency is related to time, days, year, month, hours,
seconds and minutes.
c =

3.a + 4.b + 7.c = 1


20
–b …(2)

…(3) nee Efficiency is inversely proportional to time

∈ ∝
1
t
Now, putting the value of a and b in equation (3)


1
3
 3 
– b  + 4.b + 7  – b  = 1
Illustration 16.
rin
Or,
 12 
– 6b + +
1 21
= 1
 20 
g.n
Ram can do a job in 9 days and Shyam can do the
same job in 6 days. Find efficiency and in how many
days working together they can complete the job ?

Or,
4 20
6.b =

=
5 + 21
20
26 – 20
–1
Solution :

Ram’s one day work =


e 1
9 t
20 1
Shyam’s one day work =
6 6
6.b =
20 (Ram + Shyam)’s one day work
1
b = 1 1 2+3
20 = + =
9 6 3×3×2
1 1 5–3
a = – = 5 5
12 20 4 × 3 × 5 = =
3 × 3 × 2 18
2 1
= = 1 1
4 × 3 × 5 30 Efficiency of Ram = × 100% = 11 %
9 9
3 1 1
c = – = 1 4
20 20 10 Efficiency of Shyam = × 100% = 16 %
1 1 1 6 6
∴ a, b and c are , , respectively.
30 20 10 (Ram + Shyam) can do work in
Therefore, a, b and c can do the work separately in 18 3
= days = 3 days
30, 20 and 10 days respectively. 5 5

140 | CAT Complete Course

Visit : www.EasyEngineering.net
Visit : www.EasyEngineering.net

Illustration 17. Or, 14.B = 4.G


A is twice as efficient as B and is therefore able to 7.B = 2.G …(3)
finish a piece of work in 40 days less than Q. Find the time Clearly, the work of 2 girls is equal to 7 boys.
in which A and B can complete the work individually ?
In Ist cast,
Solution : 2 12
Efficiency of A:B = 2:1 6 boys = 6 × girls = girls
7 7
A requires 2.X days and B requires X days.  12 
6 boys + 8 girls =  7 + 8  = 68 girls
According to question,   7
2.X – X = 40 68
Now, girls finish the a job in 6 days
Or, X = 40 7
∴ 2.X = 2 × 40 = 80 days 68
∴ 1 girl finish the a job in × 6 days
X = 40 days 7
Thus, A can finish the work in 80 days and B can 408
= days
finish the work in 40 days. 7

ww
Illustration 18.
If 25 persons can do a piece of work in 5 days then
1 girl finishes the a job =
408
7
7
days

Again, 8 girls + 6 boys = 8 × boys + 6 boys

w
calculate the number of persons required to complete the
2
work in 10 days ?
Solution :
We know that .Ea = 34 boys
∴34 boys finish the job in 6 days

syE
Number of persons / man / workers X days = work
We represent the person / man / worker = P
∴ 1 boys finish the job in = 34 × 6 days
1 boy finish the job in = 204 days
7

ngi
Day = D One girl’s one day job =
408
Work = W 1
One boy’s one day job =

nee
Now, For the same work 204
P 1 . D1 = W1 …(1) One boy and girl one day job
P 2 . D2 = W2 …(2) 7 1 9


P 1 . D1 = P 2 . D2
25 × 5 = P 2 × 10 rin = +
408 204 408
=

408
≅ 46

g.n
Hence, they will finish the work together in
9
25 × 5
Or, P2 = days.
10

Illustration 19.
P2 =
25
2
persons
Illustration 20.

e
Pipe X can fill a tank in 12 minutes and pipe Y can
t
fill it in 36 minutes. If both the pipes are opened to fill an
empty tank. In how many minutes will it be full ?
6 boys and 8 girls finish a job in 6 days and 16 boys
Solution :
and 10 girls finish the same job in 4 days. In how many
days working together 1 boy and 1 girl can finish the First method :
work ? In 12 minutes pipe X can fill one tank
Solution : 1
∴ In one minute pipe X can fill part of tank
We have 12
1
Job = work = Person × Day Similarly, in one minute pipe Y can fill part of
36
In Ist case, tank
6(6.B + 8.G) = Work …(1)
1 1 
4(16.B + 10.G) = Work …(2) Now, in one minute both pipes can fill  +  part
 12 36 
Now, from equation (1) and (2), we get
of tank
(– 64 + 36). B + (48 – 40).G = 0 3+1 1
– 28.B + 8.G = 0 = = part of tank
36 9
28.B = 8.G ∴ Both the pipes can fill one tank in 9 minutes.

CAT Complete Course | 141

Visit : www.EasyEngineering.net
Visit : www.EasyEngineering.net

Second Method : Clearly, Formula :


100 Original no. of boys
Efficiency of pipe X =
12 No. of additional boys × No. of days
25 taken by in 2nd case
= % =
3 No. of less days
100 25 Illustration 23.
Efficiency of pipe Y = = %
36 9 A certain number of men can complete a job in 30
Now, Efficiency of both the pipe to fill tank days. If there were 5 men more, it could be completed in
 25 25  10 days less. How many men were in the beginning ?
=  3 + 9  % = 100 %
  9 Solution :
Now, time required to fill the tank From formula :
100 Original no. of men
= = 9 minutes.
100 No. of additional men × No. of days
9 taken by in 2nd case
=
No. of less days

ww
Illustration 21.
X can do a piece of work in 12 days, Y can do it in
16 days and Z can do it in 20 days. In how many days
=
5 × 20
10

w
they can complete the work together ?
Solution :
.Ea
Efficiency of X =
100
%= %
25
∴ Original no. of men = 10.

Illustration 24.
Work and Wages

syE
Efficiency of Y =
12
100
16
%= %
3
25
4
Ram, Shyam and Rajesh can do a piece of work
in d 1, d 2 , d 3 days respectively. If they work together, in

Efficiency of Z =
100
20
% = 5%

Now, if X, Y and Z work together, then ngiwhat proportion should their earnings be divided amongst
them ?
Solution :

Efficiency = 5 +
25 25
4
+
3 nee Ram’s one day work =
1
d1
=
60 + 75 + 100 235
12
=
12
%
rin
Shyam’s one day work =
1
d2
Now, time taken to do work =
100
235
12
Rajesh’s one day work =
g.n 1
d3

Illustration 22.
=
100
235
× 12 =
240
47
days. ones day work.
e
Their earnings are also directly proportional to their

Now, Ram’s share : Shyam’s share : Rajesh’s share


1 1 1
t
A group of boys can do a certain piece of work in 25 = : :
days. If the group had 10 more boys, the work could be d1 d2 d3
done in 10 days less. How many boys are there in the = d2 × d3 : d1 × d3 : d2 × d3
group ?
The wages earned by a worker is directly propor-
Solution :
tional to the amount of work he does.
Let the number of boys = X
Wages are distributed in direct proportion to the
According to question,
amount of work done by individual workers.
X boys can do the work in 25 days
Total wages = One person’s one day work × Number
While (X + 10) can do the work in 15 days of person × Number of days
In both the cases total work done remains the same. Illustration 25.
X × 25 = (X + 10) × 15 Sita can do a work in 3 days while Gita can do it in 6
10 × X = 150 = 15 × 10 days. Both of them work together to do the work. If the
15 × 10 total amount paid for the work is Rs. 120. How much is
X = = 15 boys
10 Sita share in it ?

142 | CAT Complete Course

Visit : www.EasyEngineering.net
Visit : www.EasyEngineering.net

Solution : 5. Ram and Shyam can do a work in 16 days. If Ram


1 can do the same work in 21 days. In how many days
Sita’s one day work = Shyam alone can finish the work?
3
1 6. Sita, Gita, Radha, Shalani can do a certain work in 3,
Gita’s one day work =
6 6, 12, 9 days respectively. In how many days can
Now, shares ratio of Sita and Gita is given by they working together, finish the same job?
1 1 7. Ram and Rahim can do a work in 8 days. Rahim and
Sita’s share : Gita’s share = : = 2 : 1 Krishna in 12 days, Krishna and Ram in 16 days. In
3 6
how many days can they complete it, If they work
Since, total wage = Rs. 120
together?
2
∴ Sita’s share = × 120 = Rs. 80 8. Ram and Raju can do a work in 15 days. Raju and
3
Rakesh in 20 days. Rakesh and Ram in 12 days. In
1
∴ Gita’s share = × 120 = Rs. 40 how many days can they complete it. If (i) they work
3 together. (ii) They work separately.
Illustration 26. 9. 4 men or 5 women can do a certain piece of work in
Sanjay and Sheela contract to do a work together for 40 days. How many days will 8 men and 15 women

ww
Rs. 360. Sanjay can do it in 8 days and Sheela alone in 12
days. But with the help of Gita they finish it in 4 days.
How is the money to be divided among them ?
take to do the same work, when working together?
10. 16 men and 8 women can do a certain piece of work

w
Solution :

.Ea
Sanjay’s one day work =
1
8
in 6 days. 8 men and 6 women can do this work in 10
days. In how many days 32 men and 32 women can
finish the same work?
11. 5 men and 7 women can make 20 shirts in 6 days. 3

Similarly, syE
Sanjay’s 4 days work = =
4 1
8 2
men and 4 women can make 8 shirts in 2 days. How
many women work with 8 men. So that they can
make 40 shirts in 8 days?
Sheela’s 4 days work =
4 1
=
12 3
ngi 12. Ram, Shyam and Gita together earn Rs. 900 in 12
days. Ram and Shyam together earn Rs. 300 in 5

nee
Now, Sanjay, Sheela and Gita are working only 4 days. Shyam and Gita together earn Rs. 320 in 8
days. days. Find the earning of each?
1 1  1 13. 5 girls and 4 boys earn Rs. 660 in 3 days. 10 boys
Gita’s 4 days work = 1 –  +  =

rin
So,
2 3  6 and 20 grils earn Rs. 3500 in 5 days. In how many
days can 6 boys and 4 girls earn Rs. 1300?
1 1 1
∴ The ratio of their shares = : : =3:2:1
2 3 6
3
Sanjay’s share = × 360 = Rs. 180 g.n
14 8 taps are fitted to a water tank. Some of them are
water taps to fill the tank and the remaining are outlet
taps used to empty the tank. Each water tap can fill
6
2
Sheela’s share = × 360 = Rs. 120
6
1
Gita’s share = × 360 = Rs. 60
e
the tank in 36 hours and each outlet tap can empty it

t
in 48 hours. On opening all the taps, the tank is filled
in 6 hours. Find the number of inlet water taps?
15 The quantity of water flowing through a pipe is
6 proportional to square of its diameter. A tank has
Exercise A three inlets of diameters 2 cm., 4 cm. and 6 cm. If the
smallest inlet can fill the tank in 4 hours. In how
1. 10 men can make 20 shirts in 5 days working 5 much time can all the three inlets fill the tank?
hours/day. How many shirts can 20 men make in 10
16 Men, Women and children are employed in a factory,
days working 8 hours/day?
the amount of work done by a man, a woman and a
2. Ram can type 800 pages in 25 days working 5 hours/ child in a given being 3 : 2 : 1. They are paid wages
day. In how many days he can type 1000 pages according to the amount of work done by each. In the
working 10 hours per day? factory there are 20 men, 12 women and 6 children
3. 6 workers can make 12 toys in 5 days working 4 and their total wages amount to Rs. 270. How much
hours per day. How many toys can 12 workers make will the daily wages be if there are 30 men, 25
in 8 days working 10 hours per day? women and 20 children in all?
4. X can do a piece of work in 20 days while Y can do 17 An overhead cubical cistern with an edge equal to 2
it in 5 days. In how many days can X and Y working meter has water flowing into it from four pipes. Pipe
together it? A can pour in 80 litres per minute. While the second

CAT Complete Course | 143

Visit : www.EasyEngineering.net
Visit : www.EasyEngineering.net

pipe B can fill 30 litres per minute. There is another Rajesh started his work one second day and worked
pipe C that can fill 20 litres of water per minute and alternatively till the completion of the work. In how
the last pipe D can fill 10 litres of water per minute. many days the work will be finished ?
Of the given pipes B, C and D each of them shall act 5. A tank is connected with 10 pipes. Some of them are
as inlet pipes till the water level in the tank is below inlet pipes and other is working as outlet pipes. Each
their respective heights. The amount of water level of the inlet pipes can fill the tank in 12 hours, indi-
rises above the pipe, it starts acting as an outlet pipe vidually, which each of those that empty the tank can
with a drain capacity same as the pouring capacity empty it in 8 hours individually. If all the pipes are
assume that pipes to be of negligible diameter. kept open when the tank is full. It will take exactly 8
If pipes B, C, D are at heights of 0.28 m., 0.5 m., 0.7 hours for the tank to empty. How many of these are
m. respectively. Then find the time taken to fill the inlet pipes ?
cistern completely? 6. Two pipes X and Y can fill a tank in 25 hours and 20
18 A, B, C, D and E are five taps capacity of B is twice hours respectively. Hari opened the pipes X and Y to
that of A, capacity of C is 3 times that of A, fill an empty tank and some times later he closed the
capacities of D and E are 4 and 5 times that of A pipes X and Y when the tank was supposed to be
respectively. In First Case A, D and E act as input full. After that it was found that the tank was emptied

ww
pipes and B and C are out put pipes. In second Case
C, D and E act as input pipes and A and B act as out
put pipes. If A and D working together as input pipes
in 5 hours because an outlet pipe Z connected to the
tank was open from the beginning. If Hari closed the
pipe Z instead of closing pipes X and Y the remaining

w
can fill the tank in 4 hours, then what is the
difference in time required to fill the tank in the first

.Ea
and second cases stated above?
19 A task is assigned to a group of ‘n’ men not all of
7.
tank would have been filled in what time ?
1
Pipe P takes of the times required by pipe Q to fill
2
the empty tank individually. When an outlet pipe R is

syE
whom have the same capacity to work. Every day
exactly two men out of the group work on the tasks
with no same pair of men working together twice.
also opened simultaneously with pipe P and pipe Q.
3
It takes more time to fill the empty tank than it
4

ngi
Even after all the possible pairs have worked once, takes, when only pipe P and pipe Q are opened
all the men together had to work for exactly one day together. If it takes to fill 21 hours when all the
more to finish the task. Find the number of days that three pipes are opened simultaneously, then in what
will be required for all the men working together to
finish the work? nee
8.
time pipe R empty the full tank operating alone ?
Sona, Mona and Soni are three friends. Sona and
Mona are twins. Soni takes 2 days more than Sona to
Exercise B
1. Ram can do a piece of work in 8 days, Shyam can do
rin
complete the work. If Mona started a work and 3
days later Sona joins her, then the work gets com-
it in 10 days, with the help of Rahim, they finish it in
4 days. In how many days Rahim complete the whole
work alone ? g.n
pleted in 3 more days working together Sona, Mona
and Soni can complete thrice the original work in 6

e
days. In how many days Sona can complete twice the
2. Gita is twice as good a workman as Lila and therefore
Gita takes 5 days less than Lila to finish the work
individually. If Gita and Lila working together
complete the work in 5 days, then how many days
are required by Lila to complete the work alone ?
9.
original work with double the efficiency working
alone ?
Rohan can do a work in 15 days while Ramu can do
the same work in 10 days. They started work
t
together. After 5 days Rohan left the work and Ramu
3. Ganga, Jamuna and Janki can do a piece of work, completed it. For how many days Ramu worked
working together in one day. Ganga is five time more than the number of days required, when both
efficient as Janki and Jamuna takes the trice the worked together ?
number of days as Janki takes to do it alone. What is 10. The ratio of efficiency of A is to C is 3 : 2 the ratio
the difference between the number of days taken by of number of days taken by B & A is 2 : 1 A takes 12
Ganga and Jamuna ? days less than C when A and C complete is the work
4. Rajesh is renowned packager of fruits in Patna. He individually. B and C started the work and left after 2
packs 24 apples or 36 guavas everyday working 6 days. Find the number of days taken by A to finish
hours per day. His wife Reena also helps him. She the remaining work ?
packs 20 apples or 25 guavas working 5 hours per 11. The number of days required by X, Y and Z to work
day. Rajesh has to pack 4000 apples and 4400 guavas individually is 4, 6 and 8 respectively. They started a
with the help of his wife. They work alternatively, work doing it alternatively. If X has started then
each day 8 hours. His wife started packaging on the followed by Y and so on. How many days are needed
first day and works on every alternate days. Similarly to complete the whole work ?

144 | CAT Complete Course

Visit : www.EasyEngineering.net
Visit : www.EasyEngineering.net

12. Kamal, Krishna and Raju started a work together for completion of the work and Sheela left 4 days before
2 the completion of the work. In how many days was
Rs. 900. Kamal and Krishna did of the total work
5 the work completed ?
4
while. Krishna and Raju together did of the total 23. A piece of work can be completed by 10 boys and 6
5
girls in 18 days. Boys works 9 hours per day while
work. What is the amount of greatest efficient per-
Girls works 7.5 hours per day. Per hour efficiency of
son ?
2
13. If 2 men or 3 women or 4 boys can do a piece of a girl is rd of a boys’s efficiency. In how many
3
work in 52 days. Then the same of piece of work will more days the work will be completed by 5 boys and
be done by 1 man,1 women and 1 boy. What is the 9 girls ?
required number of days to complete the work if they 24. Pipes X and Y can fill a tank in 15 hours and 10
all work together ? hours respectively. Today morning as soon as the
14. If 2 men or 3 women or 4 boys can do a piece of pipes were opened, air bubbles were generated in
work in 104 days, then the same piece of work will both the pipes due to which pipe X could work with
be done by 1 man, 1 woman and 1 boy. What are the 1 1
only of its capacity and pipe B could work with
required days ? 2 3

ww
15. 3 children and 1 man complete a certain piece of
work in 6 days. Each child takes twice the time taken
by a man to finish the work. In how many days will 5
of its capacity. After some time, the bubbles in both
the pipes burst and then the tank could be filled in 4
hours. For how much time were the air bubbles

w
men finish the same work ?

.Ea
16. Six men and five women can do a piece of work in 4
days, while four men and two women can do the
25.
present in the pipes ?
Large, medium and small ships are used to bring
water 2 large ships carry as much water as 5 small
ships, 2 medium ship carry the same amount of

syE
same work in 7 days. If Rs. 48 is given to a women
for her contribution towards work, per day, then what
is the amount received by a man per day ?
water as one large ships and 2 small ships. 5 large, 8
1
medium and 12 small ships, each made 42 journeys
5
17. 4 boys and 6 girls can do a piece of work in 12 days.
7 boys and 8 girls can do the same work in 7 days. In ngi and brought a certain quantity of water. In how many
journey would 20 large, 14 medium and 24 small
ships, bring the same quantity of water ?
how many days 2 boys and 3 girls complete the same
work working together ?
nee
26. Two pipes X and Y can separately fill a cistern in 15
and 20 min. respectively and waste pipe C can carry

rin
18. (Y – 2) men do a piece of work in Y days and (Y + 7) off 10 litres per minutes. If all the pipes are opened
men can do 75% of the same work in (Y – 10) days. when the cistern is full, it is emptied in 2 hours. How
Then in how many days can (Y + 10) men finish the

g.n
many litres does the cistern hold?
work? 27. The tunnel-boring machines, working at the two ends
19. 33 girls can do a job in 30 days. If 44 girls started the of a tunnel, have to complete a work in 60 days. If
job together and after every day of the work, one girl
leaves. What is the minimum number of days required
to complete the whole work ?
2 e
the first machine does 30% of the work assigned to it
and the second 26 %, then both will drive 60 metres
3
2
t
20. Z is twice efficient as X, Y takes thrice as many days of tunnel. If the first machine had done of the work
3
as Z. X takes 8 days to finish the work alone. If they 3
assigned to the second one, and the second of the
work in pairs ( i.e. XY, YZ, ZX ) starting with XY 10
on the first day then YZ on the second day and ZX work assigned to the first one, then the first machine
on the third day and so on, then how many days are would have needed 6 days more than would have the
required to finish the work ? second, how many metres of the tunnel are driven by
each machine per day ?
21. Gita is twice efficient as Sita and Sita can do a piece
28. A tank is filled with three pipes with uniform flow.
of work in 15 days. Sita started the work and after a
The second pipe fills the tank 5 hours faster than the
few days Gita joined her. They completed the work
first pipe and 4 hours slower than the third pipe. First
in 11 days from the starting. For how many days they and Second Pipe open simultaneously will take as
worked together ? much time as third pipe can take individually. Find
22. Rani, Sheela and Soni can complete a piece of work the time required by the third pipe to fill the tank ?
in 9, 18, 24 days respectively. They started the work 29. Ram has the job of laying flower beds. He takes 2
together and Rani left after 2 days before the hours to lay a bed which is 12 feet × 10 feet. He asks

CAT Complete Course | 145

Visit : www.EasyEngineering.net
Visit : www.EasyEngineering.net

his friends to work with him sometimes. His friend Answers with Hints
takes 3 hours to do the same job. Ram has to pay his
Exercise A
friend on an hourly basis, so he prefers to work alone.
Ram has recently got a contract to lay 6 flower beds, 1. Given that
4 of which have dimensions 10 feet × 15 feet each P 1 = 10, P 2 = 20
and two 12 feet × 10 feet each. He has promised to W1 = 20 shrits, W2 = ?
do the job within 9 hours. For how many hours does T1 = 5 × 5 = 25 hours, T2 = 8 × 10
he need to employ his friend ?
= 80 hours
30. If a cistern generally takes 30 min. to fill by a pipe, From formula,
but due to a leak, it takes 20 extra min. to be filled,
W1
then find the time in which the lack can empty the W2 = P 2 × T2 ×
P 1 × T1
cistern fully ?
20
31. Men, Women and children are employed in a factory, W2 = 20 × 80 × = 128 shirts.
10 × 25
the amount of work done by a man a woman and
2. W1 = 800, W2 = 1000
a child in a given time being 2 : 5 : 7. They are paid
T1 = 25 × 5 = 125 hours, T2 = ?
wages according, to the amount of work done by

ww
each. In a factor there are 5 men 8 women and
10 children and their total daily wages amount to
Rs. 512. How much will the daily wages be if there
P 1 = 1,
P 2 × T2
W2
=
P 1 × T1
W1
P2 = 1

w
are 12 men, 15 women and 8 children in all ?

.Ea
32. Two pipes P and Q can fill a tank 12 hours and 18
hours respectively. If both pipe are open simulta-
neously, How much time they will take to fill the
Or,
T2
1000
=
25 × 5

5
800
T2 = × 5 × 25
4
tank ?
syE
33. Pipes A and B can fill a tank alone in 9 hours and 12
5
Now 10 × days = × 5 × 25
4
5 × 5 × 25 125

ngi
hours respectively. Pipe can empty the full tank in 15 Days = =
hours. If all the three pipe are open simultaneously, 4 × 10 8
how much time it will take them to fill the tank 5
Days = 15
completely ?
34. The Quantity of water flowing through a pipe is nee
3. Given
8

6 workers in 5 days working 4 hours per day can

rin
proportional to square of its diameter. A tank has
three inlets of diameters 1 cm., 2 cm. and 3 cm. If the make 12 toys
smallest inlet alone can fill the tank in 10 hours. In So, 1 worker in 5 days working 4 hours per day can
how much time can all the three inlets together fill
the tank ?
12
make = toys
6
g.n
35. Two pipes X and Y can fill a cistern in 14 hours and
16 hours respectively. The pipers are opened
simultaneously and it is found that due to leakage in
the bottom, it took 32 minutes more to fill the cistern.
e
Or, 12 workers in 5 days working 4 hours/day
=
12 × 12
6
toys
Or, 12 workers in one day working 4 hours/day t
When the cistern is full, in what time will the leak 12 × 12
= toys
empty it ? 6×5
Or, 12 workers in 8 days working 4 hours/day
36. Three pipes X, Y and Z can fill a tank from empty to
12 × 12
full in 30 minutes, 20 minutes and 10 minutes respec- = ×8
6×5
tively. When the tank is empty all the three pipes are
opened. X, Y and Z discharge chemical solution A, Or, 12 workers in 8 days working 10 hours/day
B and C respectively. What is the proportion of 12 × 12 × 8
= × 10 toys
solution C in the liquid in the tank after 3 minutes ? 6×5×4
37. In a group of four boys, the second boy is twice as = 96 toys
efficient as the first one. Third one is twice as effi- 4. In 20 days X can do a piece of work.
cient as the second one, and so on. All of them 1
∴ 1 day X can do a piece of th work.
working together will take 5 days to complete a job. 20
How much extra time will the second and third boy Similarly,
take, working together as compared to the fourth 1
boy, working alone to complete the same job ? In 1 day Y can do = th work
5

146 | CAT Complete Course

Visit : www.EasyEngineering.net
Visit : www.EasyEngineering.net

Now, In 1 day ( X + Y ) can do together ∴ (Ram + Rahim + Krishna)’s one day work
1 1 13
=  20 + 5  th work =
96
th work …(4)
 
4+1 5 Subtracting equation (1) from equation (4) , we get
= = 13 1 13 – 12
20 20 Krishna’s one day work = – =
1 96 8 96
= th work
4 1
= th work
1 96
Q X and Y together can do th work in one day
4 13 1 13 – 8
Ram’s one day work = – =
∴ X and Y together can do 1 work in 4 days. 96 12 96
1 5
5. Ram’s one day work = th work = th work
24 96
1 13 1 13 – 6
(Ram + Shyam)’s one day work = th work Rahim’s one day work = – =
16 96 16 96
Now, Shyam’s one day work 7
= th work

ww =
1 1 
 16 – 24  th work
  ∴ Ram can do the work in days
96
96
5

6.
w .Ea
=
3–2 1
= th work
8 × 3 × 2 48
Clearly, Shyam can do the work in 48 days.
1
Sita’s one day work = th work
∴ Rahim can do the work in days
96
7
∴ Krishna can do the work in 96 days.

syE 3 1
8. Now, (Ram + Raju)’s one day work = …(1)
1 15
Gita’s one day work = th work 1
6 (Raju + Rakesh )’s one day work = …(2)
Radha’s one day work =
1
12
1
th work
ngi (Ram + Rakesh )’s one day work =
20
1
12
…(3)
Shalani’s one day work = th work
9
So, (Sita + Gita + Radha + Salani)’s one day work neeNow, 2 (Ram + Raju + Rakesh)’s one day work

=
1
+
1
+
1

=
1 1 1 1 
 3 + 6 + 12 + 9  th work
  rin =
15 20 12
4 + 3 + 5 12
=
=
12 + 6 + 3 + 4
36
th work =
25
36 g.n 60
(Ram + Raju + Rakesh)’s one day work
60

7. (Ram + Rahim)’s one day work


1
= th work
36
∴ They can together do the work in days.
25

…(1)
Now, Rakesh’s one day work
=

=
e 12
120 10

1

=
1

1 3–2 1
= =
…(1)

t
8 10 15 30 30
(Rahim + Krishna)’s one day work Hence, Rakesh can do the work in 30 days.
1 1 1 1
= th work …(2) Ram’s one day work = – =
12 10 20 20
(Krishna + Ram)’s one day work Hence, Ram can do the work = 20 days
1 1 1 6–5 1
= th work …(3) Raju’s one day work = – = =
16 10 12 60 60
Adding equation (1) + equation (2) + equation (3), Hence, Raju can do the work in 60 days.
we get
9. According to question,
2 (Ram + Rahim + Krishna)’s one day work 4 men is equivalent to 5 women
1 1 1  ∴ 4 men = 5 women
=  8 + 12 + 16  th work
  5
1 men = women
6 + 4 + 3 13 4
= = th work
48 48 8 men = 10 women

CAT Complete Course | 147

Visit : www.EasyEngineering.net
Visit : www.EasyEngineering.net

Now, 8 men + 15 women = 10 women + 15 women 31


∴ 40 shirts in 8 days is made by × 40w
= 25 women 5×8
5 women can do a piece of work in 40 days = 31 w
40 Now, since 15 m = 11 w
∴ 25 women can do a piece of work in ×5
25 11 88
8m = × 8w = w
= 8 days. 15 15
10. According to question, ∴ No. of required women
88
16 m + 8 w = 6 days = 31 – = 31 – 5
15
6 × 16 m + 48 w = 1 day
= about 26 women
96 m + 48 w = 1 day …(1)
12. (Gita + Ram + Shyam)’s daily earning
and 8 m + 6 w = 10 days
900 300
80 m + 60 w = 1 day …(2) = = = Rs. 75 …(1)
12 4
Equation (1) and equation (2) are equal, so (Ram + Shyam)’s daily earning
96 m + 48 w = 80 m + 60 w

ww
300
= = Rs. 60 …(2)
16 m = 12 w 5
4m = 3w …(3) (Shyam + Gita)’s daily earning
Now

and
w 32 men = (3 w) × 8 = 24 w

.Ea
Clearly, 32 men + 32 women = 24 w + 32 w = 56 w
Since 16 men = 4 × 4 m = 12 w
=
320
8
= Rs. 40

Now, Gita’s daily earning = Rs. 15


Ram’s daily earning = Rs. 35
…(3)

syE
16 men + 8 women = 12 w + 8 w = 20 w
Now, 20 w can do a certain piece of work in 6 days
∴ 56 w can do a certain piece of work in
Shyam’s daily earning = Rs. 25
13. (5 G + 4 B)’s daily earning
6 × 20
56
=
15
7
days
ngi = Rs.
660
3
= Rs. 220 …(1)

11. 20 shirts in 6 days formed by 5 men + 7 women

∴ 1 shirt in 6 days formed by


5m + 7w
20
nee (10 B + 20 G )’s daily earning

= Rs.
3500
5
= Rs. 700 …(2)

6
∴ 1 shirt in 1 day formed by (5 m + 6 w) …(1)
20 rin
According to question,
5G + 4B
nd
Now, Similarly in 2 case,
2 g.n
In 1st case Rs. 1 earned by

In 2nd case Rs. 1 earned by


220
10B + 20G
One shirt in 1 day by (3 m + 4 w)

1
8
1
= (3 m + 4 w)
4
3

Or,
5G + 4B
220
=e 700
10B + 20G
700
35 G + 28 B = 22 B + 44 G
t
Now, (3 m + 7 w) = (5 m + 4 w)
4 10 6B = 9 G
5 (3 m + 7 w) = 6 ( 5 m + 4 w) ⇒ 2B = 3G …(3)
Or, 15 m + 35 w = 30 m + 24 w In Ist case,
15 m = 11 w …(2) 2 10
5G = 5 × × B = B
11 11 3 3
Now, 3m = 3 × w = w
15 5 10
5G + 4B = B + 4B
11 31 3
3m + 4w = w + 4w = w
5 5 22
= B
According to question, 3
31 22
One shirt in 1 day is made by w Since, B can earn Rs. 220 in one day
5 3
31 2 26
∴ One shirt in 8 days is made by w ∴ 6 B + 4 G = 6 B + × 4B = B
5×8 3 3

148 | CAT Complete Course

Visit : www.EasyEngineering.net
Visit : www.EasyEngineering.net

26 1
Now, B can earn Rs. 700 in one day and the inlet of diameter 4 cm can fill part of tank
3 60
3 26 in 1 minute.
= 220 × × × 26
22 3 9
The inlet of diameter 6 cm can fill part in 1
= Rs. 260 in one day 4 × 60
26 minute.
∴ B earn Rs. 260 in one day
3 Now, In one minute three inlets can fill
26 1
∴ B earn Rs. 1 in = × 1300  1 1 9 
3 260  240 + 60 +  part of tank
26 1 × 1300  4 × 60
B earn Rs. 1300 in = = 5 days
3 260 1+4+9
=
So, In 5 days 6 boys and 4 girls can earn Rs 1300. 240
14. Let the number of inlet water taps = a 14 7
= =
∴ Number of out let taps are = 8 – a 240 120
Since, the tank can be filled by each tap in 36 hours. 120
Clearly, three inlets can fill minutes = 17·15 min

ww a 7
Therefore, part filled by a taps in 1 hours =
36 16. The ratio of amount of work done by man, woman
Now, Similarly and child is 3 : 2 : 1.

w
Part emptied by (8 – a) out let taps in 1 hour =

.Ea
∴ Net part filled in 1 hour =
a 8–a

36 48
8–a
48
∴The amount of work done by a man = 3X
The amount of work done by a woman = 2X
The amount of work done by a child = X

syE =
4a – 3(8 – a)
12 × 3 × 4
7a – 24
Total work done by men = 3X × 20 = 60X
Total work done by women
= 2X × 12 = 24X

ngi
=
12 × 3 × 4
Total work done by 6 children
36 × 4
The tank will be full in hours on opening all = 6×X = 6X

nee
7a – 24
the taps together. According to question,
According to question, Total daily wages amount = Total work done by men
36 × 4
7a – 24
= 6 children
rin
+ Total work done by women + Total work done by

Or,
Or,
24 = 7a – 24
48 = 7a
g.n
270X = 60X + 24X + 6X
270 = 90X

e
48
∴ a = X = 3

a ≅ 7
7

15. In one hour the inlet of 2 cm diameter can fill of


1
4
Required daily wages = 30 × 3X + 25 × 2X

= 90X + 50X + 20X


t
+ X × 20

the tank.
= 160X
∴ In 1 hour the inlet of 4 cm diameter can fill
1 42 = 160 × 3 = Rs. 480.
× = 1 tank
4 22 17. Now, since 1 m3 = 1000 litres
Clearly, inlet of 4 cm diameter can fill tank in 1 hour. 2 m3 = 2000 litres
Similarly,
Volume up to B = 0·28 × 2000 = 560 litres
In 1 hour the inlet of 6 cm diameter can fill
1 62 36 9 1 Volume up to C = 0·5 × 2000 = 1000 litres
= × 2= = = 2 tank
4 2 16 4 4 Volume up to D = 0·7 × 2000 = 1400 litres
4
Clearly, inlet of 6 cm diameter can fill tank in hour Volume up to A = 2000 litres
9
560
1 Time to fill upto B =
Now, the inlet of diameter 2 cm can fill part of 80 + 30 + 20 + 10
4 × 60
tank in 1 minute = 4 minutes

CAT Complete Course | 149

Visit : www.EasyEngineering.net
Visit : www.EasyEngineering.net

1000 – 560 Exercise B


Time to fill from B upto C =
80 – 30 + 20 + 10
1. Ist method
440
= = 5·5 minutes 100 25
80 Efficiency of Ram = = = 12·5%
8 2
Similarly,
100
1400 – 1000 Efficiency of Shyam = = 10%
Time to fill from C upto D = 10
80 – 30 – 20 + 10
100
400 Efficiency of (Rahim + Ram + Shyam) = = 25%
= = 10 minutes 4
40
Now, Rahim’s efficiency = 25 – (10 + 12·5)
2000 – 1400
Time to fill from D upto Top = = 2·5%
80 – 30 – 20 – 10
600 100
= = 30 minutes Now, Rahim can do the same work alone in = 40
20 2·5
days
Total time = 4 + 5·5 + 10 + 30
= 49·5 minutes. Second Method :

ww
18. Let A can fill or empty X% of the tank in an hour.
So, B, C, D and E can fill / empty 2X, 3X, 4X and
Rahim’s 1 day work = Ram, Shyam and Rahim’s 1
day work – (Shyam 1 day work + Ram 1 day work)
 1 1 1  1

w
5X per cent of tank in one hour respectively. =  4 – 10 – 8  = 40
Now, A and D can fill (X + 4X) = 5X per cent of
tank in one hour.
.Ea
In 4 hours they can fill 20 X per cent of tank. But
Hence, Rahim can do the same work alone in 4 0
days.

syE
they take 4 hours to fill the tank.
20X = 100
X = 5
2.

If,
Since, Days ∝

Gita’s efficiency = 2
1
Efficiency

Case II : C, D and E act as input and A and B act as


output. ngi Then, Lila’s efficiency = 1
Let Gita requires X days then
So, 3X + 4X + 5X – (X + 2X) = 9X
= 9 × 5 per cent of tank gets filled in 1 hour nee Lila requires 2.X days to complete the same job
Now, According to question,

So, Time taken to fill tank =


100
45
hours
rin
2.X – X = 5
X = 5
Case I : A, D and E act as input and B and C act as
out put.
X + 4X + 5X – (2X + 3X) = 5X
Second Method :
g.n
Lila requires = 2.X = 10 days

So,
= 5 × 5 per cent of tank gets filled in 1 hour
Time taken =
100
25
complete the same job.

5
+
5
e
Let Gita takes X days, then Lila takes (X + 5) days to

Gita and Lila five days work = 1


= 1
t
100 100 x 5+x
Difference = –
25 45
5x + 25 + 5x = x 2 + 5x
100 10
= = hours x = 5 days
15 × 2 × 3 9
19. Let the ith person be able to do the job in Xi days. Lila takes = 10 days to work completely.
Since, ‘n’ people are working all possible pairs, each 1
3. Since, Time ∝
person works exactly (n – 1) days. Efficiency
Total work done = (n – 1) So, Janki (N) Ganga (G) Jamuna (J)
1 1 1  Efficiency 1 5 3
 X + X + X + …… + X1  1 1 1
 1 2 3 n Now, + + = 1
G J N
All persons working together in a single day would
1 1 1
be able to do + + = 1
5N 3N N
1 1 1 1  23
W= + + + …… +  of the work. N =
 x1 x2 x3 xn  15

150 | CAT Complete Course

Visit : www.EasyEngineering.net
Visit : www.EasyEngineering.net

and J = 3N and G = 5N ; Difference = 2N Part of tank emptied in one hour


46 10 – X
Difference = ≅ 3 days =
15 8
So, Number of days taken by Ganga X
Part of tank filled =
100 12
= = 3 days
100 According to question,
3 X 10 – X 1
– =
Number of days taken by Janki = 15 days 12 8 8
Number of days taken by Jamuna = 5 days Or, 5.X = 33
4. Rajesh packs 24 apples working 6 hours/day 33
X = ≅ 7 pipes.
24 5
Rajesh packs apples working 1 hrs./day 100
6 6. Efficiency of inlet X = = 4%
24 25
Rajesh packs × 8 apples working 8 hrs./day 100
6 Efficiency of inlet Y = = 5%
20
Rajesh packs 32 apples working 8 hrs./day …(1)

ww
Similarly,

Rajesh’s wife packs


20
× 8 apples working 8 hrs./day
∴ Efficiency of X and Y together

=
100
25 × 4
× 9 = 9%

w 5

.Ea
∴ Rajesh’s wife packs 32 apples working 8 hrs./day
In (Rajesh + his wife) pack 64 apples working 8
hrs./2 days …(2)
Pipes X and Y together can fill tank in
hours
25 × 4 100
9
=
9

syE
Now,64 apples are packed in 2 days

∴ 1 apple is packed in
2
days
Now, If the efficiency of Z be a%, then
the capacity of tank which will be filled
100
9
hours

64
∴ 4000 apples are packed in ngi =
100  100
 – a
9  9


1
32
× 4000 days =
1000
8
= 125 days
nee According to question,
Amount of water is being emplied by Z at a% per
hour, then
Again
rin
100  100 
 – a  × 1a = 5
9  9 
Rajesh packs 36 guavas working 6 hours/day

Rajesh packs
36
× 8 guavas working 8 hours/day Or,  g.n
20  100 – 9a 
 = 1

e
8 9a  9 
Rajesh pack 48 guavas working 8 hours/day
Similarly,

His wife packs


25
5
…(3)

× 8 guavas working 8 hours/day


Or,
Or,
2000 – 180a = 81a
261a = 2000

a =
2000
t
261
40 guavas is packed working 8 hours/day …(4) 100 2000
Hence, in hours % tank is filled only, hence
Now, (Rajesh + his wife) pack 88 guavas working 8 9 261
hours/2 day  2000 
the remaining  100 –  % of the capacity will be
Since, 88 guavas are packed in 2 days  261 
2 filled by pipes X and Y in
∴ one guava is packed in days
88 100 × 241
= × 9 ≅ 8 hours.
1 261 × 100
∴ 4400 guavas are packed in × 4400 = 100 days
44 7. Let pipe P fill the tank in X hours and then pipe Q
Clearly, in 125 + 100 = 225 days they finish the work fill it in 2X hours.
of packing. Therefore, In one hour they will fill
5. Let number of inlet pipes = X 1 1 3
= + =
Number of outlet pipes = 10 – X 2X X 2X

CAT Complete Course | 151

Visit : www.EasyEngineering.net
Visit : www.EasyEngineering.net

2X 1 10
Time required = hours Ramu can complete work in days
3 6 6
When the pipe R is also opened then it takes 10
= days ≅ 3 days
2X 2X 3 3
= + ×
3 3 4 Required days taken by Ramu = 5 + 3 ≅ 8 days
2X X 7X Now, Both together can do the same work in 6 days
= + =
3 2 6 ∴ Required difference in the number
Now, in one hour pipe P, Q and R working together = 8 – 6 = 2 days.
fill
10. A B C
6 1 1 6
= + – = Efficiency 3 6 2
7X 2X R 7X
∴ R = 2.X Number of days 3.X 6.X 2.X
Hence, in 2.X hours pipe R can empty the whole tank According to question,
7X 3.X – 2.X = 12
∴ = 21
6 Or, X = 12

ww
Now,
X = 18
2.X = 36 hours.
Number of days taken by A = 36
Number of days taken by B = 72

w
8. From the last statement, Number of days taken by C = 24
In 72 days B can completes the work
From the first statement ,
.Ea
Efficiency of Sona, Mona and Soni = 50%

Soni takes 2 days more than Sona.


So in 2 days B can completes work
1
36

From the second statement,


syE
Mona had worked for 6 days and Sona had worked
Similarly, in 2 days C can work work

Now, Total work done in 2 days


1
12

for 3 days only.


Number of days taken by Sona = 6
ngi =
1
+
1
=
36 12 36 9
4 1
= work

nee
Efficiency = 16·66% 8
Rest work =
9
It means Sona has completed 16·66 × 3 = 50% work
Since, in 36 days A can finish the work.
in 3 days
Therefore, Mona had completed 50% work in 6 days.
8

rin 8
So, work is finished by A in 36 × days = 32 days.
9 9

g.n
50 11. Days 1 2 3
Efficiency of Mona = = 8.33 %
6 X Y Z
Efficiency of Soni = 50 – (16·66 + 8·33)

∴ Sona takes 6 days.


= 25%

9. Rohan can do in 15 days one work


Work in one day
4
1
4 e
Now, in 3 days X, Y, Z can do
6
1
6
8

t
1
8

1 1 1 6 + 4 + 3 13
1 + + = = work
∴ Rohan can do in 5 days work 4 6 8 6×4 24
3 11
1 Rest work = work
Similarly, Ramu can do in 5 days work 24
2 Day 4th 5th 6th
1 1  X Y Z
Both together can do in one day  +  work
 15 10  Work in
1 1 1
4 6 8
2+3 1
= = work Now, X and Y can do
5×3×2 6
1 1 3+2 5
5 + = = work
Both together can do in 5 days = work 4 6 12 12
6
13 5 23
1 In 5 days Total work done = + = work
Rest work = work 24 12 24
6 1
Given Ramu can complete 1 work in 10 days Rest work =
24

152 | CAT Complete Course

Visit : www.EasyEngineering.net
Visit : www.EasyEngineering.net

The work is done by only Z on 6th day. Since, 2 man can finish the work in 104 days
∴ Total number of days = 6. ∴ 1 man can finish the work in 2 × 104 days
2 13
12. Kamal (Ka) + Krishan (Kr) = ∴ man can finish the work in
5 6
4 2 × 104 × 6
Krishna (Kr) + Raju (R) = = = 96 days.
5 13
Since (Ka + Kr + Kr + R) – (Ka + Kr + R) = Kr 15. According to question,
6 1 One man = 2 children
– 1 = = kr.
5 5 1M = 2C
1
Krishna = Since, (3 C + 1 M) complete work in 6 days
5
1 
Now, Kamal = – =
2 1 1  2 · 3 M + 1 M  complete work in 6 days
5 5 5  
4 1 3 5
Raju = – = · M complete the work in 6 days
5 5 5 2

ww
So, the whole amount will be distributed in the ratio
of 1 : 1 : 3.
∴ 5 M complete the work in 6 ×
5 1
× = 3 days.
2 5
16. We represent men as M and women as W –

w
Now, Raju is greatest efficient.
3 ∴ 6M + 5 W = 4 …(1)

.Ea
So, he get his own share = × 900 = Rs. 540.
5
13. 2 men can complete the job in 52 days hence 1 man
4M + 2 W = 7
In Ist case—Work will be complete in One day by
…(2)

day job is
1
104 syE
can complete same job in 108 days. So, man’s one (24 M + 20 W)
and, in IInd case—Work will be complete in One day
by (28 M + 14 W)
3 women can complete the job in 52 days hence 1
women can complete same job in 156 days.
ngi Hence, 24 M + 20 W = 28 M + 14 W
4M = 6W

nee
1
So, women’s one day job is
156 2M = 3W
4 boys can complete the job in 52 days hence 1 boy M 3
∴ =
can complete same job in 208 days. So, boy’s one
day job is
1
rinW
So, Amount received by men
2

g.n
208 3
= × 48 = Rs. 72
If all work together then their one day work 2
1 1 1

=
= +
104 156 208
+

1  1 1 1  1  6 + 4 + 3 
52  2 3 4  52  12 
+ + =
one work
Similarly,
e
17. In 12 days 4 boys and 6 girls can complete one work
∴ In one day (4 × 12 B + 6 × 12 G) can complete
…(1)
t
1 13 1 In one day (7 × 7 B + 8 × 7 G) can complete one
= × =
52 12 48 work …(2)
Hence, work will complete in = 48 days. Now, From (1) and (2), we get
14. According to question, 48 B + 72 G = 49 B + 56 G
Work done by 2 men = 3 women = 4 boys 16 G = B …(3)
3 Now, In Ist case—
1 man = women
2 4 Boys + 6 Girls = 4 × 16 G + 6 G
1 man = 2 boys = (64 + 6)G
Or, Again 1 man + 1 woman + 1 boy = 70 G
2 1
= 1 man + man + man Again, 2 Boys + 3 Girls = 2 × 16 G + 3G
3 2
= 32 G + 3 G
6 + 4 + 3 13
= = man = 35 G
6 6

CAT Complete Course | 153

Visit : www.EasyEngineering.net
Visit : www.EasyEngineering.net

Since, 70 G can complete the work in 12 days. So, X Y Z


So, 35 G can complete the work in 24 days. Efficiency 3 2 6
18. Since, Number of days 2 3 1
In Y days (Y – 2) men do a piece of work Number of days taken by X = 8
In one day (Y – 2). Y men do a piece of work …(1) 3
Number of days taken by Y = 8 × = 4
Again, 6
3 1 4
In (Y – 10) days (Y + 7) men can do work Number of days taken by Z = 8 × =
4 6 3
4 1 1 1+2 3
∴ In (Y – 10) days (Y + 7) men can do one work One day work of (X + Y) = + = =
3 8 4 8 8
4 1 3
In one day (Y – 10). (Y + 7) men one work …(2) One day work of (Y + Z) = + = 1
3 4 4
3 1 6+1 7
Similarly, One day work of (X + Z) = + = =
4 8 8 8
Equation (1) and equation (2) are equal.
Hence, the work will be finish on 2nd day.

ww
So,
2
Y – 6Y – 280 = 0
4
Y. (Y – 2) = · (Y – 10). (Y + 7)
3 21. Since, Efficiency ∝
1
Number of days

w Y = 28 or Y = – 14

.Ea
So, Acceptable value of Y = 28
Now Total men = Y + 10 = 38
So, Sita can do a piece of work in 15 days

Gita can do a piece of work in


15
2
1
days

syE
Sita’s one day work =
Since, 26 men can do the work in 28 days 15
∴ 1 men can do the work in 28 × 26 days Gita’s one day work =
2
28 × 26 15

19.
∴ 38 men can do the work in

Total work = 33 × 30
38
≅ 14 days

ngi Let Gita joined Sita after X days, then

In X days Sita can do work =


X
= 990 girls – 1 day

First day’s work =


1
× 44
neeNow,
15
Rest day = 11 – X

rin
990 (Gita + Sita)’s one day work
1 2 1
1 = + =
Second day’s work = × 43

g.n
15 15 5
990
11 – X
1 Now, Both can do in (11 – X) day work =
Third day’s work = × 42 and so on 5
990
So, the total work in 44 days

=
1
990
(44 + 43 + 42 + ……) Or,
11 – X X
5

15
+
15
3 (11 – X) + X
e
= 1

= 1 t
We know the Sum of natural Number Or, 33 – 3.X + X = 15
n (n + 1) Or, 18 = 2.X
1 + 2 + 3 + 4 + …… + n is
2 Or, X = 9 days
So, Total work done in 44 days So, they work together for 2 days.
1 44 × 45
= × =1 22. Let the work will be finished in d days
990 2
1
Hence, in 44 days Total work will be completed. Rani’s one day work =
9
20. According to question, we have 1
X Y Z Sheela’s one day work =
18
Efficiency 1 – 2 1
Soni’s one day work =
Days – 3 1 24
1 d
We know Efficiency ∝ Now, in d days Soni’s work =
Days 9

154 | CAT Complete Course

Visit : www.EasyEngineering.net
Visit : www.EasyEngineering.net

d–2 25. Let the capacities large, medium and small ships are
In (d – 2) days, Rani’s work =
18 represented by X, Y and Z respectively.
d–4 Now, According to question,
and in (d – 4) days, Sheela’s work =
24 2.X = 5.Z …(1)
d d–2 d–4
Now, + + = 1 2.Y = X + 2.Z …(2)
9 18 24
From equation (1) and equation (2), we get
8d + 4 (d – 2) + 3 (d – 4)
⇒ = 1 5.Z 9
9×2×4 2.Y = + 2.Z = · Z
2 2
92
Hence, d = ≅ 6 days. 9
15 Y = ·Z …(3)
4
3
23. Work of a boy for one hour = girl’s work for 1 hour Let the required number of journey is ‘a’, then
2
Again, Work of a boy for 1 day a (20 × X + 14 × Y + 24 × Z)
3 9   1
=  2 × 7·5  girl’s for 1 day = (5 × X + 8 × Y + 12 × Z)  42 
   5

ww =
9
5
girls work for 1 day

Or, a  20 ×

5.Z
2
9
+ 14 × · Z + 24.Z
4


w
9
So, 1 boy = girl  1   5.Z 
5 =  425   5 × 2 + 8 × 49 · Z + 12.Z 

.Ea 9
10 boy + 6 girls = 10 × + 6 = 24 girls
5
 

a  50 +
63   25

  1
+ 24  =  + 18 + 12  ×  42 

syE
Or,
9
Again, 5 boys + 9 girls = 5 × + 9 = 18 girls  2  2   5
5
 100 + 63 + 48   25   
Since, 24 girls can do the work in 18 days Or, a  =  2 + 18 + 12  ×  4251 
     
So, 1 girl can do the work in 18 × 24

18 girls can do the work in


18 × 24
= 24 days ngi Or,
2


211
=
85  1 
×  42 
18
Hence, 6 more days will be required to complete the
job with the help of 5 boys and 9 girls. nee
Or,
2

a =
2  5

85  1 
×  42 
24. Pipe X can fill =
1
15
part of tank in one hour
rin 211  5 
= 17 journeys.
Pipe Y can fill =
1
10
part of tank in one hour
g.n
26. Let a be the capacity of cistern.

Since, Pipe X in one hour can fill =


1
part of cistern
Now, in one hour both fill
=
1
+
1
=
2+3
15 10 5 × 3 × 2 6
In 4 hours both the pipes (X + Y) can fill
=
1
e
2
15

Pipe X in 2 hours can fill = part of cistern


15
2
t
Similarly, pipe Y in 2 hours can fill = part of
4 2 20
= part of the tank
6 3 cistern
2 1 According to question : Cistern if full
Now, Rest part = 1 – =
3 3
a a 
Now, According to question, So, a + 2 hr.  + – 10  = 0
 15 20 
If X be required in which both the pipes work in
case of air bubbles.  4.a + 3.a – 600 
Or, a + 2 × 60   = 0
1 1 1 1  1  5×3×4 
Now, X × + ×  =
 2 15 3 10  3  7a – 600 
2X 1 Or, a + 2 × 60 ×   = 0
Or, =  60 
30 3
2X 1 Or, a + 14.a – 1200 = 0
Or, =
30 3 a = 80 litres
X = 5 hours Hence, the cistern capacity is 80 litres.

CAT Complete Course | 155

Visit : www.EasyEngineering.net
Visit : www.EasyEngineering.net

27. Let the first machine drive a metres/day and second Now, the number of hours taken by friend
machine b metres/day. Then 300
= = 7·5 Hrs.
 30   80  40
60a   + 60b   = 60
 100   300  30. The pipe fills the cistern in one minute =
1
30
Or, 18a + 48b = 60 …(1)
1
In Second case, Due to a leak cistern in one minute =
50
First machine finishes the work in Now, leak takes required time to empty
2 60b b 1 1 20
× = 40 days = – =
3 a a 30 50 1500
and Second machine finishes the work in So, Required time taken = 75 minutes
3 60.a a
× = 18 days 31. The ratio of amount of work done by a man, women
10 b b and a child is 2 : 5 : 7.
b a
Then 40 × – 18 × = 6 The amount of work done by a man = 2.a
a b
The amount of work done by a women = 5.a

ww
We put

18
b
a
= C The amount of work done by a child = 7.a
Total work done by man = 2.a × 5 = 10.a

w
40.C – = 6
C Total work done by women = 5.a × 8 = 40.a
Or,
Or,
.Ea 20.C2 – 9 = 3.C
20.C2 – 3.C – 9 = 0
Total work done by children = 10 × 7.a = 70.a
So, Total daily wages amount = Total work done by

syE
Or, 2
20.C – 15.C + 12.C – 9 = 0 men + Total work done by women + Total work
done by children
Or, 5.C (4.C – 3) + 3 (4.C – 3) = 0
So, 512 = 120.a
Or, (4.C – 3)(5.C + 3) = 0

Or, C = =
3 b
4 a ngi a =
512 128
=
120 30

nee
Total wages of twelve men + Fifteen women + Eight
28. Let V be the volume of the tank. a be the time taken
children
by the second pipe to fill the tank.
= 12 × 2.a + 15 × 5.a + 8 × 7.a
Or, a + 5 = time taken by the first pipe to fill the pool
a – 4 = time taken by the third pipe to fill the pool
rin
= 24.a + 75.a + 56.a
= 155.a

g.n
V V V
Or, + = 128
a+5 a a–4 = 155 ×
2.a + 5 1 30
Or,

Or,
Or,
(a + 5)a
=

a2 – 8a – 20 = 0
(a – 10)(a + 2) = 0
a–4

=
e
= 31 ×
1984
3
128
6
= 31 ×
64
3
t
Or, a = 10 32. Pipe P can fill in 12 hours one tank
Hence, the time taken by the third pipe to fill the tank 1
is 10 – 4 = 6 hours. ∴ Pipe P can fill in 1 hours th tank
12
29. Total area to be laid = 4 × (10 × 15) Similarly,
+ 2 × (12 × 10) 1
In 1 hour pipe Q can fill th part of the tank
= 600 + 240 = 840 sq. feet 18
In one hour, Ram can lay 1 1 
Both the pipes (P + Q) can fill =  +  th in one
=
12 × 10
= 60 sq. feet  12 18 
2 hour
In one hour, the friend can lay In one hour (P + Q) can fill
12 × 10 3+2 5
= = 40 sq. feet = = th tank
3 4 × 9 36
In 9 hours, Ram can lay = 9 × 60 = 540 sq. feet 36
(P + Q) can fill the tank in hours
Remaining job = 840 – 540 = 300 5

156 | CAT Complete Course

Visit : www.EasyEngineering.net
Visit : www.EasyEngineering.net

33. If all the three pipes are open then 1


∴ In one hour (two pipes + leak) can fill = part of
In one hour they can fill 8
cistern
1 1 1 
=  9 + 12 – 15  part of tank Now, In one hour leak can empty water
  15 1 1
20 + 15 – 12 23 = – =
= = part of tank 112 8 112
3 × 4 × 5 × 3 9 × 20 ∴ Leak will empty the full cistern in 112 hours.
9 × 20 180
Clearly, the tank will be full in hours = 1
23 23 36. Since, In one minute pipe X can fill part of tank
30
hours.
1
34. Since, the Quantity of water is proportional to square In one minute pipe Y can fill part of tank
20
of its diameter.
1
1 In one minute pipe Z can fill part of tank
∴ In one hour in inlet of 1 cm. diameter can fill 10
10
In one minute (X + Y + Z) can fill
of the tank
1 1 1 11
In one hour in inlet of 2 cm. diameter can fill = + + =

ww
1 22 4
× =
10 12 10
of the tank
30 20 10 60
In 3 minutes (X + Y + Z) can fill
11 11

w
Similarly,
In one hour the inlet of 3 cm. diameter can fill
1 32 9
×
10 12 10
= of the tank
.Ea
= 3× =
60 20

Individually Pipe Z can fill in 3 minutes =


3
10

syE
In one hour three inlets of diameter can fill

+ =
9 14
of the tank
1
+
10 10
4 Now, Required ratio =
3 20 6
× =
10 11 11
37. Let the first boy do a% of the job in a day so 2nd will
10 10
7
= of the tank
5 ngi do 2a% 3rd will do 4a% and 4th will do 8a% in a day.
All of them working together will take 5 days to

nee
complete a job i.e. in one day they are finishing 20%
5 of the job.
Hence, the whole tank will get filled in × 60
7 So, we get a + 2.a + 4.a + 8.a = 20

rin
300
minutes = minutes 20 4
7 Or, a = =
15 3

g.n
35. Since, pipe X can fill one cistern in 14 hours.
Since, 2nd and 3rd working together will finish
1
So, In one hour pipe X can fill = th part of cistern 4
14 6.a% = × 6
1
In one hour pipe Y can fill = th part of cistern
16
Now, In one hour pipes X and Y can fill
3

e
= 8% of the job in a day
So, Time taken by 2nd and 3rd
100 25 1
t
1 1 15 = = ≅ 12 days
= + = 8 2 2
14 16 112
32
112 and fourth will finish 8a% = % of the job in a day
Time taken by these pipes to fill the tank = 3
15
100 300
= 7 hours 28 minutes and fourth will take = ×3= ≅ 9 days
32 32
Since, Due to leakage, time taken is given by 1 1
T = 7 hrs. 28 min. + 32 min. = 8 Hrs. Hence, extra days = 12 – 9 = 3 days.
2 2
●●●

CAT Complete Course | 157

Visit : www.EasyEngineering.net
Visit : www.EasyEngineering.net

11 Simple and Compound Interest


There are two types of Interest— Solution :
1. Simple Interest P× r×t
(i) S.I. =
2. Compound Interest 100
1200 × 5 × 2
If the interest on a certain sum borrowed for a certain = = Rs. 120
100
period is calculated uniformly. It is called simple interest.
P× r×t
In Compound Interest—For every unit of time (yearly, (ii) S.I. =
100
half-yearly, quarterly or monthly) principal varies. The

ww
simple interest due at the end of the first unit of time is
added to the principal and the amount so obtained
becomes the principal for the second unit of time. Simi-
2
Here, time = 2 months = year = year
12
1800 × 10 1
1
6

w
larly the amount after the second unit of time becomes

.Ea
the principal for third unit of time and so on.
SIMPLE INTEREST
Now, S.I. =

(iii) Time = t = 730 days =


100
730
365
3650 730
× = Rs. 30

year
6

syE
Transaction of money after takes places among
banks, individuals, business and other concerns.
Now, S.I. = ×
100 365
= 12 × 73 = Rs. 876

ngi
If a person X borrows some money from another
Effect of Change of P, r, t on Simple Interest
person Y for a certain period, then after that specified
period, the borrower has to return the money borrowed as (A) If P1 changes to P2 and r, t are fixed, then
well as some additional money. This additional money
that borrower has to pay is called Interest. nee Change in S.I. =
r×t
(P – P 1 ) (P2 > P1)
100 2
Here, person Y is called lender, who gives money
and person X is known as borrower, who receives money.
Then Change in S.I. = rin
(B) If P remains constant and r and t change—
P
(r t – r t )
Principal (P)—The money given by the lender is
called principal / Sum. Formula of Amount :
g.n
100 1 1 2 2

∴ Amount = Principal + Interest


Rate Percent (R)—The interest that the borrower has
to pay for every 100 rupees borrowed for every year is
known as rate per cent per annum. It is denoted by r% or
A = P+ e
Amount = Principal + Interest
P× r×t
100
 r × t
t
⇒ A = P 1 + 
R%.  100 
If P = Principal  100 
Or, A = S.I.  1 + 
r = Rate per cent per annum  r × t
t = Time Period (C) Effect of change of r or t on Amount—
Now, Simple Interest is given by If P is fixed and either r or t is variable. So, principal
P× r×t is given by
S.I. =
100 A × X2 – A2 × X1
Illustration 1. P = 1
X1 – X2
Find Simple Interest in the following cases— where X = r or t
(i) P = Rs. 1200, r = 5 % per annum, t = 2 years where A = Amount
(ii) P = Rs. 1800, r = 10% per annum, t = 2 months A1 – A2
Again, r or t = × 100
(iii) P = Rs. 3650, r = 12% per annum, t = 730 days A1 × X2 – A2 × X1

158 | CAT Complete Course

Visit : www.EasyEngineering.net
Visit : www.EasyEngineering.net

Illustration 2. 12 × 4
Or, = r
If Given P = Rs. 120, r = 2% per annum, and t = 5 5
years. Then Find A = ? 48
Or, = r
5
Solution :
r = 9·6% per annum.
From given formula,
Illustration 4.
 r × t
A = P 1 +  At what rate of interest per annum will a sum double
 100 
itself in 5 years ?
 5 × 2 Solution :
A = 120 ×  1 + 
 100  Let required principal = X
11 According to question,
= 120 × = Rs. 132.
10 Amount after 5 years = 2.X
Illustration 3. Interest = 2.X – X = X
Find the following : P× r×t
S.I. =
P=?
ww
(i) A = Rs. 7200, r = 10% per annum, t = 5 years,

(ii) P = Rs. 600,r = 3% per annum, t = 12 months,


Or, X =
100
X×r×5
100
A = ?

t=?
w .Ea
(iii) S.I. = 150, P = Rs. 2000, r = 12% per annum,

(iv) S.I. = 120, P = Rs. 5000, t = 3 months, r = ?


Or, r = 20%
(D) Repayment of debt in equal Installments—Let
Ram borrows an amount Rs. M and he returns it in equal
installments at a rate r% annum if ‘i’ is annual installment.
Solution :
syE
 r × t
A = P 1 + 
Now, Borrowed amount = M

= n.i +
r×i
×
n (n – 1)

ngi
(i) 100 × Y 2
 100 
where Y = no. of installments per annum
 10 × 5
7200 = P  1 + 
nee
Y = 1, when each installment is paid
 100  yearly.
3 Y = 2, when each installment is paid half-
Or, 7200 = P ×
2
P = 2400 × 2 = Rs. 4800 Illustration 5. rin
yearly.

(ii) From given formula



A = P 1 +
r × t
 g.n
What annual installment will discharge a debt of Rs.
4800 due in 5 years at 10% simple interest
 100 


A = 600 ×  1 +

3 12 
× 
100 12 
Solution :
Using formula,

M = n.i +
e
r×i
100 × Y
×
n (n – 1)
2
t
103
= 600 × = Rs. 618 M = Rs. 4800, n = 5, Y = 1, r = 10%, i = annual
100 installment
P× r×t 10 × i 5 × 4
(iii) S.I. = 4800 = 5.i + ×
100 100 × 1 2
2000 × 12 × t i
Or, 150 = = 5.i + × 10
100 10
15 5 5 6.i = 4800
Or, t = = = years
2 × 12 4 × 2 8 i = Rs. 800.
5 15
t = × 12 months = months COMPOUND INTEREST
8 2
P× t×t The difference between the final amount (A) obtained
(iv) S.I. =
100 at the end of the last unit of time and the original principal
3×r (P).
120 = 5000 ×
12 × 100 Compound Interest C.I. = A – P

CAT Complete Course | 159

Visit : www.EasyEngineering.net
Visit : www.EasyEngineering.net

Some important formula,  2892 – 2562 


= 1200 
We have Principal = Rs. P  2562 
Rate = r% per annum 1200 × 545 × 33
= = 329·32.
Time period = t years 256 × 256
Amount = Rs. A Illustration 8.
1. When Interest is compounded annually Shyam invests Rs. 25,000 in a bond which gives
interest at 10% per annum during the first year, 20%
 r 
t

A = P 1 +  during the second year and 25% during the third year.
 100  How much does he get at the end of third year ?
2. When Interest is compound half-yearly Solution :
 r  2t From question,
 2 
A = P  1 + P = Rs. 25,000, r1 = 10%, r2 = 20%, r3 = 25%
100
Amount is given by
 r 
2t

A = P 1 +   r1   r2   r3 
 200  A = P 1 +   1 + 100   1 + 100 

ww
Illustration 6.
C.I. = A – P
 100  

= 25000 ×  1 +
10  

20 
 ×  1 + 100 

w
Find compound interest on Rs. 2000 at 5% per

.Ea
annum, compounded yearly, for 2 years ?
Solution :
 100   

× 1 +

25 

100 

Now,

syE
P = Rs. 2000, r = 5%, t = 2 years

A = P 1 +
r 
 100 

t

Illustration 9.
= 25000 ×
11 6 5
× × = Rs. 41,250.
10 5 4


= 2000  1 +
 100 
5 

2

ngi Find the compound interest on Rs. 20000 for 3


1
2

= 2000 ×
21 × 21
400
= 5 × 441
nee
years at 20% per annum compounded yearly ?
Solution :
1

Illustration 7.
A = Rs. 2205.
Now, Amount is given by rin
Now, P = Rs. 20000, r = 20%, t = 3 years
2

Find the compound interest on Rs. 1200 at 25% per


annum, compounded quarterly for 1 year ? 
A = P 1 +
r  

3
g.n
1+
r 
2 
Solution :
Since, interest is compounded quarterly.
So,
25
Time = 4.X, r = %
4
 100 

= 20000 ×  1 +

e100

20 

100 
3
 10 
 1 + 100


 t
 r  4n A = 20000 ×
216 11
× = Rs. 38016
A = P 1 +  125 10
 4 × 100
C.I. = A – P = 38016 – 20000 = Rs. 18016
 25 
4

= 1200 ×  1 +  Illustration 10.


 4 × 100
A certain sum of money becomes n times in t years
 17  4 when compounded annually. In how many years will it
= 1200 ×  
 16  become nx times at the same rate of compound interest ?
17 × 17 × 289 Solution :
= 1200 ×
16 × 16 × 256 Using formula,
289 × 289  r 
t
= 1200 × A = P 1 + 
256 × 256  100 
C.I. = A – P
 r 
t

= 1200 ×
289 – 289
– 1200 Or, n.P = P  1 + 
256 × 256  100 

160 | CAT Complete Course

Visit : www.EasyEngineering.net
Visit : www.EasyEngineering.net

 r   r 
t t

n =  1 + 100  …(1) A = P 1 + 
   100 
Let the sum become nx times in T years. A
P =
 r 
t
 r 
T

nx =  1 +   1 + 100 
 100   
T For n years, we have
 r 
x
A = Rs. A1
n = 1 +  …(2)
 100  100 (A 2 – A1)
r = % per annum
Comparing equation (1) and equation (2), we get A1
T t = n years
= t
x A1
P =
T = x.t years  100 (A 2 – A1) n
Illustration 11.  1 + 100 × A1 
If the difference between C.I. and S.I. on a certain

ww
sum at r% per annum for 2 years is Rs. X. Find the sum ?
Solution :
P =
A1
 A2 
A 
 1
n

w
Let the sum be Rs. P.

S.I. =
P × r × 2 2Pr
100
 
=
100
.Ea 
A 
P = A1 .  1 
 A2 
n

r 
2

C.I. = P   1 +
  100 
 

2r  r 
syE
– 1

2

– 1
Illustration 13.
A certain sum of money invested at compound inter-
est, compounded annually becomes Rs. 8820 in 2 years
= P 1 + +
  100  100 


ngi and Rs. 9261 in 3 years. Find the rate of interest and the
sum ?
Solution :
= P×
r
100 

× 2 +
r 

100 
nee Rs. (9261 – 8820) = Rs. 441 is the interest on Rs.
8820 for 1 year.

∴ x = CI – SI =
Pr 
× 2 +
100 
r  2pr
–
100  100 r =
441
8820
rin
× 100

=
Pr 
100 
r
 2 + 100
 pr2
– 2 =
 1002 P =
A g.n
r = 5% per annum

P =

Illustration 12.
x (100)2
(r) 2
·

=
 r 
 1 + 100


 e
t

8820
=
8820 t
 5 
2
441
A certain sum of money invested is compounded  1 + 100  400
yearly, becomes Rs. A1 in n years and Rs. A2 in (n + 1)  
years. Find the sum and the rate of interest ? P = Rs. 8000.
Solution : Illustration 14.
Rs. (A2 – A1) is the interest on Rs. A1 in 1 year. Divide Rs. 10387 into two parts, such that the first
Using Formula : part after 5 years is equal to the second part after 7 years,
100 × I compound interest being 10% per annum compounded
r = yearly ?
P.t
I = Rs. (A2 – A1) Solution :
P = Rs. A1 Let the first part = Rs. X
Second part = 10387 – X
t = 1 year
 10 
5
100 (A 2 – A1)
r = % per annum Now, Amount after 5 years = X.  1 + 
A1  100 

CAT Complete Course | 161

Visit : www.EasyEngineering.net
Visit : www.EasyEngineering.net

 10 
7
6. In how many years Rs. 1000 will produce the same
Amount after 7 years = (10387 – X) ×  1 +  interest at the rate of 6% as Rs. 1200 produce in 5
 100 
1
years @ 5 %?
According to question, 2
 11  5  11  7 (A) 55% (B) 12·5%
X.   = (10387 – X) ×  
 10   10  (C) 5·5% (D) 16·33%
121 (E) 35%
Or, X = (10387 – X) ×
100 7. Two equal sum of money were lent in at simple
221 121 1 1
Or, × X = 10387 × interest at 12% p.a. for 2 years and 3 years respec-
2 2
100 100
tively. If the difference in interests for two periods
121
Or, X = 10387 × was Rs. 720. Then Find the each sum ?
221
(A) Rs. 650 (B) Rs. 950
= 47 × 121 = 5687
(C) Rs. 5760 (D) Rs. 1250
Second Part = Rs. 4700.

ww Exercise A
2
(E) Rs. 4000
8. Gitanjali invested a certain amount in three different

w
1. Find the simple interest on Rs. 15,000 at 2 % per schemes X, Y and Z with the rate of interest 5% p.a.,
3
10% p.a. and 15% p.a. respectively. If the total

.Ea
annum for 3 months ?
interest accrued in one year was Rs. 4500 and the
(A) Rs. 200 (B) Rs. 100
amount invested in scheme Z was 180% of the
(C) Rs. 350 (D) Rs. 50
(E) Rs. 150
syE 1
2. Find the simple interest on Rs. 20,000 at 7 % per
amount of the amount invested in scheme X and
120% of the amount invested in scheme Y. What was
the amount invested in scheme Y ?
4
annum for the period from 4th Feb., 2005 to 18th
April, 2005 ? ngi (A) Rs. 45117
(C) Rs. 90234
(B) Rs. 34468
(D) Rs. 81000

(A) Rs. 290


(C) Rs. 190
(B) Rs. 450
(D) Rs. 390 nee
(E) Rs. 17234
9. Divide Rs. 25300 into 3 parts so that their amounts 4,
(E) Rs. 150
1 rin
6 and 8 years respectively may be equal, the rate of
interest being 5% per annum at simple interest. What
3. A sum at simple interest of 6 % per annum amounts
4
to Rs. 25,000 after 4 years. Find the sum ? (A) Rs. 1110
g.n
is the difference between first and third part ?
(B) Rs. 17500
(A) Rs. 12000
(C) Rs. 8000
(E) Rs. 20000
(B) Rs. 15000
(D) Rs. 10000
(C) Rs. 8400
(E) Rs. 9100
e
(D) Rs. 1700

t
10. The Simple interest on a certain sum of money for 3
1
2
4. A sum of Rs. 1000 amounts to Rs. 1200 in 5 years at years at 8% per annum is Rs. 57 less than the simple
simple interest. If the interest rate is increased by 1
1 interest on the same sum for 5 years at 12% per
3 %, it would amount to how much ? 2
2 annum. Find the sum ?
(A) Rs. 1000 (B) Rs. 1200 (A) Rs. 600 (B) Rs. 750
(C) Rs. 850 (D) Rs. 1500 (C) Rs. 150 (D) Rs. 1200
(E) Rs. 1375 (E) Rs. 1800
5. Raju borrows Rs. 25000 for 2 years at 5% p.a. simple
1 Exercise B
interest. He immediately lends it to Rani at 7 % p.a.
2 1. Find compound interest on Rs. 225,00 at 4% per
for 2 years. Find his gain in the transaction per year ? annum for 2 years, compounded annually ?
(A) Rs. 125 (B) Rs. 225 (A) Rs. 5625 (B) Rs. 1125
(C) Rs. 625 (D) Rs. 500 (C) Rs. 1416 (D) Rs. 1500
(E) Rs. 800 (E) Rs. 1836

162 | CAT Complete Course

Visit : www.EasyEngineering.net
Visit : www.EasyEngineering.net

2. Ramu borrowed Rs. 20,000 from two persons he paid (A) Rs. 12000 (B) Rs. 14218
6% interest to one and 8% per annum to the other. In (C) Rs. 26000 (D) Rs. 24000
one year he paid total interest Rs. 1280. How much
(E) Rs. 16412
did he borrow at 8% rate ?
9. A certain sum of money amounts Rs. 4800 in 3 years
(A) Rs. 6000 (B) Rs. 8000
at 5% per annum simple interest in how many years
(C) Rs. 12000 (D) Rs. 4000 will it amount to Rs. 5600 at the same rate of
(E) Rs. 16000 interest ?
3. A borrowed Rs. 1200 at 5% per annum and Rs. 1600 (A) 5 year 10 months
at 6% per annum for the same period. He paid (B) 6 year 10 months
Rs. 312 as total interest. Find the time for which he (C) 7 year 10 months
borrowed the sum ?
(D) 3 year 10 months
(A) 0·5 years (B) 1 year
(E) 4 year 10 months
(C) 1·5 years (D) 2 years
10. An amount of Rs. 50440 borrowed at 5% per annum
(E) 2·5 years compounded yearly, is to be repaid in 3 equal install-

ww
4. Find the annual installment that will discharge a debt
of Rs. 1,60,000 due in 5 years at 10% per annum
simple interest ?
ments. Find the amount of each installment ?
(A) Rs. 18522 (B) Rs. 20202

w
(C) Rs. 22110 (D) Rs. 19202
(A) Rs. 16000 (B) Rs. 20000 (E) Rs. 23110
(C) Rs. 24000
(E) Rs. 28000 .Ea
(D) Rs. 32000 11. Rs. 24000 is lent out in three parts. The first part is
lent at 4% per annum for 5 years, the second at 5%

syE
5. When the bank reduces the rate interest from 5% to
4% per annum. Rohit with draws Rs. 1000 from his
for 3 years and the third at 8% for 4 years. The total
interest earned on each part is equal. Find the value
of biggest part ?

ngi
account. If he now Rs. 75 less interest during one
year. Find how much total money was there is Rohit’s (A) Rs. 18000 (B) Rs. 16000
account initially ? (C) Rs. 9000 (D) Rs. 10000
(A) Rs. 3500
(C) Rs. 2800
(B) Rs. 3200
(D) Rs. 2200 nee (E) Rs. 12000
1 1
12. Rakesh invests th of his capital at 5% per annum rd
(E) Rs. 2500
6. Find the compound interest on Rs. 2000 for 9 months
5

rin 3
at 2% and the remainder at 3%. If his annual income

g.n
from these is 920. Find this capital invested in
at 8% per annum being given when the interest is thousand ?
reckoned. (A) 36 (B) 24
(i) Quarterly
(ii) Half-yearly
(iii) Yearly
(C) 12
(E) 30 e
(D) 50

13. A sum of money lent out at compound interest


t
(A) Rs. 122; 121; 120 (B) Rs. 120; 121; 122 increases in value by 50% in 3 years. A person wants
(C) Rs. 122; 122; 122; (D) Rs. 122; 121; 118 to lend three different sums of money X, Y and Z for
6, 9 and 12 years respectively at the above rate, in
(E) Rs. 120; 122; 124 such a way that he gets back equal respective periods.
7. A certain sum is interested at compound. The interest Find the ratio X : Y : Z ?
accrued in the first two years is Rs. 272 and that in (A) 3 : 2 : 1 (B) 11 : 9 : 7
the first three years is Rs. 434. Find the rate per cent ? (C) 8 : 4 : 3 (D) 4 : 3 : 2
(A) 8% (B) 7% (E) 9 : 6 : 4
(C) 6% (D) 9% 14. The simple interest on a certain sum for 2 years is
(E) 10% Rs. 60 and the compound interest is Rs. 72. Find the
Sum ?
8. Rajesh set up a factory by investing Rs. 50,000.
During the first three successive years, his profit were (A) Rs. 600 (B) Rs. 100
5, 10 and 15% respectively. If each year profit calcu- (C) Rs. 75 (D) Rs. 60
lated on previous year’s capital. Find his total profit ? (E) Rs. 30

CAT Complete Course | 163

Visit : www.EasyEngineering.net
Visit : www.EasyEngineering.net

15. Divide Rs. 5115 into two parts such that the first part 1000 × r × 5
⇒ 200 =
after 10 years is equal to the second part after 7 years, 100
compound interest being 20% per annum com- r = 4%
pounded yearly and find the difference between two Now, New rate of interest
parts ?
1  7 15
(A) Rs. 3340 (B) Rs. 4440 = 4 × 3 =  4 + % = %
2  2 2
(C) Rs. 5115 (D) Rs. 1875
1000 × 15 × 5
(E) Rs. 1465 Now, New S.I. =
100 × 2
16. If the difference between C.I. and S.I. on a certain = 25 × 15 = Rs. 375
sum at p% per annum for 3 years is Rs. q. Find the
sum ? Now, Amount = Sum + S.I.
17. Krishna lends Rs. a in n parts in such a way that = 1000 + 375 = Rs. 1375.
interest on the first part is r1 % per annum for t1 years, 5. (C) Interest on Rs. 25000
on the second part r2 % for t2 years and so on. If the 25000 × 5 × 2
= = Rs. 2500
interest earned from each part for the corresponding 100

ww
periods are equal. Find the ratio of each part ?

Answers with Hints


Now, Interest on Rs. 25000

=
25000 × 2 × 15
100 × 2
= Rs. 3750

1. w
(B) Using formula,
Exercise A

S.I. =.Ea P× r×t


Now, Amount gained by Raju = 3750 – 2500
= Rs. 1250 in two years
Now, Amount gained by Raju = Rs. 625 in one year.

syE
100
6. (C) Let required time = x years
2 8 3
Here, P = Rs. 15000, r = 2 % = %, t = year According to question,
3 3 12

Now, S.I. =
15000 × 8 × 3
100 × 3 × 12
= Rs. 100
ngi 1000 × 6 × x
100
=
1200 × 11 × 5
100 × 2
55 11

nee
2. (A) Principal = P = Rs. 20000, Or, x = =
10 2
1 29
Rate = r = 7 % = % 11
4 4 ∴ x = %

rin
2
24 + 31 + 18 73 1
Time = t = = = year 7. (C) Now, Let sum of money = Rs. X
365 365 5

S.I. =
P × r × t 20000 × 29 1
100
50 × 29
=
100 × 4
×
5
Now,
nd
In 2 case,
S.I. =
g.n
X × 12 × 5 3.X
100 × 2
=
10

3. (E) Now, S.I. =


=
5
P× r×t
100
= Rs. 290.
S.I. =
e
X × 7 × 5 7.X
100 × 2
Now difference in simple interest
=
40
t
X × 25 × 4 X 3.X 7.X 5.X
= = = – =
100 × 4 4 10 40 40
Now, Amount = Sum + S.I. 5.X
X Now, 720 =
= X+ 40
4 720 × 40
5X Or, X = = Rs. 5760.
25000 = 5
4
8. (E) Let the amount be Rs. a, Rs. b and Rs. c respec-
25000 × 4
X = tively.
5
= 5000 × 4 = Rs. 20000 a × 5 × 1 b × 10 × 1 c × 15 × 1
Now, + + = 4500
100 100 100
4. (E) Let initial rate of interest = r%
Given Principal = Rs. 1000; Amount = Rs. 1200; a + 2b + 3c = 90,000 …(1)
Time = 5 years According to question,
P× r×t 180 9
S.I. = c = ·a= ·a …(2)
100 100 5

164 | CAT Complete Course

Visit : www.EasyEngineering.net
Visit : www.EasyEngineering.net

120 6 13 13
c = ·b= ·b …(3) Now, X = ·Y= × 8400 = Rs. 9100
100 5 12 12
Now, putting the values of a and b in equation (1), Z = 25300 – (9100 + 8400)
we get = 25300 – 17500 = Rs. 7800
5 5
∴ a + · c + 2 × · c + 3.c = 90000 Now, Sums are Rs. 8400, Rs. 9100, Rs. 17500
9 6
Difference between third and first is
10 + 30 + 54
∴ c× = 90000 = 17500 – 8400 = Rs. 9100.
18
94 10. (C) Let the sum be Rs. X.
∴ c× = 90000
18 In Ist case,
90000 × 18 810000 7 28
∴ c = = Rs. = Rs. 17234 S.I. = X × ×8= X …(1)
94 47 2 × 100 100
9. (E) Let the first part = Rs. X In 2nd case,
Let the second part = Rs. Y 11 12 66
S.I. = X × × = X …(2)
2 100 100
Now, Third part = 25300 – (X + Y)

ww
Since,
So, for 4 years
Amount = Sum + S.I.
According to question,
66
100
X–
28
100
X = 57

w
For 6 years,
Amount = X +

.Ea
4 × X × 5 6X
100
=
5
…(1) Or,

Or,
38
100
X = 57

X =
57 × 100
38

For 8 years,
Amount = Y +
syE Y × 5 × 6 13.Y
100
=
10
…(2) 3
= × 100 = Rs. 150.
2

Amount = [25300 – (X + Y)] + [25300 – (X + Y)]


×
5×8 ngi
1. (E)
Exercise B
C.I. = A – P
100

= [25300 – (X + Y)]  1 +

2 
5 
nee 
A = P 1 +

4 

100 
2
 26 
= 22500  
 25 
2

7 
= [25300 – (X + Y)]   rin 262
C.I. = 22500 
 25 2

– 1 

g.n
…(3)
5   262 – 252 
= 22500   = 1836
According to question,  252 
6.X
5
=
13.Y
10
12.X = 13.Y …(4)
e
2. (D) Let the sum borrowed at 6% interest be Rs. a.
Now, the sum borrowed at 8% = (20000 – a)
Now, time is one year
t
6.X   13   2  r1 = 6%, r2 = 8%
Now,
5
=  25300 –  Y + 12 · Y   1 + 5 
P ×r ×t P ×r ×t
S.I.1 = 1 1 S.I.2 = 2 2
6.X  25  7 100 100
Or, =  25300 – 12 · Y × a×6×1 (20000 – a) × 8 × 1
5   5 S.I.1 = S.I.2 =
100 100
6 13   7
Or, × ·Y =  25300 – 25 ·Y × Since, S.I.1 + S.I. 2 = 1280
5 12  12  5 6a (20000 – a) × 8
Or, + = 1280
 6 × 13 25 × 7  100 100
Or, Y  +  = 25300 × 75
 5 × 12 12 × 5  Or, – 2a + 20000 × 8 = 128000
253 7 Or, 2a = 160000 – 128000
Or, Y × = 25300 ×
5 × 12 5 2a = 32000
Or, Y = 100 × 7 × 12 a = 16000
= Rs. 8400 20000 – 16000 = 4000

CAT Complete Course | 165

Visit : www.EasyEngineering.net
Visit : www.EasyEngineering.net

3. (D) Let required time = t years 6. (D) P = Rs. 2000


Now, when rate is 5% (i) Interest is compounded quarterly
9 3
P × r × t 1200 × 5 × t So, Time = = year
So, 11 = = 12 4
100 100
  r  nt 
11 = 60t …(1) C.I. = P   1 +  – 1
  100 × n 
When rate is 6%
 4×
3 
P × r × t 1600 × 6 × t   8  4
12 = = = P  1 +  – 1
100 100   100 × 4 
12 = 96t …(2)   51  3

= 2000    – 1 = Rs. 122
According to question,   50  
60t + 96t = 312 (ii) n = 2 interests compounded Half-yearly.
t = 2 years  2×
3 
  8  4
C.I. = 2000 ×   1 +  – 1

ww
4. (D) Let each installment per annum = Rs. x
First installment is paid after 1 year and hence will
  100 × 2

 
3
2


w = 2000 ×   1 +  – 1 = Rs. 121



remain with the lender for the remaining (5 – 1) = 4 1
years.   25  

lender for 3 years. .Ea


Similarly, second installment will remain will the (iii) n = 1
 3

for one year.
syE
Third installment for 2 years and fourth installment  
C.I. = 2000 ×   1 +


8 
  100 × 1
4
 – 1



ngi
Now, fifth installment remain Rs. x as such. 3
  4
= 2000 ×    – 1 = Rs. 118.
Now, Amount = A1 + A2 + A3 + A4 + A5  27
  25  
P× r×t
Now, 1 =
100
 100 + rt  nee
7. (A) Let the sum = P
  r 
n

– 1


A = P+1=P×
 100 

100 + 10 × 4 100 + 10 × 3 rin


Using formula, C.I. = P   1 +
  100 


r 
2


Or,

 100
+
100
A = x  100 + 10 × 2 100 + 10 × 1
+



 g.n
C.I.1 = P   1 +
  100 
 – 1

e
+ + 0
 100 100   r 
16000 = x × 5
x = Rs. 32000
5. (A) Interest on Rs. 1000 at 5% for 1 year
Let us suppose  1 +

In 2nd Case,

 = a
100 
C.I.1 = P (a2 – 1) = 272 t
…(1)

1000 × 5 × 1  r 
3

= = Rs. 50 C.I.2 = P   1 +  – 1
100  100  
This Rs. 50 Rohit would have lost even if the rate of 434 = P (a3 – 1) …(2)
interest had not been reduced.
From equations (1) and (2), we get
So, the loss of interest due to reduction in the rate of
a3 – 1 434
interest. =
a2 – 1 272
= 75 – 50 = Rs. 25 2
a +a+1 434
Reduction in the rate of interest = (5 – 4)% = 1% =
a+1 272
Rs. 1 is lost on every Rs. 100 a2 434
+1 =
∴ Rs. 25 is lost when amount = Rs. 2500 a+1 272
So, Rohit’s total amount in the account initially a 2 81
=
= 2500 + 1000 = Rs. 3500 a+1 136

166 | CAT Complete Course

Visit : www.EasyEngineering.net
Visit : www.EasyEngineering.net

Or, 136a2 – 81a – 81 = 0 41 t


Or, =
9 r 120 20
Or, a = =1+
8 100 41
t = years = 6 year 10 month.
r 1 6
=
100 8 10. (A) Let each equal installment be of Rs. a.
∴ r = 8%. Since, installment is paid after one year and hence it
8. (E) Since, profit for each year is on the previous includes interest for one year and the principal sum.
year’s capital. So, the present worth of the first installment
So, its similar to compound profit a 20
= = ×a
Now, Using formula, 5 21
1+
1 + 5  1 + 10   100
The second installments includes interest for two
 100   100 
C.I. = 50000   years along with its principal sum.
  1 + 100  – 1 
 15 
  ∴ Present worth of the second installment

ww = 50000 
 21 11 23 
× × –1
 20 10 20 
=

a
5 
 1 + 100 
 
2
=
400
441
×a

w = 50000 ×

=
50
4
1313
20 × 10 × 20

.Ea
× 1313 = 16412·5.
Similarly, the present worth of the third installment

=

a
=
8000
5  9261
 1 + 100 
3
×a

9. (B) Let
For First case,
Sum = P
syE 
Now, according to question –

I1 =
P× r×t
100
P×r×t ngi ∴
50440 =
20
21
×a+
400
441
×a+
8000
9261
×a

nee
A 1 = I1 + P = +P a = Rs. 18522.
100
11. (D) Let interest of each part of sum = I
 3 × 5
Or, 4800 = P  1 +  In First case,


100 

3  rin
Let sum = P1 ; t = 5 years; r = 4%
Or, 4800 = P  1 +

4800 × 20
 = P × 23
20  20 So, I. =
100
g.n
P × r × t P1 × 5 × t P1
=
100
=
5

From Second case,


P =

I2 =
23

P× r×t
…(1) In Second case,

So, I =
e
Sum = P 2 ; t = 3 years; r = 5%
P 2 × 3 × 5 3 × P2
100
=
20
t
100
In Third case,
P×5×t
A 2 = I2 + P = P + Sum = P 3 ; t = 4 years ; r = 3%
100
P3 × 4 × 3 P3 × 3
 t  So, I = =
Or, 5600 = P  1 +  …(2) 100 25
 20 
Since, P 1 + P2 + P3 = 24000
By eqns. (1) and (2)
 25 20 
20  t  ∴ I + 5 +  = 24000
5600 = 4800 ×  1 +  3 3 
23  20 
60
Or, I× = 24000
56 23  t  3
Or, × =  1 + 20 
48 20   I = 1200
7 23  t  P 1 = Rs. 6000; P2 = Rs. 8000; P3 = Rs. 10000
Or, × =  1 + 20 
6 20   Hence, the value of biggest part is Rs. 10000.

CAT Complete Course | 167

Visit : www.EasyEngineering.net
Visit : www.EasyEngineering.net

12. (E) Let capital = Rs. a, time = 1 year Now, Solving for C.I.
Now, in First case, For First year
a P× r×1
×2×1 S.I.1 =
a 1 3 100
I1 = × ×5+
5 100 100 For Second year
  a a   P×r
 a –  5 + 3   × 3 × 1 Sum = P + S.I. = P +
100
+
100  r 
 1 S.I.2 = P  1 + × r
7 
So,
920 = a 
1  100  100
 100 150 5 × 100
+ +
Now, C.I. = S.I.1 + S.I. 2
 30 + 20 + 42 
920 = a ×  P×r  r 
 3000  72 =
100
+ P 1 + × r
 100  100
a = Rs. 30000.
P×r r 
 2 + 100 

ww
13. (E) Since, a certain sum of money increased by 50%
in 3 years.
3
In 3 years, x becomes = · X
72 =

Putting the value of


pr
100 

= 30

…(2)

w .Ea
In next 6 years, X will become
2

=
3 3 3 
× · X =   .X
2 2 2 
2
100

72 = 30  2 +


r 

100 

Similarly,
syE
3  3 r
72
30
=

12


r 
 2 + 100 

In 9 years Y will become   .Y


2 
3  4
ngi 100
P×r
=
30

nee
Or, = 30
In 12 years Z will become   .Y 100
2  12
Or, P× = 30
3  3  3  4
rin
30
Now,  2  .X =  32  .Y =  32  .Z
      Again using eq. (1)

Or,
3 9
X = ·Y= ·Z
2 4
P =
g.n
900 300
12
=
4
= Rs. 75.

Or,
2 4
X:Y:Z = 1: :
3 9
X:Y:Z = 9:6:4
14. (C) Let Sum = P; Rate = r %; Time = 2 years
Now, First part for 10 years

= a 1 +
e
15. (E) Let the first part be Rs. a and second part is Rs b.

20 
 100 
10
 6  10
 = a × 5 
 
t
P× r×t
Simple Interest = S.I. = The second part for 7 years
100
 20  6 
7 7
P× r×2 = b 1 +  = b× 
Or, S.I. = = 60  100  5 
100
P×r According to question ,
Or, = 30 …(1)
100
 6  10 6  7
Since, C.I. = 72 a ×  = b × 
5  5 
S.I. = 60
b  6  3 216
C.I. – S.I. = 12 Or, =  5  = 125
60
a  
S.I. for First year = = 30
2 Since, a + b = 5115
Since, C.I. is the sum of simple interest of First year 216
∴ a+ × a = 5115
and Second year. 125

168 | CAT Complete Course

Visit : www.EasyEngineering.net
Visit : www.EasyEngineering.net

Or, a =
125
× 5115 ap  p2 3p 
100  (100) 2 100 
341 q = + +3–3

ap2  p + 300 
a = 125 × 15
1002  100 
q =
a = Rs. 1875
b = 5115 – 1875 = Rs. 3340 q (100)3
a =
p2 (p + 300)
Hence, desired result is b – a = 3340 – 1875
17. Let Sum a is divided into n parts P1 , P2 , P3 , …… Pn
= Rs. 1465. and I be the equal interest earned on each part.
16. Let Sum be Rs. a. 100 × I
Then, P1 =
r1 × t1
Principal × rate × time
S.I. = 100 × I
100 P2 =
r2 × t2
a × p × 3 3ap
S.I. = = ………………
100 100
………………
  
ww C.I. = a
p 3
  1 + 100  – 1  Pn =
100 × I
rn × tn
100 × I 100 × I 100 × I

w C.I. = a ×

C.I. = a  1 +

p3

.Ea
p  p2
3 +
3p2
2
(100) (100) 100

3p
+
3p 
– 1 


∴ P 1 : P2 : P3 … P n = : :
r1 × t1 r2 × t2 r3 × t3

:
100 × I
rn × tn
100  (100) 2 100 
syE
+ +3 1 1 1
P 1 : P2 : P3 … P n = : : ……
r1 × t1 r2 × t2 r3 × t3
ap  p2  3ap
+ 3  –
3p 1

ngi
C.I. – S.I. = + :
100  (100) 100
2
 100 rn × tn
●●●

nee
rin
g.n
e t

CAT Complete Course | 169

Visit : www.EasyEngineering.net
Visit : www.EasyEngineering.net

12 Plane Geometry

Point
Point is a basic concept in Geometry and is
hypothetical too.
● Point is represented by a fine dot made by a sharp
pencil on a sheet of paper.
● It has no width and no length. 8. A point C is said to be on the line AB.

ww
A circle with zero radii. So, area is also zero.
Straight Line
If AC + CB = AB

w
1. At least two distinct points can define a line. 9. Two distinct lines in the same plane are either
parallel or intersecting at only one point.
A
.Ea B
Here A and B are two distinct points.

B CsyE
2. A line contains infinitely many points.

A D
A, B, C, D, E … are infinite points on the line.
3. Through a given point, there pass infinitely many ngi Lines XY and AB are parallel.
Here, AB and CD are intersecting each other.

lines.
nee
Parallel
Two lines are said to be in parallel if

rin
(i) They lie in the same plane.
(ii) They do not intersect, though how far they are

g.n
extended.
Perpendicular Lines

Through point O, infinite lines pass.


e
If two lines make an angle of 90° with each other,
then these lines are said to be perpendicular.

t
4. A line has only length no width.
5. Given two distinct points, there is one and only
one line that can contain both the given points.
If A and B are two distinct points, then there is only Clearly, AB is perpendicular to CD.
one line which contain both the points. It is represented by AB ⊥ CD or CD ⊥ AB.
6. Collinear—If two or more points are said to be Line Segment
collinear if these point lie on the same line.
A line segment is a part of a line.

A, B, C and D are four distinct points contained by 1. It has definite length and two end points.
one straight line, So these four points are collinear. 2. It can not be extend on both sides.
7. Concurrent—If more than three lines are said to be 3. Every line segment has one or only one mid point.
concurrent if these lines pass through a point. AB is the part of PQ. So, AB is line segment.

170 | CAT Complete Course

Visit : www.EasyEngineering.net
Visit : www.EasyEngineering.net

Ray 5. Supplementary Angles—If sum of two angles is


equal to 180°. then these angles are said to be supple-
A ray has one end point and extends in the other mentary angles.
direction up to infinity.
It is represented by the symbol →
Angles
An angle is the bending between two rays which
have same end points but different direction. Here, angles a and b are supplementary angles.
Here, are two rays having the same end point O. a + b = 180°
● The end point O is known as vertex. Since, XOY is a straight line.
● OA and OB are called the “arms”. 6. Complementary Angles—If sum of two angles is
● θ is known as angle. “∠” it is represented by the equal to 90°, then these angles are said to be comple-
symbol and ∠AOB = ∠ BOA = θ Every angle mentary angles.
divides the whole space around itself in two parts :

ww
w
Types of Angles

.Ea
1. Acute Angles—If measurement of an angle is less
than 90°, then such angle is said to be acute angle.
Here, ∠ABC + ∠CBD = 90°
θ1 + θ2 = 90°

syE
∠A = ∠CAB = ∠BAC is acute angle. 7. Straight Angle—A straight angles has its sides
lying along a straight line. An angle whose measure is
180° is called a straight angle.

No two acute angles can be supplementary. ngi


2. Obtuse Angle—If measurement of an angle is
greater than 90°, then it is said to be obtuse angle.
nee ∠ABC = 180°
8. Reflex Angle—An angle with measure more than

rin
180° and less than 360° is called a reflex angle.

3. Right Angle—An angle whose measure is 90° is


called a right angle. The angle between two perpendicular g.n
lines is 90°.

e
180° < a < 360°
9. Adjacent Angles—Two angles are adjacent if
they share the same vertex and a common side but no
angle is inside another angle.
t
4. Vertically Opposite Angles—When two lines
interest, four angles are formed. The angles opposite to
each other are called vertically opposite angles and are Clearly, from diagram ∠ABC and ∠CBD are adja-
equal to each other. cent angles.
10. Linear Pair of Angles—Two angles are said to
form a linear pair if they have a common side and their
other two sides are opposite rays.
Here, θ1 = θ3
and θ2 = θ4
where, θ 1 and θ 3 are vertically opposite angles.
θ2 and θ 4 are vertically opposite angles.

CAT Complete Course | 171

Visit : www.EasyEngineering.net
Visit : www.EasyEngineering.net

The Sum of the measures of the angle is 180°. The Some Important Points Related to the Line
angles that form a linear pair are always adjacent.
1. Two adjacent angles are linear pair if they are
11. Angles Made by a Transversal—A line which supplementary.
intersects two or more given coplanar lines in distinct
2. The sum of all the angles rounded at a point is
point.
equal to 360°.
3. If two parallel lines are intersected by a trans-
versal, then the bisector of any pairs of alternate interior
angles are parallel.
Clearly, OA and O' A' are parallel lines.
If these are bisectors of angles ∠1 and ∠2 which are
alternate interior angles of two parallel lines l and m.
Here, AB is a transversal line. Triangles
12. Corresponding Angles—Angles that appear in
The plane figure bounded by the union of three lines,
the same relative position in each group are called corre-
which join three non-collinear points, is called a triangle.
sponding angles.

ww A triangle is denoted by the symbol ∆.


Non-collinear points are called the vertices of the
triangle. In ∆ABC. A, B and C are the vertices of the

w .Ea
That is ∠1 and ∠5 are corresponding angles.
triangle.
Line Segments—AB, BC and AC are called sides of
the triangle.

syE
∠2 and ∠6 are corresponding angles. (1) Based on Length of the Sides—There are three
types of triangles—
∠7 and ∠3 are corresponding angles.
(a) Equilateral Triangle—Equilateral triangles have
13. Alternate Interior Angles—Interior angles on
opposite sides of the transversal are called alternate
interior angles. ngi equal sides and equal angles.

Here, ∠4 and ∠6 are alternate interior angles.


∠3 and ∠5 are alternate interior angles. nee
14. Consecutive Interior Angles—The pairs of the
interior angles on the same side of the transversal are
called the pairs of consecutive interior angles. rin
Each angle measures 60°.
Here, ∠4 and ∠5 are consecutive interior angles.
∠3 and ∠6 are consecutive interior angles. g.n
AB = BC = AC
A = B = C = 60°
Transversal Across the Two Parallel Lines
When a transversal line mn cuts the parallel lines XY
and AB, then we have the following relationship between
e
(b) Isosceles Triangle—It has two equal sides and
two equal angles two equal angles are some times called
the base angles and the third angle is called the vertex
angle.
t
angles.
(1) Corresponding angles will be equal—
Here, ∠1 = ∠5
∠2 = ∠6
∠8 = ∠4 Here, XY = YZ
∠7 = ∠3 ∠YXZ = ∠XZY
(2) Alternate interior angles will be equal— (c) Scalene—Scalene triangles have all three sides of
∠4 = ∠6 different length and all angles of different measure.
∠8 = ∠5 PR ≠ RQ ≠ PQ
The sum of consecutive interior angles will be equal (2) Based on Measure of the Angles—There are
to 180° three types of triangles—
∠5 + ∠4 = 180° (a) Acute Triangle—A triangle in which all the
∠3 + ∠6 = 180° angles are acute (< 90°) is called as an acute angle triangle.

172 | CAT Complete Course

Visit : www.EasyEngineering.net
Visit : www.EasyEngineering.net

A special case of an acute angle triangle is when all Therefore, if two angles of triangle – 1 are equal to
the three acute angles are equal. two angles of triangle – 2. Then, these triangles are said
(b) Obtuse Triangle—A triangle in which one of the to be similar.
angles is obtuse is called as an obtuse triangle. The length the sides of similar triangles are in
Here, ∠ B > 90° proportion to each other.
(c) Right Angle Triangle—Right angled triangle Here, ST || QR
contains one right angle and other two angles are comple- ∴ ∆PST ~ ∆PQR
mentary. PR PQ
∴ =
In right angle, the opposite side of the right angle is PS PT
called hypotenuse. ST
=
The other two sides are called base and perpendicular. RQ
Properties of Similar Triangles
(A) Right Triangle—ABC is a right triangle with B
as right angle.

ww Now, BD is perpendicular to AC.

w AC = Hypotenuse
BC = Base

.Ea
AB = Perpendicular

triangle.
syE
Pythagorean Theorem is based on right angled

Pythagorean Theorem—It states that the square of


(a) ∆ABC ~ ∆ABD
AC
=
AB

ngi
AB AD
the length of the hypotenuse is equal to the sum of the
squares of the lengths of the legs. (AB) 2 = AC × AD
(b) ∆ABC ~ ∆BCD
(AC) 2 = (BC)2 + (AB)2
Some Important Points
nee AC
BC
=
BC
CD
1. Sum of angles of a triangle is equal to 180°.
2. Sum of the lengths of any two sides is greater than (c) rin
(BC)2 = AC × CD
∆ABD ~ ∆BCD
the length of third side.
3. Difference of any two sides is less than the third
AD
BD
=
g.n
BD
CD

e
side. (BD) 2 = AD × CD
4. Side opposite to the greatest angle is greatest and
vice-versa.
5. A triangle must have at least two acute angles.
(B) If two triangles are equiangular, their corre-
sponding sides are proportional.
In ∆ABC ~ ∆XYZ
t
6. If a, b, c denote the sides of a triangle, then If ∠A = ∠X
(i) If c 2 < a2 + b2 , triangle is acute angled. ∠B = ∠Y
(ii) If c2 = a2 + b2 , triangle is right angled. ∠C = ∠Z
(iii) If c2 > a2 + b2 , triangle is obtuse angled. AB AC BC
Then, = =
XY XZ YZ
7. Triangles on equal bases and between the same
parallel have equal area. Congruence of Two Triangles—Two triangles are
congruent if and only if one of them can be made to
8. When two sides are extended in any direction, an superpose on the others, so as to cover if exactly.
angle is formed with another side. This is called the
(A) Side-Angle-Side (SAS) Congruence Postu-
exterior angle.
late—If the two sides and the angle included in one
There are six exterior angles of a triangle. triangle are congruent to the corresponding two sides and
Similarity—Two triangles are similar if all three the angle included in another triangle then the two
pairs of corresponding angles are equal. The sum of the triangles are congruent.
three angles of a triangle is 180°. ∴ ∆ABC ≅ ∆A'B'C'

CAT Complete Course | 173

Visit : www.EasyEngineering.net
Visit : www.EasyEngineering.net

∴ ∠B = ∠B' 3. Median—A line segment from the vertex of a


AB = A'B' triangle to the mid-point of the side opposite to it is called
a median.
BC = B'C'
Thus, every triangle has three medians.
(B) Angle-Side-Angle (ASA) Congruence Postu-
late—If two angles of one triangle and the side they 4. Centroid—In a triangle, the point of intersection
include are congruent to the corresponding angles and side of three medians is called centroid. The centroid divides
of another triangle the two triangle are congruent. any median in the ratio 2 : 1.
∠B = ∠E 5. Circumcentre—The point at which the perpen-
dicular bisectors of the sides of a triangle meet is the
∠C = ∠F
circumcentre of the triangle. The circumcentre O of a
BC = EF triangle is equidistant from the three vertices.
∴ ∆ABC ≅ ∆DEF We have OA = OB = OC
(C) Angle-Angle-Side (AAS) Congruence Postu- The circle with centre O and passing through A, B
late—If any two angles and a non-included side of one and C is called the circumcircle of ∆ABC.
triangle are equal to the corresponding angles and side of
6. Angle Bisector—A line segment from the vertex
another, then the two triangles are congruent.

ww ∠A = ∠P
∠B = ∠Q
to the opposite side such that it bisects the angle at the
vertex is called as angle bisector. Thus every triangle has
three angle bisectors.

∴ w BC = QR

.Ea
∆ABC ≅ ∆PQR
(D) Side-Side-Side (SSS) Congruence Postulate—
∠1 = ∠2 =
∠A
2
7. Incentre—The point of intersection of the angle
bisectors of a triangle is called the incentre I.

syE
If all the sides of one triangle are congruent to the
corresponding sides of another triangles then the triangles
are congruent.
The perpendicular distance of I to any one side is in
radius and the circle with centre I and radius equal to in

ngi
XY = X'Y' radius is called the in circle of the triangle.
XZ = X'Z' Clearly, IA = inradius
Exercise A

YZ = Y'Z'
∆XYZ ≅ ∆X'Y'Z'
(E) Right-Angle-Hypotenuse-Side (RHS) Congru-
nee1. In a right angle triangle ABC, right angled at A, if
AD ⊥ BC, such that AD = a’, BC = b, CA = c, AB =
ence Postulate—This postulate is applicable only to right
triangles. If the hypotenuse and any one side of a right
a, then—
1 1 1 rin 1 b2

g.n
(A) 2 = 2 + 2 (B) 2 = 2 2 2
triangle are congruent to the hypotenuse and the corre- a′ a b a′ c (b – c )
sponding side of another right another then the two 1 1 1 1 1 1
(C) 2 = 2 + 2 (D) 2 = 2 + 2
triangles are congruent.
Here, PR = P'R'
∠Q = Q' = 90°
PQ = P'Q'
a′ a c

e
a′ b c
2. If ABC is an equilateral triangle and AD ⊥ BC, then
AD2 is equal to—
3
(A) CD2 (B) 2DC2
t
2
∴ ∆PQR P'Q'R'
(C) 3DC2 (D) 4DC2
1. Altitude—An altitude is the perpendicular dropped
from one vertex to the side opposite the vertex. If 3. The circum-radius of an equilateral triangle of side
measures the distance between the vertex and the line 12 cm will be—
which is the opposite side. (A) 2√
 3 cm (B) 3√
 3 cm
Since, every triangle has three vertices, it has three (C) 4√
 3 cm (D) 22 cm

altitudes.
1
Here, AD ⊥ BC 4. PQR is a triangle, PS ⊥ QR. If RS2 – SQ2 = QR2 ,
4
∴ AB = Altitude RS
then =?
2. Orthocentre—The perpendiculars drawn from the RQ
vertices to opposite sides meet at a point called ortho- 5 5
(A) (B)
center of the triangle. 4 8
Clearly, altitudes AB, CF and BE cut at a point O. 7 3
(C) (D)
So, O is the orthocenter of ∆ABC. 3 7

174 | CAT Complete Course

Visit : www.EasyEngineering.net
Visit : www.EasyEngineering.net

5. P is a point on the base BC of the equilateral triangle 16. The three sides of a triangle are 12, 20, and 24 cm
BC AP2 respectively. What is the area of the triangle ?
ABC, such that BP = . Then =—
4 AB2
(A) 32 cm2 (B) 32√
14 cm2
13 13
(A) (B) (C) 50 cm2 (D) None of these
16 6
11 3 17. The base of a right angled triangle is 5 units and its
(C) (D) hypotenuse is 13 units. What is its area ?
7 16
6. In ∆PQR, ∠A = 60°. The internal bisectors of angles (A) 30 sq. units (B) 40 sq. units
B & C meet at D. Then ∠BDC will be— (C) 45 sq. units (D) 85 sq.units
(A) 90° (B) 115° 18. The length of three sides of a triangle are given. In
(C) 60° (D) 120° which of the following cases, a triangle cannot be
formed ?
7. The value of X in the adjoining figure will be (given
that PQ || RS)— (A) 3 cm, 4 cm, 5 cm
(A) 3 (B) 3·5 (B) 10 cm, 8 cm, 6 cm
(C) 1·5 (D) 2·0 (C) 7.8 cm, 1.8 cm, 9.5 cm

ww
8. In the figure, AB is parallel to CD. OC = OD = 18
cm, AB = OB = 12 cm and CD = 6 cm. Then the
length of AC is—
(D) 1.3 cm, 1.2 cm, 0.5 cm
19. The perimeter of a right-angled triangle is 24 cm. If
the hypotenuse of the triangle is 10 cm and one of

w
(A) 14 cm
(C) 16 cm
(B) 15 cm

.Ea
(D) 17.1 cm
9. In the triangle PQR, PR = 8 cm, PR = 20 cm and BR
the other side is 8 cm, what would be the area of the
triangle ?
(A) 48 sq.cm (B) 24 sq. cm

(A) 16 cm
(C) 32 cm
syE
= 4 cm. Then the length of AQ is—
(B) 18 cm
(D) 40 cm
(C) 72 sq.cm (D) 80 sq. cm
20. From an equilateral triangle, a new triangle was
formed by increasing one side x times and reducing
10. In the figure, AB, CD, XY are parallel. If AB = 4
cm, CD = 6 cm, then XY will be equal to— ngi the other x times, i.e, if the side was 'a' initially, then
a
making one side ax and another side . Then which

nee
x
(A) 3 cm (B) 5 cm of the following is/are true ?
(C) 2·4 cm (D) 2·405 cm
1+√
5 2
11. AB is a chord of a circle which is equal to its radius.
Then the angle subtended by this chord at the minor
I. x <
2
rin II. x >
1+√
5

g.n
arc is— III. x has no upper limit IV. x has no lower limit
(A) 60° (B) 75° (A) I only (B) I and II only
(C) 1000° (D) 150°
12. In the figure AC = CE = EG = GI. If the area of strip
DEGF = 20 units and also BC II DE II FG II HI, then
area of strip BCED will be—
(C) I and IV only

e
(D) II and III only
21. PS, UT are drawn perpendicular to QR in ∆ PQR. If
PS = 8 cms and UT = 1 cm, the ratio of area of
∆PUR that of ∆URQ is—
t
(A) 7 units (B) 5 units
(A) 8 : 3 (B) 5 : 3
(C) 12 units (D) 8 units
(C) 5 : 8 (D) 8 : 5
13. In the figure, if ST is parallel to QR, then the
measure of ∠TSP will be— 22. The value of x in the given figure is—
(A) 13° (B) 43°
(C) 33° (D) 77°
14. In the diagram, L1 and L2 are parallel. The sum of the
angles α, β, χ marked in the diagram is—
(A) 180° (B) 270°
(C) 360° (D) less than 270°
15. ABC is an equilateral triangle and PQRS is a square
inscribed in it. Which of the following is true ? (A) 12 (√ 3 – 1 ) cm (B) 6 (√ 3 – 1 ) cm
(A) AR2 = RC2 (B) 2AR2 = RC2
(C) 3AR2 = 4RC2 (D) 4AR2 = 3RC2 (C) 12√
 3 cm (D) 6√
 3 cm
CAT Complete Course | 175

Visit : www.EasyEngineering.net
Visit : www.EasyEngineering.net

23. In the figure, the area of shaded portion is— (A) 45° (B) 35°
(C) 25° (D) None of these
29. If two supplementary angles differ by 50°, then the
smaller angle is—
(A) 115° (B) 75°
(C) 65° (D) None of these
30. In the adjoining figure, if AOB is a line, then the
value of x is—
(A) 2 (π3 – √ 3) cm2 (B) 4 (π3 – √ 3) cm
2

(C) 4 ( – 3√  3) cm (D) 4 ( – √  3) cm
2π 2 2π 2
3 3
24. The sum of two angles of a triangle is 100° and their
difference is 30°. Then the smallest angle is— (A) 75° (B) 55°
(A) 40° (B) 50° (C) 45° (D) 35°

ww
(C) 65° (D) 35°
Exercise B
25. In given figure, ∠B = ∠D and CP = 2 cm, AP = 4
cm, then AD/CB is equal to— 1. The sum of interior angles of a polygon is twice the

w .Ea
sum of its exterior angles. The polygon is—
(A) Octagon
(C) Hexagon
(B) Nonagon
(D) Decagon
2. A regular polygon has 54 diagonals. The number of

syE sides of the polygon is—


(A) 9
(C) 12
(B) 10
(D) 15

(A) 5/2 (B) 3 ngi


3. The sides of a Pentagon are produced to meet so as
to form a star shaped figure, as shown below. The
sum of the angles at vertices of the star is—
(C) 2/1 (D) 3/2
26. In the given figure CD is parallel to AB then ∠y is— nee
(A) 2 right angles
(C) 4 right angles
(B) 3 right angles
(D) 5 right angles

rin
4. If each interior angle of a regular polygon is 10 times
its exterior angle, the number of sides in the polygon

g.n
is—
(A) 10 (B) 12
(C) 22 (D) 24

(A) 60° (B) 100°


(A) 6r
3
√ 2r2
e
r, then the perimeter and area hexagon will be—

(B) 5r,
3
√ 3r2 t
5. If a regular hexagon is inscribed in a circle of radius

(C) 80° (D) 40° 4 2


27. In a ∆ABC, ∠A = 50°. If the internal bisectors of
angles B and C meet in O, then the measure of 3
√ 3r2 3√ 3r2
(C) 6r, (D) 5r,
∠BOC is— 2 4
6. The radius of a circle is 20 cm. The radii (in cm) of
three concentric circles drawn in such a manner that
the whole area is divided into four equal parts, are—
(A) 20√
 2, 20√
 3, 20 (B) 10√
 3/3, 10√ 2/3, 10/3
(C) 10√
 3, 10√
 2, 10 (D) 17, 14, 10
7. Two circles of radii 10 cm and 6 cm are drawn with
centers C and C´ respectively. Their transverse com-
(A) 115° (B) 120° mon tangents meet CC´ in A. The point divides CC´
(C) 125° (D) 130° in the ratio—
28. If the angles (3x – 40)° and (2x – 20)° are comple- (A) 4 : 5 internally (B) 10 : 16 internally
mentary, then the value of x is— (C) 10 : 6 externally (D) 16 : 10 externally

176 | CAT Complete Course

Visit : www.EasyEngineering.net
Visit : www.EasyEngineering.net

8. C 1 and C2 are the centres of the two circles whose (A) 25π (B) 27π
radii are 7 cm and 5 cm respectively. The direct (C) 30π (D) 45π
common tangents to the circles meet C1 C 2 in P. Then 14. In the figure AB is tangent and AD is the diameter of
P divides C1 C 2 in the ratio— the circle. If ∠BCD = 150°, ∠ABC will be—
(A) 5 : 6 internally (B) 7 : 5 internally
(C) 7 : 5 externally (D) 5 : 7 externally
9. In the figure, the length of TR will be—

(A) 40° (B) 50°


(C) 60° (D) 70°
15. In the figure, O is centre of circle, ∠ACB = 30°. Then
∠BDC will be—

(A) 6 cm (B) 9 cm

ww
(C) 5 cm (D) 8 cm
10. In the adjoining figure, AD, AE and BC are tangents
to the circle at D, E, F respectively. Then—

w .Ea
(A) 30°
(C) 60°
(B) 45°
(D) 75°
16. A circle has two parallel chords of length 6 cm and 8
cm. If the chords are 1 cm apart and are on the same

syE side of the diameter- parallel to them, then the


diameter of the circle is—
(A) 5 cm (B) 6 cm
(A) AD = AB + BC + CA
(B) 2 AD = AB + BC + CA ngi (C) 10 cm (D) 12 cm
17. In the figure AB is the diameter of the circle with
(C) 3 AD = AB + BC + CA
(D) 4 AD = AB + BC + CA
nee centre O. CD cuts AB at E such that OE = EB. If CE
= 9 cm and ED = 3 cm, then the diameter of the
circle will be—

rin
11. The sum of the interior angles of a polygon is 7 times
the sum of its exterior angles. The number of sides in
the polygon is—
(A) 14
(C) 18
(B) 16
(D) 20 g.n
12. In the adjoining figure the radius of the smallest
circle (r) is— (A) 8 cm
(C) 11 cm
e
(B) 10 cm
(D) 12 cm t
18. In the figure, find X—

(A) 3
√ 3 cm (B) 3 cm
(C) 2 cm (D) None of these
13. In the figure, if AB = 4 cm, BD = 6
√ 3 cm, then the
area of shaded region will be—
(A) 120° (B) 135°
(C) 140° (D) 145°
19. In the figure, DC is the diameter of circle and AB is
a chord parallel to DC. If AB = AD, then the value of
∠BAD will be—

CAT Complete Course | 177

Visit : www.EasyEngineering.net
Visit : www.EasyEngineering.net

(A) 100° (B) 120°


(A) 90°, 105°,110° (B) 70°, 95°, 105°
(C) 90° (D) 130°
(C) 70°, 120°, 140° (D) 75°, 90°, 85°
20. In the figure, the value of x is—
26. Two parallel chords of a circle are 6 cm and 8 cm in
length. If the diameter of the circle is 10 cm, then
the distance between the chords is—
(A) 1 cm (B) 2 cm
(C) 3 cm (D) 3.5 cm
27. In the adjoining figure, the measure of ∠BCD is—

ww
(A) 15° (B) 65°

w
(C) 70° (D) 85°

.Ea
21. In the above question the value of y is—
(A) 15°
(C) 70°
(B) 65°
(D) 85°

syE
22. The angle of a hexagon are x, 2x + 4, 2x + 4, x – 10,
x – 10, and 2x + 21, the value of x is—
(A) 45°
(C) 90°
(B) 70°
(D) 120°

ngi
(A) 45° (B) 60°
28. In the adjoining figure. Find the value of angle a and
(C) 79° (D) 85° b—
23. In the adjoining figure, ∠ABC is—

nee
rin
g.n
e
(A) 110° and 70° (B) 120° and 60°
(A) 40°
(C) 70°
(B) 60°
(D) 120°
24. In the figure O is the center of the sector OAB,
∠BOA = 90°. A semicircle ABC is constructed with
(C) 85° and 95° (D) 80° and 100°

Answers with Hints


Exercise A
t
AB as diameter. If OA = 1 m, then the area of the 1 1 a′b
1. (B) Area of triangle = ac = a′b or a =
shaded part will be— 2 2 c
Now, a2 + c2 = b2
 a′b  2 2
⇒  c  + c = b2
 
b2
⇒ a′2 = 2 (b2 – c2)
c
1 b2
⇒ 2 = 2 2 2
a′ c (b – c )
(A) πm2 (B)
1 2
m (3) × BC

2 2. (C) We have AD =
2
(C) 300 m2 (D) 5000 cm2  3 × DC × 2 = √

=  3DC
25. In the adjoining figure, the respective values of a, b 2
and c are— or AD2 = 3 DC2

178 | CAT Complete Course

Visit : www.EasyEngineering.net
Visit : www.EasyEngineering.net

a 8. (A) =
p p1 (since OC = OD)
3. (C) Circum – radius =
3
√ =
q q1 (OB = AB)
where a is the side of equilateral triangle. ⇒ Since, AB|| CD)
⇒ =
q1 p
12 12 × √3
⇒ = ⇒ =
q1 p1
3
√  3 × √ 3
√ = q=q
= 4√  3 cm ⇒ OAB ≅ ≈ CDO (A.A.A.)
1 OA CD
4. (B) RS – SQ2 = QR2
2 ⇒ =
4 AB DO
1 OA 6
⇒ (PR – PS ) – (PQ – PS ) = QR2
2 2 2 2 ⇒ =
4 12 18
1 6 × 12
⇒ PR2 – PQ2 = QR2 ……(i) ⇒ OA = = 4 cm.
4 18
PR – PQ = RS × RQ – QS × QR
2 2
⇒ AC = OC – OA
= QR [RS – QS] …….(ii) = (18 – 4) = 14 cm.
From equations (i) and (ii) 9. (C) Triangle PAB and PRQ
1 ∠P = ∠P [Common]
⇒ QR[RS – QS] = QR2

ww
⇒ RS – (RQ – RS) – QR
1
4
4

So,
∠PAB = ∠PRQ [Given]
∠PBA = ∠PQR [Property]
∆PAB ≅ ∆PRQ [by A.A.A. property]


w
⇒ RS + RS – RQ = QR

.Ea
2RS = RQ
RS
1
4
5
4
5


PQ
PR
PQ
20
=

=
PB
PA
PR – BR
8

5.

(A)
RQ
=

syE
8
AB = BC = AC (given)
1


PQ
20
PQ =
=
20 – 4
8
20 × 16
= 40 cm.
BP = BC
AD is perpendicular to BC.
4
ngi ⇒

AQ = PQ – PA
8

= (40 – 8) = 32 cm.

nee
1 1
⇒ AD = DC = BC or AB 10. (C) ∆ABC and ADC are similar triangles
2 2
AB CA
⇒ PD = BD – BP ⇒ =
XY CY
1
= BC – BC
2
1
1
4
1
⇒ XY =
rin AB × CY 4CY
CA
=
CA
……(i)
= BC or AB
4
AP2 = AD2 + PD2
4 ⇒
DC
XY
=
g.n
AC
AY
6AY
(Pythagorean theorem)
= (AB2 – BD2 ) + PD2
= AB2 – AB2 +
AB2
1
4
1
16
AB2
⇒ XY =

4CY
CA
=
AC
From equations (i) and (ii)
e
6AY
AC
……(ii)

t
= [16 – 4 + 1] CY 6 3
16 ⇒ = =
AY 4 2
13AB2
= CY + AY 3
16 ⇒ = +1
AY 2
AP2 13
⇒ = 5
AB2 16 =
2
1
6. (D) Angle ∠BDC = 90° + ∠A 6AY 6 × 2
2 or XY = =
1 AC 5
= 90° + (60°) 12
2 ⇒ XY =
= 120°. 5
7. (C) The two triangles are similar. = 2·4 cm
3 X
Therefore, = 11. (D) Given that OA = OB = AB
4 2
4X = 6 ∴ ∠AOB = 60°
6 Now, extend AO so that it meets the circle at the
or X = = 1·5.
4 point C. Join BC,

CAT Complete Course | 179

Visit : www.EasyEngineering.net
Visit : www.EasyEngineering.net

1 12 + 20 + 24 56
∴ ∠ABC = ∠AOB 16. (B) s = =
2 2 2
1 = 28
= × 60° = 30°
2 Here, a = 10 cm,
Again, since APBC is a cyclic quadrilateral, b = 14 cm
and c = 16 cm
So, ∠APB = (180° – 30°) = 150°
12. (C) If BC = X Area = √ 
s(s – a) (s – b) (s – c)
Then, DE = 2X, = √
28 × 16 × 8 × 4
FG = 3X, = √
4 ×7×4×4×8×4
HI = 4X, [By similar triangles] = 32√ 14 cm2
Area of DEGF = 20 units 17. (A) BC2 = AC2 – AB2
1 = (13) 2 – (5) 2
= (2X + 3X) × h = 20 2
BC = 144
2
⇒ BC = 12 units.
[∴DEFG is a trapezium and h = Distance between 1
two parallel lines] Area of ∆ABC = × AB × BC
2

ww
⇒ Xh =
20 × 2

1
5
⇒ Area of BCDE = (X + 2X) × h
=8 1
= × 5 × 12
2
= 30 sq.units.

w .Ea
2
1 3
= × 3X × h = × (Xh)
2
3
2
18.

19.
(C) The sum of any two sides should be greater than
the third side.
Here, (1·8 + 9·5) cm = 7·8 cm
(B) Perimeter = 24 cm
= × 8 = 12 units.

13. (D)

syE
2
ST || QR
∠Q = ∠S,
Then third side is = 24 – (10 + 8)
1
Area of the triangle = × 8 × 6
2
∠R = ∠T
∠TPY = ∠TSP = 77° ngi 20.
= 24 sq. cm.
(A) Here, the property of the triangle i.e., sum of the

nee
two sides of a triangle must be greater than third side
14. (C) L3 is drawn parallel to L1 and L 2 . is used.
Let β = α1 + χ1 a
(i) ax < a +
α + α1 = 18°

rin
Now, x
and χ + χ 1 = 18° ⇒ x < 1+
1
∴ α + α1 + χ + χ1 = 36° x
and (α 1 + χ1) = β
Hence, α + β + χ = 360°

Therefore,
x2 – x – 1 < 0

x < g.n1+√ 5 ….. (i)

15.

Required answer = 360°
(D) Since,

Since,
SR || BC
AS = AR
∠A = 60°,
(ii)


ax + a >

x+1 >
e
a
x
1
2

t
x
∴ ∆ASR is equilateral. ⇒ x2 + x – 1 > 0
Let AR = X, –1+ √5
Therefore, x > ……(ii)
∴ PQ = SR = X 2
In ∆RQC ∠RQC, Combining equation (i) and equation (ii)
∠C = 60° 1+√  5 > x > – 1 + √ 5
RQ X 2 2
sin 60° = = Therefore, only I is the true.
RC Y
21. (B) Area of ∆PUR = Area of ∆PQR
3 = X
√ – Area of ∆UQR
or
2 Y 1 1
= QR × PS – QR × UT
or  3Y = 2X
√ 2 2
1
Squaring both side, = QR[8 – 3]
2
3Y2 = 4X2; 5
= QR
3RC2 = 4AR2 . 2

180 | CAT Complete Course

Visit : www.EasyEngineering.net
Visit : www.EasyEngineering.net

1 and ∠B = ∠D
and Area of ∆URQ = 3 × × QR
2 and ∠APD = ∠CPB
Therefore, Thus, ∆ PCB and ∆ APD are similar
5 3
∆PUR : ∆UQR = QR : . QR Hence,
CP
=
CB
2 2 PA AD
= 5:3 2 1
22. (B) AB = 12 sin 30° = =
4 2
1
⇒ AB = 12 × = 6 cm AD 2
2 ∴ =
CB 1
AB ∴
tan 45° = 26. (A) CD // AD
BD
∴ ∠ABC = ∠ DCE
⇒ AB = BD = 6 cm.
i.e., 3x° = y
⇒ (12) 2 = 62 + (6 + x)2
Now, 3x + 2x + 4x = 180°
⇒ 12 – 62 = (6 + x)2
2
9x = 180°
⇒ 144 – 36 = (6 + x)2
x = 20°
⇒ 108 = 36 + 12x + x2
∴ y = 3x

ww
2
x + 12x – 72 = 0
y = 3 × 20° = 60°
– 12 ± √
144 + 4 × 72
x2 = 27. (A) ∆ ABC
2

w x =

=
– 12 ± √

.Ea
– 12 ± √
2



144 + 288
2
432 – 12 ± 12√
=
2
3

Taking +ve sign


– 12 + 12
√3 syE ∠A + ∠B + ∠C = 180
∠B + ∠C = 180 – 50

⇒ 6√
3–6
⇒ 6 √ (
2

3–1 ) ngi In ∆ BOC


= 130

– ve is not taken because distance can be negative.


23. (D) ∴ ∠AOD =
(360° – 2 × 120°) nee ∠B
2
+ ∠O +
∠C
2
= 180
130

= 60°
2 ⇒

rin
∠O = 180 –

∠O = 115
2

So, area of shaded portion


=
θ
360°
πr2 –
r sin θ
2

2
28. (D) Given
angles is
angles
90°. g.n
are complementary i.e., sum of two

=
60°
360°

× π × 16 –
16 ×
2
√ 3
2
⇒ 3x – 40° + 2x – 20° – 90°


e
5x = 90° + 60°
= 150° t
= – 4
√3 x = 30° .
3 29. (C) Note—Two angles are said to be supplementary
= 4 ( 2π
3
–√)3 cm2 if their sum is 180°.
⇒ x° – (180 – x)° = 50°
24. (D) In ∆ABC, given that
∠A + ∠B = 100° …(i) ⇒ 2x° = 50° + 180°
∠A – ∠B = 30° …(ii) ⇒ 2x = 230°
2∠A = 130° 230°
⇒ x° = = 115°
2
130°
∠A = ∴ Smaller angle = 180° – 115° = 65°.
2
= 65° 30. (B) 2x + 30° + x – 15° = 180°
∠B = 35° ⇒ 3x + 15° = 180°
25. (C) ∴ AP × PB = DP × PC ⇒ 3x = 180° – 15°
AP PC ⇒ 3x = 165°
=
PD PB ⇒ x = 55°

CAT Complete Course | 181

Visit : www.EasyEngineering.net
Visit : www.EasyEngineering.net

9. (B) We have, Triangle TQR similar to triangle PTR


Exercise B TQ PT QR 6
1. (C) We have, Sum of exterior of a polygon = 360°. or = or TR = PT × = 3 × = 9 cm
QR TR TQ 2
Hence, sum of interior angles = 720° which is true for 10. (B) Knowing that the tangents drawn to a circle from
a hexagon. a point outside it are equal, one gets
2. (C) From the formula : AD = AE
= n C 2 – n = 54 BD = BF
∠n CF = CE
= – n = 54
∠2∠n – 2 ∴ AD = AB + BD = AB + BF
n (n – 1) ∴ Also, AD = AE = AC + CE = AC + CF
= – n = 54
2 ∴ 2AD = AB + AC + BF + CF
⇒ n2 – 3n – 108 = 0 = AB + AC + BC
⇒ n – 12n + 9n – 108 = 0
2
11. (B) Let the side of a polygon is n.
⇒ n(n – 12) + 9(n – 12) = 0 According to question,
⇒ n = 12 ⇒ 2(n – 2) × 90° = 7 × 360°
or n = –9 7 × 360
n–2 = = 14

ww
– ve is not possible
⇒ there are 12 sides.
3. (A) Formula for the sum of angles of the vertices of a


n – 2 = 14
n = 16
180

w
star of n sided polygon = (n – 4) × 180°. For pentagon
we have (5 – 4) × 180° = 180° = 2 right angles.

.Ea
4. (C) Each exterior angle of n sided polygon =
360
n
,
∴ The number of sides in the polygon = 16
12. (C) We have in triangle OBD

or
(r + 3)2 = 32 + (6 – r)2
r = 2 cm

and interior angle =  180 –
 syE
360 
n 
·
 360 
13. (B) Angle ADC is a right angle (angle in a semi-
circle).
So, BD2 = AB × BC
Hence, as per the condition given  180 –

360
 n 

ngi or
or
36 × 3 = 4 × BC
BC =
36 × 3
4
= 27 cm

⇒ 180 –
360
n
= 10 ×

= 10 ×
n
360
n
nee Now, Shaded Area = Area of bigger semi-circle

  2
– Area of smaller semi-circle
1  27 
2


11 × 360
n
= 180
1 31

1  961
1

rin
⇒ π   – π (2)2 – π  
2 2  2
729 
2 2 

⇒ n =
11 × 360
180
= 22
5. (C) We have, Radius of circle = side of hexagon = r.
⇒ π
2  4
–4–
4 
g.n
Hence, Perimeter = 6r

Area =
3√
 3 × Side2 = 3√ 3 r2
2 2


1
8
π [961 – 745]
π
8
[216] =
=
e
216π
8
27π
t
6. (C) Total Area = π (20) 2 = 400 π
Each area = 100 π 14. (C) ∠ACD = 180° – 150° = 30°
Let R1 be the radius of the smallest circle. ∠ ABC = 180° – (90° + 30°)
Then, πR1 2 = 100 π So, ∠ ABC = 180° – 120° = 60°
⇒ R 1 = 10 cm 15. (C) ∠ACB = 30°,
π(R2 2 – R 1 2 ) = 100 π ∠ABC = 90° (Angle in a semi-circle)
or R 2 = 10√ 2 cm ⇒ ∠BAC = 60°
π (R 3 2 – R 2 2) = 100 π and ∠BDC = 60° = ∠BAC
or R 3 = 10√  3 cm (Angle in a same segment)
Short cut : From options check the ratio of areas. 16. (C) We have
7. (C) Transverse common tangent divides the line
joining the centres of two circles in the ratio of their 
√ (r2 – 32) – √

(r2 – 42) = 1
radii internally i.e., 10 : 6. or r = 5 cm
8. (B) The direct common tangent divides the line C1 C 2 (on trying different options)
in the ratio of their radii i.e., 7 : 5 externally. or Diameter = 2 × 5 = 10 cm.

182 | CAT Complete Course

Visit : www.EasyEngineering.net
Visit : www.EasyEngineering.net

17. (D) Let a be the radius of the circle. 23. (D) The measure of arc
Then, OE = EB CA = 360° – 120° = 240°
and OE + EB = a Now, measure of arc CA
a = 2∠ABC = 240°
⇒ OE = EB = ∴ ∠ABC = 120°
2
⇒ AE × EB = CE × ED 24. (B) Area of segment
 a  a 1 1
ABD = πr2 – (OA) (OB)
⇒  a + 2  × 2 = 9 × 3 4 2
π 1
3a a = –
⇒ × = 9×3 4 2
2 2 Area of the shaded part
⇒ 3a2 = 4 × 9 × 3
1 π 1 
⇒ a2 = 4 × 9 = 36 = π (AB/2)2 –  – 
⇒ a = 6 cm
2 4 2 
 2
1  (12 + 12)   π 1 


So, Diameter = 2 × 6 = 12 cm
18. (A) ∠ACB and ∠ABC are equal. Since, two tangents = π   –  4 – 2 
2 2

ww
from an external point are equal and so the triangle is
equilateral because
∠A = 60°, 25. (C)
π π 1 1
= – + = m2
4 4 2 2
a = 70°

So,w ∠ACB = ∠ABC


= 60° (each)

.Ea
∠x = ∠A + ∠ACB
= 60° + 60° = 120°
Also,

a + b = 180°
b = 180° – 70° = 110°
∠C = 2∠OTS
∠C = 2 × 70° = 140°
19. (B)


syE
a = 90° (angle in semi-circle)
AB = AD (given)
b = d (base angles, loss triangle)
26. (A) OA = OB = 5 cm
AN = 4 cm

ngi
and BM = 3 cm
X = X1 (angle in same segment)
ON2 = OA2 – AN2
B = X1 (alt. angles, AB || CD)
ON2 = 52 – 42 = 25 – 16
⇒ b = d = X = X1
a + X + b + d = 180°
90° + 3X = 180°, X = 30° nee or
ON2 = 9
ON = 3 cm
In ∆OBM, by Pythagorean theorem
∠BAD = a + X
= 90° + 30° = 120°
rin
OM2 = OB2 – BM 2
OM2 = 52 – 32 = 25 – 9
20. (A)


c = 80° (angle in alt. seg.)
z + 20° + 45° = c (ext. angle of triangle) OM2 = 16,
OM = 4 g.n
e
z + 20° + 45° = 80°
∴ Distance between chords

21.
z = 15°,
x = z = 15°(angle in same seg.)
80° + y + z = 180° (adj. angle on st. line)
(B) 80° + y + 15° + 20° = 180°
27. (B)
= OM – ON
= 4 – 3 = 1 cm
∠ADB = 2∠AEB
t
⇒ ∠AEB = 70°
y = 65°
So, ∠AEB + ∠ACB = 180°
22. (C) The sum of angles of a regular hexagon is
∠ACB = 180° – 70° = 110°
= 720°
and ∠ACB + ∠BCD = 180°
⇒ x + 2x + 4 + 2x + 4 + x – 19 + x – 10 + 2x + 21
∠BCD = 180° – 110° = 70°
= 720°
28. (C) ∠BOD = 2a
⇒ 9x + 9 = 720 170
⇒ 9x = 720 – 9 ⇒ a = = 85°
2
9x = 711 and b = 180° – 85°
x = 79° = 95°
●●●

CAT Complete Course | 183

Visit : www.EasyEngineering.net
Visit : www.EasyEngineering.net

13 Mensuration
Mensuration deals with the measurements of lengths, AD = 
√ AC2 – CD2
area, volumes, surfaces etc.



a  2 √
(A) TRIANGLE = a2 –  2  = 23 · a
 
Perimeter—Perimeter of a plane figure is the mea-
sure of the length of its boundary. Perimeter = 3a
(i) Find the area of triangle when sides are given. 3· a

Height =

ww
Let a, b, c be the sides of a triangle whose opposite
vertices are A, B, C respectively.
If S be the semi perimeter of the ∆ABC
1
2

Area = × Base × Height


2
Now,
w 2S = a + b + c

S =
.Ea
a+b+c
2
…(1)
A =
1
2
×a×
3· a

2
 3 · a2

syE
Area is given by A =
4
Area = √ 
S (S – a)(S – b)(S – c) Illustration 1.
It is known as Hero’s Formula.

ngi
If a, b, c be the sides of a triangle and S the semi-
1 perimeter of the triangle. Find the area of the triangle in
Area = × Base × Height
2 terms of a, b, c and S.
(ii) Right Angled Triangle—Here, ABC is right
angled triangle of height h and base b.
1
nee
Solution :
We draw AD ⊥ BC.
Now, Area = × Base × Height
2
Perimeter = b + h + p
Now,
rin
2S = a + b + c
a+b+c

g.n
S = …(1)
2
1
Area = × p × b If CD = X
2
(iii) Isosceles right angled triangle—Let ABC is a
isosceles right angled triangle where
AB = BC

If
BD = a – X
AD = h
Now, in right angled ∆ACD
e t
Now, if BC = AB = X
b2 = h2 + X 2 …(2)
Then, AC be the hypotenuse AC = 
√ 2.X Again, in right angled ∆ABD
1
Area = × Base × Height Now, h2 = c2 – (a – X)2 …(3)
2
From equation (2) and equation (3), we get
1
Area = × X2
2 b2 – X2 = c2 – (a – X)2
(iv) Equilateral Triangle—Let ABC be an equi- Or, b2 – X2 = c2 – a2 – X2 + 2aX
lateral triangle of each side a. Or, 2aX = b2 – c2 + a2
We draw a perpendicular AD to the side BC. a2 + b2 – c2
X =
a 2a
Now, CD = BD =
2 ∴ h2 = b2 – X 2
Now, Applying Phythagoras  a2 + b2 – c2  2
In right angled triangle ACD = b2 –  
 2a 
184 | CAT Complete Course

Visit : www.EasyEngineering.net
Visit : www.EasyEngineering.net

1 The length of any of the altitude of the triangle


Now, Area of ∆ABC = × Base × Height
2 Area
=
1 Base
= ×a×h
2  3 × 25

4  3 × 5 metre2



 a2 + b2 – c2  2
1 = =
= ×a× b2 –   5 4
2  2a  Illustration 4.
1 1 Find the lengths of the three sides of a triangle 20
= ×a× × √
4a2 b2 – (a2 + b2 – c2)2
2 2a metre, 51 metre, 37 metre. Find the area of the triangle
1 and hence find the length of the perpendicular on the
= × √
(2ab) 2 – (a2 + b2 – c2)2 greatest side from the opposite vertex.
4
1 Solution :
= × √ 
(2ab + a2 + b2 – c2)(2ab – a2 – b2 + c2)
4 Let S be the semi-perimeter.
1 Then, a = 20 metre, b = 51 metre, c = 37 metre
= × √ 
{(a + b)2 – c2}{c2 – (a – b) 2 }
4 a + b + c 20 + 51 + 37
S = =

ww√
Area of ABC

= ×
1 (a + b + c)(a + b – c)
=
108
2
2
= 54 metre
2

w.E 4
1
= ×
4
(c + a – b)(c – a + b)

√
2S.(2S – 2c)(2S – 2b)(2S – 2a)
Now, Area of the triangle is given by
= √
= √

S (S – a)(S – b)(S – c)

asy
= √
Illustration 2.
S (S – a)(S – b)(S – c) .
= √
54 (54 – 20)(54 – 51)(54 – 37)

54 × 34 × 3 × 17

metres.
Solution :
E
Find the area of triangle whose sides are 5, 12 and 13

ngi
= √
6 × 9 × 6 × 4 × 3 × 17
= 6 × 3 × 2 ×
Greatest side = 51 metre
√51

We see that 52 + 12 2 = 25 + 144 = 169 = 132


So, given triangle is right angled triangle whose nee ∴ Height =
Area
Side
6 × 3 × 2 ×√ 51 = 36 metre.
hypotenuse = 13 metre
1
Now, Area of ∆ABC = × Base × Height
=

rin 51 51

g.n
2 Illustration 5.
1 The sides of a triangle are in the proportion 3 : 4 : 5
= × 5 × 12 = 30 metre2 . and its perimeter if 48 metres. Find the area of the

e
2
triangle. Is it a right angle triangle ?
Illustration 3.
The perimeter of an equilateral triangle is 15 metre.
Find its area and the length of any of its altitudes.
Solution :
Solution :
Let sides are 3X, 4X and 5X.
Now, perimeter = 3X + 4X + 5X
t
48 = 12 X
Let a be the side of equilateral triangle. 48
Or, X = =4
Since, in equilateral triangle all sides are equal. 12
So, perimeter = 3a ∴ Sides are 12 metre, 16 metre, 20 metre.
15 = 3a Area of triangle = √

S (S – a)(S – b)(S – c)
a = 5 metre Since S = Semi perimeter so S = 24
Area of triangle
 3 × (Side)2

Area of equilateral triangle =
4 = √ 
24 (24 – 12)(24 – 16)(24 – 20)
= 12 × 8 = 96 metre2
 3 × (5)2

= Now, 122 + 16 2 = 144 + 196 = 340
4
202 = 400
 3 × 25 metre2
√ Clearly, Given triangle is not right angle triangle.
=
4
CAT Complete Course | 185

Visit : www.EasyEngineering.net
Visit : www.EasyEngineering.net

Illustration 6.  3 × a2 = 24 × a

Or,
One side of a triangular lawn is 50 metre and the 4 2
other two sides are equal. If the cost of paying the lawn is 48
Rs. 3500 at 2·5/square metre. Find the length of each Or, a =
side. 3

Solution : Or, a = 16√
 3 metre
Now, we draw a perpendicular AD to the base BC.  3 × a2 × √ 3 × (16 √ 3)2

Area of ∆ABC =
Now, In right angled triangle ACD 4 4
h = 
√ a2 – 252  3 × 16 × 16 × 3

=
1 4
Now, Area of triangle = × 50 × h
2
= 168√
 3 metre2.
1
= × 50 ×  √
a2 – 252 …(1) Illustration 8.
2
Since, Rate of cost = 2·5 / metre2 A ladder 25 metre long is placed against a wall with
its foot 7 metre from the wall. How far should the foot be

ww Total cost = 3500

So, Area of the triangle =


3500
2·5
…(2)
drawn out so that the top of the ladder may come down
by half the distance that the foot is drawn out ?

w
Now, From equation (i) and equation (ii), we get

35000 2
3500
2·5
.Ea 1
= × 50 × 
2
√
a2 – 252
Solution :
Let X1 Y1 be the ladder of length l.
a = distance of point X 1 from foot.

syE
= √
Or, × a2 – 252 X2Y2 be the new position of the ladder.
25 50
Now, In X1 OY1
Or, 56 = √

a2 – 252
(X 1 Y1)2 = (OX1 )2 + (OY1)2
Or,
Or,
2 2
56 + 25 = a 2

3136 + 625 = a2 ngi Or, l 2 = h2 + 7 2 …(i)

nee
In X 2 OY2
a = 
√ 3761 metre.
(X 2 Y2)2 = (OX2 )2 + (OY2)2
Illustration 7.
 a 2
In the adjoining, equilateral triangle ABC, three per-
pendiculars OE, OF, OG are drawn from point ‘O’ to the
Or,
rin
l2 =  h – 2  + (7 + a)2
 
…(ii)

g.n
three sides. If the perpendicular measure 6 metre, 8 metre
and 10 metre respectively. Find the area of the triangle. Given l = 25
Solution : h2 = 252 – 72
Now, joining the points B and O and O and C.
We get BOC as a triangle whose heights OE and
base is BC.
e
= (25 + 7) (25 – 7)
= 32 × 18
h = 24 metre
t
1
Now, Area of ∆BOC = × BC × OE
2  2
Now, (25) 2 =  24 – a2  + (7 + a)2
1
= ×a×6  
2
a2
Similarly, Area of ∆ACO Or, 252 – 242 = – 24a + + 49 + 14a + a2
4
1
= ×a×8 5a2
2 Or, 49 = – 10a + + 49
1 4
Area of ∆AOB = × a × 10
2 a 
Now, Area of ∆ABC = Area of ∆AOB + Area of Or, 5a 4 – 2  = 0
 
∆BOC + Area of ∆AOC
Or, a = 0 not possible
 3 × a2 = 1 × 6 × a + 1 × 8 × a

4 2 2 Or, a = 8 metre
1 Hence, the foot of the ladder should be drawn out by
+ × 10 × a
2 8 metre.

186 | CAT Complete Course

Visit : www.EasyEngineering.net
Visit : www.EasyEngineering.net

Illustration 9. Then, a2 + b2 = 52 …(i)


If the perimeter of a right angled isosceles triangle is Since, Perimeter = 2 (a + b)
(2√ 2 + 2) metre. Find the hypotenuse. 14 = 2 (a + b)
Solution : a+b = 7 …(ii)
a2 + a2 = b2 Now, putting the value of b in equation (i), we get
2a2 = b2 a2 + (7 – a)2 = 52
Or, a2 + 49 – 14a + a2 = 25
b = √
 2a …(i)
Or, 2a2 – 14a + 49 – 25 = 0
Now, Perimeter = OA + OB + AB
Or, 2a2 – 14a + 24 = 0
2√
 2 + 2 = a + a + √ 2a Or, a2 – 7a + 12 = 0
Or, 2 (√ 2 + 1) = 2a + √
 2a (a – 3)(a – 4) = 0
Or, 2 (√ 2 + 1) =  2 (√
√  2 + 1)a a = 3, 4
∴ b = 4, 3
2
Or, a = =√  2 metre

ww √ 2
b = √ 2 × √ 2 = 2 metre
Now, Area of the rectangle = a × b

Illustration 11.
= 12 cm2 .

So,w b = 2 metre

.Ea
Hypotenuse = 2 metre

(B) AREA AND PERIMETER OF


The length of a rectangle exceeds its width by 20
meter and the area of the rectangle is 300 metre2 . Find the
dimensions of the rectangle.

RECTANGLE AND A SQUARE


syE
(a) Rectangle—Let l be the length of the rectangle
and b is its breadth.
Solution :
Let the breadth of the given rectangle be a metre.
Now, its length = (a + 20) metre
(i) Perimeter = l + b + l + b
= 2 (l + b) ngi Area = 300 metre2
a (a + 20) = 300
(ii) Area of rectangle ABCD = Area of ∆ACD
+ Area of ∆ABC nee Or, a2 + 20a – 300 = 0
Or, a2 + 30a – 10a – 300 = 0
Area of rectangle ABCD
1
= ×l×b+ ×l×b
1 Or,
rin
Or, a (a + 30) – 10 (a + 30) = 0
(a – 10) (a + 30) = 0

= l×b
2 2
If, g.n a = 10, – 30
a = 10 metre
Area of rectangle = Length × Breadth
(b) Square—Since, square has equal sides.
Let each side is a.
Then,
Illustration 12.
e
length = 30 metre

A table cover 4 metre × 3 metre, is spread on a


meeting table. If 35 cm of the table cover is hanging all
t
(i) Perimeter = 4a
around the table. Find the cost of polishing the table top
(ii) Area—Area of square ABCD = Area of ∆ACD at Rs. 2·05 per square metre.
+ Area of ∆ABC Solution :
1 1
= × AD × CD + × AB × BC ABCD is our required area of table.
2 2
Now, length of table = 4 – 2 × 0·35
1 1
= ×a×a+ ×a×a = 4 – 0·7 = 3·3 metre
2 2
Area of square = a 2 Breadth of the table = 3 – 2 × 0·35
= 3 – 7 = 2·3 metre
Illustration 10.
Now, Area of the table = 3·3 × 2·3
If the diagonal of a rectangle is 5 cm long and the
perimeter of the rectangle is 14 cm. Find the area of the = 7·59 metre2
rectangle. The cost of polishing the table
Solution : = 7·59 × 2·05
Let a and b be the length and breadth of the rectangle. = Rs. 15·5595

CAT Complete Course | 187

Visit : www.EasyEngineering.net
Visit : www.EasyEngineering.net

Illustration 13. Now, Cost of gravelling the roads


There is a square field whose side is 44 metre a = Rate × Area
square flower bed is prepared in its centre leaving a = 2 × 13 = Rs. 26
gravel path all round the flower bed. The total cost of
Illustration 15.
11
laying the flower bed and gravelling the path at Rs. A rectangular courtyard, 16 metre and 18 metre
4
3 2
broad, is to be paved exactly with square tiles, all of the
and Rs. /metre respectively is Rs. 4904. Find the width
2 same size. Find the largest size of such a tile and the
of the gravel path. number of tiles required to pave it.
Solution : Solution :
Let the width of the gravel path = a metre Side of the largest possible square tile
Now, Side of square flower bed = (44 – 2a) metre = H.C.F. of length and width of the hall
Area of the flower bed = (44 – 2a)2 metre2 = 2
Area of square field = 44 × 44 = 1936 metre2 Area of square tile = 22 = 4
Now, Area of the gravel path Area of Courtyard
Number of square tile =

ww = Area of the field – Area of the flower bed


= 1936 – (44 – 2X)2 =
Area of square tile
16 × 18
4
= 72.

w = 1936 – (1936 – 176X + 4X2)

.Ea
= (176 X – 4X2 ) metre 2
Cost of lying the flower bed
11
Illustration 16.
A square room is surrounded by a verandah of width
‘w’. If the arc of the verandah is A0. Find the area of the

syE
room.
= (44 – 2X)2 ×
4 Solution :
= 11 (22 – X) 2
Let the side of the room = a metre
Cost of gravelling the path

= (176X – 4X2 ) ×
3
2 ngi Area of the room = a2 metre2
Area of room + Vernadah = (a + 2w)2 metre2

According to question,
= 6 (44X – X2 )
nee Area of verandah = A0 = (a + 2w)2 – a2
A0 = (2a + 2w) × 2w

11 (22 – X)2 + 6 (44 – X2) = 4904


rin A0 = 4aw + 4w 2

a = 0
A – Aw2

g.n
Or, 5X2 – 220X + 5324 = 4908 4w
Or, X2 – 44X + 84 = 0  A0 – Aw2  2
∴ Area of the room = a2 =   metre2
 4w 


Or, X = 2, 42
But X ≠ 42 since the side of the square is 44 metre.
X = 2
Hence, the width of the travel path is 2 metre.
Illustration 17.
e
A ground of length 120 metre and breadth 100 metre
has pavements of uniform width 2·5 metre all around it,
t
Illustration 14. both on its outside and inside. Find the total are of the
pavements.
A rectangular grass plot 8 metre × 6 metre has roads,
each 1 metre wide, running in the middle of its one parallel Solution :
to length and the other parallel to breadth. Find the cost For rectangle A′B′C′D′ which is interior part of
of gravelling the road at Rs. 2 /metre2 . rectangle ABCD.
Solution : ∴ Its length = 120 metre – 2 × 2·5 = 115 metre
From figure it is clear that Area of rectangular road Breadth = 100 – 5 = 95 metre
ABCD = 1 × 8 = 8 metre2 Its Area = 115 × 95 metre2
Similar, Area of rectangular road = 6 × 1 = 6 metre2 For rectangle ABCD
But A′B′C′D′ is consider twice in the measuring of Area = 120 × 100 metre2
area of roads. For rectangle EFGH
So, Area of road = 6 + 8 – 1 × 1 Length = 120 + 5 = 125 metre
= 14 – 1 = 13 metre2 Breadth = 100 + 5 = 105 metre

188 | CAT Complete Course

Visit : www.EasyEngineering.net
Visit : www.EasyEngineering.net

Its Area = Length × Breadth Now, Area of trapezium ABCD = Area of ∆ ACD +
= 125 × 105 meter2 Area of ∆ ABC
Now, Required area is our shaded area 1
Now, Area of ∆ACD = × Height × Base
2
= 125 × 105 – 115 × 95
1
= 25 (5 × 105 – 23 × 19) = ×h×b
2
= 25 (525 – 437) 1
Area of ∆ABC = × Height × Base
= 25 × 88 = 2200 metre2 2
1
(C) AREA OF PARALLELOGRAM, = ×h×a
2
A RHOMBUS, A TRAPEZIUM, Area of trapezium ABCD
A QUADRILATERAL 1 1
= ×h×b+ ×h×a
(a) Parallelogram—In parallelogram, opposite sides 2 2
are parallel and equal to each other. 1
= × h × (a + b)
Area = Height × Base 2

ww
(b) Rhombus—In Rhombus, all sides are equal and
its diagonals perpendicular bisect to each other.
If d1 and d2 are the diagonals of the Rhombus ABCD.
1
Area of trapezium = × Height × Sum of sides
2
(d) Quadrilateral—We draw perpendiculars from

w
Now, AC = d1 , BD = d2
Now, In ∆ ACD
.Ea
Base = AC = d1
A and C to the diagonal BD whose lengths are h1 and h2
respectively.
Now, Area of Quadrilateral
= Area of ∆ABD + Area of ∆BCD

1 syE
d
Height = DO = 2
2
Area of ∆ACD = × Base × Height
1 1
= × h1 × BD + × h2 × BD
2 2
2
1 d d d
= × d1 × 2 = 1 2 ngi Illustration 18.
1
Area of Quadrilateral = × BD × (h1 + h2 )
2

nee
2 2 4
In ∆ ABC, Find the area of a quadrilateral inscribed in a circle
Base = AC = d1 whose side measures 36 metre, 77 metre, 75 metre and 40
d
Height = BO = 2
2
metre respectively.
Solution :
rin
1
Area of ∆ABC = × Height × Base
2 inscribed inside the circle.
g.n
We know that if a Quadrilateral of sides a, b, c and d

1 d d d
= × d1 × 2 = 1 2
2 2 4
Now, Area of Rhombus = Area of ∆ACD
S = e
Then, Area of the quadrilateral is given by

√ S (S – a)(S – b)(S – c)(S – d)
where S = Semi-perimeter of the quadrilateral t
+ Area of ∆ABC a + b + c + d 36 + 77 + 75 + 40
Now, S = =
d d d d 2 2
∴ Area of Rhombus = 1 2 + 1 2
4 4 S = 114
1
Area of Rhomubs = × d1 × d2 Hence, Area of quadrilateral
2
1 = √ 
114(114 – 36)(114 – 77)(114 – 75)(114 – 40)
Area of Rhombus = × Product of Diagonals
2 = 2886 metre2.
(c) Trapezium—If an quadrilateral contains two Illustration 19.
parallel and two non-parallel sides, it is called trapezium. The sides of a quadrilateral, taken in order, are 10,
If non-parallel sides are equal then it is called 10, 7, 5 metre respectively and angle contained by the
isosceles trapezium. first two sides is 60°. Find its area.
Let AB is parallel to CD Solution :
AB = a CD = b Let ABCD be the Quadrilateral.
and we draw a perpendicular AN to the side CD. Since, In ∆ACD

CAT Complete Course | 189

Visit : www.EasyEngineering.net
Visit : www.EasyEngineering.net

AD = DC and ∠ADC = 60° From the diagram clearly BE is a perpendicular


∴ ∠ DAC = ∠DCA drawn from B to the side CD.
We know that In ∆ ACD Now, ∆BCE is right angled triangle.
Sum of angles = 180° So, (BE)2 = (BC)2 – (CE)2
Or, 2∠ DAC + 60° = 180° = 41 – 25
2∠ DAC = 120° = 16
∠ DAC = 60° BE = 4 metre or Height = 4 metre
∴ ∠DCA = 60° 1
Now, Area of trapezium = × (15 + 25) × 4
Clearly, ∆ ACD is an equilateral triangle. 2
 3 × (Side)2
√ 1
= × 40 × 4 = 80 metre2 .
So, Area of ∆ ACD = 2
4
Illustration 22.
 3 × (10)2 = 25√ 3 metre2

= The perimeter of a rhombus is 20 metre and sum of
4
Now, Area of ∆ ABC : the lengths of its diagonals is 14 metre. Find the area of
the rhombus.

ww AC = 10 metre
If S be the semi-perimeter then
10 + 7 + 5
Solution :
Let d1 , d2 are the diagonals of the perimeter and a be

w S =

Area =
2

.Ea
= 11 metre

√
= √
S (S – a)(S – b)(S – c)

11(11 – 10)(11 – 7)(11 – 5)
the side.
So, Perimeter = 4a
20 = 4a
a = 5 metre …(1)

Illustration 20.
= √
syE

11 × 1 × 4 × 6 = 2√66 metre2. Sum of diagonals = 14
d1 + d2 = 14
(d1) + (d2 ) = 5 × 4
…(2)

ngi
Since, 2 2 2
The cross-section of a canal is a trapezium in shape if
the canal is 12 metre aside at the top, 18 metre wide at the (d1)2 + (14 – d1 )2 = 52 × 4
bottom and the area of the cross-section is 2400 metre2. 2 (d1)2 + 196 – 28d1 = 25 × 4
Find the depth of the canal.
Solution : nee 2 (d1)2 – 28 d1 + 96 = 0
28 ± √

(28) 2 – 4 × 96 × 2
We know that Area of trapezium is given by
1
= × Height × (Sum of parallel sides)
d1 =

rin 2×2
28 ± 2 × 2
√
49 – 48
2
Here, 12 metre, 18 metre are if depth of the canal is h
d1 =

d1 = g.n
28 ± 4
4

metre.
Now,
1
2400 = × h × (12 + 18)
2
2400 × 2 = h × 30
d1 =
4
32 24
4 4
d1 = 8, 6
,
e t
2400 × 2 d2 = 6, 8
h = = 80 × 2 We take d = 8 metre and d2 = 6 metre
30 1
1
h = 160 metre. Now, Area = × Product of diagonals
2
Illustration 21. 1
Parallel sides of a trapezium are 15 metre and 25 = × 8 metre × 6 metre
2
metre, while non-parallel sides of equal length each is √41 = 24 metre2.
metre. Now, find the area and height of the trapezium. Illustration 23.
Solution : In a four sided field, the longer diagonal is 24 metre
and the perpendicular from the opposite vertices upon the
longer diagonals are 16 metre and 18 metre. Find the area
of the field.
Solution :
Let ABCD be the such field whose diagonal AC is
24 metre.

190 | CAT Complete Course

Visit : www.EasyEngineering.net
Visit : www.EasyEngineering.net

1 Now, In ∆ ONB
Now, Area of this field = × (h1 + h2 ) × AC
2
a
1
= × 24 × (16 + 18) NB 2
2 sin ∆ BON = =
OB R
= 12 × 34 = 408 metre2 .
 180° 
(D) AREA OF REGULAR POLYGONS sin   = a
 n  2×R
(i) Polygon—A polygon is a plane figure enclosed
by the line segments which are known as the sides of the a  180° 
R = × cosec  
polygon. 2  n 
(ii) Regular Polygon—A polygon is a regular poly- 1
gon if its all sides and all angles are equal. Area of AOB = × Height × Base
2
(iii) Internal Angle of Regular Polygon—Each 1
internal angle of a regular polygon of n sides is equal to = × ON × AB
2
n – 2 °  180° 
 n × 180 1
× R × cos  ×a

ww  
(iv) Circum-Circle of a Regular Polygon : A regular
polygon can be inscribed in a circle which is known as
=
2

1 a
 n 
 180°   180° 
× a × × cosec  × 

w
=
the circum-circle. 2 2  n   n 
We draw a line ON ⊥ AB.
.Ea
Since we have a polygon of n sides whose length is a. =
1
4
 180° 
× a2 × cot 
 n 

=
360°
n
· syE
Now, Angle subtended by each sides on the centre O
If polygon has n sides, then

1  180° 
Area of polygon = n × × a2 × cot  
Now, In right angled triangle OBN

∠NOB =
360° 180°
= ngi 4

If r be the radius of in-circle.


 n 

Or, sin
180°
n
=
2×n
NB
=
OB 2 × R
a
n

nee Now, r = ON =
a
2
 180° 
× cot 
 n 

R =
a
2
 180° 
× cosec 
 n 
 Now,
rin
Area of in-circle = πr2

Area of polygon =
1 
n ×  × Height × Base 
2 
= π×
g.n a2
4
 180° 
× cot2 
 n 

=
1 
n ×  × ON × AB 
2 
Illustration 24.
=
4e
π × a2  180° 
× cot2 
 n 
·
t
1 
n ×  × R × cos × AB 
180° Find the area of a regular hexagon whose side is 5
=
2 n  metre.
1  Solution :
n ×  × R × cos × a
180°
=
2 n  Using the formula—

=
n a2
× × cosec
180°
× cos
180° √ 3
2 2 n n Area of regular hexagon = 3a2
2
1  180° 
Area of polygon = × n × a2 × cot   √ 3
4  n  = 3 × 52 ×
2
(v) In-Circle of a Regular Polygon—If a circle
3
√3
having centre at the centre of a regular polygon and = 25 ×
2
touching all sides of it is called the in-circle.
Let a is the length of the polygon and R is the radius 75
√ 3 metre2.
=
of the in-circle. 2

CAT Complete Course | 191

Visit : www.EasyEngineering.net
Visit : www.EasyEngineering.net

Illustration 25. Illustration 27.


Find the difference between the area of a regular Each side of a regular hexagon measures X cm. By
hexagon each of whose side is 24 cm and the area of the joining the mid points of each side, another hexagon is
circle inscribed in it. formed inside it. Find the ratio of areas of outer and inner
Solution : hexagons.
We know that the area of an sided regular polygon is Solution :
1  180°  Let ABCDEF is a regular hexagon of side X cm.
n × × a2 × cot  
4  n  Now, OD bisects angle EDC.
 180°  ∴ ∠ ODK = 60°
π × a2
and area of the in-circle is × cot2  
4  n  x x
√ 3 cm.
Now, OK = × tan 60° =
2 2
where a is the side of the polygon.
In regular hexagon OD = OE = ED
Now, Number of sides = n = 6
Each side of inner hexagon
1  180° 
∴ Required area = n × × a2 × cot   x
√ 3 cm.
4  n 

ww –
π × a2
4
 180° 
× cot2 
 n 
 Area of inner hexagon = 6 ×
= OK = ∠ K =

√ 3 × (∠ K)2
2

w 1
4
.Ea
= × 6 × (24)2 × cot 30°

1 22
– × × (24)2 × cot2 30°
=
3√

9
2 4
4
 3 × x2 × 3

syE
4 7

√ 3 – 144 × 22
= 144 × 6
7
×3
Area of outer hexagon = 6 ×
= ×
8
√ 3 × x2 cm2 .

√ 3 × (AB)2



√ 3 – 1447× 66 cm2 .
=  864
 ngi =
3
2
×
4
√ 3 × x2
Illustration 26.
Compare the area of an equilateral triangle, a square nee Now, Ratio =
3
2
×√
 3 × x2

rin
and a regular hexagon of equal perimeter. 9
×√
 3 × x2
8
Solution :
4
Let the perimeter of each polygon be S.

Now, each side of equilateral triangle =


S
g.n
= =4:3
3

(E) AREA OF A CIRCLE, SECTOR AND

e
3
 3 ×  S 
t
2
√ SEGMENT OF A CIRCLE
Its area =
4 3  (a) Circle—A circle is the locus of a point which
moves in a plane in such a way that its distance from a
 3 × 32
√ fixed point always remains the same.
= …(1)
36
The fixed point of a circle is called the centre.
S
Each sides of square = The perimeter of a circle is generally known as its
4
S2 circumference.
Now, Its area = (2)
16 (i) If r be the radius of the circle. Then,
S circumference = 2πr
Each side of regular hexagon =
6 Diameter = 2 × radius = d
 3 ×  S 
2
3√ ∴ Circumference = πd
Area of this regular hexagon =
2 6  d  2
(ii) Area = π r2 = π × 
Now, Area of equilateral triangle : Square : Regular 2 
S2 S2 S2 π
= : : = × d2
12√ 3 16 8√ 3 4
π 2
= 4 : 3√
 3 : 6. (iii) Area of a quadrant of a circle = ×r
4

192 | CAT Complete Course

Visit : www.EasyEngineering.net
Visit : www.EasyEngineering.net

(iv) Area enclosed by two concentric circles—If R (iv) Let us consider three identical circle of radius r.
and r are radii of two concentric circles, then Area Now, we have shaded area which is made after touching
enclosed by the two circles = πR2 – πr2 = (R 2 – r2 )π. to each other.
Some Important Points Now, Triangle formed by such geometry is equi-
lateral triangle.
(i) If two circles touch internally, then the distance
between their centers if equal to the difference of their Now, Side of ∆ABC = 2r
radii.  3 × (Side)2

Its Area =
(ii) If the circles touch externally, then the distance 4
between their centers is equal to the sum of their radii.  3 × 4r2 = √ 3 × r2

(iii) Distance moved by a rotating wheel in one =
4
revolution is equal to the circumference of the wheel. 60°
Now, Area of sector = 3 × × πr2
(iv) The number of revolutions completed by a 360°
rotating wheel is one minute πr2
=
Distance moved in one minute 2
=
Circumference
 3 × r2 – πr2
2

ww
(b) Sector of a Circle and Its Area—
Minor Sector—If minor arc ACB form the sector
Now, Shaded area =

=


√

 3 – π2  × r2

w
AOBCA then this sector is called the minor sector.

.Ea
Major Sector—The major arc ADB form the major
sector.
Since, 360° angle is made at the centre Area = πr2
 
(v) Chords and arcs—Let us consider that r be the
radius of the circle and arc ABC has length l.
Length of the arc which makes an angle θ is given by

syE
∴ θ° angle is made at the centre area =

θ°
θ°
360°
× πr2
l =
 θ 
 360°  × circumference of circle
 
So, Area of sector ∆AOB =
360°
1
Area of ∆AOB = × sin θ.r2
× πr2

ngi  θ 
l =   × 2r
 360° 
2
1
= × r2 .sin θ
2
nee
Illustration 28.
A piece of wire is bent in the shape of an equilateral

Now, Area of shaded region ACBD


= Area of the sector OACB – Area of the ∆ AOB rin
triangle of each side 1·32 metre if it is bent again to form
a circular ring. Find the diameter of the ring.

=
θ°
360°
1
× πr2 – × r2 . sin θ
2
Solution :

g.n
Each side of the equilateral triangle = 1·32 metre

e
Now, perimeter of the equilateral triangle
 π θ sin θ 
=  360° – 2  × r2
(c) Circular Pathway—
(i) Let ABC be the circle whose radius = r there is a
= 3 × 1·32 metre = 3·96 metre
Since, the some triangle piece of wire is bent again to
form a circular ring.
So, Perimeter of triangle = Circumference of the
t
circular ring
path way DEF outside this circle ABC width = w
Or, 3·96 = 2πr
Now, Area of circular part ABC = πr2
3·96 × 7
Area of circular part DEF = π. (r + w)2 r =
2 × 22
Area of circular pathway = π (r + w)2 – πr2 r = 0·63 metre.
= π w (2r + w) Illustration 29.
(ii) In Second Case—If circular path is made inside The hands of a clock are 14 cm and 7 cm respectively.
the circle ABC. Find the difference between the distances traversed by
Area of the circular path way = πw (2r – w) their extremities in 2 days 4 hours.
(iii) Semi-circle— Solution :
πr2 Total time = 2 days + 4 hours
Area of Semi-circle =
2 = 2 × 24 + 4
Perimeter = πr + 2r = r (π + 2) = 52 hours

CAT Complete Course | 193

Visit : www.EasyEngineering.net
Visit : www.EasyEngineering.net

Radius of hour hand = r = 7 cm. a


While, Path ACB = X2 =
Radius of minute hand = R = 14 cm. 2
45
Now, Total distance traversed by the extremity of Walking speed of Ram = metre/sec.
60
hour hand
3
22 = metre/sec.
= 2πr × n = 2 × × 7 × No. of revolution 4
7
Using the formula,
= 44 cm × 52
Distance = Speed × Time interval
= 2288 cm
3
Total distance traversed by the extremity of minute X2 – X1 = × 45
4
hand
= 2πR × no. of revolution  a a  135
Or, 2 – π  = 4
22  
= 2 × × 14 × 52
7 135 2 × π
Or, a = ×
= 88 × 52 = 4576 cm 4 (π – 2)
157

ww Required distance = 4576 – 2288 = 2288 cm.


Illustration 30.
A circular grass plot 42 metre in radius is surrounded
a = 67·5 ×

(F) CUBES AND CUBOIDS


57
metre

w
by a ring of gravel. Find the width of the gravel, so that

.Ea
the area of the grass and gravel may be equal.
Solution :
Parallelepiped—A solid bounded by three parallel
plane surfaces is called a parallelepiped. The plane sur-
faces are known as the faces of the parallelepiped. A
parallelepiped contains 6 sides and 12 edges.
gravel
Using formula : syE
Let r be the radius of grass plot and w = width of the
Each face of a parallelepiped is a parallelogram and
opposite faces are congruent i.e., equal in all respects.

Area of gravel = πw (2r + w)


Area of grass = πr2 ngi Let l, b , h be the length, breadth and height of the
cuboids.
Total surface area of the cuboids
According to question,
Area of grass = Area of gravel nee = Area of all the faces
= Area of ABCD + Area of BCGF
Or,
Or,
πr2 = πw (2r + w)
r2 = w.2.r + w2
rin
+ Area of EFGH + Area of CDHG
+ Area of AEDH + Area of ABFE
Or, w + 2w × 42 – (42)2 = 0
2

w =
– 42 × 2 ± 
√(42 × 2)2 + 4 × 1 (42)2 g.n
= l×h+h×b+l×h+l×b+b×h+l×b
= 2 (l × b + b × h + l × h)

w =
2
2
– 42 × 2 ± 42 × 2√
2 e
Volume of the cuboids = Area of base × Height
Volume of the cuboids = l × b × h
(iii) Diagonal of the cuboids = √

l 2 + b2 + h2 t
w = – 42 ± 42√  2 = 42 (– 1 ± √ 2) (iv) Area of the four walls = 2h (l + b)
Considering the + ve value— (v) Cube—If all the edges of a cuboids are equal in
length, it is called a cube.
w = – 42 (– 1 ±  √ 2) cm. Now, Total area of the cube = 6a2
Illustration 31. Volume of the cube = a3
Ram by walking diametrically across a circular grass
Diagonal of the circle = √ 3a
plot, finds that it has taken him 30 sec. less than if he has
kept to the path round the outside. If he walks 45 meters Illustration 32.
per minute. Find the circumference of the glass plot. Find the volume, the surface area and the diagonal of
Solution : a cuboids 12 cm long, 5 cm wide and 3 cm high.
Let ACBD is the circumference of grass plot whose Solution :
value is a. Here, we have
Now, When Ram walks across AOB l = 12 cm.
a b = 4 cm.
Distance = X1 =
π h = 3 cm.

194 | CAT Complete Course

Visit : www.EasyEngineering.net
Visit : www.EasyEngineering.net

Volume of the cuboids = l × b × h = 155·52 – (3·2 + 3)


V = 12 × 4 × 3 = 144 cm3 = 155·52 – 6·2 = 149·32 m2
Total Surface area of the cuboids Now, Cost of the colour washing
= 2 (l × b + b × h + l × h) = Required area × Cost
A = 2 (48 + 12 + 36) = 149·32 × 5
= 2 × 96 = 192 cm2 = Rs. 746·6.
Diagonal = √
l 2 + b2 + h2 Illustration 35.
The cost of white washing the walls of a room at
= √
(12) 2 + (4)2 + (3)2
Rs. 2·4 per m2 is Rs. 640 and the cost of carpeting the
= √
144 + 25 = 13 cm. floor at Rs. 6·4 per m 2 is Rs. 320. There are two almirahs
Illustration 33. each of size 2·4 m × 1 m and four doors each of size 2·4
m × 2 m. If the length and breadth are in the ratio 1 : 2.
A plot of land in the form of a rectangle has a
Find the dimensions of the room.
dimension 120 m × 112 m. A drainlet 20 m wide is dug
all around it on the outside and the earth dug out is evenly Solution :

ww
spread over the plot, increasing its surface level by 34 m.
Find the depth of the drainlet.
Solution :
Since, the ratio of length and breadth are 1 : 2.
So, Let length = a
Breadth = 2a

w
Let a m be the depth of the drainlet.
Now,Volume of drainlet
.Ea
= (160 × 10 × a + 160 × 10 × a + 112 × 10 × a
Area of the floor = a × 2a
= 2a2
Now, Cost of carpeting the floor

= (5440) m3
syE + 112 × 10 × a)

Now,Volume of earth spread out over the plot = 112


= 2a2 × 6·4
320 = 2a2 × 6·4

ngi
3200
× 120 × 34 m 3 Or, a2 =
2 × 6·4
Clearly, The Volume of earth spread over the plot a = 5m

Or,
= Volume of the drainlet
5440 × a = 112 × 120 × 34 nee Breadth = 10 m
Let height of the wall = h m
a =
112 × 120 × 34
5440
rin
Area of two almirahs = 2 × 2·4 × 1 = 4·8 m 2
Area of four doors = 4 × 2·4 × 2 = 19·2 m2

Illustration 34.
a = 84 m.

g.n
Now, Area of the four walls
= (l + b) × h
A class room is 8 m long 6.4 m wide and 5·4 m high.
It has one door 2 × 1·6 m and three windows, each
measuring 1 m × 1 m. The interior walls are to be colour
washed. The contractor charges Rs. 5 per sq. m. Find the According to question,
e
= (5 + 10) × h
= 15 h m2
t
cost of colour washing. The cost of white washing the walls = Area of the
Solution : walls excluding almirahs and door × rate
Total area of the four walls = 2 (l + b) × h 640 = {15 × h – (19·2 + 4·8)} × 2·4
640 × 10
Here, we have l = 8m Or, = 15 × h – 24
24
b = 6·4 m 800
Or, + 24 = 15 × h
h = 5·4 m 3
∴ Total area of the four walls = 2 (8 + 6·4) × 5·4 Or,
872
= h
3 × 15
= 2 × 14·4 × 5·4
872
= 155·52 m2 h = m.
3 × 15
Area of the door = 2 × 1·6 = 3·2 m2 Illustration 36.
Area of windows = 3 × 1 m × 1 m = 3 m2 One iron-solid is a cuboids of dimensions 12 m × 24
The required area to be washed = Total area of the m × 36 m. It is melted and cubes each of side 4 m are
four walls – (Area of the door + Area of windows) molded from it. Find the number of cubes formed.

CAT Complete Course | 195

Visit : www.EasyEngineering.net
Visit : www.EasyEngineering.net

Solution : Illustration 38.


Volume of iron-solid = 12 m × 24 m × 36 m A water tank of thickness 10 m measures externally
= 12 × 24 × 36 m3 220 m, in length, 180 m in breadth and 120 m in height.
Find its external and internal surface area and the maxi-
Volume of one cube = 4 × 4 × 4 m3
mum weight of water that it can store. 1 m3 of water
Now, Number of each cubes weighs 1 gm. What is the weight of the tank if 100 m3 of
Volume of iron solid its material weight 250 kg ?
=
Volume of one cube Solution :
12 × 24 × 36
= Let l, b and h be the external length, breadth and
4×4×4 height of the tank.
= 3 × 6 × 9 = 162. Similarly, l′, b′ and h′ be the internal length, breadth
Illustration 37. and height of the tank.
A cuboids has dimensions a m × b m × c m. If its Now, l = 220 m, b = 180 m, h = 120 m
length and breadth increases by 25%, then find the
l′ = 220 – 20 = 200 m
percentage increase in the lateral surface area of the
b′ = 180 – 20 = 160 m

ww
cuboids if breadth is the half of length.
Solution : h′ = 120 – 20 = 100 m
Let a be the length and b be the breadth of the Now, External volume = V = 220 × 180 × 120 m3

w
cuboids respectively.
Now, after 25% increment.

.Ea
Length = l′ = l + l ×
25 5
= l
Internal Volume = V′ = 200 × 160 × 100 m3
Volume of water in the tank = V′
Now, Weight of the water = V′ × 1 gm

syE
100 4
200 × 160 × 100
25 5 = kg
and Breadth = b′ = b + b × = b 1000
100 4
= 3200 kg
Since, lateral surface area = Area of four walls
= 2 (l + b) × h ngi Volume of material of the tank = V – V′
= 22 × 18 × 12 × 10– 3 – 20 × 16 × 103 m3
5 5 
Final Lateral surface area = 2  l + b  × h
4 4  nee = 1552 × 103 m3
1552 × 103 × 250
Since, it is given b =
l
2
rin
Now, Weight of the material =
100
= 3880000 kg.
So, Final lateral surface area
5 5 
= A′ = 2  l + b  × h
g.n
(G) RIGHT CIRCULAR CYLINDER

3
4 4 
= 5 × ×l×h
4
height of the cylinder.
(i) Area of the base = πr2 e
(a) Let r be the radius of the base and h be the

(ii) Curved surface area = 2 πrh


t
Initial lateral surface area
(iii) Total surface Area = Curved surface area +
= A = 2 (l + b) × h Area of circular parts
3 Total surface Area = 2 π rh + 2 π r2
= 2 × ×l×h
2
A = 2 π r (h + r)
15
A′ 4 (iv) Volume = Area of base × height
Now, =
A 3 V = πr2 h
15 1 5 (b) Surface Area and Volume of a hollow Cylinder
= × = = 3lh
4 3 4 A solid bounded by two co-axial cylinders of the
A′ 1 same height and different radii is called a hollow cylinder.
1– =
A 4
Let R and r be external and internal radii of the
% increase in lateral surface area cylinder respectively.
 A′  (i) Area of circular end = π(R2 – r2 )
=  1 – A  × 100 = 25%.
  (ii) Curved surface area of the hollow cylinder

196 | CAT Complete Course

Visit : www.EasyEngineering.net
Visit : www.EasyEngineering.net

= External surface + Internal surface For, Second cylinder can—


= 2 π Rh + 2 π rh Radius = 7 cm
= 2 π h (R + r) Height h = 20 cm
(iii) Total surface area of the hollow cylinder Its Volume = π r2 h
= Curved surface + 2 (Area of the base ring) 22
= × (7)2 × 20
= 2 π Rh + 2 (πR2 – πr2 ) + 2 πrh 7
= 2 π h (R + r) (h + R – r) = 154 × 20 m3
(iv) Volume of the material = External Volume – = 3080 m3
Interior Volume Since, Volume of Third Can = Volume of First Can
= π R2 h – π r2 h + Volume of Second Can
V = π h (R2 – r2 ) Or, π × (70)2 × h = 1540 + 1540 × 2
22
Illustration 39. × 70 × 70 × h = 1540 × 3
7
Find the curved surface area, volume and total sur-
70 × 3 3
face area of a cylinder whose height 5 m and circum- Or, h = = m.

ww
ference of its base is 44 m.
Solution :
Illustration 41.
700 10

A solid iron rectangular block of dimensions 2·2 m,

w
Given

Or,
h = 5m
Circumference of the base is given by = 2 πr

.Ea 22
44 = 2 × × r
7
2·1 m and 1·5 m is cast into a hollow cylindrical pipe of
internal radius 14 cm and thickness 7 cm. Find the length
of the pipe.
Solution :

Now, syEr = 7m
Curved surface area = 2 π rh
Let l be the length of the pipe.
Internal dimensions of the pipe are
= 44 × 5
= 220 m2 ngi r = 14 cm
h = l
Total Surface Area = 2 πr (r + h)
= 44 × (7 + 5) nee Internal volume = π r2 h

=
22
7
× (14)2 × l
= 44 × 12 m2
= 528 m2 rin
=
22
7
× 14 × 14 × l
Volume of the cylinder = π r2 h
22
× (7)2 × 5
g.n
= 22 × 28 × l cm3
External dimensions of the pipe

e
=
7

t
r = 14 + 7 = 21 cm
= 154 × 5 m3 h = l
= 770 m3 22
External volume = × 21 × 21 × l cm3
Illustration 40. 7
Two cylinders cans have bases of the same size, the = 22 × 3 × 21 × l cm3
diameter of each is 14 cm. One of the can is 10 cm high Volume of the material of the pipe
and the other is 20 cm. If the a can of radius r = 70 cm
= (22 × 21 × 3 × l – 22 × 28 × l )
and it has volume equal to the sum of both the cylinders
then. Find the height of this can of radius 70 cm. = 22 l (63 – 28) cm3
Solution : Volume of solid rectangular block
= 2·2 × 2·1 × 1·5 m3
For First cylinder can—
Now, we have
Diameter = 14 cm
22 × l × 35 cm3 = 2·2 × 2·1 × 1·5 × 106 cm3
Radius = 7 cm
21 × 15
Its Volume = π r2 h l = × 103 cm
35
22 45
= × (7)2 × 10 = × 103 cm
7 5
= 1540 m.3 = 9 × 103

CAT Complete Course | 197

Visit : www.EasyEngineering.net
Visit : www.EasyEngineering.net

Illustration 42. 22
= × 7 × 7 × 20 × 10– 2 cm3
A goodown building is in the form such that its upper 7
part is semi-cylindrical of radius 7 m while the inner = 440 × 7 × 10 – 2 cm3
portion of the godown is cuboidal form of dimension For Graphite,
14 m × 5 m × 21 m. Find the volume of the godown and 7
the total interior surface excluding the floor. Radius = r = = 3·5 cm
2
Solution : Height = 20 cm
Here, we have Volume of the graphite = π r2 h
22  7 
2
r + h = 21 m
= ×   × 10– 2 × 20
Since, r = 7m 7 2 
h = 14 m = 11 × 10 × 7 × 10– 2 cm3
l = 14 m breadth = b = 5 m Volume of wood = Volume of pencil
Since, Diameter of semi-cylindrical part is 14 m, so – Volume of graphite
diameter is along the length of the godown. = (44 × 7 × 10– 1 – 77 × 10– 1) cm3

ww
Now, Height of cylindrical part = b = 5 m

Volume of semi-cylindrical part =


πr2 h
2
= 7 × 10– 1 × (33) cm3
Mass of wood = Volume × Density
= 7 × 33 × 10– 1 × 0·7 gm

w =

.Ea
22 (7) × 5
7
×
2

2
= 11 × 7 × 5 m3
= 385 m.3
= 16·17 gm
Mass of graphite = 7·7 × 3·5
= 2·695 gm

syE
Volume of cuboidal form of the godown
= l×b×h
So, Total weight = 2·695 + 16·170 gm
= 18·765 gm.
(H) CONE
= 14 × 5 × 14 m3
= 980 m3 ngi A right circular cone is a solid generated by revolv-
ing a line segment which passes through a fixed point and
Total Volume of godown = 385 + 980
= 1365 m3 nee
which makes a constant angle with a fixed line.
Height of the Cone—Perpendicular drawn from
vertex C to the base on the circular base.
Total surface area excluding the floor
= Interior surface of semi-cylindrical part
rin
Slant height of the Cone—The length of the segment
BC is called the slant height of the cone.
+ Interior surface of cuboidal form
= πr (r + h) + 2 (l + b) × h g.n
Now, Curved surface area of the cone
1
=
22
7
× 7 × (7 + 5) + 2 (74 + 5) × 14

= 22 × 12 + 28 × 19
2
base e
= × length of the arc of the sector × radius of the

1
Curved surface area of cone = × 2 πr × l
2
t
= 264 + 532 = 796 m 2
Illustration 43. = π rl
A lead pencil consists of a cylinder of wood with a Total surface area of the cone = Curved surface area
solid cylinder of graphite filled into it. The diameter of the + Area of the base
pencil is 14 mm, the diameter of the graphite is 7 mm and = πrl + πr2
the length of the pencil is 20 cm. Calculate the weight of
= π r (l + r).
the whole pencil, if the specific gravity of the wood 0·7
gm/cm3 and that of the graphite is 3·5 gm/cm3 . Illustration 44.
Solution : The radius of the base and the height of a right
circular cone arc respectively 14 cm and 35 cm. Find the
Given for pencil,
curved surface area, total surface area and the volume of
Radius = 7 mm the cone.
Height = 20 cm Solution :
Now, Here, r = 14 cm
Volume of the pencil = π r2 h h = 35 cm

198 | CAT Complete Course

Visit : www.EasyEngineering.net
Visit : www.EasyEngineering.net

Let l be the lateral height or slant height, then 10


Mass of iron = 5416π ×
1000
l = 
√ r 2 + h2 = 
√
142 + 35 2
1
= 5416 × 3·14 ×
= 7 ×
√
4 + 25 = 7
√29 100
Curved surface area or lateral surface area = πrl = 170·07 kg.
22 Illustration 46.
= × 14 × 7 √29
7
The interior of a building is in the form of cylinder of
= 22 × 2 × 7 
√29 diameter 14 m and height 10 m, surmounted by a cone
whose vertical angle is a right angle. Find the area of the
= 308√
29 surface and the volume of the building.
Total surface area of the cone Solution :
= (π rl + r2 ) According to question,
= π r (l + r) Radius of the base of cylinder = 7 m = r1
22
= (
× 14 × 7√
29 + 14 ) ∴ Radius of the cone = r2 = 7 m

ww
7
Now, In ∆ AOB
= 22 × 2 × 7 (2 + √
29 ) ∠BAO = 45°

w
Value of the cone =
=

3
308 (2 + √

.Ea
1 2
29 ) cm
1 22
2

π r h = × × 14 × 14 × 35
3 7
sin 45° =

AB =
BO
AB
BO
sin 45°
=
7
1
= 7
√2 m
22 × 2 × 14 × 35 3

Illustration 45.
=
syE3
cm
AB is the lateral height of the cone.
2

cylinder and remaining in the form of a right circular


ngi
An iron pillar has part in the form of a right circular Now, AO =

=

√ (AB) 2 – (BO) 2


√ (7 
√ 2)2 – (7)2 = 7 m

nee
cone. The radius of the base of each of cone and cylinder
is 6 cm. The cylindrical part is 120 cm high and the Now, Surface area of the building = Surface area of
conical part is 8 cm high. Find the weight of the pillar if the cylinder + Surface area of cone
one cubic cm of iron weighs 10 gm.
Solution :
rin
Surface area of the building
= 2 π r1 h + π r2 l
For conical part r = 6 cm
h = 8 cm
g.n
= 2 × 3·14 × 7 × 10 + 3·14
× 7 × 7
√2
l = √

r 2 + h2 = 
l = 10 cm
1
Volume = π r2 h
3
√
62 + 8 2
e
= 440 + 154 × 1·41
= 440 + 217·14
= 657·14 m.2
t
1 Volume of the building
= × π × 62 × 8
3 = Volume of the cylinder + Volume of the cone
= 12 × 8 × π = 96 π cm3 1
= π (r1 )2h + (r2 )2h′
For cylindrical part h = 120 cm, r = 6 cm 3
Volume = π r2 h = π × 62 × 120 22 1 22
= × 7 × 7 × 10 + × × 7 × 7 × 7
7 3 7
= 36 × 120 × π
154 × 7
= 4320 π cm3 = 1540 +
3
Volume of iron = Volume of conical part
= 1540 + 359·33 = 1899·33 m.3
+ Volume of cylindrical part
Illustration 47.
= 96π + 4320π
A semi-circular sheet of metal of diameter 28 cm is
= 5416 π cm3 bent into an open conical cup. Find the depth and capa-
Mass of iron = Volume of iron × density city of cup.

CAT Complete Course | 199

Visit : www.EasyEngineering.net
Visit : www.EasyEngineering.net

Solution : (I) PYRAMIDS


When the semi-circular sheet is bent into an open Right Pyramid :
conical cup, the radius of the sheet becomes the slant 1
height of the cup and the circumference of the sheet Volume = × Base area × Height
3
becomes the circumference of the base of the cone. 1
= ×A×h
∴Slant height of the conical cup = 14 cm. 3
If r be the radius of the conical cup. 1
Slant Surface area = × Slant height × base side
∴ Circumference of the base of the conical cup = 2
Circumference of the sheet 1
= ×l×b×n
∴ 2 π r = π × 14 2
⇒ r = 7 n = number of sides
Now, l 2 = r 2 + h2 Whole Surface Area = Base Area + Total Slant Arc
For A right pyramid on a triangular base, each edge
⇒ h = √
l 2 – r2 of the pyramid is ‘a’ metre.
h = √
(14) 2 – (7) 2 = 7
√ 3 cm OR A regular tetrahedron of ‘a’ metre edge.

ww h = 7 × 1·732 = 12·12 cm
∴ Depth of the cup = 12·12 cm
Volume =
a3
6√2

w
Also, capacity of the cup = Volume of the cup

.Ea 1
= πr2 h
3
1 22
For Square Base =

Illustration 49.
a3
3√2

syE
= × × 7 × 7 × 12·12
3 7
= 602·26 cm.3
A right pyramid 6 m high has a square base of which
the diagonal is 10 m. Find its volume.
Solution :
Illustration 48.
The radii of the ends of a bucket of height 24 cm are
21 cm and 14 cm. Find its capacity. ngi Let ABCDE be our required pyramid
OA = h = 6 m.

Solution :
Using the properties of similar triangles between nee If side of the base be a.
Now, diagonal = a √ 2
AOB and AO′E.

Now,
OB
=
AO 7
or =
h′
Or,
rin
10 = a √2
10
O′E AO′ 21 h′ + 24
2
a =

g.n2

=  
2

e
2 h′ 10
Or, = Now, Area of base = A = a2

Now, Volume of cone ADE


3 h′ + 24
h′ = 48 cm.

1  21 
2
= 50 m2

Now, Volume of the pyramid = × A × h


1
3
√
 2
t
= π   × (24 + 48)
3 2  1
= × 50 × 6
1 22 21 21 3
= × × × × 72
3 7 2 2 = 100 m3
= 11 × 21 × 36 Illustration 50.
= 396 × 21 Find the volume of a pyramid formed by cutting off a
= 8316 cm3 corner of a cube whose edge is 8 m by a plane bisects
1 three conterminous edges.
Volume of cone ACB = × π × (7)2 × 48
3 Solution :
1 22
= × × 7 × 7 × 48 Let the edge of the given cube = 8 m
3 7
ABCO is our required pyramid.
= 154 × 16 = 2464 cm3
Capacity of bucket = 8316 + 2464 Now, AO = BO = OC = 4 m
= 10780 cm.3 Here, AO, BO, OC are perpendicular to each other.

200 | CAT Complete Course

Visit : www.EasyEngineering.net
Visit : www.EasyEngineering.net

If ∆BOC is base. 1
Volume of cone BCD = × (OB)2 × OC
1 3
Now its Area = × 4 × 4 = 8 m2
1  120 
2
2
= ×  × 288
1 3  13  13
Volume of pyramid = × A × h
3
1  120   50 288 
2
1
= ×8×4 Now, Total volume = ×  × + 
3 3  13   13 13 

1  120 
2
32 3
=
3
m = ×  × 388 m3
3  13  13
Illustration 51.
Illustration 52.
A right angled triangle of which the sides are 10 m
A cone is 100 m high and its slant height is inclined
and 24 m in length, is made to turn round its hypotenuse.
30° to horizon. Find the area of its curved surface.
Find the volume of the double cone thus formed.
Solution :
Solution :
According to question,
Let ABC be the right angled triangle.

ww
where ∠ ABC = 90°
AB = 10 m
∠ABO = 30°
Since, in right angle ∆ AOB
h AO

w BC = 24 m
AC =
.Ea
= √

√ (AB) 2 + (BC) 2

(10) 2 + (24)2 = 26 m
sin 30° = =

1
2
=
l AB
100
AB

syE
Since, cone is made by rotating ∆ABC along AC.
So, radius of base of the required cone = BO = OD Now,
l = 200 m

cos 30° =
BO r
AB l
=
Now, ∆ABC and ∆BOC are similar.

So,
AB
BO
=
AC
BC ngi 3 = r

2 200

Or,
10
X
=
26 13
=
24 12 nee r = 100√
Area of lateral surface = πrl
3m

X =
120
13
m
rin =
22
7
× 100 
√ 3 × 200

So, In right angled triangle AOB


AB = 10 Illustration 53. g.n
= 3·14 × 2 × 104 × √
 3 m.2

BO =
120
13
e
The base of a prism is quadrilateral ABCD if its
height is 12 m and AB = 9 m, BC = 40 m, CD = 28 m,
AB = 9 m, BC = 40 m, CD = 28 m, DA = 15 m, ∠B = 90°.
t


 120  2 Find the volume.
AO = (10) 2 – 
 13  Solution :
According to question,
√ 100 × 169 – 14400
=   AB = 9, BC = 40
 13 
Now, In right angled triangle ABC
10 50
= ×5= m (AC) 2 = (AB) 2 + (BC) 2
13 13
50 = (9) 2 + (40)2 = 1681
So, CO = 26 – AO = 26 –
13 AC = 41 m.
288 Now, Area of the base = Area of ∆ABC
=m
13 + Area of ∆ACD
1 1
Volume of cone ABC = × (BO)2 × AO = × 9 × 40 + Area of ∆ACD
3 2
1  120 
2
In ∆ ACD
= ×  × 50
3  13  13 AD = 15, DC = 28, AC = 41

CAT Complete Course | 201

Visit : www.EasyEngineering.net
Visit : www.EasyEngineering.net

15 + 28 + 41 4 22
If S = Semi-metre = = 42 m = × × 2·1 × 2·1 × 2·1
2 3 7
∴ Area = √
S (S – a)(S – b)(S – c) = 88 × 0·441 metre3
Illustration 55.
= √
42 (42 – 41)(42 – 15)(42 – 28)
A spherical lead ball of radius 10 cm is melted and
= √
42 × 1 × 27 × 14 small lead balls of radius 5 mm are made. Find the total
= √
6 ×7×9×3×2×7 number of possible lead balls.
= 6 × 7 × 3 = 42 × 3 = 126 m2 Solution :
Area of the base = 180 + 126 Since, small lead balls are melted from the spherical
lead ball.
= 306 m2
So, Volume of the spherical lead ball = Number of
Volume of the prim = Area of base × Height
small lead balls × Volume of one small lead ball
= 306 × 12 = 3672 m.3 4 4
Or, π (10 cm)3 = n × π (5 mm)3
(J) SPHERE 3 3
4 5  3

ww
A sphere is a solid figure generated by a complete 4
Or, (10 cm)3 = n × π  cm 
revolution of a Semi-circle around of its diameter which 3 3  10 
is kept fixed. 10 × 10 × 10 × 10 × 10
× 10 = n

w
The centre and the radius of the Semi-circle are also Or,
5×5×5
the centre and radius of the sphere.

.Ea
Let a sphere of radius r has centre O. Now,
(i) Surface Area = 4 πr2
Or,
Illustration 56.
n = 8000

syE
(ii) Volume of the Sphere = π r3
4
3
A hemispherical bowl of internal diameter 24 cm
contains a liquid. This liquid to be filled in cylindrical
bottles of radius 3 cm and height 5 cm. How many bottles

ngi
For a hemisphere of radius r. are required to empty the bowl ?
(i) Curved surface area = 2 π r2 Solution :
Total Surface area = 2 π r2 + πr2 2

nee
(ii) Volume of the hemispherical bowl is given by π r3
Total Surface area = 3 π r2 3
4 2
Volume = π r3 Now, Volume of hemispherical bowl = π (24)3

rin
(iii) 3
3
Let us consider a hollow spherical shell of external Volume of the cylindrical bottle = π R2 h
radius R and internal radius r.
∴ Total Surface Area = 4π (R2 – r2 ) g.n = π × 32 × 5
Now, n be the number of cylindrical bottle, then

Illustration 54.
4
Volume of the Shell = π (R3 – r3 )
3

Find the volume and surface area of a sphere of


Or,
2
3

n = ×
e
π (24)3 = n × π × 32 × 5

2 24 × 24 × 24
3 5×3×3
t
radius 2·1 metre. 2 × 512
n =
Solution : 5
Surface area of the sphere of radius r is given by Illustration 57.
S = 4π r2 A spherical ball of radius 4 cm is melted and recast
22 into three spherical balls. The radii of two of the balls
S = 4 × × (2·1)2 1·5 cm and 2 cm. Find the diameter of the third ball.
7
88 Solution :
= × 2·1 × 2·1
7 Let R be radius of the initial spherical ball.
= 88 × 0·63 metre2 Now, r1 , r2 , r3 be the radii of the three spherical balls.
Now, Volume of the sphere is given by 4 4 4 4
4 Now, π R3 = (r1 )3 + (r2 )3 + (r3 )3
V = π r3 3 3 3 3
3
Or, R 3 = (r1 )3 + (r2 )3 + (r3 )3
4 22
= × × (2·1)3 Or, 43 = (1·5)3 + 2 3 + (r3 )3
3 7

202 | CAT Complete Course

Visit : www.EasyEngineering.net
Visit : www.EasyEngineering.net

Or, 64 = 2·25 + 4 + (r3 )3 7


R 3 = 23 ×
Or, (r3 )3 = 60 – 2·25 = 57·75 4
3

√74
3
r3 = 
√ 57·75 cm. R = 2×
Illustration 58. Now, Thickness = R – r
A solid wooden toy is in the shape of a right circular  3 


2 × 7
cone mounted on a hemisphere. If the radius of the = – 2 cm.
hemisphere is 4·2 cm and the total height of the toy is  4
10·2 cm. Find the volume of the wooden toy. Illustration 60.
Solution : Find the whole surface of a hemisphere if—
Let R be the radius of solid wooden toy and h be the (i) It is a solid one with a diameter 14 cm.
height of the cone. (ii) It is 1 cm in thickness and 14 cm in external dia-
R + h = 10.2 meter.
4·2 + h = 10.2 Solution :
h = 6 cm (i) Radius of hemisphere = 7 cm.

ww 2
Volume of hemispherical part = π R3
3
Now, Total Surface area of hemisphere = 3πr2
22
Total surface area = 3 × × 72

w .Ea
2 22
= × × (4·2)3
3 7
1
Volume of conical part = π R2 h
3
7
= 66 × 7
= 462 cm2

syE 1
= × π × (4·2) 2 × 6
3
44
× 4·2 × 4·2 × 4·2
(ii)


Thickness = 1 cm
External radius R = 7 cm
Internal radius = r = R – thickness

ngi
Volume of wooden toy =
21
= (7 – 1) = 6 cm
1 22
+ × × 4·2 × 4·2 × 6 Now, Total surface area of the hemisphere = Internal

Illustration 59.
3 7
= 266·11 cm3
nee
surface area + External surface area + upper Surface area
= 2 π r2 + 2 π R2 + (R2 – r2)

Find the thickness of a hollow sphere whose inner


diameter is 4 cm, if it weigh half as much as a solid ball 22 rin
= 2π × 62 + 2π × 72 + (72 – 62 )
22
of the same diameter.
Solution : =
22
7
g.n
= 2 × × 85 + × 13
7

(170 + 13) =
22
× 183 cm2
Let R and r be the external and inner radii of the
hollow sphere.
Given r = 2 cm.
7
Exercise Ae 7

1. The foot of a ladder 13 m long is 5 m from a house t


Radius of solid sphere is also 2 cm. and its top reaches the upper part of a circular
If both are made of the same material, then density window. When the foot is drawn away to a distance
remains constant. of 7 m remote from the house, the top reaches the
lower edge of the window is—
Now, Weight = Volume × Density
77 2
4 (A) 97 m2 (B) m
Volume of solid sphere = π × 23 2
3
7
4 (C) 9 m2 (D) None of these
Volume of hollow sphere = π × (R3 – 23 ) 14
3
2. Ram by walking diametrically across a circular grass
Now, According to question, plot, finds that it has taken 30 seconds less than if he
4 4 had kept to the path round the outside. If he walks at
π × (R3 – 23 ) × d = π × 23 × d
3 3 the rate of 60 m per minute, the diameter of the grass
where d is the density of the material. plot is—
3 (A) 27 m (B) 26·25 m
Now, R 3 – 23 = 23 ×
4 (C) 28 m (D) 35 m

CAT Complete Course | 203

Visit : www.EasyEngineering.net
Visit : www.EasyEngineering.net

3. A garden is 450 m long and 200 m broad. It has two (A) 250 m (B) 270 m
roads each 5 m wide running in the middle of it, one (C) 275 m (D) 345 m
parallel to the length and the other parallel to the
breadth. The cost of gravelling them at Rs. 2·15 per 12. A path 5·5 m wide, running all round a square park,
square metre will be— has an area of 451 m 2 . The cost of covering the area
of the park enclosed by the path at Rs. 1·25 per square
(A) Rs. 5998·50 (B) Rs. 6920·50 metre will be—
(C) Rs. 6934 (D) Rs. 6933·75 (A) Rs. 250 (B) Rs. 195·90
4. The length of a room is 15 m. The cost of carpeting it (C) Rs. 281·25 (D) Rs. 300
at Rs. 1·50 per square metre is Rs. 225 and the cost
of repairing its walls at Rs. 6·00 per square metre is 13. The area of a square field is 12100 m2 . How long
Rs. 3600. The breadth and height of the room are— will it take a man to walk round the outside of it at
the rate of 3 km per hour ?
(A) 10 m, 12 m (B) 5 m, 9 m
(A) 8·8 minutes (B) 7·5 minutes
(C) 15 m, 12 m (D) 8 m, 13 m
(C) 9·0 minutes (D) 10·15 minutes
5. Within a rectangular courtyard of length 60 m, a
gravelled path 32 m wide is laid down along the four Directions (Q. 14 to 16)—Refer to the following

ww
sides. The cost of the gravel is Rs. 3 per square metre.
If the path had been twice as wide, then the gravel
would have cost Rs. 1200 more. The width of the
information to answer the question that follow—
S (x) is the area of a square where, x is side of square.

w
P (x)is the perimeter of a square where, x is side of
courtyard is—
square.
(A) 6 m
(C) 5 m
.Ea
(B) 12 m
(D) 7 m
R (x, y) is the area of a rectangle where, x is length
and y is the breadth.

(A) 66 cm2
(C) 132 cm2
syE
6. The area of the parallelogram is—
(B) 135 cm2
(D) None of these
h (x, y) is the perimeter of a rectangle where, x is
length and y is the breadth.
14. The value of h[P{R(2, 3)+ S(4)}, 4] is equal to—
7. ABCD is a parallelogram, then the value of ∠C
and ∠ B will be— ngi (A) 170 (B) 200

nee
(C) 95 (D) 184
(A) 125°, 70° (B) 90°, 75°
(C) 55°, 125° (D) 110°, 98° [S (4) – R (3, 4)] h (2‚ 3)
15. The value of × is equal to—
P (1) S (2)
8. A plot of ground, 15 m by 12 m, has a flower-bed
cut-out in the centre, 5 m by 4·5 m. What fraction of
the whole is occupied by the flower-bed ?
(A) 2
rin (B) 2·5

(A)
3
8
(B)
1
8
(C) 5

16. The value of g.n


(D) 9
 h (10‚ 20) – h (10‚ 20) S (5) 
 S (10) – P (20) – P (2·5)  is
(C)
1
11
(D)
4
11
9. If the fore-wheel of a carriage is 4 m and the hind
wheel is 6 m in circumference, then how far will the
equal to—
(A) 5·5
e
(B) 4·5 t
(C) 3·25 (D) 5·0
carriage have gone when the fore-wheel has made
2500 revolutions more than the hind wheel ? 17. A square contains 9 times the area of another square.
If one side of the larger square be 10 cm greater than
(A) 25 km (B) 27·75 km
that of smaller square, then the perimeter of smaller
(C) 15 km (D) 30 km square will be equal to—
10. The volumes of spheres are proportional to the cubes (A) 24 cm (B) 20 cm
of their radii. Two spheres of the same material
weigh 9·8 kg and 1·4 kg and the radius of the smaller (C) 18 cm (D) 12 cm
is 2 cm. If the two were melted down and formed 18. Mr. Khanna walks in morning in a square enclosure
into a single sphere, then what would be its radius ? of 900 m2. A man walks at the rate of 60 kmph along
(A) 4 cm (b) 4·3 cm one side, a diagonal, along another side, and returns
(C) 5 cm (D) 15·75 cm along the other diagonals to the starting point. The
time taken in walking the total distance is—
11. A railing encloses a rectangular field of 4500 m 2 .
The length of the field is to its breadth as 4 : 5. What (A) 1·50 minutes (B) 1·45 minutes
is the whole length of the railing ? (C) 2·50 minutes (D) 1·35 minutes

204 | CAT Complete Course

Visit : www.EasyEngineering.net
Visit : www.EasyEngineering.net

19. Three concentric circles, numbered a1 , a2 and a3 are 26. The dimensions of the floor of a hall are 12 × 48 m2.
drawn in such a way that the circumference of a2 is If square tiles of largest possible dimensions are to
the average of the circumferences of a1 and a 3 . The be used to pave the floor such that none of the tiles
ratio of the area between a1 and a2 to that between a2 need to be broken, then the number of tiles to be used
and a3 will be (given that the radii are as 10 : 11 : is—
12)— (A) 2 (B) 5
17 15
(A) (B) (C) 4 (D) 10
19 17
21 27. The length and breadth of a rectangular plot of a land
(C) (D) None of these are in the ratio of 7 : 5. The owner spent Rs. 2400 for
23
Directions (Q. 20 to 21)—Refer to the following surrounding it from all the sides at the rate of Rs. 2
information to answer the question that follow. per metre. What is the difference between the length
A piece of wire 20 m long is cut into two pieces, one and breadth of the plot ?
of which is bent into a circle and the other into the square (A) 85 m (B) 100 m
enclosing it. (C) 127 m (D) 250 m
20. The area of square outside the circle is—
28. A 8 metres wide road is to be constructed surrounding
(A) 1·2 cm2 (B) 1·68 cm2

ww
(C) 3 cm2 (D) 1 cm2
21. The ratio of the radius of circle to the perimeter of
a square plot of area 225 m2 . If the unit cost of
construction is Rs. 310 per sq. m, then what is the
total cost of construction ?

w
square is—
(A) 1 : 8
(C) 5 : 4
.Ea
(B) 8 : 1
(D) 2 : 3
22. From a square plate of side 20 cm four squares each
(A) Rs. 15,000
(C) Rs. 2,28,350
(B) Rs. 2,28,160
(D) None of the above
29. If the side of an equilateral triangle is increased by

perimeter of the figure is—syE


of side 4 cm are cut away as given in figure. The
25%, then its area will increase by—
(A) 45·15%
(C) 70%
(B) 56·25%
(D) 85%

ngi
30. In the figure drawn alongside, by how much would
the shaded area increase if the radii of both the inner

nee
and outer circles get doubled ?
(A) 300%
(C) 525%
(B) 500%
(D) 800%

rin
31. If the length, breadth and height of a cuboid are 5 m,
3 m and 2 m respectively, then its surface area is—
(A) 75 cm
(C) 64 cm
(B) 82 cm
(D) 105 cm
(A) 62 m2
(C) 30 m2 g.n
(B) 65 m2
(D) None of these

and recast into three spherical balls. The diameter of


two of these are 2 cm and 3 cm respectively. The
diameter of the third ball is—
e
23. A spherical ball of lead, 6 cm in diameter is melted 32. The length, breadth and height of a rectangular box
are 6 m, 5 m and 2 m respectively. How many cubic
metres of sand will be needed to fill the box to a
depth of 80 cm ?
t
(A) 5·65 cm (B) 6·15 cm (A) 9·85 m3 (B) 9·40 m3
(C) 7·80 cm (D) 9·05 cm (C) 30 m3 (D) 12 m3
24. A carpet 20 m long by 15 m wide is so placed on the 33. The number of bricks, each measuring 25 cm × 12 cm
floor of a room that there is a border 30 cm wide all × 45 cm, needed to construct a wall 10 m long, 5 m
round the carpet. What is the area of the floor of the high and 50 cm thick, is—
room ? (A) 1500 (B) 1852
(A) 329·64 m2 (B) 253·45 m2 (C) 1659 (D) 1955
(C) 321·36 m2 (D) 450 m2
34. Three cubes of copper are melted and formed into a
25. A rectangular park, with a side of 18 m long and 13 m single cube of edge 12 cm, if edge of one cube is 6
wide is crossed centrally by two perpendicular con- cm and edge of second cube is 10 cm then the edge
crete roads, each 2·2 m and 4·3 m wide respectively. of third cube ? The surface area of the new cube
The area of the park left is— formed is—
(A) 96·54 m2 (B) 100·25 m2 (A) 8 cm, 44 cm2 (B) 8 cm, 864 cm2
(C) 122·75 m 2 (D) 137·46 m 2
(C) 6 cm, 144 cm2 (D) 6 cm, 864 cm2

CAT Complete Course | 205

Visit : www.EasyEngineering.net
Visit : www.EasyEngineering.net

35. A rectangular room measure 15 m × 12 m × 9 m. 44. Keeping the volume of a wire the same as before, we
What is the maximum length of an iron sticks that it decrease its diameter by 95%. The per cent change in
can accommodate ? its length is—
(A) 15 m (B) 15√
2m (A) – 125% (B) 300%
(C) 135·8% (D) – 12·5%
(C) 17 m (D) 18√3m
45. The volume of a sphere is changing @ 121 cc/min.
36. The volume of a right circular cylinder is 14850 cubic
The rate at which the surface area of the sphere area
centimeters. If the vertical height of the cylinder is
of the sphere is changing when the radius of the
21 cm, then what is its lateral area ?
sphere = 11 cm, is—
(A) 1239 cm2 (B) 1276 cm2
(A) 32π cm2/ min (B) 22 cm2 / min
(D) 2250 cm2 (D) 2970 cm2 2
(C) 22π cm / min (D) 32 cm2 / min
37. A right circular solid cylinder of base radius 3 cm
and vertical height 320 cm is melted to form 6 equal 46. A right circular cone and right circular cylinder have
solid spheres. If there is a process loss of 40% during equal height and equal bases. Their curved surface
such formation, then what is the radius of each of the are in the ratio 5 : 6. The ratio of their base radius to
solid sphere so formed ? the height will be—

ww
(A) 4√
 3 cm
(C) 7 cm
(B) 6 cm
(D) 7·5 cm
(A) 3 : 6
(C) 4 : 3
(B) 2 : 3
(D) 5 : 5

w
38. A room measures 12 m ×12 m × 12 m. What is the

.Ea
maximum length of sticks (in metres) which it can
contain ?
47. From a solid cylinder of height 15 cm and radius 10
cm, a cavity to a depth of 5 cm followed by a
cylindrical bore of radius 3 cm is made. The volume
of material in the solid is—
(A) 12 m
(C) 15√ 2m syE
(B) 12√
2m
(D) None of these
(A) 1500 π
(C) 1080 π
(B) 1650 π
(D) 1200 π
48. A rectangular courtyard of dimensions 15 m × 9 m is
39. The base radius of the right circular cone is 5 cm and
its vertical height is 42 cm. Its volume is—
(A) 1156 cm3 (B) 1225 cm3 ngi surrounded an all sides by a footpath 3·5 m wide.
Since, the President of India is about to visit this site,

(C) 1100 cm3 (D) 1352 cm3


40. A sphere circumscribes a cube of side 'a'. How many nee we want to provide a proper foundation to this foot-
path and hence we need to dig it to a depth of 20 cm
and fill it completely with gravel. The amount of
times the volume of cube is the volume of the
sphere ? (A) 20·08 m3 rin
gravel (filling material) needed will be—
(B) 43·4 m3
(A) 2
(C) π
√ 3/2
(B) π
(D) 3

(C) 50·5 m 3

g.n
(D) 65·9 m3
49. The greatest possible sphere is turned from a cubical
41. A circle of radius 5 cm is cut such that a sector
PABC (sector angle 90°) is formed, where P is the
centre of the circle and PA = PB = PC are the radii.
Now the sector is folded to make PA and PC
will be—
(A) 42 in.
e
block of wood. If the volume of the block removed
be 4410 c. in., the diameter of the sphere (π = 22/7)

(B) 47 in.
t
coincident. Thus, the volume of the figure now (C) 41 in. (D) 21 in.
generated will be (cc)— 50. A sphere of 6 ft. radius resets on a table. The volume
(A) 7·1 (B) 7·9 of right hollow cone which can just cover it will be—
(C) 8·9 (D) 6·2 [The section of the cone through the axis being an
equilateral triangle (π = 22/7).]
42. Each edge of a cube is increased by 25%. The
percentage increase in the surface area is— (A) 2036·6 c.ft. (B) 2038 c.ft.
(C) 1539·9 c.ft. (D) None of these
(A) 75% (B) 120%
(C) 56·25% (D) 85% Exercise B
43 The percentage increase in the volume of a cuboid 1. A circular garden of radius 84 m is to be surrounded
when three edges are increased by 200%, 120% and by a road 8·4 metre wide. What is the ratio of the
350% respectively will be— area of the road to that of the garden ?
(A) 1650% (B) 670% (A) 1·1 (B) 1·21
(C) 470% (D) 2870% (C) 0·21 (D) 0·11

206 | CAT Complete Course

Visit : www.EasyEngineering.net
Visit : www.EasyEngineering.net

2. How many spherical balls. Each of radius 3 cm, can 11. A closed box made of wood of uniform thickness has
be made by melting a spherical ball of radius 9 cm ? length, breadth and height 15 cm, 13 cm and 11 cm
(A) 8 (B) 27 respectively. If the thickness of the wood is 2·5 cm,
then the inner surface area is—
(C) 9 (D) 81
(A) 426 cm2 (b) 376 cm2
3. The rectangular floor of a room having dimension 40
feet 60 feet needs to be carpeted. If a 2 foot margin is (C) 490 cm2 (D) 150 cm2
kept on all sides, then what % of the area of the room 12. A cylindrical rod of iron whose height is four times
would be occupied by the carpet ? its radius is melted and cast into spherical balls of the
(A) 96% (B) 98% same radius. Then there are—
(C) 60% (D) 84% (A) 5 balls (B) 8 balls
4. A hemisphere of radius 3 cm is cast into a right (C) 9 balls (D) 12 balls
circular cone of height 150 cm. What is radius of the 13. There is a cube of edge length equal to one linear
base of the cone ? unit. Then the distance (shortest) between any two of
(A) 0·3 cm (B) 0·5 cm its vertices can only be—
(C) 0·8 cm (D) 0·6 cm (A) 1 or 
√ 2 linear units

ww
5. A wire in the form of a circle of radius 7 cm is bent
to form a square. What is the area of the square ?
(B) 1, √
 2 or √ 3 linear units
(C) 1 or √ 2, √
 3 or 2 linear units

w
(A) 49 cm2
(C) 176 cm2

.Ea
(B) 121 cm2
(D) 144 cm2
6. The volume of a right circular cone varies as square
of the radius of the base when the height is constant,
(D) None of the above
14. The diameter of hollow cone is equal to the diameter
of a spherical ball. If the ball is placed at the base of

syE
and as the height when the radius is constant. When
the radius of the base is 7 cm and the height 30 cm,
the volume is 1540 cubic cm. Find the height of a
the cone, then what portion of the ball will remain
outside the cone ?
(A) 50% (B) Less than 50%
cone whose volume is 264 cubic cm and which
stands on a base whose radius is 6 cm— ngi (C) More than 50% (D) 100%
1
15. How many lead balls of radius cm can be made out
(A) 28 cm

(C) 21 cm
(B) 14 cm

(D)
5
10
cm nee (A) < 4000
4
of a solid lead sphere of diameter 8 cm ?
(B) > 3000, < 4000
7. Pipe A can fill a tank in 8 hours and pipe B can fill it
in 6 hours. If A, B and an outlet which removes 10
(C) Around 7000
rin (D) > 4000

litres of water in an hour, are opened together, the


tank is filled in 5 hours. The volume of the tank is— g.n
16. A cylindrical container, used for holding petrol, had
a diameter of 16 m and a height of 3 m. The owner
wishes to increase the volume. However, he wishes
(A) 521 litres
(C) 109 litres
(B) Data inadequate
(D) None of these
8. The external dimensions of a closed wooden box are
50 cm, 45 cm, and 35 cm. The wood (of which it is
Thus, X will be—
(A) 16 m
e
to do it such that if X m are added to either the radius
or the height, the increase in volume is the same.

(B) 5·33 m
t
made) is 2·5 cm thick. How many bricks of size 6 cm (C) 6·77 m (D) 3·56 m
× 5 cm × 4 cm can be put in this box ?
17. Mr. Badriprasad has a cylinder and a sphere. The
(A) 190 (B) 230 sphere is such that it perfectly fits in the cylinder
(C) 375 (D) 450 with no parts of its outside. Thus, the ratio of curved
9. There is an error of + 2.5% while measuring the surface area of the cylinder to the surface area of
radius of a sphere. What is the percentage error in inscribed sphere will be—
calculating the volume of the sphere ? (A) Equal to one (B) More than 1
(A) 7·7% (B) 7·5% (C) Less than 1 (D) Equal to two
(C) 3·3% (D) 5·9% 1
18. One cone of height = diameter = and another cone
10. The exact no. of cubes which can be made out of a 2
cuboid of dimensions 65 m × 26 m × 13 m is— of height = diameter = 1 are cut from the opposite
sides of a unit cube with areas at the centre. The
(A) 15 (B) 13 πld
(C) 9 (D) None of these surface area (excluding the base) of a cone is ,
2

CAT Complete Course | 207

Visit : www.EasyEngineering.net
Visit : www.EasyEngineering.net

where is the base diameter and l is the lateral height . 4. (A) Let Breadth = b and Height = h.
The surface area of the resultant cavity is— From question,
π π
(A) 35√ 5 192 (B) 19√ 5 64 (15 × x) 1·50 = 225
225
π π x = = 10 m.
(C) 5
√ 5 16 (D) 9√ 5 32 1·5 × 15
and [2h (l + b) × 6] = 3600
Answers with Hints 12h (l + b) = 3600
Exercise A 12h (15 + 10) = 3600
1. (B) Let AC = 13 m 3600
h = = 12 m.
BC = 5 m 12 × 25
5. (A) Let width of the courtyard = x m.
⇒ AB = √
(AC) 2 – (BC)2
Then, Area of courtyard with path = 60x m2
= √
(13) 2 – (5) 2 Area of courtyard without path = (60 – 4) × (x – 4) m2
= 
√ 169 – 25 = 56 (x – 4) m 2

ww
In IInd case
= 
√ 144 = 12 cm
So, Area of path = [60x – 56 (x – 4)] m2
When width of path is twice, then area of courtyard
without path

w DB =

.Ea =
=



(DE)2 – (BE)2

(13) 2 – (12) 2

√ 169 – 144

= (60 – 12)(x – 12) m2
= 48 (x – 12) m2
Area of path = [60x – 48 (x – 120] m2

So,
= √
AD = AB – DB
syE
25 = 5 cm
By the question,


3x [12x + 576] – 3x [4x + 224] = 1200

= 12 – 5 = 7 m
Diameter of circular windows = 7 m ngi ⇒

12x + 576 – 4x – 224 = 400
8x = 400 – 352

7
So, Radius of window = = 3·5 m
2 nee ⇒
8x = 48
x = 6 m.
So, Area of window = πr2
22  7 
×  =
2
22 7 7 77 2
× × =
6. (C)

rin
Area of ABCD = Area [∆ ABC + ∆ ACD]
ABCD = 2 [∆ ABC]

g.n
= m.
7 2  7 2 2 2 11 + 13 + 20 44
S = = = 22
2 2
2. (B) By the question, difference between circum-
ference of semi-circle and diameter = Distance trav-
elled in 30 seconds

⇒ πr – 2r = 60 ×
30
60
Area =
e

√ S (S – a)(S – b)(S – c)
where a, b, c are the sides length.
= 
√ t
22 × (22 – 11) × (22 – 13) × (22 – 20)
⇒ r (π – 2) = 30 = 
√ 22 × 11 × 9 × 2 = 22 × 3 = 66 cm2
 22  So, area of ABCD = 2 × 66 = 132 cm2 .
⇒ r – 2  = 30
7  7. (C) By the property of parallelogram
8  AB || CD and DA || CB
⇒ r  = 30
7  and ∠A = ∠C
30 × 7 105 ∠B = ∠D
⇒ r = = = 26·25 m.
8 4 ⇒ ∠ C = 55
3. (D) Area of roads and 360° – 110° = 250°
= 5 × 450 + 5 × 200 – (5 × 5) So, ∠ B + ∠ D = 250°
= 2250 + 1000 – 25 Or ∠ B + ∠B = 250°
= 3250 – 25 = 3225 m2 250
⇒ ∠B = = 125°
Cost of graveling = 2·15 × 3225 m2 = 6933·75 m2. 2

208 | CAT Complete Course

Visit : www.EasyEngineering.net
Visit : www.EasyEngineering.net

8. (B) Area of the plot = (12 × 15) m2 ⇒ 22x = 451 – 121


= 180 m2 ⇒ 22x = 330
Area of the flower-bed = (5 × 4·5) m2 = 22·5 m2 330
⇒ x = = 15 m.
22·5 225 75 22
So, Required fraction = = =
180 1800 600 So, Area of square = 15 × 15 = 225 m2
3 1 Cost of covering = 225 × 1·25 = Rs. 281·25
= =
24 8 13. (A) Area of square field = 12100 m2
9. (D) Let the revolutions made by hind wheel = x i.e., x2 = 12100 m2
Then revolution made by fore wheel = x + 2500 x2 = (110) 2 m2
Since, distance covered by both wheels is equal, then Side of the field = 110 m
6x = 4 (x + 2500)
∴ Perimeter = 4 × 110 = 440 m
⇒ 2x = 10000
440
∴ x = 5000 m Required time = hrs = 0·1467 hrs
3000
So, Distance travelled by carriage = 6x
= 8·799 minutes

ww = 6 × 5000 m = 30,000 m
= 30 km.
V r 3
14. (D) S(x) = x2
= 8·8 minutes.
P(x) = 4x

w
W1
10. (A) Since, = 1 = 13 R (x, y) = xy h (x, y) = 2 (x + y)
W2 V2 r 2


9·8
1·4 .Ea
r 3
= 13
r2
So, R (2, 3) = 6, S (4) = 16
P(16 + 6) = P(22) = 4 × 22 = 88

and
r1 3 =
syE
9·8 3
1·4 2
r2 = 2 cm
r H (88, 4) = 2 (88 + 4)
= 2(92) = 184

⇒ r1 3 =
9·8
1·4
× 23
ngi15. (B) S (4) = 16
R (3, 4) = 12


Let required radius = R.
R 3 = r1 3 + r2 3 nee ⇒
P(1) = 4 × 1 = 4, h (2, 3) = 2 (2 + 3) = 10
16 – 12 10
4
×
4
4 10
= × = 2·5
4 4

 98 
=
9·8
1·4
×8+8 16. (B)
rin
R (10, 20) = 200

⇒  14 + 1  × 8 = 8 × 8 = 64
 
g.n
h (10, 20) = 2 (10 + 20) = 60
S(10) = 100, P(20) = 4 × 20 = 80


11. (B)
R3
R
l:b
Length of the field
=
=
=
=
64
4 cm.
4:5
4x


200 – 60 25

100 – 80 10
140
20
e
– 2·5 = 7 – 2·5 = 4·5 t
17. (B) Let, side of smaller square = x cm.
Breadth of the field = 5x
∴ Side of larger square = (x + 10) cm
Area of field = 4x × 5x = 20x2
⇒ 21x2 = 4500 and (x + 10)2 = 9(x2 )
x2 + 100 + 20x = 9x2
⇒ x2 = 225 m2
⇒ x = 15 m 8x2 – 40x + 20x – 100 = 0
So, Length = 4 × 15 = 60 m, 20
⇒ x = 5 or x = –
8
Breadth = 5 × 15 = 75 m
So, perimeter of smaller square = 4x = 20 cm
Length of railing = 2 [l + b]
18. (B) Area of square park = 900 m2
= 2 [60 + 75]
One side = 30 m
= 2 [135] = 270 m.
AB → BD → DC → CA
12. (C) Area of park = 451 m2
⇒ (x + 11)2 – x2 = 451 m2 AC = BD = 30 √
2

CAT Complete Course | 209

Visit : www.EasyEngineering.net
Visit : www.EasyEngineering.net

Total distance = 2 (30 + 30√


 2) 22. (C) The perimeter = 2 [(20 – 8) + (12 – 8)] + 4 × 8
= 2 [12 + 4] + 32
= 60(1 + √  2) = 60 × 2.414
= 2 × 16 + 32 = 64 cm.
= 144.85
23. (A) Volume of spherical ball = volume of three
Speed = 6 kmph
spherical balls.
6 × 1000
4 6  4 2  4 3 
3 3 3
= m/minutes 4
60 ⇒ π  = π   + π   + πx3
3 2  3 2  3 2  3
= 100 m/minute.
144·85 27
∴ Time = = 1·448 minutes. ⇒ 27 = 1 + + x3
100 8
19. (C) Let the radii are be r1 , r2 , r3 respectively.  27 
⇒  27 – 8  = 1 + x3
2πr1 + 2πr3  
⇒ 2πr2 =
2 7 × 27
⇒ = x3 + 1
r + r3 8
⇒ r2 = 1 …(1)
2 189 – 8

ww
Area between a1 & a2 = πr2 2 – πr1 2
Area between a2 & a3 = πr3 2 – πr2 2
⇒ x3 =
8
= (2x)3
= ÷ (required diameter)3


w (r2 2 – r1 2 ) (r2 + r1 )(r2 – r1 )

.Ea
r2 + r1
r3 + r2
=
=
(r3 2 – r2 2 ) (r3 + r2 )(r3 – r2 )
11 + 10 21
=
11 + 12 23
24.
= 8x3 = 8 ×
181
8
= 5·65 cm.

(C) Length of the floor = (20 + 2 × 0·30)


= (20 + 0·60) = 20·60 m

syE
20. (B) Let side of square = x m = Let diameter of circle
= 20
Breadth of the floor = (15 + 2 × 0·30)
= (15 + 0·60) = 15·60 m


4x + πx = 20

4x +
22
x = 20 ngi 25.
∴ Area of floor = (20·60 × 15·60) m2 = 321·36 m2.
(D) Area of the park = 18 × 13 = 234 m2


7
28x + 22x
7
= 20 nee Area of Roads = 18 × 4·3 + 13 × 2·2 – 4·3 × 2·2
= 77·4 + 28·6 – 9·46
= 106·0 – 9·46 = 96·54 m2
⇒ 50x = 20 × 7
rin
∴ Required area of roads = 234 – 96·54

g.n
14 = 137·46 m2 .
⇒ x = m
5 26. (C) Each dimension of the tile must be the H.C.F of
 14  2 196 2 12 and 48, i.e., 12 m.
Now, Area of square =   =
 5  25
m

 r  2 22 7 7 154 2
Area of circle = π   = × × =
 2  7 5 5 25
m 27.
e
So, number of tiles required will be
12 × 48
12 × 12
(B) The perimeter of the plot = 2 (7X + 5X)
= 4.
t
= (24X) metres.
∴ Required area = Area of square – Area of circle ∴ 24X × 2 = 2400
196 22 × 7  196 – 154  2 ⇒
1200
= – = m X =
24
= 50
25 25  25 
So, length of the plot = 7 × 50 = 350 m
42
= = 1·68 m2. and breadth of the plot = 5 × 50 = 250 m.
25
∴ Required difference = 350 – 250 = 100 m.
x  14  1 7
21. (A) Radius of circle = =  = 28. (B) Required area of road = (15 + 16)2 – 152
2 5 2 5
= (31) 2 – 152
and Perimeter of square = 4x
= 961 – 225
14 56
= 4× = m. = 736 sq.m.
5 5
7 56 7 × 5 1 ∴ Total Cost of construction
∴ Required ratio = : = = ·
5 5 5 × 56 8 = 736 × 310 = Rs. 2,28,160

210 | CAT Complete Course

Visit : www.EasyEngineering.net
Visit : www.EasyEngineering.net

 3 a2
√ ⇒ x3 = 1728 – 1216
29. (B) Area of equilateral ∆ =
2 ⇒ x3 = 512 = 8 3
If a is increased by 25%. ⇒ x = 8 cm.
25 a Surface area = 6 × Side2 = 6 × 122
i.e., a× =
100 4 = 144 × 6 = 864 cm2 .
a 5a
i.e., side = a + =
4 4 35. (B) Length of longest rod = √
(152 + 12 2 + 9 2 )

 3 25a2
√ = √
(225 + 144 + 81)
New area =
2 16 = √
450 = 
√
(15) 2 × 2
 3 a2  25 – 1 
√ = 15√ 2 m.
Increment =
2  16  36. (D) Volume of cylinder = πr2 × h = 14850 cm3
 3 a2  25 – 16 
√ ⇒
22 2
× r × 21 = 14850
=
2  16  7
⇒ 66 × r2 = 14850
 3 a2  9 

ww =
2  16 
 3 a2  9 


r2 =
14850
66
= 225

w
r = 15 cm.
2  16  Lateral area = 2πr × h
Increment in % =

.Ea  3 a2

2
× 100
22
= 2 × × 15 × 21
7

30. (D)
=
syE
9
16
× 100 = 56·25%.

Shaded area = A = π (r2 2 – r1 2 )


= 1980 cm2 .
37. (B) (Vol. of solid cylinder) × 0·6 = 6 × Vol. of each
solid sphere
New shaded area = A1 = π [(3r2 )2 – (3r1 )2]
= 9π (r2 2 – r1 2 ) ngi ⇒
4
πr2 h × 0·6 = 6 × × πr1 3
3

∴ Percentage Increase = 1
A –A
A
× 100
nee ⇒ π × 9 × 320 × 0·6 = 6 × π × × r1 3

9 × 32 × 3
4
3

=
9A – A
A
× 100 = 800%.


4
rin = r1 3

r 1 3 = 23 × 33
31. (A) The surface area of cuboid whole length, breadth
and height are l, b, and h respectively. ⇒
g.n
r1 = 6 cm.



= 2 (lb + bh + hl)
= 2 (5 × 3 + 3 × 2 + 2 × 5) m2
= 2 (15 + 6 + 10)
= 2 (21 + 10) = 31 × 2 = 62 m2.
38. (D) Length =
=e 122 + 12 2 + 12 2


= 12√
3
144 + 144 + 144
t
1 2
32. (D) Volume of the speed = (l × b × h) 39. (C) Volume of right circular cone = πr h
3
= (6 × 5 × 0·40) m3
1 22
= (30 × 0·40) = 12 m3 . = × × 25 × 42
3 7
33. (B) The number of bricks are = 22 × 50 = 1100 cm3
Volume of wall 1000 × 500 × 50
= = 40. (C) If edge of cube = a, diameter of sphere = a√
3
Volume of bricks 25 × 12 × 45
2000 × 500 and radius = a
√ 3/2.
= = 1851·85
12 × 45 Now ratio = a3 : 4/3π (a√
 3/2)3 = 1 : π√ 3/2.
= 1852 (approx.). 41. (B) ABC becomes the base circumference of the
34. (B) The volume of the new formed cube cone thus generated and P becomes the vertex. Thus,
= ∑ [Volume of smaller cubes] radius of cone can be found.
⇒ 123 = 63 + 10 3 + x3 Circumference = ABC = 1/4 [2πR]
⇒ 1728 = 216 + 1000 + x3 = 1/4 [2π × 5] = 5/2π cm.

CAT Complete Course | 211

Visit : www.EasyEngineering.net
Visit : www.EasyEngineering.net

Circumference of base = 2R = 5/2π Surface area s = 4πr2


⇒ R (of cone) = 5/4 cm = OA. ⇒
ds
= 8πr
dr
dt dt
Height of cone OP =√
{AP 2 – OA2}
ds 1
⇒ = 8πr
= √
{52 – (5/4)2} dt 4π
⇒ Volume = 1/3 πR2H ds
⇒ = 2r = 2 × 11 = 22 cm2/ min
dt
= 1/3 π(5/4)2 × 
√
{252 – 25/16}
Surfance area of cone 5
= 7·9 cc. 46. (B) =
Surface area of cylinder 6
42. (C) Old Surface area = 6a2 πrll 5
New Surface area = 6 (1·25a)2 ⇒ =
2πrh 6
(New – Old) l 5
Percentage Increase = × 100% ⇒ =
Old h 3
6 (1·25)2 a2 – 6a2 5
= × 100 ⇒ l = h
6a2 3

ww = [(1·25)2 – 1] × 100
= [1·5625 – 1] × 100
= 0·5625 × 100 = 56·25%.
and we know that


l 2 = r 2 + h2
25 2
h = r 2 + h2

w
9
43. (D) Old volume = lbh 25 – 9 2

.Ea
New Volume = 2l × 2·2b × 4·5h
= (29.7) lbh


9
h

16 2
h
= r2

= r2
⇒ % increase =
syE
(New – Old)
Old
(29·7 – 1) Old
× 100%

9
r2
h2
=
16
9
=
Old
= (29·7 – 1) × 100%
× 100%

ngi ⇒
r
h
=
4
3
or 4 : 3.

44. (B)
= 28·7 × 100% = 2870%.
Wire = Cylinder nee
47. (C) Volume of solid cylinder = πr2 h
= π (10)2 × 15 = 1500π


d  2
Volume = πr2 h = π  
2 
h=
πd2h
4
rin π
Volume of frustum = h (r1 2 + r1 r2 + r2 2 )
3

According to question,


πd2h π(0·5) 2 h′ =
π
g.n
Here, r1 = 4 cm, r2 = 10 cm, H = 5 cm

× 5 × (16 + 40 + 100)

e
= 3
4 4


d h = 0·25d2 h′
2

h′
h
=
1
0·25
= 4 = h′ = 4h
=
5π × 156
3
= 260π

and the volume of the cylindrical bore = πr2 h


= π × (4) 2 × 10 = 160π
t
(New – Old)
% increase = × 100%
Old The volume of the remaining material
4h – h
= × 100% = (1500π – (260π + 160π)
h
= (1500 – 420)π
3h
= × 100% = 300% (increment)
h = 1080π.
4 48. (B) Area of the path = Outer rectangle – courtyard
45. (B) V = πr3 h
3 area
dV 4 dr = 22 × 16 – 15 × 9
⇒ = π3r2
dt 3 dt = 352 – 135
dr
⇒ 121 = 4π (17)2 = 217 m2
dt
dr 1 Amount of gravel = (217 × 0·2) m3
⇒ =
dt 4π = 43·4 m3

212 | CAT Complete Course

Visit : www.EasyEngineering.net
Visit : www.EasyEngineering.net

49. (D) Let the cube side = x in. π × 8·4 × 176·4


Required ratio =
Diameter of the sphere = x in. π × 84 × 84
176·4
Now, volume of wood removed = volume of cube – = = 0·21
84 × 10
volume of sphere
4
4 x 
3
π × 93
= x3 – π  2. (B)
Volume of big ball
=
3
3 2  Volume of small ball 4
π × 33
4x3 3
= x3 –
6 = 9 × 3 = 27
 22  Area of floor = 60 × 40
= x3  1 –
3. (D)
 7 × 6 Area of carpet = (60 – 4) × (40 – 4)
 20  = 56 × 36
= x3   56 × 36
 42  % required = × 100 = 84%
40 × 60
According to question, 2 1
π 33 = π × r2 × 150

ww x3 4. (D)
× 20 = 4410 3 3
42 2 × 27
4410 × 42 = r2
⇒ x3 = 150



w .Ea
20
x3 = 21 × 21 × 21
x = 21 in.


r2 =
27 9
75 25
3
=

r = = 0·6 cm.
5

syE
50. (A) The section of the cone through the axis cut the
sphere in a circle which is the inscribed circle of the
equilateral triangle in which it cuts the cone. Hence,
5. (B) Circumference of circle
= 2πr = 2 ×
22
× 7 = 44 cm

ngi
the side of the equilateral is 7
a = 2√  3.r Perimeter of square = 4a = 44
a = 11
where r is the radius of the inscribed circle
= 2√
 3 × 6 ft = 12√3 ft. 6. nee Area of square = 112 = 121 cm2.
(A) Let the height and radius of the base is n and r
Hence, the height of the equilateral triangle is =
√ 3 a
2
respectively in cm.

rin
and let v be the volume in c.c.

=
 3 × 12√ 3 = 18 ft.

2
g.n
Then, v = mr2 h when m is constant by question
1540 = m × 72 × 30

Also, the radius of the base of the cone =

a
=
2
= 6√ 3 ft.
1
2
a


22 2
21
m =
e
rh = v
154
3 × 49

t
By substituting v = 264 and r = 6, h = ?
1
So, volume of cone = πr2 h 22
3 ⇒ 264 = × 62 × h
21
1 22
= × × (6 √ 3)2 × 18 264 × 21
3 7 ⇒ h = = 28 cm.
22 × 36
22
= × 36 × 3 × 6 7. (C) Let the volume of tank = V
7
V
18 × 36 × 22 A's one hour work =
= 8
7
V
= 2036·57 or 2036·6 c.ft (approx.) B's one hour work =
6
Exercise B According to the given condition the volume of the
1. (C) Area of garden = π × 842 tank
Area of garden = (92·4)2 – π × 842 V V 
=  8 + 6 – 10  × 5 = V
= π × 8·4 × 176·4  

CAT Complete Course | 213

Visit : www.EasyEngineering.net
Visit : www.EasyEngineering.net

 3V + 4V – 240  4 3
πr
⇒  ×5 = V Volume of sphere =
3
 24 
12πr3 12 × 3
24V Number of sphere = = = 9 balls.
⇒ 7V – 240 = 4 3 4
5 πr
3
7V 24V
⇒ – = 240 13. (B) Obvious, Any of the adjacent vertex will have
1 5
the distance as 1.
⇒ 11V = 240 × 5
⇒ V = 109 litre. The vertex opposite on the surface will have √

12 + 1 2
=√ 2 units of distance. Also, the body diagonal will
8. (D) Thickness of the wood =
2·5 cm
be √

12 + 1 2 + 1 2 = 
√ 3 units apart.
⇒ Internal length of box =
50 – (5) = 45 cm
Internal breadth of box =
(45 – 5) cm = 40 cm 14. (C) Obviously more than 50% will remain outside.
4 8 
3
Internal height of box =
(35 – 5) = 30 cm
15. (D) Volume of solid lead sphere = π 
⇒ Volume of the box (45 × 40 × 30) cm3
= 3 2 
(6 × 5 × 4) cm3 4 1 
3

ww
Volume of one bricks =
45 × 40 × 30 Volume of 1 lead ball = π  
Number of bricks = = 450. 3 4 
4×5×6
4 3
9. (A)
w 4
V = πr3
3

.Ea
4
V′ = π (1·025r) 3
3
Number of balls =
3
π4

4 1 
π 
3 4 
3

4
syE
= πr3 (1·07689)
3
= 43 × 43 = 64 × 64
= 4096 > 4000.

ngi
V′ = V (1·0769)
16. (B) Volume = πR2 H
V′
⇒ = 1·0769 ⇒ New volume = π.82 . (3 + X)
V
⇒ Error in volume of sphere
= (1 – 1·0769) × 100 nee

= π. (8 + X)2 .3
X = 16/3 = 5·33 m.
= 0·0769 × 100 = 7·67
= 7·7% (approx.) rin
17. (A) Curved area of cylinder = (2πR) × (2R) = 4πR 2 ,
Curved area of sphere = 4πR2 ⇒ Ratio 4πR2 : 4πR2

10. (D) The exact number cubes are got when side of
cube is edge length is HCF of (65, 26, 13)
= 1 : 1.

g.n
Note that the cylinder and sphere have equal radii R

HCF (65, 26, 13) = 13

Then, exact number of cubes =


65 × 26 × 13
(13) 3
= 10.
1 e
and height of cylinder = 2R = Diameter of sphere.

1
18. (D) For smaller cone, r = , h = , l =
4 2
√ 5
4 t
11. (B) Thickness = 2·5 cm. 1 √5
For greater cone, R = , H = 1, L = 
√
R 2 + H2 =
The inner length, inner breadth, and inner height 2 4

= (15 – 5), (13 – 5), (11 – 5) cm. Required surface area = surface area of greater cone
1
= 10 cm, 8 cm, 6 cm. + 2 × · Surface area of smaller cone
4
Then the inner Surface Area
 1 
= 2 [lb + bh + hl] = (π × R × L) +  2 × × πrl 
 4 
= 2[10 × 8 + 8 × 6 + 6 × 10]
 1 √    5 
= 2[80 + 48 + 60] = 2 × 188 =  π × ×  5 +  2 × 1 × π × 1 × √
= 376 cm2 .  2 2  4 4 4 

12. (C)Volume of cylinder = πr2 h  5π + √


√  5π = √  8 + 1  9√
=  5π   =  5π ·
Here, h = 12r 4 32  32  32
⇒ = 12πr3 ●●●

214 | CAT Complete Course

Visit : www.EasyEngineering.net
Visit : www.EasyEngineering.net

14 Linear and Quadratic Equation


Pair of Linear Equations in Two Variables 8. Algebraic methods for the solution of system of
1. An equation that can be written in the form ax + linear equations
by = c, a ≠ 0, b ≠ 0 where a, b and c are real numbers, is (a) Method by Substitution
known as a linear equation in two variables x and y.
Step 1. Find the value of one the variables in terms
2. A solution is a pair of values, one for each
of the other from any one of the given equations.
variable, which satisfies the equation. Every linear
equation in two variables has infinitely many solutions. Step 2. Substitute the value of the variable obtained
3. Two linear equations, each containing the same in step 1 in the other equation.

ww
two unknown variables, e.g.,
a1 x + b1 y = c1
a2 x + b2 y = c2
Step 3. Solve the simple equation obtained in step 2
and find the value of one of the variables.
Step 4. Substitute the value of the variable obtained

w
are said to form a system of simultaneous linear
equations.
.Ea
4. The solution to a system of linear equations is the
values of x and y common to all lines in the system when
in step 3 in any one of the given equations and find the
value of the other variable.
(b) Elimination by Equating the Coefficients

the lines are drawn on a graph.


syE
If a system of equations has at least one solution, it is
said to be consistent.
Step 1. Multiply the equations by the constant and
non-zero numbers so as to make the coefficient of one
variable to be eliminated equal.

ngi
Step 2. Now, if the coefficients of the variable to be
5. When the system has a unique solution, it is called eliminated are having same sign, then subtract the equa-
consistent and the solution has to be given in the form tion obtained in step (1).
x = α, y = β.
● When there is no common solution of the two given
equations, it is called inconsistent. nee Also, if the coefficients of the variable to be
eliminated are having opposite signs, then add the
equation obtained in step (1).
● When the equations have infinitely many common
solutions, it is called system of dependent equations.
rin
Step 3. Solve the equation obtained in step (2) to
obtaine the value of one variable.
6. Graphic Method for the solution of system of linear
equations—
● When line intersect at a single point, the equations g.n
Step 4. Substitute the value of variable obtained in
step 3 in any of the given equations to find the value of

e
the other variable.
have a unique common solution.
● When the lines are parallel, the equations have no
common solution.
● When the lines are coincident, the equations have
infinitely many solutions.
(c) Method of Comparison
Step 1. From each equation, find the value of same
variable (say y) in terms of other.
Step 2. Equate the values thus found in step 1.
t
7. For the given system of linear equations a1 x + b1 y Step 3. Solve the equation obtained as linear equation
+ c 1 = 0, a2x + b2 y + c 2 = 0. in one variable.

↓ ↓
Consistent Inconsistent

→

↓ ↓
a1 b1 a1 b1 c1 a1 b1 c1
Condition → ≠ = = = ≠
a2 b2 a2 b2 c2 a2 b2 c2
↓ ↓ ↓
Graph → Intersecting Lines Coinciding Lines Parallel Lines
(Point of intersection) (Overlapping)
Solution → Unique (One) Solution Many(infinite) Solution No Solution

CAT Complete Course | 215

Visit : www.EasyEngineering.net
Visit : www.EasyEngineering.net

Step 4. Substitute the value of variable obtained in Exercise A


step 3 in given equation to find the value of the other
variable. 1. Solve for x and y—
(d) Method of Cross Multiplication ax + by = a – b
bx – ay = a + b
In the given system of equations in the form of
a1 x + b1 y + c 1 = 0 2. Solve for x and y—
57 6
a1 x + b1 y = c1 + = 5
x+y x–y
⇒ a1 x + b1 y – c 1 = 0 38 21
+ = 9
a2 x + b2 y + c 2 = 0 x+y x–y
a2 x + b2 y = c2 3. Solve for x and y—
⇒ a2 x + b2 y – c 2 = 0 44 30
+ = 10
x+y x–y
Solution of variables x and y can be found with the
55 40
help of following expressions— + = 13
x+y x–y

ww x
=
y
b1 c2 – b2 c1 c1a2 – c2a1
x y
=
1
a1 b2 – a2 b1
–1
4. Solve for x and y—
x y
+ + 1 = 15

w = = 10 5
b1 c2 – b2 c1 c1a2 – c2a1 a1 b2 – a2 b1 x y
Quadratic Equations
.Ea
1. Quadratic polynomial—A polynomial of the
8 6
5. Solve for x and y—
+ = 15

syE
form p(x) = ax2 + bx + c when a ≠ 0 and a, b, c are real
numbers and x is a real variable, is called a quadratic
polynomial. and
2x y
a b
+ = 2
x y
– = 4
2. Quadratic equation—An equation p(x) = 0 where
ngi
p(x) is a quadratic polynomial, is called zeros of quadratic
6. Solve for x and y—
a b

nee
2 2 1
equation i.e., ax 2 + bx + c = 0, where a ≠ 0. + =
x 3y 6
3. Zeros of quadratic equations—Those values of x 3 2
for which ax2 + bx + c = 0 is satisfied are called zeros of and + = 0
quadratic polynomial. If p(α) = aα2 + bα + c = 0, then α
is called the zero of quadratic polynomial.
x y

rin
Hence, find ‘a’ for which y = ax – 4.

4. Roots of quadratic equations—If α, β are the


roots of a quadratic polynomial ax2 + bx + c, then α, β are g.n
7. Places A and B are 80 km apart from each other on a
highway. A car starts from A and another car starts
from B at the same time. If they move in the same
called roots (or solutions) of the corresponding equation
ax2 + bx + c = 0, which implies that p(α) = p(β) = 0
i.e., aα 2 + bα + c = 0 and aβ2 + bβ + c = 0.
5. Solution of a quadratic equation can be found by
e
direction, they meet in 8 hours and if they move in
opposite directions they meet in 1 hour and 20
minutes. Find the speed of the cars.
8. The sum of two digit number and the number formed
t
two methods— by interchanging its digits is 110. If 10 is subtracted
(i) by factorization, (ii) by completion of square. from the first number. The new number is 4 more
than 5 times the sum of digits in the first number.
Note—By completing the square, we get roots as Find the number.
– b +
√
b2 – 4ac – b –√
b2 – 4ac 9. A man travels 370 km partly by train and partly by
α= ,β=
2a 2a car. If he covers 250 km by train and the rest by car,
Where (b2 – 4ac) is called the discriminant, denoted it takes him 4 hours. But, if he travels 130 km by
by ‘D’. train and the rest by car, he takes 18 minutes longer.
Find the speed of the train and that of the car.
6. Nature of roots—
10. A man travels 600 km partly by train and partly by
(i) When D = 0, the roots of quadratic equation are
–b car. If he covers 400 km by train and the rest by car.
real and equal and each = It takes him 6 hours and 30 minutes. But, if he
2a
travels 200 km. by train and the rest by car, he takes
(ii) When D > 0, the roots are real but unequal. half an hour longer. Find the speed of the train and
(iii) When D < 0, the no-real roots are possible. that of the car.

216 | CAT Complete Course

Visit : www.EasyEngineering.net
Visit : www.EasyEngineering.net

11. A part of monthly hostel charges in a college are 21. Taxi charges consist of fixed charges per day and the
fixed and the remaining depend on the number of remaining depending upon the distance travelled in
days, one has taken food in the mess. When a student kilometres. If a person travels 110 km he pays Rs.
A takes food for 20 days, he has to pay Rs. 1000 as 690 and for travelling 200 km he pays Rs. 1050.
hostel charges whereas a student B, who takes food Express the above statements in the form of
for 26 days, pays Rs. 1180 as hostel charges. Find simultaneous equations and hence find the fixed
the fixed charge and the cost of food per day. charges per day and the rate per km.
12. A two digit number is 4 times the sum of its digits. If 22. A number consisting of two digits is seven times the
18 is added to the number the digit is reversed. Find sum of its digits. When 27 is subtracted from the
the numbers. number, the digits are reversed. Find the number.
13. A two digit number is 3 more than 4 times the sum of 23. A number consists of two digits. When it is divided
its digits. If 18 is added to the number the digits are by the sum of the digits, the quotient if 6 with no
reserved. Find the number. remainder. When the number is diminished by 9, the
14. A two digit number is 4 more than 6 time the sum of digits are reversed. Find the number.
digits. If 18 is subtracted from the number the digits 24. There are two classrooms A and B containing
are reversed. Find the number. students. If 5 students are shifted from Room A to

ww
15. The denominator of a fraction is 4 more than twice
the numerator. When both the numerator and the
Room B, the resulting number of students in the two
rooms become equal. If 5 students are shifted from
Room B to Room A, the resulting number of students

w
denominator are decreased by 6, then the denominator
becomes 12 times the numerator. Determine the in Room A becomes double the number of students
number.
.Ea
16. A part of monthly hostel charges in a college are
left in Room B. Find the original number of students
in the two rooms separately.
25. The area of a rectangle gets reduced by 80 sq. units,

syE
fixed and the remaining depend on the number of
days one has taken food in the mess. When a student
X takes food for 25 days, he has to pay Rs. 1750 as
if its length is reduced by 5 units and the breadth is
increased by 2 units. If we increase the length by 10
units and decrease the breadth by 5 units, the areas is
hostel charges whereas a student Y, who takes food
for 28 days, pays Rs. 1900 as hostel charges. Find
the fixed charge and the cost of food per day. ngi increased by 50 sq. units. Find the length and breadth
of the rectangle.

17. The taxi charges in a city comprise of a fixed charge


together with the charge for the distance covered. For nee
26. For what value of k, does the quadratic equation 9x2
+ 8kx + 16 = 0 have equal roots.
27. If roots of the equation (a – b)x2 + (b – c)x + (c – a)
a journey of 10 km the charge paid is Rs. 75 and for
a journey of 15 km the charge paid is Rs. 110. What
will a person have to pay for traveling a distance of rin
= 0 are equal. Prove that 2a = b + c.
28. For what value of k the quadratic equation (k + 4)x2 +
25 km ?
18. The car hire charges in a city comprise of a fixed g.n
(k + 1)x + 1 = 0 has equal roots.
29. Find the value of ‘k’ so that the equation 9x2 – kx +
charge together with the charge for the distance
covered. For a journey of 12 km, the charge paid is
Rs. 89 and for a journey of 20 km the charge paid is
Rs. 145. What will a person have to pay for traveling
81 = 0 has equal roots.
e t
30. If the equation (1 + m 2 )x2 + 2mcx + (c 2 – a2 ) = 0 has
equal roots. Prove that c2 = a2 (1 + m2).
31. Find the value of c for which the quadratic equation
a distance of 30 km ? 4x2 – 2(c + 1)x + (c + 4) = 0 has equal roots.
19. Taxi charges consist of fixed charges and the 32. Find whether the quadratic equation x2 – x + 2 = 0
remaining depending upon the distance traveled in has real roots. If yes, find the roots.
kilometres. If a person travels 10 km he pay Rs. 68
33. If one root of the quadratic equation 2x 2 + ax + 3 = 0
and for traveling 15 km he pays Rs. 98. Express the
is 1, find the other root, and the value of ‘a’.
above statements with the help of simultaneous
equations and hence find the fixed charges and the 34. Find the value of α such that quadratic equation
rate per km. (α – 3)x2 + 4(α – 3)x + 4 = 0 has equal roots.
35. Find the values of k so that quadratic equation x2 –
20. The total expenditure per month of a household
2x(1 + 3k) + 7(3 + 2k) = 0 has equal roots.
consists of a fixed rent of the house and the mess
charges depending upon the number of people 36. Find the values of p for which the x2 + p(4x + p – 1)
sharing the house. The total monthly expenditures is + 2 = 0 has equal roots.
Rs. 3900 for 2 people and Rs. 7500 for 5 people. 5
37. One root of the equation 2x 2 – 8x – m = 0 is . Find
Find the rent of the house and mess charges per head 2
per month. the other root and value of m.

CAT Complete Course | 217

Visit : www.EasyEngineering.net
Visit : www.EasyEngineering.net

38. For what value of k the equation 2kx2 – 40x + 25 = 0 reduced by 100 km/hr and time increased by 30
has equal roots ? Find the roots. minutes. Find the original duration of flight.
39. For what value of k the equation 9x 2 – 24x + k = 0 55. In a flight of 3000 km an aircraft was slowed down
has equal roots ? Find them. due to bad weather. Its average speed for the bad
40. If the list price of a book is reduced by Rs. 5, a person weather. Its average speed for the trip was reduced
can buy 5 more books for Rs. 300. Find the original by 100 km/hr and time increased by one hour. Find
price of the book. the original duration of flight.
41. If the list price of a toy is reduced by Rs. 2, a person 56. In a flight of 6000 km an aircraft was slowed down
can buy 2 toys more for Rs. 360. Find the original due to bad weather. Its average speed for the trip was
price of the toy. reduced by 400 km/hr and time increased by 30
minutes. Find the original duration of flight.
42. Two numbers differ by 3 and their product is 504.
Find the numbers. 57. A rectangular field is 16 m long and 10 m wide.
There is a path of uniform width all around it, having
43. Two numbers differ by 4 and their product is 192, an area of 120 sq.m. Find the width of the path.
find the numbers.
58. X and Y are centers of circles of radius 9 cm and 2
44. Two numbers differ by 2 and their product is 360,
cm respectively and XY = 17 cm Z is the centre of a

ww
find the numbers.
45. Find two consecutive numbers, whose squares have
the sum 85.
circle of radius r cm which touches the above circles
externally. Given that ∠ XZY = 90°, write an equa-

w
tion in r and solve it for r.
46. The sum of two numbers is 15 and sum of their 59. A person on tour has Rs. 360 for his daily expenses.
3

.Ea
reciprocals is . Find the numbers.
10
If he extends his tour for 4 days. He has to cut down
his daily expense by Rs. 3. Find the original duration

syE
47. Rs. 9,000 were divided equally among a certain of the tour.
number of persons. Had there been 20 more persons,
60. A piece of cloth costs Rs. 200 if the piece was 5 m
each would have got Rs. 160 less ? Find the original
longer and each metre of cloth costs Rs. 2 less, the

ngi
number of persons.
cost of the piece would have remained unchanged.
48. A plane left 30 minutes later than the scheduled time How long is the piece and what is its original rate per
and in order to reach the destination 1,500 km away metre?
in time. It has to increase the speed by 250 km/hr
from the usual speed. Find its usual speed.
nee Exercise B

rin
49. The length of the hypotenuse of a right angled 1. Ramesh travels 760 km to his home, partly by train
triangle exceeds the length of the base by 2 cm and and partly by car. He takes 8 hours if he travels 160
exceeds the length of the altitude by 1 cm. Find the

g.n
km by train and the rest by car. He takes 12 minutes
length of each side of the triangle. more if he travels 240 km by train and the rest by
50. Some students planned a picnic. The budget for food car. Find the speed of the train and the car separately.
was Rs. 500. But 5 of these failed to go and thus the
cost of food for each member increased by Rs. 5.
How many students attended the picnic?
51. Two pipes running together can fill a cistern in 6
e
2. A person invested some amount at the rate of 12%
simple interest and some other amount at the rate of
10% simple interest. He received yearly interest of
Rs. 130. But if he had interchanged the amounts
t
minutes. If one pipe takes 5 minutes more than other invested, he would have received Rs. 4 more as
to fill the cistern. Find the time in which each pipe interest. How much amount did he invest at different
would fill the cistern. rates?
8 3. A part of monthly expenses of a family is constant
52. Two pipes running together can fill a cistern in 2
11 and the remaining varies with the price of wheat.
minutes. If one pipe takes 1 minute more than the When the rate of wheat is Rs. 250 a quintal, the total
other to fill the cistern. Find the time in which each monthly expenses of the family are Rs. 1000 and
pipe would fill the cistern. when it is Rs. 240 a quintal, the total monthly
1 expenses are Rs. 980. Find the total monthly
53. Two pipes running together can fill a cistern in 3
13 expenses of the family when the cost of wheat is Rs.
minutes. If one pipe take 3 minutes more than the 350 a quintal.
other to fill the cistern. Find the time in which each 4. The sum of the numerator and the denominator of a
pipe would fill the cistern. fraction is 18. If the denominator is increased by 2,
54. In a flight of 2800 km an aircraft was slowed down 1
the fraction is reduced to · Find the fraction.
due to bad weather, its average speed for the trip was 3

218 | CAT Complete Course

Visit : www.EasyEngineering.net
Visit : www.EasyEngineering.net

5. 2 tables and 3 chairs together cost Rs. 2000, whereas 18. It takes me 8 hours to reach city A from city B. If I
3 tables and 2 chairs together cost Rs. 2500. Find the increase my speed by 6 km per hour. It takes me 1
total cost of 1 table and 5 chairs. hour 20 minutes less. Find the distance between the
6. Five years hence, father’s age will be three times the cities A and B.
age of his son. Five years ago, father was seven times 19. Rs. 6500 were divided equally among a certain
as old as his son. Find their present ages. number of persons. Had there been 15 more persons,
7. A and B each have certain number of oranges. A each would have got Rs. 30 less ? Find the original
says to B, “If you give me 10 of your oranges. I will number of persons.
have twice the number of oranges left with you.” B 20. Two squares have sides x cm and (x + 4) cm. The
replies, “If you give me 10 of your oranges, I will sum of their areas in 656 cm2 . Find the sides of the
have the same number of oranges as left with you.” squares.
Find the number of oranges with A and B separately. 21. The side of a square exceeds the side of another
8. Jai raj travels 300 km to his home partly by train and square by 4 cm and the sum of the area of the two
partly by bus. He takes 4 hours, if he travels 60 km squares is 400 sq.m. Find the dimensions of the two
by train and the remaining distance by bus. If he squares.
travels 100 km by train and the remaining distance
22. The length of a rectangle exceeds its width by 8 cm

ww
by bus, he takes 10 minutes longer. Find the speeds
of the train and the bus separately.
9. Ten years ago, a father was twelve times as old as his
and the area of the rectangle is 240 sq.cm. Find the
dimensions of the rectangle.

w
son and ten years hence, he will be twice as old as

.Ea
his son will be. Find their present ages.
10. The present age of a father is three years more than
three times the age of the son. Three years hence
23. The area of a right angled triangle is 600 sq.cm. If
the base of the triangle exceeds the altitude by 10
cm. Find the dimensions of the triangle.
24. An express train makes a run of 240 km at a certain

syE
father’s age will be 10 years more than twice the age
of the son. Determine their present ages.
11. Solve—
speed. Another train whose speed is 12 km/hr less
takes an hour longer to cover the same distance. Find
the speed of the express train in km/hr.
2
13
(2x + 3y) = 3 +
x–y
4 ngi 25. A train covers a distance of 90 km at a uniform
speed. Had the speed be 15 km/hr more, it would

nee
have taken half an hour less for the journey ? Find
4y + 5x 1
= 2x + 7 the original speed of the train?
3 6
12. Solve for x and y— 26. A train travels a distance of 300 km at a constant

 1
bx + ay = a + b
1  rin
speed. If the speed of the train is increased by 5 km
per hour, the journey would have taken 2 hours less.
ax  – 
a – b a + b 
 1 1 
g.n
Find the original speed of the train.
27. The sum of the squares of two consecutive natural
numbers is 313. Find the numbers.
+ by  –  = 2a
b – a b + a  a + b
13. Find real values of x and y which will make—
(2x – 3y – 13)2 + (3x + 5y + 9)2 = 0
e
28. Divide 29 into two parts so that the sum of the
squares of the parts is 425.
t
29. The speed of a boat in still water is 8 km/hr. It can go
15 km upstream and 22 km downstream in 5 hours.
14. Solve for x and y— Find the speed of the stream.
2x + 3y = 17 30. The sum of two numbers is 48 and their product is
and 2(x + 2) – 3(y + 1) = 5 432. find the numbers.
15. Solve for x and y— 31. The speed of a boat in still water is 15 km/hr it can
x + y – 8 x + 2y – 14 3x + y – 12 go 30 km upstream and return downstream to the
= = original point in 4 hours 30 minutes. Find the speed
2 3 11
of the stream.
16. If two liquids are mixed in the ratio 3 : 2, a mixture is
obtained weighting 1.04 g. per c.c., while if they are 32. Find the whole number which when decreased by 20
mixed in the ratio 5 : 3, the resulting mixture weights is equal to 69 times the reciprocal of the number.
1.05 g per c.c. Find the weight of a c.c. of each of the 33. Find two consecutive natural numbers whose product
original liquids. is 20.
17. A says to B, “I am three times old as you were, when 34. One year ago, the father was 8 times as old as his son.
I was as old as you are.” The sum of their present Now, his age is square of the son’s age. Find their
ages is 64 years. Find their ages. present ages.

CAT Complete Course | 219

Visit : www.EasyEngineering.net
Visit : www.EasyEngineering.net

35. A shopkeeper buys a number of books for Rs. 80. If x y 1


Or = 2 =
he had bought 4 more for the same amount, each (– a2 + b2 ) (a + b2 ) – (a2 + b2 )
book would have cost Rs. 1 less. How many books – (a2 + b2 )
Or x =
did he buy? – (a2 + b2 )
36. If a integer is added to its square, the sum is 90. Find (a2 + b2 )
and y =
the integer with the help of a quadratic equation. – (a2 + b2 )
37. If the roots of the equation (a – b)x2 + (b – c)x + (c – a) Hence, x = 1 and y = – 1
= 0 are equal. Prove that 2a = b + c. 57 6
2. + = 5 …(i)
38. If the roots of (a2 + b2 )x2 + 2(bc – ad)x + (c 2 + d2 ) = 0 x+y x–y
are real and equal. Show that ac + bd = 0. 38 21
+ = 9 …(ii)
x+y x–y
39. If the roots of the equation p(q – r)x2 + q(r – p)x +
1 1 2 Multiplying (i) by 7 and (ii) by 2 and subtracting
r(p – q) = 0 be equal. Show that + = .
p r q 399 42
+ = 35
40. If the equation (1 + m 2 )x2 + 2mcx + (c 2 – a2 ) = 0 has x+y x–y
equal roots. Prove that c2 = a2 (1 + m 2 ). 76 42
+ = 18

ww
41. A cyclist cycles non – stop from A to B a distance of
14 km at a certain average speed. If his average
1
x+y x–y
323
x+y
=
17
1

w
speed reduces by 1 km/hr he takes hrs. more to

.Ea
3
cover the same distance. Find his average original
speed.
17(x + y) = 323
x + y = 19
Putting the value of (x + y) in (i), we get
…(iii)

syE
42. If I had walked 1 km/hr faster, I would have taken 10
minutes less to walk 2 km. Find the rate of my walk- 57 6
+ = 5
ing. 19 x – y
6
43. In a flight of 1600 km aircraft was slowed down by
bad weather. Its average speed for the trip was
reduced by 400 km/hr and the time to flight ngi⇒ 3+
x–y
6
= 5

= 5–3=2
increased by 40 minutes. Find the actual time to
flight. nee

x–y
2(x – y) = 6

rin
x–y = 3 …(iv)
44. In a group of children, each child gives a gift to
every other child. If the number of gifts is 132. Find Adding (iii) and (iv), we get
the number of children.
45. A man purchased number of books at Rs. 720. If the
g.n
2x = 22
x = 11
price of each book were Rs. 2 less, he would, then get
4 more books. How many books he had purchased.
7
46. Out of a group of swans, times the square root of
2

11 + y = 19
e
Putting the value of x in (iii),

y = 19 – 11 = 8
x = 11, y = 8
t
the total number are playing on the share of a pond.
The two remaining ones are swimming in water. 44 30
3. + = 10 …(i)
x+y x–y
Find the total number of swans.
55 40
+ = 13 …(ii)
Answers x+y x–y
Exercise A Multiplying (i) by 4 and (ii) by 3 and subtracting
1. ax + by – (a – b) = 0 176 120
+ = 40
bx – ay – (a + b) = 0 x+y x–y
Using cross multiplication method, we get— 165 120
+ = 39
x+y x–y
x y 1
= =
–b – (a – b ) – (a – b ) – b a 11
→
→

b
→

= 1

x+y

–a

–a – (a + b ) a b

– (a + b )
x y 1 ⇒ x + y = 11 …(iii)
= =
– ab – b2 – a2 + ab – ab – b2 – a2 + ab – a2– b2 Using the value of (x + y) in (i)

220 | CAT Complete Course

Visit : www.EasyEngineering.net
Visit : www.EasyEngineering.net

44 30 2 2 1
+ = 10 6. + = …(i)
11 x – y x 3y 6
30 44 3 2
= 10 – + = 0 …(ii)
x–y 11 x y
30 110 – 44 66 12 4
= = =6 (i) ⇒ + = 1 …(iii)
x–y 11 11 x y
6(x – y) = 30 (Multiplying by ‘6’)
⇒ x–y = 5 …(iv) 12 8
Now, (ii) ⇒ + = 0 …(iv)
Adding (iii) and (iv), we get x y
2x = 16 (Multiplying by ‘4’)
–4
x = 8 (iii) – (iv) ⇒ = 1 or y = – 4
y
Putting the value of x in (iii)
Substitute y = – 4 in (ii), we get
8 + y = 11
3 2
y = 11 – 8 = 3 + = 0
x –4
4.
ww x y
+ + 1 = 15
10 5
x y
…(i) ⇒
3 1
– = 0
x 2
3 1

w
+ = 15 …(ii) or =
8 6 x 2

.Ea
Multiplying (i) by 10 and (ii) by 24, we get—
x + 2y + 10 = 150
or
Also,
x = 6
y = ax – 4
or
and
syE
x + 2y = 140
3x + 4y = 360
3 × (iii) ⇒ 3x + 6y = 420
…(iii)
…(iv)
…(v)
or
or
– 4 = 6a – 4
6a = 0
a = 0
(iv) – (v) – 2y = – 60
y = 30 ngi 7. Let the speed of car, starts from A = x km/hr
and the speed of car, starts from B = y km/hr

and
x + 2y = 140
y = 30 nee According to the question,
8x = 8y + 80 (same direction)


x + 60 = 140
x = 80 4 4 rin{Q Distance = Speed × Time}

5.
2x y
a b
+ = 2 …(i)
and
3
x + y = 80
3
g.n

(opposite direction)

20 4 

By adding
x y
– = 4
a b
3x
a
= 6
…(ii)

4x + 4y = 240
4(y + 10) + 4y = 240
e
 1 hr. 20 min. = 1 60 = 3 hr. 

t
[From (i)]
⇒ 3x = 6a
4y + 40 + 4y = 240
⇒ x = 2a
4y + 4y = 240 – 40
Putting the value of x = 2a in (i), we get
8y = 200
 2a  y
2 + = 2 ⇒ y = 25
a  b
y Taking value of y in (i), we get
⇒ 4+ = 2
b x = 25 + 10 = 35
y ∴ Speeds of cars are 35 km/hr and 25 km/hr.
⇒ = 2–4
b
8. Let unit’s place digit be x
y
⇒ = –2 and ten’s place digit be y.
b
⇒ y = – 2b Then, original numbers—
⇒ x = 2a, y = – 2b Ist number = x + 10y
You may also use cross multiplication method. And Reversed number = 10x + y

CAT Complete Course | 221

Visit : www.EasyEngineering.net
Visit : www.EasyEngineering.net

According to the question, 10. Let the speed of the train = x km/hr
x + 10y + 10x + y = 110 and the speed of the car = y km/hr
11x + 11y = 110 Accroding to the question,
x + y = 10 …(i) [÷ by 11] 400 200 13
⇒ + = hr. …(i)
x + 10y – 10 = 5(x + y) + 4 x y 2
5x + 5y + 4 – x – 10y + 10 = 0  Distance 
 Time = Speed 
4x – 5y = – 14 …(ii)
Multiplying (i) by 5 and adding to (ii), we get 200 400
+ = 7 hrs. …(ii)
x y
5x + 5y = 50
Multiplying (i) by 2 and subtracting (ii) from it
4x – 5y = – 14
800 400
9x = 36 + = 13
x y
x = 4 200 400
+ = 7
Putting the value of x in (i), we get x y

ww 4 + y = 10
y = 10 – 4 = 6

600
x
= 6

6x = 600

w
Original number = x + 10y
= 4 + 10(6) ⇒ x = 100

.Ea = 64
9. Let the speed of the train = x km/hr
Putting the value of x in (ii), we get
200 400

syE
+ = 7
100 y
and the speed of the car = y km/hr
400
According to the question, = 5
y
250 120
x
+
y
= 4

…(i)

Distance 
ngi ⇒
5y = 400

y =
400

130 240 18
 Q Time = Speed 
nee ⇒ y = 80
5

rin
+ = 4+ Speed of the train = 100 km/hr
x y 60
and Speed of the car = 80 km/hr.
43

g.n
= …(ii) 11. Let the fixed hostel charges be Rs. x
10
Multiplying (i) by 2 and subtracting (ii) from it. and the cost of food per day be Rs. y
500 240
x
+
130 240
x
+
y

y
= 8

=
43
10
According to the question,

Subtracting,
x + 20y = 1000
x + 26y = 1180
e
– 6y = – 180
t
…(i)
…(ii)

370 37
= ⇒ y = 30
x 10
⇒ 37x = 3700 Putting the value of y in (i)
⇒ x = 100 x + 20(30) = 1000
Putting the value of x in (i) x + 600 = 1000
250 120 x = 400
+ = 4
100 y The fixed charge = Rs. 400
120
= 4 – 2·5 and the cost of food per day = Rs. 30
y
= 1·5 12. Let ten’s digit be = x
1·5y = 120 and Unit’s digit be = y
⇒ y = 80 Number formed = 10x + y
∴ Speed of train = 100 km/hr If digits are reversed, then number formed
and Speed of car = 80 km/hr. = 10y + x

222 | CAT Complete Course

Visit : www.EasyEngineering.net
Visit : www.EasyEngineering.net

Number = 4(sum of digits) and 10x + y – 18 = 10y + x


10x + y = 4(x + y) or 4x = 4 + 5y
10x + y = 4x + 4y and x–2 = y
6x = 3y x
15. Let the fraction be =
y = 2x …(i) y
Number + 18 = Number with reversed digits According to the conditions—
10x + y + 18 = 10y + x y = 2x + 4 and y – 6 = 12(x – 6)
9x – 9y + 18 = 0 So, 2x + 4 – 6 = 12(x – 6)
(by putting the value of y)
x–y+2 = 0 …(ii)
⇒ 12x – 2x = 72 – 2 ⇒ x(numerator) = 7
From (i) taking the value of y in (ii), we get
Y(denominator) = 18 (by putting the value of x)
x – 2x + 2 = 0
7
–x = –2 Hence, the fraction is ·
18
⇒ x = 2 16. Let the fixed hostel charges be Rs. x

ww
Hence,
y = 2x
= 2×2=4
Number = 10 × 2 + 4 = 24
and the cost of food per day be Rs. y.
According to the question—

w
13. Let the digit at ten’s place = x

.Ea
and the digit at unit’s place = y
Number formed = 10x + y
Subtracting

x + 25y = 1750
x + 28y = 1900
– 3y = – 150
y = 50
…(i)
…(ii)

syE
When digits are reversed, then new number = 10y + x
According to the question,
Putting y = 50 in (i), we get
x + 25 (50) = 1750

ngi
10x + y = 3 + 4(x + y) x + 1250 = 1750
10x + y = 3 + 4x + 4y x = 500

nee
6x = 3 + 3y Fixed charges = Rs. 500
⇒ 2x = 1 + y and Cost of food = Rs. 50 per day
⇒ y = 2x – 1 …(i)

rin
17. Let fixed charge of taxi = Rs. x
Also, 10x + y + 18 = 10y + x and running charges of taxi = Rs. y per km.
⇒ 9x + 18 = 9y
⇒ x+2 = y
Using the value of y in (ii), we get
…(ii)
g.n
According to the question—
x + 10y = 75 …(i)

x + 2 = 2x – 1
2 + 1 = 2x – x
3 = x
Subtracting,

x + 15y = 110
– 5y = – 35
y = 7
e
Putting the value of y in (i)
…(ii)

t
x = 3 x + 70 = 75
Now, y = x+2 x = 5
⇒ y = 3+2=5 ∴ Person has to pay for traveling a distance of 25 km.
So, the number = 10x + y = x + 25y
= 10 × 3 + 5 = 30 + 5 = 35 = 5 + 25 (7)
14. Let ten’s digit = x = 5 + 175
Unit’s digit = y
= Rs. 180
Number formed = 10x + y
18. Let fixed charges of taxi = Rs. x
Sum of digits = x + y
The number formed when digits are interchanged and running charges of taxi = Rs. y per km
= 10y + x According to the question—
According to the conditions— x + 12y = 89 …(i)
10x + y = 4 + 6(x + y) x + 20y = 145 …(ii)

CAT Complete Course | 223

Visit : www.EasyEngineering.net
Visit : www.EasyEngineering.net

Subtracting, – 8y = – 56 22. Let unit’s place digit = x


y = 7 and ten’s place digit = y
Putting the value of y in (i) Original Number = x + 10y
x + 84 = 89 Reversed Number = 10x + y
⇒ x = 5 According to the question—
Now, person has to pay for traveling a distance of x + 10y = 7(x + y)
30 km. ⇒ x + 10y = 7x + 7y
= x + 30y ⇒ 10y – 7y = 7x – x
= 5 + 30 (7) ⇒ 3y = 6x
= 5 + 210 ⇒ y = 2x …(i)
= Rs. 215 and x + 10y – 27 = 10x + y
19. Let fixed charges of taxi = Rs. x x + 10y – 10x – y = 27
and running charges of taxi = Rs. y per km. – 9x + 9y = 27
According to the question—

ww x + 10y = 68
x + 15y = 98
…(i)
…(ii)



–x+y = +3
– x + 2x = 3
x = 3
(Dividing by 9)
[Q y = 2x from (i)]

w
Subtracting, – 5y = – 30

.Ea
y = 6
Putting the value of y in (i), we get
Putting x = 3 in (i), we get
y = 2x
y = 2×3=6

x = 8
syE
x + 10y = 68

20. Let the monthly rent of the house = Rs. x


∴ Original number = 3 + 60 = 63
23. Let unit’s place digit = x

and the mess charges per head per month = Rs. y


According to the question— ngi and ten’s place digit = y
Original Number = x + 10y

x + 2y = 3900
x + 5y = 7500
…(i)
…(ii) nee Reversed Number = 10x + y
According to the question—
x + 10y 6
By subtracting, – 3y = – 3600
y = 1200 ⇒
x+y
rin=
1
6x + 6y = x + 10y
Putting the value of y in (i)
x + 2400 = 3900
⇒ 6x – x + 6y – 10y = 0
⇒ g.n
5x – 4y = 0
x = 3900 – 2400
x = Rs. 1500
∴ Monthly rent (x) = Rs. 1500

⇒ y =
5x
4
e
– 4y = – 5x

…(i)
t
Mess charges per head per month (y) = Rs. 1200 x + 10y – 9 = 10x + y
21. Let the fixed charges of taxi per day = Rs. x x + 10y – 10x – y = 9
and the running expenses of taxi = Rs. y per km. ⇒ – 9x + 9y = 9
According to the question— ⇒ –x+y = 1 (Dividing by 9)
x + 110y = 690 …(i)
5x  5x 
x + 200y = 1050 …(ii) ⇒ –x+ = 1  Q y = 4 from (i) 
4
Subtracting, – 90y = – 360
⇒ x = 4
⇒ y = 4 Putting x = 4 in (i), we get
Putting the value of y in (i) 5x
x + 110 (4) = 690 y =
4
x + 440 = 690 5×4
y = =5
x = 690 – 440 = Rs. 250 4
∴ Fixed charges (x) = Rs. 250 ∴ Original number = 4 + 10(5)
and Rate per km (y) = Rs. 4 = 4 + 50 = 54

224 | CAT Complete Course

Visit : www.EasyEngineering.net
Visit : www.EasyEngineering.net

24. Let the number of students in class A = x and in 28. a = k + 4, b = k + 1, c = 1


class B = y D = b2 – 4ac
According to the Ist condition— = (k + 1)2 – 4 × (k + 4)
x–5 = y+5 = k2 + 1 + 2k – 4k – 16
⇒ x – y = 10 …(i) = k2 – 2k – 15
According to the IInd condition— For equal roots. D = 0
x + 5 = 2(y – 5) k2 – 2k – 15 = 0
⇒ x – 2y = – 15 …(ii) (k – 5) (k + 3) = 0
(10 + y) – 2y = – 15 (by putting x = 10 + y) k = 5, – 3
y = 25 and x = 35 29. a = 9, b = – k, c = 81
In room A original number of students = 35 : in room D = b2 – 4ac
B original number of students = 25
= k2 – 4 × 9 × 81
25. Let length = x units and Breadth = y units
= k2 – 2916
Then area of rectangle = x y sq. units

ww
According to the Ist condition—
xy – (x – 5) (y + 2) = 80
For equal roots. D = 0
k2 – 2916 = 0
k = ±√ 

w
2916
5y – 2x = 70 …(i)
k = ± 54

(x + 10) (y – 5) – xy = 50
.Ea
According to the IInd condition—
30. a = 1 + m , b = 2mc, c = c2 – a2
2

D = b2 – 4ac
xy – 5x + 10y – 50 – xy = 50
or 2y – x = 20
Multiplying (ii) by 2
syE …(ii)
= (2mc)2 – 4 × (1 + m2 ) (c 2 – a2 )
= 4m2c2 – 4 × (c2 – a2 + m2c2 – m2a2 )

⇒ 4y – 2x = 40 and subtract from eq. (i)


We get y = 30 ngi = 4m2c2 – 4c2 + 4a2 – 4m2c2 + 4m2a2
= 4a2 + 4m2a2 – 4c2

Putting y = 30 in equation (ii), So, x = 40


Hence, length is 40 units and breadth is 30 units. nee
For equal roots. D = 0
⇒ 4a2 + 4m2a2 – 4c2 = 0
26. a = 9, b = 8k, c = 16
D = b2 – 4ac

⇒ a2 +
rin
a2 + m2a2 – c2 = 0
m 2 a2 = c2
[Dividing by 4]

= (8k)2
= 64k2 – 64 × 9
– 4(9) × (16) ⇒ a2 (1 + m2 )
g.n
= c2
31. a = 4, b = – 2(c + 1), c = c + 4
For equal roots. D = 0
64k2 – 64 × 9 = 0
64k2 = 64 × 9
D = b2 – 4ac
e t
= [ – 2(c + 1)]2 – 4 × 4 × (c + 4)
= 4(c2 + 1 + 2c) – 16c – 64
k2 = 9 ⇒ k = ± 3 = 4c2 + 4 + 8c – 16c – 64
27. A = a – b, B = b – c, C = c – a = 4c2 – 8c – 60 = 4(c 2 – 2c – 15)
D = B 2 – 4AC For equal roots. D = 0
= (b – c)2 – 4× (a – b) × (c – a) 4(c2 – 2c – 15) = 0
= b2 + c2 – 2bc – 4(ac – a2 – bc + ab) (c – 5) (c + 3) = 0
= b2 + c2 – 2bc – 4ac + 4a2 + 4bc – 4ab c = 5, – 3
= b2 + c2 + 4a2 + 2bc – 4ac – 4ab 32. a = 1, b = – 1, c = 2
= (2a – b – c)2 D = b2 – 4ac
For equal roots. D = 0 = ( – 1) 2 – 4 × 1 × 2
(2a – b – c)2 = 0 = 1–8
2a – b – c = 0 = –7
2a = b + c As D < 0, Roots are not real.

CAT Complete Course | 225

Visit : www.EasyEngineering.net
Visit : www.EasyEngineering.net

33. 2x2 + ax + 3 = 0 …(i) 36. Here, a = 1, b = 4p, c = p2 – p + 2


As ‘1’ is a root, therefore it must satisfies the equa- D = b2 – 4ac
tion. = (4p) 2 – 4 × (p2 – p + 2)
2(1) 2 + a(1) + 3 = 0 = 16p2 – 4p2 + 4p – 8
a+5 = 0 = 12p2 + 4p – 8
a = –5 For equal roots. D = 0
Put a = – 5 in (1) 12p2 + 4p – 8 = 0
2
2x – 5x + 3 = 0 12p2 + 12p – 8p – 8 = 0
2x2 – 3x – 2x + 3 = 0 12p(p + 1) – 8(p + 1) = 0
x(2x – 3) – 1(2x – 3) = 0 (12p – 8) (p + 1) = 0
(x – 1) (2x – 3) = 0 2
p = – 1,
3 3
x = 1,
2 2
37. The given equation is 2x – 8x – m = 0.
3 5

ww
Other root is . As is a root of the equation, therefore it must satis-
2 2
34. Here, a = α – 3, b = 4(α – 3). fies the equation.
5  2 5 
w
The quadratic equation will have equal roots.
⇒ 2 ×  – 8 × – m = 0
If b2 = 4ac 2  2 

⇒ .Ea
[4(α – 3)2] = 4(α – 3) × 4
(4) 2 × (α – 3)2 = (4) 2 × (α – 3)

25
2
– 20 – m = 0


⇒ (α – 3) – (α – 3) = 0
2 syE
(α – 3)2 = α – 3 ⇒

25 – 40 – 2m
– 15 – 2m
= 0
= 0
⇒ (α – 3) (α – 3 – 1) = 0
⇒ (α – 3) (α – 4) = 0
ngi ⇒ – 2m

m =
= 15
– 15
2

or
α = 3
α = 4
But α = 3 is not possible since it reduces the equation
nee The equation is 2x 2 – 8x +
15
2

to a constant. Hence, α = 4.
35. Equation is x2 – 2x(1 + 3k) + 7(3 + 2k) = 0 rin
4x2 – 16x + 15 = 0
4x2 – 10x – 6x + 15 = 0

Here, a = 1,
b = – 2(1 + 3k), g.n
2x(2x – 5) – 3(2x – 5) = 0
(2x – 3) (2x – 5) = 0

Given that the equation has equal roots.



c = 7(3 + 2k)

b2 – 4ac = 0
3
Other root is .
2
e x = ,
3 5
2 2
t
⇒ 4(1 + 3k) – 4 × 1 × 7 × (3 + 2k) = 0
2
38. Here, a = 2k, b = – 40, c = 25
⇒ 4(1 + 9k2 + 6k) – 28(3 + 2k) = 0
D = b2 – 4ac
⇒ 4 + 36k2 – 24k – 84 – 56k = 0
= (– 40)2 – 4 × 2k × 25
⇒ 36k2 – 32k – 80 = 0
= 1600 – 200k
⇒ 9k2 – 8k – 20 = 0 [Dividing
by 4] For equal roots. D = 0
⇒ 2
9k – 18k + 10k – 20 = 0 1600 – 200k = 0
⇒ 9k(k – 2) + 10(k – 2) = 0 k = 8
⇒ (9k + 10) (k – 2) = 0 ∴ Equation is 16x – 40x + 25 = 0
2

⇒ 9k = – 10 (4x – 5)2 = 0
or k = 2 5 5
x = ,
– 10 4 4
⇒ k =
9 5 5
Roots are , ·
or, k = 2 4 4

226 | CAT Complete Course

Visit : www.EasyEngineering.net
Visit : www.EasyEngineering.net

39. a = 9, b = – 24, c = k 42. Let the numbers be x and y.


D = (– 24)2 – 4 × 9 × k x–y = 3 …(i)
= 576 – 36k and x y = 504 …(ii)
For equal roots. D = 0 From (i), x = 3+y Put in (ii)
576 – 36k = 0 (3 + y)y = 504
k = 16 2
y + 3y – 504 = 0
2
Equation is 9x – 24x + 16 = 0 2
y + 24y – 21y – 504 = 0
(3x – 4)2 = 0 (y – 21) (y + 24) = 0
4 4 y = 21, – 24
x = ,
3 3 When y = 21, x = 24 and
4 4
Roots are , · When y = – 24, x = – 21
3 3
43. Let the numbers be x and y (Let x > y).
40. Let the original price of book = Rs. x
x–y = 4 …(i)
300

ww No. of books in Rs. 300 =


x
Reduced price of book = Rs. (x – 5)
and
From (i)
Put
xy = 192
x = 4+y
x = (4 + y) in (ii)
…(ii)

w
Then,

Given
No. of books =

.Ea
300 300
x–5 x

300
x–5

= 5 2
2
(4 + y)y = 192
y + 4y – 192 = 0
y + 16y – 12y – 192 = 0


x(x – 5)
syE
300x – 300(x – 5)

300x – 300x + 1500


= 5
(y + 16) (y – 12) = 0
y = – 16, 12

x(x – 5)
= 5

1500 = 5 (x2 – 5x) ngi When y = – 16, x = – 12


and y = 12, x = 16

x2 – 5x – 300 = 0
x2 – 20x + 15x – 300 = 0 nee
44. Let the numbers be x and y.
Given x–y = 2 …(i)

(x – 20) (x + 15) = 0
x = 20, – 15
and
From (i)
rin
x y = 360
x = 2+y
…(ii)

Price can never be – ve


Price = Rs. 20.
Put
g.n
x = (2 + y) in (ii), we get
(2 + y)y = 360

41. Let the original price of the toy = Rs. x

No. of toys in Rs. 360 =


360
x
2y + y 2 = 360
y2 + 2y – 360 = 0
(y + 20) (y – 18) = 0
e t
y = 18, – 20
Reduced price of toy = Rs. (x – 2)
When y = 18, x = 20
360
Then, No. of toys = When y = – 20, – 18
x–2
360 360 45. Let two consecutive numbers be x and x + 1.
Given – = 2
x–2 x According to the question—
360x – 360(x – 2) x2 + (x + 1)2 = 85
⇒ = 2
x(x – 2) x2 + x2 + 2x + 1 – 85 = 0
360x – 360x + 720 = 2x2 – 4x 2x2 + 2x – 84 = 0
x2 – 2x – 360 = 0 x2 + x – 42 = 0 [Dividing by 2]
(x + 18) (x – 20) = 0 2
x + 7x – 6x – 42 = 0
x = – 18, 20 x(x + 7) – 6(x + 7) = 0
Price can never be – ve (x – 6) (x + 7) = 0
Original price of the toy = Rs. 20. x–6 = 0

CAT Complete Course | 227

Visit : www.EasyEngineering.net
Visit : www.EasyEngineering.net

or x+7 = 0 1500x + 375000 – 1500x 1


=
x = 6 x(x + 250) 2
or x = –7 ⇒ x(x + 250) = 750000
When x = 6, numbers are 6 and 7 ⇒ x2 + 250x – 750000 = 0
When x = – 7, numbers are – 7 and – 6 ⇒ x2 + 1000x – 750x – 750000 = 0
46. Let two numbers be x and (15 – x). ⇒ x(x + 1000) – 750(x + 1000) = 0
According to the question— ⇒ (x – 750) (x + 1000) = 0
1 1 3 ⇒ x = 750 or x = – 1000
+ =
x 15 – x 10 But speed can never be – ve
15 – x + x 3 Usual speed = 750 km/hr.
⇒ =
x(15 – x) 10 49. Let length of base = x cm
⇒ 150 = 3x(15 – x) Hypotenuse = (x + 2) cm
⇒ 150 = 45x – 3x2 Altitude = (x + 1) cm
⇒ 3x2 – 45x + 150 = 0 According to Pyth. Theorem

⇒ ww x2 – 15x + 50 = 0
x2 – 5x – 10x + 50 = 0
[Dividing by 3] P 2 + B2 = H 2
x + (x + 1)2 = (x + 2)2
2



w
⇒ x (x – 5) – 10 (x – 5) = 0

.Ea
(x – 5) (x – 10) = 0
x = 5 or x = 10
When x = 5, numbers are 5 and 10
x2 + x2 + 1 + 2x = x2 + 4 + 4x
x2 – 2x – 3 = 0
(x – 3) (x + 1) = 0
x = 3, – 1

syE
When x = 10, numbers are 10 and 5.
47. Let original number of persons = x
Length can never be – ve.
x = 3 cm
The increased number of persons = x + 20
Total Amount = Rs. 9,000
ngi Here, Base = 3 cm
Altitude = (x + 1) = (3 + 1) = 4 cm

nee
According to the question— Hypotenuse = 5 cm.
9000 9000  Total Amount  50. Let the no. of students who attended picnic = x
x

x + 20
= 160  Each got = No. of Persons  Nos of students who planned picnic = (x + 5)
9000x + 180000 – 9000x
(x + 20)
=
160
1 rin
Total Budget = Rs. 500
According to the question—


160(x + 20) = 180000
x(x + 20) = 1125

500 500
x

g.n
x+5
= Rs. 5

Total Budget 
⇒ 2
x + 20x – 1125 = 0
⇒ x2 + 45x – 25x – 1125 = 0
⇒ x(x + 45) – 25(x + 45) = 0
[Dividing by 160]

500x + 2500 – 500x


x(x + 5)
e= 5 t
 Each contribution = No. of Students 

⇒ (x – 25) (x + 45) = 0 ⇒ 5x(x + 5) = 2500


⇒ x = 25 ⇒ 5x2 + 25x – 2500 = 0
or x = – 45 ⇒ x2 + 5x – 500 = 0 [Dividing by 5]
But number of persons can’t be – ve ⇒ x2 + 25x – 20x – 500 = 0
∴ The original number of persons = 25 ⇒ x(x + 25) – 20(x + 25) = 0
48. Let the usual speed of plane = x km/hr ⇒ (x + 25) (x – 20) = 0
The increased speed of plane = (x + 250) km/hr ⇒ x + 25 = 0
Distance = 1500 km or x – 20 = 0
According to the question— ⇒ x = – 25
1500 1500 1 or x = 20
– =
x x + 250 2 But number of students cannot be – ve.
 Distance  ∴ x = 20
 Time = Speed 
Nos. of students who attended picnic = 20

228 | CAT Complete Course

Visit : www.EasyEngineering.net
Visit : www.EasyEngineering.net

51. Let the two pipes fill the cistern = x and x + 5 41 ± √



(41) 2 + 4 × 13 × 120
minutes. ⇒ x =
26
1 1 1
Then, + = 41 ± √

1681 + 6240
x x+5 6 =
26
⇒ 6(x + 5) + 6x = x(x + 5)
41 ± √

7921
⇒ 6x + 30 + 6x = x2 + 5x =
26
⇒ x2 + 5x – 6x – 30 – 6x = 0
41 ± 89
⇒ x2 – 7x – 30 = 0 =
26
⇒ x2 – 10x + 3x – 30 = 0 130 48
= or
⇒ x(x – 10) + 3(x – 10) = 0 26 26
⇒ (x – 10) (x + 3) = 0 = 5 or –
24
13
⇒ x = 10 or x = – 3
Since, time cannot be – ve.
Since, time cannot be – ve.
∴ x = 5 minutes
∴ The pipes would fill the cistern in 10 and 10 + 5

ww
i.e., 15 minutes.
52. Let the time taken by one pipe = x minutes
∴ Time taken by Ist pipe = 5 minutes
and time taken by IInd pipe = 5 + 3 = 8 minutes

w Time taken by other pipe = x + 1 minutes 54. Let usual speed of aircraft = x km/hr
1 1 11 Reduced speed of aircraft = (x – 100) km/hr
Then,

.Ea +
x x+1
=
30
 8 30 
Distance = 2800 km
According to the question—

⇒ syE
30(x + 1) + 30x =
 Q 2 11 = 11 
11x(x + 1)
2800 2800
x – 100

x
= 30 minutes

 Distance 
⇒ 30x + 30 + 30x
⇒ 11x – 30x – 30x + 11x – 30
2
=
=
11x2 + 11x
0
ngi  Q Time = Speed 

nee
⇒ 11x2 – 49x – 30 = 0 2800x – 2800x + 280000 1
= hr.
x(x – 100) 2
49 ± √

(49) 2 + 4 × 11 × 30
⇒ x = ⇒ x2 – 100x – 560000 = 0

rin
22
⇒ x2 – 800x + 700x – 560000 = 0
49 ± √

2401 + 1320 49 ± √ 
3721
= = ⇒ x(x – 800) + 700(x – 800) = 0

=
22
22
49 ± 61 110
=
22
or –
12
22
22

= 5 or –
6
11

⇒ g.n
(x – 800) (x + 700)
x
=
=
0
800 or x = – 700
Since, time cannot be – ve.
∴ Time taken by Ist pipe = 5 minutes
and time taken by IInd pipe = 5 + 1 = 6 minutes
Q Speed can never be – ve.
Q Speed = 800 km/hr
D 2800 7
e 1
t
Time = = = = 3 hours.
S 800 2 2
53. Let the two pipes be A and B.
55. Let usual speed of aircraft = x km/hr
Let the time taken by pipe A = x minutes
Reduced speed of aircraft = (x – 100) km/hr
and time taken by pipe B to fill = x + 3 minutes
Distance = 3,000 km
As per the question,
According to the question—
1 1 13
+ = 3000 3000
x x+3 40 – = 1 hr
x – 100 x
 1 40  
 Q 3 13 = 13  Distance 
 Q Time = Speed 
⇒ 40(x + 3) + 40x = 13x(x + 3) 3000x – 3000x + 300000 1
=
⇒ 40x + 120 + 40x = 13x2 + 39x x(x – 100) 1
⇒ 13x2 + 39x – 40x – 120 – 40x = 0 ⇒ x(x – 100) = 300000
⇒ 13x2 – 41x – 120 = 0 ⇒ x2 – 100x – 300000 = 0

CAT Complete Course | 229

Visit : www.EasyEngineering.net
Visit : www.EasyEngineering.net

⇒ x2 – 600x + 500x – 300000 = 0 Now, XZ2 + ZY2 = XY2


⇒ x(x – 600) + 500(x – 600) = 0 ⇒ (9 + r)2 + (2 + r)2 = (17) 2
⇒ (x – 600) (x + 500) = 0 ⇒ 81 + r2 + 18r + 4 + r2 + 4r = 289
⇒ x = 600 ⇒ 2r2 + 22r + 85 = 289
or x = – 500 ⇒ 2r2 + 22r = 289 – 85 = 204
∴ Speed can never be – ve. ⇒ r2 + 11r = 102
∴ Speed = 600 km/hr ⇒ r2 + 11r – 102 = 0
D 3000 ⇒ r2 – 6r + 17r – 102 = 0
Time = = = 5 hours.
S 600 ⇒ r(r – 6) + 17(r – 6) = 0
56. Let usual speed of aircraft = x km/hr ⇒ (r – 6) (r + 17) = 0
Reduced speed of aircraft = (x – 400) km/hr ⇒ r = 6 or r = – 17
Distance = 6,000 km Rejecting the – ve value, we get
According to the question— r = 6 cm.
6000 6000

ww x – 400

x
= 30 minutes

 Distance 
 Q Time = Speed 
59. Let usual number of days of tour = x
After increasing number of days = (x + 4)
Total amount to be spent = Rs. 360

w .Ea
6000x – 6000x + 2400000
x(x – 400)
1
= hr
2
According to the question—
360 360
x

x+4
= Rs. 3



syE
x2 – 400x = 4800000
x2 – 400x – 4800000 = 0
⇒ x2 – 2400x + 2000x – 4800000 = 0

360x + 1440 – 360x
x(x + 4)
=
3
1
1440 = 3x2 + 12x

⇒ x (x – 2400) + 2000 (x – 2400) = 0



ngi ⇒

3x2 + 12x – 1440 = 0
x2 + 4x – 480 = 0 [Dividing by 3]

nee
(x – 2400) (x + 2000) = 0
⇒ 2
x + 24x – 20x – 480 = 0
⇒ x = 2400 or x = – 2000
⇒ x(x + 24) – 20(x + 24) = 0
∴ Speed can never be – ve.
∴ Speed = 2400 km/hr


rin
(x + 24) (x – 20) = 0
x = – 24 or x = 20

57. Let
Time =
D 6000 5
=
S 2400 2
1
= = 2 hours.
2
width of the path = x m g.n
But the number of days cannot be – ve.
Original duration of tour = 20 days.

Area of outer rectangle = (16 + 2x) (10 + 2x)


= 160 + 32x + 20x + 4x2
= 4x2 + 52x + 160
60. Let

e
the length of piece of cloth = x m
Increased length of piece of cloth = (x + 5)m
Total cost = Rs. 200 t
According to the question—
Area of the inner rectangle = 16 × 10 = 160 m2 200 200
– = Rs. 2
Area of the path = 4x2 + 52x + 160 – 160 x x+5
 Total Cost 
= 120
 Q Rate per metre = Length 
⇒ 4x2 + 52x – 120 = 0
200x + 1000 – 200x
⇒ x2 + 13x – 30 = 0 = Rs. 2
x(x + 5)
⇒ x2 + 15x – 2x – 30 = 0 ⇒ 1000 = 2x2 + 10x
⇒ (x + 15) (x – 2) = 0 ⇒ 2
2x + 10x – 1000 = 0
⇒ x = – 15 or x = 2 ⇒ x2 + 5x – 500 = 0 [Dividing by 2]
Since, the width cannot be negative. ⇒ x2 + 25x – 20x – 500 = 0
Hence, width of the path = 2 m. ⇒ x(x + 25) – 20(x + 25) = 0
58. From ∆ XZY, we get XZ = 9 + r cm ⇒ (x + 25) (x – 20) = 0
ZY = 2 + r cm ⇒ x = – 25 or x = 20

230 | CAT Complete Course

Visit : www.EasyEngineering.net
Visit : www.EasyEngineering.net

Length of can never be – ve. 12x 10y


+ = 130
∴ Length of cloth = 20 metre 100 100
200 ⇒ 12x + 10y = 13000
and Rate per metre = = Rs. 10
20 ⇒ 6x + 5y = 6500 …(i)
Exercise B If the amounts invested are interchanged, then yearly
interest
1. Let the speed of the train = x km/hr
10x 12y
Speed of the car = y km/hr = + = 134
100 100
Total Distance = 760 km ⇒ 10x + 12y = 13400
Distance travelled by train = 160 km or 5x + 6y = 6700 …(ii)
Distance travelled by car = 760 – 160 = 600 km Multiplying (i) by 5 and (ii) by 6, we get
According to the question— 30x + 25y = 32500 …(iii)
160 600 30x + 36y = 40200 …(iv)
+ = 8 hours …(i)
x y
Subtracting (iii) from (iv), we get
240 520 1

ww
and
x
+
y
= 8

=
5
41
hours …(ii)

11y = 7700
y = 700

w
5 Putting y = 700 in (i), we get
Multiplying (i) by 3 and (ii) by 2, we get 6x + 5 × 700 = 6500

x .Ea
480 1800
+
y
= 24 …(iii)
⇒ x = 500
3. Let the constant expenditure = Rs. x
480 1040
x
+
syE
y
=

Subtracting (iv) from (iii), we get


82
5
…(iv)
and consumption of wheat = Rs. y quintals.
Then, total expenditure = x + y × Rate per quintal
1000 = x + 250y …(i)
760
y
=
38
5 ngi and
Subtracting,
980 = x + 240y
20 = 10y
…(ii)


38y = 760 × 5

y =
760 × 5
38
nee

Putting
y = 2
y = 2 in (i), we get

= 100 km/hr ⇒
rin
1000 = x + 2 × 250
1000 = x + 500
Substituting the value of y in (i), we get
160 600
x
+
100
= 8

g.n
x = 1000 – 500
= 500


160
x
+6 = 8

160
x
= 8–6
per quintal
e
Total expenses when the price of wheat is Rs. 350
= 500 + 350 × 2
= 500 + 700
= Rs. 1200 t
x
160 4. Let the fraction be ·
⇒ = 2 y
x Then, x + y = 18 …(i)
⇒ 2x = 160 x 1
and by IInd condition = …(ii)
⇒ x = 80 km/hr y+2 3
Speed of the train = 80 km/hr y + 2 = 3x
and Speed of the car = 100 km/hr. ⇒ y = 3x – 2
Putting value of y in equation (i), we get
2. Let invested amount @ 12% be Rs. x and invested
amount @ 10% be Rs. y. x + 3x – 2 = 18
p ×r×t ⇒ x = 5
Interest =
100 Putting x = 5 in in equation (i), we get
x × 12 × 1 y × 10 × 1 y = 13
The yearly interest = +
100 100 5
Hence, required fraction is ·
= 130 13

CAT Complete Course | 231

Visit : www.EasyEngineering.net
Visit : www.EasyEngineering.net

5. Let the cost of a table be Rs. x and that of a chair From (ii),
be Rs. y. x – 10 – y – 10 = 0
Then, 2x + 3y = 2000 …(i) x – y – 20 = 0 …(iv)
and 3x + 2y = 2500 …(ii) Adding (iii) and (iv), we get
Multiplying (i) by 3 and (ii) by 2, we get 2y – x – 30 = 0
6x + 9y = 6000 – y + x – 20 = 0
6x + 4y = 5000 ⇒ y = 50
– – –
Putting the value of y in (iv), we get
5y = 1000
x – 50 – 20 = 0
⇒ y = 200
x – 70 = 0
Putting value of y in equation (i), we get
x = 70
2x + 600 = 2000
Number of oranges with A = 70
⇒ x = 700
and Number of oranges with B = 50
and hence the value of one table (1x) and 5 chair

6.
ww
(5y) is Rs 1700.
Father Son
8. Let the speed of the train = x km/hr
and speed of the bus = y km/hr
Total distance = 300 km

w
Let the present age of x years y years
5 years ago, (x – 5) years (y – 5) years Distance travelled by train = 60 km
After 5 years,
As per the question .Ea
(x + 5) years (y + 5) years ∴ Distance travelled by bus = 300 – 60 = 240 km
60 240
+ = 4 hours …(i)
After 5 years—
⇒ syE
(x + 5) = 3(y + 5)
x + 5 = 3y + 15
x y
II-Distance travelled by train = 100 km
∴ Distance travelled by bus = 300 – 100
x – 3y – 15 + 5 = 0
x – 3y – 10 = 0 …(i)
ngi 100 200
= 200 km
10
5 Years ago— (x – 5) = 7 (y – 5)
x – 5 = 7y – 35
x – 7y + 35 – 5 = 0
nee ∴
x
+
y
= 4 hours + hrs

=
25
hr
60

…(ii)

rin
6
x – 7y + 30 = 0 …(ii) Multiplying (i) by 5 and (ii) by 3, we get
Subtracting (ii) from (i)
x – 3y = 10 …(i) x
g.n
300 1200
+
y
= 20 …(iii)

e
x – 7y = – 30 …(ii) 300 600 75
+ = …(iv)
4y = 40
y = 10
Putting the value of y in (i), we get
x

600
y
y

=
6
By subtracting (iv) from (iii), we get
45
6
t
x – 3 (10) = 10
45y = 3600
x – 30 = 10
x = 40 ⇒ y = 80
∴ Present age of the father = 40 years Using the value of y in (i), we get
and Present age of the son = 10 years 60 240
+ = 4
x 80
7. Let the number of oranges with A be x
60
and number of oranges with B be y. = 4–3
x
Then, x + 10 = 2(y – 10) …(i) 60
= 1
and x – 10 = y + 10 …(ii) x
From (i), ⇒ x = 60
2y – 20 – x – 10 = 0 ∴ Speed of the train = 60 km/hr
2y – x – 30 = 0 …(iii) and Speed of the bus = 80 km/hr.

232 | CAT Complete Course

Visit : www.EasyEngineering.net
Visit : www.EasyEngineering.net

9. Father Son 8y + 10x = 12x + 43


Let the present age of x years y years or – 2x + 8y = 43
10 years ago, (x – 10) years (y – 10) years 12. Multiplying both sides of equation (ii) by (a2 – b 2 ),
After 10 years, (x + 10) years (y + 10) years we get
As per the question, ax{(a + b) – (a – b)} + by{– (a + b) – (a – b)}
Ist condition, = 2a(a – b)
x – 10 = 12(y – 10) or ax(2b) + by(– 2a) = 2a(a – b)
⇒ x – 10 = 12y – 120 or bx – by = a – b
⇒ x – 12y = – 120 + 10 = – 110 …(i) 13. (2x – 3y – 13)2 + (3x + 5y + 9)2 = 0
IInd condition, ⇒ 2x – 3y – 13 = 0
x + 10 = 2(y + 10) 3x + 5y + 9 = 0
x + 10 = 2y + 20 (Q For any real numbers a and b – a2 + b2 = 0 ⇒ a
x – 2y = 20 – 10 = 10 …(ii) = 0 and b = 0)
14. Let 2x = u and 3y = v

ww
Subtracting (ii) from (i), we get
x – 12y = – 110
x – 2y = 10
…(i)
…(ii) 15.
Then, u + v = 17 and 4u – 3v = 5
x+y–8
=
x + 2y – 14 3x + y – 12
=

⇒ w – 10y = – 120
y = 12
.Ea
Now, Putting the value of y in (i), we get
2
x+y–8
2
x + 2y – 14
=
3
x + 2y – 14
3
3x + y – 12
11

syE
and =
x – 2 (12) = 10 3 11
⇒ x – 24 = 10 ⇒ 3x + 3y – 24 = 2x + 4y – 28

ngi
x = 24 + 10 = 34 and 11x + 22y – 154 = 9x + 3y – 36
∴ Father’s age = 34 years and Son’s age = 12 years ⇒ x–y = –4
Father Son

nee
10. and 2x + 19y = 118
Let the present age of x years y years 16. Let the weights per c.c. be A and B, then
After 3 years, (x + 3) years (y + 3) years 3 2
As per the question,
x = 3(y) + 3
5
5
5
3 rin
A + B = 1·04,

x = 3y + 3
According to the question—
…(i) 8
A + B = 1·05
8
g.n
17. Let the present age of A be x years and the present
x + 3 = 2(y + 3) + 10
x + 3 = 2y + 6 + 10
x – 2y = 6 + 10 – 3
age of B be y years.
e
∴ According to the question, we have
x + y = 64 …(i) t
x – 2y = 13 …(ii) and [x – (x – y)] = 3[y – (x – y)]
Putting x value in equation (ii), we get i.e., y = 3(2y – x)
3y + 3 – 2y = 13 ⇒ 3x – 5y = 0 …(ii)
⇒ y = 10 Multiplying (i) by 3, we get
Putting this y value in equation (i), we get 3x + 3y = 192 …(iii)
x = 33 Subtracting (ii) from (iii), we get
11. Multiplying both sides of equation (i) by 52, the 8y = 192
L.C.M. of 13 and 4, we get ⇒ y = 192 ÷ 8 = 24
8(2x + 3y) = 156 + 13(x – y) Putting y = 24 in (i), we get
16x + 24y = 156 + 13x – 13y x + 24 = 64
or 3x + 37y = 156 x = 64 – 24 = 40
Multiplying both sides of equation (ii) by 6, the Hence, the present age of A is 40 years and the
L.C.M. of 3 and 6, we get present age of B is 24 years.

CAT Complete Course | 233

Visit : www.EasyEngineering.net
Visit : www.EasyEngineering.net

18. Let the original speed be x km/hr and the distance ⇒ (x + 20) (x – 16) = 0
between two cities A and B by ‘y’ km. ⇒ x = 16, – 20
Now, Distance = Speed × Time Side can never be – ve.
∴ y = x×8 Side of one square = 16 cm
⇒ y = 8x …(i) ∴ Side of another square = 16 + 4 = 20 cm.
If I increase my speed by 6 km/hr it takes me 1 hour 21. Let the side of the square be x cm.
20 minutes.
∴ Side of the another square be (x + 4) cm
i.e., 80 minutes less
Equation as per the question—
 80   
(x + 6)  8 –  = y ⇒ (x + 6)  8 – 43  x2 + (x + 4)2 = 400
 60    ⇒ x2 + (x2 + 8x + 16) = 400
20(x + 6) ⇒ x2 + x2 + 8x + 16 – 400 = 0
= y
3
⇒ 2x2 + 8x – 384 = 0
20(x + 6)
⇒ = 8x [Using (i)] ⇒ x2 + 4x – 192 = 0 [Dividing by 2]
3

⇒ ww 20x + 120 = 24x
4x = 120;



x2 + 16x – 12x – 192 = 0
x(x + 16) – 12(x + 16) = 0
(x + 16) (x – 12) = 0

w
x = 30
Putting x = 30 in (i), we get y = 8 × 30 = 240 ⇒ x = – 16

.Ea
Hence, the distance between the two cities A and B
is 240 km.
or x = 12
As the length of the square cannot be – ve.

syE
19. Let the number of persons in Ist condition = x
and the 2nd condition no. of persons = (x + 15)


x = 12
Side of the first square = 12 cm

ngi
Amount divided = Rs. 6500 and Side of the second square = 12 + 4 = 16 cm.
According to the question— 22. Let the Breadth of the rectangle be x cm.

nee
6500 6500 Then, Length = (x + 8) cm
– = Rs. 30
x x + 15 Area = Length × Breadth
6500x + 97500 – 6500x 30 x(x + 8) = 240 (given)

rin
=
x(x + 15) 1 2
x + 8x – 240 = 0
⇒ 30x2 + 450x = 97500 ⇒ x2 + 20x – 12x – 240 = 0


30x2 + 450x – 97500 = 0
x2 + 15x – 3250 = 0 [Dividing by 30] ⇒ g.n
⇒ x(x + 20) – 12(x + 20) = 0
(x – 12) (x + 20) = 0
⇒ x2 + 65x – 50x – 3250 = 0
⇒ x(x + 65) – 50(x + 65) = 0
⇒ (x + 65) (x – 50) = 0

or ex = 12,
x = – 20
Since, Breadth cannot be – ve.
t
⇒ x = – 65 x = 12 cm = Breadth
or x = 50 and Length = 12 + 8 = 20 cm.
23. Let the altitude of triangle = x cm
Since, the number of persons can never be – ve.
Then, Base = (x + 10) cm
∴ Number of persons = 50
1
20. Area of square having side x cm = x2 cm2 Area of triangle D = × B × H
2
Area of square having side (x + 4) cm = (x + 4)2 cm2 1
600 = × x × (x + 10)
Given 2
x2 + (x + 4)2 = 656 1200 = x2 + 10x
⇒ x2 + x2 + 16 + 8x = 656 x2+ 10x – 1200 = 0
⇒ 2x2 + 8x – 640 = 0 2
x + 40x – 30x – 1200 = 0
⇒ x2 + 4x – 320 = 0 [Dividing by 2] (x + 40) (x – 30) = 0
⇒ x + 20x – 16x – 320 = 0
2
x = – 40, 30

234 | CAT Complete Course

Visit : www.EasyEngineering.net
Visit : www.EasyEngineering.net

Side can never be – ve. According to the question—


∴ Altitude of triangle = 30 cm 300 300
– = 2 hr
and Base = 30 + 10 x x+5
= 40 cm.  Distance 
 Q Time = Speed 
24. Let the speed of Ist train be = x km/hr
and speed of the another train be = (x – 12) km/hr 300x + 1500 – 300x 2
=
x(x + 5) 1
Distance = 240 km
⇒ 1500 = 2x(x + 5)
According to the question—
240 240 ⇒ 2
2x + 10x – 1500 = 0
– = 1 hour
x – 12 x ⇒ x2 + 5x – 750 = 0 [Dividing by 2]
 Distance  ⇒ x2 + 30x – 25x – 750 = 0
 Q Time = Speed  ⇒ x(x + 30) – 25(x + 30) = 0
240x – 240x + 2880 1 ⇒ (x – 25) (x + 30) = 0
=
x(x – 12) 1 ⇒ x = 25

ww


x(x – 12) = 2880
x2 – 12x = 2880
or x = – 30
Q Since the speed cannot be – ve.

w
⇒ x2 – 60x + 48x – 2880 = 0

.Ea
⇒ x(x – 60) + 48(x – 60) = 0
⇒ (x – 60) (x + 48) = 0
∴ Usual Speed = 25 km/hr.
27. Let the consecutive natural numbers be n and (n + 1).
As per the question,

or syE
x = 60
x = – 48 ⇒
⇒ 2
n2 + (n + 1)2 = 313
n2 + n2 + 1 + 2n = 313
2n + 2n – 313 + 1 = 0


Speed of Ist train = 60 km/hr
Speed of IInd train = 48 km/hr.
ngi ⇒

2n2 + 2n – 312 = 0
n2 + n – 156 = 0 [Dividing by 2]
25. Let the original speed of the train be = x km/hr
and increased speed of the train be = (x + 15) km/hr
nee ⇒ n2 + 13n – 12n – 156 = 0
⇒ n(n + 13) – 12(n + 13) = 0

rin
Distance = 90 km
⇒ (n – 12) (n + 13) = 0
According to the question—
⇒ n – 12 = 0
90

90
x x + 15
1
= hr
2
 Distance 
or
⇒ g.n
n + 13 = 0
n = 12

90x + 1350 – 90x


x(x + 15)
=
1
2
 Q Time = Speed  or
e
n = – 13
Since – 13 is not a natural number.
Hence, the numbers are 12, and 12 + 1 i.e., 13. t
28. Let the parts be x and (29 – x).
⇒ 2700 = x2 + 15x
As per the question,
⇒ x2 + 15x – 2700 = 0
(x)2 + (29 – x)2 = 425
⇒ x2 + 60x – 45x – 2700 = 0
x2 + 841 + x2 – 58x = 425
⇒ x(x + 60) – 45(x + 60) = 0
⇒ 2x – 58x + 841 – 425 = 0
2
⇒ (x + 60) (x – 45) = 0 ⇒ 2x2 – 58x + 416 = 0
⇒ x = – 60 ⇒ x2 – 29x + 208 = 0 [Dividing by 2]
or x = 45 ⇒ x2 – 16x – 13x + 208 = 0
∴ Since, the speed cannot be – ve. ⇒ x(x – 16) – 13(x – 16) = 0
∴ Speed = 45 km/hr. ⇒ (x – 16) (x – 13) = 0
26. Let the usual speed of the train be = x km/hr ⇒ x = 16
and increased speed of the train be = (x + 5) km/hr or x = 13
Distance = 300 km Hence, the parts are 16 and 13.

CAT Complete Course | 235

Visit : www.EasyEngineering.net
Visit : www.EasyEngineering.net

29. Speed of boat in still water = 8 km/hr 30


Time taken by boat in upstream = hour
Let the speed of stream in still water = x km/hr 15 – x
30
∴ Speed upstream = (8 – x) km/hr Time taken by boat in downstream = hour
15 + x
and speed of downstream = (8 + x) km/hr
30 30
According to the question— Total time taken = +
15 – x 15 + x
15 22 = 4 hours 30 minutes (given)
+ = 5
8–x 8+x
30(15 + x) + 30(15 – x) 1
15(8 + x) + 22(8 – x) = 4
= 5 (15 – x) (15 + x) 2
(8 – x) (8 + x)
450 + 30x + 450 – 30x 9
120 + 15x + 176 – 22x ⇒ =
⇒ = 5 225 – x2 2
64 – x2
900 9
⇒ 120 + 15x + 176 – 22x = 320 – 5x2 ⇒ =
225 – x2 2
⇒ 5x + 15x – 22x + 120 + 176 – 320 = 0
2
⇒ 1800 = 2025 – 9x2
⇒ 5x2 – 7x – 24 = 0 ⇒ 9x2 = 2025 – 1800



ww 5x2 – 15x + 8x – 24 = 0
5x(x – 3) + 8(x – 3) = 0


9x2 = 225
x2 = 25

w
(x – 3) (5x + 8) = 0
⇒ x = √25 = ± 5
⇒ x–3 = 0
or
⇒ .Ea 5x + 8 = 0
x = 3
∴ Speed of stream = 5 km/hr.
32. Let one number be = x

syE
1
x – 20 = 69 ×
or 5x = – 8 x
⇒ x = 3 x2 – 20x – 69 = 0

or x = –
8
5
ngi ⇒

2
x – 23x + 3x – 69 = 0
x(x – 23) + 3(x – 23) = 0
Q Speed cannot be – ve.
∴ Speed of stream = 3 km/hr.
nee


(x – 23) (x + 3) = 0
x – 23 = 0

rin
30. Let one number = x or x+3 = 0
Then, other number = (48 – x) ⇒ x = 23
Product = x(48 – x) = 432
48x – x2 = 432
or

g.n x = –3
But – 3 is not a whole number

e
2
x – 48x + 432 = 0 x = 23.



2
x – 36x – 12x + 432 = 0
x(x – 36) – 12(x – 36) = 0
(x – 36) (x – 12) = 0
33. Suppose one natural number = x
IInd natural number = x + 1
According to the question—
t
⇒ x – 36 = 0 x(x + 1) = 20
or x – 12 = 0 x2 + x = 20
⇒ x = 36 ⇒ x2 + x – 20 = 0
or x = 12 ⇒ x2 – 5x – 4x – 20 = 0
∴ One number = 36, 12 ⇒ x(x + 5) – 4(x + 5) = 0
and other number = 12, 36 ⇒ (x + 5) (x – 4) = 0
∴ Two numbers are 36 and 12. ⇒ x+5 = 0
31. Let the speed of stream = x km/hr or x–4 = 0
Speed of the boat in still water = 15 km/hr (given) ⇒ x = –5
Speed of boat in downstream = (15 + x) km/hr or x = 4
Speed of boat in upstream = (15 – x) km/hr But x = – 5 is not a natural number
Distance = 30 km Hence, two consecutive natural numbers are 4, 5.

236 | CAT Complete Course

Visit : www.EasyEngineering.net
Visit : www.EasyEngineering.net

34. Let the present age of son = x years ⇒ (2a – b – c)2 = 0


Present age of father = x 2 years ⇒ 2a – (b + c) = 0
One year ago, age of son = (x – 1) year ⇒ 2a = b + c
One year ago, age of father = (x2 – 1) year Hence, proved.
According to the question— 38. Roots are real and equal.
8(x – 1) = (x2 – 1) ⇒ B 2 – 4AC = 0
8x – 8 = x2 – 1 ⇒ 4(bc – ad)2 = 4(a2 + b2 ) (c 2 + d2 )
x2 – 8x + 7 = 0 ⇒ b2 c2 + a2 d2 – 2abcd = a2 c2 + a2 d2 + b2 c2 + b2 d2
(x – 7) (x – 1) = 0 ⇒ b2 d2 + a2 c2 + 2abcd = 0
x = 7, 1 ⇒ (ac + bd)2 = 0
x = 1 is not suitable for the equation ⇒ ac + bd = 0
x = 7 39. The equation is p(q – r)x2 + q(r – p)x + r(p – q) = 0
Hence, the age of son = 7 years Since, the roots are real.

ww
Age of father = 49 years.
35. Let number of books = x
80


b2 – 4ac = 0
q2 (r – p) 2 – 4pr(q – r) (p – q) = 0
⇒ q2 (r2 + p2 – 2rp) – 4pr(pq – pr – q2 + qr) = 0

w
Cost of each book = Rs.

.Ea
If number of books = x + 4

Then, cost of each book =


x

80
x+4
⇒ q2r2 + p2 q2 – 2prq2 – 4p2 qr + 4p2 r2


+ 4pq2 r2 – 4 pqr2 = 0
p2 q2 + q2 r2 + 4 r2 p2 + 2 pq2 r – 4 p2rq = 0

According to the question—


syE
80
=
80
+1


(pq + qr – 2rp)2 = 0
pq + qr – 2rp = 0

ngi
x+4 1 1 2
x ⇒ + =
p r q
80 80 + x + 4
= 40. The equation is (1 + m2)x2 + 2mcx + (c2 – a2 ) = 0
x x+4
80x + 320 = 80x + x 2 + 4x
x2 + 4x – 320 = 0 nee⇒
The roots are real and equal. D = 0
D = B 2 – 4 AC
x2 + 20x – 16x – 320 = 0 ⇒
rin
(2mc)2 – 4(1 + m2) (c 2 – a2 ) = 0
⇒ 4m2 c2 – 4(c2 – a2 + m2 c2 – m2 a2 ) = 0

g.n
(x + 20) (x – 16) = 0
⇒ 4(– c2 + a2 + m2 a2 ) = 0
x = 16, – 20
⇒ – c2 + a2 + m2 a2 = 0
Number of books can never be – ve.
x = 16
Hence, number of books = 16
36. Let the integer = x
41.

e a2 (1 + m2) = c2
Let the original average speed of the cyclist be
x km/hr. t
Distance from A and B = 14 km
According to the question—
∴ Time taken to cover a distance of 14 km at
x2 + x = 90 14
original speed = hrs.
x2 + x – 90 = 0 x
x2 + 10x – 9x – 90 = 0 When speed is decreased by 1 km/hr new speed =
(x – 1) km/hr.
(x + 10) (x – 9) = 0
Time taken to cover 14 km at reduced speed
x = – 10, 9 14
2 = hr.
37. The equation is (a – b)x + (b – c)x + (c – a) = 0 (x – 1)
Since, the roots are real, therefore Discriminate = 0 According to the question—
⇒ B 2 – 4AC = 0 14 14 1
– =
⇒ (b – c)2 – 4(a – b) (c – a) = 0 (x – 1) x 3
⇒ b2 + c2 – 2bc – 4(ac – a2 – bc + ab) = 0 ⇒ x2 – x = 42
⇒ 4a2 + b2 + c2 – 4ac – 4ab + 2bc = 0 ⇒ 2
x – x – 42 = 0

CAT Complete Course | 237

Visit : www.EasyEngineering.net
Visit : www.EasyEngineering.net

⇒ (x – 7) (x + 6) = 0 According to the question—


⇒ x = 7, – 6 x(x – 1) = 132
Original average speed = 7 km/hr. or 2
x – x – 132 = 0
42. Let the rate of walking = x km/hr (x + 11) (x – 12) = 0
Distance = 2 km x = 12, – 11,
2 Rejecting x = – 11
Time = hr
x Number of children = 12
Again, Speed = (x + 1) km/hr 45. Let a man purchased x books at Rs. 720.
2 720
Time = hr ∴ Cost of one book = Rs.
(x + 1) x
According to the question— If each book cost Rs. 2 less, he would get 4 more
2 2 1 books.
– =
x (x + 1) 6 ∴ Now, with Rs. 720, he can purchase (x + 4) books.
2 1
= 720

ww x2 + x
x2 + x – 12 = 0
(x – 3) (x + 4) = 0
6 Cost of each book = Rs.

According to the question,


x+4

w x = 3, – 4, Rejecting x = – 4

.Ea
Rate of walking = 3 km/hr.
43. Let the average speed of the aircraft = x km/hr ⇒
720 720
x

720 

x+4
 4 
= 2

 x2 + 4x 
= 2

Time to cover 1600 km =


syE
1600
x
hrs ⇒

x2 + 4x – 1440 = 0
(x – 36) (x + 40) = 0

ngi
On decreasing the speed by 400 km, its speed will be
= (x – 400) km/hr x = 36, – 40
Time taken to cover a distance of 1600 km at Rejecting x = – 40
reduced speed =
1600
(x – 400)
hr
nee ∴ Number of books = 36
46. Let original number of swans = n
According to the question,
1600

1600
=
2 7
= √n rin
Number of swans playing on the shore of the pond


(x – 400) x
 400  2
1600 
3

 x2 – 400x  3
=
2
According to the question—
g.n

⇒ x =
x2 – 400x – 960000 = 0
400 ± 
√
(400) 2 + 4 × 960000

7
2
7
n+2 = n

 n + 4 = 2n

e t
2×1 ⇒ 7  n = 2n – 4

400 ± √

4000000 400 ± 2000 ⇒ 49 n = 4n2 + 16 – 16n
= =
2 2 ⇒ 4n2 – 65n + 16 = 0
2400 – 1600
= , 65 ± √

(65) 2 – 4 (4) (16)
2 2 n =
2×4
= 1200, – 800, Rejecting x = – 800
65 ± √
3969
∴ Average speed = 1200 km/hr n =
8
1600 4 1
Actual time = = hr = 1 hr. 128 2
1200 3 3 n = ,
8 8
44. Let the number of children in a group = x 1
n = 16, , But n ≠ become n ∈ N
∴ Each child give a gift to every other child. 4
∴ One child give (x – 1) gift to other children. ∴ Total number of swans = 16.
∴ x children gives x(x – 1) gifts. ●●●

238 | CAT Complete Course

Visit : www.EasyEngineering.net
Visit : www.EasyEngineering.net

15 Progression and Sequence


Progression implies sequence of numbers. It consists Solution :
of a set of numbers arranged in order. The numbers in the Ist term = a = – 4
sequence are termed as terms of the sequence or series. Common difference = – 7 – (– 4) = – 10 (– 7) = – 3
Examples— 10th term is given by = a + (10 – 1).d
3, 5, 7, 9, 11, 13, ………… = – 4 + 9 × (– 3) = – 31.
2, 4, 6, 8, 10, 12, ………… Illustration 3.
The General form of representing a series is 1 3 5

ww X1, X2 , X3 , X4 ……………
There are three types of progression—
term.
Given A.P. is , 1, , 2, , ……… Find its 115th

Solution :
2 2 2

w
(i) Arithmetic Progression (ii) Geometric Progression
(iii) Harmonic Progression.

.Ea
(A) Arithmetic Progression (A.P.)
Ist term = a =

Common difference = 1 – = – 1 =
1
2
1 3
2 2
1
2

syE
A sequence of numbers is said to be in Arithmetic
progression if the difference between any two consecutive
numbers is always the same. If first term of the progression
Now, nth term = tn = a + (n – 1).d
1
t115 = + (115 – 1) ×
1
is ‘a’ and the difference between two consecutive terms
of the series is termed as its common difference ‘d’. Then,
the nth term of the series is Tn. ngi t115 =
2
115
2
2

Illustration 1.
Tn = a + (n – 1)d
nee
Illustration 4.
Find the Sum of n natural numbers.

term.
Let an A.P. is 3, 6, 9, 12, 15, 18, …… Find its 15th
Solution :
We known that
rin
Solution :
Let the first term of the progression = a g.n
k2– (k – 1)2 = 2 k –1
Now, when k = 1 12 – 02 = 2 × 1 – 1
k = 2 22 – 12 = 2 × 2 – 1
Common difference = d
n – represent nth term of the series.
Ist term = a = 3
e
k = 3 32 – 22 = 2 × 3 – 1
k = 4 42 – 32 = 2 × 4 – 1
k = 5 52 – 42 = 2 × 5 – 1 t
Common difference = d = 6 – 3 = 9 – 6 = 3 ……………………
t2 = 2nd term = 3 + 3 = 6 = a + d ……………………
t3 = 3rd term = 3 + 6 = 9 = a + 2d k = n n2 – (n – 1) 2 = 2 × n – 1
t4 = 4th term = 3 + 9 = 12 = a + 3d n2 = 2(1 + 2 + 3 + 4 + … + n) – n
t5 = 5th term = 3 + 12 = 15 = a + 4d n2 = 2(Sum of natural numbers) – n
n(n + 1)
… … Sum of n natural numbers =
… … 2
… … Illustration 5.
tn = nth term = a + (n – 1) d Find the sum of n terms of an A.P. whose first
So, 15th term = a + (15 – 1).d number is ‘a’ and common difference is ‘d’.
= a + 14 d = 3 + 14 × 3 = 45. Solution :
Illustration 2. According to given condition—
Given A.P. is – 4, – 7, – 10, – 13, – 16, …… Find the Arithmetic progression is—a, a + d, a + 2d, a + 3d,
common difference and the 10th term. a + 4d, …… a + (n – 1)d

CAT Complete Course | 239

Visit : www.EasyEngineering.net
Visit : www.EasyEngineering.net

If Sn be the sum of the above numbers, then Illustration 9.


S n = a + a + d + a + 2d + a + 3d + …… + The first and 7th term of an A.P. are 7 and 49 respec-
tively. Write the A.P.
a + (n – 1) d
S n = na + d[1 + 2 + 3 + …… (n – 1) term] Solution :
Since, we know that
n(n – 1)
S n = na + d ×
2 n
Sn = [2a + (n – 1)d]
n 2
Sn = [2a + (n – 1) d]
2 n
= [a + a + (n – 1)d]
Illustration 6. 2
Find the sum of 20 terms of an A.P. which is – 5, – 3, n
= [a + nth term]
– 1, 1, 5, 7, …… 2
Solution : n
Sn = [Ist term + nth term]
According to question— 2
Ist term = a = – 5; Common difference = d = – 3 (– 5) If last term = nth term

ww
= 2; n = 20

Since, S 20 =
20
2
[2a + (20 – 1)d]
So, Sn =
n
2
[Ist term + Last term]

w
Illustration 10.
= 10[2 × (– 5) + 19 × 2] The first and last term of an A.P. consisting of 12

Illustration 7.
= 280
.Ea terms are 20 and 245 respectively. Write the A.P.
Solution :

(i) 430, 415, 400, 385 ………… syE


Find 20th term of the following A.P. Given a = 20
Last term = 245

ngi
(ii) (4x + 8y), (6x + 7y), (8x + 11y), (10x + 15y)…… n = 12
Solution : From formula—

nee
(i) First term = a = 430 12
S 12 = [2a + (n – 1)d]
Common difference = d = 415 – 430 = – 15 2
Now, tn = a + (n – 1)d 12
t20 = 430 + (20 – 1) (– 15)
=
2
rin
[Ist term + Last term]

= 6 × (20 + 245)

g.n
= 430 + 19 (– 15)
= 430 – 285 = 145 = 6 × (265)
= 1590

e
(ii) Given A.P. is (4x + 3y), (6x + 7y), (8x + 11y),
(10x+15y) ……
We consider above A.P. in two separated A.P.’s 4x,
6x, 8x, 10x, ……
and 3y, 7y, 11y, 15y, ……
Illustration 11.

t
If the first and last terms of an A.P. are 5 and 1025
and the Sum of terms is 2060, then find the value of n.
Solution :
Now, xt20 = 4x + (20 – 1)2x = 4x + 38x = 42x a = 5
and yt20 = 3y + (20 – 1)4y = 3y + 19 × 4y = 3y Last term = a + (n – 1)d = 1025
+ 76y = 79y n
S n = [Ist term + Last term]
Now, t20 = 42x + 79y 2
Illustration 8. n
2060 = [5 + 1025]
2
If Ist term of an A.P. is 10 and common difference is
5, write down the A.P. n = 4
Solution : Illustration 12.
Ist term = a = 10 Find the Sum of first n even natural numbers.
Common difference = d = 5 Solution :
So, Arithmetic Progression is 10, 15, 20, 25, 30, 35, The Ist n odd natural number start with 2.
40, …… An A.P. is 2, 4, 6, 8, 10,

240 | CAT Complete Course

Visit : www.EasyEngineering.net
Visit : www.EasyEngineering.net

So, a = 2, d = 2 The Arithmetic Mean of n Quantities is Equal


n to the Sum of the n Quantities Divided by n
S n = [2.a + (n – 1)d]
2 X1 + X2 + X3 + …… + Xn
∴ A.M. = 8 =
n n
= [2 × 2 + (n – 1) × 2]
2 Extra Shot— Several kinds of mean exist, and the
n method of calculating a mean depends upon the
= (2n + 2) relationship known or assumed to govern the other
2
Even = n(n + 1) members. The arithmetic mean, denoted 8, of a set of n
numbers X1 + X2 + X3 + …… + Xn is defined as the sum
Illustration 13.
of the numbers divided by n—
Find the Sum of first n odd natural numbers.
X + X2 + X3 + …… + Xn
Solution : ∴ 8 = 1
n
Given A.P. is 1, 3, 5, 7, 9, 11, …… The arithmetic mean represents a point about which
a = 1, d = 2 the numbers balance. For example, if unit masses are
placed on a line at points with co-ordinates X 1 , X2 , X3 ,

ww
n
S n = [2a + (n – 1)d] ……, Xn , then the arithmetic mean is the co-ordinate of
2
n the centre of gravity of the system. In statistics, the
= [2 × 1 + (n – 1) × 2] arithmetic mean is commonly used as the single value

w
Illustration 14.
2
S odd = n2

.Ea
typical of a set of data. For a system of particles having
unequal masses, the centre of gravity is determined by a
more general average, the weighted arithmetic mean. If
each number xi is assigned a positive weight wi, the
numbers.
Solution : syE
Find the sum of the squares of the first n natural
weighted arithmetic mean is defined as the sum of the
products w ixi divided by the sum of the weights. In this
case,
Given series is 12 + 2 2 + 3 2 + 4 2 + 5 2 + 6 2 + ……+ n2
Now, we have k3 – (k – 1)3 = 3k2 – 3k + 1 ngi X w + X2 w 2 + X3 w 3 + ……+ Xn w n
8 = 1 1
w 1 + w2 + w3 + …… + wn
Putting
Putting
k = 1
k = 2
13
23


03
13
=3×
=3×
12
22
–3×1+1
–3×2+1 nee The weighted arithmetic mean also is used in
statistical analysis of grouped data; each number x i is the
Putting k = 3 33 – 23 = 3 × 32 – 3 × 3 + 1
………………………………………………… rin
mid-point of an interval; and each corresponding value of
w i is the number of data points within that interval.

…………………………………………………
Putting k = n n3 – (n – 1) 3 = 3 × n2 – 3n + 1 g.n
For a given set of data, many possible means can be
defined, depending on which features of the data are of

Now, n3 = 3(12 + 2 2 + 3 2 + …… + n2) – 3 (1 + 2

n(n + 1)
+ 3 + 4 + …… + n) + n e
interest. For example, suppose five squares are given,

t
with sides 1, 1, 2, 5 and 7 inches. Their average area is
(12 + 12 + 22 + 52 + 72 )/5, or 16 square inches, the area of
a square of side 4 inches. The number 4 is the quadratic
or n3 = 3(Sn ) – 3 +n mean (or root mean square) of the numbers 1, 1, 2, 5, 7
2
1
3 and differs from their arithmetic mean, which is 3 . In
or n3 = 3S n – (n2 + n) + n 5
2 general, the quadratic mean of n numbers X1 + X2 + X3 +
3 …… + X n is the square root of the arithmetic mean of
or 3S n = n3 + (n2 + n) – n
2 their squares,
1



= [2n3 + 3n2 + 3n – 2n] X12 + X22 + X23 + …… Xn2
2 =
n
n
= [2n2 + 3n + 1] The arithmetic mean gives no indication of how
2
widely the data are spread or dispersed about the mean.
n(2n + 1) (n + 1)
Sn = Measures of the dispersion are provided by the arithmetic
6 and quadratic means of the n differences X1 – 8, X2 – 8,
n(n + 1) (2n + 1) X3 – 8 …, Xn – 8. These are called the variance and the
Sn =
6 standard deviation of X1 , X2, X3, ……, Xn.

CAT Complete Course | 241

Visit : www.EasyEngineering.net
Visit : www.EasyEngineering.net

The arithmetic and quadratic means are the special Solution :


cases p = 1 and p = 2 of the pth–power mean, Mp, Ist term = a
defined by the formula
(n + 2)th term = b
1
 Xp + Xp2 + Xp3 + … + Xnp p If d be the common difference.
Mp =  1 
 n  Now, (n + 2)th term = a + (n + 2 – 1)d
where p may be any real number except zero. The b = a + (n – 1)d
case p = – 1 is also called the harmonic mean. Weighted
b–a
pth–power means are defined by d =
n+1
1
 w1 Xp1 + w2X 2p + w3X 3p + … + wn X np p So, 2nd term = a + d
Mp =  
 w 1 + w2 + w3 + …… + wn  b – a an + a + b – a b + an
= a+ = =
If a is the arithmetic mean of x 1 and x 2 , the three n+1 n+1 n+1
numbers x 1 , a, x2 are in arithmetic progression. If h is the b – a 
harmonic mean of x 1 and x2, the numbers x1, h, x2 are in 3rd term = a + 2d = a + 2  
n + 1 
harmonic progression. A number g such that x1 , g, x2 are

x1 g ww
in geometric progression is defined by the condition that
= , or g2 = x1x2; hence g =  √x1x2 This g is called the
=
an + a + 2b – 2a 2b + (n – 1)a
n+1
=
n+1
So, the required arithmetic mean to be inserted is

w
g x2
geometric mean of x 1 and x 2 . The geometric mean of n b + an 2b + (n – 1)a 3b + (n – 2)a

.Ea
numbers x 1 , x 2 , . . . , xn is defined to be the nth root of

their product; g = 
n
√
x1x2x3……xn.
=

Illustration 17.
n+1
,
n+1
,
n+1

syE
All the means discussed are special cases of a more
general mean. If f is a function having an inverse f –1, the
Find the sum of cubes of first n natural numbers ?
Solution :

ngi
number Given S n = 13 + 2 3 + 3 3 + 4 3 + 5 3 + …… + n3

 f (x1 ) + f (x2 ) + f (x3 ) + …… + f (xn ) k4 – (k – 1)4 = 4k3 – 6k2 + 4k – 1


= f–1 
 n
is called the mean value of x 1 , x 2 , …, x n associated

nee
Putting k = 1 14 – 0 = 4 × 13 – 6 × 12 + 4 × 1 – 1
Putting k = 2 24 – 14 = 4 × 23 – 6 × 22 + 4 × 2 – 1
with f. When f (x) = xp the inverse is f – 1(x) = x1/p and the
mean value is the pth–power mean, Mp. When f (x) =
logex, the inverse is f – 1(x) = ex and the mean value is the
………………………………………………………
rin
Putting k = 3 34 – 24 = 4 × 33 – 6 × 32 + 4 × 3 – 1

geometric mean.
Illustration 15. g.n
Putting k = n n4 – (n – 1) 4 = 4 × n3 – 6 × n2 + 4n – 1
n4 = 4(13 + 2 3 + 3 3 + …… + n3) – 6(1 2 + 2 2

Find the arithmetic mean of the following—


(i) 3, 9, 12, 15, 21, 24
(ii) a, 4a, 9a, 6a, 20a or n4 = 4S n – 6
e
+ 32 + …… + n2) + 4(1 + 2 + 3 + 4

n(n + 1) (2n + 1) t
+ …… + n) – n

6
Solution :
n(n + 1)
3 + 9 + 12 + 15 + 21 + 24 +4× –n
(i) Arithmetic mean = 2
6
or n4 = n4 + n(2n3 + 3n + 1) – (2n2 – 2n) + n
84
= = 14
6 or = n4 + 2n3 + 3n2 + n – 2n2 – 2n + n
a + 4a + 9a + 6a + 20a So, 4S n = n4 + 2n3 + n2
(ii) Arithmetic mean =
5
4S n = n2 (n2 + 2n + 1)
40
= a = 8a  n(n + 1)  2
5
∴ Sn = 
Illustration 16.  2 
Insert n arithmetic means between a and b with n Illustration 18.
arithmetic means and the two given quantities a and b, Find the sum of the series of n terms—
the total number of terms will be (n + 2). Further all the
(42 – 12 ) + (52 – 22 ) + (62 – 32 ) + (72 – 42 ) + ……
terms will be in A.P.
242 | CAT Complete Course

Visit : www.EasyEngineering.net
Visit : www.EasyEngineering.net

Solution : Illustration 20.


The given series is combination of two series 42, 5 2 , Find the nth term of a G.P. whose Ist term is a and
6 , 72 ……. and 12 , 2 2 , 3 2 , 4 2 , ……
2 common ratio be r.
nth term = a + (n – 1)d = 4 + (n – 1)1 Solution :
=n+3 t1 = Ist term = a
2 2 2 2
nth term of series = 1 , 2 , 3 , 4 , …… t2 = 2nd term = t1 × r = a × r = a r2 – 1
nth term = n2 t3 = 3rd term = t2 × r = a r × r = a r2 = a r3 – 1
Now, nth term of the given series = (n + 3)2 – n2 t4 = 4th term = t3 × r = a r2 × r = a r3 = a r4 – 1
= n2 + 6n + 9 – n2 t5 = 5th term = t4 × r = a r3 × r = a r4 = a r5 – 1
= 6n + 9 ……………………………………………………
So, S n = Σn = 1 (6n + 9)
n
tn = nth term = a rn – 1
n(n + 1) Illustration 21.
= 6× + 9n
2
Find the sum of n terms of a G.P.
= 3n + 3n2 + 9n

ww = 3n2 + 12n = 3n(n + 4)


(B) Geometric Progression (G.P.)
Solution :
Let Ist term = a

w
A sequence of numbers is said to be in geometric

.Ea
progression. If the ratio of two consecutive numbers is
constant, the constant ratio is known as common ratio and
represented by r. Then nth term of the series is Tn = arn – 1.
Common ratio = r
If Sn be the sum of n terms.
So, S n = a + ar + ar2 + ar3 + ar4 + … + arn – 1…(1)

syE
So, series is a, ar, ar2 ,ar3, …… arn – 1
Here, First term = a; and common ratio = r.
Now, multiplying equation (1) with r, we get
rSn = ar + ar2 + ar3 + … + arn
Now, Sn – r Sn , then we get
Example—
(i) 3, 6, 12, 24, 48, ……
ngi S n = a + ar + ar2 + ar3 + …… + arn – 1

nee
(ii) 2, 4, 8, 16, 32, 64, …… – r Sn = – ar – ar2 – ar3 – …… – arn – 1 – arn
(i) First term = a = 3 (1 – r)Sn = a – arn
12

rin
Common ratio = r = =2 (1 – rn )
6 Sn = a × when r < 1
1–r
Tn = 3.(2)n–1
(ii) First term = a = 2
8
When r > 1

∴ Sn = a × 
r – 1 
g.n
 rn – 1

e
Common ratio = r = = 2
4

Illustration 19.
Tn = 2.(2)n – 1 = 2n

Find the common ratio of the following G.P.


Illustration 22.
t
A finite G.P. consists of 6 terms. If its first and last
terms are 5 and 84035. Find the G.P.

(i) 15a2 , 45a4 , 135a6 , 405a8 , …… Solution :


(ii) – 6, – 42, – 234, – 2058, …… Ist term = a = 5
Solution : n = 6
(i) Ist term = 15a2 Last term T 6 = 84035 = a. r5
Second term 84035 = 5 × r5
Common ratio =
First term
16807 = r5
45a4
= = 3a2 75 = r 5
15a2
(ii) Ist term = – 6 r = 7
Second term So, G.P. is 5, 35, 245, 1715, ……
Common ratio =
First term Illustration 23.
– 42 Find the sum of first 5 terms of G.P. whose first term
= =7
–6 is 2 and common ratio is 3.

CAT Complete Course | 243

Visit : www.EasyEngineering.net
Visit : www.EasyEngineering.net

Solution : (ii) G.M. = √ 


(a + b) (a2 – b2 ) (a – b)
n = 5, a = 2, r = 3 = ± (a2 – b2 )
Using Formula— Now, there are two G.M. = a2 – b2 and – (a2 – b2 )
 rn – 1 (a2 – b2 )
S5 = a ×   So, Ist common ratio = =a–b
r – 1  (a + b)
(a2 – b2 )
 35 – 1  2nd common ratio = – = – (a – b)
= 2 ×  = 243 – 1 = 242 (a + b)
 3–1  Illustration 25.
Illustration 24. Find the missing terms in the following G.P. –
Find the sum of 6 terms of a G.P. whose first term is 96, –, –, –, 486
1
5 and the common ratio is · Solution :
2
Clearly, a = 96, t5 = ar4
Solution :
a = 5 = Ist term ∴ 486 = 96 × r4
81  3 
4

ww
Common ratio (r) = 2

Sum = S n = a × 
 rn – 1
r – 1 
 r<1

r4 =

3
= 
16  2 

w
r =
2
1 – 1 
 26 
S 6 = 5 × 
1 

 1 – 2 .Ea 3
Second term = 96 × = 48 × 3 = 144
2
3  2

= 5 ×
 26 – 1 
 25  syE 63 315
 = 5 × 32 =
32
3rd term = 144 × 

3
Fourth term = 216 × = 324
 = 72 × 3 = 216
2 

Geometric Mean (G.M.)


ngiIllustration 26.
2

nee
The square root of the product of two quantities is The sum of three numbers in G.P. is 63 and their
equal to their Geometric mean. When three quantities are product is 1728. Find the numbers.
in G.P., the middle quantity is the geometric mean of the
Solution :
other two quantities.
If a and b are in G.P. So,
rin a a
Let the three numbers in G.P. be a, , 2 ·
r r
G.M. = √
ab
The Geometric mean of n quantities is equal to the
According to question—
a a g.n
nth root of the product of n quantities.

G.M. = √
n

X1.X2 .X3 .X4 . …… Xn
G.M. = (X1.X2 .X3 .X4 . …… Xn )1/n
or
a + + 2 = 63
r r
a a e
a · · 2 = 1728
r r
a3
…(1)

…(2)
t
or = 1728 = (4 × 3)3
r3
Illustration 24.
a
Find the Geometric mean, the common ratio and or = 12
r
write G.P. for each of the following paris.
 1 1
(i) 5x, 20 x3 or a 1 + +  = 63
 r r2 
(ii) (a + b), (a2 – b2 ), (a – b)
 1 1
Solution : or 12.r  1 + + 2  = 63
 r r 
(i) 5x, 20 x3
 r2 + r + 1 
G.M. = 
√ 5x. 20x3 = 10x2 or 12.r   = 63
 r2 
Clearly, 5x, 10x2, 20x3 are in G.P.
or 12.r2 + 12.r + 12 = 63.r
10x2
Common ratio = = 2x or 12.r2 – 51.r + 12 = 0
5x
Now, G.P. is 5x, 10x2 , 20x3, 40x4, …… or 12.r2 – 48.r – 3.r + 12 = 0

244 | CAT Complete Course

Visit : www.EasyEngineering.net
Visit : www.EasyEngineering.net

or 12.r(r – 4) – 3(r – 4) = 0 If H. P. is X1, X2, X3, X4, ……


1 1 1 1 1
r = 4, r = a = 48, 3 Then, , , , …… are in A.P.
4 X1 X2 X3 X4
Clearly, there are two G.P. which are – Illustration 29.
3 3 3
3, , , , …… 2 1 2 1 2 
4 16 64 Show that  ‚ ‚ ‚ ‚ ‚ ……  is a H.P.
 11 7 17 10 23 
and 48, 48 × 4, 48 × 42 , 48 × 43, ……
Solution :
Illustration 27.
The new sequence formed by reciprocals of the terms
The sum of first five terms of a G.P. is 93 and the
11 17 23
sum of first ten term is 3069. Find the G.P. of the given sequence is , 7, , 10, ·
2 2 2
Solution : 11 3
Now, 7– =
 rn – 1 2 2
Now, Sn = a ×  
r – 1  17 3
–7 =
2 2
 r5 – 1
S5 = a ×  
ww
So,
r – 1 
 r10 – 1 
S 10 = a ×  
10 –

23
17
2
=
3
2
3

w
and – 10 =
 r–1  2 2
S 10
S5
r10 – 1
= 5
r –1
.Ea
= r5 + 1 =
3069
93
Since, the difference between two consecutive terms
of the new sequence is always same.
So, they form an A.P. and Given sequence is in H.P.
r5 =
2976

r = 2
93
= 25
syE Illustration 30.

The first and 16th terms of a H.P. are


3
and
1

S5 = a × 
 25 – 1 
 2–1 
 = a × 31
ngi respectively. Find its 6th term.
Solution :
2 2

93 = a × 31
a = 3 nee We know that nth term of an A.P. is
tn = a + (n – 1)d
Illustration 28.
If A and G are A.M. and G.M. of a, b, then find the ∴ rin
where a = first term ; d = common difference
16th term = a + (16 – 1)d = a + 15d
equation having a, b as its roots.
Solution :
According to question—
1 g.n
=
3
…(1)
Since, a, b are two numbers

A.M. =
a+b

a + b = 2.A
2
=A

…(1)
1
a + 15d
a
= e2
1
2
Putting value of a in equation (2), we get
t
…(2)

1 1
=
Again, G.M. = G = √
ab 3 2
+ 5d
2
G = ab

2 1
a.b = G2 …(2) or =
3 + 10d 2
We know that if a, b are the roots of a quadratic or 4 = 3 + 10d
equation, then it will be 1
or d =
X2 – (a + b) X + ab = 0 10
X2 – 2A.X + G2 = 0 Now, 6th term = a + 5d
2 1
(C) Harmonic Progression (H.P.) = +5×
3 10
A sequence of numbers is said to be in H.P. if 2 1 7
= + =
reciprocals of its term from an arithmetic progression 3 2 6
(A.P.) conversely. If terms of a sequence are in A.P., their 6
∴ 6th term of a H.P. =
reciprocals form a H. P. 7

CAT Complete Course | 245

Visit : www.EasyEngineering.net
Visit : www.EasyEngineering.net

Harmonic Mean (H.M.) 6. If a 1 , a2 , a3 , …… a n is a G.P. of non–zero, non–


negative terms.
The Harmonic mean (H) of two given quantities a
and b is given by Log a1, Log a2 , Log a3 , …… Log an is an A.P. and
2ab vice–versa.
H = 7. If a G1 , G2, G3, …… Gn b are in G.P., where a and b
a+b
where a, H, b form a H.P. two given numbers, then G1 , G2 , G3 …… G n are
known as n geometric mean (G.M.) between a and b.
Illustration 31.
Find the harmonic mean of 4 and 12. Short Cuts
Solution : (a) If A.M., G.M. and H.M. between a and b are in A, G
Given two numbers are 4 and 12. and H respectively, then
a+b
So, H.M. =
2ab 2 × 4 × 12
= =6 A= , G = √ ab , H = a2ab
a+b 16 2 +b
a + b 2ab
SOME IMPORTANT Therefore, A × H = ×
2 a+b
POINTS IN A.P.

ww = ab = G = A × H
2

1. If a is the first and d be the common difference of an G = A×H


2
A.P. and if AP having m terms, then nth term from
(b) Always A.M. ≥ G.M. ≥ H.M.

w
the end is equal to the (m – n + 1)th term from the
beginning. So, nth term

.Ea
from the end = tm – n + 1 = a + (m – n)d
2. If a constant is added to or subtracted from an A.P.,
Hence, A ≥ G≥H
(c) The way of assuming terms in A.P.
(i) α – β, α, α + β

syE
then the resulting sequence is also an A.P. with some
common difference.
(ii) (α – 3β), (α – β), (α + β), (α + 3β)
(iii) (α – 5β), (α – 3β), (α – β), (α + β), (α + 3β),

ngi
3. In a finite A.P., the sum of terms equidistant from the (α + 5β)
beginning and end and is always equal to the sum of (d) The way of selecting terms in G.P. –
first and last term of the series.
α
tm + tn – (m – 1) = t1 + tn for all n = 1, 2, 3, ……
4. If between two given quantities a and b, we have to nee
(i)
r
, α, αr

α α
(ii) 3 , , αr, αr3
insert n quantities a, A1, A2, A3, …… An , b such that
a, A1, A2, A3, …… An , b are in A.P., then we say A1,
r r
α α
(iii) 2 , , α, αr, αr2 rin
A2, A 3 , …… An are n arithmetic means between a
and b.
r r
g.n
where α = Ist term, r = common ratio
SOME IMPORTANT
POINTS IN G.P.
1. The nth terms from the end of a finite G.P. consisting
of m terms is arm – n
Exercise A
e
1. A.M. between the roots of a quadratic equation is 5
and G.M. is 4, then find the quadratic equation. t
2. If A, G, H are A.M., G.M. and H.M. between three
where a = Ist term given numbers a, b and c, then the equation having a,
r = Common ratio 3G3
b, c as its roots is X 3 – 3 AX2 + – G3 = 0.
2. The nth term from the end of G.P. with last term l H
and common ratio r is 3. If in an A.P., the sum of m terms is equal to n and the
l  n – 1 sum of n terms is equal to m, then find the sum of
l ×   (m + n) terms.
r 
4. If (b + c), (c + a), (a + b) are in H.P., then what is the
3. a, b, c three non–zero numbers are in G.P. if b2 = ac.
series of a2 , b2 , c2 ?
4. If all the terms of a G.P. be multiplied or divided by
5. If the ath, bth and cth terms of a G.P. are P, Q, R
the same non–zero constant, then it remains a G.P.
respectively, then show that
with the same common ratio.
5. In a finite G.P., the product of the terms equidistant P b – c × Qc – a × Ra – b = 1
from the beginning and the end is always the same 6. If for a G.P. if T m + n = P and Tm – n = Q, then find out
and is equal to the product of first and the last term. Tm.

246 | CAT Complete Course

Visit : www.EasyEngineering.net
Visit : www.EasyEngineering.net

7. The ratio of the sum of m and n terms of an A.P. is 25. The sum of three numbers in A.P. is 15, whereas sum
m2 : n2 , then find the ratio of mth and nth term. of their squares is 83. Find the numbers.
8. If H be the H.M. between a, b, then prove that 26. There are two A.P.’s whose common differences
(i) (H – 2a) (H – 2b) = H2 differ by unity but sum of three consecutives terms in
each is 15. If P and P1 be the products of these terms
1 1 1 1
(ii) + = + P 7
H – a H – 2b a b such that = , then find two A.P.’s.
P1 8
H+a H+b
(iii) + =2 27. The pth term of A.P. is a and qth term is b, then find
H–a H–b
the sum of (p + q) term.
9. Find the value of Σr = 1 (3n3 + 2n).
8

28. There are two A.P.’s each of n terms


Σr = 1 (r + 1)2 .
10
10. Find the value of
a, a + d , a + 2d, ………… L
11. Express the following summations in Σ sign
p, p + q, a + 2q, ………… L1
(i) 3b + 12b2 + 48b3 + 192b4 + 768b5 These A.P.’s satisfy the following conditions—
(ii) 1·2 + 2·5 + 3·10 + 4·17 + 5·26 +……+ n(n2 + 1) Sn
L L1
12. Find the sum of the series of n terms = = 4, =2
S1n

ww
p a
(42 – 12 ) + (52 – 22 ) + (62 – 32 ) + (72 – 42 ) + …… d L
Find out and ·
13. Find the sum of first n terms of the series— q L1

14.
w
(12 + 32 + 32 ) + (22 + 42 + 52 ) + (32 + 52 + 72 ) + (42 +
62 + 9 2 ) + ……

.Ea
Find the sum of first n terms of the series
(53 + 3 3 ) + (73 + 5 3 ) + (93 + 7 3 ) + (113 + 9 3 ) + ……
29 There are n A.P.’s whose common differences are 1,
2, 3, …… n respectively, the first term of each being
unity. Find sum of their nth term.
30. If S1 , S 2 , S 3 , …… Sm are the sums of n terms of m
15.
1
+
1
+
1
3·4 4·5 5·6 6·7 7·8
+
1
+
1syE
Find the sum of first n terms of the following series
+ ……
A.P.’s whose first terms are 1, 2, 3, …… m and
common difference are 1, 3, 5, …… 2m – 1 respecti-
vely. Show that
16. Find the sum of first n terms of the series
3 – 5 + 7 – 9 + 11 – 13 + 15 – 17 + …… ngi 1
S 1 + S2 + S3 + …… + Sm = mn (mn + 1).
2

17. Find the number of terms in the series 20, 19 , 18 ,


…… of which the sum is 300.
1
3
2
3 nee
31. Find the first term and common ratio for a G.P.
1
whose sum to infinity is 3 and whose second term
8
18. The third term of an A.P. is 7 and its 7th term is 2
more than thrice of its 3rd term. Find the first term,
1
is ·
2 rin
19.
common difference and the sum of its first 20 terms.
If the sum of first 8 and 19 terms of an A.P. are 64 g.n
32. Find the sum of all odd numbers between 1 and 1000
which are divisible by 3.

20.
and 361 respectively. Find the common difference
and sum of its n terms.
The interior angles of a polygon are in arithmetic
progression. The smallest angle is 1200 and the
e
33. Find four numbers in A.P. whose
(a) Sum is 20 and Sum of their squares is 120.
(b) Sum is 32 and Sum of squares is 276. t
common difference is 50 . Find the number of sides of 34. Divide 28 into four parts in A.P., so that ratio of the
the polygon. product of first and third with the product of second
and fourth is 8 : 15.
21. Find the sum of n terms of the series
a2 a3 a4 Exercise B
log a + log + log 2 + log 3 + …… to n terms.
b b b
1. Find the Sum of series—
22. A class consists of number of boys whose ages are in
(i) (a + b) + (a2 +2b) + (a3 + 3b) + …… to n terms.
A.P. The common difference being four months. If
the youngest boy is just eight years old and if the (ii) (x + y) (x2 + xy + y2) + (x3 + x2y + xy2 + y3)
sum of the ages is 168 years. Find the number of + …… + n terms.
boys. 2. The fifth term of a G.P. is 81, whereas its second
23. The mth term of an A.P. is n and its nth term is m. term is 24. Find the series and sum of its first eight
Find the (m + n)th term. terms.
24. Between 1 and 31 are inserted m arithmetic means so 3. The first term of an infinite G.P. is 1 and any term is
that ratio of the 7th and (m – 1)th means is 5 : 9. equal to the sum of all the succeeding terms. Find the
Find the value of m. series.

CAT Complete Course | 247

Visit : www.EasyEngineering.net
Visit : www.EasyEngineering.net

4. If x = 1 + a + a2 + a3 + a4 + …… to ∞ (|a| < 1) 20. Find the sum of the series of n terms


y = 1 + b + b + b + …… to ∞
2 3 (|b| < 1) (62 – 32 ) + (72 – 42 ) + (82 – 52 ) + (92 – 62 ) …… nth
Now, find out 1 + a·b + a .b + a ·b + …… ∞
2 2 3 3 21. Sum of p, q, r terms of an A.P. be a, b and c
5. In an increasing G.P., the sum of the First and the respectively, then prove that
last term is 66. The products of the second and the a b c
(q – r) + (r – p) + (p – q) = 0
last but one term is 128 and the sum of all the terms is p q r
126. How many terms are there in the progression ? 22. Find the sum of first n terms of the series
6. The Sum of three numbers in G.P. is 14. If the first (22 + 4 2 + 5 2 ) + (32 + 5 2 + 7 2 ) + (42 + 6 2 + 9 2 ) + …
two terms are each increased by 1 and the third term 23. Find the sum of first n terms of the series
decreased by 1, the resulting numbers are in A.P.
Find the numbers. (53 + 3 3 ) + (73 + 5 3 ) + (93 + 7 3 ) + (113 + 9 3 ) + ……
7. In a set of four numbers the first three are in G.P. and 24. Find the sum of first n terms of the following series
the last three in A.P. with common difference 6. If 1 1 1 1
+ + + + ……
the first number is the same as the fourth. Find the 1·2 2·3 3·4 4·5
four numbers. 25. Find the sum of n terms of the series

ww
8. Three numbers from a G.P. If the 3rd term is
decreased by 64, then the three numbers thus
obtained will consecutive on A.P. If the second term
7 + 77 + 777 + 7777 + –––––––––– to n terms
26. Find the sum of n terms of the series—

w
12 – 22 + 3 2 – 43 + 5 2 – 62 + ……
of this A.P. is decreased by 8 a G.P. will be formed
27. Find the sum of the series 31 3 + 32 3 + …… + 503

.Ea
again determine the numbers.
9. The consecutive digits of a three digit number are in 28. Find the sum of n terms of the series
G.P. If the middle digit be increased by 2, then they 3 + 8 + 22 + 72 + 266 + 1036 + ……

syE
form an A.P. If 792 is subtracted from this number
the we get the number consisting of same three digits
but in reverse order. Find the number.
29. Find the sum of all possible products of the first n
natural numbers taken two by two.

ngi
10. If p, q, r, s, t form an A.P. Find the values of p – 4q +
6r – 4s + t.
30. Find sum of the series
n·1 + (n – 1)·2 + (n – 2)·3 + …… + 1·n

nee
31. Find the value of the expression
11. If the sum of n terms of an A.P. is (2n2 + 3n). Find
the tn term and common difference. 1·(2 – w) (2 – w 2 ) + 2 (3 – w) (3 – w2) + … + (n – 1)
(n – w) (n – w2 ) ……
12. If one G.M. G and two arithmetic means A and B be
inserted between any two given numbers, then show
that G2 = (2A – B) (2B – A). rin
where w is an imaginary cube root of unity.
32. Find the sum Sn of cubes of the first n terms of an
13. In a G.P. of real numbers, it is given that T1 + T2 +
T3 + T4 = 30 and g.n
A.P. and show that the sum of first n terms of the
A.P. is a factor of S n .
T1 2 + T2 2 + T3 2 + T4 2 = 340 determine G.P.
14. Find the sum of n terms of the series
3
+
5
+
7
+
9
12 12 + 2 2 12 + 2 2 + 3 2 12 + 2 2 + 3 2 + 4 2
+ ……
e
33. 150 workers were engaged to finish a piece of work
in a certain number of days. Four workers dropped
the second day, four more workers dropped the third
day and so on. It takes 8 more days to finish the work
t
6n now. Find the number of days in which the work was
= completed.
n+1
15. If a, b and l be the first, second and the last term of a 34. Along a road lay an odd number of stones placed at
G.P. respectively, then find the sum of the G.P. intervals of 10 metres. These stones had to be assem-
16. If Sn , S2n and S3n are the sum of first n, 2n and 3n bled around the middle stone. A person could carry
only one stone at a time. A man started the job with
terms respectively of a G.P., then show that
one of the end stones by carrying all the stones he
S n × (S3n – S2n) = (S2n – Sn )2 covered a distance of 3 km. Find the number of
17. If a, b and c are in A.P., then stones.
10ax + 10, 10bx + 10, 10cx + 10 [x ≠ 0] 35. Find the sum of integers from 1 to 100, which are
Find the series of given numbers. divisible by 2 or 5.
18. Find the value of 9 1/3 × 91/9 × 91/27 × …… ∞. 36. Find the sum of all odd integers between 2 and 100
19. Find the sum of series whose mth term is 6m(m + 2) divisible by 3.
where m = 1, 2, 3, …… and series has total number 37. Find the sum of all natural numbers which are multi-
to terms = n. ples of 7 or 3 or both and lie between 200 and 500.

248 | CAT Complete Course

Visit : www.EasyEngineering.net
Visit : www.EasyEngineering.net

Answers Sm + n =
m+n
{2a + (m + n –1)d}
2
Exercise A
By putting the value of a and
1. Let roots are a and b.
According to question— m + n  m2 + n2 + mn – m – n

2 
Sm + n =
mn
a+b
A.M. =
2 2(m + n – 1) (m + n) 
a+b –
mn 
5 =
2
m +n 1
a + b = 10 …(1) = 2× × [m 2 + n2 + mn – m – n
2 mn
G.M. = √ab – {m 2 + 2mn + n2 – m – n}]
4 = √ab =
m+n
(– mn)
mn
ab = 16 …(2)
Sm + n = – (m + n)
If a, b are the roots of a quadratic equation, then
4. Since, (b + c), (c + a), (a + b) are in H.P.

ww
X2 – (a + b) X + ab = 0
or X2 – 10 X + 16 = 0
2. According to question—

1
,
1
b+c c+a a+b
,
1
are in A.P.

w 3A = a + b + c

.Ea
G = a ×b×c
1
3

1 1 1 1 
…(1)
…(2)
Now,common difference remains constant.

So,
1

c+a b+c
1

2
=
1

1

a+b c+a
1

1 2b + a + c
=  + + 

syE …(3) = + =
H 3 a b c  c+a a + b b + c (a + b) (b + c)
3G3 or 2(ab + b2 + bc + ac) = (2b + a + c) (c + a)
a × b +b × c +c × a = or 2ab + 2b2 + 2bc + 2ac = 2bc + 2ab + a2 + c2

ngi
H
If l, m, n be the roots of a cubic equation, then + 2ac a2 + c2 = 2b2
X3 – (l + m + n)X2 + (lm + ln + nm)X – lmn = 0 That’s mean a2, b2 , c2 are in AP.

So, X3 – 3 AX2 +
3G3
H
X – G3 = 0
nee
5. Let Ist term = X
Common ratio = Y
3. Let a be the Ist term and d is the common difference.
According to question—
Now,
rin
P = X × Ya – 1
Q = X × Yb–1
S m = Sum of m terms

=
m
{2a + (m – 1)d} = n …(1)
R = X × Yc–1
g.n
Now, P b – c = (X·Ya – 1) (b – c) = Xb – c × Y(a – 1) (b – c)
2
n
S n = {2a + (n – 1)d} = m
2
2n
…(2) e t
Qc – a = (X·Yb – 1) (c – a) = Xc – a × Y(b – 1) (c – a)
R a – b = (X·Yc – 1) (a – b) = Xa – b × Y(c – 1) (a – b)
P b – c × Qc – a × Ra – b = X0 × Y0 = 1
From equation (1) = {2a + (m – 1)d}
m Hence, proved P b – c × Qc – a × Ra – b = 1
2m
From equation (2) = {2a + (n – 1)d} 6. Let a be the Ist term
n
r = common ratio
Now, equation (1) – equation (2), we get
2(n2 – m2) Now, Tm + n = P = a × r m + n – 1 …(1)
= (m – n)d
mn Tm – n = Q = a × rm – n – 1 …(2)
2(m + n) From equation (1) and equation (2), we get
d = –
mn 1
2n P  P  2n
and now, From equation (1) 2a = – (m – 1)d = r2n ⇒ r =   …(3)
m Q Q 
2n 2(m + n) Now, P = a × rm + n – 1
2a = + (m – 1) ×
m mn m+n–1
2 2
n + m + mn – m – n P  2n
a = P = a × 
mn Q 

CAT Complete Course | 249

Visit : www.EasyEngineering.net
Visit : www.EasyEngineering.net

 a – b  2ab – b2 – ab
m+n–1
Q  2n
H – b = b × =
a = P×  …(4) a + b  a+b
P 
Tm = a × rm – 1 a – b 
= – b× 
m+n–1  1  a + b 
Q    2n 
m–1
  
2n
P  
= P×    Q   1 1
= 
1 1
P  + +

 – a  a – b  b  a – b  
H–a H–b
m+n–1  m – 1
   a + b   a + b  
Q  2n  
   2n 
Q
= P×    P  
P   a + b   1 1 
=  
 a – b   – a b 
+
m+n–1 m–1
Q  –
2n 2n
= P×   (a + b) (a – b)  1 1
P   (a – b)ab  = a + b
Tm = √

P·Q
2ab  2b + a + b 
7. Let a = Ist term of an A.P. (iii) H+a = +a=a× 
a+b  a+b 

ww d = Common difference

Then, S m =
m
{2a + (m – 1)d} = a ×
 3b + a 
a+b 

w
2
n  2b + a + b 

.Ea
2ab
S n = {2a + (n – 1)d}
2 H+b = +b=b× 
a+b  a+b 
Sm m2
Given = 2  3a + b 

or
Sn n
m 2a + (m – 1)d
×
n 2a + (n – 1)d
syE m2
= 2
n H+a
= b ×
a+b 

a(3b + a)  3b + a 
or
2a + (m – 1)d
2a + (n – 1)d
=
m
n ngi H–a
=

 3a + b 
=
a(b – a)  b – a 

nee
H+b
Now, replaying m by (2m – 1) and n by (2n – 1), we =  
H–b  a–b 
get
2a + 2(m – 1)d 2m – 1 H+a H+b  3a + b   3b + a 
2a + 2(n – 1)d
a + (m – 1)d
=
2n – 1
2m – 1
H–a H–b
+ = –

rin + 
 b–a   b–a 

g.n
or = – 2a + 2b
a + (n – 1)d 2n – 1 = =2
b–a
tm 2m – 1 H+a H+b

tn
8. (i) Since, H be H.M. of a, b.

H =
2ab
=
2n – 1 +
H–a H–b
9. We know that
= 2

e  n(n + 1)  2
=
t
 2 
a+b 13 + 23 + 33 + …… + n3

 b – a – b – 2a2 Now,
8
Σr = 1 (3n3 + 2n) = 3(13 + 23 + 33 + … + 83)
H – 2a = 2a ×  =
 a+b  a+b + 2(12 + 2 2 + 3 2 + … + 82)
– 2a2  2
H – 2b =
Σr = 1 (3n3 + 2n) = 3  8(82+ 1) 
8
a+b
 
 2ab  2
4a2 b2
(H – 2a) (H – 2b) = =  8(8 + 1) (17)
+2×
(a + b)2  a + b  6
= H2 8
= 3 × 16 × 81 + × 9 × 17
3
2ab  2b – a – b
(ii) H–a = –a= a×  = 3888 + 408 = 4296
a+b  a+b  10. Σr = 1 (r + 1) = Σr = 1 (r2 + 2r + 1)
10 2 10

2ab – a2 – ab a – b  = (12 + 2 2 + 3 2 + … + 102) + (2·1 + 2·2 + 2·3 + ……


= =–a× 
a+b a + b  + 2·10) + (1 + 1 + 1 + 1 + 1 + 1 + 1 + 1 + 1 + 1)

250 | CAT Complete Course

Visit : www.EasyEngineering.net
Visit : www.EasyEngineering.net

10(10+ 1) (21) 10 × 11 14. (53 + 3 3 ) + (73 + 5 3 ) + (93 + 7 3 ) + (113 + 9 3 ) + …


= +2× + 10
6 2 = (53 + 7 3 + 9 3 + …) = its rth term = (2r + 3)3
5
= × 11 × 21 + 55 × 2 + 10 + (33 + 5 3 + 7 3 + …) = it rth term = (2r + 1)3
3
Now, rth term of the series = (2r + 1)3 + (2r + 3)3
= 35 × 11 + 110 + 10 = 385 + 120 = 505
tr = 16r3 + 48r2 + 60r + 28
11. (i) 3b + 12b2 + 48b3 + 192b4 + 768b5
Σr = 1 (16r3 + 48r2 + 60r + 28)
n
Sn =
1st term = 3b
 n(n + 1)  2  
2nd term = 12b2 = 16 ×   + 48 ×  n(n + 1)6(2n + 1) 
Common ratio = 4b  2   
∴ 3b + 12b2 + 48b3 + 192b4 + 768b5  n(n + 1) 
+ 60 ×   + 28n
= 3b + 3·(4b)·b + 3b·(4b)2 + 3b·(4b)3 + ……  2 
= Σr = 1 3b(4b)n – 1
n
= 4 × n2 (n2 + 2n + 1) + 8 × (n2 + n) (2n + 1)
We have 5 term, so n = 5 + 30 (n2 + n) + 28n
Σr = 1 3b(4b)n – 1
5
= 2n(2n3 + 12n2 + 29n + 33)

ww
(ii) 1·2 + 2·5 + 3·10 + 4·17 + 5·26 + …… + n(n2 + 1)
Ist digits of each term forms an A.P., so its rth term = r
15. rth term of the given series is represented by
1
(r + 2) (r + 3)

w
2nd digits of each term are 2, 5, 10, 17, 26,

.Ea
2, 22 + 1, 3 2 + 1, 4 2 + 1, 5 2 + 1, ……,
12 + 1, 2 2 + 1, 3 2 + 1, 4 2 + 1, 5 2 + 1, ……
tr =

t1 = –
1

1 1

r+2 r+3
1

∴ rth term = r2 + 1

syE
Now, rth term of above series = r(r2 + 1)
Now, Σr = 1 r(r2 + 1)
n
t2 = –
3 4
1 1
4 5

12. Clearly, rth term =


(r + 3)2 – r2
r2
= 6r + 9
ngi t3 = –
………… = …………
1 1
5 6

Now, S n = Σr = 1 (6r + 9)
n

= 6×
n(n + 1)
+ 9n
nee ………… = …………
………… = …………

rin
2 1 1
tn = –
n+2 n+3
= 3n(n + 1 + 3)

g.n
1 1
= 3n(n + 4) S n = t1 + t2 + t3 + …… + tn = –
3 n–3
13. We can write the above series as n
(12 + 2 2 + 3 2 + 4 2 + ……)
+ (32 + 4 2 + 5 2 + 6 2 + ……)
2 2 2
+ (3 + 5 + 7 + 9 + ……)
2
2
…(1)
…(2)
…(3)
Sn =
3(n + 3)
16. When n is even n = 2m e
Then, 3 – 5 + 7 – 9 + 11 – 13 + 15 – 17 + …… t
Now, rth of 1st = r S n = (– 2) + (– 2) + (– 2) + …… m times
rth of 2nd = (r + 1)2 S n = – 2m
rth of 3rd = (2r + 1)2 n
S n = – 2 × = – n when n is even.
Now, rth term = r2 + (r + 1)2 + (2r + 1)2 2
tr term = r2 + r2 + 4r + 4 + 4r2 + 4r + 1 When n is odd n = 2m – 1
= 6r2 + 8r + 5 3 – 5 + 7 – 9 + 11 – 13 + 15 – 17 + ……
Σr = 1 tr = Σr = 1 (6r2 + 8r + 5)
n n
Now, = (3 + 7 + 11 + 15 + …… + m terms)
n(n + 1) (2n + 1) – (5 + 9 + 13 + 17 + …… (m – 1) terms)
= 6× +8
6 2m – 2
S n = 3 + (– 5 + 7) + (– 9 + 11) + ……
n(n + 1) 2
× + 5n
2 = 3 + 2 + 2 + 2 + …… (m – 1) times
= (n2 + n) (2n + 1) + 4n2 + 4n + 5n = 3 + 2 (m – 1)
= 2n3 + n2 + 2n2 + n + 4n2 + 9n = 2m + 1
= 2n3 + 7n2 + 10n S n = n + 1 where n is odd

CAT Complete Course | 251

Visit : www.EasyEngineering.net
Visit : www.EasyEngineering.net

17. Given Ist term (a) = 20 or 22 = 11d


1 2 or d = 2 and a = 1
Common difference (d) = 19 – 20 = –
3 3 n
So, Sn = [2 × 1 + (n – 1) × 2]
Let n be the number of terms. 2
Using formula— = n2
n 20. Let the number of the sides of the polygon = n
S n = [2a + (n – 1)d]
2 Ist angle of the polygon = 120°
n  2   d = 5°
or 300 =  40 + (n – 1)  – 3  
2 Now, Sum of all the interior angles = (n – 2)
n  120 – 2n + 2  × 180 …(1)
or 300 = ×  
2  3 n
Sum of angles = [2a + (n – 1)d]
2
n 122 – 2n n(61 – n)
or 300 = × = n
2 3 3 = [240 + (n – 1) × 5] …(2)
2
or n2 – 61n + 900 = 0

ww
or n2 – 36n – 25n + 900 = 0
or n(n – 36) – 25(n – 36) = 0
Since, equation (1) and equation (2) are equal.
n
(n – 2) × 180 = [235 + 5n]
2

w
It has double answers.
n = 25, n = 36

.Ea
Because, the last eleven terms (36 – 25) are present
whose sum is zero.
or (n – 2) × 180 × 2 = 5n(47 + n)
or n2 + 47n = (n – 2) × 72
or n2 – 25n + 144 = 0

18. Let Ist term = a


Common difference = d syE or
If n = 16
n = 9, 16
Tn = a + (16 – 1)d
= 120 + 15 × 5 = 195°
According to question—
t3 = a + 2d = 7 …(1) ngi Since, interior angle can not be greater than 180°.

nee
Hence, only n = 9 acceptable.
t7 = a + 6d = 3 × 7 + 2 = 23 …(2)
21. Ist term of the series = log a
Now, from equations (1) and (2), we get
a2 a

rin
4d = 16 Common difference = log – log a = log
b b
d = 4 = Constant

Now, S 20 =
a = 7 – 2d = 7 – 8 = – 1
20
× [2a + (n – 1)d]
So, Sn =
n
2 g.n
[2a + (n – 1)d]
2
= 10 × [2 (– 1) + 19 (4)]
= 740
19. Let Ist term = a
=

=
n 
2 

n 
e
2 log a + (n – 1) × log

a 
 n log b + log ab 
a
b


t
2
Common difference = d
22. According to question—
8
Now, S 8 = × [2a + (8 – 1)d] First term = a = 8 years
2
= 4 × [2a + 7d] Common difference = d =
4 1
= years
12 3
64 = 8a + 28d
16 = 2a + 7d …(1) Let n be the number of boys in the class.
19 n
S 19 = [2a + (19 – 1)d] So, S n = [2a + (n – 1)d]
2 2
19 n  1 
361 = × [2a + 18d] or 168 =  2 × 8 + (n – 1) × 3 
2 2
19 = a + 9d …(2) (48 + n – 1)
or 168 × 2 = n ×
Now, equation (1) – equation (2), we get 3
38 – 16 = 18d – 7d or (n + 47) × n = 168 × 2 × 3

252 | CAT Complete Course

Visit : www.EasyEngineering.net
Visit : www.EasyEngineering.net

or n2 + 47n – 168 × 2 × 3 = 0 Now, the number are when β = 2, 3, 5, 7, ……


or n2 + 63n – 16n – 168 × 6 = 0 When β = – 2, 7, 5, 3, ……
or n(n + 63) – 16 (n + 63) = 0 26. Let the three consecutive terms of Ist series are a – d,
n = 16 a, a + d
So, 16 boys are present in class. and 2nd series has A – b, A, A + b
23. Let Ist term = a Given b = d+1 …(1)
P 2 2
(a – d )a
Common difference = d =
P1 (A 2 – b2 )A
tm = a + (m – 1)d = n …(1)
7 (a2 – d2 )a
tn = a + (n – 1)d = m …(2) or = …(2)
8 [A 2 – (b + 1)2 ]A
From equations (1) and (2), we get Since, a – d + a + a + d = S = 15
(m – 1)d – (n – 1)d = n – m 3a = 15, a = 5
or (m – n)d = n – m and A – b + A + A + b = 15, A = 5
d = –1 …(3) Now, putting the value of a and A in equation (2),

ww
or
a + (m – 1) (– 1) = n
a = n+m–1
Now, tm + n = a + (m + n – 1)d
…(4)
we get
7
8
=
(25 – d2)5
[25 – (d + 1)2 ]5

w = (m + n – 1) + (m + n – 1) (–1)
tm + n = 0
.Ea
24. Let d be the common difference.
or

or
7
8
=
25 – d2
25 – (d + 1)2
7 × 25 – 7(d + 1)2 = 25 × 8 – 8d2

syE
First term = 1
Last term = 31
or
or
8d2 – 7(d + 1)2 = 25
d2 – 14d – 7 = 25
Since, m terms are inserted between 1 and 31 so, 31
is the (m + 2)th term of A.P.
Now, 31 = tm + 2 = a + (m + 2 – 1)d ngi or
or
d2 – 14d – 32 = 0
d2 – 16d + 2d – 32 = 0

a + 7d
31 = a + (m + 1)d
5
…(1)
nee or d(d – 16) + 2(d – 16) = 0
d = 16

rin
= Now, Ist series is –11, 5, 21, ……
a + (m – 1)d 9
4a = 5md – 5d – 63d 2nd series is –12, 5, 22, ……
or
or
4a = 5md – 68d
(5m – 68)d = 4 So, g.n
27. A be the Ist term and d be the common difference.
tp = A + (p – 1)d = a …(1)

or (5m – 68) ×

or
or
30
m+1
= 4

(5m – 68) × 30 = 4m + 4
75m – 68 × 15 = 2m + 2
e
tq = A + (q – 1)d = b
Now, equation (1) – equation (2), we get
(p – q)d = a – b
…(2)

t
a–b
or 73m = 1022 d = …(3)
p–q
1022
or m = = 14 A + (p – 1)d = a
73
25. Let the three numbers of A.P. are α – β, α, α + β. a – b 
A = a – (p – 1)  
According to question— p – q 
α – β + α + α + β = 15 a(p – q) – (p – 1) (a – b)
Now, A =
3α = 15 ⇒ α = 5 p–q
and (α – β) + α + (α + β)2 = 83
2 2
or A =
– aq + pb + a – b
or (5 – β)2 + 25 + (5 + β)2 = 83 p–q
or 2(25 + β2) = 83 – 25 = 58 p+q
Now, Sp + q = [2A + (p + q – 1)d]
2
or 25 + β2 = 29
or β2 = 4 = 2 2 p + q  a(1 – q) + b(p – 1) a – b 
 2×
p – q 
= + (p + q – 1)
or β = ±2 2  p–q

CAT Complete Course | 253

Visit : www.EasyEngineering.net
Visit : www.EasyEngineering.net

p+q 1 n
= × [2a(1 – q) + 2b(p – 1) + (p + q – 1) = [(2 + 2 × 2 + 2 × 3 + 2 × 4 + …… + 2 × m)
2 p–q 2
(a – b)]
+ (n – 1) (1 + 3 + 5 + 7 + …… + (2m – 1)]
p + q  a – b 
=  a+b+
p – q  n  m(m + 1) 
2  =  2× + (n – 1)m 2 
2  2 
28. From Ist series
n
L = a + (n – 1)d …(1) = (m2 + m + n × m2 – m2)
2
L1 = p + (n – 1)q …(2) n
L L1 = m(mn + 1)
Given = =4 2
p a
31. Let 1st term = a
L p
From First two = …(3) Common ratio = r
L1 a
Here, r < 1
a + (n – 1)d = 4p
a
p + (n – 1)q = 4a Now, Sum of infinity terms =
1–r
4p – a

ww
d 1 a
= 3 =
4a – p q 8 1–r
n 25 a
(a + b)

w
Sn 2 a + 4p or = …(1)
and = = =2 8 1–r
S1n n p + 4a

2p = 7a .Ea
2
(p + L1 ) and ar =
1
2
1
⇒ a=
1
2r

Or
a
2
d
p
= =k
7
syE
28k – 2k or
25
8
=
2r
1–r
25(1 – r)r = 4


L
q
=
8k – 7k
p 7
= 26

ngi or
or 2
2
25r – 25r + 4 =
25r – 20r – 5r + 4 =
0
0

nee
= =
L1 a 2
or 5r(5r – 4) – r(5r – 4) = 0
29. Ist term of all A.P.’s = 1 4 1
or r = ,
Now, nth term of 1st A.P. = 1 + (n – 1) 1 = n
nth term of 2nd A.P. = 1 + (n – 1) 2 = 2n – 1
or rin ar =
5 5
1
nth term of 3rd A.P. = 1 + (n – 1) 3 = 3n – 2
Similarly, where g.nr =
2
4
5
a=
5
8
nth term of nth A.P. = 1 + (n – 1)n = n2 – (n – 1)
Now, their sum is
S n = [n + 2n – 1 + 2n – 2 + …… + n2 – (n – 1)]
When
e
r =
1
5
32. Odd numbers between 1 and 1000 are
a=
5
2
t
n(n + 1) n 2 3, 5, 7, 9 11, 13, 15, 17, 19, …… 999
S n = n + (n – 1) × = (n + 1)
2 2
Now, odd numbers divisible by 3 are 3, 9, 15, 21,
n n …… 993, 999
30. Now, S 1 = [2 × 1 + (n – 1)1] = [2 × 1 + (n – 1)1]
2 2
Now, Ist term = 3
n
S2 = [2 × 2 + (n – 1)3] Common difference = 6
2
Last term = 999
n
S3 = [2 × 3 + (n – 1)5] If nth term be 999
2
Now, S1 + S2 + S3 + …… + Sm Then, 3 + (n – 1)6 = 999
n n or 6n – 3 = 999
= [2 × 1 + (n – 1)1] + [2 × 2 + (n – 1)3]
2 2 or 6n = 1002
n n or n = 167
+ [2 × 3 + (n – 1)5] + …… + [2 × m
2 2
167
+ (n – 1) (2m – 1)] Sn = (first + Last terms)
2

254 | CAT Complete Course

Visit : www.EasyEngineering.net
Visit : www.EasyEngineering.net

167 167 or 3β2 + 46β – 49 = 0


Sn = (3 + 999) = × 501
2 2 49
or β = 1, –
S n = 501 × 167 3
S n = 83667 Suitable value of β = 1
33. (a) Let the four terms of A. P. are α – 3β, α – β, Now, numbers are 4, 6, 8, 10.
α + β, α + 3β. Exercise B
According to questions—
1. Given series can be written as
α – 3β + α – β + α + β + α + 3β = 20
(a + a2 + a3 +…+ n terms) + (b + 2b + 3b +…+ n terms)
4α = 20
First Series Second Series
α = 5
First Series—Is in G.P. whose First term = a
and (α – 3β)2 + (α – β)2 + (α + β)2 + (α + 3β)2 = 120
Common ratio = a
or 2(α + 5β2 ) + 2(α 2 + β2) = 120
 rn – 1  an – 1 
or α2 + 5β2 = 30 Now, its Sum = a r – 1  = a  a – 1 
   
or 25 + 5β2 = 30

ww
When
β = ±1
β = 1
Second Series—b + 2b + 3b + 4b + …… + n terms
b(1 + 2 + 3 + 4 + …… + n terms)
b × Sum of n first natural numbers

w
Numbers are 2, 4, 6, 8
When
Numbers are 8, 6, 4, 2
.Ea β = –1

So,
= b×

Net Sum = a × 
n(n + 1)
2
 an – 1  b
 + n(n + 1)
α + β, α + 3β
syE
(b) Let the four terms of A. P. are α – 3β , α – β,

α – 3β + α + β + α – β + α + 3β = 32
 a–1  2
Dividing each term by (x – y) and multiplying each
by (x – y), we get
4α = 32
α = 8 ngi Sn =
x2 – y2 x3 – y3 x4 – y4
x–y
+
x–y
+
x–y
+ …… + n terms

and (α – 3β)2
or
+ (α – β)2 + (α
4(2 +
+ β)2
5β2)
+ (α +
= 276
3β)2 = 276
nee =
1
x–y
[(x2 + x3 + x4 + …… n terms) – (y2 + y3 + y4
+ …… n terms)]
1  x2(1 – xn ) y2(1 – yn ) 
or
or
α2 + 5β2
64 + 5 β 2 = 69
= 69 =
x – y  1 – x

rin
1 – y 

or
or
5 β2 = 5
β = ±1
2. Let First term = a
Common ratio = r g.n
When
Numbers are 5, 7, 9, 11
When
β = 1

β = –1
Now,
Fifth term = ar4 = 81

r3 = = e
81 27  3 
= 
24 8  2 
3

t
24
Numbers are 11, 9, 7, 5. Now, a = × 2 = 16
3
34. Let four consecutive terms of A.P. are α – 3β, α – β, Series is 16, 24, 36, 54, ……
α + β, α + 3β.
Sum of its first eight terms
According to question—
 r8 – 1 
α + β + α – 3β + α – β + α + 3β = 28 = a r – 1 
 
or 4α = 28
3  8 – 1 
or = 7 …(1)
 2  
(α – 3β) (α + β)
=
8 = 16 ×  
 
(α – β) (α + 3β) 15 3
–1
2
or 15(α – 2αβ – 3β ) = 8(α 2 + 2αβ
2 2
38 – 28
– 3β2) = 16 × ×2
28
or 7α2 – 46αβ – 21β2 = 0 81 × 81 – 32 × 8
= 16 × ×2
or 7 × 49 – 46 × 7β – 21β2 = 0 32 × 8

CAT Complete Course | 255

Visit : www.EasyEngineering.net
Visit : www.EasyEngineering.net

6561 – 256 rn – 1
= ×2 Sum = a × = 126
16 r–1
1 2(32r – 1)
= × 6305 or = 126
8 r–1
3. Let First term = a = 1 or 32r – 1 = 63 (r – 1)
Common ratio = r or 31r = 63 – 1 = 62
According to question— r = 2 (r > 1)
Tn = Tn + 1 + Tn + 2 + Tn + 3 + … + ∞ So, rn – 1 = 32, 2n – 1 = 32 = 25
or a × rn – 1 = a × rn + a × rn + 1 + … + ∞ n = 6; r = 2; a = 2
arn 6. Let First term of G.P. = a
or a × rn – 1 =
1–r Common ratio = r
or (1 – r) × r n – 1 = rn Now, according to question—
or rn – 1 – rn = rn a + a.r + a.r2 = 14
rn
or = 2 × rn or a × (1 + r + r2 ) = 14

ww r

r =
1
2
Now, (a + 1), (ar + 1), (a.r 2 – 1) these are in A.P.
So, 2(ar + 1) = (a + 1) + (a.r 2 – 1)

4.
w 1 1 1
So, series is 1, , , ……
2 4 8

.Ea
x = 1 + a + a2 + a3 + a4 + …… ∞
= Sum of infinity series whose common ratio is less
or
or
2(ar + 1) = a(1 + r2 )
a(1 + r2 – 2r) = 2
1 + r + r2
=
14
=7
than 1.

=
1 syE 1 + r2 – 2r 2
1 + r2 + r = 7 + 7 × r2 – 14r
6r2 – 15r + 6 = 0

y =
–a+1
1
1–b
or y (1 – b) = 1 ngi 2r2 – 5r + 2 = 0
1

b = 1– =
1 y–1
y y nee r = 2,
2
So, numbers are 2, 4, 8 or 8, 4, 2.
Now, 1 + a.b +
1
a2.b2
+ + …… to ∞
a3 .b3
1
7.
rin
Since, the last three are in A.P.
So, these numbers are (a – d), a, (a + d).

g.n
= =
1 – a.b y–1 x–1 Since, these number are also in G.P.
1– ×
y x
Since, first number is same as that of last.

5. Let
=
x.y
x+y–1
First term = a
Common ratio = r (r > 1)
e
So, numbers in G.P. are (a + d), (a – d), a
(a – d) 2 = a(a + d) (d = 6)
(a – 6) 2 = a(a + 6)
t
36 – 12a = 6a
Number of terms = n
18a = 36
Now, according to question—
a = 2
a + a × rn – 1 = 66
So, Four numbers are 8, – 4, 2, 8.
a × r × a × rn – 2 = a2 × rn – 1 = 128
128 8. Let three numbers a, a, r, ar2 are in G.P. and
a × rn – 1 =
a a, a, r, (ar2 – 64) are in A.P.
128 Now, 2ar = a + ar2 – 64
66 – a =
a
a × (r2 – 2r + 1) = 64 …(1)
or 2
a – 66a + 128 = 0
Again, a, (ar – 8), (ar2 – 64) are in G.P.
or (a – 2) (a – 64) = 0
(ar – 8) 2 = a(ar2 – 64)
or a = 2, 64
– 16ar + 64 = – 64a
1
rn – 1 = 32,
32 a × (16r – 64) = 64 …(2)

256 | CAT Complete Course

Visit : www.EasyEngineering.net
Visit : www.EasyEngineering.net

From equations (1) and (2), we get Then, X, A, B, Y are in A.P.


r2 – 2r + 1 So, 2A = X + B
= 1
16r – 64 2B = A + Y
or r2 – 2r + 1 = 16r – 64
2A – B = X …(3)
or 2
r – 18r + 65 = 0
2B – A = Y …(2)
or (r – 13) (r – 5) = 0
From equations (1), (2) and (3), we get
r = 13, 5
G2 = (2A – B) (2B – A)
So, numbers are 4, 20, 100 are in G.P.
13. Let First term = a
4, 20, 36 are in A.P.
9. Let digits of the three digit number are x, y, z. Common ratio = b
Three numbers in G.P. are a, a.b, ab 2 . a + ab + ab2 + ab3 = 30
According to question— a(1 + b + b2 + b3 ) = 30
a, (ab + 2), (ab2 ) are in A.P. Similarly, a2 (1 + b2 + b4 + b6 ) = 340
So, 2(ab + 2) = a + ab2  1 – b4 

ww
Or 2ab + 4 = a(1 + b2 )
a(b – 1) 2 = 4 …(1)
a ×
1–b
 = 30

 1 – b8 

Or
w
Again, 100 a + 10 ab + ab – 792 = 100 ab2 +
10 ab + a

.Ea
99(b2 – 1)a = – 792
2

or
1 – b2
×
a2 × 
 1 – b2 
a2 (1 – b8)  1 – b 
 a(1 – b4) 
 = 340
2

=
340
30 × 30

syE
Or (b2 – 1) (b + 1)a = – 8 …(2)
From equations (1) and (2), we get (1 + b4 ) 1 – b 17
or × =
1 – b4 1 + b 45
b+1

ngi
= –2 or 17(1 + b + b2 + b3 ) (1 + b) = 45(1 + r4 )
b–1
1 or 14b4 – 17b3 – 17b2 – 17b + 14 = 0
b =

∴ a = 9
3

Hence, numbers are 9, 3, 1 or 931.


nee or

14 ×  b2 +

1
b2 



 – 17  b + 1b  – 17 = 0

10. Let d be common difference of the given A.P. Put


rin  1
b + b  = X
 
Now, p – 4q + 6r – 4s + t
or = p – 4(p + d) + 6(p + 2d) – 4(p + 3d) + (p + 4d) or 14(X 2
g.n
– 2) – 17X – 17 = 0
1
= p – 4p + 6p – 4p + p – 4d + 12d – 12d + 4d
= 0
11. Given S n = sum of nth term = Sn
S n = 2n2 + 3n …(1)
or

e
Required G.P.’s are 2, 4, 8, 16, 32
X = 2,

a = 2
2

t
and 16, 8, 4, 2, 1, ……
S n – 1 = 2(n – 1) 2 + 3(n – 1) …(2) 14. rth term of the given series is
nth term = S n – S n – 1 3 + (r – 1)2
tr = 2
= 2(n – 1) 2 + 3(n – 1) – (2n2 + 3n) 1 + 2 2 + 3 2 + …… + r2
= 2(2n – 1) (– 1) + 3 (– 1) 2r + 1
tr =
r(r + 1) (2r + 1)
= – 4n + 2 – 3n 6
tn = 2 – 7n
6(2r + 1) 6
where we put n = 1 = =
r(r + 1) (2r + 1) r(r + 1)
First term = 2 – 7 × 1 = 2 – 7 = – 5  r + 1 – r  1 1 
12. Let G be G.M. of X and Y, then = 6 ×   =6× – 
 r(r + 1)  r r + 1 
G2 = X.Y …(1)
1 1 
Since, A and B be two arithmetic means between X t1 = 6 ×  – 
and Y. 1 2 

CAT Complete Course | 257

Visit : www.EasyEngineering.net
Visit : www.EasyEngineering.net

1 1  Now, 102(bx + 10) = 102bx + 20


t2 = 6 ×  – 
2 3  = 10(c + a)x + 20
1 1  = 10cx + 10 × 10ax + 10
t3 = 6 ×  –  Clearly, 10ax + 10, 10bx + 10, 10cx + 10 are in G.P.
3 4  1 1 1
………………………… 18. Given 9 3 × 9 9 × 9 27 × …… ∞
…………………………
= 9
( )
1 +1 + 1 + ……
3 9 27
1 1 
tn = 6 ×  –  1 1 1 1
 n n + 1 + + + + …… are in G.P.
3 9 27 81
 1  1 1 1 3 1
t1 + t2 + t3 + …… + tn = 6 ×  1 –  = × = × =
 n+1 3
1–
1 3 2 2
3
6n
= S n = 91/2 = 3
n+1
15. Let last term will be nth term so l = arn – 1 19. Given tm = 6m(m + 2)

= ww
Since, b is second term so r = common difference
b
a
Sn = Σn m = 1 tm = Σn m = 1 6m(m + 2)
= Σ m = 1 (6m2 + 12m)
n

w l = a×  
b  n – 1
a 
.Ea b  – 1  n
= 6×
n(n + 1) (2n + 1)
6
= n(n + 1) (2n + 7)
+ 12 ×
n(n + 1)
2

 a   20. We divide the series in two parts such as


Sum = a× 
syE
 r – 1
n

r – 1 
=a× 
 ba – 1 
 62 + 7 2 + 8 2 + 9 2 + …… nth
and 32 + 4 2 + 5 2 + 6 2 + …… nth
…(1)
…(2)
a(bl – a2 )
= 2
a (b – a)
×a=
bl – a2
b–a
ngi rth term of First series
= [6 + (r – 1) × 1]2

nee
16. Let First term = a = (r + 5)2
Common ratio = r rth term of Second series
 rn – 1 = [3 + (r – 1) × 1]2
Now, sum is given by S n = a r – 1 
 
rin
= (r + 2)2
Now, Sn (S3n – S2n)

=a×
 rn – 1  r3n – 1
  a× –a×
r2n – 1 
r – 1 
Now, rth term the net series

g.n
= (r + 5)2 – (r + 2)2

= a2 × 
r – 1  


 r – 1  
r–1
 rn – 1  r3n – 1 – r2n + 1 
r–1  Now,
e
= (2r + 7) (3)
= 6r + 21
t
S n = Σ r = 1 tr = Σ r = 1 (6r + 21)
n n

n(n +1)
 rn – 1 2 2n = 6× + 21n
= a2 ×   ×r 2
r – 1 
= 3(n2 + n) + 21n
 rn – 1 2
=  a × × rn  Sn = 3n2 + 24n
 r–1  21. Let First term = X
 r2n – rn  2

=  a × Common difference = Y
 r – 1  p
a = × [2X + (p – 1)Y]
 r2n – 1 rn – 1
2
2
= a × –a×
 r–1 r – 1  a 1
or = X + × (p – 1)Y
p 2
= [S2n – Sn ]2
q
17. Since, a, b, c are in A.P. b = × [2X + (q – 1)Y]
2
So, 2b = c + a b 1
or = X + × (q – 1)Y
2bx = (c + a) x q 2

258 | CAT Complete Course

Visit : www.EasyEngineering.net
Visit : www.EasyEngineering.net


n
r (16r3 + 48 r2 + 60 r + 28)
c = × [2X + (r – 1)Y] r=1
2
 n(n + 1)  2
c 1 = 16 ×   + 48 × n(n + 1)6(2n + 1)
or
r
= X + (r – 1)Y
2  2 
n(n + 1)
a b c + 60 × + 28n
= (q – r) + (r – p) + (p – q) 2
p q r
    = 4 × n2 (n + 1)2 + 8n(n + 1) (2n + 1) + 30n(n + 1)
1 1
=  X + 2 × (p – 1)Y  +  X + 2 × (q – 1)Y  + 28n
= 4 × n2 (n2 + 2n + 1) + 8n(2n 2 + 3n + 1) + 30n 2 +
 
+  X + × (r – 1)Y 
1 30n + 28n
 2  = 2n[2n(n2 + 2n + 1) + 4(2n2 + 3n) + 15n + 33]
= 0 = 2n[2n3 + 4n2 + 2n + 8n2 + 12n + 15n + 33]
22. Given series is divided into three different parts, such = 2n[2n3 + 12n2 + 29n + 33]
as 24. Now, rth term of the given series is
22 + 3 2 + 4 2 + ………… …(1) 1 r+1–r

ww
tr = =
42 + 5 2 + 6 2 + ………… …(2) r(r + 1) r(r + 1)
2 2 2
5 + 7 + 9 + ………… …(3) 1 1
= –
r r+1

w
rth term of series first
= [2 + (r – 1) × 1]2 = (r + 1)2
rth term of series second
.Ea
= [4 + (r – 1) × 1]2 = (r + 3)2
Now, t1 = –

t2 = –
1 1
1 2
1 1

syE
2 3
rth term of series third 1 1
t3 = –
= [5 + (r – 1) × 1]2 = (r + 4)2 3 4
Now, rth term of net series
= tr = (r + 1)2 + (r + 3)2 + (r + 4)2
ngi t4 = –
………………
1 1
4 5

Sn = Σn r = 1 tr
S n = Σ r = 1 [(r + 1)2 + (r + 3)2 + (r + 4)2 ]
n
nee ………………
tn = –
1
n n+1
1
= Σ r = 1 (3r2 + 16r + 26)

rin
n
1 1
n(n + 1) (2n + 1) n(n + 1) t1 + t2 + t3 + …… tn = 1 – =
= 3× + 16 × n+1 n+1

g.n
+ 26n
6 6 1
Sn =
n(n + 1) (2n + 1) n+1
= + 8n(n + 1) + 26n
2
= n[2n2 + 3n + 1 + 16n + 16 + 52]
= n(2n2 + 19n + 69)
23. Given series can be written as
e
25. Given series is 7 + 77 + 777 + 7777 + …… n terms
= 7[1 + 11 + 111 + 1111 + …… n terms]
7
= [9 + 99 + 999 + 9999 + …… n terms]
9
t
7
(53 + 7 3 + 9 3 + 11 3 + …) + (33 + 5 3 + 7 3 + 9 3 + …) = [10 –1 + 102 –1 + 103 – 1 + 10 4 – 1 + … n terms]
9
Its rth term of first series 7
= [5 + (r – 1) × 2]3 = [(10 + 102 + 10 3 + 10 4 + …… + 10n) – n]
9
= (2r + 3)3 7   10n – 1  
=  10 ×  – n 
Its rth term of second series 9   10 – 1 
= [3 + (r – 1) × 2]3
7  10 
= (2r + 1)3 =  × (10n – 1) – n 
9 9 
Now, rth term of the given series
26. Given series can be written as
= (2r + 3)3 + (2r + 1)3 (12 + 3 2 + 5 2 + ……) – (22 + 4 2 + 6 2 + ……)
= 8r3 + 27 + 36r2 + 54 r + 8r2 + 1 + 12r2 + 6r Now, First series = S 1 = 1 2 + 3 2 + 5 2 + ……
= 16r3 + 48r2 + 60 r + 28 and second series = S 2 = 2 2 + 4 2 + 6 2 + ……
Now, Sum = Sn = Σ r = 1 tr
n
Sn = S1 – S2

CAT Complete Course | 259

Visit : www.EasyEngineering.net
Visit : www.EasyEngineering.net

Now, rth term of S 1 series 28. Given series is 3 + 8 + 22 + 72 + 266 + 1036 + ……


= [1 + (r – 1) × 2]2
= (2r – 1)2 First difference 5, 14, 50, 194, 770
S n = Σ r = 1 (2r – 1) = Σ r = 1 (4r2 – 4r + 1)
m m
2 Second difference 9, 36, 144, 576
m(m + 1) (2m + 1) m(m + 1) They are in G.P. where nth term is
S1 = 4 × –4× +m
6 2 a.rn – 1 = a.4 n – 1
2 Now, T n term of the given series will be of the form
= × m(2m2 + 3m + 1) – 2m(m + 1) + m
3 Tn = a 4 n – 1 + b n + c
m m
= [4m2 + 6m + 2 – 6m – 6 + 3] = (4m 2 – 1) When n = 1 T1 = a + b + c = 3
3 3
m n = 2 T2 = 4a + 2b + c = 8
= (2m – 1) (2m + 1)
3 n = 3 T3 = 16a + 3b + c = 22
and, rth term of S 2 series Solving them in First equation, we get
= [2 + (r – 1) × 2]2 = (2r)2 a = 1, b = 2, c = 0
= 4r2 Tn = 4n – 1 + 2n

ww
It’s Sum

= Σmr = 1 4r2 = 4 × m(m + 1)6 (2m + 1)


S n = Σ4n – 1 + Σ2n

Sn = 1 ×
4n – 1
+2×
n(n + 1)

w
4–1 2
2 1
=
3
.Ea
m(m + 1) (2m + 1)

Now, Sum of the given series 29. We know that


=
3
× (4n – 1) + n2 + n

= S1 – S2

=
m
3
syE 2
(2m – 1) (2m + 1) – m(m + 1) (2m + 1)
3
= (X 1 + X2 + X3 + …… + Xn)2
= (X 1 2 + X2 2 + X3 2 + …… + Xn 2 ) + 2[X 1 X2 + X2 X3

Sum = – m(2m + 1)
Case-I—When n is odd.
ngi = ΣXi2 + 2ΣXiXj
+ X3X4 + …… + X1Xn]

Let
where
n = 2m + 1
m = 0, 1, 2, 3
nee Now, We put X1 = 1, X2 = 2, X3 = 3, X4 = 4,
Xn = n, we get

Sum = – m(2m + 1)
rin
= (1 + 2 + 3 + …… + n)2
= 12 + 2 2 + 3 2 + …… + n2
n – 1 
= –
 2 
 (n) = – n2 (n – 1)
 n(n + 1)  2 g.n + 2Σ XiXj

Case-II—n is even.
Let n = 2m
Sum = – m (2m + 1)
(m = 1, 2, 3, ……)
or

or
 2  =

2Σ XiXj =
e
n(n + 1) (2n + 1)
6
+ 2Σ XiXj

t
n(n + 1)  n(n + 1) (2n + 1) 
× – 
n 2  2 3
= – (n + 1)
2 n(n + 1) 3n2 + 3n – 2n – 1
= ×
27. We can write 313+ 32 3 + …… + 503 as 2 6
2
n(n + 1) 3n + n – 1
(13 + 2 3 + 4 3 + …… + 303 + 31 3 + 32 3 + …… + 503) = ×
2 6
– (13 + 2 3 + …… + 303)
30. rth term of the given series is
 50(50 + 1)  2  30(30 + 1)  2
=   –   = [n + (r – 1) × 1]r
 2 2
= (n + r – 1)r
 50 × 51  30 × 31
2 2
= nr + r2 – r
=   – 
 2   2  Now, Sum = Σ (nr + r2 – r)
= (25 × 51)2 – (15 × 31)2 n(n + 1) n(n + 1) (2n + 1)
= n× +
2 6
= 25[(255)2 – (93) 2 ] n(n + 1)

= 25 × 348 × 162 2

260 | CAT Complete Course

Visit : www.EasyEngineering.net
Visit : www.EasyEngineering.net

n(n + 1)   Had the workers not dropped then the work would
– 1 
2n + 1
=
2 

n+
3  have finished in (n – 8) days with 150 workers work-
ing on each day
n(n + 1)  3n + 2n + 1 – 3 
= ×  i.e. 150 (n – 8)
2  3  n(152 – 2n) = 150 (n – 8)
n(n + 1) 5n – 2 n2 – n – 600 = 0
= ×
2 3
n = 25
31. (r–1)th term of the given series is
34. Let the number of stones be (2n + 1) so that there is
tr – 1 = (r – 1) (r – w) (r – w 2 ) one mid–stone and n stones each on either side of it.
n n If P be mid stone and A, B be last stones on the left
Now, S = Σ tr = Σ [(r – 1) (r – w) (r – w2)] and right of P respectively. There will be (n) stones
r=1 r=1
n on the left and (n) stones on the right side of P or n
= Σ (r – 1) [r2 – r(w + w2) + w3 ] intervals each of 10 metres both on the right and left
r=1 side of mid–stone. Now, he starts from one of the end
n
= Σ (r – 1) (r2 + r + 1) (w3 = 1, w2 + w = 1) stones, picks it up, goes to mid–stone, drops it and
goes to last stone on the other side, picks it and
r=1

ww
=
n
Σ (r3 – 1)
r=1
1
comes back to mid–stone.
In all the travels n intervals of 10 metres each 3 times.
Now, from centre he will go to 2nd stone on L.H.S.

32.
=

w 4
× n2 (n + 1)2 – n

.Ea
S n = Sum of cubes of the first n terms of an A.P.
= 13 + 2 3 + 3 3 + …… + n3
then comes back and again go to 2nd last on R.H.S.
and again come back.
Thus, he will travel (n – 1) intervals of 10 metres
each 4 times.
 n(n + 1)  2
S n = 
 2  syE Similarly, (n – 2) intervals of 10 metres each times
for 3rd and so on for the last.
Hence, the total distance covered as given = 3 km =

ngi
Let First term of an A.P. = a + d 3000 metre
Common difference = d 3 × 10n + 4 × [10 (n – 1) + 10 (n – 2) + …… + 10]
or 30 n + 40 × [1 + 2 + 3 + …… + (n – 1)] = 3000

nee
S 1 n = (a + d) + (a + 2d) + (a + 3d) + …… + (a + nd)
n or 2n2 + n – 300 = 0
= × [2a + (n – 1)d]
2 or (n – 12) (2n + 25) = 0
S n = (a + d)3 + (a + 2d)3 + …… + (a + nd)3
= na3 + 3a2 d Σn + 3ad2 Σn2 + d3 Σn3
or
rin n = 12
Hence, the number of stones = 2n + 1

= na3 + 3a2 d ×
n(n + 1)
2
+ 3ad2
35. L.C.M. of 2 and 5 is 10 g.n = 25

n
×
n(n + 1) (2n + 1)
6
 n(n + 1) 
+ d3 × 
 2 
2

are also divisible by 10.


Similarly,
e
Numbers divisible by 2 will contain numbers which

t
= × [4a3 + 6a2 d(n + 1) + 2ad2 (n + 1) (2n + 1) Numbers divisible by 5 will contain numbers which
4
are also divisible by 10.
+ d3 n(n + 1)2 ]
Thus, the number divisible by 10 will occur twice.
1 n
= × × [2a + (n + 1)d] [2a2 + 2ad(n + 1) So, Hence we can write to
2 2 S = S 2 + S5 – S 10
+ d2 n(n + 1)]
S 2 = Number divisible by 2 between 1 to 100
S1
= × [2a2 + 2ad(n + 1) + d2 n(n + 1)] = 2 + 4 + 6 + 8 + …… + 100
2
= 2 (1 + 2 + 3 + 4 + …… + 50)
33. Let the work finish in n days when the workers 50 × 51
started dropping, so that the total number of workers = 2× = 2550
2
who worked all these days is the sum of A.P. S 5 = Numbers divisible by 5 between 1 to 100
= 150 + 146 + 142 + …… + n terms = 5 + 10 + 15 + 20 + …… + 100
n
= × [2 × 150 + (n – 1) × (– 4)] = 5 (1 + 2 + 3 + 4 + …… + 20)
2 20 × 21
= n(152 – 2n) = 5× = 1050
2

CAT Complete Course | 261

Visit : www.EasyEngineering.net
Visit : www.EasyEngineering.net

S 10 = Numbers divisible by 10 between 1 to 100 43


= × [2 × 203 + (43 – 1) × 7]
= 10 + 20 + 30 + …… + 100 2
= 10 (1 + 2 + 3 + …… + 10) 43
= × [406 + 42 × 7]
2
10 × 11
= 10 × = 550 43
2 = × [406 + 294]
2
Now, S = 2550 + 1050 – 550
43
= 3050 = × 700 = 43 × 350
2
36. Odd integers between 2 and 100 are 3, 5, 7, 9, 11, 13,
…… 97, 99 = 15050
So, those odd numbers which are divisible by 3 are 3, Numbers divisible by 3 are 201, 204, 207, …… 498
9, 15, 21, …… 99 First term = 201
Now, First term = 3 Common difference = 3
2nd term = 9 nth term = a + (n – 1) × d
Common difference = 6 498 = 201 + (n – 1) × 3
If n term will be 99.

ww
So, tn = 3 + (n – 1) × 6 = 5n – 3
99 = 6n – 3
or
or
297 + 3 = 3n
n = 100
100

Now,
w 102 = 6n

.Ea
n = 17
n
Sum = × [2a + (n – 1) × d]
S3 =

=
2
100
2
× [2 × 201 + 99 × 3]

× [402 + 297]

=
2
17
2 syE
× [2 × 3 + (17 – 1) × 6]
= 50 × 699 = 34950
Numbers divisible 7 and 3 are 210, 231, 252, … 483
483 = 210 + (n – 1) × 21
= 17 [3 + 16 × 3]
= 17 × 51 ngi or 23 = 10 + n – 1

nee
or n = 14
= 867
n
37. L.C.M. of 7 and 3 is 21. S 21 = × [2 × 210 + (n – 1) × 21]
2

rin
Between 200 and 500 numbers divisible by 7 are
14
203, 217, …… 497 S 21 = × [2 × 210 + 13 × 21]
2
If nth term of series is 497.
tn = 203 + (n – 1) × 7
g.n
= 7 × (420 + 273)
= 7 × 693 = 4851

or
497 = 203 + 7n – 7
301 = 7n
n = 43
S 7 = Sum of numbers divisible
e
Now, numbers divisible by 7 or 3.
= S 7 + S3 – S 21
= 15050 + 34950 – 4851
t
by 7 between 200 and 500 = 45149
●●●

262 | CAT Complete Course

Visit : www.EasyEngineering.net
Visit : www.EasyEngineering.net

ONE DAY CAPSULE OF NUMERICAL APTITUDE

1. Average Example 1. In a class, the age of four students are 18


years,20 years , 22 years ,and 24 years , then what is the
The numerical result obtained by dividing the sum of
average age of the students in the class ?
two or more quantities by the number of quantities is
called Average. Solution : B y the above definition, average age
20 + 22 + 18 + 24 84
An arithmetic mean of given observations is called = = = 21years.
4 4
Average.
Therefore, average age of students = 21 years.
Average is defined in so many ways. We can say
average means Usual or Normal Kind, amount, quality, Weighted Average

ww
rate, etc. Hence, normal or ordinary capability of student
makes him an average student. An above the Average, is
called intelligent.
The concept of weighted Average is used when we
have two or more groups whose individual averages are
known.

w
Average is a number or value of a set of values

.Ea
carefully defined to typify the set, as a median or mode.
Average refers to the result obtained by dividing a
sum by the number of quantities added. For example the
Suppose in a class, there are 2 students each of 20
years, 3 of 21 years, 4 of 22 years and 5 of 23 years, then
their average age is given by :
(2 × 20) + (3 × 21) + (4 × 22) + (5 + 23)
average of 7, 9, 17 is
3 syE
7 + 9 + 17 33
= = 11 and in extended
3
use is applied to the usual or ordinary kind, instance, etc. =
2
2+3+4+5
3 4
× 20 + × 21 + × 22 + × 23
5

Average is different from mean and median.


ngi =
14
306
14
years.
14 14 14

nee
The Average of a given set of numbers is a measure
of the central tendency of the set . In other words, it is the 2 3 4 5
Here, , , and are called the weights of
mean value of a set of numbers or values. Therefore, 14 14 14 14
average of a set of numbers is given by :

Average =
(x1 + x2 + x3 + …… + xn )
each category of students.

rin
Example 2. What is the average concentration of a
n
Or in other words average of some observations :
24% sulphuric acid solution ?
g.n
mixture if 3 L of 36 % sulphuric acid is added to 9 L of

e
Solution : The average concentration of the com-
Sum of all observations
Average =
Number of observations
Mean commonly designates a figure intermediate
between two extremes; for example the mean temperature
for a day with a high of 34°C and a low of 18°C is
bined mixture is the weighted average

= ( )
3
12
× 36 +

= 9 + 18 = 27%
9
12 ()
× 24 t
34 + 18 In other words, weights are the fraction of the number
= 26°C and the median is the middle number or in that category with respect to the total students in that
2
m point in a series arranged in order of size i.e., the class. This average is also called the weighted average of
median grade in the group 50, 55, 85, 88, 92 is 85; the that class.
average is 74. Average Speed
Norm implies a standard of average performance for a If a (body) certain distance is covered in parts at
given group i.e., a child below the norm for his age in different speeds, the average speed is given by :
reading comprehension.
Total distance covered
(x + x2 + x3 + …… + xn ) Average speed =
Average = 1 Total time taken
n As, if a body travels d1, d2 , d3 …, dn distances, with
Or in other words average of some observations : speeds s1 , s2 , s3 …, sn in time t1, t2 , t3 …, tn respectively,
Sum of all observations then the average speed of the body through the total
=
Number of observations distance is given by :

CAT Complete Course | 263

Visit : www.EasyEngineering.net
Visit : www.EasyEngineering.net

Total distance covered This weight of the group is reduced because the man
Average speed =
Total time taken weighing 80 kg is replaced by a man who is 18 kg lighter
Always remember that, Average speed than him. Therefore, weight of new man = (80 – 18)
Sum of speeds s + s2 + s3 + … + sn = 62 kg.
≠ ≠ 1
Number of different speeds n Runs and Average
d + d2 + … + dn
Average speed = 1 Example 6. A cricketer has a certain average of 9
t1 + t2 + tn
innings. In the tenth inning he scores 100 runs, thereby
s1 t1 + s2 t2 + … sn tn increasing his average by 8 runs. Calculate his new
=
t1 + t2 + t3 + t4 + tn average.
d1 + d2 + d3 + … + dn Solution : Let the average of 9 innings be x runs,
=
d1 d2 d3 d hence new average will be (x + 8) runs.
+ + +…+ n
s1 s2 s3 sn Total runs scored for 9 innings = 9x
Short Cut Total runs scored for 10 innings = (9x +100)
If you travel equal distances with speeds u and v, Total runs
Average for 10 innings =

ww
then the average speed over the entire journey is
2uv
(u + v)
·

If a man changes his speed in the ratio m : n, then the


⇒ (x + 8) =
10
(9x + 100)
10

w
ratio of times taken becomes n : m .

.Ea
Example 3. Let the distance between two points A
and B be d and speed in travelling from point A to B be x
⇒ x = 20
Therefore, new average = ( 20 + 8) = 28 runs.
Average of Some Important Series of Num-
Solution :
syE
km/hr and from point B to A be y km/hr.

Total distance
bers
(a) The average of odd numbers from 1 to n is
Then, average speed =
Total time
2d ngi (n + 1) ,
2
when n = last odd number.

(b) The average of even numbers from 2 to n is

nee
=
d d
+ (n + 2) ,
x y when n = last even number.
2
If two speeds are given as x km/hr an y km/hr, then
Average speed (distance being same) n is
n (n + 1)(2n + 1)
·rin
(c) The average of square of natural numbers till

g.n
6n
2xy (n +1)(2n + 1)
= ⇒
x+y 6
Example 4. If a person travels two equal distances at
10 km/hr and 30 km/hr .What is the average speed for the
entire journey ?
Solution :
is =
n2 (n + 1)2
4n
·

n (n + 1)2
e
(d) The average of cubes of natural numbers till n

t

2 × 30 × 10 600 4
Average speed = = = 15 km/hr.
30 + 10 40 (e) The average of first n consecutive even num-
bers is (n +1).
Age and Average
( f ) The average of first n consecutive odd num-
If the average age of n persons decreases by x years. bers is n.
Then, the sum of age of n persons decreases by (n × x) (g) The average of squares of first n consecutive
years. Also, if the average age of n persons increases by 2(n + 1)(2n + 1)
x years. Then, the sum of age of n persons increases by even numbers is ·
3
(n × x) years.
(h) The average of squares of consecutive even
Example 5. The average weight of 6 men decrease (n + 1)(n + 2)
by 3 kg when one of them weighing 80 kg is replaced by numbers till n is ·
3
a new man. Calculate the weight of the new man. (i) The average of squares of consecutive odd
Solution : Total weight reduced of 6 men = 6 × 3 = n (n + 2)
numbers till n is ·
18 kg. 3

264 | CAT Complete Course

Visit : www.EasyEngineering.net
Visit : www.EasyEngineering.net

Example 7. What is the average of odd numbers We are giving here under the number chart, which is
from 1 to 25 ? self—explanatory in its nature and use for the practical
25 + 1 application.
Solution : Average = = 13
2 I. Complex Number : Complex number is also
Example 8. What is the average of even numbers referred as imaginary. The form in which complex
from 1 to 40 ? number is written as a + i b , where a and b are real
40 + 2 number and i is the imaginary unit whose value is  √– 1.
Solution : Average = = 21
2 In real number system the square root of negative number
Example 9. What is the average of square of natural does not exist.
numbers from 1 to 20 ? II. Real Number : Set of all numbers that can be
(20 + 1)(40 + 1) represented on the number line is called real numbers.
Solution : Average = = 143·5 9 ,
6 For example : 4, – 8, 0, 3·92, 2 + √ 11, 11 etc .
Example 10. What is the average of cubes of natural
A number line is a straight line with an arbitrary defined
numbers from 1 to 5 ?
point zero. To the right of this point lie all positive
5(5 + 1)2 numbers and to the left, all negative numbers.
Solution : Average = = 45

ww 4
Example 11. What is the average of first 49
consecutive even numbers ?
Real Number line : Now, real numbers can be
divided into two categories, rational numbers and irra-
tional number .

w
Solution : Average = 49 + 1

.Ea
Example 12. What is the average of first 19 con-
secutive odd numbers ?
(a) Rational Number : If a number can be
p
expressed in the form of where q ≠ 0 and where p and q
q
are integers, then the number is called rational number
Solution : Average = 19

syE
Example 13. What is the average of square of first
10 consecutive even numbers ?
9 16 8 27
e.g., , , , – etc.
25 7 1 51
All integers are also rational numbers. Every termi-
Solution : Average =
2(10 + 1)(20 + 1)
3
ngi nating decimal or a repeating decimal is also a rational
number, e. g. 3·1, 7·323232…., etc.
=
2 × 11 × 21
3
= 154

Example 14. What is the average of square of nee (b) Irrational Number : If a numbers cannot be
p
expressed in the form of , q ≠ 0, then the number is

rin
q
consecutive even numbers till 10? called irrational number.
(10 + 1)(10 + 2)

g.n
Solution : Average = In other words, non-repeating as well as non-
3 terminating type of decimals are called irrational numbers.
11 × 12
e.g., 
√
= = 44 2‚ 3 √
 4,… 4·965896…, 3·14592 …
3
Example 15. What is the average of square of
consecutive odd numbers till 12?
12(12 + 2)
e
Rational numbers can be further sub-divided into
two parts-integers and fractions.
t
(i) Integers : Integers are the set of all non–frac-
Solution : Average = tional numbers lying between – ∞ and + ∞. Hence,
3
integers include negative as well as positive non–frac-
12 × 14
= = 56 tional numbers. Integer is denoted by Z or I.
3
I = {– ∞ …, – 4, – 3, – 2, – 1, 0, 1, 2, 3, 4,…, + ∞}.
2. Number System (Note that 0 is neither a positive nor a negative
integer)
Number system is the key concept in every branch of
mathematics. The use and scope of number system is (ii) Fractions : A fraction includes two parts,
3 9 11
unlimited. The system deals with the nomenclature, use numerator and denominator – , , etc.
7 5 7
and properties of number. The chapter is a brief intro-
duction of number and its application in different Integers can be further subdivided into negative
competitive questions. Its scope in this book is limited number and whole numbers have two sections zero and
keeping in view our domain of competitive examination. positive numbers popularly called as natural number.
The number system that we use in over every day life is Natural Numbers : Set of natural numbers is
called decimal system. This is because there are 10 digits denoted by N
(0, 1, 2,…, 9,) . N = {1, 2, 3, 4, 5,…, ∞}

CAT Complete Course | 265

Visit : www.EasyEngineering.net
Visit : www.EasyEngineering.net

1. Even numbers : All numbers that are divisible (366 × 641 × 753) = (9 × 6 × 7) = 378.
by 2 are called even numbers e.g. {2, 4, 6, 8, 10, 12,…, ∴ Unit digit in the given product = 8.
∞}. (b) Representation of rational numbers : Rational
2. Odd numbers : All number that are not divisible numbers when converted into decimal form can be either
by 2 are called odd numbers e.g. {1, 3, 5, 7, 9, 11,…, ∞}. a recurring and non-terminating or a terminating decimal.
3. Prime numbers : The numbers that have only two For example, terminating decimal = 2·6, non-terminating
factors, 1 and the number itself, are called prime num- and recurring decimal = 2.636363…
bers e.g., {2, 3, 5, 7, 11, 13, 17, 19,…} (c) Remainder theorem :
Note— (I) Dividend = (Divisor × Quotient) + Remainder
● Number 1 is not a prime number . If a number when divided by 5 leaves a remainder 3,
● There are 25 prime numbers from 1 to 100 i.e., 2, 3, the number can be written as 5x+3, where x is a whole
5, 7, 11, 13, 17, 19, 23, 29, …, 97. number . This is also written as number or N = 3 mod 5 .
It means the number when divided by 5 leaves the
SOME IMPORTANT POINTS ABOUT PRIME remainder 3 .
NUMBERS
Example 1. There are two positive numbers x, y.


ww
The smallest prime number is 2, which is the only
even prime number .
All prime numbers can be written in the form of
Each of them when divided by 6 leaves the remainders 2
and 3 respectively. Find the remainder when (x + y) is
divided by 6.


w
(6N – 1) or (6N + 1) .The converse is not necessarily

.Ea
true . This means any number of the form (6N – 1) or
(6N + 1) is not necessarily a prime number .
The remainder when a prime number P ≥ 5 is divided
Solution : In case of number x,
x = 6a + 2
In case of number y,
…(i)

syE
by 6 is 1 or however, if a number on being divided
by 6 gives a remainder of 1 or 5 . The number need
not be prime.
y = 6b + 3
Adding equation (i) and (ii)
We get is x + y = (6a + 6b) + 5
…(ii)

● The remainder when the square of prime number


P≥ 5 divided by 24 is 1. ngi ⇒ x + y = 6(a + b ) + 5
So, it is clear that on dividing numbers (x + y) by 6 it


For prime number > 3 , P2 - 1 is divisible by 24.
The reminder of the division of the square of a prime nee
leaves a reminder 5.
Example 2. A number when divided by 256 give a

rin
P≥ 5 divided by 12 is 1. reminder 77. when the same number is divided by 16,
Operation of Numbers what would be the remainder ?

(a) Finding a unit digit in a product :


Example 1. Find the unit digit in the product (289 ×
256, leaves the remainder 77.
g.n
Solution : We know that if a number N is divided by

Then, N = 256x + 77 where x is the quotient


156 × 439 × 151).
Solution : Product of unit’s digits in given numbers
= (9 × 6 × 9 × 1)
= 486
e
= (16 × 16x + 16 × 4 + 13)
N = 16 (16x + 4) + 13
Therefore, number N leaves remainder 13, when it ist
divided by number 16.
∴ Unit digit in the given product is 6. Example 3 : If N ≡ 3 mod 6, what is the remainder
Example 2. Find the unit digit in the product (366 × when N2 is divided by 6 ?
6 × 753).
41
Solution : N ≡ 3 mod 6, means that number divided
Solution : We know, unit digit in 34 is 1. by 6 leaves a remainder 3.
∴ unit digit in 364 is1 In other words, N = 6x + 3
Hence, unit digit in 366 = 1 × 3 × 3 = 9 N2 = (6x + 3)2 = ( 6x)2 + 2 (6x) (3) + (3)
Unit digit in every power of 6 is 6. = 36x2 + 36x + 9 = 36x2 + 36x + (6 + 3)
Therefore, unit digit in 6 41 = 6 = 6 (6x2 + 6x +1 ) + 3
Unit digit, in 74 is 1 = 6M + 3 (where M is a natural number )
∴ Unit digit in 752 is1 0×P×q
Hence, Unit digit in 753 = 1 × 7 = 7 (II) The expression [ n ] will give the same
0r × Pr × qr
Therefore, product of unit’s digit in the given
numbers
remainders as [ n ]
where 0 r, Pr and qr are the

266 | CAT Complete Course

Visit : www.EasyEngineering.net
Visit : www.EasyEngineering.net

remainders when O, P and q are respectively divided by Some Important Formulae


n. ● (a + b)2 = a2 + b2 + 2ab
Example 4. Find the remainder for [(85 × 73 × 69)]/ ● (a – b) 2 = a2 + b2 –2ab
12.
● (a + b)2 – (a – b) 2 = 4ab
Solution : According to the theorem , the remainder
for the expression [(85 × 73 × 69]/12 will be the same as ● (a + b)2 + (a – b)2 = 2 (a2 + b2 )
9 ● (a2 – b2 ) = (a + b) (a – b)
the remainder for [(1 × 1 × 9)]/12 = ⇒ remainder = 9.
12 ● (a3 + b3 ) = (a – b) (a2 – ab + b2 )
Example 5. What is the remainder if 825 is divided ● (a3 – b3 ) = (a – b) (a2 + ab + b2)
by 7 ?
● a2 + b2 + c2 – ab – bc – ca
Solution : 1 1 1
Method I. 825 can be written as ( 7+ 1)25 . In this = (a – b)2 + (b – c)2 + (c – a)2
2 2 2
binomial expansion there are total 26 terms. All the first ● a4 + a2 b2 + b4 = (a2 + ab + b2) (a2 – ab + b2 )
25 terms will have 6 as a multiple in it .
● a3 + b3 + c 3 – 3abc = (a + b + c) (a2 + b2 + c 2 – ac –
The 26th term is (1)25 = 1. Hence, the expansion can bc – ab)

ww
be written as 7x + 1(where 7x is the sum of all first 25
terms). It is now clear (7x + 1), if divided by 6, leaves a
reminder 1 . ●
If a + b + c = 0 then a3 + b3 + c3 = 3abc
(a + b + c)3 – a3 – b3 – c3 = 3 (a + b) (b + c) (c + a)

w
Method II. Using the remainder theorem we find
that reminder for
825
7
.Ea
is the same as the reminder for

i.e., reminder 1. Therefore, remainder for


825
(1) 25
7
will be 1.
,
● a4 + b4 + c4 – 2a2 b2 – 2b2 c2 – 2c2a2
= – (a + b + c) (a + b – c) (b + c – a) (c + a – b)

3. Partnership

Factorisation syE 7
Partnership : When two or more than two persons
run a business jointly, they are called partners and the deal

ngi
It is the process of splitting any number into the form is known as partnerships. It can be simple or compound
of its basic prime factors. type.
For example, 24 = 2 × 2 × 2 × 3 = 23 × 3 Simple Partnership : When investments of all the
24 is expressed in the factorised from in terms of its
basic prime factors. This is the factorisation form of 24. nee
partners are for the same time, the profit or loss is divided
among them in the ratio of their investments.

rin
Compound Partnership : When investments are for
Example 6. Calculate the total number of factors of
different times, then equivalent capitals are calculated for
12.
a unit of time by multiplying the capital with the number
Solution : 12 can be expressed as 2 n × 3
Here, the powers of 2 can be one of (0, 1, 2) and that
of units of time.
g.n
1. If A and B are two partners and A invest MA
of 3 can be one of (0, 1) . So, the total possibilities, if you
take the two combinations are 3 × 2 = 6.
In general, for any composite number C, which can
be expressed as C = am × bn × c p × … where a, b, c, …
e
money for TA time and B invest MB money for TB time ,

t
the ratio of share of profit of A and share of profit of B is
T A × MA : TB × MB
TAMA
are all prime factors and m, n, p, are positive integers, the Share of Profit of A =
TAMA + TB MB
number of factors is equal to (m + 1) (n + 1) (p + 1)…
TBMB
Example 7. Find the total number of factors of 576 . Share of Profit of B =
TAMA + TB MB
Solution : The factorised form of 576 is 2. If A, B and C are three partners and A invests MA
(24 × 24) = (23 × 3) (23 × 3) = (26 × 32) money for T A time, B invests MB money for T B time and
Therefore , the total number of factors is C invests MC money for TC time , the ratio of share of
(6+ 1) (2+1) = 21. profit of A, share of profit of B and share of profit of C is
T A × MA : TB × MB : TC × MC
Important Result TAMA
Share of Profit of A =
A number having odd factors is always a perfect TAMA + TB MB + TC MC
square. TBMB
e.g. , 64 = 26 = (6 + 1) = 7 Share of Profit of B =
TAMA + TB MB + TC MC
144 = 24 × 32 = (4 + 1)(2 + 1) = 15 TCMC
Share of Profit of C =
36 = 22 × 32 = (2 + 1)(2 + 1) = 9 TAMA + TB MB + TC MC

CAT Complete Course | 267

Visit : www.EasyEngineering.net
Visit : www.EasyEngineering.net

A partner who participates in the working and Divisibility by 6 : A number is divisible by 6, if the
manages the business, is called a working partner while number is divisible by both 2 and 3 simultaneously.
the one only invests capital but does not participate in the Note—
working of the business, is called a sleeping partner.
● A number is divisible by ab only when it is divisible
A working partner gets either monthly payment or a by co-prime of that number .
part in the profit for his contribution in the management
● CO–Prime : Two number s are said to be co-prime,
of the business.
if their HCF is 1
This payment is deduced from the total profit before
its distribution. e.g., (2, 3) (8, 11) , (7, 9) (2, 7) etc .
Example 5. What least number should be added to
4. Divisibility 345670 in order to make it divisible by 6 ?
Rules of Divisibility Solution : The number will be divisible by 6, if it is
divisible by 2 and 3 both .
Divisibility by 2 : A number is divisible by 2, if its
last digit ( unit’s place ) is either 0, 2, 4, 6, or 8 e.g., each To make it divisible by 3 the sum of digits should be
of the numbers 21674 , 31856, 20018 , 43560 is divisible divisible by 3 .The sum of digits is 25 . If two is added to

ww
by 2. We note that all even numbers are divisible by 2.
Divisibility by 3 : A number is divisible by 3, if the
sum of its digits is divisible by 3 .
the number it becomes divisible 3 and satisfies the
condition of divisibility by 2 as well .
Example 6 : If N is divisible by 2 but not 3, what is

w
Example 1. Which of the following numbers is
divisible by 3 ?
(i) 98730527
.Ea
(ii) 17096528 (iii) 93476388.
the remainder when N is divided by 6 ?
Solution : Since, the number is divisible by 2, it has
to be an even number .

syE
Solution : Sum of digit in (i) is 41, since 41 is not N = 6a + x
divisible by 3, hence the given number is not divisible by x (remainder) will be even and can take the value
3. either 2 or 4 .
Sum of digit in (ii) is 38 , since 38 is not divisible by
3, hence the number (ii) is not divisible by 3.
ngi Divisibility by 8 : A number is divisible by 8 , if the
last 3 digits taken together, is divisible by 8 .

nee
Sum of digit in (iii) is 48, since 48 is divisible by 3 , Divisibility by 9 : A number is divisible by 9, if the
hence number (iii) is divisible by 3. sum of the digits of given number is divisible by 9.
Divisibility by 4 : A number is divisible by 4, if the Divisibility by 11 : A number is divisible by 11, if
number formed by the last two digits is divisible by 4.
Example 2. Which of the following numbers is rin
the difference of the sum of its digits at odd places and the
sum of its digits at even places, is either 0 or a number
divisible by 4?
(i) 6897956 (ii) 6893573
divisible by 11.
g.n
Example 7. Is number 96858256 divisible by 11 ?
Solution : It is clear that number formed by the last
two digits in (i) is 56 , which is divisible by 4, hence the
entire number is divisible by 4. In (ii) the last two digits is
73, which is not divisible by 4 hence the number is not
e
Solution : The divisibility rule of 11
= (sum of digits at even place )
t
– ( sum of digits at odd places )
divisible by 4.
= (9 + 8 + 8 + 5) – (6 + 5 + 2 + 6)
Example 3. If abc3d is divisible by 4, find the value
of d. = (30) – (19) = 11
Solution : For the number to be divisible by 4, the ∴ 96858256 is divisible by 11.
number formed by the last two digit i.e., 3d should be
divisible by 4. Hence, d should be replaced by 2 or 6.
5. Area
Divisibility by 5 : A number is divisible by 5, if the 1. Area
last digit is either 0 or 5. For example, 2635, 12970, 1. Rectangle = bh
38525, …..
Area = 4·7 = 28
Example 4. What is the remainder if a number 3ab9
is divisible by 5 ? 7
Solution : For the number 3 ab 9 to be divisible by 4
5, the last digit should be either 0 or 5. Now, since the
last digit 9 exceeds 5. Therefore, (9 – 5) = 4 would be the 2. Parallelogram = bh
remainder . Area = 8·4 = 32

268 | CAT Complete Course

Visit : www.EasyEngineering.net
Visit : www.EasyEngineering.net

m ∠A + m ∠B + m ∠C + m ∠D + m ∠E = (5 – 2)
5
4 180 = 540

8
1
3. Rhombus = d d
2 1 2
1
Area = 2 . 8·10 = 40

2. In a parallelogram :
(a) Opposite sides are parallel.
(b) Opposite sides are congruent.
(c) Opposite angles are congruent.
4. Square = side × side (d) Consecutive angles are supplementary.
Area = 4 × 4 = 16 (e) Diagonals bisect each other.

ww 4
4 (f) Each diagonal bisects the parallelogram into two
congruent triangles.

w
3. In a rectangle, in addition to the properties listed
in (2) above;

1 .Ea (a) All angle are right angles.


(b) Diagonals are congruent.
5. Triangle =

1
2
bh

Area = 2 (6)(22) = 66 syE 4. In a rhombus, in addition to the properties listed


in (2), above;
(a) All sides are congruent.

ngi (b) Diagonals are perpendicular.


(c) Diagonals bisect the angles.

nee 5. A square has all of the properties listed in (2) (3)


and (4) above.

6. Equilateral Triangle =
 3 × a2

4
rin
6. The apothem of a regular polygon is perpendicular
to a side, bisect that side, and also bisect a central angle.

Area =
√ 3 × 62 = 9√
4
3
OX is an apothem.
g.n
It bisects AB, is perpendicular to AB,

6
and bisect angle AOB

e t
1
7. Trapezoid = h (b1 + b2 )
2
1 7. The area of a regular polygon is equal to one-half
Area = × 5 × (6 + 10) = 40 the product of its apothem and perimeter.
2
6 1
A = (3) (6 × 5) = 45
5
2

10

Polygons
1. The sum of the measures of the angles of a
polygon of n sides is (n – 2) × 18°.
Since, ABCDE has 5 sides.

CAT Complete Course | 269

Visit : www.EasyEngineering.net
Visit : www.EasyEngineering.net

1 1 1 1 1 5
6. Simplification Note—2 + 3 + 4 + 23 + 68 + 22 + 34 = ?
3 6 2 2 3 6
Fractions (2 + 3 + 4 + 23 + 68 + 22 + 34 = 156)
A fraction is a number which represents a ratio or
division of two whole number (0, 1, 2, 3, 4……)
+ ( 1 1 1 1 1 1 5
+ + + + + + =3
3 6 2 2 3 3 6 ) = 159
(integers). A fraction is written in the form of () p
q Operation with Fractions
p( numerator) Adding and Subtracting : For adding and sub-
·
q (denominator) tracting, the number must have the same (common)
6 denominator.
For example, is a fraction, it represents taking 6 of
7 a c ad cb ad + cb
7 equal parts, or dividing 6 into 7. For example, + = + =
b d bd bd bd
A fraction with 1 as the denominator is the same as Multiplying : To multiply fractions, always try to
the whole number which its numerator. divide common factors from any numerators and any
0 23 denominators where possible before actually multiplying.
For Example, = 0 or = 23

ww
1 1 In multiplying mixed numbers, always rename them as
Note—q cannot be equal to zero. improper fraction first.
Fraction are primarily of five types— 2 21 88
For example, Multiply – × × = 11 or Multiply

of
p,
q
w
1. Proper Fraction : A rational number in the form

.Ea
where q ≠ 0, where the numerator is less than
3
1
2
2 6
– 4 × 3 × = 11
3 9
7 16 3

In word problems, of, usually indicates multiplica-


the denominator, e.g., ·
7
syE
2. Improper Fraction : A rational number in the
p
tion.
For example, Dr. Dim, director of Institute of Perfec-
1
form of , where q ≠ 0, where the numerator is more than
q
7
the denominator, e.g., · ngi tion donates of $ 690.
3
1 690 690
It means ×

nee
3 = = $ 230 is the amount Dr.
3 1 3
3. Mixed Fraction : Mixed fraction consist of inte- Dim donates.
3 3 17

rin
gral as well as the fractional part, e.g., 2 = 2 + = Note—Multiply the numerator and denominator of a
7 7 7 fraction by the same non-zero number, the fraction remains
It means that a mixed fraction is always an improper the same.
fraction.
4. Compound Fraction : A fraction of a fraction is
g.n
Dividing : To divide fractions or mixed numbers,
remember to multiply by the reciprocal of the divisor (the

e
4 9 4 9
known as compound fraction, e.g., of = × · number after the division sign).
5 11 5 11
5. Complex Fraction : Any complicated combina-
tion of the other type of fractions, e.g.,
1 3 4 3
1 2 22 3
For example, 7 ÷ =
3 3 3 2
× = 11
t
To divide one fraction (dividend) by another fraction
2 of ; of (divisor), invert divisor and multiply.
3 2 7 2
1+ 3+ 5 3 5 4 5
3 1 For example, ÷ = × =
2+ 8 4 8 3 6
1+3
Note—Dividing the numerator and denominator of a
Mixed Number
fraction by the same non-zero number the fraction remains
A mixed number consists of a whole number and a the same.
fraction.
Reducing a fraction to lowest term: A fraction has
3 3
For example, 11 is a mixed number. It means 11 + been reduced to lowest terms when the numerator and
4 4
denominator have no common factors.
Any mixed number can be changed into a fraction.
4 6
3 For example, is reduced to lowest terms but is
For example, write 11 as a fraction 5 21
4 not because 3 is a common factor of 6 and 21. Hence, the
47 3×2 2
11 × 4 = 44 and 44 + 3 = 47 so, fraction is · lowest term of fraction is = ·
4 3×7 7

270 | CAT Complete Course

Visit : www.EasyEngineering.net
Visit : www.EasyEngineering.net

7. Ratio and Proportion Some Results on Ratio and Proportion


Ratio 1. Invertendo—If a : b : : c : d, then b : a :: d : c
a 2. Alternendo—If a : b :: c : d, then a : c :: b : d.
If a and b are two quantities of the same kind, then
b
3. Componendo—If a : b :: c : d, then (a + b) : b ::
is known as the ratio of a and b. Therefore, the ratio of
two quantities in the same units is a fraction that one (c+ d) : d.
quantity is to the other. 4. Dividendo—If a : b :: c : d, then (a – b) : b ::

()
Thus, a to b is a ratio
a
b
, written as a : b.
(c – d ) : d.
5. Componendo and Dividendo—If a : b :: c : d,
The first term of the ratio is called antecedent, while then (a + b) : (a – b) :: (c + d) : (c – d).
the second term is called consequent.
Direct Proportion
Ratio between 60 kg and 100 kg is 3 : 5.
The multiplication or division of each term of ratio If A is directly proportional to B then as A increases,
by a same non-zero number does not affect the ratio. B also increases proportionally. For example, the relation
Hence, 3 : 5 is the same as 6 : 10 or 9 : 15 or 12 : 20 etc. between speed, distance and time, speed is directly pro-

ww
Ratio can be expressed as percentages. To express
the value of a ratio as a Percentage, we multiply the ratio
portional to distance when time is kept constant.
It is therefore important to note here that the variation

w
by 100 .
3
Therefore, = 0·6 = 60%.

Proportion
5
.Ea
is direct and proportional. If one quantity is doubled the
related quantity will also be doubled.
Other examples of direct proportion are :

syE
(a) Simple Interest Vs Time (principal and rate being
The equality of two ratios is called proportion. a, b, constant).
c, d are said to be in proportion if a : b = c : d (b) Density Vs Mass (volume being constant).
or a : b :: c : d.
In a proportion , the first and fourth terms are known
as extremes, while second and third terms are known as ngi (c) Force Vs Acceleration (mass being constant).
Direct Variation
means. Hence, a and d are extremes and b and c are means
of the proportion a : b :: c : d. nee If A is said to vary directly as B , then as A increases
B also increases but not proportionally. This variation is
In a proportion we always have :
Product of extremes = Product of means
a×d = b×c rin
denoted by A ∝ B or A = KB, where K is a constant.
For example, the total cost of production is directly

Continued Proportion
g.n
related to the number of items being produced.
Here, the variation is direct but not proportional .
Four quantities : a , b , c , d , are said to be in a
continued proportion, if
a b c
a : b = b : c = c : d or = =
b c d
Inverse Proportion
e
A is inversely proportional to B means if A increases
B decreases proportionally. If speed is doubled, time
t
Three quantities are said to be in continued pro- taken to cover the same distance is reduced to half.
portion, if
a : b = b : c or, ac = b2 Other example of inverse proportion are :
In this relationship, b is said to be the mean pro- (a) Density Vs volume (mass being constant).
portional between a and c and c is said to be a third (b) Number of person Vs time taken to complete the
proportional to a and b. work (work being same).
Example 1. An object 1·6 m long casts a shadow
1·4 m long. At the same time another object kept nearby Inverse Variation
casts a shadow 6·2 m long. Find the length of the second If A is inversely related to (or) varies inversely as B,
object. then if B increases as A decreases but not proportionally .
Solution : Ratio of length of the object to its shadow 1
This relation can be expressed mathematically as A ∝
would be same. B
∴ 1·6 : 1·4 = x : 6·2 1
⇒ A = K × , where K is a constant.
B
1·6 × 6·2
or x = = 7·08 m
1·4 Here, the variation is inverse but not proportional.

CAT Complete Course | 271

Visit : www.EasyEngineering.net
Visit : www.EasyEngineering.net

8. Progression 99 – 12
= (n – 1)
3
A sequence of numbers, each of which is obtained n = 30
from its predecessor by the same rule.
∴ The required sum
Arithmetic Progression (AP)—A sequence of terms n
each of which, after the first, is derived by adding to the S n = (12 + 99) = 1665
2
preceding one , a common difference (Ex. 5, 9, 13, 17,
etc.) form an arithmetic progression. Important Result of an AP
And also, a number series which progresses in such a (i) Sum of n consecutive natural numbers
way that the difference between two consecutive numbers n (n + 1)
(1 + 2 + 3 + 4 + ……….. + n) =
is common, is called the arithmetic progression. 2
In the above Ex (series) the difference between two (ii) Sum of squares of n consecutive, natural num-
consecutive number is 4. bers
(a) The nth term of the AP is given by n (n + 1)(2n + 1)
(12 + 2 2 + 3 2 + …….. + n2) =
Tn = a + (n – 1) d 6
where a = First term of the series (iii) Sum of cubes of n consecutive natural numbers

ww n = Number of the terms in the series


d = Common difference of the series
(13 + 2 3 + 3 3 + 4 3 + …… + n3) =
n2 (n + 1)2

Geometric Progression (GP)—A sequence of terms


4

w
(b) The sum of n terms of the AP is given by :
n
2
.Ea n
S n = [2a + (n – 11d ] = (first term + last term)
2
in which the ratio of each term to the preceding one is the
same throughout the sequence (Ex. : 1, 2, 4, 8, 16, 32).
And also, a progression of numbers in which every

syE
Example 1. Find the sum of even numbers between term bears a constant ratio with its preceding term, is
1 and 100 (including 100). called geometrical progression.
Solution : The series of even numbers between 1 and The constant ratio is called the common ratio of the

ngi
100 will be 2, 4, 6, 8,……., 100. GP.
Since, the difference between two consecutive terms Therefore, a ar, ar2 is a GP
is common, it is arithmetic progression. where a = first term, r = common ratio
n
S n = [first name + last name]
2 nee The nth term of GP is given by Tn = ar n – 1
And sum of n terms of GP is given by :
There will be 50 even numbers between 1 and 100 .
n = 50 rin
Sn =
a (1 – rn )
(1 – r)

∴ Sn =
50
2
[2 + 100] = 25 × 102 = 2550. …, 1024?
Solution : a = 2, r = 2 g.n
Example 4. How many terms are there in 2, 4 8, 16,

Example 2. The sum of first natural number from


one to fifty, is divisible by
(a) 3
Solution :
(b) 51 (c) 5 (d) 25 (e) all
or, 2n – 1 =
e
Then, nth term is given by ar n – 1
1024 = 2 × 2n – 1
1024
= 512
t
2
n
Sum Sn = [first name + last name] ⇒ 2n × 2– 1 = 512
2
50 ⇒ 2n = 512 × 2 = 1024 = 210
= [1 + 50] = 25 × 51 = 1275
2 ∴ 2n = 210 or n = 10.
Now, 1275 is divisible by each one of 3, 51, 5 and 25.
9. Percentage
Hence option (e) is correct.
Example 3. Find the sum, if all 2 digit numbers are The word percentage means per 100 or for each
divisible by 3. hundred. A fraction whose denominator is 100 is called a
Solution : All 2 digit numbers divisible by 3 are : percentage and the numerator of the fraction is called the
rate per cent. It is denoted by the symbol %.
12, 15, 18, 21, 24,………….99.
19
The above series is an AP, where a = 12 and d = 3. 19% = = 0·19
100
To find the number of terms in the above series 12
Tn = a + (n – 1)d 12 7 12 1 3
% = = × = = 0·017
99 = 12 + (n – 1) 3 7 100 7 100 175

272 | CAT Complete Course

Visit : www.EasyEngineering.net
Visit : www.EasyEngineering.net

Questions based on Percentage Example : Find out what is new number after
decreasing 40 by 50%.
(1) To find the % equivalent of a fraction or as a
Solution :
decimal, add the % sign and multiply by 100.
4
Example : To rename a fraction as a %.
25
40 ×[ 100 ]
100 – 50 1
= 40 × = 20
2
(5) To find the % increase of a number, use formula
4 Total Increase
Solution : × 100% = 16% % increase = × 100
25 Initial Value
Example : To rename a decimal 0·074 as a %. Example : Over a five year period, the enrollment at
Solution : This has the effect of moving the decimal Institute of Perfection, Haridwar; increases from 800 to
point two places to the right. 1000. Find the per cent of increase.
Solution :
0·074 = 7·4%
1000 – 800
(2) To find the fraction or number or decimal % increase = × 100%
800
equivalent of a %, remove the % sign and divide by 100. 200 1
= × 100% = × 100% = 25%
Example : To rename a 25% as a fraction or decimal. 800 4

ww
Solution : 25% =
25 1
= = 0·25
100 4
(6) To find the % decrease of a number, use formula
% decrease =
Total Decrease
× 100

w
Initial Value
Most percentage problems can be solved by using the
following relation : Example : Over a five year period, the enrollment at
%
100
=
.Ea
Part
Whole
Institute of Perfection, Haridwar; decreases from 1000 to
800. Find the per cent of decrease.

Example : Find 27% of 92.

Solution :
27
=
x syE Solution :
% decrease =
1000 – 800
100
× 100%

ngi
100 92 200 1
= × 100% = × 100% = 20%
X = 24·84 1000 5
Example : 7 is 5 % of what number?

Solution :
5
100
=
7
x nee
Growth Rate
Growth is normally in absolute values whereas
growth rate is expressed in percentage terms. It could be
X = 140
Example : 90 is what % of 1500? rin
positive as well as negative. It is always with respect to
the previous value unless mentioned otherwise.

Solution :
x
100
=
90
1500
Growth % =
g.n
Change in Growth
Initial Value
× 100

X = 6
Example : Find 125% of 16.
125 x
Rule of Successive Changes
e
Let a% and b% are the first and second changes
t
respectively than the net change could be expressed as a
single percentage.
Solution : =
100 16
X = 20 (
Net Change = a + b +
ab
100 )%

(3) To increase a number by a given %, multiply the Note—You can use this formula, where the product
of two equal to one.
number by the factor [
100 + rate
100 ]
·
For example, Area = Length × Breadth, Revenue =
Price × Sale, Expenditure = Consumption × Rate of
Example : Find out what is new number after
Commodity etc.
increasing 22 by 200%.
Example 1. Institute of Perfection, Haridwar nor-
Solution : mally employs 100 people. During a slow spell, it fired
22 × [100100+ 200] = 22 × 3 = 66 20% if its employees. By what per cent must it now
increase its staff to return to full capacity ?
(4) To decrease a number by a given %, multiply the 1
Solution : 20% means of 100 = 20
5
number by the factor [
100 – rate
100 ] IOP now has 100 – 20 = 80 employees.

CAT Complete Course | 273

Visit : www.EasyEngineering.net
Visit : www.EasyEngineering.net

If it then increases by 20, the percent of increase is Solution :


20 1
80
× 100% = × 100% = 25%.
4 Percentage change in area = (a + b + 100
ab
)%
Example 2. The production of a company is increased 200
= 20 – 10 – = 8%
from 200 crores to 350 in a financial years 2005-06 and 100
decreased by 35 crore in the subsequent year. What is Example 6. The price of a Maruti car rises by 30% ,
percentage increase and decrease in production in the while the sales of the car goes down by 20% . What is the
respective years ? percentage change in the total revenue ?
Solution : Increase in production in 2005-2006 Solution : Revenue = Price × Sale
= (350-200) = 150 crores Hence, Percentage change in revenue
30 × 20
% increase in production
Increase 150
(
= 30 – 20 –
100 ) % = 4%
= × 100 = × 100
Base value 200 Therefore, revenue will increase by 4%.
= 75% Example 7. When the price of sugar was increased
Decrease in production in 2006-07 = 35 crores by 32%, a family reduced its consumption in such a way

ww
% decrease in production during 2006-07

=
Decrease
× 100
that the expenditure on sugar was only 10% more than
before. If 30 kg were consumed per month before, find
the new monthly consumption.

w
Base value Solution : Let the original price be Rs. x per kg.
35 We know that Price × Consumption = Expenses
=
350
.Ea
× 100 = 10%

Percentage increase / decrease


∴ Original expenses = x × 30 = Rs. 30x
After increases, let the new consumption be y kg.
=

[
syE]
Increase/decrease
Base value
× 100

Final value – Initial value


Given, new price = 1·32x
New expenses = 1·1 × 30x
Or, % =
Initial value
× 100

Example 3. If Vivek’s income is 20% more than that ngi ∴ 1·32x × y = 1·1 × 30x ⇒ y =
1·1 × 30x
1·32x
= 25 kg.

nee
Hence, new monthly consumption = 25 kg .
of Tony then what per cent is Tony’s income less than
Example 8. From a man’s salary, 10% is deducted
that of Vivek ?
on tax , 20% of the rest is spent on education and 25% of
Solution : Let the income of Tony be Rs. 100,
Income of Vivek = Rs. 120
rin
the rest is spent on food. After all these expenditures, he
is left with Rs. 2700. Find his salary .
In the question Tony’s income is being compared
with that of Vivek’s and hence base value to find the %
decrease will be the income of Vivek. g.n
Solution : Let the salary of the man be Rs. 100.
Then , after all the expenses he is left with :
100 × 0·9 × 0·8 × 0·75 = Rs. 54.
% Decrease =

=
Decrease
Base value
(120 – 100)
× 100

× 100 =
20
× 100 =
100
e
If he is left with Rs. 54, his salary = Rs. 100
If he is left with Rs. 2700, his salary would be
× 2700 = Rs. 5000.
t
120 120 54
50 2 Some Short Cuts
= % or 16 %
3 3
If, A’s income is r% more than that of B, then :
Example 4. If the price of petrol increases succes-
sively by 20 % and then by 10% , what is the net change
in percentage terms ?
B’s income is ( r
100 + r )
× 100 % less than that of A.

If A’s income is r% less than that of B, then :


Solution :

Net change in % = [20 + 10 + 100


200
]% = 32% B’s income is ( r
100 – r )
× 100 % more than that of
A.
It means that the successive increase of 20% and If, the price of a commodity increases by r%, then
10% are equal to a single increase of 32% . reduction in consumption, so as not to increase the
Example 5. The length of a rectangle is increased by expenditure :
20% and breath is decreased by 10%. Calculate the
percentage change in the area . ( r
100 + r
× 100 %)
274 | CAT Complete Course

Visit : www.EasyEngineering.net
Visit : www.EasyEngineering.net

If, the price of a commodity decreases by r%, then Marked Price (MP) : The price on the label is called
increase in consumption, the marked price or list price .

( r
100 – r )
× 100 % Discount : The reduction made on the ‘Marked Price’
of an article is called discount. When no discount is given,
Let, the per cent population of a town be P and let ‘Selling Price’ is same as ‘Marked Price’.
there be an increase of R% per annum, then Commission : Many sales people earn money on a
R n commission basis. In order to encourage sales, they are
(
(a) Population after n years = P 1 +
100 ) paid a percentage of the value of goods sold. This amount
is called commission.
P
(b) Population n years ago = List of important short cuts and formulae—
R n
( 1+
100 ) 1. Profit = SP – CP (if SP > CP otherwise it is loss)
Let, the present value of a machine be P and let it Profit SP – CP
2. % profit = × 100 = × 100
depreciate at R% per annum, then CP CP
R n % Profit × CP % Loss × CP
(a) Value of machine after n years = P 1 – (
100 ) 3. Profit =
100
or Loss
100

ww
(c) Value of machine n years ago =
(
1–
P
E n
100 )
4. SP =
100 + Profit %
100
100 – Loss %
× CP

w
Let, Maximum marks = M, Marks obtained = x %
and Pass marks = P %
.Ea
If a student obtains x % marks and fails by y marks,
5. SP =
100
× CP

6. SP = Marked price – Discount

7. CP = SP × ( 100
)
syE
100 + Profit %
y
M = × 100
P–x
If marks obtained is given as z and fails by y marks ,
or SP × ( 100
)
100 – Loss %

M =
z+y
P
× 100
ngi 8. % Discount =
Discount
Marked Price
× 100

A scores x % marks but fails by y marks. B scores z%


marks which is w marks more than pass marks . Then nee(
Some Short Cuts
In case of successive discount a% and b%, the
ab
)
M = ( )
w+y
z–x
× 100
rin
effective discount is a + b –
100
%.

If two items are sold, each at Rs. X, one at a profit of


In a class test x% student failed in English, y% failed
in Hindi. If z% failed in both subjects the percentage of
students who passed in both subjects is given by : (i) In percentage = g.n
p% and another at a loss of p%, there is an overall loss
p2
%
[100 – (x + y – z ]%

10. Profit and Loss


(ii) In value =
2p2
100

1002 – p2 e
X
t
Buy A and get B free, i.e., if A + B items are sold at
Definition of Some Very Important Words cost price of A items then the percentage discount is
B
Cost Price (CP) : The price for which an article is calculated by = × 100.
A+B
bought, is called its cost price.
If (i) CP of two items is the same and (ii) % loss and
Selling Price (SP) : The price at which an article is % profit on the two articles are equal then net loss or net
sold, is called its selling price. profit is zero.
Profit or Gain : The difference between the selling If someone (i) buys a table for Rs. l and (ii) sells b
price and cost price, is called the profit (If SP > CP). tables for Rs. m
Otherwise it is called the loss. (if CP > SP)
Profit and loss are generally represented as a per cent
Then net profit = ( )
m
b
a –1 …(i)
of the cost price, unless otherwise stated.
Overhead Charges : If an individual has to spend In percentage profit =
[ ]
ma
bl
–l
× 100%
some money on transportation etc., then this extra expen- l
diture, is called overhead charges, i.e., which is not
directly connected with production.
or = [ ma
bl ]
– 1 × 100%

CAT Complete Course | 275

Visit : www.EasyEngineering.net
Visit : www.EasyEngineering.net

N.B. If the result is negative ,it represents loss. Marked Price or LIST Price or PRINT Price
Visualization : Quantity price ( CP or SP) In a sale transaction , the seller marks the goods more
a 1 than the cost price in order to earn a profit . This addition
to the cost price is called the mark up price and this mark
b m up value added to cost price is called the marked price or
list price.
% Profit = (ambl – 1) × 100% Therefore, Print price = CP + Mark up price
Now, the seller may sell the product on the marked
Example 1. A person sells an article for a price which
price. In such case,
gives him a profit of 15% on cost price of Rs. 600. Cal-
culate the selling price of the article. Print price = Selling price
Solution : SP = CP + Profit He may also sell the product after allowing a discount
on the marked price. In such case
= CP + 15% of CP
Selling price = Print price – Discount
(
= CP 1 +
15
100 ) Example 3. A person marks his goods 30% more

ww =
CP × 115
100
than the cost price and allows some discount on it. He
still makes a profit of 10%. Find the discount percentage.
Solution : Let the CP of the article be Rs. 100.

w
115
= × 600 = 690
100 Then, MP of the article = Rs. 130

.Ea
∴ Selling price of the article = Rs. 690 .
Hence, we conclude that , if gain percentage is given
Since, profit = 10%, i.e., Rs. 10 (Profit is always on
CP hence profit is 10% of Rs. 100 = Rs. 10).

]sy
alongwith the CP, then ∴ SP = CP + Profit

[
E
Gain % = 100 + 10 = Rs. 110.
SP = 1 + × CP
100

ngi
The difference between marked price and selling
(100 + Gain %) × CP price is the discount.
=
100 ∴ Discount = MP – SP
In case of loss the following formula are applic-
able— nee Discount % =
= 130 – 110 = Rs. 20
Discount
× 100
Loss = CP – SP
Loss rin
=
Marked Price
20
× 100 =
200
= 15·38
Loss percentage =
CP
× 100

Like Profit, Loss percentage is also calculated on CP g.n


130 13
Note : Discount percentage is always calculated on
print price unless specified .
unless specified.
Example 2. A person buys an article for Rs. 600 and
sells the same at a loss of 20%. Find the selling price of
the article .
False Weight
e
Example 4. A dishonest dealer professes to sell his
goods at cost price but uses a weight of 960 g for a kg
t
weight. Find his gain per cent.
Solution : SP = CP – loss
Solution : Suppose the cost price of 1kg of goods is
= CP – 20% of CP
Rs. 100.
(
= CP 1 –
20
100 ) Therefore, cost price of 960 g of goods will be
Rs. 96.
4
= × 600 = 480 Since, he is selling 960 g (Using a false weight
5 instead of 1 kg ), the selling price of 1 kg would be, i.e.,
Therefore, the selling price of the article is Rs. 480. Rs. 100.
Hence, we conclude that if loss percentage is given Therefore, profit = SP – CP
alongwith the CP, then = (100 – 96) = Rs. 4
SP = 1 – [ Loss %
100 ] × CP Hence, Profit % =
Profit
CP
× 100

= [ ] × CP
100 – Loss % 4 1
= × 100 = 4 %
100 96 6

276 | CAT Complete Course

Visit : www.EasyEngineering.net
Visit : www.EasyEngineering.net

II Method 100
and SP of 1 poster = Rs. = Rs. 4
25
Gain % = [(True Value)
Error
– (Error)
× 100] Since, SP > CP hence this transaction will yield a
profit.
This formula can be used in questions of false weight.
∴ Profit = Rs. (5 – 4)
∴ Gain % =
40
(
960 )
× 100 % = 4 %
1
6 = Re. 1 and profit %
1
Example 5. By selling two articles for Rs. 180 each, = × 100 = 25%
4
a shopkeeper gains 20% on one and losses 20% on the
other, find the percentage profit / loss. Example 8. Toffees are bought at 12 for a rupee and
are sold 10 for a rupee. Find the profit or loss percentage.
Solution : SP = CP + 20% (profit)
Solution : CP of 12 toffees = SP of 10 toffees = Re. 1
and SP = CP – 20% (loss)
1
∴ 180 = 1·2 × CP ∴ CP of 1 toffee = Re.
12
and 180 = 0·8 × CP 1
and SP of 1 toffee = Re.
180 10
∴ CP = = Rs. 150

ww
and CP =
1·2
180
0·8
= Rs. 225
tion.
Since, SP > CP, hence there is a gain in this transac-

Gain = SP – CP

wTotal cost price = (150 + 225) = Rs. 375

Hence,
.Ea
Total selling price = (180 + 180) = Rs. 360
loss = CP – SP
= Re.

1
60
( 1 1
–)
10 12

× 100 = 20%
= Re.
1
60

syE
= (375 – 360) = Rs. 15 Gain % =
1
15 12
and Loss % = × 100 = 4%
375 Example 9. If a commission of 20% is given on

and one is sold at the loss of x % and another is sold at a


ngi
In case, where the selling price of two articles is same retail price, the profit is 60%. Find the profit percentage
when the commission is increased by 5% of the retail

nee
profit of x% . Or in other words, the profit % and loss % price.
is same and selling price is same. This transaction always Solution : Let the retail price be Rs. 100 .
has a loss and such loss % is
Then, SP = Rs. 80 (after a discount of 20% )
[
= a + (– a) +
(a) × (– a)
100 ] Since,
rin
Profit = 60%
SP 80
= ( Common loss or gain % 2

20 2
( )
10 ) ∴

New
CP =

g.n
1·6
SP = Rs. 75
=
1·6
= Rs. 50

In the above case loss % =


10
= 4%

Example 6. A Shopkeeper sells two items at the


same price. If he sells one of them at a profit of 10%. Find

e
(after a commission of 25%)
Profit = Rs. (75 – 50) = Rs. 25
Hence, Profit % =
25
50
× 100 = 50% t
the percentage profit / loss.
Solution : In such transaction, there will always be a 11. Mixture of Alligations
loss and Loss % This Topic deals with a specific type of questions
Common loss or grain % 2
= ( 10 ) % which can be solved quickly using the method of Alliga-
tions. Hence, it is very necessary to understand and
= 1% loss. identify the pattern of such questions so as to apply the
Example 7. By selling 20 posters, a person recovers rules of alligations.
the cost price of 25 posters. Find the gain or loss per- Alligation is the rule to find the proportion in which
centage. the two or more quantities at the given price must be
mixed to produce a mixture at a given price.
Solution : Given, SP of 20 poster = CP of 25 poster
Now, suppose SP of 20 poster = CP of 25 poster Rule of Alligation
= Rs. 100 If two quantities are mixed in a ratio, then
100 Quantity of cheaper CP of dearer – Mean price
∴ SP of 1 poster = Rs. = Rs. 5 =
20 Quantity of dearer Mean price– CP of cheaper

CAT Complete Course | 277

Visit : www.EasyEngineering.net
Visit : www.EasyEngineering.net

Some Short Cuts 2. Time is inversely proportional to speed. If the


distance remains the same and speed is doubled, then
(1) m gm of sugar solution has x % sugar in it. To time taken to travel the same distance becomes half of the
increase the sugar content in the solution of y % original time taken at the original speed.
m (y – x)
Quantity of sugar needs to be added = 3. Speed is directly proportional to distance. If the
100 – y speed is doubled, then distance travelled in the same time,
(2) The ingredients of a mixture in its pure form has will also be doubled.
percentage value 100% and fraction value 1.
Conversion of Units or Standard Conversion
(3) When X1 quantity of ingredient A of cost C1 and
Factors
X2 quantity of ingredient B of cost C 2 are mixed, cost of
the mixture Cm is given by In general : Time is measured in seconds, minutes
C X + C2 X 2 or hours.
Cm = 1 1 Distance is usually measured in metres, kilometres,
X1 + X 2
miles or feet .
(4) Similarly, when more than two …….than
C X + C2 X 2 + C3 X 3 + C4 X 4 + C5 X 5 + … (i) 1 hr = 60 minutes = 60 × 60 seconds
Cm = 1 1 (ii) 1 km = 1000 m or 1km = 0·6214 mile

ww
X1 + X 2 + X 3 + X 4 + X 5 + …
(5) When two mixtures M1 and M2 , each containing or 1 mile = 1·609 km
ingredient A and B in the ratio a : b and x : y, respectively i.e., 8 km ≅ 5 mile (aprox.)

w
are mixed, the proportion of the ingredients A and B, i.e.,
QA : QB in the compound mixture is given by—

QA
=
M1 ×
.Ea (( ))
( ) a+b
a
+ M2 ×
x+y
x
(iii)

or y m/s =
( )
x km/hr = x ×

18
5
5
8

y km/hr
m/sec

QB
M1 × ( )
syE
a+b
b
+ M2 ×
x+y
y

and, the quantity in which M1 and M2 is to be mixed when


(iv) ( )
x km/hr = x × x ×
5
8
mile /hr

ngi ( ) 22
the quantity of A and B, i.e., QA : QB in the compound (v) x mile/hr = x × ft/s
15
mixture is given by
Example 1. If a man walks at the rate of 5 km/hr, he
 QA 
Quantity of M1
Quantity of M2
=
( ) x
x+y
 QA 
–
 QA + QB

nee
misses a train by 7 minutes. However, if he walks at the
rate of 6 km/hr, he reaches the station 5 minutes before

( )
rin
a the arrival of the train. Find the distance covered by him
Q + Q  – a + b
 A B to reach the station.
Solution : Difference in the times taken at two speeds
If a container originally contain x units of liquid and
y unit of liquid is taken out. If this operation is repeated n
times. Then, final quantity of the liquid in the container g.n
= (7 + 5) = 12 min = hr
1
5
left is :

( )
x 1–
y n
x
units ∴
x x
– =
5 6
1
5
e
Now, let the required distance be x km.

or 6x – 5x = 6
t)
12. Time, Speed and Distance x = 6 km ( Q time =
distance
speed
The speed of a body is the defined as the distance Example 2. If a student walks from his house to
covered by it in unit time. Or the speed of a body is the school at 5 km/hr, he is late by 30 minutes. However, if
rate at which it is moving . he walks at 6 km/hr, he is late by 5 minutes only. Find the
(i) More distance, more time, at same speed. distance of his school from his house.
(ii) More speed, less time, for same distance. Solution : In both case the student is late.
(iii) More speed, more distance for same time. Therefore, difference in timings :
Distance
Speed = = (30 – 5) = 25 min
Time
or Distance = Speed × Time 25 5
= hr = hr
60 12
The expression shows that :
1. Distance and time are directly proportional. If Let the distance be x km.
distances to be travelled were doubled, then the time taken x x 5
∴ – =
would also be doubled at the same speed. 5 6 12

278 | CAT Complete Course

Visit : www.EasyEngineering.net
Visit : www.EasyEngineering.net

5 × 30 Therefore, if C takes 54 minute to cover a distance,


6x – 5x =
12 54
then time taken by B to cover the same distance = ×2
⇒ x = 12·5 km. 6
Example 3. A and B are two stations 350 km apart. = 18 min.
A train starts from A at 7 a.m. and travels towards B at 40 Average Speed
km/hr. Another train starts from B at 8 a.m. and travels
# If a body convered certain distance in parts at
towards A at 60 km/hr. At what time do they meet ?
different speeds, the average speed is given by :
Solution : Let both the trains meet x hours after
Total distance covered
7 a.m. Average speed =
Total time taken
Then,
# As, if a body travels d1 , d2 , d3 , …, dn distance, with
Distance moved by first train in x hours + Distance speed s1 in time t1 , t2 , t3 , …, tn respectively then the
moved by other train in (x – 1) hours = 350 km.
average speed of the body through the total distance
40x + 60 (x – 1) = 350 is given by :
100x = 350 + 60 Total distance travelled
x = 4·1 hours Average speed =
Total time taken

wwHence, both the trains will meet at 6 minute past 11.


Example 4. Mr. X arrives at his office 30 minutes
late everyday. On a particular day, he reduces his speed
#


Always remember that , Average speed
Sum of speeds
Number of different speeds
s + s2 + … + sn
≠ 1
n

w
by 25% and hence arrives 50 min late instead. Find how

.Ea
much time would he take to travel to his office if he
decides to be on time on a particular day ?
Solution :
# Average speed =

=
d1 + d2 + … + dn
t1 + t2 + … + tn
s1 t1 + s2 t2 + … + sn tn

New speed = S
3
4
4
syE = 1
t1 + t2 + t3 + t4 … + tn
d + d2 + d3 + … + dn
d1 d2 d3 d
∴ New time = T
4
3
T – T = 50 – 30 ngi Short cut
+ + +…+ n
s1 s2 s3 sn

nee
3
T (1) If you travel equal distance with speeds u and v,
or = 20
3 2uv
then the average speed over the entire journey is ·
T = 60 minutes.
It was the time taken by X when he was 30 min late.
rin (u + v)
(2) If a man changes his speed in the ratio m : n,
So, if he decides to come on time, he would take 30
minutes to travel.
Example 5. Excluding stoppages, the speed of bus is g.n
then the ratio of times taken becomes n : m.
Example 7. A person goes to Mansha Devi from Har
ki Pauri at the speed of 40 km/hr and comes back at the
54 km/hr and including stoppages, it is 45 km/hr. For how
many minutes does the bus stops per hour?

less.
Solution : Due to his stoppages the bus travels 9 km
person for the entire trip. e
speed of 60 km/hr. Calculate the average speed of the

Solution : It is given that distance travelled by thet


person at different speeds are the same.
Hence, time taken by the bus to cover 9 km is the 60 + 40
time used up at stoppages and time taken to cover 9 km Hence, average speed calculated by = 50
2
9
= × 60 = 10 minutes. km/hr does not give the correct average speed.
54
Total distance
Therefore, the bus stops 10 minutes per hour. Thus, average speed =
Total time
Example 6. A is twice as fast as B and B is thrice as In travelling equal distance with speeds u and v, the
fast as C. The journey covered by B in— 2uv
(a) 18 min (b) 27 min (c) 38 min (d) 9 min average speed is expressed as ·
(u + v)
Solution : Let the speed of C be x km/hr, then speed 2 × 60 × 40
of B = 3x km/hr and speed of A = 6x km/hr. Therefore, Hence, Average speed = = 48 km/hr.
60 + 40
the ratio of speed of A, B and C = 6 : 3 : 1 and ratio of Example 8. What is the average speed if a person
1
time taken by A, B and C to cover the same distance = : travels at the speed of 20 km/hr and 30 km/hr ?
6
1 (a) For the equal interval of time.
: 1 = 1 : 2 : 6.
3 (b) For equal distance.

CAT Complete Course | 279

Visit : www.EasyEngineering.net
Visit : www.EasyEngineering.net

Solution : we use (observe) every day. Relative speed means the


20 + 30 speed of an object A with respect to another object B,
(a) Average speed = = 25 km/hr which may be stationary, moving in the same direction as
2
A or in the opposite direction as A.
However, it can also be solved by the basic formula
of average speed. It is often seen, while sitting in a moving train, speed
of the train seems to increase when another train moving
Total distance
Average speed = in the opposite direction crosses it. The speed appears
Total time
decreasing when another train moving in the parallel
20 km + 30 km
= track in the same direction passes it. Hence, this feeling
1 hr + 1 hr of change in speed of train is nothing but its relative
50 speed in relation to the another moving train .
= = 25 km/hr.
2
Case I : When one object is stationary and the
2uv 2 × 20 × 30 other is moving
(b) Average speed = =
(u + v) 20 + 30
For example : If one Ambulance (A) is standing and
1200
= = 24 km/hr. second Ambulance (B) passing by, the relative speed of
50 the Ambulance A and the B will be the speed of B. If B’s

ww
Example 9. A car travelled for 30% of time at a
speed of 20 km/hr, 40% of time at a speed of 30 km/hr
and rest of the journey at a speed of 40 km/hr. What is
speed is 60 km/hr, then the relative speed is also 60 km/hr.
Case II : When both moving in the same direction

w
the average speed of the car for the entire journey ?

.Ea
Solution : Let the total time taken in the entire
journey be 1 hr.
Then distance travelled in 30% of time at 20 km/hr
As both Ambulances (A and B) move in the same
direction at speeds of 40 km/hr and 60 km/hr respectively,
the relative speed of B with respect to A is (60 – 40) = 20
km/hr .

syE
= 0·3 × 20 = 6 km
Distance travelled in 40% of time at 30 km/hr
Case III : When both moving in the opposite
direction
As both A and B move in the opposite direction,
= 0·4 × 30 = 12 km
Distance travelled in 30 % of time at 40 km/hr ngi relative speed of B with respect to A is (60 + 40) = 100
km/hr .

∴ Average speed =
= 0·3 × 40 = 12 km
Total distance
Total time
nee Example 11. A person starts from city A towards B
at 8·00 a.m. at a speed of 40 km/hr. Another person starts
from city B towards A at 8·30 a.m. at a speed of 50 km/hr.

=
6 + 12 + 12
1
= 30 km/hr. and B is 200 km.
rin
At what time will they meet, if the distance between A

Example 10. A car covers four successive 3 km


stretches at speeds of 10 km/hr, 20 km/hr, 30 km/hr and g.n
Solution : It is clear from the above question that
both persons are simultaneously in motion at 8·30 a.m.
Hence, concept of relative speed will apply only when
60 km/hr respectively. What is the average speed of the
car for the entire journey ?
Solution :
Total distance = (3 + 3 + 3 + 3) = 12 km.
e
both the bodies are in motion i.e., at 8·30 a.m. In half
hour, person starting from A would have travelled 20 km.
Hence, at 8·30 a.m. the distance between A and B is (200
– 20) = 180 km.
t
3 3 3 3 Distance = 180 km,
Total time = + + +
10 20 30 60 Relative speed = (50 + 40) = 90 km/hr
108
= 3 ( 6+3+2–1
60 ) Hence, time when they meet =
90
= 2hr.

12 × 3 3 Therefore, 2 hours after 8·30 a.m. i.e., 10·30 a.m.


= = hr both the person will meet.
60 5
Total distance Example 12. A thief steals a car at 2·30 p.m. and
∴ Average speed = drives it at 60 km/hr. The theft is discovered at 3 p.m. and
Total time
the owner sets off in another car at 75 km/hr. When will
12
= = 20 km/hr he overtake the thief ?
3/5
Solution : Both the persons are in motion at 3 p.m.,
Relative Speed hence distance between the two persons at 3 p.m. = 30 km
The word ‘relative’ means one with respect to (because the thief has travelled 30 km in half an hour).
another. Basically relative speed is the phenomenon that Relative speed = (75 – 60) = 15 km/hr

280 | CAT Complete Course

Visit : www.EasyEngineering.net
Visit : www.EasyEngineering.net

Distance to be covered = 30 km (iv) Two trains A and B start from two points p and
Hence, time taken by the owner in overtaking the q and move towards each other, after crossing they take
thief time a and b in reaching q and p respectively. Then the
ratio of their speed is given by :
30
= = 2 hrs.

√ba
15
A’s speed : B’s speed =  b : √ a =

Therefore, owner will overtake the thief at 2 hours
after 3 p.m., i.e., at 5 p.m. Example 14. A train 140 m long is running at 60
Example 13. Mohit and Ajay are two friends whose km/hr. In how much time will it pass a platform 260 m
homes are 20 km apart. Both of them decide to meet long ?
somewhere between their houses. Mohit rides at 8 km/hr Solution :
and Ajay at 10 km/hr. Mohit leaves his house at 8·00 a.m.
5 50
and Ajay leaves his house at 9·00 a.m. Speed of train = 60 × = m/s
18 3
(a) At what time they meet ?
Distance covered by train in crossing the platform
(b) At what distance from Ajay’s house ?
= (140 + 260) = 400 m
Solution : Ajay leaves one hour after Mohit has

ww
started. Hence at 9.00 a.m. Mohit would have travelled 8
km at a speed of 8 km/hr. Now, at 9.00 a.m. the distance
between Mohit and Ajay =(20-8)=12 km. which is to be

3
Time taken = 400 × = 24 s
50
Example 15. Two trains are running on parallel lines

∴ w
covered at a relative speed of (8+10)=18 km/hr.

Time taken =
.Ea 12
18
× 60 = 40 min.
in the same direction at speeds of 40 km/hr and 20 km/hr
respectively. The faster train crosses a man in the second
train in 36 seconds. Find the length of faster train.
Solution : Let the length of the faster train be x
a.m., i.e., at 9·40 a.m.
syE
Thus, both of them will meet 40 minutes after 9·00

(b) Distance travelled by Ajay in 40 minutes


metre. Since, train crosses a man sitting in another train,
hence we are concerned with the length of one train only.

ngi
Speed = (40 – 20)
40 400 20 2
= 10 × = = km or 6 km 5
60 60 3 3 = 20 × m/s
18
Concept of Relative Speed in Motion of
Trains
nee Time =
Distance x × 18
Speed
=
20 × 5
1. Train
(i) Time taken (t) by a train x metres long to cross a
or
rin
36 =
18x
100
stationary person or pole is equal to time taken by the
train to cover the distance x metres (equal to its length )
with its own speed (v)

g.n
x = 200 m
Example 16. A train speeds past a pole in 15 second

t =
x
v
(ii) Time taken by a train x metres long to cross a
stationary objects (Like Railway station , bridge, tunnel,
of train.
e
and a platform 100 m long in 25 seconds. Find the length

t
Solution : Let the length of train be x metre and speed
be y m/s.
another standing train etc.) y metres long is equal to time Distance x x
∴ Time = = or 15 =
Speed y y
taken by the train to cover a total distance (x + y) meters
as its own speeds (v) x
∴ Speed (y) = m/s
x+y 15
t =
v The train passes the platform 100 m long in 25
(iii) If two trains of length ‘x’ and ‘y’ metres move in seconds.
the same direction at ‘a’ and ‘b’ m/sec, then the time Therefore, train will have to cover a distance of
taken to cross each other from the time they meet x
(x + 100) metre with a speed of m/s.
Sum of their length x + y 15
t = =
Relative speed a–b (x + 100)
Speed =
N.B. If above trains move in the opposite direction, 25
then x (x + 100)
=
Sum of their length x + y 15 25
T = =
Relative speed a+b x = 150 m

CAT Complete Course | 281

Visit : www.EasyEngineering.net
Visit : www.EasyEngineering.net

Example 17. A train running at 54 km/hr takes 13. Work and Time
20 seconds to pass a platform. Next it takes 12 seconds to
pass man walking at 6 km/hr in the same direction in Some Basic Relations
which the train is going. Find the length of the train and 1. If A can do a piece of work in ‘m’ number of days,
the length of the platform.
Solution : ()
then in one day
1
m
th of a work is done. Conversely, if

Speed of train in relation to man = (54 – 6) km/hr


5
a man does () 1
m
th of a work in 1 day, then he can
= 48 × 1
18 complete the work in 1 ÷ = m days.
m
40
= m/sec. 2. If A is ‘m’ times as good a workman as B, then he
3
Length of train (T) = Speed of Train (Relative)
× Time to pass a man
(A) will take () 1
m
th of the time taken by B to do the
same work .
40
= × 12 = 160 m 3. Amount of work done, basically depends on
3
various variables, like number of persons, time period,

ww
Train + Platform (T + P) their efficiency etc.
54 × 5
= × 20 L1 T1 LT
18 = 2 2 = k (constant)
W1 W2

w .Ea
Hence, length of platform
= 300 m

= 300 – 160
= 140 m
[L means number of Labour doing the work (W) in
Time T]
4. If A and B can do a piece of work in ‘l’ and ‘m’

2. Boats and Streams


syE ( () )
Upstream : If a boat (or person) moves against the
days respectively, then working together, they will take
lm
l+m
days to finish the work and in one day, they will

n
l+m
stream (in opposite direction) it is called upstream (→ ←). finish th part to the work.

( ) gi
lm
Downstream : If a boat (or person) moves with the
5. If A, B and C can do a certain piece of work in l,

n(ee )
stream (in same direction) it is called downstream → .

(i) If the speed of a boat in still water is B km/hr,


m and n days respectively, then they can together do the
same work in
l.m.n
lm + mn + nl
days.
and the speed of the stream is S km/hr.

( ) →
rin
Speed of boat in downstream → = (B + S) km/hr.
6. A and B together can do a work in l days, B and
C together in m days, C and A together in n days, then the

)g.
same work can be done :
Speed of boat in upstream (→ ←) = (B – S) km/hr.

( ) (→
(ii) Let boat’s speed downstream → = x km/hr
and boat’s speed upstream (→ ←) = y km/hr. Then,
By A alone in

By B alone in (
2lmn
lm + mn – nl
2lmn
)
mn + nl – lm
days.

days.
net
1
(a) rate of boat in still water = (x + y) km/hr.
1
2 and by (A + B + C) together in ( 2lmn
)
lm + mn + nl
(b) rate of stream = (x – y) km/hr. days.
2
7. A group of ‘m’ persons can do a work in ‘d’ days.
Example 18. A man can row 6 km/hr in still water If the group had ‘n’ more persons the work could be
1
and the river is running at 4 km/hr. If a man takes 1 hour finished in ‘t’ days less. Then, the number of persons
2
initially present in the group m :
to row to a place and back, how far is the place?
n × (d – t)
Solution : Downstream Speed = ( 6 + 4 ) = 10 km/hr =
t
Upstream Speed = ( 6 – 4) = 2 km/hr 8. ‘m’ persons start to do a work in ‘d’ days. After
Let the distance be x, then ‘t’ days, n more men had to join in order to complete the
x x 3 work in scheduled time. If ‘n’ men had not joined them,
+ =
10 2 2 the number of additional days needed to complete the
30 work is given by :
⇒ 6x =
2 n × (d – t)
=
Hence, x = 2·5 km. m

282 | CAT Complete Course

Visit : www.EasyEngineering.net
Visit : www.EasyEngineering.net

14. Simple Interest and Compound Solution : Amount ( Principal + Interest ) for 2 years
Interest = Rs. 1560
Amount (Principal + Interest ) for 5 years = Rs. 2100
Simple interest is the interest accrued on a certain
Hence, interest for 3 years = (2100 –1560) = Rs.540
sum at a certain rate of interest on flat basis irrespective
of any time. It means for first and subsequent years the Simple interest for 2 years = Rs .360
amount on which the interest is calculated remains the ∴ Principal = (1560 – 360) = Rs. 1200
same and hence no benefit on the interest calculated on 360 × 100
the previous years is given in the subsequent years. R = = 15%
1200 × 2
Simple interest is given by the following formula. R = 15%
P×R×T Example 5. A certain sum of money amounts to
S.I. =
100 1
Rs. 1008 in 2 years and to Rs. 1164 in 3 years. Find the
100 × S.I. 100 × S.I. 2
∴ P= ,R = ,
R×T P×T sum and the rate of interest.
100 × S.I. Solution :
T =
P×R
( 1
) 1

ww
where P = Principal amount or amount on which the
interest is calculated .
SI for 3 – 2 = 1 years
2 2
= Rs. (1164 – 1008) = Rs. 156

w
R = Rate of interest (per annum)

.Ea
T = Time period for which the interest is calculated .
S.I. ∝ P × R × T

SI for 2 year = Rs. 208
Principal = Rs. (1008 – 208) = Rs. 800
Now, P = 800, T = 2 years and SI = Rs. 208

syE
Simple interest is directly proportional to principal, 100 × SI 100 × 208
rate and time. It means if the interest on a sum at a given ∴ Rate = = = 13%
P×T 800 × 2
rate of interest for 1 year is Rs. 100, then interest for 2 The difference in amount for two different time

ngi
years, 3 years and 4 years (other things being same) will periods is equal to the simple interest for the difference in
be Rs. 200, Rs. 300 and Rs. 400 respectively. two different time periods .
Example 1. Calculate the simple interest on
3
Rs. 7200 at 12 % per annum for 9 months.
4
P ×R×T
n ee
Example 6. A man borrowed Rs. 24000 from two
money lenders. For one loan he paid 15% annum and for
the other 18% per annum. At the end of one year, he paid
[ ]
rin
51 9 Rs. 4050. How much did he borrow at each rate ?
Solution : S.I. = R = %‚ t = years
100 4 12
Solution : Let the sum borrowed at 15% be Rs. x
=
7200 × 51 × 9
4 × 12 × 100
= Rs. 688·50.

Example 2. At what rate percent per annum will a Therefore, g.n


then sum borrowed at 18% will be Rs. (24000 – x).
x × 15 × 1 (24000 – x) × 18 × 1
+ = 4050
sum of money double in 8 years ?
Solution : Let P = x, then a = 2x , S.I. = (2x – x) = x,
t = 8 years
Or,
100

e 100
15x + 432000 – 18x = 405000 or x = 9000
∴ Money borrowed at 15% = Rs. 9000
and money borrowed at 18% = Rs. (24000 – 9000)
t
S.I. × 100 x × 100
∴ R = = = 12·5%
P×T x×8 = Rs. 15000
Example 3. The simple interest on a sum of money Example 7. Rs.800 amounts to Rs. 920 in 3 years at
is 25% of the principal and the rate per annum is equal to simple interest. If the interest is increased by 3%, it
number of years. Find the rate per cent. would amount to how much ?
x Solution : SI = (920 – 800) = Rs. 120
Solution : Let the principal be Rs. x, then S.I. = Rs.
4 800 × r × 3
120 =
T = R, if R = rate per annum 100
x x×R×R ∴ r = 5%
∴ =
4 100 800 × 8 × 3
Interest at 8% =
100
⇒ R 2 = 25 or R = 5%
= Rs. 192
Example 4. A certain sum of money amounts to
Rs.1560 in 2 years and Rs. 2100 in 5 years. Find the rate ∴ Amount = (800 + 192)
percentage per annum. = Rs. 992

CAT Complete Course | 283

Visit : www.EasyEngineering.net
Visit : www.EasyEngineering.net

n
Example 8. What annual installment will discharge a
debt of Rs. 1092 due in 3 years at 12% simple interest ?
Amount = Principal (1 + 100R )
Solution : Let each installment be Rs. x, then first or A = P (1 + R × 0·01)
installment paid after 1 year will earn an interest for 2 Compound interest = Amount – Principal.
years at 12% and second installment paid after 2 year will Conversion Period : The time period after which the
earn an interest for 1 year at the same rate as the debt has interest is added each time to form a new principal, is
to be squared off in 3 years. called conversion period. It may be one year, six months
x × 12 × 1 x × 12 × 2
∴ [ x+
100 ] [
+ x+
100 ]+x or three months, i.e., annually, half-yearly or quarterly etc.
The following table will illustrate the conceptual
= Rs. 1092 working of simple interest and compound interest.
28x 31x Rate of interest per annum is 10%.
+ + x = 1092
25 25 For the
Simple Interest Compound Interest
or 28x + 31x + 25x = 1092 × 25 Year
∴ x = Rs. 325 Principal SI Principal CI

Short cut : The annual payment that will discharge a 1 1000 100 1000 100

ww
debt of Rs. A due in t years at r% rate of interest per
annum is :
100A 
= 
100 × 1092 
2

3
1000

1000
100

100
1000 + 100
= 1100
1100 + 110
110

121

[ w]100t +
Rt (t – 1)
2

.Ea
=
 100 × 3 +
109200
336
3 × 12 (3 – 1)

= Rs. 325
2

= 1210
On the basis of above calculation, it is clear that :
(1) Simple interest for each year is constant.

syE
(2) Compound interest calculated for each year
Example 9. A sum was put at simple interest at a includes—Simple interest on principal and simple interest
certain rate for 2 years. Had it been put 3% higher rate, it on interest calculated for previous year.

ngi
would have fetched Rs. 72 more ? Calculate the sum. Let principal = p, rate = R% per annum, time = n
Solution : Let the sum be Rs. x and rate be r%. years and amount = A
x × (r + 3) × 2 x × r – 2 (i) When interest is compounded annually :
Then, 1280 =

= 72
100

100
nee Amount = p 1 + [ R n
100 ]
Or, 2rx + 6x – 2rx = 7200
⇒ x = Rs. 1200 rin
(ii) When interest is compounded half- yearly :
Half-yearly : n = 2 × (given time in years) and R

g.n
= 1/2 ( given rate of interest per annum)
Example 10. The rate of interest on a sum of money 2n
 R 
is 4% per annum for the first 2 years, 6% per annum for  2 
Amount = p  1 + 
the next 3 years and 8% per annum for the period beyond
5 years. If the simple interest accrued by the sum for a
total period of 8 years is Rs. 1280, what is the sum ?
Solution :
e 100
(iii) When interest is compounded quarterly :
Quarterly : n = 4 × (given time in years) and R = 4 t
1

Rate of interest for first 2 years = 4 × 2 = 8% (given rate of interest per annum )
4n
Rate of interest for next 3 years = 6 × 3 = 18%  R 
Rate of interest for last 3 years = 8 × 3 = 24%  4 
Amount = p  1 + 
100
Total rate of interest for 8 years = 50%
(iv) When interest is compounded annually but time
P × 50 2
Then, 1280 = is in fraction , say 3 years
100 5
∴ P = Rs. 2560  2 R 
3  5 
(Time of 8 years is already adjusted in the total rate
of interest calculation )
then, Amount = p 1 + [ R
100 ]×  1 + 
100
(v) Present worth of Rs. x due in n years, hence is
Compound Interest given by :
Compound Interest : The interest charged every x
Present worth =
R n
year on the amount of last year is called compound
interest. [ 1+
100 ]
284 | CAT Complete Course

Visit : www.EasyEngineering.net
Visit : www.EasyEngineering.net

Arrangement for Different Conversion 12


Solution : R = = 3% per quarterly, P = 12000
Periods 4
3 4
# The ratio of change in an object over a particular
period to the measurement of that object for that particular
∴ Amount = 12000 1 + (100 )
Period . = 12000 (1·03) 4
# Rate of growth of objects can be positive in some = Rs. 13506·11
cases as in case of population; and in some cases it may
∴ CI = (13506·11 – 12000) = 1506·11
be negative as in the case of depreciation of machinery
over a period of time . Example 14. Find the compound interest on
Rs. 16000 at 20% per annum for 9 months compounded
# In compound interest , the amount at rate R and n
quarterly .
time can be calculated by the formula
A = P (1 + R × 0·01)n Solution : R = 5% per quarter, T = 9 months or 3
quarterly
# If the rate of growth is negative, in that case
5 3
formula becomes
A = P (1 – R × 0·01)n
∴ Amount = 16000 × 1 +( )
100

ww (where, n is the number of conversion periods)


# The rate of growth may vary from one to the other
= Rs. 18522
21 21 21
= 16000 × × ×
20 20 20

w
conversion period . In that case the formula becomes :
A = P (1 + R1 × 0·01) (1 + R2 × 0·01) (1 + R3 × 0·01) ∴ C.I. = (18522 – 16000) = Rs. 2522
… (1 + Rn × 0·01)
.Ea
# To find rate (R), principal (P), time (n) ,
Example 15. The difference between the compound
interest and simple interest on a certain sum at 10% per

Or (1 + R × 0·01)n = syE
P = A ÷ (1 + R × 0·01)n
A
P
annum for 2 years is Rs. 631. Find the sum.
Solution : Let the sum be Rs. x.

( 10
) 21x
Or C.I. = P {(1 + R × 0·0) – 1}
n

Example 11. Calculate the population of a town afterngi C.I. = x 1 +


100
x × 10 × 2 x
–x=
100

nee
SI = =
2 years, if it grows at a rate of 10% per annum. The 100 5
present population is 1200000. 21x x
Given, – = 631
100 5
Solution : Population of the town after 2 years
= 12,00,000 × 1·1 × 1·1 ⇒
rin
x = Rs. 63100

= 1452000

(1·1 =
100 + 10
100 )
g.n
Concept : Simple interest and compound interest
for the first year is the same. Difference in the second
year’s interest is due to the fact that compound
The same question can be calculated by the method
of compound interest .

Amount = P 1 +( r n
100 ) e
interest is calculated over the first year’s interest also .

year at 10% . Hence , interest on first year = Rs. 6310.t


Hence, Rs. 631 is the interest on the interest of first

10 2
= 1200000 1 +( 100 )
= 1452000
Now, if interest for first year is Rs. 6310 at 10% then
principal = Rs. 63100.
Example 12. Find the compound interest on Example 16. Rs. 25000 is borrowed at CI at the rate
Rs. 30500 at 15% per annum for 2 years compounded of 3% for the first year, 4% for the second year and 5%
annually. for the third year. Find the amount to be paid after 3 years.
15 2
Solution : A = 30500 1 + (
100 ) Solution : Amount = 25000 × 1·03 × 1·04 × 1·05
= Rs. 28119
= 30500 × 1·15 × 1·15 Example 17. At what rate per cent per annum will
= 40336·25 Rs. 1000 amount to Rs. 1331 in 3 years ? The interest is
∴ CI = (40336·25 – 30500) compounded yearly.
R 3
= Rs. 9836·25
Example 13. Find the compound interest on
Solution :
1331
1000 ( = 1+
100 )
3
R 3
Rs. 12000 at 12% per annum for 1 years, compounded
quarterly. ( ) (
11
10
= 1+
100 )
CAT Complete Course | 285

Visit : www.EasyEngineering.net
Visit : www.EasyEngineering.net

11 R When difference between the CI and SI on a certain


⇒ = 1+
10 100 sum of money for 2 years at r% rate is x, then the sum is
⇒ R = 10% given by :
Difference × 100 × 100 100 2
Example 18. A sum of money placed at compound
interest doubles itself in 4 years. In how many years will
Sum =
Rate × Rate
=x× ( )r
it amount to eight times itself ? Example 21. The difference between the compound
r 4
Solution : Given, 2P = P 1 + (100 ) interest and the simple interest on a certain sum of money
at 5% per annum for 2 years is Rs. 1·50. Find the sum.
4 Solution : Using the above formula :
or, ( 1+ )
r
= 2
100 2
100
4 3 Sum = 1·5 ( ) 5
= 1·5 × 400 = Rs. 600
or, [(1 + 100r ) ] = (2) 3 = 8
On a certain sum of money, the difference between
12 compound interest and simple interest for 2 years at r%
P (1 +
100)
r
or, = 8P r 2

Hence, required time is 12 years.


rate is given by sum( )
100
.

ww
Remember : If a sum becomes n times in t years at
compound interest then it will be (n) m times in mt years.
Example 22. Find the difference between the com-
pound interest and simple interest for Rs. 2500 at 10%
per annum for 2 years.

w
Thus, if a sum of money doubles itself in 4 years ,
then it will become (2)3 . Times in 4 × 3 = 12 years..

.Ea
Example 19. If the compound interest on a certain
sum for 2 years at 3% be Rs. 101·50, what would be the
Solution : Using the above formula :

Difference = sum ( )R 2
100
10 2
( )
simple interest ?
Solution :
s
( ) yE
3 2 609
= 2500 ×
100
= Rs. 25

If the difference between CI an SI on a certain sum


CI on 1 rupee = 1 +

2×3 6
100
–1=
1000
ngifor 3 years at r% is Rs x, the sum is given by:

Sum =
Difference × (100)3
R2 (300 + R)

nee
SI on 1 rupee = =
100 100
Example 23. If the difference between CI and SI on
SI 6 10000 200
= × = a certain sum of money for 3 years at 5% per annum is.

rin
CI 100 609 203 122, find the sum.
200 200 Solution : Using the above formula :
∴ SI = × CI = × 101·50

g.n
203 203
122 × 100 × 100 × 100
= Rs. 100 Sum =
52 (300 + 5)

e
If on a certain sum of money , the SI of 2 years at On a certain sum of money, the difference between
the rate r% per annum is Rs. X, then the difference in
the compound interest and simple interest is given by
Rs.( )Xr
200
. This formula is applicable only for 2 years .
per annum is given by :

Difference =
Sum × R2 (300 + R)
(100) 3
t
compound interest and simple interest for 3 years at r%

Example 20. On a certain sum of money, the simple


Example 24. Find the difference between CI and SI
interest for 2 years is Rs . 50 at the rate of 5% per annum.
on Rs. 8000 for 3 years at 2·5 % per annum.
Find the difference in CI and SI.
Solution : Using above formula : Solution : Using above formula
50 × 5 8000 × (2·5)2 × (300 + 2·5)
Difference in CI and SI = = Rs. 1·25 Difference =
200 100 × 100 × 100
P×2×5 8 × 25 × 25 × 3025
Concept : 50 = =
100 100 × 100 × 100
121
∴ P = Rs. 500 = = Rs. 15·125
8
105 × 105
A = 500 × = Rs. 551·25 Hire Purchase : In a hire purchase plan, a customer
100 × 100
can make use of the goods while paying for them. The
∴ CI = (551·25 – 500) = Rs. 51·25 amount paid at the time of purchase is called the down
Hence, CI – SI = 51·25 – 50 = Rs. 1·25 payment. The remainder is paid in equal installments and

286 | CAT Complete Course

Visit : www.EasyEngineering.net
Visit : www.EasyEngineering.net

each is the monthly installment. The difference between 2. If pipe A is ‘x’ time bigger than pipe B, then pipe
the total amount to be paid and the cash price is called the 1
A will take th of the time taken by pipe B to fill the
installment charges. x
Amount to be Paid cistern.
– Down Payment 3. If A and B fill a cistern in ‘m ’ and ‘n’ hours,
Monthly Installment =
Number of Instalments mn
respectively then together they will take hours to
Example 25. If a LG Refrigerator is available at m+n
th
Rs 4,000 cash or Rs. 1,000 down payment and Rs. 700
per month for 5 months. Find : (i) Total Amount paid for
fill the cistern and in one hour ( ) m+n
mn
part of the

it, (ii) The Installment Charge. cistern will be filled.


Solution : 4. If an inlet pipe fills a cistern in ‘m’ hours and an
(i) Amount paid = 1000 + 700 × 5 = 4500 outlet pipe empties the cistern in ‘n’ hours, then the net
part filled in 1 hour when both the pipes are opened, is
(ii) Installment Charge = 4500 – 4000 = 500

15. Pipe and Cistern


( )1 1

m n
i.e.,
n–m
mn
and the cistern will get filled in

( ) mn
hours.

wwProblems on pipes and cisterns are closely related to


problems on work.
A pipe connected with a cistern is called an inlet, if it
m–n
If more then one pipe is working for inlet or as outlet
pipes then you can use following formula.

w
fills the cistern.

.Ea
A pipe connected with a cistern is called an outlet, if
it empties the cistern.
Net part filled of a cistern = (Sum of work done by
inlets pipes) – (Sum of work done by outlet pipes)
Some Short Cuts

syE
In fact, filling or emptying a cistern can be considered
as work done. 1. If an inlet pipe fills a cistern in ‘a’ minutes, takes
‘x’ minutes longer to fill the cistern due to a leak in the
Some Useful Points : cistern, then the time in which the leak will empty the
1
1. If an inlet pipe fills a cistern in ‘a’ hours, then th
a ngi ( )
cistern is given by a × 1 +
a
x
.

part is filled in 1 hour.


Similarly, if an outlet pipe empties a cistern in ‘a’
1 nee 2. If two pipes A and B can fill a cistern in ‘x’
minutes and if A alone can fill it in ‘a’ minutes more than
‘x’ minutes and B alone can fill it in ‘b’ minutes more

rin
hour, then th part is emptied in 1 hour.
a than ‘x’ minutes, then x = √
ab .
●●●

g.n
e t

CAT Complete Course | 287

Visit : www.EasyEngineering.net
Visit : www.EasyEngineering.net

Part B

ADVANCE TOPICS OF MBA MATHEMATICS

1 Numbers
Numbers are collection of certain symbols or figures SOME IMPORTANT POINTS ABOUT PRIME
called digits. The common number system is use in NUMBERS
Decimal system. In this system we use ten symbols each
1. 1 is not a prime number.
representing a digit.

ww
These are 0, 1, 2, 3, 4, 5, 6, 7, 8 and 9. A combination
or single of these figures representing a number is called
a natural number.
2.
3.
4.
There is only one even prime number i.e., 2.
A composite number may be even or odd.
All prime number can be written in the form

w
Number system is the key concept in every branch of
mathematics. The use and scope of number system is

and properties of numbers.


.Ea
unlimited. The system deals with the nomenclature, use
(6N – 1) or (6N + 1). The converse is not neces-
sarily true. This means any number of the form
(6N – 1) or (6N + 1) is not necessarily a prime
number.
CLASSIFICATION OF NUMBERS

syE
1. Natural Number—These are also called counting
numbers as these numbers are the ones which we use for
5. The remainder when a prime number p ≥ 5 is
divided by 6 or 5.
4. Fractional Number—A number which represents
counting purpose. It is represented by
N : { 1, 2, 3, 4, …………10,000 …………}
ngi a ratio or division of two numbers is called fractional
number.
3 5 1 7

nee
2. Whole Number—It include all Natural numbers Example— – ‚ , ,
plus zero and we can denote it by 2 2 2 2
W : { 0, 1, 2, 3, ………10256, …………} A fractional number has two parts Numerator and
Denominator
3. Integer—It includes all whole numbers alongwith
negative numbers. It is represented by 3
2 rin
= 3 is = Numerator and 2 is denominator
I : {………, -5, -4, -1, 0, 1, 2, 3, 4, …………}
Natural numbers are categorized into the following
numbers. g.n
(a) Equivalent Fraction—Two fractions are said to
be equivalent if they represent the same ratio or number.

(a) Even Number—A number which is completely


divisible by 2 is called an even number.
Example—{ 2, 4, 6, 8, 10, ………,102056,………}
e
So if we multiply or divide the numerator and denom-
inator of a fraction by the same non-zero, integer, the
t
result obtained will be equivalent to the original fraction.

Example—
20
is equivalent to
4
(b) Odd Number— A number which is not com- 25 5
pletely divisible by 2 is called odd number.
1 3
Example—{ 1, 3, 5, 7, ………,10001, ………} is equivalent to
3 9
(c) Prime Number—The numbers that have only (b) Proper Fraction—When denominator is greater
two factors 1 and number itself are called prime numbers. than numerator then such fraction number is known as
Example—{ 2, 3, 5, 7, 11, 13, 17, 19, ………} proper fraction.
(d) Twin Prime Number—If the difference between 1 1 3 7
two consecutive prime number is 2 then Example— ‚ ‚ ‚
3 5 4 9
Both prime numbers are known as twin prime num- (c) Improper Fraction—Those fractional number
bers. whose numerator is greater than denominator are called
Example—{ 5, 7 },{ 17, 19 } improper fraction.
(e) Composite Number—A composite number is 7 5 7 17
Example— ‚ ‚ ‚
one which has other factors besides itself and 2 2 3 4
unity. (d) Mixed Fraction—It consists of integral as well
Example—4, 6, 9, 14, 15, ………etc. as the fractional part.

288 | CAT Complete Course

Visit : www.EasyEngineering.net
Visit : www.EasyEngineering.net

3 11 Example— 
√– 3, 2 + 3i, 3 – i √
3
Example— 2 =
4 4
3
where 2 is integral part and is fractional part.
4
In general, for any composite number C, which can
be expressed as
C = am × bn × c p ……
where a, b, c ……… are all prime factors and m, n, p
are positive integers.
So, the total number of factor is given by = (m + 1)
(n + 1) (p + 1)………
(e) Compound Fraction—A fraction of a fraction
is known as compound fraction.
4 9
Example— or
5 11
4 9 36
× =

ww 5 11 55
2 4 8
× =
3 3 9

w
(f) Complex Fraction—Any complicated combina-
tion of other type of fractions.
Example— 2 or
1
3 .Ea
1+
3
2

4
3
of
3+
syE
3
3
2
1+
1
2+3
5. Rational Number—If a number can be expressed ngi
p
in the form of where q ≠ 0 and where p and q are
q
integers, then the number is called rational number.
nee
9 1 5
Example— , , , –
25 3 2
27 3
,–
51 5 × 151 × 289 × 156. rin
Illustration 1. Find the unit digit in the product 439

Important points—Fractional numbers and integers


are the part of rational number.
g.n
Solution : Product of units digits in given numbers
= 9 ×1×9×6
6. Irrational Number— If a number can not be
p
expressed in the form of , where q ≠ 0, then the number
q
is called irrational number.
In other words—
= 486

e
∴ Unit digit in the given product is 6.

t
Illustration 2. Find the unit digit in the product 366 ×
641 × 753.
Solution : We know, unit digit in 31 = 3; 32 = 9; 33 =
Non-repeating and non-terminating type of decimals 7; 34 = 1 .
are called irrational numbers. So, unit digit of 364 is 1.
3 Then unit digit in 366 is 364 + 2 = 1 × 3 × 3 = 9
Example—  √ 2, √ 4, 4·3767, 0·3333 Unit digit in 641 = 6
7. Real Number—Set of all numbers that can be Unit digit in 71 =7 ; 72 = 9; 73= 3; 74 =1
represented on the number line are called real numbers. It
So, unit digit in 752 is 1
includes all number such as whole, fractional, integer,
rational, natural numbers. Hence, unit digit in 753 is 752 + 1 = 1 × 7 = 7
8. Complex Number—It is also known as imaginary Therefore, product of unit digit in the given numbers
number. 366 × 641 × 753
= 9 × 6 × 7 = 378
Imaginary number is written in the form a + ib,
where a, b are real numbers. ∴ Unit digit in the given product = 8
Illustration 3. Convert 1.33333………into a rational
i = imaginary unit whose value is √
– 1. number.
Those numbers which are inside root symbol with Solution : Let X = 1.33333 ………
negative sign are known as complex number. 10 X = 13.33333

CAT Complete Course | 289

Visit : www.EasyEngineering.net
Visit : www.EasyEngineering.net

10 X – X = (13.3333……) Illustration 6. Find the triplicate ratio of 4 : 5.


– (1.3333………) Solution : The triplicate ratio of 4 : 5 is 64 : 125.
9X = 12
Sub-Duplicate Ratio : For any ratio a : b, its sub-
4
X =
3 duplicate ratio is defined as 
√ a : √ b.
Inverse proportion : A is inversely proportion to B Illustration 7. What is the sub-duplicate ratio of 16 :
means if A increases B decreases proportionally. 25 ?
Distance Solution : The sub-duplicate ratio of 16 : 25 is
Example : Speed = · If speed is doubled,
Time 16 : √25 = 4 : 5

time taken to cover the same distance is reduced to half.
Sub-Triplicate Ratio : For any ratio a : b, its sub-
1
Inverse Proportion : If A ∝ 3 3
B triplicate ratio is defined as 
√ a : √ b.
It means A is inversely related to B. Illustration 8. Find the sub-triplicate ratio of 27 :
1 64.
A = k× Where k = constant
B Solution : The sub-triplicate ratio of 27 : 64 is
Here, variation is inverse but not proportional. 3 3
27 : √64 = 3 : 4

ww
Illustration 4. Ram can do a piece of work in 24
days, Shyam is 60% more efficient than Ram. Find the
number of days that Shyam takes to do the same piece of
Illustration 9. If a : b is the duplicate ratio of (a + y)
: (b + y) show that y2 = a.b.
work.

w .Ea
Solution : Ratio of efficiencies of A and B = 100 : 160
= 5 : 8
Solution : Given

Or
a
b
a
b
= ( )
= 2
a+y 2
b+y
a + y2 + 2ay
2

b + y2 + 2by

syE
Since, efficiency is inversely proportional to the
Or, a.b2 + a.y2 + 2 a.b.y
number of days.
= a.y2 + 2 a.b.y + b.a2
Hence, ratio of days taken to complete to the job is
Or, y2 (a – b) = a.b (– b + a)
8:5
So, number of days taken by Shyam
ngi Or, y2 = a.b
Illustration 10. Find the compound ratio of (a + y) :

nee
5
= × 24 = 15 days (a – y), (a2 + y2) : (a + y)2 and (a2 – y2)2 : (a4 – y4).
8
a + y a2 + y2 (a2 – y2)2
Commensurable Ratio : It is the ratio of two Solution : × ×
a – y (a + y)2 a4 – y4
fractions or any two quantities which can be expressed
exactly by the ratio of two integers.
rin =
(a2 – y2)2
(a – y)(a + y)2 (a2 – y2)
Example : The ratio of 10 m to 40 m. is 10 : 40 i.e.,
1 : 4 which is the ratio of two integers, So these are
commensurable quantities. g.n
= 1
Illustration 11. If ax + cy + bz = 0, cx + by + az = 0
Incommensurable Ratio : It is the ratio of two
fractions or any two quantities in which one or both the
terms is a surd quantity. No integers can be found which
will exactly measure their ratio i.e., cannot be exactly
Solution :
Given ax + cy + bz = 0
cx + by + az = 0
e
and bx + ay + cz = 0, then show that a3 + b3 + c3 = 3 abc.

t
…(1)
…(2)
expressed by any two integers. bx + ay + cz = 0 …(3)
Example : The ratio of a side of a square to its From equation (1) and (2), we get
diagonal is 1 : √
 2 which can not be expressed as a ratio
of two integers. Thus, 1 and  √ 2 are incommensurable
quantities.
ac – b2 : bc – a2 : ab – c2
Duplicate Ratio : It is compounded ratio of two x y z
a2 Let = = =k
equal ratios. Thus, duplicate ratio of a : b is 2 or a2 : b2 . ac – b2 bc – a2 ab – c2
b
Then, x = k (ac – b2 )
Illustration 5. Find the duplicate ratio of 4 : 5.
y = k (bc – a2 )
Solution : The duplicate ratio of 4 : 5 is 16 : 25.
z = k ( ab – c2)
Triplicate Ratio : It is the compounded ratio of
a3 Put them in equation (3) we get,
three equal ratios. Thus, the triplicate ratio of a : b is 3 bk (ac – b2 ) + ak (bc – a2 ) + ck (ab – c2) = 0
b
or a3 : b3 . 3 abc = a3 + b3 + c3

290 | CAT Complete Course

Visit : www.EasyEngineering.net
Visit : www.EasyEngineering.net

Test of Divisibility (K) Divisibility test by 15 : A number is divisible by


(A) Divisibility by 2 : A number is divisible by 2 15 when it is simultaneously divisibly by 3 as well as 5.
when its unit digit is either even or zero. Ex. : 4875; 3840 are divisible by 15.
Ex. : 4, 6, 8, 112, 13256 are divisible by 2. (L) Divisibility test by 16 : A number is divisible by
3, 5, 7, 111, 11567 are not divisible by 2. 16, when the number formed by its four extreme right
digits is divisible by 16 or these last four digits are zeroes.
(B) Divisibility by 3 : A number is divisible by 3,
Ex. : 50248, 200768,10000 are divisible by 16.
when the sum of its digits is divisible by 3.
(M) Divisibility by 18 : A number is divisible by 18
Ex. : 426 : 4 + 2 + 6 = 12 which is divisible by 3.
when it is divisible by 9 as well as by 2 that is the sum of
Hence, 426 is the divisible by 3. its digits is divisible by 9 and also the number has either
5436 : 5 + 4 + 3 + 6 = 18 which is divisible by 3 zero or even digit at its unit’s place.
Hence, 5436 is divisible by 3. Ex. : 2610, 781812.
(C) Divisibility by 4 : A number is divisible by 4 BINARY NUMBER SYSTEM
when the number formed by its two extreme right digit is
either divisible by 4 or both these digits are zero. Binary Number system contains only 0 and 1. Every
Ex. : 524; 1032; 111524; 200; 400 are divisible by 4. decimal number can be represented by binary numbers.

ww(D) Divisibility test by 5 : A number is divisible by


5, when its unit digit is either 5 or zero.
Our practical numbers such as 10, 12, 13, 14, ……
are called the decimal number system. This is because
there are 10 digits in the system 0 – 9.

w
Ex. : 50,4500, 5155, 735 are divisible by 5.

.Ea
(E) Divisibility test by 6 : A number is divisible by
6, when it is divisible by 2 as well as 3.
Ex. : 72; 840 are divisible by 6.
(A) Conversion of a decimal Number to a binary
system:
1. Divide the decimal number by 2.
2. Keep dividing the quotient by 2 still the quotient is

syE
(F) Divisibility test by 8 : A number is divisible by
8, when the number formed by its three extreme right
digits is divisible by 8 or when these last three digits are
0.
3. Write down the remainders on the right side after
each of the above division.
zeroes.
Ex. : 13248;11600;1000 are divisible by 8. ngi 4. Arrange the remainders in the reverse order to get
the required equivalent binary number.
(G) Divisibility test by 9 : A number is divisible by
9 when the sum of its digits is divisible by 9. nee Conversion of (31)10 to binary number
2
2
31
15 1
Ex. : 53973 : 5 + 3 + 9 + 7 + 3 = 27 is divisible by 9.
(H) Divisibility test by 11 : A number is divisible by
rin 2
2
7
3
1
1
11 when the difference between the sum of the digits at
odd places and the sum of the digits at even places is
either 0 or divisible by 11. g.n
So (31)10 = (10111)2
1 0

Ex. : 34381567 sum of digits at odd places = 3 + 3 +


1 + 6 = 13
Sum of digits at even places = 4 + 8 + 5 + 7 = 24
e
(B) Conversion of binary number to decimal number :
1. Write down the binary number in a series.
2. Multiply the extreme right digit by 0 power of 2 t
Difference = 24 – 13 =11 i.e., 20.
Difference is multiple of 11 So, the given number is 3. Multiply the digit next to extreme right by 1 power
divisible by 11. of 2 i.e., 21 .
4. Multiply the third digit from right by 2 power of 2
(I) Divisibility test by 12 : A number is divisible by
i.e., 22 or 4.
12, when it is divisible by 3 or 4.
5. Follow the same procedure till the extreme left
Ex. : 4716 ; 4 + 7 + 1 + 6 = 18 divisible by 3 and
digit.
last two digits 16 also divisible by 4.
6. Add all the number so obtained to find the
3108 last digit 08 is divisible 4 and sum 12 is equivalent decimal number
divisible by 3. For Example : Conversion of (110001)2 equivalent
So 4716, 3108 are divisible by 12. to decimal number :
(J) Divisibility test by 14 : A number is divisible by (110001) 2 = 1 × 25 + 1 × 24 + 0 × 23
14 when it is simultaneously divisible by both 2 as well + 0 × 2 2 + 0 × 21 + 2 0
as 7. = 32 + 16 + 1
Ex. : 460992, 2352 are divisible by 14. = 49

CAT Complete Course | 291

Visit : www.EasyEngineering.net
Visit : www.EasyEngineering.net

SOME IMPORTANT FORMULA IMPORTANT POINTS


1. (X + Y)2 = X2 + Y2 + 2.X.Y 1. In a Generalized Fibonacci sequence, the sum of the
2. (X – Y ) 2 = X2 + Y2 – 2.X.Y first n terms is Fn+2 minus the second term of the
3. (X + Y)2 – (X – Y)2 = 4.X.Y series.
2. The square of any Fibonacci number differs by 1
4. (X + Y)2 + (X – Y)2 = 2 (X 2 + Y2 ) from the product of the two Fibonacci numbers on
5. X2 – Y2 = (X + Y) (X – Y) each side.
6. X3 – Y3 = (X + Y) ( X2 + Y2 – X.Y) 3. (Fn )2 + (F n + 1)2 = F2n+1
= (X + Y)3 – 3.X.Y (X + Y) 4. For any four consecutive Fibonacci number A, B, C,
D
7. X – Y = ( X – Y) (X2 + Y2 + X.Y)
3 3
C2 – B2 = A × D
= (X – Y)3 + 3.X.Y ( X – Y) Golden Ratio : It is obtained by having the sum of 1
8. X + Y + Z3 = (X + Y + Z) (X2 + Y2 + Z2 – X.Y –
3 3
and square root of 5
Y.Z – X.Z)
1+√ 5
1 i.e., Golden Ratio =
9. X2 + Y2 + Z2 – X.Y – Y.Z – X.Z = [(X – Y) 2 + 2
2
(Y – Z)2 + (Z – X) 2 ] Reverse Century : Now, using all the non-zero digits

ww
10. (X + Y + Z) 3 – X3 – Y3 – Z 3 = 3 (X + Y) (Y + Z) (Z
+ X)
i.e., 1, 2, 3, 4, 5, 6, 7, and 9 in reverse sequence, place
plus and minus signs between them that the result of the
arithmetic operation will be 100.

w
11. (X 2 + XY + Y2 ) (X2 – XY + Y2) = X4 + X2 .Y2 + Y4

.Ea
Perfect Number : If the sum of the divisions of N
excluding N itself is equal to N1, then N is called a perfect
number.
(i) 98 – 76 + 54 + 3 + 21 = 100
(ii) 98 + 7 + 6 – 5 – 4 – 3 + 2 – 1 = 100
(iii) 9 – 8 + 76 + 54 – 32 + 1 = 100
(iv) 98 – 7 + 6 + 5 – 4 + 3 – 2 + 1 = 100
Example : 6, 28, 496, 8128
syE
6 = 1 + 2 + 3 where 1, 2, 3 are the divisors of 6.
28 = 1 + 2 + 4 + 7 + 14, where 1, 2, 4, 7, 14 are
Similarly, we can do and get 100.
BODMAS : When we have to perform a series of

ngi
mathematical operations, there is a rule regarding the
divisors of 28. order in which we should perform these operation. This
The sum of the reciprocals of the divisors of a perfect rule is BODMAS rule.
→
numbers including that of its own is always = 2.
Example : For the perfect number 28; + + +
1 1 1 1
1 2 4 7 nee B = Bracket ( ), { }, [ ]
O = of Order of

+ +
1
14 28
1
=2
D = Division
M = Multiplication
rin performing the
operations
Every even perfect number is of the form 2n - 1 (2n – 1)
where 2n – 1 is a prime number.
A = Addition
S = Subtraction
g.n
Example : N = 2, 3, 5, 7, 11, 13, 17, 19, …………
Largest prime number 2132048 (2132049 – 1) which
consists of 39,751 digits.
Fibonacci Number : Form a sequence {an } where
1. If X =
2+
1
1
e
Exercise

t
an + 2 = an + 1 + an 1
3+
where a1 = 1, a2 = 1 1
2+
3 + ………∞
e.g. , 1, 1, 2, 3, 5, 8, 13, 21, ………
What the value of X ?
nth Fibonacci number Fn is given by
  n n – 6 ±√
15 – 3 ±√
15

√   
√   (A) (B)
Fn =
1  1 + 5
 +  1 – 5   6 4
 5   2   2  
√ – 3 ±√
15 – 6 ±√
15
(C) (D)
Given any Fibonacci number greater than 1, you can 2 4
calculate the next Fibonacci number. (E) 1
Call the given number A, the next Fibonacci number
is 2. The smallest number which, when divided by 4, 6 or
7 leave a remainder of 2 is ?
A+1+A √ 5
(A) 84 (B) 86
2
where bracket indicate rounding down to the nearest (C) 68 (D) 88
integer if A = 13 the next Fibonacci number is 21.54 ≈ 21 (E) 48

292 | CAT Complete Course

Visit : www.EasyEngineering.net
Visit : www.EasyEngineering.net

3. Three bells chime at intervals of 18 min., 24 min. 12. 3 – [2 – {7 – (6 – 3 – 2)}]


and 32 min. respectively. At a certain time, they (A) 1 (B) 3
begin together. What length of time will elapse (C) 5 (D) 7
before they chime together again ?
(E) 9
(A) 4 hr. and 24 min. (B) 2 hr. and 24 min.
 
(C) 4 hr. and 48 min. (D) 2 hr. and 48 min. 13. Simplify 0·63 + 0·37
(E) None of these (A) 1.00 (B) 1.05
4. A number when divided by the sum of 555 and 445 (C) 1.01 (D) 1.10
gives two times their difference as quotient and 30 as (E) 1.21
the remainder. Find the number?
14. What is the smallest number which must be added to
(A) 220040 (B) 220080 1953701 to obtain such which is
(C) 220015 (D) 220030 (i) Divisible by 3 or multiple of 3.
(E) 220055 (ii) Divisible by 11 or multiple of 11.
5. A number when divided by sum of 55 and 45 gives (A) 1; 6 (B) 2; 7

ww
two times their difference as quotient and 11 as the
remainder. Find the number?
(A) 2011 (B) 2022
(C) 2; 8
(E) 1; 2
(D) 1; 9

w
15. What is the minimum value of the expression (X2 +
(C) 2033 (D) 2044
X + 1) (Y2 + Y + 1) (Z2 + Z + 1), where X, Y, Z are
(E) 2055

.Ea
6. The largest number which exactly divides 522, 1276
all positive integer ?
(A) 1 (B) 8
and 1624 is ?
(A) 116
(C) 29 syE
(B) 232
(D) 58
(C) 27
(E) Cannot be determine
(D) Data insufficient

ngi
16. If a, b, c are real numbers such that a + b + c = 5 and
(E) 64 ab + b c + a c = 3, then which of the following
7. Find the unit’s digit in the product 152 × 169 × 171 × equations best describes the largest value of a ?
144 ?
(A) 2 (B) 9 nee
(A)

(C) 13
13
3
(B)

(D) 1
1
3
(C) 1
(E) 3
(D) 4
(E) 3
rin
8. Find the unit’s digit in the product (36 × 641 × 759) ?
(A) 3 (B) 6
17. We have

g.n
A (a, b) = a + b
B (a, b) = a × b
(C) 7
(E) 1
(D) 2

9. Find the total number of factors 512 ?


e
C (a, b) = a – b
For convenience A (a, b) is represented as A and so
on …………, Now t
(A) 10 (B) 11 Which of the following is equal to a ?
(C) 8 (D) 9 1

(E) 2 (A) [ A3 + C3 + 3B (C – A)
2 ] 3

10. A number when divided by the sum of 255 and 345 (B) [A 3 + C3 + 3B (C – A)] 1/3
gives two times their difference as quotient and 20 as 1
the remainder. Find the number ?
(A) 108060 (B) 108020
(C) [ A3 + C3 + 3B (C + A)
2 ] 3

(C) 108000 (D) 106020 (D) 1


(E) 106080 (E) None of these

11. [ 1 4
of {(2 × 3) + (4 × 5)} +
4 3
1
3 ] 18. If a2 + b2 = 1 the value of 2 (a6 + b6) – 3 (a4 + b4) +
1?
(A) 7 (B) 9 (A) 0 (B) 1
(C) 11 (D) 13 (C) – 1 (D) 2
(E) 15 (E) – 2

CAT Complete Course | 293

Visit : www.EasyEngineering.net
Visit : www.EasyEngineering.net

19. A number (22222222) 11 is written in a number 26. In a division sum, the divisor is 16 times the quotient
system which uses 11 as its base. What is the and five times the remainder if the remainder is 16.
remainder when this number is divided by number 11 Determine the dividend?
in that number system? (A) 256 (B) 5
(A) 0 (B) 1 (C) 16 (D) 80
(C) 2 (D) – 1 (E) 416
(E) – 2
Solution
20. If a number 774958A96B is to be divisible by 8 and
9. The respective value of A and B will be? 1
1. (A) X =
(A) 8; 0 (B) 1; 4 1
2+
3+X
(C) 2; 8 (D) 8; 4
3+X
(E) 0; 2 Or, =
6 + 2X + 1
21. If the unit digit in the product ( 459 × 46 × 28 * × Or, 6X + 2X2 + X = 3 + X
594 ) is 2. Find the digit in place of *? Or, 2X2 + 6X – 3 = 0

ww
(i) 2 only
(iii) 3 only
(ii) 7 only
(iv) 2 & 7 both
Or, X =
– 6 ±√
62 + 4 × 3 × 2
2×2
 ∴ If equation is ax2 + bx + c = 0‚ then 

w
(v) 3 & 7 both
(A) (i) & (v)
(C) (ii) & (vi)
(vi) 2 & 3 both

.Ea
(B) (iv) only
(D) (v) only


 x =
– b ±√

– 6 ± 2√

b2 – 4ac
2a
15 = – 3 ± √15




(E) (i), (ii) & (iii)

syE
22. Find the unit digit in the expression (256252 + 36529 +
7352) ?
Or, X =
2. (B) L.C.M. of 4, 6 or 7 is
4, 6, 7 2
2×2 2

(A) 1
(C) 3
(B) 2
(D) 4 ngi 2, 3, 7
L.C.M. = 2 × 3 × 7 × 2 = 84
(E) 5
23. Find the unit digit in the expression 71 + 82 + 93 + nee Required number = 84 + 2 = 86
3. (C) First of all we find the L.C.M. of 18, 24, 32
104 + 11 5 ?
(A) 2 (B) 3 rin
18, 24, 32 2
9, 12, 16 2

(C) 1
(E) 5
(D) 4
g.n
9, 16, 18 3
3, 12, 18 2

24. Find the value of

+
1
?
1
+
1
+
1
(22 – 1) (42 – 1) (62 – 1)
+…
3, 11, 14

e
L.C.M. = 2 × 2 × 3 × 2 × 4 × 1 × 3 = 288
The bells will chime together again after 288 min.
= 4 hr. and 48 min.
t
(202
– 1)
4. (D) According to question :
2 20
(A) (B) Divisor = 555 + 445 = 1000
21 21
Divident = ?
1 3
(C) (D) Quotient = (555 – 445) × 2
2 21
= 110 × 2 = 220
6 Remainder = 30
(E)
7 Divident = (Divisor × Quotient) + Remainder
25. Two different numbers when divided by a certain = (1000 × 220 ) + 30
divisors leave remainder 47 and 59 respectively. = 220000 + 30
When the sum of the two numbers is divided by the = 220030
same divisor remainder is 19. Find the divisor? 5. (A) According to question :
(A) 2 (B) 19 Divisor = 55 + 45 = 100
(C) 59 (D) 87 Quotient = (55 – 45) × 2 = 20
(E) 47 Remainder = 11

294 | CAT Complete Course

Visit : www.EasyEngineering.net
Visit : www.EasyEngineering.net

Divident = (Divisor × Quotient)


+ Remainder
11. (B) [14 of 43 {6 + 20} + 31]
= 20 × 100 + 11
= [ of × 26 + ]
1 4 1
= 2000 + 11 4 3 3

= [ × 26 + ]
= 2011 1 1
6. (D) 522 ) 1276 ( 2 58 ) 1624 ( 28 3 3
1044 116 26 1
= +
232 ) 522 ( 2 464 3 3
464 464 27
= =9
58 ) 232 ( 4 × 3
232 12. (B) 3 – [2 – {7 – (6 – 1)}]
× = 3 – [2 – {7 – 5}]
7. (A) Product of any digits in given numbers = 3 – [2 – 2]
= 2 ×9×1×4 = 3–0
= 72 = 3

8. ww
∴ Unit digit in the given product = 2
(B) We know the unit’s digit in 34 = 1
So, unit’s digit of 36 = 1 × 32 = 9
13. (C) Let

X = 0·63
100 X = 63.63636363 …(1)

w Unit’s digit of 6n = 6

.Ea
So, unit’s digit of 641 = 6
Unit’s digit of 74 is 1
X = 0.63636363 …(2)
Subtracting equation (2) from equation (1), we get
99.X = 63

syE
Unit’s digit of (74)14 = 1 63
∴ X =
Now, unit’s digit of 759 = 1 × 73 = 3 99
Now, product of unit’s digit in the given numbers  37
Similarly, 0·37 =
= 36 × 641 × 759
= 9×6×3
= 54 × 3 ngi 
∴ 0·63 + 0·37 =

99
63 37
+
99 99
= 162
So, required unit’s digit = 2 nee =
100
99
=1+
1
99
9. (A) Factorization of 512 is given by
2 512
rin
= 1 + 0.01
= 1.01

g.n
2 256 14. (D) Since, divisible by a and multiple of a are
2 128 equivalent expression i.e., if a number is divisible by
2 64 a then that number is a multiple of a.
2 32
2 16
2 8
2 4
the number.
(i) For divisibility by 3,
e t
We can also say that X is a factor or sub-multiple of

2 Sum of digits must be divisible by 3.


∴ 512 = 29 So, 1 + 9 + 5 + 3 + 7 + 0 + 1 = 26
Total number of factors = 9 + 1 = 10 Since, we have to added smallest number
10. (B) According to question : So, required number = 1
Divisor = 255 + 345 = 600 (ii) For divisibility by 11, the difference of sums of
We have to find dividend digits at odd and even places must be either zero
Now, Quotient = (345 – 255) × 2 or multiple of 11.
= 90 × 2 So, Difference = (1 + 5 + 7 + 1) – (9 + 3 + 0)
= 180 = 2
Since, Remainder = 20 Since, Here unit’s place is at even place.
So, Dividend = Divisor × Quotient + Remainder So, we have 11 – 2 = 9
= 600 × 180 + 20 and we add 9 to the number.
= 108000 + 20 1953701 + 9 = 1953710
Number = 108020 Clearly, 1953710 is divided by 11.

CAT Complete Course | 295

Visit : www.EasyEngineering.net
Visit : www.EasyEngineering.net

15. (C) Given Question is-


(X 2 + X + 1) (Y2 + Y + 1) (Z2 + Z + 1) 19. (A) The number (22222222)11 can be written as
1 2 3 1 2 3
= [( ) ] [( ) ]
X+
2
+
4
× Y+
2
+
4
(2 × 110 + 2 × 111 + 2 × 112 + ……… + 2 × 117)
in decimal system
1 2 3
[( ) ] × Z+
2
+
4
= 2 (110 + 11 1 + 11 2 + 11 3 + ……… + 117)

= minimum values of X, Y, Z are 1, 1, 1


= 2×1 [ ]
118 – 1
11 – 1
respectively. 8
Now, (11 – 1) is divisible by (11 – 1) and (11 + 1)
So, value = 3 × 3 × 3 = 27 i.e., 10 and 12 both, thus the number is divisible by
16. (A) We know that 12 in the decimal system which is written as 11 in
the system with base 11.
(a + b + c)2 = a2 + b2 + c2 + 2 (ab + bc + ca)
∴ Ans. 0.
Or, 52 = a2 + b2 + c2 + 2 × 3
20. (A) 774958A96B is divisible by 8 if 96B is divisible
Or, a + b + c2 = 25 – 6 = 19
2 2
by 8 and 96B is divisible by 8 if B is either 0 or 8.
For the maximum value of a, the value of (b2 + c2)
Now, to be become the same.
must be minimum.
Divisible by 9 sum of all the digits should be divisible
Now, b2 + c2 = (b + c)2 – 2bc

ww
Hence, b must be maximum for which b = c
Now, a + b + c = 5
by 9.
If (A + B) is 8.
⇒ more either A = 0 or B = 8, or A = 8 or B = 0

w a + 2b = 5
b =
ab + bc + ac = 3
5–a

.Ea
2
…(1)
Since, the number is divisible by both A and B,
hence A and B may take either values i.e., 8 or 0.
21. (B) Unit digit in the product of 9 × 6 × 4 = 6
Now, to get a digit 2 in the unit place 6 should be
Or, ab + b2 + ab = 3
Or, b2 + 2ab = 3
syE
From equation (1) and equation (2), we get
…(2)
multiplied by either 2 or 7. So, Answer is 2 or 7.
22. (B) We know that unit digit in the product of any
number with 5 at unit place is always 5. 5 n = 5
5–a 2
( ) 2
+ 2ac ×
5–a
2
= 3
ngi So, unit place of (25)6252 = 5
Similarly, unit digit in the product of any number in

nee
Or, 25 – 10a + a2 + 4 (5a – a2 ) = 12 the product of any number with 6 at unit place is
always. 6n = 6
Or, – 3a2 + 10a + 25 = 12
2 Unit digit in (36)529 = 6
Or, 3a – 13a + 3a – 13 = 0
a (3a – 13) + 1 (3a – 13) = 0
Or, (a + 1) (3a – 13) = 0
and

rin
Unit digit in 34 = 1
So, unit digit in [(73)4 ]13 = 1

Or, a = – 1,
13
3 g.n
Now, Unit digit for the expression
(25) 6252 + (36)529 + (73)52 = 5 + 6 + 1 = 12
17. (A) A3 = (a + b)3 = a3 + b3 + 3ab (a + b)
A3 = a3 + b3 + 2BA
C 3 = (a – b) 3 = a3 – b3 – 3ab (a – b)
= a3 – b3 – 3 BC
23. (E)
e
Required unit digit = 2
Unit digit of 115 = 1
Unit digit of 104 = 0
Unit digit of 93 = 9
t
A + C = 2a3 + 3B (A – C)
3 3 Unit digit of 82 = 4
2a3 = A3 + C3 – 3B (A – C) and Unit digit of 71 = 7
Now, Unit digit in the expression
a3 = [ A3 + C3 + 3B (C – A)
2 ] 71 + 8 2 + 9 3 + 10 4 + 11 5 = 7 + 4 + 9 + 0 + 1 = 21
1 Required unit digit = 1
a = [ A3 + C3 + 3B (C – A) 3
2 ] 24. (B) We have Tn 2
(n
1
– 1)
=
(n –
1
1)(n + 1)
18. (A) 2 (a6 + b6 ) – 3 (a4 + b4 ) + 1
(n + 1) – (n – 1)
= 2 [(a2 )3 + (b2 )3] – 3 [(a2)2 + (b2 )2] + 1 =
(n – 1)(n + 1)
= 2 [(a2 + b2 )3 – 3a2 b2 (a2 + b2 )] – 3 [(a2 + b2 )2 1 1
– 2a2 b2 ] + 1 Tn = –
(n – 1) (n + 1)
= 2 [1 – 3a2b2 ] – 3 (1 – 2a2 b2 ) + 1 where n is even number.
= 2 – 6a2 b2 – 3 + 6a2 b2 + 1 1 1 1
Now, we can write T1 = 2 = –
= 0 (2 – 1) 1 3

296 | CAT Complete Course

Visit : www.EasyEngineering.net
Visit : www.EasyEngineering.net

1 1 1 ∴ X – 47 is exactly divisible by the divisor z.


and T2 = = –
42 – 1 3 5 Y – 59 is completely divided by z and
1 1 1
Similarly, T3 = 2 = – X + Y – 19 is completely divided by z.
(6 – 1) 5 7
1 1 1 Now, X + Y – 47 – 59 = X + Y –106 must be
T4 = 2 = – divided by the same divisor.
(8 – 1) 7 9
…………………… So, Divisor = (X + Y – 19) – (X + Y – 106)
…………………… = 87
1 1 1
T20 = = – 26. (E) Let a be the quotient and b be the remainder.
(202 – 1) 19 21

ΣT1–20 = (22 1– 1) + 42 1– 1 Divisor = 16a = 5b
5 5
1 1 a = × b= × 16 = 5
+ 2 +…+ 16 16
(6 – 1) (202 – 1) Divisor = 16a = 16 × 5 = 80
1 20
= 1– = Dividend = Divisor × Quotient + Remainder
21 21
= 80 × 5 + 16

ww
25. (D) Let the two numbers be X and Y.
According to question—
Let certain divisor = z
= 400 + 16
= 416

w .Ea
●●●

syE
ngi
nee
rin
g.n
e t

CAT Complete Course | 297

Visit : www.EasyEngineering.net
Visit : www.EasyEngineering.net

2 Set Theory
Sets : A set is collection of well defined objects or Types of Sets
elements. By well defined, means that the object follow a
(1) Finite Set : Finite set means number of element
given definition or rule.
is fixed or constant.
For example : The collection of short students in a
Example : Set of vowels of English alphabets {a, e,
class is not defined – hence is not a set. i, o, u}.
But collection of students whose height is less than 5 (2) Infinite Set : The number of elements is infinite.
feet is a set. Example : Set of natural numbers {1, 2, 3, 4, 5……

ww
Between the years 1874 and 1897, the German
mathematician and logician Georg Cantor created a
theory of abstract sets of entities and made it into a
10,000 ……}.
(3) Empty Set or Null Set : The set which contains

w
no elements at all, is called an empty set or Null set. The
mathematical discipline. This theory grew out of his empty set is written as { } or φ.

.Ea
investigations of certain concrete problems regarding
certain types of infinite sets of real numbers. A set, wrote
Example : Set of even prime number greater than 5
is null set.

syE
Cantor, is a collection of definite, distinguishable objects
of perception or thought conceived as a whole. The
objects are called elements or members of the set.
(4) Singleton Set : A set containing only one element
is a singleton set.
Example : Even number between 2 and 6 is 4. {4}.
Fundamental Set Concepts
ngi
If the elements and sets to be considered are restricted
(5) Equal Sets : Two sets are said to be equal if every
elements of one set is in the other set and vice-versa. So,
two elements A and B are equal if x ∈ A ⇒ x ∈ B
to some fixed class of objects, such as the letters of the
alphabet, the universal set (or the universe), which is nee ⇒ A = B
(6) Equivalent Sets : Two sets A and B are equiva-

rin
commonly denoted by U, can then be defined as that lent if the elements of A can be paired with elements of
which includes all of the elements—in this case, the set of B, so that to each element of A there corresponds exactly

g.n
all of the 26 letters. Thus, if A is one of the sets being one element of B.
considered, it will be understood that A is a subset of U. In other words number of elements in both the sets
Another set may now be defined that includes all of the are equal.
elements of U that are not elements of A. This set, which
is called the complement of A, is denoted by A´. (Some
writers, employing the convention of “difference sets,”
speak of “the complement of A with respect to U,” which
Example :
e
A = {a, b, c},
B = {1, 3, 5}
Then, A and B are equivalent. t
(7) Subsets : Let A = {1, 3, 5 },
they denote by “U - A.”)
B = {1, 3, 5, 7, 9, 11}
A set can be represented in two forms :
Since, each elements of set A is present in the set B.
(a) Tabular Form : A set is described by listing all So, A is subset of B.
its elements enclosed in curly brackets the elements are Clearly, A set X is said to be subset of Y if and only
separated by commas and each element is written only if each element of X is an element of set Y.
once. Clearly, X is subset of Y. And Y is super set of X.
Example : A set of even natural numbers less than
It has some important properties :
10 is represented by {2, 4, 6, 8}.
(a) Every set is a subset of itself.
(b) Set Builder Form : A set is defined by specifying
the property which determines the elements of the set (b) A set having n elements has 2 n subsets.
uniquely. (c) The empty set is a subset of every set φ ⊂ A, φ ⊂ φ.
Example : Set of odd natural numbers more than 1 is (d) If A ⊆ B and B ⊆ C then A ⊆ C.
represented by (8) Universal Sets (U) : A set which contains all the
{x : x = 2n + 1} where n ≥ 1 n ∈ N set in a given context is a universal sets.

298 | CAT Complete Course

Visit : www.EasyEngineering.net
Visit : www.EasyEngineering.net

Example : Set of natural number is a universal sets. Example : A = {1, 3, 5, 4},


Note : Universal set contains more than one subsets. B = {1, 3, 5, 8, 9, 10, 12, 13}
SOME OPERATION APPLIED ON SETS Now, A ∩ B = {1, 3, 5 }
(6) Disjoint Sets : Two sets are said to be disjoint
(1) Difference of Sets : If we have two sets X and Y
sets if they have no common elements. That is A ∩ B = φ
then the set of all elements which belong to A but do not
belong to B is called the difference of sets A and B Example : A = {1, 3, 5, 9, 11 },
denoted by A – B. B = {2, 4, 6, 8, 10 }
Example : A = { 1, 3, 5, 7,4 }, Now, A ∩B = φ
B = {1, 3, 5, 6,7,8, 9,10} Some Important Formula :
Clearly, A – B = {4} If A, B, C are three infinite sets, then
Some Important Points on Difference : (i) n (A ∪ B) = n(A) + n(B) – n(A ∩ B)
(i) A – B ≠ B – A (ii) n (A ∪ B) = n(A) + n(B)
(ii) If A ⊂ B, A – B = φ if A and B are disjoint.
(iii) (A – B) ∪ B = A ∪ B (iii) n(A – B) = n(A) – n(A ∩ B)
(iv) A – B = A ∩ B1 (iv) n(A ∪ B ∪ C ) = n(A) + n(B) + n(C)

ww
(v) A – B ⊆ A
(vi) The sets (A – B), (A ∩ B ) and (B – A) are
– n(A ∩ B) – n(B ∩ C)
– n(A ∩ C) + n(A ∩ B ∩ C)

w
mutually disjoint.
(v) n(A 1 ∪ B1) = n{ (A ∪ B)1 }
(2) Cardinal Number : The number of elements in a

.Ea
set is called the cardinal number of the set. The cardinal
number is represented as n(A). Some Important Laws :
= n(∪) – n (A ∩ B)

If
n(A) = 5
(3) Complement of a Set :syE
A = {a, b, c, d, e} (1) Commutative Laws : If A and B are two sets,
then
A∪B = B∪A
Let A = {1, 3, 5 },
B = {1, 3, 5, 7, 9, 10, 11, 12}
ngi and A∩B = B∩A
(2) Associative Laws : If A, B and C are three sets,
Now, complement of set A = {7, 9, 10, 11, 12 }
where B is universal set and A ⊆ B.
Complement of a set is determined relative to other nee then

and
(A ∪ B) ∪ C = A ∪ (B ∪ C)
A ∩ (B ∪ C) = (A ∩ B) ∪ (A ∩ C)
when Ist set is subset of 2nd set.
Some Important Points : rin
(3) Distributive Laws : If A, B and C are three
sets, then
(i) Complement of the universal set is the null set φ
and vice-versa.
g.n
A ∪ (B ∩ C ) = (A ∪ B) ∩ (A ∪ C)
A ∩ (B ∪ C) = (A ∩ B) ∪ (A ∩ C)
(ii) (A1)1 = A
(iii) If A ⊆ B, then B1 ⊆ A1
A1 represent the complement of set A.
(4) Union of A set : The union of two sets A and B
(4) Identity Laws : A ∪ φ
e t
= A where φ is null set.
A ∩ U = A U is universal.
(5) De-Margan’s Laws : If A and B are two sets
is a set which contains all the elements of A and the
elements of B. (A ∪ B)1 = A1 ∩ B1
Symbol ∪ denotes union (A ∩ B)1 = A1 ∪ B1
A ∪ B = {x/x ∈ A or x ∈ B} The notion of set belongs to primary mathematical
Example : A = {1, 3, 5}, concepts. Here, are some examples of sets: the sets of
tables in a classroom, the set of residential houses in a
B = {2, 4, 6}
certain town, the set of whole numbers, the set of all
A ∪ B = {1, 2, 3, 4, 5, 6} triangles which can be inscribed in a given circle, and so
Some Points : on.
(i) A∪B = B∪A Objects contained in a set are elements of this set.
(ii) A ∪ A1 = Universal set For instance, the set of natural numbers less than 7
(iii) A ∪φ = A consists of the following elements : 1, 2, 3, 4, 5 and 6.
(5) Intersection of Sets : The intersection of the two We distinguish between finite and infinite sets. The
sets A and B is the set of common elements of A and B. set of table in a classroom and the set of residential
The Symbol ∩ denotes intersection. houses in a town are finite: the tables and houses can be

CAT Complete Course | 299

Visit : www.EasyEngineering.net
Visit : www.EasyEngineering.net

counted; there is a definite number of them, whereas the The union of two sets is the set of elements that are
set of whole numbers and the set of all triangles inscribed members of at least one of the given sets. If C is the union
in a given circle are infinite. of the sets A and B then we write C = A ∪ B (∪ being
We shall denote sets by capital Latin letters and the the union symbol).
elements belonging to them by lower- case letters. For instance, the union of the set of even natural
For instance, the set notation A = {a, b, c, d} means numbers and the set of odd natural numbers is the set of
that the set A consists of the elements a, b, c and d. all natural numbers.
If the elements x is an element of the set E, then we If the set B is a subset of the set A, then the union of
use the notions x ∈E (read : “x belongs to the set E”) , the sets A and B is the set A, i.e., if B ⊂ A, then A ∪ B =
where ∈is the membership symbol. And if x is not a A.
member of the set E, then we write x ∉ E (read : “x does For instance, if A = {a, b, c, d, e} and B = {a, b, c,
not belong to the set E”). d} then B ⊂ A, therefore A ∪ B = A.
For example : If N is the set of natural numbers, For sake of obviousness, we shall regard sets as
1
then 1 ∈ N, 2 ∈N, ∉N, – 2 ∉N , and π ∉ N. certain sets of points in the plane. Figure 1 shows an
3 intersection and a union of two sets.
A set that contains no elements is called the empty

ww
set (or null set) and is denoted by φ.
Here, are some example of empty sets : the set of

w
natural roots of the equation x + 1 = – 2, the set of real
solutions of the inequality x2 + 1 < 0, the set of common

.Ea
points of two distinct parallel straight lines, and so on.
Sometimes, we have to consider not the entire set but Fig. 1

syE
only its part. For instance, we consider not the entire set
of natural numbers but only the set of prime numbers.
Instead of the words “a part of a set” we often say “a
In arithmetic and algebra use is made of number sets,
that is, of sets whose elements are numbers.
First the natural numbers1, 2, 3, 4,…. are considered.

ngi
subset”.
The addition and multiplication of natural numbers
A and B is a subset of a set A if the set B has the
always yield natural numbers. But the subtraction of two
property that each element of B is also an element of A.
In this case we write B ⊂ A (read : “B is included in A”).
This definition implies that any set is a subset of nee
natural numbers does not always yield a natural number.
Therefore, negative whole numbers (integers) and the
number zero were introduced, thus extending the set of
itself : A ⊂ A. And the empty set is regarded as a subset
of any set : φ ⊂ A.
rin
natural numbers to the set of whole numbers 0, ± 1, ± 2, ±
3, ± 4…….
If for two sets A and B the statements A ⊂ B and B
⊂ A hold simultaneously, then this means that these two
sets have precisely the same elements. Such sets are said g.n
The addition, multiplication and subtraction of whole
numbers always yield whole numbers. But the result of
division of one whole number by another is not always a
to be equal sets, and we write A = B.
For instance, {a, b, c} = {b, c, a} = {c, a, b} and so
on. Finding all the subsets of the set M = {a, b, c}; we
obtain {a}, {b}, {c}, {a, b} {a, c}, {b, c}, {a, b, c}, φ.
e
whole number, therefore fractions were introduced. There
appears the notion of a rational number i.e., of a number
a
of the form , where a and b are whole numbers, and b ≠
b
t
We can perform various operations with sets, 0. The set of whole numbers has thus extended to the set
intersection and union being the simplest of them. of rational numbers. The set of rational numbers is the
union of whole numbers and fractions. The four
The intersection of two sets is defined as the set arithmetic operations (except for division by zero)
which consists of the elements that both sets have in performed on rational numbers always yield rational
common, if C is the intersection of the sets A and B, then numbers. But when taking a square root of a rational
we write C = A ∩ B (∩ being the intersection symbol). number we do not always obtain a rational number.
For instance, the intersection of the set of even Therefore, new numbers are added to rational numbers
natural numbers and the set of prime numbers is the set which are called irrational.
consisting of one elements, i.e., of the number 2. Rational and irrational numbers from the set of real
If the set B is a subset of the set A, then the inter- numbers.
section of the sets A and B is the set B, i.e., if B ⊂ A, The modulus (or absolute value) of a real number a
then A ∩ B = B. is defined as the number a itself if a is positive; as zero if
For instance, if A = {a, b, c, d} and B = {ab, d}, then a = 0; as the number (– a) if a is negative. The modulus
B⊂ A, and therefore A ∩ B = B. of a real number of a is denoted by |a| thus,

300 | CAT Complete Course

Visit : www.EasyEngineering.net
Visit : www.EasyEngineering.net

A= {–– aa ifif aa ≥≥ 0‚0. }


For instance |4| = 4, |0| = 0, |– 6| = – (– 6) = 6
Fig. 3

Fig. 2
Fig. 4
Now, we proceed to the geometric representation of
Besides finite interval discussed above, we often deal
numbers. Let us take a straight line and a point O on it as
with infinite intervals. For instance [a, + ∞] is the set of
the initial point (the origin) for reckoning lengths. We
numbers x satisfying the inequalities x ≥ a, (a, + ∞) is the
also choose a scale that is unit length, and a positive
set of numbers x satisfying the inequality x > a and so on;
direction along the line (Fig. 2). Then every real number
(– ∞, + ∞) is the set of all real numbers.
x will be associated with a definite point M, the abscissa
of which is equal to x. Infinite intervals are respected by rays on the
numbers axis.
In analysis, numbers are depicted in this way (for
For example, [– 2, + ∞) is the set of numbers x
greater pictorialness) by points.
satisfying the condition x ≥ – 2; (– ∞, 5) is the set of

ww
A straight line with origin, scale, and positive
direction for reckoning lengths is called a number axis
(or a number line).
numbers x satisfying the condition x < 5. (Fig. 4)
Venn Diagram—Venn diagrams are the pictorial
representation of the inner-relationship among two or

w
As is known from geometry any line segment OM

.Ea
has a length expressed by a rational or irrational number.
Therefore, to every point M on the number axis there
corresponds a quite definite real number x, which is
more than two sets.
● The universal set is represented by a rectangle.
● The subset of the universal set is shown by

syE
circles.
positive if M lies on the right of O and negative if M lies (i) Universal set and one subset.
on the left of O. The modulus of the number x is equal to
the length of the line segment OM.
Conversely, to every real number x there corresponds
a definite point M which lies on the number axis at a ngi
distance equal to |x| from the point O and is found on the
right of O if x > O and on the left of O if x < 0. For x = 0
the point M coincides with the point O. Thus, one-to-one nee (ii) Universal set and two disjoint set.
correspondence has been established between real
numbers and points of the number axis.
rin
Consider the following number sets. If a < b, then the
set of real numbers x satisfying the inequalities a ≤ x ≤ b
is called a number interval (or simply interval) and is g.n
denoted by the symbol [a, b].
If a < b, then the set of real numbers x satisfying the
inequalities a < x < b is spoken of as an open (or non
e
(iii) Universal set and two intersecting sets :

t
closed) interval, its commonly used notation being (a, b).
If one of the end points is included into the interval
while the other is not, the resultant set is specified by the
inequalities a ≤ x < b (if the end point a is added to the Illustration 1. In a coaching institute, 80 students are
interval) or by a < x ≤ b (if the end point b is joint to the selected in a banking exam coaching, 50 students selected
in staff selection exam coaching and 40 students selected
interval). Respectively, the half- interval thus obtained
in both the examination coaching. How many students are
are denoted as [a, b) and (a, b].
there in the institute?
When there is no need in distinguishing whether or How many students are selected ?
not an end point is included into the interval in question (i) In banking exam coaching only.
we simply speak of an interval. (ii) In staff selection exam coaching only.
For instance, the closed interval [– 2, 1] is the set of Solution : The number of students in banking exam
numbers x satisfying the inequalities – 2 ≤ x ≤ 1, and the = n (B) = 80
half- interval (2, 5] is the set off number x satisfying the The number of students in staff selection exam.
inequalities 2 < x ≤ 5. (Fig 3) = n (S) = 50

CAT Complete Course | 301

Visit : www.EasyEngineering.net
Visit : www.EasyEngineering.net

Now, n (B ∩ S ) = 40
Total students in the institute
= n (B ∪ S)
= n (B) + n (S) – n (B ∩ S)
= 80 + 50 – 40 = 90
Now given, 100 = (20 + w) + (5 + w) + (5 + w)
+ (20 – w) + (10 – w) + (15 – w) + w
Or, w = 25
Illustration 4. In group of 1000 people, there are 750
people who can speak Hindi and 400 who can speak
(i) Number of students selected in banking exam. English. How many people can speak only Hindi ?
= n (B) – n (B ∩ S) (1) 600 (2) 500
= 80 – 40 = 40 (3) 650 (4) 700
(ii) Number of students selected in staff selection Solution : (1) Here n (H ∪ E)
exam.

ww = n (S) – n (B ∩ S)
= 50 – 40 = 10
= 1000, n(H) = 750, n(E) = 400
Using n (H ∪ E) = n(H) + n(E) – n (H ∩ E);
1000 = 750 + 400 – n (H ∩ E)

w
Illustration 2. 65% of students in a class like cartoon
movies, 75% like horror movies and 85% like war

.Ea
movies. What is the smallest percent of student liking all
the three types of movies?
⇒ n(H ∩ E) = 1150 – 1000 = 150.
Number of people who can speak Hindi only
= n(H) – n (H ∩ E)

syE
Solution : Total number of students
= X+Y+Z+a+b+c+w
= 100
= 750 – 150 = 600.
Short-cut : by Venn diagram.

ngi
Now, 750 – x + x + 400 – x = 1000.
From the Venn diagram it is clear that k will be
∴ x = 150. Only Hindi
minimum if a = b = c = 0

nee
= 750 – 150 = 600.
Illustration 5 & 6 : Read of the information given
and answer the question that follow—

rin
In a school with 727 students, 600 students offer
Mathematics and 173 students offer both Mathematics
∴ X + Y + Z + w = 100
Now, X + a + c + w = 65
…(1)
…(2)
two subjects.
g.n
and Physics. Each student is enrolled in at least one of the

e
5. How many students are enrolled in Physics ?
Y + b + c + w = 75 …(3)
Z + a + b + w = 85
Adding equations (2), (3) and (4), we get
X + Y + Z + 3w = 225
…(4)
(1) 250
(3) 280
(2) 300
(4) 290
t
Solution : (2) Let M be the set of students offering
Mathematics and P the set of students offering Physics.
2 × w + 100 = 225 We are given that :
w = 125/2 = 62·5 n(M∪ P) = 727,
Hence, at least 62.5% of students like all the three n(M) = 600, n (M ∩ P) = 173
type of movies.
Using n (M ∪ P) = n (M) + n(P) – n (M ∩ P)
Illustration 3. In a survey of 100 students it was
found that 50 used the college library, 40 had their own 727 = 600 + n (P) – 173
library and 30 borrowed books of these, 20 used both the ⇒ 727 = 427 + n (P)
college library and their own, 15 used their own library ⇒ n (P) = 727 – 427 = 300
and borrowed books and 10 used the college library books
⇒ Number of students enrolled in Physics
and 10 used the college library and borrowed books.
How many students used all the three sources of books ? = n(P) = 300.
Solution : Let college library, own library, borrow 6. The students who enrolled in only Physics are :
books are represented by A, B and C respectively. (1) 300 (2) 280
So, Let w = All the three sources (3) 127 (4) 173

302 | CAT Complete Course

Visit : www.EasyEngineering.net
Visit : www.EasyEngineering.net

Solution : (3) Number of students enrolled in Physics Complaints about M alone


only = 110 – 20 – X – 11 = 79 – X
= n(P) – n (P ∩ M) Complaints about F alone
= 300 – 173 = 127. = 55 – 20 – X – 16 = 19 – X
Illustration 7. In a school, 21 students are on the Complaints about S alone
basket ball team, 26 students are on the hockey team, 29 = 67 – 16 – X – 11 = 40 – X
students are on the football team. 14 students play hockey
Total numbers of complaints
and basketball, 15 students play hockey and football, 12
students play football and basketball & 8 students are on = 79 – X + 20 + 19 – X + 16 + X
all the three teams. How many members are there + 11 + 40 – X
altogether ? = 185 – 2X.
(1) 38 (2) 47 But the total number of complaints = 173.
(3) 51 (4) 43 185 – 2X = 173
Solution : (4) Let B, H, F denote the sets of members ⇒ – 2X = 173 – 185 = – 12
who are on the basketball team, hockey team and football ⇒ 2X = 12 ⇒ X = 6.
team respectively.

ww
Then, n(B) = 21, n (H) = 26,
n(F) = 29, n (H ∩ B) = 14,
There are 6 complaints about all the three.
Illustration 9. The total number of complaints about
two or more than two categories is :

w n (H ∩ F) = 15, n (F ∩ B) = 12
and n (B ∩ H ∩ F) = 8.

.Ea
We have to find n (B ∪ H ∪ F).
Using the result n (B ∪ H ∪ F) = n(B) + n(H) + n(F)
(1) 48
(3) 53
(2) 51
(4) 63
Solution : (3) Numbers of complaints about 2 or
more than two = 20 + 6 +16 + 11 = 53.

We get n (B ∩ H ∩ F) syE
– n (B ∩ H) – n (H ∩ F) – n (F ∩ B) + n (B ∩ H ∩ F). 79 – x + 20 + 19 – x + 11 + x + 16 + 40 – x
= 173.
= 21 + 26 + 29 – 14 – 15 – 12 + 8
= 84 – 41 = 43.
Short-cut : by Venn diagram. ngi ⇒

185 – 2x = 173.
x = 6.

nee
Illustration 10. In a group of 70 people, 37 like
Total numbers = 3 + 6 + 5 + 4 + 8 + 7 + 10 coffee, 52 like tea and each person likes at least one of
= 43. the two drinks. How many people like both coffee and
For Q. 8 & 9 : Read of the information given and
answer the question that follow—
tea ?
(1) 19
rin (2) 21
A survey regarding the complaints of students was
conducted. All complaints of students fell into three
(3) 17

g.n
(4) 15
Solution : (1) Let A be the set of people who like
coffee and B be the set of people who like tea. Then, n
categories :
Complaint about :
Mess (M), Food (F) and Service (S).
The total number of complaints received was 173
We know that e
(A ∪ B) = 70, n(A) = 37 and n(B) = 52.

n (A ∪ B) = n(A) + n(B) – n (A ∩ B) t
70 = 37 + 52 – n (A ∩ B)
and were as follows :
or (A ∩ B) = 37 + 52 – 70 = 89 – 70 = 19
n(M) = 110, n(F) = 55, n(S) = 67, ⇒ 19 people like both coffee and tea.
n (M ∩ F ∩ S") = 20, Illustration 11. In a group of 65, people, 40 like
n (M ∩ S ∩ F’) = 11, n (F ∩ S ∩ M') = 16. cricket and 10 like both cricket and tennis. Each one likes
Illustration 8. The number of complaints about all at least one of the two games. How many people like only
three categories is : tennis ?
(1) 35 (2) 25
(1) 8 (2) 5
(3) 30 (4) 20
(3) 4 (4) 6
Solution : (2) Let A be the set of people who like
Solution : (4) M ∩ F ∩ S’ means complaints about cricket and B be the set of people who like tennis. Then,
both M and F but not about S. n (A ∪ B) = 65, n(A) = 40 and n (A ∩ B) = 10.
M ∩ F ∩ S" = 20, M ∩ F ∩ S = ?, ⇒ n (A ∪ B) = n(A) + n(B) – n (A ∩ B)
F ∩ S ∩ M' = 16 ⇒ 65 = 40 + n(B) – 10
Let M ∩ S ∩ F = X, ⇒ n(B) = 65 – 40 + 10 = 35.

CAT Complete Course | 303

Visit : www.EasyEngineering.net
Visit : www.EasyEngineering.net

Number of people who like tennis (A) 43 (B) 76


= n (B) – n (A ∩ B) (C) 49 (D) None of these
= 35 – 10 = 25 7. The relation “congruence modulo m” is—
⇒ Number of people who like tennis only and not (A) Reflexive only (B) Transitive only
cricket = 25. (C) Symmetric only (D) An equivalence relation
Illustration 12. In a committee, 50 people speak 8. R is a relation over the set of real number and it is
French, 20 speak Spanish and 10 speak both Spanish and given by mn ≥ 0. Then R is—
French. How many people speak at least one of these two (A) Symmetric and transitive
languages ? (B) Reflexive and symmetric
(1) 55 (2) 50 (C) A partial order relation
(3) 70 (4) 60 (D) An equivalence relation
Sol. (4) Let A be the set of people who speak French 9. Let A = {1, 2, 3} and B = {a, b, c}. If f is a function
and B be the set of people who speak Spanish. Then n(A) from A to B and g is a one-one function from A to B
= 50, n(B) = 20. then the maximum number of definitions of—
n (A ∩ B) = 10 (A) f is 9 (B) g is 9

ww
⇒ n (A ∪ B) = n(A) + n(B) – n (A ∩ B)
= 50 + 20 – 10 = 70 – 10 = 60.
Hence, 60 people speak at least one of these two
(C) f is 27 (D) g is 6
10. Let r be a relation over the set NxN and it is defined
by (a, b) r(c, d) ⇒ a + b = b + c. Then r is—

w
languages.

.Ea
People speak at least one of these two languages
= 10 + 10 + 40 = 60.
(A) Reflexive only
(C) Transitive only
(B) Symmetric only
(D) An equivalence relation
11. Let A = {1,2,3}. The total number of distinct rela-

syE
Short-cut : by Venn diagram. tions that can be defined over A is—
(A) 29 (B) 6
Exercise (C) 8 (D) None of these
1. If A and B are two sets then (A – B) ∪ (B – A) ∪ (A
∩ B) is equal to—
(A) A ∪ B (B) A B ngi 12. Let R = set of real numbers and Rc = set of real
angles in radian measure. If f: Rc → R be a mapping
such that f(x) = sin x, x ∈ Rc, then f is—
(C) A (D) B’
2. Let ∪ be the universal set and A ∪ B ∪ C = ∪. Then
{(A – B) ∪ (B – C) ∪ (C – A)}’ is equal to—
nee (A) One-one and into
(B) One-one and onto

(A) A ∪ B ∪ C
(C) A ∩ B ∩ C
(B) A ∪ (B ∩ C)
(D) A ∩ (B ∪ C)
(C) Many-one and onto
(D) Many-one and into
rin
3. Let A and B be two sets, then (A ∪ B)’ ∪ (A’ ∩ B)
is equal to— is— g.n
13. Let f : R → R such that f(x) =
1
1 + x2
, x ∈ R. Then f

(A) A’
(C) B’

A ∩ B is equal to—
(B) A
(D) None of these
4. Let A and B be two sets such that A ∪ B = A. Then
(A) Injective
(C) Bijective e
(B) Surjective
(D) None of these

14. f : R x R → R such that f(x + iy) = + √



t
x2 + y2 . Then f
is—
(A) φ (B) B
(A) Many-one and into
(C) A (D) None of these
(B) One-one and onto
5. 20 teachers of a school either teach mathematics or (C) Many-one and onto
physics. 12 of them teach mathematics while 4 teach
both the subjects. Then the number of teachers (D) One-one and into
teaching physics only is— 15. Let A = {x : – 1 < x < 1} = B. If f : A → B be
(A) 12 (B) 8 bijective, then a possible definition of f(x) is—
(C) 16 (D) None of these (A) |x| (B) x |x|
6. Of the numbers of three athletic teams in a school 21 (C) sin πx (D) None of these
are in the cricket team, 26 are in the hockey team and 16. Let A = {1, 2, 3} and B = {a, b}. Which of the
29 are in the football team. Among them, 14 play following subsets of A x B is a mapping from A to
hockey and cricket, 15 play hockey and football, and B?
12 play football and cricket. Eight play all the three
games. The total number of members in the three (A) {(1, a), (3, b), (2, a), (2, b)}
athletic teams is— (B) {(1, b), (2, a), (3, a)}

304 | CAT Complete Course

Visit : www.EasyEngineering.net
Visit : www.EasyEngineering.net

(C) {(1, a), (2, b)} 6. (A) n(C) = 21, n(H) = 26, n(F) = 29,
(D) None of these n(H ∩ C) = 14, n(H ∩ F) = 15, n(F ∩ C)
Directions—From Question 17 to 21 one or more = 12, n(C ∩ H ∩ F) = 8.
option may be correct. n(C ∩ H ∩ F) = n(C) + n(H) + n(F) – n(H ∩ C)
17. Let R be the relation over the set of straight lines of a – n(H ∩ F) – n(F ∩ C) + n(C ∩ H ∩ F)
plane such that— = 21 + 26 + 29 – 14 – 15 – 12 + 8
l1 Rl2 ⇔ l1 ⊥ l2 . Then R is = 43
(A) Symmetric (B) Reflexive 7. (D) If R be the relation, xRy ⇔ x – y is divisible by m.
(C) Transitive (D) An equivalence relation xRx because x - x is divisible by m. So, R is reflexive.
18. Let R be the relation over the set of integers such that xRy ⇒ y Rx. So, R is symmetric.
mRn if and only if m is a multiple of n. Then R is— xRy and yRz
(A) Reflexive (B) Symmetric ⇒ x – y = k1m, y – z = k2 m
(C) Transitive (D) An equivalence relation
∴ x – z = (k1 + k2 )m. So, R is transitive.
19. Let A = {1,2,3,4} and R be a relation in A given by As R is reflexive, symmetric and transitive, it is an

ww
R = {(1,1),(2,2),(3,3),(4,4),(1,2),(2,1),(3,1),(1,3)}
Then R is—
(A) Reflexive (B) Symmetric
8.
equivalence relation.
(D) R is reflexive, symmetric and transitive. So, the
most appropriate option is (D).

w
(C) Transitive (D) An equivalence relation

.Ea
20. Let f : R → R be a mapping such that f(x) =
Then f is—
x2
1 + x2
·
9. (C, D) Every element of A can have image in B in 3
ways. So, the total number of ways in which 3
elements of A can have images in B = maximum
number of definitions of f = 3 × 3 × 3.
(A) Many – one
(C) Into syE
(B) One-one
(D) Onto
The number of ways of arranging 1, 2, 3 in places of
a, b, c is 3! = the maximum number of definitions of

Solutions
1. (A) Draw the Venn diagram. From the Venn diagram, ngi 10.
one-one function g.
(D) (a, b) r(a, b) because a + b = b + a . So, r is
reflexive.
(A – B) ∪ (B – A) ∪ (A ∩ B) = A ∪ B
2. (C) (A – B) ∪ (B – C) ∪ (C – A) is represented by the nee (a, b) r(c, d)
⇒ a+d = b+c
shaded portion in the figure. The unshaded portion is
A ∩ B ∩ C.

rin
c+b = d+a
⇒ (c, d) r(a, b)
So, r is symmetric.

g.n
(a, b) r(c, d) and (c, d) r(e, f)

e
a + d = b + c, c + f = d + e
Adding , a + b + c + f = b + c + d + e
⇒ a+f = b+e
⇒ (a, b) r(e, f).
t
∴ R is transitive
∴ {(A – B) ∪ (B – C) ∪ (C – A)}’ = A ∩ B ∩ C
11. (A) n(AxA) = n(A).n(A)
3. (A) (A ∪ B)’ ∪ (A’ ∩ B) = (A’ ∩ B’) ∪ (A’ ∩ B)
= 32 = 9.
= (A’ ∪ A') ∩ (A’ ∪ B) ∩ (B’ ∪ A') ∩ (B’ ∪ B)
So, the total number of subsets of AxA is 29 and a
= A’ ∩ {A’ ∪ (B ∩ B’)} ∩ U = A' ∩ (A' ∪ φ) ∩ U subset of AxA is a relation over the set A.
= A’ ∩ A’ ∩ U =A’ ∩ =A’ 12. (D) 13. (D) 14. (A)
4. (B) A ∪ B = A 15. (D) f(x) = |x| is many-one and into.
⇒ B⊆ A ⇒ A ∩ B = B f(x) = x |x| is one-one but into as f(x) will have only
5. (B) n(M ∪ P) = 20, n(M) = 12, n(M ∩ P) = 4 rational values.
n(M ∪ P) = n(M) + n(P) – n(M ∩ P) f(x) = sin πx is onto but many – one
⇒ 20 = 12 + n(P) – 4 ∴ 3
∴ n(P) = 12. [ ( ) ( )]
f
4
=f
1
4
So, the required number 16. (B) 17. (A) 18. (A, C) 19. (A, B) 20. (A, C)
= n(P) – n(M ∩ P) = 12 – 4 = 8. ●●●

CAT Complete Course | 305

Visit : www.EasyEngineering.net
Visit : www.EasyEngineering.net

3 Logarithms
When studying logarithms it is important to note that of the number N to the base a, then by definition, the
all the properties of logarithms are consequences of the following equations would hold true :
corresponding properties of powers, which means that the aα = N and aβ = N
aspirant should have a good working knowledge of power whence a α = aβ. But then, by the properties of
as a foundation for tackling logarithms. This close powers with positive base different from unity, we should
relationship between logarithms and powers stems from arrive at the equation α = β. Thus, if the number N has a
the definition of a logarithm in terms of the concept of a logarithms to a base a, then this logarithm is unique; we
power. denote it by the symbol loga N.

ww
Here is a definition taken from a commonly used
textbook : “The logarithm of a given number to a given
Thus, by definition,
X = log a N if ax = N

w
base is the exponent of the power to which the base must
be raised in order to obtain the given number”. Thus, a

if ax = N. .Ea
number x is the logarithm of a number ‘N’ to the base ‘a’
Consequently, the equation x = loga N and a x = N
(provided the restriction imposed earlier on hold true)
express one and same relationship between the numbers
x, a, N: in logarithmic form in the former case and in

syE
There is one very essential detail in this definition:
no restrictions are imposed on the phrases “to a given
base”, and so if we are to follow this definition literally
equivalent exponential form in the latter.
It is easy to prove that negative numbers and zero do
not have logarithms to any base a (with the usual
(and a definition must always be followed literally), then
we will have to concede that 3 is the logarithms of – 8 to
the base-2 (since (–2)3 = 8), 2 is the logarithms of – 4 tongi provision that a > 0 and a ≠ 1). Indeed, if N ≤ 0 and x =
loga N, then ax = N ≤ 0, which contradicts the property of
powers having a positive base.
the base-2 (since (– 2)2 = 4), and so forth. As for the base
1, the situation is stranger still : any number x is the
logarithm of 1 to the base 1 because 1 x =1 for every x.
nee As for positive numbers, we assume without proof
that any positive number to any base has a logarithm.
This assertion is taken in school to be self- evident and is
Any person acquainted with the school course of
rin
not even stated. Although, it is no easy job to establish its
validity (this would require invoking a highly developed

g.n
mathematics will say that these examples are meaningless
theory of real numbers and the theory of limits).
since we have to consider only logarithms to a positive
Quite naturally, the aspirant must have a through
base different from 1. True enough, that is the convention,
but it is much better to impose this restriction on the base
directly in the definition. And so the definition should
read :
Let there be a number a > 0 and a ≠ 1. A number x is
e
knowledge of the definition and of the properties of
logarithms and must be able to prove them.
First of all, note the so-called fundamental loga-
rithmic identity t
aloga N = N
called the logarithm of a number N to the base a if ax = N.
which is valid for every N and a such that a > 0, a ≠
The more attentive readers have perhaps noticed that 1, N > 0. This identity follows directly from the equation.
we have not once written x = loga N but have always Here are some formulas that are frequently used in
stated : x is the logarithms of N to the base a. The problem solving (we stress once aging that, according to
explanation is very simple. Until we are sure that no the definition of a logarithm, all bases are positive and
number can have two distinct logarithms to a given base, different from unity).
we have no right to use the equal sign. Indeed, imagine I. loga MN = loga M + loga N (M > 0, N > 0)
for a moment that some number N has two distinct M
logarithms to the same base a : then, using the equals II. loga = loga M – loga N (M > 0, N > 0)
N
sign, we should be able to write α = loga N and β = loga III. loga Nα = α loga N (N > 0, α any number)
N, whence α = β. α
IV. loga β Nα = loga N (N > 0, α ≠ 0, β ≠ 0)
For this reason we will introduce a notation for β
logarithms only when we are convinced that no number loga N
V. logb N = (N > 0)
can have two distinct logarithms to the same base. loga b
Indeed, If two distinct numbers α and β were logarithms VI. logb a.loga b = 1.

306 | CAT Complete Course

Visit : www.EasyEngineering.net
Visit : www.EasyEngineering.net

Let us prove formula I. raise a to the power of VIII. If 0 < a < 1, then from 0 < x 1 < x 2 it follows that
loga M+ loga N. By the property of powers and by the loga x 1 > loga x2 and from loga x 1 > loga x2 it
fundamental logarithmic identity, we have follows that 0 < x1 < x2 . In other words, when a < 1
alogam+logaN=alogaM.alogaN = MN the inequalities 0 < x1 < x2 and log a x1 > loga x2 are
equivalent.
The resulting equation
These two properties are proved in exactly the same
aloga M +loga N = MN
way, and so we confine ourselves to proving property
May be rewritten in logarithmic thus: loga M + Loga VIII.
N = log a MN which signifies the validity of formula I.
Let a number a be positive and less than unity. If the
Formula II is proved similarly. inequality 0 < x1 < x2 holds, then there exist numbers loga
To prove equation III, raise a to the power α loga N x1 and loga x2. Using the fundamental logarithmic identity,
and utilize the properties of powers : rewrite the inequality x1 < x2 in the form
α a = (α a ) α = N
a log N log N α aloga x1 < aloga x2
From this, by the definition of a logarithm, we obtain whence, by the properties of a power to a base less
than unity, we conclude that loga x1 > loga x2.
the required equation
Conversely, if the inequality loga x1 > loga x2 is true,

ww
Equation IV follows from the manipulations
α then, firstly, both numbers x1 and x2 are positive.
(α β)β loga N = aα loga N = (aloga N)α = Nα Secondly, raising the number a, 0 < a < 1, to the powers
It will prove useful to memorize the following two loga x 1 and loga x2 we get (again by the properties of

w
special cases of formula IV :
1
IV. (a) logaβ N = loga N
β
V. (b) loga α Nα = logα N
.Ea (N > 0, β ≠ 0)
powers to a base less than 1) the inequality
aloga x1 < aloga x2
or x1 < x2. Now since, as we have already mentioned,
(N > 0, α ≠ 0) the numbers x1 and x2 are positive, it follows that 0 < x1 <

syE
To prove V, let us first write it in the form logα N = x2 which completes the proof.
loga b. logb N. The proof is similar to that of the preceding The following statements are consequences of the

ngi
case : properties that have just been proved :
αlogα b.logb N = (αlogα b)logb N = blogb N = N VII (a) If a > 1, then the inequalities loga x < α and 0
< x < aα are equivalent.

nee
We can reason differently. Writing the fundamental
VIIL (b) If a > 1, then the inequalities loga x > α and x
logarithmic identity
> aα are equivalent.
blogb N = N
VIII (a) If 0 < a < 1, then the inequalities loga x < α
we derive from it the equation
loga (blogb N) = loga N rin
and x > aα are equivalent.
VIII (b) If 0 < a < 1, then the inequalities loga x > α
(equal number have the same logarithms!). Now, using
property III, we convince ourselves of the validity of g.n
and 0 < x < aα are equivalent.
To prove this it suffices to note that α = logα αα.
formula V.

e
From these statements it is easy to drive that
Formula VI is a special case of the preceding one logarithms of numbers exceeding 1 to bases exceeding 1
obtained for N = b. Equation V is usually called the rule are positive and logarithm of numbers less than 1 (but
for changing the base of a logarithm. This rule makes positive !) are negative; and, conversely, logarithms to
t
different tables of logarithms to various bases unnec- bases less than 1 are negative for numbers exceeding 1
essary; it suffices to have, say, tables of common and positive for numbers less than 1.
logarithms (base 10). For instance, suppose it is required
to compute log5 13. On the basis of property V, we can Let us now solve some illustration involving the basic
log10 13 properties of logarithms.
write log5 13 = · Using logarithmic tables, we Illustration 1. Compute log3√3 27.
log10 5
find log10 13 ≈ 1·1139 and log10 5 ≈ 0·6990, and thus By formula IV, we have
log5 13 ≈ 1·5937. (Using Logarithmic Table) 3
log3√3 27 = log3 3/2 33 = log3 3 = 2
Some other properties of logarithms that are abso- 3/2
lutely necessary in the solution of inequalities are : Illustration 2. Compute 2log2√2 15
VII. If a > 1, then from 0 < x1 < x2 it follows that loga By formula IV (a), we have
x 1 < loga x2 and from loga x 1 < loga x2 it follows 2
Log2 √2 15 = log2 3/2 15 = log2 15
that 0 < x 1 < x 2 . In other words, for a > 1 the 3
inequalities 0 < x 1 < x2 and loga x 1 < loga x2 are Applying the fundamental logarithmic identity, we
equivalent. get

CAT Complete Course | 307

Visit : www.EasyEngineering.net
Visit : www.EasyEngineering.net

2
2log2√2 15 = 23 log2 15 loga N1 logb N1
= (N 1 > 0, N2 > 0, N2 ≠ 1)
loga N2 logb N2
3
= (2log2 15)2/3 = 15 2/3 = 
√
225 By formula V, we have
Illustration 3. Compute log3 5.log25 27. loga N1 log N
By formula IV, we have = logN2 N1 and b 1 = log N2 N1
loga N2 logb N2
3
Log3 5.log25 27 = log3 5. log52 33 = log3 5. log5 3 whence it is clear that our equation holds true.
2
Illustration 9. Which is greater, log2 3 or log1/4 5?
And since, by formula VI, log3 5.log5 3 = 1, it follows
that log3 5.log25 27 = 3/2. Since, log 2 3 > 0 and log1/4 5 < 0, it follows that
1 log2 3 > log1/4 5.
Illustration 4. Compute (√ 9)
3 5 log5 3
. Illustration 10. Which is greater, log5 7 or log 8 3 ?
By formula VI, we have Since, log5 7 > 1 and log8 3 < 1, it follows that
1 1 log5 7 > log8 3.
= log3 5
5 log5 3 5 3
√ a if log a = 4.
It then only remains to take advantage of the funda- Illustration 11. Compute logab ab
√ b

w(w)
mental logarithmic identity and the laws of exponents :
3
9

1
5 log5 3
= (9 ) 1 1 log 5
3 5
3
1
= (32/3)5 log 3 5
By the laws of logarithms, we have
3
√ a = 1 log a – 1 log b = 4 – 1 log b
w (3log3 5)2/3·1/5 = 52/15 =

.√E
15
25

 1 
2–
log5 13
logab
16 3

ab
2 ab
3 2 ab

It remains to find the quality logab b. Since,

asy
2log59
Illustration 5. Compute   · 1 = logab ab = logab a + logab b = 4 + logab b
√27 It follows that logab b = – 3 and so

exponents we compute the radicand :

 1 
 
2–
log5 13
2 log5 9
=
1
·
E
Using in succession the laws of logarithms and

(√27) 1/2 log 9 13


ngi logab
3
√ a = 4 – 1 (– 3) = 17
b 3 2
√ 6
Illustration 12. Compute log6 16 if log12 27 = a.

nee
√27 27
1 The chain of transformations
· (3log3 13)3/8 = 3 –3 · 13 3/8
= Log 6 16 = 4 log6 2
27
whence it is clear that the given number is equal to
3–3/2 · 133/16 ·
=
4
=
rin 4
log2 6 1 + log2 3

Illustration 6. Which is greater, log4 5 or log1/16 ?


1
25 g.n
Shows us that we have to know log2 3 in order to
find log6 16. We find it from the condition log12 27 = a :
By formula IV b, we have
1
Log1/16 = log4–2 5 –2 = log4 5
25
So, that the two numbers are equal.
a = log12 27 = 3 log12 3 =

=
1+
3
2
=
e3

3 log2 3
2 + log2 3
=
3
log3 12 1 + 2 log3 2

t
log2 3
Illustration 7. Compute log3 2.log43…………… 2a
log109.log1110. which means that log2 3 = (note that, obviously,
3–a
By formula V, 4 (3 – a)
a ≠ 3). we finally have log6 16 = ·
log11 2 3+a
Log3 2 = ; log 4 –2
log11 3 Illustration 13. Compute log 25 24 if log6 15 = α and
log11 3 log11 9 log12 18 = β.
= ; … log10 9 =
log11 4 log11 10 We have the equation
whence Log3 2. log4 3……..log1110 1
Log25 24 = (log 5 3 + 3 log5 2)
log11 2 log11 3 log11 9 2
= · … · log1110
log11 3 log11 4 log11 10 3 1
= log5 2 + log5 3
= log112 2 2
Illustration 8. Prove that the ratio of the logarithms which shows us that we have to determine log5 2 and
of the two numbers is not dependent on the base; that is, log5 3. The equation log6 15 = α yields.

308 | CAT Complete Course

Visit : www.EasyEngineering.net
Visit : www.EasyEngineering.net

α = log6 15 = log6 3 + log6 5 Illustration 14. Solve the equation x + log10 (1 + 2x)
1 1 = x log10 5 + log10 6.
= +
1 + log3 2 log5 2 + log5 3 Transposing x log10 5 to the left member of the
and the equation log12 18 = β yields equation and utilizing the laws of logarithms, we get
x + log10 (1 + 2x) – x log10 5 = x log10 10 – x log10 5 +
β = log12 18 = log12 2 + 2log12 3 log10 (1 + 2x) = log10 2 x (1 + 2x).
1 1 The equation can thus be rewritten as log10 2x (1 +
= +
2 + log2 3 1 + 2 log3 2 2x) = log10 6, whence
Taking logs to base 5 in all cases we find, by formula (2x) + 2x – 6 = 0
V, Denoting z = 2 x , we arrive at the quadratic equation
1 1 z2 + z – 6 = 0 which has the roots z1 = – 3, z2 = 2. Since
α = +
1 + log3 2 log5 2 + log5 3 the equation 2x = – 3 is impossible (because 2 x is positive
1 1 for all values of x), it remains to solve the equation 2x =
= + 2. It has the root x = 1, which is the sole root of the
log5 2 log5 2 + log5 3
1+ original equation.
log5 3

ww 1 + log5 3 Illustration 15. Solve the equation


= ,
log5 2 + log5 3 1
loga (ax).logx (ax) = loga 2 , where a > 0, a ≠ 1.
1 2 a
β =

w
+
2 + log2 3 1 + 2 log3 2 Clearly, the roots must satisfy the condition x > 0, x
≠ 1. Using the properties of logarithms, transform the
=
2+
1

.Ea
log5 3
log5 2
+
1+2
2
log5 2
log5 3
expression that enter into this equation :
1 loga x + 1

=
syE
log5 2 + 2 log5 3
log5 3 + 2 log5 2
logx (ax) = 1 + logx a = 1 +

1 1 1
loga x
=
loga x

loga 2 = – loga a = – , loga (ax) = 1 + loga x


a 2 2
,

ngi
The last two equations may be regarded as a system
of equations for determining log5 2 and log5 3 : Our equation can now be rewritten as
α log 5 2 + (α – 1) log5 3 = 1 (loga x + 1)2 1
(2β – 1) log5 2 + (β – 2) log 5 3 = 0
If α (β – 2) – (α – 1) (2β – 1) = – α – αβ + 2β – 1 ≠ 0 nee loga x
= –
2
5
whence (loga x)2 + loga x + 1 = 0 solving this
then this system has the solution
log5 2 =
2–β
, log5 3
equation, we get
rin2

α + αβ – 2β + 1

g.n
1 1
x1 = 2 , x2 =
=
2β – 1 a a

α + αβ – 2β + 1
We finally get
log25 24 =
5–β
2α + 2αβ – 4β + 2
Now, let us verify that the expression α + αβ – 2β + 1
5 (log y x + logx y) = 26
xy = 64
e
Illustration 16. Solve the system of equations

t
It is clear that it must be true that x > 0, y > 0, x ≠ 1,
is indeed different from zero. Thus, we have y ≠ 1. Denoting z = logx y and using Formula VI, we find
α + αβ – 2β + 1 = log6 15 + log6 15. log12 18 – 2 that the first equation of the system can be rewritten as 5
(z + 1/z) = 26, whence z1 = 5, z2 = 1/5. This means that
log12 18 + 1
the solutions of the original system must be sought
= (log6 15 – log12 18 + 1) + log12 18. among the solutions of the system
(log6 15 – 1) logx y = 5
The second summand here is positive since log12 18 xy = 64
> 0 and log6 15 > 1. As to the first summand, using the
properties of logarithms, we can write log6 15 > 1, log12 and of the system
18 < 2 and so log6 15 – log12 18 + 1 > 0. Thus, the logx y = 1/5
expression α + αβ – 2β + 1 is positive. xy = 64
The properties of logarithms, among them the pro- Solving these systems and choosing those solutions
perties I to VIII given above, are widely used in solving a which satisfy the conditions x > 0, x ≠ 1, y > 0 y ≠ 1, we
broad range of problems such as logarithmic equations obtain the answer. The original system has two solutions :
and system logarithmic inequality and so on. x1 = 2, y1 = 32, x2 = 32, y2 = 2.

CAT Complete Course | 309

Visit : www.EasyEngineering.net
Visit : www.EasyEngineering.net

Illustration 17. What can be said about the number x For example, in formula I, loga MN has meaning
if it is known that for every real a ≠ 0 when the numbers M and N are both positive as well as
logx (a2 + 1) < 0 ? when they are both negative. By contrast, the right-hand
For every a ≠ 0 the number 1 + a2 > 1. But since the member of this formula is meaningful only in the first
logarithm of a number greater than unity is negative only instance. But this mean that if we transform an equation
to a base less than unity, it follows that x < 1. Further and replace the logarithm of a product of two expression
more, since logarithms are only considered to a positive M and N containing the unknown by the sum of the
base, x > 0. And so finally we see that the number x of logarithm of these expression, then for values of the
our problem is taken in the interval 0 < x < 1. unknowns which make M and N negative numbers, we
change the meaningful expression loga MN into a
Illustration 18. Find all x such that log1/2 x > log1/3 x.
meaningless expression loga M + loga N. The very same
From Formula V, we have goes for formulas II and III.
log1/2 x 1
log1/3 x = = log1/3 · · log1/2 x For these reasons, formulas of a more general nature
1 2 are used in solving problems containing unknowns :
log1/2
3 I* loga MN = loga |M| + loga |N| (MN > 0)
and so our inequality can be rewritten as M
II* loga = loga |M| – loga |N| (MN > 0)

ww 1
(
log1/2 x 1 – log1/3 )
1
2
>0

Since, 1 – log1/3 > 0, from the letter inequality we


N
III* loga N2k = 2k loga |N| (N ≠ 0, k an integer)
1

w 2
obtain log1/2 x > 0, whence x < 1. But the original in-

.Ea
equality is meaningful only when x > 0. Therefore, all x
that satisfy the original inequality lie in the interval 0 < x
IV* logx2k N = log|x|k N
2
(N > 0, k an integer, x ≠ 0, |x| ≠ 1)
It should be noted that formula I* and II* also have
drawbacks stated above: their left and right members are
< 1.

syE
Illustration 19. Solve the inequality
1
loga x
> 1, a > 1.
meaningful for different restrictions on the values of the
letters that enter into them. Namely, the right-hand
members have meaning for arbitrary M and N different

ngi
The fraction 1/p is greater than unity if its denominator p from zero, while the left-hand members are only
lies between zero and unity. Thus, our task is to find meaningful for M and N having the same sign, which

nee
values of x such that their logarithms (to the base a > 1) means that they are subjects to more stringent restrictions.
lie between zero and unity, that is to say, so that the For this reason, replacing loga MN = log a |M| + loga |N|
following two conditions hold true simultaneously : 0 < when solving equations can lead to extraneous solutions

rin
loga x and loga x < 1. The first states that the values of x but not to the loss of solutions, as can happen when using
must exceed unity, the second that they must be less than formulas I-IV. Since, acquiring extraneous solutions of an

g.n
a. Hence, the solution of the original inequality is the equation is preferable to losing solutions (superfluous
interval 1 < x < a. solutions may be discarded by verification, but lost
We can also reason differently. The left member of solution cannot be found), one should use formulas I* to
the proposed inequality is meaningful only for positive IV * when manipulating literal expressions.
values of x different from unity, and so the inequality

only for values of x which are greater than unity (since


e
Here, are some problems which illustrate the impor-
may be rewritten as logx a > 1. This inequality holds true tance of utilizing these properties.
Illustration 20. Simplify the expression
t
for 0 < x < 1 we have logx a < 0 when a > 1) but less than x2
a (since for x > a >1 we have, by the logarithmic laws, log 4 – 2log4 4x4 and then compute its value for
4
logx a < 1). x = – 2.
In the foregoing examples, formulas I to VI were It is quite evident here that computations by formulas
used successfully to transform a variety of expressions I and III, that is,
both with concrete number and literal data. Such x2
log4 – 2 log4 4x4 = 2 log4 x – log4 4 – 2 log4 4 – 8
manipulations are necessary primarily in the solution of 4
equations and inequalities. log4 x = – 3 – 6 log4 x
But in many such cases these formulas are not are erroneous because the letter expression for x = – 2 is
sufficient. First of all, this is due to the fact that the letters meaningless, whereas the original one is meaningful and
in the formulas have to satisfy very stringent restrictions. is equal to – 6.
A still greater drawback of formulas I to IV is that the This paradoxical result is due to the fact that formulas
right and left members are meaningful for different I and III are only applicable to positive values of the
restrictions on the values of the literal elements that enter letters. Now, if we use formulas I* and III* in which the
into them. values of the letters may be negative as well, we get

310 | CAT Complete Course

Visit : www.EasyEngineering.net
Visit : www.EasyEngineering.net

x2 1
log4 – 2 log4 4x4 = 2log4 |x| – 1 – 2 – 8 log4 |x| 4. If log 3M + 3log 3N = 1 + log0·008 5, then—
4 3
= – 3 – 6 log4 |x| 9 9
(A) M9 = (B) N9 =
N M
It is clear that for x = – 2 this expression is equal to – 6.
3 3
Illustration 21. Solve the system of equations (C) M3 = (D) N9 =
N M
Log2 xy = 5
x 5. If log 10 x – log10√  x = 2 logx 10, then a possible
log1/2 = 1 value of x is given by—
y
1
Using formulas I* and II* rewrite the system as (A) 10 (B)
100
log2 |x| + log2 |y| = 5
1
log1/2 |x| – log1/2 |y| = 1 (C) (D) None of these
1000
Denoting z1 = loga |x|, z2 = log2 |y|, we get 6. Let u = (log2 x)2 – 6 log2 x + 12, where x is a real
z1 + z2 = 5 number. Then the equation x n = 256, has—
z1 – z2 = – 1 (A) No solution for x
whence z1 = 2, z2 = 3, and so |x| = 4, |y| = 8. (B) Exactly one solution for x

ww
But this does not mean that the original system has
four solutions :
(C) Exactly two distinct solutions for x
(D) Exactly three distinct solutions for x
7. If log 4 5 = a and log5 6 = b, then log 3 2 is equal to—

w
x1 = 4, y1 = 8, x2 = – 4, y2 = – 8
x3 = 4, y3 = – 8. x4 = – 4 y4 = 8 1 1

.Ea
(A) (B)
Because it is required that the expressions log 2 xy and 2a + 1 2b + 1
x 1
log1/22 be meaningful. They will clearly have meaning (C) 2ab + 1 (D)
2ab – 1

syE
y
only for x and y having the same signs. And so our 8. If log ax x, logbx x, logcx x are in HP, where a, b, c, x
system will only have two solutions : x1 = 4, y1 = 8, and belong to (1, + ∞), then a, b, c are in—
x2 = – 4, y 2 = – 8.

ngi
(A) AP (B) GP
Thus, using formulas I* and II* we acquired (C) HP (D) None of these
extraneous solutions which were readily discarded in a 9. If log5 a.loga x = 2, then x is equal to—
verification; now if we had used formulas I and II and
had rewritten the system as
log2 x + log2 y = 5
nee (A) 125
(C) 25
(B) a2
(D) None of these

log1/1 x – log1/2 y = 1
we would have lost the solutions x2 = – 4, y 2 = – 8.
10. The value of 3log
rin
81
80
25
+ 5log + 7log
24
16
15
is—

g.n
(A) log 2 (B) log 3
Note also that the original system may be solved in a (C) 1 (D) 0
different way by reducing it directly to the system
11. Let f(x) = √

xy = 32, =
x 1
y 2
whence the required answer is obtained.
Exercise
which f(x) is real, is—
(A) [– 1, 1]
(C) (– ∞, 1]
e
log10 x2. The set of all values of x for

(B) [1, + ∞)
(D) (– ∞, – 1] ∪ [1, + ∞) t
12. The set of real values of x for which 2log√2 (x – 1) >
1. If log7 log5 (√
x + 5 + √x ) = 0, find the value of x— x + 5 is—
(A) (– ∞, – 1) ∪ (4, + ∞)
(A) 1 (B) 0
(B) (4, + ∞)
(C) 2 (D) None of these
(C) (–1, 4)
2. If log3 2, log3 (2x – 5), log3 (2x – 7/2) are in arithmetic (D) None of these
progression, then the value of x is equal to—
13. If a1 , a2 , a3 , …….. are positive numbers in GP, then
(A) 5 (B) 4
logan , logan+1, logan+2 are in—
(C) 2 (D) 3
(A) AP (B) GP
3. What is the sum of ‘n’ terms in the series: log m +
log(m2/n) + log m3/n2 ) + log (m4/n3 ) + ……. (C) HP (D) None of these
n(n – 1) n/2 mm n/2
(A) log
m[ (n + 1)] (B) log
nn[ ] 14. If x = loga(bc), y = logb(ca) and z = log c(ab), then
which of the following is equal to 1?
n/2
m(1 + n) n/2
[ ] [ ] (A) x + y + z
m (1 – n)
(C) log (1 – (D) log
n m) n(n – 1) (B) (1 + x)– 1 + (1 + y)– 1 + (1 + z)– 1

CAT Complete Course | 311

Visit : www.EasyEngineering.net
Visit : www.EasyEngineering.net

(C) xyz 26. The solution set of log2 |4 – 5x| > 2 is—
(D) None of these 8 
(A)  ‚ + ∞
15. If x n > xn - 1 > ………….. > x 2 > x 1 > 1, then the 5 
value of 4 8 
logx1 logx2 logx3 …… logxn xnxn – 1 is equal to— (B)  ‚ 
5 5 
(A) 0 (B) 1 8 
(C) 2 (D) None of these (C) (– ∞, 0) ∪  ‚ + ∞ 
5 
16. If logx : logy : logz = (y – z) : (z – x) : (x – y), (D) None of these
then—
x+2
(A) xy.yz.zx = 1 (B) xxyyzz = 1 27. The set of real values of x for which log0.2 ≤1
x
x y z is—
(C) √
 x · √ y · √ z = 1 (D) None of these
 
17. xlogx a × log a y × logy z is equal to— (A)  – ∞‚ –5  ∪ (0, + ∞)
 2 
(A) x (B) y
5
(B) [ , + ∞)

ww
(C) z (D) None of these
18. The number of zeroes coming immediately after the
decimal point in the value of (0·2)25 is (given log102
2
(C) (– ∞, – 2) ∪ (0, + ∞)
(D) None of these

w
= 0·30103)—
(A) 16
(C) 18
(B) 17

.Ea
(D) None of these
19. If [x] = the greatest integer less than or equal to x,
28. The set of real values of x satisfying log1/2 (x 2 – 6x +
12) ≥ – 2 is—
(A) (– ∞, 2]
(C) [4, + ∞)
(B) [2,4]
(D) None of these
then [log10 6730·4]has the value—
(A) 6
(C) 5
(B) 4 syE
(D) None of these
29. If log0.04 (x – 1) ≥ log0.2 (x – 1), then x belongs to the
interval—

20. The number of solutions of log2 (x + 5) = 6 – x is—


(A) 2 (B) 0 ngi (A) [2, 1]
(C) [2, 1)
(B) (2, 1)
(D) (2, 1]
30. If log1/√2 sin x > 0, x ∈ [0, 4π], then the number of
(C) 3 (D) None of these
21. The number of real values of the parameter k for nee π
values of x which are integral multiples of , is—
4
which (log16 x)2 – log16 x + log16 k = 0 with real
coefficient will have exactly one solution is—
(A) 6
(C) 3
rin (B) 12
(D) None of these
(A) 2
(C) 4
(B) 1
(D) None of these interval—
 √
 5 – 1 
g.n
31. If logcosx sinx ≥ 2 and 0 ≤ x ≤ 3π, then sin x lies in the

 √ 5 – 1

e
22. If log0.5 sinx = 1 – log0.5 cosx, then the number of
solutions of x ∈ [– 2π, 2π] is— (A)  2 ‚ 1 (B)  0‚
2 
(A) 3
(C) 1
(B) 2
(D) 4
23. If logcosx tanx + logsinx cotx = 0, then the most general
(C) [0, 1/2]
32. If log√3 (sinx + 2√

(D) None of these
 2 cosx) ≥ 2, – 2π ≤ x ≤ 2π, then
the number of solutions of x is—
t
solutions of x are— (A) 0 (B) Infinite
π π (C) 3 (D) None of these
(A) nπ + , n ∈ Z (B) 2nπ + , n ∈ Z
4 4
Directions—(Q. 33 to 36) Choose the correct options.
3π One or more options may be correct.
(C) 2nπ – , n ∈ Z (D) None of these
4 33. If log k x.log5 k = logx 5, k ≠ 1, k > 0, then x is equal
24. The number of values of x ∈ [0, nπ], n ∈ Z, that to—
satisfy log|sin x| (1 + cos x) = 2, is— (A) k (B) 1/5
(A) 0 (B) n (C) 5 (D) None of these
(C) 2n (D) None of these 34. If x 2 + 4y2 = 12xy, x ∈ [1, 4], y ∈ [1, 4], then—
89 (A) The greatest value of log2 (x + 2y) is 4
25. The value of Σ log10 tan
πr
180
is equal to— (B) The least value of log 2 (x + 2y) is 3
r=1 (C) The range of value of log2 (x + 2y) is [2, 4]
(A) 10 (B) 1 (D) The number of integral values of (x, y) is 2 such
(C) 0 (D) None of these that log2 (x + 2y) is equal to 3

312 | CAT Complete Course

Visit : www.EasyEngineering.net
Visit : www.EasyEngineering.net

1 Applying log to base 2 on both sides, we get u log2 x


35. If ≤ log0·1 x ≤ 2, then—
2 = log2 2 8
1 ulog2 x = 8 …(2)
(A) The maximum value of x is
10
√ Dividing eq. 2 by eq. 1, we get
1 1 u = 8/p
(B) x lies between and
100 ⇒ 8/p = p2 – 6p + 12
10

1 1 ⇒ 8 – p3 – 6p2 + 12p
(C) x does not lie between and ⇒ (p – 2) 3 = 0
100 10
√ ⇒ p = 2
1
(D) The minimum value of x is ⇒ log2 x = 2 ⇒ x = 4
100
Thus, we have exactly one solution.
36. If x 3/4 (log3 x) + log 3 x – 5/4 = √
 3, then x has—
2
7. (D) Here, 5 = 4a and 6 = 5b
(A) One positive integral value
Let log32 = x. Then, 2 = 3 x
(B) One irrational value
(C) Two positive rational values Now, 6 = 5b = (4a )b = 4 ab or 3 = 22ab – 1
(D) None of these ∴ 2 = (22ab – 1)x = 2x(2ab – 1)
⇒ x(2ab – 1) = 1

ww
1. (B) log7 log5 (
Solutions
√ x + 5 + √ x) = 0
8. (B) Clearly, logx (ax), logx (bx), logx (cx) are in AP
⇒ 1 +logx a, 1 + logx b, 1 + logx c are in AP


w log5 (
√ x + 5 + √ x ) = 70 = 1

.Ea
 x + 5 + √x = 51 = 5
2√x = 0
⇒ logx a, logx b, logx c are in AP

log a log c
+
log x log x
= 2
⇒ log a + log c = 2log b
log b
log x

2.

syE
x = 0
(D) In an AP, the three terms a, b, c are related as 2b
=a+c

9. (C) 10. (A)
ac = b2

11. (D) log10 x2 ≥ 0 ⇒ log 10 x2 ≥ log10 1


Hence, 2 [log 3 (2x – 5)] = log3 2 + log3 (2x – 7/2)
log3 (2x – 5)2 = (2(x + 1) – 7)
Substitute the choices, only x = 3 satisfies the con- ngi ⇒ x2 ≥ 1 ⇒ x ≥ 1 or x ≤ – 1
12. (A) {( 2)2 }log 2 (x – 1)

log 2 (x – 1)2
> x+5

3.
ditions.
(D) S = log m + log
m2 m3
+ log 2 + ….. n terms nee or ( 2)
2

13. (A) 14. (B)


> x+5
or (x – 1) > x + 5 or (x – 4) (x + 1) > 0

rin
n n x1
m 2 m3

mn
= log (m · · … n –1) 15. (B) log x1 logx2 logx3 …… logxn (xn– 1 x n – 2 logxn xn)
n n2 n
 n(n + 1)
( )
(n + 1) n/2

g.n x1

m 2
= log   = log m(n – 1)
 n(n – 1)
 n = logx1 logx2 logx3 …… logxn xn– 1 x n – 2

e
n 2
= ………
4. (B)


1
3
log3 M + 3 log3 N = 1 + log0·008 5

log3 (M.N9)1/3 = 1 +
log3 5
log3 ·
8
= logx1 x1 = 1.
16. (B) 17. (C) 18. (B)
19. (D) log10 7630·4 = 3.xxxx
t
1000 ⇒ [log10 6730] = 3
= 1 – 1/3 = 2/3 20. (D) Here, x + 5 = 26 – x .
⇒ (M.N9 )1/3 = 32/3 Clearly, there cannot be more than one solution and
⇒ N9 = 9/M by trial, the solution is x = 3.

5. (B) log10 x – log10 


√ x = 2 logx 10 1 ±√
1 – 4 log16 k
21. (B) log16 x = · For exactly one
2
⇒ 1/2 log10 x = 2 logx 10
solution, 4log16 k = 1
⇒ log10 x /logx 10 = 4
∴ K4 = 16
⇒ log10 x = 2 ∴ k = 2, – 2, 2i, – 2i. But k is positive and real.
⇒ x = 1/100 22. (B) log 0.5 sinx + log0.5 cosx = 1
6. (B) u = (log2 x)2 – 6(log2 x) + 12 ⇒ log (sinx. cosx) = 1/2
Let (log2 x) = p …(1) ⇒ sin 2x = 1
⇒ u = p2 – 6p + 12 π π π
xu = 256 = 2 8 ∴ 2x = , ± 2π + , ± 4π + ,……
2 2 2

CAT Complete Course | 313

Visit : www.EasyEngineering.net
Visit : www.EasyEngineering.net

0
30. (A) sinx < 

π π π
, ± π + , ± 2π + , …
1 = 1 and sin x > 0
x =
4 4 4 
√
 2
But log0.5 sinx, log0.5 cosx have to be real at the same ∴ sin x has all values in (0, π), (2π, 3π), (4π, 5π),…
time. So, angles in the first quadrant only can be π π π π
considered. ∴ required x = , 2 × , 3 × , 9 × ,
4 4 4 4
π π π π
∴ x = , – 2π + · 10 × , 11 ×
4 4 4 4
23. (B) log cosx sinx – 1 + logsinx cosx – 1 = 0 31. (B) sin x ≤ cos2 x, because cos x must be a positive
Or y + 1/y = 2 , where y = logcosx sinx proper fraction.
1 2 5
∴ (y – 1)2 = 0 or logcosx sinx = 1 or sinx = cos x
Also, sinx, cosx, tanx must be positive and sinx ≠ 1, (
Or sin2 x + sin x – 1 ≤ 0 or sin x +
2 )
– ≤0
4
cosx ≠ 1. From the definition of logarithm, sin x > 0, cos x > 0,
24. (A) 1 + cosx = |sin x|2 cos x ≠ 1
= sin 2 x or cosx (1 + cosx) = 0 √ 5 ∴ 0 < sin x ≤ √ 5 – 1
∴ sin x + 1/2 ≤
But 1 + cosx ≠ 0 for the value of the logarithm to be 2 2
real. 2

cosx = 0
32. (D) sin x + 2√ 2 cos x ≥ √ ( )
3

ww
⇒ sin x = 1 (not possible, because the base of the
logarithm cannot be 1)
Or sin x + 2

(
√ 2 cos x ≥ 3
Or sin x + cos–1
1
3
≥ 1)
w
25. (C) Value = log10 {tan1°. tan 2°. tan 3°…..tan 89°)
= log10 {(tan1°. tan 89°)…… (
⇒ sin x + cos–1
1
= 1)
.Ea
= log10 1 = 0
(tan 44°.tan46°). tan 45°}

3
x = n π + (– 1)n
π
2
– cos–1
1
3
26. (C)
⇒ | 5x
4 |
–1 > 1
syE
|4 – 5x| > 2 = 4 For solutions in [– 2π, 2π], n = 0, 1, – 1, – 2
33. (B, C)

log5 x = logx 5
(log5 x)2 = 1

5x
4
5x
–1 > 1
ngi ⇒

34. (A, C, D)
log5 x = ± 1
x = 5, 5– 1

nee
or –1 < –1
4 (x + 2y)2 = 16xy
∴ x > 8/5 or x < 0. So, the solution set or 2log2 (x + 2y) = 4 + log2 x + log2y

( 8
) 1

x+2
= (– ∞, 0) ∪

≥ (0·2)1 or
x+2 1

5
‚+∞
But 1 ≤ x ≤ 4, 1 ≤ y ≤ 4.
rin
∴ log2(x + 2y) = 2 + (log2 x + log2 y)
2

g.n
27. (A) 1
x x 5 ∴ maxlog2(x + 2y) = 2 + (log 2 4 + log2 4)
Multiplying by 5x 2 2
5x (x + 2) ≥ x2 or 4x2 + 10x ≥ 0 1
∴ x ≥ 0 or x ≤ –

Also,
x+2
x
>0
5
2 Also log2 (x + 2y) = 3
⇒ x + 2y = 8
e
minlog2(x + 2y) = 2 + (log 2 1+log21)
2

This is satisfied by x = 2, y = 3; x = 4, y = 2.
t
1 1/2 1 2
⇒ x (x + 2) >0
∴ x < – 2 or x > 0
1
35. (A, B, D) ≤ log1/10 x ≤ 2 ⇒
2 10 ( )≥ x,
10() ≤x
5 1 1
∴ The solution set is (– ∞, – ] ∪ (0, + ∞). So, ≤x≤
2 100 10

1 –2
28. (B) x2 – 6x + 12 ≤ () 2
or x2 – 6x + 8 ≤ 0 36. (A, B, C)
or (x – 2)(x – 4) ≤ 0
log10 (x – 1) log10 (x – 1)
Taking logarithm, {34 (log x) + log x –54} log x
3
2
3 3

29. (C) ≥
log10 0·04 log10 (0·2) = log3 √3
⇒ log10 (x – 1) · { 1

1
2log10(0·2) log10 (0·2) } ≥0 Or, 3/4y3 + y2 – 5/4y
Or (y -1)(3y2 + 7y + 2)
=
=
1/2 (let log3 x = y)
0
–1
⇒ log10 (x – 1) · ≥0 Or (y – 1)(3y + 1)(y + 2) = 0
2log10(0·2)
⇒ log10 (x – 1)≥ 0 because log10 (0.2) < 0 Hence, log3 x = 1, – 2, – 1/3
∴ x – 1 ≥ 100 ∴ x ≥ 2 ⇒ x = 3, 3– 1/3, 3– 2.
Also x – 1 > 0, i.e., x > 1. ●●●

314 | CAT Complete Course

Visit : www.EasyEngineering.net
Visit : www.EasyEngineering.net

4 Functions and Graphs


CONCEPT OF A FUNCTION For instance,
Methods of Representing Functions
When studying various phenomena of nature and in
Y = 3x, y = {xx ifif xx >≤ c0}
2

our everyday practical activity, we come across quantities Such a method of representing a function is called
of various character, such as length, area, volume, mass, analytical.
temperature, time, and so on. Depending on concrete In the general case, when a function is specified
condition, some quantities have constant and variable, analytically, its domain of definition is usually under-

ww
respectively.
Mathematics studies the dependence between variable
quantities (or simply, variables) in the process of their
stood (provided those are no additional conditions) as the
maximum set of values of x for which the formula
representing the function makes sense.
change.
w .Ea
For instance, with a change in the radius of a circle,
its area also changes, and we consider the question of
how the circle changes depending on the change in its
For instance, the function y = x2 is defined throughout
the number axis, the same as the analytical expression
which represents it. But if this function express the
dependence of the area of a square on the length of its
radius.
syE
Let the variable x take on numerical values from this
set E.
side, then the function y = x2 is specified for any x > 0.
(2) Tabular method : When specifying a function

ngi
by means of a table, we simply write down a sequence of
Consider the concept of a function. A function is a valuable of the argument x 1 ,x2,….,xn and the corres-
rule which attributes to every number x from E one ponding values of the function y1,y2,……….yn. This
definite number y.
Here x is called the independent variable, or the
argument of the function, and y is called the dependent
nee
method of representing a function is called tabular.
This way of representing functions is widely used;
variable, the set E is spoken of as the domain of
definition of the function. The set of all values attained rin
for instance, the reader is undoubtedly familiar with tables
of logarithms, tables of trigonometric functions and their
logarithms, etc.
by the variable y is called the range of the function.
The above given definition can be formulated in g.n
The tabular method is particularly often used in
natural sciences and technology. The numerical results
other words : A variable y is said to be a function of a
variable x in the domain of definition E if to each value of
x belonging to this domain there corresponds a definite
value of the variable y.
e
obtained in a sequence of observations (measurements) of
a process are usually complied in a table, which thus
shows relation between the quantities under investi-
gations.
t
The notation y = f(x) or y(x) means that y depends on
An advantage of a tabular representation of a function
x. The letter f symbolizes the rule according to which we
is that for any value of the independent variable included
obtain the value of y corresponding to a given value of x
into the table the corresponding value of the function is
from the set E.
immediately found without any additional measurement
Instead of the letter x, E,Y, f(x) any other letters and or calculation. But it also has an essential demerit:
notations are also used. To represent or, which is the usually it is impossible to specify a function by a table in
same, to specify a function y = f(x) on the set E means to a complete manner, since there are some values of the
indicate the rule according to which for every x from E independent variable that do not enter into the table.
the corresponding value of y is found.
(3) Graphical representation : On the co-ordinate
Consider basic methods of representing functions : xy-plane for every value of x from the set E (that is, from
(1) Analytical representation (by means of for- the domain of definition of a function) a point M(x, y) is
mula). constructed whose abscissa is equal to x and whose
A function can be given by a single formula in the obtained to the corresponding value of the function y(x).
entire domain of its definition or by several formulas, Points thus constructed plot a certain line which is called
different for different parts of the domain of its definition. the graph of a given function.

CAT Complete Course | 315

Visit : www.EasyEngineering.net
Visit : www.EasyEngineering.net

In general, the graph of a function y = f(x) specified Examples of odd functions : y = x3, y = x3 + x,
on the set E is the set of points M(x, f(x)), where x ∈ E, x
y= 2
belonging to the co-ordinate plane. x +1
In general, the graph of a function y = f(x) specified Indeed, (– x)3 = – x 3 , (– x) 3 + (– x) = – (x3 + x ),
on the set E is the set of points M(x, f(x) where x ∈ E (– x) x
=– 2 of any x.
belonging to the co-ordinate plane. (– x)2 + 1 x +1
The method of representation of a function with the The sum and difference of an odd function is again
aid of a graph is called graphical. To find the value of a an odd function, but the product and quotient of an odd
function y(x) for a definite value of x by a given graph of function is an even function.
the function, let us proceed as follows. Through the point It should not be regarded that every function is even
x on the axis of abscissas erect a perpendicular to this axis or odd. Most functions do not posses the property to be
and find the point of intersection of this perpendicular even or odd.
with the graph of the given function. The ordinate of the
For instance such is the function y = x3 + x2
point of intersection just yields the corresponding value
of the function. Indeed (– x)3 + (– x)2 = – x3 + x2 , i.e., (– x)3 + (– x)2 ≠
x + x2 and also (– x)3 + (– x)2 ≠ – (x3 + x2).
3
The graphical method of representation a function is
The definition of even and odd functions implies that

ww
widely used in scientific investigations as well as in mod-
ern production. Graphs of change of various quantities
are automatically drawn by recording instrument
the graph of an even function is symmetric about the axis
of ordinates, and that of an odd function about the origin.

w
(recorders). Really let the point M(x 0 , y0 ) be a point of the graph
Illustration 1. Find the domain of definition of the of an even function y = f(x), i.e., y0 = f(x0).
following functions :
(a) y = 2
1 .Ea
; (b) y = √

4 – x2; (c) y =
1
.
x – 3x + 2

syE √

4 – x2
Solution : (a) The function is defined for all values
of x, except for those for which x2 – 3x + 2 = 0. Solving
this quadratic equation, we find x = 1 and x = 2.
Thus, the domain of definition of the given function ngi
consists of three intervals : ( – ∞, 1), (1, 2) and (2, + ∞).
(b) The domain of definition of the given function is
determined from the condition 4 – x2 ≥ 0. Solving this nee
rin
Fig. 1
inequality we obtain – 2 ≤ x ≤ 2. Thus, the domain of
Consider he point N(– x0, y0) which is symmetric to
definition of the given function is the interval [– 2, 2].
the point M(x 0 y0) about the y-axis (Fig. 1). Since, the
(c) The domain of definition of the function is
determined from the condition 4 – x2 > 0, whence – 2 < x
< 2. Consequently, the domain of definition of the given
given function is even, we have
g.n
f(– x0) = f(x0 ) = y0
function is the interval (– 2, 2).
We shall most often consider function represented
analytically with an interval or half-interval as the domain
of their definition.
e
which means that the point N(– x 0 , y0) also belongs
to the graph of the functions y = f(x).
t
Properties of Functions
1. Even and Odd Functions : Let a function y = f(x)
be given in a certain interval symmetric with respect to
the point O (in particular, throughout the entire x-axis).
The function y = f(x) is said to be even if for any
f(– x) = f(x).
Examples of even functions : y = x2, y = x2 + 3, y =
– 3x2 + 4, y= |x|, y = 4.
Indeed, (– x)2 = x2, (– x)2 + 3 = x2 + 3, – 3(– x)2 + 1 =
2
– 3x + 1, |– x| = |x|, y = 4 for any x. Fig. 2
The sum difference, product and quotient of an even The symmetry of the graph of an odd function about
function is again an even function. the origin follows from the fact that along with the point
A function y = f(x) is said to be odd if for any x M(x 0 , y 0 ) of the graph of an odd function, there also
f(– x) = – f(x). belongs to this graph the point N(– x 0 , – y0) which is

316 | CAT Complete Course

Visit : www.EasyEngineering.net
Visit : www.EasyEngineering.net

symmetric to the point M(x 0 , y0) about the origin. The In general, to every value y 0 from D there will
point O bisects the line segment MN. (Fig. 2) correspond one or several values of x0 from E for which
2. Monotone Functions : A function y = f(x) is said f(x0 ) = y0. To obtain these values of x 0 , we may draw
to be increasing on a certain interval if for any two through the point y 0 on the axis of ordinate a straight line
values of x from this interval to greater values of the parallel to the axis of abscissas. This straight line will
argument there correspond greater values of the function, intersect the graph of the function y = f(x) at one or
that is, the condition x1,< x 2 implies that f(x1) < f(x2) for several points. The abscissas of these points just yield the
any x1 and x2 from the given interval. desired values of x (one of them being x0) for which the
function is equal to y0 . (Fig. 5)
Suppose that the function y = f (x) is such that to
every value y0 from D there corresponds one value x0
from E for which f(x0 ) = y 0 . In this way the function x =
g(y) is defined in the domain D and is called the inverse
of the function y = f(x).
Fig. 3 Fig. 4 If E is an interval and the function y = f(x) is mono-
tone (increasing or decreasing), then it has an inverse

ww
The ordinate of the graph of an increasing function
increase with an increase in x. (Fig. 3)
Similarly, a function y = f(x) is called decreasing on
function x = g(y), increasing or decreasing, respectively.

w
some interval if for any two values of x from this interval

.Ea
to greater values of the argument there correspond
smaller values of the function, that is, the condition x1 <
x2 implies that f(x1 ) >> f(x2) for any x 1 and x2 from this
interval.
syE
The ordinate of the graph of a decreasing function
decreases with an increases in x. (Fig. 4)
Increasing and decreasing functions are referred to as
monotone functions. ngi
3. Interval of constant sign and roots of a function :
Interval within which a function keeps its sign unchanged
(that is, remain positive or negative) are called intervals
nee Fig. 5

of constant sign of the function.


For instance, the function y = x 2 + 1 is positive rin
throughout the x-axis; the functions y = x 3 is positive for
x > 0 and negative for x < 0, its intervals of constant sign g.n
are (0, + ∞) and (– ∞, 0) consequently the graph of the
function y = x3 is situated above the x-axis for x > 0 and
below the x-axis for x < 0.
The values of the argument x for which f(x) = 0 are
e t
called the roots (or zeros) of the function f(x). Thus, the
root of the function f(x) is the same as the root of the
equation f(x) = 0. The roots of a function are the points of Fig. 6. Gives an example of an increasing function.
intersection of its graph with the x-axis. The function graphed in Fig. 5. has no inverse.
The root of the function y = x3 is x = 0 the function In a function given by a formula has an inverse, then
y = x 2 + 1 has no real root. to find the formula defining the inverse, we have to
Inverse of a Function and its Graph express x in terms of y from the given formula.
For instance, derive the formula specifying in the
Let there be given a function y = f(x), and let E be its
y+1
domain of definition and D the set of its values (or the inverse of the given function y = 2x – 1, we have x =
2
range). Then to every value x0 from E there will
which is the inverse.
correspond one definite value y0 = f(x0) from D.
y+1
Let us take an arbitrary number y0 from D. In the The graphs of the functions y = 2x – 1 and x =
2
domain E there is necessarily at least one number x0 for coincide, since both function express one and the same
which f(x0) = y0. relation between the variables x and y. In general, the

CAT Complete Course | 317

Visit : www.EasyEngineering.net
Visit : www.EasyEngineering.net

graph of the function y = f(x) and the graph of its inverse and then by its shape, to find out the properties of the
x = g ( y) is one and the same curve. Usually, when function.
studying the inverse function, its argument is denoted by 1. Linear Function y = kx + b and its Graph : A
x and the dependent variable by y, i.e., instead of x = g(y), linear function is defined as a function of the from
we write y = g(x). In such a notation the inverse of the Y = kx + b,
x+1
function y = 2x – 1 will be y = · where k and b are given numbers,
2
(1) consider a particular case when k = 0, then
Y = b
This function is defined throughout the entire x-axis
and attains one and the same value b for all x’s. Con-
sequently, its graph is a straight line parallel to the x-axis
and passing at a distance of |b| units from it (if b > 0 and
below it if b < 0) (Fig. 8). If b = 0, then the graph of the
function y = 0 is a straight the coinciding with the x-axis.
(2) If b = 0, then y = kx. For k ≠ 0 the function y = kx

ww
is called the direct proportionality relation (or depend-
ence). This function is defined everywhere. It increase
monotonically for k > 0 and decrease for k < 0. Let us

w
prove that the function y = kx is monotone.

.Ea
Fig. 7
The graph of two inverse (or reciprocal) functions y
= f(x) and y = g(x) are symmetric about the bisector y = x

syE
of the first and third quadrants. (Fig. 7)
Proof : Let M (x0, y 0 ) be a point belonging to the graph
of the function y = f(x). Then, by the definition of the
inverse of a function, the point N(y0, x 0 ) will belong to
the graph of the inverse function y = g(x). We have to ngi
prove that the points M (x0, y 0 ) and N (y0, x 0 ) are
symmetric with respect to the straight line y = x. For this nee Fig. 8

rin
purpose, let us consider the triangle MON. In this
triangle, |OM| = |ON|, the line segment OP being the
bisector.
Indeed, |OM| = |ON| = √
x2
0 + y2
0 by the formula of
the distance between two points on a co-ordinate plane, g.n
the equality |OM| = |ON| can be obtained from the triangle
MPM1 and NON1 . From the same triangle we have :
MOM1 = NON 1 . The bisector OP in the isosceles triangle
MON also serve as the median and altitude i.e., |MP| =
e t
|NP| and |OP| ⊥ |MN|, which was required to be proved.
Properties and Graph of Certain Simplest
Functions
In the general case, a function y = f (x) is investigated Fig. 9
according to the following plan : We take two arbitrary values x1 and x 2 and find the
1. Find the domain of definition of the function and corresponding values y1 and y2 :
the set of its values. y1 = kx1 , y2 = kx2 .
2. Check to see whether the function is even or odd.
Subtracting y1 from y2 ,we obtain
3. Find the intervals of monotonicity and the intervals
y2 – y1 = k(x2 – x1).
of constant sign of the function.
4. Determine the points of intersection of the graph If x2 > x 1 and k > 0 then y2 – y 1 > 0; then y 2 > y 1 and
of the function with the co-ordinate axes and so on. the function y = kx increases.
Then we can plot the graph of the function. Some- If x2 > x 1 and k < 0 then y2 – y1 < 0; then y2 < y 1 and
time, it is simpler to construct the graph of the function the function y = kx decrease.

318 | CAT Complete Course

Visit : www.EasyEngineering.net
Visit : www.EasyEngineering.net

Consequently, the function y = kx is monotone.


If x = 0, then the value of the function y = kx is also
equal to zero, hence, the point (0, 0) belongs to the
function. For k > 0 the sign of x and y coincide; for k < 0
the signs of x and y are opposite.
Hence, we conclude that for k > 0 the points of the
graph of the function y = kx belongs to the first and third
quadrants, and for k < 0 to the second and forth quadrants.
Let us now prove that the graph of direct proport-
ionality is a straight line passing through the origin. Fig. 11
(3) General case; y = kx + b. Every point of the graph
Let us take x = 1. then y = k. the straight line passing of this function is obtained by shifting (or translating) the
through the point P(1, k) and the origin (0, 0) is the graph corresponding point of the graph of the function y = kx by
of the function y = kx indeed let k > 0. |b| units along the axis of ordinates (upward if b > 0 and
The triangle MON and POQ are similar for any po- downward if b < 0). Therefore, the graph of a linear func-
sition of the point M on the constructed straight line. The tion is a straight line parallel to the straight line y = kx.

ww
similarity implies that
|MN| |PQ|
=
|ON| |OQ|
or
y k
= , i.e., y = kx
x 1
(Fig. 10)
The coefficient k is called the slope of the straight
line y = kx. The slope determines the angle of inclination

w
The result is also retained for any point M which lies

.Ea
on the straight line under consideration situated in the
third quadrant (in this case, its distances from the x and y-
α of this straight line to the x-axis : k = tan α. If k > 0,
then the angle α is acute; If k < 0, then the angle α is
obtuse. The ordinate of the point of intersection of the
straight line and the y-axis equal to b.

< 0 and x < 0.


syE
axis are respectively equal to |y| = – y and |x| = – x since y

Thus, it has been proved that any point situated on


Thus, the location of the straight line y = kx + b on
the co-ordinate plane depends on the values of k and b.
To construct the graph of a linear function, one has
the straight line passing through the points P(1, k) and
O(0, 0) belongs to the graph of the function y = kx. No ngi to plot two points belonging to this graph and then to
draw a straight line through these points.
For instance, let us construct the graph of the

nee
other points M1 situated outside this straight line can
1
belong to the graph y = kx.(See Fig. 9) If we assume that function y = – x + 1.
the point M 1 (x, y1) belongs to this graph then it must be y1 2

rin
For x = 0, y = 1; for y = 0, x = 2. joining the found
= kx. At the same time, the point M(x, y) obtained as the
points by a straight line, we obtain the graph of the given
intersection of the straight line drawn from the point M1
1 1
parallel to the y-axis and the straight line OP belongs (as
it was proved) to the sought for graph. Hence, y = k x,
which contradicts the equality y1 = k x : their right-hand
k
g.n
function. (Fig. 11) here k = – and tan α = – ·
2 2
2. The Function y = and its Graph : A function

e
x
members are equal, while the left hand members are dif-

t
k
ferent, since y ≠ y1 . Thus, the graph of the function y = kx of the form y = , where k ≠ 0 is a given number is called
x
is the straight line OP. the inverse proportionality relation.
The case k < 0 is considered analogously. Consider the case k > 0 :
(1) The function is defined everywhere, except for x
= 0, the domain of its definition being the intervals (– ∞,
0) and (0, ∞) :
(2) The function is odd, since
k k
f(– x) = = – = – f(x);
–x x
k
Consequently, the graph of the function y = is
x
symmetric about the origin and, therefore, the further
investigation is conducted for x > 0;
(3) The sign of y coincide with the sign of x;
(4) The function is decreasing, since for 0 < x 1 < x 2
we have
k k k(x1 – x2)
– = < 0 (k > 0)
Fig. 10 x2 x1 x1x2

CAT Complete Course | 319

Visit : www.EasyEngineering.net
Visit : www.EasyEngineering.net

X –3 –2 –1 1 0 1 1 2 3

2 2
Y = X2 9 4 1 1 0 1 1 4 9
4 4

The graph of the function y = x2 is shown in Fig. 14


and is called the parabola. For x = 0 the value of the
function y = x2 equals zero. For x ≠ 0 the values off the
function are positive. This means that the parabola y = x2
touches the x-axis at the origin, its remaining points lying
above the x-axis.
Fig. 12
(it is obvious that for k > 0 the function also decrease
on the interval (– ∞, 0))
Using these properties, we construct the graph of the
k

ww
function y = for k > 0 (Fig. 12). The obtained curve is
x
called the hyperbola. It consists of two branches situated
in the first and third quadrants. Similarly, it is proved that

w k
if k < 0, then the function y = is monotone: it increases

.Ea
x
on each of the intervals (– ∞, 0) and (0, + ∞) the graph is
also a hyperbola. Its branches are situated in the second Fig. 14

syE
and fourth quadrants. (See Fig. 12)
Thus, the graph of the inverse proportionality y =
k
x
The parabola y = x2 is symmetric about the y-axis.
The point of intersection of the parabola with its axis of
symmetry is called the vertex of the parabola. The vertex
(k ≠ 0) is a hyperbola whose location on the co-ordinate
plane depends on the values of k.
ngi
1
of parabola y = x2 is the origin.
Let us now compare the functions y = 2x2 and y = x 2 .
For one and the same x the value of the function y = 2x2 is

nee
For instance, Fig. 13 depicts two hyperbolas; y = –
x twice the value of the function y = x2 consequently, the
2
and y = – , the origin being the centre of symmetry of graph of the function y = 2x2 can be obtained by two –
x
these hyperbolas.
rin
fold extension of the parabola y = x2 along the y-axis.
In general, the graph of the function y = ax2 for a > 0

g.n
can be obtained by stretching the parabola y = x2a times
along the y-axis (more precisely by stretching for a > 1
and by compressing for 0 < a < 1). Note the following

e
properties of the function y = ax2 for a > 0;

t
(a) The function is defined for any x, and also y = ax2
≥ 0; consequently, the least value of the function is equal
to zero and is attained for x = 0;
(b) The function is even, since f(– x) = a(– x)2 = ax2
= f(x). Therefore, the y-axis is the axis of symmetry of the
graph;
Fig. 13 (c) The function is increase on the interval (0, + ∞)
3. Quadratic Trinomial and Its Graph : A quadratic and decrease on the interval (– ∞ , 0). Let us prove an
trinomial is defined as a function of the form increase in the function for x > 0. For 0 < x1 < x2 we have
Y = ax2 + bx + c ax21 < ax2 2 (a > 0) (by the property of inequalities) and,
where a , b and c are given numbers and a ≠ 0 hence, the function y = ax2 is increasing for a > 0 on the
sometimes, the function y = ax2 + bx + c where a ≠ 0, is interval (0, + ∞). A decreases in the function for x < 0
called a quadratic function. follows from the evenness of the function and its
Let us first consider particular cases of functional increases for x > 0.
depends y = x2 + bx + c, a ≠ 0. Compare the function y = – x2 and y = x2. For one
(1) Quadratic function y = ax2 and the same x the values of these functions are equal by
For a =1 we have y = x2. To construct the graph of modulus and opposite by sign. Consequently, the graph
the function y = x2 let us compile a table of its values : of the function y = – x 2 can be obtained by the symmetry

320 | CAT Complete Course

Visit : www.EasyEngineering.net
Visit : www.EasyEngineering.net

of the parabola y = x 2 about the x-axis. The branches of This parabola can be obtained by shifting the
the parabola y = x 2 are said to be directed upward, and parabola y = ax2 along the x-axis by |x0 | units (rightward
those of the parabola y = – x2 downward. if x0 > 0 and leftward if x0 < 0). (Fig. 16)
(3) Quadratic function y = ax2 + c

ww Fig. 15
The graph of the function y = ax2 for any a ≠ 0 is Fig. 17

w
also a parabola with the y-axis as the axis of symmetry

.Ea
and the origin as its vertex for a > 0 the branches of the
parabola are directed upward and a < 0 downward
(2) Quadratic function y = a(x – x0)2.
The graph of the function y = ax 2 + c is a parabola
with the vertex (0, c ) and the y-axis as its axis of
symmetry. This parabola can be obtained by shifting the

syE
Compare the functions y = 2(x – 1)2 and y = 2x2 . The
function y = 2(x – 1)2 takes on the same value as the
parabola y = ax2 along the y-axis by |c| units (upward if c
> 0 and downward if c < 0) (Fig. 17)
(4) General case : y = ax2 + bx + c (a ≠ 0).
function y = 2x2, but with the corresponding value of the
argument increased by unity. Consequently, the graph of
the function y = 2(x – 1)2 can be obtained by displacing ngi Isolating a perfect square in the trinomial ax 2 + bx + c
we rewrite the function y = ax2 + bx + c as follows :
(or shifting) the parabola y = 2x2 along the x-axis
rightward by unity. As a result, we shall get the parabola
y = 2(x – 1)2 whose axis of symmetry is parallel to the y-
nee Y = a(x – x0)2 + y0.

rin
From the above considered particular cases it follows
axis and whose vertex is the point (1, 0). that the graph of a quadratic trinomial is a parabola with
vertex at the point C(x0 , y0) whose axis is a straight line

g.n
Proceeding in a similar way, that is shifting the
parabola y = 2(x)2 along the x-axis leftward by unity, we passing through its vertex parallel to the y-axis.
obtain the parabola y = 2(x + 1)2 whose axis of symmetry The branches of the parabola y = ax2 + b x + c are
is parallel to the y-axis and whose vertex is the point
(– 1, 0).
In general, the graph of the function y = a(x – x0 )2 is
a parabola with the vertex (x0, 0) whose axis of symmetry
e
directed upward if a > 0 and downward if a < 0. Note that
the abscissa x 0 of the parabola y = ax2 + b x + c can be
found by the formula t
b
is a straight line passing through the vertex parallel to the X0 = –
2a
y-axis.
The ordinate of the vertex of the parabola
Y0 = ax02 + bx0 + c.
The graph of a quadratic trinomial can be constructed
with the aid of the following technique:
1. Reduce the quadratic trinomial to the form y = a(x
– x0)2 + y0 by isolating a perfect square.
2. Construct the vertex of the parabola i.e., the point
C(x0, y 0 ) and draw through it a straight line parallel to the
y-axis which will be the axis of symmetry of the parabola.
3. Construct the point if intersection of the parabola
Fig. 16 and the y-axis.

CAT Complete Course | 321

Visit : www.EasyEngineering.net
Visit : www.EasyEngineering.net

4. Find the real root of the quadratic trinomial, if any, Consider Several Examples
and plot the corresponding points of the parabola on the Illustration 2.
x-axis. Represent a given positive number a in the form of
5. Join the constructed points to get a parabola. the sum of two addends so that their products is the
x +x greatest possible.
Remark. It is easy to check that X0 = 1 2 ,
2 Solution : Let us denote one of the required addends
where x1 and x 2 are the roots of the quadratic trino- by x. Then the other addends will be equal to (a – x).
mial. Their product x(a – x) is a quadratic trinomial. We now
Since, y = ax2 + bx + c = a(x – x0)2 + y0 we have: transform the trinomial by isolating a perfect square :
(1) If a > 0, then for x = x 0 the quadratic trinomial
takes on the least value equal to y 0 indeed, if a > 0, then
( )
x(a – x) = – x2 + ax = – x –
a 2 a2
2
+
4
for any x a
Hence, it is seen that for x = the quadratic trinomial
a(x – x0)2 ≥ 0. 2
a2
takes on the greatest value equal to , thus each of the
4
a
desired addends is equal to ·

ww Illustration 3.
2

Construct the graphs of the following functions :

w .Ea
(a) y = x2 + 2x + 3; (b) y = – 2x2 + 4x + 1; (c) y = –
2(x – 1) (x + 3).

syE
Fig. 18 ngi
Therefore, y ≥ y0, where y = y0 only for x = x0. Graph-
ically, this means that of all points of the parabola y = ax2 nee
rin
+ bx + c for a > 0 the least ordinate is possessed by the
point C (x0, y0) i.e., by the vertex of the parabola. (Fig.18)
(2) If a < 0, then for x = x 0 the quadratic trinomial
attains the greatest value equal to v0 Indeed, if a < 0, then
for any x g.n
Fig. 20
Solution : (a) Separate a perfect square y = x2 + 2x +
3 = (x + 1)2 + 2. Consequently, the vertex of the parabola
a(x – x0)2 ≤ 0.
e
is C(– 1, 2); (0, 3) is the point intersection of the parabola
and the y-axis; the branches of the parabola are directed
upward. (Fig. 20)
(b) Transform of the trinomial; y = – 2x2 + 4x + 1 =
t
(– 2x2 + 4x – 2) + 3 = – 2(x – 1)2 + 3. Hence, the vertex of
the parabola is C(1, 3) (Fig. 21).

Fig. 19
Therefore, y ≤ Y0 , where y = y0 only for x = x0.
Graphically this means that of all points of the parabola y
= ax 2 + bx + c for a < 0 the point C(x0 , y 0 ) i.e., the vertex
of the parabola has the greatest ordinate. (Fig. 19) Fig. 21

322 | CAT Complete Course

Visit : www.EasyEngineering.net
Visit : www.EasyEngineering.net

The roots of the trinomial are x1 = 1 –


√ 32 and
x2 = 1 +
√ 32.

Fig. 24
4. Power Function with Integral Exponent and its
Graph : A power function with an integral exponent is
defined as a function of the form

ww Fig. 22
(c) The roots of the trinomial y = – 2(x – 1)(x + 3) are
Y = xn,
where n ≠ 0 is an arbitrary integer.
The function is defined for any x (except x = 0 for

w
x1 = 1 and x 2 = – 3, and, consequently, x0 =

.Ea
1–3
2
= – 1 is
the abscissa of the vertex of the parabola C. Finally, we
find its ordinate y0 : y 0 = – 2(– 1 – 1) (– 1 + 3) = 8. Thus,
n < 0).
For n = 1, n = 2, and n = – 1 we have y = x, y = x2
and y = x– 1 = ,
1
x

Illustration 4. syE
the vertex of the parabola is C(– 1, 8) (Fig. 22) Respectively, their graphs are : a straight line (the
bisector of the first and third quadrants), a parabola and a
hyperbola, respectively.

ngi
Construct the graph of the following functions : If n is an even number, then the power function y =
(a) y = |x2 – 1 :| (b) y = x2 + 2 |x|. xn is an even function : (– x)n = (xn) for any x. If n is an
Solution : (a) First construct the parabola y = x2 – 1. odd number, then the function y = xn is an odd function :
since |x2 – 1 :| = x2 – 1 for x 2 – 1 ≥ 0 and |x2 – 1:| =
– (x 2 – 1) for x2 – 1 < 0, we shall proceed as follows : we
map the part parabola situated below the x-axis symmet-
nee
(– x)n = (xn). Consequently the graph of the function y =
xn is symmetric about the y-axis for an even n and it is
symmetric about the origin for an odd n.
rically about this axis. The graph of the function y =
|x2 – 1 :| is depicted in (Fig. 23). rin
If n is a positive integer, then the graph of the func-
tion y = x n is a parabola. For n = 2 this is simply a para-

g.n
bola for n = 3 a cubical parabola and so on. If n > 0, then
by the property of inequalities, the condition 0 < x 1 < x 2
implies x1n < x2n < i.e., the function y = xn where n is a

e
natural number, increase on the interval (0, + ∞ );
consequently, for an even n it decrease on the interval
t
(– ∞, 0) and for an odd n increase on the interval (– ∞, 0)
and, hence, throughout the x-axis. The graphs of the
function y = x n for n = 2k and n = 2k + 1 are given in Fig.
25 and 26, respectively.

Fig. 23
(b) The given function is even :
(– x)2 + 2 |– x| = x2 + 2|x|.
Consequently, its graph is symmetric about the y-
axis. For x ≥ 0 we obtain y = x2 + 2x = (x + 1)2 – 1 which
is a parabola with the vertex (– 1, – 1). Its points with the
abscissa x ≥ 0 are also points of the graph of the function
y = x 2 + 2 |x|. (Fig. 24) Fig. 25

CAT Complete Course | 323

Visit : www.EasyEngineering.net
Visit : www.EasyEngineering.net

5. The Graph of the Function y =  x. The Func-



n
tion y = 
√ x and its Graph : We know the prosperities of
the arithmetical square root. Hence, we have the follow-
ing properties of the function y = 
√x :
(1) The function is defined for all x ≥ 0.
(2) The value of the function is equal to zero only for
x = 0 and is positive for any x > 0.
(3) The function is monotone, it increases in the
entire domain of its definition. The graph of the function
y =√  x is shown in Fig 29. It is the inverse of the
function y = x 2 on the interval (0, + ∞). Therefore, its
graph is symmetric to the parabola y = x2 about the
Fig. 26 bisector of the first quadrant. The inverse of the function
In general, the graph of the power function y = xn y = x 2 on the interval (– ∞, 0) is y = – √
 x.

ww
with an integer positive exponent has for an even n the
same shape as the graph of the function y = x2 , and for an
odd n as the graph of the function y = x3 .
Due to the properties of the arithmetical n th root.
Hence, we conclude that the function y = √
n
 x, where n ≥ 2

w
Consider now the power function y = x n with an
integral negative exponent. If n = – 1 then we have a
1
.Ea
hyperbola y = (Fig. 27). If n = – 2, then we have the
x
is a natural number, has the same properties as the
function y = √
 x.

syE
function y = 2 whose graph is shown is Fig. 28 for an odd
x
n the graph of the Function y = x n looks like the graph of

ngi
the function y = x – 1 and for an even n like the graph of
the function y = x – 2.

nee Fig. 29

rin
n
The function y =  x can be understood in the

following way.

g.n
The power function y = x n with an integer positive
exponent n ≥ 2 increases on the interval (0, + ∞).

e
Consequently, on this interval the function y = xn has an
inverse which is also an increasing function. This
function is specified by the formula y = √
n
t
 x, where x ≥ 0,
and its graph is symmetric to the graph of the function y =
xn about the bisector of the first quadrant. (Fig. 30)
Fig. 27

Fig. 28 Fig. 30

324 | CAT Complete Course

Visit : www.EasyEngineering.net
Visit : www.EasyEngineering.net

6. Exponential Function its Properties and Graph :


An exponential function is defined as a function of the
Indeed, by the property of a power,
ax
b x =
a x
b ()
,
form a ax ax
where < 1. Therefore, x < 1 for x > 0 and x > 1 for x <
Y = ax, b b b
where a is a given positive number not equal to unity. 0 (by property 4).
The exponential function is considered only a > 0,
since for a < 0 and a = 0 the expression ax loses sense for
some values of the variable x.
For instance, for a = – 4 the expression (– 4)x has no
1
sense for x = (among the real numbers). If a = 1, then ax
2
=1 for any x. The case a = 1 is not considered since it is
not interesting.
The properties of the exponential function are as
follows :
(1) The exponential function Y = ax, is defined for all

ww
real values of the argument x, that is, its domain of
definition is the entire number axis (– ∞ , + ∞ ). This
follows from the definition of the power of a positive real
Fig. 31
The graph of the exponential function Y = a x is

w
number with any real exponent. represented in Fig. 31. In the computation practice an
important role is played by the exponential function y =
(2) a0 = 1 for any base a ≠ 0.

.Ea
(3) The exponential function Y = ax is positive in the
entire domain of definition and attains all positive values.
10x. Let us show that, knowing its values for x from the
interval [0, 1], it is easy to compute its values for any x.

syE
Let there be required to find an approximate value of
The latter means that for any y > 0 there exists a value of
the expression 102.36. We write the exponent 2.36 in the
x such that ax = y.
form of the sum of the integral and fractional parts 2.36 =
The first part of the statement follows from the prop-
2 + 0.36, then
erties of a power with a rational exponent (for a power
with an irrational exponent the proof is omitted).
(4) For a >1 we have ax >1 for x > 0 and ax < 1 for x ngi 102.36 = 102 + 0.36 = 102 . 10 0.36 = 100.10 0.36
Consequently, to find an approximate value of 102.36,
< 0; for 0 < a < 1, vice versa.
Let a >1. Consider the case of a rational x. nee
it remains to find the value of 100.36.
Suppose we have to find an approximate value of
10–3.24 to this end we write the exponent – 3.24 in the
If x = n, where n is natural, then it is obvious that an
> 1 for a > 1.
p
ap/q
– 3.24 = – 4 + 0.76, then rin
form of the sum of the integral and fractional parts :

g.n
p
If x = , where p and q are natural then a q = q 10–3.24 = 10 –4 + 0.76 = 10 –4. 100.76.
q
ap
√ It remains to find the value of 100.76.

e
p x
> 1, since a > 1 for a > 1, thus a > 1 for x > 0 and > 1.
7. Logarithmic Function and its Graph :

p
a
1
If x = – n, where n is natural then a – n = n < 1.
p
If x = – , where p and q are natural then aq = q
q
1

ap

Definition : The logarithm of a number b to a base

to which a must be raised to obtain b.


t
a(a > 0 and a ≠ 1) is the exponential indicating the power

Notation : loga b (read : “The logarithm of the number


q
< 1. √ap > 1 for a > 1. ax1 ax2 ax2 – x1 . b to the base a”).
The case 0 < a < 1 is proved in a similar way. For the logarithm of a number b to the base a = 10
x we use the symbol log b to mean log10 b.
(5) The exponential function Y = a is monotone, it
For instance, instead of log 1027 we write log 27.
increasing for a > 1 and decreases for 0 < a < 1.
x By the definition of logarithm, log216 is the exponent
Let us prove that the function y = a is increasing for
any a > 1. We take two arbitrary values x1 and x2 (x1 < indicating the power to which 2 must be raised to obtain
x x x x – x
x2). Then a 2 – a 1 = a 1 ( a 2 1 – 1 ) . By the 16, that is log 2 16 = 4, since 2 4 = 16.
1
property of positiveness of an exponential function, ax1 > Similarly, log 3 27 = 3, since 33 = 27; log 2 = – 2,
0, by property (4), – ax2 –x1 – 1 > 0, since a > 1 and x 2 – x 1 4
1
> 0. Consequently, ax2–x1 > 0 or ax2 > ax1 for x1 < x 2 , that since 2– 2 = ; log5 1 = 0,
4
is, the function Y = ax increase if a > 1. 1
(6) If a < b, then ax < bx for x > 0 and ax > bx for x < Since, 50 = 1; log 10 = 1, log = – 2. Note that
100
x x
0. for x = 0 a = b = 1. x
such expression as log3 (– 27) and 6 = 0 have no roots.

CAT Complete Course | 325

Visit : www.EasyEngineering.net
Visit : www.EasyEngineering.net

In general the expression loga b, where a > 0 and a (2) Log 1 = 0, that is, the graph intersects the x-axis
≠ 1, has sense only for b > 0. at the point (1, 0).
From the definition of the logarithm it follows that (3) The function y = log x is a monotone increasing
aloga b = b (a > 0, a ≠ 1) function (a greatest logarithm corresponds to a greatest
For any b > 0. This equality is an identity on the set number).
of positive numbers. (4) The function y = log x attains all real value, that
For instance, 10log x = x for x > 0. is, the range of values of the function is the set of all real
The exponential function y = a x(a > 0 and a ≠ 1) is numbers.
monotone throughout the entire x-axis. Consequently, it
8. Graph of Function Containing a Modulus :
has the inverse.
To specify by a formula the inverse of an exponential
function, let us express from the formula Y = a x the
1. y = |x| = {–– xx ifif xx <≥ 00}
variable x in terms of y : We know that the straight line y = x is the bisector of
X = loga y. the first and third quadrants, and the straight line y = – x
Passing over to the customary notation we obtain is the bisector of the second and fourth quadrants. Hence,
Y = loga x. we obtain the graph of the function y = |x| (represented in

ww
A logarithmic function is a function of the from
Y = loga x.(a > 0, a ≠ 1)
Fig. 34).

w .Ea
syE
ngi
Fig. 32
nee 
Fig. 34
1 
– 2x + 1 if x ≥ – 2 

2. y = |2x + 1| =  
rin 1

– (2x + 1) if x < – 2

g.n
Let us first construct the straight lines y = 2x + 1 and
y = – 2x – 1, by determining their points of intersection

e
with the co-ordinate axes. On the straight line y = 2x + 1
1
we take only points with abscissa x ≥ – , and on the line
1
2
y = – 2x – 1 points with abscissa x < – . Thus, we obtain
t
2
the graph of the function y = |2x + 1| (Fig. 35).
Fig. 33
This is the inverse of the exponential function.
Therefore, their graphs are symmetric to about the
straight line y = x. Knowing the graph of an exponential
function, we obtain the graph of a logarithmic function.
(Fig. 32)
In particular, the graph of the function y = log x is
symmetric to the graph of the function y = 10x about the
straight line y = x (Fig. 33). The property of the loga-
rithmic function y = log x can be obtained from its graph.
They are listed below :
(1) The function y = log x is defined for all positive
numbers (therefore, all negative numbers and zero are
said to have no logarithms). Fig. 35

326 | CAT Complete Course

Visit : www.EasyEngineering.net
Visit : www.EasyEngineering.net

3. y = x |x| = {–– xx ifif xx ≥< 00}


2
2

The function is odd; the graph is symmetric about the


point O (Fig. 36).

Fig. 39
Knowing the graph y = log x, we obtain the graph y =
|log x| (Fig. 39).
7. y = |x + 1| – |x – 2|. By the definition of the

ww
4. y = √
Fig. 36

|x|. This function is defined for any x. The


modulus,

{
|x + 1| = }
– x + 1 if x ≥ – 1
– (x + 1) if x < – 1

w
function is even; for x ≥ 0; y = √
plotting the graph (Fig. 37).
 x. Hence, the method of

.Ea
{
|x – 2| = }
– x – 2 if x ≥ 2
– (x – 2) if x < 2
The points x = – 1 and x = 2 divided the entire
number axis into three intervals : (– ∞, – 1), (– 1, 2) and

syE (2, + ∞).


Consider the function on each of the intervals.

ngi Let x ≤ – 1, then y = – (x + 1) + (x – 2) = – 3.


If – 1 ≤ x ≤ 2, then y = x + 1 + (x – 2) = 2x – 1.
For x ≥ 2, y = x + 1 – (x – 2) = 3.

nee Consequently, the given function can be written in


the form

rin
Y = 
– 3 if x ≤ – 1
2x – 1 if
3 if x ≥ 2 – 1 ≤ x ≤ 2



Fig. 37
g.n
5. y = The function is even; for x ≥ 0, y = 2x. Its
2|x|.
graph is shown in (Fig. 38).
e t
Fig. 40
Hence, it is clear that on each of the intervals under
consideration the graph of the given function is a straight
line (Fig. 40).
Graphical Method of Solving Equations and
Fig. 38 Systems of Equations. Equation of a Circle

6. y = |log x| = {–– log


log x if log x < 0 }
x if log x ≥ 0 Consider the equation in one unknown
f(x) = 0,

CAT Complete Course | 327

Visit : www.EasyEngineering.net
Visit : www.EasyEngineering.net

where f(x) is a function of the variable x. where a, b and c are given real numbers, where at
For a graphical solution of an equation it is necessary least one of the numbers a and b is not equal to zero. Let
to construct the graph of the function y = f(x) and find its b ≠ 0. Then the equation can be transformed to the form
points of intersection with the axis of abscissa. The a c
Y = – x +
abscissa of these points yield the values of the real roots b b
of the equation f(x) = 0. In particular, the graphical a c
The graph of the linear function y = – x + is a
method can be applied to solving the linear equation ax + b b
b = 0 and quadratic equation ax2 + bx + c = 0 (a ≠ 0). In straight line, which will just be the graph of the equation
certain cases it is possible to transform the equation f(x) = ax + by = c if b ≠ 0.
0 to an equivalent equation of the form g(x) = h(x). In Let now b = 0. Then the equation takes the form ax =
such cases we construct the graphs of the functions y = c
c or x = – (if b = 0, then it follows that a ≠ 0). The set of
g(x) and y = h(x) and find the abscissa of their points of a
intersection. points in the plane whose co-ordinate satisfy the equation
Illustration 5. c
x = is a straight line parallel to the y-axis (Fig. 42).
a
Solve graphically the equation x2 + x – 2 = 0.

ww
Solution :
It is possible to construct the parabola y = x2 + x – 2

w
and find the abscissa of the points at which it intersects
the x-axis. But it is simpler to proceed in a different way.

.Ea
Let us rewrite the given quadratic equation as follows :
x2 = 2 – x

syE
ngi
nee Fig. 42
Thus, the graph of any linear equation ax + by = c is
a straight line.

rin
Let us construct the graph of the equation xy = – 1.
1

g.n
We transform the equation to the form y = – and
1
x
construct the graph of the function y = – (Fig. 43).
Fig. 41
We then construct the parabola y = x2 and the straight
line y = 2 – x and find the abscissa of their points of
intersection : x = – 2 and x = 1 (Fig. 41). Hence, the given
e x

t
equation has the roots : x1 = – 2 and x2 = 1.

Consider an equation in two unknowns x and y. The


graph of an equation in two unknowns is the set of points
of a plane whose co-ordinates reduce the given equation
to a true equality.
Let, for instance there be given the equation 2x – 3y
2 Fig. 43
= 6. We transform it to the form y = x – 2 and construct
3 The constructed hyperbola is thus the graph of the
2 equation xy = – 1. In the above considered example we
the graph of the linear function y = x – 2, which is a
3 were seeking for the graph of an equation to obtain in the
straight line. plane the corresponding line- a straight line or hyperbola.
Consider now as arbitrary linear equation There arises an inverse problem : for a line in the
ax + by = c, plane set-up the equation whose graph is this line.

328 | CAT Complete Course

Visit : www.EasyEngineering.net
Visit : www.EasyEngineering.net

Fig. 44 Fig. 46
Consider the circle of radius r with centre at the The graph of the equation x 2 + y 2 = 25 is a circle of
origin (Fig. 44). Let us write the equation whose graph is radius 5 centered at the origin, and the graph of the
represented by this circle.
equation x + y = 5 is a straight line. The circle and the
Let M(x, y) be an arbitrary points of the given circle. straight line intersect at the points A(0, 5) and B (5, 0).
Its distance from the centre of the circle O is equal to the
Consequently, the given system has two solutions: (0, 5)
radius of the circle: |OM| = r.

ww
On the other hand, by the formula for the distance of
a point in the plane from the origin, we have


and (5, 0).
Consider the system of two linear equations in two
unknowns x and y :

w
Therefore,

√ x2+
|OM| =

y2
.Ea
x2 + y2

= r or + = x2 y2 r2
Thus, the co-ordinate of any point of a circle satisfy
a1x + b1 y = c 1

a2 x + b2 y = c 2
where a1 , b1, c 1 , a2, b2 and c2 are given real numbers

syE
the equation x2 + y2 = r2 . If a point does not belong to the
given circle, then its co-ordinates do not satisfy the
equation x2 + y2 = r2 .
and also at least one of the numbers a1 and b1 and at least
one of the numbers a 2 and b2 are not equal to zero to

Indeed, then |OM| ≠ r, 


√
x2 + y2 ≠ r, and x 2 + y2 ≠ r2 .
Conclusion : A circle with centre at the origin whose ngisolve this system graphically, we have to construct two
straight lines which are the graphs of the equations
entering into the system. The solution of the given system
radius is equal to r is the graph of the equation x2 + y2 =
r2 , where x and y are variables and r is a given positive
number. nee
depends on the mutual position of two straight lines in the
plane. The following three cases are possible here :
To solve graphically a system of two equations in
two unknowns we have to construct (in one and same co- unique solution.
rin
(1) The lines intersect; in the case, the system will

ordinate system) the graphs of the given equations and


find the co-ordinate of the points of intersection of these
graphs.
system will have no solution; g.n
(2) The lines are distinct and parallel; in this case, the

Illustration 6.
Solve graphically the system of equation

{x2 + y2 = 25
e
(3) The lines coincide; in this case, the system will

any point on the coincident lines are a solution of the


system.
t
have an infinite set of solutions, since the co-ordinate of

x+y=5
Solution : Such is the geometrical interpretation of the solution
Construct (in one and the same co-ordinate system) of a system of two linear equations in two unknowns.
the graph of the equations x2 + y2 = 25 and x + y = 5 (Fig. Illustration 7.
45).
Solve graphically the following system of equations :

{x2x––2y4y==14
Solution :
The straight lines x – 2y = 1 and 2x – 4y = 4 are
parallel (Fig. 46) and hence, the system has no solution.
Constructing Graph (Solving Problems)
Consider the below given examples on constructing
Fig. 45 the graphs of functions and equations.

CAT Complete Course | 329

Visit : www.EasyEngineering.net
Visit : www.EasyEngineering.net

Illustration 8. Construct the straight lines y = – (x – 2) and y = x –


Construct the graph of the function 2. Taking the points with the abscissa x ≥ 2, on the first of
them and with the abscissa x < 2 on the second, we obtain
Y = { x‚ x ≤ 0
x2‚ x > 0 the graph of the given function (Fig. 49).
Solution :
The graph of this function given by different formulas
on different intervals of variation of the argument consists
of the bisector of the third quadrant and a branch of a
parabola (Fig. 47).

Fig. 49
Note that the graph of y = – |x – 2| can be obtained by
shifting the graph of y = |x | by 2 along the x-axis

ww
rightward and then mapping it symmetrically about the
x-axis.
Illustration 11.

w .Ea
Fig. 47
Construct the graph of the equation |y| = |x|.
Solution :
The given equation decomposes into two equalities :
y = x and y = – x, since if two numbers are equal
Illustration 9.
syE
Construct the graph of the function y =
|x|
x
.
modulus, then the numbers are either equal or differ only
in sign. The graph of the equation |y| = |x| consists of the
bisectors of the quadrants (Fig. 50).
Solution :
The given function is defined for any x ≠ 0. Here, ngi
Y = {1 if x > 0.
– 1 if x < 0.
The desired graph is represented in Fig. 48.
nee
rin
g.n
Fig. 50
Illustration 12.
e
Construct the graph of the equation |x| + |y| = 1.
Solution : t
Since, |– x| = |x|, then if (x, y) is a point of the graph,
Fig. 48 the point (– x, y) will also be a point belonging to the
graph. Hence, the graph is symmetric about the y-axis.
Illustration 10.
The given equation contains y only under the modulus
Construct the graph of the function
sign and, consequently, along with the point (x, y) of the
y = – 
√ x2 – 4x + 4. graph, the point (x, – y) will also belong to the graph, that
Solution : is, the graph is also symmetric about the x-axis.
Since, x 2 – 4x + 4 = (x – 2)2 , y = – |x – 2|, we have Let x ≥ 0 and y ≥ 0. Then for the points of the first
quadrant the equation takes the form x + y = 1. Construct
|x – 2| = {x– –2 2(xif–x2)≥if2.x < 2. the straight line x + y = 1 and take on it only the points
situated in the first quadrant. Then map the obtained line
Consequently Y = 
√ x2 – 4x + 4 . segment symmetrically about the co-ordinate axes. The
= (– (x – 2) if x < 2) graph of the equation |x| + |y| = 1 is the contour of a
(x – 2 if x ≥ 2). square (Fig. 51).

330 | CAT Complete Course

Visit : www.EasyEngineering.net
Visit : www.EasyEngineering.net

Fig. 51
Illustration 13. Fig. 53
Illustration 16.
Construct the graph of the equation |x| + |y| = 0.
Solution : Construct the graph of the function y = √
1 – x2.
Solution :
Since, |x| ≥ 0 and |y| ≥ 0, the given equation is

ww
satisfied only by the numbers x = 0 and y = 0. The graph
consists of only one point- the origin.
Squaring both sides, we obtain

Since, y = √
Y2 = 1 – x2, or x2 + y2 = 1.

1 – x2 ≥ 0, for constructing the graph of

w
Illustration 14.

Solution : .Ea
Construct the graph of the function y =
x–1
x
.
the equation x 2 + y 2 = 1 it is necessary to leave only the
points with the ordinate y ≥ 0. The graph of equation x 2 +
y2 = 1 is a circle of radius 1 centered at the origin.
Consequently, the graph of the function y = √ 
1 – x2 is the

1
syE
We have y = 1 – . Therefore, the graph of the given
x
upper semi-circle (Fig. 54).

ngi
1
function can be obtained by shifting the hyperbola y = –
x
along the y-axis by unity upward (Fig. 52).

nee
rin
g.n
Fig. 54
Illustration 17.
e t
Construct the graph of the function y = |x2 + 2| x |– 3|.
Solution :
The given function is even; its graph is symmetric
about the y-axis. For x ≥ 0, we have y = |x2 + 2x – 3|.
Fig. 52
Illustration 15.
Construct the graph of the function y = x3 – x.
Solution :
The given function is odd :
(– x)3 – (– x) = – (x3 – x).
Consequently, the graph is symmetric about the
origin.
Let x ≥ 0. Since, y = x3 – x = x(x + 1) (x – 1), we have
for 0 ≤ x ≤ 1, y ≤ 0; for x ≥ 1, y ≥ 0 and at the point x = 0,
x = 1, and x = – 1 the graph will intersect the x-axis.
Taking into account the oddness of given function and
intervals of constant sign, we construct the graph of the
function (Fig. 53). Fig. 55

CAT Complete Course | 331

Visit : www.EasyEngineering.net
Visit : www.EasyEngineering.net

Fig. 58
Fig. 56 Illustration 20.
Hence, the method of constructing the graph of the Construct the graph of the function y = 10–logx .
given function : first construct the parabola y = x2 + 2x – Solution :
3, then the graph of the function y = |x2 + 2x – 3| (Fig. 55) We use the basic logarithmic identity
and finally, the graph of the function y = |x 2 + 2| x |– 3| 10logx = x if x > 0.

ww
(Fig. 56). The graph y = x2 + 2x – 3 = (x + 1)2 – 4 is a
parabola with the vertex (– 1, – 4) and branches directed
1
Then, 10 –logx = (10logx)– 1 = x – 1 = , where x > 0.
x

w
upward, the ordinate of the point of intersection of the Therefore, the graph of the function y =10-logx is the
parabola and the y-axis being (– 3). Solving the quadratic 1

.Ea
branch of the hyperbola y = situated in the first quadrant
equation x2 + 2x – 3 = 0, we find its roots : x1 = 1 and x2 = x
– 3, which are the abscissa of the points at which the (Fig. 59).
parabola intersects the x-axis.
Illustration 18.
syE
Construct the graph of the function

y= (12) – 1.
x
ngi
Solution :
nee
rin
Construct the graph of the exponential function y =
Fig. 59
1 x
() with the base less than 1; shift the obtained graph Illustration 21.
2
along the y-axis by 1 downward, and, finally, taking into
g.n
Construct the graph of the equation |y| = log x.
Solution :

e
consideration the modulus, we have the graph of the
Note that :
given function (Fig. 57).
(1) Since, |y| ≥ 0, log x ≥ 0, i.e., x ≥ 1.
t
(2) If (x, y) is a point belonging to the graph, then
(x, – y) will also be its point, that is, the graph is sym-
metric about the x-axis. Therefore, using the graph y = log
x, we obtain the graph of the equation |y| = log x. (Fig. 60).

Fig. 57
Illustration 19.
Construct the graph y = log(x – 1). Fig. 60
Solution : Application of Graph to Solving Inequalities
The desired graph can be obtained by displacing the The knowledge of how to construct a parabola (the
known graph y = log x along the x-axis by 1 rightwards graph of a quadratic trinomial) can be used for the
(Fig. 58). graphical method solving quadratic inequalities.

332 | CAT Complete Course

Visit : www.EasyEngineering.net
Visit : www.EasyEngineering.net

Illustration 22. 1 1
function y = – x. The set of solution of the inequality
Solve graphically the inequalities – 3x2 – 5x + 2 > 0. 2 2
Solution : x + 2y – 1 > 0 consists of the points in the plane lying
1 1
The graph of the trinomial y = – 3x 2 – 5x + 2 is a above the straight line y = – x. (Fig. 63).
2 2
parabola whose branches are directed downward. We find
1
the roots of the trinomial : x1 = – 2 and x2 = . Therefore,
3
the parabola intersects the x-axis at these point (Fig. 61).
The inequalities – 3x2 – 5x + 2 > 0 is satisfied by
those values of x for which the points of the parabola lie
above the x-axis, that is, numbers x are such that – 2 < x <
1
·
3

Fig. 63

ww Illustration 26.
Represent the set of points specified by the system of

w
inequalities
Fig. 61
{ x2 + y ≤ 1‚

.Ea
A system of inequalities in one unknown can also be
solved graphically. Solution :
y–x≥–1

Illustration 23.

{ syE
Solve graphically the system of inequalities
x – 1 > 0‚
We have the inequalities y ≤ 1 – x2 and y ≥ x – 1.
Let us construct the parabola y = 1 – x2 and the
straight line y = x – 1. The set given by the system of

Solution :
3 – x > 0.

Let us construct the graphs of the functions y = x – 1ngi inequalities consist of the points lying on the parabola y =
1 – x2 or below it and, simultaneously, on the straight line
y = x – 1 or above it (Fig. 64).
and y = 3 – x in one and the same co-ordinate system
(Fig. 62). Both graphs lie above the x-axis for the value
of x from the interval (1, 3).
nee
rin
g.n
e t
Fig. 64
Illustration 27.
Represented the set of points in the plane defined by
the system of inequalities
Fig. 62
We are now going to show how graphs are applied to
solving inequalities and system of inequalities unknowns.
{x2x+–yy<<1‚2
Illustration 24. Solution :
Solve graphically the inequalities x + 2y – 1 > 0. Since, x + y < 1, we get y < 1 – x; since 2x – y < 2,
Solution : we obtain y > 2x – 2. The set specified by the given
To solve graphically the inequalities x + 2y – 1 > 0 or system of inequalities consist of the points lying below
1 1 the straight line y =1 – x and, simultaneously, above the
y > – – x, first construct the graph of the linear
2 2 straight line y = 2x – 2 (Fig. 65).

CAT Complete Course | 333

Visit : www.EasyEngineering.net
Visit : www.EasyEngineering.net

Q. 16. Construct the graph of the function


y = (|x + 1| + 1) (x – 3) …(6)
Q. 17. Construct the graph of the function
|x – 3| + |x + 1|
Y = …(7)
|x + 3| + |x – 1|
Q. 18. Construct the graph of the function
y |sin x| + |cos x|.
Q. 19. Graph the function
sin x cos x
Y = + …(8)

√ 1 + tan x √
2 
1 + cot2 x
x2 + 1
Q. 20. Graph the function y = .
x
Fig. 65 Q. 21. Construct the graph of the function y = x sin x.
i.e., the set of solution of each of these linear inequalities Q. 22. Sketch the graph of the function y = 21/x .
is a half-plane. The set defined by the system of these
Q. 23. Construct the graph of the function

ww
inequalities is the intersection of the two half-planes.
Exercise – A
y = 1 – 21 + sin (x + 1)
Q. 24. Construct the graph of the function

w
Q. 1. Find the domain of definition (or, simply, do-
main) of the function y = logx cos x.

.Ea
Q. 2. Find the domain of the function
cot x
…(1)
y = log2 (1 – x2).
Q. 25. Construct the graph of the function
y = logsinx 1/2.
Q. 26. Construct the graph of the function y = sin x2.

syE
Y =

√ sin x – cos x Q. 27. Find a set of points, in the plane, whose co-
Q. 3. Find the domain of the function ordinate x and y satisfy the system of inequalities

ngi 5x + 3y ≥ 0
1 + x2
Y=√ 
cos(cos x) + arcsin …(3)
2x y – 2x > 2 …(11)
Q. 4. Construct the graph of the function y = 2 – 1/x. Q. 28. Determine the set of prints, in a plane, whose
Q. 5. Construct the graph of the function y =
3
x+4
.
nee
co-ordinate x and y satisfy the relation
|x + y| = |y| – x …(12)
Q. 6. Construct the graph of the function y =
–x+5
3x – 2
.
rin
Q. 29. A system of Cartesian co-ordinates is given in
a plane. Represent the region of this plane filled with all

g.n
Q. 7. Draw the graph of the function y = log4 (– x). the points whose co-ordinates satisfy the inequality
Q. 8. Construct in a single drawing the graphs of the Logx logy x > 0 …(13)
functions
y1 = sin x, Y2 = sin 2x, Y3 = – 2 sin x
Q. 9. Construct the graph of the function
y = sin [2x – (π/3n)].
e
Q. 30. Find all the points in the plane, whose co-
ordinate x and y satisfy the inequality
cos x – cos y > 0
Exercise – B
t
Q. 10. Construct the graph of the function Directions (Q. 1–2) : Read the information given
y = sin 2 x. below and answer the questions that follows—
Q. 11. Draw the graph of the function If md (x) = |x|, mn (x, y) = minimum of x and y and
1 Ma (a, b, c, …) = minimum of a, b, c …
Y = …(4) 1. Value of Ma [m d (a), mn (md(b), a), mn (ab, md
x log10x
(ac)], where a = – 2, b = – 3, c = 4 is—
Q. 12. Sketch the graph of the function
(A) 2 (B) 6
Y = log1/2 x –( ) 1
2
+ log2 
√
4x2 – 4x + 1 …(5) (C) 8 (D) – 2
2. Give that a > b then the relation Ma [md (a). mn (a,
Q. 13. Construct the graph of the function y = |2 – 2x|. b)] = mn [a, md(Ma (a, b)] does not hold if—
Q. 14. Construct the graph of the function (A) a < 0, b < 0
y = ||x + 1| – 2|. (b) a > 0, b > 0
Q. 15. Construct the graph of the function (C) a > 0, b < 0, |a| < |b|
y = x2 – 2 |x| – 3. (D) a > 0, b < 0 |a| > |b|

334 | CAT Complete Course

Visit : www.EasyEngineering.net
Visit : www.EasyEngineering.net

Directions (Q. 3–6) : Read the information given M (x, y) = xy


below and answer the questions that follows— D (x, y) = x/y where y ≠ 0.
x–3 12. What is the value of M (M (A(M (x, y), S(y, x))
If f(x) = 2x + 3 and g(x) = , then
2 A(y, x)) for x = 2, y = 3 ?
3. fog (x) = (A) 50 (B) 140
(A) 1 (B) go f (x) (C) 25 (D) 70
15x + 9 1 13. What is the value of S [M (D (A (a, b), 2)), D (A (a,
(C) (D)
16x – 5 x b), 2), M (D(S (a, b), 2), D (S (a, b), 2))] ?
4. For what value of x; f(x) = g (x – 3) ? (A) a2 + b2 (B) ab
(A) – 3 (B) 1/4 (C) a2 – b2 (D) a/b
(C) – 4 (D) None of these Directions (Q. 14–16) : Read the information given
5. What is value of (gofofogogof) (x) (fogofog) (x) below and answer the question that follows—
the ? The following functions have been defined
(A) x (B) x2
la (x, y, z) = min (x + y, y + z)
5x + 3 (x + 3) (5x + 3)
(C) (D) le (x, y, z) = max (x – y, y – z)

ww 4x – 1 (4x – 5) (4x – 1)
6. What is the value of fo (fog) 0 (gof) (x) ?
(A) x (B) x2
ma (x, y, z) = (1/2) [le (x, y, z) + la (x, y, z)]
14. Given that x > y > z > 0, which of the following is

w
(C) 2x + 3 (D)

.Ea
x+3
4x – 5
Directions (Q. 7–10) : Read the information given
necessarily true?
(A) la (x, y, z) < le (x, y, z)
(B) ma (x, y, z)< la (x, y, z)
(C) ma (x, y, z) < le (x, y, z)

syE
below and answer the questions that follows—
Le (x, y) = least of (x, y), mo (x) = |x|, me (x, y) = (D) None of these
maximum of (x, y) 15. What is the value of ma (10, 4, le (la (10, 5, 3), 5, 3)) ?
7. Find the value of me (a + mo (le (a, b)); mo (a + me
(mo (a) mo (b)))), at a = – 2 and b = – 3—
(A) 1 (B) 0 ngi (A) 7.0
(C) 8.0
(B) 6.5
(D) 7.5
(C) 5 (D) 3
8. Which of the following must always be correct for a, nee
16. For x = 15, y = 10 and z = 9, find the value of : le (x,
min (y, x – z), le (9, 8, ma (x, y, z))—
(A) 5 (B) 12
b>0?
(A) mo (le (a, b)) ≥ (me (a), mo (b)) (C) 9
rin (D) 4

g.n
(B) mo (le (a, b)) > (me (mo (a), mo (b)) Directions (Q. 17–19) : Read the information given
(C) mo (le (a, b)) < (le (mo (a)), mo (b)) below and answer the questions that follows—
(D) mo (le (a, b)) = le (mo (a), mo (b))
9. For what value of a is me (a2 – 3a, a – 3) > 0 ?
(A) 1 < a < 3
(C) a < 0 and a < 3
(B) 0 < a < 3
(D) a < 0 or a < 3
e
The following operations are defined for real number

∇ b = (a + b)a + b if ab is positive else a ∇ b = 1.


17. (2 # 1)/(1 ∇ 2)— t
a # b = a + b if a and b both are positive else a # b = 1. a

10. For what values of a le (a2 – 3a, a – 3) > 0 ? (A) 1/8 (B) 1
(A) 1 < a < 3 (B) 0 < a < 3 (C) 3/8 (D) 3
(C) a < 0 and a < 3 (D) a < 0 or a < 3 18. {((1 # 1) # 2) – (101.3 ∇ log10 0.1)}/(1 ∇ 2)—
Directions (Q. 11) : Answer the questions indepen- (A) 3/8 (B) 4 log10 0.1/8
dently. (C) (4 + 101.3)/8 (D) None of these
11. Largest value of min (2 + x2 , 6 – 3x), when x > 0 is—
(A) 1 (B) 2 19. ((x # – y)/(– x ∇ Y)) = 3/8, then which of the follow-
ing must be true?
(C) 3 (D) 4
(A) x = 2, y = 1 (B) x > 0, y < 0
Directions (Q. 12–13) : Read the information given
(C) x, y both positive (D) x, y both negative
below and answer the question that follows—
A, S, M and D are functions of x and y and they are Directions (Q. 20–22) : Read the information given
defined as follows : below and answer the questions that follows—
A (x, y) = x + y If x and y are real numbers, the functions are defined
S (x, y) = x – y as f(x, y) = |x, y|, F (x, y) = – f(x, y) and G (x, y) = – F (x,

CAT Complete Course | 335

Visit : www.EasyEngineering.net
Visit : www.EasyEngineering.net

y). Now, with the help of this information answer the


following questions—
20. Which of the following will be necessarily true ?
26.
(A) G(f(x, y), F(x, y)) > F (f(x, y), G(x, y))
(B) F (F(x, y), F(x, y)) = F (G(x, y), G(x, y))
(C) F(G(x, y), (x + y) ≠ G(F(x, y), (x – y)))
(D) f(f(x, y), F(x – y)) = G (F(x, y), f(x – y))
21. If y = which of the following will give x2 as the final Directions (Q. 27–28) : Read the information given
value ? below and answer the questions that follows—
(A) f(x, y) G(x, y)4 Certain relation is defined among variable A and B
(B) G(f(x, y)f(x, y))F (x, y)/8 Using the relation answer the questions given below
@ (A, B) = average of A and B
(C) – F(x, y)G(x, y)/log2 16
∴ (A, B) = product of A and B,
(D) – f(x, y) G(x, y) F(x, y)/F(3x, 3y) X (A, B) = the result when A is divided by B
22. What will be the final value given by the function 27. The sum of A and B is given by—

ww
G(f(G(F(f(2, – 3),0) – 2).0)) ?
(A) 2 (B) – 2
(A) /(@ (A, B), 2)
(C) @(X(A, B), 2)
(B) @(/(A, B), 2)
(D) None of these

w
(C) 1 (D) – 1 28. The average of A, B and C is given by—
(A) @(× (/(@(A, B), 2), C), 3)

.Ea
Directions (Q. 23–26) : Read the information given
below and answer the questions that follows—
Any function has been defined for a variable x,
(B) /(x (/(@(A, B)), C2))
(C) X(@(/(@A, B), 2), C, 3))

syE
(D) X(/(@(/(@(A, B), 2), C), 2), 3)
where range of x ∈ (– 2, 2). Directions (Q. 29–31) : Read the information given
Mark (a) if F1 (x) = – F (x) below and answer the questions that follows—
Mark (b) if F1 (x) = F (– x)
Mark (c) if F1 (x) = – F (– x)
ngi x and y are non-zero real numbers
f(x, y) = + (x + y)0.5, if (x + y)0.5 is real otherwise
= (x + y)2
Otherwise mark (d).
nee 2 0.5
g(x, y) = (x + y) if (x + y) is real, otherwise
= – (x + y)

23. greater than g (x, y) ?


rin
29. For which of the following is F (x, y) necessarily

(A) x and y are positive


(B) x and y are negative
g.n
(C) x and y are greater than–1
(D) None of these
e
30. Which of the following is necessarily false ?
(A) f(x, y) ≥ g(x, y) for 0 ≤ x, y < 0.5
(B) f(x, y) > g(x, y) when x, y < – 1
t
(C) f(x, y) > g(x, y) for x, y > 1
24. (D) None of these
31. If f(x, y) = g(x, y) then—
(A) x = y (B) x + y = 1
(C) x + y = – 2 (D) Both (B) and (C)
Directions (Q. 32–33) : Answer the questions inde-
pendent of each other—
32. Which of the following question will be best fit for
above data ?
25.
X 1 2 3 4 5 6
Y 4 8 14 22 32 44
(A) y = ax + b (B) y = a + bx + cx2
(C) y = e ax + b (D) None of these

336 | CAT Complete Course

Visit : www.EasyEngineering.net
Visit : www.EasyEngineering.net

33. If f(0, y) = y + 1, and f(x + 1, y) = f(x, f(x, y)). Then Directions (Q. 42–43) : Read the information given
what is the value of f(1, 2)? below and answer the question that follows—
(A) 1 (B) 2 The batting average (BA) of a test batsman is com-
(C) 3 (D) 4 puted from runs scored and inning played-completed
inning and incomplete inning (not out) in the following
Directions (Q. 34–36) : Read the information given
manner :
below and answer the questions that follows—
r1 = number of runs scored in completed innings
Graph of some functions are given mark the options.
n1 = number of completed innings
(A) If f(x) = 3f(– x)
r2 = number of runs scored in incomplete innings
(B) If f(x) = f(– x)
(C) If f(x) = – f(– x) n2 = number of incomplete innings
(D) If 3f(x) = 6f(– x) for x > 0 r + r2
BA = 1
n1
34.
To better assess a batsman’s accomplishment, the
ICC is considering two other measures MB A1 and MB
A2 defined as follows :
35.
ww MB A1 =
n1 n2
 r r 
  n2 n1  
r + r2
+ max  0‚  2 – 1   : MB A2 = 1
r1 r2
n1 + n2

36. w .Ea
42. Based on the information provided which of the
following is true?
(A) MB A1 ≤ BA ≤ MBA 2
(B) BA ≤ MB A 2 ≤ MB A1

syE
Directions (Q. 37–39) : Read the information given
(C) MB A2 ≤ BA ≤ MB A1
(D) None of these
below and answers the questions that follows—
Follows m and M are defined as follows :
ngi 43. An experienced cricketer with no incomplete innings
has a BA of 50. The next time he bats, the innings is
incomplete and he scores 45 runs. It can be inferred

nee
m (a, b, c) = min (a + b, c, a)
that—
M (a, b, c) = max (a + b, c, a)
(A) BA and MB A 1 will both increase
37. If a = – 2, b = – 3 and c = 2 what is the maximum
between [m(a, b, c) + M (a, b, c)]/2 and [m (a, b, c) –
M (a, b, c)]/2 ?
rin
(B) BA will increase and MB A2 will increases
(C) BA will increase and not enough data is

g.n
(A) 3/2 (B) 7/2 available to assess change in MB A1 and MB A2
(C) – 3/2 (D) – 7/2 (D) None of these
38. If a and b, c are negative, than what gives the mini-
mum of a and b?
(A) m(a, b, c)
(C) m (a + b, bc)
(B) –M (– a, a, – b)
(D) None of these
pendent of each other.
44. If f(x) = log { }1+x
1–x
e
Directions (Q. 44–49) : Answer the questions inde-

, then f(x) + f(y) is : t


39. What is m (M (a – b, b, c)) m (a + b, c, b), – M(a, b, c)
for a = 2, b = 4, c = 3? (A) f(x + y) (B) f {x+y
1 + xy }
(A) – 4 (B) 0 1 f(x) + f(y)
(C) (x + y) f (D)
(C) – 6 (D) 3 1 + xy 1 + xy
Directions (Q. 40–41) : Read the information given 45. Suppose. For any real number x, [x] denotes the
below and answer the question that follows— greatest integer less than or equal to x. Let L (x, y) =
[x] + [y] + [x + y] and R (x, y) = [2x] + [2y]. Then it’s
f(x) = 1/(1 + x) if x is positive = 1 + x is negative or impossible to find any two positive real numbers x
zero f n (x) = f(f n – 1 (x)) and y for which—
40. If x = 1 find f1 (x) f2 (x) f3 (x) f4 (x) …f9 (x)— (A) L (x, y) = R (x, y) (B) L (x, y) ≠ R (x, y)
(A) 1/5 (B) 1/6 (C) L (x, y) < R (x, y) (D) L (x, y) > (R x, y)
(C) 1/7 (D) 1/8 46. Let g(x) = max (50x, x + 2). The smallest possible
41. If x = – 1 what will f(x) be ? value of g(x) is—
(A) 2/3 (B) 1/2 (A) 4.0 (B) 4.5
(C) 3/5 (D) 1/8 (C) 1.5 (D) None of these

CAT Complete Course | 337

Visit : www.EasyEngineering.net
Visit : www.EasyEngineering.net

47. Let f(x) = |x – 2| + |2.5 – x| + |3.6 – x|, where x is real (C) Minimized whenever a > 0, b > 0
number attains a minimum at— (D) Minimized whenever a > 0, b < 0.
(A) x = 2.3 (B) x = 2.5 54. If f(x) = x3 – 4x + p, and f(0) and f(1) are of opposite
(C) x = 2.7 (D) None of these signs, then which of the following is necessarily
48. When the curves y = log10 x and y = x – 1 are drawn in true?
the x – y plane, how many times do they intersects (A) – 1 < p < 2 (B) 0 < p < 3
for values x ≥ 1 ? (C) – 2 < p < 1 (D) – 3 < p < 0
(A) Never (B) Once Directions (Q. 55–56) : Answer the questions on the
(C) Twice (D) More than twice basis of the information given below—
49. Consider the following two curves in the x – y plane; f1 (x) = x 0≤x≤1
y = x3 + x2 + 5; y = x2 + x + 5 which of the following = 1 x≥1
statement is true for – 2 ≤ x ≤ 2 ?
= 0 otherwise
(A) The two curves intersect once
f2 (x) = f1 (– x) for all x
(B) The two curves intersect twice
(C) The two curves do not intersect f3 (x) = – f2(x) for all x
(D) The two curves intersect thrice f4 (x) = f3 (– x) for all x

ww
Directions (Q. 50–52) : Answer the question on the
basis of the table given below—
55. How many of the following products are necessarily
zero for every x :

a
w
Two binary operations ⊕ and * are defined over the

e f g h
.Ea
set (a, e, f, g, h) as per the following tables : (A) 0
(C) 2
f1 (x) f2(x), f2 (x) f3(x), f2 (x) f4(x)?
(B) 1
(D) 3

syE
a a e f g h 56. Which of the following is necessarily true?
e e f g h a (A) f4 (x) = f1 (x) for all x
f f g h a e (B) f1 (x) = – f3 (– x) for all x
g
h
g
h
h
a
a
e
e
f
f
g ngi (C) f2 (– x) = f4 (x) for all x
(D) f1 (x) + f3 (x) = 0 for all x

* a e f
a a a a a a
g h
nee Solutions
Exercise – A
e a e f
f a f
g h
h e g rin
Ans. 1. It is obvious that the domain of this function
includes only those values of x for which the following
g a g e h f
h a h g f e g.n
conditions are simultaneously valid: (a) x > 0, x ≠ 1 (since
the logarithmic base must be positive and non-zero); (b)
cos x > 0 (since negative numbers and zero do not have
Thus, according to the first f ⊕ g–a, while according
to the second table g * h = f, and so on.
Also, let f2 = f* f, g3 = g * g* G, and so on.
50. What is the smallest positive integer n such that gn =
logarithms). '
e t
Solving this system of inequalities, we find that the
domain of the function at hand is the following set of
numbers :
e? π π π
0 < x < 1, 1 < x < , – + 2kπ < x < + 2kπ
(A) 4 (B) 5 2 2 2
(C) 2 (D) 3 where k = l, 2, 3, …(represent it on the number line).
Ans. 2. This function is not defined for those values
51. Upon simplification, f⊕[f*{f⊕(f*f)}] equals— of x for which sin x-cos x =0 (the denominator of the
(A) e (B) f fraction must be different from zero), and, besides, for
(C) g (D) h those x for which sin x-cos x < 0 (because for these values
of x the denominator assumes imaginary values). Thus,
52. Upon simplification, {a10*(f10⊕g9 )} ⊕ e8 equals—
the domain of function (1) consists only of those values
(A) e (B) f of x for which the inequality sin x-cos x > 0 is valid;
(C) g (D) h solving this inequality , we find
53. Let f(x) = a x2 – b |x|, where a and b are constants. π 5π
Then at x = 0, f(x) is— + 2kπ < x < + 2kπ, k = 0, ± 1, ± 2 …(2)
4 4
(A) Maximized whenever a > 0, b > 0 However, it must be further noted that cot x is not
(B) Maximized whenever a > 0, b < 0 defined for x = nπ, Where n is any integer. And so all the

338 | CAT Complete Course

Visit : www.EasyEngineering.net
Visit : www.EasyEngineering.net

values of x = nπ, n = 0, ± 1 ....................... likewise do not sine), the aspirant sometimes sketches the appropriate
belong to the domain of the function and must be graph (sine curve) and then makes the mistake of saying
excluded from the resulting sequence of intervals (2). "this property is evident from the drawing." Such
Thus, for the domain of function (1) we finally get the reasoning is faulty because it is precisely by using the
following set of real numbers : property of the function that one can more or less
π 5π accurately sketch its graph. For this reason, all the
+ 2kπ < x < π + 2kπ, π + 2kπ < x < + 2kπ, properties of a function must be demonstrated by rigorous
4 4
analysis as is done in the standard text book.
K = 0, ± 1, ± 2, ...........
Ans. 3. We consider each summand separately. The Let us examine some problems in which graphs are
domain of this function can only embrace those values of constructed by translation or a specific deformation of the
the argument for which the first term assumes real values, graphs of basic functions.
i.e., those values of x for which the radicand cos (cos x) is Ans. 4. The domain of this function consists of all
non-negative: cos (cos x) ≥ 0. It is easy to see that this real values of x except x = 0. If we consider the function
inequality holds true for all real values of x. Y1 = – 1/x (which is a hyperbola whose branches are
Now, let us examine the second summand. By located in the second and fourth quadrants), it is obvious
definition, the expression arcsin a is meaningful only for that for each value x = x 0 the value of y is greater by 2

ww
|a| ≤ 1; in other words, only those values of x belong to
the domain of function (3) for which |(1 + x2)/2x| ≤ 1.
However, it may be proved directly that the inequality |(1
than the value of y 1 for the same value x0 of the argument.
It therefore suffices to translate the graph of the function

w
+ x 2 )/2x| ≥ 1. holds for all non-zero real values of x,

.Ea
equality being achieved only when x =1 and x = – 1.
Consequently, the domain of (3) consists of two
points only : x = – 1 and x = l.
y1 2 units upwards along the axis of ordinates to get the
desired graph of the function y (Fig. 1).
It is easy to see that this device enables us to
construct the graph of the function y = a + f(x), where a is

syE
The foregoing examples show that in finding the
domain of definition of a function one has to invoke
a given number, if we have already constructed the graph
of the function y1 = f(x): it is sufficient to translate the
graph of the function y1 a units upwards if a > 0 or |a|
various branches of algebra and trigonometry. Only when
these sections are fully mastered can the aspirant tackle
such problems with ease. ngi units down if a < 0.

The aspirant should have a firm knowledge of the


definitions and be able to investigate such general nee
rin
properties of functions as bounded ness, monotonicity
(intervals over which a function is increasing or
decreasing), evenness and oddness, periodicity, and be
able to find the range of a function, its zeroes, external
values, and the like. g.n
The investigation of the properties of functions is
carried out without invoking the concept of a derivative,
which is an element of mathematical analysis and is
outside the school curriculum.
Fig. 1
e t
The aspirant should have a clear idea of a system of
co-ordinates in the plane and be able to sketch, by
memory, the graphs of the basic functions y = k x + b
(straight line); y = ax 2 + b x + c (parabola); y = k /x
(hyperbola); y = |x – a|; y = x3; y =√  x ; y = 1/x2; y = ax
(a > 0, a ≠ 1); y = loga x(a > 0, a ≠ 1); y = sin x (sine
curve); y = cos x; y = tan x; y = cot x. The aspirant should
Fig. 2
be able to sketch the graphs of these functions in each
concrete case, giving a general picture and the cha- Ans. 5. Evidently, x can assume all values except – 4.
racteristic peculiarities of behaviour of the curve and not Compare this function with the function y 1 = 3/x. It is
be forced to compute each time a table of values and plot clear that the value of the function y corresponding to
the curve. some value x = x0 coincides with the value of y1, which
The aspirant should also be able to illustrate corresponds to the value of its argument equal to x0 + 4.
geometrically on the graph the properties of a function. For example, the function y = 3/(x + 4) when, x0 =1, takes
When relating some property (say, the oddness of the the value y = 3/5, and the function y1 = 3/x assumes this

CAT Complete Course | 339

Visit : www.EasyEngineering.net
Visit : www.EasyEngineering.net

very same value for the value of its argument equal to 5 =


x0 + 4. And so if we translate the graph of the function y1
four units to the left along the x-axis, we get the graph of
the function y that we want (Fig. 2).
It is not so hard to figure out that this same method
enables one to draw the graph of the function y = f(x + b),
where b is a given number, if we already have the graph
of the function y1 = f(x) : it suffices to translate the graph
of y 1 b units leftwards if b > 0 or |b| units rightwards if b
< 0.
Ans. 6. To draw the graph, first transform the fraction
and represent the function as Fig. 3
1 13/9 Ans. 8. The aspirant is not always able to give a
Y = – +
3 x – (2/3) proper representation of all three curves in a single
Arguing as in Problems 4, and 5, we see that the drawing and correctly to indicate their mutual positions

ww
graph of the proposed function is an "ordinary" hyperbola
y = (l3/9)/x translated 2/3 unit rightwards along the x-axis
and 1/3 unit down along the y-axis (make the drawing!).
(Fig. 4), to indicate the peculiarities of each of the sine
curves and to explain how they are obtained one from
another.

w
A similar device permits drawing the graph of any
function

Y = .Ea
ax + b
cx + d

syE
This is called a linear fractional function. Indeed, a
simple transformation permits writing this function as

a
Y = +
ad – bc
c2 ngi Fig. 4
c
x+
d
c
It then remains to invoke the arguments given above
nee For one thing, it is useful to remember that the
smallest positive period of the function y = A sin ωx,
in solving Problems 4 and 5.
Note that in the very same way, by combining rin
where ω ≠ 0 and A ≠ 0 are given numbers, is equal to 2
π/|ω| (for instance the smallest positive period of the
function y = – 3 sin π x is the number 2π/π = 2, and for
remarks pertaining to Problems 4 and 5, we can readily
represent the graph of a function y = a + f(x + b), where a g.n
the function y = 1/4 sin (– x/3) is the number 2π/|– 1/3| =
6π), while its "amplitude" is equal to |A| (thus, the
and b are specified numbers, if the graph of the function
y1 = f(x) has already been constructed.
Ans. 7. Sometimes aspirant gives answer like this :
that it is
e
"amplitude" of the function y = – 1/2 sin 3x is 1/2).
The foregoing of course refers also to all the other
trigonometric t
functions. It is important to stress the fact
possible to construct the graph of the function y
"The graph of the function does not exist since negative
= A f(ωx) where ω ≠ 0 and A ≠ 0 are given numbers, if
numbers do not have logarithms." The mistake here is the we know the graph of the function y = f(x). First
failure to grasp the fact that – x does not by any, means compress the graph of y along the x-axis1ω times if ω >
1
always represents a negative number. 0; but if ω < 0, then compress the graph of the function y1
The domain of the function y under consideration is |ω| times along the x-axis and perform a reflection with
the set x < 0. It is immediately clear that the value of this respect to the y-axis (see the solution of problem 7). Then
function, when x = – x0 , x 0 > 0, coincides with that of the take the resulting curve and stretch it along the y-axis A
function y 1 = log4 x for the value x0 of its argument. times if A > 0; but if A < 0, then perform an |A| -fold
Hence, to obtain the graph of the function y it is sufficient stretching along the y-axis and a reflection about the x-
to reflect the graph of the function y 1 about the y-axis axis. Of course, if |ω| < 1, then the compression along the
(Fig. 3). x-axis is actually a stretching; in the same way, the |A|
It will be noted that this same device permits -fold stretching along the y-axis for |A| < 1 is actually a
constructing the graph of the function y = f(– x) if we compression.
have the graph of the function Y1 = f(x); it suffices to Note particularly an important special case : if the
reflect the graph of y1 about the y-axis. graph of the function y1 = f(x) has been sketched, the

340 | CAT Complete Course

Visit : www.EasyEngineering.net
Visit : www.EasyEngineering.net

graph of the function y = – f(x) can be obtained from it by It is sometimes useful to first transform the formula
a reflection about the x-axis. defining the functional relation; then the graph is readily
drawn. In particular, it is always desirable to represent a
complicated functional relationship as an easily sur-
veyable combination of elementary functions, the graph
of which combination is obtainable by familiar tech-
niques (that was precisely how we constructed the graph
in Problem 6).

Fig. 5
Ans. 9. Representing the given function in the form
y = sin 2 [x – (n/6)], we see immediately that for every Fig. 7
value X = X0 the value of y coincides with the value of

ww
Y1 = sin 2x, which corresponds to the value X0 – (π/6) of
its argument. And so to construct the graph of y, draw the
graph of y 1 and then translated it π/6 units rightwards
Ans. 10. Since, this function may be written as Y = 1/2
– 1/2 cos 2x, the graph of the function Y is obtained by
familiar techniques: the cosine curve y1 = – 1/2 cos 2x,

w
along the x-axis (Fig. 5).

.Ea
A very common mistake in constructing the graph of
the function y is as follows : the graph is drawn of the
which is constructed by the technique described in the
solution of Problem 8, must be translated 1/2 unit upwards
(Fig. 7).
Ans. 11. Employing familiar formulas involving

units along the x-axis.


syE
function y1 and it is then translated rightwards by π/3 logarithms, we see that x 1/log10x = x logx 10 = 10, whence
aspirants often conclude immediately that the graph of
the function (4) is the straight line Y = 10.

ngi This conclusion is incorrect however. It is necessary


to take into account the domain of definition of the

nee
function and the conditions under which the transforma-
tions that are carried out are legitimate.
The domain of the function (4) consists of the real

rin
numbers which satisfies the conditions : x > 0, x ≠ 1.
Under these conditions, it is legitimate to carry out the

Fig. 6 g.n
transformation indicated above. And so graph of the
function (4) is the half-line y = 10, x > 0 with the point
(1·10) deleted (Fig. 8).
It is easy to see that this construction is incorrect,
because the graph crosses the x-axis at the point π/3
(since the graph of the function y 1 cuts this axis at the
origin and is then translated π/3 units rightwards!). Yet
e t
the value of the function y is clearly non-zero for the
value of the argument x = π/3.
The technique used in this specific instance enables
one to construct the graph of any function of the form y = Fig. 8
A sin (ωx + ϕ), y = A cos (ωx + ϕ) etc., and also y = a sin The arrowhead at any point indicates, that point does
ωx + b cos ωx.. not belong to the graph).
This technique is of a general nature and permits Ans. 12. First of all, perform an identity transforma-
obtaining the graph of a function y = f (ωx + ϕ), where ω tion of the second summand:
≠ 0 and ϕ are specified numbers, if the graph of the
function y 1 = f(x) has already been drawn : it is sufficient log2 
√
4x2 – 4x + 1 = log2  √
(2x – 1)2
to draw the graph of the function y2 = f (ωx) (it may be = log2 | 2x – 1|
obtained the method indicated in the solution of Problem
8) and then to translate it along the x-axis rightwards by
| |
= 1 + log2 x –
1
2
an amount |ϕ/ω| if ϕ/ω < 0 leftwards by ϕ/ω if ϕ/ω < 0 It is now clear that the domain of the function y is the
(see Problem 5). set x > 1/2 (because the second term in the formula

CAT Complete Course | 341

Visit : www.EasyEngineering.net
Visit : www.EasyEngineering.net

defining this function is (Fig. 8) meaningful for all x ≠ techniques indicated in the solution of problems 6 and 13.
1/2, while the first is meaningful only for x > 1/2). Indeed, taking the graph of the function y1 = |x | (Fig. 11),
However, the equation log1/2 (x – 1/2) = – log2 (x –
1/2) is true for x > 1/2, and, hence, in its domain (x > 1/2)
function (5) can be written y = 1.
Thus the graph of the function y is the ray y = 1, x >
1/2 (Fig. 9) the arrowhead at the point (1/2, 1) signifies
that this point does not belong to the graph of the function
(5)].

Fig. 11
translate it one unit leftwards along the x-axis and two
units Downwards along the y-axis. This yields the graph
of the function y2 = |x + 1| – 2. Then replace the portion
of the graph below the x-axis corresponding to – 3 ≤ x ≤

ww Fig. 9
Aspirant often find it difficult to construct graphs of
1, by the portion symmetric to it about the x-axis. The
resulting polygonal line is the graph of the function y.
The general technique for constructing the graph of a

w
functions whose analytic expressions involve the absolute- function whose analytical expression contains an absolute-
value sign. The next few illustrations illustrate how the value sign consists in rewriting the expression of the

.Ea
graphs of such functions are constructed. functional relationship without using the absolute value
Ans. 13 First note that the proposed function can sign. In this case, the functional relationship on different

syE
obviously be written in the form y = | 2x – 2 | portions of variation of the argument is, as a rule,
Consider the auxiliary function y1 = 2x – 2, the graph described by different formulas. Quite naturally, on each
of which is readily drawn (by the technique described in of these portions, the graph must be constructed on the

ngi
the solution of Problem 4). How does the graph of the basis of the appropriate formula.
function y differ? Ans. 15. To get rid of the absolute-value sign, con-
Recall the definition of absolute value; from this sider separately two cases. x ≥ 0 and x < 0. If x ≥ 0, then
definition it follows that
2x – 2 for values of x for which 

 2x – 2 ≥ 0 that is‚for x ≥ 1‚  
nee
y = x 2 – 2x – 3. It is easy to draw this parabola, then we
take that portion which corresponds to non-negative
values of x. But if x < 0, then y = x2 + 2x – 3. Draw this
Y = – (2x – 2) for values of x for which 
 2x – 2 < 0‚ that is‚ for x < 1
 rin
parabola and take that portion which corresponds to
negative values of x. Taken together, the two pieces of the

g.n
It is then clear that the graph of the function y, for x ≥ parabolas constitute the graph that interests us (Fig. 12).
1, coincides with the graph of the function y1 and, for x <

e
1, is a curve symmetric to the graph of the function y1
with respect to the x-axis (Fig. 10).

t
Fig. 12
Fig. 10
Ans. 16. By the definition of absolute value we can
In precisely the same way we can obtain the graph of
represent this function in the form
the function y = |f (x)| if the graph of the function y1 = f (x)
 [(x + 1) + 1] (x – 3) = (x + 2)(x – 3) if x ≥ – 1‚ 
has been drawn. It suffices (Fig. 10) to replace the portions y =  [– (x + 1) + 1] (x – 3) = – x (x – 3) if x > – 1  .
of the graph of y 1 lying below the x-axis by corresponding  
portions symmetric with respect to the x-axis [to find It now remains simply to sketch the curve, using the
these portions we have to solve the inequality f (x) < 0]. appropriate formula, for each of the indicated intervals
Ans. 14 Here, without dropping the absolute-value (x ≥ – 1 and x < – 1). Together, the two curves yield the
signs, we can carry out the construction using the graph of function (6).

342 | CAT Complete Course

Visit : www.EasyEngineering.net
Visit : www.EasyEngineering.net

Let us first consider the function y1 = (x + 2) (x – 3). The subsequent construction now follows familiar
Ordinarily, aspirants remove the brackets and perform a techniques (Fig. 14).
rather lengthy procedure of isolating a perfect square,
whereas it is better not to remove the brackets because it
is at once clear that this is a parabola, the graph of a
quadratic trinomial; the parabola intersects the x-axis at
the points A (– 2, 0) and B (3, 0) (because – 2 and 3 are
the roots of the trinomial) and its branches are directed
Fig. 14
upwards (since the leading coefficient is positive).
It will be noted that if x increases without bound, the
Substituting the value x = 0 into the formula for the
graph of the function (7) approaches without bound the
function y1, we get the co-ordinates of the point C of
straight line y = 1, remaining all the time below it; but if x
intersection of this parabola with the axis of ordinates
decreases without bound, then the graph approaches the
(y-axis): C (0,– 6). It is also easy to find the co-ordinates
same line without bound, remaining all the time above it.
of the vertex D of this parabola. Since, the parabola is
Ans. 18. When drawing the graph of a periodic function
symmetric about the vertical straight line passing through
it is often helpful to know that all values of such a func-
the vertex, its axis of symmetry bisects the line segment
tion are repeated in every period. Thus, if a periodic func-
AB. It is therefore clear that the abscissa of the vertex is

ww
equal to 1/2; the ordinate is computed directly and we get
D (1/2, – 25/4).
tion is given with period T, then it is sufficient to construct
the graph on some segment of length T; for 0 ≤ x ≤ T, the
portions of the graph on the intervals T ≤ x ≤ 2T, 2T ≤ x

w
Having constructed the parabola—the graph of the
function y1—we must isolate that portion which corre-

.Ea
sponds to the values x ≥ 1 of the argument (Fig. 13).
≤ 3T, – T ≤ x ≤ 0, etc., will have the very same shape.
It is clear that the number 2π is the period of the
function y under consideration so that we can confine
ourselves to the interval 0 ≤ x ≤ 2n. Partitioning this

syE interval into four parts in each of which both sin x and
cos x preserve sign, we get

√ 2 sin (x + 4π) if 0 ≤ x ≤ π4‚ 


ngi   2 sin (x – 4π) if π2 ≤ x ≤ π‚ 

nee Y =
– √ 2 sin (x + 4π) if π ≤ x ≤ 3π2 ‚
– √ 2 sin x – π if 3π ≤ x ≤ 2π 
rin
( 4) 2
Fig. 13 (π/4)] and y2 =  g.n
We now construct the graphs of y 1 = √ 2 sin [x +
√ 2 × sin [x – (π/4)] and sin [x – (π/4)] and
The construction of the graph of the function y2 = – x
(x – 3) is similar. Take only that portion of the parabola
which corresponds to the values x < – 1 of the argument.
The graph of function (6) is shown in Fig. 13 by the solid
e
then we take the portion of the curve y1 on the interval
from 0 to π/2, the portion of the curve y2 on the interval
from π/2 to π; and on the intervals from π to 3π/2 and
from 3π/2 to 2π, we take the curves that are symmetric, t
line. about the x-axis, to the corresponding portions of the
Ans. 17. We first find the values of x for which each curves y1 and y2. Then, taking advantage of periodicity,
of the expressions under the absolute-value sign vanishes; we extend the resulting curve beyond the interval 0 ≤ x ≤
they are – 3, – 1, 1, 3. By considering function (7) on 2π (the solid line in Fig. 15).
each of the five intervals into which these values partition
the number line, we obtain the following notation:

 1 – x +2 1 if x<–3

 – x2 + 12 if – 3 ≤ x < – 1,

Y=  1 if – 1 ≤ x < 1, 
 x+1 2 1 ≤x<3 
 1– 2 
if

 x+1 if 3≤x  Fig. 15

CAT Complete Course | 343

Visit : www.EasyEngineering.net
Visit : www.EasyEngineering.net

It is clear from this graph that π/2 is the period of the


given function so that we were overcautious in consider-
ing the interval from 0 to 2π. If we had realized from the
very start that π/2 is the period of this function-and this is
easy to demonstrate :
| ( )
sin x +
π
2 ( )|
+ cos x +
π
2
= |cos x | + |sin x |
then the graph could be constructed much faster. This
example shows that a careful preliminary analysis of the
properties of a given function very often appreciably
simplifies the construction of the graph.
Ans. 19. At first glance this function might appear to
be very complicated. However, by transforming the
formula defining the given function we obtain a simpler
notation for (8) that will permit drawing the required
graph with comparative ease. Fig. 17
First of all, note that the domain of the function (8) is It is easy to figure out the shape of the graph of the

ww
the entire number line with the exception of the points x =
nπ/2, where n is any integer (at each of these points,
either tan x or cot x becomes meaningless).
function on the positive x-axis: for each value x > 0, the
corresponding ordinate of the straight line y1 = x has to be
increased by an ordinate of the hyperbola y 2 = 1/x corre-

w
Since for x ≠ nπ/2 the equations

√ 1 + tan2 x =

.Ea
1 ,
| cos x |
√
1 + cot2 x =
1
| sin x |
are true, it is clear that the function (8) can, in its domain
sponding to the same value of x. It is quite obvious (Fig.
45) that for a positive x tending to zero, the expression x
+ (1/x) tends to + ∞ (increases without bound), and for x
tending to + ∞, the desired graph approaches the bisector
of definition, be written as

syE
y = sin x. | cos x | + cos x. | sin x |
This is a periodic function with period 2π. The graph
y1 = x without bound, since the summand 1/x becomes
smaller and smaller. It is easy in this case to determine
the smallest value of the function y (recall that so far we

ngi
can be constructed as was done In Problem 18. It is
shown in Fig. 16. are only considering positive values of x) : indeed, when
x > 0 the inequality x + (l/x) ≥ 2 holds true, which is to say

nee
the smallest value is equal to 2 and is reached when x = 1.
Construction of the graph is similar on the negative
x-axis as well. Incidentally, we could take advantage of

rin
the fact that the function y is odd and, hence, its graph is
symmetric about the origin.
Ans. 21 Take advantage of the fact that the formula
Fig. 16
Note once again, however, that the function (8) is not g.n
defining this function is a product and we apply a
technique called multiplication of graphs. The required
defined at the points x = nπ/2, n = 0, ± 1, ± 2, …… In the
figure this is indicated by arrowheads the endpoints of the
segments of the curve. We will now consider some
instances of the construction of complicated in which the
e
graph will be constructed by "multiplying" two auxiliary
graphs y1 = x and y 2 = sin x. In other words, for each
value of the argument, the corresponding ordinate y is
constructed as a product of the ordinates y1 and y2 whicht
foregoing elementary devices do not suffice. Each of correspond to the same value of the argument (Fig. 18).
these examples has its peculiarities that must be taken
into account when sketching the graph. In solving
problems like those given below, one often has to reason
in quite an unorthodox manner.
So to speak. The approach should be to get onto an
item that will give some clue to the construction.
Ans. 20. Representing the given function as y = x +
(1/x), we apply a technique called addition of graphs,
which means that the desired graph is constructed by
"combining" two auxiliary graphs, y1 = x and y2 = 1/x. In
other words, for each admissible value of the argument
(that is, for every x ≠ 0) the corresponding ordinate y is
built up as an algebraic sum of the ordinates y 1 and y2
corresponding to the same value of the argument (Fig.
17). Fig. 18

344 | CAT Complete Course

Visit : www.EasyEngineering.net
Visit : www.EasyEngineering.net

We first construct the graph of the function y for It is easy to demonstrate that the inequality 0 < y < 1 is
non-negative value; of the argument. For each value of x true on the negative x-axis. Using similar reasoning, we
we multiply the value of the corresponding ordinate of construct the graph of the function y for x < 0 as well
the straight line y 1 = x and the value of the ordinate of the (Fig. 19). The arrowhead at the origin indicates that the
sine curve y 2 = sin x , and are thus able to construct a origin does not belong to the graph).
smooth curve that gives an approximate idea of the Ans. 23. If we represent this function as
behaviour of the graph of the function y on the non- y = 1 + (– 2). 2sin (x + 1) …(9)
negative x-axis .The aspect of the we can be improved then it is clear that having the graph of the function y1 =
somewhat by plotting a few characteristic points. First of
2 sin x it is easy to obtain the graph of the function y by
all, it is clear that y = 0 for those values of x for which
means of techniques discussed in the solution of
since x = 0, and so the graph of the function y crosses the
Problems 4, 5, and 8.
positive x-axis at the points x = kπ, k = 0, 1, 2,…
Furthermore, for x > o the obvious inequality – x ≤ x sin x And so let us first tackle the graph of the function
≤ x holds true; this means that for Fig. 18. Positive values Y1. This is a periodic function with period 2π; therefore,
of the argument the graph of the function y does not all we need to do is draw the graph on the interval 0 ≤ x ≤
extend above the straight line y = x or below the straight 2n (see Problem 18).
line y = x. or the below the straight line y = – x. In this For x = 0 the function y1 assumes the value 1. If x is

ww
case, the points of the graph of the function y that
correspond to the values of x > 0 for which sin x = 1 i.e.,
increased from 0 to π /2, then sin x monotonically
increases from 0 to 1 and 2sin x monotonically increases
from 1 to 2.If x then increases from π/2 to 3π/2, then sin x

w
to the values x = (π/2) + 2kπ, k = 0, 1, 2,…… lie on the
straight line y = x, and the points corresponding to the monotonically decreases from 1 to – 1 and 2sin x

.Ea
values of x > 0 for which sin x = – l, i.e., to the values x =
(3π/2) + 2kπ, k = 0, 1, 2,… , lie on the straight line y = – x.
monotonically decreases from 2 to 1/2; in particular, for x
= π the function y1 takes on the value 1. Finally, if x
increases from 3π/2 to 2π , then sin x monotonically

syE
Ans. 22. Here we have to construct the graph of a
function. To graph such a function, the aspirant must
know the properties of the basic elementary functions and
increases from – 1 to 0 and 2sin x monotonically increases
from 1/2 to 1: the function Y1 has the value 1 for x = 2π.
All these statements about the behaviour of the function

ngi
have a clear-cut idea of the consequent properties of
combinations of these functions. y1 follow from the properties of a sine function and an
exponential function (it is left to the reader to put a
The domain of the function y consists of all real
numbers except x = 0. Since, for x > 0 the exponent 1/x >
0, it follows that y > 1 for all positive values of the
argument, by the property of an exponential function.
nee
rigorous foundation under these statements). They permit
determining the approximate behaviour of the graph of
the function y1 when 0 ≤ x ≤ 2π, it then remains to extend

Note that y = 2 when x = 1. If x increases without


bound, then the expression l/x decreases to zero mono- rin
the resulting curve periodically over the entire x-axis (in
Fig. 20, the graph of y1 is shown as a dashed line).

tonically, remaining, and so 2 1/x decreases to unity mono-


tonically, remaining greater than unity (by the property of g.n
an exponential function). When x is positive and tends to
zero, the exponent 1/x increases without bound and,
hence, 21/x also increases without bound. This enables us
to sketch a graph of the function y when x > 0.
e t
Fig. 20
Everything is now ready for the construction, first
translate the graph of the function Y 1 one unit leftwards
along the x-axis; this yield a curve which is the graph of
the function y 2 = 2 sin(x + 1) (see Problem 5). It is also a
periodic function (with period 2π). It has a maximal value
of 2 which it assumes at the points x = (π/2) – 1 + 2kπ, k
= 0, ± 1, ± 2, …, it has a minimal value equal to 1/2
which it takes on at the points x = – (π/2) – 1 + 2 kπ, k =
0, + 1, ± 1, ± 2, …… (Fig. 20). Stretching the curve y2 by
Fig. 19 a factor of 2 along the y-axis and then reflecting it about

CAT Complete Course | 345

Visit : www.EasyEngineering.net
Visit : www.EasyEngineering.net

the x-axis, we construct the graph of the function y3 = (– 2). π π


2kπ < x < + 2kπ, + 2kπ < x < (2k + 1)π
2sin (x + 1) (See Problem 8). Note that this periodic function 2 2
has a maximal value of – 1 and a minimal value of – 4. k = 0, ± 1, ± 2, ……
(Fig. 20). Finally, the graph of the function y is obtained The function y is clearly periodic with period 2π.
by translating the curve of Y3 up one unit on y-axis (see And so we can confine ourselves to an interval of length
Problem 4). 2π, say, the interval 0≤ x ≤ 2π. (Fig. 21)
The graph (solid line in Fig. 20) conveys the basic But not the whole of this interval lies in the domain
features of the behaviour of the function y. This is a of the function : the function is meaningful (over this
periodic function (with period 2π) which vanishes at the interval) only for 0 < x < π/2, π/2 < x < π. It is precisely
points x = – (π/2) – 1 + 2kπ, k = 0, ± 1, ± 2… (with 0 the on these intervals that we first of all have to construct the
maximal value) and assumes a minimal value of – 3 at the graph (then we can simply extend it over the entire
points x = (π/2) – 1 +2kπ, k = 0, ± 1, ± 2, … The function domain because of its periodicity).
y varies monotonically in between the external values.
It will be seen that the function y can, in its domain,
When x = 0 the function y is equal to 1 – 21 + sin1 (note
be rewritten as
that sin 1 is the sine of an angle of one radian!)
1
Of course, Fig. 20 gives only a rough idea of the Y = …(10)
log1/2 sin x

ww
graph of the function y, but that is as much as is ordinarily
required at an examination.
Ans. 24. First draw the graph of the auxiliary function
We first of all construct the graph of the auxiliary
function y1 = log 1/2 sin x. It will only interest us over the
interval 0< x < π. Taking the piece of the sine curve y2 =

w
y1 = 1 – x2. This parabola is shown in Fig. 21

.Ea
sin x corresponding to this interval, we can use the same
method as in the preceding problem (don't forget that the
base of the logarithm 1/2 < 1) to obtain the graph of the
composite function y1 (Fig. 22),

syE
ngi
nee
rin
g.n
Fig. 21 Fig. 22
the auxiliary graphs y 1 and y2 are depicted by dashed
by the dashed line. It is then necessary to construct the
lines).
graph of the logarithm of this function.
For x = 0 we have y = log2 1 = 0. If x is increased
from 0 to 1, then as may be seen from the graph of the
auxiliary function, 1 – x 2 decrease from 1 to 0 and so
any
the
e
We now consider the interval 0 < x < π/2. Since for
value of x in this interval, the corresponding value of
function
corresponding
y is the reciprocal of the value of y1
to the same value of the argument [see
t
log2 (1 – x2) decreases from 0 to – ∞.
(10)], it is easy to obtain a rough sketch of the graph of y
Similarly, if x decreases from 0 to – 1, then 1 – x2 for 0 < x < π/2 (the solid line in Fig 22; the arrowhead on
decreases from 1 to 0 and log2 (1 – x 2 ) decreases from 0 the curve at the origin indicated that this point does not to
to – ∞. For the remaining values x, that is, for x ≤ – 1 and the graph).
x ≥ 1, we have 1 – x2 ≤ 0 so that log2 (1 – x2 ) is meaning-
It is easy to prove that by using familiar properties of
less. The graph of the function y is shown in Fig. 21 as a
elementary function the function y monotonically
solid line.
increases when x varies from 0 to π/2; if x increases from
Note that in the construction of this graph we did not 0 to π/2, then sin x increases monotonically from 0 to 1,
start out by finding domain of the function, which was and then log 1/2 sin x decreases monotonically from + ∞ to
obtained almost automatically. A preliminary determina- 0; and hence, [see (10)], the value of y increases
tion of the domain of a function is frequently very useful, monotonically from 0 to + ∞ . Let us stress that if x
however. approaches π/2, remaining all the time less than this
Ans. 25. The domain of this function is the collection value, then the value of the function y 1 tends to zero,
of all values of x for which, simultaneously, sin x > 0 and remaining all the time positive, and therefore the value of
sin x ≠ 1, that is, the set y increases without bound. But if x approaches zero and

346 | CAT Complete Course

Visit : www.EasyEngineering.net
Visit : www.EasyEngineering.net

remains positive , then the value of y1 increases without obtain the x-intercept of this graph : all you have to do is
bound and so the value of y tends to zero (although it solve the equation sin x 2 = 0. It is clear that the
does not take on the value 0). nonnegative roots of this equation are the numbers x =
The construction is similar for the graph of the kπ, k = 0, 1, 2,……

function at hand when π/2 < x < π.
On the negative x-axis, the graph is drawn at once
It is useful to note that just about the same reasoning since the function y is even.
as is used, in construction of the graph of the function y We conclude this section with some problems of a
on the basis of the Graph of the auxiliary function y1 different nature, but also connected with graphical con-
enables us to construct the graph of y = l/f (x) if the graph struction in the plane with a specified co-ordinate system.
of the function y1 = f(x) is known.
Ans. 27. From the first inequality we have y ≥ – 5x/3.
Considering Fig. 22 in more detail, we note that we We first of all graph the function y = – 5x/3 (Fig. 24).
did not obtain a complete description of the behaviour of
the graph of the given function in the foregoing solution
(for instance, the fact that this graph is bent in the specific
way as it is shown in the figure was not even discussed).
But this is not required since it goes beyond the scope of

ww
the elementary means at the disposal of the aspirant: And
a rough Sketch of the graph can, as we have seen, be
made with relative ease.

w
True, the shape of the curve could be improved a bit

.Ea
by computing a table of values of the function for
"convenient" values of the argument and taking these into

syE
account when drawing the curve. As a rule, examination
questions do not require such improvement. The
important thing is to be able to sketch a curve that
conveys the general aspect and characteristic features of
the graph.
Ans. 26. First of all, do not confuse the notation sin
ngi Fig. 24
Then the points whose co-ordinates satisfy the
x2 with sin 2 x : the former means sin (x2 ), the latter means
(sin x)2. nee
equation
the points
y = – 5x/3 lie on the constructed straight line and
whose co-ordinate y exceeds – 5x/3 will lie
above this line. Thus, the set of points whose co-ordinates

rin
Let us first consider the non-negative values of the satisfy the first inequality of (11) will constitute the half-
argument and partition the semi-axis x ≥ 0 into intervals Plane lying above the straight line y = – 5x/3 (the straight

increases from 0 to π/2 (which is to say that x increases hatching).


from 0 to √ π/2, then sin x2 increases from 0 to 1; if x2 g.n
over which the function y increases or decreases. If x2 line included; in Fig. 24 this region is denoted by vertical

e
Similarly, from the second inequality of (11) we
increases from π/2 to 3π/2 (i.e., x increases from 
to √


to 
√
3π/2), then sin x 2 decreases from 1 to – 1; if x 2
increases from 3π/2 to 5π/2 (i.e., x increases from 
√π/2

√
have

half-plane lying below the straight line y = 2x + 2 (the


3π/2 line itself is not included; in Fig. 24 this region is
5π/2), then sin x2 increases from – 1 to 1, and so on. indicated by horizontal hatching).
t
y < 2x + 2 so that the set of points whose co-ordinates
satisfy the second inequality of (11) will constitute the

The graph of the function y therefore is of a wavelike Hence, the points of the plane whose co-ordinates x
nature with amplitude of 1 (Fig. 23). It is easy to and y satisfy of inequalities (11) lie in the common
portion (intersection) of the two resulting half-planes; this
is an angular region (in Fig. 24 the desired set is indicated
by double cross-hatching). Here, one of the bounding
rays of the region-a piece of the straight line y = – 5x/3 –
is included in the sought for set, while the other—a piece
of the straight line y = 2x + 2 – is not included (the vertex
A of the angular region, the intersection point of the
straight lines y = – 5x/3 and y = 2x + 2, does not belong to
this set either).
Ans. 28. As in the solution of other problems involv-
ing absolute values, it is useful first of all to attempt to
Fig. 23 get rid of the absolute-value sign.

CAT Complete Course | 347

Visit : www.EasyEngineering.net
Visit : www.EasyEngineering.net

To do this (see Fig. 25), construct straight lines in the form x + y = – y – x, or x + y = 0. This latter equation is
clearly satisfied by those points of Region IV which lie
on the bisector of the fourth quadrant.
Thus, in the plane, the set of points whose co-
ordinates x and y satisfy relation (12) is the angular
region between the negative x-axis and the bisector of the
fourth quadrant (including the bounding rays) and the
positive y-axis (Fig. 25).
Ans. 29. Note right off that x and y which satisfy
condition (13) are such that x > 0, y > 0, x ≠ 1 and y ≠ 1.
Since the properties of logarithms are different for bases
that exceed unity or are less than unity, it is natural to
consider two cases.
(a) Let x > 1. then by the properties of logarithms,
the inequality (13) will hold true if the inequality logy x >
1 is valid. It will be recalled the logarithms of numbers
greater than unity to a base less than unity are negative.

ww Fig. 25
plane: x + y = 0 (the bisector AOB of the second and
fourth quadrants) and y = 0 (the axis COD of abscissas).
And so the inequality logy x > 1 cannot be valid for y in
the interval 0 < y < l.
Thus, the inequality logy x > 1 can hold only when y

w
Clearly, the co-ordinates x and y of any point above the
straight line x + y = 0 satisfy the inequality x + y > 0, and

.Ea
for any below this line the inequality x + y < 0 holds.
Similarly, any point in the upper (relative to the x-axis)
> l. But if y > 1, then all x > y will be solutions to the
inequality logy x >1.
Thus, if x > 1, then for inequality (13) to hold, y must
be greater than unity : y > 1, and the original inequality

syE
half-plane has a positive ordinate, and any point in the
lower half-plane has a negative ordinate.
These straight lines partition the plane into four
will be satisfied by those points for whose co-ordinates
the condition x > y is also valid.
If we depict the set of these points in a drawing, it
regions (Fig. 25) and it is clear that in each one of these
regions the expressions x + y and y preserve sign for all
ngi will be seen that the set is the interior of the angular
region CBD (Fig. 26).

nee
points (x, y). It is therefore advisable, in each of these
regions, to seek separately the points whose co-ordinates
x and y satisfy the relation (12).
For any point (x, y) of Region I (the angular region
DOA including the bounding rays as well) we have the
rin
inequalities x + y ≥ 0, y ≥ 0. Hence, in Region I relation
(12) takes the form x + y = y – x, or x = 0. But this last
equation is satisfied by the co-ordinates of the point of g.n
the positive y-axis (by no means all the points of the
y-axis, since we are only interested in those points which
lie in Region I, and the negative y-axis does not belong to
this region).
e t
For any point of Region II (the angular region AOC;
of the boundary rays only the ray CO is included) we
Fig. 26
have the inequalities x + y < 0, y ≥ 0, and for this reason
(b) Now let 0 < x < 1. Reasoning in similar fashion,
relation (12) takes the form – (x + y) = y – x, or y = 0, in
we find that condition of the problem is satisfied by
Region II. This latter equation is satisfied by the points of
points for whose co-ordinates the conditions 0 < y < 1 and
the negative x-axis (the other points the x-axis do not lie
y < x are fulfilled. The set of these points is the interior of
in Region II).
the triangle AOB (Fig. 26).
The inequalities x + y < 0, y < 0 hold for any point of Consequently points whose co-ordinates satisfy
Region III (the angular region COB excluding the bound- inequality (13) from the region cross- lined in Fig. 26 [the
ing rays), and so relation (12) in Region III assumes the co-ordinates of the boundary points of this region do not
form – (x + y) = – y – x, or 0 = 0. This means that the co- satisfy relation (13)].
ordinates of any point of Region III satisfy relation (12). Ans. 30. Using a familiar trigonometric identity,
Finally, for any point of Region IV (the angular rewrite the given inequality as
region BOD including only the ray BO) we have x + y ≥ x+y y–x
sin sin >0
0, y < 0, and so relation (12) in Region IV assumes the 2 2

348 | CAT Complete Course

Visit : www.EasyEngineering.net
Visit : www.EasyEngineering.net

This inequality holds true for all points whose co- each of the strips {M2.n – 1, M2n}, n = 0. ± 1, ± 2…… In
ordinates x and y are such that the expressions A = sin each of the strips {Mk, Mk + 1} of Fig. 27 is indicated the
[(x + y)/2] and B = sin [(y – x)/2] have the same signs. sign of expression A: the strips where A > 0 are marked
Let us first that the expression A. solving the with horizontal lines, the strips where A < 0 are marked
equation sin [(x + y)/2] = 0, we find x + y = 2kπ, k = 0, with vertical lines.
± 1, ± 2,…… Geometrically, this signifies that the Let us now examine expression B. Similar reasoning
expression A reduces to zero only the co-ordinates x and shows that expression B is made to vanish by the co-
y of points in the plane which lie on one of the straight ordinates x and y of point lying on the straight lines y = x
lines y = – x + 2kπ, k = 0, ± 1, ± 2,…… (in Fig. 27 these + 2mπ, m = 0 ± 1, ± 2…… (these are shown dashed in
Fig. 27). For integral m we denote the straight line y = x +
2mπ, Nm , and we will agree to call the strip between
adjacent lines Nm and Nm +1 the strip {Nm , Nm + 1 } (the
straight lines N m and Nm + 1 themselves do not belong to
this strip). It is easy to verify that the strip {N m, Nm + 1} is
a set of points whose co-ordinate x and y satisfy the
inequality 2mπ < y – x < 2 (m + 1)π.

ww Solving the inequalities B> 0 and B < 0, we see that


B is positive for the co-ordinates x and y of all points lying

w .Ea
in each of the strips {N2p, N2p + 1}, p = 0, ± 1, ± 2……,
which is to say in each strip bounded from below by the
line N2 p with even index and from above by N2p + 1.
Furthermore, the expression B is negative for the co-

syE ordinates x and y of all points lying in each of the strip


{Nm, Nm + 1}, p = 0, ± 1, ± 2…… In Fig. 27, in each of the
strip {Nm, Nm + 1} is indicated the sign of the expression

ngi
Fig. 27
straight lines are indicated by solid lines). For the sake of B : strips with B > 0 are marked with vertical lines, those
brevity, we denote by Mk the straight line y = – x + 2kπ with B < 0 are marked with horizontal lines.
for integral k (thus, the straight line M0 is the bisector of
the second and fourth quadrants, M– 1 has equation y =
– x – 2π etc.).
nee It is now easy to describe the set of points, in the
plane, whose co-ordinates x and y satisfy the inequality
A.B > 0 : it includes all rectangles (excluding their
All the straight lines Mk are parallel and partition the
plane into strips. Let us agree to call the strip between rin
contours) which are double cross-hatched in Fig. 27).

two adjacent lines M k and Mk + 1 the strip {Mk, Mk + 1},


the lines Mk and Mk + 1 themselves not being included in
g.n
Exercise – B
1. (B) Ma [md (a), mn (md (b), a), mn (ab, md (ac))]

e
this strip. For example, {M 0 , M1 } is the strip between the Ma [|– 2|‚ mn (|– 3|‚ – 2) mn (6‚ | – 8|) ]
straight lines y = – x and y = – x + 2π, that is, the set of
points whose co-ordinates x and y satisfy the inequality
0 < x + y < 2π.
Analogously, in the general case, the strip {Mk, Mk+1 }
Ma [2, mn (3, – 2), mn (6, 8)]
Ma [2, – 2, 6] = 6
2. (A) Ma [md (a), mn (a, b)] = mn [a, md(Ma(a, b)]
t
is the set of points whose co-ordinates x and y satisfy the Ma [2, – 3] = mn [– 2, md(– 2)]
inequality 2kπ < x + y < 2 (k + 1)π. 2 = mn (– 2, 2)
Now, let us determine the set of points whose 2=–2
coordinates x and y satisfy the inequality sin [(x + y)/2] >
Relation does not hold for a = – 2 and b = – 3 or a < 0,
0. This inequality can easily be solved; it is valid for
b<0
2.2nπ < x + y < 2 (2n + 1)π, n = 0, ± 1, ± 2……
Geometrically, this signifies that the expression A is
positive for it coordinates x and y of all points lying in
3. (B) fog (x) = f {g(x)} = f ( )
x–3
2
each of the strips {M2n, M2n + 1}, n = 0, ± 1, ± 2,…… i.e.,
in each strip bounded from below by the straight line M2n
= 2 ( )
x–3
2
+3=x

with even index and from above by M2n + 1. gof (x) = g {f (x)} = g(2x + 3)
In the same way, by solving the inequality sin [(x 2x + 3 – 3
= =x
+y)/2] < 0 we convince ourselves that expression A is 2
negative for the co-ordinates x and y of all points lying in ∴ fog (x) = gof (x)

CAT Complete Course | 349

Visit : www.EasyEngineering.net
Visit : www.EasyEngineering.net

4. (C) f (x) = g (x – 3) 11. (C) Equating 2 + x 2 = 6 – 3x


x–3–3 x–6 ⇒ x2 + 3x – 4 = 0
2x + 3 = =
2 2 ⇒ x + 4x –x – 4 = 0
2

4x + 6 = x – 6 Or (x + 4) (x – 1) = 0
3x = – 12 ⇒ x = – 4 or 1
x = –4 But x > 0 so x = 1, so LHS = RHS = 2 + 1 = 3. It
5. (B) {go fo fo go go f (x)} {fo go g (x)} from Q.3, we means the largest value of function min (2 + x2, 6 –
have fog (x) = go f (x) = x. 3x) is 3.
Therefore, above expression becomes (x). (x) = x 2 12. (D) M (M (A (M (x, y),S (y, x)), x),A (y, x)
6. (C) fo (fog) o (gof) (x) M (M (A (6, 1), 2), A (3, 2))
We have, fog (x) = gof (x) = x M (M (7, 2), A (3, 2))
So, given expression reduces to f (x) that is 2x + 3 M (14 ,5) = 70.
7. (A) me (a + mo (le (a, b)), mo (a + me (mo (a), mo 13. (B) S[M(D(A(a, b), 2), D(A(a, b), 2)), M(D(S(a, b),
(b))) 2), D(S(a, b), 2))]
Given a = – 2, b = – 3 ⇒ S[M(D(a + b, 2), D(a + b, 2)),

ww
a + mo (le (a, b)) = – 2 + mo(le(– 2, – 3))
= – 2 + mo (– 3) ⇒ S M [ (( )( )) (
a+b
M(D(a – b, 2)), D(a – b, 2))]
a+b
‚M )]
a – b‚ a – b

w
2 2 2 2
= – 2 + 3 =1 2 2
[( ) ( ) ]
a+b ‚ a–b

.Ea
mo (a + me (mo(a), mo(b)))
= mo (– 2 + me (mo (– 2), mo (– 3)))
⇒ S
2

=
2
(a + b)2 – (a – b) 2

8.
syE
= mo (– 2 + me (2, 3)) = mo (– 2, + 3) = mo (1) = 1
me (1,1) = 1
(D, A) mo (le(a, b)) ≥ me (mo (a), mo(b))
=
(2a)(2b)
4
22
= ab

= le (a, b)> me (a,b) as a, b > 0 which is false.


(B) mo (le(a, b))> me (mo(a), mo(b)) which is again ngi 14. (D) Since, x > y > z > 0
∴ la (x, y, z) = y + z

false.
Can be true only for a = b. nee and le = max(x – y, y – z)
We cannot find the value of le. Therefore we can’t
say whether la > le or le > la.
(C) mo (le (a, b))< le (mo(a), mo(b))
Or le (a, b) < le (a, b) which is false.
rin
Hence, we can’t comment, as data is insufficient.
15. (B) la (10, 5, 3) = 8
(D) mo (le(a, b) = le (mo(a), mo (b))
Or le (a, b)) = le (a, b) TRUE
le (8,5,3) = 3
g.n
1 13
9. (B) me (a2 – 3a, a – 3) < 0 or me [a(a – 3), a – 3] < 0
Case I. a < 0, a3 – 3a > a – 3
⇒ a (a – 3) < 0 or 0 < a <3 which is not true.
Case II. 0 < a < 3, a (a – 3) < 0 or 0 < a < 3 which is
2
1 e
ma (10, 4, 3) = [7 + 6] = = 6·5

16. (C) ma (15,10,9) = [19 + 5] = 12


2
2

t
min (10,6) = 6
true. le (9,8,12) = 1
Case III. a = 3, me (0,0)< not true. le (15,6,1) = 9
Case IV. a>3, a (a – 3) < 0 or 0 < a < 3 not true. 2+1 3
17. (C) (2 # 1)/(1 ∆ 2) = 2 + 1 =
Alternatively, it can also be found by putting some 2 8
values of a, say a = – 1 in case I. a =1 in case II and 18. (A) Numerator = 4– [(101·3 ∆log10) 0·1]
a = 4 in case IV.
= 4 – (101·3 ∆ (– 1) = 4 – 1 = 3
10. (B) le (a (a – 3), (a – 3))< 0
Denominator = 1∇2 = 21 + 2
Again in case I, a < 0; a – 3 < 0 or a < 3 (from last 3
question) can be true = 8 hence answer
8
In case II, 0 < a < 3; a – 3 < 0 or a < 3 can be true 19. (B) Try for (A), (C) and (D) all give numerator and
In case III, a = 3, le (0,0) = 0 < 0, not true Num 1
denominators as 1 i.e., = = 1.
In case IV, a > 3, a – 3< 0 or a < 3 not true. Hence Den 2
(B) and (C) are correct. Hence, (B) is the answer.

350 | CAT Complete Course

Visit : www.EasyEngineering.net
Visit : www.EasyEngineering.net

20. (B) Going by option elimination. 32. (B) It is not linear in x and y, that’s why option (a) is
(A) will be invalid when x + y = 0 neglected. It also can’t be exponential. By substitut-
(B) is the correct option as both sided gives – 2 |x + y ing X and Y in y = a + bx + cx2 we see that it gets
| as the result. satisfied.
(C) will be equal when (x + y) = 0. 33. (D) f (x + 1, y) = f [f, f (x, y)]
(D) is not necessarily equal (plug values and check) Put x = 0, f (1, y) = f [0, f (0, y)] = f [0, y + 1]
21. (C) consider option (C) as – F (x, y). G (x, y) = [– |x + = y+1+1=y +2
y |.|x + y|] = 4x 2 for x = y. Put y = 2, f (1 ,2) = 4.
And log2 16 = log2 24 = 4, which gives values of 34. (B) As graph is symmetrical about y-axis, we can say
option (C) as x 2. function is even so f (x) = f (– x).
22. (B) Solve sequentially from innermost bracket to get 35. (D) We see from the graph. Value of f (x) in the left
the answer is (B). region is twice the value of f (x) in the right region.
23. (D) From the graph F1 (x) = F(x) for x ∈ (– 2, 0) but, So, 2f (x) = f (– x) or 6f (x) = 3f (– x).
F 1 (x) = – F (x) for x ∈ (0, 2).
36. (C) f (– x) is replication of f (x) about y-axis – f (x) is
24. (D) From the graphs, F1 (x) = – F (x) and also F1 (x) =
replication of f (x) about x-axis and – f (– x) is replica-

ww
F (– x). So, both (A) and (B) are satisfied which is
not given in any of the option.
25. (D) By observation F1 (x) = – F (x) and also F1 (x) = F
tion of f (x) about y-axis followed by replication
about x-axis. Thus, given graph is of f (x) = – f (– x).

w
(– x). So, both (A) and (B) are satisfied. Since no

.Ea
option is given mark (D) as the answer.
26. (C) By observation F1 (x) = – F (– x). This can be
checked by taking any value of x say 1, 2 so answer
37. (C) putting the actual values in the functions, we get
the required answers.
m (a, b, c) = – 5, M (a, b, c) = 2

is (C).

27. (A) @ (A, B) = syE


A+B
2
So, [m(a, b, c)+ M (a, b, c)]/2 is maximum.
38. (C) m (a, b, c) = min (a + b, c, a); – M (– a, a, – b) = –
max (0, – b, – a); m (a + b, b, c) = min (a + 2b, c, a +

/(@ (A, B), 2) = ( ) A+B


2
×2=A+B
ngi b)
39. (C) m (M (a – b, b, c), m (a + b, c, b), – M (a, b, c))=
28. (D) X(/(@(/(@ (A, B), 2), C), 2), 3)

= (((( (A + B)
) ) ) )
* 2 + C /2 *2 /3 nee m (3, 4, – 6) = – 6.

40. (D) f (1) =


1
1+1 2
1
= as x is positive.

rin
2
1 2
A+B+C f 2 (1) = f (f (1)) = = ,
= 1 3

g.n
3 1+
2
= average of A, B and C. 3
f 3 (1) = f (f 2 (1)) = f [2/3] = ,
{ x2 < x‚ 0 < x < 1 f (x, y) = (x + y)0·5
}
29. (D) x2 > x 1 < x g (x, y) = (x + y)2
when x and y are positive. Thus for x + y > 1, (x +
y)0.5 < (x + y)2 therefore, f (x, y) < g (x, y). We can
5
e 5

f 4 (1) = thus f 1 (1) f 2 (x) f 3 (1)


8
t
… f 9 (1) =
1
therefore eliminate answer option a if x and y are 8
both negative then f (x, y) = (x + y)2 and g (x, y) = – (x 41. (C) When x is negative, f (x) = 1 + x
+ y) now for – 1 < x + y < 0, then (x + y)2 < – 1x + y
f (– 1) = 1 – 1 = 0;
Therefore, f (x, y) < g (x, y) thus answer option (B) is
f 2 (– 1) = f (f (-1)) = f (0) =1;
eliminate. As in evident from the above discussion,
for x and y > 1, we cannot again guarantee that f (x, y) 1 1
f 3 (– 1) = f (f 2 (– 1)) f (1) = = ;
> g (x, y). 1+1 2
30. (C) When 0 ≤ x, y < 0·5, x + y may be < 1 or 1, so f 4 (– 1) = f (f3 (-1)) f (1/2)
given statement (A) cannot be true or false. when x, y = 2/3 and f 5 (– 1) = 3/5.
< – 1, again statement (B) can be true or false. 42. (D) Clearly, BA ≥ MB A1 and MB A2 ≤ BA as n1 ≥
When x, y > 1, x + y > 1, hence f (x, y) < g(x, y). f (x, n2 + n2 .
y) > g (x, y). So option (A), (B) and (C) are neglected.
Thus, statement (C) given is necessarily false.  r r 
+ ≥ + max  0‚ 2 – 1 
r 1 r 2 r 1 n2
31. (B) when x + y = 1, we have (x + y)2 = (x + y)0·5 i.e., See BA =
f (x, y) = g (x, y). Thus answer is (B).
n1 n1 n1 n1  n2 n1 

CAT Complete Course | 351

Visit : www.EasyEngineering.net
Visit : www.EasyEngineering.net

r2 r n r n r  The dark lines represent the function g (x). It clearly


because ≥ 0 and 2 ≥  2 × 2 – 2 × 1 
n1 n1  n1 n2 n1 n1  shows the smallest value of g(x) = 3·5
r 2 r 2 n2 r 1 47. (B) f (x) = |x – 2 | + | 2·5 – x | + | 3·6 – x | can attain
or ≥ – minimum value when either of the terms = 0.
n1 n1 n1 2
So, none of the answers match. Case I : when |x – 2 | = 0 ⇒ x = 2, value of f (x)
43. (B) initial BA = 50, BA increases as numerator = 0·5 + 1·6 = 2·1
increases with denominator remaining the same. Case II : when |2·5 – x | = 0 ⇒ x = 2·5 value of f (x)
r + r2
MB A2 = 1 decrease as average of total runs = 0·5 + 0 + 1·1 = 1·6
n1 + n2
decrease from 50, as runs scored in this inning are Case III : when | 3·6 – x | = 0 ⇒ x = 3·6
less than 50. ⇒ f (x) = 1·6 + 1·1 + 0 = 2·7
44. (B) f (x) = log ( )
1+x
1–x
Hence, the minimum value of f (x) is 1·6 at x = 2·5.
48. (B) The curves can be plotted as follows :
f (y) = log (
1 – y)
1+y
and

ww
∴ f (x) + f (y) = log (
1 – x)
1+x
+ log (
1 – y)
1+y

w = log {(

.Ea
= log (
1 – x) ( 1 – y)}
1+x 1+y
Fig. 29

1 – x – y – xy )
1 + x + y + xy We see that they meet once.

syE
(1 + xy) (1 +
1 + xy)
x+y
49. (D) substitute values – 2 ≤ x ≤ 2 in the given curves,
we find the curves will intersect at x = 0, 1 and – 1.

ngi
= log 50. (A) From the table, we have g*g = h (this is g
(1 + xy) (1 –
1 + xy)
x+y squared) h *g = f (this is g cubed) f *g = e.( this is g to
the power 4)
[divided the Nr and Dr by (1+xy)]

1+
x+y n
51. (D) f ⊕ [f * {f ⊕ (f * f)}] is to be simplified so we

ee
start from the innermost bracket

= log
1–
1 + xy
x+y
=f (1x++xyy ) rin
f *f = h
f ⊕h = e
1 + xy
45. (D) [x] means if x = 5·5, then [x] = 5
f *e = f
f ⊕f = h.g.n
L [x, y] = [x] + [y] + [x + y]
R (x, y) = [2x] + [2y]
Relationship between L (x, y) and R (x, y) can be
52. (A) {a10 * (f10
f*f=hg*g=ha*a=ae*e=e
⊕ g9 )} ⊕ e8

h*f = gh *g = fa10 = a e8 =e
e t
found by putting various values of x and y. Put
x = 1·6 and y = 1·8; L (x, y) = 1 + 1 + 3 = 5 and R (x, g*f = ef * g =e
y) = 3 + 3 = 6, so (B) and (C) are wrong. e* f = fe * g=g
If x = 1·2 and y = 2·3 f 5 = fg5= g
L (x, y) = 1 + 2 + 3 = 6 and R (x, y) = 2 + 4 = 6 or So, f 10 = f 5 & f 5 = f *f= h so, g9 = g5 * g4 =g*e=g
R (x, y) = L (x, y), so (A) is not true. Q a10 * (f 10 ⊕ g9 )} ⊕ e8
We see that (D) will never be possible.
{a*(h ⊕ g)} ⊕ e
46. (D) g(x) = max (5 – x, x + 2). Drawing the graph.
{a*f} ⊕ e ⇒ e.
53. (D) y = ax2 – b | x |
As the options (A) and (C) include a > 0, b > 0 we
take a = b =1.
Accordingly the equation becomes y = x2 – | x |. A
Fig. 28 quick plot gives us.

352 | CAT Complete Course

Visit : www.EasyEngineering.net
Visit : www.EasyEngineering.net

55. (C) Consider the product f1 (x) f2 (x); for x ≥ 0, f 2 (x) =


0, hence f 1 (x) f2 (x) = 0 and for x < f 1 (x) = 0, hence
f 1 (x) f 2 (x) = 0 consider the product f 2 (x) f 3 (x);
for x ≥ 0, f2 (x) = 0, f3 (x) = 0, hence f2 (x) f3 (x) = 0
for x < 0, f2 (x) > 0, f3 (x) < 0, hence f2 (x) f3 (x) < 0
Fig. 30
Consider the product f2 (x) f4 (x)
So, at x = 0, we neither a maxima nor a minima As
the options (B) and (D) include a > 0, b < 0 we take for x ≥ 0, f2 (x) = 0, hence f2 (x) f4 (x) = 0
a = 1, b = – 1 for x < 0, f4 (x) = 0, hence f2 . f4 (x) = 4
Accordingly the equation becomes y = x2 + |x | ∴ f1 (x). f2 (x) and f3 (x). f4 (x) always take a zero
value.
56. (B) Choice (A) : from the graph it can be observed
that f1 (x) = f4 (x), for x ≤ 0 but f1 (x) ≠ f4 (x) for x > 0.
Choice (B) : The graph of f3 (x) is to be reflected in
Fig. 31 x-axis followed by a reflection in y-axis (in either

ww
so at r = 0, we have a minima.
54. (B) f (x) = x3 – 4x +p
f (0) = p, f (1) = p – 3
order), to obtain the graph of – f3 (– x) this would
give the graph of f1 (x).

w
Choice (C) : The graph of f2 (– x) is obtained by the
Given f (0) and f (1) are of opposite signs, reflection of the graph of f2 (x) in y-axis, which gives

.Ea
p (p – 3) < 0
If p < 0 then p – 3 is also less than 0.
us the graph of f1 (x) and not f4 (x), hence option 3 is
ruled out.

syE
∴ P (p – 3)>0 i.e., p cannot be negative
∴ Choices (A), (C) and (D) are eliminated 0 < p < 3.
Choice (D) : For x > 0, f1 (x) > 0 and f3 = 0, hence
f1 (x) + f3 (x) > 0.
●●●

ngi
nee
rin
g.n
e t

CAT Complete Course | 353

Visit : www.EasyEngineering.net
Visit : www.EasyEngineering.net

5 Inequalities
It is quite common, at examinations, for the aspirant Such a sum is greater than two. Hence, the original
to be asked to solve certain numerical or literal inequality holds true.
inequalities. This section is devoted to an analysis of Illustration 2. Prove that if a > 0, b > 0, c > 0, then
proof of literal and numerical inequalities. It would be bc ac ab
nice of course if there were some unified method for + + ≥a+b+c
a b c
providing all inequalities. Unfortunately, no such method
exists. But we will give below a number of techniques We take advantage of the following inequalities :
that are of use in proving a rather large number of
(
1 bc ac
) (
1 ac ab
) (
1 bc ab
)
ww
inequalities.
First we take up some inequalities that are frequently
2 a
+
b
≥ c,
2 b
+
c
≥ a,
2 a
+
c
≥b

w
used in problem solving, such as the inequalities between (these inequalities are valid because the left members
an arithmetic mean and a geometric mean, a are arithmetic means and the right members are

.Ea
consequences of this inequalities concerning the sum of
reciprocal quantities, and also the following trigonometric
inequality :
geometric means of positive numbers). Combining them
term by term, we get inequality that we wish to prove.

–√
syE
a2 + b2 ≤ a sin x + b cos x ≤ 
√
a2 + b2
The relation between an arithmetic mean and a
…(1)
Illustration 3. Prove that if a > 0, b > 0, c > 0, then
(a + b) (b + c) (a + c) ≥ 8 abc
Taking the following inequalities (see formula [2])
geometric mean of two numbers reads :
For any two non-negative numbers a and b the ngi a+b≥2√ ab, b + c ≥ 2 √bc, a + c ≥ 2 √ac

nee
inequality and multiplying them term wise, we get the desired
inequality.
ab ≤ a +2 b
√ …(2) This inequality may be proved in a different manner
holds true; equality occurs only, when a = b
A special case of (2) is the inequality rin
by using the inequality between the arithmetic mean and
the geometric mean for 8 positive numbers (see formula

1
x+ ≥2
x g.n
(5)). Indeed, removing brackets in the left member of our
inequality, we find that it can be rewritten as follows :
a2 b + b2 c + c 2 b + a2 c + b2 a + c2a + abc + abc
which is valid for all x > 0. In this inequality, the
equals sign holds for x = 1 only. It is useful to remember
the verbal statement of this inequality.
The sum of two positive reciprocals does not exceed
e 8

On the left side we have the arithmetical mean of 8 t


≥ abc

positive numbers; on the right, as can readily be verified,


two, and is equal to two only when both numbers are we have their geometric mean, which completes the proof
equal to unity. of the original inequality.
Also, note that for any x ≠ 0 the inequality holds true. Before going on to the next problem, let us dwell on

| |x+
1
x
≥ 2
a typical mistake that is rather often made in proving
inequality. It is this. The aspirant writes the inequality to
be proved, then performs certain (quite legitimate)
| |1 + x2
2x
≥ 1 …(3) manipulations and finally arrives at an obviously valid
inequality (say 1 < 2 or (a – b) 2 ≥ 0) and then concludes :
Illustration 1. Prove the inequality
“hence, the inequality is proved.” This is a crude logical
1 1
+ > 2. error : from the fact that a true inequality has been
log2 π logπ 2 obtained, we can by no means conclude that the original
By the properties of logarithms,1/logπ 2= log2 π > 0, inequality was true! To be more exact, we proved the
which means that the left member of our inequality is the following : if one assumes that the proposed inequality is
sum of two positive reciprocal different from unity (log2 true then the inequality obtained v i a a chain of
π ≠ 1). transformations is true, then the inequality obtained via a

354 | CAT Complete Course

Visit : www.EasyEngineering.net
Visit : www.EasyEngineering.net

chain of transformations is also true. But it is obvious that Illustration 5. Prove that if a > 0, b > 0, then for any
the final inequality is true as it stands; and we continue to x and y the following inequality holds true :
know nothing about the inequality which we set out to a.2x + b.3y + 1
prove.
It is logically correct to reason in the reverse order. It ≤√
4x + 9y + 1 · 
√ a2 + b2 + 1
is necessary to take some obviously valid inequality and By hypothesis, both sides of this inequality are
perform manipulations (which of course must be legiti- positive and so it is equivalent to the following :
mate from the viewpoint of algebra and trigonometry) (a·2x + b·3y + 1)2 ≤ (4x + 9y + 1) (a2 + b2 + 1)
that will bring us to the inequality to be proved. This is or to
justified reasoning : we started with a valid inequality and
via a chain of legitimate transformations arrived at the a2 .4 x + b2 .9 y + 1 + 2ab 2x 3 y + 2a 2 x + 2b3y
new inequality, which, hence must also be valid. ≤ 4xa2 + 4 xb2 + 4 x + 9ya2 + 9 yb2 + 9 y + a2 + b2 + 1
Of course there remains the most important question. Transposing all terms of this inequality to the right
From what inequality are we to proceed so as to side, and then collecting like terms and regrouping, we
transform it into the required inequality? To answer his can write it in the equivalent from
question we can perform the transformation of the (a29y – 2ab2x3y + 4xb2) + (4x – 2a2x + a2 )

ww
proceed inequality that leads us to an obviously valid
inequality. However, this stage in the solution of the
problem must be regarded as an exploratory search for
+ (9y – 2b·3y + b2 ) ≥ 0
Since, each parenthesis is a perfect square, the

w
the proof, as an attempt to get the proper approach, but original inequality is equivalent to the following obvious
not as proof in itself. If as a result of this exploration inequality :

.Ea
(manipulations) we have obtained an obviously true
inequality, then we can begin the proof proper : take this
(a3y – b2x)2 + (2x – a)2 + (3y – b)2 ≥ 0
Hence, the original inequality is true.

syE
obviously correct inequality and manipulate it as we did
in the exploratory search, but in reverse order; inverse the
manipulations, so to say. If this “work backward
Note that this inequality is also true for any real
values of a and b (the proof of this fact is left to the
reader).
procedure” is everywhere legitimate, then the inequality
being proved is indeed valid.
Incidentally, a somewhat different procedure is often ngi Illustration 6. Prove that the inequality

 3 sin x ≤ 1 is valid for arbitrary x.
followed. If, in the process of exploring the proof via a
reduction of our inequality to an obvious inequality, we nee –1≤
2 + cos x
Since, both members of this inequality are non- nega-

rin
always replaced the given inequality by an equivalent tive, then after squaring and multiplying by the positive
one, then the last inequality will be equivalent to the expression (2 + cos x)2 , we get an equivalent inequality :
original one, and therefore its validity implies the validity
of the original inequality. Hence, if at each stage in the
transformation we specially verified and stressed the g.n
3 sin2 x ≤ (2 + cos x)2. Replacing sin2 x by 1 – cos2x and
grouping, we finally get (2 cos x +1)2 ≥ 0. This inequality
holds true for all x, and since it is equivalent to the
equivalence of the inequality, then the “ work backward
procedure” is not necessary.
We shall follow this reasoning in carrying out the
proof of the following inequalities.
what we set out to prove.
e
original one, the original inequality is also true, which is

The original inequality may be proved differently by


making use of inequality (1). Indeed since 2 + cos x > 0
t
Illustration 4. Prove the inequality for all x, then, after multiplying by 2 + cos x, we get the
a3 + b3
2
≥( )a+b 3
2
following double inequality which is equivalent to the
original one :
where a > 0, b > 0 – 2 – cos x ≤ √

3 sin x ≤ 2 + cos x
Replace this inequality by the equivalent one The inequality on the left may be written as
a3 + b3
2
– ( )
a+b 3
2
≥0 –2≤√
 3 sin x + 1·cos x
Removing brackets and regrouping, we can write it It is now evident that this is a special case of
in the equivalent form inequality (1) which is a true inequality. The validity of
3 the inequality on the right is proved similarly.
(a + b) (a – b)2 ≥ 0
8 Illustration 7. Prove that for arbitrary α the in-
Since, a > 0 and b > 0, this inequality is obvious and, equality 4 sin 3α + 5 ≥ 4 cos 2α + 5 sin α is valid.
thus, the validity of the equivalent original inequality is One of the crudest errors made in proving this
proved. inequality is the “proof” by substitution of specific values.

CAT Complete Course | 355

Visit : www.EasyEngineering.net
Visit : www.EasyEngineering.net

At an examination, a number of aspirants reasoned for all x. A similar proof is given that
something like this : “for α = 0° the inequality holds π sin x – cos x
because 5>4, for α = 30° the inequality is true because sin ( 4
+
2 ) >0
1 1 A decisive factor in the example which is the use of
4 + 5 > 4· + 5· , for α = 45°,60°, 90° it is also
2 2 properties of the exponential function y = a x if a > 1, a
obviously true, which means it holds true for all values of larger value of the argument is associated with a larger
α.” value of the function and, hence, a greater value of the
Actually, of course, these aspirants proved the function is associated with a greater value of the
inequality only for several separate values of α and argument; if a < 1, to a greater value of the argument
offered no proof what so ever for the remaining values of corresponds a smaller value of the function and, hence, to
α.A correct proof is as follows. a larger value of the function corresponds a smaller value
We know that sin 3 α = 3sin α – 4 sin3 α, cos 2 α = of the argument.
1 – 2 sin 2 α and so the original inequality may be rewritten Illustration 10. Prove that possible numbers c and d
as and arbitrary α > 0, the inequalities c < d and cα < dα are
4 (3 sin α – 4 sin3 α) + 5 ≥ 4 (1 – 2 sin2 α) + 5 sin α equivalent.
Let c and d be positive numbers and α > 0. Consider

ww
or, 16 sin3 α – 8 sin2α – 7sin α – 1 ≤ 0
The latter inequality should be valid for all values of
α. Denoting sin α by x, we rewrite it as
the function y = (c/d) x.
If c < d, then 0 < c/d <1. By the property of an

w 16x3 – 8x2 – 7x – 1 ≤ 0

.Ea
We now have to prove that this inequality is valid for
arbitrary values of x in the interval – 1 ≤ x ≤ 1
exponential function with base less than unity, we have
c α
() ()
d
<
c °
d
whence follows cα/dα < 1, or cα < dα

syE
or (x – 1) (4x + 1)2 ≤ 0. This inequality is clearly
valid, and so the original inequality is proved. Conversely, if c α < dα then cα/dα < 1,
c α c °
The proof of certain inequalities requires skill in or () () <

ngi
utilizing the properties of functions that enter into the d d
inequalities. This means that the larger value of argument (α > 0)
Illustration 8. Prove that the inequality of our function is associated with a smaller value of the
cos (cos x) > 0 is true for all x.
For all x we have – 1 ≤ cos x ≤ 1. Put α = cos x to get nee
function. But this is true only when the base is less then
unity, that is c/d <1, whence c<d.

– 1 ≤ α ≤ 1. Since – π/2 < – 1 and 1 < π/2, it follows all


the more so that α satisfies the condition – π/2 < α < π/2.
rin
The statement we have just proved is ordinarily
formulated as follows : an inequality between positive
numbers may be raised to any positive power : in
The properties of the function y = cos x imply that cos α
is positive for all these values of α , which actually is
what we set out to prove. g.n
particular, a root of any degree may be extracted.
Illustration 11. Prove the inequality
Illustration 9. Prove the inequality cos (sin x) > sin
(cos x).
It can be rewritten as
(α α + bα)1/α ≤ (aβ + bβ)1/β
For a ≥ 0, b ≥ 0, α > β > 0 e
If a = 0 or b = 0, then the proposition is obvious. t
Now, let a > 0 and b > 0. It is clear that one of these
(
cos (sin x) – cos
π
2 )
– cos x > 0 numbers does not exceed the other.
or, 2 sin Suppose, say,
0 < a ≤ b. Then 0 < a/b ≤ 1, and since α > β,
( π sin x – cos x
4
+
2
sin (π4 – sin x +2 cos x) > 0 ) it follows that
We shall that the factors in the left-hand member are 0 < (a/b)α ≤ (a/b)β and 1 + (a/b) α ≤ 1 + (a/b) β
positive since From the latter inequality we get (see example 10)
[1 + (a/b)α]1/β ≤ [1 + (a/b)β]1/β
|sin x – cos x | = | √
 2 sin (x – π/4) ≤ √ 2 < π/2
Furthermore, since
π sin x – cos x π
it follows that – < < 1 + (a/b)α ≥ 1 and 0 < 1/α < 1/β
4 2 4
It follows that
π sin x – cos x π
and therefore 0 < + < [1 + (a/b)α]1/α ≤ [1 + (a/b)α]1/β
4 2 2
π sin x – cos x Now, we can write
Consequently sin ( 4
+
2 ) >0 [1 + (a/b)α]1/α ≤ [1 + (a/b)α]1/β ≤ [1 + (a/b)β]1/β

356 | CAT Complete Course

Visit : www.EasyEngineering.net
Visit : www.EasyEngineering.net

whence To prove this, reduce each term of the sum in the


left-handed member :
(
aα + bα
bα ) ≤(
1/α aβ +
bβ )
bβ 1/β

1
>
2
=2 √ (
k + 1– √
k )
Since, b > 0, the inequality being proved follows k √
√  k + √
k+1
from the last inequality.
Therefore, the left side of the inequality we want to
Illustration 12. Prove the inequality 0 < sin8 x + prove can be reduced :
cos14 x ≤ 1.
1 1 1
It is quite obvious that sin8 x + cos14 x ≥ 0. But the 1+ + +… >2 (√ 2 – √ 1)
equality sin8 x + cos1 4 x = 0 is valid only if we 2 √
√ 3 n

simultaneously have sin8 x = 0 and cos14 x = 0, which, of
course, is impossible. Therefore, the strict inequality
+2 (√ 3 – √ 2) + …… + 2 (√ n – √
n – 1)
sin 8 x + cos14 x > 0 holds true. + 2 (√  n)
n+1– √
The properties of trigonometric functions imply that
Since, the right side of this latter inequality is exactly
sin 2 x ≤ 1 and cos2 ≤ 1. for arbitrary real x. But since 8 > 2
and 14 > 2, it follows there from that equal to 2 √

n + 1 – 2, in the original inequality is valid.

wwsin 8 x ≤ sin 2 x and cos14 x ≤ cos2 x


Combining these inequalities term wise and noting
that sin2 x + cos2x = 1, we obtain
In the next example, an apt combining of factors
leads us to the result we need.

( )
n+1 n

w
Illustration 15. Prove that n! < where n is
2
sin8 x + cos14x ≤ 1.

.Ea
It is obvious here that, say, for x = π /2 we have
equality; in other words, the weak inequality cannot be
an integer exceeding unity.
The validity of this inequality will follows from the
validity of the equivalent inequality

syE
replaced by the strict inequality sin8x + cos14x ≤ 1.
One of the techniques used in proving inequalities
consists in the following. For instance, let it be required
(n!)2 < ( )
n + 1 2n
2
to prove the inequality A < B, where A and B are certain
expressions. If we succeed in finding an expression C
such that A < C and at the same time C ≤ B then the ngi Let us multiply the number n! = 1.2…………k ……
(n – 1)n by the number n! = n (n – 1) …..(n – k + 1) ……
2.1 arranging them one above the other :
required inequality A < B will have thus been proved.
Illustration 13. Prove that for every positive integer nee 1
n
2 …………k ……….(n – 1) n
(n – 1) ……(n – k + 1) ……….2 1
n the following inequality holds true :
1 1
+ + …… +
1
<
1
rin
Multiplying the numbers in each column, we get

g.n
9 25 (2n + 1)2 4 (1.n) [2(n – 1)] … [k (n – k + 1)] … [(n – 1)·2] (n·1)
2 1 1 In order to obtain (n!) 2 we have to multiply the terms
Noting that < –
(2k + 1) 2 2k 2k + 2 of this row. Applying inequality (2) to each term of this
we replace the sum in the left member of the
inequality to be proved by the greater expression
1 1
+ + ……+
32 52
1
(2n + 1)2
row, we get


√ k (n – k + 1) ≤
k+– k+1
2
e t
n+1
= , k = 1, 2, ………, n
<
1
2[( ) ( )
1 1
2 4
– +
1 1

4 6
+…
1
(

1
2n 2n + 2 )] 2
Equality being achieved only when k = n – k + 1, that
However, this latter expression is equal to is to say, for k = (n + 1)/2. In other words, only for n odd;
and only then for one term of our row in this inequality is
1 1
[–
1
2 2 2n + 2 ]
= –
1 1
4 4n + 4 equality possible. Hence, for all brackets except possibly
one, the inequalities
1
and, obviously, is less than 1/4. Hence, the sum +
1 1
9 [k (n – k + 1)] < ( )
n+1 2
2
hold true.
+…+ is all the more so less than 1/4.
25 (2n + 1)2 Since, there n terms in the row, we get
Illustration 14. Prove that for any positive integer
n >1 the inequality (n!)2 < [( ) ]
n+1 2 n
2
1 1 1
1+ + +… >2 √ (
n + 1 – 1 holds true. ) A sufficiently large number of inequalities can be
2 √
√ 3 n
√ proved by the method of mathematical induction.

CAT Complete Course | 357

Visit : www.EasyEngineering.net
Visit : www.EasyEngineering.net

Illustration 16. Prove that for any real number α ≥ – 1 Note that we also established the fact that equally in
and any positive integer n the inequality the relation at hand is only possible if all x i = 1; now if
(1 + α) n ≥ 1 + nα ……………………….. (4) not all xi are equal to unity, then the strict- inequality sign
holds true in this relation.
holds true.
From this theorem follows the generalized inequality
The inequality is clearly true for n = 1. Suppose that
between the arithmetic mean and the geometric mean for
the inequality (1 + α)k ≥ 1 + kα holds true; we will prove
n ≥ 2 positive numbers :
that in that case in inequality (1 + α)k+1 ≥ 1 + (k + 1) α is
valid indeed : x1 + …… + xn n
≥√

x1……xn, x1 > 0,
(1 + α)k+1 = (1 + α)k (1 + α) ≥ (1 + kα) (1 + α) = 1 n
+ (k + 1) α + kα2 ≥ 1 + (k + 1) α. ………xn > 0 …(5)
This means that the original inequality holds true. n
Indeed denote  √
x1 …… xn by c and xi/c by y i. Then
Illustration 17. Prove that the inequality |sin nx| ≤
yi…….yn = (x1……xn)/c n . By what has been proved, y1 +
n|sin x|, is valid for any positive integer n.
…….yn ≥ n, whence (x 1 +……..+ xn)/n ≥ c and this
For n = 1 the inequality is obviously true. Assuming completes the proof.
that |sin kx| ≤ k|sin x|, we will prove that |sin (k + 1)x| ≤ (k

ww
+ 1)|sin x|. Indeed, taking advantage of the inequality |cos
kx| ≤ 1 we have
This inequality is widely used in the proof of other
inequalities. For example, if we apply it to the numbers
1,2,…….,n, then we immediately get the inequality

w
|sin (k + 1) x| = |sin kx·cos x + sin x·cos kx|
n 1 + 2 + …… + n
≤ |sin kx|·|cos x| + |sin x|·|cos kx| 
√ 1·2……n <

.Ea
≤ |sin kx| + |sin x| ≤ k|sin x|+|sin x|
= (k+1).|sin x| or √
n
n

n! < (n + 1)/2, whence n ! < [(n + 1)/2]n . We

syE
Hence, the original inequality is true.
Illustration 18. Prove the following theorem : if the
proved this inequality in problem 15 via a special
technique. This new proof is clearly simpler.
The foregoing examples show that the method of

ngi
product of n ≥ positive numbers is equal to 1, then the
mathematical induction can successfully be applied in the
sum of the numbers is greater than or equal to n, that is if,
proof of a variety of inequalities. At the same time, one
x1x2……..xn = 1, x1 > 0, x2 > 0, ……., xn > 0,

nee
should not overestimated the power of the induction
then x1 + x2 +……… + xn ≥ n method : there are many problems that would seem
If n = 2 we have to prove the statement : if x1 x2 = 1, particularly suited to this method, whereas attempts to
then x 1 + x2 ≥ 2. but this is obvious since the arithmetic
mean (x 1 + x2)/2 of two positive numbers is greater than
rin
employs it encounter insuperable difficulties.
To illustrate, let us try to use in induction on the
or equal to the geometric mean √

when x 1 = x2 = 1.
x1x2 = 1 or x 1 + x2 ≥ 2.
Besides, equality (that is, x 1 + x 2 = 2) is attained only
inequality
1 1
+
9 25
+ …… + g.n 1
(2n + 1)2 4
<
1

Using induction, we take any positive numbers


x1…….x k , x k+1 which satisfy the condition x 1 ……..xk- that this inequality is valid for n = k :
1 .xk.xk+1 = 1. If each of these number equals 1, then the
sum x 1 + ……..xk + x k+1 = k +1 so that in this case the
1 1
+
9 25
+ …… +
e
For n = 1 it has form 1/9<1/4, which is true. Suppose

1
<
(2k + 1) 2 4
1 t
original inequality is valid. For n = k + 1, the left side is
If this is not so, there will be a number among them 1 1 1
+ + ……+
less than 1 and a number greater than 1. Suppose that 9 25 (2k + 3) 2
xk>1. xk+1<1 we have the equality
x1………xk-1 (xkxk + 1) = 1 = [
1 1
+
9 25
+ ……+
1
(2k + 1) 2 ]+
1
(2k + 3) 2
This is a product of k numbers and so the induction By the induction hypothesis, the sum in the square
hypothesis is applicable and we can assert that brackets is less than 1/4 and therefore
x1 +…….+ xk-1 + x kxk + 1 ≥ 1 1 1 1 1 1
– + …… < +
But then 9 25 (2k + 3) 2 4 (2k + 3) 2
x1 +………..+xk–1 + xk + x k +1 ≥ k – xkxk+1 + xk + xk+1 Quite obviously this inequality does not in the least
imply that the left- hand member is less than 1/4. Thus,
= k + 1 + (xk – 1) (1 – xk+1 ) > k + 1
proof by induction has come to an impasses, whereas the
Since, xk– 1 > 0 and 1 – xk +1 > 0 which completes the inequality is very simply proved by an entirely different
proof. method. This is done in Illustration 13.

358 | CAT Complete Course

Visit : www.EasyEngineering.net
Visit : www.EasyEngineering.net

In conclusion we offer two inequality in the proof of  2 – x and y = – √


√  2 – x (these lines included). But the
which the techniques suggested above and certain others circle x2 + y2 = 1 lies entirely insides this strip and so the
are used; these inequality can be solved by several methods co-ordinates of any points of it satisfy the inequality – √2
involving algebra, trigonometry and even geometry.
≤x+y≤√  2 the proof is complete.
Illustration 19. Prove that if x2 +y2 = 1, then – √ 2 ≤ Illustration 20. Let a + b = 2, where a and b are real
x+y≤√  2. numbers. Prove that a4 + b4 ≥ 2.
Algebraic solution. Let us write the obvious Note that if one of the numbers, a or b, is negative
inequality (x – y)2 ≥ 0 or x2 + y 2 ≥ 2xy, whence 2(x2 + y 2 ) the inequality is almost obvious. Suppose, say, b < 0.
≥ x2 + 2xy + y 2 . In so far as x 2 + y2 = 1, from the latter Then a > 2 and the inequality a4 + b4 ≥ 2 is true, since b4
inequality we have (x + y)2 ≤ 2, whence, > 0 and a4 > 16. We will therefore assume that a ≥ 0 and
|x + y| ≤ 
√ 2 or – √ 2 ≤ x + y ≤ √ 2 b ≥ 0.
First solution. Since a + b = 2, then (a + b) 2 = 4.
Trigonometric solution. If x and y satisfy the
condition x 2 + y 2 = 1, then we can find an angle α such Using the inequality between the arithmetic mean and the
geometries mean, ab ≤ (a 2 + b2)/2, we have 4 = (a + b)2 =
that x = cos α, y = sin α. Then we have to prove that for
a2 + b2 + 2ab ≤ 2(a2 + b2 ), or 2 ≤ a2 + b2 squaring this
any value of α

ww –√ 2 ≤ cos α + sin α ≤ √ 2


Since cos α + sin α = √  2 sin (α + π/4) and – 1 ≤ sin
inequality (this is legitimate since the numbers on the
right and left are positive), we get
4 ≤ (a2 + b2 )2

–√w
(α + π/4) ≤ 1, it follows that –

.Ea
On the basis of the inequality between the arithmetic
mean and the geometric mean a2 b2 ≤ (a 4 + b4 )/2.
 2 ≤ √ 2 sin (α + π/4) ≤ √ 2 for all values of α, Therefore we have 4 ≤ (a2 + b2 )2 = a4 + b4 + 2a2 b2 ≤ 2 (a4
which completes the proof of the required inequality. + b4 ) whence 2 ≤ a4 + b4 , and the proof is complete.

syE Second solution. We again assume that a ≥ 0 and b


≥ 0. Since a + b =2, then (a + b)4 = 16 or
(a + b)4 = (a2 + 2ab + b2) (a2 + 2ab + b2 )

ngi But since


= a4 + b4 + 4ab (a2 + b2 ) + 6a2 b2 =16

nee a2 + b2 = 4 – 2ab, the last equality can be


rewritten

rin
4 4
a +b = 16 – 16ab + 2a b 2 2

If we are able to demonstrate that 16 – 16ab – 2a2b2


Fig. 1
Geometric solution. We will consider x and y as co- g.n
≥ 2, then our inequality will have been proved.
By hypothesis, ab ≤ 1. Indeed, 
Since a + b = 2, it follows that√
√ab ≤ (a + b)/2.
ab ≤ 1., whence ab ≤ 1.
ordinate of point in a plane in a given system of co-
ordinate. Then the condition x2+y2 =1 is satisfied by the
points(x, y) lying on a circle of radius 1 centered at the
origin (Fig. 1). The points which satisfy the inequality x +
e t
And so we have to prove the inequality 16 – 16ab + 2a2 b2
≥ 2, provided that ab ≤ 1. We set x = ab. Then we have to
prove of the inequality x2 – 8x + 7 ≥ 0 with the provison
y ≤√ 2 lie on the straight line y = √ 2 – x and below that that x ≥ 1. The roots of the quadratic trinomial x2 – 8x + 7
line. are x1 = 1, x2 = 7. Therefore, the last inequality may be
written as (x – 1) (x – 7) ≥ 0.
Let B be the point of intersection of this straight line
with axis of abscissas and OA a perpendicular dropped on But the last x ≤ 1 this inequality is obvious. We have
thus obtained 16 – 16ab +2a2 b2 ≥ 2, which is what we set
this line from the origin. Then OB =  √ 2, ∠ABO = 45° out to prove.
and therefore OA =1. Hence, the point A lies on the circle
Third solution. Let a =1 + c, b = 1 – c. Since, we
and the straight line y = √ 2 – x is perpendicular to the earlier assumed that a ≥ 0 and b ≥ 0, it follows that – 1 ≤
radius OA at its end point, which is to say it is tangent to c ≤ 1 and so we can take advantage of inequality (4) (see
the circle. problem 16 of this section) :
Similarly, the inequality – √ 2 ≤ x + y is satisfied by (1 + c)4 ≥ 1 + 4c, (1 – c)4 ≥ 1 – 4c
points lying on the straight line y = – √  2 – x and above Thus,
it; this line is also tangent to the circle x2 + y2 = 1. a + b4 = (1 + c)4 + (1 – c)4 ≥ (1 + 4c) + (1 – 4c) = 2
4

Thus, the double inequality to be proved is satisfied In conclusion we note that a more general statement
by points lying in the strip between the straight line y = – is valid : if a + b = 2, then a n + bn ≥ 2 for any positive

CAT Complete Course | 359

Visit : www.EasyEngineering.net
Visit : www.EasyEngineering.net

integer n. This can easily be proved by, say, the third It is a matter of wonder that so many mistakes are
method given above. made by students when solving the simplest kind of
inequality. Apparently this is due to a formally
Solving Inequalities understood analogy between equations and inequality.
A great deal of mistake are made in the solution of The reasoning goes roughly like this : "Since the solution
inequalities. The point is that in most cases the solution of of the equation log 1/2 x =1 is x = 1/2, the solution of the
inequalities given at examination does not require any inequity log1/2 X > 1 constitutes the values x > 1/2."
particular ingenuity or artificial techniques, and so, as a Similarly, solutions to the inequalities (1/5)x < 2 are
rule, the student sees at a glance what steps must be written as x < log1/5 2, and so on. Yet the actual solutions
taken. However, in carrying out the manipulations, the to the two foregoing inequalities are different : in the first
students makes serious mistakes due to a failure to case, 0 < x < 1/2, in the second, x > log1/5 2. A false
recognize the fundamental theoretical propositions analogy between equations and inequalities led to these
involving inequalities. mistakes.
Actually, solving inequalities hardly requires Actually, when the student tackles an elementary
anything more than the ability to reduce an inequality to inequality, he should consciously take advantage of the
the solution of elementary inequalities (without either properties of the functions participating in the inequality.

ww
losing a solution or introducing any extraneous ones), and
then to solve these elementary inequalities. To carry out
the latter part, the student has to know the fundamental
Let us now consider examples in solving some elemen-
tary inequalities.
We wish to note first of all that the solution of linear

w
properties of the functions studied at school (algebraic,

.Ea
exponential, logarithmic and trigonometric functions); to
carry out the former part, the student must be able to
handle the basic concepts involving the equivalence of
(first-degree) I and quadratic (second-degree) algebraic
inequalities is usually quite thoroughly explained in text-
books and hardly ever causes any trouble.
Here we wish to dwell on elementary exponential,

syE
inequalities, the sources of loss of solutions and of the
introduction of extraneous solutions.
The basic definitions needed in the solution of
logarithmic, and trigonometric inequalities.
An elementary exponential inequality is an inequality
of the type a x >ab (a x < a b ). When handling such
inequalities repeat almost word for word those required
for equations. Note the following two differences in ngi inequalities, it must be remembered that the properties of
an exponential function differ for bases greater than unity
terminology however : the term "root" is not used when
speaking of inequalities; one always uses the term
"solution"; also, for the sake of brevity, one speaks of the nee
and less than unity.
Illustration 21. Solve the inequality – 1 ≤ (1/3)x <2.
To solve a double inequality means to find all the
solution being a certain set of values of x, for instance,
the interval a < x < b, whereas in actuality every value of
rin
values of x which simultaneously satisfy the two
inequalities : (1/3)x ≥ – 1 and (1/3)x < 2. (Fig. 2)

g.n
x of the set is a solution.
The similarity of equations and inequalities is quite
naturally not confined to that of the basic definitions. It is
obvious, for example, that everything that has been said
about transforming equations which extend or restrict the
domain of the variable is just as valid when applied to
inequalities.
e t
However, it must be stressed that solving inequalities
has its peculiarities in that the same manipulations
applied to equations and inequalities lead to different
results. For instance, when multiplying both members of
an equation by some nonzero factor (which is meaningful
in the domain of the variable), an equation is replaced by
an equivalent equation, whereas for inequalities we have
to deal with the additional restriction that the factor be
non-negative in the domain of the variable. In the same
way, squaring both sides of an equation does not lead to a
loss of roots, while squaring an inequality can lead either Fig. 2
to a loss of solutions or to the introduction of solutions. Since, an exponential function is always positive, the
Students often lose sight of these peculiarities and make first of these inequalities is valid for all values of x.
mistakes in the solution of inequalities that they never
would make when solving equations. Rewriting the second inequality as (1/3)x< ()
1
3
log 1/3 2.

360 | CAT Complete Course

Visit : www.EasyEngineering.net
Visit : www.EasyEngineering.net

We take advantage of the property of an exponential Taking into account the simplicity of solving
function : to a base less than unity, the greater value of logarithmic inequalities by means of Properties VII and
the function is associated with the smaller value of the VIII, we will henceforth solve such inequalities by using
argument and conversely, to the smaller value of the these properties.
function correspond the greater value of the argument. Illustration 22. Solve the inequality log1/2.x >log1/3 x.
This inequality is therefore equivalent to the inequality x >
log1/2 2. Taking the logarithm of the right member to the base
1/2 (Rule V), we get an equivalent inequality :
This solution is well illustrated by the graph shown
in Fig. 2, namely, the solutions are those values of x for
which the graph of the function y = (1/3)x lies below the (
log1/2 x 1 – log1/3
2
1
)>0

horizontal straight line y = 2; that is, all x to the right of Since, 1/2 > 1/3, it follows that log 1/2 1/2 >log1/3 1/3
the abscissa of the point of intersection of these graphs or 1- log1/3 1/2> 0.
(this abscissa is a solution of the equation (1/3)x = 2).
Noting that 0 =log1/2 1, we find that the original
Thus, the solution of our inequality is the interval x >
inequality is Equivalent to log 1/2x > log1/21.
log1/3 2.
When solving inequalities containing the unknown Applying Property VIII to this inequality, we get the

ww
under the sign of the logarithm, one must also bear in solution of the original in quality : 0 < x< 1.
mind that the properties of a logarithmic function differ Now, let us examine trigonometric inequalities.
depending on whether the base is less than or greater than Despite the fact that the solutions of the more elementary

w
unity. However, another essential point in solving these
inequalities is that the logarithmic function is not defined

.Ea
for all values of x. This is lost sight of by many students
when solving an inequality like log2 x < 1. They reason
trigonometric inequalities are thoroughly explained in the
standard textbooks, students continue to make serious
mistakes even when solving the simplest inequalities. We
now examine a few typical mistakes of this nature.

syE
this way : "We rewrite the inequality as log 2 x < log2 2.
The greater number to a base greater than 1 has the larger
logarithm, and so the inequality is valid for x < 2."
(a) Knowing that the solutions of the equation sin x = a
(|a| ≤ 1) are given by the formula x = (– 1)k arc sin a + k
π, where k = 0, ± 1, ± 2, ..., many students write that "the
Nothing would seem to be wrong in this argument,
but still the answer is faulty because extraneous solutions
were introduced. Indeed, any negative number is less than ngi solution of the inequality sin x < a consists of all values
of x < (– 1)k arc sin a + k π, k = 0, ± l, ± 2,……”

2, but the original inequality is meaningless for negative


values of x (because negative numbers do not have neeIt is quite often difficult to convince the student of the
absurdity of such an answer.
(b) Many mistakes are made that are connected with

rin
logarithms).
the formal use of the symbols arcsin a, arccos a, etc.
Why were extraneous solutions introduced? When
These symbols are frequently employed when the student

g.n
"solving" the inequality, we passed from log2 x < log2 2 to
has not yet investigated whether they are (Fig. 3).
x < 2. The latter inequality is meaningful for all values of
x while the original inequality has meaning only for those
values of x for which log2 x is meaningful, that is to say,
for x > 0. Hence, extraneous solutions were introduced
simply because the fact was disregarded that a logarithmic
function is defined only for positive values of x.
e t
A correct answer is obtained if we choose from
among the solutions of the latter inequality those whose
values of x > 0; thus, the solution of our inequality is the
interval 0< x <2.
This simple example makes it abundantly clear that
one should bear in mind, when solving logarithmic Fig. 3
inequalities in this manner, that a logarithmic function is
Meaningful or not. For instance, the solution to the
only defined for positive values of x. However, these inequality sin x ≤ log4 5 is written as arcsin (log4 5),
inequalities may be solved in a different way : instead of which is meaningless since log4 5 > 1. Yet this inequality
using the domain of definition of the logarithmic function
is valid for all values of x; this is evident from, the very
and its property of monotonicity we can immediately take start because log 4 5 > 1.
advantage of Properties VII and VIII of logarithms.
(c) Mistakes occur due to improper use of the
Thus, using Property VII in the above example, we
trigonometric circle. For example, when solving an
can directly replace the inequality log2 x < log2 2 by the
equivalent inequality 0 < x < 2, which yields the answer. inequality like sin x ≤ – √ 2/2, the students correctly

CAT Complete Course | 361

Visit : www.EasyEngineering.net
Visit : www.EasyEngineering.net

indicate the angles that yield the solutions of the


inequality (Fig. 3) but err when they give the analytic
notation as
5π π
+ 2k π ≤ x ≤ – + 2k π, k = 0, ± 1, ± 2,……
4 4
It is clear that this notation is meaningless since the
left member of the inequality is greater than the right
member for ail values of k.
Fig. 5
When solving elementary trigonometric inequalities,
it is best to make use of the graphs of trigonometric Fig : 5 the drawing shows us that it is no longer
functions. This is a practical guarantee against mistakes convenient to take the interval from 0 to 2π for the basic
and makes for a pictorial representation of the regions in interval because the solution of the inequality there will
which the inequality is valid. When giving their analytic consist of two "pieces". It is therefore more convenient to
notation, it is convenient to take advantage of the seek the solution of this inequality on the interval from
following fact : if f (x) is a periodic function, then to solve – π to π. This is the interval – 2π/3 ≤ x ≤ 2π/3. Conse-
the inequality f(x) > a it suffices to find the solution in quently, the complete solution is

ww
any interval that is equal to the length of the period of the
function f(x), then all values of x thus found and also all x
that differ from these values by an integral number of the


3

+ 2k π ≤ x ≤ + 2k π, k = 0, ± 1, ± 2,……
3

w
Illustration 25. Solve the inequality |tan x| < 1/7.
periods of the function f (x) constitute a solution of our
The period of the function |tan x| is equal to π. We
inequality.

.Ea
Illustration 23. Solve the inequality sin x > 1/2.
consider the inequality on the interval from – π/2 to π/2
and construct the graphs of the functions y1 = |tan x| and

syE
We construct the graphs of the functions y1= sin x
and y2 =1/2 (Fig. 4). This inequality is satisfied for all
values of x for which the (Fig. 4).
y2 = 1/7.

ngi
nee
rin
g.n
Fig. 4
First graph lies above the second one. Since the
period of the function sin x is 2π, it is sufficient for us to
e
Fig. 6 t
solve the proposed inequality on some interval of length
It is evident that the solution will consist of all x
2π. It is easy to see that the most convenient interval is lying in the interval – x0 < x < x0 , where x 0 is the abscissa
that from 0 to 2π : the solutions can most simply be of the intersection point of the graphs under consideration
written then as π/6 < x < 5π/6. that lies between 0 and π/2, that is, the root of the equa-
Thus, the complete solution of the inequality is tion tan x = l/7 located in the interval 0 < x < π/2. Hence,
x0 = arctan (1/7). Taking into account the period of the
π 5π
+ 2kπ < x < + 2kπ, k = 0, ± 1, ± 2,…… function y = |tan x|, we find that the solution of our
6 3
inequality consists of all values of x located in the
This notation is to be understood as follows : there is intervals
a certain interval for each integer k, and the set of all 1 1
these intervals constitutes the solution of the inequality. – arctan + k π < x < arctan + kπ, where k = 0, ± 1,
7 7
Illustration 24. Solve the inequality cos x ≥ – 1/2. ± 2,...
We construct the graphs of the functions y1 = cos x Note that the original inequality can be written as a
and y2 = – 1/2 (Fig. 5). The period of the function cos x is double inequality – 1/7 < tan x < 1/7 and solved by using
also equal to 2π, but the graph of the function y = tan x.

362 | CAT Complete Course

Visit : www.EasyEngineering.net
Visit : www.EasyEngineering.net

Now, let us consider raising to a power. In the sequel trinomial in the left-hand member of this equation does
we will frequently make use of the following statement. not have any real roots and therefore this inequality holds
Theorem—If f (x) ≥ 0 and φ > (x) ≥ 0 on some set of true for a1l real values of x. It then follows, because the
values of x, then the inequalities f (x) >φ (x) and [f (X)]2 > inequalities are equivalent, that the original inequality too
[φ (X)]2 are equivalent on that set. holds true for all values of x”. This reasoning appears to
be correct, but there is one serious defect. It is true in the
Proof–Let x 0 be an arbitrary solution of the first
domain of the variable of the original inequality.
inequality taken from the set of values of x under
consideration. If φ (x0 ) > 0, then from the validity of the The proper solution is in the domain of the variable,
inequality f (x 0 ) > φ (x 0 ) follows, on the basis of the both member of the original inequality are non-negative;
theorem on raising numerical inequalities to a power, the for this reason it is equivalent, in the domain, to the
validity of the inequality [f (x0)] 2 > [φ (x0 )] 2 . But if φ (x0 ) inequality 5x 2 + 7x + 17 > 0 and hence is true for all
= 0, then it is obvious that the validity of the inequality Values of x in the domain. It is now easy to find domain
f(x0 )>0 implies [f(x0)] 2 > 0. This proves that every of the original inequality and thus to obtain the answer : 2
solution of the inequality f (x) > φ (x) is a solution of the ≤ x ≤ 3.
inequality [f(x)]2 > [φ (x)]2 . In the problem that follows, extraneous solutions are
The converse is proved in similar fashion : that every introduced not because of an extension of the domain of

ww
solution of the inequality [f(x)]2> [ϕ > (x)] 2 is a solution of
the inequality f (x) > φ (x).
the variable but because of raising to a power without
investigating the signs of both members of the inequality.
Illustration 27. Solve the inequality x + 1 >  √
x + 3.

w
The proof of the theorem is complete.
Note that in the statement of the theorem the strict Here is an instance of reasoning that gives rise to

.Ea
inequalities f (x) > φ (x) and [f (x)]2 > [φ (x)]2 may be
replaced by the weak inequalities f (x) ≥ φ (x) and [f(x)]2
extraneous solutions : "The domain of the variable of our
inequality is x ≥ – 3. For any x in the domain we have a

syE
≥ [φ (x)]2 . The proof of this fact is carried out in the same non-negative number (principal square root) on the right;
way as the proof of the theorem. hence, the number on the left is a positive number. For
When equations are raised to a power, it is only this reason, squaring yields the equivalent inequality x2 +

ngi
possible to introduce extraneous solutions, which may x – 2 > 0, the solution of which is x > 1 and also x < – 2.
occur due to an extension of the domain of the variable or Taking into account the domain of the original inequality,
when the signs of the two sides of the equation are we get the answer : the solution of the original inequality
disregarded. Similarly, extraneous solutions can be
introduced in the solution of inequalities; they too are
introduced because of an extension of the domain of the
nee
consists of all values of x > 1 and also of all values of x
located in the interval – 3 ≤ x < – 2.’’
Actually, all values of x in the interval – 3 ≤ x < – 2 are
variable and also when the signs of the two members of
the inequality are disregarded. Below are some examples rin
not solutions, to the original inequality. The point is that
for x in the domain, the right member of the inequality is
which illustrate how extraneous solutions are introduced
in both cases.
g.n
indeed non-negative, whereas the left member is negative
for certain values of x located in the domain and is non-

e
However, unlike the case of equations, raising an negative for others. It is clear that for those values of x in
inequality to a power can result in the loss of solutions as
well. The reason why student, make mistakes here is that
they remember that raising an equation to a power cannot
result in the loss of a solution but forget that raising an
the domain for which the left member is negative, the

inequality among them. It is thus necessary to seek t


inequality is invalid and so there are no solutions of our

solutions of the original inequality among those values of


x in the domain for which the left-hand member of the
inequality to a power can result in the loss of solutions.
We will show below how it is possible to lose a solution inequality is non-negative, which is to say among x ≥ – 1.
when raising an inequality to a power . For these x, both members of the inequality are indeed
Let us begin with an example that illustrates how non-negative, and it can be squared to obtain the
extraneous solutions are introduced due to extension of inequality x2 + x – 2 > 0, which is equivalent to the
the domain of the variable when raising an inequality to a original inequality on the set x ≥ – 1. It is now necessary
power. to choose from among the solutions of the inequality x 2 +
x – 2 > 0 those which satisfy the condition x ≥ – 1. They
Illustration 26. Solve the inequality 
√
(x – 3) (2 – x) will yield the solutions of the original inequality, which
> √
2
4x + 12x + 11. are x > 1.
Some aspirants gave this solution : "Since the right The mistake that was made in the earlier reasoning
and left members of this inequality are non-negative (this was due to the fact that the student did not notice the shift
is because we have principal square roots on the right and in concepts. It is true that for any value of x which is a
left), the inequality may be squared to obtain the solution of the original inequality there is a non-negative
equivalent inequality 5x 2 + 7x + 17 > 0. The quadratic number (principal square root) on the right and a positive

CAT Complete Course | 363

Visit : www.EasyEngineering.net
Visit : www.EasyEngineering.net

number on the left. However, it is obvious that not all solution will consist of all the values of x in the interval
values of x located in the domain will be solutions of the – 1 < x < 2. All these values of x enter into the domain,
original inequality, and so the number on the left will not and so some students wrote that these values constitute
be positive for all x of the domain. The student replaced the answer to the problem.
the words "for any x which is a solution" by the phrase Actually, in thus reasoning they lost the solutions – 2
"for every value of x in the domain." This was his ≤ x ≤ – 1, because it is easy to see that for any number in
mistake. this interval the left member of the inequality is non-
Illustration 28. Solve the inequality negative, while the right member is negative.

√ 4 –√

1 – x– √

2 – x> 0 So as not to lose solutions, the student must keep
careful watch of the signs of the left and right members.
Difficulties here spring up when we begin to The proper solution of this inequality is as follows.
compute the domain of the variable. The domain of this
inequality is defined from the conditions : 2 – x ≥ 0, 1 – x The domain of the variable in this inequality consists
of all x ≥ – 2. The left member of the given inequality is
≥ 0, 4 ≥ √
1 – x.. The first two of these inequalities are
non-negative in the domain, while the right member may
true for x ≤ 1. But both sides of the third inequality are
be positive or negative. Clearly, the original inequality
non-negative for these values of x, and so it can be

ww
squared to get an equivalent inequality : x ≥ – 15. Thus,
the domain of the original inequality is – 15 ≤ x ≤. We
rewrite our inequality thus : √
4 –√
1 – x> √

will be true for all those values of x in the domain for
which the right member is negative. Hence, all the values
of x in the interval 0 > x ≥ – 2 are solutions of the

w
2 – x. Within original inequality.
the domain, both members o f this inequality are non-

.Ea
negative; therefore squaring yields an inequality are non-
negative, therefore squaring yields an inequality, 2 + x >
√
Now let us consider the remaining values of x, that
is, x ≥ 0. Both members of the original inequality are
nonnegative for all these x, and so the inequality can be

syE
1 – x, that is equivalent in the domain. For values of x <
– 2 and such that enter into the domain, the left member
of this inequality is negative, while the right member is
squared to obtain x + 2 > x 2, which is an equivalent
inequality for all x ≥ 0.
The solution of the last inequality consists of all x in
non-negative, which means that there are no solutions to
the original inequality among these values of x. It remains
to consider the values of x in the interval – 2≤ x ≤ 1. For ngi the interval – 1 < x < 2. In this case, the solution of the
original inequality consists of all values of x in the
these x, both members of the inequality 2 + x > √
non-negative, and so squaring yields the quadratic
1 – x are

inequality x 2 + 5x + 3 > 0, which is equivalent to the


nee
interval 0 ≤ x < 2.
Combining these two cases, we find that the solution
to the original inequality will consist of all values of x
original inequality on the set – 2≤ x ≤ 1. This latter
inequality holds true for x > (– 5 + √ 13)/2 and for x < rin
lying in the interval – 2 ≤ x < 2.

g.n
In the next problem, it will be possible to lose
(– 5 – √
13)/2. Now to get the answer we have to choose solutions if one fails to take into account the signs of the
from among these solutions those which lie in the interval right and left members of an intermediate inequality.
– 2 ≤ x ≤ 1. These consist of all values of x in the interval
(– 5 + 
√13)/2 < x ≤ 1. They are the ones which constitute
the answer to this problem. √

x2 + 3x + 2 < 1 + 
√ e
Illustration 30. Solve the inequality

x2 – x + 1
The domain of the variable here consists of two
t
Note that if we had not taken the domain of the
intervals : x ≤ – 2 and x ≤ – l. In the domain, both
variable into account, we would have introduced extrane-
members of our inequality are non-negative and so
ous solutions, for example, all x > 1; and if we had not
squaring yields the equivalent (in the domain) inequality
taken into consideration that the inequality 2 + x > √
1–x
has solutions only for – 2 ≤ x ≤1, we would also have 2x < √ 
x2 – x + 1.
introduced extraneous solutions, for example, all the (a) For x ≤ – 2 and – 1 ≤ x < 0, this inequality is true
values of x < (– 5 – √
13)/2. since for each of these values of x there is a negative
number on the left and a positive number on the right.
Let us now examine some problems in which one Thus, all these values of x are solutions to the original
can lose solutions by raising the inequality to a power. inequality.
Illustration 29. Solve the inequality 
√
x + 2 > x. (b) For x ≥ 0, both members of the inequality 2x <
If we square this inequality at once, we will lose √ 
2
x – x + 1 are non-negative and so squaring yields the
solutions even if we take into account the domain of the equivalent (for these x) inequality 3x2 + x – 1 < 0. The
variable. Indeed, the domain for this inequality is x ≥ – 2. solution of this inequality consists of values of x in the
Squaring, we get the inequality x + 2 > > x2, whose interval (– 1 – √ 13)/6 < x < (– 1 + √13)/6.
364 | CAT Complete Course

Visit : www.EasyEngineering.net
Visit : www.EasyEngineering.net

Taking Condition (b) into account, we find that in the 17. Solve the inequality
latter case the solution of the original inequality will (logx 2) (log2x 2) (log2 4x) > 1
consist of all values of x in the interval 0 ≤ x < (– 1 + 18. Solve the inequality
√13)/6. 1 + 3 tan x

√ 3 + 2 tan x – tan2 x ≥
Combining both cases we get the answer : x ≤ – 2 2
and also – 1 ≤ x < (1 + √13)/6. 19. Solve the inequality

(24 –142x – x ) > 1


2
It will be noted that those students who did not log25 – x2
consider the cases (a) and (b) and squared the inequality 16
2x <  √
x2 – x + 1 from the start naturally lost some of the 20. Solve the inequality
solutions. Most likely what happened was that since at
the beginning of the solution of the inequality the signs of log2 cos x √

1 + 2 cos 2x < 1
the left and right members had already been investigated, 3

there was a kind of loss of "vigilance" in the second 21. Solve the inequality
squaring. x4·7 log 71/3 5 ≤ 5–log1/x 5
Exercise

ww
22. Solve the inequality
1. Solve the inequality sin x – cos x > 0. (x2 + x + 1)x < 1
2. Solve the inequality 23. Solve the inequality

w
x (x + 1) (– x + √
 2) (x2 – x + 1) (3x + 1)2 (x +

.Ea
× (1 – x) (2x – π2 ) (– x + π) (x – sinx) < 0
3. Solve the inequality
17)3
√ Logx2

24. Solve the inequality


( )
4x – 5
|x – 2|

1
2

4. Solve the inequality syE


9x – 10·3 x + 9 ≤ 0

5
logx 2x ≤ √
25. Solve the inequality

logx (2x3)

x–5
Log 2 2 x + 3log2 x ≥ log4√2 16

5. Solve the inequality


2
ngi 
√ 4 sin2 x – 1 logsin x

Solutions
2x – 1
≥0

1 (x6 – 2x3 + 1)1/2


2() nee 1. Using a consequence of the addition formula and
π/4 as an auxiliary angle (we call this the auxiliary-angle
< ()
1 1–x
2
rin
formula), we get the inequality √  2 sin [x – (π/4)] > 0. Of
course it can be solved by considering the graph of the
6. Solve the inequality
5 + 2 cos 2x ≤ 3|2 sin x – 1|
g.n
function y = sin [x – (π/4)]. However, it is best to do
otherwise. Denoting x – (π/4) by z, let us consider the
inequality sin z > 0. Its solution 2π k < z < π + 2πk, k = 0,
7. Solve the inequality
log5 .sinx > log125 (3sin x – 2)
8. Solve the inequality,
function y = sin z. Now, substitutinge
± 1, ± 2,.,... is directly obtained from the graph of the

x – (π/4) in place of z, we find the appropriate t


cos [π(x2 – 10x)] – √
 3 sin [π (x2 – l0x)] > 1 intervals of variation of x :
π 5π
9. Solve the inequality 
√ x > – 1. + 2k π < x < + 2k π , k = 0, ± 1, ± 2.
4 4
10. Solve the inequality 
√
log10 x > 0. This technique-replacing x – (π/4) by z-enabled us to
11. Solve the inequality log2-x (x – 3) ≥ – 5. dispense with constructing the graph of the function y =
sin [x – (π/4) . Its convenience is still more evident when
12. Solve the inequality 
√
x + 2+ 
√
x – 5≥ 
√
5 – x. solving elementary trigonometric inequalities with a
13. Solve the inequality 
√
2 + x – x2 > x – 4. complicated argument. For example, it allows us to get
around constructing an extremely involved graph when
14. Solve the inequality 
√
sin x + 2 cot x < – 1.
solving inequalities like sin (√  2 x + 7) –1/2. Here of
15. Solve the inequality course it is easier to denote √2x + 7 by z and solve the
(x – 2)/(x + 2) ≥ (2x – 3)/(4x – l) inequality sin z > – 1/2 using the graph of the function y =
16. Solve the inequality sin z, and then pass to x.
5x + 4 5π Higher-degree algebraic inequalities can also be
log1/2 > tan
x–2 4 classed as elementary inequalities. Students sometimes

CAT Complete Course | 365

Visit : www.EasyEngineering.net
Visit : www.EasyEngineering.net

solve them by investigating various cases, which is to which is equivalent to the original one for all x #
say, by passing to a solution of several systems of – 1/3(since we multiplied three parentheses by – 1, the
inequalities. Confusion often begins when the student is sense, of the inequality is reversed).
not able to find the common portion of the solutions and
Plot the numbers 0, – 1, – √
17 1, π2/2, √ 2 and sin2 1
is undecided about whether or not to combine these
on the real number line
solutions, yet there is a unified standard method for
solving such inequalities. It is the so-called method of in-
tervals that we now give.
Suppose, for example, we have to solve the inequality
(x – x1) (x – x2) … (x – xn–1) (x – xn) < 0
Fig. 8
Where x 1 , x2, ..., xn are distinct real numbers. We will
(Fig. 8). Then the last inequality is true for x located
assume that
in the intervals
x1< x 2 <…. < xn–1 < xn
Plot these points on the real number line x<–√ 17, – l < x < 0, sin2 1 < x < 1,
 2 < x < π, π2 < x
2

ww The solutions of the original inequality are these


values of x, with the exception of x = –l/3, that is,

w
Fig. 7
(Fig. 7) and consider the polynomial x<–√ 17, – 1 < x < – 13 , – 31 < x < 0,

(x – xn–1) (x – xn) .Ea


P (x) = (x – x1) (x – x2)….
…………………………(1)  2< x < π, π2 < x
sin 2 1 < x <1 ,√
2

syE
It is clear that for all x > xn all the parenthetic
expressions in (1) are positive and, hence, for x > xn we
have P (x) > 0. Since, for x n–1 < x < x n the last parenthesis
It is also to be noted that the weak inequality
(x – x1) (x – x2) .,. (x – xn) ≤ 0
in the expression P (x ) is negative, and all the other
parentheses are positive, it follows that for Xn -1 < X < Xn
we have P (x) < 0. Similarly, we obtain P (x) > 0 for Xn-2 ngi can also be solved by the method of intervals, but the
answer is written in the form of the intervals x i ≤ x ≤ xi+1
with the endpoints included.
< X < X n-1 and so on. That is the underlying idea of the
method of intervals. On the number line, the numbers X1
X2,……, Xn must be arranged in order of increasing
nee Frequently, problems involving inequalities can be
reduced to elementary inequalities by means of simple
magnitude. Then place the plus sign in the interval to the
right of the largest number. In the next interval (from
unknown.
rin
algebraic manipulations and the introduction of a new

right to left) place the minus sign, then the plus sign, then
the minus sign, etc. The solution of the inequality P (x) < g.n
3. Denoting 3 by y, rewrite the inequality thus : y2 –
10y + 9 ≤ 0. This quadratic inequality is true for all values
of y in the interval 1 ≤ y ≤ 9.
0 will then consists of intervals having the minus sign.
2. It is quite obvious that if we reduce this inequality
to systems of inequalities, then we will have a large
number of cases to consider.
e
Substituting 3x in place of y, we obtain that the
original inequality holds true for all x satisfying the
double inequality 1 ≤ 3x ≤ 9. t
Let us solve it by the method of intervals. First, we Solving this elementary exponential inequality, we get
have to reduce it to the proper form. Note that x2 – x + l the answer 0 ≤ x ≤ 2.
> 0 for any value of x and for this reason this factor can
4. Denoting y = log2 x and noting that 5/2 log 4 √2 16 =
be cancelled from both members of the inequalities.
4, we rewrite our inequality thus : y 2 + 3y – 4 ≥ 0. The
Further note that (3x + 1)2 > 0 for x ≠ – 1/3 and therefore
solution set of this quadratic inequality is made up of all y
this factor can likewise be cancelled. Remember however
≥ 1 and also all y ≤ – 4. Hence, the original inequality
that x = – 1/3 is not a solution of the inequality. Besides, it
will hold true for all x for which log 2 x ≥ 1 and also for
is clear that the sign of (x + √
17)3 coincides with that of x those x for which log2 x ≤ – 4. Solving these elementary
+ √17 and therefore we can replace (x + √17)3 by x + √ 17 logarithmic inequalities by means of Property VIII of
without impairing the inequality. Finally, represent each logarithms, we get the answer : x ≥ 2, 0 < x ≤ 2–4.
factor as x – a, where a is a number. 5. If we disregard the exponents, we can say that this
All these manipulations result in the inequality is an elementary exponential inequality with base less
(x – 0) [x – (– 1)] (x –√ 2) [x – ( – 17)] × (x – 1)
√ than unity : (1/2) a < (1/2)b . Solving it, we find that the
π2 original inequality is equivalent to the inequality (x6 – 2x 3
(x – ) (x – π) (x – sin2 1) > 0
2 + 1)1/2 > 1 – x.

366 | CAT Complete Course

Visit : www.EasyEngineering.net
Visit : www.EasyEngineering.net

Since, (x 6 – 2x3 + 1) 1/2 = √ 


(x3 – 1)2 = |x 3 – 1| , it sin x – 2 > 0. Denoting y =sin x, we arrive at the system
follows that we have yet to solve the inequality of inequalities
|x3 – 1| > 1 – x y3 – 3y + 2 > 0
Since, the left member here is non-negative, it is 3y – 2 > 0
automatically satisfied for 1 – x < 0, that is, when x > l. Regrouping, represent the left member of the first
We now consider x ≤ 1. In this case, x 3 ≤ 1, and so inequality as
|x3 – 1| = 1 – x3. y3 – 3y + 2 = y (y2 – 1) – 2 (y – l)
and we have the inequality l – x3 > 1 – x or x (x – 1) = (y – l) (y2 + y – 2)
(x + 1) < 0
= (y – l)2 (y + 2)
Solving this inequality by the method of intervals,
It then follows that this inequality is true for all y >
we find that it is true for x < – 1 and for x located in the
– 2 with the exception of y = 1.
interval 0 < x < 1. All these values of x lie in the domain
x ≤ 1 under consideration and so are solutions of the The second inequality of this system is valid for y >
original inequality. 2/3. Hence, the solution of the system includes all y > 2/3,
Thus, the original inequality is valid for x < – 1, 0<x except y = 1.

ww
<1, x > 1.
6. Taking advantage of the formula for the cosine of
a double angle and denoting sin x by y, we can rewrite
Returning to x, we find that the original inequality is
equivalent to the following double inequality : 2/3 < sin x
< 1.

1/2. w
our inequality as 7 – 4y 2 ≤ 3 |2y – 1|. To get rid of the
absolute-value sign, consider two cases : y ≥ 1/2 and y <

.Ea
(a) Suppose y ≥ 1/2, then our inequality is written 7 –
The solutions of this elementary trigonometric ine-
quality are given by the intervals
arcsin 2/3 + 2kπ < x < π/2 + 2kn,
n/2 + 2kπ < x < π – arcsin 2/3 + 2kπ,

syE
4y2 ≤ 3 (2y – l) or 2y2 + 3y – 5 ≥ 0. The solution set of the
latter inequality is y ≥ 1 and y – 5/2. But taking into
account that we only consider y ≥ 1/2, we find that this
k = 0, ± 1, ± 2, ...
8. Putting y = n (x2 – l0x), rewrite the inequality as
condition is satisfied by y ≥ 1 alone.
(b) Let y < 1/2. Then the original inequality is
rewritten 7 – 4y2 ≤ – 3 (2y – l) or 2y2 – 3y – 2≥ 0. The
ngi 1
2
cos y –
 3 sin y > 1

2 2

nee
Using the auxiliary-angle formula, we get cos [y +
solution set of this last inequality consists of y ≥ 2 and y (π/3)]> 1/2. The solution of this elementary inequality
≤ – 1/2. But condition (b) is satisfied solely by y ≤ – 1/2. consists of the intervals
Thus, the solutions of the inequality in y are y ≤– 1/2
and y ≥ 1. If in these inequalities we replace y by sin x,
π π π

rin
– + 2k π < y + < + 2k π, k = 0, ± 1, ± 2……
3 3 3

g.n
we find the solutions of the original inequality to be all x Reverting to x, we find that for every integer k we
that satisfy the elementary trigonometric inequality sin x have to solve the following system of quadratic
≤ –1/2 and all x satisfying the inequality sin x ≥ 1. inequalities :
The solution set of the first inequality consists of all
x lying in the intervals


6
π
+ 2k π ≤ x ≤ – + 2k π, k = 0, ± 1, ± 2,……
6
x2 – 10x – 2k < 0
e
x2 – 10x – 2k + 2/3 > 0
t
The first inequality has solutions if and only if the
discriminate of the quadratic expression x2 – 10x – 2k is
The second inequality will be true only for those positive, that is, 25 + 2k > 0 or k ≥ – 12 (k an integer).
values of x for which sin x = 1; that is, for And so the second inequality of the system will also be
π considered only for k ≥ – 12.
x = + 2k π k = 0, ± 1, ± 2……
2
Note that for these k, the discriminate of the second
Thus, finally, the solution set of the original inequality is also positive. For any fixed k ≥ – 12, the
inequality consists of all solution of the first quadratic inequality is the interval 5 –
x = π/2+2k π and all x located in the intervals 
√ 25 + 2k < x < 5 +  √
25 + 2k while the solutions of the
5π π second one consist of two infinite intervals : x > 5 +
– + 2k π ≤ x ≤ – + 2k π, k = 0, ± 1, ± 2,……
6 6

√ 25 + 2k – (2/3) and x < 5 – √ 
25 + 2k – (2/3). The
7. Noting that log5 sin x = log125 sin3 x, we rewrite common portions of the solutions of these two
our inequality as inequalities (in the terminology of set theory we would
Log 125 sin3 x > log125 (3 sin x – 2) say "the intersection of the solutions of these two
Now, applying Property VIII of logarithms , we see inequalities") yield the solution of the system and, hence,
that our inequality is equivalent to the inequality sin3 x >3 of the original inequality. Clearly, √ 
25 + 2k – (2/3)

CAT Complete Course | 367

Visit : www.EasyEngineering.net
Visit : www.EasyEngineering.net

<√
25 + 2k for all k ≥ – 12. Taking this remark into found the solutions without computing the domain, in the
account, it is easy to write out the answer : third, fourth and fifth we first found the domain and this
immediately gave us our answer. On the contrary, in the
5–√

25 + 2k < x < 5 – √

25 + 2k – 2/3, sixth example it would have been a complicated job to
5+√ 
25 + 2k – 2/3 < x < 5 +  √
25 + 2k find the domain; what is more, it would have been
where k is an integer ≥ – 12. senseless since there were no solutions anyway among
the permissible values of x.
Besides inequalities that are combinations of
elementary inequalities, the student often has to deal with For this reason, when solving complicated problems,
inequalities in the solution of which he has to apply it is sometimes useful to find the domain at the start, but
various transformations and the associated concepts. occasionally this is useless since later on it turns out to be
superfluous for the given case. A general piece of advice
9. In this question we will show how the concept of
may be given : if computing the domain is not compli-
the domain of the variable is used. Since, the left member
cated, then it is best to do so (since it will never do any
is a non-negative expression, the inequality is true for all
harm), but if it is a complicated affair, then put off
values of x for which it is meaningful, that is to say, in the
computing the domain until it is really needed.
domain of the variable x. But the domain of this
inequality consists of the set x ≥ 0; this is the solution of At competitions one often encounters problems that

ww
the inequality.
10. This time again, the expression on the left-hand
side is non-negative and so the inequality holds true for
require transformations which can result in a loss of
solutions or the introduction of extraneous solutions. Here
again, as in the case of equation solving, a principal role

w
all x in the domain of the variable, with the exception of

.Ea
those for which the left member vanishes. This domain is
determined by the condition log10 x ≥ 0; which is to say it
is the set x ≥ 1. But when x =1, the left member vanishes
is played by the concept of equivalence. Previously we
examined the equivalence of equations and demonstrated
why the student has to be sure that the newly derived
equations and the original equations are equivalent. All

syE
and so this value of the unknown is not a solution of the
inequality; the interval x > 1 constitutes the solution of
the original inequality.
this basically holds true for inequalities as well, in fact it
is still more important than for equations.
Indeed, for equations it usually suffices to point out
11. The domain of the variable here is defined by the
conditions x – 3 > 0, 2 – x > 0, 2 – x ≠ 1. But the
ngi that for a certain transformation certain extraneous roots
may be introduced and then to check the roots. In the case
of inequalities, it is not possible to verify solutions by

nee
inequalities x – 3 > 0 and 2 – x > 0 do not have common
substitution since ordinarily there are an infinity of
solutions. Hence, the domain of our inequality does not
solutions. It is therefore necessary to pay special attention
contain a single number and so the inequality does not
to the derived and original inequalities being equivalent.
have a solution.
12. The domain of the variable is defined by the
rin
It is to be noted that the transformations which lead to
non-equivalence of equations naturally lead to non-
inequalities x + 2 ≥ 0, x – 5 ≥ 0, 5 – x ≥ 0. But this system
of inequalities has the solution x = 5. Hence, the domain
of the original inequality consists of the unique solution x
equivalent inequalities.
g.n
Certain manipulations only extend or restrict the
= 5. Therefore, no transformations are needed to solve
this inequality since it is sufficient to verify that it is satis-
fied for x = 5. A direct verification shows that x = 5 is the
solution.
e
domain of the variable of the inequalities. A general
procedure can be suggested for such transformations :
manipulations restricting the domain are forbidden since
t
that might result in a loss of solutions; as for manipula-
tions extending the domain, first carry them out and then
13. The domain of this inequality is the interval – 1 ≤ choose from the solutions of the final inequality those
x ≤ 2. Thus, the left member of the original inequality values which enter into the domain of the original
assumes real and non-negative values for – 1 ≤ x ≤ 2. It inequality. These will yield the answer.
is meaningless for other values of x. But it is obvious that The most common types of transformations that alter
the right member of the inequality is negative of all x < 4 domains are the "identity transformations", which have
and, in particular, for all x in the interval – 1 ≤ x ≤ 2; thus already been mentioned. Besides these, the solution of
the proposed inequality is valid. Hence, the solution of inequalities involves other transformations as well :
the inequality is the interval – 1 ≤ x ≤ 2. clearing of fractions, taking certain functions of both
14. The left member of this inequality is non-negative members. These include powering, taking logarithms,
for all permissible x and, consequently, it cannot be true antilogarithms, and the like. We will now take these up in
for any value of x, which means there are no solutions. more detail.
The foregoing examples make it clear that we cannot We start with the most "harmless" one, that of
give a general recipe of how to employ the notion of the clearing fractions. Recall equations. There is no loss of
domain of the variable of an inequality in various specific solutions when clearing fractions, and extraneous
cases. In the first two examples we simply could not have solutions are introduced only due to the extension of the

368 | CAT Complete Course

Visit : www.EasyEngineering.net
Visit : www.EasyEngineering.net

domain of the variable, which is to say, via adding to the The roots of the quadratic expression x2 – 5x + 4, i.e.,
domain of the original equation those values of the x1 = 1 and x 2 = 4, are the solutions of our inequality.
unknown which make the denominator vanish. We will now assume that x ≠ 4 and x ≠ l. and we will
Many think that the same holds true of inequalities, solve the inequality
and so they "solve" the inequality l/x < 1 this way : (x – 1) (x – 4)
"clearing fractions we get 1 <x; all these values of x yield ≥0 …(3)
(x + 2) (x – 1/4)
the solutions of the original inequality since the
At this point, students often reduce the inequality to
denominator of the original inequality does not vanish for
two systems of inequalities : the numerator and
any value."
denominator are both greater than zero or are both less
But it is easy to see that the original/inequality holds than zero. It is simpler however to solve it by the method
true for all negative values of x as well. All these of intervals.
solutions are thus lost by the student because clearing of
Multiply both sides of the last inequality by the
fractions in equations is quite different from that
expression (x + 2)2 × (x – 1/4)2 , which is positive for the
operation in inequalities.
x under consideration. Then for all these values of x our
Actually, clearing of fractions in an equation (or inequality will be equivalent to the following one :
inequality) consists in multiplying both members of the

ww
equation (or inequality) by the expression in the
denominator. In this operation, equations remain
equivalent if they are multiplied by a non-zero expression,
(x + 2) (x – 1/4) (x – 1) (x – 4) > 0 …(4)
This inequality is in a form convenient for applica-
tion of the method of intervals.

w
but for inequalities this property is more involved :

.Ea
multiplication of both members of an inequality by a
positive expression does not change the sense of the
inequality, multiplication by a negative expression re-

syE
verses the sense of the inequality.
Therefore, when multiplying both members of the
inequality at hand by x, one should have taken into
Fig. 9
Fig. 9. shows us that the solutions of the last
account that the x could have assumed negative values as
well as positive values, and then he should have reversed
the sense of the inequality in the latter case. ngi inequality consist of all x in the intervals x < – 2, 1/4 < x
< 1, 4 < x. Since, we have already found that x = 1 and x
= 4 are solutions to the original inequality, we get the
Thus, in every case when we wish to multiply both
members of an inequality by an expression that is nee
answer :
x < – 2, 1/4 < x ≤ 1, 4 ≤ x
dependent on x and assumes both positive and negative
values, the student should examine the two appropriate
rin
In the foregoing solution, we replaced the inequality
(3) by the Inequality (4) by multiplying, the first one by

g.n
cases. This rule is often forgotten and is the cause of a lot the square of the denominator. Similarly, we can assure
of trouble. ourselves that, generally, the inequalities
15. The domain of the variable in this inequality p (x)
consists of all values of x except x = – 2 and x = 1/4. From
now on we will consider only those values of x which lie
in the domain. At the examination, many students cleared
fractions and wrote that it can be replaced by the
q (x)
e
> 0 and p (x) q (x) > 0

are equivalent. Therefore, to solve the inequality


p (x)
>0
t
following inequality : q (x)
(x – 2) (4x – 1) ≥ (2x – 3) (x + 2)………………. (2) where P (x) and Q (x) are polynomials, one applies
This is clearly wrong since the manipulation actually the method of intervals to the inequality P (x) Q (x) > 0,
amounts to multiplying both members of the original which need not even be written out explicitly, it being
inequality by the expression (x + 2} (4x – l), which may sufficient to locate the roots of the polynomials P (x) and
be negative as well as positive. The original inequality Q (x) on the number line and affix the appropriate sign to
may be replaced by (2) if and only if the expression (x + each of the resulting intervals.
2) (4x – 1) is positive, and so also we have to consider the 16. Noting that tan (5 π/4) = 1 and applying Property
case. Then it is negative. Thus, the solution of the original VII of logarithms, we see that our inequality is equivalent
inequality reduces to solving systems of inequalities. to the double inequality 0 < (5x + 4)/(x – 2) < 1/2 or, what
It is simpler however to do as follows. Transpose all is the same thing, to the system of inequalities
terms of the original inequality to the left side and reduce 5x + 4
>0
it to a common denominator : x–2
2 (x2 – 5x + 4) 5x + 4 1
≥0 <
(x + 2) (4x – 1) x–2 2

CAT Complete Course | 369

Visit : www.EasyEngineering.net
Visit : www.EasyEngineering.net

Transposing 1/2 to the left member of the second The domain of the variable in (5) is the interval – 1 ≤
inequality and carrying out the obvious manipulations, y ≤ 3. Our inequality is obvious for those values of y in
we rewrite it in the form [x + (10/9)]/(x – 2) < 0. the domain for which 1 + 3y < 0; that is, all values of y in
We use the method of intervals to solve each of the the interval – 1 ≤ y <– 1/3 are solutions of inequality (5).
inequalities of this system and find that the first inequality It remains to consider Case (b) : – 1/3 ≤ y ≤ 3. Here
holds true for x > 2 and for x < – 4/5, while the second both members of (5), are non-negative and so squaring in
one is valid for x located in the interval – 10/9 < x < 2. the case at hand yields the equivalent inequality 13y 2 – 2y
We now have to find the common part of these solutions – 11 ≤ 0.
(their intersection, in the terminology of set theory). This The solution of the last inequality consists of all
is conveniently done on the number line (Fig. 10). values of y in the interval – 11/13 ≤ y ≤ 1. Taking into
consideration Condition (b), we find that in this case the
solution of inequality (5) consists of all values of y in the
interval – 1/3 ≤ y ≤ 1.
Combining both cases we find the solution to
Fig. 10 inequality (5) to be all values of y in the interval – 1 ≤ y
≤ 1.

ww
Plotting the points – 10/9, – 4/5 and 2, we denote the
solutions of the first inequality by the broken line and the
solutions of the second one by the solid line. The
Returning to x, we find the solution of the original
inequality to be all x satisfying the inequality – 1 ≤ tan x
≤ 1. We solve this elementary trigonometric inequality to

w
overlapping (common) portion of these two ranges is
readily found to be – 10/9 < x < – 4/5. This is the solution
of the original inequality.
.Ea
17. Using the properties of logarithms, this inequality
get the answer :
π
4
π
– + k π ≤ x ≤ + k π, where k = 0, ± 1, ± 2, ……
4
may be rewritten
log2 4x
syE
log2 x·log2 2x
>1
Ordinarily, taking antilogarithms of inequalities is
only employed in the solution of inequalities involving
the unknown under the logarithmic sign. We have
Denoting log2 x by y, we rewrite the last inequality as
2+y
>1 ngi already considered the solution of elementary logarithmic
inequalities and have seen that they are very simply
solved by taking advantage of Properties VII and VIII of
y (1 + y)
Transposing all terms to the right and reducing to a
common denominator, we get
nee
logarithms.
The more complicated logarithmic inequalities should

rin
therefore also be solved on the basis of these properties.
y2 – 2 This will help the student to avoid many mistakes.
<0
y (1 + y)

g.n
One more remark is in order : despite the fact that
Factoring the numerator, we locate the roots of the taking antilogs is always involved in the solution of
numerator and denominator on the number line (Fig. 11) logarithmic inequalities (either with regard for the
and then apply the method of intervals to get the solution
of the inequality : all values of y in the intervals
–  2 < y < – 1 and 0 < y < √ 2

e
domain of definition of the logarithmic function or by

t
Properties VII and VIII), that term is not always used and
we find phrases like this : "on the basis of the properties
of logarithms (or the logarithmic function) we have.. .".
Taking antilogarithms is investigated in the next few
problems.
19. The natural thing to do is to take antilogarithms.
Fig. 11 Since, the logarithmic base contains x and since the
Recalling that y = log2 x, we see that the solution of properties of a logarithmic function differ according as
the original inequality includes all values of x that satisfy the base is greater than or less than unity, we cannot take
antilogs straight off and will have to consider two cases.
either the inequality – √
 2 < log2x < – 1 or the inequality 25 – x2
0 < log2 x <  2. The solution of the first inequality
√ (a) Let >1, that is, x 2 < 9. In this case the
16
– √2
consists of the x in the interval 2 < x < 2–1; the solution given inequality is equivalent to
of the second inequality consists of the values of x lo- 24 – 2x – x2 25 – x2
>

√2 14 16
cated in the interval 1 < x < 2 .
This inequality may be rewritten as x2 + 16x – 17 <
18. Denoting tan x by y, rewrite the inequality as 0. The solution of this inequality consists of all values of
2 
√ 3 + 2y – y2 ≥ 1 + 3y …(5) x in the interval – 17 < x < 1, but the condition of this

370 | CAT Complete Course

Visit : www.EasyEngineering.net
Visit : www.EasyEngineering.net

case (that is, – 3 < x < 3) is only satisfied by those x √ 6 + 2k π < x < – π + 2k π,
located in the interval – 3 < x < 1. All these values of x – arc cos
4 6
constitute the solution of the original inequality in the
case at hand. π √ 6 + 2k π
+ 2k π < x < arc cos
6 4
(b) Now, let 0 < (25 – x2)/16 < 1. Here, the original
inequality is equivalent to the double inequality where k is any integer (Fig. 12).
24 – 2x – x2 25 – x2
0< <
14 16
Thus, in this case we have to solve the following
system of double inequalities :
25 – x2
0< <1
16
24 – 2x – x2 25 – x2
0< <
14 16
The first one is readily reduced to 9 < x 2 < 25 and its
solution consists of two intervals : – 5 < x < – 3 and 3 < x

ww
< 5. The second double inequality is equivalent to the
system of inequalities
x2 + 2x – 24 < 0
Fig. 12
With respect to taking logarithms of inequalities, it is

w x2 + 16x – 17 > 0

.Ea
The first inequality of this system has the solution – 6
< x < 4, the solution of the second one consists of two
easy to see in which cases this operation leads to an
equivalent inequality. However, it is well to bear in mind
that unwise logarithm-taking of inequalities can result in
a restriction of the domain of the variable and a loss of

syE
infinite intervals x > 1 and x < – 17, so the solution of the
latter system is the interval 1 < x < 4.
We now have to choose from these values of x those
solutions. Therefore, prior to taking logarithms always
check to see that both members of the inequality are
positive. Only then (and naturally with regard for the base
which satisfy the first double inequality; they are the
values of x in the interval 3 < x < 4.
Thus, combining the two cases, we have the solution ngi of the logarithm) are we able to generate an equivalent
inequality.
Earlier we solved the elementary inequality (1/3)x <
of the original inequality which consists of two intervals :
– 3 <x < 1 and 3 < x < 4.
20. Denoting cos x by y and taking advantage of the
nee
2, using the properties of the exponential function let us
now solve this inequality by taking logarithms. Since,
both members of the inequality (1/3)x < 2 are positive, we
formula for the cosine a double angle, we rewrite the
inequality as rin
can take advantage of property VIII of logarithms and
take logarithms of the inequality to the base 1/3 to get
log2y/√3 √

4y2 –1 <1
The domain of this inequality is defined by the
…(6)
g.n
log1/3 (1/3)x > log1/32 (note that the sense of the inequality
has been reversed!) whence x > log1/32.

conditions y2 > 1/4, y > 0, y ≠ √  3/2, or y > 1/2 and y ≠


√ 3/2. Since, the logarithmic base may be greater than 1
or less than 1 for values of y, we consider two cases :
log1/32.
e
Thus, the solution set of the inequality is the set x >

Note that all the elementary exponential inequality


discussed above could have been solved via logarithms. t
(a) Let 1/2 < y < √ 3/2. Then the base is less than 1 Let us solve a few problems by taking logarithms.
and we obtain the equivalent inequality √ 
4y2 – 1 > 2y/√3 21. The domain of the variable here consists of all x
or, since both members positive, 4y2 – 1 > 4y 2 /3. This > 0, except x =1. Since, 7log71/3 5 = 53 and log1/x5 = – logx 5 ,
inequality is true for y 2 > 3/8, or y > √6/4. Taking into our inequality may be rewritten in the form
account the condition of the case at hand, we find the x4·5 3 ≤ 5logx 5
solution of inequality (6) to be the interval 
√6/4 < √ 3/2. Both members are positive within the domain of the
variable and so we can take logarithms of both sides of
(b) Let y > √  3/2. Then we get the equivalent the inequality to the base 5 (greater than unity) and obtain
inequality √ 
4y – 1 < 2y / √
2  3 whence, after squaring, the equivalent (in the domain) inequality 4log5 x + 3 ≤ logx
follows y2 < 3/8. But under the condition of our case y2 > 5. Denoting logx5 by y and transposing all terms to the
3/4, that is, we do not obtain any new solutions to left-hand side, we rewrite the inequality thus; 4y + 3 – 1/y
inequality (6). ≤ 0 or, reducing to a common denominator, thus : (y + 1)
It thus remains to solve the elementary trigonometric (y – 1/4)/y ≤ 0.
inequality √6/4 < cos x < √3/2 which is satisfied for all Now, we apply the method of intervals and find the
values of x in the intervals solution to be y ≤ – 1 and y in the interval 0 < y ≤ 1/4.

CAT Complete Course | 371

Visit : www.EasyEngineering.net
Visit : www.EasyEngineering.net

Now, reverting to x, we see that the original inequality Let x < – 1. Then x 2 + x + 1 > 1. If the quadratic
is true for those values of x for which log5 x ≤ – 1 and also expression x2 + x + 1 is now raised to a negative power x,
for those x for which 0 < log5 x ≤ 1/4. Solving these the result will be less than unity, which means that for all
elementary logarithmic inequalities, we get the answer : x < – 1 we have (x2 + x + l) x < 1.
4
0 < x ≤ 1/5, 1 < x ≤ √ 5. Thus, the solution set of the original inequality
consists of all values of x < – 1.
22. For arbitrary real x, the quadratic expression x2 +
x +1 is positive and therefore the domain of the variable We have been making considerable use of the
consists of all real values of x. concept of domain of the variable of our inequalities.
However, with the exception of just a few very
Since, both sides of the original inequality are elementary cases, we did not stress whether this has been
positive for all x, we take logs to the base 10 to get the helpful or not in solving inequalities. We will therefore
equivalent inequality x log1 0 (x2 + x + 1) < 0. This
consider two examples involving inequalities to see
inequality holds true in two cases : when x satisfies the whether it is necessary to compute the domain of the
system of inequalities variable beforehand.
x>0 23. The domain of x here is defined from the
log10(x2 + x + 1) < 0 conditions (4x – 5)/|x – 2| > 0, x2 > 0, x2 ≠ 1, whence x >

ww
and when x satisfies the system of inequalities
x<0
5/4 and x ≠ 2. But for all these values of x we have x 2 > 1
and so our inequality, by the property of logarithms to a
base exceeding unity, is equivalent (within the domain of

w log10(x2 + x + 1) > 0

.Ea
Let us solve the first system of inequalities. From the
properties of logarithms we find that it is equivalent of
the system
x) to
4x – 5
|x – 2|
≥ x

Since, x ≠ 2, the expression |x – 2| is positive and


x>0
syE
(x2 + x + 1) < 1
therefore the original inequality is equivalent, within the
domain of x, to
Since, the solution of the second inequality of the
system is – 1 < x < 0 and the solution of the first is x > 0,
this system is inconsistent, which means that in this case ngi 4x – 5 ≥ x |x – 2|
we now consider two cases.

original inequality does not have a solution.


The second system is equivalent to nee (a) Let x > 2. Then our inequality will be rewritten as
4x – 5 ≥ x2 – 2x or x2 – 6x + 5 ≤ 0. The solution set of the
last inequality consists of all x in the interval 1 ≤ x ≤ 5,
x<0
(x2 + x + 1) > 1
rin
and the solution set of the original inequality in this case
consists of all values of x in the interval 2 < x ≤ 5.
The solution set of this system consists of all x < – 1,
2
g.n
(b) Now, let 5/4 ≤ x < 2. Then our inequality takes the
whence the solution of the original inequality is the set of form 4x – 5 ≥ – x + 2x or x + 2x – 5 ≥ 0. Its solution set
2

will consist of all values of x in the intervals x ≥  √6 – 1

e
all values of x < – 1.
A different solution of this inequality may be and x ≤ – √

whether the base is greater or less than unity, it is natural interval 


to consider two cases.
 6 – 1. The solution set of the original
suggested. Since the properties of a power depend on inequality will in this case consist of all values of x in the
√ 6 – 1 ≤ x < 2.
Combining both cases, we find the solution of the
t
(a) Suppose that x 2 + x + 1 < 1, or – l < x < 0. For all original inequality to consist of all values of x in the
these values of x, the quadratic x2 + x + 1 is raised to a intervals √  6 – 1 ≤ x < 2 and 2 < x ≤ 5.
negative power x. And since for all these values of x the
24. In this problem, it is not advisable to establish the
trinomial x 2 + x + 1 < 1, it follows that for them (x2 + x +
domain of the variable beforehand since it does not
1) x > 1, which contradicts the condition. Hence, these
simplify the solution and is a rather complicated matter.
values of x cannot be solutions of our inequality.
Without finding the domain of x, we note only that in
(b) Suppose that x2 + x + 1 > 1. This is clearly valid
the domain, x > 0 and x ≠ 1.
for x > 0 and for x < – 1. Therefore we have to consider
two cases here. Denote logx 2 by y and rewrite the inequality as
Let x > 0. Then x 2 + x + 1 > 1 and after raising the y+1≤√ 
y+3 …(7)
expression to a positive power x the sense of the Here, the domain of y consists of all y≥ – 3. But the
inequality remains unchanged, which means that for these inequality is obvious for y in the interval – 3 ≤ y < – 1,
x we have (x 2 + x + 1) x > 1. Hence, neither can these which means that all these values of y constitute the
values of x be solutions of our inequality. solution.

372 | CAT Complete Course

Visit : www.EasyEngineering.net
Visit : www.EasyEngineering.net

Now, let y ≥ – 1. Then both members of (7) are non- zero. Then, we naturally have to take only those solutions
negative; this inequality may be squared to obtain the of the resulting inequality and equation which enter into
equivalent (for y ≥ – 1) inequality (y + 1)2 ≤ y + 3, whose the domain of the variable of the original inequality.
solution consists of all y in the interval – 2 ≤ y ≤ 1. In this This domain is defined by the system of inequalities
case, the solution of (7) consists of all values of y in the x–5
interval – 1 ≤ y ≤ 1. 4 sin2 x – 1 ≥ 0, 0 < sin x < 1, >0
2x – 1
Combining both cases we see that inequality (7) is From the first two inequalities it follows that 1/2 ≤
satisfied for – 3 ≤ y ≤ 1. sin x < 1 ; the third is satisfied for x < 1/2 and x >5.
Now, returning to x we find that the original Let us begin with the inequality logsinx [(x – 5)/(2x –
inequality will have solutions for all x that satisfy the 1)] ≥ 0. Since, sin x < 1, it follows that the inequality is
double inequality – 3 ≤ logx 2 ≤ 1. equivalent, in the domain, to the inequality (x – 5)/(2x –
This inequality may be solved in two ways. 1)≤1, the solutions of which are x ≤ – 4 and x ≥ 1/2 of
First solution : Since, the properties of logarithms these values of x, only x ≤ – 4 and x > 5 lie in the domain
differ for bases greater than or less than unity, we of x. It remains to take into account the inequality 1/2
consider two cases : x > 1 and 0< x< 1. ≤ sin x < 1.
(a) Let x > 1. Then logx 2 > 0, and all the more so The graph shown in Fig. 13 makes it evident that the

ww
logx 2 ≥ – 3. It remains to solve the inequality logx 2 ≤ 1,
whence 2 ≤ x. Thus, here the solution set of the original
inequality consists of all values of x ≥ 2.
solution of this double inequality consists of the intervals
π
6
+ 2n π ≤ x ≤

6
+ 2n π, n = 0, ± 1, ± 2,……

w
(b) Let 0 < x < 1. Then logx 2 < 0 and all the more so

.Ea
logx 2 ≤ 1. It remains to solve the inequality – 3 ≤ logx 2,
whence x –3 ≥ 2, or x ≤ 2–1/3. Thus in this case the solution
set of the original inequality consists of all x in the
with the points x = π/2 + 2n π eliminated. Since, we only
need x ≤ – 4 and x > 5, it follows (this too is found from
the graph) that the values n = 0 and n = – 1 do not satisfy
us, and there remains a portion of the interval
3
interval 0 < x ≤ √
1/2.
syE
Combining both cases we have the solution of the
corresponding to n = – 1 : – (11π)/6 ≤ x ≤ – 4 without x =
– 3 π/2.

3
original inequality : 0 < x < √
1/2 and x ≥ 2.
Second solution : Since, logx 2 = 1/log2 x, then by ngi
denoting log2 x by z, we get the system of inequalities
1–z
≤0 nee
rin
z
1 + 3z
≥0
z
Solving each of these inequalities by the method of
intervals, we find that the system will hold true for z ≥ 1 g.n
Fig. 13
We now consider the equation 4 sin2 x – 1 = 0, from
and also for z ≤ – 1/3.
To get the answer we have to solve the elementary
logarithmic inequalities log2x ≥ 1 and log2 x ≤ – 1/3,
3
e
which, taking into account that sin x, > 0 in the domain,
we get sin x = 1/2. However, we have just solved an
t
inequality which is clearly satisfied by the solutions of
the equation sin x = 1/2. Therefore, all the solutions of the
whence we obtain x ≥ 2 and 0 < x ≤  √1/2. equation at hand have been obtained and there would be
25. At the very start, many students made the mistake no reason to include them if (and this is yet another
of discarding the first factor. Their reasoning probably underwater reef of the given problem) the solutions of the
went like this : since √

4 sin2 x – 1 ≥ 0, it is necessary for double inequality 1/2 ≤ sin x < 1 had not partially been
the second factor to be non-negative as well. This discarded because of the conditions x ≤ – 4 and x > 5. In
argument contains two mistakes at once : firstly, the this operation, the values – 7π/6, π/6 and 5 π /6 were
domain of the variable is extended when the radical is eliminated; the latter two do not enter into the domain of
discarded; secondly, when the first factor is zero, the the original inequality, and the first one, that is, x = – 7
inequality is valid even when the second factor is negative. π/6, is to be adjoined to the intervals obtained above.
The first mistake introduces extraneous solutions, the We thus get the answer :
second mistake results in a loss of solutions. π 5π
+ 2nπ ≤ x ≤ + 2n π, x ≠ π/2 + 2n π
A proper solution must take into account both of 6 6
these items and can be carried out as follows. The given where n is any integer except 0 and 1, – 11 π/6 ≤ x ≤
inequality is valid in two cases : when the second factor is – 4 and also x = – 7π/6.
greater than or equal to zero and when the first factor is ●●●

CAT Complete Course | 373

Visit : www.EasyEngineering.net
Visit : www.EasyEngineering.net

6 Permutations and Combinations


Permutation means ‘arrangement’; Combination 8P 8! 8!
6 = =
means ‘selection’. (8 – 6) ! 2 !
There are two fundamental principle of counting : 8 ×7×6×5×4×3×2×1
=
2×1
1. Product Rule : If one process can be performed in
m ways, and if corresponding to each of the m ways of = 20160
performing this process there are n ways of performing a nP n! n!
n = = =n!
second process, then the number of ways of performing (n – n) ! 0 !

ww
the two operations together is m × n.
Example : If Ram can go from his house to Sita’s
house in 3 different ways and from Sita’s house to the
Or,
0! =
0 = 1
1

w
school in 5 different ways.

.Ea
Then total number of ways of going from his house
to the school is 3 × 5 = 15 ways.
2. Addition Rule : If a work w consists of two parts
Illustration 1. Find the total number of arrangement
possible with 6 letters (A, B, C, D, E, F) taking 4 of them
at a time.
Solution : The total number of arrangements

syE
w 1 and w2 of which one part can be done in m ways and
the other part in n ways, then the work w can be done in
(m + n ) ways.
possible with 6 letters taking 4 of them at a time is given
by
6! 6 ×5×4×3×2×1

ngi
6P = =
4
(6 – 4) ! 2×1
Example : If Ram can go from Meerut to Delhi in 5
different ways and from Meerut to Faridabad in 3 = 30 × 12 = 360
different ways, then total number of different ways he can
go to Delhi or Faridabad from Meerut = 5 + 3 = 8
nee Illustration 2. If n P 4 = 56 × n P 2 . Find the value of n.

Solution :
n!
= 56 ×
n!

rin
Factorial : A factorial is the name obtained by (n – 4) ! (n – 2) !
multiplying all the positive integer less than or equal to a n! n!
given positive integer. = 56 ×
(n – 4) ! (n – 2) (n – 3) (n – 4) !
The factorial of a given integer n is usually written as
n ! or n
Or,
Or, g.n
n2 – 5n + 6 = 56
n2 – 5n – 50 = 0
∴ n ! = n × (n – 1) × (n – 2) × (n – 3) × …2 × 1
Example :
6! = 6 ×5×4×3×2×1
Or,
Or,
Or,
e
n2 – 10n + 5n – 50 = 0
n (n – 10) + 5 (n – 10) = 0
(n – 10) (n + 5) = 0
t
= 30 × 24 n = 10, n ≠ – 5
= 720 Negative value of n is not allowed.
Permutation : ‘n’ number of given things can be nP
5
arranged in different orders by taking some or all of them Illustration 3. n–1 3 = · Find the value of n.
P 4 21
at a time each of these different arrangements is called a
n! (n – 5) ! 5
permutation. Solution : × =
(n – 3) ! (n – 1) ! 21
The number of permutations of ‘n’ different things
taking ‘r’ of them at a time is denoted by n P r where P n (n – 1) ! (n – 5) ! 5
Or, × =
(n – 3) (n – 4) (n – 5) ! (n – 1) ! 21
stands for permutation.
n P is defined as n 5
r Or, =
(n – 3) (n – 4) 21
nP = n !
r
(n – r) ! Or, 21n = 5 (n2 – 7n + 12)
n P = n (n – 1) (n – 2) (n – 3)
r Or, 5n2 – 35n + 60 – 21n = 0
------------- (n – r + 2) (n – r + 1) Or, 5n2 – 50 n – 6n + 60 = 0

374 | CAT Complete Course

Visit : www.EasyEngineering.net
Visit : www.EasyEngineering.net

Or, 5n (n – 10) – 6 (n – 10) = 0 Illustration 6. At an annual meet of a company there


Or, (n – 10) (5n – 6) = 0 are 40 males and 25 females. Each male exchange greet-
ing only with all the males and each female exchange
6
n = 10, n = greeting only with all females. Find the total number of
5
greeting exchanged at the meet.
Since, n represents number of things so it can’t be a
Solution : Each exchange of greeting occurs
fraction.
between two persons, either two males or two females.
∴ Answer n = 10 Exchange of greeting among males from one class while
Combination : Each of different selections or the exchange of greeting among females from another
collections or groups that can be formed by taking some class and the two classes are independent of each other.
or all of a number of given things at a time irrespective of Number of greeting exchanged among males
the order in which the things appear in the group is called
40C = 40 !
a combination. 2
2 ! × (40 – 2) !
The number of combinations of n different things r 40 × 39 × 38 !
of them at a time is denoted by n C r, =
2 × 1 × 38 !
where ‘C’ stands for combinations.

ww
= 780
n!
∴ n Cr = No. of greeting exchanged among females
r ! × (n – r) !
25C = 25 !
Relation between nP r and nC r :

w
= 300
2
2 ! × 23 !
n!

.Ea
n P is written as = Total number of greeting exchanged
r
(n – r) !
= 780 + 300 = 1080
n!
while n C r is written as


∴ n
n P = r ! × nC
r
n
Cn = Co = 1
syE
r ! × (n – r) !
r
Illustration 7. In a class there are 20 boys and 16
girls. They are to be divided into subgroups each con-
taining either 2 boys or 3 girls, what is the maximum

ngi
number of ways of forming such subgroups ?
Illustration 4. Find the total number of possible Solution : Boys have to be divided into subgroups of
groups of 3 men that can be formed out of 5 men A, B, C, 2 each while girls have to be divided into subgroups of 3
D and E.
Solution : We have five men A, B, C, D and E.
∴ Combination of 3 men can be formed as
nee
each. The two types of subgroups are independent of each
other.

ABC ACD BCE = 20C 2 = rin


Total number of possible subgroups among boys
20 !
2 ! × 18 !
=
20 × 19

g.n
ABD ACE BDE 2
CDE ADE = 190
Total number of possible subgroups among girls
ABE BCD
Clearly, number of groups = 10
Alternate Method : The number of groups can be
formed by taking 3 men from 5 men at a time is given by
= 16C 3 =

=
3×2
16 !
e
3 ! × 13 !
16 × 15 × 14
= 16 × 5 × 7 t
5! 5×4×3!
5C = = = 80 × 7 = 560
3
3 ! × 2 ! 3 ! × (2 × 1)
Maximum number of ways of forming such sub-
5×4
= = 10 groups
2
= 190 + 560
Illustration 5 . How many triangles can be formed
with in 5 points which are co-planar and no three points = 750
are collinear? Illustration 8 . A palace has 10 entry gates but and 8
Solution : A triangle is formed by joining 3 points exit gates. Entry gates cannot be used for exit and vice-
which are co-planar but not collinear clearly using versa. In how many ways can a tourist enter the palace
property of combination. We have to select three points and then come out of it?
out of 5 points. Solution : The whole process can be broken into two
So, number of triangles formed = 5 C 3 part—
5! (i) entry and (ii) exit.
=
3!×2! Number of ways in which the tourist can enter the
= 10 palace = m = 10

CAT Complete Course | 375

Visit : www.EasyEngineering.net
Visit : www.EasyEngineering.net

Number of ways in which the tourist can exit from 5P 5!


4 =
the palace = n = 8 (5 – 4) !
5 P = 5 × 4 × 3 × 2 = 120
Once he has entered the palace through any of the 10 4
entry gates. He has 8 ways open to him to exit from the
palace. In other words, we can say that for each way of PERMUTATION
entering the palace he has 8 ways of coming out of it. (A) General Permutation : When ‘n’ different
∴ For 10 ways of entering the palace there will be 10 objects and taken all at a time.
× 8 = 80 ways of coming out of it.
Place 1st 2nd nth
Illustration 9. A young lady has 10 sarees, 8 pairs of Number of ways N (n – 1) n – (n – 1) = 1
shoes, 6 sets of jewellery and 4 watches. In how many to fill
different ways can she dress up? Using multiplication
rule find the number of permutations of n different things ∴ To fill all the places total number of ways
taking r of them at a time or derive the relation = n × (n – 1) × (n – 2) -------------- × 1
n! n = nPn
nP =
r for r ≤ n =
(n – r) !
Example : How many number of 4-digits formed

ww
Solution : We have all n different things and none of
the n things is used more than once.
Let us select a space and divide it into r column and
from four digits 9, 5, 7 and 6?
Solution : According to above formula—

I IIw
two rows.

III
.Ea
------------------------- (r – 1) th rth
Required number of numbers =
= 4 ×3×2×1
= 24
4

syE
Now, from n different things, r of them are to be
selected for putting one of each column.
(B) Permutation without Repetition : We have ‘n’
different objects and ‘r’ objects are taken at a time.

ngi
(I) The first column can be filled in n ways as any Place Ist 2nd 3rd -------- rth
one of the given n things can be chosen to fill up this Number n (n – 1) (n – 2) ------- (n – r + 1)
column. of ways
(II) When the first column has been filled has been
filled up in any one of the n ways. We are left with (n – 1)
things. Then the second column can be filled up in (n – 1)
nee Total number of ways
= n (n – 1) (n – 2) ------------ [ n – (r – 1)]
ways with any one of the remaining (n – 1) things.
(III) When the first two columns have been filled up
= n Pr
rin
g.n
n!
we are left with (n – 2) things. Therefore, the third = n Pr =
(n – r) !
column can be filled up in (n – 2) ways with any one of
Illustration 11. How many of different numbers of 3
(n – 2) things that are remaining.
Similarly, (r – 1)th column can be filled up in {n – (r –
1) – 1} = [n – (r – 2)] ways with any one of the remaining
(n – r + 2) things.
repeating any of them in any number? e
digits can be formed with five digits 1, 2, 3, 4, 5 without

Solution : 3-digit number can be formed as t


∴ So, all the r columns are to be filled in succession, Hundred’s place Ten’s place Unit’s place
we can use the generalized multiplication rule to find the
5 ways 4 ways 3 ways
possible number of ways of filling these r columns.
∴ Total number of ways filling r columns Total number of 3-digit numbers
= n (n – 1) (n – 2) ---------------- (n – r + 2) (n – r + 1) = 5 × 4 × 3 = 60
Total number of permutation of n different things (C) Permutation with Repetition : We have ‘n’
taking r of them at a time. different objects and r blank spaces. Now,
i.e. n P r = n (n – 1) (n – 2) ------------- (n – r + 1)
Place Ist 2nd 3rd --------------- rth
Illustration 10. How many different numbers of 4
Number of ways n n n --------------- n
digits can be formed with five digits 2, 5, 7, 6, 9 without
to fill with n
repeating any of them in any number ?
objects
Solution : Total possible ways of forming these
numbers is equal to the number of permutations of 5 ∴ Total number of ways = n × n × n × ------------ n
digits taking any 4 of them at a time = nr

376 | CAT Complete Course

Visit : www.EasyEngineering.net
Visit : www.EasyEngineering.net

Illustration 12. How many four digit numbers can (F) Objects of some group occur together : ‘n’
be formed with digit 5, 6, 7, 8, 9 repetitions of digits different groups (n ≥ 2) where each group consists of
allowed any number of times? number of the objects ( ≥ 1) such that the objects of the
Solution : The four places in a four digit numbers are some group are always together.

Thousand’s Hundred’s We have,


Ten’s place Unit’s place
place place Number of different groups = N
It can be It can be It can be It can be Number of objects in group I = n1
filled by one filled by one filled by one filled by one
of digits 5, 6, of digits 5, of digits 5, 6, of digits 5, 6, Number of objects in group II = n2
7, 8, 9 in 5 6, 7, 8, 9 in 7, 8, 9 in 5 7, 8, 9 in 5
Number of different objects in group w = nw
ways 5 ways ways ways

∴ Total number of ways of forming four digits Group 1st 2nd 3rd wth
numbers with given 5 digits (5, 6, 7, 8, 9) repetition of No. of objects n1 n2 N3 nw
digits being allowed any number of ways.
No of ways n1 n2 N3 nw
= 5 × 5 × 5 × 5 = 54 = 625

ww
(D) Permutation of Certain Things Occur To-
gether : ‘n’ different objects of which ‘t’ number of
objects are to be always together ( t > 1)
Number of ways objects can be arranged in each
group = n1 × n2 × n3 …… ×

w Block ‘A’

.Ea
Block ‘B’
No. of objects remaining = n – t Objects always together = t

If objects in block ‘B’ is considered as 1.


Number of ways each group can be arranged among
themselves = n
Required number of ways = n × n1 × n2 ……nw

syE
∴ Total number of objects in block = A + B
Hence, number of arrangements = (n – t + 1)
Illustration 14. There are two books each of four
volumes and two books each of three volumes to how
many ways can these books be arranged in a shelf so that
Number of ways ‘t’ objects can be arranged in block
‘B’ itself = t
ngi
the volumes of the some book remain together?
Solution : Formula , Required arrangement
Using product rule :
Required number of arrangements = (n – t + 1) × t
nee = 4 × 4 × 3 × 3

= 24 × 24 × 6 × 6
Illustration 13. Find the total number of words
formed with 6 different alphabets, taking any four of
rin
= 36 × 24 × 24

g.n
them at a time. (G) All objects are not different : n objects of
Solution : We consider 4-different alphabets taking which all are not different but ‘s1 ’ are alike, ‘s2’ are alike,
as a single alphabet. ‘s3 ’ are alike and rest ‘k’ are all different
Now, Total alphabet = (6 – 4) + 1 = 2 + 1 = 3
∴ 4-different alphabets can be arrangements in 4
ways.
=
n!
(s1 !) × (s2 !) × (s3 !)
e
∴ Required number of permutations

t
(H) Arrangements in a row (linear permutation) :
Block ‘A’ Block ‘B’
Two different groups- one group consisting of ‘n’ distinct
3 ways 4 ways objects and other group have r number are arranged in a
row such that
Now, Total number of words from 6-different
alphabets taking any four of them at a time (a) All of ‘r’ are together :
= 3 × 4 Group I Group II
= 3 ×2×4×3×2×1 No of object = n No of objects together = r
= 6 × 24
= 144
If objects in Group II is considered as one, then total
(E) Certain things never together :
no of objects = n + 1
When r number of objects among ‘n’ different
objects never together Required number of arrangement
Required permutation = n – ( n – r +1) × r = n +1× r

CAT Complete Course | 377

Visit : www.EasyEngineering.net
Visit : www.EasyEngineering.net

(b) All of ‘r’ are ever together : Required number = 4 ×3×5×4×3×2


of arrangements = 120 × 12 = 1440
= n + r – (n + 1) × r (I) Circular Permutation : ‘n’ different things taken
all at a time :
(c) ‘r’ objects under restriction :
(1) No two of ‘r’ are together :

1 2 3 4

Given circle can be filled by ‘r’objects to full fill the


restriction . Here, having fixed ‘n’ things, the remaining (n – 1)
Required number of arrangements = (n !) × n+1 Pr things can be arranged round the table in (n – 1) ! Ways.
(d) Objects of both groups are alternate : Particular case : necklace

ww
(i) If n ≠ +
Required arrangements = n × +
Number of arrangements of n beads all different to
1
form a necklace or on a circular wire will be (n – 1) !
2

w
Or (ii)

.Ea
n = +
Required arrangements = 2 ( n )2
Illustration 15. How many different words can be
Explanation : R = Ram G = Gopal S = Sita

syE
formed with the letters of the word ‘illusion’ taken all at a
time, such that all vowels occur together ?

ngi
Solution : In ‘illusion’
We have four (i, u, i, o) vowels

nee
In which there are two
Now,
Required number of words in which all 4 vowels
occur together

= 5×
4 rin
2
= 5 ×4×3×2×1×4×3 g.n
= 120 × 12
= 1440
Illustration 16. In the above question if no two
vowels occur together. Then find the how many different
e t
words can be formed? Round Table
Solution : From the above formula : The above seating arrangement of Ram, Gopal and
1 Sita on the round table are different as shown in figure
No. of words formed = 4 n+1 Pr × and that is why we say (n – 1) ! Clockwise and anti-
2 clockwise make different arrangements.
Here, we have n = 4 (1, 1, s, n)
R = 4 (I, I, o, u)
There are two I, 2
∴ Number of words formed
1
= 4 × 45 P ×
2
5 1
= 4 × ×
1 2

378 | CAT Complete Course

Visit : www.EasyEngineering.net
Visit : www.EasyEngineering.net

Again if repetition of digits not allowed


Number of six-digit telephone number = 9 × 9 P 5
Required number of telephone number
= 9 × 105 – 9 × 9 P5
= 7,63,920

COMBINATION
(A) Combination of ‘n’ distinct objects and taken
all at a time
R = red flower, G = green flower, B = blue flower Required number of combination = n C n = 1
The above arrangements of three flowers to form a (B) Combination without repetition : Here we
necklace is the some because on the necklace we get the have ‘n’ distinct objects and taken ‘r’ objects at a time but
arrangement and that is why we say the total number of no object is repeated.
1 Required number of combinations = n C r
arrangements of n beads for forming a necklace is
2
n (n – 1) ……t or terms

ww
(n – 1) !
Here, clockwise or anticlockwise does not change the
character of the necklace. It remarks the same.
=
r
Illustration 19. Find the number of ways 6 identical

colors?
w
Illustration 17. In how many ways a wedding

.Ea
garland can be formed out of 15 flowers of different
balls can be distributed among 10 identical boxes if not
more than one ball can go into a box.
Solution : Here balls are identical and boxes are also
identical. So, it is the case of combination it also has been

syE
Solution : Number of wedding garland formed from given that one ball can go into one box only i.e., there is
1
out of 15 flowers = (15 – 1) ! no repetition
2
Required number of ways = n C r

ngi
1
= × (14) !
2 Here, n = 10; r = 6
Illustration 18. Find the number of six-digit tele- 10 !
phone numbers in a city if at least one of their digit is
repeated and (i) Zero (0) is allowed at the beginning of
telephone number (ii) Zero (0) cannot initiate the number.
nee Required number of ways = 10C 6 =
6!4!
= 210

(C) Combination of with repetition : ‘n’ distinct

Solution : Here, all the ten digits 0,1,2,----------9


have equal importance since ‘0’ can also start the
may be repeated.
rin
objects and taking ‘r’ objects at a time when each objects

g.n
Required number of combination = n+r–1C r
telephone number.
Illustration 20. A box has 12 balls find the how
Number of six- digit telephone number many ways 8 balls can be selected if any ball may be
= 106 (when digit may be repeated)
If repetition of digit is not allowed
= 10P 6
repeated any number of times.
e
Solution : Using above formula-Required number of
combination = n+r–1C r t
Hence, required number of telephone numbers Here, n = 12, r = 8
= 10 – 10P 6 = 8,48,800 ∴ Required number of combination
(ii) Here , 0 (zero) cannot come at the start of the 19 !
telephone number. = C12+8–1
8 = C819 =
(8 !) (11 !)
Six- digit telephone no
19·18·17·16·15·14·13·12
9 digits except 0 = = 19.18.17.13
8·7·6·5·4·3·2·1
(D) Certain things always occur :

↓ ‘n’ distinct objects taken ‘r’ at a time r ≤ n such that


↓ particular ‘P’ no of objects always occur.
9 ways 9P
5
BlockA Block B
Now,
Number of six- digit telephone number = 9 × 105 Remaining no. of objects No. of objects always
=n–p occur = P
(if repetition of digits allowed)

CAT Complete Course | 379

Visit : www.EasyEngineering.net
Visit : www.EasyEngineering.net

Number of combination in Block B = p C p Illustration 23. Find the number of ways 10 different
Since, all P objects to occur in each combination, things can be divided in two equal groups.
remaining r – p objects are to be taken from n – p objects Solution : Now here n = 10
in block A Number of groups = r = 2
Number of combination in Block A = n–p C r–p 10 !
Required number of ways =
∴ = n–pC r–p· p C p = n–pC r–p
Illustration 21. A basket contain 14 tennis balls, 6
2( )
10
2
!
2

green 4 yellow and 2 blue and 2 white. Find the no. of 10 10·9·8·6
=
ways a player can select some or all of tennis balls. 2·5 !·5 ! 5·4·3·2·2
Solution : Total number of balls n = 14 = 126
Number of green balls, m = 6 (G) Distribution of given different objects in
No. of yellow balls, p = 4 several groups/packets :
No. of blue balls, q = 2 Number of different objects = n
No. of white balls, r = 2 Number of groups = p

ww
∴ Required no. of ways selecting some or all of
tennis balls
Distribution of different objects is made in ‘p’
number of groups
Again each group can be arranged among themselves

w
[(m + 1) (p + 1) (q + 1) (r + 1)] – 1
= [(6 + 1) (4 + 1) (2 + 1) (2 + 1)] – 1 in p! ways
= 7.5.3.3 – 1
= 314 .Ea Group 1st 2nd 3rd -------------- pth

number of objects never occur.


syE
(E) Certain things never occur : ‘n’ distinct object
taken ‘r’ at a time (r ≤ n) when particular ‘p’ (1 ≤ p ≤ n)
↓ ↓ ↓ ↓
Required number of combination = n–pC r
Illustration 22. A person has 10 friends and he ngi n
p
n
p
n
p
n
p
wants to invite 8 of them to a birthday party. How many
times 2, particular friends will never attend the parties ?
Solution : Here, total number of friends, n = 10 ne(e)
Total number of ways for such distribution

=
n!

rin
n p
Particular friends = p = 2 ! ×p!
p
We have to select only 8 friends
So, required number of combination 10–2C 8
= 8C8 = 1 g.n
Illustration 24. Find the number of ways 6- different
sarees can be divided equally among 3 ladies.

e
Solution : Number of sarees = 6 ; Total ladies = 3;
(F) Equal Distribution of given different objects
among persons :
Number of different objects = n
Number of persons = r
they are different.

So, required of ways =


6!
(2 !)3
= 90

Illustration 25. How many different numbers can be


t
Since, distribution is made equally. So, each person formed from the digits 1, 3, 5, 7, 9 (without repetition)
n
gets equal number of objects = when taken all at a time and what is their sum ?
r
Solution : 1, 3, 5, 7, 9 – five digits
Number of numbers = 5! = 120
Persons 1st 2nd 3rd ------------- rth Suppose 9 is the unit place, then the remaining 4 can
be arranged in 4!= 24 ways.
Hence, sum of due to the unit place all the 120
↓ ↓ ↓ ↓
number = 24(9+7+5+3+1) = 600 units.
n n n n
r r r r Again, suppose 9 is in the 2nd place i.e., ten’s place
Total number of ways for such distribution and it will be so in 24 numbers. Similarly, each digit will
n be in ten’s place 24 times. Hence, the sum of digits due to
= ten’s place of all the 120 numbers is
n r
r ( )
!
= 24 (9 + 7 + 5 + 3 + 1) ten = 6000

380 | CAT Complete Course

Visit : www.EasyEngineering.net
Visit : www.EasyEngineering.net

Proceeding exactly for hundreds, thousands and ten ∴ Total number of different numbers formed by 9 –
hundreds. We have the sum of the numbers digits
= 600( 1+10+100+1000+10000) = 9 ×8×7×6
= 600 × 11111 = 6666600 = 72 × 42 = 3024
Illustration 26. A number of 4- different digits is
Illustration 28. On a shelf there is space for 4 books
formed by using the digits 1,2,3, 4,5,6,7 in all possible
in how many ways can it be filled 12 different books?
ways.
Find : Solution : There is space for 4 books we have to fill
12 different books on that space.
(a) How many such numbers can be formed ?
(b) How many of them are greater than 3400 ? ∴ Number of ways the space of the shelf can be
Solution : 4 – digit number filled = 12P 4 = 11880.
thousand hundred ten one Illustration 29. Husband has 12 relatives (7m, 5f)
and his wife has 12 relatives (5m, 7f) in how many ways
σ can they invite 12 people (6m, 6f) for a dinner party so
that each invites 6 of his / her relations?
We have 4-places and 7- digits
Solution : Husband Wife

ww
So, number of numbers formed
= 7P4 =
7!
3!
(7m, 5f)
M - F
(6m, 6f) (5m, 7f)
M- F

(b)
w
thousand hundred
3
= 7 × 6 × 5 × 4 = 840

.Eaten one
6
5
4
3
0
1
2
3
0
1
2
3
6
5
4
3

When 3 Hundred’s syE


can be can be filled
2
1
4
5
4
5
2
1
filled 5 P 2

ngi ∴ Total number of ways = (7 C 6 · 5 C 0 ) H × (5 C 0 · 7 C 6 )W


is fixed place can be 5P
2
At filled from + (7C 5 · 5 C 1 )H × (5 C 1 × 7C 5 )W + (7 C 4 · 5 C 2 )H × (5 C 2 · 7 C 4 )W
thousand any of digits + (7 C 3 · 5 C 3 )H × (5 C 3 · 7 C 3 )W + (7C 2 · 5 C 4 )H × (5C 4 · 7 C 2 )W
place 4, 5, 6, 7
So, number of numbers formed when 3+5 fixed at nee
+ (7C 1 × 5C 5 )H × (5 C 5 · 7 C 1 )W
Illustration 30. In how many ways three girls and
thousands place = 4 × 5 P 2 = 80
Again
rin
nine boys can be seated in two vans, each having
numbered seats, 3 in the front and 4 at the back?

g.n
How many seating arrangements are possible if 3
girls should sit together in a back row on adjacent seats?

e
↓ Now, if all the seating arrangements are equally

4,5,6,7
Required number
= 4 × 6 P 3 = 4 × 6 × 5 × 4 = 24 × 20 = 480
Required number of numbers formed
likely, what is the probability of 3 girls sitting together in
a back row on adjacent seat ?
Solution : 3G, 9 B = 12 person
3,3 front, 4,4 back = 14 seats
t
= 480 + 80 = 560 (if no condition) number of arrangement = 14P 12
Illustration 27. How many different numbers of 4 Take out 3G are taken as a single number of seating
digits can be formed with nine digits 1,2,3,4,5,6,7,8,9 of 3G together on adjacent seats, there are 4 ways of
without repeating any of them in any number ? selecting three seats 1,2,3 or 2,3,4 Ist van and 1,2,3 or
Solution : A four digits numbers has four places 2,3,4 of 2 nd van in each set the three girls can be arranged
thousand hundred ten one in 3! ways each
Total number of ways = 4·3 ! = 24
After seating the three girls as desired we are left
↓ ↓ ↓ ↓ with 9 boys to be seated on remaining 11 seats which can
be done in 11P9 ways
Thousand’s Hundred’s Similarly ten’s One’s can
place can filled place be place can be be filled in ∴ Hence by fundamental the or m total number of
from one of filled from filled in 7 ways 6 ways seating arrangement is 11P9 ·24
the nine digits one of 8 1
Required probability = 11P 9 ·24 ÷ 14P 12 =
in 9 ways digits in 8 ways 91

CAT Complete Course | 381

Visit : www.EasyEngineering.net
Visit : www.EasyEngineering.net

Illustration 31. A tea party is arranged for 16 people Comparing, we get 10 – r = 5


along two sides of a long table with 8 chairs on each r = 5
sides. Four men wish to sit on one particular side and two
on the other side. In how many ways can they be seated ? Illustration 35. 15C 8 + 15C 9 – 15C 6 – 15C 7
Solution : Having seated 4 on side A and 2 on side Solution : 15C 8 + 15C 9 – 15C 6 – 15C 7
B we are left 10 person. nC nC
We know that r = n–r
Out of which we choose 4 for side A in 10C 4 ways
and now for side B we are left with 6 persons and 6 have ∴ 15C
8 + 15C
15–8 = 15C
7

to be seated so that they can be seated ∴ 15C


8 + 15C
9 – 15C
6 – 15C
7
In 6 C 6 = 1 way 15C 15C 15C 15C
= 7 + 6 – 6 – 7 = 0
Hence the number of selection for the two sides is
Illustration 36. If nC + 3 nC 3 n C n–r+2
n–r n–r+ 1
10C × 1 = 10 ! + nC p
4 n–r+3 = C r
6!4!
Again 8 person on each side can be arranged Find the value of P in terms of n.
amongst themselves in 8! ways Solution : We know that nC r + n C r–1 = n+1Cr

ww
Total number of seating arrangement is

=
10 !
6!4!
×8!×8!
∴ n C n–r + 3 nC n–r+1 + 3 nC n–r+2 + n C n–r+3 = p C r
We can write

w = 341397504000

.Ea
Illustration 32. Find r if 15C3r = 15Cr+3.
Solution : We know that nC x = n C y
+
nC
nC
n–r
n–r+3
+ n C n–(r–1) + 2nC n–(r–1) + 2nC n–(r–2) + n C n–(r–2)
= p Cr
Or, | n C n–r + n C n–(r–1)| + 2 |nC n–(r–1) + n C n–(r–2)|


x+y = n
15C
3r =
15C
r+3
syE + |n C n–(r–2) + n C n–(r–3)|
Or, n+1C
n–(r–1) +2 n+1C
n–(r–2) +
=
n+1C
pC
r

n–(r–3)


3r + r + 3 = 15
4r = 15 – 3 = 12 ngi Or, n+1C + n+1Cn–(r–2) + n+1Cn–(r–2) + n+1Cn–(r–3)
= p Cr

nee
n–(r–1)
12
⇒ R = =3 = pC
r
4
Or, n+2C + n+2C = pC
Illustration 33. If n C8 find n C 17 and 22Cn .

rin
n– (r–2) n–(r–3) r

Solution : We know that nC x = n C y Or, n+2C


n–(r–3) = p Cr


x+y = n
12 + 8n = 20
Or,
Or, n+3Cg.n
n+3C
n–(r–3)

= pC
= pC

∴ P=n+3
r

e
r r
20 × 19 × 18

t
Now, n C = 20C
17 17 = = 1140 Illustration 37 . If n C r–1 = 36, n C r = 84 and n C r+1
3·2
= 126
22 × 21
Now, 22C n = 22C8 = 22C 20 =
2 Find the value of n and r.
= 11 × 21 = 232 84 nC
r n–r+1
9P = =
Illustration 34. 5 + 5·9 P
4 =
10 P find r.
r 36 nC
r–1 r
∠9 ∠9 14 n–r+1
Solution : + 5· = 10 Pr ⇒ =
∠5∠4 ∠4∠5 6 r
∠9 ∠9 ∠10 7 n–r+1
+ = Or, = …(1)
∠4 ∠4 ∠10 – r 3 r

(∠55 = ∠41 ) and


nC
nC
r+
=
126
84
r
∠9 ∠10
2. = n–r 21 3
∠4 ∠10 – r Or, = = …(2)
r+1 14 2
2 10
=
∠4 ∠10 – r Solving eq. (1) & (2), we get
∠10 – r = ∠5 n=9 r=3

382 | CAT Complete Course

Visit : www.EasyEngineering.net
Visit : www.EasyEngineering.net

SOME IMPORTANT FORMULA Illustration 40. Find the number of ways in which 8-
n! nP different colored pencil can be arranged among
(1) nC
r = = r themselves red and black colours are never together.
(n – r) ! r ! r !
r ! nC r = n P r Solution : Considering red and black colors as a
single colors.
(2) n C n = nC
0 =1
nc nc So, we have n – 1 = 8 – 1 = 7 different colors
(3) r1 = r2
∴ Required number of permutations = n ! – n ! × 2 !
⇒ r1 + r2 = n
⇒ nc = nC =8!–7!×2!
r1 n–r 2
nc
= 7 ! (8 – 2)
(4) r + n cr–1 = n+1c
r
= 7·6·5·4·3·2·1 × 6
n–r+1
(5) nc
r ÷ ncr–1 = = 42 × 120 × 6 = 30240
r
n–1P + r·n–1P n Illustration 41. How many six digit numbers can be
(6) r r–1 = P r
formed with 1 to 8 without repetition of any digit in any
Illustration 38. Permutations with all n things taken number such that digits 2 and 5 always occur together?

ww
at a time in which 2 specified things always occur
together.
Solution : Let us consider the 2 specified things as a
Solution : Here, n = 8
Here, 2 and 5 always occur together we have to form

w
six digit number
single entity. The number of things in the given sample
∴ r = 6

.Ea
reduces to (n – 1).
Number of possible permutations with these (n – 1) So, Required number of seven digit numbers
things among themselves = (n – 1). = 2(r – 1) n–1P r–1

ing their places = 2.


syE
For each of these (n – 1) permutations the 2 specified
things can be rearrangement in two ways by interchang-
= 2 (6 – 1) 6 P 4

= 2×5×
6·5
= 150

ngi
2
∴ Total number of possible permutation of n things Illustration 42. How many 4-digit numbers can be
among themselves such that 2 specified things always formed with digits 1to 8 without repetition any digit in
occur together (n – 1) ! × 2.
Illustration 39. Permutations of n different things
taken r at a time in which 2 specified things occur
nee
any number such that digit 3 and 4 never occur together ?
Solution: n = 8 r=4

together.
Solution : We consider 2- specified things as a single
n P – 2 (r – 1)n–2 P
r rin
Required number of 4 digit number
8 6
r–2 = P 4 – 2 (4 – 1) P2

thing /entity
We have (n – 2) things g.n
= 8.7.6.5-2.3.6.5 = 56.30-6.30
=1680 – 180 = 1500
1 2 3 ------ ----- ---- r places
e
Illustration 43. How many four digit numbers can

t
be formed with digits 1, 3, 5, 7, 9 such that 3 is always at
hundred’s place ?
Solution :
But taking two things as a single thing/entity we have Thousand hundred ten one
1 2 3 r-2 places

Rest digit = 4
Now we have to fill 3 place by the digits 4 (1,5,7,9).
Considering the extremes we have r – 1 places we
have to fill (r – 1) places from (n – 2) digits So, required number of numbers formed
∴ Required number of permutation = 4 P 3 = 4! = 4.3.2 = 24
= n–2P r–2 × (r – 1) × 2 ! Illustration 44. How many seven digit number can
Because the single entity /thing can be be formed with digit 1, 2, 3, 4, 5, 6, 7, 8, 9. Such that they
start with 2 and end with 6?
Added at (r – 1) places
Solution :
and the 2 specified things can be arranged in 2! = 2
ways by inter changing their places. 6 2

CAT Complete Course | 383

Visit : www.EasyEngineering.net
Visit : www.EasyEngineering.net

Here, 2, and 6 are fixed so we have to fill 5- places the number of ways he can make his select his dresses for
with only 7-digits (1, 3, 4, 5, 7, 8, 9). the hour.
Required number of 7-digit number Solution : (i) Number of ways four suits can be
7! selected out of 8 suits
= 7 P 5 = = 7·6·5·4·3 = 2520
2! 8·7·6·5
= 8C4 = = 70
Illustration 45. How many non-zero numbers can be 4·3·2
formed with digits 0, 1, 2, 3, 4, 5 and 6 repetition of digits Number of ways 8 ties can be selected out 12 ties
not being allowed in any number ? 12 !
= 12C8 =
Solution : (i) Single digit number. Total ways of 8!4!
forming single digit numbers 12·11·10·9 990
= = = 445
= 6 4·3·2·1 2
(ii) Two digits numbers. Number of permutations of ∴ Total number of ways to select 4 suits and 8 ties
7 digits taken 2 at a time 7 P 2 = 70 × 495 = 34650
But this also includes 6 P 1 two digit numbers starting (ii) First we will find the number of selections in
with 0 and hence are not two digit numbers. which both green tie and blue suit appear together. In

ww
Total number of such two digit numbers starting with
0 is equal to the number of permutation of remaining six
digits to fill units place this is given by 6 P 1
such cases, 3 suits more are to be selected from remaining
7 suits and 7 ties are to be selected from remaining 11
ties.

w
Total number of two digit numbers

.Ea
= 7P2 – 6P1
Similarly, total number of three digit numbers
Number of ways = 7 C 3 × 11C

=
7!
×
3!4! 7!4!
11 !

= 7P3 – 6P2
syE
Total number of four digit numbers
=
11·10·9·8·7·6·5
3·2·4·3·2·1
= 11.10.7.3.5 = 11550

ngi
= 7 P 4 – 6 P 3 and so on
∴ Total number of non- zero numbers Exercise – A
= 6 P 1 + (7P 2 – 6 P 1 ) + (7P 3 – 6 P 2 ) + (7P 4 + 6P 3 )

nee
1. If 2n+1Cn–1 : 2n–1C n = 7 : 4, then the possible value of
+ (7P 5 – 6 P 4 ) + (7P 6 – 6 P 5 ) + (7P 7 – 6 P 6 ) n will be—
= (7P 2 + 7P 3 + 7P 4 + 7P 5 + 7 P 6 + 7P 7 ) (A) 3 (B) 5

– ( 6 P 2 + 6P 3 + 6P 4 + 6P 5 + 6P 6 ) (C) 4
rin (D) 2

g.n
= 11742 2. In how many ways can the letters of the word
"IGNOU" be arranged so that the vowels are never
Illustration 46. How many four digits numbers separated ?
greater than 6000 can be formed with the digits 0, 3, 4, 6,
7 and 9 repetition not being allowed ?
Solution : Four digit numbers greater than 6000 must
start with 6,7 or 9.
(A) 36
(C) 1·8 e (B) 72
(D) 18
3. How many different ways are possible to arrange the t
letters of the word "GRACE" so that the vowels may
Suppose, the number starts with 6, the other three occupy only the even positions ?
places in the number can be filled up by any of the 3-
(A) 10 (B) 12
digits from remaining 5 digits (0,3,4,7,9).
(C) 14 (D) 15
The number of permutation of 5 digits taken 3 of
them at a time is 4. The total number of ways in which letters of the
word “DISCIPLINE” can be arranged so that the
= 5 P 3 = 60
three I's never come together will be—
This hold for in each case of number starting with 7 (A) 4500 (B) 60,000
and 9 too.
(C) 564480 (D) 9500
Required number of numbers = 3 × 60 = 180
5. The total number of ways in which 5 books can be
Illustration 47. Shyam has 8 different color suits
distributed amongst 4 students (when repetitions are
and 12 ties of different colors he has to take 4 suits and 8
allowed) are—
ties on a tour. In how many ways he can select his dresses
for the tour ? If a grey tie is not be selected in case he (A) 52 (B) 45
selects a blue suit, find the number of ways he can find (C) 1 (D) 93

384 | CAT Complete Course

Visit : www.EasyEngineering.net
Visit : www.EasyEngineering.net

6. A man invites 3 men and 3 women to a party. In how 17. The letters T and E respectively occupy the first and
many ways can they sit at a round table so that no the last places ?
two men are together ? (A) 7 ! (B) 6 !
(A) 10 (B) 12 (C) 5 ! (D) 2 × 6 !
(C) 16 (D) 24
18. The letters T, H, O are never together ?
7. 4 men and 5 women have to be seated in a straight (A) 2560 (B) 4320
row so that no two women are together. The number
of ways in which this can be done is— (C) 5420 (D) None of these
(A) 1440 (B) 2880 19. The vowels always occupy even places—
(C) 7200 (D) 7505 (A) 3 ! × 3 ! (B) 4 ! × 4 !
8. In how many ways can 11 different alphabets (1, 2, (C) 3 ! × 4 ! (D) 5 ! × 4 !
3, …..) be arranged so that the alphabets f and g 20. How many words of 4 letters each can be formed,
never come together ?
each containing 2 consonants and 2 vowels ?
1
(A) |11 – |10 (B) |11 – |10 × (A) 432 (B) 5 ! × 4 !
2
(C) |11 – 2 × |10 (D) None of these (C) 4 ! × 4 ! (D) 241

ww
9. There are 9 points in a plane out of which 6 are
collinear. The number of straight lines then can be
drawn by joining these points will be—
21. How many different words beginning with Y and
ending with I can be formed with the letters of the
word “INDUSTRY”?

w
(A) 22
(C) 36
(B) 24

.Ea
(D) 38
10. In the above problem the number of triangles that
(A) 8 !

(C) 6 !
(B)
7!
2!
(D) 2 × 5 !
can be drawn will be—
(A) 35
(C) 46 syE
(B) 40
(D) 64
22. The number of different permutations of the letters of
the word ‘MISSISIPPI’—
10 ! 10 !

ngi
(A) (B)
5!4! 4!3!
11. How many numbers greater than 6000000 can be
formed with 1, 1, 6, 6, 9, 9, 0 ? (repetitions not 10 ! 10 !
(C) (D)

nee
allowed) 5!3! 4!3!2!
(A) 360 (B) 720 23. How many of them are greater than 3400 ?
(C) 180 (D) 240

rin
(A) 840 (B) 560
12. Out of 6 consonants and 3 vowels, all distinct, how (C) 480 (D) 120
many words can be formed each having 3 consonants
and 1 vowel ?
(A) 2400 (B) 4800 (A) 20
g.n
24. How many of them are exactly divisible by 25 ?
(B) 35
(C) 3600 (D) 7200
13. How many numbers greater than one million can be
formed with 3, 4, 5, 0, 2, 3 ? (repetitions not allowed)
(C) 40

e
(D) 50
25. How many of them are exactly divisible by 4 ?
(A) 150 (B) 160 t
(A) 575 (B) 300 (C) 120 (D) 200
(C) 625 (D) 675
26. From 7 men and 3 ladies, a committee of 4 is to be
14. How many numbers lying between 10 and 1000 can formed. The number of ways in which this can be
be formed with 2, 3, 4, 0, 8, 9 ? (repetitions not done such that at least one lady is included, is—
allowed)
(A) 70 (B) 72
(A) 360 (B) 120
(C) 75 (D) 92
(C) 125 (D) 142
27. The total number of seats at a particular management
15. Each consisting of 4 official and 5 non-official college is X, out of which Y seats are reserved for
members—
SC/ST candidates. If a total of 100 candidates
(A) 1260 (B) 1800 including 17 SC/ST candidates take the MBA
(C) 3360 (D) 1600 entrance test, then how many ways are there in which
16. Each containing at least two non-official members— the section can be done ? (Assume Y < 17)
(A) 368 (B) 456 (A) 17C Y × 11C(X–Y) (B) 17C Y × (100–Y)C(X–Y)
(C) 1029 (D) 1120 (C) C (X–Y) × CX
17 17 (D) (100–17) C X × 17C(X–Y)

CAT Complete Course | 385

Visit : www.EasyEngineering.net
Visit : www.EasyEngineering.net

28. There are 5 boys and 6 girls in a family. They are 7. 5 men and 6 women have to be seated in a straight
photographed in groups of 6 children such that there row so that no two women are together. The number
are not least girl. The number of different photo- of ways in which this can be done is—
graphs will be— (A) 48400 (B) 39600
(A) 220 (B) 260 (C) 9900 (D) 86400
(C) 350 (D) 386 8. In how many ways can 13 different alphabets (a, b, c,
29. An army code languages consists of 4 symbol codes ….m) be arranged so that the alphabets f and g never
out of which the first two have to be numbers (digits) come together ?
and the last two have to be alphabets. What is the (A) 13! – 12! (B) 13! – 12!/2!
total number of codes possible under such a scheme? (C) 13! – 2 × 12! (D) None of these
(A) 84656 (B) 24230
9. There are 10 points in a plane out of which 5 are
(C) 56346 (D) 67600 collinear. The number of straight lines than can be
30. There are 40 volunteers for earthquake relief who drawn by joining these points will be—
have to be sent to four small villages requiring 12, 3, (A) 35 (B) 36
11, and 9 men respectively. The number of ways in (C) 45 (D) 24
which this can be done is—

ww
(A) 40C 12 × 28C3 × 25C11 × 14C9
(B) 40C 11 × 28C3 × 25C11 × 14C9
(C) 40C 12 × 28C3 × 25C11 × 14C8
10. In the above problem the number of triangles that
can be drawn will be—
(A) 120 (B) 110

1. If
w
(D) 40C 12 × 28C3 × 25C10 × 14C9

2n+1 Pn–1 : 2n–1 .Ea


Exercise – B
Pn = 3 : 5, then the possible value of
(C) 100 (D) 78
11. How many numbers greater than 6000000 can be
formed with 1, 1, 6, 6, 9, 9, 0 ? (repetitions not
allowed)
n will be—
(A) 3 syE
(B) 5
(A) 360
(C) 180
(B) 720
(D) 240
12. Out of 5 consonants and 2 vowels, all distinct, how
(C) 4 (D) 2
2. In how many ways can the letters of the word
ngi many words can be formed each having 2 consonants
and 1 vowel ?

nee
"VALEDICTORY" be arranged so that the vowels (A) 120 (B) 240
are never separated ? (C) 360 (D) 150
(A) 883490 (B) 967680 13. How many numbers greater than one million can be
(C) 563680 (D) 483840
3. How many different ways are possible to arrange the rin
formed with 2, 3, 0, 3, 4, 2, 3 ? (repetitions not
allowed)
letters of the word "PINAK" so that the vowels may
occupy only the even positions ?
(A) 720
(C) 120
g.n
(B) 360
(D) 240

(A) 10
(C) 24
(B) 16
(D) 12
4. The total number of ways in which letters of the
allowed)
(A) 360
(C) 125
e
14. How many numbers lying between 10 and 1000 can

(B) 120
(D) 142
t
be formed with 2, 3, 4, 0, 8, 9 ? (repetitions not

word ACCOST can be arranged so that the two C's


never come together will be— 15. Each consisting of 3 official and 2 non-official
(A) 120 (B) 360 members—
(C) 240 (D) 6! – 2! (A) 360 (B) 180
(C) 336 (D) 160
5. The total number of ways in which 'X' things can be
16. Each containing at least two non-official members—
distributed amongst N persons (when repetitions are
(A) 368 (B) 456
allowed) are—
(C) 366 (D) 360
(A) Xn (B) Nx
(C) n P x × X ! (D) XP N × N ! For Q. 17 – 20 : How many different words can be
formed from the letters of the words GANESHPURI
6. A man invites 4 men and 4 women to a party. In how when :
many ways can they sit at a round table so that no 17. The letters P and I respectively occupy the first and
two men are together ? the last places ?
(A) 24 (B) 6 (A) 9 ! (B) 8 !
(C) 144 (D) 120 (C) 8 !/2 ! (D) 2 × 8 !

386 | CAT Complete Course

Visit : www.EasyEngineering.net
Visit : www.EasyEngineering.net

18. The letters E, H, P are never together ? 29. An army code languages consists of 4 symbol codes
(A) 8 ! (B) 8 ! × 42 out of which the first two have to be numbers (digits)
(C) 8 ! × 28 (D) 8 ! × 84 and the last two have to be alphabets. What is the
total number of codes possible under such a scheme?
19. The vowels always occupy even places ?
(A) 84656 (B) 24230
(A) 7 ! × 6 ! (B) 5 ! × 6 ! (C) 56346 (D) 67600
(C) 6 ! × 5 !/2! (D) 2 × 6 ! × 7!
30. There are 40 volunteers for earthquake relief who
20. How many words of 5 letters each can be formed, have to be sent to four small villages requiring 12, 3,
each containing 3 consonants and 2 vowels ? 11, and 9 men respectively. The number of ways in
(A) 5 ! × 5 ! (B) 5 ! × 6 ! which this can be done is—
(C) 7 ! × 9 ! (D) 2 × 5 ! (A) 40C 12 × 28C3 × 25C11 × 14C9
(B) 40C 11 × 28C3 × 25C11 × 14C9
21. How many different words beginning with O and
(C) 40C 12 × 28C3 × 25C11 × 14C8
ending with E can be formed with the letters of the
word ORDINATE ? (D) 40C 12 × 28C3 × 25C10 × 14C9
(A) 8 ! (B) 6 ! Solutions – A

ww
(C) 7 ! (D) 7 !/2 !
22. The number of different permutations of the letters of
the word 'BANANA' are—
Solution : 1. (A)
[2n + 1

w
(A) 120
(C) 180
(B) 60

.Ea
(D) 100
23. How many of them are greater than 3400 ?
Given


2n+1Cn–1 : 2n–1C n =

[2n + 1 [n [n
=
[n + 1 [n 7

7
[2n
[n [n
=
4

(A) 840
(C) 480 syE
(B) 560
(D) 120 ⇒
[n + 1 [n [2n
(2n + 1) [2n [n
(n + 1) [n [2n
=
4
7
4
24. How many of them are exactly divisible by 25 ?
(A) 20 (B) 35
ngi ⇒
Solution 2. (A)
2n + 1
n+1
=
7
4
⇒n=3

nee
(C) 40 (D) 50
Consider the vowels to be one entity (i, o, u), g, n
25. How many of them are exactly divisible by 4 ?
have to be permuted and the 3 vowels can be also
(A) 150 (B) 160

rin
permute in the set.
(C) 120 (D) 200 ⇒ Total number of arrangements possible
= 3 P 3 × 3P 3

g.n
26. From six men and 4 ladies, a committee of 5 is to be
formed. The number of ways in which this can be L3 L3
⇒ × = 6 × 6 = 36.
done such that at least one lady is included, is— L0 L0
(A) 382
(C) 482
(B) 246
(D) 336
27. The total number of seats at a particular management
Solution 3. (B)
e
'GRACE' has 3 consonants and 2 vowels.
The vowels can be placed in position no. 2 or 4
∴ Total ways possible
t
college is X, out of which Y seats are reserved for
SC/ST candidates. If a total of 100 candidates = 2! = 2.
including 17 SC/ST candidates take the MBA The consonants can occupy the position in 3! Ways.
entrance test, then how many ways are there in which So, the required number of ways
the section can be done ? (Assume Y < 17) = 2! × 3! = 12.
(A) 17C Y × 11C(X–Y) Solution 4. (C)
(B) 17C Y × (100–Y)C(X–Y) Total number of ways to permute 10 alphabets 3 of
which are common
(C) 17C (X–Y) × 17CX
L
(D) (100–17) C X × 17C(X–Y) = 10 = 10 × 9 × 8 × 7 × 6 × 5 × 4
L3
28. There are 5 boys and 3 girls in a family. They are = 720 × 840
photographed in groups of 2 boys and one girl. The = 604800.
number of different photographs will be— Assume three I’s as one
(A) 360 (B) 180 ⇒ Number of possible ways = 8 P 8
(C) 30 (D) 60 = L8 = 40320

CAT Complete Course | 387

Visit : www.EasyEngineering.net
Visit : www.EasyEngineering.net

So, required number of ways = Total arrangements – Solution 12. (D)


Number of arrangements in which they always come Total number of ways in which 3 consonants can be
together selected out of 6 = 6C 3
= 604800 – 40320 = 564480. Total number of ways in which 1 vowel can be
Solution 5. (B) chosen out of 3 = 3 C 2
Books 1 books 2 books 3 books 4 books 5 ⇒ Total numbers = 6 C 3 × 3 C2 × 5P 5 = 7200.
4 4 4 4 4
(The last term denotes the internal arrangements in
⇒ Total number of ways = 4 × 4 × 4 × 4 × 4 = 45 . each word)
Solution 6. (C)
Solution 13. (B)
Fix the position of 1 women M
Required number is greater than 1 million (6 digits).
⇒ Remaining women can sit in 2 P 2 ways
From given digits, total numbers which can be
⇒ Remaining men can sit in 3P 3 ways. formed = |6
⇒ Total 3 P 3 × 2 P2 = L3 × L2 Number starting with zero = |5
= 6 × 2 = 12 ⇒ Required number = |6 – |5 = 720 – 120
Solution 7. (B) = 600

ww
The arrangements will be : W M W M W M W M W
⇒ Total possible arrangements will be
= 4 P 4 × 5 P5
Since, repetition not allowed, so required answer

=
600
|2
= 300.

w
Solution 8. (C)

.Ea
Total possible arrangements = |11
= L4 × L5 = 2880. Solution 14. (C)
Between 10 and 100,
Possible numbers

So, syE
Total number in which 6 and 7 are together
= 2 × 10P10 = 2 × |10
required arrangement = |11 – 2 × |10
= 6 P 2 – 5 (those starting with a zero) = 25.
Between 100 and 1000, possible numbers
= 6 P 3 – 6 P 2 (starting with a zero) = 100
Solution 9. (A)
To draw a straight line, we need two points. ngi ⇒ Total = 25 + 100 = 125.
For Q. 15 & 16 : How many committees of 6 mem-
Hence, 9 C 2 lines are possible.
But 6 points are collinear, hence we subtract 6C 2
nee
bers each can be formed from 9 official and 5 non-official
members in the following cases :

rin
But these 6 points give 1 straight line. Solution 15. (A)
⇒ Number of straight lines possible 4 official and 2 non-officials

g.n
= 9C2 – 6 C2 + 1
4 official out of 9 officials
|9 |6
= – +1 |9 9 ×8×7×6
|2|7 |2|4 = 9C4 = = = 126

Solution 10. (B)


=9×4–3×5+1
= 37 – 15 = 22.
|4|5 4 ×3×2×1

e
2 non-official out of 5 can be selected in 5 C 2
= 10 ways
⇒ The number of ways in which the committee can
t
A triangle requires 3 non collinear points, 9 C 3 com-
be formed is
binations. But 6 points give us straight line .
Hence, number of triangles = 126 × 10 = 1260.
= 9C3 – 6 C3 Solution 16. (B)
9×8×7 6×5×4 Three non-official and 3 official
= –
6 6 = 5 C 3 × 9 C3 = 10 × 84 = 840
= 84 – 20 = 64. Four non-official and 2 officials
Solution 11. (A) 9×8
= 5 C 4 × 9 C2 = 5 ×
Case 1 : First place is occupied by 6 2
⇒ Total arrangements = 6P 6 / (2! 2!) = 36 × 5 = 180.
Because 9 and 1 each appear twice 5 non-official and 1 official
Case 2 : First place is occupied by 9, then total ways = 5 C 5 × 9 C 1 = 1 × 9 = 9.
are exactly similar to case 1. Total 840 + 180 + 9
⇒ Total numbers = 2 × (6 P 6 / 2! 2!) = 360. = 1029.

388 | CAT Complete Course

Visit : www.EasyEngineering.net
Visit : www.EasyEngineering.net

For Q.17 – 20 : How many different words can be in the first two places we will have to choose 2 more out
formed from the letters of the words CATHODE when : of remaining 5 and the number will be
Solution 17. (C) 5 P = 5 !/3 ! = 5 × 4 = 20.
2
T occupies first and E occupies the last place and so Therefore, total as above will be 20 × 4 = 80 …(b)
we have to arrange the remaining 5 letters which can be Hence, all the numbers greater than 3400 will be
done in 5 ! ways.
480 + 80 = 560.
Solution 18. (B)
Short-cut : Numbers less than 3400 will have 1 or 2
Take T, H, O as one letter and so the number of in 1st place or 31, 32 in the first two positions.
letters will be 5 ! 3 ! 6 P + 6 P = 120 + 120 = 240.
3 3
The number of words in which T, H, O are together 5 P + 5 P = 20 + 20 = 40.
2 2
will be 5 ! 3 !
Total numbers which are less than 3400
The total number of arrangements is 7 !
= 240 + 40
Hence the number of words when T, H, O are never
together is = 280
= 7! – 5! 3! = 5! [7 × 69 – 6] = 5!(42 - 6) Also total number of numbers formed is 7 P 4

ww
= 36 × 120 = 4320
Solution 19. (B)
We have 7 places, out of which 4 places are odd and
= 840.
Hence, numbers greater than 3400 is 840 – 280
= 560.

w
3 places are even i.e., 1st , 3rd , 5th , 7th are odd places

with 3P 3 = 3 ! ways.
.Ea
and 2nd, 4th , and 6th place are even places. We have

Total number of ways is = 3 ! × 4 !


Solution 24. (C)
A number will be divisible by 25 if the last two digits
are divisible by 25 and this can be done in two ways for
either 25 or 75 can be there and remaining two places out
Solution 20. (A)
syE
Each word is to contain 2 consonants out of 4 and 2
vowels out of 3 in 4 C 2 and 3 C 2 ways. Thus, by fundamental
of 5 digits can be filled in 5 P 2 ways. Hence, the required
number = 2 × 5P 2 = 2 × 20 = 40.
Solution 25. (D)
theorem, the total number of combinations (groups) of 5
ngi
letters will be 4 C 2 × 3 C 2 = 6 × 3 = 18 ways. Thus, we have
A number divisible by 4 if the last two digits are
divisible by 4 which can be done in 10 ways (12, 16, 24,

nee
18 groups each containing 4 letters i.e., 2 consonants and 32, 36, 52, 56, 64, 72, 76)
2 vowels. Now, the 4 letters in each group can be arranged Hence, required numbers
amongst themselves in 4! Ways, i.e., 24 ways. Hence, the = 10 × 5 P 2 = 10 × 20 = 200.
total number of different words will be 24 × 18 = 432.
Solution 21. (C)
Solution 26. (A)
rin
This can be done in 2 ways
Fix the position of Y and I. Then here are 6! Ways of
arranging the words in different letters. g.n
(1) There are 2 ladies and 2 men 3 C 2 × 7C 2
(2) There are 3 ladies and 1 men 3 C 3 × 7C 1
Solution 22. (D)
In the word MISSISIPPI, 3S15, 2P15, 4T5 and 1m.

Number of words taken all at a time is


10 !
3!2!4!1!
· Solution 27. (B)
e
Total number of ways = 3 × 21 + 1 × 7
= 63 + 7 = 70
t
Total numbers of possible ways
For Q. 23 – 25 : Refer to the following information = 17C Y × (100–Y)C (X–Y).
to answer the question that follow—
Solution 28. (D)
A number of 4 different digits is formed by using the
Different photographed in following ways :
digits 1, 2, 3, 4, 5, 6, 7 in all possible ways without
repetition. (1) 3 girls and 3 boys = 6 C 3 × 5C 3 = 200
Solution 23. (B) (2) 4 girls and 2 boys = 6 C 4 × 5 C 2 = 150
Numbers greater than 3400 will have, 4 or 5 or 6 or 7 (3) 5 girls and 1 boy = 6 C 5 × 5 C 1 = 6 × 5 = 30
in the first place. Having filled the first place say by 4, we (4) 6 girls = 6 C 6 = 6
have to choose 3 digits out of the remaining 6 and the Total number of ways = 200 + 150 + 30 + 6 = 386
number will be 6 P 3 = 6!/3! = 6 × 5 × 4 = 120. Solution 29. (D)
Therefore, total of such numbers will be Each code is of the form NN' AA' where first two are
4 × 120 = 480 … (a) digits and last two are alphabets.
Numbers greater than 3400 can be those which have ⇒ Total number of codes possible
34, 34, 36, 37 in the first two places. Having filled up 34 = (10)2 × (26)2 = 67600.

CAT Complete Course | 389

Visit : www.EasyEngineering.net
Visit : www.EasyEngineering.net

Solution 30. (A) Solution 7. (D)


For the first village, we can select volunteers in 40C 12 Total seats = 5 + 6 = 11
ways, for the second village, we have to select from the Arrangements will be : W M W M W M W M W
remaining volunteers 28 and so on. MW
Total number of ways = 40C 12 × 28C3 × 25C11 × 14C9 . ⇒ Total possible arrangements will be : 6P 6 × 5 P5
Solutions – B = 86400.
Solution 8. (C)
Solution 1. (C)
Total possible arrangements
Given 2n+1Pn–1 : 2n–1Pn = 3/5
= 13P 13 = 13 !
(2n + 1) ! (2n – 1) ! 3
⇒ ÷ = Total number in which f and g are together
(2n + 1 – n + 1) ! (2n – 1 – n) ! 5
(2n + 1) × 2n × (2n – 1) ! (n – 1) ! 3 = 2 × 12 P12 = 2 × 12 !
⇒ × =
(n + 2) (n + 1) n (n – 1 !) (2n – 1) ! 5 Hence Answer is 13P 13 – 2. 12P 12
2 (2n + 1) 3 = 13 ! – 2 × 12 !.
⇒ =
(n + 2) (n + 1) 5
Solution 9. (B)
⇒ 5 (4n + 2) = 3 (n2 + 3n + 2)
To draw a straight line, we need two points. Hence,

ww



20n + 10 = 3n2 + 9n + 6
3n2 – 11n – 4 = 0
10C lines are possible. But 5 points are collinear, hence
2
we subtract 5 C 2 . But these 5 points give 1 straight line.

w
(3n + 1) (n – 4) = 0 ⇒ Number of straight lines possible
⇒ n = 4. = 10C2 - 5 C 2 + 1 = 45 – 10 + 1 = 36.

Solution 2. (B) .Ea


Short-cut : Work from the option. Solution 10. (B)
A triangle requires 3 non-collinear points, 10C 3 com-

syE
Consider the vowels to be one entity (a, e, i, o), v, l,
d, c, t, r, y have to be permuted and the 4 vowels can be
also permute in the set.
binations. But 5 points give us straight line .
Hence, number of triangles = 10C 3 – 5 C 3
= 120 – 10 = 110.
⇒ Total number of arrangements possible
= 8 P 8 × 4 P4 = 967680.
ngi Solution 11. (A)
Case 1 : First place is occupied by 6
Solution 3. (D)
'PINAK' has 3 consonants and 2 vowels.
The vowels can be placed in position no. 2 or 4 nee ⇒ Total arrangements = 6C 6 / (2! 2!)
Because 9 and 1 each appear twice
∴ Total ways possible = 2! = 2.
The consonants can occupy the position in 3! ways. rin
Case 2 : First place is occupied by 9, then total ways
are exactly similar to case 1.

So, the required number of ways


= 2! × 3! = 2 × 6 = 12. Solution 12. (A) g.n
⇒ Total numbers = 2 × (6 C 6 / 2! 2!) = 360.

Solution 4. (C)
Total number of ways to permute 6 alphabets 2 of
which are common = 6! / 2! = 360.
Treat the two C's as one
selected out of 5 = 5 C2
e
Total number of ways in which 2 consonants can be

Total number of ways in which 1 vowel can be


chosen out of 2 = 2 C1
t
⇒ Number of possible ways = 5 P 5 = 120 ⇒ Total numbers = 5 C 2 × 2 C 1 × 3 P 3 = 120.
So, reqd. number of ways = Total arrangements – (The last term denotes the internal arrangements in
Number of arrangements in which they always come each word)
together Solution 13. (B)
= 360 – 120 = 240. Required number is greater than 1 million (7 digits).
Solution 5. (B) From given digits, total numbers which can be
The first can be given in N ways; the second thing formed
can also be given in N ways; the third thing can also be = 7!
given in N ways etc.
Number starting with zero = 6 !
⇒ Total number of ways = N.N.N…. times = Nx.
Solution 6. (C) ⇒ Required number = 7 ! – 6 !
Fixed the position of one woman Q Repetition not allowed, so required answer
⇒ Remaining women can sit in 3 P 3 ways 7!–6!
= = 360.
⇒ Total 3 P 3 × 4 P 4 = 144 ways. 2!3!

390 | CAT Complete Course

Visit : www.EasyEngineering.net
Visit : www.EasyEngineering.net

Solution 14. (C) Solution 20. (A)


Between 10 and 100, Each word is to contain 3 consonants out of 6 and 2
Possible numbers vowels out of 4. We can select them in 6 C 3 and 4 C 2 ways.
= 6 P 2 - 5 (those starting with a zero) = 25. Thus, by fundamental theorem, the total number of
Between 100 and 1000, possible numbers combinations (groups) of 5 letters will be 6C 3 × 4 C 2 . But
6 C × 4 C = 120 ways. Thus, we have 120 groups each
= 6 P 3 - 6P 2 (starting with a zero) = 100 3 2
containing 5 letters i.e., 3 consonants and 2 vowels. Now,
⇒ Total = 25 + 100 = 125.
the 5 letters in each group can be arranged amongst
For Q. 15 & 16 : How many committees of 5 themselves in 5! Ways, i.e., 120 ways. Hence, the total
members each can be formed from 8 official and 4 non-
number of different words will be 120 × 120 = 14400.
official members in the following cases :
Solution 15. (C) Solution 21. (B)
3 official and 2 non-officials 6 ! ways, O fixed in 1st and E fixed in last.
3 official out of 8 can be selected 8C 3 = 56. Solution 22. (B)
2 non-official out of 4 can be selected in 4 C 2 BANANA.3 As, 2Ns, 1B, i.e., 6 letters, 3 alike of
= 6 ways one type and 2 of another type.
⇒ The number of ways in which the committee can 6!

ww
be formed is 56 × 6 = 336.
Solution 16. (B) =
Number of words taken all at a time is
6×5×4
2
= 60.
3!2!

w
At least two non-official members

.Ea
⇒ Two non-official and 3 officials
= 4 C 2 × 8C 3 = 6 × 56 = 336.
For Q. 23–25 : Refer to the following information
to answer the question that follow—
A number of 4 different digits is formed by using the

syE
Three non-official and 2 officials digits 1, 2, 3, 4, 5, 6, 7 in all possible ways without
= 4 C 3 × 8C 2 = 4 × 28 = 112. repetition.
Solution 23. (B)
Four non-official and 1 official

Total 336 + 112 + 8 = 456


= 4 C 4 × 8C 1 = 1 × 8 = 8.
ngi Numbers greater than 3400 will have, 4 or 5 or 6 or 7
in the first place. Having filled the first place say by 4, we
have to choose 3 digits out of the remaining 6 and the
Short-cut : At least two non-officials
= Total – One non-official – No non-official. nee
number will be 6 P 3 = 6!/3! = 6 × 5 × 4 = 120.
Therefore, total of such numbers will be 4 × 120
= C 5 - ( C 4 × C1 ) - C 5 = 792 – 280 – 56 = 456.
12 8

Solution 17. (B)


4 8 = 480

rin
Numbers greater than 3400 can be those which have
…. (1)

g.n
34, 34, 36, 37 in the first two places. Having filled up 34
P occupies 1st and I occupies the last place and so we in the first two places we will have to choose 2 more out
have to arrange the remaining 8 letters which can be done of remaining 5 and the number will be 5 P = 5 !/3 !
2

e
in 8! ways.
= 5 × 4 = 20.
Solution 18. (D)
Take E, H, P as one letter and so the number of
letters will be 10 – 3 + 1 = 8.
The number of words in which E, H, P are together + 80 = 560.
Therefore, total as above will be 20 × 4
= 80
Hence, all the numbers greater than 3400 will be 480
t
…(2)

will be 8!, 3! Short-cut : Numbers less than 3400 will have 1 or 2


The total number of arrangements is 10! in 1st place or 31, 32 in the first two positions. 6 P 3 + 6 P 3
Hence, the number of words when E, H, P are never = 120 + 120 = 240.
together is 10! – 8! 3! = 8! [10 × 9 – 6] = 84! × 8!. 5 P + 5 P = 20 + 20 = 40. Total numbers which are
2 2
Solution 19. (B) less than 3400 = 240 + 40 = 280 Also, total number of
7
We have ten places out of which 5 places are odd numbers formed is P 4 = 840.
i.e., 1st , 3rd , 5th , 7th , 9th, and five are even i.e., 2nd, Hence, numbers greater than 3400 is 840 – 280
fourth, sixth, eight, and tenth. In the five even places we = 560.
have to fix up 4 vowels which can be done in 5 P 4 ways. Solution 24. (C)
Having fixed up the vowels in even places, we will be left A number will be divisible by 25 if the last two digits
with six places namely 5 odd and one even after fixing are divisible by 25 and this can be done in two ways for
the four vowels. In these six places we have to fix six either 25 or 75 can be there and remaining two places out
consonants which can be done in 6 P 6 i.e., 6! ways. Thus, of 5 digits can be filled in 5 P 2 ways. Hence, the required
the total number of ways is 5 P 4 × 6 P6 or 5! × 6! number = 2 × 5P 2 = 2 × 20 = 40.

CAT Complete Course | 391

Visit : www.EasyEngineering.net
Visit : www.EasyEngineering.net

Solution 25. (D) Solution 28. (B)


A number divisible by 4 if the last two digits are The number of ways of forming the group
divisible by 4 which can be done in 10 ways (12, 16, 24, = 5 C 2 × 3 C1 = 30.
32, 36, 52, 56, 64, 72, 76) Members of each group can be arranged among
Hence, required numbers = 10 × 5P2 = 10 × 20 = 200. themselves in 3! ways
Solution 26. (B) = 6 ways
Therefore, the number of photographs 30 × 6
There are 6 men and 4 ladies, in forming the groups,
they may be classified as follows : = 180
1. 1 lady, 4 men ⇒ 4 C 1 × 6 C 4 = 60 Solution 29. (D)
Each code is of the form NN' AA' where first two are
2. l lady, 3 men ⇒ 4C

6C
3 = 120
digits and last two are alphabets.
3. 3 ladies, 2 men ⇒ 4C

6C
2 = 60 ⇒ Total number of codes possible = (10)2 × (26)2
4. 4 ladies, 1 man ⇒ 4C

6C
1 =6 = 67600.
Total = 60 + 120 + 60 + 6 = 246. Solution 30. (A)
For the first village, we can select volunteers in 40C 12

ww
Solution 27. (B)
Total numbers of possible ways
= 17CY × (100–Y)C (X–Y) .
ways, for the second village, we have to select from the
remaining volunteers 28 and so on.

w .Ea
Total number of ways = 40C 12 × 28C3 × 25C11 × 14C9 .
●●●

syE
ngi
nee
rin
g.n
e t

392 | CAT Complete Course

Visit : www.EasyEngineering.net
Visit : www.EasyEngineering.net

7 Probability
Introduction : Human life is full of uncertainties in Event : A subset of the sample space S is called an
our day to day life very often we make guess and use event.
statements like : “possibility of a particular party to win Example : When two coins are tossed
this election is more………….” Or “Most probably it will
Sample Space S = {HH, HT, TH, TT }
rain today.”
Here HT denotes the occurrence of head on first coin
Whenever we use such statement we have intuition
and tail on second coin.
which enables us to claim that one event is more likely to
Simple Event or Elementary Event : An event is

ww
happen than the other.
called a sample event if it is a singleton subset of the
In probability theory, the degree of certainty or
sample space S.
uncertainty of such events is measured in terms of

w
numbers lying between 0 and 1. Example : When a coin is tossed,
Sample Space S = { H, T }

.Ea
SOME IMPORTANT TERMS
1.Experiment : An operation which result in some
Let A = {H} = the event of occurrence of head
B = { T } = the event of occurrence of tail

syE
well defined out come is called an experiment.
2.Random Experiment : An experiment whose out
come can not be predicted with certainty is called a
Here A and B are simple events.
Mixed Event or Compound Event or Composite
random experiment.
In other words if an experiment is performed many ngi Event : A subset of the sample space S which contains
more than one element is called a mixed event.

times under similar condition and the out come each time
is not the same, then this experiment is called a random
experiment.
nee Example : When a die is thrown, Sample Space
S = { 1, 2, 3, 4, 5, 6 }

rin
Let A = { 1, 3, 5 } = the event of occurrence of
Example : Tossing of a fair coin, is a random an odd number and
experiment because if we toss a coin either a head or a

g.n
B = { 5, 6 } = the event of occurrence of a
tail will come up. But if we toss a coin again and again number greater than 4
the out come each time will not be the same.
Trial : When an experiment is repeated under
Sample Space : The set of all possible out comes of
a random experiment is called the sample space for that
experiment. It is usually denoted by S.
Example : (i) When a coin is tossed either a head or
e
conditions and it does not give the same result each time
but may result in any one of the several possible. Out
t
comes, the experiment is called a trial and the out comes
are called cases.
a tail will come up. If H denotes the occurrence of head The number of times the experiment is repeated is
and T denotes the occurrence of tail. called the number of trials.
Sample Space, S = { H, T } Example : One toss of a coin is a trial when the coin
(ii) When a die is thrown any one of the numbers 1, is tossed 5 times.
2, 3, 4, 5 and 6 will come up. Occurrence of an Event : For a random experiment,
∴ Sample Space, S = { 1, 2, 3, 4, 5, 6 } Let E be an event.
Sample Point or Event Point : Each element of the Let E = { a, b, c } if the out come of the
sample space is called a sample point or an event point. experiment is a or b or c then we say that event E has
occurred.
Example : When a die is thrown sample space.
Equally Likely Cases (Events) : Cases (out comes)
S = { 1, 2, 3, 4, 5, 6 }
are said to be equally likely when we have no reason to
Here 1, 2, 3, 4, 5, 6 are the sample points. believe that one is more likely to occur than the other.
Discrete Sample Space : A sample space S is called Thus when an unbiased die is thrown all the six faces 1,
a discrete sample space if S is a finite set. 2, 3, 4, 5 and 6 are equally likely to come up.

CAT Complete Course | 393

Visit : www.EasyEngineering.net
Visit : www.EasyEngineering.net

Exhaustive Cases (Events) : For a random (i) Odds in favour of event E


experiment A, Set cases (event) is said to be Exhaustive if n (E) Number of cases favourable to event E
= =
one of them must necessarily happen every time the n (E1) Number of cases against E
experiment is performed. (ii) Odds against an event E
Example : When a die is thrown cases or events 1, 2, n (E1) Number of cases against the event E
3, 4, 5, 6 form on exhaustive set of events. = =
n (E) Number of cases favourable to event E
Probability of Occurrence of an Event Some Important information about playing cards :
Let S be the sample space, then the probability of (a) A pack of 52 playing cards has 4 suits
occurrence of an event E is denoted by P (E) and it is (i) Spade (ii) Hearts
defined as (iii) Diamonds (iv) Clubs
n (E) number of elements in E
P (E) = = (b) Spades and clubs are black faced cards.
n (S) number of elements in S
(c) Heart and diamonds are red faced cards.
number of cases favourable to event E
P (E) = (d) Each suit consists of 13 cards.
Total Number of Cases
Illustration 1. When a die is tossed, sample space, S (e) The aces, kings, queens, jacks are called face

ww
= { 1, 2, 3, 4, 5, 6 }?
Solution : Let A = The event of occurrence of an
odd Number = { 1, 3, 5 }
cards or honors cards.
Illustration 3. One ticket is drawn from a bag
containing 30 tickets numbered 1 to 30. Represent the

w
B = The event of occurrence of a number greater
than 4 = { 5, 6 }

P (A) = .Ea
n (A) 3 1
= =
sample space and the event of drawing a ticket containing
number which is a multiple of 5.
Solution : Since, the bag contains 30 tickets num-
bered 1 to 30.

P (B) =
n (S) 6 2
n (B) 2 1
= =
n (S) 6 3syE Here Random experiment is :- Drawing of one ticket
from the bag.
Illustration 2. Probability of occurrence of an event
is a number lying between 0 and 1?
ngi Let S be the sample space and E be event of drawing
a ticket containing a number which is a multiple of 5.
Let 1, 2, 3, ----------- 30 denotes the out comes of drawing
Solution : Let S be the sample space and E be an
event.
Then, φ ⊆Ε⊆S nee
tickets containing numbers 1, 2, 3, --------- 30
respectively.

Or,
n (φ) ⊆ n (E) ⊆ n (S)
0 ⊆ n (E) ⊆ n (S) rin
Then, S = {1, 2, 3, 4, -------------- 30 }
and E = {5, 10, 15, 20, 25, 30 }

Or,
0
n (S)

n (E)
n (S)
⊆1
g.n
Illustration 4. A coin is tossed successively three
times. Find the probability of getting exactly one head or

e
two heads.
∴ 0 ≤ P (E) ≤ 1
Now, it is clear that
(i) φ is the impossible event.
(ii) P (S) =
n (S)
=1
then
Solution : Let S be the sample space and E be the
event of getting exactly one head or exactly two heads,

S = {HHH, HHT, HTH, THH, TTH,


t
n (S)
THT, HTT, TTT }
(iii) P (E) = 0
and E = {HHT, HTH, THH, HTT, THT,
⇒ E = φ
(iv) P (E) = 1 TTH }
⇒ E = S n (E) = 6 and n (S) = 8
Complement of an event : Let S be the sample n (E)
Now, required probability, P (E) =
space for a random experiment. n (S)
Let E be an event complement of event E is 6 3
= =
– 8 4
denoted by E1 or Ec or ∈
Illustration 5. If a leap year is selected at random,
E1 means non occurrence of event E
what is the chance that it will contain 53 Tuesdays?
Odds in favour and odds against on event :
Solution : A leap year has 366 days i.e., 52 complete
Let S be the sample space and E be an event. weeks and two days more. These two days will be two
Let E1 denotes the complement of event E, then consecutive days of a week. A leap year will have 53

394 | CAT Complete Course

Visit : www.EasyEngineering.net
Visit : www.EasyEngineering.net

Tuesdays if out of the two consecutive days of a week Solution : n (S) = Sample Space = Total number of
selected at random one is a Tuesday. selections of 8 persons out of 13 persons
Now, S = { (Monday, Tuesday) (Tuesday, Wednes- = 13C 8
day) (Wednesday, Thursday) (Thursday, Friday) (Friday, n (E) = Number of selection of 6 men and 2
Saturday) (Saturday, Sunday) (Sunday, Monday)} ladies out of 8 men and 5 ladies
n (S) = 7 = 8 C 6 × 5 C2
and E = {(Monday, Tuesday) (Tuesday, So Required probability
Wednesday) } n (∈)
= P (E) =
n (E) = 2 n (S)
8C × 5 C
∴ Required probability, P (E) 6 2
P (E) = 13C
n (E) 2 8
= =
n (S) 7 8!×5! 8 ! ×5 !
= ×
Illustration 6. What are the odds in favour of 6!×2!×2!×3! 13 !
throwing at least 8 in a single throw with two dice? 8 ×7×5×4 5·3·4·2·1
= ×
2×2 13 × 12 × 11 × 10 × 9

ww
Solution : Here random experiment is : Throwing of
two dice 280
=
S = Sample Space 1287

w = {1, 2, 3, 4, 5, 6 } × {1, 2, 3, 4, 5, 6}
n (S) = 6 × 6

.Ea
Throwing at least 8 with two dice means throwing 8
or 9 or 10 or 11 or 12.
Illustration 9. If from a pack of 52 playing cards
one card is drawn at random, what is the probability that
it is either a king or a queen?
Solution : Random experiment : Drawing of one

syE
∴ E = {(6, 2) (5, 3) (4, 4) (3, 5) (2, 6) (6, 3) (5, 4)
(4, 5) (3, 6) (6, 4) (5, 5) (4, 6) (6, 5) (5, 6)
card from a pack of 52 playing cards.
Let n (S)= Number of ways of selecting one
card out of 52 playing cards

n (E) = 15
(6, 6)}
ngi = 52 C1 = 52
n (E) = Number of selection of a card which
n (E1 ) = 36 – 15 = 21
Now, odds in favour of E nee
is either a king or a queen
= 4 C1 + 4C1 = 4 + 4 = 8

=
n (E) 15 5
=
n (E1) 21 7
=
= rin
∴ Required probability, P (E)
n (E) 8
= =
2

g.n
n (S) 52 13
Illustration 7. If a number of two digits is formed
with the digits 2, 3, 5, 7, 9 without repetition of digits. Probability in terms of Symbols
What is the probability that the number formed is 35?
Solution : Here random experiment is : Formation of
two digit number with the digits 2, 3, 5, 7, 9 without
repetition.
e
Let A and B any two events.

t
(a) A ∩ B or AB denotes the event of simultaneous
occurrence of both the events A and B.
Thus, A ∩ B occurs if and only if both A and B
Let S = Sample Space occurs.
E = The event that the number formed is 35 (b) A ∪ B or A + B denotes the events of occurrence
of at least one of the events A and B.
Now,
Thus, A ∪ B occurs
n (E) = 1
⇒ at least one of A and B occurs.
n (S) = Total number of numbers of two digits (c) A – B denotes the occurrence of events A but not
formed with the digits 2, 3, 5, 7, 9 without repetition. B.
= 5 P 2 = 5 × 4 = 20 Thus, A – B occurs
∴ Required probability, ⇒ A occurs and B does not occur.
n (E) 1 Clearly, A – B = A ∩ B1
P (E) = = ·
n (S) 20 B – A = B ∩ A1
Illustration 8. From a group of 8 men and 5 ladies a (A) Mutually Exclusive or Disjoint events :
committee of 8 persons is formed. What is the probability Two or more events are said to be mutually exclusive
that the committee will consist of exactly 2 ladies? if one of them occurs, others cannot occur.

CAT Complete Course | 395

Visit : www.EasyEngineering.net
Visit : www.EasyEngineering.net

Thus, two or more events are said to be mutually Solution : From Set-theory we know that–
exclusive if no two of them can occur together. n (A ∪ B) = n (A) + n (B) – n (A ∩ B)
Thus, events A1 , A2 , --------------- An are mutually
n (A ∪ B)
exclusive if and only if Or,
n (S)
Ai ∩ AJ = φ for i ≠ j
n (A) n (B) n (A ∩ B)
Example : When a die is thrown sample S = + –
n (S) n (S) n (S)
= {1, 2, 3, 4, 5, 6 }
P (A ∪ B) = P (A) + P (B) – P (A ∩ B)
Let A = the event of occurrence of a number greater
than 4 = { 5, 6 } Case-I
B = the event of occurrence of an odd number If A and B are mutually exclusive events.
= {1, 3, 5 } Then, A∩B = φ
C = the event of occurrence of an even number Hence, P (A ∩ B) = 0
= {2, 4, 6 } ∴ P (A ∪ B) = P (A) + P (B)
Then B and C are mutually exclusive events but A
Case-II
and B are not mutually exclusive because they can occur

ww
together.
(B) Independent or Mutually Independent Events :
Two events are mutually exclusive if and only if
P (A ∪ B) = P (A) + P (B)

w
Two or more events are said to be independent if Case-III
occurrence or non-occurrence of any of them does not 1 = P (S) = P (A ∪ A1 ) = P (A) + P(A 1 )
other events.
.Ea
affect the probability of occurrence or non-occurrence of
P (A 1 ) = 1 – P (A)
Illustration 11. If A, B and C are any three events in

syE
Example : Let a bag contain 3 red and 2 black balls.
Two balls are drawn one by one with replacement.
Let A = The event of occurrence of a red ball in
a sample space S, then prove that
P(A ∪ B ∪ C) = P(A) + P(B) + P(C) – P(A ∩
first draw
B = The event of occurrence of a black ball in
the second draw ngi B) – P(B ∩ C) – P(A ∩ C) + P(A ∩ B ∩ C).
Solution : From Set-theory we know that

3
P (A) = when a red ball is drawn in the first raw.
5 nee n (A ∪ B) = n (A) + n (B) – n (A ∩ B)
Now, n ( A ∪ B ∪ C )

P (B) =
2
5
rin
= n [ A ∪ (B ∪ C) ]
= n (A) + n (B ∪ C) – n [ A ∩ (B ∪ C)]

g.n
Here probability of occurrence of event B is not
= n (A) + n (B ∪ C) – n [ (A ∩ B) ∪ (A ∩ C)]
affected by occurrence or non-occurrence of event A.
Here A and B are independent events. = n (A) + n (B ∪ C) – n (X ∪ Y)
But if two balls are drawn one by one without
replacement, then probability of occurrence of a black
ball in the second draw when a red ball has been drawn in
2
the first draw = probability of occurrence of a black ball
e
{where X = A ∩ B, Y = A ∩ C}
= n (A) + n (B ∪ C) – [ n (X) + n (Y)
– n (X ∩ Y)] t
4 = n (A) + n (B) + n (C) – n (B ∩ C)
in the second draw when a red ball is not drawn in the – n (X) – n (Y) + n (X ∩ Y)
1
first draw = = n (A) + n (B) + n (C) – n (B ∩ C)
4
Here the events of drawing a red ball in the first – n (A ∩ B) – n (A ∩ C) + n (A ∩ B ∩ C)
draw and the event of drawing a black ball in the second n (A ∪ B ∪ C)
draw are not independent. ∴
n (S)
So Clearly, n (A) n (B) n (C) n (B ∩ C)
(i) A and B1 are independent events. = + + –
n (S) n (S) n (S) n (S)
(ii) A1 and B are independent events. n (A ∩ C) n (A ∩ B) n (A ∩ B ∩ C)
Illustration 10. To prove P (A ∪ B) = P (A) + P (B) – – +
n (S) n (S) n (S)
– P (A ∩ B)
∴ P (A ∪ B ∪ C)
where A and B be any two events in a sample space
S, then the probability of occurrence of at least one of the = P(A) + P(B) + P(C) – P (A ∩ B)
events A and B is given by P (A ∪ B). – P (B ∩ C) – P (A ∩ C) + P (A ∩ B ∩ C)

396 | CAT Complete Course

Visit : www.EasyEngineering.net
Visit : www.EasyEngineering.net

Illustration 12. Two cards are drawn at random 1 3


P (E1) = 6 X = 6 × =
from a pack of cards. Find the probability that both the 10 5
cards are of red colour or they are queen. 3
P(E2 ) = 3 X =
Solution : Here random experiment is – 10
Drawing of two cards from a pack of 52 cards. Illustration 14. The odds in favour of standing first
of three students appearing at an examination are 1 : 2,
Let S be the sample space
2 : 7 and 1 : 9 respectively. What is the probability that
A = The event that the two cards drawn are red either of them will stand first?
B = The event that the two cards drawn are Queen. Solution : Let the three students be P, Q and R.
Number of cards drawn are both red or both queen Let A, B and C denotes the events of standing first of
= A∪B the three students P, Q and R respectively.
A ∩ B = the event that the two cards drawn are Given, Odds in favour of A = 1 : 2
queen of red colour.
Odds in favour of B = 2 : 7
Total number of cards = 52
and Odds in favour of C = 1 : 9
Number of red cards = 26 1

ww

Number of queen = 4
n (S) = 52C 2
So, P (A) =

P (B) =
3
2
9

w
n (A) = 26C
2

n (B) = 4C 1

∴ Required probability
.Ea 2

P (A ∪ B) = P (A) + P (B) – P (A ∩ B)
P(C) =
10
Since, events A, B, C are mutually exclusive.

26C 4C 2C
syE
= 52 2 + 52 2 – 52 2
C2 C2 C2
∴ P (A ∪ B ∪ C) = P (A) + P (B) + P (C)

=
1 2 1
+ +
3 9 10

=
26 × 25
+
4×3

2×1
52 × 51 52 × 51 52 × 51 ngi =
30 + 10 + 9 49
90
=
90

=
660
2652
nee Fundamental Principal of Counting
1. (a) Addition Rule : If a work is done when
Illustration 13. For a post three selected is twice
that of B and the probability of B being selected is thrice
that of C, what are the individual probabilities of A, B rin
exactly one of a number of works A1, A2 , A3 --------- An
is done, then number of ways of doing the work.

and C being selected?


Solution : Let E1 , E2 , E3 be the events of selection of
works A1 , A2, ----------- An
g.n
A = Sum of the number of ways of doing all the

(b) Multiplication Rule : If a work A is done when


A, B and C respectively.
Let P (E2) = X
Then, According to question –
e
all of a number of works A1 , A2, A3, --------- An are done,
then number of ways of doing the work.
A = Product of the number of ways of doing all the
works A1 , A2, A3 ---------- An
t
P (E2) = 3 P (E3 ) = 3 X
2. (a) Number of permutations of n different things
P (E1) = 2 P (E2 ) = 6 X taken r at a time is given by
Here there are three candidates A, B and C one must nP = n!
r
be selected and exactly one will be selected. (n – r) !
(b) Number of ways of arranging of n different
∴ P (E1 ∪ E2 ∪ E3 ) = 1
things = n !
and E1 , E2 , E3 are mutually exclusive. (c) Number of ways of arranging n things out of
Now, 1 = P (E1 ∪ E2 ∪ E3 ) which P are like and are of one type, Q are like and are of
= P (E1) + P (E2) + P (E3) second type and rest are all different.
n!
1 = 6 X + 3 X + X = 10 X =
P!Q!
1 (d) Number of ways of arranging n different things
X =
10 along a circle when clockwise and anticlockwise
1 arrangements are different i.e. when observations can be
∴ P(E3 ) = X =
10 made from one side only. = (n – 1) !

CAT Complete Course | 397

Visit : www.EasyEngineering.net
Visit : www.EasyEngineering.net

(e) Number of ways of arranging n different things n (E) = Number of selection of 4 cards out of which
along a circle when clockwise and anticlockwise arrange- two is a kings, one is a queen and one is a jack
ments are not different i.e., when observation can be made = 4 C2 × 4 C1 × 4 C1 = 6 × 4 × 4
(n – 1) !
from both sides = Now, P (E)
2
n (E) 6 × 4 × 4 6 × 4 × 4 × 4 × 3 × 2
3. Number of ways of selecting r different things out = = 52 =
n (S) C4 52 × 51 × 50 × 49
of n different things
2 1 8 2
= n Cr = × × ×
13 17 25 49
Illustration 15. If 12 persons are seated at a round
table, what is the probability that three particular persons BINOMIAL PROBABILITY
sit together? If an experiment is repeated n times under similar
Solution : Let S be the sample space conditions, we say that n trials of the experiment have
been made.
n (S) = (12 – 1) ! = (11) !
To solve such conditional problem we use Binomial
= Total number of ways of seating 12 persons along Theorem.
around table. Binomial Theorem on Probability :

wwNow, considering the three persons as one person.


Now, We have only 9 + 1 = 10 persons
Statement : Let E be the event and P be the
probability of occurrence of event E in one trial.
q = 1 – P = Probability of non occurrence of

w
These 10 persons can be seated at a round table in
(9)! ways

.Ea
But three particular persons can be arranged among
themselves in 3! ways
event E in one trial.
Again Let A be the number of success or number of
times event E occurs in n trials.

syE
Then, the probability of occurrence of event E exactly
∴ n (E) = (9) ! . (3) ! r times in n trials is given by
n (E) 3 !·9 ! 3×2 3 P (r) = n Cr Pr q(n – r)
P (E) = = = =
n (S) 11 ! 11 × 10 35
Illustration 16. If from a pack of 52 playing cards,
one card is drawn at random, what is the probability that ngi trial
= (r + 1)th term in the expansion (q + p)n
Proof : Probability of occurrence of event E in one

it is either two kings or two queens?


Solution : S = Sample space nee = P
Probability of non-occurrence of event E in one trial
n (S) = Number of ways of selecting two
cards out of 52 playing cards rin
= q = 1–p
Event E occurs exactly r times in n trials means that

g.n
the event E occurs r times and it does not occur (n – r)
= 52C 2 times in n trials.
n (E) = Number of selecting of a card Now, r trials in which event E occurs can be
which is either a king or a queen

= 4 C2 + 4 C2 =
4×3 4×3
2
+
2
e
selected out of n trials in nC r ways.
Also, the n trials are independent.
Therefore, probability of occurrence of event E
exactly r times out of n trials is given by
t
= 6 + 6 = 12 P (A = r) = n C r (P.P.P. -------- r times) [ q.q. --------
Required probability ---- (n – r) times]
n (E) 12 P (A = r) = n C r Pr q(n – r)
= P (E) = =
n (S) 52C 2
Probability Distribution
12 × 2 6 6
= = = Random Variable : A random variable is a real
52 × 51 13 × 51 663
valued function defined over the sample space of an
2
= experiment. A random variable is usually denoted by the
221
capital letters X, A, B, C, Z etc.
Illustration 17. Find the probability of drawing two
Discrete Random Variable : A random variable
kings a queen and a jack from a pack of 52 playing cards?
which can take finite or countable infinite number of
Solution : n (S) values is called a discrete random variable.
= Total number of ways of selecting 4 card. Continuous Random Variable : A random variable
from a pack of 52 cards which can take any value between two given limits is
= 52 C4 called a continuous random variable.

398 | CAT Complete Course

Visit : www.EasyEngineering.net
Visit : www.EasyEngineering.net

Probability distribution of a random variable : Let E = the event = a success


If X denote the number of success in n trials, then P
If the values of a random variable together with the (X = r) = probability of r success
corresponding probabilities are given, then this
description is called a probability distribution of the Therefore, the probability distribution of the random
random variable. variable X in as given below :
Probability distribution when two coins to seed : X 0 1 2
P(X) qn n C P q(n – 1) n C P2 q(n – 2)
1 2
Let A denote the number of heads occurred then
P (A = 0) = Probability of occurrence of zero head -------- r -------- n
1 1 1 --------- nC r (n – r) Pn
rP q --------
= P (T T) = × =
2 2 4
Mean and Variance of a Random Variable :
P (A = 1) = Probability of occurrence of one head
Let X be a random variable which takes values X1,
= P (H T) + P (T H) X2, ------------ Xn with corresponding probabilities P1 , P2 ,
1 1 1 1 1 ----------- P n , then
= × + × =
2 2 2 2 2 n

ww
P (A = 2) = Probability of occurrence of two head

= P (H H) = × =
1 1
2 2
1
4
Mean, µ =
Σ Pi Xi
X=1
Σ Pi
= Σ Pi X i {ΣPi = 1}

w
Thus, the probability distribution when two coins are
tossed is as given below :
X 0 .Ea1 2
and Variance,

(σ) 2 =
n
Σ (Xi – µ)2 Pi
i=1
ΣPi
P (A)

Another Form :
1
4
syE
1
2
1
4
Mean of the Binomial Distribution :
n
= Σ (Xi – µ)2 Pi
i=1

ngi
n
A  → 0 1 2  Mean, µ = Σ Pi Xi = Σ r nC r P r q (n – r)

    r=0
 P (A) →  14 12 14  Here xi = r and Pi rP q
r (n – r)
= nC

Binomial Probability of throwing a die : nee = Σn ×


=nP Σ
(n –
n
1) C (r – 1) × P × q
(n – 1)C
r (n –

(r – 1)
r)

× P(r – 1) × qn – 1 – ( r – 1)

rin
r=1
When a die is thrown,
Sample Space = {1, 2, 3, 4, 5, 6 } = n P ( q + P )n

g.n
Let E = the event of occurrence of a number greater = nP ( P + q = 1)
than 4 Variance = Σ(Xi – µ) 2 × Pi
n
E = { 5, 6 } = Σ (r – n P)2 × nC r × Pr × q(n – r)
n (E) = 2
P = probability of occurrence of event E when a die
is thrown once
n (E) 2 1
r=0
n

r=0
e
= Σ r2 × nC r Pr q(n – r) – 2n P Σ r n C r P r q(n – r)
n

r=0
n
+ n2 P2 Σ n C r Pr q(n – r)
t
= = = r=0
n (S) 6 3 n
1 2 = nP Σ (n – 1)C
(r – 1) P(r – 1) q(n – r) – 2 n P . n P + n2 p2
q = 1–P = 1– = r=1
3 3 (P + q)n
Now, if probability of occurrence of event E three n
times when a die is thrown 10 times is given by = n P Σ ( r – 1 + 1) × (n – 1)C
(r – 1) P(r – 1) q(n – r) – 2 n2
r=1
P (A = 3) = 10C 3 × P3 × q(10 – 3)
P 2 + n2 P2 (P+q = 1)
= 10C 3 ×
3() ()
1 3
×
2 7
3
n
= nP Σ (r – 1) ×
r=1
(n – 1)C
(r – 1) P(r – 1) q(n – r) + nP Σ
n

r=1

Binomial Distribution : (n – 1)C P(r – 1) q(n – r) – n2 P2


(r – 1)
n
Let P = probability of occurrence of event E in one = n P (n – 1) P Σ (n – 2)C (r – 2) P (r – 2) qn – 2 – ( r – 2) + n P
trial r=2

and (P + q)(n – 1) – n2 P2
q = 1 – P = probability of non-occurrence of event = n (n – 1) ( P + P2 q)(n – 2) + n P – n2 P2
E in one trial = n (n – 1) P2 + n P – n2 P2

CAT Complete Course | 399

Visit : www.EasyEngineering.net
Visit : www.EasyEngineering.net

= – n P2 + n P ∴ probability that out of 8 bulbs selected 3 will be


= nP(1–P) defective and 5 non-defective is given by
= nPq P ( X = 3) or P (3)
Illustration 18. 10 coins are tossed what is the = 8 C 3 P 3 q5
probability that exactly 5 heads appear ? Also, find the
probability of getting at least 8 heads. =
8!
3!5! ( ) ()
×
1 3
10
×
9 5
10
Solution : Let E = the event of occurrence of head
3306744
on one coin =
100000000
1 1
∴ P = P (E) = and q = 1 – P = Probability that no bulb will be defective.
2 2
Here n = 10 ( Since 10 coins have been tossed) = 1 – P (0) = 1 – ( ) 9 8
10
Now, P (r) = Probability that event 6 will occur
43046721 56953279
exactly r times in 5 trials. = 1– =
100000000 100000000
= n Cr × Pr × q(n – r)
Illustration 20. The first of three urns contains 7
Required probability white and 10 black balls, the second contains 5 white and

ww ( ) ( ) = 10C 5 × P2 × q5

=
10 !
×
1 2
×
1 5
12 black balls and the third contains 17 white balls. A
person chooses an urn at random and draws a ball from it
and finds it to be white. Find the probability that the ball

w.E
5!5! 2 2
come from the second urn.
63
= Solution : Urn – I = 7 white, 10 Black
256

asy
Second Part : Required Probability
= P (8) + P (9) + P (10)
= 10C 8 × P8 × q2 + 10C9 × P9 × q1
( ) A
Urn – II = 4 white, 12 Black
Urn – III = 17 white

E
= 45 × () ()
1 8
2
×
1 2
2
+ 10C10 × P10 × q0

+ 10 ×() ()
1 9
2
×
1
2 ngi
Let P
A1
denote the probability of drawing a
white ball when the ball is drawn from the second urn.
A denote the event of drawing a white ball. When

()
+1×
1 10
2 nee
one ball drawn at random from one of the three urns.
A1 = the event that one ball drawn came from the
second urn.
= 56 × ()1 10
2
urn. rin
A2 = the event that the ball drawn is from the first

7
= 7 =
2
7
128
urn. g.n
A 3 = the event that the ball drawn is from the third

Illustration 19. A lot of 100 bulbs from a manu-


facturing process is known to contain 10 defective and 90
non-defective bulbs. If 8 bulbs are selected at random,
what is the probability that
Baye’s theorem :

( )
P
A1
A
e t
(i) There will be 3 defective and 5 non-defective
bulbs.
=
( )
P (A 1 ) × P
A
A1
(ii) There will be at least one defective bulb.
Solution : Since, out of 100 bulbs, 10 bulbs are
( )
P (A 1 ) × P
A
A1
+ P (A2) × P( ) A
A2
+ P (A3) × P( ) A
A3
defective, therefore probability of drawing a defective
bulb when one bulb is selected is given by Now, P(AA ) = 175
1

P( ) =
10 1 A 2 7
P = = Similarly,
100 10 A 17
9
P( ) =
∴ q=1–P = A 3 17
10 A 17
= probability that a bulb selected is We assume that the probabilities of choosing first,
non defective. second and third urn are equal.
Since, 8 bulbs are selected, therefore number of trails 1
P (A 1 ) = P (A 2 ) = P (A 3 ) =
n = 8 3

400 | CAT Complete Course

Visit : www.EasyEngineering.net
Visit : www.EasyEngineering.net

From equation (1), Required probability Compound events are two types :
1 5 (i) Independent events
×
P( ) A1
A
=
3 17
1 5 1 7 1 7
× + × + ×
=
5
29
(ii) Dependent events
Compound Probability : Let A and B be any two
3 17 3 17 3 17
Illustration 21. An urn contains 4 red and 6 black
balls and another urn contains 3 red and 7 black balls. If
events B ≠ φ , then P () A
B
denotes the conditional
probability of occurrence of event A when B has already
one ball is drawn from each urn. Find the probability that
occurred.
(i) both balls are of same colour.
Example : Let a bag contain 2 red balls and 3 black
(ii) both balls are of different colours.
balls. One ball is drawn from the big and this ball is not
Solution : Let replaced in the bag. Then second ball is drawn from the
I- 4R : 6B bag.
P (E1) = probability of drawing red ball Let B denote the event of occurrence of a red ball in
4 2 the first draw and A denote the event of occurrence of a
= =
10 5 black in the second draw.
II- 3R : 7B When a red ball has been drawn, the number of balls

ww P (E2) = probability of drawing black ball

=
6 3
10 5
=
left is 4 and out of these four balls one is red ball and
three are black balls.

w
Illustration 22. If a dice is thrown then find the
Similarly, probability of occurrence of a number greater than 4,
3
P (E3) = ; P (E4) =
10
.Ea 7
10
Now, P (E1 ∩ E3 ) = P (E1) × P (E3 )
when an odd number has occurred.
Solution : When a die is thrown :

P (E1 ∩ E3 ) = ×
syE
E1 and E 3 are independent.
2 3
=
3
Sample Space S = {1, 2, 3, 4, 5, 6}
Let A = the event of occurrence of a
number greater than 4.
5 10 25
P (E2 ∩ E4 ) = P (E2) × P (E4 )
3 7 21 ngi = {5, 6}
B = the event of occurrence of a
= ×
5 10 50
=
Now, probability of drawing the same colour n(e)e Then, P
A
B
number greater than 4.
= probability of occurrence of a
=
3
25 50
+
21

rin
number greater than 4 when an odd number has occurred.
But among odd number only 5 is greater than 4.

g
6 + 21 27
= = So, here when an odd number has occurred total

.ne
50 50
number of case is only 3 and favourable case is 1.
Now, P (E1 ∩ E4 )
= probability of drawing the one Hence, P() A
=
1

= ×
red and one black ball
2 7
5 10 25
=
7
B

of Compound Probability :
3
(A) Multiplication Theorem of Probability or Theorem

Let S = Sample Space. In case of occurrence of


t
3 3 9
P (E2 ∩ E3 ) = × = event A when B has already occurred, B works as the
5 10 50 sample space and A ∩ B works as the event.
∴ Probability of drawing the different colour n (A ∩ B) n (A∩ B)/n (S)
=
7
+
9 Now, P () A
B
=
n (B)
=
n (B)/n (S)
25 50 P (A ∩ B)
=
14 + 9 23
=
P() A
B
=
P (B)
50 50 Case–I : If A and B are independent events, then
Compound and Conditional Probability probability of occurrence of event A is not affected by
occurrence or nor occurrence of event B.
Compound Events : When two or more events occur
together their joint occurrence is called a compound
event.
∴ P() A
B
= P (A)

∴ P (A ∩ B) = P (A) × P (B)
Example : Drawing a red and a black ball from a
bag containing 5 red and 6 black balls when two balls are Case–II : If A, B, C are any three independent events
drawn from the bag is a compound event. P (A ∩B ∩ C) = P (A) × P (B) × P (C)

CAT Complete Course | 401

Visit : www.EasyEngineering.net
Visit : www.EasyEngineering.net

(B) Complementation Rule : Let E1 = A ∩ A1


Case–I : If A and B are two independent events. E2 = B ∩ B1
∴ P (A ∪ B) = 1 – P (A 1 ) × P (B1 ) 4 2
Now, P (A) = =
Similarly, if A1, A2, A3 …………An are independent 6 3
events 2 1
P (B) = =
P (A 1 ∪ A2 ∪ A3 ∪ A4 ………∪ An ) 6 3
= 1 – P(A11 ) P(A1 2 ) P(A1 3 ) …… P (A1 n ) 3
P(A1 ) =
8
Illustration 23.
5
A dice is thrown twice. What is the chance of P(B 1 ) =
8
coming up of the number 6 in the first throw and an odd
(i) P(E1 ) = P(A ∩ A1 ) = P (A) × P(A1)
number in the second throw?
Since, A and A1 are independent.
Solution :
2 3 1
For first throw of the die P (E1) = × =
3 8 4
Let S = Sample Space = {1, 2, 3, 4, 5, 6} (ii) P(E2 ) = P (B ∩ B1)

ww A = the event of occurrence of 6


Then, n (A) = 1
= P (B) × P (B1 )
Since B and B1 are independent.

w
n(A) 1
∴ P(A) = = 1 5 5
n(S) 6 ∴ P(E2 ) = × =

.Ea
3 8 24
For 2nd throw of die :
(iii) P (E) = probability of occurrence of one is
Its contain sample space
white and one is black.

syE
= {1, 2, 3, 4, 5, 6}
B = the event of occurrence of an odd number
= {1, 3, 5}
black
Probability of occurrence of one is white and one is

n (B) = 3
n(B) 3 1 ngi = P (A ∩ B1 ) + P (B ∩ A1 )
4 5 2 3
= × + ×
6 8 6 8

nee
P(B) = = =
n(S) 6 2
5 1
Clearly, A ∩ B = the event of occurrence of 6 in the = +
12 8
first throw and occurrence of an odd number in the
second throw.
Now, since A, B are independent events.
=
rin
10 + 3 13
24
=
24

∴ P(A∩B) = P (A) × P (B) = × =


1 1 1
6 2 12
∴ P (E) =
13
24
g.n
e
Illustration 25.
Illustration 24.
A bag contains 4 white and 2 black balls. Another
contains 3 white and 5 black balls. If one ball is drawn
from each bag. Find probability that :
(i) both are white
If X and Y are independent events then
(i) X and Y1 are independent events.
(ii) X1 and Y are independent events.
(iii) X1 and Y 1 are independent events.
t
(ii) both are black Solution :
(iii) one is white and one is black. Since, X and Y are independent events.
Solution :
∴ P (A ∩ B) = P (A) × P (B)
Bag 1 = 4 white and 2 black
(i) X = (X ∩ Y) ∪ (X ∩ Y1 )
Bag 2 = 3 white and 5 black
P (X) = P (X ∩ Y) + P (X ∩ Y1 )
A = the event of drawing a white ball from first
bag Since, X ∩ Y and X ∩ Y1 are mutually exclusive
events.
B = the event of drawing a black ball from first
bag. P (X) = P (X) × P (Y) + P (X ∩ Y1 )
1
A = the event of drawing a white ball from Or, P (X ∩ Y1 ) = P (X) – P (X) × P (Y)
second bag. = P (X) {1 – P (Y)}
B 1 = the event of drawing a black ball from ⇒ P (X ∩ Y1 ) = P (X) × P (Y 1 )
second bag. Clearly, X and Y1 are independent events.

402 | CAT Complete Course

Visit : www.EasyEngineering.net
Visit : www.EasyEngineering.net

(ii) Similarly, we can show that X1 and Y are inde- Let E1 = the event that Ram wins
pendent events. E2 = the event that Shyam wins.
⇒ P (X 1 ∩ Y) = P (X 1 ) × P(Y) Clearly, exactly one of E 1 a n d E 2 w i l l certainly
(iii) P (X1 ∩ Y1 ) = P (X ∪ Y)1 happen.
= 1 – P (X ∪ Y) ∴ E2 = E1 1
= 1 – [ P (X) + P (Y) – P (X ∩ Y)] For Ram to win, he must throw a head in first, third,
= [1 – P (X)] + P (Y) [1 – P (X)] fifth ……… throws and Shyam should not throw a head
in second, fourth, sixth ……… throws.
P (X ∩ Y ) = P (X 1 ) × P (Y1)
1 1

Hence, clearly X1 and Y 1 are independent events. ( ) ( )


∴ P (E1) = + 1 –
1
2
1
2
× 1–
1
2
1
× + …+ ∞
2
Illustration 26.
A speaks the truth in 75% cases and B in 80% of the ( ) ( ) ( ) ( )
1–
1
2
× 1–
1
2
× 1–
1
2
× 1–
1
2
1
× + …+ ∞
2
cases, in what percentage of cases they likely to contradict 2 4
= + ( ) × + ( ) × + …+ ∞
1 1 1 1 1
each other in starting the same fact?
2 2 2 2 2
Solution :
1   2 4

ww
Let E1 = the event that A speaks the truth
E2 = the event that B speaks the truth
= ×  1 + ( ) + ( ) + ………
2 
1
2
1
2 
1  1  1 4 2

w E 1 1 = the event that A tells a lie


E 1 2 = the event B tells a lie
X = E1 ∩ E21
.Ea
= the event that A speaks the truth and B
= ×
2  1 2 3 3
1
 2–

and P (E2) = P (E11 ) = 1 – P (E)


= × =

tells a lie
Y = E1 1 ∩ E2 syE =
1
3
2
P (E1) = , P (E2 ) =
1
= the event that A tells a lie and B speaks
the truth
ngi Illustration 28.
3 3

nee
Z = the event that A and B contradict each If a coin is tossed n times, what is the probability that
other in starting the same fact then head will appear an odd number of times ?
Z = X ∪Y Solution :
Clearly, A and B are independent events because A
and B are independent with cases.
rin
Here number of trials = n
P = probability of occurrence of head when coin is
So, E 1 and E 2 are independent events.
E1 and E 2 1 are independent events.
tossed once =
1
2 g.n
E1 1 and E 2 are independent events.

Z = X∪Y= [34 ×15 + 14 ×45] = 207


∴ q = 1–P=
1
2
e
Now, probability of occurrence of head exactly r
times in n trials is given by
t
Required percentage P(r) = n C r × Pr × q(n – r)
7 ∴ Probability of occurrence of head odd number of
= × 100 = 35%
20 times in n trials
Illustration 27. = P (1) + P (3) + P (5) + ……
1 n–1
Two persons Ram and Shyam throw a coin alter-
nately till one of them gets ‘head’ and wins the game. = n C1 × () ()1
2
×
1
2
Find their respective probabilities of winning. 3 n–3

Solution : () ()
+ nC3 ×
1
2
×
1
2
+ ……
n
Let E = the event of occurrence of head in one throw
of a coin = () 1
2
× (n C 1 + n C 3 + n C 5 + ……)
n
E1 = the event of occurrence of tail in one throw of
a coin
= () 1
2
× 2n – 1 {·.· C1 + C3 + … = 2n – 1}

1 1 1
∴ P (E) = and P (E1 ) = 1 – P (E) = =
2 2 2

CAT Complete Course | 403

Visit : www.EasyEngineering.net
Visit : www.EasyEngineering.net

BAYES’S THEOREM Illustration 29.


A man known to speak the truth 3 out of 4 times. He
Partition of a Set : A family of sets A1, A 2 , ……A n throws a die and reports that it is a six. Find the pro-
is said to form a partition of a set A if bability that it is actually a six.
(i) A1, A2, ……… An are non-empty Solution :
(ii) Ai ∩ Aj = φ for i ≠ j Let A = the event that man reports occurrence of E
(iii) A = A1 ∪ A2 ∪ ……… ∪ An A1 = the event of occurrence of 6 when a die is
Baye’s Theorem : If A1 , A2 , A3 , ……… An be n thrown
mutually exclusive and exhaustive events and A is an A2 = the event of non-occurrence of 6 when a die is
event which occurs together ( in conjunction) with either thrown
of Ai i.e. if A1 , A2, …… A n form a partition of the sample Now, By Baye’s Theorem :
space S and A be any event, then ( )
P(A1 ) × P
A

( )
P(Ak) × P A
A
( )
P
A1
=
A1
( )A
P Ak =
( ) A
( ) A
k

P(A1 ) P A + P(A2) P A + …… + P (An ) P A


1 2
( ) A
n
A
( )
P(A1 ) P
A
A1 ( )
+ P(A2 ) P
A
A2
1. If P (A1 ) = P (A 2 ) = P (A 3 ) = ……… = P (An) 1 5
P (A 1 ) = , P (A2 ) =

w( )w( ) ( () ) ( )
P
Ak
A
=
A
P
A
A
Ak
A
( )
P
A
A1
6 6
= Probability that man reports occurrence

w.E P
A1
+P
A2
+ …… + P
2. The probabilities P (A1), P (A2 ) …… P (An ) which
are known before the experiment takes place are
An of 6 when 6 has actually occurred =

( )
P
A
3
4
= Robability that man reports occurrence of

a(s)y
A2
Ai
called priori probabilities and P are called 6 when 6 has not actually occurred.
A
posteriori probabilities. ( )
P
A
= Probability that the man tells a lie =
1

then prove that

P ( )Ak
=
E
● If A1 , A2 , ……… An form a partition of an event A

P(Ak) ngi ( )
P
A2

A1
A
=
1
1 3

3
×
6 4
× + ×
5 1
=
3
8
4

A P(A1 ) + P(A2 ) + …… + P(An )

nee
Proof : Since A1 , A2, …… An for a partition of A Illustration 30.
therefore 3
6 4 6 4

2
(i) A1, A2, ……An are non-empty.
(ii) They are pair wise disjoint i.e. no two of A1,
If P (A) = ,
5
(a) P (A ∪ B) ≥ rin
2
P (B) = then
3

A2, ……An have any common element.


(iii) A = A1 ∪ A2 ∪ A3 …… ∪ An (b)
4
15
3
≤ P (A ∩ B) ≤ g.n 5
3

From (i), (ii) and (iii) it is clear that


(A ∩ A1), (A ∩ A2), …… (A ∩ An) are non-empty
pair wise disjoint (they are mutually exclusive)
and By using additional theorem :
Solution :
Since,
∴ P (A ∪ B) ≥
e
P (A ∪ B) ≥ P (B)
2
3
t
…… (1)
P (A) = P (A ∩ A1 ) + P (A ∩ A2) + … + P (A ∩ An ) Or, P (A ∩ B) ≤ P (A)
P (A k ∩ A)
Now, P ( ) Ak
A
=
P (A)
⇒ P (A ∩ B) ≤
3
5
…… (2)

P (A k ∩ A) P (A ∩ B) = P (A) + P (B) – P (A ∪ B)
= ≥ P (A) + P (B) – 1
P (A ∩ A1 ) + P(A ∩ A2 ) + ……+ P (A ∩ An )
3 2 4

=
( )
P(Ak) × P
A
Ak
≥ + –1=
5 3 15
…(3)
Now, from equation (2) and equation (3), we get
( )
P(A1 ) × P
A
A1 ( )
+ P(A2 ) × P
A
A2
+ …… 4
≤ P (A ∩ B) ≤
3
Since, A1, A2, ……and An are subsets of A. 15 5
Illustration 31.
∴ P () A
Ai
= 1 for i = 1, 2, 3, …… n In a test, an examinee either guesses or copies or
known the answer to a multiple choice question with four
∴P ( ) Ak
A
=
P(Ak)
P(A1 ) + P(A2 ) + ……+ P(An) choice the probability that he makes a guess is · The
1
3

404 | CAT Complete Course

Visit : www.EasyEngineering.net
Visit : www.EasyEngineering.net

1 where r = wins exactly 4 games


probability that he copies the answer is · The probability
6 2 2 1
1 P = q=1–P=1– =
that the answer is correct, given that he copies, is · Find 3 3 3
8 4 2

the probability that he knows the answer to the question,


given that he correctly answered it.
P (4) = 6 C 4 × () ()
2
3
×
4
1
3
6×5
Solution :
Let A = the event that the examinee gives the
=
2 ()
×
2
3
×
1
9
80
correct answer =
A1 = the event that the examinee knows the 243
answer (ii) X wins at least games means r ≥ 4
A2 = the event that the examinee guesses the i.e. X = 4, 5, 6
answer ∴ P (r ≥ 4) = P (r = 4) or P (r = 5) or P (r = 6)
4 2 5
A3 = the event that the examinee copies the
answer
= 6 C4 × () ()
2
3
×
1
3 ()
+ 6 C5 ×
2
3
6
By Baye’s theorem,

( )
A1 () ×
1
3 ()
+ 6 C6 ×
2
3

ww
Required probability = P
A

()
P(A1) × P
A
A1
=
80
+
16
+
243 243 243 × 3
240 + 48 + 64 352
64

(w
= = =
)P(A1) × P
A
()
+ P(A2) × P
A
+ P(A3) × P ()
A 243 × 3 729
A1
According to question :
.Ea
1
A2 A3

1
Exercise – A
1. A bag contains 6 black balls and an unknown

syE
P (A 2 ) = P (A 3 ) =
3 6 number (not greater than six) of white balls. Three
∴ P (A 1 ) = 1 – P (A 2 ) – P (A3 ) balls are drawn successively and not replaced and all
1 1 1 are found to be white. The chance that a black ball
= 1– – =

Also, P
3 6 2

(AA ) = probability that the examinee gives ngi will be drawn next is—
(A)
677
909
(B)
609
677

nee
1
the correct answer. 480 280
(C) (D)
Since, probability of occurrence of a sure event = 1 909 909
( )A
∴P
A2
= probability that the examinee gives the
1 rin
2. It is known that at noon at a certain place the sun is
hidden by clouds on an average two days out of

g.n
correct answer. When he makes a gives = every three. The chance that the sun will be shining
4 at noon on at least four out of five specified future
P ( )
A
=
1 days is—

e
A3 8 21 19
(A) (B)

P ( )
A1
A
=
1
2
1
2
×1
1 1 1 1
×1+ × + ×
3 4 6 8
(C)
4096
20
4096
(D)
4096
17
4096 t
3. Supposing that it is 4 to 7 against a person A who is
24
= now 30 years of age, living till he is 80, and 5 to 6
29 against a person B now 25 living till he is 75. The
Illustration 32. chance that at least one of these persons will be alive
X and Y play a series of 6 games in badminton. X1 s 30 years hence is—
2 57 91
chance of winning a single game against Y is · Find the (A) (B)
3 121 121
chance that X wins 101
(C) (D) None of these
(i) exactly 4 games 121
(ii) at least 4 games 4. Three dice are thrown together. The probability that
the sum of all faces showing up will be more than 16
Solution :
is—
Here each game is trial and X1 s winning the game is 1 1
a success. All games are independent. So, using the (A) (B)
54 27
Binomial Distribution. 1 3
P (r) = n C r × Pr × (1 – P)n – r (C) (D)
108 154

CAT Complete Course | 405

Visit : www.EasyEngineering.net
Visit : www.EasyEngineering.net

5. A bag contains 5 white and 6 black balls, and 6 are 45 46


(A) (B)
successively drawn out and not replaced. What's the 87 87
chance that they alternately of different colours ? 47 87
(C) (D)
15 30 87 45
(A) (B)
77 77 13. From a bag containing 5 white and 13 black balls a
31 16 man drawn 5 at a random; what are the odds against
(C) (D)
77 77 these being all black ?
6. Three balls are drawn successively from box con- (A) 8567 : 1 (B) 1 : 8567
taining 9 white, 10 red, 5 blue balls. If each drawn (C) 5423 : 1 (D) 1 : 5423
balls is not replaced, then the probability of these are 14. The sincere to casual ratio in a class is 12 : 10. If this
drawn in order red, white and black is— trend is expected to continue, the probability that a
11 12 newcomer is casual, will be—
(A) (B)
2024 2024 11 5
(A) (B)
25 29 12 11
(C) (D)
2024 2024 6 12
(C) (D)
11 11

ww
7. X, Y, Z in order, cut a pack of cards, replacing them
after each cut, on condition that the first who cuts a
heart shall win a prize. Then A's chance of winning
15. Two fruits are to be picked from a basket having 6
mangoes and 10 oranges. The probability that both of

w
is— them are mango is—
(A) 16/37 (B) 12/37 2 5

.Ea
(A) (B)
21 42
(C) 9/37 (D) 14/37
3 6
(C) (D)
8. A party of n persons is sitting at a round table. The 21 21

each other are—


(A) 2 : (n – 1)
syE
odds against two specified individuals sitting next to

(B) (n – 3) : 2
16. Out of integers from 5 to 40 (both 5 & 40 included),
a number is picked at random. The probability that

(C) 2 : (n – 3) (D) (n – 1) : 2
9. Triangles are formed by joining vertices of a octagon. ngi the number is divisible by 5 is—
(A)
2
9
(B)
3
5
Any one those triangles is selected at random. What
is the probability that the selected triangle has no nee (C)
7
9
(D)
6
7

rin
side common with the octagon ? 17. In a plane, 6 lines of lengths 2, 3, 4, 5 & 6 cm are
5 14 lying. What is the probability that by joining the three
(A) (B)
7 5 randomly chosen lines end to end a triangle can't be

(C)
7
9
(D)
5
14
formed ?
(A)
2 g.n
(B)
3

10. If on an average 1 bulb out of every 7 is fused, then


the chance that out of 5 bulbs expected, at least 3 will
arrive lighed, is—
16416 16529
(C)
5
5
7
(D)
e10
7
10
18. If a rod is marked at random in n points and divided t
(A) (B) at those points, then the chance that none of the parts
75 75 1
75 910 shall be greater than th of the rod is—
(C) (D) n
16416 75 1 1
(A) n – 1 (B) n
11. In an examination, there are 500 students. 120 passed n n
n!
the first paper and 250 passed the second paper. 150 (C) n (D) 2nPn–1
n
students passed both the papers. The probability that
a student at random has failed in both the papers is— 19. Two drawing, each of 2 balls, are made from a bag
2 1 containing 6 black and 14 yellow balls, the balls not
(A) (B) being replaced before the second trial. The chance
25 25
that the first drawing will be give 2 yellow and the
4 3
(C) (D) second drawing will give 2 black balls is—
25 50
16 15
12. A person is asked to randomly pick two balls which (A) (B)
323 323
has 20 black & 10 red balls. The probability that the 14 16
persons pick two balls of the same colour is— (C) (D)
323 350

406 | CAT Complete Course

Visit : www.EasyEngineering.net
Visit : www.EasyEngineering.net

20. X and Y throw one die for a stake of Rs. 22, which 27. Eight children are standing in a line outside a ticket
will be won by the player who first throws a 1. If X window at Appu Ghar, New Delhi. Four of these
has the first throw, then what could be their children have a one a rupee coin each and the
respective expectations ? remaining 4 children have a two rupee coin each.
(A) Rs. 10, Rs. 12 (B) Rs. 11, Rs. 11 The entry ticket is priced at rupee one. If all the
(C) Rs. 13, Rs. 9 (D) Rs. 12, Rs. 10 arrangements of the eight children are equally likely,
then the probability that no child will have to wait for
21. Before a race the chance of three runners, A, B, C, change is—(assume that the cashier at the ticket
were estimated to be proportional to 5, 3, 2; but window begins with no change)
during the race A meets with an accident which 1 1
reduces his chance to one-third . What is the chance (A) (B)
2 5
of B ?
1 1
2 3 (C) (D)
(A) (B) 8 6
5 5
4 1 28. A box contains 13 white balls and 12 black balls.
(C) (D)
15 5 One ball is taken out from the box and not replaced
back. If a ball is now taken at random from the box,

ww
22. In the above question (Q. 21), what is the chance of
C?
(A)
4
(B)
2
then what is the probability that it is black ?

(A)
13
(B)
12

w
25 25
15 15
11 14

.Ea
3 2 (C) (D)
(C) (D) 25 25
5 5
23. 10 persons draw lots for the occupancy of the six 29. A closed box contains two balls whose colours are

syE
seats in a first class railway compartment the chance
that two specified persons obtain opposite seats is—
9 7
not known. [Each an be either white or black.] A
white ball has been put [which is indistinguishable
from the other size] into the box. What is the
(A)

(C)
10
1
(B)

(D)
10
3 ngi probability of drawing a white ball ?

(A)
1
(B) 1

nee
10 10 3
2 1
24. Karunesh Verma and Kaloesh Sharma appear for an (C) (D)
3 6
interview for two vacancies in an organisation for
the same post. The probabilities of their selection are
3 2 rin
30. Three dice are thrown. What is the probability that
the numbers shown on the dice are not same?

g.n
and respectively. What is the probability that none
5 3 103 105
of them will be selected ? (A) (B)
108 108
(A)

(C)
3
15
2
15
(B)

(D)
1
15
4
15
(C)
21
54
(D)
e
7
36

Exercise – B t
25. The odds against a certain events are 3 : 5 and odds 1. A bag contains 6 black balls and an unknown number
in favour of another independent events are 4 : 7. (not greater than six) of white balls. Three balls are
The probability that none of the events will occur is— drawn successively and not replaced and all are
3 5 found to be white. The chance that a black ball will
(A) (B) be drawn next is—
22 22
7 9 (A) 677/909 (B) 609/677
(C) (D)
22 22 (C) 480/909 (D) 280/909
26. Rahul and Shahil throw a die alternately till one of 2. It is known that at noon at a certain place the sun is
them gets ‘6’ and wins the game. What are their res- hidden by clouds on an average two days out of
pectively probabilities of winning ? every three. The chance that the sun will be shining
(A) ( )
6 5

11 11
(B) ( )
7 4

11 11
at noon on at least four out of five specified future
days is—

(C) ( )
3 2

5 7
(D) ( )
7 2

11 7
(A) 1/81
(C) 11/243
(B) 16/243
(D) 7/20

CAT Complete Course | 407

Visit : www.EasyEngineering.net
Visit : www.EasyEngineering.net

3. Supposing that it is 9 to 7 against a person A who is 11. In an examination, there are 500 students. 150 passed
now 35 years of age, living till he is 65, and 3 to 2 the first paper and 350 passed the second paper. 50
against a person B now 45 living till he is 75. The students passed both the papers. The probability that
chance that at least one of these persons will be alive a student at random has failed in both the papers is—
30 years hence is— (A) 1/5 (B) 1/10
(A) 1/2 (B) 27/80 (C) 3/10 (D) 3/5
(C) 3/4 (D) 53/80 12. A person is asked to randomly pick two balls from a
4. Three dice are thrown together. The probability that bag which has 15 yellow & 5 red balls. The
the sum of all faces showing up will be more than 15 probability that the persons pick two balls of the
is— same colour is—
(A) 5/108 (B) 5/6 (A) 33/38 (B) 23/38
(C) 1/54 (D) 5/18 (C) 38/43 (D) 15/38
13. From a bag containing 4 white and 5 black balls a
5. A bag contains 5 white and 3 black balls, and 4 are
man drawn 3 at a random; what are the odds against
successively drawn out and not replaced. What's the
these being all black ?
chance that they alternately of different colours ?

ww
(A) 1/6
(C) 1/4
(B) 1/5
(D) 1/7
(A) 37 : 5
(C) 23 : 19
(B) 5 : 37
(D) 19 : 23
14. The sincere to casual ratio in a class is 10 : 27. If this

w
6. Three balls are drawn successively from an urn
containing 8 red balls, 5 white balls and 7 black

.Ea
balls. If each drawn balls is not replaced, then the
probability of these are drawn in order red, white and
trend is expected to continue, the probability that a
newcomer is casual, will be—
(A) 10/37
(C) 10/27
(B) 27/37
(D) 17/27
black is—

(A)
7
19 syE
(B)
1
171
15. Two balls are to be picked from a bag having 6 white
balls and 9 black balls. The probability that both of
them are black is—
(C)
7
171
(D)
19
7
ngi (A) 17/49
(C) 12/35
(B) 27/35
(D) 23/35
7. A, B, C in order, cut a pack of cards, replacing them
after each cut, on condition that the first who cuts a
spade shall win a prize. Then A's chance of winning nee
16. Out of integers from 10 to 50 (both 10 & 50
included), a number is divisible by 7 is—

rin
(A) 6/40 (B) 6/41
is—
(C) 6/39 (D) 7/41
(A) 16/37 (B) 12/37
(C) 9/37 (D) 14/37
8. A party of n persons is sitting at a round table. The g.n
17. In a plane, 5 lines of lengths 2, 3, 4, 5 & 6 cm are
lying. What is the probability that by joining the
three randomly chosen lines end to end a triangle
odds against two specified individuals sitting next to
each other are—
(A) 2 : (n – 1) (B) (n – 3) : 2
can't be formed ?
(A) 3/10
(C) 1/2
e
(B) 7/10
(D) 1 t
(C) 2 : (n – 3) (D) (n – 1) : 2 18. If a rod is marked at random in n points and divided
9. Triangles are formed by joining vertices of a at those points, then the chance that none of the parts
octagon. Any one those triangles is selected at 1
shall be greater than th of the rod is—
random. What is the probability that the selected n
triangle has no side common with the octagon ? 1 1
(A) n–1 (B) n
n n
(A) 3/7 (B) 2/7
n!
(C) 5/7 (D) 1/7 (C) n (D) 2nPn–1
n
10. If on an average 1 vessel out of every 10 is broken, 19. Two drawings, each of 3 balls are made from a bag
then the chance that out of 5 vessels expected, at containing 5 white and 8 black balls, the balls not
least 4 will arrive safely is— being replaced before the second trial. The chance
(A) 34712/50000 that the first drawing will be give 3 white and the
(B) (9/10)5 second drawing will give 3 black balls is—
(C) 30618/50000 (A) 7/429 (B) 7/15
(D) 45927/50000 (C) 5/143 (D) 8/15

408 | CAT Complete Course

Visit : www.EasyEngineering.net
Visit : www.EasyEngineering.net

20. A and B throw one die for a stake of Rs. 11, which 1 1
(A) (B)
will be won by the player who first throws a 6. If A 2 5
has the first throw, then what could be their 1 1
(C) (D)
respective expectations ? 8 6
(A) Rs. 7, Rs. 4 28. A box contains m white balls and n black balls. One
(B) Rs. 6, Rs. 5 ball is taken out from the box and not replaced back.
(C) Rs. 4, Rs. 7 If a ball is now taken at random from the box, then
(D) Rs. 5, Rs. 6 what is the probability that it is white ?
m m
21. Before a race the chance of three runners, A, B, C, (A) (B)
m+n m+n–1
were estimated to be proportional to 5, 3, 2; but
m–1 m–1
during the race A meets with an accident which (C) (D)
m+n–1 m+n
reduces his chance to one-third . What is the chance
of B ? 29. A closed box contains two balls whose colours are
(A) 2/5 (B) 3/5 not known. [Each n be either white or black.] A
white ball has been put [which is indistinguishable
(C) 4/15 (D) 1/5
from the other size] into the box. What is the

ww
22. In the above question (Q. 21), what is the chance of
C?
probability of drawing a white ball ?

(A)
1
(B) 1

w
(A) 4/15 (B) 2/15 3
(C) 3/5 (D) 2/5 2 1

.Ea
23. Seven persons draw lots for the occupancy of the six
seats in a first class railway compartment the chance
(C)
3
(D)
6
30. Three dices are thrown. What is the probability that

(A) 6/7
(C) 1/7
syE
that two specified persons obtain opposite seats is—
(B) 5/7
(D) 3/7
the sum of numbers shown on the dice is neither 3
nor 18 ?
(A) 1/36 (B) 5/36
24. Ram Gopal Verma and Shyam Gopal Sharma appear
for an interview for two vacancies in an organisation ngi (C) 1/108 (D) 107/108

for the same post. The probabilities of their selection


are 1/6 and 2/5 respectively. What is the probability
that none of them will be selected ? nee Solutions – A
1. (A) The bag contains 6 black balls and there are four

rin
way of the following containing white balls :
(A) 5/6 (B) 1/5 (a) It may contain three white balls.
(C) 1/2 (D) 3/5

g.n
(b) It may contain four white balls.
25. The odds against a certain events are 5 : 2 and odds (c) It may contain five white balls.
in favour of another independent events are 6 : 5. (d) It may contain six white balls.
The probability that none of the events will occur is—
(A) 6/11
(C) 52/77
(B) 5/11
(D) 25/77
26. Two persons A and B throw a coin alternately till
give white ball is :
3 2 1
e
drawn = p1 = × × because balls are not
t
(a) The probability that at the three successive draws

9 8 7
one of them gets head and wins the game. What are
replaced.
their respectively probabilities of winning ?
2 1 1 2 (b) The probability of drawing white balls at three
(A) , (B) , successive draws is
3 3 5 7
1 2 1 1 4 3 2
(C) , (D) , p2 = × × ·
3 7 5 3 10 9 8
27. Eight children are standing in a line outside a ticket Similarly,
window at Appu Ghar, New Delhi. Four of these
children have a one a rupee coin each and the
p3 = ( ) ( ) ()
5
11
×
4
10
×
3
9
remaining 4 children have a two rupee coin each. 6 2 4
The entry ticket is priced at rupee one. If all the p4 = × ×
12 11 10
arrangements of the eight children are equally likely,
then the probability that no child will have to wait for When there are 6 white balls, i.e.,
change is— (assume that the cashier at the ticket 1 1 2 1
p1 = ,p = ,p = ,p =
window begins with no change) 84 2 30 3 33 4 11

CAT Complete Course | 409

Visit : www.EasyEngineering.net
Visit : www.EasyEngineering.net

If Q1 , Q2 , Q3 , Q4 be denoting posteriori prob- Total probability


ability of (a), (b), (c) and (d) cases. 15 15 2 × 15 30
= + = = ·
Q Q2 Q Q 1 77 77 77 77
∴ 1 = = 3 = 4 =
55 154 280 420 909 6. (C) On first drawing, Probability of drawing red
Therefore, the chance of drawing the black ball 10C
10 5
next = 24 1 = =
C 1 24 12
6 3 2
Q 1 × 1 + Q2 × + Q 3 × + Q 4 × On second drawing, Probability of drawing white
7 4 3 9C
9
55 154 6 280 3 420 2 = 23 1 =
= ×1+ × + × + × C 1 23
909 909 7 909 4 909 3
677 On third drawing, Probability of drawing black
= · 5C
909 5
= 22 1 =
3 C 1 22
2. (C) The probability that the sun is hidden =
4 ∴ Required probability
1 5 9 5 25
∴ The probability that it is out = = × × = ·

ww 4
Now, at least 5 days shining = 5 days out and 1 day
hidden + 6 days out
12 23 22 2024
7. (A) There are 13 cards of spade in a pack of 52 cards.
So, the chance that any of them will cut's a spade is

w
() =

= ()
1
4
1
46
5

.Ea
× 6 C1 +()
3
4

[6 × 3 + 1]
1
4
6
13/54 = 1/4 ⇒ Probability that it is not a spade = 1 –
1/4 = 3/4. Consider the following mutually exclusive
ways in which A may win : A wins in the first cut,
OR X, Y, Z have failed and then A wins, OR X, Y,

=
19
=
syE
46 16 × 16 × 16
19
Z, X, Y, Z have failed and then A wins and so on
upto infinity ⇒ Respective chance of these events
are = 1/4, [(3/4) × (3/4) × (3/4) × (1/4)], [(3/4) × (3/4)
=
19
4096
4ngi × (3/4) × (3/4) × (3/4) × (3/4) × (1/4)] …. To infinity
⇒ A's chance of succeeding = (1/4) + (3/4)3 × (1/4)
+ (3/4) 6 × (1/4) + …. = 1/4 [1/(1 – 27/64)] = 16/37.
3. (C) The chance that A will die within 50 years is ;

5
the chance that B will die within 50 years is ; there-
11
nee
8. (B) Total numbers of ways in which n persons can sit
a round table = (n – 1)! As the two specified

fore the chance that both will die is


4 5
11

× or
20
; rin
individuals are to sit together, we may consider them
to be a single individuals and then we have (n – 2)!

g.n
11 11 121 Different arrangements in which the two specified
Therefore, the chance that both will not be dead, that individuals can sit together in the same order. Since,
20 101 they can permute among themselves, the total

4.
is at least one will be alive, is 1 –

(A) Total possible outcomes


= 6 × 6 × 6 = 216
or
121 121
·

e
possible ways favourable to the events are 2 × (n –
2)! ⇒ Probability of two specific individuals sitting
together = [2 × (n – 2)!]/(n – 1)! = 2/(2 + n – 3).
Hence, the odds against the events are (n – 3) to 2.
t
Favourable cases are (6, 6, 6), (6, 6, 5), (6, 5, 6), (5,
9. (D) Here number of sides are = 9
6, 6)
Total number of triangles are
Total number of favourable cases = 4
4 1 9 9×8×7
Probability = = · 9C = = = 84
216 54 3
6
3 6
5. (B) Total number of balls = 11.
Total number of triangles with no sides common
Let the first drawn ball is white.
= n C 3 – (n – 4) n – n
So, required probability
= 9 C 3 – (9 – 4) 9 – 9
5 6 4 5 3 4 15
= × × × × × = · = 84 – 45 – 9
11 10 9 8 7 6 77
When we start with a black ball, the required = 84 – 54 = 30
5 5 5 4 4 3 15 So, required probability
Probability = × × × × × = ·
11 10 9 8 7 6 77 30 5
= = ·
Since, these two cases are mutually exclusive. 84 14

410 | CAT Complete Course

Visit : www.EasyEngineering.net
Visit : www.EasyEngineering.net

1 15. (C) The total number of ways of selecting two fruits


10. (A) The chance that the bulb is fused =
7 = 15C 2
6 Favourable case = 5 C 2
⇒ Chance that the bulb is lighted = at least 3 will
7 5C
5×2 2
be lighted if all 5 are light or 4 are lighted and one is ⇒ Probability = 15 2 = = ·
fused or 3 will be lighted and 2 are fused. C 2 15 × 7 21
Required Probability 16. (A) Possible comes = 36.
5 4 3 2 Favorable case = 8
= () ()
6
7
+5
6
7() () 1
× + 5 C2
7
6
7
×
1
7 Required probability =
8 2
= ·
5 4 3 36 9
= ( ) + ×( ) + ×( )
6 5 6 10 6 ∴
7 7 7 7 7 2 17. (B) The sum of two sides in a triangle is greater
63 than the third side.
= [36 + 30 + 10]
75 ∴ In following cases, triangle cannot be formed.
76 × 216 16416 2 3 5
= =
75 75 2 3 6

ww
11. (B) Total number of students = Number passing in I
paper only + Number of student passing in II paper
only + Number passing in both + Number failed in
2
2
2
4
4
4
7
6
7

w
both

.Ea
⇒ 500 = (120 – x) + (250 – x) + 150 + x
⇒ 500 = 520 – x ⇒ x = 20
⇒ Required probability
2 5 7
∴ Required probability = 6 =
6 6×6
C3 6 × 5 × 4

=
20
500 25
=
1
·
syE 18.
=
3
10
·

(b) The care, none of the parts shall be greater than


12. (C) Total possible outcomes = 30C 2
The person can pick up two black balls in 20C 2 ways
and two red balls in 10C2 ways ngi 1
n
th of the rod is possible if the rod is divided in n
1

nee
equal parts. The probability of one part being th of
n
Required probability 1
19 × 20 10 × 9 the rod = .
+ n

rin
20C 10C
2 2 2 2 1
= 30 + 30 = ∴ Required probability of n parts being th of the
C2 C2 30 × 29 n

=
190 + 45 235 47
435
= =
435 87
2 rod each
1 1 1
= × × × …… = g( .)n 1 1
n
.
13. (A) The total number of ways in which 5 balls can be
drawn is 18C 5 and the number of ways of drawing 5 19.
white ball is 5 C 3 ; therefore the chance of drawing 5
white balls
n n n

etn n
14C
(C) The chance of drawing 2 yellow balls = 20 2 and
without replacing, the chance that of drawing two
6C
C2

5C
1 black balls = 18 2 ·
= 18 5 = C2
C 5 |18 14C 6C
13 5 13 × 5 × 4 × 3 × 2 × 1 Required Probability = 20 2 × 18 2
C2 C2
= =
18 18 × 17 × 16 × 15 × 14 13 14 × 6 3 × 5
= ×
1 10 × 19 9 × 17
=
8568 14 14
= = ·
Thus, the odds against the event are 8568 to 1. 19 × 17 323
1
14. (B) Net cases = 12 + 10 = 22. 20. (D) In his first throw X's chance is ; in his second
6
The required Probability 5 5 1
throw it is × × , because each player must have

=
1
22 ( )=
10 5
= ·
6 6 6
failed once before X can have a second throw; in his
4
1
10 ( ) 22 11 third throw his chance = () 5
6
1
× because each
6

CAT Complete Course | 411

Visit : www.EasyEngineering.net
Visit : www.EasyEngineering.net

player must have failed twice; and so on. Thus, X's and Probability of coming another then 6
chance is the sum of the infinite series 5
2 4 = = P (F)
1 
6 
1+ ( ) ( ) + …… ·
5
6
+
5
6 

6
Now, Rahul wins if he throws a six in 1st or 3rd or
Similarly, Y's chance is the sum of the infinite series 5th or 7th …..
∴ Probability of Rahul wins = P[S or (FFS) or
5 1 
( ) ( ) + …
2 4
5 5
6 6 
, 1+ + (FFFFS) or ……]
6 6
= P(S) + P(FFS) + P(FFFFS) + ……..
∴ X's chance is to Y's as 6 is to 5. Their respective = P(S) + P(F) . P(F) . P(S) + P(F) . P(F) . P(F) . P(F)
6 5
chances are therefore and , and their expecta- . P(SH) + ……..
11 11
6 1 5 5 1 5 5 5 5 1
tions are Rs. × 22 = 12 and Rs. 10 respectively. = + · · + · · · · + ……
11 6 6 6 6 6 6 6 6 6
2 4

21. (A) The chance of A, B, C before the starts are ,


5
10
= +
1 1 5
6 6 6 () ()+
1 5
6 6
+ ……

1  
3 2 1 3 1 4
1 + ( ) + ( ) + …
ww , i.e., , , respectively. A can lose in two 5 5
6 
= 2
10 10 2 10 5
ways; either by the winning of B or C.
6 6 
As A's chance of winning is 1/2; therefore A's chance 1 1 36 6

w
of losing is 1/2. After the accident his chance of

.Ea
winning is 1/3, and hence his chance of losing
becomes 2/3. ⇒ his chance of losing is increased in
the ratio of 4 to 3, also B's and C's chances of
=
6
×
1– () 5
6
2=

P(Rahul) =
6
11
=
6 (36 – 25) 11

3 4 2
syE
winning are increased in the same ratio. Thus B's
chance of winning = × = ; and C's chance of
10 3 5
P(Shahil) = 1 –
6
=
11 11
5

winning =
2 4 4
× =
10 3 15
·
ngi ⇒ Required Probability = ( 6 5

11 11 ) ·

22.
23.
(A)
(C) Let the X & Y are two specified persons. The
chance that X occupies any seat in the first class nee
27. (b) Total possible combinations of distribution of
ticket that no child will have to wait for change are
as follows :
9
railway compartment is , X has occupied his seat
10
1. 1 1 1 1 2 2 2 2
3. 1 1 1 2 2 1 2 2 rin2. 1 1 1 2 1 2 2 2
4. 1 1 1 2 2 2 1 2
there is only one case favourable to the event that Y
occupies is seat opposite to X. Thus, the chance of the
compound events that Y gains an opposite seat to X
5. 1 1 2 1 1 2 2 2
7. 1 1 2 1 2 1 2 2 g.n
6. 1 1 2 2 1 1 2 2
8. 1 1 2 1 2 2 1 2

24.
is
9 1 1
× =
10 9 10
·

(C) Required Probability


9. 1 1 2 2 1 2 1 2
11. 1 2 1 1 1 2 2 2
13. 1 2 1 1 2 2 1 2
e
10. 1 2 1 2 1 2 1 2
12. 1 2 1 1 2 1 2 2
14. 1 2 1 2 1 1 2 2 t
( ) ( )
= 1–
3
5
× 1–
2
3
Now, these 14 combinations can again be arranged in
4 ! × 4 ! Ways.
2 1 2 So, total favourable combinations
= × = ·
5 3 15
= 14 × 4 ! × 4 !
25. (B) Let A & B be the event.
And total possible combinations = 8 !
5
⇒ P(A) = , 14 × 4 ! × 4 ! 1
8 Required Probability = = ·
8! 5
4
⇒ P(B) = · 28. (B) Case I : Let the first ball removed be black
11
∴ Required probability ⇒ Black balls left = 11 and white balls left
5 4 5 = 13
× =
8 11 22 Probability of getting a black ball in the first and
26. (A) Probability of coming six second draw
1 12 1 1
= = P(S) = × =
6 25 24 50

412 | CAT Complete Course

Visit : www.EasyEngineering.net
Visit : www.EasyEngineering.net

Case II : Let the first ball removed be white 3 2 1


drawn = p1 = × × because balls
⇒ Black balls left = 12 9 8 7
are not replaced.
White ball left = n – 1
(b) The probability of drawing white balls at three
Probability of getting a white ball in the first and 4 3 2
black in second draw successive draws is p2 = × × ·
10 9 8
13 12 13 Similarly, p3 = (5/11) × (4/10) × (3/9)
= × =
25 24 50 6 2 4
Required Probability = Sum of these two p4 = × ×
12 11 10
11 13 12 When there are 6 white balls, i.e.,
= + = ·
50 50 25 1 1 2 1
p1 = ,p = ,p = ,p =
29. (C) Case I : Assume that the box has two white balls. 84 2 30 3 33 4 11
When a white ball is added, it now has 3 white balls If Q1, Q2, Q3, Q4, be denoting posteriori probability of
⇒ Probability of drawing a white ball = Probability (a), (b), (c) and (d) cases.
of drawing one ball × Probability of white ball Q1 Q2 Q Q 1
1 3 1 ∴ = = 3 = 4 =
55 154 280 420 909

ww = × =
3 3 3
Case II : Assume that the box has one white and one
Therefore, the chance of drawing the black ball next
6 3
Q 1 × 1 + Q2 × + Q 3 × + Q 4 ×
2

w
Black ball. 7 4 3
When a white ball is added the box will then contain 55 154 6 280 3 420 2

.Ea
2 white balls and 1 Black ball ⇒ Probability of draw-
ing a white ball = Probability of drawing one ball ×
=

=
909
677
×1+

·
× +
909 7 909 4 909 3
× + ×

Probability of white ball

syE 1 2 2
= × = ·
3 3 9
2.
909
(C) Here the probability that the sun is hidden = 2/3
∴ The probability that it out = 1/3
Case III : Assume that the box has two Black balls.
When a white ball is added, it now has 1 white ball
and 2 Black balls ngi ( ) ( ) ( )
Now, at least 4 days shining = 4 days out and 1 day
hidden + 5 days out
4 5

⇒ Probability of drawing a white = Probability of


drawing one ball × Probability of white ball nee = 5

5×2
2
3
×

1
1
3
11
+
1
3

rin
= + = ·
1 1 1 243 243 243
× = ⇒ Overall Probability
3 3 9 9
3. (D) The chance that A will die within 30 years is ;
1 2 1 2
= + + =
3 9 9 3
g.n
the chance that B will die within 30 years is ;
16
3
5
[because all these are mutually exclusive cases].
30. (B) Possible outcomes = 63 = 6 × 6 × 6 = 216
The outcomes which are not favourable are (1, 1, 1),
(2, 2, 2), (3, 3, 3), (4, 4, 4), (5, 5, 5) and (6, 6, 6)
e
therefore the chance that both will die is × or ;
9 3 27
16 5 80
therefore the chance that both will not be dead, that
is at least one will be alive, is 1 –
27
or
53
·
t
(216 – 6) 80 80
⇒ Required probability =
216 4. (A) (a) Total possible outcomes = 6 × 6 × 6 = 216
210 105 (b) Favourable cases are (6, 6, 6), (6, 6, 5), (6, 5, 6),
= = ·
216 108 (6, 4, 6), (5, 6, 6), (4, 6, 6), (6, 5, 5), (5, 5, 6), (5, 6, 5)
Solutions – B ⇒ Total number of favourable cases = 10
⇒ Probability = 10 / 216 = 5 / 108.
1. (A) The bag contains 6 black balls and there are four
ways of following containing white balls : 5. (D) Total number of balls = 8. Let the first drawn
ball is white.
(a) It may contain three white balls.
5 3 4 2 1
(b) It may contain four white balls. So, required probability = × × × = · But here
8 7 6 5 14
(c) It may contain five white balls. we had started with a white ball. When we start with
(d) It may contain six white balls. a black ball, the required
(a) The probability that at the three successive draws 3 5 2 4 1
Probability = × × × = ·
give white ball is : 8 7 6 5 14

CAT Complete Course | 413

Visit : www.EasyEngineering.net
Visit : www.EasyEngineering.net

Since, these two cases are mutually exclusive. ⇒ Required probability


1 1 2 1 5 4
Total probability = + =
14 14 14 7
= · = ( ) 9
10
+5× ( ) × 101
9
10
6. (C) On first drawing, Probability of drawing red = 45927/50000.
8C
8 2
= 20 1 = = 11. (B) Total number of students = Number passing in I
C 1 20 5 paper only + Number passing in II paper only + Num-
On second drawing, Probability of drawing white ber passing in both + Number failed in both
5C
5 ⇒ 500 = (150 – 50) + (350 – 50) + 50 + X
= 19 1 =
C 1 19 ⇒ X = 50
On third drawing, Probability of drawing black ⇒ Required probability
7C
7 = 50/500 = 1/10.
= 18 1 =
C 1 18 12. (B) Total probability
∴ Required probability 15C 5C
= 20 2 + 20 2
2 5 7 7 C2 C2
= × × = ·

ww 5 19 18 171
7. (A) There are 13 cards of spade in a pack of 52 cards.
So, the chance that any of them will cut's a spade is
=
105 10 23
+
190 190 38
= ·

13. (A) The total number of ways in which 3 balls can be

w
13/52 = 1/4 ⇒ Probability that it is not a spade = 1 –

.Ea
1/4 = 3/4. Consider the following mutually exclusive
ways in which A may win : A wins in the first cut,
OR A, B, C have failed and then A wins, OR A, B,
drawn is 9 C 3 and the number of ways of drawing 3
black ball is 5 C 3 ; therefore the chance of drawing 3
5C
black balls = 9 3 =
C3
5×4×3 5
=
9 × 8 × 7 42
· Thus, the odds

syE
C, A, B, C have failed and then A wins and so on
upto infinity ⇒ Respective chance of these events
are = 1/4, [(3/4) × (3/4) × (3/4) × (1/4)], [(3/4) × (3/4)
against the event are 37 to 5.
14. (B) Net cases = 27 + 10 = 37.
× (3/4) × (3/4) × (3/4) × (3/4) × (1/4)] …. To infinity
⇒ A's chance of succeeding = (1/4) + (3/4)3 × (1/4)
+ (3/4) 6 × (1/4) + …. = 1/4[ 1/(1 – 27/64)] = 16/37.
ngi Required probability = (1/37) /(1/27) = 27/37
15. (C) Total possible outcomes

8. (B) Total numbers of ways in which n persons can sit


a round table = (n – 1)! As the two specified indi- nee = 15C 2 ; Favourable outcomes
= 9C
2

⇒ Probability =

rin
9C 15C
viduals are to sit together, we may consider them to 2/ 2 = 36/105 = 12/35.
be a single individuals and then we have (n – 2)! 16. (B) Total possible outcomes = 41. Therefore, the
Different arrangements in which the two specified
individuals can sit together in the same order. Since,
they can permute among themselves, the total pos-

g.n
required probability = 6/41.
17. (A) The sum of two sides in a triangle is greater
sible ways favourable to the events are 2 × (n – 2)!
⇒ Probability of two specific individuals sitting
together = [2 × (n – 2)!]/(n – 1)! = 2/(2 + n – 3). Hence,
the odds against the events are (n – 3) to 2.
than the third side.

e
∴ In following 3 cases, triangle is not formed.
2, 3, 5 t
2, 3, 6
9. (B) Total number of triangles that can be formed
2, 4, 6
= 8 C 3 = 56.
3 3
Total number of triangles with no sides common = ∴ Required probability = 5 = ·
C 3 10
n C – (n – 4)n – n = 16, where n = 8 here. S, required
3
probability = 16/56 = 2/7. 18. (B) The care, none of the parts shall be greater than
1
10. (D) The chance that the vessel is broken = 1/10 ⇒ th of the rod is possible if the rod is divided in n
n
Chance that the vessel is safe = 1 – 1/10 = 9/10. At 1
least 4 will arrive safely if all 5 are safe or if 4 are equal parts. The probability of one part being th of
n
safe and 1 is broken. Probability that all five are safe 1
3 the rod = .
= ( )9
10
; Probability that 4 are safe and 1 is broken n
1
4 ∴ Required probability of n parts being th of the
=5× () 9
10
×
1
10
because the 4 may be safe in 5 n
n

mutually exclusive ways.


1 1 1
rod each = × × × …… =
n n n
1
n
1
n
. ()
414 | CAT Complete Course

Visit : www.EasyEngineering.net
Visit : www.EasyEngineering.net

19. (A) At the first trial 3 balls may be drawn in 13 C3 The chance that A occupies any seat in the first class
ways and 3 white balls may be drawn in 5 C 3 ways. 6
railway compartment is , because there are seven
Therefore, the chance of 3 white balls at first trial 7
5 × 4 13 × 12 × 11 5 persons and the first class seats in the compartment
= ÷ = · When 3 white balls
1×2 1×2×3 143 are 6. Wherever, A has occupied his seat there is
have been drawn and removed, the bag contains 2 only one case favourable to the event that B occupies
white and 8 black balls; therefore at the second trial 3 is seat opposite to A. Thus, the chance of the
balls may be drawn in 10C 3 ways; and 3 black balls compound events that B gains an opposite seat to A
may be drawn in 8 C 3 ways; therefore the chance of 3 6 1 1
is × = ·
black balls at the second trial 7 6 7

8 × 7 × 6 10 × 9 × 8 7 24. (C) Required probability


= ÷ = ·
1 × 2 × 3 1 × 2 × 3 15
So, the chance of the compound event
= (1 – 16) × (1 – 25)
5 3 1
5 7 7 = × = ·
= × = · 6 5 2
143 15 429

ww
25. (D) Let the events be A & B. Now, P(A) = 2/7, P(B)
1
20. (B) In his first throw A's chance is ; in his second = 6/11.
6
5 5 1 ∴ The probability that none of the events (A & B)
throw it is × × , because each player must have

w 6 6 6
failed once before A can have a second throw; in

.Ea
his third throw his chance =
5
6
4

() 1
× because each
6
will occur

= 1–
2
7
× 1–
6
( ) ( )
11
5 5 25

1
() ()
s
player must have failed twice; and so on . Thus,

yE
A's chance is the sum of the infinite series
2 4

= × =
7 11 77

26. (A) Since, Probability of coming head =


·

1
= prob-

ngi
5 5 2
6 
1+ + + … ·
6 6  ability of coming tail

nee
Similarly, B's chance is the sum of the infinite series 1
⇒ P(H) = = P(T).
2
2 4
5 1 
6 6 
, 1+ (56) + (56) + …  Now, A wins if he throws a head in 1st or 3rd or 5th

∴ A's chance is to B's as 6 is to 5. Their respective


or 7th …..
∴ Probability of A wins rin
chances are therefore
6
11
5
and , and their expec-
11
g.n
= P[H or (TTH) or (TTTTH) or (TTTTTTH),……]

e
tations are Rs. 6 and Rs. 5 respectively. = P(H) + P(TTH) + P(TTTTH) + ……..
21. (A) The chance of A, B, C before the starts are
3 2
,
10 10
1 3 1
i.e., , , respectively. A can lose in two
2 10 5
5
10
, = P(H) + P(T) . P(T) . P(H) . P(T) . P(T) . P(T) . P(T)
. P(H) + ……..
1 1 1 1 1 1 1 1 1
= + · · + · · · · + ………
t
ways; either by the winning of B or C. 2 2 2 2 2 2 2 2 2
2 5
As A's chance of winning is 1/2; therefore A's chance
of losing is 1/2. After the accident his chance of
1
= +
2 ( ) ( ) + ……
1
2
+
1
2
winning is 1/3, and hence his chance of losing
1  2 4
=  1 + ( ) + ( ) + …
becomes 2/3. ⇒ his chance of losing is increased in 1 1
the ratio of 4 to 3, also B's and C's chances of 2 2 2 
winning are increased in the same ratio. Thus B's 1 1 1 4 2
3 4 2 = × = × =
chance of winning = × = ; and C's chance of
()
2 1 2 2 4–1 3
10 3 5 1–
2
2 4 4
winning = × = ·
10 3 15 2 2 1
P(A) = , P(B) = 1 – =
22. (A) 3 3 3
23. (C) Let the two specified persons be denoted by A
and B.
⇒ Required Probability = (23 ‚ 13) ·
CAT Complete Course | 415

Visit : www.EasyEngineering.net
Visit : www.EasyEngineering.net

27. (B) Total possible combinations of distribution of m × (m – 1) mn


= +
ticket that no child will have to wait for change are (m + n)(m – 1 + n) (m + n)(m + n – 1)
as follows : m2 – m + mn
=
1. 1 1 1 1 2 2 2 2 2. 1 1 1 2 1 2 2 2 (m + n)(m – 1 + n)
3. 1 1 1 2 2 1 2 2 4. 1 1 1 2 2 2 1 2 m(m – 1 + n) m
= = ·
(m + n)(m – 1 + n) m + n
5. 1 1 2 1 1 2 2 2 6. 1 1 2 2 1 1 2 2
29. (C) Case I : Assume that the box has two white
7. 1 1 2 1 2 1 2 2 8. 1 1 2 1 2 2 1 2 balls. When a white ball is added, it now has 3 white
9. 1 1 2 2 1 2 1 2 10. 1 2 1 2 1 2 1 2 balls ⇒ Probability of drawing a white ball = Prob-
11. 1 2 1 1 1 2 2 2 12. 1 2 1 1 2 1 2 2 ability of drawing one ball × Probability of white ball
13. 1 2 1 1 2 2 1 2 14. 1 2 1 2 1 1 2 2 1 3 1
= × =
3 3 3
Now these 14 combinations can again be arranged in
Case II : Assume that the box has one white and one
4! × 4! Ways.
Black ball.
So, total favourable combinations = 14 × 4! × 4! When a white ball is added the box will then contain
And total possible combinations = 8! 2 white balls and 1 Black ball ⇒ Probability of draw-

ww Required Probability =
14 × 4! × 4! 1
8! 5
·
ing a white ball = Probability of drawing one ball
× Probability of white ball
1 2 2

w
28. (A) Case I : Let the first ball removed be White

.Ea
⇒ White balls left = (m – 1) Black balls left = n
Probability of getting a white ball in the first and
m (m – 1)
= × = ·
3 3 9
Case III : Assume that the box has two Black balls.
When a white ball is added, it now has 1 white ball
and 2 Black balls
second draw = ×

syE
(m + n) (m – 1 + n)
Case II : Let the first ball removed be Black
⇒ Probability of drawing a white = Probability of
drawing one ball × Probability of white ball

ngi
⇒ White balls left = m 1 1 1
× = ⇒ Overall Probabiity
3 3 9
Black ball left = n – 1 1 2 1 2

nee
= + + =
Probability of getting a Black ball in the first and 3 9 9 3
White in second draw [because all these are mutually exclusive cases].
n m 30. (D) There will be 6 × 6 × 6 = 216 outcomes.
= ×
(m + n) (m + n – 1)
⇒ Total Probability rin
The outcomes which are not favourable are (1, 1, 1)

g.n
and (6, 6, 6)
= Sum of these two ⇒ Required probability = [214/216] = [107/108].
●●●

e t

416 | CAT Complete Course

Visit : www.EasyEngineering.net
Visit : www.EasyEngineering.net

8 Co-ordinate Geometry
(Lines in two dimensions)

1. Co-ordinate axes : Two perpendicular lines XOX′ 5. Distance formula : The distance between two
and YOY′ are intersecting at the point O. XOX′ and YOY′ points P (x1 , y1) and Q (x2, y2) is given by
are called co-ordinate axes. The point O is called origin. PQ = 
√ (x2 – x1)2 + (y2 – y1)2.
XOX′ is called x-axis and YOY′ is called y axis.
Note : Distance of a point P (x, y) from the origin
(0, 0) = √

x2 + y2 .

ww
w .Ea 6. Section formula : The co-ordinate of point P,
dividing the line segment joining A (x1, y 1 ) and B (x 2 , y 2 )

syE
2. Co-ordinate of a point : The perpendicular dis-
tance of a point P from x-axis and y-axis is shown by P
(x, y) and also known as the co-ordinate of point P.
internally in the ratio m : n are given by

The length OM or LP is called x-co-ordinate or


abscissa of point P and OL or MP is called y-co-ordinate ngi
or ordinate of point P.
3. Quadrants : The x-axis and y-axis divide the plane
into four parts called quadrants. Regions XOY, YOX′, nee x =
mx 2 + nx1
m+n
my + my1
,y= 2
m+n
X′OY′ and Y′OX are respectively called the first, second,
third and fourth quadrants.
rin
Note : The mid-point P (x, y ) of a line segment
joining the points A (x1, y2) and B (x2 , y2) is given by
4. Signs of the Co-ordinates of Points in Different
Quadrants : x =
2 g.n
x1 + x2 y + y2
,y= 1
2
Quadrant

First quadrant
Second quadrant
x-
co-ordinate
+

y-
co-ordinate
+
+
Point

(+, +)
(–, +)
K : 1 is
kx2 + x1 ky2 + y1
K+1 K+1

e
The co-ordinates of a point dividing the line segment

t
joining the points A (x 1 , y1) and B (x 2 , y2) in the ratio of

( )
Third quadrant – – (–, –) 7. The co-ordinate of the centroid of a triangle whose
Fourth quadrant + – (+, –) vertices are A (x1 , y1) B (x2 , y2) and C (x3 , y3) are

Note : 1. Abscissa is the perpendicular distance of a (x + x3 + x ‚ y + y3 + y )


1 2 3 1 2 3

point from y-axis.


8. Four points will form : (i) a parallelogram, if its
2. Ordinate is the perpendicular distance of a point opposite sides are equal.
from x-axis.
(ii) a rectangle, if its opposite sides are equal and two
3. Abscissa is positive to the right of y-axis and is
diagonals are also equal.
negative to the left of y-axis.
(iii) a rhombus, if its all the four sides are equal.
4. Ordinate is positive above x-axis and negative
below x-axis. (iv) a square, if its all sides are equal and diagonals
5. Abscissa of any point on y-axis is zero. are also equal.
6. Ordinate of any point on x-axis is zero. 9. Three points will form : (i) an equilateral triangle,
7. Co-ordinate of the origin are (0, 0). if its all the three sides are equal.

CAT Complete Course | 417

Visit : www.EasyEngineering.net
Visit : www.EasyEngineering.net

(ii) an isosceles triangle, if its any two sides are 6. Point P is equidistant from A (1, – 8) and B (4, 5) and
equal. its ordinate is twice its abscissa, then its co-ordinates
are :
(iii) a right angled triangle, if the sum of square of its
any two sides is equal to the square of third largest side. (A) (1, 3) (B) (3, 9)
10. Three points A (x1 , y 1 ), B (x2, y 2 ) and C (x3 , y 3 ) (C) (2, 6) (D) (5, 15)
are collinear or a lie on a line if any one of the follow- 7. The vertices of triangle ABC are A (2, 2), B (3, 4)
ing holds : and C (6, 6), then angle ABC is equal to :
(i) AB + BC = AC (A) 45° (B) 60°
(ii) AC + CB = AB
(iii) CA + AB = CB
(C) 120° (D) tan–1
1
3()
(iv) [x1 (y2 – y3) + x2 (y3 – y1) + x3 (y1 – y2)] = 0 8. The equation of the line passing through the points
11. Area of a triangle: Let the vertical of a triangle (1, –1) and (2, 4) is :
be A (x1, y1) B (x2 , y2) and C (x3 , y3). Then, area of ∆ABC (A) 5x – y + 6 = 0 (B) 5x – y + 6 = 0
1
= [x1 (y2 – y3) + x2 (y3 – y1) + x3 (y1 – y2)] (C) 5x – y – 6 = 0 (D) 5x + y – 6 = 0
2

ww
If the value of area calculation is negative then we
take its absolute value.
9. The slope of the line 9x – 5y + 16 = 0, where a is
constant, is :
9 5

w
12. (a) If we have to show that three given points are
collinear using area of a triangle, then the area of
triangle formed by them is 0.
.Ea
i.e.,, [x1 (y2 – y3) + x2 (y3 – y1) + x3 (y1 – y2)] = 0
(A) –

(C)
9
5
5
(B) –

(D)
5
9
9

syE
(b) If we have to find area of a quadrilateral, then we
divide the quadrilateral into appropriate number of
triangles and after finding areas of triangles, add them.
10. The equation of the line passing through (1, – 3) and
parallel to X + 2Y + 5 = 0, is :
(A) 2X – Y – 1 = 0 (B) 2X + Y + 1 = 0
(c) If in question, it is given that the area of a triangle
is p square units and we have to find same condition, then ngi (C) 2X + 2Y – 1 = 0 (D) 2X – Y + 1 = 0
11. If (a, – 2) lies on the line which passes through (2, 7)
we must take two cases as ± p after removing the modulus
sign from the formula of triangle.
Exercise – A
nee and which is parallel to 3x + 5y – 6 = 0, then the
value of P is :

1. The area of quadrilateral with vertices (3, 5), (0, 5),


(A)
1
16
rin (B) 16

g.n
(0, – 6), (3, – 6) is equal to (sq. units) : 2
(C) 8 (D)
(A) 12 (B) 20 9
(C) 33 (D) 50 12. The vertices of a ∆ABC are A (0, 2), B (3, 0) and
2. The points (0, 0), (1, 2), (2, 0) are vertices of a/an :
(A) equilateral triangle.
(B) isosceles triangle.
(A)
7 7
3 3

e
C (4, 0). The orthocenter H of the triangle is :

( ) (B) 1‚( )4
3 t
(C) right triangle. (C) (0, – 6) (D) None of the above
(D) scalene triangle.
13. If A (a, b) lies on 3x + 2y – 12 = 0 and B(b, a) lies on
3. The co-ordinates of a point which divides the join of 2x – 5y + 4 = 0, then the equation of the line AB is :
(7, – 7) and (4, – 5) in the ratio 5 : 4, externally, are :
(A) y + x = 5 (B) y – a = 10
(A) (5, 4) (B) (– 8, 6)
(C) x + y + 5 = 0 (D) x + y – 10 = 0
(C) (– 8, 3) (D) (5, 3)
4. The centroid of the triangle whose vertices are (5, 7), 14. The equation of perpendicular bisector of the line
(8, 9), (2, 11), is : joining A (3, 6) and B (5, 2) is :
(A) (5, 4) (B) (3, 9) (A) x + 2y + 4 = 0 (B) x – 2y + 4 = 0
(C) (5, 9) (D) None of these (C) x + 2y - 4 = 0 (D) x – 2y – 4 = 0
5. If A (0, 0), B (3, 4), C (5, 8) are three vertices of a 15. A triangle ABC is given by A (5, 4), B (2, -7), C (4,
parallelogram ABCD, then the fourth vertex D will 5). The equation of median drawn on BC from A, is :
be :
(A) y + 3x = 19 (B) 5x – 2y = 17
(A) (1, 2) (B) (2, 3)
(C) (2, 4) (D) (4 – 4) (C) 5x + 3y = 17 (D) 3x – 5y = 20

CAT Complete Course | 418

Visit : www.EasyEngineering.net
Visit : www.EasyEngineering.net

16. The equation of the line perpendicular to the line L : 25. The points (2, 4) and (6, – 2) are the opposite vertices
3X + 5Y + 5 = 0 and having Y intercept same as L is : of a rectangle. The other two vertices lie on the line
(A) 5X + 3Y = 17 (B) 5X + 3Y = 3 2y = 3x + k, then the value of k will be :
(C) 5X – 3Y = 3 (D) 5X – 3Y = 13 (A) 10 (B) – 10
(C) 5 (D) – 8
17. The line with X intercept = – 2 and passing through
the intersection of x + 5y – 5 = 0 and x – y + 2 = 0, 26. The extremities of a diagonal of a parallelogram are
will be : the points (– 1, 3) and (3, 2) and (1, 0). If the vertex
is (– 2, 1), then fourth vertex is :
(A) 8x – 5y = 3 (B) 3x + 9y = 7
(A) (– 1, 5) (B) (2, 4)
(C) 7x – 5y = 17 (D) 7x + 17y = 14
(C) (1, 5) (D) (– 1, – 5)
18. The distance between the lines 2x + 3y – 6 = 0 and
27. The equation of the straight line passing through the
24x + 36y + 84 = 0, will be :
points (– 1, 2) and perpendicular to the line y = – x is :
14
(A) (B) √13 (A) y – x + 5 (B) y = x + 3
13
√ (C) 2y = x + 7 (D) y = 5x + 5
49

ww
(C)
13

(D) None of these

19. The point whose abscissa is equal to its ordinate and


28. The angle between the lines y = 
3 is :
(A) 45° (B) 60°
√ 3 + 9 and √ 3 + x =

w
which is equidistant from the point (3, 5) and (1, 3) is :
(A) (2, 1)
(C) (6, 3)
.Ea
(B) (4, 2)
(D) (8, 4)
(C) 90° (D) 120°
29. A line passes through the point (4, 6) and cuts off
intercepts from the co-ordinates axes such that their

syE
20. The co-ordinates of the points A, B, C, D, are (1, 3),
(2, 4), (6, 8) and (3, b) respectively. If the line AC
and BD are perpendicular, then a = ?
sum is 20. The equation of the line is :
(A) 3x – 2y = 24
(C) x + y = 10
(B) x - y = 20
(D) 2x + 3y = 15
(A) 1

(C) 3
(B) – 2

(D)
4 ngi 30. Point of intersection of the diagonals of square is at
origin and co-ordinate axes are drawn along the
5
21. If the middle points of the sides of a triangle be
(3, – 2), (– 3, 4) and (7, 9), then the centroid of the
nee diagonals. If the side is of length a, then which one is
not the vertex of square ?

triangle is :

( )
5
( )
7 6
(A) (a
√ 2, 0)
(C) (a/√
 2, 0) rin (B) (0, a/√
(D) (– a/√
 2)
 2, 0)
(A)

(C)
3

( )
7
‚4

‚4
(B)

(D)

5 5

‚( )
7 11 g.n
Exercise – B
3 3 3
22. The mid-points of sides of a triangle are (0, – 4), (1, 4)
and (3, 2). Then the co-ordinates of its vertices are : (A) 8
(C) 16
e
1. The area of quadrilateral with vertices (2, 4), (0, 4),
(0, – 4), (2, – 4) is equal to (sq. units) :
(B) 12
(D) 32
t
(A) (3, 6), (– 3, – 14), (5, 6)
(B) (2, 3), (0, 1), (4, 5) 2. The points (3, 0), (– 3, 0), (0, – 3 √
 3) are vertices of
(C) (2, 3), (– 2, 4), (5, 9) a/an :
(D) (2, -3) (– 2, – 5) (4, 7) (A) equilateral triangle (B) isosceles triangle
(C) right triangle (D) scalene triangle
23. If the points (2, 0), (0, 5) and (a, b) are collinear, then :
5 9 7 15 3. The co-ordinates of a point which divides the join of
(A) a = , b = (B) a = , b = (5, – 5) and (2, – 3) in the ratio 4 : 3, externally, are :
2 5 3 2
15 (A) (3, 4) (B) (– 7, 3)
(C) for b = (D) none of the above
2 (C) (– 7, 9) (D) (8, 3)
24. Area of a triangle whose vertices are (a sin θ, – b 4. The centroid of the triangle whose vertices are
cos θ), (– a sin θ, b cos θ), (a sin θ, – b cos θ) is : (3, 10), (7, 7), (– 2, 1), is :
(A) 4ab sin θ cos θ (B) ab sin 2θ cos θ (A) (8/3, 6) (B) (6, 8/3)
(C) 3ab sin θ cos θ (D) 4ab sin 2θ (C) (– 4, – 7/3) (D) None of these

CAT Complete Course | 419

Visit : www.EasyEngineering.net
Visit : www.EasyEngineering.net

5. If A (0, 2), B (– 2, – 2), C (1, – 1) are three vertices 16. The equation of the line perpendicular to the line L1 :
of a parallelogram ABCD, then the fourth vertex D 2X – 5Y = 0 and having X intercept same as L1 is :
will be : (A) 5X – 2Y + 3 = 0 (B) 2X – 5Y + 9 = 0
(A) (1, 2) (B) (2, 3) (C) 5X – 2Y + 9 = 0 (D) 10X + 4Y - 5 = 0
(C) (3, 3) (D) (2, -2) 17. The line with Y intercept = – 1 and passing through
6. A point is equidistant from A (3, 1) and B(5, 3) and the intersection of 7X + 2Y = 20 and 2X – 3Y + 12 = 0,
its abscissa is twice its ordinate, then its co-ordinates will be :
are : (A) 149X – 137Y + 9 = 0
(A) (2, 1) (B) (1, 2) (B) 153Y – 137X + 9 = 0
(C) (4, 2) (D) (2, 4) (C) 4X – 5Y + 8 = 0
7. The vertices of triangle ABC are A (4, 4), B (6, 3) (D) 149X – 36Y – 36 = 0
and C (2, -1) ; then angle ABC is equal to : 18. The distance between the lines X + 2Y – 3 = 0 and
(A) 45° (B) 90° 11X + 22Y + 88 = 0, will be :
(C) 60° (D) None of these (A) 7/5 (B) 49/√
5

ww
8. The equation of the line passing through the points
(5, 3) and (3, 5) is :
(A) X – Y + 8 = 0 (B) X – Y + 8 = 0
(C) 7/√
5 (D) None of these
19. The point whose abscissa is equal to its ordinate and

w
which is equidistant from the point (1, 0) and (0, 3) is :
(C) X – Y – 8 = 0 (D) X + Y – 8 = 0
(A) (1, 1) (B) (2, 2)

constant, is : .Ea
9. The slope of the line a2 X – aY + 1 = 0, where a is (C) (3, 3) (D) (4, 4)
20. The co-ordinates of the points A, B, C, D, are (2, a),
(A) – a2
(C) a
syE
(B) – a
(D) None of these
(3, 5), (3, 4) and (0, 6) respectively. If the line AC
and BD are perpendicular, then a = ?
(A) 7 (B) 1

ngi
10. The equation of the line passing through (2, – 4) and
parallel to X – 2Y – 5 = 0, is : (C) – 1 (D) – 7
(A) 2X + Y + 3 = 0 (B) X – 2Y – 10 = 0 21. If the middle points of the sides of a triangle be (– 2,
(C) X – 2Y + 8 = 0 (D) X – 2Y + 13 = 0
11. If (4, P) lies on the line which passes through (2, 3) nee 3), (4, – 3) and (4, 5), then the centroid of the triangle
is :
and which is parallel to 4X + 3Y – 6 = 0, then the
value of P is :
(A) (5/3, 2)
(C) (2, 5/3)
rin (B) (5/6, 1)
(D) (1, 5/6)
(A) 3
(C) 1/3
(B) 1
(D) 1/2
g.n
22. The mid-points of sides of a triangle are (2, 1), (– 1,
– 3) and (4, 5). Then the co-ordinates of its vertices
are :
12. The vertices of a ∆ABC are A (0, a), B (b, 0) and C
(c, 0). The orthocenter H of the triangle is :
(A) (b + c)/3, a/3
(C) (0, – bc/a)
(B) (b + c)/c, (a2 + bc)/2a
(D) None of these
(C) (1, 1), (2, 3), (– 5, 8)
(D) None of the above
e
(A) (7, 9), (– 3, – 7), (1, 1)
(B) (– 3, – 7), (1, 1), (2, 3)
t
13. If R (r, s) lies on 6X – Y = 1 and S (s, r) lies on 2X – 23. If the points (a, 0), (0, b) and (1, 1) are collinear,
5Y = 5, then the equation of the line RS is : then :
(A) Y + X = 6 (B) X + Y + 2 = 0 (A) 1/a2 + 1/b2 = 1 (B) 1/a2 - 1/b2 = 1
(C) X + Y + 1 = 0 (D) None of these (C) 1/a + 1/b = 1 (D) 1/a – 1/b = 1
14. The equation of perpendicular bisector of the line 24. Area of a triangle whose vertices are (a cos θ, – b
joining A (1, 2) and B (7, 4) is : sin θ), (– a sin θ, – b cos θ), (– a cos θ, – b sin θ) is :
(A) 3X + Y = 0 (B) – 13X + 13Y – 11 = 0 (A) a cos θ sin θ (B) ab sin θ cos θ
(C) 3X + Y – 15 = 0 (D) None of these (C) 1/2 (D) ab
15. A triangle ABC is given by A (2, 5), B(– 1, – 1), 25. The points (1, 3) and (5, 1) are the opposite vertices
C (3, 1). The equation of median drawn on BC from of a rectangle. The other two vertices lie on the line y
A, is : = 2x + c, then the value of c will be :
(A) 2X + Y – 9 = 0 (B) 5X – Y – 5 = 0 (A) 4 (B) – 4
(C) 3X + Y = – 9 (D) None of these (C) 2 (D) – 2

CAT Complete Course | 420

Visit : www.EasyEngineering.net
Visit : www.EasyEngineering.net

26. The extremities of a diagonal of a parallelogram are 3. (C) (x 1 , y1) (x2, y2) in m : n (7, – 7), (4, – 5), 5 : 4
the points (3, – 4) and (3, – 4) and (– 6, 5). If the m × x2 – m × x1
vertex is (– 2, 1), then fourth vertex is : x =
m–n
(A) (1, 0) (B) (– 1, 0) 5 × 4 – 4 × 7 20 – 28
= =
(C) (1, 1) (D) None of these 5–4 1
27. The equation of the straight line passing through the = –8
points (3, 2) and perpendicular to the line y = x is : m × y2 – m × y1
y =
(A) x – y = 5 (B) x + y = 5 m–n
5 ×–5–4×7
(C) x + y = 5 (D) None of these =
5–4
28. The angle between the lines y = (2 –  3) X + 5 and
√ 28 – 25
= =3
y = (2 + √
 3) X – 7 is : 1
(A) 30° (B) 60° (x, y) = (– 8, 3)
(C) 45° (D) tan-1 3 4. (A) Centroid (x, y), where x & y are given by
5+8+2
29. A line passes through the point (3, 4) and cuts off x = =5

ww
intercepts from the co-ordinates axes such that their
sum is 14. The equation of the line is :
(A) 4x – 3y = 24 (B) 4x + 3y = 24
y =
3
7 + 9 + 11
3
=9

w
(C) 3x – 4y = 24 (D) 3x + 4y = 24

.Ea
30. Point of intersection of the diagonals of square is at
origin and co-ordinate axes are drawn along the
Centroid = (5, 9)

5. (C) As diagonals bisect in a parallelogram, the mid


point of AC will be the same as mid point of BD.

not the vertex of square ?


(A) (a
√ 2, 0)
syE
diagonals. If the side is of length a, then which one is

(B) (0, a/√


 2)
Let D = (X, Y)
Then we have mid point of AC

( )
ngi
0+5 0+8
= ‚
(C) (a/√
 2, 0) (D) (– a/√
 2, 0) 2 2
5
( )
nee
Solutions – A = ‚4
2
1. (C) Area of quadrilateral ABCD = Area of ∆ABD x+3 5
⇒ = ⇒x=2
+ ∆BCD 2 2
A (3, 5), B (0, 5), D (3, – 6),
1
C (0, – 6)
Area ∆ABD = [x1(y2 – y3) + x2(y3 – y1)

y+4
2 rin
= 4 ⇒y=4
2
+ x 3 (y1 – y2)]
So, D = (2, 4)
g.n
6. (C) Let co-ordinate of P is (x, y).

1
1
= [3(5 + 6) + 0( ) + 3(5 – 5)]
2

33
×˚3 × 11 = sq. unit.
=
As given


y = 3x.
(x – 1)2 + (3x + 8)2 = √ e

(x – 4)2 + (3x – 5)2
⇒ x2 – 2x + 1 + 9x2 + 64 – 48x t
2 2 = x2 + 16 – 8x + 9x2 + 25 – 30x
1 ⇒ – 50x + 65 = – 39x + 41
Area of ∆BCD = [0( ) + 0( ) + 3(5 + 6)] ⇒ 65 – 41 = (50 – 38)x
2
11 × 3 33 ⇒ 24 = 12x
= = sq. unit. ⇒ x = 2, y = 6
2 2
7. (D) ∠ABC is the angle between the lines AB & BC.
Area of ABCD =
2 (
33 33
+
2 )
sq. units. = 33. y –y
Now slope of line AB(m1) = 2 1 =
4–2
x2 – x1 3 – 2
2. (A) Here A (0, 0),B (1, 2), C (2, 0)
= 2
AB = √
(1 – 0) 2 + (2 – 0)2= 
√
1 + 4 =
√5 6–4 2
& slope of line BC (m 2 ) = = =1
BC = √
(2 – 1) 2 + (0 – 2)2 = 
√1 + 4 = √ 5 5–3 2
m1 – m2
CA = √
22 + 0 2 = 2 tan θ =
1 + m1 m2
AB = BC = CA 2–1 1
= = ·
So, ABC is isosceles triangle. 1+2×1 3

CAT Complete Course | 421

Visit : www.EasyEngineering.net
Visit : www.EasyEngineering.net

y2 – y1 Slope of BC’s altitude


8. (D) y – y1 = (x – x1 )
x2 – x1 1
= = 0]
Here (x1 , y1) = (1, – 1) ∞
(x2 , y2) = (2, 4) Equation of BC’s altitude
4 – (– 1) 1
⇒ y – (– 1) = (x – 1) = y – 2 = (x – 0)
0
2–1
5 ⇒ x = 0 …(ii)
⇒ y + 1 = (x –1)
1 From equations (i) & (ii)
⇒ y + 1 = 5x – 5 x = 0
⇒ 5x – y = 6 y = –6
coefficient of X Hence orthocenter (0, – 6)
9. (C) Slope = –
coefficient of Y 13. (A) (a, b) lies on 3x + 2y = 12
9 9
= – = · ⇒ 3a + 2b = 12 …(i)
–5 5
(b, a) lies on 2x – 5x + 4 = 0
10. (B) Slope of given line(m1)
⇒ 2b – 5a + 4 = 0 …(ii)

ww =

m1 = –
coefficient of X
coefficient of Y
1
From equations (i) & (ii)
3a + 2b = 12


w .Ea
m1 × m2 = – 1
1
2
Let slope of perpendicular line = m 2
– 5a + 2b = – 4
8a = 16
a = 2



– × m2 = – 1
2
syE
m2 = 2
So, A(2, 3), B(3, 2)
Equation of line AB
b = 3

ngi
So, equation of line that passes through (1, 3) and
2–3
having slope 2 is y–3 = (x – 2)
3–2
y – y1 = slope (x – x1 )

nee
y – 3 = – (x – 2)
⇒ y – 3 = 2(x – 1)
⇒ x+y–5 = 0
⇒ 2x – 2 = y – 3 ⇒ x+y = 5

11.
⇒ 2x – y + 1 = 0
(C) Equation of line parallel to 3x + 54 – 6 = 0 is
14. (C)
rin
The slope AB =
2–6 –4
5–3 2
= =–2

3x + 5y + λ = 0
it is given it passes through (1, 7)
…..(i)
g.n
Slope of line perpendicular to AB

=
–1 1
=
⇒ 3×1+5×7+λ = 0


3 + 35 + λ = 0
λ = – 38
e =
–2 2
and this passes through mid point of AB i.e.,

(
3+5 6+2
2

2 ) t
So, equation of line is 3x + 5y – 38 = 0
= (4, 4)
Since (a, – 2) lines on this line.
⇒ Equation of perpendicular bisector
⇒ 3a – 10 – 38 1
⇒ 3a = 0 ⇒ a = 16 ⇒ y – 4 = (x – 4)
2
12. (C) Slope of AB =
0–2
=–
2 ⇒ 2y – 8 = x – 4
3–0 3 ⇒ x – 2y + 4 = 0
1 3 15. (B)
Slope of AB’s altitude = =
(2/3) 2
Median will be the line through A & mid-point of
Equation of AB's altitude
3
= (y – 0) = (x – 4)
BC (2+4 –7+5
2

2 )
2 i.e, (3, –1). Hence equation through (5, 4) & (3, – 1)
= 2y = 3x – 12 …(i) will be
0–0 0 –1–4
Slope of BC = = =0 y–4 = (x - 5)
4–3 1 3–5

CAT Complete Course | 422

Visit : www.EasyEngineering.net
Visit : www.EasyEngineering.net

⇒ 2(y – 4) + 5(x – 5) 20. (B) Slope AC =


8–3
=1
⇒ 2y – 8 = 5x – 25 6–1
b–4
⇒ 5x – 2y = 17 Slope BD = =b–4
3–2
16. (C) 5Y = – 3X – 5 1 × (b – 4) = – 1
3 5 ⇒ b = –1+4=3
Y = – X–
5 5 21. (C)
1 5
So slope of L 1 is = =
(– 3/5) 3
Put x = 0 in L. Y = –1
Now equation of L1 is = Y – (– 1)
5
= (X – 0)
3
= 3(Y + 1) = 5X
= 5X – 3Y = 3
17. (D) Equation of line which passes through the inter

ww
section of x + 5y – 5 = 0 and x – y + 2 = 0 is
x + 5y – 5 + λ (x – y + 2) = 0
(a + λ) x + (5 – λ)y + (– 5 + 2λ) = 0
x1 + x2
2
x1 + x2 =
= 3,
6
Put


w .Ea
y = 0
(a + λ) x = 2λ - 5
x =
2λ – 5

y1 + y2
2
y1 + y2 =
x2 + x3
= –2
–4 …(i)

⇒ syE = –2
1+λ

2λ – 5 = – 2 – 2λ
given

2
x2 + x3 =
= –3
–6 …(ii)

ngi
y2 + y3
⇒ 4λ = – 2 + 5 ⇒ = 4
2
⇒ 4λ = 3 y2 + y3 = 8

⇒ Equation is
λ =
3
4
nee and

x1 + x3
2
=
x1 + x3 =
7
14
3
(a + 5y - 5) + (x – y + 2) = 0
4
and
rin
y2 + y3 =
From (i) + (ii) + (iii)
18 …(iii)

4x + 20y – 20 + 3x - 3y + 6 = 0
7x + 17x – 14 = 0 g.n
x1 + x2 + x2 + x3 + x3 + x1
=6 – 6 + 14

18. (B)

Here
Distance =
7x + 17y = 14
C1 – C2

√ (a2 + b2 )
C 1 = 6, C 2 = – 7


and
2(x1 + x2 + x3) =
x1 + x2 + x3 =
2(y1 + y2 + y3) =
y1 + y2 + y3 =
14
7
e
– 4 + 8 + 18
22
= 11
t
2
6 – (– 7) 13 x + x2 + x3 y1 + y2 + y3
= = So, Centroid is = 1 ,

√ (4 + 9) √ 13 3 3
= √
19. (B) Let the point is (x, y)
13 = ‚( )
7 11
3 3
22. (D)
Given that x = 2y ⇒ points (2y, y)
and 
√ (2y – 3)2 + (y – 5)2 = √
(2y – 1)2 + (y – 3)2
4y + 9 – 12y + y + 25 – 10y = 4y2 – 4y + 1 + y2
2 2

+ 9 – 6y
⇒ – 22y + 34 = – 10y + 10
⇒ 34 – 10 = 12y
⇒ y = 2.
So, abscise = 4
So, point is (4, 2).

CAT Complete Course | 423

Visit : www.EasyEngineering.net
Visit : www.EasyEngineering.net

x1 + x2 = 0 …(i) ⇒ x+1 = 2 ⇒ x=1


x2 + x3 = 2 …(ii) y+0 3+2
⇒ =
x3 + x1 = 6 …(iii) 2 2
y1 + y2 = –8 …(iv) ⇒ y = 5
y2 + y3 = 2 …(v) ⇒ = (1, 5)
27. (B) Slope of y = – x is
y3 + y1 = 4 …(vi)
m = -1
From Eqns. (i) & (ii) Equation of line with slope 1 and passing through
x1 + x3 = –2 point (– 1, 2) is
x1 + x3 = 6 y – 2 = 1 (x + 1)
⇒ y–2 = x+1
2x1 = 4 ⇒ x–y+3 = 0
x1 = 2
⇒ x2 = –2 28. (C) m1 = √ 3, m2 = -– 1
3

From Eqns. (iv) & (v)
y1 – y3 = – 10  3 –  1 

y1 + y3 = 4 m – m2 √
 3 2
tan θ = 1 =∞

ww 2y1 = – 6
y1 = – 3
1 + m1 · m2
Short cut : here m1 × m2
=
1–1
=

 3 × – 1 = – 1.
3 ×0

w
(4, 7)
y3 = 7 & y2 = – 5
⇒ Co-ordinate of vertices are (2, – 3), (– 2, – 5) and

.Ea
23. (C) For three points to be collinear
29. (B) Here

a + b = 20
= √

a = 20 – b
3


2 0 1
1
2
a b 1
syE
 0 5 1 = 0
Equation of line in intercept’s form is

x
x y
+
a b
y
= 1

2 – 5 0
⇒ 0 5 – b 0 R1 → R1 → R2 → R2 → R2 → R3
a b 1 ngi ⇒
20 – b b
It passes through (4, 6)
+ = 1



Expand along C3
+1 (2 × 5 – b) – (–5) = 0 nee ⇒

4
20 – b b
6
+ = 1
4b + 120 – 6b = (20 – b)b


10 – 2b + 5 = 0
b =
15 ⇒
rin
⇒ – 2b + 120 – 20b + b2 = 0
b2 – 22b + 120 = 0
⇒ b - 12b – 10b + 120 = 0

g.n
2 2
24. (B) Area ∆ whose co-ordinates are (x1, y 1 ), (x 2, y 2 ) ⇒ b(b – 12) – 10(b – 12) = 0
and (x3 , y3) are ⇒ (b – 12) (b – 10) = 0


1
= [x1 (y2 – y3) + x2(y3 -–y1) + x3 (y1 – y2)
2
1
= [a sin θ (2b cos θ) – a sin θ
2

So,
Equation of line is

x y
+
e b = 12 or 10
a = 8 or 10

= 1
t
(– 2b cos θ) + a sin θ(0)] 8 12
1 ⇒
⇒ = [4ab sin θ cos θ] 12x + 8y = 96
2 or 3x + 2y = 24
⇒ = 2ab sin θ cos θ x y
⇒ = ab sin θ. or it can be + = 1
10 10
25. (B) We know that the mid-point of diagonals lies on ⇒ x + y = 10
line 2y = 3x + k. 30. (A) Obviously from right angled triangle BOA
Here mid-point is (4, 1) hence 2 × 1 = 3 × 4 + k
OA = OB = a/ √2
⇒ k = – 10.
26. (C) Let A (– 1, 3) and C (3, 2) be the ends of Hence the vertex (a/√ 2, 0) is not the vertex of
diagonals of parallelogram ABCD. Let B (1, 0) and square.
D be (x, y). Then mid-points of diagonals AC and Solutions – B
BD coincide. 1. (C) Let A, B, C, D be the four vertices. Then Area of
So, quadrilateral = Area of two triangles ABD and BCD.
x+1 –1+3 Now Area of triangle ABD with A (2, 4), B (0, 4), D
⇒ =
2 2 (0, – 4)

CAT Complete Course | 424

Visit : www.EasyEngineering.net
Visit : www.EasyEngineering.net

1 8. (D) Direct formula. The equation will be


= [x (y – y ) + x2 (y3 – y1) + x3 (y1 – y2)]
2 1 2 3 (5 – 3)
1 Y–3 = × (X – 5)
= [2 (4 + 4) + 0 (– 4 – 4) + 2 (4 – 4)] (3 – 5)
2 ⇒ Y – 3 = (X – 5)
1
= × 16 = 8 square units. or X+Y–8 = 0
2
9. (C) Slope = – coefficient of X/coefficient of Y
Area of Triangle BCD with B (0, 4), C (0, – 4), D(2,
– 4) will be = – a2 /– a = a.
1 10. (B) Line parallel to X – 2Y – 5 = 0 will be X – 2Y +
= [0 (– 4 + 4) + 0 (– 4 – 4) + 2 (4 + 4)] K = 0 Put the point (2, -4) in this equation we have 2
2
1 + 8 + K = 0 or K = -10 or line is X – 2Y – 10 = 0.
= × 16 = 8 square units.
2 Short-cut : Substitute (2, -4) in the options. Only (b)
⇒ Total Area satisfies.
= 8 + 8 = 16 square units. 11. (C) Line parallel to 4X + 3Y – 6 = 0 will be 4X + 3Y
2. (A) Find the three length separately +K=0
AB = 6, BC = √
32 + (3
√ 3)2 = 6 Put (2, 3)
⇒ 8 + 9 + K = 0 or K = – 17.

√ 32 + (3
√ 3)2 = 6.
ww AC =
Hence the point are the vertices of equilateral
triangle.
Hence the required line is 4X + 3Y – 17 = 0
Now (4, P) lies on this line so, 4 × 4 + 3 × P – 17 = 0

w
3. (B) Let the ratio be 4 : 3 or 4/3 : 1. Or 3P = 1 or P = 1/3.
Now X = (4/3 × 2 – 5)/(4/3 – 1) 12. (C) We have the triangle as in the figure.

.Ea= (8/3 – 5)/(1/3)


= (– 7/3)/(1/3) = – 7
We have slope of BC = 0
Hence slope of BC's altitude = ∞

Hence (– 7, 3). syE


Y = [(4/3X – 3]/(4/3 – 1)
= 1/(1/3) = 3. ⇒
Equation of BC's altitude = (Y – a) = m(X)
(Y – a) = 1/0 × X
X = 0 … (a)
4.

5.
(A) Centroid = (3 + 7 – 2), (10 + 7 + 1)/3
= (8/3, 6).
(C) As diagonals bisect in a parallelogram, the mid ngi Now slope of AB = (a – 0)/(0 – b)
= – a/b and
point of AC will be the same as mid point of BD. Let
D = (X, Y) Then we have mid point of AC = (1/2, nee Slope of AB's altitude = b/a
⇒ Equation of AB's altitude = (Y – 0)

rin
1/2) & mid point of BD = [(– 2 + X)/2, (– 2 + Y)/2] = b/a(X – c)
Equating (– 2 + X)/2 = 1/2 or -2 + X = 1 or X = 3 aY – bX + bc = 0 …(b)
Similarly Y = 3.

6.
Hence (3, 3).
(C) Let the point be P (2X, X).
13. (A) We have g.n
Solve (a) & (b) to get X = 0
Y = – bc/a.
6r – s = 1 … (1)
AP = √
BP = √

(3 – 2X)2 + (1 + X)2 ,

(5 – 2X)2 + (3 – X) 2
AP = BP (3 – 2X) 2 + (1 + X)2
= (5 – 2X)2 + (3 – X) 2
e
2s – 5r = 5

t
… (2)
Solving r = 1, s = 5. Hence equation of line through
(1, 5), (5, 1) will be Y – 5 = (1 – 5)/(5 – 1) × (X – 1)
⇒ Y–5 = 1–X
10 – 14X + 5X2 = 34 – 26X + 5X2
2X = 24 or Y + X = 6.
⇒ x = 2, 2x = 4 14. (C) The slope of given line = (4 – 2) /( 7 – 1) = 2/6 =
P(2X, X) = P(4, 2). 1/3, hence slope of line perpendicular to this will be
7. (D) Angle ABC is the angle between the lines AB & – 3 & it passes through mid point of AB (1 + 7)/2, (2
BC. + 4)/2, = (4, 3).
Now slope of line AB = m1= (3 – 4)/(6 – 4) = – 1/2
Hence equation is Y – 3 = – 3(X – 4)
and slope of line BC = m2 = (– 1 – 3)/(2 – 6)
= – 4/- 4 = 1 or Y – 3 = – 3X + 12
1 or Y + 3X = 15.
– –1
m 1 – m2 2 15. (B) Median will be the line through A & mid-point
Now tan θ = =
1 + m1 m2
1+ – ( )
1
2
(1) of BC ( –1+3 –1+1
2

2 )
– 3/2 i.e, (1, 0) . Hence equation through (2, 5) & (1, 0)
= = – 3.
1–
1 will be Y – 5 = (0 – 5)/(1 – 2) × (X – 2) or 5X – Y –
2 5 = 0.

CAT Complete Course | 425

Visit : www.EasyEngineering.net
Visit : www.EasyEngineering.net

16. (D) Let the required line be L. Similarly y1, y2, y3 can be found
Now 2X – 5Y – 1 = 0 or Y = 2/5 X – 1/5. Y1 + Y2 = 2, Y2 + Y3 = – 6, Y3 + 4 1 = 10
Slope of L1 = 2/5, ∴ Y1 = 9, Y2 = – 7, Y3 = 1.
Slope of L = – 5/2, Put Y = 0 into L 1 . a 0 1
X = 1/2 (X intercept of L = X intercept of L1 = 1/2) 23. (C) 1/2 0 b 1 = 0
Now equation of L is Y – 0 1 1 1
= – 5/2 (X – 1/2) ⇒ a (b – 1) + 0 + 1 (– b) = 0
or 10X + 4Y – 5 = 0 ⇒ ab – a – b = 0
17. (D) The required line ⇒ 1/a + 1/b = 1
= 7X + 2Y – 20 + K (2X – 3Y + 12) – a cos θ – b sin θ 1

= 0 …(1) 24. (D) Area = 1/2– a sin θ – b cos θ 1
Or (7 + 2K) X + (2 – 3K)Y – 20 + 12 K = 0 – a cos θ – b sin θ 1

Now Y intercept is obtained by putting X = 0 in the – cos θ – sin θ 1
above equation. = 1/2 (a × b) – sin θ – cos θ 1
or Y = (20 – 12K) / (2 – 3K) = -1 – cos θ – sin θ 1

ww
or 20 – 12 K = 3K – 2 or 15K = 22 or K = 22/15.
Putting K into (1) we have the line as 149X – 36Y –
36 = 0.
= ab/2 [cos θ ( cos θ + sin θ) – sin θ
(–sin θ + cos θ) + 1 ( sin2 θ + cos2 θ)
= ab/2 (1 + 1) = ab.

w
18. (D) We have the lines as

Now the distance =


.Ea
X + 2Y – 3 = 0 & X +2Y + 8 = 0
C1 – C2


+
(a + b ) √
2 2
8+3

12 + 2 2
25. (B) We know that the mid-point of diagonals lies on
line y = 2x + c.
Here mid-point is (3, 2), hence c = – 4.

=
syE
11
5

·
26. (B) Let A (3, – 4) and C (– 6, 5) be the ends of
diagonals of parallelogram ABCD. Let B (– 2, 1) and
D be (X, Y). Then mid-points of diagonals AC and
19. (B) Let the point be (X, X), so according to the
condition
(X – 1)2 + (X – 0) 2 = (X – 0)2 + (X – 3) 2 ngi BD coincide.
So (X – 2)/2 = (– 6 + 3)/2 and (Y + 1)/2 =
5–4
2


– 2X + 1 = – 6X + 9
X = 2. nee ⇒ X = –1, Y = 0
⇒ Co-ordinates of D are (–1, 0).

rin
Hence the point is (2, 2). 27. (B) Let the required equation be y = – x + c which is
perpendicular to y = x and passes through (3, 2). So
20. (B) Slope AC = (4 – a)/(3 – 2) = 4 – a
2 = –3+c ⇒c=5
Slope BD = (6 – 5)/ (0 – 3) = – 1/3
For perpendicular (4 – a)(– 1/3) = – 1 ⇒ a = 1.
θ = tan–1 
g.n
Hence required equation is x + y = 5.
2 – √
3–2–√  3
21. (C) Let the vertices of the triangle are A (X1, Y1),
B (X2 , Y2) and C (X3, Y3), then
X1 + X 2 = 8
Y1 + Y2 = 10
…(i)
…(ii)
28. (B)

= tan–1 (– √e
 1+4–3 
 3) = 120°
Considering smaller angle θ = 60°. t
X2 + X 1 = – 4 …(iii) 29. (B) Given a + b = 14 ⇒ a = 14 – b.
Hence the equation of straight line is
Y2 + Y 3 = 6 …(iv)
X/(14 – b) + y/b = 1.
X3 + X 1 = 8 …(v)
Also it passes through (3, 4) ⇒ 3/(14 – b) + 4/b = 1
Y3 + Y 1 = – 6 …(vi) ⇒ b = 8 or 7
On solving these equations, we get X1 = 10, X2 = – 2, Therefore equations are
X3 = – 2Y1 = – 1, Y2 = 11, Y3 = – 5. 4x + 3y = 24 and x + y = 7.
Hence the centroid is (2, 5/3). Shortcut : This equation can be checked with the
Shortcut : As we know that the centroid of the options as the line 4x + 3y = 24 passes through (3, 4)
triangle ABC and that of the triangle formed by and also cuts the intercepts from the axes whose sum
joining the middle points of the sides of triangle is 14.
ABC is same. [(4 + 4 – 2) /3, (5 – 3 + 3)] ≡ (2, 5/3). 30. (a) Obviously from right angled triangle
22. (A) (X 1 + X2 )/2 = 2 (X2 + X3 )/2 = – 1, BOA, OA = OB = a/
√2
(X 3 + X1 )/2 = 4 Hence the vertex (a/√
 2, 0) is not the vertex of square.
⇒ X1 = 7, X2 = – 3, X3 = 1. ●●●

CAT Complete Course | 426

Visit : www.EasyEngineering.net
Visit : www.EasyEngineering.net

9 Puzzles, Verbal Problems and Equations


We use the term Puzzle, verbal, or story, problems Since components A and B are in the ratio 3: 5 by
for those which traditionally are called problems leading weight in the first mixture, each gram of the first mixture
to equations. Examination problems nowadays frequently contains 3/8 gram of component A and 5/8 gram of
require the use not only of equations but of inequalities as component B. Similarly, 1 gram of the second mixture
well, and sometimes of other conditions which are not contains 1/3 gram of B and 2/3 gram of C, and 1 gram of
written in the form of equations and inequalities. The the third mixture contains 2/5 gram of A and 3/5 gram of
most common feature of problems of this type is that the C.
condition is given in verbal form without any formulas or If we take x grams of the first mixture, y grams of the

ww
even without any literal designations of the unknowns.
The habit of most aspirants to regard any verbal problem
as a problem which involves the setting up of equations is
second, and z grams of the third and mix them, we get
(x + y + z) grams of the new mixture, which will contain

( 5 1
) ( 2 3
)
w
sometimes a drawback in that they are psychologically

.Ea
unprepared when it turns out that the equations alone are
not sufficient to obtain a solution.
The ordinary type of problem in which all the
(3/8x+ 2/5z) grams of A,
8
x + y of B and
3 3
y+ z
grams of C. We have to take the first, second and third
mixtures in quantities such that in the new mixture the
components A, Band C are in the weight ratio 3: 5 : 2,
5

syE
conditions can be written down in the form of equations
do not, as a rule, cause any particular difficulty, though
even in these problems there are certain items that,
that is, in one gram of the new mixture there must be 3/10
gram of A, 5/10 gram of B and 2/10 gram of C. But then
x + y + z grams of the new mixture will contain
occasionally are troublesome. Usually the real difficulty
of more complicated problems lies in their unusual ngi 3(x + y + z)
10
grams of A,
5(x + y + z)
10
grams of B and

nee
aspect, the necessity to reason and not merely to solve 2(x + y + z)
grams of C. Equating the different expressions
certain systems of equations or inequalities. 10
for one and the same quantity of components A, B, C, we
It frequently happens that simple arguments without
any setting up of equations and inequalities, even if that is
possible, proves to be faster and simpler. What is more,
3 2
x+ z = rin
get the following system of equations:
3
(x + y + z)

g.n
8 5 10
an occasional problem is more amenable to simple,
5 1 5
everyday reasoning than to ordinary mathematical x+ y = (x + y + z) …(1)
8 3 10

e
techniques. Incidentally, the "commonsense" solution is
2 3 2
not always rigorous and so one has to supplement it with
rigorous mathematical arguments.
Let us begin with some mixture problems. The
aspirant frequently finds difficulty in setting up the
3
y+ z =
5 10
(x + y + z)

Note from the start that although we have three


equations in three variables, there are only two
independent equations in this system.
t
equations in these problems. This can easily be shown, for example, by
Illustration 1. subtracting from the equation x + y + z = x + y + z the
Given three mixtures consisting of three components sum of the first two equations; we get the third equation.
For this reason, from system (1) we find only the ratio x:
A, B and C. The first mixture only contains components
y: z, and not x, y and z themselves. For instance,
A and B in the weight 3 : 5, the second mixture contains
eliminating from the first two equations of (1), we find
components B and C in the weight ratio of 1 : 2, and the that y = 2z. Substituting this value of y into any equation
third mixture contains only the components A and C in of the system, we get x = (20/3)z.
the weight ratio 2 : 3. In what ratio must we take these Consequently, x: y: z = 20 : 6 : 3, which means the
mixtures so that resulting mixture of components A, B mixtures, have to be taken in the weight ratio 20 : 6 : 3.
and C stand in the weight ratio 3 : 5 : 2 ? Percentage problems represent another difficult type
Some aspirants have found the phrase "weight ratio" of problem for aspirants. Yet there is nothing hard about
difficult to grasp others fear the word "mixture". Actually, the notion of a per cent. We can get rid of percentages
the problem is not at all difficult. simply by considering hundredths of a number. The

CAT Complete Course | 427

Visit : www.EasyEngineering.net
Visit : www.EasyEngineering.net

following problem is one involving both mixtures and mouth of the tributary. The time required for a motorboat
percentage. to cover the distance from A to the mouth of the tributary
Illustration 2. and back stands in the ratio of 32 to 35 to the time
required to go from B to the mouth of the tributary and
The percentages (by weight) of alcohol in three back. If the rate of the motorboat were 2 km/hr more,
solutions form a geometric progression. If we mix the then this ratio would be 15: 16, and if the rate of the
first, second and third solutions in the weight ratio of 2: motorboat were 2 km/hr less, then the ratio would be 7: 8.
3: 4, we obtain a solution containing 32% alcohol. If we Find the rate of flow of the river (distances are measured
mix them in the weight ratio 3 : 2 : I, we obtain a solution along the tributary and river, respectively).
containing 22 % alcohol. What is the percentage of
Let the rate of river flow be u km/hr, the rate of the
alcohol in each solution ?
motorboat in still water v km/hr and the rate of flow of
Let there be x% alcohol in the first solution, y% in the tributary, w km/hr. Furthermore, let the distance from
the second, and z%in the third. This means that 1 gram of A to the mouth of the tributary be equal to s km. Then the
the first solution contains x/100gram of alcohol, 1 gram time the boat takes to cover the distance from A to the
of the second solution, y/100gram of alcohol, and 1 gram mouth of the tributary and back is t1 = s/(v + w) + s/(v –w)
of the third solution, z/100gram of alcohol. If we take 2 = 2sv/(v2– w2) (hrs.).
grams of the first solution, 3 grams of the second, and 4

w(w
grams of the third, we get 9 grams of a mixture
containing 2·
x
100
+ 3·
y
100
+ 4·
z
100) grams of alcohol.
Since the distance from B to the mouth of the
tributary is also s km, the time required for the boat to go
from B to the mouth of the tributary and back is t2 = s/(v +

w
By the statement of the problem, the resulting mixture

mixture contains 9.
32
100 .Ea
contains 32% alcohol, which means that 9 grams of the
grams of alcohol. From this
u) + s/(v – u) = 2sv/(v2 – u2) (hrs.). From the statement of
the problem, t1 : t2 = 32 :35 and we get the first equation
v2 – u2 32
=
v2 – w 2 35
condition we get equation
2x + 3y + 4z
100
= syE
9·32
100
Two more equations are set up in similar fashion:
(v + 2)2 – u2 15
=
(v + 2)2 – w 2 16
In similar fashion, we get yet another equation :
3x + 2y + z
=
6·22 ngi (v + 2)2 – u2 7
=
(v + 2)2 – w 2 8

nee
100 100 Simplifying we get the following system of equa-
Finally, by hypothesis, the numbers x, y, z form a tions :
geometric progression and so y2 = xz. 3v2 = 35u2 – 32w 2
It now remains to solve the system of equations
2x +3y + 4z = 288 rin
(v + 2)2 = 16u2 – 15w 2
(v – 2)2 = 8u2 – 7w 2
3x +2y + z = 132
y2 = xz g.n
We have to find u. This is best done by first
eliminating u, which is just the unknown we are seeking.
Solving the first two equations for y and z and
substituting the resulting expressions into the third
equation, we get the equation x 2 – 76x + 768 = 0 with
roots X1 = 64 and X2 = 12.
e
Eliminating u, we obtain the system
2(v – 2)2 – (v + 2)2 = w 2
35(v – 2)2 – 24v2 = 11w 2
t
Now eliminating w, we get the equation 13 (v – 2)2 +
But the value X1 = 64 does not satisfy the conditions 11 (v + 2)2 – 24v2= 0, whence v = 12. Now it is easy to
of the problem since the corresponding value of Y = 48 – find w = 2 and, finally.
2x is negative. Hence, there remains only x = 12, from u = 4. We thus have the answer: the rate of flow of
which we easily find y = 24 and z = 48.Thus, the first the river is equal to 4 km/hr.
solution contains 12% alcohol, the second 24%, and the The next problem involves a system of three linear
third 48% . equations in three unknowns. This would appear to be
In many cases difficulties arise in the solution of the easy to solve, but at the examinations many aspirants
systems, particularly when finding the unknown requires failed because they got entangled in the manipulations
a certain amount of guesswork or an artificial device. with the literal coefficients. Problems involving literal
Such techniques often simplify computations or even data come up rather often at competitions and the aspirant
suggest the only way to solve the problem. . should be able to handle them.
Illustration 3. Illustration 4.
A tributary flows into a river. On the tributary, at a Two rivers flow into a lake. A steamship leaves port
certain distance from the mouth of the tributary, lies point M on the first river, steams downstream to the lake then
A. On the river, point B lies at the same distance from the across the lake (still water) and up the second river

428 | CAT Complete Course

Visit : www.EasyEngineering.net
Visit : www.EasyEngineering.net

(upstream) to port N, and then makes the return trip. The Now adding (3) and (4) we have 2v 2 S – 2v 1 v2s =
ship has a speed of v in still water, the rate of flow of the tv(2v2 – 2v1v2), whence follows the distance over the lake:
first river is v 1 that of the second river v 2 , the transit time vS – v2t + v1 v2t S – vt
between M and N is t, the distance from M to N is S. The S=v vt + v2 …(5)
v1v2 v1v2
return-trip time from N to M along the same route is
equal to t. What distance does the steamship cover across The solution of the problem is complete. Some
the lake in one direction? aspirants, however, considered it necessary, after
Denote by S1 and S2 respectively the distances from obtaining the answer with literal data [formula (5), say],
to determine under what relationships of the data the
ports M and N to the lake, and by s the distance over the
lake. By the statement of the problem, we have S1 + s + answer has "real meaning" (such requirements are impo-
S 2 = 8. It is then easy to see that the time required for the sed as positivity of rates, distances, etc.; conditions are
introduced that guarantee nonvanishing denominators,
steamship to cover the distance from M to N is
and the like). Quite naturally, a properly conducted
s1 s s
+ + 2 =t investigation does not detract from the solution of a
v + v 1 v v – v2 problem, but this investigation is not logically necessary,
The time required for the return trip is figured since it is usually taken for granted that the events
similarly. We thus obtain a system of three equations in actually took place and, hence, the literal data already

ww
three unknowns S 1 , S 2 , s :

s1
s1 + s + s2 = S
s s
satisfy the necessary relations. Quite naturally, such an
investigation must be carried out if it is explicitly
required by the conditions of the problem.

w s1
+ + 2 = t
v + v 1 v v – v2
s
+ + 2 = t
v – v1 v v + v 2
s
.Ea
…(2)
It rather often happens that in problems demanding
the setting up of equations, the resulting system has
second-degree homogeneous equations in two variables
(equations of the type ax2+ bxy + cy2 = 0, where a, b and
We are interested in s.

syE
This system looks rather imposing, though actually
there is nothing really complicated in it, particularly if we
c are certain numbers). Unfortunately, however, few
aspirants realize that the presence of homogeneous
equations helps to solve a system of equations. A homo-
recall that v, v1 , v2, S, t are given constants; it is quite
obvious that the system (2) is a system of three equations
in three unknowns, which of course can always be solved, ngi geneous equation of second degree in two unknowns
directly defines the relationship of the unknowns, and this
say, by successive elimination of the unknowns.
However, it often happens that what is simple in nee naturally simplifies subsequent computations. Let us
examine a problem whose solution makes essential use of
this fact.

rin
theory turns out to be extremely awkward in practice. In
Illustration 5.
this problem, such an approach would be unwieldy in the
extreme, involving cumbersome manipulations merely An automobile leaves A for B and a motorcycle
because the coefficients of system (2) are rather compli-
cated. g.n
leaves B for A at the same time and with a smaller speed.
After a time they meet, and at that instant another
We will solve (2) in a somewhat artificial manner,
but one which is shorter. The second equation of the
system can be rewritten as
v2s1 – vv2 s1 + v 2 s + (v1 – v2)vs – v1 v2s + v 2 s2 + vv1 s2
e
motorcycle leaves B for A and encounters the automobile

t
at a point which is distant from the meeting point of the
automobile and first motorcycle 2/9 of the distance from
A to B. If the speed of the automobile were 20 km/hr less,
= tv(v2 + vv1 – vv2 – v1v2) the distance between the meeting points would be 72 km
and the first meeting would have occurred 3 hours after
Replacing the sum v2 s1 + v2s + v2s2 in the left member
the automobile left A. Find the distance between A and B
by v2S on the basis of the first equation, and collecting
(the motorcycles have the same speeds).
terms, we get the equation
v 2 s + v[v1 s2 – v 2 s1 + (v 1 – v 2 )s] – v1v2s = tv(v2 + vv1 Let the speed of the automobile, be u km/hr, that of
– vv2 – v1v2) …(3) the motorcycle v km/hr, the distance AB, s km, and let t
hours elapse before the automobile and first motorcycle
In the same way we can transform the third equation meet. We readily set up the following system of
of our system. But we can save on manipulations if we equations :
notice that the third equation is very much like the
second: simply replace s1 by s 2 and v1 by v 2 and vice- tu + tv = s
versa, and we have the second equation. In (3) replace s1 3(u – 20) + 3v = s
by s2 and v1 by v2 and vice versa to get the third trans- 3(u – 20) + 3v = s
formed equation : 2 2
v2S + v [v2 s1 – v1 s2 + (v2 – v1 )s] – v2v1s = tv (v 2 + vv2 s vt – s
9 9
– vv1 – v2v1) …(4) =
u v

CAT Complete Course | 429

Visit : www.EasyEngineering.net
Visit : www.EasyEngineering.net

72 3v – 73 set out at same time and arrived at the same time yield the
=
u – 20 v first equation :
Eliminating the auxiliary unknown t and simplifying, a s–a a s–a
+ = +
we get the following system : 20 v w 20
s = 3(u + v – 20) The information on the B to A trip enables us to set
9uv = 2(u + v)2 up the second equation in similar fashion :
v (u – 20) = 24(u + v – 20) a+1 s–a–1 a+1 s–a–1 7
+ = + –
To find s we have to find u and v from the last two 20 v w 20 20
equations. Noting that the second equation is a homo- (21 minutes = 7/20 hr).
geneous second-degree equation in two variables, we can
Since the first boy spent 1/v hours per km and the
easily find the ratio u : v.
second 1/w hours, we immediately get (from the
Since we are interested in u and v different from statement of the problem) the third equation :
zero, then dividing the second equation by v 2 , we get a
quadratic equation in the new variable z = u/v : 1 1 1
– =
w v 20
2z2 – 5z + 2 = 0
We now have a system of three equations in four

wwThe roots of this equations z1 = 2 and z2 = 1/2, and so


either u = 2v or u = v/2.
But from the statement of the problem u > v, and so
unknowns. It is impossible to determine all the
unknowns s, a, v, w from this system. In this sense, the

w
system is indeterminate. But does this mean we are not
we take u = 2v.
able to solve the problems ? Of course not. This is

.Ea
Substituting this value of u into the third equation,
we get either v = 40 or v = 6. But if v = 6, then u = 12, but
it is given that u > 20. The problem is satisfied only by
because we only have to find two unknown quantities, the
rates v and w, and those can be found uniquely from the
system of equations.

syE
v =40. But then u = 80 and s = 300. Therefore, the
distance AB is found to be 300 km.
Almost insuperable difficulties stem from problems
To do this, subtract the first equation from the second
1 1 9
to get + = and consider the result together with the
in which the aspirant finds, after correctly setting up the
system of equations, that the number of unknowns is
ngi w v 20
third equation. Obvious computations yield v = 5 km/hr
and w = 4 km/hr.

nee
greater than the number of equations. The following
problem illustrates this point. In this problem we were able to find the unknowns
Illustration 6. we needed despite the fact that there were fewer
Two boys with one bicycle between them set out
from A in the direction at B, one by bicycle and the other
rin
equations than unknowns. In the next problem we will
obtain a system of equations without being able to

g.n
on foot. At a certain distance from A the one riding the determine any of the unknowns, though we will be able to
bicycle left it by the road and continued towards B on determine the greater one, which is what is required in the
foot. The one who had started out on foot reached the problem.
bicycle and rode the rest of the distance. Both reached B
at the same time. On the return trip from B to A, they did
as before, but this time the cyclist rode one kilometer
more than the first time and so his comrade arrived in A
Illustration 7.
e
A aspirant spends a certain sum of money on a
booking, a fountain pen and a book. If the cost of the t
bookbag were less by a factor of 5, the pen by a factor of
21 minutes after he did. Find the rate of each of the boys 2 and the book by a factor of 2.5, the overall cost would
on foot if they both did 20 km/hr cycling, and, on foot, be 8 dinar. Now if, compared to the original cost the
the first takes 3 minutes less to cover each kilometer than prices were reduced- twofold for the bookbag, fourfold
the second. for the pen, and threefold for the book-then the total
Let us introduce the following notation : outlay would be 12 dinar. How much money was spent
s km for the distance between A and B; and what item cost more, the book bag or the fountain
v km/hr for the rate on the foot of the first boy; pen ?
w km/hr for the rate on the foot of the second boy; Suppose the bookbag cost x dinar, the pen y dinar
and the book z dinar. Together they cost x + y + z, and
a km for the distance that the first boy cycled from A
that is what we wish to find out.
to B (he thus first his bicycle a km from A and covered
the rest of the distance to B on foot). The first equation is set up on the condition that
under the original assumption the outlay was 8 dinar :
It is quite clear that the whole trip from A to B was
made by the first boy a/20 + (s – a ) /v hours, by the x y z
+ + = 8
second boy in a/w + (s – a)/20 hours. The fact that they 5 2 2·5

430 | CAT Complete Course

Visit : www.EasyEngineering.net
Visit : www.EasyEngineering.net

Similarly we set up the second equation : km. From the statement that the fast train meets the first
x y z passenger train not later than three hours after departure,
+ + = 12 we get
2 4 3
We have a system of two equations in three s
≤ 3
unknowns and cannot of course determine all the v1 + v2
unknowns, but we can find the total sum, which is what From the condition that the fast train arrived in B not
we need. To do this, rewrite the equations thus: earlier than 5 hours after departure, we obtain
2x + 5y + 4z = 80 s
≥ 5
6x + 3y + 4z = 144 …(6) v1
Adding these two equations, we get the sum of the Since the time elapse prior to the first meeting is
unknowns : x + y + z = 28. The total outlay comes 28 s/(v1 + v2) hours, it follows that the fast train will overtake
dinar, thus answering the first question. the second passenger train in 12+ [s/(v 1 + v2)] hours, and
Now let us try to determine which item, bookbag or so
fountain pen, is more expensive: in other words we want
to know which of the inequalities x > y or y > x is valid.
( 12 +
s
)
v1 + v2
(v1 – v2) = s

We have to find x = s/v2 , whence s = xv2 . Substituting

ww
If we subtract the first equation of (6) from the
second, we obtain this expression for s in the preceding equation and
inequalities and denoting v1/v2 α we get the system
2x – y = 36 …(7)
x ≤ 3(α + 1)

w
From this it is clear that x > y/2, because otherwise

.Ea
we would have 32 = 2x – y < 0. But the inequality x > y/2
does not yet answer the question given in the problem.
This is because we have not yet made full use of equation
x ≥ 5α
x = 6(α 2 – 1)
It was precisely this system that stumped many

syE
(7). Namely, we merely noticed that the difference 2x – y
is positive. Now let us try to make use of the fact that it is
equal to 32 and also take into account that x + y + z = 28
aspirants.
Actually it is not so complicated. It is necessary to
eliminate either x or α and go over to a system of two
and that all the unknowns, x, y, z must, realistically, be
positive numbers.
ngi inequalities in one unknown. Since it appears easier at
first glance to eliminate x, let us do so. Putting x = 6(α2 –

nee
Rewrite (7) as x + (x – y) = 32. Since the total outlay 1) into the first two inequalities, we obtain the system of
is 28 dinar, then certainly x < 28 and from the latter inequalities
equations x – y > 0 which states that the bookbag is more 2α2 – α – 3 ≤ 0
expensive than the fountain pen.
Nearly all the foregoing problems implicitly involve
rin
6α2 – 5α – 6 ≥ 0
The solutions of the first inequality are – 1≤ α ≤ 3/2,
inequalities. In problem 5 for instance there were even
two : u >v and u > 20. The inequalities that appear in such
problems do not ordinarily upset the aspirant; what does g.n
the solutions of the second are α ≥ 3/2, and α ≤ – 2/3.
Hence, the solution of the system is α =3/2, and also all α
lying in the interval – 1 ≤ α ≤ – 2/3. Since we are only
cause a lot of trouble is when the condition of the
problem have to be written out explicitly as inequalities.
Many aspirants get as far as writing down the system of
equations and inequalities, but no farther. Apparently
e
interested in positive values of α, the condition of the

t
problem is satisfied by the sole value α, the condition of
the problem is satisfied by the sole value α = 3/2. From
this it is easy to find x = 15/2 and we get the answer : the
they are not ready psychologically to solve systems of first passenger train arrives at A at 16:30.
this kind to illustrate, take the following problem. This problem allows for a solution in which all the
Illustration 8. conditions of the problem are written down in the form of
A first train leaves A for C at 9 A.M. At same time equations. This is achieved by introducing supplementary
two passenger trains leave B (located between A and C), unknowns and obtaining a system of equations in which
the first with destination A, the second with destination the number of unknowns is greater than the number of
C. The two passenger trains have the same speed. The equations. However, it is more difficult to solve that type
fast train meets the first passenger train not later then 3 of system of equations than it is to solve a system of
hours after departure, then passes B not earlier than 14:00 inequalities.
of the same day and, finally, arrives in C together with Let us solve the problem the second way, retaining
the second passenger train exactly 12 hours after meeting all the earlier notation. Let the fast train meet the first
the first passenger train. Find the time of arrival at A of passenger train after an elapse of (3 – t1 ) hours (t1 ≥ 0), let
the first passenger train. it pass B in (5 + t2) hours (t2 ≥ 0) and let it catch up with
Let the speed of the fast train be v1 km/hr, that of the the second passenger train in [(3 – t1 ) + 12] hours. Then
passenger train, v 2 km/hr, and let the distance AB be s we can easily set up the equations

CAT Complete Course | 431

Visit : www.EasyEngineering.net
Visit : www.EasyEngineering.net

(v1 + v2) (3 – t1 ) = s prior to the second meeting he rode 8 hours and hence
v1(5 + t2 ) = s covered 96 km. It is given that the car driver met the
(15 – t1 ) (v 1 – v2) = s cyclist before the latter had covered 96 km, which means
the motorist has less than 96 km left to drive to A. He
xv2 = s
will spend less than 96/50 hours. Since the cyclist will
Eliminating s and denoting v1/v2 by α, we get the spend less than 8 hours before meeting the car, their
system of equations meeting will occur earlier than 17:00. Thus, after meeting
(α + 1) (3 – t1 ) = x the cyclist there remain over two hours for the car driver
α(5 + t2 ) = s …(8) to reach A before 19:00 hours. But he requires less than
(α + 1) (15 – t1 ) = x 96/50 hours, that is less than two hours, to cover this
distance. And so the motorist will reach A before 19:00.
This is a system of three equations in four unknowns.
We only want x. proceeding as before, eliminate x to get At competitions, problems appear in which the
αt2 + (α +1) t1 = 3 – 2α aspirant is asked to find an optimal solution, say, to buy
the largest possible quantity of goods for a given sum of
(1 – α)t1 – α t2 = 15 – 10α …(9)
money or to choose the best possible (cheapest) variant in
Noting that the right member of the second equation transportation of goods, and the like.
is five times the right member of the first, multiply the

ww
first by 5 and subtract the second to get
6αt2 + (6α + 4) t1 = 0 …(10)
Since α > 0, t1 ≥ 0, t2 ≥ 0, it follows that this
Solutions of this type of problem may consist of
setting up systems of equations and inequalities and their
solution. However, the most necessary element in such

w
equation can only be valid for t1 = 0 and t2 = 0. But then

.Ea
from (9) it is easy to find α = 3/2 and from (8), x =15/2.
We get the same answer. Many aspirants failed to notice
cases is the reasoning that helps to choose the best
possible variant.
Illustration 10.
The task is to construct a number of identical

syE
that (10) can be derived from (9) and therefore they could dwelling houses with a total floor-space of 40,000 sq.
not conclude, from (9), that t 1 = t2 = 0 and so could not
metres. The cost of one house of N sq. metres (m 2 ) of
solve the problem. dwelling floor-space consists of the cost of the above
The foregoing shows that the first method of solution
is much easier than the second method.
Illustration 9. ngi ground portion (superstruction), which is proportional to
N√ N and the cost of the foundation, which is proportional

A cyclist starts out from city A at 9 A. M. and


proceeds at a constant rate of 12 km/hr. Two hours later, nee
to √
 N. The cost of a house of 1600 m2 is set at 176,800
dinar, in which case the cost of the above ground portion
is 36 % of that of the foundation. Determine how many
a motorcyclist starts out with an initial speed of 22 km/hr
and proceeds with uniformly decelerated motion so that
rin
houses should be put up so that the overall cost is the
smallest possible and find that sum.

g.n
in one hour the speed diminishes by 2 km/hr. A man in a Suppose we decide to construct n identical structures
car driving to A at 50 km/hr meets first the motorcyclist each of which y m 2 of dwelling floor-space. Then the
and then the cyclist. Will the car driver be able to reach A equation yn = 40,000 is valid. Let the cost of one
by 19: 00 that same day ?
This problem can likewise be solved by setting up a
system of equations and inequalities, but it would require
extended reasoning. It is best to approach the problem by
e
structure of y square metres of floor-space be z thousand

t
dinar; then the cost x of the construction job as a whole is
computed from the equation x = zn.
The cost of a house consists of the cost, v, of the
simple reasoning instead of attempting to solve it by above-ground portion of the structure and the cost, w, of
formally setting up of a system of equations and the foundation: thus, z = v + w.
inequalities. It is given that the cost of the above-ground portion
If follows from the statement of the problem that first of a house of y squares metres is proportional to y √ y, that
the motorcyclist catches up with the cyclist and then the
is, v = αy √  y , where α is a certain coefficient. Similarly,
cyclist catches up with the motorcyclist. Let the cyclist
spend t hours prior to a meeting (first or second, no w = β√  y where β is a certain coefficient.
matter which). Then the motorcyclist will spend (t – 2) In the particular case of the construction of a house
hours over the same distance. Since they cover the same of 1600 square metres, taking into account that the cost of
distance before meeting, we can equate their paths to the superstructure is 36% of the cost of the foundation,
36
obtain.
(t – 2)2
we get α·1600· √
1600 =
100
(
· β·√ )
1600 and taking into
12t = 22 (t – 2) – 2 consideration that the construction of a house of 1600
2
Solving this equation, we see that up to the first square metres costs 176,800 dinar, we have
meeting the cyclist rode 6 hours and thus covered 72 km; 176,800 = α·1600√

1600 + β
√
1600

432 | CAT Complete Course

Visit : www.EasyEngineering.net
Visit : www.EasyEngineering.net

All he conditions of the problem have been written than one set of the third kind. We have thus obtained the
down, we now have to determine x as a function of n and inequalities x ≤ 2, z ≤ 1.
then determine for which n, will be a minimum. Now there are fewer cases and we can run through
It is easy to find α and β from the last two equations: them. For x = 0 we get the equation 6y + 9z = 100 to
α = 117/160000, β = 13/4. Substituting v and w into the determine y and z. It clearly does not have a solution
expression for z, we get z = (117/160000) y√  y + (13/4) since the left member is divisible by 3 but the Right is
not. Furthermore, when x = 1 we get the equation 2y + 3z
√ y. Now putting this value of z and the value y=40,000/n = 32, which [taking into account the inequality z ≤ 1) has
from the first equation into the second, the unique solution y = 16. z = 0. Finally, when x = 2,
we get there is no solution, as in the case of x = 0.
X = 650   n
9
+√ Thus, to ensure the largest number of decorations, we
√n  have to buy 1 set of 20 items and 16 sets of 35 items.
We, thus conclude that x, the cost of construction, is It is possible in this solution to dispense with
the above function of n, the number of houses. We now examining all cases if a more detailed use is made of
have to determine the smallest value of x. Applying to divisibility, as witness: from the given equation it follows
the right member of this equation between the arithmetic that the number x yields a remainder of 1 upon division

ww
mean and the geometric mean, we find X ≥ 2·650 √
3900 equality being attained only when 8/ √n = √
9 =
 n, that
by 3, and the number z is even. It therefore follows
immediately from the inequalities x ≤ 2 and z ≤ 1 that x =
1 and z = 0; from the equation we get y = 16.

w
is, for n = 9. In other words, the cost of construction will
always be at least 3.9 million dinar and will be exactly

.Ea
equal to this figure only when n =9.
Thus, in this housing construction project, the
Note in conclusion that in actuality the foregoing
reasoning signifies that the optimality condition of the
solution may be written in the form of the following
system of equations and inequalities :

Illustration 11. syE


smallest expenditure will be in building 9 houses and the
total cost in that case will be 3·9 million dinar.
4x + 6y + 9z = 100,
0 ≤ x ≤ 2, 0 ≤ y, 0 ≤ z ≤ 1 …(11)
With the supplementary provison that x, y and z are
It is agreed to spend 100 dinar on Christmas tree
decorations. Such decorations are sold in sets consisting
ngi integers. Now the condition of x, y and z being integers
means that z = 2n and x =1 + 3k, where n and k are also

nee
of 20 items per set costing 4 dinar a set, sets of 35 items integers. Substituting these values of z and x into the
at 6 dinar, and sets of 50 items at 9 dinar. How many of appropriate equations, we get n = k = 0, or x = 1 and z =
what kinds of sets should be bought in order to have the 0. It is now easy to find y = 16 from equation (11).
largest number of items ?
Let x, y, z be the number of sets, respectively, of the
Illustration 12.
rin
A forestry has to deliver 1590 trees. The vehicles
first, second and third kind which must be bought to
ensure a maximum quantity of items (this solution is
termed the optimal solution of the problem) Then
1
g.n
assigned to this job are 1 – ton, three-ton and five-ton
2

e
1
4x + 6y + 9z = 100 trucks. A1 – ton truck carries 26 trees at a time, a three-
This is the only equation that can be set up on the
basis of the statement of the problem. But we also know
that x, y and z are nonnegative integers and that the
number of items of decoration in this purchase is greater
2

t
ton truck carries 45 trees, and a five-ton truck, 75 trees.
1
The cost of one run of a 1 – ton truck is 9 dinar, that of a
2
three-ton truck, 15 dinar, and of a five-ton truck, 24 dinar.
than any other. These conditions, it turns out, are quite is The forestry wishes to minimize the overall cost of the
greater sufficient to ensure an unambiguous determination deliveries. How is this to be done (it is assumed all trucks
of all unknowns. The first idea is to solve the equation by are fully loaded) ?
running through all possible values of the unknowns but 1
Let x, y, z be the number of 1 -ton, three-ton, and
this is rather hopeless due to the enormous number of 2
cases. five-ton trucks, respectively, in the case of optimal
However, this number may be appreciably reduced distribution. Since all vehicles are fully loaded, the
with the aid of arguments of an economic nature. Indeed, number of transported trees in this setup will be 26x + 45y
12 dinar can buy 3 sets of the first type or 2 sets of the + 75z and thus we get the equation 26x + 45 y + 75z =
second; in the former case we get 60 items, in the latter 1590.
70. It is then clear that in an optimal solution the number We are now in the same position as in Problem 11,
of sets of the first type should not exceed 2. Comparing but attempts to reduce the number of cases that succeeded
sets of the second and third type in similar fashion, we then do not yield any substantial simplification. For
find that in the optimal solution there should not be more example, x is divisible by 15. That's about all. We might

CAT Complete Course | 433

Visit : www.EasyEngineering.net
Visit : www.EasyEngineering.net

1 summarize, then, we set out in the ordinary way, but in


add that 45 1 -ton trucks would cost 405 dinar a trip, 26
2 the process of solution we found an approach which made
three-ton trucks carrying the same number of trees would all the earlier arguments unnecessary. Quite obviously the
cost 390 dinar so that in an optimal solution the number method given here would be quite sufficient at any
1 competition.
of 1 -ton trucks should not exceed 44. And so for x we
2
get three possibilities : x = 0, x = 15, x = 30. For each of SOLVING EQUATION
these values we would have to solve the equation for y At competitions a rather strange situation would
and z, which also have a good many solutions. appear to develop around equation. These problems are
This is a very tedious approach, though in the ordinarily not considered difficult and most aspirants
absence of any other, it is acceptable. handle them fairly well. Yet many serious mistakes are
Here's an attractive idea, which, unfortunately, made.
doesn't do the job. From the statement of the problem it is This situation is strange only at first glance. The
1 point is that very often there is a big gap between
easy to figure out that for 45 dinar using 5 1 -ton trucks
2 computational techniques and a conscious grasp of the
we can deliver 130 trees and using 3 three-ton trucks, 135 logical foundations that underlie them.
1

ww
trees. It would therefore appear that the number of 1 -ton
trucks should not exceed 4, otherwise those same trees
could be delivered more cheaply. From this and from
2
Most aspirants can of course simplify an equation by
means of clear-cut manipulations, but by far not every
aspirant is capable of realizing that a solution has been

w
earlier considerations it follows that x = 0 and the number

.Ea
of cases left to be examined is much less. .
Actually, however, this argument only implies that
for a given sum of money we can deliver a larger number
lost or acquired, and many don’t even give thought to
such things still others may know certain parts of the
theory pertaining to these matters but the knowledge is
only formal and such aspirants are often completely

syE
of trees, but our aim is to deliver a given number of trees
at minimum cost. Nevertheless, it is still possible to get
around brute-force tactics by just a little commonsense
helpless in a slightly altered situation.
Let us say that the aspirant is quite familiar with fact
that squaring both sides of an irrational equation can give

reasoning.
Any reasonable person would first estimate which of ngi rise to extraneous roots. Yet time and again aspirants
square trigonometric equations and then fail to discard
extraneous roots! This mistake would not be made if the
the given types of trucks is the most efficient by
determining the cost of delivering a single tree. We find
1 nee
aspirant realized why squaring leads to the introduction
of extraneous roots.

rin
the following : for a 1 -ton truck, a three-tonner and a Or take checking. There seem to be two opposing
2 opinions among aspirants here. Some regard checking as
five-tonner the cost is 9/26, 1/3 and 8/25 dinar a whim of the teacher, some thing that simply has to be
respectively. Since 9/26 > 1/3 > 8/25, it is clear that it is
more profitable to use five-ton trucks first, then, if
1 g.n
done in order to pass while others regard checking as
necessary in all cases without exception. They even go on
necessary, three-tonners and, finally, 1 -ton trucks.
2
It is easy to see that the greatest number of trees that
can be delivered by five-ton trucks comes out to 1575.
However, seeing that all vehicles must be fully loaded,
e
to check the roots of quadratic equations. Both views are
based on a total misconception of what checking really is
and what place it occupies in problem solving.
t
In short, the aspirant should have a firm grasp of the
fundamentals of the theory that is needed in the solution
we get 1500 trees for delivery by five-ton trucks. Then 90 of equations. Let us examine this minimum of theoretical
trees can be delivered by three-tonners and so it is natural knowledge.
to suppose that the optimal distribution will be 20 five-
First some definitions.
ton and 2 three-ton trucks.
1. The domain of the variable of an equation is the
It is easy to demonstrate that this plan is indeed an
set of values of the variable (unknown) for which its left
optimal plan : if we reduce the number of five-tonners,
and right members are meaningful (defined ); it is thus
then the "undelivered" trees apportioned to these vehicles
the set of all eligible replacement for a variable in an
1
would have to be delivered by 1 -ton and three-ton equation.
2
trucks, but delivery costs per tree on these trucks are 2. A number is a solution (root) of an equation if
higher, and so the total cost or the assignment would when substituted for the unknown (variable) makes the
increase. equation a true statement (converts it into a true
numerical equation).
We thus have the optimal distribution of 20 five-ton
and 2 three-ton trucks, while all the unknowns and the According to this definition, the solution set (all the
single equation that was set up remain unused! To solutions) of an equation is a subset (a part) of the domain

434 | CAT Complete Course

Visit : www.EasyEngineering.net
Visit : www.EasyEngineering.net

of the variable, otherwise substitution in place of the new solution set (new roots). The prime task in a correct
unknown would not yield a true statement and would be solution of any equation is to follow the variations in the
meaningless. solution set and not to allow for any loss of roots or any
3. To solve an equation means to find all the roots or failure to discard extraneous roots.
prove that there are no roots. The best method is clearly , each time to replace the
4. If all the roots (solution set) of one equation are given equation with an equivalent equation. Then the
the roots of another equation, then the latter equation is a roots (solution set) of the last equation will be the roots
consequences of the former. (solution set) of the original equation. In practice
however this ideal version is rare. As a rule, an equation
5. Two equations are termed equivalent if each is a
is replaced by a consequence that is not equivalent ; then,
consequence of the other. From this definition it follows
by the definition of consequence, all the roots of the first
immediately that equivalent equations have the same
equation are the roots of the second, that is to say there is
solution sets.
no loss of roots, but extraneous roots may appear (on the
6. Two equations are equivalent on some set of other hand, they may not). And when in the process of
values of the variable (unknown) if they have exactly the manipulations the equation is replaced by a nonequivalent
same solution belonging to this set. consequence, the roots have to be investigated. This is a

ww
Let us illustrate these concepts with two examples.
This domain of x in the equation x – 3 = √  3 consists,
according to the definition, of all x for which the left
check and it is necessary. Note here and now that, as we
shall see later on, this investigation does not at all mean
that we have to substitute the roots obtained into the

w
member x – 3 and the right member √

.Ea
 x are meaningful.
Clearly, the left member is defined for any x and the
right member for x ≥ 0. Therefore the domain of x in this
equation consists of x ≥ 0.
original equation.
To summarize then, if a solution is carried through
without an investigation of equivalence and source of
extraneous roots, then verification is an necessary part of

syE
Yet many aspirants erroneously state that the domain
of the variable is x ≥ 3, since “for x < 3 the left member is
negative and the right member cannot be negative.” The
the solution without which it cannot be regarded as
complete even if no extraneous roots appeared in actual
fact. Of course if they did appear and were not discarded,
quoted part of the statement is true and it is used in the
solution of the given equation. It shows that the roots of ngi the solution is simply incorrect. On the other hand, if
each time the equation was replaced by an equivalent
equation (which, incidentally, is an extremely rare case),
the equation are not less than 3. But it does not follow
there from that all permissible values are less then 3,
because not all permissible (eligible) values are roots ! nee
and this fact is stipulated at each stage in the solution,
then no verification is required. We thus see that the

rin
notion of checking plays a very definite and extremely
Consider the two equations.
essential role in the solution of equation and does not by
log6 (x – 2) + log6 (x + 3) = 2 and log6 (x – 2) (x + 3) = 2
any means merely reduce to a simple checking through of
Obviously every root of the former equation is a root
of the latter one, so that the latter equation is consequence
of the former the latter equation can readily be solved to
computations.
g.n
As for checking computations, that is up to the aspirant.
yield the solution set (roots) x1 = 6 and x 2 = – 7. The root
x2 does not satisfy the first equation, it is not even in the
domain of the variable. Thus, the two given equations are
not equivalent, but they are equivalent in the domain of x
e
He may do it or he may not according to how carefully he
feels the computation have been carried through. It is of
course always best to check oneself at an examination,
but this should be done on a separate sheet of paper and
t
of the first equation (in this domain they have the one there is no need to include it in the solution.
root x = 6). It must be stressed that it is not permissible to replace
It is easy to see why this is so. The domain of x in the a given equation by one which is not a consequence of
first equation consist of x > 2,while the domain of x in the the first, because then there is a root of the first equation
second equation is broader, including these x and also x < which is not a root of the second, and so solving the
– 3. It is therefore natural that in passing from the first second equation does not yield all the roots of the first. A
equation to the second an extraneous root x = – 7 root will be lost for good. That is the essential difference
appeared that does not belong in the domain of the between loss of roots and the introduction of extraneous
variable of the first equation. roots.
How do these newly introduced concepts operate in Such is the theory. In practical situation, one has to
the solution of equations? The point is that in most cases know the specific sources of introducing or losing roots.
a solution is obtained after a long chain of manipulations In the main, these sources are of two types: the so-called
and transformations from one equation to the next. Thus, “identity transformations” and the performance of
in the solution process, each equation is replaced by a operations such as raising to a power, taking logarithms,
new one, and quite naturally the new equation can have a antilogarithms, etc. in both members of an equation.

CAT Complete Course | 435

Visit : www.EasyEngineering.net
Visit : www.EasyEngineering.net

At first glance,” identity transformations” are quite the variable, this case and no other. As to talking function
harmless, but actually they often lead to nonequivalent of both members of an equation, we consider only two of
equations since they change the domain of the variable the more important cases: squaring and the taking of
(unknown). Say, if in the solution of an irrational equation antilogarithms.
2
we replace √(
 )
2x + 1 by 2x + 1, we immediately extend It often happens (especially in the solution of
irrational equations) that one has to pass from a certain
the domain of the variable since 2x + 1 is meaningful for
2 equation f(x) = g(x) to the equation {f(x)}2 = [g(x)]2. What
(
all x, while √  )
2x + 1 is valid only for x ≥ – 1/2. The happens to the roots in this transition ? First of all it is
same goes for the example that we analyzed earlier; the clear that the second equation is a consequence of the
use of the formula for the logarithm of a product led to an first: if the number a is a root of the first equation, that is
extension of the domain and, as a result, to the introduc- f(a) = g(a), then {f(a)}2 = [g(a)]2 and a is a root of the
tion of an extraneous root. second equation. But, generally speaking, the converse is
There is nothing strange in this : it is simply that most not true: the second equation is satisfied also by the roots
formulas used in transformations are such that their left are not lost but extraneous roots may appear.
and right members are meaningful for different values of A very important practical consequence follows from
the letters used. Such, for example, are the formulas this statement.

ww  a √ b, (√
ab = √
√  x)
2

= x, loga xy = loga x + loga y


B. If both members of an equation are non-negative
on some set of values of the argument, then, upon
squaring, we obtain an equation that is equivalent to the

w loga xn = n logax,

.Ea
alogab = b, cot x =
1
tan x
, sin 2x
original one on that set.
Indeed, in this case the “extraneous” equation clearly
has no roots, except for those for which both sides vanish,
but such are not extraneous for our equation. How this is

syE
2 tanx ,
= done practically will be demonstrated in concrete
1 + tan2 x
examples below.
tan x + tan y
tan (x + y) = Similarly we consider taking antilogarithms in
1 – tan x tan y
Therefore, replacing one part of a formula by another
ngi equations, that is, passing from the equation Logcf(x) =
logc g(x) to the equation f (x) = g(x). Let a be a root of the

nee
leads to an extension or narrowing of the domain of the original equation, that is Logc f (a) = logc g(a) then c Log0
variable. Extending the domain opens the way to f(a) = c Log0 g(a) , or f (a ) = g(a). Hence, any root of the
acquiring extraneous roots, while narrowing (restricting) original equating is a root of the second equation. On the
the domain makes possible the loss of roots, so that
narrowing is never permissible. As for extraneous roots,
if they are acquired through the extension of the domain, rin
other hand the domain of the variable of the second
equation is greater then that of the first and so it is natural

then it is not necessary to substitute them directly in the


original equation in order to separate them from the roots g.n
to expect extraneous roots, but they will be due precisely
to the extension of the domain. Hence, it suffices to find
the roots of the second equation and test them for
of the original equation; it suffices to check to see
whether they are in the domain of the variable. If they are
not, discard them, if they are, leave them.
This fact is of exceptional importance in the solution
e
membership in the domain of the first equation.
Such is the theoretical foundation that he aspirant
needs. It is also worth stressing that it is not always
advisable to employ the whole theory; moderation is the
t
of equations and so we will dwell on it in more detail. best advice and one should strive for the simplest
A. If in the process of transforming an equation, solution. If, say, in the solving process it becomes
extraneous roots appear only due to extending the domain apparent that a simple verification of the resulting roots is
of the variable, then those roots (and only those) which not difficult, then there is no reason to seek out the
appear in the domain will be the roots of the original sources of introducing roots or take an interest in the
equation. variation of the domain in the solution process, or even to
This rule relieves us of the necessity to substitute the find the domain at all. But if this verification is
roots found into the equation the purely mechanical complicated, then theoretical reasoning can save the day:
checking of numerical equations, which at times is at the proper time (and in the final version) the aspirant
exceedingly difficult or even sometimes completely should investigate the transformation that might lead to
impossible due to the fact that there are an infinite of extraneous roots.
numbers to be verified. At the same time, in any solution we must make sure
Thus, in place of direct substitution we can employ a that no loss of roots occurs. It is useful to state this
test for membership in the domain, but only when the explicitly, particularly if the transformation employed is
source of extraneous roots is extension of the domain of sufficiently complicated.

436 | CAT Complete Course

Visit : www.EasyEngineering.net
Visit : www.EasyEngineering.net

Below will illustrate the more typical cases and also Illustration 15.
the most insidious sources (though not all of them,
naturally) that give rise to extraneous roots. They include Solve the equation √
x + 3 + 
√2x – 1 = 4.
the formulas for transforming radicals the fundamental Both members of this equation are non-negative in the
logarithmic identity, and the formulas for taking domain of the variable, and so after squaring we obtain an
logarithms of a product and a power, clearing of fraction, equation which, according to statement B is equivalent to
the cancellation of similar terms, the replacement of an the original one in the domain of the variable :
2 2
equation by a collection of simpler equations, and certain
“verbal” arguments. We will then consider some sources
(√
x + 3) + 2√
x + 3 
√2x – 1 + √ (
 )
2x – 1 = 16
of loss of roots. Some of the last and more involved Using the formulas for transforming radicals, which
problems will be analyzed with the aim of indicating clearly extend the domain, we get the equation
certain difficulties of a different nature, not connected 2√

2x2 + 5x – 3 = 14 – 3x …(2)
with the acquisition or loss of roots In these transforming, extraneous roots could appear
Illustration 13. only due to the extension of the domain of the variable.
Then we reason as follows. The left member of (2) is
Solve the equation √

2x – 6 + 
√
x + 4 = 5. non-negative for every (permissible) value of x; but the

ww
Squaring both sides and using the formulas for
transforming radicals, we get the equation
2x – 6 + 2√

(2x – 6) (x + 4) + x + 4 = 25
right member is negative for x >14/3. It is quite obvious
that these values of x cannot be solutions of the equation,
and so from now on we will consider equation (2) only in

Or 2√
w

2x2 + 2x – 24 = 27 – 3x

.Ea
Again squaring and getting rid of the radicals we
…(1)

arrive at the equation x2 – 170x + 825 = 0 whose roots are


the domain x ≤ 14/3. But in this domain both member of
(2) are non-negative (for the permissible values of x with
respect to (2), naturally) and according to the statement
B, squaring yields an equation that is equivalent to (2) on
the set x ≤ 14/3 :

syE
x1 = 5 and x 2 = 165. Direct substitution of these values in
the original equation shows that x1 is a root and x 2 is not.
Illustration 14.
(2√
2x + 5x – 3)
2
2
– (14 – 3x)2
From this, once more extending the domain of the

Solve the equation :


√ 5x + 7 – √
3x + 1 = 
√x + 3
ngi variable, we get the quadratic equation x2 – 104x + 208 =
0 whose roots are x1,2 = 52 ± 8 √39. Both these roots, as

Squaring both members and manipulating we get


nee
will readily be seen, lie in the domain of the variable of
the original equation and for this reason we have only to
check that they satisfy the condition x ≤ 14/3 : It is easy
2√

(5x + 7) (3x + 1) = 7x + 5
Whence, again squaring we obtain the quadratic rin
to compute that x1> 14/3 and x2 ≤ 14/3 so that x 2 is the
only root of equation (2) and consequently, of the original
equation 11x2 + 34x + 3 = 0, whose roots are x1 = – 1/11
and x2 = – 3 direct verification shows that x = – 1/11 is
equation.

g.n
We once again stress that one should resort to this
kind of detailed, “theoretical”, solution only in case of

e
the root of the original equation. Checking the value x = –
necessity, only when working through the rough draft as
3 many aspirant at the examination got the equation √
–√
– 8
– 8 = 0. They considered this statement to be true on
the ground that the left member is a case of “equals
subtracted from equals”. Thus the value x = – 3 proved to
the aspirant sees that the roots are “bad” which is to say
t
that a direct substitution into the equation leads to a rather
complicated problem: the proof or disproof of the
equations
be a root of the original equation. But this argument is
√ 39 + √
55 + 8 √ 
103 + 16 √
39 = 4,
baseless since the expression √– 8 is devoid of meaning:
as we know, irrational equations are only considered in 
√ 39 + √
55 + 8 √ 
103 – 16√
39 = 4,
the domain of real numbers, and the symbol √  a is used Incidentally, the first of these equations is clearly not
for real a only to denote the principal square root of a true. The second one can easily be proved if one knows
non-negative number a. Therefore, the value x = – 3 does the formula for transforming expressions of the form
not lie in the domain of the variable and hence, is not a 
√ A±√
 B. The alternative approach of squaring involves
root of the original equation. considerable computational difficulties. It is quite clear
The situation is quite different in the problem which that both these methods are more complicated than the
now follows. Here, checking the “bad” roots is very one we gave, where all we had to do was to test the roots
involved and the simplest approach to a solution is by x1 and x 2 for their validity under the condition x ≤ 14/3.
applying the theory we have developed when the sources Nevertheless, in this problem it is still possible to over-
of extraneous roots are taken into account in the very come the difficulties of direct verification and avoid
process of solution. application of the theory.

CAT Complete Course | 437

Visit : www.EasyEngineering.net
Visit : www.EasyEngineering.net

However, in equations containing a parameter, direct replacing log a f(x) + loga g(x) by loga f (x) g(x), we extend
verification is appreciably more difficult and practically the domain of the variable, permitting values of the
the only way out is to employ the theory. unknown x for which we simultaneously have f(x) < 0 and
g(x) < 0. And so extraneous roots can appear, but only due
Illustration 16.
to extension of the domain of the variable, so that to
Solve the equation x – 1 = √
a – x2. discard them, on the basis of statement A, it suffices to
The right side is non-negative for all (permissible) x, verify their membership in the domain. Also note that the
and the left side is non-negative for x ≥ 1. Therefore, the converse replacement- the logarithm of a product by the
given equation is, in the domain x ≥ 1, equivalent to the sum of the logarithms- can lead to a narrowing of the
domain of the variable and so is not permissible.
equation (x – 1)2 = (√
a – x ) which can be reduced to
2
Illustration 17.
2x2 – 2x + 1 – a = 0 …(3)
Solve the equation log2 (x + 2) + log2 (3x - 4) = 4.
(in the process, the domain of the variable was extended
and we will finally have to check the resulting roots to Passing to the logarithms of a product we get log2
see if they lie in the domain). And so we have to solve (x + 2) (3x - 4) = 4 whence (x + 2) (3x - 4) =16 the roots
equation (3) and choose the roots for which x ≥ 1 and a – of this equation are x1,2 = (– 1 ± √
73)/3. It is easy to see
x2 ≥ 0. The discriminate of this equation is equal to 2a – that only x 1 lies in the required domain of the original

ww
1, so that for a <1/2 it does not have any real roots; all the
more so, the original equation has no roots for these
values of a.
equation and, on the basis of statement A, is its root.
A direct substitution of the “bad” root x1 would not

(
= 1 ±√ w
Now we assume that a ≥ 1/2; the roots of (3) are x 1,2
 )
.Ea
2a – 1 /2. The root x 2 clearly does not satisfy the
condition x ≥ 1 and so is not a root of the original
have required very cumbersome computations but there
would unpleasant enough “irrational-logarithmic” mani-
pulations, whereas the employment of statement A
yielded the answer at once.

(
the inequality 1 ± √ ) syE
equation. In order to find out about x1 we have to solve
2a – 1 /2. ≥ 1 or √ 
2a – 1 ≥ 1; It
clearly holds true a ≥ 1. And so for a<1the original
The appearance of extraneous roots as a result of
applying the fundamental logarithmic identity ordinarily
surprises the aspirant, although there is actually nothing

equation does not have any roots, but for a ≥ 1 we still


have to verify the validity of the inequality a – x12 ≥ 0 ngi strange here. It is due to the extension of the domain
when replacing the expression alogab by b if a or b contain
the unknown.
which is equivalent to the inequality a ≥≥ √
2a – 1. Both
member of this inequality are nonnegative (we are
considering a ≥ 1) and they can be squared, yielding a2 ≥
nee
Illustration 18.
Solve the equation x log√ x2x = 4.
2a – 1 or a2 – 2a + 1 ≥ 0, which is valid for all values of
a. rin
Replacing log√ x2x by logx (2x)2, we get xlogx(2x)2 = 4.

g.n
Now employing the fundamental identity, we get
And so for a < 1 the original equation has no roots, (2x)2 = 4, which means x1 = – 1, x2 = 1. But neither x 1 nor
(
but for a ≥ 1 it has the root x = 1 ± √
 )
2a – 1 /2. x2 lie in the domain of the variable of the original equa-
Note that the verification of the last condition a – x2
≥ 0, which logically speaking is obligatory, can be
conducted without any computations at all. Indeed, x1 and
x2 have been obtained as roots of the equation (x – 1)2 = a
tion: x 1 < 0, and √
e
x2 = 1 cannot be a logarithmic base.
Hence, the given equation does not have any roots.
The appearance of extraneous roots may not be so t
noticeable as in the problems given above. As a rule, this
– x2 and hence for x = x1 and x = x2 the right side is non- is due to the fact that the reasoning and computations
negative. employed lead to an extension of the domain of the
We once again stress the fact that a direct substitution variable. In the next problem, extraneous roots appear in
as a check of the roots would have reduced to equations a mutual canceling of like terms. Again there is no cause
in a : for surprise: in canceling, we remove the restriction that
the eliminated terms must be meaningful and thus extend



–
a ±√

2a – 1 a+ √
2a – 1 the domain of the variable.
– =1
2 2 Illustration 19.
the outward aspect alone of which is quite saddening. Solve the equation
Thus, without a conscious mastering of the approach
given here to the solution of equations, such problems log10 
√
1 + x + 3 log10 √

1 – x = log10 
√
1 – x2 + 2. we
can cause great difficulties. transform log10 √

1 – x2
One of the most common sources of extraneous roots log10 √

1 – x2 = log10 √

1+x
√
1 – x = log10 
√ 1 – x+
is the use of various logarithmic formulas, in particular,
the formula for taking logarithms of a product. Indeed, log10 
√
1+x

438 | CAT Complete Course

Visit : www.EasyEngineering.net
Visit : www.EasyEngineering.net

It is easy to see that this manipulation does not sin 2x


= 0
change the domain of the variable, and the transformed cos 3x cos 5x
equation log10 
√
1 + x + 3 log10 
√
1 – x = log10 √

1–x+ Now, solving the equation sin 2x = 0, we get x = kπ /
2, k = 0, ± 1,…… Now discard extraneous solutions,
log1 0
√ 1 + x + 2 is equivalent to the given one.
which is to say, those for which the denominator cos 3x
Eliminating log10 
√
1 + x in both members, we obtain the cos 5x vanishes, which obviously happens when the
equation 2 log10 
√
1 – x = 2 whose domain consists of the values of k are odd. The solution of the original equation
number x < 1, which, as is evident, is greater than that of will then be the angles x = kπ/2, where k is even: k = 2n,
the original equation. We should thus expect extraneous n = 0, ± 1, ± 2……
roots. Solving the last equation we get the root x = – 99, That is, x = nπ, n = 0, ± 1, ± 2,……
which does not lie in the domain of the original equation Quite obviously it is a grave mistake to take the set
and therefore is not its root. Thus, the given equation of values x = kπ / 2 for the answer.
does not have any roots. It is the same disregard for the domain of the variable
One of the sources of mistakes is the (explicit or that accounts for mistakes in equation solving in which
implicit) clearing of fractions. But this causes an the left-hand member has been factored and the right-
extension of the domain of the variable-those values of x hand member is zero. In solving such an equation, the

ww
are included for which the denominator is equal to 0.
Illustration 20.
aspirant ordinarily equates each factor to zero in succe-
ssion and combines the solutions obtained, completely
discarding the fact that for certain values of x which

w
Solve the equation,
1
log6 (3 + x)
+
2log0·25 (4 – x)

.Ea
log2 (3 + x)
= 1

Taking all logarithms to the base 2 and manipulating,


make one of the factors vanish the other factor may be
meaningless, and in that case such values of x will not be
roots of the proposed equation. Therefore a proper solution
requires a check to see that all the values of x obtained do
we get the equivalent equation

syE
log2 6 – log2 (4 – x)
log2 (3 + x)
= 1 …(4)
indeed lie within the domain of the variable. This can
occasionally give rise to considerable difficulties.
Illustration 22.
Whence log2 6 – log2 (4 – x) = log2 (3 + x) and then
6
= 3+x ngi Solve the equation
Sin 2x cos2 2x sin2 6x tan x cot 3x = 0
4–x
This last equation is reduced to a quadratic equation
and its roots are found to be : x1 = 3, x2 = – 2. nee Equating each factor to zero in succession, we finally
get five groups of roots :

rin
kπ π kπ π
During the solution process, extraneous roots could x = , x = (2k + 1) , x = , x = kπ, x = (2k + 1)
2 4 6 6
have appeared only due to an extension of the domain of

g.n
Where, throughout, k is any integer.
the variable because of clearing of fractions in equation
But this is not yet the answer, the point being that
(4) and (5). It is therefore sufficient to test the resulting
tan x and cos 3x are not defined for all values of x and

e
roots for membership in the domain of the variable of the
therefore many values of x in these groups may prove to

t
original equation. We thus find that x2 does not lie in the
domain, but x 1 does and hence, it is a root of the original be extraneous. Let us consider each group separately.
equation. (1) x = kπ/2. If k is even, k = 2l, then x = l π and
cot 3x is meaningless ; if k is odd, k = 2l + 1, then x = lπ +
Mistake that occur in solving equations in which the
π/2 and tan x is meaningless.
left member is a fraction and the right member is zero are
due to this disregard for the domain of the variable. Thus, not a single angle x of the first group is
Frequently, the aspirant simply discards the denominator actually a solution of the equation
in such cases and equates the numerator to zero. For a (2) x = (2k + 1)π/4. Then, as is evident, tan x is
correct solution, one should equate the numerator to zero, meaningless. Besides, 3x(6k + 3) π/4 so that cot 3x is
find the roots of the resulting equation and discard those likewise meaningful.
for which the denominator vanishes. Thus, all angles x of the second group are solutions
Illustration 21. of the equation.
(3) x = kπ/6. It is easy to see that, in the trigono-
Solve the equation tan 3x = tan 5x.
metric circle, the terminal side of the angle x coincide
Rewrite the equation as with the vertical diameter for x = 6l + 3 and hence tan x is
sin 3x sin 5x meaningful for k ≠ 6l + 3. Furthermore, 3x = kπ/2 and cot
– = 0
cos 3x cos 5x 3x has meaning only for odd values of k. Hence, the only
Whence, after a few elementary manipulations, we suitable values are odd numbers k not equal to 6l + 3, or
get numbers k of the form k = 6l + 1, k = 6l + 5.

CAT Complete Course | 439

Visit : www.EasyEngineering.net
Visit : www.EasyEngineering.net

Thus, of the angles of the third group, the following Illustration 23.
angles are solutions :
Solve the equation
x = lπ + π/6, x = lπ + 5π/6
3
Where l is any integer. log1(x + 2)2 – 3 = log1 (4 – x)3 + log1(x + 6)3
2 4 4 4
(4) x = kπ. In this case cot 3x is meaningless and so
there are no solutions. Since, log1 (x + 2)2 = 2log1 |x + 2|
4 4
(5) x = (2k + 1) π/6. By the trigonometric circle, it is
log1 (4 – x)3 = 3log1 (4 – x),
evident that the terminal side of the angle x coincides 4 4
with the vertical diameter for k = 3l + 1, and hence, tan x log1 (x + 6)3 = 3log1 (x + 6)
will be meaningless for k = 3l, k= 3l + 2. 4 4
Thus, in the fifth group we have the angles The equation takes the form
x = 1π + π/6, x = lπ + 5π/6
log1 |x + 2| – 1 = log1 (4 – x) + log1 (x + 6)
where l is any integer; these are the same angles as in 4 4 4
the third group. The final answer can be written as Whence, log1 |x + 2| = log1 (4 – x)·(x + 6)
follows : 4 4
x = (2n + 1)π/4, x = nπ + π/6, x = nπ + 5π/6, n an

ww
and consequently,
arbitrary integer, or more compactly,
4|x + 2| = 4(4 – x) (x + 6)
x = (2n + 1) π/4, x = nπ ± π/6, n an arbitrary integer.
(there is an extension of the domain in the last two

w
Acquiring extraneous roots is not always so explicit
as occurs in the last two examples. Sometimes the cause transformations and so we can expect the appearance

reasoning.
.Ea
is what would appear at first glance to be quite harmless

For example the equation tan 3x = tan 5x that was


of extraneous roots). The roots of this equation are x1 = 2,
x2 = 1 – √
33

syE
analyzed earlier is frequently solved as follows: “The
tangents of two angles are equal if and only if the
difference of the angles is equal to an integral multiple of
Extraneous roots could appear during the process of
solution only because of an extension of the domain of
the variable and so, on the basis of statement A, all we

ngi
need to do is test the values x1 and x2 for membership in
π; hence 2x = kπ, x = kπ/2, k = 0, ± 1, ± 2, … But we the domain. It is readily seen that all the expression under
know that this answer is not correct. the sign of the logarithms in the given equation for x = x1
Where does the mistake lie ?
The explanation is rather simple: the assertion on
which the solution is based is correct although it is quite nee
and are roots of the equation.
The restriction of the domain and, hence, the loss of

rin
roots can also occur when passing to a new logarithm
common among aspirants. Indeed, if tan α = tan β, then α base.
– β = kπ, where k is an integer, but the converse is not

g.n
true : if α – β = kπ, then the equation tan α = tan β may Illustration 24.
simply be meaningless (say if α = π/2, β = – π/2). Solve the equation
Therefore, in the replacement of equation tan 3x = tan 5x
by 2x = kπ, there was no loss of roots, but certain
extraneous roots appeared.
Let us now consider some sources of the loss of roots
and appropriate measures for avoiding such loss. have
e
log0.5x x2 – 14 log16x x3 + 40 log4x √
x=0
Here is a aspirant’s solution. Talking of the change-
of-base rule and taking x as new logarithmic base, we t
Aspirants most often lose roots when replacing a given logx x2 14 logx x3 40 logx √
x
equation by a new one having a more restricted domain – + =0
logx 0·5x logx 16x logx 4x
of the variable. Such a restriction (narrowing) of the
domain results, as we have already seen, from the use of But it is quite evident that the new equation is devoid
logarithmic formulas, trigonometric formulas and certain of meaning for x =1, whereas the original equation not
“verbal” reasoning. only is meaningful for x =1 but has unity as its root. This
As we have already noted, replacing the logarithms is precisely where most aspirants lose the root x =1.
of a product by a sum of the logarithmic leads to a We must therefore reason as follows : we want to
narrowing of the domain, just as does rule III which has pass to the base x; to do this we must be sure that x > 0
to do with taking the logarithm of a power. To avoid such and x ≠ 0. Since all the x of our domain are positive the
restrictions, one should employ Rule I and III instead of first condition x > 0 is satisfied; on the other hand, unity
Rules I and III. The use of the former can at worst extend lies in the domain and substitution shows that x =1 is a
the domain, that is, lead to extraneous roots. And we root. Thus, one root of the original equation has been
already know what to do with extraneous roots. found: x = 1. Now let us seek roots that differ from unity.
That is how we will solve the following problem. Then we can pass to the base x without losing roots.

440 | CAT Complete Course

Visit : www.EasyEngineering.net
Visit : www.EasyEngineering.net

From now on the solution is not difficult using the Illustration 26.
properties of logarithms and denoting logx2 by y we have
2 42 20 Solve the equation x√x = √xx .
– + =0 This equation may be rewritten as
1 – y 1 + 4y 1 + 2y
This equation is reduced to the quadratic 2y2 + 3y – 2 x√x = xx/2
= 0, whose roots are y1 = 1/2, y2 = – 2. Then we get logx2 Thus, the powers are equal and the bases are equal.
= 1/2, whence x = 4 and logx2 = – 2 , whence x = 1/ √
 2. So as not to lose any roots let us see whether the base can
be 0 or 1. Since the expression 0° is meaningless, the
Both of these values, 4 and 1/√ 2, are roots of the original number 0 is not an element in the domain set and
equation. Hence, the original equation has the roots.
therefore x = 0 is not a root of the equation. Contrariwise,
A common and very grave mistake that results in a x = 1 is obviously a root. Now let us seek roots that are
loss of roots is the canceling of a common factor from different from 0 and 1. Using the indicated rule, we
sides of an equation. It is clear that in the process, roots
obtain √ x = x/2, whence we find the second rot of the
may be lost which make the common factor vanish.
equation, x = 4.
In such cases it is best to transpose all terms to the
One sometimes hears the erroneous assertion: “If the
left side, take out the common factor and consider two
power of a number is 1, then the exponent is equal to

ww
cases: (1) the common factor is equal to zero; (2) the
common factor is not equal to zero; then of course the
expression in the brackets is zero. It is also possible to
zero.” This is only true if the base is different from 1, but
if the base is 1, then for any exponent the power will be 1.

w
consider first the case when the common factor is equal Problem involving logical difficulties
to zero and then cancel the common factor. Very considerable difficulties of a logical nature are
Illustration 25.
.Ea
Find all the solutions of the equation
ordinarily caused by equations, inequalities or systems
containing parameters, in which it is required to find the

We consider two cases. syE


x22x + 1 + 2 |x – 3| + 2 = x22|x – 3| + 4 + 2x – 1

(a) Let x ≥ 3. Here we have the equation


values of the parameters for which certain supplementary
requirements are fulfilled (say, the equation has a unique
solution or, contrariwise, is satisfied by all admissible

x22x + 1 + 2 x – 1 = x22x + 1 + 2 x – 1
Which is evidently satisfied for every x, and so in ngi values of x, or every solution of one system of equations
is a solution of another system, or every solution of one
inequality is a solution of another, and the like)
this case the solutions of the given equation will be all
values of x ≥ 3. nee This type of problem is probably the most difficult,
for it requires a high degree of logical culture. The

rin
(b) Let x <3. Then the equation takes the form student must at every step clearly realize what has been
x22x + 1 + 2 5 – x = x227 – x + 2x – 1 done and what still remains to be done, and what the
results obtained signify.
whence
2x – 1 (4x2 – 1) = 25– x (4x2 – 1)
It was precisely at this point where many of the
Illustration 27.
g.n
For what values of a does the equation 1 + sin2 ax =
aspirants at the examination were taken in by the
exponential expressions and disregarded the
“insignificant” power expression and simply cancelled
them obtaining the equation 2x – 1 = 2 5 – x . They then
cos x have a unique solution ?
e
be expressed in terms of sin x and cos x. For this reason,
the equation at hand cannot be solved by ordinary
t
It is clear that sin2 ax cannot, for arbitrary values of a,

obtained the root x =3 and, what is more, failed to noticed methods; a new idea for the solution is needed.
that it does not satisfy the condition x < 3. Due to the fact that we have the inequality cos x ≤ 1
It is abundantly clear that before canceling out 4x2 – 1 ≤ 1 + sin 2 ax, the original equation is valid if and only if
the aspirants should have considered the case of 4x 2 – 1 = 0 one of the following systems of equations is fulfilled :
then they would have found the roots x1,2 = ± 1/2 which 1 + sin2 ax = 1 sin ax = 0
also satisfy the condition x < 3. cos x = 1 or cos x = 1
Thus, the solutions of the given equation are: any We thus have to solve the last system and investigate
x ≥ 3, x1 = 1/2, x2 = – 1/2. for which values of a it has a unique solution. Since the
A common mistake made by aspirants is the original equation is equivalent to this system, the values
incorrect use of the following statement: “If two powers of a thus found will be the required values.
are equal and if their bases are equal and different from 0 Here is where the most serious logical complications
and 1, then their exponent are equal as well:” What is begin. It is precisely at this point that we see which
usually forgotten is the phrase “differently from 0 and 1”. students understands the problem and which merely
The result is a loss of roots, namely those of which the performs the manipulations without realizing what he is
base is equal to 0 or 1. doing and why it is necessary.

CAT Complete Course | 441

Visit : www.EasyEngineering.net
Visit : www.EasyEngineering.net

Here is an instance of one student’s solution of the Obviously, x = 0. is a root of the equation for any a.
system : we will demonstrate that for a irrational there are no other
k solutions and for a rational, there are. Indeed, first
“ax =πk, x = 2πn, 2aπn = πk, a = ”
2π suppose that a is irrational. From the inequality cos x ≤ 1
That and nothing else! Not a single word, merely the ≤ 1 + sin2 ax it follows that x is a solution if and only of
equation a = k/(2n) was underlined and this was the following system is satisfied :
apparently taken to mean the answer. This is no solution 1 + sin2 ax = 1 sin ax = 0
of course. cos x = 1 that is cos x = 1
We now give a real solution, which repeats the If x ≠ 0 is a solution of the last system, then, firstly,
manipulations of the preceding “solution” but is supported ax = πk, k an integer and secondly x = 2π n, n an integer,
by arguments that were lacking there. n ≠ 0. But then 2aπn = π k, whence a = k/(2n), that is to
The solutions of the first equation of the latter system say, a is a rational number, which runs counter to the
are assumption. Now let a be rational, a = p/q. Then x = 2πq
Ax = kπ, k = 0, ± 1, ± 2…… will clearly be a solution and, moreover, one different
from zero. Thus, the given equation has a unique solution
The solutions of the second equation are also
if and only if the number a is irrational.
obvious:

ww X = 2nπ, n = 0, ± 1, ± 2…
We need x such that satisfy simultaneously both
This problem can also be solved graphically. We
assume that a ≠ 0, since for a = 0 the equation clearly has
infinite of solutions let us rewritten our equation thus :

w
equations that is, we have to find the numbers k and n for
which we obtain one and the same value of x for both

.Ea
sets. Thus, we have to solve one equation in two
unknown’s n and k (and also with the parameters a) :
And denote
sin2 ax = cos x - 1

Y1 = cos x – 1, y2 = sin2ax =
1 – cos2ax

syE
2anπ = kπ …(1) 2
It is obvious that for any a the number pair n = 0, k =
0, is a solution of this equation. To it corresponds the root
x = 0. Thus, for arbitrary a the original equation has the
solution x = 0. If n ≠ 0, then equation
ngi
nee
(1) can be rewritten
A = k/(2n) …(2)
We draw both graphs on one drawing. The original
Now let us recall our basic problem : not to solve the

rin
equation clearly has a solution if and only if the graphs of
equation but merely to determine for which values of a it the functions y 1 and y 2 have a point in common. It is
has a unique solution. Now any pair of numbers k and n evident from the drawing that for arbitrary a there is a
which satisfies (2) will yields a solution of the original
equation x = 2πn = πk/a. Since for arbitrary a we have
already found one root of the original equation (x = 0), g.n
point of intersection of the graphs: x = 0. The drawing
also shows that subsequent intersection of graphs is only
we must now seek values of a for which no integers k and
n exist such that relation(2) is valid. Clearly, if a is
irrational, then no such k and n relation (2) is valid.
Clearly if a is irrational then no such k and n exist the
e
possible at points where both curves touch the x-axis, that
is, at points where, simultaneously, sin2 ax = 0 and cosx

t
=1. But sin2 ax = 0 for ax = nπ, where n = 0, ± 1, ± 2 and
cos x = 1. for x = 2kπ, where k = 0, ± 1, ± 2, … therefore
the point x = 0 will be the sole meeting point of the
first result is obtained: if a is an irrational number, then graphs only when 2akπ ≠ nπ for no non zero integers n
the given equation has a unique solution. and k. In other words, we have shown that there will be a
Is the problem solved ? Of course not, since we have unique solution only when a ≠ n/(2k), where n and k are
not yet investigated the rational values of a. However, if nonzero integers. Then, as before, it is demonstrate that
a is rational, that is a = p/q, then it can be written in the this occurs only when a is an irrational number.
form a = (2p)/(2q) and in equation (2) we get the solution In solving problems involving parameters, one often
k = 2p, n = q. Hence, in this case, with the exception of reasons as follows. Let a parameter a be some fixed
x = 0, there will at least be one solution (actually there number that satisfies the condition of the problem; such
will even be infinitely many). To summarize then: for a values of a will be called suitable. We then derive
rational the original equation has more than one solution. consequences from the statement of the problem and
The problem is solved. assumptions concerning a . We thus obtain certain
All these steps are needed so as to solve the problem conditions which the suitable values of the parameter
for ourselves and obtain the answer. This might be called must satisfy. The values of the parameter which do not
the rough solution. We now show what the final version satisfy these consequences are automatically classed as
might look like. unsuitable, and we have only to consider the values of the

442 | CAT Complete Course

Visit : www.EasyEngineering.net
Visit : www.EasyEngineering.net

parameter that satisfy the consequence obtained. In And this is true only for x = 0, y = 1. Thus, for a = 0
particular, if these consequences are satisfied solely by the given system has the unique solution of (0, 1).
certain concrete values, then the problem reduces to a We now consider the value a = 2. In this case we
verification of these values. have the system
Illustration 28. 2|x| + |x| = y + x 2 + 2
Find all the values of a for which the system x2 + y2 = 1
2|x| + |x| = y + x 2 + a As before, we note that the pair (0, – 1) is a solution
x2 + y2 = 1 and again we have to find out whether there are any other
has only one solution (a, x, y are real numbers). solutions. But by substituting x = 1, y = 0, we are assured
In accordance with the above, we first assume that a that the pair (1, 0) is also a solution of the system and,
is some suitable number, that is, a number that satisfies hence, for a = 2 the system has more than one solution.
the statement of the problem. In other words, for this To summarize then, the given system has a unique
values of a the given system of equation has exactly one solution for a = 0 alone.
solution; denote it by (x 0 , y 0 ). But it is easy to notice that The foregoing solution requires a remark or two, not
both equations of the system remain unchanged upon of a mathematical but rather of a psychological nature. As

ww
replacing x by – x, which means that the pair (– x 0 , y 0 ) is so often happens, the solution is easy to understand, but
also a solution of the system for the value under how is it found ? There is of course no cut and-dry
consideration. The original assumption, however, was answer to that.

w
that the system has a unique solution with respect to a.
There is only one way out, (x0 , y0) and (– x0, y0) are one

.Ea
and the same pair. This then simply means that x0 = – x 0 ,
that is x 0 = 0. So far this reasoning does not yields any upon
Anyone
In our solution there are three possible guesses.
Firstly, we noticed that the system does not change
replacement of x by – x. This was an essential hint.
with some idea about the evenness and oddness

syE
information about y0 . But if we put the solution (0, y0)
into the original system, we get the equations of a function and with some experience in handling
functions would realize this.
1 = y0 + a, y2 0 = 1

ngi
Secondly, we started out by working system (3) by
Whence it follows that y0 is equal either to 1 or – 1;
nonstandard procedures using inequalities. This
accordingly, a is equal either to 0 or to 2.
conjecture is somewhat more complicated, but the

nee
We have thus demonstrated that if a suitable number, examples of earlier sections showed us that it is often
then either a = 0 or a = 2. It must be stressed that the necessary to employ inequalities in equation solving.
foregoing reasoning has in no way proved that the

rin
Finally, we realized that for a = 2 the original system
numbers 0 and 2 are suitable. Quite the contrary, that is
has yet another solution: x = 1, y = 0. We therefore tried
precisely what we must now find out.
simply to pick a solution, and it worked out. This

the system
g.n
We first consider the value a = 0 in this case we have approach proved successful merely due to the existence
of “good” integral solutions. In certain cases, such a
2|x| + |x| = y + x 2
x2 + y2 = 1
If we can prove that this system has a unique
solution, then this will signify that the value a = 0
…(3) Illustration 29.
e
choice is the only possible route for solving a problem.

Find all the values of a and b for which the system


xyz + z = a
t
satisfies the condition of the problem. Note that the value
xyz 2 + z = b
a = 0 was obtained above when substituted the pair (0,1)
into the original system. It is easy to verify that this pair x + y + z2 = 4
2 2

does indeed satisfy the system (3) and thus for a = 0 the Has only one solution (a,b,x,y,z are real numbers)
original system already has one solution. Now let us find
Let (a , b ) be a suitable pair of values of the
out whether (3) have any other solutions. parameters and (x0, y 0 , z 0 ) the corresponding unique
This system is not solvable by ordinary procedures. solution. It is readily seen that the system remain
We will have to reason in a special way. From the second unchanged if, simultaneously, we replace x by – x and y
equation of the system it follows that |x| ≤ 1, |y| ≤ 1, by – y. This implies that the triple (– x0. – y 0 , z0 ) is also a
whence x 2 ≤ |x| and y ≤ 1. Beside 2|x| ≥ 1, since |x| ≥ 0. solution of the system and, as in the preceding problem,
From all these inequalties we get, we conclude that x 0 = y 0 = 0. Substituting the triple (0, 0,
2|x| + |x| ≥ 1 + x2 ≥ y + x2 z0 ) into the system, we get z0 = a, z0 = b, z0 2 = 4, whence
And, hence, the first equation is satisfied only when z0 = ± 2 and a = b = ± 2.
equality occurs in both weak inequalities; that is, when Thus, if the pair (a, b) is suitable then either a = b =
2|x| = 1, |x| = x2, y = 1 2 or a = b = – 2

CAT Complete Course | 443

Visit : www.EasyEngineering.net
Visit : www.EasyEngineering.net

Again, as in the preceding problem, we have to indeed suitable or not. When a = 0 the system is of the
establish whether these pairs of values of the parameters form
are suitable or not. For a = b = 2 we have the system (b2 + 1 )y = 1
xyz + z = 2 bxy + x2y = 1
xyz2 + z = 2 We now have to find out whether this system has any
x2 + y2 + z2 = 4 solution for arbitrary values of b. For b ≠ 0 it follows
One of the solution of which, as can readily be from the first equation that y = 0, and then the second
verified, is x = 0, y = 0, z = 2. From the second and first equation is inconsistent. Hence, the value a = 0 is not a
equation it follows that xy (z2 – z) = 0. If x = 0, then from suitable value.
the second and third equations we get z = 2 and y = 0. We Let a =1, then the system is
already know this solution. The same solution is obtained x2 + (b2 + 1)y = 1
if y = 0. bxy + x2y = 0
We will now assume that z2 – z = 0 i.e, z = 0 or z = 1. clearly, x = y = 0 for any b is a solution and so a = 1 is a
However, when z = 0 we see that the first two equations suitable value.
are contradictory, and for z = 1 we get the system Thus, the condition of the problem is satisfied by the
xy = 1

ww x + y2 = 3
2

Which, as it easy to see, have four real solutions.


unique value a = 1.
Illustration 31.
Find all the numbers a for each of which any root of

w
Thus, for a = b = 2 the original system has five solutions
and therefore the pair a = b = 2 is not a suitable one.

.Ea
Now let a = b = – 2. We have the system
xyz + z = – 2
the equation
sin 3x = a sin x + (4 – 2) |a| sin2 x
is a root of the equation
…(4)

xyz2 + z = – 2
x + y2 + z2 = 4
2
syE sin 3x + cos 2x = 1 + 2sin x cos 2x …(5)
and, contrariwise, any root of the latter equation is a root
of the former.

ngi
One of the solutions of which, as we can readily see,
The problem can more briefly be stated thus : for
is x = 0, y = 0, z = – 2. Reasoning as before, we see that
which values of a are the equation (4) and (5) equivalent?
the system does not have any other real solutions and so

nee
There are fundamentally two ways of determining the
for a = b = – 2 the original system has a unique solution,
equivalence of two equations : the first is to obtain each
which means this pair of values of the parameters is
equation from the other by means of certain manipula-
suitable.
Hence the condition of the problem is satisfied only
by the values a = b = – 2. rin
tion, the second, in accord with the definition of equi-
valence, is to prove that every root of one equation is a
root of the other, and vice-versa.
Illustration 30.
Find all the values of a for which the system g.n
In our example, the first approach is apparently
inapplicable and we have to take advantage of the second
(x2 + 1)a + (b2 + 1)y = 2
a + bxy + x2 y = 1
has at least one solution for any value of b (a, b, x, y are
real numbers).
e
approach. Here, too, however things are not so simple. It

equations which are so unlike. The only thing that can


t
is hard to reason about the coincidence of the roots of two

save us is knowledge of all these roots or the roots of at


Let a be a suitable value of the parameter, that is, a least one of the equations.
value for which the given system has at least one solution In our case, equation (5) has a simple solution and so
for any value of b. We choose some value of b; this can the problem readily reduces to the following one: for
be done in arbitrary fashion, but we will choose b so that which values of a does equation (4) have exactly the
the system takes on the simplest possible aspect. Clearly same roots as (5) ?
the best to choose is b = 0. Then the system looks like this To simplify computations, denote sin x by y. then (5)
(x2 + 1)a = 1 becomes
a + x2y = 1 2y2 - y = 0 …(6)
and since a is a suitable value, the system has at least one This equation has the roots y1 = 0, y2 = 1/2.
solution, which we denote by (x0 , y0). Similarly, upon replacing sin 3x = 3y – 4y2, equation (4)
In this solution, x 0 is either zero or non-zero. if x0 = 0 becomes
then from the second equation we get a = 1 and x0 ≠ 0 [4y2 + (4 – 2|a|)y + a – 3] y = 0 …(7)
then x02 + 1 ≠ 1 and from the first equation we get a = 0. Many aspirants replaced sin x by y and this “helped”
Thus, if a is a suitable number then either a = 0 or them to make two serious mistakes. Thus, many decided
a = 1. Now we have to determine whether these value are immediately that the required values of a do not exist

444 | CAT Complete Course

Visit : www.EasyEngineering.net
Visit : www.EasyEngineering.net

since equation (6) is quadratic and equation (7) is cubic chosen from among only three numbers : 0, 1 and – 1 the
and, hence, they are not equivalent because they have task now is to verify all three values.
different numbers of roots. This argument contains two Let a = 0, then the equation will assume the form
mistakes at once. Firstly, a quadratic and a cubic equation sinx (sin2 x – 1) (2sin4x + 2sin2 x + 1) = 0
can be equivalent (for instance, the equations x2 = 0 and
sin 2 x (sin2 x – 1) (sin2 x +1) = 0
x3 = 0 both have the unique root x = 0) and, secondly, we
will see for ourselves below, (4) and (5) can be equivalent Since 1 + sin x > 0 and 2 sin4 x + 2 sin2 x + 1 > 0,
2

even if (6) and (7) are not. these equations are equivalent.
There lies the second mistake. At first glance it Let a = 1, then the equations can be rewritten as
would appear quit obvious that our problem was reduced sinx (2 sin6 x - 1) = 0 and sin2x (2 sin4x – 1) = 0
to the following : for which values of a does not equation 6
Since the first equation has the solution sin x = √
1/2,
(7) have only the root 0 and 1/2. But actually, if we recall
which does not satisfy the second equation, these
that y = sin x, we can indicate yet another possibility for
equations are not equivalent,
the value of a to be suitable: if (7) has the roots 0, 1/2,
and its third root y3 is greater than unity in absolute value, Let a = – 1. We then have the equations
then (4) and (5) are equivalent because the corresponding sin x(2 sin6x – 2 sin 2 x – 1) = 0
value sin x = y3 will not given equation (4) any additional sin 2 x (2sin4 x – 3) = 0

ww
and
solutions. Naturally, the equation (4) and (5) are equi- Since 2 sin x – 3 < 0 and 2sin6 x – 2 sin2 x – 1 = 2
4

valent when the third root of (7) is equal to 0 or to 1/2. sin x(sin4 x – 1) – 1 < 0, it is clear that the equations are
2

w
Our problem is now clear: we have to find values of
a such that (7) has the roots 0, 1/2 and its third root is

.Ea
either 0 or 1/2 or exceeds 1 in absolute value.
It is evident at once that 0 is a root of equation (7) so
equivalent. Thus, the condition of the problem is satisfied
only by a = 0 and a = – 1. In this problem too, many
aspirants replaced sin2x and y and could not figure out
what to do with the value a = – 1, since in the inequalities

syE
that have to be proved in this case, essential use is made
that we will henceforth consider the equation
of the fact that 0 ≤ y ≤ 1.
4y2 + (4 – 2|a|)y + a – 3 = 0 …(8) 7. Find all number pairs a, b for which every number
One of the roots of this equation must be 1/2. pair x, y (x ≠ π/2 + kπ, y ≠ π/2; k, n = 0, ± 1, ± 2,………)

a = |a|, or a ≥ 0. By Viete’s theorem, the second root is ngi


Substituting y = 1/2 into it, we find that 1/2 is a root when that satisfies the equation x + y = a also satisfies the
equation,
equal to (a – 3)/2 and according to what has just been
said, the value of a will be suitable in the following three
cases : nee tan x + tan y + tan x tan y = b
Let a and b be a suitable pair of values of the
parameters. We take a number pair x = 0, y = a which
…(11)

(1)
a–3
2
= 0, (2)
a–3 1
2
= ,
2 | |
(3)
a–3
2
>1
rin
clearly satisfies the equation x + y = a. If a ≠ π/2 + nπ,
then this pair satisfies the restrictions imposed on x and y
(also bear in mind that a ≥ 0).
We than have the answer : g.n
in the statement of the problem and, for this reason, by
virtue of (11), the equation tan a = b must be valid. Let us
now take the number pair x = π/4, y = a – π/4, must also
a = 3, a = 4, 0 ≤ a < 1, a > 5
Illustration 32.
Find all the numbers a such that for every root of the
e
satisfies the equation x + y = a. If a ≠ 3π/4 + kπ, then this
pair likewise satisfies the restrictions on x and y and
therefore (since a and b are assumed to be suitable) the
following equation must hold :
t
equation
π
2 sin7 x – (1 – a) sin3x + (2a3 – 2a – 1) sin x = 0 …(9)
is a root of the equation
( )
1 + 2 tan a –
4
= b …(12)

Since b = tan a, then a thus satisfies the equation


2 sin6 x + cos 2x = 1 + a – 2a3 + a cos2x …(10) π
and, contrariwise, every root of the second equation is a ( )
1 + 2 tan a –
4
= tan a
root of the first equation. Which can readily be reduced to the quadratic
Here, both the given equations are complicated and equation tan 2 a – 2 × tan a + 1 = 0. Hence, tan a = 1 and
so we cannot proceed as in the preceding problem. But the suitable pairs a , b must be sought among the
we can note that equation (9) has solution of the form x = infinitude of pairs of the form
kπ, where k is any integer, and perhaps some other π
solutions. This remark will enable us to solve the problem. a = + mπ, b = 1, m = 0, ± 1, ± 2, ……
4
Let a be a suitable value of the parameter. Then the Let us determine which of these pairs are actually
values x = kπ – roots of (9) – are roots of (10), and this π
suitable. Let x + y = + mπ for some integer m, and x ≠
immediately yields the equation a3 = a (since sin6 kπ = 0, 4
cos2 kπ = 1). Therefore, the suitable values are to be π/2 + kπ, y ≠ π/2 + nπ, tan x + tan y + tan x tan y

CAT Complete Course | 445

Visit : www.EasyEngineering.net
Visit : www.EasyEngineering.net

follows the inequalities – 2 ≤ x ≤ 2. However, the method


= tan x + tan (π4 + mπ – x) + tan x tan (π4 + mπ – x) of solution applied in the preceding problem is not
π π suitable here mainly because we have to do with
= tan x + tan ( – x) + tan x tan ( – x) inequalities and not with equations.
4 4
In this problem we have to determine for which
1 – tan x 1 – tan x
= tan x + + tan x values of a all the solutions of given inequality lie in the
1 + tan x 1 + tan x
interval – 2 ≤ x ≤ 2. If a ≠ 0, then the given inequality is
The last expression is equal to unity. Thus all pairs a quadratic, and we first consider this general case. Thus,
= π/4 + mπ, m = 0, ± 1, ± 2, …, b = 1 are suitable. let a ≠ 0, we know that the solutions of a quadratic
However, the solution is not yet complete since in inequality, if they exist, form on the number line either a
the course of our discussion we excluded the values a = finite interval, two infinite intervals, or the entire set of
π/2 + nπ and a = 3π/4 + kπ, k, n = 0, ± 1, ± 2,… it remain real numbers; and this depends on the signs of the
to consider these values as well. discriminate and the leading coefficient. We therefore
Let a = π /2 + n π , n an integer. In this case, compute the discriminate of the quadratic trinomial in the
obviously, a ≠ 3π/4 + kπ and therefore (12) must be valid, left member of the inequality :
from it follows, for the values of a under consideration, D = (1 – a2 )2 + 4a2 = a4 + 2a2 + 1 = (a2 + 1)2

ww
that b = 3. We will attempt to find suitable pairs among
π
a = + nπ, b = 3, n = 0, ± 1, ± 2, ……
2
Thus, D > 0 for any a, so that the roots of the
trinomial are real and distinct and easily found: x1 = a, x2
= – 1/a.

w π
π
The pairs x = – ,y = 3π/4 + nπ satisfies the equation
4

.E(a) ( )
x + y = a; on the other hand,
π
Now, depending on the sign of the number a, the
solutions of the given inequalities form an interval
between roots (for a < 0 when the parabola is concave

( ) ( 3π
) 3π downwards) or two infinite intervals (for a > 0).
tan –
4
+ tan
4
syE
+ nπ + tan –
4
tan
4
+ nπ
= – 1 ≠ 3 and therefore there are no suitable pairs among
the number pairs a, b under consideration.
By hypothesis, we need values of a for which all
solutions of the inequalities lay in the interval – 2 ≤ x ≤ 2.
Therefore the values a > 0 are not suitable: two infinite

ngi
Now let a = 3π/4 + kπ, k an integer. Since in this
case a ≠ π/2 + nπ, the equation tan a =b must be true, i.e.,
intervals cannot fit into a finite interval. It remains only to
consider the values a < 0. In this case, x1 < 0 < x 2 and the

a=

4 nee
b = – 1. We wish to find suitable pairs among the pairs

+ kπ, b = – 1, k = 0, ± 1, ± 2, ……
solution of the given inequality is the interval a < x < –
1/a.
We want the entire interval a < x < – 1/a to lie in the
The pair x = 3π /8, y = 3π /8 + kπ satisfies the
equation x + y = a on the other hand, rin
interval – 2 ≤ x ≤ 2 and this occurs obviously if and only
if the endpoints of the first interval lie on the interval – 2

tan

8
+ tan (3π
8 )
+ kπ + tan tan

8 ( 3π
8 )
+ kπ if the inequalities
g.n
≤ x ≤ 2. (coincidence of endpoints is admissible), that is,


= 2 tan + tan2
8

8
>0

(because the angle 3π/8 lies in the first quadrant) and so


the left member of (11) is different from – 1 and thus
–2≤a<– ≤2
e 1
a
Hold true. From the inequality – 1/a ≤ 2, taking into
account that a < 0, we get a ≤ – 1/2 and, hence – 2 ≤ a ≤ t
– 1/2.
there are no suitable pairs among the pairs a, b under
consideration. Thus, any solution of the complete the solution we
The final answer is this: the condition of the problem have not exceed two in absolute value when – 2 ≤ a ≤ –
is satisfied by infinity of pairs 1/2, but we obtained this on the assumption that a ≠ 0. To
complete the solution we have yet to consider this special
π
a = + mπ, b = 1, m = 0, ± 1, ± 2,…… case. For a = 0, the original inequality assumes the form x
4
> 0 and not all its solutions fail to exceed two in absolute
Illustration 33. value so that the value a = 0 is not a suitable value. To
Find all the values of a for which any values of x that summarize, then, the inequality obtained above – 2 ≤ a ≤
satisfies the inequality ax2 + (1 – a 2 ) x – a > 0 does not – 1/2 is the final answer.
exceed two in absolute value. Illustration 34.
In its logical form, the statement of this problem is Find all the values of a for which, for all x not
quite analogous to that of the preceding problem. exceeding unity in absolute value, we find the inequality
Namely, it is required to find the values of the parameters ax – a(1 – a)
a for which from the inequalities ax2 + (1 – a2 ) x – a > 0 a2 – ax – 1 > 0 to be valid.

446 | CAT Complete Course

Visit : www.EasyEngineering.net
Visit : www.EasyEngineering.net

First replace the given inequality by the equivalent From 1 – a < 1 we have a > 2 and then from (a2 –
but more customary quadratic inequality (ax + 1 – a2) [ax 1)/a > 1 follows (a2 – 1) > a, or a 2 – a – 1 > 0. The
– a(1 – a)] < 0 solutions of this inequalities are the values a < (– 1 –
We have been a bit hasty in calling this inequality  5)/2 and a > (1 + √
√  5)/2. Since a > 2, we only leave the
quadratic, since we have not yet checked to see if, when values a > 2. As in Case (a), these values automatically
the brackets are removed, the coefficient of x 2 is different satisfy condition (b). Thus, the condition of the problem
from zero. This coefficient is equal to a2 and is zero when is satisfied by the values a < (– 1 – √
 5)/2 and a > 2.
a = 0, but for a = 0 the given inequality takes the form 0
< 0, which is to say that it does not hold for any x. Exercises
Therefore, a = 0 is not a suitable value and we discard it,
considering henceforth a ≠ 0 everywhere. 1. Three cyclists start out simultaneously from the same
place in one direction around a circular track 1 km in
We will solve the resulting quadratic inequality by
length. The rates of the cyclists form, in a certain
following the same ideas as in the preceding problem. We
order, an arithmetic progression with common
see at once that the roots of the trinomial are real and so
difference 5 km/hr. After some time, the second one
the discriminate need not be computed. Beside, the
catches up with the first, having made one extra
leading coefficient a2 is positive and, hence, the solutions

ww
of the quadratic inequality form an interval between its
roots x1 = (a2 – 1)/a, x2 = 1 – a, if these roots are distinct.
But if the roots coincide, the quadratic inequality is not
circuit; 4 minutes later the third arrives at that point,
having covered the same distance that the first did at
the time he caught up with the second cyclist. Find

w
satisfied for a single value of x and therefore the

.Ea
corresponding values do not interest us.
We thus need the values of a for which the entire
interval – 1 ≤ a ≤ 1 lies between the number a – 1/a and 1
their rates.
2. Three brothers, whose ages form a geometric
progression, divide among themselves a certain sum
of money in direct proportion to the age of each. If

syE
– a. But in order to wire this geometric condition in the
language of inequality, we have to know which of the two
numbers is greater. This clearly depends on the number a,
this were done in three years time, when the
youngest becomes one-half the age of the oldest,
then the youngest would receive 105 dinar, and the
and so we consider two cases.
(a) (a2 – 1)/a < 1 – a. As in the preceding problem, ngi middle one, 15 dinar more than at the present time.
Find the ages of the brothers.

for the interval – 1 ≤ a ≤ 1 to lie entirely within the


interval (a2 – 1)/a < x < 1 – a, it is necessary that the
endpoints (-1 and 1) lie in the interval; thus, the following
nee3. Two groups of tourists start out from A in the
direction of B at the same time. The first group goes
by bus (at 20 km/hr) and reaches C, midway between
inequalities must be valid :
a2 – 1 rin
A and B, and then continues on foot. The second
group starts out on foot, but in one hour boards a car

a
<–1<1<1–a

(Coincidences of extreme points, that is, the equation g.n


which proceeds at 30 km/hr and reaches B. The first
group passes C 35 minutes before the second group,
but arrives at B 1 hr and 25 minutes later than the
a – 1/a = – 1 and 1 – a = 1 are not admissible, since, for
instance, when a – 1/a = – 1, the number – 1 in the
interval – 1 ≤ x ≤ 1 is exterior to the interval – 1 ≤ x ≤ 1 –
a.)
e
second group. What is the distance from A to B if the
rate of the first group (on foot) is 1 km/hr greater
than that of the second group ? t
4. Two identical vessels are filled with alcohol. We
From the inequalities 1<1 – a it follows that a < 0, draw off a litres of alcohol from the first vessel and
and then from the inequality (a2 – 1)/a < – 1 we get a2 – 1 add that amount of water. Then we draw off a litres
> – a or a2 + a – 1 > 0. The solution of this inequality are of the resulting mixture of water and alcohol and add
the values of a < (– 1– √ 5)/2 and a > (– 1 – √ 5)/2, Since that amount of water. In the case of the second
a < 0, we only leave the values a < (– 1 – √  5)/2. Now, vessel, we draw off 2a litres of alcohol and add that
from the resulting values of a we have to choose which amount of water, and then draw off 2a litres of the
satisfy condition (a), i.e., the inequality a – 1/a < 1 – a. resulting mixture and add that amount of water.
But this condition is automatically satisfied for a < (– 1 – Determine what part of the volume of the vessel is
taken up by a litres if the strength of final mixture in
√ 5)/2. Indeed the indicated values are obtained as the first vessel is 25/16 times the strength of the final
solutions of the inequalities a – 1/a < – 1 < 1 – a. mixture in the second vessel. (By strength is meant
(b) 1 – a < (a 2 – 1)/a. in this case we have to solve the ratio of volume of pure alcohol in the mixture to
the inequalities the total volume of the mixture. It is assumed that the
a2 – 1 volume of the mixture is equal to the sum of the
1–a<–1<1<
a volumes of its components.)

CAT Complete Course | 447

Visit : www.EasyEngineering.net
Visit : www.EasyEngineering.net

5. Two bodies are in uniform motion around a circle in number of 4's is divisible by 10 and the number of 5's
the same direction, and one of them catches up with is even. Determine the number of the various marks
the other every 46 seconds. If these bodies were in received by the thirty aspirants.
motion in opposite directions, they would meet every 12. A motorcycle and a car (Volga) leave A for B at the
8 seconds. Determine the rates of the bodies if we same time; and at the Same time another car
know that the radius of the circle is equal to 184 cm. (Moskvich) leaves B for A. the Moskvich arrives in
6. Towns A and B are located on a river, B downstream A in 5 hours and 50 minutes. The cars meet 2 hours
from A. At 9 A.M. a raft starts floating downstream 30 minutes later, and the motorcycle and Moskvich
from A in the direction of B (the rate of the relative meet 140 km from A. If the rate of the motorcycle
to the bank of the river is the same as the rate of the were twice what it was, it would have met the
current). At that time, a boat starts out from B for A Moskvich car 200 km from A. Find the speeds of the
and meets the raft in 5 hours. Upon reaching A, the motorcycle and the two cars.
boat round and returns to B arriving at the same time 13. An empty tank is being filled through two pipelines
as the raft. Did the boat and raft at B by 9 P.M. (of simultaneously with pure water and a constant
the same day) ? concentration of an acid solution. When filled, the
7. Three workers receive an assignment, which each tank has a 5% solution of the acid. If when the tanks

ww
separately completes in a specified time, the third
completing the job one hour faster than the first.
Working together they can complete the job in one
were half full, the water supply were cut off, the full
tank would have a 10% solution of the acid.
Determine which pipeline delivers liquid faster and

w
hour. But if the first worker does one hour and then
the second 4 hours, together they can complete the

.Ea
job. How long does it take each worker separately to
complete the full assignment ?
14.
how much faster ?
A car leaves point A for B. At the same time, a
cyclist starts out in the opposite direction (from B).
Three minutes after they meet, the car turns around

syE
8. We have two solutions of a salt in water. To obtain a
mixture containing 10 grams of salt and 90 grams of
water, one takes twice as much (by weight) of the
and follows the cyclist; after catching up with the
cyclist, it turns around and goes to B. If the car had
turned around 1 minute after the meeting, and the
first solution as the second. One week elapses and
200 grams of water has evaporated from each
ngi cyclist (after the meeting) had increased his speed
15/7 times, the car would have spent the same amo-
unt of time for the entire trip. Find the ratio of the

nee
kilogram of the first and second solution. Now to
obtain the same mixture as before, we require four speeds of the bicycle and the car.
times more (by weight) of the first solution than of 15. Two men start out at the same time from A in the

rin
the second. How many grams of salt did 100 grams direction of B, which is 100 km from A, one on a
of each solution originally contain ? bicycle, the other on foot. Also at the same time, a
car starts out from B and goes in the direction of A.
9. A freight train going from A to B arrives at station C
together with a passenger train going from B to A
with a speed m times that of the freight train. The g.n
An hour later the car meets the cyclist and,
2
continuing another 14 km, it meets the man on
two trains stop at C for of hours and then continue on
their ways, each train increasing its 25% over the
original speed it had prior to arrival at C. Then the
freight train arrives at B later by t1 hours and the
17

e
foot, who boards the car, and the car overtakes the
cyclist. Compute the speeds of the bicycle and the
car if we know that the man on foot was going 5 t
km/hr. The time required to get into the car and turn
passenger train arrives at A later by t2 hours than if
the car around is disregarded.
they had gone nonstop at their original speeds. How
16. A laboratory has to order a certain number of
much earlier did the freight train start out from A
identical spherical flasks with a total capacity of 100
than the passenger train from B ?
litres. The cost of one flask consists of the cost of
10. Three points A, B and C are connected by straight labour, which is proportional to the square of the
roads. Adjoining section AB of the road is a square surface area of the flask, and the cost of the material,
field with a side of 1/2 AB; adjoining BC is a square which is proportional to the surface area. A flask of
field with a side equal to BC, and adjoining CA is a capacity 1 litre costs 1 rouble 25 kopeks, and in this
rectangular section of the woods of length equal to case the labour cost is 20% of the cost of the flask
AC and of width 4 km. The wooded area is 20 square (the wall thickness of the flask is negligible). Will
kilometres greater than the sum of the areas of the 100 rubal (100 kopeks to a rubal) be enough to cover
square fields. Find the area of the woods. the cost ?
11. Thirty aspirants received marks of 2, 3, 4, 5 at a 17. Bus No. 1, which a aspirant uses to get to his place of
competition. The sum of the marks is 93, there are study (Institute of Perfection, Hardwar) without
more 3's than 5's and fewer 3's than 4's. Besides, the changing buses, covers the distance in 2 hours and 1

448 | CAT Complete Course

Visit : www.EasyEngineering.net
Visit : www.EasyEngineering.net

minute. He can also get to the institute by anyone of a skating circuit. After a time, the second one
the buses No.2, 3, ... , No. K, but the only way to overtakes the first, covering 400 metres more. The
make a change to Bus No. P is from Bus No. (P – 1). third skater covers the same distance that the first did
The routes of these buses are such that if the aspirant when the latter was overtaken by the second during a
gets to the institute on one of them, he will spend en time that is 2/3 minutes more than the first. Find the
route (disregarding transfers) a time inversely rate of the first skater.
proportional to the number of buses used. Moreover, 23. A farm has tractors of four models, A, B, C, D. Four
he will have to spend 4 minutes at each transfer. Is tractors (2 of model B and one of models C and D)
there a route he can take such that the total time is plough a field in two days. Two model A tractors and
less than 40.1 minutes ? one model C tractor take three days to do this job,
18. Between town A and city E is a petrol pump 0 and a and three tractors of models A, B and C take four
water supply station B, which divide the distance days. How long will it take to do the job if a team is
between A and E into three equal parts made up of four tractors of different models?
(AO=OB=BE). A motorist and cyclist start out 24. Grass was mowed on three fields in the course of
simultaneously from A in the direction of E and, at three days. On the first day, all the grass of the first
the same time, a truck starts out from E in the field was moved in 16 hours. On the second day, all

ww
direction of A and at the water supply station passes the grass of the second field was mowed in 11 hours.
the car and at the petrol pump passes the cyclist. At On the third day, all the grass was cut on the third
the petrol pump, the cyclist increases his speed 5 field in 5 hours, four hours of which the work was

w
km/hr. The motorist reaches E and then sets out on done with scythes and one hour by a moving
the return trip at 8 km/hr slower than before. As a

.Ea
machine. During the second and third days, together,
result, when the truck arrives in A, the cyclist still four times mare grass was cut than on the first day.
has 7.5 km to go, to B, while the motorist is in Haw many hours was the mowing machine in
between 0 and A, 14 km from O. Find the distance

car, truck and bicycle. syE


from the town to the city and also the speeds of the
operation if in one hour it mows five times as much
grass as is cut by hand. It is assumed that the hand
and machine operations were separate in time (did

ngi
19. A rectangular plot of area 900 square metres is to be not overlap) and there were no breaks in the work.
fenced in; two adjoining sides to be brick and the two
25. A factory has to deliver 1100 parts to a client. The
others to be wooden. One metre of the wooden fence

nee
parts are packed in boxes of three types. One box of
casts 10 dinar and one metre of the brick fence costs
type one holds 70 parts, one of type Two holds 40,
25 dinar. A total of 2000 dinar has been allotted for
and one of type Three holds 25 parts. The cast of
the job. Will this sum be sufficient?
20. The tank at a water supply station is filled with water
by several pumps. At first, three pumps of the same rin
delivery in a type one box is 20 dinar, in a type two
box, 10 dinar, in a type three box, 7 dinar. What kind

g.n
of boxes should be used in order to minimize the cost
capacity are turned on; 2.5 hours later, two more of delivery? All boxes must be used to full capacity.
pumps (both the same) of a different capacity are set
26. A stamp collector decides to put all his stamps in a

e
into operation. One hour after the additional pumps
new album. If he puts 20 stamps on one sheet, there
were set into operation the tank was almost filled to
capacity (15 cubic metres were still lacking), in
another hour the tank was full. One of the two
additional pumps could have filled the tank in 40
hours. Find the volume .of the tank.
will be some left over; if he puts 23 stamps on a

album. If the stamp collector is presented with an


album of the same kind, each sheet holding 21
t
sheet, there will be at least one empty sheet left in the

stamps, he will have a total of 500 stamps How many


21. At a 10,000-metre ski race, the first skier starts out
sheets are there in the album?
and is followed shortly by a second one, the rate of
the second skier being 1m/sec more than that of the 27. Two pipelines operating together fill a pool 3/4 full
first one. When the second skier catches up with the of water If one pipeline fills one-fourth of the pool
first, the latter increases his rate by 2 m/sec, while first, and then the second (the first is then switched
the rate of the second skier remains unchanged. As a off) brings the volume of water up to 3/4 the capacity
result, the second skier finishes 7 minutes and 8 of tile pool, then this will require 2.5 hours. Now if
seconds after the first one. If the distance had been the first pipeline is in operation for one hour and the
500 metres longer, then the second skier would have second for half an hour, they will bring the water
finished 7 minutes and 33 seconds after the first one. level up to more than one-half the pool. How long
Find the time lapse between the start of the first and will it take each pipeline separately to fill the pool?
second participants. 28. Points A and B are located on a river so that a raft
22. Three skaters whose rates, taken in same order, form floating downstream from A to B with the rate of the
a geometric progression, start out at the same time on current covers the distance in 24 hours. A motorboat

CAT Complete Course | 449

Visit : www.EasyEngineering.net
Visit : www.EasyEngineering.net

goes from A to B and returns in less than 10 hours. If 9. 5m (t – t2 ) – 5m– 1 (t – t1)


the rate of the motorboat in Still water were 40% 10. The area of the forest is 40 km2 . Obtain the equation
greater, the same distance (from A to B and back) AC = 5 + 1/4BC2 + 1/16 AB2 from the statement of
could be covered in not more than 7 hours. Find the the problem; besides for any three points A, B and C
time it takes the motorboat to go from A to B at the the inequality AC ≤ AB + BC is valid, whence 5 +
original rate (not increased). 1/4 BC2 + 1/16 AB 2 ≤ AB + BC or (1/2BC – 1)2 +
29. At 8 A.M. a fast train leaves A for B. At the same (1/4AB – 2)2 ≤ 0, which is possible only for AB = 8
time a passenger train and an express train leave B and BC = 2.
for A, the speed of the former being one half that of 11. The number of marks 2, 3, 4 and 5 are equal,
the latter. The fast train meets the express train not respectively, to 11, 7, 10 and 2.
earlier than 10:30 A.M. and arrives at B at 13:50 the 12. Velocities are : motorcycle, 40 km/hr, Moskvich car,
same day. Find the time of arrival of the passenger 60 km/hr and Volga car, 80 km/hr
train at A if we know that not less than an hour
13. The water is delivered twice as fast
elapsed between the meetings of the fast train and
express train and the fast train and the passenger 14. 1 : 3
train. 15. 20 km/hr and 80km/hr

ww
30. At 9 A.M. a cyclist starts out from A in the direction
of B. Two hours later a motorist sets out and
overtakes the cyclist not later than 12:00 noon. The
16. No
17. No
18. The rate of cyclist is 20 km/hr, that of the truck, 40

w
motorist continues on and reaches B, then
immediately turns round and heads back for A. On

.Ea
the way, the motorist meets the cyclist and arrives in
A at 17:00 hours that same day. When does the
km/ hr, of the Volga car 80 km/hr. The distance from
A to D is 60 km.
19. No
20. 60 m3

syE
cyclist arrive in B if we know that no more than 3
hours elapsed between the two encounters of the
cyclist and motorist?
21. 2 minutes
22. 0.6 km/min.

1.
Answer
20 km/hr; 25 km/hr; 15 km/hr ngi 23. 12/7 days.
24. 12 hours
25. Four Boxes of the third type and 25 boxes of the
2.
3.
27, 18 and 12 years old
30 km nee second type.
26. 12 sheets
4.
5.
1/6
19π cm/sec and 27π cm/sec
rin
27. The first pipe will fill the pool in 2 hours, the second
in 4 hours
6.
7.
No
3 hours, 6 hours, 2 hours
28. 4 hours
29. 16 hours and 45 minutes
g.n
8. 5 grams and 20 grams 30. 18 hours

e t
●●●

450 | CAT Complete Course

Visit : www.EasyEngineering.net
Visit : www.EasyEngineering.net

PART–II : VERBAL

ww
w.Ea
syE
ngi
nee
rin
g.n
e t

Visit : www.EasyEngineering.net
Visit : www.EasyEngineering.net

ww
w.Ea
syE
ngi
nee
rin
g.n
e t

Visit : www.EasyEngineering.net
Visit : www.EasyEngineering.net

Part A : English Verbal

1 Structure of Sentence
5. Interrogative (Question) Sentences—Such sen-
A. Sentences – Types and Interchange tences inquire about something. They ask, what ? Where
1. The art of arranging words, phrases and clauses in ? When ? Why ? Who ? Whose ? How ? Etc.
correct order so that they make complete sense is called a As,
sentence. The arrangement of such words, phrases and
Where are you working ?
clauses in a systematic and proper manner is called the

ww
structure of sentence.
2. Every sentence consists of two parts—
What are you doing ?
Do you know me ?
When are you returning ?

w
(a) Subject
Why do you sleep so much ?
(b) Predicate

.Ea
The word or words about which we say something, is
called the subject.
Who are you ?
Whose book is this ?

of the sentence.
For Example—
syE
What is said about the subject is called the Predicate
How are you ?
How far is Jaipur ?
6. Imperative Sentences—A sentence that exp-

Raj

killed a tiger.

ngi resses a command, request or wish is called an imperative
sentence. They are also called optative sentences when
they express a wish, prayer or curse.
Subject Predicate
Raj is the person the sentence is saying something nee As,
Close the door please. (Request)
about; therefore, ‘Raj’ is the subject. What is said about
the subject (Raj) ‘killed a tiger’. This then is called the
rin
Don’t walk on the grass. (Probation)
Stand to attention. (Command / Order)
Predicate of the sentence.
3. There are Four types of sentences—
g.n
May you live long? (Wish / Optative)
Wish you a happy birthday. (Wish)
(a) Assertive
(b) Interrogative
(c) Imperative/ Optative
(d) Exclamatory
e
Note—In sentences 1, 2, 3 the subject is understood.
Close the door please = You close the door.
7. Exclamatory Sentences—Sentences which
express strong emotion or feeling or reaction in con-
t
nection with a statement are called exclamatory sen-
4. Assertive/Statement (Affirmative or Negative) tences.
Declarative Sentences—
As,
When a statement that gives sense information or
What a goal ! (Surprise)
description whether in the negative or positive it is called
an Assertive Sentence. Bravo ! What spirit. (Joy)
As, How strong he is ! (Surprise)
He is a very handsome man. (Positive) How cute ! (Joy)
Harry can never be depended upon. (Negative) Alas ! We are ruined. (Sorrow)
Honesty is the best policy. (Positive) What a pity! (Regret)
She is not working properly. (Negative) How dare he! (Anger)
A positive sentence does not contain negative words Tut! Tut! (Disapproval)
such as, ‘not’, ‘never’, ‘hardly’, ‘seldom’, ‘rarely’ etc. Such sentences or words are followed by
Whereas, a negative sentence utilizes such words. exclamation marks (!).

CAT Complete Course | 453

Visit : www.EasyEngineering.net
Visit : www.EasyEngineering.net

8. Structure Sentences—To structure a sentences As,


one must be familiar with Auxiliary Verbs, Subject, Main
Verbs, Object (Direct and Indirect) and other words.
8. (a) Auxiliary Verbs—Words (Auxiliary) of the
Present, Past and Future Tense.

Present Subject Main Verb


(Continuous)
Am I
Are You, we, they (Plural) V1 + ing
Is Name, he, she, it (Singular)
(Indefinite)
Do I, you, we, they (Plural)
Does Name, he, she, it (Singular) V1
(Perfect continuous)
Has been Name, he, she, it (Singular)

ww
Have been
(Perfect)
Has
I, you, we, they (Plural)

Name, he, she, it (Singular)


V1 + ing + for / since

Have
8(b) w Past
.Ea
I, you, we, they (Plural)

Subject
V3

Main Verb
Was
Were
(Indefinite) syE You, we, they (Plural)
Name, he, she, it, I (Singular)
V1 + ing

Did
(Perfect continuous)
Had been
All subjects

All subjects ngi * 1 V1

V1 + ing + for / since


(Perfect)
Had All subjects
nee V3 + (V2 )

rin
8(c)
Future Subject Main Verb
Will
Shall
You, we, they, Name, he, she, it
I, we
g.n
V1

e
(Continuous)
Will be
Shall be
(Perfect continuous)
Will + have + been
You, we, they, Name, he, she, it
I, we

You, we, they, name, he, she, it,


V1

V1 + ing + for *2
t
Shall + have + been I, we
(Perfect)
Will have You, we, they, name, he, she, it, V3 + * 3 (V1 , or + s, es, ies)
Shall have I, we

Note— Remember,
*1 In case of the affirmative the second form of the verb The Present Indefinite in affirmative statements or
is utilized. answers add s, es or ies to the first form of the verb
*2 The Perfect Continuous Future utilizes only ‘for’ according to the nature of the word.
(period of time) and not ‘since’ as it is in the future As,
and hence since (point of time) is not applicable.
He runs. (V1 + s)
*3 The Future Perfect tense indicates an action that will
have finished before another starts and is followed by She goes to school. (V1 + es)
the Present Indefinite with affirmative statements add Tom replies to all my letters. (V 1 + ies)
s, es or ies to the first form of the verb.

454 | CAT Complete Course

Visit : www.EasyEngineering.net
Visit : www.EasyEngineering.net

* However, in Interrogative and negative sentences As,


Do, Does + (not) or simply V1 Not is negative. 1 + 2 + not + 3
9. Structure of Interrogative (Question) Sen- I am not going
tence—Helping Verb (Aux.) + Subject + Verb + others ?
Recall—
9(a) Do You read books?
Question = 2 + 1 + 3?
Formula—
Positive Answer = Yes + 1 + 2 + 3
It becomes simpler if you follow and understand it in
Negative Answer = No + 1 + 2 + not + 3
an easy mathematical formula.
Positive Statement = 1 + 2 + 3
Think of the subject as 1, the Auxiliary verb as 2 and
the main verb as 3. Got it. Negative Statement = 1 + 2 + not + 3
Now consider this. 10. Question Words—The interrogative sentence
may begin with (a) an auxiliary verb (b) a main verb or
To form a question use the formula. 2 + 1 + 3, and
(c) a question word.
you have created a question. 2 stands for the Auxiliary
Verb, 1 stands for its subject and 3 stands for the main As,
verb. (a) Is he coming ? (Auxiliary Verb)

ww
The above formula changes its auxiliary verb and
main verb according to the time frame required. Viz.
(b) Has he a car ? (Main Verb)
(c) Where are you going ? (Question word)

w
Present, Past and Future. There are basically nine question words.
As, They are—
(a) 2
Do
+ 1
you.Ea + 3?
exercise? (Present)
1. What
2. Where
(b) 2
Did
+ 1
he syE
+ 3?
play ? (Past)
3. When
4. Why
(c) 2
Will
+ 1
they
+ 3?
talk ? (Future)
ngi 5. Who
6. Whom (with)
There isn’t it easy just remember 1, 2, 3 and you can
structure an Interrogative sentence. Not to mention the
assertive (positive and negative) which follows. nee 7. Which
8. Whose
9. How
How to make a question? Easy, 2 + 1 + 3? And
you’re got a question. Simple, isn’t it? Lets move on to rin
11. The question word structure is simple—You
now know that the formula for a question is 2 + 1 + 3.
the assertive (positive answer / statement / Declaration)
formula. It’s easy as 1, 2, 3.
g.n
However, while making an interrogative sentence with a
question word place the question word/s before 2.
9(b) The formula for the assertive is as shown below:
Positive Answer = Yes + 1 + 2 + 3
(Are you going ?) = Yes, I am going
There, by remembering 1, 2, 3 and placing yes before
Example—
QW
Where
Q.word
+ 2
do
Aux.
+
e 1
you
Sub.
+ 3?
live ?
M.Verb
t
it you have a positive answer. To make a positive state- 12. (a) What (Pronoun / Adjective)—Inquires about
ment, simply remove the ‘yes’. persons / things etc.
As, (b) Where (Adverb)—Inquires about place.
1 + 2 + 3 (c) When (Adverb)—Inquires about time.
I am going (d) Why (Adverb)—Inquires about reason.
9(c) To make a negative sentence (answer) (e) Who (Pronoun)—Inquires about persons.
Use, (f) Whom (Pronoun)—Inquires about persons
(Object).
No + 1 + 2 + not + 3
(g) Which (Pronoun / Adjective)—Inquires about
No I am not going persons or things when options are limited.
Just remember, No, 1, 2 not 3, and you have a (h) Whose (Pronoun / Adjective)—Inquires about
negative answer. possession (Subject or object)
If you desire to create a negative statement or (i) How (Adjective / Adverb)—To find out method-
declaration simply remove the ‘no’ from the formula. ology, health, distance, route, size etc.

CAT Complete Course | 455

Visit : www.EasyEngineering.net
Visit : www.EasyEngineering.net

13. How + Adjective / Adverb— (c) Who are you ?


1. How many—(Countable) number. (d) Who saw me cheating ?
2. How much—Quantity (Quantitative–Uncount- (e) Who told a lie ?
able) (f) Who stole the cream ?
3. How far— Distance 14. (f) Examples of the question word ‘Whom /
4. How long— Duration With whom’—
5. How often— Frequency (a) Whom do you consider the best ?
6. How quickly—Time span (b) With whom is she going ?
Also (c) Whom do you know that speaks Greek ?
7. At what point—(exact place) (d) Whom are you talking to ?
8. At what time—(exact time) 14. (g) Examples of the question word ‘Which’—
9. What brand of / sort of / etc. (variety) (a) Which book are you reading?
14. (a) Examples of the Question word ‘What’— (b) Which of these mangoes do you want?
(c) Which movie is being shown?

ww
(a) What do you play ?
(b) What does your father do ? (d) Which coat do you like?
(c) What is the time ? (e) Which actor is your favourite?
(d)
(e)
(f)
w What is the matter ?

.Ea
What is he doing ?
What can you see ?
(f) Which day is auspicious?
* Which is generally followed by a noun or a noun
phrase.
(g)
(h)
14.
What did she ask ?

syE
What are you doing this evening ?
(b) Examples of the Question word ‘Where’—
14. (h) Examples of the question word ‘Whose’—
(a) Whose watch are you wearing ?

(a)
(b)
Where is my pen ?
Where are your brothers staying ? ngi (b) Whose books are those ?

(c) Whose house are you living in ?

(c) Where is Iceland ?


(d) Where can I find a petrol pump ? nee (d) Whose orders are you following ?

(e) Whose cell phone is that ?


(e)
(f)
Where is the railway station ?
Where do you put your shoes ? rin
(f) Whose money are you spending ?

14.
(a)
(c) Examples of the Question word ‘When’—
When does Aunty visit you ? g.n
* ‘Whose’ like ‘Which’ is followed by a noun.
14. (i) Examples of the question word ‘How’ :

(b) When will the train arrive ?


(c) When shall I see you again ?
(d) When did the police catch the thief ?
e
The question word ‘How’ covers a wide range of
areas when attached to an adjective or adverb.
Single use of ‘How’—
(a) How are you ? (Health / well being)
t
(e) When do these flowers bloom ? (b) How did you fix it ? (Methodology / way)
(f) When does Hari exercise ? (c) How do you reach the station ? (Route)
14. (d) Examples of the question word ‘Why’— How + Adjective
(a) Why are you leaving ? (a) How far is Delhi ? (Distance)
(b) Why is she sad ? (b) How handsome is your brother ? (Degree of
(c) Why are they rejoicing ? Beauty)
(d) Why do bird fly and not humans ? (c) How tall is Ravi ? (Height)
(d) How much oil do you use ? (Quantity
(e) Why did she resign ?
uncountable)
(f) Why are they quarrelling ?
(e) How many spoonfuls of sugar do you take ?
14. (e) Examples of the question word ‘Who’— (Number countable)
(a) Who is calling ? (f) How long have you been in Haridwar ?
(b) Who is your father ? (Duration)

456 | CAT Complete Course

Visit : www.EasyEngineering.net
Visit : www.EasyEngineering.net

(g) How often do you visit Raj ? (Frequency) 7. How ……………… is the river ? (Distance)
(h) How many times have I warned you ? 8. How ……………… petrol is there ? (Quantity)
(Frequency) 9. How ……………… books do you have ?
Now, that you have an idea and a certain grasp of (Number)
question words test your skill by doing the following
10. How ……………… your sister is! (Beauty)
exercises.
Note—All words except question words and words
Exercise—1 of quality and number a wide range of options.
Fill in the blanks with the appropriate Question Exercise—4
words—
Fillers—
1. .………………. are you doing ?
1. .……………… will you buy me a present ?
2. .……………….. is she going ?
3. .……………….. school do you study in ? (Question Word)
4. .………………. far is the railway station ? 2. .……………… watch are you wearing ?
5. .………………. pen is this ? (Question Word)

ww
6. .………………. are you talking to ?
7. .………………. beautiful the mountain look ?
3. .……………… are they doing ? (Question Word)
4. .……………… is that boy ? (Question Word)

w
8. .………………. close are we to Delhi ?

.Ea
9. .………………. is the doctor coming ?
10. .………………. is the matter ?
5. With ……………… are you going ?
(Question Word)
6. .……………… hotel are you staying in ?
Exercise—2

syE
Fill in the blanks with appropriate Adjectives
or Adverbs—
7. .……………… old are you ?
(Question Word)
(Question Word)

ngi
8. .……………… are you watching ?
1. How ……………… boys are in the class ? (Question Word)
2. How ……………… money do you have ?

nee
9. .……………… are you laughing ?
3. How ……………… can you count ?
(Question Word)
4. How ……………… is America ?
10. .……………… is the child crying ?
5. How ……………… your brother is !
6. How ……………… the crow is ! rin (Question Word)

7. How ……………… can you come ?


8. How ……………… he has become ! 1. g.n
Exercise—5
……………… fast can you run ?
9. How ……………… to your house does he live ?
10. How ……………… from your house, is he ?
Note—The answer page will give you an idea of the
2.
3.
4.
e
……………… do you think you are going ?
……………… is the computer not working ?
……………… does the shop close ? t
kinds of words you can use. But, it should suit the sense 5. ……………… house is being sold ?
of your question or exclamation. 6. ……………… is the matter ?
Exercise—3 7. ……………… stupid she is !
Fill in suitable words : 8. How ……………… knowledge do you have ?
1. How ……………… the rises. (Adverb of time) 9. How ……………… litres of milk did you buy ?
2. Whose ……………… have they stolen ? 10. ……………… met your father in the market ?
(Common Noun) Answers
3. .……………… have you done ? (Question Word) Exercise—1
4. Which ……………… has been selected ? 1. What 2. Where 3. Which 4. How 5. Whose 6.
(Common Noun) Who 7. How 8. How 9. When 10. What
5. .……………… is the bus arriving ? Exercise—2
(Question Word) 1. Many 2. Much 3. Much/many 4. Far 5. Handsome
6. .……………… is my bag ? (Question Word) 6. Clever 7. quickly 8. thin 9. close 10. far

CAT Complete Course | 457

Visit : www.EasyEngineering.net
Visit : www.EasyEngineering.net

Exercise—3 Case D—If so/too used before adjective, then


articles can be used before or after to adjective. Like—So
1. early 2. watch 3. what 4. girl 5. when 6. where 7. serious an attempt or a so serious attempt.
deep 8. much 9. many 10. beautiful
Rules of Articles
Exercise—4
Rule 1—The Article ‘An’ is used in the following
1. When 2. Whose 3. What 4. Who 5. Whom 6.
cases:-
Which 7. How 8. What 9. Why 10. Why
Rule 1(a)—Before words beginning with an open
Exercise—5
vowel which is sounded as open vowel.
1. How 2. Where 3. Why 4. When 5. Which 6. What
Illustrationsı—An ass; An apple; An umbrella; An
7. How 8. much 9. many 10. Who
owl; An ear;
B. ARTICLES - Types of Articles Exception—But if an open vowel is not sounded as
A, AN and THE are the three articles of two types in open vowel we used ‘a’ before such vowel- A useful
the English Language. book; a university; a one rupee note.
(i) Indefinite Article – ‘A and An’ Rule 1(b)—Before a word beginning with consonants

ww
(ii) Definite Article- ‘The’
Which one is right, wrong, preferable and acceptable
which sounded as open vowels.
Illustrations—An MBA, An SP, An FIR, An LLB.
Rule1(c)—Before word beginning with silent ‘H’

w
1. Horse is a faithful animal.
2. A horse is a faithful animal (A horse = Any Illustrations—An hour; An heir; An honour; An
horse)
.Ea
3. An horse is a faithful animal
honest; An honorary
NB:- Incorrect An Humble; Correct: A Humble

horse)
syE
4. The horse is a faithful animal (The horse = All (Due to its present pronounce as Hotel etc.)
Rule 2—The Article ‘A’ is used in the following
cases—
Sentence
1. is right and acceptable
ngi Rule 2(a)—Before a verb which is used as a noun
Illustrations—You should have a rest now; He has
2. is right and preferable
3. is wrong and unacceptable here ‘H’ sounded as
consonants nee
gone for a walk
Rule 2(b)—Before a common noun in the singular to
suggest the sense of one.
4. is right and preferable
In 1st sentence Horse is used in its widest sense. In rin
Illustrations—Wheat sells twelve rupees a kilo;
Wait for a minute.
2nd sentence Horse used as a common noun and it is
work of representative of class mean ‘Any Horse’ Hence g.n
Rule 2(c)—Before abstract noun when we use
abstract noun as a kind of quality.
omission of article before horse is writing and the sense
of 4th sentence described the quality of whole class.
Mean each horse is a faithful animal.
So, it is clear that in Standard English the use of
grammar. e
Illustrations—He has a working knowledge of Thai

Rule 2(d)—Before the common noun in the singular t


Articles play a vital role to describe the meaning of sen- to suggest the sense of any.
tences. For better understanding we present rules of the Illustrations—A bird has wings (a bird = any bird)
articles. Rule 2(e)—Before a common noun in singular to
Position of Articles suggest the sense certain.

Case A—Articles are used before noun like – a Illustrations—While going to church, I saw a thief
book, the day, an apple; but if noun is use with adjective being beaten by the police.
then article should placed before adjective. Like—a Rule 2(f)—Before a proper noun when it is used as
good library. common noun.
Case B—Always use article after many/such/what Illustrations—He thinks. He is a Hercules. He is a
not before these specific words. Like—many a man; Milkha Singh.
such an event; what a song. Rule 2(g)—Before a uncountable noun when it is
Case C—If as/how is used before adjective then used as a countable
articles should be used after adjective. Like—How good Illustrations—I have a good news for you. I have a
a book; How nice a girl you are; As good a show as that. milk bar.

458 | CAT Complete Course

Visit : www.EasyEngineering.net
Visit : www.EasyEngineering.net

Rule 2(h)—Some more sense are define by ‘a’ are as (iv) Before the nationality, community or political
follows— party
Sense of each—we observe holiday once a week. The Indian, the Hindus
Sense of same—Elephant and mouse are of an age. (v) Before the unique things
Some idiomatic phrases—To make a five, a piece of The sun, the moon, the air, the sea,
advice, I had a headache, a dozens eggs. (vi) before the name of river sea, ocean, mountain
Rule 3—The definite Article ‘The’ is used. ranges, group of islands, famous buildings, ships, famous
Rule 3(a)—Before a particular person or things, or boots and news papers
one already referred to or known as the speaker. The Taj, The Times of India, The Ramayana,
Bring a ticket from the station. (only station of city) The West
Let us go to the institute. (both or all are the students (vii) Before the name of some profession
of same institute) The author, the poet, the press.
Rule 3(b)—When a singular noun is meant to (viii) Before the name of invented things
represent a whole class. The radio, the barometer.

ww
The horse is a faithful animal.
The cow is a useful animal.
Rule 3(c)—Before a common noun to give it the
(ix) Before the name of post
The director will decide the matter.

w
He is the president and CEO of the ganga gloves
force of superlative. maker. (mean one persone holding two offices)
This is a thing to do
.Ea
He is the leader (he is the best leader)
The president and the CEO of the company are
twins. (mean two different person with different

meaning of an abstract noun.


syE
Rule 3(d)—Before a common noun to give it the

The mother ( parented affection) in her pitied the


posts )
(x) In certain fixed idiomatic phrases.
In the town; Off the mark; In the worry; At the
child.
He is the lover of the good, the pure and the ngi time
Rule 4—Omission of articles
beautiful.
Rule 3(e)—Before a common noun instead of
possessive adjectives— nee Rule 4(a)—Before name of disease, names of regular
meals seasons, games, language, colour and name of
something single kind.
He caught her by the arm. (instead of by her arm)
Rule 3(f)—When the thing or noun is understood rin
Fever, consumption. Like—she is suffering from
fever.
We should help the poor.
Kindly return the book. g.n
But when the diseases are plural in their form, the
article is not omitted. Like—The measles is a contagious

Rule 3(g)—With superlative


She is the wiser of the two
He is the greatest philosopher of the world.
disease.
—Breakfast, lunch, dinner
e
—This summer is very hot in comparison to last
summer
t
Rule 3(h)—As an adverb with a comparative —I play chess
The sooner, the better. —He does not know Thai that but he knows French
The more you eat, the fatter you become. —I like pink and yellow
The higher you go the colder is it. — Hell, heaven, god, parliament (but the pope, the
Rule 3(i)—I Some more uses of ‘the’ Before devil are exceptions)
(i) Circus/ picture/ theater/ cinema/ office/ station/ Rule 4(b)—Before such as are plural in their sense
bus stop—station though singular in form.
he is at the office Furniture, scenery, cattle, gentry, business, society,
(ii) Before the name of Musical instruments. folk, people, mankind
he plays the Violin; the sitar; the table Rule 4(c)—Before a noun used in its widest sense
(iii) Before the name of clubs or foundations. Birds’ fly, Dogs have very keen noise
Dr. Ramesh is a member of the JRD Young Rule 4(d)—‘The’ is not used before market, library,
Club institute, church, hospital, school, prison, table, bed,

CAT Complete Course | 459

Visit : www.EasyEngineering.net
Visit : www.EasyEngineering.net

temple, parliament, and university etc, when these places On the ground floor of this Rs.750 crore company, in a
are visited or used for their primary purpose. glass cabin overlooking the work stations of 100
—He went to library (it means that he went to library associates, 42 year old Anoop Thataj, Joint managing
to study or for books) Director and CEO of the company, is busy discussing the
new spring collection for a US customer, Finally, after
—They went to the library and from there they hours of discussions, a few cuts, silhouettes and fabrics
joined the rally. (it means their purpose was thought of are short–listed. Then the design team of around 100,
rather than actual building) along with a support staff of 700, begins work on rolling
Rule 4(e)—Before relatives (like brother, sister, out the products. Say Anoop Thataj. “The team has to
father, nurse, or cook) days, months and material complete the projects in the next 14 days. Then we begin
—Father has returned from the office work for a major European retail brand. I am running at
full capacity. Besides manufacturing prototypes, we are
—Sunday is a holiday developing our own design lines”.
—Gold is a precious metal For Orient Craft, it has been an eventful journey, for,
Rule 4(f)—Before abstract noun that express quality, just 10 years back it was manufacturing apparel for
feelings, action or processes of thought international clients with little value addition. But the

ww
Honesty is the best policy
Rule 4(g)—Before a noun following the expression
‘kind of’
company has climbed up the value chain. Says Sudhir
Dhingra, Chairman and Managing Director, Orient Craft:
“ Out of the 65% woman’s wear produced by us, almost

w
What kind of suit do you like?

.Ea
But if kind of/ sort of indicate any quality/ capacity then
use of a / an is must.
40% have our own design input and we produced 2,000
design samples a day. This differentiates from competition
and certain clients get back to us for particular designs.”
The design element in the apparels and accessories

syE
—What kind of an artist is she?
Rule 4(h)—Some more places where articles are
prohibited
industry – apparel alone is a Rs.30,000 crore market–has
risen by almost 80%. Graduating from assembly line
operations for Western labels, Indian design firms are
(i) When noun in pairs
—Door to door, neck and neck, brother & sister, face ngi now creating there own lines based on strong in–house R
and D capabilities. Says Devangshu Dutta, Chief Execu-
tive, Third Eyesight, a Delhi based fashion consulting
to face
(ii) When the post is only one
nee
firm. “The days of cut, copy, paste are coming to an end
as every exporter looks for a distinct image. This is
possible only if you innovate in design.”

rin
—he has been appointed headmaster
—he was appointed as a teacher (because teacher can While big export houses like Orient Craft are enhancing
their business by emphasizing on design, international

g.n
be more than one)
firms are looking at India as an outsourcing hub. This is
(iii) When noun follows by preposition spawning many start–ups, such as Bangalore–based
—on demand, by hand, by train, by courier, on foot,
on earth
(iv) When objects are followed by transitive verb
—To send word, to cast anchor, to lose heart, to
e
Munch Design and Delhi–based Bricolage, which are

t
developing lines of apparel and accessories for global
brands like Nike, Reebok, esprit, Adidas, Zara, Guess,
Macy’s and Gucci. Says Narinder Mahajan, Founder,
leave home Bricolage: “Clients depend on us for forecasts and trends.
Right from deciding on the theme–based collection
(v) Before some title or name names to the final sampling, every thing is done by us.”
—Emperor Ashoka, President APJ Kalam, Captain Bricolage is now developing a casual clothing division
George. for Reebok and a range of shirts and Tees for Benton.
Exercise—1 A Cut above the Rest
In each of the following passage there are Articles, Design in apparel as a key differentiates comes at a
each of which has been bold and italic. Identify the premium. According to industry sources, a prototype
rule of article by which you can justify its position. consignment of 10,000 shirts to the US would cost $ 10
VEERING to the left of the Hero Honda roundabout per shirt. But with elements of design like embroidery,
in Manesar, Gurgoan is a nondescript road. A 100 meters embellishments and cuts, the same shirt would cost $20
down this path stands a building, which symbolizes the or more. Says Vijay Agarwal, President of Apparel Export
rising global acceptance of India’s fashion design industry Promotion Council–India’s strength is design; where is
–the 3,50,000–square feet designing and manufacturing mass producer. We need to balance the two–numbers and
unit of Orient Craft, one of India’s largest export houses. design innovations–for enhancing exports.”

460 | CAT Complete Course

Visit : www.EasyEngineering.net
Visit : www.EasyEngineering.net

However, not just exports, the design elements are future as Indian consumers get more in sync with global
slowly creeping into the lives of domestic consumers too. trends. Indian companies will have to compete in design,
No more the plain shirts for the Indian male. The choice branding and retail. I see this as the key differentiate of
has widened to embroidered, pleated, crushed, crystal– the future.” Madura Garments owns brands like Allen
laden and metallic shirts. For women though, there’s Solly, Allen Solly Womens, Peter England, Van Heusen,
practically no end to the need for choice. SF Jeans, Louis Philippe, Byford, Elements and San
Homegrown companies like Pantaloon and Madura Frisco. “We try to balance fashion, Innovations and
garments are busy satiating the design needs of Indian Commercial logic,” he adds.
consumers. Says Hemchandra Jaweri, Senior Executive Pantaloon Retail too offers a variety of apparel
President, Madura Garments lifestyle Brands and retail: and accessories targeted at men, women and kids. Says
“The importance of design will be further heightened in 43 year old Kishor Biyani, MD of pantaloon.
Discussion—Rules of Articles
STANZA–1
Sentence–1 VEERING to the left of t h e Hero Honda roundabout in Manesar, Gurgoan is a Rule 3(i)-x; Rule 3
nondescript road. (i)-vi; Rule 2(b)

ww
Sentence–2 A 100 meters down this path stands a building, which symbolizes the rising global Rule 2(b); Rule 2(e);
acceptance of India’s fashion design industry – the 3,50,000 – square feet designing and Rule 3(c); Rule 3(i)-x
manufacturing unit of Orient Craft, one of India’s largest export houses.

w
Sentence–3 On the ground floor of this Rs.750 crore company, in a glass cabin overlooking the work

.Ea
stations of 100 associates, 42 year old Anoop Thataj, Joint managing Director and CEO
of the company, is busy discussing the new spring collection for a US customer, Finally,
after hours of discussions, a few cuts, silhouettes and fabrics are short - listed.
Rule 3(i)-x;2(e); Rule
3(a); Rule 3(a); Rule
3(c); Rule 2(e); Rule
2(d)
Sentence–4

Sentence–5
syE
Then the design team of around 100, along with a support staff of 700, begins work on Rule 3(i)-x; Rule 2(b);
rolling out the products.
Say Anoop Thataj. “The team has to complete the projects in the next 14 days.
Rule 3(a)
Rule 3(a); Rule 3(a);

Sentence–6 ngi
Then we begin work for a major European retail brand.
Rule 3(i)-x
Rule 2(e)
Sentence–7
Sentence–8
I am running at full capacity
nee
Besides manufacturing prototypes, we are developing our own design lines”.
NA
NA

STANZA–2
rin
Sentence–1
g.n
For Orient Craft, it has been a n eventful journey, for just 10 years back it was Rule 1(a)
manufacturing apparel for international clients with little value addition chain.
Sentence–2
Sentence–3
But the company has climbed up the value chain.

e Rule 3(a); Rule 3(i)-x


Says Sudhir Dhingra, Chairman and Managing Director, Orient Craft: “Out of the 65% Rule 3(a); Rule 2(d)
women’s wear produced by us, almost 40% have our own design input and we produced
2,000 design samples a day.
t
Sentence–4 This differentiates from competition and certain clients get back to us for particular NA
designs.”
STANZA–3
Sentence–1 The design element in the apparels and accessories industry – appel alone is a Rs.30,000 Rule 3(a); Rule 3(i)-
crore market – has risen by almost 80%. x; Rule 2(b)
Sentence–2 Graduating from assembly line operations for Western labels, Indian design firms are NA
now creating their own lines based on strong in – house R & D capabilities.
Sentence–3 Says Devangshu Dutta , Chief Executive, Third Eyesight, a Delhi based fashion Rule 2(b)
consulting firm.
Sentence–4 “The days of cut, copy, paste are coming to an end as every exporter looks for a distinct Rule 3(i)-x; Rule
image. 1(a); Rule 2(e)
Sentence–5 This is possible only if you innovate in design.” NA

CAT Complete Course | 461

Visit : www.EasyEngineering.net
Visit : www.EasyEngineering.net

STANZA–4

Sentence–1 While big export houses like Orient Craft are enhancing their business by emphasizing Rule 1(a)
on design, international firms are looking at India as an outsourcing hub.

Sentence–2 This is spawning many start - ups, such as Bangalore – based Munch Design and Delhi NA
– based Bricolage, which are developing lines of apparel and accessories for global
brands like Nike, Reebok, esprit, Adidas, Zara, Guess, Macy’s and Gucci.

Sentence–3 Says Narinder Mahajan, Founder, Bricolage: “Clients depend on us for forecasts and NA
trends.

Sentence–4 Right from deciding on the theme – based collection names to the final sampling, every Rule 3(i)-x; Rule 3(i)-x
thing is done by us.”

Sentence–5 Bricolage is now developing a casual clothing division for Reebok and a range of shirts Rule 2(b); Rule 2(b)
and Tees for Benton.

STANZA–5

ww
Sentence–1 Design in apparel as a key differentiates comes at a premium. Rule 2(b); Rule 2(b)

w
Sentence–2 According to industry sources, a prototype consignment of 10,000 shirts to the US Rule 2(b); Rule 3(i) –iv
would cost $ 10 per shirt.

Sentence–3
.Ea
But with elements of design like embroidery, embellishments and cuts, the same shirt Rule 3(a)
would cost $20 or more.

Sentence–4
syE
Says Vijay Agarwal, President of Apparel Export Promotion Council – India’s strength Rule 2(b)
is design; where is a mass producer.

Sentence–5
ngi
We need to balance the two – numbers and design innovation – for enhancing exports.”

STANZA–6
Rule 3(a)

Sentence–1
nee
However, not just exports, the design elements are slowly creeping into the lives of Rule 3(a); Rule 3(a)
domestic consumers too.

Sentence–2 No more the plain shirts for the Indian male.


rin Rule 3(a); Rule 3(a)

Sentence–3

g.n
The choice has widened to embroidered, pleated, crushed, crystal – laden and metallic Rule 3(a)
shirts.

Sentence–4

Sentence–1
For women though, there’s practically no end to the need for choice.

STANZA–7 e
Homegrown companies like Pantaloon and Madura garments are busy satiating the Rule 3(a)
Rule 3(a)

t
design needs of Indian consumers.
Sentence–2 Says Hemchandra Jaweri, Senior Executive President, Madura Garments lifestyle Rule 3(a)
Brands and retail: “The importance of design will be further heightened in future as
Indian consumers get more in sync with global trends.
Sentence–3 Indian companies will have to compete in design, branding and retail. NA
Sentence–4 I see this as the key differentiate of the future.” Rule 3(a)
Sentence–5 Madura Garments owns brands like Allen Solly, Allen Solly Womens, Peter England, NA
Van Heusen, SF Jeans, Louis Philippe, Byford, Elements and San Frisco.
Sentence–6 “We try to balance fashion, Innovations and Commercial logic,” he adds. NA

STANZA–8

Sentence–1 Pantaloon Retail too offers a variety of apparel and accessories targeted at men, women Rule 2(e)
and kids. Says 43 year old Kishor Biyani, MD of pantaloon.

462 | CAT Complete Course

Visit : www.EasyEngineering.net
Visit : www.EasyEngineering.net

Exercise –2 respect to the allotment of land and also for the allotment
In each of the following passage there are Articles, of iron ore. Once these issues are resolved, we will start
each of which has been bold and italic. Identify the work on the construction site”. He added that there was
rule of article by which you can justify its position. work in progress on a detailed project reports. “We
expect that this to be ready in 18 months.”
Lakshmi mittal is keen to invest over $ 15 billion in
India. Mittal, of course, knows he isn’t the only one in
EVEN AS KASHMI MITTAL goes about the chal- expansion mode. Large players like Tata Steel, Essar
lenging task of making synergies work for Arcelor Mittal, Steel, Jindal Steel & Power and SAIL have already
the president and CEO of the world’s largest steel com- announced expansion plans. “We expect the total capacity
pany hasn’t lost track of his plans for manufacturing in in the steel sector to be at around 90 – 100 million tonnes
India. Mittal is looking to set up two Greenfield projects by 2015(from the current 40 million tonnes),” says Mittal.
in India in the states of Jharkhand and Orissa. The total Meanwhile, Mittal also gave an update on the
investment across these projects is expected to be in Arcelor–Mittal merger and said the objective was to
excess of $ 15 billion (Rs.61, 500 crore). complete it as soon as possible during the course of 2007.
Announcing the first quarter results of Arcelor “The integration process has been in line with our plans
Mittal, Mittal in a conference call with the media said and there have been savings from synergies to the extent
that discussions and negotiations were in progress with of $ 573 million (Rs.2, 349.3 crore) during the first
the two state governments in India. Responding to a quarter of 2007. This is against our expectation of $ 500

ww
query from BT on the states of these too much talked
about projects, Mittal said: “The negotiations are with
million (Rs.2, 050 crore),” he told the media. Mittal isn’t
called a steel magnate for nothing.

w
Sentence–1

.Ea
STANZA–1
EVEN AS KASHMI MITTAL goes about the challenging task of making synergies Rule-3(i)-x Rule-3(i)-
work for Arcelor Mittal, the president & CEO of the world’s largest steel company
hasn’t lost track of his plans for manufacturing in India.
x Rule3(g)

syE
Sentence–2 Mittal is looking to set up two Greenfield projects in India in the states of Jharkhand and Rule – 3(i) –iv
Orissa.
Sentence–3 The total investment across these projects is expected to be in excess of $ 15 billion Rule- 3(i)-x
(Rs.61, 500 crore).

ngi
STANZA–2

nee
Sentence–1 Announcing the first quarter results of Arcelor Mittal, Mittal in a conference call with Rule-3(i)- x Rule -2
the media said that discussions and negotiations were in progress with the two state
(e) Rule -3(b) Rule- 3
governments in India.
(i)- iv
Sentence–2

rin
Responding to a query from BT on the states of these too much talked about projects, Rule- 2(e) Rule- 3(i)-
Mittal said: “The negotiations are with respect to the allotment of land and also for the
allotment of iron ore.
x Rule 3(a) Rule-3(a)

Sentence–3 Once these issues are resolved, we will start work on the construction site”.
g.n Rule-3(a)
Rule- 3(a)
Sentence–4
Sentence–5
He added that there was work in progress on a detailed project reports.
“We expect that this to be ready in 18 months.”

STANZA–3
e Rule- 2(e)

t
Sentence–1 Mittal, of course, knows he isn’t the only one in expansion mode. Rule- 3(i)-iv
Sentence–2 Large players like Tata Steel, Essar Steel, Jindal Steel & Power and SAIL have already
announced expansion plans.
Sentence–3 “We expect the total capacity in the steel sector to be at around 90 – 100 million tonnes Rule- 3(b) Rule- 3(b)
by 2015(from the current 40 million tonnes),” says Mittal.
Rule- 3(i)-iv
STANZA–4
Sentence–1 Meanwhile, Mittal also gave an update on the Arcelor – Mittal merger and said the Rule-3(a) Rule-3(a)
objective was to complete it as soon as possible during the course of 2007.
Rule-3(i)-x
Sentence–2 “The integration process has been in line with our plans and there have been savings Rule- 3(i)-x Rule-3(i)-
from synergies to the extent of $ 573 million (Rs.2, 349.3 crore) during the first quarter
x Rule-3(i)-x
of 2007.
Sentence–3 This is against our expectation of $ 500 million (Rs.2, 050 crore),” he told the media. Rule- 3(i)-iv
Sentence–4 Mittal isn’t called a steel magnate for nothing. Rule- 2(b)

●●●

CAT Complete Course | 463

Visit : www.EasyEngineering.net
Visit : www.EasyEngineering.net

2 Parts of Speech
1. In order to become adept in Grammar one must (c) He spotted a plant bug. (Here ‘plant’ is used as
develop a strong grounding in the basic rules and appli- an adjective)
cation of English grammar.
NOUN
First and foremost, the student must be well versed in
A Noun is a naming word (derived from Latin nomen
the Eight Parts of Speech.
which means name.)
Words are slotted into different kinds or classes in
accordance to the purpose for which they are utilized. Such as, tree, house, car, woman, Raj, Ravi, Sita,
These are called parts of speech and they consist of 8 Gold, Silver, Hatred, Anger, Team, bunch etc.

ww
parts in all.
1. Noun
PRONOUN
A Pronoun (the second part of speech) is used in the

w
2. Pronoun
3. Adjective
4. Verb
5. Adverb .Ea
place of a noun.
It refers to a noun, an individual/s or thing/s whose
identity has been determined earlier in a sentence/the text.
Consider these examples—
6. Preposition
7. Conjunction syE me.
(a) I met a boy on the way. He was pleased to meet

ngi
8. Interjection (Here ‘he’ refers to ‘a boy’ and works as a pronoun)
As, (b) My uncle and aunt are avid readers. They love to
Horse Noun read whenever they can.
The Horse
The Horse stands
Article + Noun
Article + Noun + Verb nee (Here ‘they’ refer to ‘uncle and aunt’ and works as a
pronoun)
The Horse stands Article + Noun + Verb +
firmly. Adverb
rin
In the first sentence, the pronoun ‘he’ is used instead
of repeating ‘boy’. In the second example, the pronoun
The Horse stands firmly Article + Noun + Verb +
on the Hill. Adverb + Preposition +
Article + Noun g.n
‘they’ is used in the place of ‘uncle and aunt’ instead of
repeating uncle and aunt over and over again.

The black Horse stands Article + Adjective +


firmly on the Hill. Noun + Verb + Adverb +
Preposition + Article +
Noun
ADJECTIVE

e
An Adjective is a qualifying word. That is, it adds
something to the meaning of a noun. It is also called a
describing word. t
commonwealth 2010 Live
It stands on the Hill. Pronoun + Verb + Prepo-
sition + Article + Noun Adjectives usually precede a noun.
Since it stands on the Conjunction + Pronoun + As,
hill it overlooks the Verb + Preposition + (a) The tall man with black hair.
plain. Article + Noun + Pronoun (b) A beautiful flower has many hues.
+ Verb + Article + Noun
It is also used as the object of a noun or pronoun.
It is quite often very difficult to say what part of
speech a word belongs to. Unless and until we see the As,
placement of the word in a sentence as it can assume (a) Ram is tall.
different roles based on its usage in a sentence. (b) That flower is beautiful. Its hues are many.
Consider these sentences— VERB
(a) Plant the tulips (Here ‘plant’ is used as a verb) A Verb is an action word. It does some-thing. Nouns
(b) Fetch some plants. (Here ‘plants’ are used as a names things; persons things etc. Verbs predicate or say
Noun) something about the Noun or Pronoun.

464 | CAT Complete Course

Visit : www.EasyEngineering.net
Visit : www.EasyEngineering.net

As, INTERJECTION
(a) A group of boys played football. (Here it tells us An Interjection, so to speak, is not a part of speech as
what the group did) it has no grammatical connection with any word or words
in a sentence.
(b) He walked into the room. (Here it tells us what
‘he’ (pronoun) did) It is simply an exclamatory sound used in a sentence
to indicate strong emotion or feeling.
(c) She dances well. (Here the action of the pronoun
An Interjection is involuntary and unplanned and
‘she’ is expressed)
expresses strong emotion that overtakes one suddenly. It
ADVERB can be of joy, sorrow, admiration, disgust or disapproval.
An Adverb modifies the verb; it indicates how the Some examples,
action of a verb is carried out. Joy – Hurrah ! (We won)
As, Grief – Alas ! Oh ! (We lost)
(a) The horse stands firmly. Amusement – Ha ! Ha ! (Good joke)
(b) She speaks well. Approval – Bravo ! (Well done)

ww
(c) He dresses beautifully .
It can also modify an adjective or another adverb.
Contempt
Ridicule
Disapproval



Bah !
Bosh !
Tut ! Tut !
(How stupid)
(How stupid)
(Bad boy)

w
The house is very firm.
To call – Hi ! Hello !
She answered most considerately
PREPOSITION
.Ea A Closer look at Noun
1. A noun is a naming word. It is employed to name

syE
A Preposition connects a noun (with or without an
article) or a pronoun to some other word.
As,
a person or thing.
A noun consists of 5 kinds or parts.

(a) It stands on a hill.


(b) Australia is over the sea. ngi They are as follows—
1. Proper Noun

(c) She told the good news to him.


CONJUCTION nee 2. Common Noun
3 Collective Noun
4. Material Noun
A Conjunction is a joining word. It is used for no
other purpose.
5. Abstract Noun
rin
g.n
2. Proper Noun—A Proper Noun stands for one
A conjunction is never connected with an object as a particular person, place or thing as distinct from any
Preposition. other; as Raman (a person) Jamuna (a river), Haridwar (a
A conjunction doesn’t qualify a word as an Adverb
does. Its function is to merely join words or sentences.
Therefore, the same word can be an Adverb in one
place, a Preposition in another, or a conjunction in yet
city) America (a country).
e
A Point to note—The writing of a Proper Noun
should always begin with a capital letter.
As,
t
another place. 1. Shane and Sita went to bathe in the Ganges.
As, 2. Tom met Harry in Haridwar.
(a) I have never seen him before. ( as Adverb) 3. Betty went to America where she met Blake in
(b) They worked here before I joined. (as Preposi- Beverly Hills.
tion) 4. The Prime Minister of Britain lives at 10,
(c) The sun rose before we reached the park. (as Downing Street.
Conjunction) 3. Common Noun—A common noun indicates no
one person or thing in particulars, instead it is common to
Conjunctions are sub. divided into two main clauses.
any and every person or thing of the same kind, As
(a) Co-ordinating conjunctions : These are so named ‘woman’ ‘table’.
as they join words, phrases or clauses of equal rank. Thus, woman does not indicate any one particulars
(b) Subordinating conjunctions : These are called so woman, such as Julia but can be employed for any and
because they join a dependant clause to a principle every woman. Similarly, ‘river’ does not point out to any
clause. (That is to a clause of higher rank). particular river such as Jamuna and can be used for any

CAT Complete Course | 465

Visit : www.EasyEngineering.net
Visit : www.EasyEngineering.net

and every river. Like ‘country’ does not point out to any Iron Fruit Leather Steel
one country, such as America but can be used for any and Lead Cotton Jute etc.
every country anywhere in the world.
6. Abstract Noun—An Abstract Noun denotes some
Some common nouns, quality state or action apart from anything possessing
1. He is a boy. such quality, state or action, and can be perceived
2. That table is made of wood. through the five senses.
3. Woman makes the man. Such as,
4. Roads are wide in London. Quality = Kindness, Bravery, Honesty
5. Shirts and pants are western attire. State = Childhood, Poverty, Love
4. Collective Noun—A collective Noun stands for a Action = Hatred, Laughter, Love.
group or collection of individuals, animals, places or Note—The Abstract Noun takes singular verbs.
things as one complete unit. 6.(a) Abstract Nouns can be formed from Adjec-
For example, There may be many buffaloes in the tives, Common Nouns or from verbs.
posture, but only one herd. Here herd is a collective noun Take a look at the following—
because it stands for all the buffaloes together.

ww
A point to note—
A distinction is made between a collective noun and a
Adjective
Deep
Abstract Noun
Depth

w
Brave Bravery
noun of multitude. A collective Noun denotes one united
whole, and thus the verb following it is singular. False Falsehood
As,
.Ea
The class (Collective Noun) consists (Singular Verb)
True
Proud
Truth
Pride
of 25 students.
syE
The team (Collective Noun) has (Singular Verb) 11
Wise
Good
Wisdom
Goodness

ngi
members. Mad Madness etc.
A noun of multitude indicates the individuals of the 6.(b) From verb to Abstract Noun
group. Thus, the verb that follows it is plural
even though the noun is singular.
As, nee Verb
Order
Love
Abstract Noun
order
Love
The class (Collective Noun) are (Plural Verb) divided
in their hobbies.
Regret
rin Regret

The jury (Collective Noun) are (Plural Verb) divided


in their verdict
Sing
Taste
g.nSing
Taste

5. Material Noun—A material noun denotes the


matter or substance of which things are made.
Hence, ‘chicken’ is a common noun; but chicken (or
the flesh of chicken) is a Material Noun.
Sleep
Talk

Common Noun
e
Sleep
Talk
6. (c) From Common Noun to Abstract Noun
Abstract Noun
t
A wood may be a Material Noun or a common noun Boy Boyhood
according to the context it is used in. Child Childhood
As, Man Manhood
1. Chickens (Common Noun) live in coops. Mother Motherhood
2. Chicken (Material Noun) is good for cancer
7. Some Abstract Nouns are considered as male,
patients.
like, sun, thunder, death.
In sentence 1 the noun indicates individual chicken
As,
or chickens, and is therefore a common noun.
The sun showers his light on the brave war places his
In sentence 2 it denotes the matter of which the
cold barrel at the head of Emperors.
bodies of chickens are made, and is thus a Material Noun.
On the flip side, states or qualities expressed by
Some common Material Nouns—
Abstract Noun and that which is assumed to possess
Gold Mutton Wood Fur beauty, fertility, grace and inferiority etc, are regarded as
Silver Chicken Clay Wool females, such as, the earth (Mother Earth) fertility

466 | CAT Complete Course

Visit : www.EasyEngineering.net
Visit : www.EasyEngineering.net

(bearing), mercy (forgiveness), charity (giving) virtue Milk-man Milk-maid


(pure) etc. Grand-uncle Grand-aunt
Consider the following: Grand-father Grand-mother
A ship is always spoken of as she even though the Other ways to identify the masculine from the
noun does not begin with a capital letter. The same is also feminine.
said of trains, motorcycles and other machines.
(d) By adding–ess to the Masculine and not alter-
As, nating its form.
1. The ship sailed along merrily on her maiden As,
journey.
Masculine Feminine
2. The Betsy will get her first test drive tomorrow.
Shepherd Shepherdess
Noun Gender—The difference of sex in grammar is
Host Hostess
called the difference of gender.
God Goddess
There are four different types of gender—
Peer Peeress
1. Masculine gender – indicates the male form.
Lion Lioness

ww2. Feminine gender – indicates the female form.


3. Common gender – denotes persons or animals of
either sex.
(e) By adding ess and dropping the vowel of the last
syllable of the Masculine.

w
4. Neuter gender – denotes things of either sex, that
is, inanimate things.

.Ea
There are three different ways by which the
Masculine Noun can be identified from the Feminine
As,
Masculine
Tiger
Feminine
Tigress

syE
Director Directress
Noun.
Waiter Waitress
(a) By changing the word.
Founder Foundress
As,
Masculine Feminine
ngi Inspector
Hunter
Inspectress
Huntress
Boy
Father
Brother
Girl
Mother
Sister
nee Actor
Instructor
Actress
Instructress
Dog Bitch Poster
rin Postress

g.n
Horse Mare Mister Mistress
King Queen Sorcerer Sorceress

e
Sir Madam (f) Exceptional Cases—
Drake
Boar
Husband
(b) By Adding a Prefix—
Duck
Sow
Wife
Masculine
Bridegroom
Widower
Feminine
Bride
Widow
t
(g) Foreign Feminines—
As, Masculine Feminine
Masculine Feminine
Czar Czarina
Cock – sparrow Hen – sparrow
Beau Belle
He – goat She – goat
Signer Signora
Jack – ass She – ass
Man – servant Maid – servant Hero Heroine
Buck – rabbit Doe – rabbit Administrator Administrators
(c) By changing the Suffix— Noun Number—There are singular nouns and plural
As, nouns. In order to recognize the single number from the
Masculine Feminine plural number.
Washer man Washer woman Read and retain the following rules and points—
Pea-cock Pea-hen Rule 1—Nouns that add – s to the singular.

CAT Complete Course | 467

Visit : www.EasyEngineering.net
Visit : www.EasyEngineering.net

Singular Plural As,


Boy Boys 1. Girls’ hostel; Dogs’ ear.
Girl Girls 2. Men’s toilet; Children’s toys.
Cat Cats Rule 6—The possessive does not carry the possessive
Dog Dogs (apostrophe) sign (’) words like—yours, ours, its, his,
hers, theirs, mine, omit the possessive sign.
Fan Fans
As,
Goat Goats
1. This is my pen and not your’s / yours.
Hen Hens
Incorrect / correct
Jackal Jackals
2. Either it is his or her’s / her pen. Incorrect / correct
King Kings
Rule 7—With a noun or title of several words. The
Queen Queens
apostrophe (possessive) sign is used only with the last
Ship Ships word.
Rule 2—Nouns which are used in the plural. As,

ww
Spectacles
Scissors
Measles
Credentials
Alms
Gymnastics etc.
1. His daughter-in-law’s mother.
2. The Governor-General’s house.

w
3. Ram and Deshmukh’s company.
Wages In case two nouns in the possessive case are joined
Riches
Gallows .Ea by ‘and’ the apostrophe is added to both to indicate
individual possession. But the use of the apostrophe in

employed in the singular. syE


However, some forms may be plural in form but are
the last word indicates joint ownership.
As,
1. Sarita and Rohan’s collection. (Joint Possession)
Like, Mathematics, Economics, Physics, News etc.
As,
ngi 2. Sarita’s and Rohan’s collection. (Separate Pos-
session)
1. Mathematics is a good subject (not ‘are’)
2. The news was good. (not ‘were’)
neeRule 8—Some nouns have the same form in the
singular as well as the plural. Like Fish, Deer, Sheep.
Ex.

rin
3. His innings was sparkling. (not ‘were’)
Rule 3—Some nouns are used in the singular only 1. Fishes live in water. (x)

g.n
and are not led by ‘a’ or ‘an’ neither are they pluralized. 2. Fish live in the water. ()
As, 3. Sheeps give us wool. (x)
1. The information is correct. (not informations’)
2. Susan’s hair is red. (not ‘hairs’)
3. Tom is up to mischief again. (not ‘mischiefs’)
4. Sheep give us wool. ()

e
Rule 9—Nouns consisting of several words take the
plural in the first word or last word.
As,
t
4. I took his advice. (not ‘advices’)
Rule 4—When a noun does the job of a compound Singular Plural
word it is not pluralized though it may be preceded by a Sister-in-law Sisters-in-law
plural number.
Commander-in-chief Commanders-in-chief
As,
B.A. B.As.
1. A three year old.
Lord Justice Lord Justices
2. A ten rupee note.
Major General Major Generals
3. A weekend holiday.
Rule 10—Noun words that end in ‘f’ get pluralized
4. A six mile track. by adding ‘ves’ and dropping the ‘f’.
Rule 5—Use of apostrophe As,
Don’t put the apostrophe (’) sign or put an ‘s’ after Knife Knives
plurals that end in ‘s’/ Other old ‘ren’, en; es; ies or change the vowels to
It may be used after plurals that do not end with ‘s’. form the plural number.

468 | CAT Complete Course

Visit : www.EasyEngineering.net
Visit : www.EasyEngineering.net

Singular Plural Cardinals Ordinals


Child Children One First
Ox Oxen Two Second
Potato Potatoes Three Third
Foot Feet Four Fourth
Goose Geese Five Fifth
Exceptions Six Sixth
Chief Chiefs Seven Seventh
Gulf Gulfs Eight Eighth
Chef Chefs Nine Ninth
Zoo Zoos Ten Tenth
Determiners Note—Cardinals can be used at random or as one
desires whereas, ordinals have to follow a serial order.
1. Definite Determiners (Demonstrative)
As,

ww
When a person or thing is pointed out precisely it is
called a Definite Demonstrative.
The most common being this, that take singular
He bought 5 apples but I bought 2. (The shaded
numbers are cardinals)

w
nouns. These, those take plural nouns.

.Ea
2. When the adjective points out in an inexact
fashion it is called an Indefinite Demonstrative.
He stood first in line whereas I stood last. (The
shaded words are ordinals)
6. Indefinite Numerals indicate number of some
kind without specifically saying what the number is.

syE
Common Indefinite Demonstrative are—any, a
certain, some, other, any other etc.
3. Take a look at the following to get a clearer under-
The main Indefinite Numerals are; all, some, enough,
no or none, many, few, several etc.

ngi
Examples—
standing of Definite Demonstratives—
All men are equal.
1. This boy met me today. These books are for him.

nee
No body attended the seminar.
2. That house is on fire. Those people will be burnt
Many women are biased.
alive.
Several horses trotted by.
3. This is not the shirt I wanted. Bring the other
shirt.
rin
Some of the teachers protested.
Enough bread was available.
4. Such people (dishonest people) can not be trusted.
4. Take a look at the following examples of Indefi-
g.n
Few people are born rich.
Note—The words some, enough, all, no or none are

e
nite Demonstratives
all adjectives of number or Adjectives of Quantity
1. Harry met some people at the party.
2. I do not know if there are any houses vacant in
Haridwar.
3. There is a certain amount of oil I can not find.
according to its structure.
t
If the noun is material or abstract the adjective is
Quantitative. However, if the noun is common, the
Adjective then is Numeral.
4. Do you know of any other camps? Some more examples:
5. Adjective : Numerals 1. She ate some (a certain quantity) cake.
Adjective Numerals show how many persons or 2. She did not eat any (any quantity of) cake.
things or in which order they stand—They identify the 3. She ate enough or sufficient cake.
number or the position. 4. She ate all (the whole quantity of) cake.
Such adjectives are divided into two main classes. 5. Half a loaf of bread is better than none (no
(a) Definite (b) Indefinite quantity).
(a) Definite Numerals indicate an exact number, Note—‘No’ is used when the noun that it qualifies is
which show how many persons or things. Viz. one, two, expressed. ‘None’ is used when the noun is understood
three etc. These are called Cardinals. (as in example ‘e’)
(b) Those which demonstrate serial order such as, Such adjectives must be followed by a singular noun
first, second, third etc. are called Ordinals. which is either a material noun or an abstract noun.

CAT Complete Course | 469

Visit : www.EasyEngineering.net
Visit : www.EasyEngineering.net

7. The Adjectives of number can be classed into two Example—


parts. Quantitative (Uncountable) and Numerals (Count- 1. After Rajiv Gandhi was declared Prime Minister
able) (Nominative), the LTTE assassinated him (Accusative).
Lets consider the following— 2. He must return the cycle (Accusative) which
1. She has much wealth. (Quantitative) (Nominative) you lent him.
2. He has many 500 rupee notes. (Numerals) 2. The objective form of the Pronoun (me, him, her,
The first example (1) has an uncountable number and us, them) is used when; (i) it is the object of a Preposition
thus is quantitative. Whereas, the second example (2) a (ii) It is placed after the Verb ‘to be’ and this infinitive is
countable number and therefore is a Numeral. preceded by a transitive verb with its object (iii) It is
placed after the adjectives; like, unlike or near.
To make things a little more clear. Consider this.
Example—
Quantitative (Uncountable) Numerals (Countable)
1. Between you and me (not I), he is quite stupid.
1. She has little wealth. She has a few rupees.
2. This is ample food for you and me (not I).
2. She has enough wealth. She has enough rupees.
3. She has some wealth. She has some rupees. 3. She invited my friend and me (not I) to lunch.
4. No one can dance like him (not he).

ww
4. She has no wealth. She has no rupees.
5. She has all the wealth. She has all the rupees.
6. Has she any wealth. Has she any 100 rupees
5. Let you and me (not I) play chess.
3. A Relative Pronoun, when it has two Antecedents

w(Abstract Noun).

.Ea
notes. (Common noun)
To make sure you have understood the difference
between quantitative and Numeral Adjectives. Test your
skills by filling in the blanks of the following exercises.
which are not of the same person, it then agrees with the
antecedent nearest to it—
Example—
1. You are the girl who has been selected.

syE
Stating whether they are quantitative or numeral (Q or N).
1. John has many cars. (…………….)
2. He is the man who wishes to help.
3. I, who am speaking said these words.
2. He has no gold. (…………….)
3. She has some money. (…………….) ngi 4. I am the person who is most affected.
5. This is one of the most tastiest dishes that have
4. Has she any 10 rupee notes. (…………….)
5. Susan has some wealth. (…………….) nee
(not has) ever been cooked. (The Antecedent of that is
dishes not one.)
6. He is one of those who know (not knows) nothing.
6. Ray has enough knowledge. (…………….)
7. Harry and Tom have all the food. (…………….)
Rules of Pronouns
rin
8. Betty has a few dollars. (…………….)
9. He has little taste. (…………….) g.n
Rule 1—Relative and Demonstrative pronouns must
be of the same number, person and gender as their
antecedents.
10. We have no regret. (…………….)
Ans. 1. N, 2. Q, 3. Q, 4. N, 5. Q, 6. Q, 7. Q, 8. N,
9. Q, 10.Q.
As,
One must not waste his time.
(Change ‘his’ to one’s)
e t
Mention whether the following adjectives according I am not one of those who can not practise what I
to the sentence are quantitative (Q) or Numeral (N) preach.
Adjectives.
(change ‘I’ to they)
1. He has no money. (Q / N)
Rule 2—When two singular nouns joined by ‘and’
2. They have all the coins. (Q / N) are preceded by ‘each’ or ‘every’ the pronoun must to
3. She has enough wealth. (Q / N) singular.
4. I have no cars. (Q / N) As,
5. Ram has a few rupees. (Q / N) Each boy and each man was in his uniform.
Ans. 1. Q, 2. N, 3. Q, 4. N, 5. N. Every winter and every summer has its charm.
A Closer look at Pronoun Rule 3—‘Neither………nor’; ‘Either…..or’; ‘or’
when they join singular nouns the pronoun is singular.
1. A Pronoun must agree in person, number and
gender with the antecedent or noun it stands for; but its As,
case relies solely upon its own sentence. Tom or Harry will enjoy his holiday.

470 | CAT Complete Course

Visit : www.EasyEngineering.net
Visit : www.EasyEngineering.net

Either Greg or Robert lost his wallet. Rule 9—When the object of a verb or a preposition
Neither Sheila nor Hema bought her bag. is a pronoun, it should be in the objective form.
Rule 4—But if a plural noun and a singular noun As,
are joined by ‘or’, or ‘nor’, the Pronoun agrees with the He gave it to you and me. (not I)
closest noun. (object of the verb ‘gave’)
As, Between you and me. (not I)
Ram or his friends must finish their work. (object of the preposition between)
Either the boys or mother will do her chores. Let(mean allow) you and me do it together.
Neither he nor we have done our job well. (object of the verb ‘let’ allow)
Rule 5—It is considered that well mannered and Rule 10—When a pronoun follows ‘than’ and ‘as’
proper to place the first personal pronoun in the singular the verb is mentally placed.
last. The second should come prior to the third. Whereas
As,
in the plural, ‘we’ come before ‘you’ and ‘you’ before
‘they’. Such is the preferred order. Tom is taller than I. (I am not ‘me’)
He runs faster than I. (I run, not me)

ww
You and I must go quickly. (Not ‘I’ and ‘You’)
You and She should join the club. (not ‘she’ and I love you more than she. (loves you)
‘you’) I run as fast as he. (he runs)

‘he’) w
You, he and I must play poker. (not ‘you’, ‘I’ and

.Ea
We and you should work together. (not ‘you’ and
I love you as much as (I love) her.
Rule 11—Try to avoid the usage of ‘the same’ in
place of a personal pronoun.

syE
‘we’) As,
You and they have failed. (not ‘they’ and ‘you’) After you have examined the contents kindly return
Rule 6—When a Pronoun refers to more than one the same to the office.
noun or pronoun of different person, the first person
plural in preference to the second, and the second in
ngi (use ‘them’)
Rule 12—‘This’ when used as a pronoun usually

nee
preference to the third.
follows the sentence or idea it refers to. It is also used in
As, the place of ‘it’ for emphasis.
You and I have completed our task. (our is the first
person plural)
You and Tom played your cards. (your is the second
As,

rin
His son stole the purse and this (Refers to the act)
person plural)
She and I did our jobs. (our is the first person plural)
hurt him deeply.

g.n
This (For Emphasis in place of ‘it’) is definite (I will

Rule 7—Who / Whom


‘Who’ is used when it is the subject of a verb.
‘Whom’ is used when it is the object of a verb.
go to Goa)

e
Rule 13—A Reflexive Pronoun does not stand alone
a noun or pronoun must precede it.
As,
t
He is the boy who stole your watch. You and myself will go there. (Use I)
(Here ‘who’ is the subject of the verb ‘stole’) Both Tom and myself won the prize. (Use I)
When I talked to him whom do you reckon inter- Both himself and Greg completed the project. (Place
rupted ? ‘he’ before himself)
(Here ‘whom’ is the object ‘talked’) Myself cooked the food.(Place ‘I’ before myself)
Rule 8—When the complement of the verb ‘to be’ is Rule 14—Don’t omit the Reflexive Pronoun when
expressed by the pronoun it should be in the Nominative the following verbs are used reflexively ‘exert’; ‘resign’;
form. ‘apply’; ‘revenge’; ‘oversleep’; ‘over-reach’; ‘acquit’;
As, ‘absent’; ‘enjoy’; ‘drink’; ‘avail’.
It was she (not ‘her’). As
It must be he (not ‘him’). She enjoyed (Insert ‘herself’) at the party.
I am he you desire (not ‘him’) I applied (Insert ‘myself’) to the job.
It was I (not ‘me’) He revenged (Insert ‘himself’) by killing his enemy.

CAT Complete Course | 471

Visit : www.EasyEngineering.net
Visit : www.EasyEngineering.net

Rule 15—The Reflexive Pronoun is omitted after, 6. Is this the book for ……………. you were
such verbs as ‘steel’, ‘stop’, ‘lengthen’, ‘make’, ‘gather’, asking?
‘bathe’, ‘move’, ‘open’, ‘spread’, ‘feed’, ‘draw’, ‘rest’, 7. This is the girl ……………. ring you bought.
‘role’, ‘burst’, ‘keep’, ‘turn’, ‘set’, ‘break’, etc. The verb
8. I do not believe ……………. you say.
is then considered intransitive.
9. My son ……………. you have not met, is here.
As,
10. I need such a man ……………. he is.
He stopped himself eating meat. (omit)
11. ……………. wants to meet me ?
She kept herself far from him. (omit)
12. ……………. umbrella is this ?
He made himself away from office. (omit)
13. ……………. are you waiting for ?
Rule 16—Each other / One anothe—While talking
about two persons or things use ‘each other’ and while 14. The is the Taj ……………. Shah Jahan built.
speaking of more than two persons or things use ‘one 15. I gave a coin to a man ……………. leg was
another’. However, both forms are now commonly used broken.
nowadays. But, the above, is considered grammatically Answer : 1. which, 2. who, 3. whose, 4. what,
correct. 5. who, 6. who, 7. whose, 8. what, 9. whom, 10. that,

ww
As,
Tom and Susan are not speaking to each other.
(Between two)
11. who, 12. whose, 13. what, 14. that, 15. whose
A Closer look at Adjective

w
The guests at the seminar spoke to one another.
(More than two)
.Ea
Rule 17—Either / Anyone—In reference to two
1. The primary use of an adjective is to modify a
noun or pronoun. It is a part of speech which describes,
qualifies and identifies a noun or pronoun.
In modern day grammar, grammarians are doing
reference to more than two.
As, syE
persons or things either is used while ‘anyone’ is used in away with the separation of Articles from its big brother
the Adjective.
The new concept or creation now includes Deter-
Either of the two girls must be selected.
Has anyone in the class seen my mobile? ngi miners, as a class a part or distinct from what was
formerly clubbed under ‘Adjectives’.
Rita was smarter than anyone in her group.
Rule 18—What—‘What’ refers to things solely, and
is used without an antecedent. It is equivalent to ‘that
nee Let us start with determiners. They can be cate-
gorized as shown below.
(a) Articles : A, An, The
which’ or the ‘thing which’.
As,
(b) Quantifiers
rin : All, Few, Many, Several

g.n
(c) Genitive or Posses- : His, Her, Its, My, Our,
What the eye can not see, the heart does not feel. sives Their, Your
(‘What’ is equal to that which) (d) Demonstratives : This, That, These, Those.
He can do anything what you like.
(Wrong, as ‘what’ is used with an antecedent
‘anything’. Change ‘what’ to ‘that’)
(e) Numbers
(f) Negative e
: No, None
Note—They indicate the number and gender of the
t
: One, Two, Ten, Hundred.

Rule 19—Whose / Which—As a rule ‘whose’ is used Noun. Thus, they possess several forms.
for persons and ‘which’ for non living things. Exercises for Determiners are on pages ………. To
As, ……… after further detailed study and examples.
2. Adjectives are divided into eight parts or cate-
Meet the man whose exploits are legendary.
gories.
This is the watch which he gave me.
(a) Proper : In which it describes a thing by
Drill—Use the correct form of the relative pronoun referring it to a Proper Noun.
in each of the following sentences.
(b) Descriptive : Shows of what quality or state a
1. The answer ……………. you gave is correct. thing is.
2. I saw the man ……………. was hurt. (c) *Quantitatives : Show “how much” ?
3. I know the woman ……………. child was lost. (d) *Numerals : Tells us “how many” or in which
4. Listen to ……………. I say. order.
5. This is the magician ……………. we saw last (e) *Demonstratives : Show which or what thing is
night. intend or meant.

472 | CAT Complete Course

Visit : www.EasyEngineering.net
Visit : www.EasyEngineering.net

(f) Distributives : Tell us whether things are taken Positive Comparative Superlative
separately or in separate lots. Beautiful More Beautiful Most Beautiful
(g) Interrogative : Inquires which or what thing is Intelligent More Intelligent Most Intelligent
meant.
Courageous More Courageous Most Courageous
(h) *Possessives : Show relationship or ownership.
Pleasant More Pleasant Most Pleasant
* As mentioned earlier these adjectives are now
Magnificent More Magnificent Most Magnificent
classed as Determiners.
Rule 6—Some adjectives form the comparatives and
Degrees of Comparison superlatives in an irregular manner.
Rule 1—Most adjectives of one syllable form the Positive Comparative Superlative
comparative by adding ‘er’ to the positive. Whereas ‘est’ Far Farther Farthest
is added to the superlative.
Good Better Best
Positive Comparative Superlative
Late Later, Latter Latest, Last
Bold Bolder Boldest
Many More Most
Plain Plainer Plainest
Much More Most

ww
Deep
Cool
Deeper
Cooler
Deepest
Coolest
Old
Little
Older, Elder
Less
Oldest, Eldest
Least

w
Strong

Positive
Stronger

.Ea
Strongest
Rule 2—When the positive end ‘e’ only ‘r’ and ‘st’
are added to form the comparative and superlative.
Comparative Superlative
It is to be remembered that ‘than’ follows the com-
parative while ‘the’ precedes the superlative.
As,

Able
Brave
Abler
Braver syE Ablest
Bravest
Comparative
1. He is taller than me.
2. Ram is stronger than Shyam.
True
Wise
Truer
Wiser
Truest
Wisest
ngi 3. Sita is more beautiful than Rita.
4. Rishikesh is farther than Haridwar.
Rude Ruder Rudest
Rule 3—Where the positive ends in one consonant
and the consonant is preceded by a short vowel, the final nee Superlative
1. He is the tallest boy in the class.

consonant is doubled.
Positive Comparative Superlative rin
2. Ram is the strongest of all.
3. Sita is the most beautiful girl in Delhi.

Big
Thin
Bigger
Thinner
Biggest
Thinnest g.n
4. Rishikesh is the farthest town in our area.
Adjectives of Degree
Wet
Red
Fit
Wetter
Redder
Fitter
Wettest
Reddest
Fittest
Superlative Degree.
e
Adjectives of Degree are divided into three classes.
The Positive Degree, The Comparative Degree and The

1. Positive Degree—The Positive Degree is used


t
Rule 4—When a positive ends in ‘y’ and the ‘y’ is when two persons or things are said to be equal in regard
preceded by a consonant the ‘y’ is changed into ‘i’ and to some quality. The Positive Degree is then used with
‘er’ and ‘est’ are added to form the comparative and as…..as, or the comparative Degree can be applied with
superlative. ‘not’.
Positive Comparative Superlative Consider these sentences—
Dry Drier Driest (a) She is as clever as Sita.
Pretty Prettier Prettiest (b) She is no less clever than Sita.
Lovely Lovelier Loveliest (c) She is not more clever than Sita.
Healthy Healthier Healthiest Again,
Merry Merrier Merriest (a) She is as beautiful as Sita.
Rule 5—All adjectives of more than two syllables (b) She is no less beautiful than Sita.
and adjectives of two syllables are preceded by ‘more’ to (c) She not more beautiful than Sita.
the positive to form the comparative and ‘most’ to form These sentences depict the same idea or message that
the superlative. Sita and ‘she’ are equal in regard to beauty.

CAT Complete Course | 473

Visit : www.EasyEngineering.net
Visit : www.EasyEngineering.net

2. Comparative Degree—The Comparative Degree As,


is usually used to compare two unequal qualities of 1. He is better than I. (Comparative)
persons or things, therein showing a difference between
2. He the best student in the class. (Superlative)
two persons or things in regard to some quality.
5. Some Important Adjectives—At times, some
Examples—
important adjectives are misused or misunder-stood. This
1. Shyam is heavier than Lakshman. leads to confusion and incorrect usage.
2. This sword is sharper than that sword. Lets take a look at the following adjectives. Elder,
3. Lakshman is not as heavy as Shyam. (Positive Eldest, Older, Oldest.
Degree with ‘not’) 1. She is my elder sister.
4. Shyam is the heaviest of all. (Superlative Degree) 2. She is older than her sister.
You can now see that the Positive Degree and the 3. Her elder daughter is a doctor.
Superlative Degree can be used as a Comparative while
Older and oldest are employed for things as well as
comparing two unequal qualities of persons or things.
persons and indicate age.
3. Superlative Degree—The Superlative Degree is
But, we can not say,
used when one person or thing is greater than all other

ww
persons or things of a similar kind the Superlative is used
with ‘the’, ‘of’.
1. Mumbai is the eldest city in Maharastra. (Wrong)
Instead use, ‘oldest’
2. That is the eldest building in the city. (wrong)

w
As,
Use ‘oldest’
1.She is the most beautiful girl of her class.

.Ea
2.Paras is the cleverest of all.
3.Rome is one of the most beautiful countries in the
By this we can by now guess that older, oldest, elder,
eldest can be used for persons and older, oldest for things.

world.
syE
4.Raj is the fastest runner of the team.
6. Further / Furthest / Farther / Farthest—The
word ‘further’ ‘denotes something additional ‘er’ extra
while ‘farther’ denotes a greater distance between two

ngi
Note—The word inferior, superior, senior, junior, points. However, nowadays the two forms ‘further’ and
prior etc. are followed by ‘to’ instead of ‘than’. ‘farther’ can be used to indicate distance.
Examples— As,
1. This paper is inferior to that.
2. Mittal is senior to Manoj.
nee 1. Kashmir is further / farther than Manali or
Vashist.

3. Gold is superior to Iron.


rin
2. Kashmir is the furthest / farthest of the three.
3. Kashmir is the furthest / farthest town.
4. She is junior to me.
5. Prior to this job I worked in Wipro.
g.n
7. Later / Latest / Latter / Last / Former—‘Later’
and ‘Latest’ indicate time whereas ‘Latter’ and ‘Last’,
4. Use of the Comparative Degree—The Com-
parative Degree is preferred to the Superlative Degree
when comparing two things of the same quality.
Consider this,
As,
e
‘former’ and ‘Latter’ denote position.

1. The student walked in later than his teacher.


2. This dress is the latest fashion.
t
1. She is the cleverest girl in the class. (It would be 3. Rohan and Sohan took an entrance examination.
better to say, ‘the cleverer’) The former (Rohan) passed while the latter (Sohan) failed.
But, 4. Ravi walked into the room last.
2. This boy is the fastest of the three. (correct) 8. Nearest / Next—‘Nearest’ denotes space or
As, there is no comparison between two persons or distance. ‘Next’ indicates order or position.
things. As,
Thus, when the number exceeds two it is correct to 1. The cinema hall is nearest to my house. (distance)
use the superlative Degree. 2. Harry’s house is next to mine. (Position)
3. Who was the most famous king Ashoka or 9. Fewer / Less—‘Less’ usually denotes quantity
Akbar ? while ‘fewer’ denotes number.
(The preferred use is ‘more famous’) But, we can say, ‘No less than a thousand guest
The Comparative Degree is generally followed by attended their marriage’ when we are thinking not of
‘than’ and ‘the’ goes before the superlative.

474 | CAT Complete Course

Visit : www.EasyEngineering.net
Visit : www.EasyEngineering.net

individual guests but of the number as a mathematical A Closer look at Verb


quantity.
1. The Infinitive is regularly used with ‘to’ except in
‘Fewer’ and ‘Less’ are usually followed by ‘than’.
the following cases—
Consider the following—
(a) The bare infinite that is,
1. No fewer than 50 girls failed in a class of 70
The infinitive without ‘to’ is applied after auxiliary
students. (Number)
verbs like, ‘shall’, ‘will’, ‘can’, ‘may’, ‘did’, ‘should’.
2. Mr. Gupta does not purchase less than 50 kgs. of However, ‘ought’ is the exception.
sugar. (Quantity)
(b) The bare infinitive is employed in the Active
3. There are fewer boys than girls in my school. voice but not in the Passive Voice after words of percep-
(Number) tion, feeling etc. such as, see, laugh, feel, watch.
4. Factories in Noida do not purchase less than a ton As,
of raw material. (Quantity)
I saw her dance. (not ‘to dance’)
10. Countable and Uncountable Adjectives—Many
She made him laugh (not ‘to laugh’)
/ Much—‘Many’ is used before countable nouns.‘Much’
is placed before uncountable nouns. We felt her touch our feet. (not ‘to touch’)

ww
As,
1. He has read many books.
However, the bare infinitive is used after ‘let’ in both
the active and passive voice.
I let her loose. (not ‘to loose)

w
2. Sheetal has many friends.

.Ea
Both 1 and 2 have countable nouns.
3. I have not much time to spare.
She was let loose. (not ‘to loose’)
(c) The bare infinitive is used after ‘need’ and
‘dare’ in the negative and interrogative forms.

syE
4. How much oil do you need?
Both 3 and 4 denote quantity and are uncountable.
As,
She need not speak (not ‘to speak’)

ngi
11. Many / Much / More / Most—These words can I dare not ask (not ‘to ask’)
be used as Pronouns when the Noun is understood. Need you go ? (not ‘to go’)
As, Dare for argue ? (not ‘to argue’)
1. He buys a lot of books but I do not buy many.
2. He spends a lot of money but I do not spend nee (d) After phrases like, ‘as soon as’, ‘had better’ and
‘had rather’ the bare infinitive is used.
much.
3. He bought many books but I bought more.
As,
rin
He would as soon sleep as work (not ‘to work’)
4. He bought a large amount of sugar but I bought
most. g.n
She had better talk soon (not ‘to talk).
(e) After ‘but’ and ‘than’ use the bare infinitive.
12. ‘Good’ and ‘Well’—At times we get into a sticky
issue over the application of Good and Well, the Adjec-
tive and the Adverb respectively. Clearly, when modify-
ing a verb it is not a problem. Use the Adverb!.
e
They boys did everything but study. (not ‘to study’)
I did nothing more than sing (not ‘to sing’)
2. The split Infinitive—When the ‘to’ is separated t
from its verb by inserting between them an adverb or
Consider this, adverbial phrase, it is not acceptable to grammarians.
He runs well. (However, in modern times such insertions are now
She knows well enough not to disturb her father at commonly used and accepted).
this time. (a) It is necessary to swiftly act. (use – to act
While modifying the verbs ‘runs’ and ‘knows’ swiftly)
simply use the Adverb ‘well’. He is to quickly leave. (use – to leave quickly)
But, when using a linking verb that involves the five (b) But, if the insertion makes the meaning more
senses, the sensible thing to do is to employ the adjective clearly expressed it can be done.
‘Good’. As,
As,
To really appreciate the play one must sit in the front
1 He is not feeling good this morning. row.
2. The boy’s vision is pretty good. (c) The ‘to’ of one infinitive can serve as ‘to’ for
3. He feels quite good after exercising. another infinitive if they are synonymous.

CAT Complete Course | 475

Visit : www.EasyEngineering.net
Visit : www.EasyEngineering.net

But, if separate ideas are expressed by the two As,


infinitives then, the ‘to’ of the first infinitive can not They served me ice-cream after having taken a bath.
serve the second infinitive. (change ‘after having taken’ to after ‘I had taken’) If
As, not ‘after having’ taken will refer to the subject ‘they’ not
He went on to succeed and prosper. ‘I’.
It is not my intention either to kill or to cure. (g) A gerund governed by a noun or pronoun should
be in the possessive case.
(d) An infinitive should be in the present tense
unless it denotes an action prior to an action of the As,
Principal verb. I disapprove my brother coming late. (wrong)
As, I disapprove my brother’s coming late. (correct)
She appears to have enjoyed herself at the party. Please excuse me being untidy. (wrong)
3. The Verbal Noun and The Gerund— Please excuse my being untidy. (correct)
(a) The Verbal Noun is preceded by the Definite 4. Participle—
Article ‘the’ and followed by ‘of’, whereas, a gerund does (a) A participle should not be alone without any
not attach them.

ww
As,
The humming of bees frightened him. (Verbal Noun)
proper agreement. It must be joined with a noun or
pronoun to which it refers.
As,

w
Humming bees frightened him. (Gerund)

.Ea
(b) A Verbal Noun is qualified by an adjective and a
gerund by an Adverb.
Walking on the footpath, a man tripped him.
(Rewrite; Walking on the footpath he was tripped by
a man.)

(wrong)
syE
The reading of a holy book quickly is not advisable.

The quick reading of a holy book is not advisable.


Being my day off, I overslept.
(Rewrite; It being my day off, I overslept.)
(b) But, words like, ‘considering’ ‘concerning’,
(Right)
Quick reading a holy book is not advisable. (wrong) ngi ‘taking’, ‘speaking’, ‘touching’, ‘owing to’ are not used
in agreement to any noun or pronoun.

Reading a holy book quickly is not advisable. (Right)


(c) A Gerund and a Verbal Noun have distinct posi- nee Taking all possibilities into consideration.
Broadly speaking it is difficult to assess.
Here the unexpressed pronoun (subject) is indefinite
tions in English Grammar no error should be made in
their composition.
rin
e.g. It means if one takes, ‘speaks’ etc.
(c) A present participle is not used to express an
The driving a car gives one a feeling of control.
(Here driving is not a verbal noun as it does not have of
g.n
action which is not in accordance with the Principal verb.
He walked home this evening and arriving there at

e
after it. So, cancel the before driving and it changes into
night. (wrong)

t
a gerund).
He walked home this evening and arrived there at
(d) Don’t mix a Gerund with a Verbal Noun in a
night. (correct)
sentence. Consider the following.
She joined me in the applying of my desires and in A Closer look at Preposition
attaining my goals. A Preposition is a word placed before a Noun or its
(‘the applying of’ is a verbal noun and ‘attaining’ a equivalent to indicate what relation the person or thing
gerund. Either cancel the ‘of’ to make applying a gerund denoted therein stands to something else. The Noun or
or place ‘the’ and ‘of’ to the gerund to make Verbal Noun equivalent is called the object of the Preposition.
Noun.) As,
(e) Do not confuse or substitute a gerund for an I put the book under my pillow.
infinitive. He sat on the chair.
It is easy recognizing him from her. (wrong) They dived into the river.
It is easy to recognize him from her. (Right) If the words ‘under’, ‘on’ and ‘into’ are removed, the
(f) When a Gerund is preceded by a preposition and sentences make no proper sense.
has reference to the subject of the sentence instead of any The book can be placed on or under the pillow. Thus,
other word, the gerund should not be used with a until a Preposition is used, the relationship between the
preposition. book and the pillow is not known.

476 | CAT Complete Course

Visit : www.EasyEngineering.net
Visit : www.EasyEngineering.net

Participle Prepositions : These were originally required by the sense. Prepositions must conform to
present or past participles used sometimes with the Noun the sense intended by the author as well as the idiom laid
expressed and sometimes with the Noun understood. down by custom. A case in point, when we speak of a
As, death caused by a disease, we say, ‘she died of T.B.’ but
if a death was caused by something else, we say, ‘she
Noun Expressed died from a kick’. Here both prepositions indicate cause,
During the winter, the winter during or still lasting. but one fits in one sense while the other is suitable to the
Noun Understood other. Likewise, we ‘agree with’ him but ‘agree to’ dis-
Owing to heavy rains, the crops were destroyed. agree. The lists of such types are endless.
However, if one uses the following appropriate
Compound Prepositions prepositions containing such words as take dissimilar
Two or more words frequently thrown together and prepositions after them, then one can retain a fair amount
ending with a single Preposition may be termed com- of suitable prepositions that are appropriate.
pound Preposition. Suitable Prepositions—
Examples— 1. Abide with (a person)
In spite of ; Because of; By means of; On account of; 2. Abide by (a thing)

of
ww
With reference to; On behalf of ; In the event of; In place

Use of ‘But’ as a Preposition


3. Abide at (a place)
4. Accused of (a crime)

w
Example—

.Ea
(a) All but (except) Hari were present.
(b) They were all but (everything except) destroyed.
5. Accused by (a person)
6. Apply for (a thing)
7. Apply to (a person)

Kinds of Preposition
At / In / On syE 8. Appeal to (a person)
9. Appeal against (a thing)
10. Angry at (a thing)
(a) ‘At’ is used for an exact time.
(b) ‘In’ is used for months, years, seasons.
ngi 11. Angry for (refers to action)
12. Angry with (a person)
(c) ‘On’ is used for days and dates.
As,
AT—at 10’clock ; at noon; at bedtime; at sunset; at
nee 13. Arrive at (a person)
14. Arrive in (a country)
the moment;
IN—In June; In winter; In 2010; In the next century; rin
15. Admit of (a thing)
16. Admit to / into (a place)
In the past; In the future;
ON—On Sunday; On Monday; On 7th January; On g.n
17. Alight on (a ground, a thing)
18. Alight at (a place)
Ram’s birthday; On Diwali;
The use of Preposition in reference to place.
(a) ‘At’ is used for a point.
20. Amused at (a thing)
21. Annoyed with (an action)
e
19. Alight from (a vehicle, animal (car, house etc.)

t
(b) ‘In’ is used for enclosed places. 22. Annoyed at (a thing)
(c) ‘On’ is used for surface. 23. Annoyed with (a person)
Examples— 24. Answer to (a person)
(a) At the corner; At the bus stop; At the end of the 25. Answer for (a thing)
lane; At my house; At the door
26. Antipathy to (a thing)
(b) In Haidwar; In the Box; In my coat pocket; in
27. Antipathy against (a person)
America
28. Anxious about (result of something)
(c) On the shelf; On the table; On the bed; On the
desk; On the floor 29. Anxious for (safety, health)
30. Ask from (a person)
Suitable Prepositions
31. Ask for (a thing)
A preposition shows relationship—what one person 32. Affiliated to (a board, university)
or thing has to do with another person or thing. It
33. Affiliated with (a part of)
expresses some relation between them. But, one must be
aware that preposition employed expresses the relation 34. Arm against (a danger)

CAT Complete Course | 477

Visit : www.EasyEngineering.net
Visit : www.EasyEngineering.net

35. Arm with (weapons) 80. Complain against (a person)


36. Atone to (a person) 81. Complain of (a thing)
37. Atone for (a thing) 82. Consult with (a person)
38. Attend to (a thing) 83. Consult on (a thing)
39. Attend upon (serve a person) 84. Clothed in (garment)
40. Award for (a thing) 85. Clothed with (some quality)
41. Award to (a person) 86. Care for (to like)
42. Account for (a thing) 87. Charge (a person) with (a crime)
43. Account to (a person) 88. Condemn to (punishment)
44. Argue against / about (a thing) 89. Condemn for (a certain crime)
45. Argue with (a person) 90. Confer with (consult with a person)
46. Apologize for (a thing) 91. Confer about (consult about a thing)
47. Apologize to (a person) 92. Converse with (a person)

ww
48.
49.
50.
Appoint (a person)
Appoint to (a position)
Arbitrate between (two parties)
93.
94.
95.
Correspond with (a person)
Correspond about (a thing)
Concerned at / about (affected by)
51.
52.
53.
w Arbitrate in (a dispute)

.Ea
Accomplice with (a person)
Accomplice in (a crime)
96.
97.
98.
Convenient for (means suitable)
Convenient to (means easy)
Clash with (means to strike against)
54.
55. syE
Affinity with (something)
Antidote to (medicine for poison)
99.
100.
Capable of (a thing)
Capacity for (doing something)

ngi
56. Antidote against (infection) Preposition Rules
57. Authority over (a person) Rule 1—As a preposition is not powerful enough to

nee
58. Authority on (a subject) stand in for much emphasis and importance, it is not
59. Aspire after (fame) positioned at the end of a sentence. But, there are the
60. Agree with (a person) following exceptions—
61.
62.
Agree to (a proposal)
Agree on (a subject) rin
(a) When if combines a preceding intransitive verb
to form a compound transitive verb.

63.
64.
Beg for (a thing)
Beloved of (noun) (a person)
As,
g.n
Francis loves being talked to.

65.
66.
67.
Blind in (one eye)
Blind to (faults / defects)
Born in (a country)
She hates being ogled at.

‘that’.
e
(b) When the sentence object is a relative pronoun

There are the keys that he was looking for.


t
68. Born of (parents) Such was the end that he came to.
69. Busy at (a thing) (c) When the object is an interrogative pronoun that
70. Busy with (refers to an action) is understood.
71. Cause for (anxiety) As,
72. Cause of (problems) What is he searching for ?
73. Claim to (a thing) What are you getting at ?
Where are you going to ?
74. Claim on (a person)
Rule 2—‘In’ and ‘at’ / ‘to’ and ‘into’
75. Compete with (a person)
When speaking of things that are fixed or at rest. ‘In’
76. Conform to (a rule) and ‘at’ are employed.
77. Contribute to (a fund) As,
78. Controversy with (a person) The lawyer is in his chamber.
79. Controversy on / about (a thing) He is at the peak of his carrier.

478 | CAT Complete Course

Visit : www.EasyEngineering.net
Visit : www.EasyEngineering.net

‘To’ and ‘into’ are used while speaking things in except the perfect tense ‘for’ refers to a period of time
motion. unlike ‘since’ which denotes some point of time. There-
As, fore, ‘for’ should not be substituted by ‘since’ or from.
The boys dived into the river. As,
He traveled to China. Robert has not eaten since morning.
Rule 3—Similar types of prepositions should not be She has been waiting in line since 8 a.m.
employed with two words. Until and unless it is suited to We were closed from Friday.
each word. I will resume working from January.
As, She will start work from today.
Cotton is similar to and superior to jute. He has been talking to him for 2 hours.
(This is inappropriate as the words ‘similar’ and Rule 9—Ago / Before
superior take different prepositions. As one preposition ‘Ago’ refers to past time and ‘before’ shows
can not exercise two, functions) precedence between two events.
Instead say, ‘Cotton is better then jute and superior to
As,
it’.

ww
Rule 4—Between / Among
‘Between’ is used when referring to two persons or
He left town five years ago.
She joined us before she got married.

w
things and ‘among’ when referring to more than two.
As,

.Ea
Share the ice cream between John and Mary.
Rule 10—It is incorrect to place prepositions after
the following words; ‘accompany’, ‘attack’, ‘assist’,
‘combat’, ‘order’, ‘reach’, ‘resist’, ‘violate’, ‘pick’,
‘afford’, ‘precede’, ‘request’, ‘inform’, ‘succeed’ etc.

Rule 5—On / To
syE
Distribute the books among members of the library.

‘On’ is also used of time / day / event and ‘to’


When used in the active voice.
As,
indicates place.
As, ngi 1. We wanted for(x) 4 pies.
2. She reached to(x) office late.
Tim arrived on my birthday.
She visited us on Saturday. nee 3. He stabbed on(x) her mercilessly.
A Closer look at Adverbs
He came on Christmas day.
He is going to the market.
rin
Rule 1— Adverbs should be placed as close as
possible to the word or words they qualify.
Don is walking to China.
She is coming to the theatre.
As,
They walked slowly. g.n
doer.
Rule 6—With / By
‘With’ frequently indicates instrument and ‘by’ the

As,
We screamed loudly.

e
The couple danced beautifully.
Rule 2— With an intransitive verb the adverb or t
adverbial phrase is positioned after it and not before.
He shot Tom with an AK 47. She heartily laughed. (×)
She was injured by a car. She laughed heartily. (√)
Harry was stabbed with a dagger by his friend, Joe.
The boys happily played. (×)
Rule 7—In / At
The boys played happily. (√)
‘In’ is used with names of countries, big towns while
‘at’ is utilized for small villages and towns. They excitedly screamed. (×)
As, They screamed excitedly. (√)
Greg lives at Ranipur More in Haridwar. Rule 3—With a transitive verb the adverb should be
placed either before or after it, never between the verb
Susan lives in America.
and object.
Rule 8—Since / For / From
He sadly mourned his father’s death. (√)
‘Since’ denotes some point of time like ‘from’ and is
employed before a noun or phrase. But ‘since’ is preceded He mourned his father’s death sadly. (√)
by a verb in a perfect tense, ‘from’ is used with all tenses He mourned sadly his father’s death. (×)

CAT Complete Course | 479

Visit : www.EasyEngineering.net
Visit : www.EasyEngineering.net

Rule 4—Adverbs which denote time ‘like’, ‘never’, He speaks nothing else than Spanish. (Use ‘but’)
‘ever’, ‘always’, ‘sometime’, ‘often’, ‘frequently’, She does nothing else than sleep. (Use ‘but’)
‘seldom’, ‘during’, ‘rarely’, etc. are positioned before the
Rule 13— The use of ‘never’ in place of ‘not’ is
verbs they modify.
wrong as ‘never’ means ‘not ever’ whereas ‘not’ means
She never talks nonsense. negative.
He rarely tells a lie. During our conversation he never mentioned her
They frequently visit the Gupta’s. name. (Use ‘did not’)
When an auxiliary verb functions as a main verb the (Never indicates ‘for all time’ while ‘not’ indicates
adverb of time is positioned after it. during the conversation there was no mention of her)
She is seldom late. I never remember talking to you. (Use ‘do not’)
He is never happy. Rule 14—‘Very’ / ‘Much’ / ‘So’
Rule 5—The Adverb enough is positioned after the
As an adverb of degree ‘so’ should not be used
word it qualifies.
without a correlative.
He was strong enough to compete. (√)
Tigers are so strong. (Use ‘very’)

ww
He was enough strong to compete. (×)
Rule 6—‘Only’ and ‘even’ are placed immediately
before the word it means to qualify.
Tigers are so strong that they can kill man easily.
(Correct as it is followed by its correlative ‘that’)

w
I played only one round. (√)

.Ea
I only played one round. (×)
Rule 7—When an adverb intends to qualify not any
‘Very’ modifies adjectives or adverbs in the positive
degree and ‘much’ in the comparative degree.
She sang very softly.
She is much faster than I.
(Not ‘much’)
(Not ‘very’)

syE
word in particular but a sentence as a whole it is
positioned at the beginning of the sentences.
Sadly, their plan did not work.
‘Very’ modifies present participles while ‘much’
modifies past participles.
Surprisingly, they gave up all hope.
Rule 8—The Adverbs, ‘enough’, ‘not’, ‘how’ and ngi The game was very interesting. (Present Participle)
He was much disturbed. (Past Participle)
‘more’ should not be left out when their presence is
required.
As, nee ‘Very’ is frequently employed to modify the adverb.
This dish is very much tastier than that.

She is not smart enough to pass.


They know how to fix it. rin (adverb) (adverb)
Rule 15—While employing the adverb ‘too’ and

We are expecting more grants from them after our


sterling performances.
getting confused.
g.n
‘very’ understand the implication of the words to avoid

He knows not what to do.


Rule 9—Adjectives should not replace adverbs.
He moved slower than I expected. (Used more
e
‘Too’ means excess of some kind denoting more than
sufficient something that goes beyond what is meant to be
expected. ‘Very’ on the hand means to ‘a great extent’ or
‘really’. t
slowly) As,
The birds flew away swifter when the cat arrived.
Sharon is too generous to feed the poor children.
( Swiftly)
(Change to ‘very’)
Rule 10—An Adverb does not qualify a Noun.
Quite a bunch of flowers were given to the chief The news is very good to be true. (Change to ‘too’)
guest.(Change ‘quite’ to ‘many’) Enough / Too
Explain the above paragraph. ‘Enough’ means sufficient or indicate a proper limit
(Change to above mentioned) has been reached whereas ‘too’ means more than enough.
Rule 11—In place of ‘firstly’ use first as ‘first’, itself He is enough weak to walk. (Change to ‘too’)
is an adverb. He is too old to get married.
Firstly, we must plan and secondly activate the plan.
(Change to ‘old’ enough)
(Use first)
He is too dark to be an African.
Rule 12—‘Else’ is followed by the adverb ‘but’. Do
not follow ‘else’ with ‘than’. (Change to ‘dark enough’)

480 | CAT Complete Course

Visit : www.EasyEngineering.net
Visit : www.EasyEngineering.net

Exercises Types of conjunctions—There are two types of


conjunctions, coordinating conjunctions and subordi-
1. They were intelligent than the others. nating conjunctions.
2. Sarah seldom finds something in the fridge. 1. Coordinating conjunctions—Coordinating con-
3. They are so exhausted to exercise. junctions knit words, phrases, clauses and sentences of
4. He does not know to play tennis. equal status or grammatical unit of a similar kind.
5. They did not fortunately get injured. Consider this.
6. The boys are happily playing. (a) John and Mary are dancing. (Two subjects)
7. She reached quick than I. Noun Noun
8. Rarely he visits the church. Subject Subject
9. I never spoke to him about my salary. The above conjunction joins two sentences.
10. Tom only came to my house this morning. E.g. ‘John is dancing, Mary is dancing’.
11. He has faithfully obeyed my orders. (b) You and I are intelligent. (Two subjects)
12. No one hardly talks to me. Pronoun Pronoun

ww
13.
14.
I am very surprised at your behaviour.
He always is happy.
Subject Subject
Here ‘and’ joins ‘you are intelligent’ and ‘I am
intelligent’.
15.

w The officer was much angry with us.

.Ea
Answers
1. place ‘more’ before ‘intelligent’, 2. Change
‘something’ to ‘anything’, 3. Change ‘so’ to much, 4.
(c) Harry and I
Noun
Subject
Pronoun
Subject
took a walk. (Two subjects)

syE
Insert ‘how’ before ‘know’, 5. Place ‘fortunately’ at the
beginning of the sentence, 6. Place ‘happily’ before
‘playing’, 7. Place ‘much’ before ‘quicker’, 8. Place
Here the conjunction and joins a noun and a pronoun
to form a single sentence.
(d) He is rich but generous.
‘rarely’ before ‘visits’, 9. Change ‘never spoke’ to ‘did
not speak’, 10. Place ‘only’ before ‘this morning’, ngi Adjective Adjective
Here the conjunction ‘but’ joins two adjectives to
11. Place ‘faithfully’ before ‘has’, 12. Change ‘no one
hardly’ to ‘Hardly anyone’, 13. Change ‘very’ to ‘much’,
14. Place ‘always’ after ‘is’, 15. Change ‘much’ to ‘very. nee
form a single sentence. ‘He is rich’, ‘He is generous’ to
express a contrast.

Exercises Adverb
rin
(e) He swims fast and perfectly.
Adverb

g.n
Tick the most suitable word. Here, the conjunction ‘and’ joins two adverbs and
1. Harry won the toss ………….. Tom lost. sentences into a single sentence, ‘He swims fast’, ‘He
swims perfectly’ to express manner. (How did he swim ?)

e
(a) not (b) whereas (c) nor (d) where
2.

3.
She studies hard ………….. she may pass.
(a) so

(a) when
(b) because (c) so that (d) but
He waited …………. I returned.
(b) till (c) unless (d) so
(f) We study during the day and play in the evening.
Phrase Phrase
Here, ‘and’ joins adverbial phrases of time.
t
(g) She is the girl who works hard and whose father is
4. He was not informed …………. He was terminated. employed.
(a) so (b) so that (c) that (d) but Two Clauses—Clause Clause
5. He has never eaten meat ……….. hopes to do so. The above sentence shows that the conjunctions
(a) or (b) never (c) always (d) neither ‘and’ and ‘but join two nouns, two pronouns, a noun and
a pronoun, two clauses and two sentences. These
6. I shall join you ………… you want me to.
conjunctions join these parts of speech, phrases, clause
(a) whether (b) if (c) only (d) lest and sentences of equal rank. And they are called coor-
Answers dinating conjunctions.
Important coordinating conjunctions are as follows:
1. (b) 2. (c) 3. (b) 4. (c) 5. (a) 6. (b)
And, but, nor, too, yet, so, as well as, only, then,
A Closer look at Conjunction therefore, no less than, otherwise, else, still, while,
A conjunctions function is solely to join words, whereas, nevertheless, either……or, neither……nor,
phrases, clauses and sentences together. both…..and, not only……but also, nonetheless etc.

CAT Complete Course | 481

Visit : www.EasyEngineering.net
Visit : www.EasyEngineering.net

Coordinating conjunctions are divided into (c) Time will make you forget for time is a great
four classes. healer.
(a) Cumulative or Copulative 2. Subordinating Conjunctions—Subordinating
(b) Alternative or Disjunctive conjunctions are used when words, phrases, clauses and
sentences depend on a Principal Clause. Conjunctions
(c) Adversative such as ‘since’, ‘that’, ‘lest’, ‘until’, ‘even if’, ‘as’, ‘since’,
(d) Illative ‘as’, ‘when’, are a case in point.
(a) Cumulative or Copulative—Such conjunctions As,
simply join two statements or facts. (a) He knows that the grass is greener in the other
And, also, too, not less than, as well as, not side.
only……but also, both…and. (b) Greg can not enjoy life since his wife has died.
Consider the following— (c) Study well lest you should fail.
He is a king and I am a pauper. (d) Wait until dark.
Sam is both ugly and stupid. (e) Do not be afraid even if I’m not here.
You as well as your father is a thief. (f) She is not so kind as her mother.

ww
The police no less than the politicians are guilty.
My father went to Delhi too.
(g) I talked to him when he was angry.
Points to note—
(a) Subordinating conjunctions assist in joining one

w
He is not only dishonest but also mean.

.Ea
Note—1. ‘Also’ and ‘as well as’ are never used after
‘both’.
2. When two subjects are joined by ‘as well as’ the
independed clause to another dependent clause (subor-
dinate clause)
(b) Important Dependent Clauses—

syE
‘If’, ‘unless’, ‘until’, ‘in case’, ‘that’, ‘so that’, ‘till’,
verb ought to agree with the first subject. ‘until’, ‘before’, ‘after’, ‘so long as’, ‘because’, ‘why’,
(b) Alternative or Disjunctive—Conjunctions like, ‘when’, ‘where’, ‘while’, ‘whether’, ‘how’, ‘whence’,
‘either……or’, ‘neither…..nor’, ‘or’, ‘else’, ‘otherwise’,

ngi
‘who’, ‘which’, ‘what’, ‘as if’, ‘as soon as’, ‘than’, ‘as’,
that give an alternative choice or offer are called Alterna- ‘since’, etc.
tive or Disjunctive conjunctions. 1. Principal Clause—A Principal Clause expresses
As,
(a) Either you or I am lying. nee
a complete meaning. It is also called independent clause.
Subordinate / Dependent Clause—A subordinate or
Dependent clause unlike a Principal Clause does not
(b) Neither England nor America is my country.
(c) Dress quickly or (else) you will miss your bus.
rin
express complete meaning and is dependent on the Prin-
cipal Clause to make sense.
(d) Confess, otherwise you will be punished.
(c) Adversative—Conjunctions like, ‘but’, ‘still’,
g.n
2. Time—Conjunctions ‘like’, ‘when’, ‘as’, ‘while’,
‘before’, ‘since’, ‘after’, ‘until’, ‘till’, ‘as soon as’, ‘as
‘yet’, ‘nevertheless’, ‘while’, ‘however’, ‘only’, ‘whereas’
show contrast or opposition and are called Adversative
Conjunctions.
Consider the following sentences—
Consider these sentences—
(a) I witnessed as accident
while (Conjunction)
e
long as’ etc. indicate the time of an action, state etc.

(Principal clause)
I was driving
t
(Dependent
(a) Meera is gentle while / whereas her sister is
coarse. Clause).
(b) Tell the truth only don’t involve me. (b) He phoned me (Principal Clause) when
(c) We had no money, nevertheless we survived. (Conjunction) I was dressing. (Dependent Clause)
(d) She was unhappy still / yet she smiled. (c) I shall be faithful to him (Principal Clause)
(e) Bill Gates is very wealthy but he is humble.
as long as (Conjunction) he is alive. (Dependent
(d) Illative—Conjunctions such as ‘hence’, ‘there-
Clause)
fore’, ‘thus’, ‘so’, ‘consequently’, ‘then’, ‘so’, ‘for’,
prove or infer statement or fact from another. Such (d) Stay there (Independent Clause) till
conjunctions are called Illative Conjunctions. (Conjunction) I call. (Dependent Clause)
Consider the following sentences—
(e) Mother had cooked the food (Independent
(a) He stole the money therefore / so he was beaten.
(b) Raymond did not pass the entrance examination; clause) before (conjunction) father arrived. (Depen-
hence he could not join the college. dent Clause)

482 | CAT Complete Course

Visit : www.EasyEngineering.net
Visit : www.EasyEngineering.net

3. Place—Conjunctions like, ‘where’, ‘wherever’ (d) ‘Lest’ is, followed by ‘should’ and ‘not’ is never
indicate place. employed in the ‘lest’ clause.
As, 7. Result or Effect—Conjunctions like, ‘so…….
that’, ‘such…..that’, ‘that’ etc. express result or effect.
(a) He knows (Independent clause) where
As,
(Conjunction) I went. (Dependent clause)
(a) He was so lazy (Principal Clause) that
(b) She runs (independent clause) wherever
(Conjunction) he failed all the tests. (Subordinate
(conjunction) she wants. (Dependent clause) clause)
4. Cause or Reason—Conjunctions like, ‘because’, (b) She ate so much (Principal Clause) that
‘since’, ‘as’, etc. express cause or reason.
(Conjunction) she vomited. (Subordinate clause)
Consider the following—
(a) She came late (Principal Clause) because (c) They are such fools (Principal Clause) that

(Conjunction) she missed the bus. (Subordinate clause) (Conjunction) everyone fools them. (Subordinate
clause)

ww
(b) Sam is heard broken (Principal Clause) since
(Conjunction) his wife has died. (Subordinate clause)
8. Comparison—Conjunctions like, ‘as much as’,
‘as…..as’, ‘than’, etc. express comparison.
(c)
w They retired early

.Ea
(Principal Clause) as
(Conjunction) they were tired. (Subordinate clause)
5. Condition—Conjunctions like, ‘if’, ‘as if’,
(a) Ray is taller (Principal Clause)
(Conjunction) his brother (is). (Subordinate clause)

(b) Our college is as good (Principal Clause) as


than

As,
(a) I will work for you
syE
‘unless’, ‘provided’ etc. express condition.

(Principal Clause) if
(Conjunction) your college (is). (Subordinate clause)

(c) I like pepsi as much (Principal Clause) as

(Conjunction) you pay me well. (Subordinate clause)


ngi
(Conjunction) I like tea. (Subordinate clause)
9. Concession or Contrast—Conjunctions like,
(b) She walked (Principal Clause)
(Conjunction) she were a queen. (Subordinate clause)
as if

nee
‘though’ / ‘although’, ‘however’, ‘even if’, ‘notwith-
standing that’, ‘even though’, etc. express contrast or
concession.
(c) He will marry me (Principal Clause) whether
(Conjunction) his parents agree or not. (Subordinate rin
Consider the following sentences—

clause)
(d) Harry won’t get selected
(d)

g.n
He is a miser (Principal Clause) even though
(Conjunction) he is very wealthy. (Subordinate clause)

e
(Principal Clause)
(e) You won’t get selected

t
(Principal Clause)
unless (Conjunction) he improves his acting. (Subor-
dinate clause) however (Conjunction) good you may be. (Subor-
dinate clause)
(e) I will help you (Principal Clause) provided
(f) He will give his consent (Principal Clause)
(Conjunction) you are sincere. (Subordinate clause)
even if Cconjunction) it hurt him. (Subordinate
6. Purpose—Conjunctions like, ‘that’, ‘so that’, in
order that ‘lest’ etc. indicate purpose. clause)
As, (g) She is not mean (Principal Clause)
(a) I breathe (Principal Clause) that (Conjunc- though / although (Conjunction) she is very poor.
tion) I may live. (Subordinate clause) (Subordinate clause)
10. Extent or Manner—Conjunctions like, ‘so….as’,
(b) She studied hard (Principal Clause) lest ‘as if’, ‘according to’, ‘as far as’ etc. indicate manner or
(Conjunction) she should fail. (Subordinate clause) extent.
As,
(c) They whispered (Principal Clause) so that
(h) He behaves (Principal Clause) as if (Conjunc-
(Conjunction) they wouldn’t be heard. (Subordinate
clause) tion) he were a king. (subordinate clause)

CAT Complete Course | 483

Visit : www.EasyEngineering.net
Visit : www.EasyEngineering.net

(i) They will be placed (Principal Clause) (e) Though he is poor (Sentence) yet he is honest.
(Sentence)
according to (Conjunction) their abilities. (Subor-
13. Usage of Correlative Conjunctions—
dinate clause)
Either…….or, Neither……..nor
(j) He talks (Principal Clause) as (Conjunction)
(a) Harry got both a car and a cheque. (√)
the French do. (Subordinate clause) Harry both got a car and a cheque. (×)
Note—Subordinate clauses can come before a Prin- (b) He is both handsome and rich. (√)
cipal Clause sometimes. He both is handsome and rich. (×)
As, (c) They are not only rich but also famous. (√)
Subordinate Clause Principal Clause Not only they are rich but also famous. (×)
(a) Since he is sincere. I must help him. (d) He is not only a doctor but also a surgeon. (√)
(b) Unless you study. You can not clear the test. He not only is a doctor but also a surgeon. (×)
(c) As far as I know. He is a gentleman. 14. Whether …………..or / not—
(d) When he entered He found her asleep. (a) They don’t know whether she is alive or not.

ww the bedroom.
11. Correlatives—Conjunctions that come in pairs
are called correlatives. These are also coordinating con-
(b) Whether he sings or not is uncertain.
15. Although / though ……….yet—

w
(a) Although (though) he was talented yet he is
junctions some important correlatives— unemployed.

.Ea
‘neither….nor’, ‘either…...or’, ‘though……yet’,
‘both…….and’, ‘such…….as’, ‘such…..that’, ‘as…..as’,
(b) Although (though) she studied hard yet she
failed.

syE
‘as…..so’, ‘so……..that’, ‘scarcely……when’, ‘not 16. Rather / other …… than—
only……but also’, ‘rather…..than’, ‘no sooner………. (a) He has no other friend than Ravi.
than’, ‘whether…….or’.
(b) I would rather die than sweep floors.
12. The functions of correlatives—
To join—
ngi 17. Such………that / Such……….as—
(a) There was such a noise that everyone evacuated
I. A noun / pronoun to another noun / pronouns.
II. An Adjective to another Adjective.
III. An Adverb to another Adverb.
nee
the area.
(b) He is such an idiot that no one ever invites him.
(c) She is not such a fool as I thought.
IV. A Phrase to another Phrase.
rin
18. No sooner……..than—
(a) No sooner did it thunder than it started to rain.

g.n
V. A Clause to another Clause.
VI. A Sentence to another Sentence. (b) No sooner does she come than she starts
working.

e
VII. A Subject to another Subject.
19. Scarcely / Hardly……… when / before—
VIII. An Object to another Object.
IX. A Complement to another Complement.
As,
(a) Either he (Pronoun) or she (Pronoun) is inno-
(a) Scarcely had she lay down when someone
knocked at the door.
(b) Scarcely had we reached home before the rain
started.
t
cent. Note—If a sentence contains NO / NOT / NEVER
(b) He can neither read (Verb) nor write. (Verb) and followed by a clause, use ‘or’ and not ‘nor’.
(c) He is not only rich (Adjective) but also modest. As,
(Adjective) (a) He has no cycle or scooter. (not ‘nor’)
(d) She is both a nurse (Complement) and a (b) John did not talk or look at me. (not ‘nor’)
compounder. (Complement) (c) They never read or write. (not ‘nor’)
●●●

484 | CAT Complete Course

Visit : www.EasyEngineering.net
Visit : www.EasyEngineering.net

3 Sentence of Connectors–Linkers
1. A word or words that join two sentences together (b) Reason
is called a sentence connector. There are some conjunc- I had to join as they needed me.
tions which do the work of connectors.
As, (c) ‘As’ used as ‘while’
(a) This restaurant is both popular and inexpensive. As he was playing, they cheered him.
(b) These pills are not only effective but also (d) In the way / same way or manner
reasonably priced. They act as you do.

ww
(c) He will either phone or write to you.
(d) Go by taxi or (otherwise) you will be late.
(e) Used as though / although
Stupid as she is, she’s careful with her money.

w
(e) He knows that his father is dead. 5. Usage and functions of ‘since’
(f) She was too late to attend the meeting. (a) After a past time

.Ea
2. Some more important sentence connectors—
(a) He has eaten his lunch so has she.
He has become a drunkard since his wife left him.
(b) From that time hence

syE
(b) He is exhausted yet he will work.
(c) I can’t go to Srinagar, moreover it is not safe.
We have been meeting every weekend ever since our
school days.

ngi
(c) Because / Seeing (that)
(d) He had lost a fortune; nevertheless he kept
Since you are injured, you should take rest.
hoping.

nee
(d) From a certain time till now
(e) I can’t lift a 100 kgs. nor can you.
(f) I know their plans; you also know it. I have not heard of him since the 1982 Asiad.

rin
(g) I hate gambling, besides it’s very risky. (e) From which time
3. Words and phrases also serve as connectors— Since when have you been working here?
Like, generally, Particularly, Similarly, Fortunately,
Sadly, For example, likewise, at last, Surely, In general
etc. g.n
6. Usage and functions of ‘while’
(a) During a time / period (that)

However, it must be noted that conjunctions alone do


not serve as sentence connectors, Adverb / Adverbial
phrases also serve as connectors sentence connectors join
two sentences and not two phrases.
(b) To show contrast
e
While he was reading he listened to music.
They joined me while. I was playing golf.

I enjoying playing chess while my brother enjoys


t
football.
As,
(c) In place of although
Attend the party or you will be sorry.
While she tries fervently, she does not succeed.
Here ‘or you will by sorry’ is not a separate sentence.
It is a co-ordinate clause. 7. Usage and function of ‘than’.
Raj is my friend. He is great fun. ‘Than’ is used as a conjunction and also as a
In the sentence he connects with Raj is my friend. He comparative Adjective or Adverb. It always follows the
is a sentence connector (linkers), and not a conjunction. comparative degree.
But words like, ‘because’, ‘as’, ‘since’, ‘while’, ‘than’, Consider the following sentences—
‘just then’, ‘just’, ‘until’, in the form of conjunctions (a) Jim is taller (comparative Adjective) than Harry.
work as sentence connectors.
(b) He is much smarter (comparative Adjective) than
4. As and its functions— I (am).
(a) As–to an equal degree. (c) I love you more than she (more than she loves
Station was as popular as Mark. you).

CAT Complete Course | 485

Visit : www.EasyEngineering.net
Visit : www.EasyEngineering.net

(Here, in sentence C, than takes the form of a Prepo- It was hardly / scarcely noon when I spoke to him. (√)
sition and is placed before a noun/pronoun to show However, be the problem, I will support you. (x)
comparison)
Whatever be the problem, I will support you. (√)
8. Just then, then, Just—
No sooner I reached the office when he called. (x)
As,
No sooner I reached the office than he called. (√)
(a) I was about to leave, just then is started to snow.
Until you are injured, you must rest. (x)
(b) I was backing into the car park, just then a cat
As long as, you are injured, you must rest. (√)
jumped in the way.
(c) Let me finish my breakfast, then I will leave. Exercise
(d) They have just left (a moment ago). 1. The food is ……… tasty ……… nutrition.
9. Until— 2. The mall is ……… inexpensive ……… decora-
Until–upto the time when tive.
(a) Wait until she leaves. 3. He will ……… walk ……… commute by train.
(b) Don’t talk until the lecture is over. 4. Harry up ……… you will be late.

ww
(c) He will keep trying until he succeeds.
10. Some common errors (conjunctions)—
5. She was ……… full ……… anymore.
6. She is tried ……… is he.
7. Jake can’t buy the book ……… it is too expensive
(x)
w
Unless you are recommended, you will be selected.

.Ea
If you are recommended, you will be selected. (v)
Until you are recommended, you will not be selected.
8. He lost his empire; ……… he still hoped.
9. I dislike bungee jumping ……… it’s very dan-
gerous.
(x)
syE
Unless you are recommended, you will not be
selected. (√)
10. I can’t fly ……… can you.
11. Harold has lost hope ……… he was sacked.

ngi
12. They phoned me ……… I was eating lunch.
He will be selected if that he is recommended. (x) 13. Don’t speak ……… the end of the lecture.
He will be selected provided that he is recom- 14. You will be selected ……… you are recom-
mended. (√)
Because he is recommended, he will be selected. (x) nee
mended.
15. Tom studied hard ……… he may pass.
Provided that he is recommended, he will be
selected. (√)
rin
16. ……… the issue I am not interested.
17. ……… I reached the station ……… the train
Because its’ a holiday, I will relax. (x)
Since / as its’ a holiday, I will relax. (√)
arrived.

g.n
18. you must rest ……… you are ill.
19. It was ……… sundown ……… he rang me.
When he played the spectators cheered. (x)
While he played the spectators cheered. (√)
She studied hard because she may pass. (x)
She studied hard in order that she may pass. (√)
Answers e
20. Let me complete my work ……… I will relax.

1. both ;and 2. not only; but also 3. either; or 4.


t
otherwise/ or 5. too; to eat 6. so 7. as 8. never the less 9.
A good woman if she is, must be respected. (x)
nor 10. besides 11. since 12. while 13. until 14. provided
A good woman that she is must be respected. (√) that 15. in order that 16. Whatever 17. No sooner; than
It was rather noon while I spoke to him. (x) 18. as long as 19, hardly/scarcely; when 20. then
●●●

486 | CAT Complete Course

Visit : www.EasyEngineering.net
Visit : www.EasyEngineering.net

4 Subject Verb Agreement


1. The Subject verb Agreement is vital to both the As,
spoken and the written form. If mismatched it will sound Present
and look wrong.
1. Are you reading ? (Present Continuous)
The Main thing to remember is that, singular
2. Do you watch movies ? (Present Indefinite)
subjects take singular verbs whereas, Plural subjects take
Plural Verbs. 3. Have they enjoyed the play ? (Present Perfect)
Noun + s / es / ies = Plural 4. Have the boys been playing ? (Present Perfect
Continuous)

ww
As,
Noun Plural
Fans
Verb (Plural)
are / do / have/ were / ……
Do the auxiliary which ends in ‘o’ is an exception
like ‘am’ which takes only ‘I’ as its sole subject.

w
Dishes
Ponies
.Ea
Verb + s / es / ies = Singular
3. Similarly the Past Tense auxiliaries like was and
has take singular subjects and ‘were’ takes plural
subjects.
As,
As,
Singular Verb
Runs
syE
Plural Verb
Run
Singular
Was he ?
Plural
Were they ?

Goes
Replies
Go
Reply ngi Has she ? Were we ?
Note—that other auxiliaries that end in ‘d’ like ‘did’,
‘had’ and ‘l’ like, ‘will’ / ‘shall’ take all subjects singular
Singular Subject + Singular Verb
Plural Subject + Plural Verb nee
and plural alike.
Past Indefinite
As,
Singular Plural
Did he play ?
Did they play ? rin
He runs
It goes
They run
We go
Did she play ?
Did we play ? g.n
She replies You reply
By these examples, you must have understood, by
now, that Singular Noun + Singular Verb and Plural
Noun + Plural Verb. Keep this important rule in mind. It
Past Perfect Continuous
Had I been reading ?
Had she been reading ?
e t
will help you avoid making subject – Verb Agreement Had it been reading ?
mistakes. Had we been reading ?
To makes a quick and correct choice of the subject-
Past Perfect
Verb combination. Remember this easy point.
Had I slept ?
2. The Auxiliaries (Helping Verb) that end in ‘s’ take
singular subjects. Had you slept ?
As, Had she slept ?
Present Had he slept ?
1. Is Ram going home ? (Present Continuous) Future Indefinite
2. Does she play chess ? (Present Indefinite) Shall I go ?
3. Has he come ? (Present Perfect) Shall we go ?
4. Has Rchana been studying ? (Present Perfect Will he go ?
Continuous) Will they go ?
And auxiliaries that end is ‘e’ take plural subjects. Will it go ?

CAT Complete Course | 487

Visit : www.EasyEngineering.net
Visit : www.EasyEngineering.net

Note that in Modern grammar ‘will’ takes all subjects ‘an’, ‘the’ to recognize Plural Forms. Also, keep an eye
and ‘shall’ is no longer used for I and We subjects. How- for possessives like his, her, our, your, its etc.
ever, in the written form shall is employed with I and We. As,
But, the discretion lies solely with the writer (or speaker). Mr. Smith, her uncle has (singular verb) arrived.
4. When two singular nouns talk about the sense Mr. Smith and her uncle have (Plural verb) arrived.
person or thing and possess only one article or other
qualifying word before them the verb is singular. Note—Here ‘and’ join two subjects and does not
refer to the same person. Unlike 7.
As,
So, when two subjects are joined by a connector (in
The king and philanthropist is here. this case ‘and’) look out for the article or possessive to
When he was born, his father and predecessor was identify plural forms.
famous. 12. When two objects give the idea of being one unit
5. When two nouns express the same idea and one is then, the singular verb is used.
added to the other for the sake of emphasis the verb is As,
Singular.
Curry and Chawal is a good dish.
As,

ww
Their success and fame is notable.
Etiquette and manners is the mark of a gentleman.
(Curry and Chawal, which means a ‘dish’ containing
curry and chawal)
Similarly,

w
6. When two singular nouns are not similar but
express together a single idea, the verb is singular.
As,
.Ea
Bread and butter is our staple food.
Slow and steady wins the race.
Winning and losing is part of the game.
However, there are exceptions to the rule.

syE
Slow and steady wins the race.
7. When a collective noun is used as a common noun
As,
Tears and laugher go (Plural verb) hand in hand.
Time and Tide wait (Plural verb) for no man.

ngi
(but shows some division) the verb is plural.
As, 13. Adjectives when followed, as it mostly does, by a
noun and describes a single noun takes the singular verb.

nee
The jury are divided in their verdict.
As,
The Ministry are divided in their opinion.
The white and blue check shirt is (Singular verb)
8. But when a collective noun is thought of as a unit

rin
mine.
the verb is singular.
The black and blue mark on you face shows (Singular
As,

g.n
verb) you have been beaten.
The team gathers together. The tall fat man is (Singular verb) insane.
The mob moves forward.

e
14. Each / Every / Neither / Either
9. When the plural noun is a proper name for a
collective unit or single object, the verb is singular.
The U.S.A. has failed in its attempt to curb terrorism.
The Andaman and Nicobar Islands is a wonderful
singular verbs.
As,
Each of us is going to protest.
t
Words such as each, every, neither, either one; take

sight.
Every one of the protesters was arrested.
10. At times the subject is joined by the conjunction
‘and’ but refers to one and the same person then the Either of the two is lying.
singular verb is used. Neither of the two brothers is guilty.
As, One of them is at fault.
The king and benefactor has (singular verb) arrived. 15. But, when the there is a singular subject and a
The king and the benefactor have (Plural verb) plural subject the subject closest to the verb agrees with
arrived. the verb.
The first sentence refers to one person as king and As,
benefactor. Thus, singular verb is used. Whereas, the Tom or his friends are guilty.
second sentence treats the king and the benefactor as
separate entities. Therefore, Plural verb is used. Either she or the girls are lying.
11. In case there is a problem in identifying a As,
singular or plural subject. Look out for the Articles ‘a’, You or he is guilty.

488 | CAT Complete Course

Visit : www.EasyEngineering.net
Visit : www.EasyEngineering.net

Either he or I am guilty. Not only your friend but also you (second subject)
Neither they nor he is guilty. are speaking the truth.
16. After ‘or’ and ‘nor’ the verb agrees with the 22. Noun words like the following are only in the
subject that follows. (Nearest to the verb) singular and have no plural form.
As, Furniture, Advice, Information, Equipment, Knowl-
edge, Work, Weather.
That boy or the girls are (Plural verb) wrong.
Diseases : Mumps, Measles etc.
Neither the teacher nor the students are (Plural verb)
present. Sport : Darts, Billiards, Caroms etc.
He or I am (Singular verb) strong. Proper Nouns : The United States, Algiers etc.
Neither you nor he is (Singular verb) capable. Others : Physics, Athletics, Politics, News, Innings
etc.
17. In case, two or more are joined by ‘and’ the
plural verb is utilized. All the above take singular verbs.
As, As,
You, he and I are (Plural verb) fit. Mumps is common among children.

ww
You and he are (Plural verb) going to Delhi.
She and I are (Plural verb) winning.
Mathematics is not a difficult subject.
No news is good news.

w
18. When a subject express distance, time, mass, Furniture is mostly made of wood.
weight, amount as a single unit. It takes the singular verb. Knowledge is invaluable.
As,
.Ea
Twenty kilometers is not a short walk.
The United States is a wealthy nation.
Politics is a dirty game.
Fifty quintals is enough.
syE
Twenty years is a long time.
23. Some, some of, enough, enough of, half, half of,
most, most of, a lot, a lot of, lots of, plenty, plenty of, not
enough of are words, phrases that are followed by plural
Five hundred dollars is a good reward.
19. But, when taken as separate units. It is followed
ngi countable that are followed by plural and those that are
followed by uncountable nouns that are followed by

nee
by a plural verb. uncountable nouns take singular verbs.
As, As,
Ten thousand dollars were (plural verb) expended. Plural Verb
Twenty miles are (plural verb) to be trekked.
20. Subjects that precede words like–besides, as
Some boys are clever.
rin
well as, except alongwith, with together with, not, in
addition to etc are followed by verbs which agree with it,
generation.
Lots of apples were sold.g.n
Not enough of books are read by the modern

according to its number Singular or Plural.


As,
Ram, besides his assistants has failed the test.
I, with my children am going to Delhi.
Singular Verb
Some food was distributed.
Half of the land was barren.
e t
Not enough rice was stored.
Roymond together with his uncles plays cricket.
24. Collective Nouns when placed between a ……..
I, no less than he am guilty of the crime.
of, followed by countable nouns take singular verbs.
Knowledge, in addition to wisdom was Solomon’s
As,
strength.
A bunch (collective noun) of flowers (Countable
He not you is clever.
noun) was (Singular verb) eaten by cow.
You as well as the officer is ignored of the fact.
A gang of thieves is in the area.
21. Where different numbers and persons are the
A troupe of dancers is performing.
subject of words like, ‘but’ not only….. but also the verb
agrees with the second subject. A group of boys was present.
As, 25. Though singular in form words like–sheep,
Not father but I am (second subject) the culprit. poultry, people, cattle etc. take plural verbs always.
Not only your from but also your friend (second As,
subject) is lying. The poultry (chicken / ducks) are missing.

CAT Complete Course | 489

Visit : www.EasyEngineering.net
Visit : www.EasyEngineering.net

The police (members of the police force) were 7. His father and mentor ……… (was/were) present.
informed. 8. Our fame and power ……… (is/am/are) remar-
The cattle (cows and buffalos) are straying. kable.
26. The ‘to infinitive and Gerund take a singular 9. Charm and gallantry ……… (is/am/are) his forte.
verb after it. 10. Bread and butter ……… (am/are/is) our favorite
As, shack.
Dancing keeps one fit. (Gerund + singular verb) 11. Slow and steady ……… (win/wins) the race.
To dance keeps one fit. (to infinitive + singular verb) 12. The jury ……… (is/are) divided in their decision.
27. In optative / conditional and imagination sen- 13. The ministry ……… (is/are) confused about their
tences the plural verb is utilized. posting.
As, 14. The unruly gang ……… (move/moves) on.
Long live the king ! (optative) 15. The U.S.A ……… (have/has)failed to gain our
May God smile upon you ! (Optative) confidence.
If, I were a butterfly. (Condition) 16. The king and the benefactor ……… (has/have)

ww
I wish he were alive. (Imagination)
28. When a noun is in apposition, the first subject
arrived.
17. Mr.gupta, her father ……… (has/have) spoken.
18. Winning and losing ……… (is/are) part of life.

w
agrees with the verb.
As, 19. Time and tide ……… (wait/waits) for no one.

.Ea
I, you brother, am happy.
You, teacher, are wise.
20. The black and white striped shirt ……… (is/are)
yours.

may be used. syE


29. In mathematics both the singular and plural verb
21. Each of you ……… (is/are) clever.
22. Either of the two ……… (is/ are) lying.
23. His friends or I ……… (am/are) innocent.

ngi
As,
One and one is two. (Singular verb) 24. Neither you nor he ……… (is/are) guilt.
25. Twenty kilometers ……… (is/are) a long walk.

nee
One and one are two. (Plural verb)
26. Twenty miles ……… (is/are) to be covered.
Exercise 27. Hari besides his friends ……… (has/have)failed.

rin
Fill in the appropriate words that are in subject - verb 28. No news ……… (is/are) good news.
agreement. 29. Some food ……… (was/were) distributed.

g.n
1. Tommy ……… (go/goes) to school daily. 30. A gang of thieves ……… (is/are) in our locality.
2. ……… (has/have)it been working perfectly ?
Answers

e
3. The children ……… (be/are) playing hopscotch.
1. goes 2. has 3. are 4. shall 5. travels 6. were 7. was
4. I ……… (will/shall) be going home in an hour.
5. Jon ……… (travel/travels) to work by bus.
6. We ……… (were/ was) in a great hurry.
t
8. is 9. is 10. is 11.wins 12. are 13. are 14. moves 15. has
16. have 17. has 18. is 19. waits 20. is 21. is 22. is 23.
am 24. is 25. is 26. are 27. have 28.is 29. was 30. is
●●●

490 | CAT Complete Course

Visit : www.EasyEngineering.net
Visit : www.EasyEngineering.net

Simple, Compound and Complex


5 Sentences
1. Introduction 2. Jack is too impatient to wait.
The conversion of a sentence is to change it from one Jack is so impatient that he can not wait.
grammatical form to another without altering its sense. A 3. The weather is too cold to bathe.
simple sentence contains a finite verb. If we say, ‘Raj is The weather is so cold that we can not bathe.
there’ we form an idea of Raj, of a place and Raj being in
that place. When we say, ‘Raj bit his friends’ ear’ we (B) There are many ways in which one can transform
picture Raj and a bite passing from Raj to his friend. The a conditional sentence. By using ‘if’.

ww
verb in the first statement is intransitive and in the second
the verb is transitive. In each we make more than one
assertion. This is called a Simple Sentence.
1. Allow me to leave and I will not return.
I will not return if you allow me to leave.

w
But, if we say, ‘Raj and Hari talked to Sam’. We

.Ea
make two statements James met Sam, James and Hari
were together. Here we have a compound Sentence.
2. Let the band be superb and I will perform.
I will perform if the band is superb.
(C) By employing a conjunctional phrase—
1. Pay me Rs. 10,000.00 and I will work for you.

syE
Often we make statements modified by some quali-
fication expressed by a clause. A clause is said to be co-
ordinate, when it is possible to separate one from another. you.
In case you pay me Rs. 10,000.00 I will work for

Such that each makes an independent sentence and


independent sense e.g. ‘They left their work and returned
to their homes. A subordinate clause on the other hand is ngi (D) Where the ‘if’ is understood.
1. If I had met him, he would have recognized me.

dependent on the Principal clause or leading assertion.


E.g. She studied hard that she might pass.
nee Had I met him, he would have recognized me.
2. If you had been to America. You would have
understood.
A sentence of this type containing a subordinate or
secondary clause is called a complex sentence.
rin
Had you been to America, you would have under-
stood.
Given below are sentences which show how the
same sense or meaning can be retained and yet conveyed
in a Simple, a Compound or a Complex sentence. have been chaos. g.n
3. If the Government had collapsed, there would

Simple—The problems facing him from all sides did


not deter him.
Compound—He was faced by problems from all
been chaos.
e
Had the Government collapsed, there would have

(E) By employing a participle phrase: t


sides but he was not deterred by them. 1. You can come to the party, if you dress well.
Complex—He was not deterred by the problems that You can come to the party provided that you dress
faced him from all sides. well.
Note that the above changes are in the form and 2. If that were the case, what should we do ?
structure but the meaning remains unchanged. The close Supposing that were the case, what should we do ?
study of the following will be of immense value in
Provided that were the case. What should we do ?
learning a variety of expression and it will definitely add
colour and flavour in preparation of competitive English. (F) By employing ‘but’ followed by a phrase:
2. Means of transforming conditional sentences 1. Had they not been innocent, they would have
(A) A sentence containing the word ‘too’ can be been jailed.
transformed in the following manner. But for their innocence, the would have been jailed.
As, 2. If he does not help you, you would be destroyed.
1. He is too honest to tell a lie. But that he helped you, you were not destroyed.
He is so honest that he can not tell a lie. But for his help, you would have been ruined.

CAT Complete Course | 491

Visit : www.EasyEngineering.net
Visit : www.EasyEngineering.net

(G) By employing the phrases ‘one more’ and ‘were (F) Use the absolute participle—
to’. 1. Though he was very honest at time he would tell
1. If we have one more such failure, we are lost. a lie.
One more such failure and we are lost. Admitting that he was very honest, he at times, tells a
2. If he questioned me, he would recognize me. lie.
If he were to question me, he would recognize me. 2. Though he was not a bright student, they
promoted him.
(H) By employing the Imperative state:
Knowing he was not a bright student, they promoted
1. If you labour, you will achieve.
him.
Labours and you will achieve.
(G) Use the preposition ‘notwithstanding’ followed
2. If you shut the door, strangers will not enter. by a Noun Clause.
Shut the door, and strangers will not enter. 1. Raj is still working although he not rested for 12
(I) By using the interrogatives sentence. hours.
1. If you dance well you will be selected. Raj is still working, notwithstanding that he has not
Do you dance well? Then they you will be selected. rested for 12 hours.

ww
2. You may leave, if you have seen the film.
Have you seen the film? Then you may leave.
2. He still borrows money although he earns more
than enough now.

w
3. To change a sentence that denotes contrast or
concession

.Ea
(A) Use the conjunction ‘though’ or ‘although’.
He still borrows money, not with standing that he
earns more than enough now.
(H) Use ‘if’ or ‘even’ or ‘even if’—
1. He was a poor man, but he was generous.
1. He is rich but just.
He is just though rich.
syE
2. Sarah is beautiful but she is still single. bills.
Even if he was a poor, he was generous.
2. Though he drank in excess, he always paid his

Although Sarah is beautiful, she is still single.


(B) Use ‘as’— ngi If he drank in excess, he always paid his bills.
3. Although he changed his job, yet he is not happy.
1. Though he was healthy, he died.
Healthy as he was, he died. nee He changed his job indeed, but he is not happy.
(I) Use ‘nonetheless’ or ‘nevertheless’—
2. Though he is powerful, he is a coward.
Powerful as he is, he’s a coward.
rin
1. Though they are very wealthy, yet they are stingy.
They are very wealthy, nevertheless they are stingy.
(C) Use ‘however’ and follow it by some adjective
or adverb— envious.
g.n
2. Though he was selected instead of me. I am not

e
1. Though he was healthy, he died. He was selected instead of me, nonetheless I am not
However healthy he might have been, he died.
2. Though he is rich, he is a miser.
However rich he may be, he is a miser.
envious.

thing.
(J) Use the pronoun ‘whatever’—
They will not attend, though I may offer them any-t
(D) Use the phrase ‘all the same’. They will not attend, whatever I may offer them.
He is very weak but he will work. 4. You can transform a sentence by simply changing
He is very weak, all the same he will work. the degree of comparision.
(E) Use the phrase ‘for all that’ and follow it with a (A) 1. Positive – Ram is as smart as Ravi.
Noun clause— Comparative – Ravi is not smarter than Ram.
1. A father will protect his son, though he may be a 2. Comparative – The weather is cooler in
criminal. Bangalore than Chennai.
A father will protect his son, for all that he may be a Positive – The weather in Chennai in not
criminal. so cool as Bangalore.
2. Though you may be innocent, the jury will not 3. Positive – No author writes as well as
believe you. Sidney Sheldon.
You may be innocent; for all that the jury will not Superlative – Sidney Sheldon writes the
believe you. best of all.

492 | CAT Complete Course

Visit : www.EasyEngineering.net
Visit : www.EasyEngineering.net

4. Superlative – She is the most beautiful girl (C) It is a very beautiful flower. (Assertive)
in the class. (D) How beautiful the flower is ! (Exclamatory)
Positive – No girl is as beautiful as her (E) If I could win her hand ! (Exclamatory)
in the class.
(F) I strongly desire to win her hand. (Assertive)
(B) Positive – No other gem is as expensive
as diamond. 9. You may transform a sentence, such that by
changing the Part of Speech of a leading word.
Comparative – Diamond is more expensive
than any other gem. (A) Verb – He failed in his Endeavour.
Superlative – Diamond is the most Noun – His Endeavour was shrouded in
expensive of all gems. failure.
(C) Superlative – U.P. is the largest state in Adjective – His Endeavour was a failure.
India. (B) Verb – I can not permit you to go.
Comparative – U.P. is larger than any other Noun – I can not give you permission to
state in India. go.
Positive – No state in India is as large as (C) Noun – They accepted our proposal.

ww U.P.
5. Another way is to transform the Active form into
Passive form and vice-versa.
Verb – They accepted what we pro-
posed.

w
(D) Adverb – She acts intelligently.
(A) Raymond shot a leopard. (Active)
Noun – She acts with intelligence.

.Ea
(B) A leopard was shot by Raymond. (Passive)
(C) The food was prepared by Aunt Jane. (Passive)
(E) Use ‘not only ………… but also’ in place of
‘besides’.

syE
(D) Aunt Jane prepared the food. (Active)
(E) Who killed your cat ? (Active)
(F) By whom was your cat killed. (Passive)
Simple – Besides being handsome, he was
intelligent too.
Compound – He was not only handsome, but
(G) Free the prisoner. (Active)
(H) Let the prisoner be freed. (Passive) ngi Simple
he was intelligent too.
– Besides scoring a century, he
6. You can change a sentence from the affirmative to
the Negative and also the other way round. nee took five wickets.
Compound – He not only scored a century, but
he also took five wickets.
(A) Mary loves Tim. (Affirmative)
(B) Mary is not without love for Tim. (Negative)
rin
(F) By using the affirmative conjunction ‘or’.
(C) None but the pure hearts will see heaven.
(Negative)
Simple

g.n
– He must surrender, to avoid
being shot.
Compound – He must surrender or he will be
tive)
(D) The pure hearts alone will see heaven. (Affirma-

(E) No other member of the club was as honest as


Paul. (Negative)
Simple
shot.
e
– You must marry him to avoid a
scandal. t
Compound – You must marry him or face a
(F) Paul was the most honest member of the club.
scandal.
(Affirmative)
(G) By using the conjunction ‘and so’.
7. An Interrogatives sentence can be transformed into
Simple – He was arrested for killing his
the Assertive form and vice versa.
wife.
(A) When will they lose a match ? (Interrogative)
Compound – He killed his wife and so they
(B) They will never lose a match. (Assertive) arrested him.
(C) Everyone would hate a heartless father. (Asser- Simple – Owing to an illness he failed the
tive) test.
(D) Who would not hate a heartless father ? (Interro- Compound – He was unwell and so he failed
gative) the test.
8. The exclamatory sentence can be transformed into (H) Change a compound sentence into a simple
the assertive form. sentence by changing a Participle for a finite verb.
(A) Alas! She is undone. (Exclamatory) Compound – The sun set and the cows went
(B) It is sad that she is undone. (Assertive) home.

CAT Complete Course | 493

Visit : www.EasyEngineering.net
Visit : www.EasyEngineering.net

Simple – The sun having set, the cows (B) Substitute a noun for a noun clause preceded by
went home. ‘what’—
Compound – He completed his work and took Complex – They must reveal what he intends
a nap. to do.
Simple – Having completed his work he Simple – They must reveal his plans.
took a nap. (C) Use a Compound Noun.
(I) By using a gerund or infinitive. Complex – We stopped on a hill where the
Compound – He must not be foolish or he will farmer lived.
regret it. Simple – We stopped on the farmer’s
Simple – In the event of his being foolish, residence.
he will regret it. 12. Transform a compound sentence into a complex
Compound – We must fight or we can not win. one.
Simple – We must fight to win. (A) Using a conjunction which adds one statement
10. Transforming a Simple Sentence into a Complex to another.
sentence. Compound – Be brave and you will win.

ww
(A) Use a noun clause.
Simple – I am sure to win.
Complex – If you are brave, you will win.
Compound – Study well or you will fail.

w Complex – I am sure that I will win.


Simple
.Ea
– He admitted his mistake.
Complex – He admitted that he had made a
Complex – Unless you study well, you will
fail.
(B) Use a conjunction which expresses contrast or
opposition.

syE
mistake.
(B) Use an adjective clause.
Simple – He gifted her diamond chain.
Compound – He is a poor but he is always
generous.

Complex – He gifted her a chain which was


made of gold. ngi Complex – He is always generous although
he is poor.
Compound – He had lost but he was not with-
Simple – Serious students are successful.
Complex – Students who are serious are
successful.
nee out hope.
Complex – Although he lost, he was not

(C) Use an adverb clause.


rin
without hope.
13. Transform a Complex Sentence into a Compound
Simple – On his command, the troops will
march.
Complex – The troops will march as soon as g.n
Sentence as illustrated below.
(A) Complex – He is sure you have failed the
test.

Simple
he commands them.
– She owned her fame to her agent.
Complex – It was owing to her agent that
e
Compound – You have failed the test and of
this he is sure.
(B) Complex – As soon as he ate breakfast. He t
she got famous. left.
11. To Transform a Complex Sentence into a Simple Compound – He ate his breakfast and
sentence. immediately left.
Substitute a noun for the Noun Clause preceded by (C) Complex – If you do not earn, you will
‘that’. starve.
Compound – You must work or you will
(A) Complex – She said that she was guilty.
starve.
Simple – She declared her guilt.
(D) Complex – We eat that we may live.
Complex – She prayed that he lived long. Compound – We desire to live, therefore we
Simple – She wished a long life for him. eat.
●●●

494 | CAT Complete Course

Visit : www.EasyEngineering.net
Visit : www.EasyEngineering.net

6 Modals

1. Introduction (D) Shortened form of Shall not, Will not


Shall not = shan’t
The words shall, should, will, would, can, could,
must, used, need and dare are called Modal or Modal Will not = won’t
Auxiliaries. Shortened Form (Positive)
Marginalized Modal Auxiliaries—Marginal Modal I Shall = I’ll
Auxiliaries are Need, Dare, Ought to, Used to. They are We shall = We’ll
not a part of a main or important group and marginalize

ww
You will = You’ll
themselves.
They will = They’ll
2. Shall / Will
He will = He’ll

w
These modals are used to express the future tense.

.Ea
‘Shall’ agrees with the First person subject, I, We
whereas, ‘will’ agrees with Second and third person
subjects.
She will
It will
=
=
3. Uses of Should / Would
She’ll
It’ll

syE
The modals shall and will express more than just the
future. They also express Promise, Intention, Warning,
Determination etc.
Should, is the past form of shall and would, is the
past form of will. The indirect speech employs the use of
should, would in place of shall, will in the Direct speech
(A) Future Tense
I shall go to Delhi. ngi pattern.
(A) Use of Should
We shall stay at the Oberoi’s.
He will pay the bill. nee Should apart from being used in Indirect speech also
expresses Advice and suggestion.

You will go by train.


(B) Intention / Determination / Promise / Warning are
As,

rin
He should attend school regularly.
expressed when the subjects I, We
joins with ‘will’. g.n
One should not tell lies.
You should see a doctor.
As,
I will not tell anyone (promise)
We will try until we succeed. (Determination)
We will report you to the Principal. (Warning)
As,
You should respect elders.
e
(B) To express Moral Obligation.

We should help the disabled.


t
I will buy some more shares. (Intention) We should obey our parents.
(C) When ‘shall’ is used with the second and third We should obey traffic rules.
person it can imply order, Promise, Determination, Threat (C) Lest ….. should to express ‘Purpose’
or Warning.
As,
As,
Be careful lest you should fall.
Rajesh shall be jailed if he breaks the law. (Warning Work hard lest you fail.
/ Threat)
Be alert lest you fall asleep.
They shall find the trip exciting. (Promise)
(D) To express a need
You shall finish it. (Order)
As,
Tom insists he shall become a Judge. (Intention /
There should garbage cans on the sheets.
Determination)
There should be schools in every village.
Note—According to Modern grammar it is now
accepted and alright to use will with all subjects. So, There should be a doctor in every school.
when in doubt just use will. There should be transparency to remove corruption.

CAT Complete Course | 495

Visit : www.EasyEngineering.net
Visit : www.EasyEngineering.net

(E) To express ‘Probability’ As,


As, (A) I must see a doctor. (Need)
He should be in the library. We must leave now. (Immediately due to …)
They should arrive by 7 p.m. I must talk to him now. (Urgency)
Mother should be in the kitchen. (B) Necessity / Compulsion (Future)
(F) To express ‘condition’ I must pass before lest I be expelled.
Here it takes the role of if. He must reach Delhi by tomorrow.
As, He must have at least 70% to enter the college next
Should it snow, I shall stay at home. month.
Should he not arrive, we shall phone him. (C) Emphatic Advice / Prohibition
Should you repent, you will be saved. You must stop drinking alcohol.
(G) To express a polite wish We must fight corruption.
I should like to speak to the Director. We must obey the law.

ww
We should like your assent on this issue. 6. Use of May
(H) To express Moral Duty / Obligation in the Past (A) May like most modals has many uses. It
As, expresses; Possibility, Permission, Request,

w
We should have visited uncle when he was in
hospital.

.Ea
You should have asked your parents.
You should not have lied to your wife.
Wish, Prayer, Purpose, Assumption etc.
Permission (May + V1)
May I use your phone ? (Am I permitted to use your

4. Use of Might syE


The Congress should not have passed such a bill.
phone ?)
May I come in ? (Am I allowed to enter ?)
He may join us for lunch. (I permit him to join us)
As, should, would, could, might is also, used in
Indirect speech. Apart from expressing Possibility, ngi (B) Possibility (May + V1 )
Thomas may come here. (It is possible that he will
Request and Permission, Purpose in the Past and such.
(A) Possibility (Might / be + V1 ) Present / Future
As, nee
come here)
It may rain today. (It is possible that it will rain

It might be a holiday.
He might pass the exam.
today)

rin
Sharon may be at home. (It is possible that Sharon

Sita might visit today.


(B) Request / Permission
will be at home)
(C) Request (May + V1 )
g.n
Might I come in ?
Might he close the door ?
He might come in ?
e
May I shut the window, please ?
May I go out to play, please ?
May I borrow your lawn mower ? t
(C) It also expresses Purpose in the past. (So + Might (D) Offer (May + V1 )
+ V1 ) May I help you ?
He ate so quickly that he might finish first.
May I carry your bag ?
They worked so hard that they might finish in time.
May I fix your motor ?
Jeef studied all night so that he might pass the test.
(D) Unfilled Possibility in the Past (Might + have + (E) Purpose (May + V1)
V3 ) He works so hard that he may qualify.
They might have seen the movie. She is singing so beautifully that she may win the
The children might have eaten. contest.
He might have got the job. (F) Assumption (May + have + V 3 )
5. Use of Must She may have stolen the watch.
Must express Necessity, Compulsion, Prohibition, He may not have forgotten the file.
Emphatic, Advice or determination, duty or possibility in (G) Wish / Prayer / Curse (Optative Sentence) (May
the Present. + V1 )

496 | CAT Complete Course

Visit : www.EasyEngineering.net
Visit : www.EasyEngineering.net

May God curse you ! 8. Use of Can


May God bless you ! Can expresses present ability, Can = able to.
May you live long. Positive statement = Sub. + can + V1 + other words
(H) Uncertainty (May + be) = I can eat ten cakes.
Who may be lurking around ? Interrogative Sentence = Can + Sub. + V 1 + OW ?
Who may be it at this hour ? = Can you lift a 100 kgs. ?
Who may be stealing our hens ? Negative Sentence = Sub. + can + not + V1 + OW
The above usages and examples of Modals are meant = He can not drink 5 litres
to give the reader a clearer picture and view into the many
uses of Modals. However, these can confuse and puzzle. of milk.
To understand Modal usage. Keep it simple. Examples—
For example— I can pass this test.
Use Shall / Will simply as the future tense. He can speak French and Russia.
Use Can for present ability, informal permission and Can you break the lock?

ww
possibility.
Use Could for past ability, Polite request and
possibility.
He can not fix the motor.
(A) Can is also used to express the future tense =
Can = Shall / will be able to.

w
Use May, for permission and possibility.

.Ea
Use might for possibility.
Use should for advice / suggestion.
As,
He can meet one tomorrow.
I can give you a loan next week.

Use Dare for challenge. syE


Use would for past habit / routine. (B) Can also operates as–know how to.
He can solve this equation. (Knows how to)

ngi
Use need / to for necessity. I can train the cadets. (Know how to)
Use ought to for advice / suggestion. (C) Can also expresses probability.
7. Use of Would
As mentioned earlier ‘would’ is the past equivalent
of will and is used in Indirect
nee
As,
Mr. Gupta can arrive anytime.
The rumor can be true.
speech besides, other functions.
rin
(D) To express or ask permission (Informal).

g.n
(A) Direct Speech As,
They said, “We will pay the bill”. Can I go out to play ?

e
Indirect Can I borrow your car ?
They said that they would pay the bill.
Direct
She said to me, “You will regret tomorrow”.
You can stay as long as you like.
(E) It also expresses offer / request.
As,
Can I help you ?
t
Indirect
Can you close the window ?
She told me that I would regret the next day.
(B) To express Past Habits 9. Use of Could
As, Could is in the Indirect Speech.
When I was younger I would walk 10 mile everyday. As, Direct

Ten years ago I would smoke and packs of cigarettes. She said, “I can solve this issue”.
Now, I don’t smoke at all. Indirect
(C) To express Hypothetical Conditions She said that she could solve that issue.
If I were a butterfly, I would kiss all the flowers. Direct
If you had entered the contest, you would have won. The boss said, “I can employ you”.
If I were the President of America, I would not Indirect
support Pakistan. The boss said that he could employ him.

CAT Complete Course | 497

Visit : www.EasyEngineering.net
Visit : www.EasyEngineering.net

(A) ‘Could’ is also utilized for Past Ability. (B) Dare (Challenge)
As, You dare to say so.
I could lift a 100 kgs. When I was younger. How dare you!
He could speak Bengali when he loved in West (I) dare you ask her.
Bengal.
Exercise
He could run 10 miles when he was fit.
Place the most suitable modal in the blanks—
(B) Could used as expression / possibility
1. I ………… go to Agra this month.
As,
2. He ………… answer the phone.
They could help the victims of the earthquake.
3. I ………… not tell a soul.
Ram could have got the job.
4. we ………… continue fighting until we win.
(C) ‘Could’ as a request.
5. Raymond ………… be punished if he cheats.
Could I help you ?
6. You ………… brush your teeth daily.
Could I come in ?
7. ………… it rain, I shall not go …………
(D) To express Condition

ww
If I were the P.M., I could pass the bill.
If I were the Principal, I could pass you.
8. It ………… rain today.
9. I ………… meet him now.

w
(E) For Suggestion

.Ea
You could discuss it with your father.
You could join a health club.
10. ………… I use your computer ?
11. He ………… go for long walks in the woods.
12. I ………… lift 50 kgs of weights
13. He ………… work for 8 hours non-stop.

syE
(G) Possibility in the past
I could have entered the contest.
You could have joined our team.
14. He said that he ………… return the next day.
(indirect speech).
15. Harry ………… a pair of shoes.
10. Use of Need / Dare (Necessity)
ngi 16. How ………… you!
17. I ………… smoke 6 packs of cigarettes a day.
Ravi needs a pair of shoes. His old ones are torn.
(Present)
Tom needed Harry’s help. (Past) nee 18. I ………… meet you tomorrow.
19. ………… I play outside?
They will need monetary support. (Future)
Thus, the above examples show that ‘need’ can be rin
20. You ………… obey your parents.
Answers
utilized in all three tenses.
(A) Need + not denote absence of necessity. g.n
1. shall 2. will 3. will (promise) 4. will (determi-
nation) 5. shall (warning) 6. should (advice) 7. should 8.
As,
You need not take a bath.
They need not visit us.
e
might (possibility) 9. must (urgency) 10. may (permis-

t
sion) 11. would (used to ) 12. ability (present) 13. ability
(past) 14. could / would 15. needs(necessity) 16. dare
(challenge) 17. would (past habit) 18. shall (simple future)
She need not enter the contest. 19. can (informal request) 20. suggestion
●●●

498 | CAT Complete Course

Visit : www.EasyEngineering.net
Visit : www.EasyEngineering.net

7 Tenses

A. Verb Forms Stand Stood Stood


Win Won Won
1. Before we go into the tenses it is imperative to
understand the importance of verb forms which are the (b) In which all three forms are similar.
pillars on which the tenses can stand. Present Past Past Participle
There are two types of verb forms— Bet Bet Bet
(a) Regular (b) Irregular Cut Cut Cut

ww
Regular verbs form their past tense and past participle
by adding ‘ed’ to its first form. As, play–played (past
Let
Put
Let
Put
Let
Put

w
tense) played (Past Participle). Split Split Split
Irregular verbs are different as they form their past

to regular verbs. .Ea


tense and past participle in a different manner as opposed
(c) In which all three forms are dissimilar.
Present Past Past Participle

syE
They change their form in the following ways.
(a) In which two forms are similar.
Be
Do
Eat
was / were
Did
Ate
Been
Done
Eaten
(b) In which all three forms are the some.
(c) In which all three form are dissimilar.
ngi Rise
Write
Rose
Wrote
Resin
Written
2. Examples of Irregular Forms
(a) In which two of the form are similar.
nee Also,
(d) In which the vowels are changed.

rin
Present Past Past Participle Present Past Past Participle
Awake Awoke Awoke Swim Swam Swum
Bleed
Catch
Bled
Caught
Bled
Caught
Drink
Wring
Drank
Wrung g.n Drunk
Wrung
B. Tenses

1. Formation of Sentences
PRESENT TENSE e t
Positive Interrogate Negative
A I go to school. Do I go to school ? I do not (don’t) go to school.
He / she / it / Ram goes to school. Does he / she / it / Ram go to school ? He / she / it / Ram does not (doesn’t)
I, You, We, They take the plural form go to school.
of the verb (do)
Subject + V1 + o / ow A.Verb1 + Subject + V 1 Subject + Do/Does + not + V1
B I am going to school. Am I going to school ? I am not going to school.
You / We / They are going to school. (You / We / They + are) You / We / They are not (aren’t) going
He / She / It / Ram is going to school. Are we going to school ? to school.

Is he / she / it / Ram going to school ? He / She / It / Ram is not (isn’t) going


to school.
Subject + Aux. + V1 + ing + ow Aux. + Subject + V1 + ing + ow Subject + Aux. + not + V1 + ing + ow

CAT Complete Course | 499

Visit : www.EasyEngineering.net
Visit : www.EasyEngineering.net

C I / You / We / They have been to Have I / You / We / They been to I / You / We / They have not (haven’t)
school. school ? been to school.
He / She / It / Ram has been to school. Has he / she / it / Ram been to school ? He / She / It / Ram has not (hasn’t)
been to school.
Subject + Aux. + V3 + ow Aux. + Subject + V3 + ow Subject + Aux. + not + V3 + ow
D I / You / We / They have been going Have I / You / We / They been going to I / You / We / They have not been
to school + (for / since) school + (for / since) ? going to school + (for / since)
He / She / It / Ram has been going to Has He / She / It / Ram been going to He / She / It / Ram has not been going
school + (for / since) school + (for / since) ? to school + (for / since)
Subject + Aux. + V3 (been) + V 1 + Aux. + Subject + been (V3 ) + V 1 + ing Subject + Aux. + not + been + V1 +
ing + ow + (For / Since) ing + ow + (For / Since)

Past Tense
2. Formation of Sentences
Positive Interrogate Negative
A

ww I / You / We / They went to school.


He / she / it / Ram went to school.
Did I / You / We / They go to school ?
Did he / she / it / Ram go to school ?
I / You / We / They / He / She / It /
Ram did not (didn’t) go to school.

w
Subject + V2 + o / ow Aux. (Did) + Subject + V1 + ow Subject + Aux. (did) + not + V1 + ow
B You / We / They were going to Were You / We / They going to You / We / They were not (weren’t)
school.
.Ea school ? going to school.
I / He / She / It / Ram was going to Was I / he / she / it / Ram going to I / He / She / It / Ram was not (wasn’t)
school.

syE
Subject + Aux. + V1 + ing + ow
school ? going to school.
Aux. (was / were) + Subject + V 1 + ing Subject + Aux. (was / were) + not + V1
+ ow + ing + ow
C
ngi
I / You / We / They / He / She / It / Had I / You / We / They / He / She / It / I / You / We / They / He / She / It /
Ram had been to school + *(before + Ram been to school ? Ram had not (hadn’t) been to school.
V2)
*when one action was completed
before another starts. nee
V2) subject + V 2 ) rin
Subject + had + V3 + ow + (before + Had + Subject + V3 + ow + (before + Subject + Aux. (Had) + not + V3 + ow
+ (before + subject + V2)
D
g.n
I / You / We / They / He / She / It / Had I / You / We / They / He / She / It / I / You / We / They / He / She / It /
Ram had been going to school + (for / Ram been going to school + (for / Ram had not been going to school.
since) since)?

e
Subject + had + (been) + V1 + ing + Aux. (had) + Subject + been + V1 + ing Subject + Aux. (had) + not + been +
(for / Since) + (For / Since) V1 + ing + ow
t
Future Tense
3. Formation of Sentences
Positive Interrogate Negative
A I / We shall go to school. Shall I / We go to school ? I / We shall not (shan’t) go to school.
You / They / He / she / it / Ram will Will You / They / He / She / It / Ram You / They / He / She / It / Ram will
go to school. go to school ? not (won’t) go to school.
Subject + Aux. (shall / will) + V1 + Aux. (shall / will) + Subject + V 1 + ow Subject + Aux. + not + V1 + ow
ow
B I / We shall be going to school. Shall I / We be going to school ? I / We shall not (shan’t) be going to
You / They / He / She / It / Ram will Will You / They / He / She / It / Ram school.
be going to school. be going to school ? You / They / He / She / It / Ram will
not (won’t) be going to school.
Subject + Aux. (shall / will) + be + V1 Aux. (shall / will) + Subject + V 1 + + Subject + Aux. + not + be + V 1 + ing +
+ ing + ow ow ow

500 | CAT Complete Course

Visit : www.EasyEngineering.net
Visit : www.EasyEngineering.net

C I / We shall have been going to school Shall I / We have been going to school I / We shall not (shan’t) have been
+ (*for) (for) ? going to school.
You / They / He / She / It / Ram will Will You / They / He / She / It / Ram You / They / He / She / It / Ram will
have been going to school. (*for) have been going to school (for) ? not have been going to school.
*only ‘for’ is used in this tense, since
is not used.
Subject + Aux. (shall / will) + be + V1 Aux. (shall / will) + Subject + have + Subject + Aux. + not + Have + been+
+ ing + ow been + V1 + ing + ow V1 + ing + ow
D I / We shall have been to school. Shall I / We have been to school ? I / We shall not have been to school.
You / They / He / She / It / Ram will You / They / He / She / It / Ram have You / They / He / She / It / Ram will
have been to school. been to school ? not have been to school.
Subject + Aux. (shall / will) + have + Aux. (will / shall) + Subject + have + Subject + Aux. + not + been + V3 + ow
V3 + ow V3 + ow

Fill in the appropriate Present Perfect Continuous 3. No sooner than the music ………. (start / starts)
Tense—
ww
1. The boy …………. (cry) …………. 2 hours.
he …………. (begin / begins) laughing.
4. Normally, 2nd class travelers ………. (do / does)
not stand in line.

w
2. Students …………. (study) …………. Morning.
3. A car …………. (honk) …………. 30 minute.

.Ea
4. Dogs …………. (bark) at people …………. Last
night.
5. Those who …………. (work / works) in temples
…………. (get / gets) free meals.
6. The baby ………. (laugh / laughs) like an angel.

immemorial. syE
5. Beggars ………………. (beg) ……………. Time 7. A loud blast ………. (sound / sounds) and a train
…………. (roll / rolls) in.
8. The wind ……………. (blow / blows) and dust

ngi
6. You …………. (exercise) …………. 4 years.
…………. (rise / rises).
7. The grass ……………. (grow) ……………. Last
winter. 9. .…………. (do / does) you play chess ?
8. Cats …………. (mew) all night.
9. It …………. (work) …………. Last week. nee 10. …………. (do / does) Sita dance?
11. Birds …………. (fly / flies) and cattle ………….

rin
(walk / walks).
10. Tom and Jill …………. (argue) …………. Days.
12. We …………. (love / loves) our country as much
11. Shane …………. (writer) …………. Years.

g.n
as we …………. (respect / respects) it.
12. They ……………. (swim) non-stop …………….
13. Hari always …………. (speak / speaks) the truth.
Yesterday.
13. King David …………. (rule) …………. 20 years.
14. He …………. (walk) …………. 2008.
15. People …………. (vote) …………. Ages.
e
14. Those flowers …………. (look / looks) beautiful.
15. He …………. (count / counts) his pennies and
…………. (waste / wastes) his pounds.
16. God …………. (help / helps) those who
t
Answers …………. (help / helps) themselves.
1. has been crying for 2. have been studying since 17. The office …………. (open / opens) at 9 a.m.
3. has been honking, for (the past) 4. have been barking,
18. The teacher …………. (give / gives) us a test
since 5. have been begging, since 6. have been exercising,
every month.
for 7. has been growing, since 8. have been mewing
9. has been working, since 10. have been arguing, for 19. The sun …………. (rise / rises) in the east and
11. has been writing, for 12. have been swimming, since …………. (set / sets) in the west.
13. has been ruling, for 14. has been walking, since 20. She …………. (go / goes) to school daily.
15. have been voting, for
21. A baby …………. (cry / cries) when its hungry.
Simple Present Tense (Exercises) 22. Raj Sir, our Maths teacher …………. (teach /
Fill in the blanks with the appropriate word. teaches) us but when he is absent Ravi sir
Remember subject verb agreement— …………. (take / takes) our class.
1. Sometimes she also ………. (sing / sings) songs. 23. Mother …………. (cook / cooks) our food.
2. Hermits …………. (lead / leads) simple lives. 24. I …………. (do / does) my work well.

CAT Complete Course | 501

Visit : www.EasyEngineering.net
Visit : www.EasyEngineering.net

25. The neighbors dog …………. (bark / barks) for 3. Does she reply ?
no rhyme or reason. Yes, she replies (By adding ‘ies’)
26. The cattle …………. (eat / eats) all the paddy in However, in the case of the Interrogative and negative
our field. the auxiliary Do and Does can not be omitted.
27. The furniture …………. (do / does) not match. As,
28. We …………. (drive / drives) very carefully. Do you play ?
29. The cock …………. (crew / crows) first before No, I do not (don’t) play.
dawn.
Does she sing?
30. You …………. (talk / talks) to your parents very
No, she does not (doesn’t) sing.
rudely.
Looks simple, doesn’t it ? Well, it is. But you need to
Answer practice in order to become perfect. Try, the exercises
1. sings 2. lead 3. starts–begins 4. do 5.work given hereafter to familiarize yourself with the simple
6. laughs 7. sounds 8. blows–rises 9. do 10. does 11. fly Present Tense.
12. walk 13. speaks 14. look 15. counts–wastes 16. helps Fill in the blanks (Interrogative)—

ww
17. opens 18. gives 19. rises–sets 20. goes 21. cries
22. teaches–takes 23. cooks 24. do 25. barkes 26. eats
27. does 28. drive 29. crows 30. talks.
1. …………. she watch T.V. ? (Do / Does)
2. …………. You like movies ? (Do / Does)

w
3. …………. it work ? (Do / Does)
Simple Present
4. …………. I teach well ? (Do / Does)

.Ea
Structure—To make a question (Interrogative) all
you have to do is follow these simple steps.
5. …………. they dance ? (Do / Does)
* Examples of Interrogative, Positive, Negative will
Aux. + subject + V 1

syE
The auxiliary verbs (Helping) in the simple tense are
Do and Does.
be added.
Present Continuous Tense (Exercises)
Thus,
Aux. + Subject + V1 ngi Fill in the blanks with the correct Present Continuous
Tense—
Do
Does
you
he
play ?
sing ? nee 1. Jamie …………. (exercise) at the moment.
2. She …………. (sing) a song.
‘Do’ takes plural subjects while ‘Does’ takes
singular ones.
rin
3. The sun …………. (rise) now.
4. They …………. (play) our song.
Positive Answer—
Yes + Subject + Aux. + V1 g.n
5. It …………. (work) perfectly.
6. He …………. (talk) on the phone.
Yes, I do play.
In the simple Present Tense one usually omits. Do
while making a positive reply or Positive Assertive
statement.
e
7. The crowd …………. (gather) around him.
8. Mother …………. (cook) the breakfast.
9. The baby …………. (cry) as it is hungry. t
10. You ………. (watch) this movie and I ……….
Examples—
(watch) that one.
Do you play ?
11. His children …………. (tear) their books.
Yes, I play.
12. The troupe …………. (entertain) the guests.
In a positive statement a simple ‘I play’ is sufficient.
13. The plot …………. (unfold) after all.
But, when one omits the Auxiliary ‘Does’ in a
14. The jury …………. (decide) the case.
positive answer or statement remember to add, s, es, or
ies, as applicable. 15. The class …………. (take) an exam.
Examples— Answer
1. Does he sing ? 1. is exercising 2. is singing 3. is rising 4. are playing
Yes, he sings (Add ‘s’ to the V1 ) 5. is working 6. is talking 7. is gathering 8. is cooking
9. is crying 10. are watching, am watching 11. are tearing
2. Does she go ?
12. is entertaining 13. is unfolding 14. is deciding 15. is
Yes, she goes (By adding ‘es’) taking.

502 | CAT Complete Course

Visit : www.EasyEngineering.net
Visit : www.EasyEngineering.net

Present Perfect (Exercises) Change the following sentences into Present Perfect
Continuous Tense. Period of time and point of time is
Fill in the blanks with the appropriate Present Perfect given in brackets.
Tense—
1. I am walking. (2 hours)
1. The sun …………. (rise).
2. He is playing. (morning)
2. I …………. (eat) lunch just now.
3. She is cooking. (childhood)
3. …………. You …………. (wear) jeans ever ?
4. The students are studying. (February)
4. …………. She …………. (post) the letter?
5. They are constructing. (2003)
5. They …………. (see) the TajMahal.
6. The baby is crying. (last night)
6. Rohit …………. (be) to France.
7. The dogs are barking. (evening)
7. The sweeper …………. (sweep) the floor.
8. They are listening to music. (5 hours)
8. Father’s oxen …………. (destroy) the crop.
9. The soldiers are fighting. (winter)
9. I …………. (know) him for 5 years.
10. The boys are jogging. (a long time)
10. The gardener …………. (cut) the grass.
Answer

ww
11. The boys …………. (play) football.
12. The temperature …………. (rise) sharply.
13. The food …………. (be) laid out.
1. I have been walking for two hours 2. He has been
playing since morning 3.she has been cooking since

w
14. The fish …………. (catch) the baits.

.Ea
15. The furniture …………. (crack).
Answer
childhood. 4. The students have been studying since
February . 5. They have been constructing since 2003.
6. The baby has been crying since last month. 7. The dogs
have been barking since evening. 8. They have been

syE
1. Has risen 2. Have eaten 3. Have you worn 4. Has
she posted 5. Have seen 6. Has been 7. Has swept 8. have
listening to music for 5 hours. 9. The soldiers have been
fighting since winter. 10. The boys have been jogging for
a long time.
destroyed 9. Have known 10. Has cut 11. Have played
12. Has risen 13. Has been 14. Have caught 15. Has
cracked. ngi Present Perfect Tense (Exercises)
Fill in the blanks with the appropriate Present Perfect
Present Perfect Continuous Tense (Exercises)
Fill in the blanks with the appropriate Present Perfect nee
Tense—
1. He …………. (eat) his lunch.

Continuous Tense—
1. He ………. (work) 2 hours. rin
2. The food …………. (serve).
3. The Washerwoman …………. (wash) the clothes.

2. They ………. (teach) ………. Morning.


3. She ………. (exercise) ………. Last week. g.n
4. Jamie …………. (see) the Taj Mahal.
5. I …………. (be) to America.
4. The doctor …………. (operate) on the patient
………. 3 hours.
5. The labourers ………. (dig) the fields ……….
2005.
e
6. The children …………. (finish) playing.
7. She …………. (watch) that clip.
8. You …………. (hear) this song. t
9. …………. You …………. (wear) jeans.
6. Shanti …………. (talk) non stop …………. 30
10. …………. She …………. (visit) you ?
minutes.
7. Robi ………. (visit) us ………. Last year. Answer
8. They ………. (close) friends ………. Childhood. 1. has eaten 2. has been served 3. has washed 4. has
9. The Ganga ………. (flow) ………. Time seen 5. have been 6. have finished 7. has watched 8. have
immemorial. heard 9. have; worn 10. has; visited
10. That man ………. (shout) ………. Yesterday.
Change the following sentences into Present Perfect
Answer Tense.
1. has been working for 2. Have been teaching since 1. I read the Bhagvad Gita.
3. has been exercising since 4. has been operating; for
2. They are walking.
5. have been digging; since 6. has been talking; for 7. Has
been visiting; since 8. have been close ; since 9.has been 3. She is singing a sing.
flowing; since 10. has been shouting; since 4. Doctors examine patients.

CAT Complete Course | 503

Visit : www.EasyEngineering.net
Visit : www.EasyEngineering.net

5. The baby is drinking milk. Answer


6. The Gardener is watering the plants. 1. wept 2. jumped 3. rode 4. took 5. ran 6. screamed
7. Mother stitches our clothes. 7. cut 8. grew 9. dug 10. swept
8. Mike sells fruits. Simple Past Tense (Structure)
9. He phones his mother.
The following timer (Auxiliary verb) is used in the
10. Sister cuts vegetables. simple past tense—
Answer Timer Subject Verb Form
1. I have read the Bhagvad Gita. Did I, You, We, They (Plural) V1
2. They have walked. Name, He, She, It (Singular) (V 2 is applied
3. She has sung. for positive state-
4. Doctors have examined patients. ment or answer)
5. The baby has drunk milk. To form the Interrogative, Positive statement or
answer follow these simple guidelines—
6. The Gardener has watered the plants.
Interrogative

ww
7.
8.
Mother has stitched our clothes.
Mike has sold fruits.
Timer + Subject + V1
Did you go to school ?
9.
10.
w He has phoned his mother.
Sister has cut vegetables.
Simple Past Tense (Exercises)
.Ea
Positive Answer
(a) Yes, Subject + Timer + V1
(b) Yes, I did go (to school)
(c) Yes, subject + V2
Tense—
1. Tom …………. (play) cricket. syE
Fill in the blanks with the appropriate Simple Present
(d) Yes, I went (to school)
In case of a positive statement or answer the second
2. He …………. (walk) to school.
3. The peon …………. (post) the letters. ngi form of the verb is usually used. However, while forming
the interrogative and the negative ‘Did’ can not be
omitted.
4. The boy …………. (brush) his shoes.
5. She …………. (sing) sweetly. nee Positive Statement
(a) Subject + Timer + V1 = I did go.
6. The noise …………. (awake) him.
7. The man …………. (eat) a pizza.
(b) Subject + V2
Negative Statement
rin = I went.

8. Michael …………. (swim) across the river.


9. Tom …………. (drink) a glass of milk. g.n
Subject + Timer + not + V1
I did not go.
10. Jessica …………. (beat) her brother.
Answer
1. Plays 2. walks 3. posts 4. brushes 5. sings
Tenses (Past Tense) (Exercises)
Exercises–1 e t
1. Tick the appropriate (Simple) Past Indefinite Tense.
6. awakes 7. eats 8. swims 9. drinks 10. beats 1. He ………… (walk) down the road when I met him.
Now, change the following into Simple Past Tense— (A) were walking (B) had been walking
1. He was weeping. (C) was walking (D) walked
2. They jump on the bed. 2. They ………… (wait) for the train to arrive.
3. She rides a motorcycle. (A) had been waiting (B) were waiting
4. They take a bath daily. (C) had waited (D) waited
5. The boys were running. 3. Mother and I ………… (speak) to them.
6. Jack screams at Jane. (A) were speaking (B) spoke
7. He cuts her hair. (C) had spoken (D) am speaking
8. Trees grow everywhere. 4. The newly wed couple ………… (dance) all night.
9. Jacklyn is digging a hole. (A) danced (B) had been dancing
10. Mother sweeps the temple. (C) are dancing (D) dance

504 | CAT Complete Course

Visit : www.EasyEngineering.net
Visit : www.EasyEngineering.net

5. The guards ………… (keep) a watch 24 hours for a 15. Tom ………… (lift) 100 kgs. of wheat.
week. 16. The cat ………… (drink) all the milk.
(A) are keeping (B) kept 17. We ………… (sit) on the carpet.
(C) had been keeping (D) keep 18. Mother ………… (fry) the potatoes.
Answers 19. The cobbler ………… (stitch) our shoes.
1. (D) 2. (D) 3. (B) 4. (A) 5. (B) 20. The Hostess ………… (pour) out the tea.

Exercises–2 Answers
Employing the appropriate simple Past Tense. Choose 1. Fetched 2. Slept 3. Crept 4. Jumped 5. Talked
the correct option. 6. Examined 7. Fell 8. Repaired 9. Wrote 10. Sped
11. Chopped 12. Danced 13. slapped 14. Scolded
1. He ………… (run) very fast.
15. Lifted 16. Drank 17. Sat 18. Fired 19. stitched
(A) runs (B) is running 20. Poured.
(C) has run (D) ran
Exercises–4
2. The gardener ………… (cut) the grass.

ww
Change the following sentences to ‘simple present
(A) has cut (B) is cutting tense’. (Tenses have been underlined to assist you.)
(C) had cut (D) cut
1. They are running away from the police.

w
3. Ricky ………… (swim) across the river in an hour.
(A) swam
(C) is swimming
.Ea
(B) swum
(D) has swum
2. He eats two dozen apples in 10 minutes.

3. Children have burst all the balloons.

(A) win
(C) won
syE
4. Toe ………… (win) the 100 metre sprint race.
(B) wins
(D) has won
4. He was walking towards the river.

5. The dogs were barking loudly.

5. Jack ………… (carry) his son on his shoulders.


(A) carries (B) is carrying ngi 6. He has been sleeping all day.

7. She races to buy chocolates.


(C) carried (D) has carried

Answers
nee
8. The gatekeeper is opening the door.

1. (D) 2. (D) 3. (A) 4. (C) 5. (C)


rin
9. The man has been shouting at me.
10. The man had pleaded guilty.
Exercises–3
Fill in the blanks using the correct ‘simple past’ g.n
11. The officer is shooting the dacoit.
form—
1. Till ………… (fetch) a pail of water.
2. Hari ………… (sleep) soundly.
3. The cat ………… (creep) into the room.
14. Father is snoring so
e
12. The mice will nibble the cheese.
13. The girl will be plaiting her hair.
loudly that t
everyone
is waking up.
4. The horse ………… (jump) over the fence.
5. She ………… (talk) to me all night. 15. The teacher has torn our homework books.
6. The doctor ………… (examine) the patient. Answers
7. Snow ………… (fall) heavily in Shimla last night. 1. Ran 2. Ate 3. Burst 4. Walked 5. Barked 6. Slept
8. Jack ………… (repair) his neighbours car. 7. Raced 8. Opened 9. Shouted 10. Pleaded 11. Shot
12. Nibbled 13. Plaited 14. Shored, Woke 15. Tore.
9. I ………… (write) a letter to my sister.
10. The motorcade ………… (speed) by us.
11. The woodcutter ………… (chop) all the wood Exercises – 5
quickly. Simple Past Continuous Tense (Exercises)
12. She ………… (dance) all day. Fill in the blanks. Assertive (Positive)—
13. He ………… (slap) his younger brother. 1. It ………… (rain) heavily today.
14. Father ………… (scold) us for arriving late. 2. They ………… (play) football..

CAT Complete Course | 505

Visit : www.EasyEngineering.net
Visit : www.EasyEngineering.net

3. I ………… writing a letter. Exercises Mixed


4. He ………… laughing loudly. 1. ……… you ……… (go) to school ? (Interrogative)
5. She ………… (sing) a song. 2. ……… she ……….. (mix) the flour ? (Interrogative)
6. We ………… (call) our friends. 3. That man ………… (talk) to father. (Positive)
7. Uncle and Aunty ………… (visit) their relativies the 4. The boys ………… (kick) the ball. (Negative)
evening.
5. The dogs ………… (bark) at him. (Positive)
8. You ………… (clean) your room.
6. Teacher ………… (teach) us. (Negative)
9. The boy ………… (ride) a cycle.
7. ………… I ………… (work) hard ? (Interrogative,
10. The gardener ………… (dig) a hole. Negative)
Answer 8. The hen ………… (lay) an egg ? (Positive)
1. was raining 2. were playing 3. was 4. was 5. was 9. That boy ………… (study). (Negative)
sing 6. were calling 7. were visiting 8. were cleaning 10. I ………… (read) these books. (Negative)
9.was riding 10. was digging
Answer

ww
Interrogative
Exercises
1. were; going 2. was; mixing 3. was talking
4. weren’t kicking 5. were barking 6. wasn’t teaching
7. wasn’t ; working 8. was laying 9. wasn’t studying

w
1. ………… she ………… (buy) books ?
10. wasn’t reading
2. ………… you ………… (wash) the dishes ?

.Ea
3. ………… I ………… (leave) today ?
4. ………… we ………… (learn) computers ?
Past Continuous Tense
Exercises–1

syE
5. ………… they ………… (teach) English ?
6. ………… he ………… (cook) the food ?
Tick the correct Past Continuous Tense option.
1. Jake ………… (wait) for Jill by the river.
7. ………… it ………… (work) ?
8. ………… father ………… (visit) the kapoors’ ?
ngi (A) waited
(C) had waited
(B) was waiting
(D) waits
9. ………… Jill and Jane ……….. (brush) their teeth ?
10. …………… John …………… (eat) a burger ?
nee
2. He ………… (dig) a hole in the ground.
(A) was digging (B) has dug

rin
Answer
(C) will dog (D) has been digging
1. was; buying 2. were ; washing 3. am; leaving
3. The wealthy man ………… (spend) a lot of money

g.n
4. are ; learning 5. are; teaching 6. is; cooking 7. was;
working 8. is; visiting 9. were; brushing 10. was ; eating to help the poor.
(A) spent (B) will spend
Simple Continuous Tenses (Exercise)
Assertive Negative
1.
2.
He ………… (go) to the market.
They ………… (peel) the potatoes.
(C) does spend
e
(D) was spending
4. She ………… (pluck) flowers to make a garland.
(A) had plucked t
(B) was plucking
3. The Gardener ………… (water) the plants.
(C) had been plucking
4. The Doctor ………… (examine) the patient.
(D) will have been plucking
5. The girls ………… (dance) on time.
6. The boy ………… (trick) the whole day. 5. They ………… (attack) us from all sides.
7. Mother ………… (stitch) the clothes. (A) have been attacking (B) will attack
8. It ………… (move) fast. (C) were attacking (D) attacked
9. You ………… (listen) to me. 6. She ………… (comb) her hair when the phone rang.
10. The driver ………… (drive) very slowly. (A) was combing (B) combed
(C) combs (D) has combed
Answer
7. The pilot ………… (fly) at 500 miles an hour.
1. was going 2. were peeling 3. was watering 4. was
examing 5. were dancing 6. was trekking 7. was stitching (A) flew (B) was flying
8. was moving 9. were listening 10. was driving (C) flown (D) has flown

506 | CAT Complete Course

Visit : www.EasyEngineering.net
Visit : www.EasyEngineering.net

8. We ………… (drive) through a dark and dangerous 19. Shane ………… (pull) up the chain.
forest. 20. The bells ………… (ring) in the temples.
(A) drove (B) had driven
Answers
(C) were driving (D) drive
1. was spreading 2. were peeling
9. Sally ………… (sit) on the road.
3. was reading 4. were washing
(A) sits (B) was sitting
5. was trying 6. was buttoning
(C) sat (D) has sat
7. was ironing 8. was blowing
10. The children ………… (swallow) their food lest they 9. was draping 10. were standing
be late for school.
11. were frisking 12. was punching
(A) was swallowing (B) were swallowing
13. was guessing 14. were guiding
(C) have been swallowing (D) swallows
15. were viewing 16. was pouring
11. Cars …………… (zoom) by very fast. 17. was twisting, turning 18. was banging
(A) is zooming (B) were zooming 19. was pulling 20. were ringing

ww
(C) was zooming (D) zoom
12. Waiters ………… (serve) the guests very efficiently.
(A) serve (B) serves
Exercises–3
Change the following sentences using the ‘Past Con-

w
(C) have been serving (D) were serving

.Ea
Answers
tinuous Tense’. Tense indicators have been underlined to
assist you.
1. Harry and Sue have eaten their lunch.

syE
1. (B) 2. (A) 3. (D) 4. (B) 5. (C) 6. (A) 2. Tom has been working with us.
7. (B) 8. (C) 9. (B) 10. (B) 11. (B) 12. (D)
3. She scrubbed the floor.
Exercises–2

1.
Fillers (use Past Continuous Tense)—
Mother ………… (spread) the sheets on the beds. ngi 4. Father had slapped him for the second time.

5. The police tracked down the thief.


2.
3.
Sisters ………… (peel) the potatoes.
Sam ………… (read) a new novel. nee
6. The policeman has been issuing tickets to traffic
offenders.
4. The washerwoman ………… (wash) clothes by the
river.
rin
7. The officials stamped visas on our passports.
5.
6.
The boy ………… (tie) his shoelaces.
I ………… (button) my shirt. g.n
8. The rascals have sprinkled dirty water on the girls.

9. The vandals destroyed all the furniture in the office.


7.
8.
9.
She ………… (iron) her brother’s shirt.
The South wind ………… (blow) ferociously.
Sita ………… (drape) herself in a saree.
e
10. A spider will scale the wall.

11. The Principal and founder will have distributed the t


10. The soldiers ………… (stand) to attention. sweets among the students.
11. The security guards ………… (frisk) the visitors. 12. Sonam stirred the soup expertly.
12. The machine ………… (punch) holes in the sheets
automatically. 13. Sam and Ray had been discussing the project.
13. He ………… (guess) as he did not know the correct 14. He jogs around the park.
answers.
14. The professor and his assistants ………… (guide) the 15. Simi is buying a new dress for her daughter.
students.
Answer
15. We ………… (view) the famous Eiffel Tower in
Paris. 1. were eating 2. was working
16. It ………… (pour) cats and dogs. 3. was scrubbing 4. was slapping
17. The patient ………… (twist) and ………… (turn) in 5. were tracking 6. was issuing
his sleep. 7. were stamping 8. were sprinkling
18. Someone ………… (bang) on the door. 9. were destroying 10. was scaling

CAT Complete Course | 507

Visit : www.EasyEngineering.net
Visit : www.EasyEngineering.net

11. was distributing 12. was stirring Example—


13. were discussing 14. was jogging If I were a butterfly.
15. was buying Here, ‘I’ takes were instead of ‘was’.
Simple Past Continuous Tense (Exercises) The structure for the Interrogative, Positive and
Negative are as follows.
Fill in the blanks—
Interrogative—To form a question follow this
1. The girls ………… (hope) on one leg. simple rule.
2. The teacher ………… (read) from the lesson. Timer + Subject + V1 + ing
3. The crowds ………… (protest) at the police station. Were you going (home) ?
4. We ………… (watch) the test match. Positive Answer
5. Sita ………… (wrap) a saree. Yes, Subject + Timer + V1 + ing
6. Mother ………… (fold) the sheets. Yes, I was going (home).
7. The carpenters ………… (hammer) nails in the Negative Answer
planks.
No, Subject + Timer + not + V 1 + ing
8.
9.
10.
ww
The girls ………… (sweep) the rooms.
The waiter ………… (serve) tea to the guests.
Harry ………… (rub) oil on his hair.
No, I was not going (home) ?
Past Perfect Continuous Tense (Exercises)

w Answer

.Ea
1. were hopping 2. was reading 3. were protesting
4. were watching 5. was wrapping 6.was folding 7. were
Fill in the blanks using Past Perfect Continuous
Tense—
1. They ……….. (working) ……….. 2 days.

rubbing.
syE
hammering 8. were sweeping 9. was serving 10. was

Change the following sentences into Simple Past


2. The driver ……….. (drive) ……….. last week.
3. The boys ……….. (play) ……….. a week.
4. Mother …….. (sew) clothes …….. her school days.
Continuous Tense—
1. He played cricket. ngi 5. Rick ……….. (write) ……….. 2002.
6. The grass ……….. (grow) ……….. last summer.
2. The girls cut their nails.
3. Silvia plaited her hair. nee
7. He ……….. (pour) tea ……….. morning.
8. The police …….. (whip) him …….. he was caught.
4. Mother peeled the bananas.
5. Kalidas wrote many books. rin
9. You ……….. (read) this book ……….. 30 minutes.
10. The birds ……….. (chirp) ……….. birth.
6. Sidney wore a tuxedo with a hat.
7. The shoeshine boy shone my shoes. g.n
Answer
1. had been working; for 2. had been driving; since
8. The cat drank all the milk.
9. The thief stole all the jewellery.
10. The boys crept into the candy store.
e
been writing ; since 6. had been growing;since 7. had
t
3. had been playing ; for 4. had been sewing ; since 5. had

been pouring ; since 8. had been whipping ; since 9. had


been reading; for 10. had been chirping ; since.
Answer Change the following into Past Perfect Continuous
1. was playing 2. were cutting 3. was plaiting Tense—
4. was peeling 5. was writing 6. was wearing 7. was 1. She has been exercising.
shining 8. was drinking 9. was stealing 10. were creeping.
2. They have been praying.
Simple Past Continuous Tense (Structure) 3. John has been listening to music.
The timers (Auxiliary Verbs) in the Simple Past 4. He has been brushing shoes.
Continuous Tense are ‘was’ and ‘were’. The subjects 5. The dentists have been drilling teeth.
which agree with them are as follows. 6. The men have been tying all day.
Timer Subject Verb Form 7. The music has been blaring all night.
Were You, We, They (Plural) V1 + ing 8. The masseur has been massaging people all morning.
Was Name, He, She, It (Singular) V1 + ing 9. They have been playing cards ever since.
However, in a conditional clause—All singular 10. The rag picker has been picking garbage since.
subjects take ‘were’. Answer 1 to 10 change the shaded words into had.

508 | CAT Complete Course

Visit : www.EasyEngineering.net
Visit : www.EasyEngineering.net

Past Perfect Continuous Tense (Structure) 5. The teacher will have been teaching for 2 hours.
Go form the Interrogative, Assertive (Positive and (A) had been teach (B) had
Negative) Positive or Negative answer. Follow the (C) is (D) will had
structure given below.
6. They are exercising for days before they left.
Interrogative
Had + Subject + been + V 1 + ing + for / since (A) is (B) were
Had he been working for 2 hours? (C) had been (D) had been exercise
Positive Answer 7. Jack has drilled holes in the angles.
Yes, Subject + had + been + V1 + ing + for (A) drilling (B) is drilling
Yes, he had been working for 2 hours. (C) was drilling (D) had been drilling
Negative Answer
8. Jennifer was combing her mother’s hair when the
No + Subject + had + not + been + V1 + ing + for
comb broke.
No, he had not been working for 2 hours.
(A) combed (B) has combing

ww
Positive Statement
Subject + had + been + V1 + ing + for
He had been working for 2 hours.
(C) had been combing (D) will comb
9. He has played cricket for years.

w
Negative Statement

.Ea
Subject + had + not + been + V1 + ing + for
He had not been working for 2 hours.
(A) has been playing
(C) played
(B) is playing
(D) had been playing
10. Sean had not talk to sue since (the year) 2000.

syE
Past Perfect Continuous Tense tells us the period of
time or point of time an action lasted in the past.
(A) has not talking
(C) had not been talking
(B) have not talked
(D) had not talking
If, for example you say, ‘I was exercising’, It tells of
a past action but it does not mention the period or point of
time elapsed. Whereas, I had been exercising for 1 hour ngi1. (B) 2. (C)
Answers
3. (D) 4. (A) 5. (B) 6. (C)
or since morning pinpoints the period or point of time the
action lasted. nee
7. (D) 8. (C) 9. (D) 10. (C)
Exercises–2
Past Perfect Continuous Tense
Exercises–1 rin
Fillers : Fill in the blanks using the appropriate Past
Perfect Continuous Tense—
Tick the most suitable option by. Changing the
underlined tenses into the Past Perfect Continuous Tense. g.n
1. Mother ……….. (cook) since morning.
2. The crowds ……….. (arrive) by the hundreds for 3
1. He eats apples for 1 week.
(A) has eaten
(C) will eat
(B) had been eating
(D) ate
days.
e
3. She ……….. (read) the Bible for 1 month.
4. The tourists ……….. (drive) for weeks around India. t
5. Mother …….. (argue) with father over a petty issue
2. They work hard since morning. for weeks.
(A) had been work (B) had worked 6. The Judge ………….. (issue) summons to all and
Sunday.
(C) had been working (D) works
7. The group members ……….. (practice) two songs
3. She will peel potatoes for 5 hours. daily.
(A) will have been peeling 8. We ……….. (contemplate) buying a new house.
(B) peels 9. I ……….. (plead) with quit his job for years.
(C) will peeling 10. He ……….. (hit) his wife continually for years.
(D) had been peeling
Answer
4. Tom and Dick fought since 2007. 1. had been cooking 2. had been arriving
(A) had been fighting (B) is fighting 3. had been reading 4. had been driving
(C) was fighting (D) are fighting 5. had been arguing 6. had been issuing

CAT Complete Course | 509

Visit : www.EasyEngineering.net
Visit : www.EasyEngineering.net

7. had been practicing 8. had been contemplating 5. The man died before the doctor arrives.
9. had been pleading 10. had been hitting
6. She is writing her will before she leaves.
Exercises–3
7. The pirates kill the crew before they sink the
Change the underlined words into the ‘Past Perfect
ship.
Continuous Tense’.
1. She was dancing since childhood. 8. Mother was cooking lunch when the bell wings.

2. They are not talking to each other for 2 days. 9. He will reach Mumbai we speak to him.

3. The Doctor prescribes wrong medicines since May. 10. Roy sang the song before we were stopping
him.
4. The poacher will kill tigers for a decade.
Answer
5. Harold taught for 5 years before he was fired.
1. had spoken, died 2. had arrived, reached
6. They train dogs for ages.
3. had already drunk, caught 4. had finished, set
7. We win when suddenly our luck changed.

ww
8. The preacher corrupts young minds before he was
caught.
5.
7.
9.
had died, arrived
had killed, sank
had reached, spoke
6.
8.
10.
had written, left
had cooked, rang
had sung, stopped

w
9. Sally lies to us since God knows when.

.Ea
10. The gas attendants stole gas for months.
Exercises–2
Fillers : Fill in the blanks with the ‘Past Perfect
Tense’—

syE
11. The thieves are removing bolts from the rails.
12. The deer graze when they were shot.
1. He ………. (abuse) me before I ……… (slap) him.
2. The train …….. (arrive) before we …….. (reach) the
station.
13. I am protesting but to no avail.
14. They will call us regularly. ngi 3. Tom …….. (eat) an apple before he …….. (sleep).
4. The terrorists …….. (shoot) 5 people before they
15. She combs her hair.

Answer nee were …….. (kill).


5. Tom and Jerry …….. (break) the vase before we

1. had been dancing 2. had not been talking


…….. (know) it.

rin
6. The flower …….. (blossom) before the sun ……..
3.
5.
had been prescribing
had been teaching
4.
6.
had been killing
had been training
(set).

g.n
7. Mary ………….. (inform) me when she …………..
(leave).
7.
9.
11.
13.
had been winning
had been lying
had been removing
had been protesting
8.
10.
12.
14.
had been corrupting
had been stealing
had been grazing
had been calling
e
8. The author …….. (complete) the book before he
………. (discover) the mistake.
9. The couple …….. (dine) before I …….. (meet) them. t
10. The old woman ……..… (trip) before ……… (catch)
15. had been combing her.
Past Perfect Tense (Exercises) Answer
Exercises–1 1. had abused; slapped 2. had arrived; reached
Tick the most suitable option to change the following 3. had eaten; slept 4. had shot; killed
tenses (underlined) in the sentences into the ‘Past Perfect 5. had broken; knew 6. had blossomed; set
Tense’. 7. had informed; left 8. had completed; discovered
1. He spoke to me before he die. 9. had dined; met 10. had tripped; caught
2. The bus arrives before we reach. Exercises–3
3. The cat already drank the milk when we Change the following sentences into the Past Perfect.
were catching it. 1. He is coming to meet us before he leaves.

4. They finished playing before the Sun was set. 2. The train arrived before she has come.

510 | CAT Complete Course

Visit : www.EasyEngineering.net
Visit : www.EasyEngineering.net

3. Mother will cook the food before we reach. 9. Place of had before ‘surrounded’ and ‘they fired’ in
place of ‘firing’.
4. The teacher taught the students before he ate 10. Use had given in place of ‘give’ and ‘he consulted’
dinner. in place of ‘consulting’.
5. Mary is killing her husband before he recovers. Past Perfect Tense (Exercises)
6. Mary will kiss her mother before she elopes. Fill in the blanks with the appropriate Simple Past
Perfect Tense—
7. He was killed before he confesses.
1. Mother ……….…. (cook) the food before father
8. She wipes the furniture before she sweeps the ……….. (arrive).
room. 2. The boy ……….. (finish) playing before it ………..
(rain).
9. He explains the formula before giving us a task.
3. The man ……….. (die) before help ……….. (come).
10. He spoke to her before phoning me. 4. Jerry ……….. (give) the massage before he ………..
(leave).
Answer
1.
3. ww
had come, left
had cooked, reached
2.
4.
had arrived
had taught, ate
5. Susan ……….……. (cut) the cake before the roof
……….. (fall).
6. Father ……….. (shut) the door before the wind
5.
7.
9.
w
had killed, recovered
had killed, confessed
had explained, he gave
6.

.Ea
8.
10.
had kissed, eloped
had wiped, swept
had spoken, he phoned
……….. (blow).
7. Mother ……….. (lay) the table before we ………..
(eat) the food.
Exercises–4
syE
Change the following into the Past Perfect Tense—
8. The Muncipality ……….. (clean) the road before the
Prime Minister’s convoy ……….. (pass).
9. The soldiers ……….. (dig) a tunnel before the firing

ngi
1. They are playing before they sleep.
……….. (start).
2. She was leaving before he arrives.
10. The host ……….. (serve) the drinks before the
3. The workers will build a wall before they start

4.
working.
He has eaten a light snack before he eats lunch. nee movie ……….. (start).
Answer
5.
6.
We turned on the music before dancing.
Charlie completed his work before relaxing. rin
1. had cooked; arrived 2. had finished; rained 3. had
died; came 4. had given; left 5. had cut; fell 6. had shut;

g.n
blew 7. had laid; ate 8. had cleaned; passed 9. had dug;
7. He left the office before the bomb exploded. started 10. had served.
8. She met him before meeting her. Change the following into Past Perfect Tense—
9.
10.
The police surrounded the area before firing.
He gave her the job before consulting his senior.
Answer
e
1. We took shelter. The storm arrives.
2. Mother cooked the food. The guests come.
t
3. The crowd started clapping. The Hero reach the stage.
1. Change ‘are playing’, and ‘sleep’ to ‘had played’ and 4. Sister dried the clothes. The rain come down.
‘slept’. 5. The thief ran 100 metres. The police catch him.
2. Change ‘was leaving’ and ‘arrives’ to ‘had left’ and 6. The car crashed. The driver realize.
‘arrived’. 7. The roof fell. Anyone escape.
3. Change ‘will build’ and ‘start’ to ‘had built’ and 8. The fruit ripened. We pluck them.
‘started’.
9. The train left the station. We arrive.
4. Change ‘has eaten’ and ‘eats’ to ‘had eaten’ and ate’.
10. We drank water. We eat food.
5. Change ‘turned’ and ‘dancing’ to ‘had turned’ and
‘they danced’. Answer
6. Place ‘had’ before completed and use ‘he relaxed’ in 1. We had taken shelter before the storm arrived.
place of ‘relaxing’. 2. Mother had cooked the food before the guest arrived.
7. Insert ‘had’ before ‘left’. 3. The crowd had started clapping before the hero reached
8. Place ‘had’ before ‘met’ and ‘she met’ in place of the stage. 4. Sister had dried the clothes before the rain
‘meeting’. came down. 5. The thief had run a100 metres before the

CAT Complete Course | 511

Visit : www.EasyEngineering.net
Visit : www.EasyEngineering.net

police caught him. 6. The car had crashed before the Simple Future Continuous Tense
driver realized 7. The roof had fallen before anyone
escaped. 8. The fruit had ripened before we plucked Choose the most appropriate ‘simple continuous
them. 9. The train had left the station before we arrived tense’ from the 4 options given below the following
10. We had drunk water before we ate food. sentences.
1. This man ……….. (accompany) you to the market.
Past Perfect Tense (Structure)
(A) will accompany
Structure
(B) is accompanying
Timer Subject Verb Form
(C) will be accompanying
(Aux. Verb)
(D) accompanies
Had I, You, We, They III Form
2. We ……….. (go) to Delhi tonight.
Name, He, She, It
(A) will have been going (B) will be going
Singular and Pluarl
(C) shall be going (D) will go
* Had agrees with all subjects.
3. The farmers ……….. (grow) wheat next month.
Interrogative

ww
Had + Subject + V3
Had she eaten (lunch) ?
(A) will grow
(C) had grown
(B) would grow
(D) will be growing

w
Positive Answer 4. If I’m not wrong, we ……….. (get) married in two
months time.
Yes, Subject + had + V3
Yes, She
Negative Answer
.Ea
had eaten (lunch).
(A) will get
(C) shall be getting
(B) should be get
(D) will have got

No she syE
No + Subject + had + not + V3
had not eaten (lunch).
5. Jake ……….. (ring) me in an hour.
(A) shall be ringing (B) would ring
Positive Statement
Subject + had + V3 ngi (C) will be ringing (D) should ring
6. The plane ……….. (land) at Heathrow Airport is five
She had eaten (lunch).
Negative Statement nee
minutes.
(A) will be landing
(C) will land
(B) should be landing
(D) shall have landed
Subject + had + not + V3
She had not eaten (lunch).
rin
7. I ……….. (play) the next match.

Simple Future Tense (Exercise)


From the two given options choose the most suitable
(A) should play
(C) would play
g.n
(B) shall be playing
(D) will be playing

‘simple future tense’ option to fill in the blanks.


Options—Shall and Will
1. Terry ……….. run all the way to Dehradun.
2. I ……….. tell you what is to be done.
(A) shall be expecting
(B) would be expecting
(C) should be expecting
e
8. We ……….. (expect) your call soon.

t
3. Reid and Douglas ……….. play the doubles match. (D) will be expecting
4. We ……….. discuss the matter later. 9. The cook ……….. (not) (work) for us any longer.
5. When he meets you he ………… inform us. (A) shall not work
6. If the thief escapes, the police ……….. be held (B) will not be working
responsible. (C) shall not be working
7. My house ……….. not be sold. (D) will not work
8. Trespassers ……….. be prosecuted. 10. Samanta ……….. (give) us a treat today.
9. ……….. We break the news? (A) shall give (B) would give
10. ……….. I meet her at home? (C) should be giving (D) will be giving
Answer Answer
1. Will 2. Shall 3. Will 4. Shall 5.Will 1. (C) 2. (C) 3. (D) 4. (C) 5. (C) 6. (A)
6. Will 7. Will 8. Will 9. Shall 10. Shall 7. (B) 8. (A) 9. (B) 10. (D)

512 | CAT Complete Course

Visit : www.EasyEngineering.net
Visit : www.EasyEngineering.net

Exercises–3 4. Rain ……… (fall) before the match ……… (finish).


Change the following sentences into ‘simple future (A) will fall; finishes
continuous tense’ by substituting the underlined words. (B) shall have fallen; finishes
1. They are going home in an hour. (C) will have fallen; finishes
2. I am buying a new car. (D) will be falling; finishes

3. He gave a lecture. 5. Thomas ……… (marry) before the year ………


(end).
4. We grow tulips in our garden. (A) will have married; ends
5. They thanked me for this. (B) shall be marrying; ends
(C) will marry; ends
6. The team has been playing all over the world.
(D) would marry; ends
7. The computer does not work.
6. We ……… (see) the movies before anyone ………
8. Same and Fred joined us from today. (find) out.

ww
9. The children are playing on the lawn.
(A) will have been seeing; find
(B) shall be seeing; finds

w
10. The helicopter has landed now.

1. will be going .Ea


Answer
2. shall be buying
(C) shall have seen; finds
(D) should have seen; found
7. He ………….. (escape) the police dragnet.
3. will be giving
5. will be thanking syE
4. shall be growing
6. will be playing
(A) shall be escaping
(B) will have escaped
7. will not be working
9. will be playing
8. will be joining
10. will be landing ngi (C) shall escape
(D) will have escape

Future Perfect Tense


Exercises–1 nee
8. Harry ……… (write) the book before he …………
(return) home.

rin
(A) will have written; returns
Out of the form given options which follow each
(B) shall be writing; return
sentence choose the most appropriate future perfect tense.
1. We ………. (clinch) the deal before they ………
(do). (D) would write; returnsg.n
(C) should write; was returning

(A) shall clinch; did


(B) will clinch; do
(C) shall have clinched; do
(question) them.
(A) will read; questions
e
9. They ………… (read) the book before he …………

t
(D) shall clinch; do (B) shall have read; questions
(C) will have read; questions
2. Mother ……… (cook) the food before father ………
(arrive). (D) will be reading; questioned
(A) will have cooked; arrives 10. John and Joe ……… (apply) for the post before he
(B) shall be cooking; arrived ……… (apply).
(C) shall have cooked; arrives (A) will have applied; applies
(D) would have cook; arriving (B) shall have applied; applied
(C) will be applying; applied
3. The plane ……… (land) before we ……… (reach).
(D) will apply; applying
(A) shall be landed; reached
(B) will have landed; arrive Answers
(C) would be landing; arrives 1. (C) 2. (A) 3. (B) 4. (C) 5. (A) 6. (C)
(D) should have landed; arrived 7. (B) 8. (A) 9. (C) 10. (A)

CAT Complete Course | 513

Visit : www.EasyEngineering.net
Visit : www.EasyEngineering.net

Exercises–2 Future Perfect Continuous (Solved)


Change the following sentences into ‘future perfect 1. She ……… (type) for 6 hours.
tense’ by changing the underlined words. 2. They ……… (dance) for 2 hours.
1. We shall be recruiting this year ends. 3. I ……… (teach) for 3 hours.
2. I am employing my relatives before my boss returns.
4. We ……… (play) many games by now.
3. Mother knits our sweaters before winter begins.
5. He ……… (check) out of the hotel at that time.
4. They police is arresting all the protesters.
6. Shane ……… (call) me all night.
5. Ricky gobbles his food very quickly.
7. They ……… (eat) by now.
6. The train departed the station before we reached.
8. The gardeners ……… (water) the plants for 8 hours.
7. Sammy eats his tiffin before the bell goes off.
8. Our General Manager closed the deal before 9. The men ……… (cut) the trees for 1 week.
consulted us. 10. The surgeon ……… (operate) on the patient for 3
9. The ships sinks before it starts. hours.
Answer

ww
10. I posted the letter then.
Answers 1 will have been typing
3. shall have been teaching
2. will have been dancing
4. will have been playing

w
1. Shall have recruited 2. Shall have employed
3. Will have knitted 4. Will have arrested 5. will have been checking 6. will have been calling
5.
7.
Will have gobbled
Will have eaten .Ea
6. Will have departed; reach 7. will have been eating
8. will have been watering
8.
9. Will have sunk syE
Will have closed; he consults
10. Shall have posted; by then
9. will have been cutting
10. will have been operating

ngi ●●●

nee
rin
g.n
e t

514 | CAT Complete Course

Visit : www.EasyEngineering.net
Visit : www.EasyEngineering.net

8 Para Jumbles
1. Para jumbles are quite difficult to arrange. As, they (c) If I asked you what he was like.
depend on the writers train of thought. So, it is not an (d) My guest is that many of you.
easy task. However, a few indications and rules will
(a) d, c, a, b (b) b, c, d, a
help consider this paragraph.
(c) c, d, a, b (d) d, a, c, b (√)
(a) Like this is called the adaptive unconscious.
(b) Is one of the most important new fields in tech- 4. (a) But Bargh and his colleagues were wrong.
nology. (b) The people primed to be rude eventually
interrupted.

ww
(c) The part of our brain that leads to conclusions.
(d) And the study of this kind of decision making. (c) on average about five minutes.
(a) c, a, d, b (√) (b) c, d, b, a (d) But of the people primed to be polite–82 percent

w
(c) b, a, d, c

.Ea
(d) b, a, d, c
In such cases one has to consider, whether a con-
tinuation is or it’s a lead sentence and how does it fall
into a series of ideas, in accordance with the flow of the
never interrupted at all.
Options—
(a) d, b, a, c (b) a, b, c, d (√)

central idea.
syE
Take option (b). It starts with as one of ……… and
(c) a, b, d, c (d) b, a, d, c
5. (a) But the Emergency Department seemed to cry
out for special attention.

ngi
answers the question ‘what’ therefore eliminating its
capacity as a lead sentence. (b) most of the patients entered the hospital through
Option (d) is similar in its opening and this the Emergency Department.
eliminated.
Option (a) makes sense but answers the question nee (c) The list of problems Reilly faced was endless.
(d) Because so few cook county patients had health
‘what’ and so fails to connect whereas.
Option (c) follows a definite path of cohesive
insurance.
(a) c, b, d, a
rin (b) c, a, d, b (√)
sentences ‘c’ is followed by ‘a’ like this which shows a
connection with ‘c’ and ‘d’ is joined by the conjunction
‘and’ which is apt and relative to ‘d’. Logically, ‘b’ will
(c) c, a, b, d
g.n (d) c, d, b, a
6. (a) Cook County’s big experiment began.
follow as it the last and correct sentence which has to fall
into place.
Now, try your hand at the given para jumbles to hone
your skills.
e
(b) named Brendan Reilly came to Chicago to
become Chairman of the Institution.
t
(c) The hospital that Reilly inherited was a mess.
(d) In 1996, a year after a remarkable man.
2. (a) They didn’t weigh
(a) a, c, b, d (b) d, b, a, c
(b) The second strategy was the path taken by
Evelyn and Having. (c) d, a, b, c (d) a, d, b, c (√)
(c) Every conceivable stand of evidence and it 7. (a) Braden found the same problem with the
ended in failure. basketball.
(d) and the Greek scholars. (b) A man revered for his knowledge and insight
Note—Look out for commas and full stops they into the art of hitting.
indicates continuation or the end of a sentence. (c) {layer Ted Williams was perhaps the greatest
(a) c, d, b, a (b) a, b, c, d hitter of all time.
(c) b, d, a, c (√) (d) d, c, a, b (d) He always said that the ball to the point where
3. (a) Have the same impression of Tom Hanks. he made contact.
(b) You would say he is decent and trustworthy and (a) a, c, b, d (b) a, c, b, d (√)
down to earth and funny. (c) d, a, b, c (d) a, b, d, c

CAT Complete Course | 515

Visit : www.EasyEngineering.net
Visit : www.EasyEngineering.net

8. (a) First impressions don’t work because Directions for questions 1 to 31 : The sentences
(b) The blind sip test is the wrong contact for thin given in each question, when properly sequenced, form a
slicing coke. coherent paragraph. Each sentence is labeled with a letter.
Choose the most logical order of sentences from among
(c) In the case of a blind sip test,
the given choices to coherent paragraph.
(d) Colas are not supposed to be sipped blind.
1. (A) These are early days yet, but these are signs that
(a) c, a, b, d (√) (b) b, a, d, c the appreciation of the rupee and the Reserve Bank
(c) c, b, a, d (d) b, a, c, d of India’s monetary policy
9. (a) What Ekman is describing. (B) The economy seems to be showing sign of wear
and tear; the latest index of Industrial Production (IP)
(b) We can all mind read effortlessly and automati-
data for September are disappointing, IP growth is
cally.
down to 6.4 percent compared to 12 percent last
(c) In a very real sense, is the physiological basis of September. This is the lowest recorded since October
how we thin slice other people. last year.
(d) Because the clues we need to make sense of (C) Which is resulting in higher interest rates, are
someone on the faces of those in front of us. hurting overall economic growth

ww
(a) c, b, a, d
(c) a, c, b, d
(b) a, b, d, c
(√) (d) c, a, d, b
(D) Which the index rose 4.5 percent. The main
culprit is the manufacturing sector, which decelerated

w
10. (a) If you are like most people. to 6.6% compared to about 13% in September last
year. The consumer durables segments, in particular,
incredible.
.Ea
(b) I imagine that you find goslings conclusion quite

(c) But the truth is that they shouldn’t be.


has been hit hard, registering a 7.6% fall in output.
There’s more bad news for the first half of their
financial year, IP growth fell to 9.2%, against 11

(a) a, c, d, b syE
(d) Not after the lessons of John Gothman.
(b) a, b, c, d (√)
percent last year.
(1) ACBD (2) ADCB
(c) c, a, d, b (d) b, a, c, d
11. (a) Can be very confusing, for the simple reason. ngi (3) ACBD
(5) DCBA
(4) DACB

(b) That most of us aren’t very objective.


(c) What people say about themselves. nee
2. (A) Typically, real estate grows 2-3 times the GDP
growth of a country. So, it is no exaggeration when J
P Morgan forecasts the industry size to grow from
(d) About ourselves.
(a) b, a, c, d (b) c, d, a, b rin
$50 billion (Rs 2,00,000 crore) per annum at present
to $90 billion (Rs 3,60,000 crore) by 2010-11,

(c) b, a, d, c (d) c, a, b, d
12. (a) What we reveal about ourselves can confuse.
(√)
g.n
showing a growth rate of 13 per cent over the period.
(B)What is driving this enormous investor appetite?
Economic growth lapping 9 per cent has spurred
(b) That’s why, when we measure personality, we
don’t just ask people point blank.
(c) What they think about themselves.
e
demand for office space, retail outlets and hotels.
(C) Kaushal Sampat, Chief Operating Officer of fi-
nancial data services firm, Dun & Bradstreet, says:
"The demand drivers for real estate are on a firm
t
(d) We give them a questioner, like the Big Five
footing. To satiate this demand, the sector will re-
Inventory, carefully designed to elicit responses.
quire continuous capital infusion." And with demand
(a) a, b, c, d (√) (b) b, c, d, a showing strength, Sampat does not see any let- up in
(c) a, d, c, b (d) b, c, a, d investor interest in the next 3-5 years.
13. (a) These kinds of unconscious – or, as they like to (D) And even as the Special Economic Zones get
call them. planned and executed, it is the coming together of
rising demand in diverse segments that is driving.
(b) Over the past few years.
(1) ABCD (2) BDAC
(c) Implicit associations play in our beliefs and
(3) BADC (4) ACDB
behaviour based on the IAT test.
(5) ACBD
(d) A number of psychologists have begun to look
more closely at the role. 3. (A) Khanna’s gamble paid off for a while as soaring
stock prices saw his returns increase several-fold.
(a) d, b, a, c (b) b, a, c, d Emboldened, he tripled his stock purchases and
(c) d, b, c, a (d) b, d, a, c (√) likewise extended his margin credit. But , Khanna's,

516 | CAT Complete Course

Visit : www.EasyEngineering.net
Visit : www.EasyEngineering.net

luck ran out: the stock he bought plunged over 50 invest in West Bengal. He has even declared that the
percent in a day, his broker quickly squared. state's economic salvation lies, not in socialism but,
(B) Off his shares without intimation to avoid further in enlightened capitalism.
losses, and Khanna ended up with dues that wiped (B) Already the doomsayers are predicting the with-
the entire gains he made since he started investing ering away of the Front. But that's not why Editor is
through margin funds. writing this editorial. Bhattacharjee has been at the
(C) Khanna is one of many who have tried margin forefront of efforts to reinvent the Marxist movement
credit and have not been successful. Margin funding in India. He has gone on record saying that he doesn't
is not always a win-win situation as it might appear have time for "dogmatic Marxism" and would rather
for most investors. practice a more “pragmatic” version of the ideology.
(D) Six months ago, small–time metal trader sunil (C) His initiative is bearing fruit. The prestigious
khanna began investing in the stock market through Tata small car (nano) project is coming up in Singur;
margin funding with assistance from his broker. This the Jindals and the Jai Balaji Group are setting up
credit facility paid for a part of Khanna's stock multi-thousand crore steel plants in the state and IT
purchases, while his broker funded the rest to be set- majors of all hues have set up large campuses in
tled at a future date. Kolkata. In doing all this, he has had to fight stiff

ww
(1) DCBA
(3) ACDB
(2) DCAB
(4) ADCB
opposition from hardliners in his own party who
have still not read the writing on the wall.
(D) These are not good times to be in WEST

w
(5) BACD

.Ea
4. (A) This quant fund will allocate and re-allocate
assets, if necessary on a monthly basis, after identi-
fying new trends in the market. Earlier too, Lotus
BENGAL Chief Minister Buddhadeb Bhattacharjee's
shoes. The flare-up in Nandigram, and before that, in
Singur, the ration riots across the state, the allegations
of police complicity in graphic designer Rizwanur

syE
India introduced an Agile Saver fund that closed for
subscription only recently.
(B) World over, quantitative fund management
Rehman’s death and reports of rifts within the till
now monolithic Left Front have all combined to sully
his squeaky clean and suave image.
models that identify trends, and use them to make
investment decisions have been gaining popularity; ngi (1) DCBA
(3) DBAC
(2) BDAC
(4) BADC
and India is no exception. Already, investors have
found a few quant-funds, as they are popularly called,
coming their way. A latest offering from Lotus India nee (5) ABCD
6. (A) Some countries, which were lazier than others,
Mutual Fund, the Agile Tax Saver fund, however, is
an ELSS fund where the investments will be eligible
rin
merely reused expressions already in vogue. In
Swahili, Afya ! (to health) is used while drinking and
for a tax deduction under Section 80C of up to a
maximum of Rs l lakh.
(C) The AGILE Tax Saver Fund will invest in the g.n
also when someone sneezes and so it is for the term
na zdrowie, which is used across many parts of
Central Europe.
same stocks as the AGILE Saver Fund with no
difference in the asset allocation mix, but the target
audience for the tax saver funds are long term
investors looking for tax saving. "We are just
e
(B) As only clinking glasses seemed too solemn an
act, especially considering that you were drinking
alcohol, people soon began to say something along
with the act, with "to your good health" being one of
t
targeting different segments of investors," says Rajiv the more popular early sayings, As races intermingled
Shastri, Head, Business Development and Strategic and travelled, this dradition soon spread from one
Initiatives. Lotus India AMC. part of the world to another, with each country soon
(D) In both the newly introduced quant funds, AGILE acquiring their own distinctive nationalistic way of
is an acronym for Alpha Generated from Industry saying cheers.
Leader's Fund. Fund managers try to generate Alpha, (C) Did you know that it's considered bad luck to
which essentially means out-performance, by inves- toast with an empty glass and, worse still, to toast
ting in stocks that beat the market or rise faster than with water, as the person so honoured will be
the market. doomed to a watery2. Toasts have been around since
(1) DABC (2) BCDA time immemorial. The more popular legend regarding
their origin is that the practice of toasting originated
(3) DCBA (4) BADC
in ancient Greece, when the host and the guest would
(5) ABCD clink their glasses together, so that a part of each's
5. (A) That includes inviting private capital-and even drink would go in to the other, and thus provide
foreign investors like Indonesia's Salim Group-to assurance that the drink was not poisoned.

CAT Complete Course | 517

Visit : www.EasyEngineering.net
Visit : www.EasyEngineering.net

(D) Do remember, however, that if you're in a selling them to banks and financial institutions. This
foreign country, always try and stay true to the has led to a grave credibility crisis in the US cor-
country's traditions by using its local toast and not porate bond market and eroded the liquidity from the
using a generic expression, say like the British chin- system
chin, especially not in Japan, where ,chin-chin (B) Already, the top US banks have reported aggre-
sounds remarkably like a word describing male gate losses and write-downs of around $50 billion for
genitalia. Also verify that what you've been told is the quarter, with worldwide, losses amounting to
actually the correct word and not someone stringing ground $300 billion-and still rising.
you along. (C) The word is finally out. The sub-prime impact
(1) ADCB (2) CABD has been far deeper and incisive than what was earlier
(3) ACBD (4) CBAD estimated. The seriousness of the problem can be
(5) ABCD gauged from the fact that two consecutive rate cuts
by the US Fed, totalling 75 bps, have failed to deliver
7. (A) The original Bhullam-buthur (means 'making a
the, desired outcome of boosting consumer expen-
gurgling sound') later Sanskritised into Brahmaputra,
diture, especially in the realty market. Consequently,
was once a very busy inland waterway, but asphalt
the decline of the housing sector, coupled with the
roads snatched all its cargo. Its only travellers, till

ww
fairly recently, were paddling fishermen. Now, the
river is on course for revival; and it is possible to
cruise almost the entire length of the river in Assam
rising banking sector woes, pose a recession threat to
the US.
(D) As stated earlier, the underlying market for CDOs

w
on cruise ships.

.Ea
(B) The BRAHMAPUTRA, one of the most spec-
tacular rivers in India, runs right through Assam. On
its way through the state, it goes past Majuli-the
has gone completely illiquid, making it difficult to
'mark to market' the securities in possession of the
large banks. So in order to reactivate the market
segment, major US banks, under the behest of the US

syE
largest riverine island in the world and a major
cultural centre, winding past the dense forests of the
Kaziranga National Park and idyllic Country villages
Treasury Secretary, aim to float a Special Investment
Vehicle Fund with a corpus of around $80-100
billion. This fund will buy securities from the market
and paddy fields. At some places the river resembles
an ocean-nearly 10 km wide. ngi and flush liquidity into the CDO market segment.
However, in my opinion, the aim seems more to get
these securities offloaded from the balance sheets of
(C) On the second day of the cruise, they take you on
a road trip to Sibsagar, the ancient capital of the
Ahom kings of Assam. These kings ruled Assam for nee the large banks and cover up their potential losses - a
case of creative accounting.

rin
(1) ABCD (2) CDAB
nearly 700 years and their culture and architecture
are a delightful blend of the Indian and the South- (3) CABD (4) CBAD
east Asian. The temples in town have stupa-like
profiles and palaces like the Rang Mahal are
decorated with beautiful carvings of river creatures
(5) DCBA

g.n
9. (A) If the reviews are overwhelmingly positive, it is
advisable to put a cap on publicity and advertising
like crocodiles. Beyond Sibsagar, the river widens
and a few hours down bifurcates into two channels,
forming Majuli.
(D) The 10 night Assam Despatch Cruise run by the
e
spend-instead of the carpet bombing being practiced.
However, if the review is overwhelmingly bad, the
producer, with money to spend, may not have much t
option but to spend it on publicity and advertising.
Assam Bengal Navigation Company takes you on its (B) so, what decides the publicity and advertising
aire-conditioned luxury river boat, from Dibrugarh in approach of a film? If you have the money, should
the Upper Assam down to Guwahati, covering some you always use it to publicise the movie? This
of the most exciting parts of the state. Dibrugarh is a depends partially on the perception of the likely
typical colonial town and was a major American base success of the movies by the producer- this leads to
during World War II. The cruise starts from either the question as to how a movie-maker can arrive at
Dikhou Mukh, about 2.5 hours from Dibrugarh or an accurate estimate of the likely success of the film
from Neamati, which is closer to the town of Jorhat. ex-ante.
(1) DABC (2) BADC (C) But what happens, if the reviews are in the middle
(3) DBAC (4) BDAC range-which might be the case for a large number of
(5) BACD films? The ideal way would be to spend, which is
8. (A) At the heart of this down-turn lies the massive why we see many filmmakers engaging in a spate of
credit abuse that resulted from the innovative pre-release publicity and marketing efforts. One
packaging (Collateralised Debt Obligation) of sub- sometimes gets the impression that money is being
quality home mortgages into high quality paper and spent because it is available. The need to rise above

518 | CAT Complete Course

Visit : www.EasyEngineering.net
Visit : www.EasyEngineering.net

the increasing clutter of course, another reason for (1) DBAC (2) DCBA
the increased spending . (3) BADC (4) CABD
(D) Films with mega stars have often bombed at the (5) DABC
box office and small budget movies have become big
12. (A) As more people buy higher-end washing ma-
hits. The challenge is to get the "right" type of viewer
chines, consumption of superior quality detergents
to view the first cut of the edited movie that is
will rise and that should increase enzyme usage.
reasonably close to the final version. Many previews
Similarly, as Indians eat more packaged foods, the
tend to be for "people like us" i.e., people who are
food-processing industry will buy enzymes in larger
like the movie-makers -close friends and family of
quantities.
the movie-makers -and this would need to change to
obtain a more accurate forecast. Getting the right (B) But if you consider the money spent and the
"pre-viewer" is not as simple as it seems. advantage you get on product performance, enzyme
(1) ACBD (2) BCDA solutions offer an advantage," says GS Krishnan,
MD of Novozymes South Asia. But this mind block
(3) BADC (4) BDAC
is now beginning to change; "In our other markets,
(5 ABCD
we have experienced that economic growth changes
10. (A) After the ceremonies Ellsworth Toohey took enzyme consumption patterns. India will be no

ww
keating away to the retreat of a pale-orchid booth in a
quite, expensive restaurant. Many brilliant parties
were being given in honor of the opening, but
exception," he adds. Take detergents, for example.
(C) Despite being around for a long time, the Indian
market for enzymes is relatively nascent. "It's all

w
Keating grasped Toohey’s offer and declined all the
other invitations.

.Ea
(B) He had to admit that he was bored. But he smiled
and shook hands and let himself be photographed.
about the mindset. Our biggest challenge is to
convince some of the potential users in certain
industries to start using enzymes-businesses are very
reluctant to change. They feel enzymes are expensive

syE
The Cosmo- Slotnick building rose ponderously over
the street. Like a big white bromide.
(C) I should be happy, peter keating told himself---
compared to chemicals.
(D) Says an industry source: "We, in India, consume
almost all the enzymes that are available globally,
and wasn’t. He watched from a window the solid
spread of faces filling broadway from curb to crub.
He tried to talk himself into joy. He felt nothing. ngi but in very small quantities. That should change-but
it won't be overnight. "

(D) In December the Cosmo- Slotnick Building was


opened with great ceremony. There were celebrities, nee (1) ABCD
(3) ACBD
(5) DABC
(2) CABD
(4) CBAD

rin
flower horseshoes, newsreel cameras, revolving
searchlights and three hours of speeches, all alike. 13. (A) Getting into that list is the equivalent of getting a
calling card to start meeting clients looking for

g.n
(1) CBAD (2) DCBA
(3) CADB (4) CDAB supercomputing services. At the conference, the Tata
supercomputer christened EKA ('one' in Sanskrit)
(5) BACD
11. (A) The Banner was first to get the newest typo-
graphical equipment. The Banner was last to get the
newspapermen—Last, because it kept them. Wynand
raided his competitions’ city rooms; nobody could
e
was ranked the fourth fastest in the world.
(B) There are two reasons why many would call
Ratan Tata's decision to build a $30-million super-
computer outrageously bold. (Yes, he personally t
steered the project.) First, none of the top ten super-
meet the salaries he offered. His produced evolved computers in the world has been fully funded by the
into a simple formula. companies that built it. They were built only after
(B) He spent money faster than it came in—and he users like the US Nuclear Security Administration
spent it all on the Banner. The paper was like a and New Mexico Applications Center placed an
luxurious mistress whose every need was satisfied order, and put the money on the table. Bluntly put,
without inquiry about the price. no one has built a supercomputer in order to build a
(C) He paid them well; he got nothing but his rent business around it.
and meals. He lived in a furnished room at the time (C) Second, none of them has had the audacity to try
when his best reporters lived in sites at expensive and build such a machine in six weeks. So why did
hotels. Tata take such a gamble ? The answer is simple: Tata
(D) In the first years of the Banner’s existence Gail and Computational Research Labs (CRL), the Tata
Wynand spent more nights on his office couch than Sons subsidiary that built the machine, wanted to
in his bedroom. The effort he demanded of himself make it to the Nevada SC07 conference last month.
was hard to believe. He drove them like an army; he The bi-annual event ranks the world's 100 fastest
drove himself like a slave. supercomputers.

CAT Complete Course | 519

Visit : www.EasyEngineering.net
Visit : www.EasyEngineering.net

(D) It can perform 120 trillion calculations per the existing airport remains operational," says
second-or 120 teraflops) "If we had missed the con- CAPA's Kaul. But to look for ways to resolve the
ference, we would have had to wait for the next issue, "an Oversight Committee involving all the
edition, which is a matter of several months," says N stake-holders has to begin negotiations with the
Seetha Rama Krishna, Project Manager at CRL. The operators," he says.
team, led by Chief Executive Officer of Tata (C) Although industry analysts aver that the airport
Consultancy Services S Ramadorai, didn't want that operators are on a sound wicket legally, developers
delay. may have to seize the initiative to ensure that the
(1) BCAD (2) ABCD long-term interests of their airport are not
(3) BCDA (4) ABDC jeopardized.
(5) DCAB (D) Senior officials, however, point out that they
cannot be perceived to be sending negative signals to
14. (A) Sometimes, acquisitions can provide control of a
investors by going back on contractual obligations,
global brand. Welspun India, a private label supplier
"at a time when we are inviting private and foreign
of terry towels to big retailers like Wal-Mart and JC
capital to invest in our airport infrastructure.
Penney, was eying the branded market (where
margins are at least 2-3 % higher.) That is where the (1) CADB (2) DCAB

ww
acquisition of 150-year old towel brand Christy’s ,
with a annual turnover of $65 million, helped.
(B) Similar is the case with Rain Calcining. It was
(3) ADCB
(5) CBDA
(4) ABCD

w
16. (A)Though the nascent FM radio sector has no data
struggling to gain access into large customers like on the demand-supply gap, human resources

.Ea
Alcan and Alcoa. That was when it acquired CII
Holding, a US-based company. "After this, access to
Alcan and Alcoa became far easier," says Kalpesh
managers admit difficulty in finding suitable staff.
Industry sources say common referral programmes
and direct hiring are there but RJs are normally

syE
Kikani, Head of Global Investment at ICICI Bank,
who advised Rain Calcining in the deal. The
acquisition has helped Rain Calcining grow from Rs
spotted during talent hunts.
(B) radio gaga, a jarring saga? Talent crunch has hit
the airwaves. FM radios, just when picking up
365 cror (2005) to over Rs 4,000 crore (2008
forecast) in revenues. ngi listener signals across the cities, are finding it hard to
get radio jockeys (RJs). FM channels are mush-
(C) In 2002, Amtek Auto was struggling to sell its
castings and forgings to customers in the UK and us.
The two countries account for two-thirds of global nee rooming; by this December, there will be 245 FM
stations across 87 cities. And the Rs 500-crore
industry is poised for a growth of 28 % till 2011. But
automotive volumes. "The biggies want supply
security and the vendor to be close by. There was no
rin
at the moment, it looks like it will most probably be
an anchor-less sail.
way we could supply from India," recalls Santosh
Singhi, Director of Finance at Amtek Auto.
(D) Its export earning that year was zilch. "That's why g.n
(C) Estimates of the attrition level in FM radio vary
between 25 and 30 %. Radio companies attribute this
to the fast addition of newer stations. In the absence
we decided to acquire globally. Our first acquisition
gave us a Backdoor entry into General Motors, Ford
and Toyota," he adds. Today 80% of Amtek Auto's
sales or roughly $ 770 Million comes from
e
of non-poaching agreements, the industry is also
seeing rampant raid on talent, pushing up salaries
further. On an average a RJ joins at a monthly pay of
Rs 5,000 to Rs 12,000, depending on talent, and this
t
acquisitions. can go up to lakhs in a short time, depending on the
(1) CDBA (2) CADB public appeal.
(3) CDAB (4) CBDA (D) For format-specific channels like, Meow-a chat-
(5) CABD based women's channel-finding right anchors is
harder. Says Anil Srivatsa, Chief Operating Officer
15. (A) Already, ministry mandarins say they have
of Meow: "For a music-oriented channel, it is easier
incorporated a special chapter in the new aviation
to hire talent. But our case is different. Though
policy that would make metro connectivity to major
everyone loves talking, on radio, it is not quite the
airports mandatory. BIAL and GMR Group are now
same."
even willing to part-finance a metro rail hook-up
(1) ABCD (2) BADC
from the airport to their respective cities. But it will
still take three years to run the course. "Only if traffic (3) DBAC (4) ADBC
growth exceeds projections would we look at the (5) BACD
option to keep the existing airport open. 17. (A) One of his current research topics is exploring
(B) “Both the airport operators would be within their connections between biological cell mechanics and
right to claim compensation from the government if human disease states." Prof Suresh is optimistic the

520 | CAT Complete Course

Visit : www.EasyEngineering.net
Visit : www.EasyEngineering.net

new responsibilities wouldn't shut out his time for much capital expenditure is required for maintenance
research. to secure reasonable profitability in future," says
(B) Indian institutions take notice, for Prof Suresh is Rajat Datta, General Manager of GE Shipping.
a proponent of international collaborations. Says he: (1) ABCD (2) CBDA
"There are potentially many possibilities to explore (3) ACBD (4) DBCA
interactions with Asia, including Indian institutions. (5) BCDA
But they must be very carefully planned in the sense 19. (A) The feeling that industry, and also the govern-
that both sides should derive significant benefits ment, is not doing enough is growing. The promises
from such partnerships. " made by the industry about creating and mentoring
(C) JULY 23 will be marked in golden letters at the dalit entrepreneurs haven't impressed many. The
Massachusetts Institute of Technology (MIT). For on Confederation of Indian Industry (CII) has promised
that day, an Indian, Subra Suresh, would become the to mentor 100 SC/ST entrepreneurs this year.
dean of its prestigious college of engineering. An IIT (B) However, these sporadic moves by the Indian
Madras alumnus, Prof Suresh's bonds with MIT are industry are few and far between to address the
deep; he completed his doctorate there, and years larger issues. "Why shouldn't SC/ST graduates be
later, came back to teach, and did post -doctoral taken aboard and given on-the-job or in-house

ww
research.
(D) Between IIT and MIT, Prof Suresh had a stop-
over at Iowa State University for his masters. Prof
training," asks Chandrabhan Prasad, author and dalit
ideologue. "These so-called initiatives only send a
message that these guys are not employable till they

w
Suresh is now the Ford Professor of Engineering at
the Department of Materials Science and Engineer-

.Ea
ing, where hundreds of projects on nanotechnology
to biotechnology are taken up at an average annual
are trained. What they may lack is articulation or soft
skills, not core skills".
(C) Affirmative action is once again under the scan-
ner. The recent initiative by Infosys, in which a batch
budget of $18 million.
(1) DCAB
(3) BCAD syE (2) CADB
(4) CDBA
of scheduled caste/scheduled tribe graduates were
trained and subsequently absorbed by IT majors, has
provoked the industry to take notice. It is seen as yet
(5) CDAB
18. (A) Companies are also getting good realisalions for
old ships. According to Clarksons Research, prices. ngi another validation of the argument that 'employ-
ability' is at the core of the issue and that it can be
addressed with training and skill programmes.
of 5-10-year-old ships are 16% higher compared to
new, building ships. "This will continue as long as nee (D) “These students were graded using the same
stringent norms used by Infosys internally for

rin
there is delay in ship acquisition," says Atul Kulkarni, assessing our trainees," says T V Mohandas Pai,
Senior Manager (Consulting) with Deloitte. Tanker Director Human Resources, Infosys. The company
prices are already up 17 % compared to last year and may institutionalise this mechanism and take it to
shipping firms believe freights will continue to be
high for at least two to three years more. Shipping g.n
other cities. A best-practice document is being
prepared. In the same vein, Assocham has announced
companies can enjoy this delightful dilemma while
the good times last.
(B) It’s that time of the shipping cycle once again
when companies have to decide whether they want to
e
a programme with Rai Foundation to put 300 girls

MBA courses and help those secure jobs.


(1) ABDC (2) CDBA t
from below poverty line families through graduate or

sail or sell. Last financial year, four companies took (3) CBDA (4) ACDB
the sell decision. At least ten ships were sold and Rs (5) ACBD
271.3 crore booked as profits, making up 13 % of the 20. (A) these theme-based malls have varying positioning
combined FYO7 profits of these companies. structures that could be adopted depending upon the
(C) at the peak of the current economic cycle, development potential, catchments characteristics
capacities of ship building firms are fully booked and and product penetration.
there is a three to four years' wait for new ships. (B) similarly, developers are also coming up with
Freight rates are also higher. That leaves shipping different mall formats to attract consumers. A notable
lines with two happy options-sell old vessels and emerging format is the specialty malls that are
book good profits or sail them and improve earnings. planned as unique centers.
(D) Surbhi Chawla, an analyst with Angel Broking, (C) It is critical for each retail development to
says in FYO7, GE Shipping sold its single hull ships, position itself clearly in reference to the neigh-
and Shipping Corporation of India offloaded its old bourhood, trade and tenant mix. Specialty malls take
fleet as it is expecting delivery of 52 new vessels. a step further and create a niche and water-tight
"Selling ships are the functions of acquisition price, positioning for themselves. Specialty malls are
what EBIDTA is earned by the company and how generally developed around a product category

CAT Complete Course | 521

Visit : www.EasyEngineering.net
Visit : www.EasyEngineering.net

(automobile, electronics, furniture, jewellery, health, dividend. That was clear admission that dividends
etc) or a target segment (women, kids, western, were a substantial part of promoter earnings. This
ethnic, discount, etc). link between executive pay and promoter dividend
(D) The evolution of modern retailing, from a may not be entirely appropriate, though.
nascent industry to an organised sector, has been a (B) for many years now, HCL Technologies has
subject of discussion in recent times. Indian retailers lagged behind Wipro in the IT sector sweepstakes.
are still experimenting with various modern retail But last financial year, HCL group founder Shiv
formats; we have seen the emergence of various new Nadar got the better of Wipro Chairman Azim Premji
retail formats such as stationery, laundry, health and on one count. Nadar, through a holding company,
beauty, medical, etc in almost all parts of the took home a dividend cheque of Rs 717.26 crore, a
country. tad higher than Azim Premji's dividend earnings of
(1) ADCB (2) DBAC Rs 696.96 crore. Remember, dividends are taxfree in
(3) ABCD (4) CABD the hands of the recipients- promoters, in this case.
(5) DBCA (C) HCL Technologies gives-away almost two-thirds
21. (A) Everybody in the courtroom was touched–except of its profits as dividends, the highest among leading
Steven Mallory. Steven Mallory listened and looked IT companies. Promoters, who hold over 68 % of the

ww
as if he were enduring some special process of company's equity, are the biggest beneficiaries.
cruelty. (D) Even as questions are being raised about CEO
salaries in India, attention is slowly shifting to
(B) At his trial for the assault on Ellsworth Toohey,

w
dividends, another big source of promoter earnings.
Steven Mallory refused to disclose his motive. He
At the recent annual general meeting of Infosys, a

.Ea
made no statement. He seemed indifferent to any
shareholder wanted to know why director Mohandas
possible sentence.
Pai was getting more salary than others like co-
(C) The judge gave him two years and suspended the

syE
chairman Nandan Nilekani.
sentence
(1) ACBD (2) BACD
(D) But Ellsworth Toohey created a minor sensation (3) ABDC (4) BCDA
when he appeared, unsolicited, in Mallory’s defense.

ngi
(5) BDCA
He pleaded with the judge for leniency; he explained
that he had no desire to see Mallory’s future and 24. (A) Since paying off a loan takes the debt burden off
career destroyed. our shoulders, in the consequent sense of relief, we
(1) CADB
(3) CADB
(2) BACD
(4) BDAC nee often forget to tie up all the loose ends. It is also
because the lender, after recovering the money,
shows little of the interest it did when it was trying to
(5) BDCA
22. (A) The young men talked a great deal about
get you to borrow.
rin
(B) most of us worry whether our paperr work is
injustices, unfairness, the cruelty of society towards
youth, and suggested that everyone should have his
future commissions guarantied when he left college. g.n
right when we are applying for a bank loan for a
house, car, or an MBA programme. What we should

(B) The woman architect shrieked briefly something


about the iniquity of the rich. the contractor barked
that it was a hard world and that “fellows gotta help
one another.” The boy with the innocent eyes
finished paying off a loan. e
be equally careful about are the procedures' we
follow and the paperwork we do when we have

(C) Apart from the conventional ones, we can look att


credit cards as loans too. Most cards today are 'life
pleaded that “we could do so much good……...” time free'. But just in case it has to be returned,
(C) His voice had a note of desperate sincerity which there are certain steps that need to be followed to
seemed embarrassing and out of place.Goldon ensure that ending the relationship with the issuing
L.Prescott declared that the A.G.A was a bunch of bank is hassle-free.
old fogies with no conception of social responsibility (D) So, most of the time, the onus of sorting out the
and not a drop of virile blood in the lot of them, and papers and recovering documents such as house
that it was time to kick them in the plants anyway. ownership deeds will be on the borrower, Unless
(D) The woman of indefinite occupation spoke about these things are sorted out, you may be in for a
ideals and causes, though nobody could gather just surprise later when you try to sell or mortgage your
what these were. house or car.
(1) ACDB (2) ABDC (1) ADCB (2) BCDA
(3) DBAC (4) DACB (3) BCAD (4) DCBA
(5) ABCD (5) BADC
23. (A) Chairman NR Narayana Murthy’s answer- unlike 25. (A) It appeared in the Banner and had started as a
founders like Nilekani, Pai was not getting much as department of art criticism, but grown into an

522 | CAT Complete Course

Visit : www.EasyEngineering.net
Visit : www.EasyEngineering.net

informal tribune from which Ellsworth M. Toohey 27. (A) We knew nothing about money, and so we had
pronounced verdicts on art, Literature, New York this sort of gang mentality toward anybody who
restaurants, international cries and sociology- mainly worked for us," he recalls. "It sounds funny now, but
sociology. that's all we had to rely on." But he didn't know the
(B) He had heard the latest story about Ellsworth difference between a stock and a bond and lost
Toohey, but he did not want to think of it because it money in real estate. So at the height of his career, he
brought him back to the annoying subject of the gave up partying and went back to school in 2000 to
strike. Six months ago, on the wave of his success study business
with sermons in stone, Ellsworth Toohey had been (B) As Torrent’s of money streamed into his wallet
signed to wire “one small voice,” a daily syndicated from multiplatinum albums in the '80s and '90s, Duff
column for the Wynand papers. McKagan, then the bass player for the hard rock
(C) But he had said nothing in his column, for no one band Guns N' Roses, had little interest in tracking his
could say what he pleased on the papers owned by cash. Instead, he relied on intimidation and his
Gail Wyned save Gail Wyned. However, a mass group's reputation as the "most dangerous band in the
meeting of strike sympathizers had been called for world" to prevent managers from ripping him off.
this evening. Many famous men were to speak, (C) Today, McKagan, 43, tightly monitors the

ww
Ellsworth Toohey among them . At least, Toohey’s finances of his current band, Velvet Revolver. Like
name had been announced. other rockers easting into middle age or seniorhood,
(D) It had been a great success. But the building McKagan is also experimenting with new partnership,

w
strike had placed Ellsworth M. Toohey in a difficult in response to a music business in flux. Amid plung-
position. He made no secret of his sympathy with he ing record sales and internet file Sharing, rockers are
strikers.
(1) BCDA
(3) BACD
.Ea (2) ACBD
(4) BADC
eagerly putting their names everywhere.
(D) Their "brands" are now found in television

syE
commercials, tour sponsorships and merchandise as
(5) ABCD diverse as cars, private-label wines and celebrity
26. (A) It was Roark’s building on the sketch, very cruises. The rock band Kiss has been among the most

ngi
neatly drawn. It was his building, but it had a prolific merchandisers, selling products ranging from
simplified Doric portico in front a cornice on top, condoms to the "Kiss Kasket”' a limited-edition
and his ornament was replaced by a stylized Greek coffin. The band's latest offerings include musical
ornament.
(B) A great deal more was said by the three men.
Roark barely heard it. He was thinking of the first
nee toothbrushes, pool cues, window blinds and baby
booties. “It’s a different ballgame now,” compared
with rock’s baby boomer heyday, says, Joseph
bite of machine into earth that begins an excavation.
Then he heard the chairmen saying “…and so it’s rin
Bongiovi, who handles merchandise and partnerships
for the rock group Bon Jovi.
yours, on one minor condition.” He heard that and
looked at the chairman.
(1) CBAD
(3) BACD
(5) ABCD g.n (2) BCAD
(4) BADC
(C) “It’s a small compromise, and when you agree to
it we can sign the contract. It’s only an inconse-
quential matter of the building’s appearance. I under-
stand that you modernists attach no great importance
to a mere façade, it’s the plan that counts with you,
e
28. (A) The Shipping Corporation of India has applied to
the government for permission to enter shipbuilding
t
with an investment of at least Rs. 1,000 crore. Others
like Jindal, Essar, Mercator Lines and Garware
quite rightly, and we wouldn’t think of altering your Offshore are in various stages of planning an entry.
plan in any way, it’s the logic of the plan that sold on “About 15 new players have shown interest. At least
the building. So I’m sure you won’t mind?” 10 are likely to enter.” Says Ramesh Agarwal, CEO
(D) “What do you want?” “It’s only a matter of a of i-maritime, a ship and port-building consultancy.
slight alteration in the façade. I’ll show you. Our Mr. (B) On the face of it, this seems to be a simple
Parker’s son is studying architecture and we had him enough business decision. The industry has seen
draw us up a sketch, just a rough sketch to illustrate frenetic growth in recent times. Outstanding orders
what we had in mind and to show the members of the have grown to 269 million gross tonnage (GT) in
board, because they couldn’t have visualized the june 2007 from 75 million GT in 2002. That could
compromise we offered. Here it is.” He pulled a have a big bearing on the current 721 million GT
sketch from under the drawings on the table and capacity of commercial ships plying the seas.
handed it to Roark. (C) Ship- building, a $ 500-billion business globally,
(1) BCDA (2) DABC has many takers in India Inc. Larsen & Toubro is
(3) CADB (4) CDAB pumping in Rs. 2,000 crore in the hope of earning a
(5) BACD billion dollars in revenue by 2015. It has started

CAT Complete Course | 523

Visit : www.EasyEngineering.net
Visit : www.EasyEngineering.net

building its first ship in Hazira and has orders for ten private equity syndications in the last two years,
ship s, two of them big, worth about $400 million. indicating that the segment is building up
(D) Pipavav Shipyard has already spent $250 million momentum.
in its shipbuilding project and plans to invest another (D) The segment has over 400 players and employs
$400 million. It has orders for 22 ships worth $900 more than 410,000 people. Industry body Nasscom
million. estimates the IT-BPO sector to rake in more than
(1) CBDA (2) ACDB $47.8 billion in revenue in FY07-almost a 10-fold in-
crease-against $4.8 billion in FY98.
(3) ACDB (4) CDAB
(1) CDBA (2) ABDC
(5) DCBA
(3) ABCD (4) CDAB
29. (A) “It’s all nonsense. It’s all a lot of childish (5) BDCA
nonsense. I can’t say that I feel much sympathy for
31. (A) Two major areas of interest in IDM are: enabling
Mr.Hopton Stoddard. He should have known better.
user access; and user lifecycle management. While
It is a scientific fact that the architectural style of the
users focus on efficiency of the experience (with one
Renaissance is the only one appropriate to our age.
ID you can sign-in and use many applications) and
(B) None of the witness looked at Roark. He looked apparent security, system administrators look at the

ww
at them. He listened to the testimony. He said: “No
question,” to each one. Ralston Holcombe on the
stand, with flowing tie and gold headed cane, had the
efficiency of management (the user to administrators
ratio), service level user administration turnaround
time and actual security.

w
appearance of a Grand Duke or a beer garden
composer. His testimony was long scholarly, but it
came down to:
.Ea
(C) The attorney gave them leads like an expert press
(B) So, organisations have to ensure that only the
intended recipients can access information. This has
changed the scope and importance of security
technologies and identity management.

syE
agent. Austen Heller remarked that architects must
have fought for the privilege of being called to the
witness stand, since it was the grandest spree of
(C) With reports of security breaches, data thefts and
electronic privacy violations happening daily,
business managers have scaled up their IT security
publicity in a usually silent profession.
(D) In the next two days a succession of witnesses ngi measures and adopted newer methods of encryption.
Earlier, IT security was designed to restrict access to
outsiders, but today, more internal applications and
testified for the plaintiff. Every examination began
questions that brought out the professional
achievements of the witness. nee resources need to be shared with partners, customers
and field staff in remote locations.

rin
(D) Organisations in India have begun to realise the
(1) ABDC (2) CABD
importance of identity management (IDM),
(3) CADB (4) DCBA especially in ITES, retail, telecom, manufacturing
(5) ADCB
30. (A) M and As were the flavour of the season with g.n
and financial services. IDM solutions can raise
productivity, lower costs and meet today's stringent
Genpact, WNS, Firstsource, Transworks, eFunds and
Techbooks, all acquiring companies. In February,
Pune-based HOV Services, in a large deal, snapped
up US-based Lason for $148 million. Among other
compliance need.
(1) BCDA
(3) ACBD
(5) CBDA
e (2) DCAB
(4) BDAC
t
big deals, Infosys acquired Citibank's 23 % stake in Answers
BPO arm Progeon last April for $115.13 million. 1. (3) ACBD 2. (2) BDAC 3. (1) DCBA
(B) The sector clocked 28 % growth-including
4. (4)BADC 5. (3) DBAC 6. (4) CBAD
domestic and export segments-in FY07. Nasscom
sees exports in the BPO segment at $8.3 billion in 7. (2) BADC 8. (3) CABD 9. (4) BDAC
FY07, a rise of 32% year-on-year. The IT-BPO 10. (2) DCBA 11. (2) DCBA 12. (4) CBAD
sector, Nasscom says, is well on track to hit the 13. (1) BCAD 14. (1) CDBA 15. (5) CBDA
target of $60 billion in export revenues by 2010. It
16. (2) BADC 17. (5) CDAB 18. (5) BCDA
contributes roughly 5.4% to India's GDP. A quick
snapshot reveals more interesting trends. Domestic 19. (2) CDBA 20. (2) DBAC 21. (4) BDAC
BPO operations grew 53 %, faster than the rate of 22. (5) ABCD 23. (4) BCDA 24. (3) BCAD
exports. 25. (4) BADC 26. (1) BCDA 27. (3) BACD
(C) Globalisation and favourable demographics have
made India a sought after ITES-BPO destination. 28. (4) CDAB 29. (4) DCBA 30. (3) CDBA
The ITES-BPO industry has seen a flurry of 31. (5) CBDA
domestic and cross-border deals, IPO listings and ●●●

524 | CAT Complete Course

Visit : www.EasyEngineering.net
Visit : www.EasyEngineering.net

Part B : English Reading Comprehensive

1 Reading Comprehensive
What is in test of Reading Comprehension ? 5. It can be inferred that xyz.
CAT Question Format 6. The primary purpose of the passage is to
Eye Span Map (ESM) 7. The author raises the questions at the beginning of
39 Example Reading Comprehension the x para in order to/ to highlight etc.
15 Lengthy Type Reading Comprehension 8. It can be inferred that the most important difference

ww
10 Revision Test of Reading Comprehension
What is in test of Reading Comprehension ?
between x and y mentioned in line z is that …………
9. Which of the following would be the most logical
continuation of the passage a, b, c, d, e ……………

w
CAT Reading Comprehension is a test of your ability

.Ea
to read and understand unfamiliar materials and to answer
questions about them. Reading Comprehension passages
and questions appear in the Reading Comprehension
10. The author most likely places the word/s in quota-
tions in line x in order to remind/ highlight the reader
that/ xyz issue, point

Section.
syE
The Reading Comprehension passages are approxi-
mately 150 to 2500 words long. Each one is followed by
11. The slayer of monsters introduced in line ‘x’ func-
tions primarily /………../……….as..
12. According to the passage the main / primary differ-
three or more question about its content and some
valuable tips from me ‘There’s no need to worry about ngi ence between x and y is that …
13. In the study in lines x,y the ……… because………
what you know or don’t know about the topic in a reading
comprehension. The answers are all based on information
in the Passage, and you won’t be required to draw on nee
14. The author attitude towards xyz can best be
described as……
Concern, resignation, anxiety, disinterest, approval,
outside knowledge.
Question Format rin
pessimism, optimism, anger, bigoted, egoistical
etc……….

Reading Comprehension question follow the standard


multiple—choice format with four-five answer choice 16. The ………… leads ………… to g.n
15. In the passage expression “……………” attends to

each. All of the question fall into on of the following


types :
● The main idea of the passage
● Specific details mentioned in the passage
to
e
17. According to the passage, the term “………“ refers

18. What is the thematic highlight of this passage?


19. In the study* described in lines # prove that:
t
● The author’s attitude or tone or aim 20. Which of the following would have been true.
● The logical and informational structure of the If ……… ?
passage 21. According to the author which of the following will
● Future inferences that might be drawn from the text correspond with (or vice versa)
● Application of the ideas in the text to new situations 22. The reason behind ………… are.
For Example 23. The primary reason that ………… was.
1. The author cites xyz/line # in order to a, b, c, d 24. Consider / peruse the following statements and
2. The author mention all the following ways by which choose the correctly stated one from the options:
………… EXCEPT 25. The stages connected to ……… of ………… is/are
3. With which of the following statements about xyz A,BC,D ………… (1) (2) (3) (4).
would the author most likely agree? 26. The basic symptom of ………… is reflected when
4. The author makes which of the following criticism of …………
the xyz, 27. The idea that ………… is directly linked will.

CAT Complete Course | 525

Visit : www.EasyEngineering.net
Visit : www.EasyEngineering.net

28. What is the basic idea behind xyz in xyz? 58. What is the basic idea behind ……… in xyz ?
29. Which of the following cannot be inferred from the 59. Why is ‘x’ in compatible to ‘y’ and is its significance
following 60. An appropriate title for the passage would be—
30. ‘………’ mentioned in line # para ……… comes 61. ‘………’ mentioned in para x/line * [ come close in
close in meaning to / is synonymous with meaning to] [signifies] [portrays] [depicts]
31. According to the passage ……… came before / after 62. The example of ……… goes to prove.
etc.
63. Which of the following is true of the xyz system?
32. Which of the following has not caused ………… ?
64. What does the phrase/sentence ……… imply.
33. The attitude of the author towards ………… is
65. In which context is the phrase/ sentence used:
—energetic, nonchalant, sardonic, penetrative, dis-
66. The author employs the metaphor ( ) to suggest.
believing, nostalgic, complaisant- acquiescent
67. What is the contradiction in para ‘x’ the author uses
34. Which of the following is true as depicted in para/
it to: pick out the correct pair—
lines—
68. The author implies that xyz in not necessarily prof-
35. According to the author the main cause for ………
itable for/ of because—
as ……… was.

ww
36. Which of the following can’t be described as
“………” ?
69. The writer recommended which of the following
changes*/ solutions etc be made in ‘ ‘

w
70. The passage supplies information to all the questions
37. Which of the following is not ‘Z’s idea ?
with the exception of.

best characterized by.


.Ea
38. The difference in concept xyz between ‘x’ and ‘y’ is

39. Going by the author’s exposition of ‘xyz’ which of


Eye Span Map (ESM)—A slow reader reads a line
as follows (show by ESM)

syE
the following is true/ untrue?
40. Which of the following views of ’xyz’ can/ cannot be A fast reader reads a line as follows (show by ESM)

ngi
attributed to the author ?
41. The author holds that.
Very Important—Vocabulary is a vital part of

nee
42. It is the author’s contention that.
43. The author makes a distinction between perception* reading comprehension.
and creation in terms of. Examples of Reading Comprehension.
44. A writer* as an artist*.
45. The writer argues that ……… because they feet* that. rin
Passage–1

g.n
Word Count–600
46. The author acknowledges xyz because— As a writer or editor today, you have a lot more at
47. The main difference between ’x and z’ in the matter your disposal than pencils and erasers.
of ‘X’ and ‘Y’ according to the author is:
48. In the statements “…………”, it refers to.
49. The importance of xyz is that it:
e
● Word-processing software simplifies writing and
revising, from note-taking and drafts to final copy.
t
● The Internet is a reference library at your fingertips.
50. ………… can be exemplified by : ● E-mail makes keeping in touch with colleagues,
51. Which of the following statements best / strongly friends and family at the press of a button besides
conveys the overall idea of the passage ? easing the interaction among authors, editors, peers,
52. Which one of the following best describes he reviewers, project managers, and production staff.
approach of the author ? These powerful tools help you do it, but a solid
53. Which of the following parallels between ……… x foundation in the principal of revising tells you what
………, is not clamed by the author ? needs to be done.
54. In developing ………… to ………… these are based Rewrite Right ! Is for people who must write at work
on: or in school and for freelance writers struggling to make
55. The areas in which ……… has / have excelled over sales. Editing is improving something written-making it
……… societies are/is easier to follow, snappier, more interesting. Knowing
56. Pick/choose out the correctly stated statements from how to edit means knowing what good writing is in the
the following— first place. And good writing comes from knowing how
57. The idea / illusion / presumption etc that is (not) to revise, how to tug on words and adjust them until they
susceptible to ……… is directly linked to: say what you want them to say. In other words, writing

526 | CAT Complete Course

Visit : www.EasyEngineering.net
Visit : www.EasyEngineering.net

and editing are facets of the some subjects : doing a good 1. Which of the following reason/s does the author state
job of putting ideas into words. to suggest that modern day technological inventions
Have advances in telecommunications reduced the are helpful but still require editing and good writing
need for good writing ? Not at all. The medium may skills—
change but the language still needs to be well crafted. 1. Through E-mail is handy, quick and cheap and
People who used to step down the hall to brainstrom with people’s e-mail style is breezy and informal.
a colleague now sit down at a keyboard and whip out a However, scant attention is paid to spelling,
message on e- mail. grammar & omitted words such sloppiness can
E-mail has many advantages it’s convenient, fast, be hazardous in any communication.
cheap, and less intrusive than a phone call; it provides a 2. People wonder if you have any idea of what you
paper trail and is easily distributed to a large number of are saying as they don’t.
people to work place from their homes. 3. Advance in telecommunications have not
But the very ease of e-mail can be a hazard. Many reduced the need for good skilful writing. As,
people have developed an e-mail style that’s breezy and they still need to be well crafted.
informal; they pay less attention to spelling and grammar 4. The informality of e-mail can lead one in
than they would in a standard letter. Should they care thinking sloppiness is right on spontaneity is fine

ww
about good writing? You bet! E- mail recipients shouldn’t
have to puzzle their way through a message riddled with
misspelled or omitted words and confusing reference.
sloppiness is a wet squib in any communication.
(A) 1,2,4 (B) 2,3,4

w
Don’t let the informally of e-mail fool you into thinking

.Ea
sloppiness is okay. Spontaneity is fine; Sloppiness can be
dangerous in any communication.
Muddled instructions create confusion. Costly
(C) 3,4
(E) 1,2,3,4
(D) 1,3,4

2. According to the author muddled instructions create

syE
confusion as (All of the following are true Except)—
research is repeated because results are buried in an
(A) Thousands o f pages o f costly research is
obscure two pound report. Boring writing is tossed aside
repeated due to nebulous and ambiguous
unread. Slipshod writing breeds distrust, prompting

ngi
readers to wonder if language is the writer’s only area of
incompetence.
reporting style
(B) Writing that cause ennui is put down unread
At the other end of the spectrum good writing gets
things done, its crisp, clear style requires less of the
reader’s time. Good writing lowers administrative
nee (C) Slipshod writing creates doubts and distrust in
the readers’ minds
(D) Though well edited and crafted skilfully lack of
expenses, lightens work-loads, and suggests the writer is
competent in other areas as well. the work insipid.
rin
personal brainstorming instead of e-mail make

Yet when it comes to writing, many capable people


falter. They may be experts at marketing or high-energy g.n
(E) The readers ponder over whether the writer’s
sole skill is language incompetence.

physics, but ask them to write it up, and they rely on


worn-out expressions and stilted prose.
As a consulting editor, I’ve learned where most
(A) Instructive
(C) Panegyric
e
3. The author’s tone in the passage shows that it is—
(B) Critical
(D) Persuasive t
people need help. My first two books, Write Right ! and (E) Hard sell
better letters, provide simple advice. 4. The Passage is intended for those who want—
Rewrite Right ! is for those who want more. It 1. Simple advice on how write well
describes two levels of editing; (1) to improve style and 2. To improve style and content
content and (2) to correct language.
3. To learn how to write better letters
Rewrite Right ! includes a variety of reference 4. To use a proper and correct language
materials; lists of accepted abbreviations, hackneyed
5. Methods to make documents look better with a
expressions, common redundancies, and irregular plural. glossary of definitions of terms not familiar
It suggests ways to make a document look better and
(A) 1, 2, 5 (B) 2, 4, 5
explores sophisticated tools available to today’s writers
and editors. A glossary provides definitions of unfamiliar (C) 1 and 2 (D) 1, 2, 3, 4
terms. (E) All of the above
I hope Rewrite Right ! helps you learn not only to Answers
write and rewrite well, but to enjoy doing it. 1. (E) 1, 2, 3 and 4 2. (D) 3. (A) 4. (D) 1, 2, 3 and 4

CAT Complete Course | 527

Visit : www.EasyEngineering.net
Visit : www.EasyEngineering.net

Passage–2 Pakistani officials interviewed by the New York


Words–541 Times denied their government had overcharged the
United States for the "War-on-terror" military aid it gets.
More than five billion dollars in US aid to Pakistan
But US officials cited helicopter maintenance as an
often never reached the military units it was intended for
example of the funding programme's failure.
to fight al-Qaida and the Taliban - instead it went into
weapon systems aimed at India. Much of the money While Pakistan received $ 55 million for helicopter
meant, to reimburse frontline Pakistani units was maintenance for an eight-month period in 2007, the
channeled to weapons systems aimed at India and to pay officials said they found out that only 25 million had been
inflated Pakistani reimbursement claims for fuel, received by the Pakistani army for helicopter maintenance
ammunition and other costs, unnamed US government for 2007.
and military officials to daily. Allegations that generous military aid to Pakistan has
Pakistanis critical of president Pervez Musharraf said been squandered represent another setback for President
he used the reimbursements to prop up his government George Bush's administration, which has viewed Pakistan
and one European diplomat said the United States should as an important ally in the “war on terror”.
have been more careful with its money. "I wonder if the 1. What was the strongest factor in which Pakistan was
Americans have been taken for a ride," said the diplomat, accused of misuse of funds a U.S. write up—

ww
who spoke on condition of anonymity.
Money intended to repay Pakistan. for maintaining
100,000 troops in the restive tribal areas apparently does
(A) Pak used the funds for other reasons than
intended

w
not reach the troops who need it, officials said.

.Ea
"It is not making its way, for certain, we know, to the
broader part of the armed forces which is carrying out the
brunt of the operations on the border” with Afghanistan, a
(B) To strengthen his (Prevez Musharraf) govern-
ment
(C) To create weapons systems armed at India
(D) To reinforce its frontline military forces

syE
senior for US military official said ?
Despite the vast funds flowing to Pakistan, a US
official visiting the border recounted finding members of
(E) To create disturbance between India and
Pakistan

the country’s frontier corps “standing there in the snow in


sandals”. ngi 2. Which one of the following statements is not True—
(A) The money was not used for maintaining troops

nee
in restive tribal area.
Several soldiers were wearing world war-I era pith
helmets and had battered Kalashnikov rifles with only 10 (B) Pak’s frontier troops have inadequate gear and
rounds of ammunition each, the official said. The two adequate weapon to conn terrorist attacks.
countries have forgotten clear strategic goals as to how
the US military aid should be spent or how Pakistan
rin
(C) US Funds were not used to reimburse Pakistan
for carrying out military operations against
could show it was meeting Washington’s expectations,
according to US and Pakistani officials US aid to
Pakistan has come under scrutiny recently in the United
terrorism.

g.n
(D) To build equipment to counter attack any Indian

States given the strength of the al-qaida and Taliban cells


in Pakistan’s northwestern tribal areas as well as the
failure to secure the
Capture of Osama bin Laden.
assault
(E) All of the above
e
3. In the last paragraph the writer implies—
(A) Pakistan has lost favour with the USA
t
Musharraf has also been forced by US pressure to (B) Pakistan will face severe restrictions in military
ease back on repressive measures, lift emergency rule, aid
shed his military uniform and move the country toward
(C) The USA wants Pakistan to restore democracy
greater democracy. The US provides the five billion in
aid to reimburse Pakistan for carrying out military opera- (D) The President of America who portrayed
tions against terrorist threats. A separate US programme Pakistan as an ally will not face political and
delivers 300 million every year to pay for equipment and country wide flak
training for the Pakistan military. (E) The country (Pak) will get further aid from
The US Congress on Thurs day slapped restrictions USA.
on the 300 million in traditional military aid, 50 million
4. The areas in which the US found Pakistan’s misuse
of which will be withheld until Pakistan shows it is
of billions of dollars in chronological order are
restoring democratic rights.
US funds are vital for Pakistan's military, with (A) To provide gear and state of art weapons to
American aid accounting for about a quarter of the combat terrorism.
military's entire budget, the paper said. (B) Maintenance of troops in restive area terrorism

528 | CAT Complete Course

Visit : www.EasyEngineering.net
Visit : www.EasyEngineering.net

(C) Reimbursement claims for fuel and ammunation (B) Nations always stand for ideals that are
(D) Helicopter maintenance. important to the human race.
(C) It is the religious element in patriotism that
(E) All of the above
motivates us for sacrificing ourselves for our
Answers nation.
1. (C) 2. (D) 3. (A) 4. (D) (D) Our pride of the community is bound with the
community’s success.
Passage–3
Words–159 Answers
Patriotism is a very complex feeling built up out of 1. (A) 2. (B) 3. (D) 4. (D) 5. (B)
primitive instincts and highly intellectual convictions. Passage–4
There is love of home and family and friends, making us
Words–389
peculiarly anxious to preserve our own country from
invasion. There is the mild instinctive liking for com- The public distribution system,. Which provides food
patriots as against foreigners. There is pride, which is at low prices, is a subject of vital concern. There is a
bound up with the success of the community to which we growing realization that though India has enough food to

ww
feel that we belong. There is belief, suggested by pride
but reinforced by history, that one’s own nation
represents a great tradition and stands for ideals that are
feed its masses two square meals a day, the monster of
starvation and food insecurity continues to haunt the poor
in our country.

w
important to the human race. But beside all these, there is

.Ea
another element, at once nobler and more open to attack,
an element of worship, of willing sacrifice, of joyful
merging of the individual of life of the nation. This
Increasing the purchasing power of the poor through
providing productive employment leading to rising
income, and thus good standard of living is the ultimate
objective of public policy. However, till then, there is a

syE
religious element in patriotism is essential to the strength
of the state, since it enlists the best that is in most men on
the side of national sacrifice.
need to provide assured supply of food through a
restructured, more efficient and decentralized public
distribution system (PDS).
1. A suitable title for the passage could be —
(A) Elements of patriotism ngi Although the PDS is extensive-it is one of the
largest system in the world it has yet to reach the rural
poor and the far-off places. It remains an urban
(B) Historical development of a nation
(C) The role of religion and history in patriotism nee
phenomenon, with the majority of the rural poor still out
of its reach due to lack of economic and physical access.

rin
(D) Religion and patriotism The poorest in the cities and the migrants are left out, for
they generally do not possess ration cards. The allocation
2. Describing the element of worship “open to attack,” of PDS supplies in big cities is larger than in rural areas.
the author implies that it—
(A) Is unnecessary
g.n
In view of such deficiencies in the system, the PDS
urgently need to be streamlined. Also considering the
(B) Leads to national sacrifice
(C) Has no historical basis
(D) Can not be justified on rational ground
e
large food grains production combined with food subsidy
on one hand and the continuing slow starvation and

t
dismal poverty of the rural population on the other, there
is a strong case of making PDS target group oriented.
3. The tone of the passage can be best described as— The growing salaried class is provided job security,
(A) Critical (B) Descriptive regular income, and social security. It enjoys almost
hundred percent insulation against inflation. The gains of
(C) Persuasive (D) Analytical
development have not percolated down to the vast
4. Which of the following can clearly be grouped under majority of our working population if one compares only
“intellectual convictions” that the author mentions in dearness allowance to the employees in public and
the opening sentence ? private sector and looks at its growth in the past few
(A) Love of family years, the rising food subsidy is insignificant to the point
of inequity. The food subsidy is a kind of D.A. to the
(B) Love of compatriots poor the self-employed and those in the unorganized
(C) The element of worship sector of the economy. However, what is most unfortunate
(D) None of these is that out of the large budget of the so called food
subsidy, the major part of it is administrative cost and
5. Which one of the following statements is false ? wastages. A small portion of the above budget goes to the
(A) We tend to like or own countrymen better than real consumer and an even lesser portion to the poor who
we like foreigners are in real need.

CAT Complete Course | 529

Visit : www.EasyEngineering.net
Visit : www.EasyEngineering.net

1. Which of the following, according to the passage, is 7. The author’s writing style is
true of public distribution system ? (A) Simplistic (B) Argumentative
(A) It is unique in the world because of its effective- (C) Verbose (D) Descriptive
ness
(E) Analytic
(B) It has remained effective only in the cities
8. What, according to the passage, would be the out-
(C) It has reached the remotest corners of the country come of making the PDS target group oriented ?
(D) It has improved its effectiveness over the years (A) It will remove poverty
(E) It develops self-confidence among the people (B) It will give food to the poorest without addi-
2. Which of the following, according to the passage, is tional cost
the main reason for insufficient supply of enough (C) It will abolish the imbalance of urban and rural
food to the poorest ? sector
(A) Production of food is less than the demand (D) It will motivate the target group population to
(B) Government’s apathy towards the poor work
(C) Absence of proper public distribution system (E) None of these

ww
(D) Mismanagement of food stocks
(E) None of these
3. What, according to the passage, is the main purpose
9. Which of the following words ids the same in mean-
ing as the word ‘system’ as used in the passage ?

w
(A) Mechanism (B) Routine
of public policy in the long run ? (C) Machine (D) Procedure

employment
.Ea
(A) Good standard of living through productive
(E) Collection
10. According to the passage, food subsidy leads to

syE
(B) Providing enough food to all the citizens
(C) Reducing the cost of living index by increasing
supplies
which of the following ?
(A) Shortage of food grains
(D) Equalizing per capita income across different
starts of society
ngi (B) Decrease in food grains production
(C) Increased dependence

nee
(E) None of these (D) Sense of insecurity
4. Which of the following in the passage, is compared (E) None of these
with dearness allowance ?
(A) Food for work programme
(B) Unemployment allowance
1. (B) 2. (C)
rin
Answers
3. (A) 4. (D) 5. (D) 6. (E)

(C) Procurement price of food grains


(D) Food subsidy
7. (E) 8. (D) 9. (D)
g.n
Passage–5
10. (E)

(E) None of these


5. What, according to the passage, should be an
appropriate step to make the PDS effective ?
e Words–409

t
Of the many monarchs that ruled in ancient India, it
is Ashoka that we remember most. And this for at least
two reasons : He is the first on whom reliable historical
(A) To increase the amount of food grains available records exist to this day. Secondly, he was emperor and
for distribution hero, conquering and noble, powerful and pious. No
matter what he did in his earlier years , in the end he
(B) To increase the amount of food grains per ration
proved to be uncommonly virtuous and wise, remarkably
card
gentle and compassionate. His very name has become a
(C) To reduce administrative cost matter of pride for the people of India. Yes it has been
(D) To make it target group oriented stated in Buddhist lore that Ashoka was once a blood-
(E) To decrease the allotment of food grains to thirsty prince who rose to his throne by the slaughter of
urban sector his ninetyeight brothers; This may be a slight exaggera-
tion, but it is well established that his emperor of the
6. Which of the following words is the same in meaning third century B.C., which extended the boundries of his
as the word ‘power’ as used in the passage ? Magadha kingdom far and wide, engaged in a bloody
(A) Energy (B) Vigor battle to subjugate Kalinga on the eastern coast of India.
(C) Authority (D) Influence But when it was all over, Ashoka saw the cruely of
(E) Capacity war, the inhumanity of humanity of conquest. He was

530 | CAT Complete Course

Visit : www.EasyEngineering.net
Visit : www.EasyEngineering.net

deeply touched by the sufferings he had inflicated this (A) Ashoka sent out missionaries to distance places
was no crafty strategy to win over the people he had for the propagation of his massage
subdued. Ashok’s feeling were as genuine as they were (B) Ashoka built roads and hospitals
profound. He adopted the Buddhist religion which insisted
(C) Ashoka became a Monk
on compassion and non injury, became a serious student
of the faith. Years later he even relinquished pomp and (D) Ashoka followed the faith of compassion and
power, and became and ascetic. Never in human history non-injury
has a victorious potentate given up pride to become a (E) Ashoka introduced public announcements and
meek monk, as did Ashoka. billboards to expand his kingdom
And while he was still in authority, he set up channels 5. What did Ashoka do which had never happened in
for the systematic propagation of the civilized principles human history ?
to which he had been converted . He sent out missionaries (A) He had slaughtered a hundred thousand people
to distance lands, to Syria and Egypt, to Macedonia and
Cyrene, as well as to Sri Lanka and beyond. He assumed (B) A powerful Emperor had become a quiet and
tremendous responsibilities towards his people. He gentle Buddhist
worked for their material and moral welfare, and built (C) He had engraved messages in rocks, caves and

ww
many roads and hospitals. Today we see billboards and
public announcements to sell commercial products.
Ashoka adopted a very similar mode over two thousand
pillars
(D) He had built roads and hospitals
(E) None of these

w
years ago. For within his own realm he carved his
messages in rocks and pillars, in public places and in

.Ea
caves, here and there a n d everywhere, so that people
would see them and reflect over them. The idea worked ,
6. Why did Ashoka’s name become a matter of pride
for Indians ?
(A) He was an Emperor

syE
and many joined the Buddhist faith. Even more impor-
tantly, those inscriptions have left for posterity indelible
records of Ashok’s thoughts and deeds.
(B) He became, in the end, a gentle, compassionate
and wise human being

1. Which one of the two reasons does the author give


for remembering Ashoka ?
ngi (C) He did a lot of charity work then he became , an
Emperor
(D) He expended his kingdom
(A) He was an emperor and hero, powerful and
pious
(B) He slaughtered ninety eight of his brothers nee (E) He had become a Monk
7. Which of the following lands did Ashoka not send
(C) He set up channels for the systematic propaga-
tion of civilized principles rin
his missionaries to ?

g.n
(A) SriLanka (B) Macedonia
(D) He adopted the Buddhist religion
(C) Persia (D) Egypt
(E) He was the youngest emperor

e
(E) Syria

t
2. How did Ashoka attract people to the Buddhist faith ?
Answers
(A) He worked for the welfare of the people
1. (A) 2. (C) 3. (D) 4. (E) 5. (B) 6. (B)
(B) He gave up his throne
7. (C)
(C) He carved messages in rocks, pillars and caves
(D) He coerced his subjects to adopt the religion Passage–6
(E) None of these Words–662
3. Ashoka adopted the Buddhist religion because Richard Dawkins's international bestseller The God
……… Delusion has got it all wrong. Far from being a delusion,
at least in India God is alive and kicking butt daily. Week
(A) It was his strategy to win over people
before last the butt He was kicking was mine, in
(B) He wanted to become a serious student of faith Bangalore. In my case, God had manifested Himself in
(C) He wanted to preach Buddhism the avatar of a sub-registrar. A sub-registrar is a divine
(D) He was moved by the suffering he had inflicted manifestation who sub-registrars things. As I needed to
on people have a thing (a small property) sub-registrared in
Bangalore I'd had for months sought a darshan with this
(E) He had become a powerful king
exalted being. Finally, my prayers and entreaties prevailed
4. Which of the following statements is not true in the and I was informed that my long sought darshan had
context of the passage ? been granted (at the BDA office, a place of transcendental

CAT Complete Course | 531

Visit : www.EasyEngineering.net
Visit : www.EasyEngineering.net

malodorous ness, through which more wealth daily babu exists. Tough lumps, Dawkins. But you're the
passes than through Tirupati) for a date falling six weeks delusion. And I have my sub-registrar to prove it.
later. I duly booked my flight to Bangalore and back, well 1. The Author is primarily concerned with—
ahead of the darshan date. Two days before the appointed
(A) God
day, I was informed that the darshan had been postponed
by three days. Why ? Because the sub-registrar had (B) Bureaucracy
decided to go on C/L. (C/L is a divine right of sub- (C) Mr.Richard Dawkins
registrars. No, it's more than a divine right. Tirupati, for. (D) Atheism
example, can't suddenly decide to go on C/L and shut up
(E) Bangalore
shop for three days. There'd be questions in Parliament,
riots, army called out. But sub-registrars and other 2. The most appropriate title for the passage would
Dominions and Powers of his ilk go on C/L as a matter of be—
scriptural., routine.) Anyway, i now faced the prospect of (A) God is red tape
canceling two non-refundable air tickets and buying two (B) God, its’ a babu
fresh ones. So i decided to cancel only my return ticket
(and buy a new one for a later date) while retaining my (C) Bureaucracy and God
ticket for the outward journey to Bangalore. This meant (D) Man is greater than God

ww
that I'd have to spend almost a whole week in Bangalore
for the sake of a 30-minute darshan, max. Hey, who said
meeting God is easy?
(E) Babus in God’s shoes
3. The author justifies his viewpoint with—

w
Now I like Bangalore. So much so, that i once

.Ea
wanted to shift there permanently from Delhi, and might
still do so one of these years. Despite having been turned
(A) Facts
(C) Assumption
(E) Comparison
(B) Statistics
(D) Personal Experience

syE
from Garden City to Concrete Bungle by greedy land
4. The author is most likely to agree with which one of
'developers' and builders, Bangalore still enjoys a pleasant
the following:
climate and a cosmopolitan charm of manner. I like
(A) God is on C/L

ngi
Bangalore. But an enforced stay can make even paradise
seem a prison. As I served my time in Bangalore I (B) Mr. Dawkins has got it all wrong
thought about how the gods rule us in India. Not the 33 (C) God is manifested in the form of babus
million and still counting gods up in Heaven. But the
somewhat less than 33 million (but not much less) even
more powerful gods down here on earth whom we daily nee (D) God is alive and kicking
(E) None of these
pay homage to and who are generically known as babus.
Forget politicians, MPs, the judiciary, the cops, whatever
rin
5. The tone of the article is—

g.n
figures of supposed authority you care to name. The (A) Witty (B) Critical
entities who really rule us are our babus, cousin-brothers (C) Light (D) Admonishing
and cousin-sisters, one and all, to my absconding C/L- (E) Dominating
taking sub-registrar. Leave alone an insignificant profes-
sional clown like me; captains of industry and barons of
business needs must genuflect at the altar of babudom.
And when you think of the time and the money and the
1. (E) 2. (B)
Answers
3. (D)
e
4. (C) 5. (A) t
effort we all of us expend in thrall to babudom, Tirupati Passage–7
looks like a flop show in comparison. Words–189
Dawkins and fellow atheists like Sam Harris have Of the many aspects of public administration, the
argued that God doesn't exist because there's no evidence ethical aspects are perhaps the most important but the
of His existence. What the dumb clucks don’t realise is least codified. While administrative rules and procedures
that the Guy’s just gone on C/L for a couple of millennia have been codified in various public documents and
or so. Then there's the ontological paradox questioning manuals, there is no manual for the ethics of public
God's existence: Can God build a wall so high that He servants.
can't jump over it? In other words: Can God, universal While organizational behaviour analyses the factors,
butt-kicker, kick His own butt? i don't know about which influence the behaviour of individuals in an
Dawkins's God. But i do know about my sub-registrar. organization, ethics refers to those norms and standards
Who (back from C/L) will doubtless one day have to go which behaviour of individuals in an organization, ethics
see his sub-registrar to get some sub-registration done refers to those norms and standards which behaviour of
only to discover that his sub-registrar has gone on C/L. the people in an organization must conform to. While
Babudom has just kicked it's own butt. Ergo, God-as- behaviour analysis deals with factual aspects, ethics

532 | CAT Complete Course

Visit : www.EasyEngineering.net
Visit : www.EasyEngineering.net

relates to the normative aspects of administration. The small but it was money for jam. After three months, it
normative aspects are of the greatest significance. Just as was stopped. But I made a few lakh of rupees. I don't
for an individual if character is lost., everything is lost, so believe in not taking opportunities."
also for an administration if the ethics is lost, everything Even in a world overflowing with rags-to-riches
is lost. Neither efficiency nor loyalty could be substitute stories, there is only one Dhirubhai. A man who rose from
for high ethical standards. In India, through there is no the position of a petrol pump attendant to set up the
ethical code for public administrators, there are what are largest grass root oil refinery in the world; from bor-
called, the government servant’ conduct RULES. These rowing Rs 100 to buy clothes in order to go to Aden for a
rules lay down what constitutes misconduct for the public job, to set up the most modern textile mill in the country;
servants. It is not permitted, is also unethical conduct. from being the one who was asked to wait for his paltry
1. As per the passage, organizations bills outside the cabins of purchase officers and cashiers
(A) Differ in ethics in mills to become the greatest industrialist in the country
and from being a member of a lower middle class Indian
(B) Human behaviour in organizations include ethics
family to become a member of the Forbes' list of richest
(C) Ethics do not relate to normative aspects of men in the world.
administration
And in this long eventful journey spanning four-odd

ww
(D) None of these
2. Ethics is to an administration, what character is for
(A) An administrator (B) An official
decades, not only did he convert his fledging business he
started with about Rs 13,000 as seed capital for his first
textile plant in Naroda, near Ahmedabad into a Rs 60,000

w
(C) An individual (D) None of these

.Ea
3. Government Servants Conduct Rules are meant for
(A) Guiding the ethical conduct of government
crore powerhouse but also enlisted the support of four
million Indians weaned on socialism, in an adventure in
can-do capitalism, convincing them to ride the Reliance
story. Not for nothing that Dhirubhai is dubbed the high
servants
syE
(B) Guiding what constitutes misconduct for public
priest of the equity cult, that began in 1977 when Reliance
Industries first went public with 58,000 shareholders, and
continues even today under his two sons, Mukesh and
(C) Guiding what constitutes misconduct for govern-
ment servants
(D) None of these ngi Anil, with over four million shareholders. A fact that has
been acknowledged even by the Wharton School of Busi-
ness in 1998 while describing "his path-breaking contri-

4. The underlined word ‘manual’ in the context of the


given passage means nee
bution towards the concept of equity investing in India by
creating wealth and value for millions of shareholders."

rin
The financial acumen of a poor school teacher's son
(A) Hand operated (B) Physical can also be gauged from the fact that Reliance Industries
(C) Guide book (D) None of these did not pay a paisa in taxes on its corporate earnings till

Answers
g.n
1996, when Finance Minister P Chidambaram introduced
the Minimum Alternate Tax. His other innovations ranged
1. (C) 2. (C) 3. (C)
Passage – 8
4. (C)

Words–1206
e
from raising funds through the global depository route,
introducing convertible debentures and issuing l00-year
bonds in the US market.
Beginnings of A Business Genius
t
The story may be apocryphal but has never been
denied. During the 1950s, the Yemeni administration But for a true analysis of his genius, we have to
discovered that some denominations of its currency, the travel back a few decades and trace Dhirubhai's journey
Rial, were disappearing from the market. The adminis- from Aden, where he worked as an attendant in a Shell
tration traced the shortage to Aden, a port Yemen, and outlet and then returned to India in 1956 to begin his new
found to its surprise that a young man in his 20s had journey. With the money earned from the silver content
placed an unlimited buy order for the rial. The rial was a of Aden's coinage, he started a trading house called
solid silver coin. The young man simply bought the rials, Reliance Commercial Corporation in Mumbai, importing
melted them into silver ingots and sold them to bullion polyester fiber, Dhirubhai opened his first textile mill in
dealers in London at a much higher price because of the Naroda near Ahmedabad, in 1966 and concentrated on
exchange rate arbitrage. The name of the young man: building up his business, giving birth to the Vimal brand.
Dhirajlal Hirachand Ambani, or Dhirubhai Ambani. Here In 1982, India saw another first from Dhirubhai
were the early signs of the making of an iconic Ambani, the beginning of the concept of backward inte-
entrepreneur who was always looking for a business gration with Reliance Industries setting up a 10,00 million
opportunity and to make a quick buck on the side. Years tonne poly ester plant in Patalganga, some 80 km from
later, Dhirubhai told an interviewer: "The margins were Mumbai for polyester yarn and fiber intermediates and

CAT Complete Course | 533

Visit : www.EasyEngineering.net
Visit : www.EasyEngineering.net

finally to the basic raw material, oil, by setting up a 27 the skillful blend of head and heart and his legendary
million tonne refinery in Jamnagar, Gujarat, in 1998. He management techniques, which A.G. Krishnamurthy,
subsequently diversified into chemicals, gas, petro- founder Chairman and former MD of Mudra Communi-
chemicals, plastics, power and telecommunications. Yet cations, the advertising arm of Reliance, calls
his success always had a whiff of controversy to it. Rivals "Dhirubhaism."
alleged that his success had as much to do with his So, how will history judge Dhirubhai Ambani ? Will
business acumen as his ability to get official rules and he be talked in the same breath as Henry Ford or Bill
regulations tweaked to undercut his rivals and push his Gates when the economic history of India is written.
own business interests. His fight-to-the-finish battle with Possibly yes, because unlike others in the West he had to
the fiery proprietor of Indian Express, Ramnath Goenka, contend with red tapism, serious credit crunch, an abysmal
the war with industrialist Nusli Wadia, the allegations infrastructure and a poorly developed capital market to
against some Ambani staffers over a plot to murder build a formidable, globally competitive industrial empire.
Wadia and his travails during the VP Singh government, And that’s quite an achievement.
controversies over licensed capacities, export manipu-
1. Which of the following would be the most suitable
lation and share switching are part of the Ambani plot.
heading for the passage—
As Gita Piramal wrote in her book Business
(A) Entrepreneur – Apart

ww
Maharajas : "The corporate world is sharply divided
between those who feel he is a visionary and those who
consider him a manipulator." Mukesh Ambani, Chairman
(B) Vision of a Visionary
(C) Visionary of a different kind

w
of Reliance Industries, qualifies it : "What galled
Dhirubhai's critics was his success in outwitting them at

.Ea
every turn. Not only did he dream bigger but he also
found novel methods to realise them. One such way was
(D) Visionary Extraordinary
(E) Leading light
2. The following attributes made Dhirubhai Ambani a

syE
to ensure that even ordinary citizen shared in the wealth multibillionaire and iconic entrepreneur ‘EXCEPT’—
he created."
(A) Strong visionary
Thinking Big (B) Sound business acumen
So what was it that makes Dhirubhai a respected as
well as reviled figure? For R Ravi mohan, Managing ngi (C) Astute manipulator skills
(D) Willingness to take risks and flawless imple-
Director of Standard and Poor's South Asia operations,
his success could be attributed to his penchant for global
benchmarking, using state-of-the-art technology while nee mentation
(E) Ensuring others did not share in his creation of
cutting costs, pursuing ambitious goals, flawless imple-
mentation and a willingness to take calculated risks. "But
wealth

rin
3. The author writes that Dhirubhai Ambanis success
his biggest plus point was his ability to align the goal of
his enterprise to the needs of the market and pass on that
vision to every member of the Reliance family," he says.
implies that Dhirubhai—
g.n
always had a whiff of controversy to it . By this, he

His ambitious goals could leave others shell shocked.


For instance, in the early 1980s, when Dhirubhai told
Ravimohan, then a project officer with ICICI, that he
achieve his goals
e
(A) Was ruthless and would use any means to

(B) He was an unscrupulous person


(C) Dhirubhai had criminal tendencies
t
planned to sell polyester yarn at less than the price of
groundnut, his jaw nearly fell. But within just three to (D) He was petty mined and spiteful
four years, Dhirubhai brought down its prices from (E) All of the above
Rs. 180 to Rs 17 per Kg. Ravimohan says Dhirubhai was
4. Dhirubhai can be best summed up by which of the
responsible for the telecom revolution in India. When the
following positive points attributed to Dhirubhai
call rates were Rs10 per minute and consultants told him
Ambani—
to keep it at RS 6 a minute, Dhirubhai talked of bringing
it down to 40 paisa a minute. "It was Reliance Infocomm (A) His ability to converse with all levels of people
which slashed prices, and the whole industry followed," (B) The adroit mix of heart and mind
he adds. (C) His munificent nature
Soft Spots (D) His legendary management skills and techniques
But there are certain softer aspects of Dhirubhai's (E) All of the above
life, too. For instance, his ability to strike up conversation
with any body, his willingness to reward talented people Answers
and give them a free hand, his open-handed generosity, 1. (C) 2. (E) 3. (A) 4. (E)

534 | CAT Complete Course

Visit : www.EasyEngineering.net
Visit : www.EasyEngineering.net

Passage - 9 Questions
Words–511 The author’s primary objective of writing the passage
Globalisation, liberalization and free market are is to belie the popular belief that the free market helps
some of the most significant modern trends in economy. enhance development of industrial societies advocate that
Most economists in our country seem captivated by the price fixing is unavoidable and it is beneficial to the
spell of the free market. Consequently, nothing seems economy of any industrialized society explain the
good or normal that does not accord with the requirements methodology of fixing price to stabilize free market prove
of the free market. A price that is determined by the seller that price fixing and free market are compatible and
or, of for that matter, established by anyone other than the mutually beneficial to industrialized societies create
aggregate of consumers seems pernicious. awareness among the general public regarding combating
price by large firms
Accordingly, it requires a major act of will to think of
1. Considering the literal meaning and connotations of
price-fixing as both normal and having a valuable eco-
the words used in the passage the author’s attitude
nomic function. In fact price fixing is normal in all
towards “most economists” can best be described as
industrialized societies because the industrial system
itself provides, as an effortless consequence of its own (A) Derogatory and antagonistic

ww
development, the price fixing that it requires. Modern
industrial planning requires and rewards great size. Hence
a comparatively small number of large firms will be
(B) Impartial and unbiased
(C) Spiteful and envious
(D) Critical and condescending

w
competing for the same group of consumers. That each
large firm will act with consideration of its own needs

.Ea
and thus avoid selling its product for more than its
competitors charge is commonly recognized by advocates
(E) Indifferent
2. The author feels that price fixed by seller seems
pernicious because

syE
of free-market economic theories. But each firm will also
act with full consideration of the needs that it has in
common with the other large firms competing for the
(A) People don’t have faith in large firms
(B) People don’t want the Government to fix prices
same customer. Each large firm will thus avoid significant
stable price cutting, because price cutting will be preju-
dicial to the common interest in a stable demand for ngi (C) Most economists believe that price fixing should
be in accord with free market
(D) Most economists believe that no one group
products. Most economists do not see price fixing when it
occurs because they expect it to be brought about by a
number of explicit agreements among large firms, More-
nee should determine price
(E) People do not want to decide prices
over, those economists who argue that allowing the free-
market to operate without interference is the most efficient rin
3. Which of the following statements is definitely TRUE
in the context of the passage? Price fixing is—
method of establishing prices have not considered the
economics of non-socialist countries. Most of these eco-
nomies employ intentional price-fixing, usually in an g.n
(A) A profitable result of economic development
(B) An inevitable result of the industrial system

overt fashion. Formal price-fixing by cartel and informal


price-fixing by agreements covering the members of an
industry are common place. Was there something pecu-
liarly efficient about the free market and inefficient about
nized decisions
e
(C) The joint result of a number of carefully orga-

(D) A phenomenon uncommon to industrialized


societies
t
price-fixing, the countries that have avoided the first and (E) A result of joint venture of the Government and
used the second would have suffered drastically in their industry
economic development. There is no indication that they 4. According to the passage, price fixing in non-
have. socialistic countries is generally—
Socialist industry also works within a framework of (A) Intentional and widespread
controlled prices. In the early 1970’s the Soviet Union (B) Illegitimate but beneficial
began to give firms and industries some flexibility in (C) Conservative and inflexible
adjusting prices that a more informal evolution has (D) Legitimate and innovative
accorded the capitalist system. Economists in the USA
(E) Conservative and scarce
have hailed the change as a return to the free-market. But
the Soviet firms were not in favour of the price established 5. What was the result of the then Soviet Union’s
by a free market over which they exercised little influ- change in economic policy in the 1970’s?
ence; rather, Soviet firm acquired some power to fix (A) They showed greater profits
price. (B) They had less control over the free market

CAT Complete Course | 535

Visit : www.EasyEngineering.net
Visit : www.EasyEngineering.net

(C) They were able to adjust to techno advancement be these enterprises but under one or the other argument
(D) They acquired some authority to fix prices these remain unattended, may be it involves a tough task.
If these cannot be sold lock, stock and barrel asset
(E) They became more responsive to free market stripping is the only option. Obviously the government
6. The author’s primary concern seems to cannot realize good price from these assets but their dis-
(A) Summaries conflicting viewpoints posal will help stop the drain. If the assets are depreciated
or become obsolete, then there is no point in holding on
(B) Make people aware of recent discoveries
to them indefinitely and take to softer option of selling
(C) Criticize a point of view the vibrant and highly profit making organizations to
(D) Predict the probable result of a practice reduce the budgetary deficit. Non-performers exist in both
(E) Prepare a research proposal the sectors. Why condemn the public sector as whole?
Better option will be closure or privatization of loss-
7. Which of the following statements about the socialist making and non-viable units supporting PSU’s which
industry is/are false? could be turned around and become healthy and viable
1. It has works under certain price restrictions and providing autonomy to the board of PSU’s which are
2. It has no authority to determine price performing well and have potential to be globally com-
petitive be welcome. With public participation in the

ww
3. It hails the strategy of price fixing, as a major
deviation
(A) Only 1 is false (B) Only 2 is false
PCU’s there will be a good does of accountability in the
system. What need to be reviewed are some basic issues;
the priorities allocated to the enterprises. Selected for

w
(C) Only 3 is false
(E) 2 and 3 are false
(D) 1 and 2 are false

.Ea
Answers
disinvestments, a comprehensive road map delineating
the route, the modes and modalities timing and its con-
sequences. These basic issues require greater discussion
and participative decision making. In any event, the
1. (B)
7. (C)
2. (C) 3. (B)
syE
4. (A) 5. (E) 6. (A) disinvestments programme in respect of the closed and
non-revivable units is a must if the drain of further
resources is to be prevented.
Passage - 10
Words–550 ngi Let it be understood that PCU’s are a big repository
of value and it will take quite some time for privatisation
The public sector is at the cross roads ever since the
launch of economic reforms programme in India. The
pendulum has been swinging between survival and sur- nee
programme to materialize despite the desire to expedite
the process. Until then if a vacuums emerges attended by
uncertainty it will do a great harm to the investments,
render. It is the result of a confluence of several factors :
a shift in global economic environment, the emergence of desired now.
rin
which were made with such great dedication although

the market economy and myths surrounding the perfor-


mance of the public sector. So virulent has been onslaught
that it is becoming axiomatic that by the very concept, the g.n
The government has withdrawn a budgetary support
over the last decade. If some support is extended it is
largely directed to closed or losing enterprise, which have
public sector is inefficient and resource waster whereas
private enterprise is resource efficient.
The reform programme in India commenced with the
policy of restricting of the public participation with the
no fortune.
e
1. The basic issue(s) requiring greater discussion and
t
participative decision making regarding the disinvest-
ments programme is / are—
passage of time, the process of liberalization has shifted (A) The priorities allocated to the enterprises selected
to privatization in a disguised form couched as strategic for disinvestments
role. In the wake of the recent hot pursuit of the whole- (B) A comprehensive road map delineating the route
sale privatization programme a lively and poignant debate (C) The modes are modalities, timings and its con-
has emerged. It provides a golden opportunity to intro- sequence
spect and revisit the issue. (D) All of these
At the very outset, it must be made clear that in the 2. “The public sector is inefficient and resources waster
worldwide-liberalized economic environment and very whereas private enterprise is resource-efficient”. This
high stake of the state in most public undertakings dis- opinion is due to—
investments policy seeks to differentiate closed or
(A) A shift in global economic environment
bankrupt enterprise from the private sector–a fact delib-
erately overlooked by the champions of privatization. (B) The emergence of market economy
These undertakings need immediate attention. They are (C) The myths surrounding the performance of the
an unnecessary drain on the public exchequer. A high public sector.
priority area for the disinvestments programme ought to (D) All of these

536 | CAT Complete Course

Visit : www.EasyEngineering.net
Visit : www.EasyEngineering.net

3. The reform programme in India started with the 3. According to the writer, a person is impelled to write
policy of restructuring of PSUs has got shifted to a book, because—
(A) Liberalisation (B) Privatisation (A) he wishes to satisfy his ego
(C) Globalization (D) None of these (B) he has something nice and pleasing to say
(C) he is capable of expressing whatever he wants
4. What was made with great dedication earlier, but
to say.
now derided ?
(D) he has discovered something unique, true and
(A) Disinvestments
good which he must convey distinctly and
(B) Investments in PSUs musically
(C) Wholesale privatization 4. Which of the following is not implied in the passage ?
(D) Strategic plans (A) A writer is motivated to write a book if he dis-
5. According to the author, non performers exist in cerns a great truth
(A) Government (B) An author of a book generally gathers some
common truths and gives them a popular and
(B) Public sector pleasing expression

ww
(C) Private sector (C) A great writer is convinced that whatever he
(D) Public and private sectors says is not an echo or imitation of what others
have said
6. An appropriate title to the passage will be

w
(A) “Disinvestments of PSUs”

.Ea
(B) “Economic Reforms Programmes in India”
(C) “Liberalised Economic Environment”
(D) An eminent writer's message is conveyed
through plain unambiguous language
5. Which of the following is opposite in meaning to the
word 'manifest' given in the passage ?

syE
(D) “Non –performing Assets”

Answers
(A) Unclear
(C) Pure
(B) Dark
(D) Hard

1. (D) 2. (D) 3. (B) 4. (B)


Passage - 11
5. (D) 6. (A)
ngi 1. (D) 2. (C)
Answers
3. (D) 4. (B) 5. (A)

Words–115
A book is written, 'not to multiply the voice merely,
nee Passage - 12
Words–185
not to carry it merely but to perpetuate it. The author has
something to say which he perceives to be true and useful rin
The second thing we must do is to observe the
caution which John Mill has given to all who are

g.n
interested in the maintenance of democracy, namely not
or helpfully beautiful. So far he knows no one has said it,
to lay their liberties at the feet of even a great man, or to
so far as he knows no one else can say it. He is bound to
trust him with powers which enable him to subvert their
say it clearly and melodiously if he may; clearly at all
events. In the sum of his life, he finds this to be the thing
or group of things, manifest him; this, the piece of true
knowledge, or sight, which his share of sunshine and
earth has permitted him to seize. That is a book.
e
institutions. There is nothing wrong in being grateful to
great men who have rendered lifelong services to the
t
country. But there are limits to gratefulness. As has been
well said by the Irish patriot Daniel O’Connell, no man
can be grateful at the cost of his honor, no women can be
1. The opening sentence of the passage implies that the grateful at the cost of her `chastity, and no nation can be
aim of writing a book is to— grateful at the cost of her liberty. This caution is far more
(A) repeat the message it contains necessary in the case of India than in the case of any
other country. For in India, hero worshipping plays a part
(B) enable the author to express his ideas in writing
in our politics unequalled in magnitude by the part it
(C) preserve from extinction the message it contains plays in the politics of any other country of the world. In
(D) propagate the ideology of the author politics this hero worshipping is a sure road to
degradation and to eventual dictatorship.
2. Which of the following would be the most suitable
title for the passage ? 1. John Stuart Mill cautioned the lovers of democracy
against
(A) Contribution of an author
(A) Subversion of democracy
(B) Aim of writing a book (B) Entrusting powers to even great men
(C) Book–the source of true knowledge (C) To sacrifice their liberty
(D) Writers and their books (D) None of these

CAT Complete Course | 537

Visit : www.EasyEngineering.net
Visit : www.EasyEngineering.net

2. One should be grateful my wife(Meenakshi) and sons on a weekend to stock up


(A) To great men for the week ahead and treat ourselves," he says.
(B) To these who render service There's a growing up market population like Desh
Panday that likes a hypermarket but without the chaos
(C) To those who have long life and milling crowds. And retailers are increasingly tai-
(D) All the above loring hypermarkets to suit this audience. The K. Raheja
3. Gratefulness cannot be group launched HyperCity at Mumbai, Landmark group's
(A) Unlimited (B) Limited Max Hypermarkets launched Spar in Bangalore, Tata's
Trent now has three Star Bazaars in Ahmedabad, Mumbai
(C) Confined (D) None of these and Bangalore and the RPG group is turning their large
4. One should be grateful without losing stores into the Spencer's Hyper models at Gurgaon and
(A) Hon our (B) Chastity Mumbai.
(C) Liberty (D) None of these "There can be only so many players in the hard core
5. People pf India are value space. There's no end to price slashing and it's not
such a sustainable model in the long term," says Samar
(A) Honorable (B) Political Sheikhawat, VP (marketing), RPG Retail. "So it makes

ww
(C) Hero-worshippers (D) None of these
6. Hero worshipping and politics are linked in
(A) India (B) In day to day life
more sense for hypermarkets to start segmenting them-
selves."
These hypermarkets offer value and deals, but not the

w
(C) Daily routine
7. ‘Magnitude’ means
(A) Degree
(D) None of these

.Ea
(B) Intensity
jaw- dropping too- good- to- be- true variety. “When you
offer potatoes or onions at ridiculous prices, you get a lot
of customers only for those products. The shopfloor turns
chaotic and you’ve got the customer but only till your
(C) Power
8. Hero worshipping always syE
(D) None of these competitor offers a better price. So we purposely don 't
over-advertise our deals," reasons Vine Singh, MD of
Max Hypermarkets "We choose to harbour customer

ngi
(A) Degree (B) Exalts loyalty by offering reasonable value and back it up with
(C) Degenerates (D) None of these rich experience, variety, freshness and service.”

nee
9. Hero worshipping often leads to There's a marked difference in the ambience at these
(A) Dictatorship (B) Democracy hypermarkets targeting the SEC B to A audience when,
compared to the price-driven ones like Big Bazaar, Vishal
(C) End of freedom (D) None of these
10. The whole passage refers to
(A) Conditions for the success of democracy rin
Megamart and Reliance Hyper Mart. Live bakeries, delis
and cafes, wide variety in cheeses, cold cuts and meats
and well-stocked wine sections are common features.
(B) Democracy
(C) Democratic ideal g.n
Aisle spaces are wider, shelves are well lit, there's more
focus on visual merchandising, staff tends to be better

(D) None of these

1. (C) 2. (A)
Answers
3. (A) 4. (D) 5. (C) 6. (A)
e
trained and the merchandising not just the
Branded goods, but even the private labels–has an
upmarket look and feel and better variety.
t
There's greater attention to detail and these stores are
7. (A) 8. (B) 9. (A) 10. (A) better designed. For instance, there more cash tills to
reduce waiting time-Spar has 25 cash tills on the foods
Passage 13 floor alone.
Words–722 “By spacing out aisles, it may seem like we're
Hyper markets take the up market route compromising on our per square foot productivity.
Makrand Desh Panday (35) likes variety when he But at stores like ours, the per bill value tends to be
goes to a hyper market. "I'm a bit of an impulsive shopper much higher than price-driven hypermarkets," Points out
and it puts me off if there's not enough variety. Good Neeti Chopra, head of marketing, Trent.
ambience even for grocery shopping is essential since I The average value of the bill at premium hyper-
don't like being jostled around”, says the associate VP at markets is around Rs. 600 to Rs. 700 a figure, Which
an MNC bank. experts say; is at least 30% higher than the average per
Desh Panday’s pick in hypermarkets are Mumbai's bill, value of price driven hypermarket, excluding special
HyperCity and Bangalore's Spar. "From exotic veggies in offer days.
the fresh section to good deals in grocery and consumer These stores also ensure that they keep other sections
durables to variety in clothes, these are places to go with well stocked since they offer higher margins compared to

538 | CAT Complete Course

Visit : www.EasyEngineering.net
Visit : www.EasyEngineering.net

grocery - at least 15%-25% on garments, 10%-20% on 2. Hypermarkets don’t have chaos and milling
electronics, 10% on home decor and 30% on books and crowds.
music. Even in the foods section traditionally considered 3. Hyper markets offer too good to be true deals.
a low margin business at 5%-10% profits are boosted by
4. Aisle spaces wider well lit shelves.
offering imported, organic and exotic ranges.
“These players will create niche micro markets (A) 1 and 3 (B) 2 and 4
among each other. But with good ambience becoming (C) 3 and 4 (D) 1 and 2
common, it's service that will become the next key differ- (E) 1 and 4
entiator, " says R. Kannan, president director of retail
consultancy firm Ramms. "Wal Mart, for instance, has Answers
post offices, salons, movie ticket booking and pharmacies 1. (B) 2. (E) 3. (A) 4. (D)
attached to their stores.”
Passage–14
In India, services are picking up. RPG Retail has a
bill-paying facility and tie-up With laundrettes in some of Words–680
their stores. A couple of other retailers are planning to Old age, they say, is the most dreaded period of a
train shopfloor staff to take goods from the billing counter person’s lifetime. Illness and ailments become a part of

ww
to the customer's vehicle. Some are planning to tie-up
with multiplexes to offer booking counters and discounts
on a certain amount of purchase. Most are also looking to
daily routine. Depletion in the quality of vision and power
of eyes is one of the first thing that comes with growing
age. And cataract is the most widespread eye ailment

w
offer co-branded credit cards with special benefits.

.Ea
1. The most appropriate title for the passage would most
likely be—
(A) Hyper markets are booming
which is common among all elderly people, but is not just
restricted to them-youngsters too suffer from it.
Many technological breakthroughs have been made
in the way cataract surgery is performed and it’s getting

syE
(B) Hyper markets take the up market route
(C) Hyper models are the upmarket
better by the day. Anyone who suffers from this ailment
can chose what kind of surgery one wants to undergo and
the amount of money that they want to shell out.

ngi
(D) Hyper markets segmenting
The most recent breakthrough in the cataract surgery
(E) Hyper models emerge in the retail scenario
is the 1.8mm micro-incision surgery known as Stellaris-

nee
2. Pick the correct options from the following state- MICS. This high-end technology requires only a 1.8mm
ments as to what will eventually make the ultimate hole (or even smaller) in the cornea of the eye to pull out
difference— cataract and insert a new lens.
1. Good ambience.
2. Post office, saloons, movie ticketing bookings
rin
This technology has been launched in India at the
same time as it has been launched internationally, which
and pharmacies attached to their stores.
3. Taking goods from the billing counter to the
customers vehicle. g.n
is generally not the case. Being a Third-World country
and despite having some very talented scientists, India is
still on the backburner of scientific inventions and
4. Offering co-branded credit cards with special
benefits.
5. Efficient service will be the primary differen-
e
development. But Stellaris-MICS was the brainchild of
one of our eye surgeons from Chennia.
Dr. Amar Aggarwal, ophthalmic surgeon, thought the
bi-manual way of surgery was a little complex which
t
tiator.
prevented it from being popular and suggested that there
(A) 1 only (B) 1 and 5
should be a machine that can perform suction and
(C) 3 and 4 (D) 1 and 5 cleaning with the help of one tool and at the same time.
(E) 5 only This is how the idea for this outstanding surgical machine
3. The author’s attitude towards the trend can best be came into being, which was noticed by international com-
described as— panies like Bosch and Lomb who went on to manufacture
(A) Optimism (B) Pessimism it and train the ophthalmologists in its usage.
(C) Bigoted (D) Concern “The process of cataract surgery started with 12mm
incision in the cornea, which is 12mm to 13mm wide. So,
(E) Resignation
the cut went right across the eye because of which there
4. Accounting to the passage the primary difference were more chances of infection and a lot of post-surgery
between upmarket route and Hyper market routes precaution were needed “explains Dr Samir Sud, eye
are— surgeon.
1. Up markets don’t have enough variety whereas Phaco, the technology to minimize the length of the
Hyper markets do. hole in the cornea, then came as a welcome change. It is

CAT Complete Course | 539

Visit : www.EasyEngineering.net
Visit : www.EasyEngineering.net

used to break the cataract inside the eye and suck it out 3. The author introduces the following lines “With this
with the help of the needle. With this it became possible it became possible to bring down the size of the cut
to bring down the size of the cut to 3mm, but still the to 3mm, but still the fresh lens that is 5mm wide
fresh lens that is 5mm wide could not be inserted through could not be inserted through that hole” to—
that hole. So the incision had to be 5mm big (A) a logical build up to show case astounding
Then came the idea of making foldable lens which benefits of the latest cataract operation machine
could be implanted through the smaller hole and then (B) to show the progress of science break through
fitted in the eye. The 3mm hole became possible with the inventions in cataract surgery
help of the intraocular micro incision lens. The surgery (C) to authenticate the invention of Micro incision
became a boon for the patients. Post-surgery precautions lens
became a thing of the past. They could get back to their
(D) applaud Phaco, the technology to minimise the
normal lifestyle within a day. But better was in the offing.
length of the hole in the cornea
1·8 mm incision technique made it all the more
4. The author in the concluding lines infers that—
patient-friendly, wherein the patient can even drive back
to his destination only 15 minutes after the surgery has 1. India has some very talented scientists
been performed. “I did not feel any irritation or that my 2. Although India has talent but fails to excel on an

ww
eye had been tampered with. It’s only much better. I can
see everything properly with bright colours,” says 75
years old Mohan Raj, who now wants to get his second
international level
3. India still lags behind in scientific inventions
and developments

w
eye operated too, so that he has a perfect vision.

.Ea
“The reason for this quick recovery is that with the
help of a smaller incision, the corneal tissue is being
disrupted at all,” says Dr Sud. Also a large part of the
4. The 12 mm incision technique was time con-
suming and often painful
5. Technology will continue to leave us spell bound

syE
reasons for an improved vision depends on the Micro-
Incision lens which is designed in a way the profession
camera lens is. It has a single point focus and is aberration
Chose the correct answer or combination :
(A) 1 and 4 (B) 3 and 5

ngi
(C) 2 and 4 (D) 5
free, aspheric lens. The lens enables good colours, con-
(E) 1 and 5
trasts and hues improving the quality of vision, after the
cataract almost blinds the patient.
Technology always leaves you wondering whether
any thing better can ever come up to make the lives easier. nee
1. (D) 2. (D)
Answers
3. (B)
Passage–15
4. (D)

And it always has a surprise in store.


1. The most appropriate title for the passage would be— rin Words–150

(A) The cut gets tinier


(B) Perfect vision is only a small a cut away g.n
American use archaisms such as the preservation of
‘gotten’ as the past participle of “get” ‘fall’ for autumn,
‘aim to’ for the English ‘aim at’ and faucet for tap. The
(C) Cut and drive home brighter
(D) A cut above the rest
2. The authors theme song is—
e
word, politician, is used in a disparaging sense in
America. Solicitor in America means a canvasser or
visiting agent or beggar and the word clerk indicates a
shop assistant, usually a female.
t
1. Technological breakthrough have helped make a The use of the word cut meant for ‘education’ was
tedious and often painful operations, painless originally American but became acceptable in good
while cutting down the time involved to a great English largely because of the 1931 financial slump in
extent. Britain.
2. The invention of the suck and clean surgical The Americans ‘visit with’ friends, in Britain one
machine visits them. In Britain the word, welcome, finds use in this
3. Technology and scientific inventions will way ‘anyone who cares to come will be welcome’ or ‘the
improve still further and make our lives easier. guests were welcome by her. In the United States, anyone
begging another person’s pardon may receive the reply,
4. The invention of Micro incision lens “you’re welcome” which in Britain would be considered
Chose the correct answer or combination— a sign of ill breeding.
(A) 3 only (B) 3 and 4 1. The American use ……… for autumn.
(C) 2 and 3 (D) 1 and 2 (A) Past participle of get
(E) 1 and 4 (B) Gotten

540 | CAT Complete Course

Visit : www.EasyEngineering.net
Visit : www.EasyEngineering.net

(C) Fall late seventies and the eighties and particularly the
(D) None of these relaxation of some of the restrictive provision of MRTP
Act. and FERA, brought about a qualitative change in the
2. Americans use faucet for— mergers and amalgamations of companies. Even so, the
(A) Tap (B) Aim incentive to grow was almost nonexistent and in fact
(C) Gotten (D) None of these some companies preferred to “demerge” by splitting one
3. The word ‘politician’ is used in— company into two or more so as to escape form the harsh
provision of the MRTP Act.
(A) Bad sense in America
The fast pace of liberalization since July 1991 and
(B) Hateful sense
the time-bound programme of structural reforms under
(C) Criticising sense form the IMF and the World Bank have shaken the Indian
(D) None of these industry form a slumber by exposing it to internal as well
4. The shop assistant in America is— as international competition. Not surprisingly, the
(A) Usually female (B) Clerk pressure is building up on every enterprise to mordernise
and expand to cut costs. Gone are the days of the license
(C) Good person (D) None of these
and permit raj, high import duties and the prosperity

ww
5. ‘Visit with’ the friends, in America, mean guaranteed by a “ sellers’ market” with the rising threat
(A) Visit them (B) Go with them of competitions and the “seller market” giving way to a
(C) See them (D) None of these “buyer market” in a large number of industries, the

is—w
6. In America “You’are welcome” is said when a person

.Ea
(A) Begging another person’s pardon
compulsion to look for economies of scale in production
and cutting down the selling cost is increasing.
Simultaneously, the virtual scrapping of the MRTP
provision and relaxation in FER have removed the

syE
(B) When a person is received disincentive to grow. Hence mergers, amalgamations and
(C) When a person is insulted take-over have assumed greater importance, mergers and
(D) None of these acquisitions have now come to represent a short cut for

ngi
companies to achieve accelerated growth. This is the
7. The meaning of ‘ill breeding’ is— trend world over and India cannot remain an exception as
(A) Bad manners (b) Uncultured it moves towards globalization.
(C) Uncivilized (d) None of these
8. Which of the following sentences use the word ‘sign’ nee
1. What was the motive of some companies to resort to
demerger ?

rin
correctly— (A) To boost their productivity and profitability
(A) Please sign this paper (B) To bypass the unfavorable legal provision
(B) This is a sure sign of success
(C) He called him for sign
(D) None of these g.n
(C) To bring about qualitative changes
(D) To increase the number of companies under their

1. (C)
7. (B)
2. (A)
Answers
3. (A)
8. (A), (B)
4. (A) 5. (A) 6. (A)
(E) None of these

market” means—
e
2. The phrase “sellers’ market giving way to Buyers’
t
(A) Increase in production is proportionate to the
Passage–16 demand
Words–354 (B) Increase in demand is disproportionate higher
than the supply
The surge witnessed in mergers, amalgamations and
take-over of companies during the past few years is (C) Market is financially in favor of consumers as
indicative of the shape of thing to come. While these compared to in the past
concepts are not new and were recognised even in the (D) Industry’s profit margin is enhanced
Companies Act of 1913 compulsion have undergone a (E) None of these
dramatic change. In the past mergers and acquisition were
used largely as an instrument for revival of sick units or 3. Which of the following is true about Indian Industry’s
for obtaining tax benefits. It was not uncommon for a scenario prior to July 1991 ?
business house to merge a sick company with a profit (A) There had been pressures form the World Bank
making one claim tax benefits. and the IMF
The objective was not necessarily to achieve faster (B) It was exposed to severe competition on national
growth. The liberalization process witnessed during the and international fronts

CAT Complete Course | 541

Visit : www.EasyEngineering.net
Visit : www.EasyEngineering.net

(C) Liberalization process was at its peak 9. The term “demerge” as used in the passage means
(D) Structural reform programme were planned and (A) Formulation of two or more companies out of
implemented an existing one
(E) None of these (B) Re-union of companies which had split up of
one company
4. Which of the following is not an outcome of Indian
Industry’s exposure to competition ? (C) Separation of two or more companies which had
merged into one
(A) Switching over to expansion
(D) Renaming a company to claim tax benefits
(B) Reduction in selling cost
(E) None of these
(C) Adopting new technologies
(D) Guarantee for Profits and Prosperity 10. Which of the following inferences can be drawn
form the passage?
(E) Need for obtaining licenses and permits
(A) In the eighties, the change in FERA and MRTP
5. For which of the following were the amalgamations provision the necessary impetus for growth
largely used in the past ?
(B) FERA provisions were counter-productive to
(A) Saving on Taxes payable to the Government

ww
(B) Forcing the Government to adopt liberalization
process
industrial growth earlier
(C) It is only the external financing agencies’ pres-
sure that has compelled Indian industry to adopt

w
(C) To achieve accelerated growth

units
(E) None of these .Ea
(D) Overcoming the provision of revival of sick
the present structure
(D) Most of the business house were not inclined to
using the merger-tactics for revival of sick units
(E) None of these

syE
6. The mergers of companies in the past and present
doffer in respect of which of the following ?
11. Which of the following groups of statements is true
in the context of the passage ?
(A) Tax benefits
(C) Objectives
(B) Pace of growth
(D) Modalities
ngi Statement (i) : FERA and MRTP provisions were
not conductive to industrial growth earlier.
Statement (ii) : Unlike in the rest of the world, in
(E) Profit percentage
7. Which of the following is true about Government’s
on import duties ?
nee India merge of companies is a way to achieve
accelerated growth.

(A) Import duty is lowered in order encourage


imports rin
Statement (iii) : The Indian industry shall have to
find out profit sources other than form customers
pocket.
(B) Import duty is raised in order to discourage
buying of foreign good g.n
(A) Only (i) and (iii) are correct
(B) All the three statements are correct
(C) Import duty is now lowered to encourage healthy
competition
(D) Import duty is raised in order to earn substantial
e
(C) Only (i) and (ii) are correct
(D) Only (ii) and (iii) are correct
(E) None of these three statements is correct
t
revenue
Answers
(E) Import duty is lowered in order to enable Indian
industrialists to adopt foreign technology 1. (B) 2. (C) 3. (E) 4. (E) 5. (A) 6. (B)
7. (C) 8. (D) 9. (A) 10. (C) 11. (A)
8. The changing scenario as described in the passage is
most likely to result into— Passage–17
(A) Exorbitant profit margin to industry despite Words–1199
financial respite to consumers A lot of boom, a little gloom, but no doom. That
(B) Reasonable profit margin to industry and perhaps sums up the trajectory of India's growth story so
marginally higher cost to buyers far, and which is also a fair commentary on what's in store
in the coming days. For, nothing-not even higher interest
(C) Marginal losses to industry and considerable
rates, inflationary pressures and an appreciating rupee-
benefits to buyers
appear potent enough to arrest the economy's growth,
(D) Adequate profit margin to industry despite particularly investment growth of the corporate sector.
lower prices Hence, by all reckoning, economic expansion should
Shift of Govt. taxes form industry to buyers touch 8 % in the current financial year.

542 | CAT Complete Course

Visit : www.EasyEngineering.net
Visit : www.EasyEngineering.net

The growth momentum could be maintained beyond Opening The Flood Gates
2007-08. In any case, a lower than 7.5 % growth is un-
The higher interest rate in India has attracted large
likely, even if the agriculture sector's performance dis-
inflows over the last few months. With RBI ceasing to
appoints and corporate investments slacken.
buy dollars, given its attack on inflation, the rupee
While the Reserve Bank's monetary tightening, aimed appreciated about 6 % in April. While this has given the
at dousing inflation, and currency appreciation, triggered jitters to exporters and companies with high overseas
fears of an economic slowdown, confidence is intact. income, the appreciation is unlikely to hurt growth.
While export earnings are taking a hit, exports account “Recent history indicates that exports have grown even in
for just about 12 % of the GDP. So while earnings growth the years the rupee has appreciated. There s not enough
of some companies may come under pressure, there is evidence to suggest the strengthening rupee would hurt
little concern about a slowdown in demand. In contrast growth, “point out vyas.
the appreciating rupee may benefit companies that intend
Holland adds that income growth of exporters and
to import equipment for capacity expansion.
software companies may remain subdued, given the
Belt Tightening-Just that translation effect on earnings. According to Ashima
The monetary tightening over the last one year has Goyal, Professor at Indira Gandhi Institute of Develop-
ment Research, “There has been no real slump in exports

ww
not hampered demand for investment credit. Non-food
credit grew by 28% in 2006-07 against 31.8% in 2005- and most IT companies have shown better-than-expected
06. The Reserve Bank of India hopes to moderate that results. An appreciation of the rupee means that there will
be greater focus on the domestic market.” Also, a stronger

w
growth to 24-25 % in the current financial year, even as it
targets about 8.5% GDP. rupee would aid the country’s infrastructure investment

.Ea
"Interest rate hike has not impacted investment
growth as yet," points out Manesh Vyas, Managing
needs, Holland notes. Companies and project developers
can access equipment and technology at a lower cost, at
the same time capacity building would not stoke inflation.

syE
Director and Chief Executive Officer of Centre for Moni-
toring Indian Economy. He reckons that another round of
increase in the interest rate may not have a negative
There are other benefits too at the same time. The oil
import bill would be lower, as a stronger currency would

ngi
impact on investment growth, as the consumer demand is partly offset the increase in oil prices. Consequentially,
very strong. “The recent hike in the cash reserve ratio is a the government would have to spend less on subsidizing
lot more severe than all the measures taken by RBI earlier, diesel, Kerosene and liquefied petroleum gas.
and even these don't seem to be hurting credit growth
rate," Vyas explains. So what are the chances of interest
rates rising further? "Interest rate are close to peaking,"
nee However, there is no saying how oil prices will move
in the months ahead. Much would depends on the geopo-
litical scenario. Holland does not expect any sharp move-
declares DSP Andrew Holland, Merrill Lynch Managing
Director. rin
ment in either direction, and estimates crude prices to be
in the range of $ 55 to $ 65 a barrel. S & P’s wyss puts it
But he cautions that inflation remains a risk and thus
another increase of perhaps 25 bps in interest rates seems g.n
in a narrower band of $ 60-65 a barrel. Nymex futures
market forecasts oil price would rise to $ 65.7 a barrel by
probable. A Deutsch Bank report on India published mid
April too suggests that policy rates of RBI were near
peaking. Holland believes globally, inflation would begin
to cool off leading to a fall in interest rates. "We should
July 2007.

e
Another area of concern is a possible slowdown in
housing, caused by the rise in the cost of funds coupled
with an escalation of property prices. but the housing t
story remains strong, given the employment growth in the
see interest rate start to come down in the second half of
services sector. Moreover, with speculation in the sector
the year in the US and Europe." In contrast, Standard &
declining, buyers may find prices more realistic. Demand
Poor's Chief Economist David Wyss is less optimistic for housing loans remains high despite recent rate hikes,
about an easing of interest rates. "I don't expect any points out Vyas. A slowdown in housing could have
significant rate cuts this year. The US may cut by year- dampened demand for cement and steel, but given the
end, but Europe and Japan are still tightening." Wyss rising incomes, the fear may be far fetched.
reckons the rate cuts may begin in 2010.
The Party Poopers
The monetary stance of the US Federal Reserve
The growth story is, however, not without any risk.
normally serves as a cue for other central banks. The Fed For one, political indecisiveness has the potential to
he held interest rates steady since Jun 2006 amid concerns hamper growth. As Kristin Lindow, Vice President and
of a slowing economy. In contrast, many central banks, senior credit officer of Moody’s investor service, puts it :
including the Bank of England the European Central Bank “India’s continuing inflationary pressures and its widening
and the Bank of Japan, have hiked rates over the last 10 external trade resemblances are emblematic of two of the
months. economy’s capacity constraints remain unresolved amidst

CAT Complete Course | 543

Visit : www.EasyEngineering.net
Visit : www.EasyEngineering.net

an obstructive political climate. The second issue relates Answers


to the insufficiency of fiscal adjustment, exacerbated
recently by an overly accommodative fiscal stance that 1. (E) 2. (E) 3. (C) 4. (C)
puts the onus of inflation control squarely on the Reserve Passage–18
Bank.” Infrastructure deficit and slow progress on
Words–925
addressing the problem would be another issue. “ The
rate of growth was running way ahead of its potential,” Our parliamentary system has created a unique breed
states Yes bank’s Chief Economist Shubhada Rao. Agri- of legislator, largely unqualified to legislate, who has
culture output would remain a concern, given that the sought election in order to wield (Or influence) executive
sector supports nearly 70% of the population but contri- power. It has produced governments more skilled at
butes less than 20% to GDP. However, companies are politics than at policy or performance. It has distorted the
going ahead with their capital expenditure plans, a reflec- voting preferences of an electorate that knows which
tion that investment growth is strong. “with the invest- individuals it wants but not necessarily which policies.
ment to GDP ratio hovering at 33%, sustaining a 9% It has permitted parties that are shifting alliances of
growth is not an issue,” explain Goyal. individuals rather than vehicles of coherent sets of ideas.
Supplementing the domestic sources of investment is It has forced governments to concentrate not on governing
but on staying in office, and obliged them to cater to the

ww
the strong inflow of foreign direct investments. “we
should get about $15 billion in foreign direct investment
this year, and about $20 billion next year,” notes Holland.
lowest common denominator of their coalitions. It is time
for a change.

w
Further, the retail boom would give a thrust to growth.
Also, the government’s expenditure is unlikely to decel-

.Ea
erate as elections are less than two years away and the
popularity of the PA- led government is waning. That
The fact that the principal reason for entering parlia-
ment is to attain governmental office poses two specific
problems. First, it limits executive posts to those who are
elect able rather than to those who are able. The prime

syE
would provide some impetus to growth. minister cannot appoint a cabinet of his choice; he has to
1. The most appropriate title for the passage would cater to the wishes of the political leaders of 20 parties.
most likely be: Second, it puts a premium on defections and horse-

ngi
trading. The Anti-Defection Law of 1984 was necessary
(A) Economy surge
because in many states (and, after 1979, at the Centre)
(B) Rising higher and swifter parliamentary floor crossing had become a popular
(C) Boom ,boom on the rise
(D) The sky’s the limit
(E) No slowdown in sight nee
pastime, with lakhs of rupees, and many ministerial posts,
changing hands. Now, musical chairs is an organized
sport, with “party splits” instead of defections, and for
2. According to the author which of the following rea-
sons are no cause for alarm to the ever rising boom— rin
much the same motives. I shudder to think of what will
happen after the next elections produce a parliament of 40

(A) High interest rates


(B) The appreciating rupee g.n
odd parties jostling to see which permutation of their
numbers will get them the best rewards.

e
The case for a presidential system of either the
(C) Pressures of inflation
(D) Shortfall in FDI and FII’s
(E) All of the above
3. According to the author what could cause the fall in
French or the American style has, in my view, never been
clearer. The French version, by combining presidential
rule with a parliamentary government headed by a prime t
minister, is superficially more attractive, sine it resembles
our own system, except for reversing the balance of
interest rates in the near future as started by
power between the president and the council of ministers.
Holland—
This is what the Sri Lankans opted for when they
(A) Depreciation of the rupee jettisoned the British model.
(B) A surge in FDI’s But, given India's fragmented party system, the
(C) US federal interest rates have been steady which prospects for parliamentary chaos distracting the elected
would signal global inflation rates will cool off president are considerable. An American or Latin Ameri-
(D) Bank of Japan will raise interest rates further can model, with a president serving both as head of state
(E) None of the above and head of government, might better evade the problems
we have experienced with political factionalism. A
4. The tone of the author in the passage can best be
directly-elected chief executive in New Delhi, instead of
described as—
being vulnerable to the shifting sands of coalition,
(A) Canteous (B) Pessinistic support politics, would have stability of tenure free from
(C) Encouraging (D) Didactic legislative whim, be able to appoint a cabinet of talents,
(E) Pragnatic and above all, be able to devote his or her energies to

544 | CAT Complete Course

Visit : www.EasyEngineering.net
Visit : www.EasyEngineering.net

governance, and not just to government. The Indian voter 1. According to the author which of the followings
will be able to vote directly for the individual he or she statements can be deemed to be true ?
wants to be ruled by; and the president will truly be able (A) India’s parliamentary system require no change
to claim to speak for a majority of Indians rather than a
(B) India’s parliamentary system needs to switch
majority of MPs. At the end of a fixed period of time -let
over to a different system of legislature
us say five years- the public would be able to judge the
individual on performance in improving the lives of Indi- (C) The Parliamentary system is perfect. However
ans, rather than on political skill at keeping a government its implantation lacks either honest or unquali-
in office. It is a compelling case. fied. (d) The French or American Presidential
system is clearer for India.
Why; then, do the arguments for a presidential system
get such short shrift from our political class ? At the most (E) All of the above
basic level, our parliamentarians' fondness for the par- 2. The author suggests that—
liamentary system rests on familiarity: this is the system (A) The French version of governance is suitable for
they know. They are comfortable with it, they know how India.
to make it work for themselves, they have polished the (B) India counter part legislature in England is a
skills required to triumph in it. Most non politicians in perfect model to ape.

ww
India would see this as a disqualification, rather than as a
recommendation for a decaying status quo.
(C) An American or Latin American model might
serve us better

w
The more serious argument advanced by liberal
democrats is that the presidential system carries with it

.Ea
the risk of dictatorship. They conjure up the image of an
imperious president, immune to parliamentary defeat and
(D) French or American system suits India best
(E) None of the above
3. The attitude of the passage is—

syE
impervious to public opinion, ruling the country by fiat.
Of course, it does not help that Mrs Gandhi, during the
Emergency, contemplated abandoning the parliamentary
(A) Critical of India’s legislation
(B) Appreciative of India’s legislation
(C) Cynical of India’s legislation
system for a modified form of Gaullism, thereby, dis-
crediting the idea of presidential government in ngi (D) Dissatisfied with India’s legislation
(E) Confused with India’s legislation
many democratic Indian eye: Mrs Gandhi is herself
the best answer to such fears: she demonstrated with her
Emergency rule that even a parliamentary system can be nee
4. The loopholes and shortcoming of India’s parlia-
mentary system is due to—
distorted to permit autocratic rule Dictatorship is not the
result of a particular type of governmental system.
rin
(A) The ruling party having to cater to the demands
of its coalition partners
In any case, to offset the temptation for a national
president to become all-powerful, and to give real preference
g.n
(B) The P.M. cannot appoint a cabinet of his

e
substance to the decentralisation essential for a country of (C) Floor crossing and horse trading have immense
India's size, an executive chief minister or governor
should also be directly elected in each of the states. The
case for such a system in the states is even stronger than
in the Centre. Those who reject a presidential system on
the grounds that it might lead to dictatorship may be
value
(D) Most part of constitution builts by British
(E) All of the above
t
Answers
assured that the powers of the president would thus, be
1. (B) 2. (E) 3. (D) 4. (A and C)
balanced by those of the directly elected chief executives
in the states. Passage–19
Democracy, as i have argued elsewhere, is vital for Words–924
India's survival: our chronic pluralism is a basic element
Happiness is a Warm Sun
of what we are. Yes, democracy is an end in itself, and
we are right to be proud of it. But few Indians are proud I watched the rerun of a BBC documentary, called
of the kind of politics our democracy has inflicted upon The Happiness Formula, over the week end. It delved into
us. With the needs and challenges of one-sixth of scientific research on the subjects and noted how close
humanity before our leaders, we must have a democracy we were actually to quantifying happiness in individuals I
that, froze in anxiety as the show ended.
delivers progress to our people. Changing to a Would doctors be telling us soon how happy we
presidential system is the best way of ensuring, a democ- were? According to the documentary quite possibly. And
racy that works. poof would go a chunk of our individual right to examine

CAT Complete Course | 545

Visit : www.EasyEngineering.net
Visit : www.EasyEngineering.net

and decide how exactly we felt on a particular day. Our You might assume we in India are a pretty happy
moods would fit into a formula devised by scientists, who bunch going by the delirious, musical acrobatics that
would blithely define what it meant to be happy, sad or inundate our TV screens day in and night out or by the
otherwise. sheer number of supposedly happy festivals we celebrate
Mind you, the scientists are doing astounding work. year in and year out. Well, India fingers a lowly 125th on
The documentary showed a series of baby faces in various the index. If that makes you sad, you can feel happy by
stages of emotion some were radiating delightfully looking at Pakistan, which is 166th ,just step above
toothless grains, others were puckered in distress. Each of Russia. But then again, China is higher at 82nd.
those moods had been closely scanned by electrodes to Interestingly, the countries that fare well on the index
produce changing colours in various parts of their brains. are almost all well–off. Being healthy and wealthy boosts
The happy babies showed distinctly different, and national moods. But does it make people wise? If being
apparently quantifiable, blobs of blue in the area of the healthy and wealthy made nations wise, the Iraq war may
brain where happy moods reside you can are or so the not have happened capitalism however is a winner all the
scientists how happy you are or so the scientist claimed nations listed high on the index are market economics.
adding they were on the job of precisely measuring Democracy, curiously does not seem to feature in the
happiness. global happiness equation, much to the chagrin of us

ww
Well, good luck, men in white. You are not going to
crowd my skull with electrodes any day soon to tell my
mood. And you are not going to define for me what it
liberals who value freedom - of expression, in particular
as a key ingredient of any national happiness formula.
Although most national in the top 50 in the list are

w
mean to be happy at a particular point of time, on a
particular day, in a particular situation and in a particular

.Ea
place, I will decide for myself thank you; or may be my
mercurial temperament will. And, will you measure me
democracies countries like Saudi Arabia, Brunei and
kuwati are also up there, underscoring the wealth factor
but raising doubt about popular yearning for freedom.
The one intriguing entry in the list, at number 8, is

syE
after stuffing me full of chocolates or will you deprive me
of the one human invention that can make me hit instant
high? Besides, do you know we sometimes smile when
the happy state of Bhutan. It is neither wealthy nor is it a
market economy. But it is the only country which
officially measures its well being not by GDP alone but
we’re actually fuming inside? I do often it’s stressful but
controlling one’s temper with a smile is a wise forward
deal on long-term happiness. ngi also by a national happiness index. Bhutan’s culture
minister explained on the BBC show people were kept
I am happy at new year’s eve parties, positively jolly,
as such occasions demand but to raise another problem
the degree of happiness can very from year to year. And,
nee
happy by strict government supervision of what they
could see on TV and what outside influence would be
allowed into the landlocked kingdom. Clearly, if you
if you measure my mood in the middle of the festive
hours, you might find the happiness area colourful, but rin
don’t let people know what excitement lies out there in
the wide world, ignorance can indeed be bliss.
with a marked decrease in intensity as evening wears into
the late night. The next morning could show a brainful of
g.n
I, on the other hand, would rather be sad knowing
thing out there under the warm sun then be happy in
remorse at having indulged too much for too long. I crawl
out of bed swearing never again until the next party that
is. So, what do you measure, and exactly when and
where, if you want to declare me happy or unhappy ?
plato’s cave.
e
1. The author makes which of the following critical of
scientists research into qualifying happiness ? t
But common sense arguments never stopped 1. That a huge amount of one’s individual also right
scientists, Neurologists, Psychiatrists and geneticists will to decide on how one feels would be usurped.
carry on turning common sense on its head as indeed they
2. Neither wealth nor economy play an important
must if they are true scientists. And now political scientist
role in the happiness quotient.
and social psychologist have joined the band they have
gone one gone beyond to classify whole nations in a 3. Controlling one’s anger by masking it with an
global happiness index appearance of approval is wise on the term
Researchers at Britain’s university of Leicester used happiness scale.
a battery of statistical data and surveyed 80,000 people 4. The degree of happiness various from time to
worldwide in 2006 to chart the state of happiness in 178 time. So, when, where and what does one
countries. The happiest nations ? Denmark and Switzer- measure to slot one as happy or unhappy
land, which tied for first place. The most unhappy ? 5. Happiness cannot be put to a scale.
Zimbawe and Burundi. The united states came in 23rd in
(A) 1 and 2 (B) 3 and 4
the list, even though it was the first republican democracy
in the world to incorporate the pursuit of happiness in its (C) 1 and 4 (D) 1 and 5
constitution as a worthy national goal. (D) 5 and 3

546 | CAT Complete Course

Visit : www.EasyEngineering.net
Visit : www.EasyEngineering.net

2. The author state all of the following argument against These governmental and other approaches have had
scientific research to quality happiness in individuals mixed results. For example SEWA’s (Self Employed
EXCEPT— Women’s Association) and WWF’s (Working Women’s
(A) An individual’s right to decide one’s feelings Forum) approach of using bank credit to organize self-
can be dented employed women has worked well. So also women’s
literacy programmes in Kerala. On the other hand, some
(B) One cannot enjoy festivals as such occasions
of the legislated reforms like dowry prohibition have not
demands
worked well. The issue of whether women should be
(C) The degree of happiness varies from year to year identified as separate participants in the development
(D) Democracy does not feature in the global process is also controversial.
equation of happiness In this context, it is disappointing to read in one of
(E) Happiness can be defined by scientists the reports that one of the government’s targets for 1991-
3. The author attitude can be best described as one of: 95 ought to be foster “an ethos of caring in the com-
(A) Approval (B) Cynicism munity, not to let a child go to bed hungry, be subjected
to a preventable disease or remain without learning oppor-
(C) Anger (D) Denial
tunity this could mark the beginning of a social process
(E) Criticism

ww
4. The most appropriate title for the passage would
undoubtedly be—
towards a more humane order.” Is the report saying then
the Indians do not love their children as a matter of policy
makers. It is very unfortunate that these reports have

w
(A) Transferred happiness

.Ea
(B) Cloned happiness pines
(C) Programmed happiness
(D) Happiness on call
become superficial and does not delve sufficiently deep
into the problems.
1. Which of the following is correct in the context of
the passage in regard to development schemes ?

(E) Happiness a warm sun.


AnswerssyE (A) All people are the opinion that women should
be given special status
(B) Some people h o l d the opinion t h a t women
1. (D) 2. (B) 3. (E)
Passage–20
4. (B)
ngi should not be given special status
(C) No one feels that women should be given

nee
special status
Words–385 (D) There is no controversy regarding women being
A recent report titled ‘Women and Children in India’ given special status
and a nearly simultaneous report on ‘Indian Women-their
health and economic productivity’ highlight the interna-
(E) None of these
rin
2. Which of the following is specifically put forward as
tional importance being given to the problems of the
progress of Indian women.
g.n
a reason for the unequal sex ratio in the population ?
(A) Natural causes

e
Unfortunately, both the report fail to either convey a
(B) Economic development
fresh understanding of the issue or to suggest innovative
and workable approaches for women’s development. The
former is because the reports restrict themselves to old
statistics. We are told for example, that the ratio of
women to men remains below its natural level ant that it
(C) Literacy level
(D) Socio economic programmes
(E) Poverty alleviation programme
t
3. Which of the following is not put forward by the
is related to other parameters, such as the level of literacy author of the passage as a defect of the reports ?
and the availability of primary health care.
(A) They highlight the problems of Indian women
The failure to suggest a fresh approach is less under-
(B) They contain outdated statistical data
standable given that a variety of approaches to women’s
development has been tried out all over the world. In (C) The reports have not suggested any new
India, the state has made a strong political and economic measures
commitment to women. In 1990 the National Com- (D) Some of them suggested new measures
mission of Women was established. In 1989 two major (E) The report are not analytical
policy documents were released. Ever since the Sixth- 4. Which of the following is true in the context of the
Five plan, there has been a special section on socio- passage ?
economic programmes for women. Poverty alleviation
programmes have a 30 percent target for women. Special (A) Culturally, Indians do not love their children
programmes like Development of Women and Children (B) All developmental programmes and schemes
in Rural Areas have also been started. are generally a success

CAT Complete Course | 547

Visit : www.EasyEngineering.net
Visit : www.EasyEngineering.net

(C) Indian women do not manage their finances (C) No social organization were involved in this
well (D) Reason not mentioned in the passage
(D) Women literacy programme have failed in India (E) Literacy rate among women is poor
(E) Some developmental programme have mixed 11. Which of the following would indicate the same
result all over India meaning as the phrase “go to bed hungry” as it has
5. Which of the following has not been mentioned as been used in the passage ?
necessary, in one of the reports , to start new social (A) Malnutrition (B) Insomnia
order ? (C) Famine (D) Undernourishment
(A) Prevent hunger of children (E) Unsatisfied
(B) Prevent disease in children 12. Which of the following would correctly reflect the
(C) Increase educational opportunities position of the author of the passage to the two report
(D) Community care of children mentioned in the first paragraph ?
(E) All of these have been mentioned in the report (A) Evaluative (B) Critical
6. Why does the author say that the reports are unable (C) Neutral (D) Praiseful

ww
to convey a ‘fresh understanding of the issues’ ?
(A) The problem are dealt with superficially
(E) Appreciative
Answers

w
(B) Old data are used in the reports
1. (B) 2. (C) 3. (A) 4. (E) 5. (C) 6. (B)
(C) Some of the development schemes are not dealt
with
.Ea
(D) Indian problems cannot be understood by
7. (A) 8. (C) 9. (D) 10. (D)
Passage–21
11. (E) 12. (B)

foreigners
(E) None of these
syE Words–249
It is strange that, according to his position in life, an

ngi
7. Which of the following is conveying the same extravagant man is admired or despised. A successful
meaning as the ‘target’ as is used in the passage ? business man does nothing to increase his popularity by
(A) Aim (B) Margin being careful with his money. He is expected to display
(C) Attempt
(E) Quota
(D) Schemes
nee
his success, to have a smart car, an expensive life, and to
be lavish with his hospitality. If he is not so, he is
considered mean, and his reputation in business may even
8. Which of the following is conveying the same mean-
ing as the word ‘mark’ as is used in the passage ? rin
suffer in consequence. The paradox remains that if he had
not been careful with his money in the first place, he
(A) Symbol
(C) Signal
(B) Sign
(D) Notice g.n
would never have achieved his present wealth.
Among the low-income group, a different set of

(E) Start
9. Which of the following is false according to the
passage ?
e
values exists. The young clerk, who makes his wife a

is condemned as extravagant. Carefulness with money to


t
present of a new dress when he hasn't paid his house rent,

the point of meanness is applauded as a virtue. Nothing in


his life is considered more worthy than paying his bills.
(A) The problem of women in general is attention
the world over The ideal wife for such a man separates her housekeeping
money into joyless little piles so much for rent, for food,
(B) National Commission of Women is an attempt
for the children's shoes; she is able to face the milkman
for women’s development
with equanimity every month, satisfied with her
(C) Indian people love their children and try to take economising ways, and never knows the guilt of buying
care of them something she can't really afford.
(D) Right from independence women have been As for myself, I fall into neither of these categories.
receiving special attention for development If I have money to spare, I can be extravagant, but when,
(E) Care of children is an essential starting point for as is usually the case, I am hard up, then I am the meanest
social development man imaginable.
10. Why has, according to the passage the legislation 1. Which of the following would be the most suitable
prohibiting dowry failed in India ? title for the passage?
(A) Due to the cultural ethos in the country (A) Extravagance is always condemnable
(B) Because of the fact that it was passed as law (B) Extravagance leads to poverty

548 | CAT Complete Course

Visit : www.EasyEngineering.net
Visit : www.EasyEngineering.net

(C) Extravagance in the life of the rich and the poor 9. We understand from the passage that—
(D) Miserly habits of the poor (A) thrift may lead to success
2. In the opinion of the writer, a successful businessman (B) wealthy people are invariably successful
(A) should not bother about popularity, (C) all mean people are wealthy
(B) is expected to have expensive tastes (D) carefulness generally leads to failure
(C) is more popular if he appears to be doing 10. As far as money is concerned, we get the impression
nothing that the writer—
(D) must be extravagant before achieving success (A) doesn't often have any money to save
3. The phrase ‘lavish with his hospitality' in the third (B) would like to be considered extravagant
sentence of the first paragraph, signifies (C) is never inclined to be extravagant
(A) considerateness in spending on guests and (D) is incapable of saving anything
strangers Answers
(B) indifference in treating his friends and relatives
1. (C) 2. (B) 3. (D) 4. (B) 5. (A) 6. (D)
(C) miserliness in dealing with his friends

ww
(D) extravagance in entertaining guests
4. The word 'paradox' in the last sentence of the first
7. (A) 8. (C) 9. (A) 10. (A)
Passage–22

w
Words–66
paragraph means
The recent rapid growth of industry has, in some

.Ea
(A) statement based on the popular opinion
(B) that which is contrary to received opinion
cases, been so excessive that too much manufacturing
capacity has been developed in some fields of production,

syE
(C) statement based on facts
(D) that which brings out the inner meaning
5. It seems that low paid people should
which forces companies to sell their surplus products in
world markets at prices lower than normal. This will
make it almost impossible to develop local industries

ngi
producing the same items because consumers will prefer
(A) feel guilty if they overspend to buy the cheaper' imported product.
(B) borrow money to meet their essential needs 1. Why is it necessary for companies to sell products at
(C) not keep their creditors waiting
(D) not pay their bills promptly nee cheaper prices ?
(A) The cost of production has been considerably

6. How does the housewife, described by the writer,


feel when she saves money ? She.....
low.

rin
(B) The local industries also manufacture the same

(A) wishes she could sometimes be extravagant


product.

g.n
(C) There is a heavy demand for these products.

e
(B) is still troubled by a sense of guilt (D) The demand has been lowered significantly.
(C) wishes life were less burdensome
(D) is content to be so thrifty
7. The statement "she is able to face the milk man with
(E) None of these

t
2. According to the passage, the situation resulting from
the rapid industrial growth is
(A) favourable to the manufacturers
equanimity" implies that
(A) she is not upset as she has been paying the (B) disastrous to the exporters
milkman his dues regularly (C) conducive to the growth of local industries
(B) she loses her nerve at the sight of the milk man (D) unfavourable to the consumers
who always demands his dues (E) None of these
(C) she manages to keep cool as she has to pay the 3. 'This will make' in this sentence, 'This' refers most
milkman only a month's dues closely and directly to—
(D) she remains composed and confident as she (A) Development of local industries
knows that she can handle the milkman tactfully (B) The recent rapid growth of industry
8. Which of the following is opposite in meaning to the (C) Selling products of excessively higher prices
word 'applauded' in the passage ? (D) Companies manufacturing surplus products
(A) suppressed (B) cherished (E) Companies selling their production at cheaper
(C) decried (D) humiliated prices

CAT Complete Course | 549

Visit : www.EasyEngineering.net
Visit : www.EasyEngineering.net

4. Which of the following is/are most likely to hamper 3. “Accomation of interest” means—
the development of local industries ? (A) A place for living of interests
1. Availability of imported product at cheaper rates (B) Adjustment of individual interests
2. Consumer's tendency to refrain from using (C) Adjustment of personal ends
imported products
(D) None of these
3. Excessive production capacity and low pro-
4. One may be as free as one likes in—
duction cost
(A) Matters which do not encroach on ‘another’s
(A) Only A (B) Only B
liberty
(C) Only C (D) A and B
(B) Laughing at others
(E) A and C
(C) Laughing at oneself
5. 'Imported product' as used in the last product refers
(D) None of these
to'—
(A) product manufactured locally but of export 5. “Indifferent to“ can be replaced by—
quality (A) Having no interest in

ww
(B) product sold to such, other country which can't
locally manufacture it,
(C) product of a foreign country available at a
(B) Unconcerned about
(C) Not worried about
(D) None of these

w below normal price

.Ea
(D) surplus product manufactured by foreign
country and sold at a normal price
6. The word ‘fancy’ has been used twice in the sentence
number 4. It means—
(A) Liking; liking

1. (E) 2. (E)
Answers
3. (B)
syE
Passage–23
4. (E) 5. (C)
(B) Imagination; Imaginative
(C) Liking; whim
(D) None of these
Words–169
Liberty is not a personal affair only, but a social ngi 7. ‘Inquire of you’ means—
(A) Inquire form you
contract. It is an accommodation of interests. In matters
which do not touch anybody else’s liberty of course, I nee (B) Inquire about you
(C) Inquire into your views

rin
may be as free as I like. If I choose to go down the Strand (D) None of these
in a dressing-gown, with long hair and bare feet, who
shall say me nay, You have liberty to laugh at me, but I 8. ‘Dark’ is contrasted with the word …….…. in the
have to be indifferent to you. And if I have a fancy for
dyeing my hair. Or waxing my moustache (which heaven
last sentence
(A) Prefer g.n
(B) Whether
forbid), or wearing a tall hat, a frock coat and sandals, or
going to bed late or getting up early, I shall follow my
fancy and ask no man’s permission. I shall not inquire of
you whether I may eat mustard with my mutton. And you
(C) Fair

e
(D) None of these

t
9. ‘Prefer’ takes the proposition ……… as is clear form
the passage.
(A) To (B) On
will not ask me whether you may follow this religion or
that, whether you may marry the dark lady or the fair (C) With (D) None of these
lady, whether you prefer Longfellow to Wordsworth, or 10. ‘Champagne’ is a type of—
champagne to coca cola. (A) Cold drink (B) Soft drink
1. ‘Not a personal affair’ means— (C) Wine (D) None of these
(A) Not enjoyed in person
Answers
(B) Not having individual affair
1. (C) 2. (B) 3. (B) 4. (A) 5. (B) 6. (C)
(C) Not concerned with individuals
7. (A) 8. (C) 9. (A) 10. (C)
(D) None of these
Passage 24
2. ‘Social contract’ implies—
Words–577
(A) A contract for the sake of society
Naseema Begagum, a government school teacher in
(B) A contract among the members of society Karachi's middle income Garden Road locality, is not a
(C) A contract of social interests happy woman. She has been assigned election duties in
(D) None of these Karachi South, one of the city's more troubled areas.

550 | CAT Complete Course

Visit : www.EasyEngineering.net
Visit : www.EasyEngineering.net

Here the fight will be between Altaf Hussain's However, a recent rally in Hyderabad was a
Muttahida Qaumi Movement (MQM) and Benazir disappointment. In such uncertain times, there are
Bhutto's Pakistan Peoples Party. Naseema Begum and expectations that voter turnout will be much lower than
other teachers at the Government Girls Secondary School expected. The boycott by main religious parties will lead
on Garden Road say that it is the polling agents who have to lower voter turnout. Despite all the hype created by
to bear the brunt of election violence. "Hooligans from major politicians that this may be the election that will
one party came in when I was on duty and pointed a gun decide the future of President Musharraf, the chances are
at me and told me to get lost. It was so humiliating," she most Pakistanis will stay away from the polls.
says, recalling a previous election. 1. Which of the following headings can aptly sums up
Naseema believes that the MQM holds sway in most the passage ?
parts of Karachi, but points out that trouble arises in those (A) Terror stalks Pak polls
constituencies where there is opposition. On previous
occasions, the government has deputed the army and (B) Pak polls in the eye of terror
paramilitary Rangers force to protect polling stations but (C) Bullet not ballot will decide Pak poll fate
this time when polling takes place, the government has (D) Pak polls under terror shadow
instead deputed civilians like firemen, civil defence
(E) Pakistanis are disillusioned by past and present

ww
volunteers and private security guards.
But the teachers are not impressed. "Yeh aik chamaat
kay hain" (They are worth one slap) comments Muhibba
political scenario
2. According to the author low voter turnout will be due

w
Bano, a teacher also doing poll duty in January.

.Ea
Bano says that they feel insecure when local police
take charge as they run away at the slightest hint of
trouble. "In fact they are also part of the rigging when it
to—
(A) Booth rigging
(B) Fear of terror attacks and boycott by main

syE
religious parties
takes place," she alleges. Government employees
(C) Lack of voters faith in political parties
complain that they are forcibly drafted to election duties
without any proper arrangements. "We get very little (D) Fear of religions ostracise and repercussion
allowance, no transport and no security," comments Bano.
This time round, they say, there is another danger ngi (E) because Government employees not get proper
allowances and security
because of which many are being advised to stay away
despite strict government instructions to be present at
their stations on polling day. The danger comes from
nee
3. Polling agents fear for their safety because of the
following reason—

religious militants who have threatened to blow up


polling stations, particularly in the troubled tribal areas. rin
(A) Inadequate security
(B) Opposing political cadre clashes

President Musharraf has alluded to this threat in his


speeches and vowed that the democratic process "will not (D) Terror attacks g.n
(C) Intimidation by hooligans

be hijacked by those who believe in extremism."


However, the rise in suicide attacks over the past week
have again raised fears of election related attacks.
The attack on the welcome rally of Ms. Bhutto in
(E) none of these
e
4. The writer introduces which of the following to
support the argument of polling agents ? t
October 07 in which hundreds died is fresh in the minds (A) Examples
of people. “we are afraid of what can happen at rallies,” (B) past history of Pak
says Nasheed Ahmad, (C) News clips
a resident of Rawalpindi Who said he did not go to (D) premise
the airport to receive Ms Bhutto as he feared a possible
suicide attack. "I cant help it," he says, adding "my family (E) Interviews and number of terrorist attacks
members do not let me go." That is one reason why 5. The author is most likely to sympathizes with which
election rallies are poorly attended, say observers. statement—
The government has also issued a code of conduct (A) A few Pakistani will turn up for voting
under which political rallies can only beheld in the (B) The majority of Pakistanis will not exercise the
daytime. "Our priority in the elections is to ensure law right to vote
and order," comments Brigadier Akhtar Zamin, the home
(C) A fifty percent turnout is a fair assumption
minister for Sindh province where the most largely
attended rallies are those in which Ms Bhutto is the main (D) Pakistanis will in masse boycott the polls
speaker. (E) A huge voter turnout, say about 70%

CAT Complete Course | 551

Visit : www.EasyEngineering.net
Visit : www.EasyEngineering.net

Answers The two managed to leave a lasting impact, despite


1. (D) 2. (B) 3. (A) 4. (E) 5. (B) doing away usual romantic track that Bollywood heroes
can’t seem to do without. In the girlie category, our vote
Passage–25 for hot and happening would go to the chak De girls who
Words–627 showed what woman power was all about; and then for
Year of Experiments and ‘Neo-Wave’ Cinema the two debutants, Deepika Padukone and Sonam Kapoor
The year goes down in Bollywood history as the who matched dew fresh charisma with Ranbir Kapoor
‘year of living Dangerously’. Forget the regular hits like and Neil Mukesh, the male debutants of the year.
Om Shanti Om, Namastey London, Partner and Heyy Watch out for them in the next two years, they will
Babby. These were the usual blockbuster that one find in be heading the power list soon. But only next to the brand
Bollywood annual roster, year after year. What made new hero of Bollywood: Innovation, experimentation,
2007 special was the spirit of Bollywood. Like a new ideation.
–age Columbus, it embarked boldly on a voyage into the And how do we close account without gleefully
unknown, caring little about the common concern of cash anointing the duhs! Of the year. Leading the sad pack is
inflows and revenue curves. indeed Ram Gopal Verma ki Aag which enunciated how
The idea was the lodestar which guided this doughty not to remake a classic, followed by Nikhil Advani’s

ww
ship into unchartered terrain, choppy waters and swirling
whirpools. And the end result of all this experimentation
was a neo- wave cinema that boasted of films like Bheja
Salaame-e-Ishaq and Shaad Ali’s Jhoom Baraber Jhoom
,two over hyped films that bit the dust, despite the stars
and the reputed directors.

w
fry, Black Friday, Parzanta, Johny gaddar, Manorama six
feet under and perhaps, the biggest venture of them all,
Taare zameen par.
.Ea
Think about it and you will realize that even the two
Remains of the day? Year 2008 is going to be tough
formula won’t work taking the audience for granted will
be suicidal, adventurism will be the must have attitude.
Truly a year to find out: Bollywood tussi great ho or

syE
biggies that charmed your heart chak de India and Jab we
met were formula busting films. Chak de struck and
instant chord and became the national slogan with its
nahin.
1. The main theme of the passage can be summed up by
which of the following—

ngi
underplayed celebration of patriotism, only because this
brand of desh bhakti was so different from the chest (A) block buster are an annual feature
beating, flag waving nationalism of films like Border, (B) Hits like ‘Om Shanti Om’ ‘Namaste London’
Gaddar and even Lagaan. Similarity jab we met turned
the Dilwale Dulhaniya le jayenge formula on its head,
merely because its lead characters streered clear of the nee and Partner tell a big story
(C) New age entrepreneurs step into unknown
frontiers
stereotype, as did the dhamakedar dialogue which the
daily banter of today’s youth
rin
(D) Popular actors like Amitabh Bachan Hrithik
Roshan Shah Rukh Khan are respectively for
Films like Dharma,Black Friday and Parzania
displaed the power of docufeatures and showed how
cinema could still sell soul stirring messages about peace,
hits
g.n
(E) Living on conservative lines accounts for
humanity, oneness and tolerance. Bheja fry lead the brat
pack with its attempts to redefine comedy completely and
make the laugh act brain teasing rather than rib tickling.
Of course, India still laughed on the banana-peel skids
blockbusters

e
2. By these lines in the 2nd paragraph, “this doughty
t
ship into uncharted terrain and swirling whirlpool”
the authors implies—
(partner, Heyy Babby, Dhamaal and Dhol)But the Rajat (A) There are fluke films that made it big
Kapoor Vinay Pathak banter gave the comic a cutting (B) These brand of films are bold and neo
edge satire and black humour. wave cinema which will sparkle and pioneer a
But the real teasers of the maverick bunch were two new trend of cinema that are different and
films which made the criminal king. Johny gaddar and ground breaking.
shootout at lokhandwala stood out for their ekdum human (C) Though innovative cannot set a precedent
insights into evil, making the anti hero alluring, yet (D) Wishful thinking to cash in on the fickle publics
wicked. mind
Surely, Vivek Oberoi’s crook act in shootout at
(E) Are not worth the while
Lokhandwala and Neil Mukesh’s Johny gaddar are the
two most mesmerising character of the year in the 3. The author infers that the two films Chak de India
‘ignoble’ class, even as Shah Rukh khan’s coach Kabir and Tare Zameen par the hottest characters are coach
Khan in Chak De India and Aamir Khan’s Nikumb sir in Kabir Khan and Nikumb sir because—
Taare zameen par are the hottest characters in the ‘noble’ (A) They are the hottest character in two noble
category. category

552 | CAT Complete Course

Visit : www.EasyEngineering.net
Visit : www.EasyEngineering.net

(B) The left an indelible impression without having (C) Humanitarian considerations will be the only
to depend on the emotional remarks quotient test of brotherly feeling
(C) Stood out for their insights into evil (D) None of these
(D) They are the two most mesmerising character of 5. The writer wants of say that nations should—
the year (A) Not spend on education
(E) Two films which made the criminal king
(B) Not manufacture atoms
4. Taking the public for granted in the closing lines of (C) Not wage wars
the passage implies—
(D) None of these
(A) Formula will work
6. Nations in new world would have—
(B) Taking the public for granted is suicidal.
Adventurism will tip the scales. (A) Different ideologies
(C) Over hyped films have hit the dust (B) Friendly relation
(D) Only time will tell whether Bollywood age old (C) Different ideals
formulas will work (D) None of these
(E) Stars will make all the difference 7. In the new society everyone will have—

ww
1. (C) 2. (B)
Answers
3. (B) 4. (B)
(A) Divine power
(B) Ideologies

w Passage–26

.Ea Words–138
It is firm resolve that we should strive together to
(C) Light of spiritualism
(D) None of these
8. ‘Refulgent’ means—

syE
build a new world—a world where the differences of rich
and poor, colour and castes of brotherhood, where every
religion will be respected, where the wealth of the nations
(A) Dazzle
(C) Radiant
(B) Glare
(D) None of these

would be employed for the developmental works and for


the improvement of education, health and nutrition of the
children, instead of building up atomic piles for waging ngi 9. ‘Road’ in the last sentence refers to—
(A) Build new world

nee
(B) Achieving spiritualism
wars, where nations would have friendly relations with (C) Getting divine power
one another even though they might have subscribed to
(D) None of these

rin
different ideologies, where the structure of divine power
in everyman would be converted into the refulgent light 10. ‘Perforce’ means—
of spiritualism. The road is difficult now like the razor’s
edge, but if you want to preserve the human race, we will
perforce have to walk on this path – with courage, with (C) Forcefully
g.n
(A) Under compulsion (B) By force
(D) None of these
patience and with self confidence.
1. ‘Firm resolve’ in the first sentence means—
(A) Definite resolution (B) Accepted view
(C) Unflinching (D) None of these
e
11. If we do not follow this path—
(A) Human race will come to an end
(B) We will be forced to follow it
(C) Will have to take courage
t
2. What is ‘our firm resolve’ ? (D) None of these
(A) To work together
Answers
(B) To make a collection effort to build a new world
1. (C) 2. (B) 3. (A) 4. (C) 5. (C) 6. (B)
(C) To build a new world
7. (C) 8. (C) 9. (A) 10. (A) 11. (A)
(D) None of these
3. One of the features of the new world will be— Passage–27
(A) No distinction of colour and caste will exist Words–230
(B) No person belonging to castes will exist When we are suddenly confronted with any terrible
(C) Poor and rich will live together danger, the change of nature we undergo is equally great.
In some cases fear paralyses us. Like animals, we stand
(D) None of these
still, powerless to move step in fright or to lift a hand in
4. ‘Humanity will be sole test of brotherhood’ means— defence of our lives, and' sometimes we are seized with
(A) Human being will judge brotherhood panic, and again, act more like the inferior animals than
(B) Human feeling will develop brotherhood rational beings. On the other hand, frequently in cases of

CAT Complete Course | 553

Visit : www.EasyEngineering.net
Visit : www.EasyEngineering.net

sudden extreme peril, which cannot be escaped by flight, Answers


and must be instantly faced, even the most timid' men at 1. (B) 2. (C) 3. (D) 4. (B) 5. (B)
once as if by miracle, become possessed of the necessary
courage, sharp quick apprehension, and swift decision. Passage–28
This is a miracle very common in nature. Man and the Words–93
inferior animals alike, when confronted with almost The artificial ways of inducing sleep are legion, and
certain death 'gather resolution from despair' but there can are only alike in their ineffectuality. In Lavengro there is
really be no trace of so debilitating a feeling in the person an impossible character, a victim of insomnia, who finds
fighting, or prepared to fight for dear life. At such times that a volume of Wordsworth's poems is the only sure
the mind is clearer than it has ever been; the nerves are soporific, but that was Borrow's Malice. The famous old
steel, there is nothing felt but a wonderful strength and plan of counting sheep jumping over a stile has never
daring. Looking back at certain perilous moments in my served a turn. I have herded imaginary sheep until they
own life, I remember them with a kind of joy, not that insisted on turning themselves into white bears or blue
there was any joyful excitement then; but because they pigs, and I defy any reasonable man to fall asleep while
brought me a new experience - a new nature, as it were mustering a herd of stupid swine.
and lifted me for a time above myself.
1. The author points out that—

ww
1. An appropriate title for the above passage would
be— (A) sleep can-easily be induced
(A) The Will to Fight (B) the artificial means of inducing sleep are not

w
(B) The Miracle of Confronting Danger good
(C) The Change of Nature (C) artificial ways of inducing sleep are ineffective
(D) Courage and Panic
.Ea
2. The author names three different ways in which a
(D) artificial ways of inducing sleep are expensive
2. According to the author the character in Lavengro

syE
man may react to sudden danger. What are they ?
(A) He may flee in panic, or fight back or stand still.
(A) resorts to external aids to get some sleep
(B) is an admirer of Wordsworth

ngi
(B) He may be paralyzed with fear, seized with (C) spends sleepless nights reading Wordsworth
panic or act like an inferior animal.
(D) is an avid reader of poetry
(C) He may be paralyzed with fear, or seized with
panic, or as if by miracle, become possessed of
the necessary courage, and face the danger.
nee
3. The author uses "impossible" for the character of
Lavengro in the sense of—

rin
(D) He may be paralyzed with fear, run away or (A) funny (B) unrealistic
fight. (C) queer (D) imaginary
3. The distinction between inferior animal and 'rational
beings' is that—
(A) the former are incapable of fighting.
(A) Sleeplessness
g.n
4. Borrow's malice is most probably directed at—

(B) the latter are clever.


(C) the latter are stronger.
(D) the latter are capable of reasoning things out
(C) Wordsworth's poetry
(D) Poetry in general
e
(B) The artificial ways of inducing sleep

t
whereas the former cannot do so. 5. In order to cure his insomnia, the writer—
4. Explain the phrase 'gather resolution from danger'— (A) does a lot of reading
(A) Find hope and courage (B) vainly tries to concentrate on imaginary sit-
(B) A state of utter hopelessness steels one to fight uations
out the danger (C) keeps a flock of sheep
(C) Not to lose hope, but fight (D) counts sheep jumping over a stile
(D) Find courage to face the danger
Answers
5. The author feels happy in the recollection of dangers
1. (C) 2. (D) 3. (B) 4. (B) 5. (D)
faced and overcome because—
(A) they brought him a new experience Passage–29
(B) they brought him a new experience, and lifted Words–135
him above himself for a time Experiments with the Sulfonamides have made clear
(C) he survived his ordeal a fact about germs which is gaining increasing importance
(D) he was lucky to be alive in fighting them. Germs, it seems, have the same ability

554 | CAT Complete Course

Visit : www.EasyEngineering.net
Visit : www.EasyEngineering.net

as all the other living things gradually to change them- Passage–30


selves to suit new conditions. But, as the generation of
germs lasts only twenty, twenty-five or thirty minutes, Words–220
before all the germs divide to form new ones, changes, Though supposed to be the beginning of woman’s
that would take many years in animals can be achieved liberation decade, recent months have been far form
by germs in a few hours. Perhaps, then you give the being kind to top ladies. All the three really prominent
attacking germ a dose of Sulfonamides which upsets once only a short while ago firmly in power, are now out
them somewhat but is not strong enough to prevent them it. In Argentina, the widow of the former dictator, Juan
from multiplying; if so, they very rapidly develop new Peron, was displaced by a military coup; but both Mrs.
powers which enable them to resist the effects of the Indra Ghandhi early this and Mrs. Bandaranaike now,
drug. After this has happened, even the strongest dose have been defeated in a democratic election. Speculation
will fail to disturb them. will be immediate about the extent to which the Indian
1. Experiments with Sulfonamides have led to the example influenced even farther south. Mrs. Gandhi had
important discovery that— governed by Emergency and extensively rewritten Con-
stitution. Mrs. Bandaranaike used a similar steamroller
(A) germs are living things, and can change thems- majority in the last Parliament to give Sri Lanka an
elves to suit new conditions. entirely new one. Both extended the natural life of

ww
(B) one generation of germs lasts only twenty,
twenty five or thirty minutes.
Parliament; both imposed severe curbs on the judiciary
and the press, both were eventually embarrassed by the
prominence given to their sons. The main difference

w
(C) germs can adjust themselves to live and mul-
tiply in new conditions.

.Ea
(D) germs are not disturbed even by the strongest
possible dose of Sulfonamides.
seems to be that Mrs. Bandaranaike at least won her own
seat, whereas Mrs. Gandhi did not. It will be a matter of
wide satisfaction at home and abroad, for the subcontinent
to have once again demonstrated, that special power are

syE
2. Like all other living things, germs can change
themselves to suit new conditions. This adjustment is
not necessarily eternal that public patience is not
inexhaustible, and that it is a pure and interested myth to
assert that Asia is not a favourable ground for effective

ngi
possible because the germs have—
democracy.
(A) the power of fluctuation
1. ‘Liberation decade’ means—

nee
(B) the power of compliance
(A) Decay of liberation movement
(C) the power of adaptability
(B) Liberation has come to an end
(D) the power of adaptability
3. Since germs can change themselves to suit new
rin
(C) A period of 10 year during which liberation
movement goes on
conditions, the task of fighting them has become—
(A) absolutely impossible
(D) None of these

g.n
2. ‘Far form being kind’ means—
(B) much easier
(C) much more difficult
(D) increasingly important
4. Germs which are not disturbed even by the strongest
(C) Kind enough
3. Juan Peron was the—
e
(A) Has not been kind (B) Not kind
(D) None of these
t
(A) Prime Minister of Argentina
possible dose of the Sulfonamides are said to have (B) Wife of the former dictator of Argentina
become—
(C) Lady who brought military coup
(A) immortal (B) immune
(D) None of these
(C) improvised (D) immobile
4. Speculation will be immediate. About what ?
5. One generation of germs expires, bringing into (A) About democratic elections
existence the next generation—
(B) About Mrs. Bandaranaike’s defeat
(A) in twenty minutes
(C) About the extent to which Indian elections
(B) in twenty five minutes effected Sri Lanka
(C) in not more than half an hour (D) None of these
(D) in a few hours 5. Mrs. Gandhi ruled—
Answers (A) By changing the Constitution
1. (A) 2. (D) 3. (C) 4. (B) 5. (C) (B) Through emergency

CAT Complete Course | 555

Visit : www.EasyEngineering.net
Visit : www.EasyEngineering.net

(C) Through emergency and by rewriting consti- 3. “Fullest description of God” means—
tution (A) Describing God completely
(D) None of these (B) Giving proper description of God
6. ‘Steamroller majority’ means— (C) Complete description of God
(A) Overwhelming majority (D) None of these
(B) Overriding majority 4. Why does he use the phrase “a step further” ?
(C) Happy majority (A) In order to show that his earlier view less
(D) None of these comprehensive
7. Mrs. Gandhi and Mrs. Bandaranaike resembled— (B) In order to prove the second view is larger one
(A) In curbing judiciary and press (C) In order to say that Truth makes God higher
(B) In being ousted (D) None of these
(C) In resigning 5. ‘Atheists’ are the persons—
(D) None of these (A) Who do not believe in the existence of God

ww
8. Public patience is—
(A) Boundless
(C) Pure
(B) Not boundless
(D) None of these
(B) Who worship many gods
(C) Who are irreligious
(D) None of these

w
9. Myth means—
(A) Fiction
(C) Mythology .Ea
(B) Legend
(D) None of these
6. Power of Truth—
(A) Cannot be denied even by the atheists
(B) Can be understood

1. (C) 2. (A)
Answers
3. (B)
syE
4. (C) 5. (C) 6. (A)
(C) Can be realized
(D) None of these

7. (A) 8. (A) 9. (A)


Passage–31 ngi 7. ‘Atheists’ have not hesitated—
(A) To disbelieve the existence of God

Words–144
I would say with those who say God is Love. But nee (B) Deny Gods’ power
(C) Deny life
(D) None of these
deep down in me I used to say that through God be Love,
God is Truth-above all. If it is possible for the human
rin
8. In the last sentence the word ‘this’ refers to—
tongue to give the fullest description of God, I have come
to the conclusion that for myself, God is Truth. But two
years ago I went a step further and said that .Truth is
(B) Existence of God
g.n
(A) Argument given in the previous sentence

God. I never found a double meaning in connection with


Truth, and even atheists had not demurred to the
necessity or power of Truth but in their passion for
discovering Truth the atheists have not hesitated to deny
(C) Hesitate
(D) None of these
e
9. The main idea of the passage is—
(A) Truth is God (B) God is Truth
t
the very existence of God form there own point of view (C) God is Love (D) None of these
rightly. And it was because of this reasoning that I saw
that rather than say that God is Truth, I should say that Answers
Truth is God. 1. (D) 2. (A) 3. (C) 4. (A) 5. (A) 6. (A)
1. The writer believed— 7. (A) 8. (A) 9. (A)
(A) What others said about God Passage–32
(B) That God is Love Words–188
(C) That Love is God It will be a mistake to think that he was given only
(D) None of these 'bouquets', he also received many 'brickbats'. The
2. “Deep down in me” means— Christian missionaries took alarm at his popularity. They
used to raise funds 'by preaching that India was a land of
(A) In the hearts of my heart
heathens waiting to be saved by Christianity. The
(B) When I was down American press now began to say that it was a shame that
(C) In the depth of down any body should try to teach India religion, rather the

556 | CAT Complete Course

Visit : www.EasyEngineering.net
Visit : www.EasyEngineering.net

world should sit at her feet to learn it. Vivekananda also living by something so 'advanced as the principles of
said that India did not need religion but material support. Christianity. Yet, there is a common phenomenon. It is
The missionaries found that the subscriptions they had so not, however, in my province to talk about religion, but
long been receiving from the people were steadily de- rather to stress the power which great literature and the
clining. They blamed it on Swamiji. They now started great personal and the great personalities whom we meet
denigrating him in all manner of ways. They even began in it and in it and in history have to open and enlarge over
to spread scandals against his personal character. minds, and to show us what is first rate in human
Strangely enough, even some of his own countrymen personality and human character by showing us goodness
joined them in this for reasons of their own. But 'Truth and greatness.
alone prevails', as Swamiji always preached. He did not 1. In the passage, the author’s ultimate intention is to
try to defend himself, but others stood up for him and talk about—
vehemently protested. Finally, all such mean attempts
(A) religion (B) history
failed and his reputation only rose higher and higher.
(C) education (D) character
1. The passage teaches us
(A) not to believe in religions other than our own 2. The phrase "so raising us" means—
(B) not to get involved in scandals (A) giving us a sense of spiritual superiority

ww
(C) not to visit foreign lands
(D) not to deviate from the path of truth
(B) making us feel that we are more important than
we really are
(C) improving our mental abilities

w
2. Vivekananda was criticised by missionaries in
America because—

.Ea
(A) he was a bad student of Western theology.
(B) he opposed the tenets of Christianity
(D) making us realise that we all are children of
God
3. What surprises the author is that—

syE
(C) Americans had become very fond of him.
(D) he did not allow them to raise funds in India.
(A) Even uneducated people are attracted towards
Christianity.
(B) Christianity is practiced by a large number of
3. Swami Vivckananda told the American people that
India— ngi people
(C) Despite being difficult and complex, the, prin-
(A) did not approve of the Catholic Church
(B) would teach religion to those who sit at her feet nee ciples of Christianity are practiced by so many
people
(D) Even very intelligent people cannot understand
(C) required religious and material help
(D) was self sufficient in religion though poor
rin
the principles of Christianity
4. The author hesitates to talk about religion be cause
4. Vivekananda's rapport with the American people—
(A) helped India get substantial aid g.n
(A) he does not feel himself competent to talk about
it.
(B) made his friends desert him
(C) annoyed the American Government
(D) caused a drop in Church's collections
e
(B) nobody around him likes to talk about it.
(C) he does not believe in any religion.
(D) he does not fully understand its importance. t
5. Vivekananda did not defend himself because—
5. According to the author, we come across examples of
(A) he believed in the ultimate triumph of truth. greatness and nobility in—
(B) he was in a foreign land. (A) great works of literature
(C) some of his countrymen were opposing him. (B) literary and historical works
(D) he had brought many friends along to fight for
(C) historical records
him.
(D) books on Christianity
Answers
1. (D) 2. (C) 3. (D) 4. (D) 5. (A) Answers
1. (C) 2. (D) 3. (C) 4. (A) 5. (B)
Passage–33
Passage–34
Words–109
Religion is the greatest instrument for so raising us. Words–691
It is amazing that a person not intellectually bright, Judiciary has enjoyed years of activism. Public
perhaps not even educated, is capable of grasping and adulation, emanating from the instant judicial solution to

CAT Complete Course | 557

Visit : www.EasyEngineering.net
Visit : www.EasyEngineering.net

their problems, has oxygenated filling of PILs in the apex already ensured payment of Rs123crore to the contractor
and high court. A fact-over 90% of the PILs are dismissed with the direction that it would look into the final
by the courts. The exercise of its prime role as protector estimates of the project cost. Who fixes the project cost-
of fundamental rights has made the judiciary wander judiciary or executive? Justice Katju has consistently
deep into executive’s and sometimes, into legislature’s advocated the principle of judicial restraint. This is
domains. As long as the executive was at the receiving evident from his judgment right in the beginning of the
end, there was hardly any protest. year.
The first ever clear-cut legislative action by the SC In Veam china Koteshwar Rao case judgment
happened in Vishakha case. It gave a new guideline, to pronounced by him on February 15, he had said, “ it is
deal with perverted Romeos at work-place. Though the well settled that there must be judicial restraint regarding
judgment came more than 10 years ago, it still holds administrative decision. “ He is not a judge known for
good. For, the legislature has not bothered to enact a law keeping any restraint in his utterances during hearings.
to make sexual harassment at workplace a statutory The famous “hang them from the nearest lamp post”
offence mooted by him for corrupt officials had hit headlines.
Protests from legislature became loud after the Well, we are sure that he said it out of anger against
judgment on President’s rule in Bihar castigating the rampant corruption in the country and did not actually

ww
governor and the central government for preventing a
coalition from staking claim. This was followed by SC’s
intervention in Jharkhand trust motion, an order that
mean it to be a law. For, it had striking resemblances with
Taliban laws. Few days back, he again observed that if a
girl was of marriageable age, there was nothing wrong in

w
made legislators hoist the red flag for judicial
encroachment into their domain. Buoying the legislative

.Ea
and executive protests, a two judge bench of Justices AK
Mathur & Markandey Katju recently penned a judgment
her eloping with the person she wanted to marry.
Judicial restraint should also from part of their
speech at the time of hearing cases. If not, then
incongruities-like the two-judge bench terming previous

syE
warning of impending political anger over judicial
activism that could lead to curtailment of judiciary’s
independence. It also urged judges to exercise restraint so
larger bench judgment as judicial aberrations-are bound
to creep in .
1. The author states which of the following examples
as not to breach the lakshman rekha demarcating
boundaries.
ngi prove that the legislature is of logger heads with the
judiciary ?

nee
The judgmenrt, delivered on December 6, did not 1. New policies and ruler to deal with workplace
specify which of the judges-justice Mathur or Justice Romes who are perverted advised by the judi-
Katju-authored it. It was a unanimous one- both agreeing ciary go Central unheeded a decade on
to it whole heartedly. The Justices Mathur-Katju bench
has in the last four months, given similar judgments. In
Union of India vs MS Mohd Rawther the bench on
2.
rin
Berating a government and governor for
disbarring a coalition from staking a claim to
August 16 said, “The court has only judicial power to
review the executive order on Wendnesbury principle, 3.
rule
g.n
Anger over judicial activism
but it cannot arrogate itself the power of the executive
………. The court must exercise judicial restraint in such
matters. There is a board separation of powers under the
Constitution, and one organ of the state should not the
4.

5.
boundaries
All of the above
e
Judges do not restrain in breaching demarcating

t
ordinarily encroach into the domain of another”. Chose the correct answer or combination :
So, the judicial non-encroachment into executive (A) 1 and 2 (B) 1, 2 and
domain is held to be a cardinal principle, by which both
Justices Mathur & Katju swear. (C) 1, 3 and 4 (D) only 3
If the restraint dice is rolled further, one comes (E) only 5
across the Ghazipur abattoir case pending before the 2. In which areap according to the passage the court has
court for the last three years. The abattoir is being no power to execute ?
constructed by the MCD. A three- judge bench of the SC, (A) To peruse the executive order on Wednesbury
including Justice Mathur, has been passing orders for principles
speeding up the construction, the money to be paid to the
(B) Beston upon itself the power of the executive
contractor and the release of machinery imported from
abroad. (C) One domain of the state should not trespass into
the area of another
Whose function is it to monitor construction of an
abattoir- judiciary or executive? The cost of the abattoir (D) Encroachment into executive domain
was fixed by an expert at Rs113crore, but the bench has (E) None of these

558 | CAT Complete Course

Visit : www.EasyEngineering.net
Visit : www.EasyEngineering.net

3. What would be the most suitable and appropriate title Thus according to the law, at the time of declaring a
for the passage ? ‘forest’ a settlement officer surveys and settles the rights
(A) Should judger look the other way fearing of people. Unsurprisingly, these settlement officers either
political wreath ? did nothing or only recorded the rights of those who were
powerful. Million of people, mostly tribal, lost their rights
(B) Judiciary fearing judiciary & executive back
and were deemed ‘encroachers’ in their own homes. To
lash
this day, 82 per cent of Madhya Pradesh’s forest blocks
(C) Executive reigns, judiciary bows have not been surveyed, while 40 per cent of Orissa’s
(D) Executive and judiciary: heads on forests and 60 per cent of our national parks have not
(E) Toe the line, executive to the judiciary completed settlement of rights. In Gujarat, a survey found
4. What are the problems which may crop up, accord- that even when the Government claimed to recognize
ing to the author if judicial restrained does not land title, only 5 per cent of the eligible claimants actually
became a part of judgment during or at the time of got pattas.
hearing cases ? As a result of this, neither forests nor the people
(A) Differ cultists are likely to arise gained. Those deemed ‘illegal’ are subjected to extortion,
assault, jail or arbitrary eviction. It is no accident that
(B) A smaller bench accusing larger benches of

ww making wrong decisions


(C) Executives will take the advantage
tribals are the poorest communities in the country today.
Forests, in turn, came under the highly centralized
control of the bureaucracy, which treats them as property

w
(D) Judge will lose face and executives will stand to to be sold to the highest bidder.
gain
(E) None of the above
.Ea
Answers
This happens through corruption and legal diversion
of forests for industrial, mining and development purpose,
which has already destroyed five lakh hectares of forest
1. (E) 2. (B) 3. (B)
syE
Passage–35
4. (B) between 2001 and 2006. Forest dwellers who oppose
destruction of their homelands find themselves facing
criminal cases, arrest and eviction.
Words–808
The Dust over the Gujarat election has begun to ngi The Forest Rights Act aims to address this through
two steps. First, it recognizes the trilbals’ rights to land
settle and the post-mortems have begun. The pundits have
zeroed in on three or four reasons for the Congress’
defeat. One of them is the stalling of the Forest Rights nee
and forest resources that they were using as of 2005 (non-
tribals have to prove 75 years of residence) No land is
‘given’; no one receives title to land that they are not
Act, which apparently cost the congress a large number
of potential tribal supporters and which-belatedly-will be
rin
already cultivating. Second, it gives communities the
right to protect forests, empowering them to be partners
notified today. Is this diagnosis correct? Why should
what seems to be an arcane welfare scheme make such a
difference to a state election? g.n
in conversations rather than its victims.
So why has the government delayed the Act? Such
change is not liked by the forest bureaucracy by the
The Scheduled Tribes and Other Traditional Forest
Dwellers (Recognition of Forest Rights) Act, 2006 is not
a welfare scheme. Rather, it aims at correcting the cen-
tury-long State takeover of resources belonging to the
e
forest bureaucracy with conversation the firestorm of t
industrial groups that currently enjoy easy access to forest
land or by a few conservationists who now equate the

opposition to this Act has taken place in the name of tiger


country’s tribal and forest communities. Contrary to conservation, but its essential theme has been that all
allegations, this Act does not ‘hand out’ or distribute power should remain with the bureaucracy. Anything
land to anyone . It addresses one basic problem : what are else, we are told will lead to catastrophe for tigers, forests,
called ‘forests ‘ in India law presumed by many to be ‘ or both this is despite the fact that the act itself provides
pristine wilderness’ are nothing of the kind; crores of safeguards against land-grabbing and a procedure for how
people live inside these forests. people can be resettled where necessary for wildlife, a
Why do many of them not have rights to the land procedure supported by environmental organizations.
they live on ? The reason is India’s forest laws. The forest Given such powerful opposition, even when passing
Acts aimed to bring all forests under a centralized Forest the Act, the government modified some key clauses
Department’s control and to take over the lands and rights severely weakening the law’s provisions. For a full year
of forest-dwellers. At first this was justified in the name after its passage, the government stalled the Law’s imple-
of easier timber extraction; then post-Independence., for mentation. Today, according to the minister, the Act will
the nation’s industrial requirements; and finally, for be notified into force. But even this may not end the
conservation. Whatever the justification, the policy has charade, for the rules to the Act will likely again be
remained the same. heavily diluted.

CAT Complete Course | 559

Visit : www.EasyEngineering.net
Visit : www.EasyEngineering.net

Meanwhile, 10 days ago, more than 100 families Answers


were evicted in Madhya Pradesh and their homes 1. (E) 2. (E) 3. (A) 4. (B)
demolished; forest guards opened fire twice, seriously
injuring six people. Forest destruction also continues, Passage–36
with the ministry hastily clearing new industrial and Words–875
mining projects. The notification today will be a step Catch Them Young
forward. But if the rules are diluted further, it will only
Education system in India, all the way from adult
show that mere electoral defeat is insufficient to shake the
literacy programmes and rural schools to vocational
government’s commitment to vested interests. It is a
institutes and research universities, need a comprehensive
commitment that has already extracted a heavy price.
makeover. This is uppermost amongst the country’s
1. Which of the following would be the most suitable development priorities.
title for the passage ?
Literacy and basic education are required for people
(A) Don’t dilute the Forest Act to manage their daily lives and participate in democratic
(B) Out of the wilderness processes; vocational skills enable participation in the
(C) Arcane scheme rock the congress boat economy; and higher education enable Indians to play a
(D) Protect tribunal interests or lose more effective role in the global knowledge economy and

ww
(E) Rights of tribals must not be denied or watered
down
international affairs. However there is another need
which runs through all forms of education: it is the need
to inculcate the right values to develop responsible

w
2. Which of the following the author does not mention
at the time of declaring a ‘forest’ ?

.Ea
(A) The settlement officers surveys and settler the
rights of people
citizens of society.
Changing societal values is a recurring theme across
the world the erosion of ‘family values’ has been
concerning leader in the USA where both republican and

syE
(B) The settlement officer did nothing
(C) The settlement officers only recorded the rights
of those who were powerful
Democratic political leaders regularly call for their
restoration. The decay of ‘Community life’ also bothers
Americans. Social studies reveal that family and
(D) Tens of thousands, mostly tribals lost their right
(E) 82 percent of MP’s forest blocks have been ngi community values contribute greatly to happiness, and
their depletion in economically advanced countries is a
surveyed
3. In the starting statement of paragraph six the author
implies—
nee
significant reason why people in these countries are not
happier even when they are richer.
Though children brought up in the education systems
(A) That corruption and legal methods of distribu-
tion deprive tribals of their rights and make rin
of these countries learn to stand on their feet, and be
politically and economically independent, often they end
bureaucrats rich
(B) Tribals are charged with criminal cases, arrested g.n
up “Bowling Alone” (in sociologist Robert Putnam’s
memorable phrase) and unhappy. Therefore as, we strive
to improve India’s education systems to enable our
and convicted
(C) It recognizer the tribals rights to land and forest
resources
(D) Communities are given the right to protect
e
children to become economically independent, they
should also imbibe better family and community values
in them if we want a more harmonious, happy society. t
At the heart of family and community values is the
forest
value of inclusion of considering not only one’s own
(E) It empowers them to be co-owners in conserva-
needs, but also the needs of others. Some suggest that
tion and not destroyers of forests.
good values are built at home, in the family and the
4. The firestorm of opposition to the proposed Forest community rather than in school. However, when good
Rights Act is summarized by which of the follow- values are disappearing from families and communities
ing— themselves, schools must become the place where they
(A) Power shall remain with the bureaucracy under are strengthened, and from which required values will be
the name of tiger conservation reinserted into society. In India, citizens must think and
(B) That it will lead to the destruction and exploita- act more inclusively than many in any other country of
tion of tigers instead of conservation of tigers the world perhaps, because we are very diverse and a
(C) The act provides provisions against land snatch- democracy with many inequities.
ing Values cannot be taught like texts, nor tested in
(D) As to how the tribals can be resettled smoothly written examinations. They are learned by living. Two
and efficiently Indian schools that are good example of how children
(E) None of the above imbibe values of inclusion are the Loreto Day school

560 | CAT Complete Course

Visit : www.EasyEngineering.net
Visit : www.EasyEngineering.net

besides Sealdah station in Kolkata, and the Kathalaya 1. The need of the hour according to the author is their
schools in Delhi’s enormous Govindpuri slum. The India needs to stress on above all—
Loreto Day school has about 1’500 children, half from (A) To change its societal values
fee paying middle class families, and the rest from poor
(B) Literacy and basic education
families.
Amongst the poor are about 350 ‘rainbow’ children, (C) Vocational training
who are taken off the streets, many of whom are orphans. (D) Adult literacy programmes
The children come from very different circumstance but (E) All of the above
no distinction is made amongst them when they are in 2. By the phrase decaying of community life the author
school. Learning is in teams. The rainbow mix with the means—
rest. The school enters inter-school debates and other
competitions and does well. (A) Erosion of family values
However it, has an unusual system. If a child wins a (B) Community value play as vital role in contri-
debate, that child does not compete in the next one. This buting to happiness
may affect the school’s chances at winning. But it gives (C) That depletion of community values have an
another child a chance to win and shine. Imagine adverse effect on the overall happiness quotient

ww
Implementing this system in Indian cricket. If a player
scores a centaury, he must step down from the next
match. There would be fewer superstars but there may be
(D) Restoration of such values are the need of the
hour
(E) People are not happier even when they are richer

w
many more stars!
The Kathalaya school is in the middle of the huge,

.Ea
congested, and trash- strewn slum. It has several hundred
children, all poor. Here too, children learn in teams. They
3. The main concern of the author is which of the
following—
(A) Literacy and basic education

syE
begin every semester exploring a new theme, and weave
all their lessons, in language, history, geography, science
and Vocational skills, around topics that they discover
(B) To inculcate correct values to make happier and
better citizen
(C) Vocational training; so that our can earn a

ngi
and want to learn about as they explore this theme.
livelihood
The school is buzzing with activity computer screens
flashing, painting sketched on boards, colleges of words (D) Improve our educational system to make our
and pictures, and voice of children. No bored children
sitting in row in this school! The unusual mode of
learning in the school not only enables the kids to easily
nee children economically independent
(E) India is diverse and a country with many
inequities.
pass the certified central school exams, it also gives them
a great ability and desire to learn. rin
4. The most appropriate title for the passage would
be—
The themes the kids explore in school lead them to
the concerns of their communities. While looking into
(A) Values are invaluable
g.n
(B) Children our future leaders
these, they are guided into structured projects. For
example, teams are working on improving sanitation and
availability of water in the slums of Delhi.They have
surveyed families needs and the physical realities of
infrastructure. They have discussed solutions with the
(D) Catch them young
(E) Child, the father of man
e
(C) Community values our strength

t
municipal authorities, some of which are being imple- Answers
mented. They are also persuading their community to 1. (A) 2. (C) 3. (D) 4. (D)
change behaviours, regarding sanitary practices for
example. The confidence of these 12-14 years old children Passage–37
in the face of staggering odds- that they will make a
Words–878
difference to the lives of their own communalities is most
inspiring. A bunch of no-hopers from Iraq-spurred on by their
Both these schools are showing how, along with all platitude-spilling Brazilian coach and some crazy idea of
the utilization knowledge and skills children need to pass deliverance - provided the most poignant moment in all
exams and get jobs, the children are also learning values sport in 2007, and not just football.
of caring for others and for their surrounding, so that they When a hastily-whipped up, scarred-by-the war, no-
will become not only responsible citizens, but even place-to-call-home team won the Asia Cup this summer,
leaders. These children are some of the millions of it helped a sport that has become an industry, remember
fireflies, carrying their own light for others. They can that you cannot manufacture romance. Football become a
transform India into a more inclusive and better society. sport again, albeit briefly...

CAT Complete Course | 561

Visit : www.EasyEngineering.net
Visit : www.EasyEngineering.net

Because soon, the wheels that turn football today unfolding. Having quit the National team after the World
were back in business. And the Iraq story being the story Cup last year, since his mother couldn't take the pressure,
of the year, 2007 actually belonged to two characters - so the moody, publicity-shy Argentine playmaker Riquelme
different and yet so similar with their unique sub-plots in was having Problems at his club, Villarreal which had,
the bigger picture. David Beckham and Juan Roman for three seasons, revolved around him. Written off, out
Riquelme may have crossed paths a few times in the of favour and still on loan from FC Barcelona, he was
Spanish League turning out for Real Madrid and Villarreal farmed out to Boca Juniors in his homeland Argentina
respectively, and the odd footwear commercial, but you where he recovered his joy for the sport leading the side
couldn't have more diverse personalities living the same to the South American championships, the Copa
life. And that irony was the story of the year in football- Liberator’s.
pretty -face, money-spinning, most-time model, part-time But then that is the Americas and only Beckham
footballer who can't hide meets out of work misfit who must shine there. In Europe with his club future still
actually happens to be the best footballer in the world at uncertain, he returned to the Argentine National fold,
the moment, but there's no one to confirm the fact. donning the No 10 once again, and dishing out some
Much before he finally won his only trophy in his scintillating football as Argentina marched in fear some
high-profile, much-sought-after four years with Real fashion to the Copa America finals only to self-destruct

ww
Madrid, Beckham had already signed up to take soccer in
the US to its natural plane, i.e. help it break the bank.
And so, even as sanity -read, football - was returning to
before a clinical Brazil side in a manner on1y they can.
Typically Requiem faded in the big game, but then
seemed to regain his groove as the mammoth South

w
Rea1, Beckham was sulking over not being allowed to
attend newest bestest pal Tom Cruise's wedding. That

.Ea
was last year, and, the man who clamped-the curfew back
then on1y Cause of the minor detail of Real playing over
American qualifiers for the 2010 World Cup got under-
way, majestically dictating play and scoring spectacular
goals–off regular play and free-kicks.
So like Beckham and unlike him, here was Riquelme.

syE
that weekend–now happens to be Beckham's boss in the A man whose bank balance is nothing home to write
England set-up. After making public that he had nothing about-he, lived in the social apartment block in downtown
to do with him, tough talking disciplinarian Fabio Capello Villarreal without a club, but so important for his country

ngi
did a most undiplomatically turnaround when he was that they wouldn't dream of taking, the field without him.
forced to re-induct Beckham into the squad as ,his team
As the year faded, Boca had offered him a contract
floundered; now he faces the dilemma again. But for a
which Riquelme had gladly accepted. On the other side,
while, the onerous task, was his predecessor's, the out-of-
depth, Steve Mc-Claren.
As Beckham brought out his best suits to sit on the
nee
the Italian Capello has admitted he's taking English
lessons for his new job. Whether he wants to tell off

rin
Beckham in the Queen's best English, or, welcome him
LA Galaxy bench- ostensibly honouring his $250 million back, some how Ciao sounds so much better. Either way.
(£128m) five year deal - Mc Claren was doing a Capello,

g.n
1. The author here in the passage implies that David
dropping Beckham from the England line-up as they went
Beckham with millions in the banks—
about the task of qualifying for the European champion-
ships next year Futilely it was to prove on both fronts as (A) Earns an enormous sum by his sheer
an inept England superstar/ celebrity side failed to come
to terms with the changed standard in Europe, Beck ham
returned to the England right wing – hilariously much to
the mild consternation of Galaxy who had failed to make
e
scintillating football skills
(B) Performed superbly job for LA galaxy in the
qualifiers for the European championships
(C) Is a better footballer than Roman Riquelme
t
the MLS playoffs. He was to be a bit player in the scheme, (D) Earns more in sponsorship deals than by his
but apart from cornering all the attention, did little as the football genius
most over-hyped football team in the world tumbled out (E) None of the above
of qualifying.
So, Beckham with millions in the bank over the next 2. The author points out to the ironical situation between
five years $ 50m as salary, $50m from club profits, $ 50 sheer talent and celebrity status through which of the
from shirt sales merchandising and $ 100m from his following—
various sponsorship deals which include Adidas, Pepsi, (A) Pure raw talent besides, being the best footballer
Police, Diesel and Rage software, putting his worth for in the world is no match for money churning,
2007 at 11m pounds - was without a country and one goal semi footballer and pretty model celebrity
for his weekend football club (which incidentally brought (B) An out of work, talented footballer is no match
out Ruud Gullit from the wood work to help spruce up for the likes of Beck ham who has earned fame
their shexy shoccer) in the football fraternity
At the same time, south of the equator from the (C) The diverse personalities of the out of form
sunny climes of California, an eerily similar story was back ham and inform Riquelme

562 | CAT Complete Course

Visit : www.EasyEngineering.net
Visit : www.EasyEngineering.net

(D) Requilme was offered sevrals big money con- The rebels within the BJP ranks proved toothless
tracts while Back ham was paid enormous sum when it came to the crunch. The Patel vote did not, as
to endorse big products like Adidas, Pepsi, expected, go against him. The development platform he
Diesel and Rage initially campaigned on has a firm foundation. Sonia
(E) All of the above Gandhi’s ‘merchant of death’ accusation backfired,
wounding ‘Gujarati pride’ which was reinvigorated by
3. The tone of the author in the passage can be best Modi’s Soharabuddin speech. All these, and many more,
described by which of the following word/s ? reasons will be adduced to explain the spectacular elec-
1. Encouraging 2. Ironical toral success of ‘Moditva’, an addition to India’s political
3. Disgusted 4. Didactic lexicon which seems to have come to stay, to the great
glee of Narendrabhai’s supporters and the dismay of his
5. Cynical
equally vehement opponents, not all of whom are outside
(A) 1 and 2 (B) 3 and 4 the BJP.
(C) 2 and 3 (D) 2 and 5 But in this welter of instant analysis and even faster
(E) 3 and 5 comment, when cause and effect often switch place, one
4. Which of the following headings according to the factor can perhaps be identified as the underlying theme

ww
passage would be most apt ?
(A) Win-some lose some
(B) How the west was won
which links these many disparate notes to compose
Modi’s victory march: the yearning for a ‘strong leader’;
a hero, a slayer of monsters. India and Indians-and not

w
(C) Glamour vs Raw talent

.Ea
(D) Pretty face -ugly football
(E) Moderate talent- Model success
just Gujaratis, but all of us–have, ever since the British
decamped, longed for strong men (or women) to lead us
to our manifest destiny, whatever that might be. The goal
(prosperity, literacy, better public health) is not as

1. (D) 2. (A)
Answers
3. (E)
syE
4. (B)
important as the emotional and psychological reassurance
that we are being led, inexorably, unswervingly, without
any of the diversionary debate which robbed us of our

ngi
innate robustness of single-minded energy and enterprise
Passage–38 thanks to the legacy of Nehruvian social liberalism.
Nehru, despite his patrician mien was after all an end
Words–931
The hero is a slayer of monsters. That is not just his
corer competence, but his only competence. According to nee
product of the European Enlightenment, as such he was
the antithesis of the mythic hero, a description more likely
to fit Sardar Patel, the ‘strongman’ that many even today
Roberta Calasso, deconstructor of the cross-cultural
mythological hero, the destruction of the monstrous is not
rin
wish had become India’s first prime minister and rewritten
our post-independence history by doing so. Subhas Bose
an attribute of heroism but its constitution; the very stuff
heroism is made of. It is Hercules slaying the Nemean
lion or the many-headed Hydra; Bal Krishna crushing the g.n
also belonged to the mythic-heroic mould, and even now
commands a near idolatrous veneration and not just in

e
Bengal.
giant serpent; These us killing the minotaur; Rama
vanquishing Ravana.
It is not incumbent on the hero qua hero to be particu-
larly wise, or intelligent, or indeed decisive. In fact, the
Indira Gandhi succeed, but only too well, in giving
us a ‘strong’ leader : Like a true hero, she slew the
t
demons of privilege and pelf by cutting the princely privy
purses and nationalizing banks, and, as Ma Durga,
hero as hero is the least decisive of persons : his killing of ‘liberating’ Bangladesh from Pakistani tyranny. In the
monsters being a form of monomania or an obsessive- end, she met the fate of most heroes who must forever
compulsive disorder. The hero must destroy monsters, created monster to destroy in order to perpetuate their
even when, like a post-modern Beowulf, he might intuit heroic role : she created one monster too many, which
that the monster he destroys may in fact be somewhat less turned around and devoured her.
monstrous than he himself is. At first glance, the ‘monster’ that Modi created, post-
At the moment of his triumph-when he not only Godhra, so as to cast himself as protective hero is all too
bestrides Gujarat like a colossus but casts a prophetic easy to see : the demoniacal ‘other’ the lurking terrorist in
shadow over Delhi-Narendra Modi recalls the great our midst who threatens every single member of the
heroes of mythology as defined by Calasso: slayers of majority community. Naipaul described the rise of
monsters and demons. The many, interlocking reasons Hindutva as a response to the ‘700-year-old sense of
behind Modi’s victory-which totally confounded most revenge’ that Hindus harboured against Muslim
supposedly expert predictions-will be discussed and ‘invaders’, particularly such marauders like Mahmud
analysed till kingdom come, or at least till the next Ghazni who destroyed Gujarat’s Somnath temple. The
government comes. meat-eating Muslim was seen as a carnivorous predator;

CAT Complete Course | 563

Visit : www.EasyEngineering.net
Visit : www.EasyEngineering.net

the ‘grass-eating’ (vegetarian) Hind as effete, herbivorous (C) Fear of being helpless
prey. By ‘semitising’ Hinduism (one people, one book, (D) Fear of invasion
one faith) Hindutva developed an aggressive muscularity.
But it had yet to find its implacable, invincible avenging 4. The line “must preserve the monster in order repeat-
hero (Vajpayee ? Too sweet and too found of jalebis. edly to kill it” finds true repre station in which of the
Advani ? All rath and no wrath; too soft on jinnah) .In following—
Modi and Moditva, the true Saffron Avenger has finally (A) Killing the enemy and preserving the body to
been found. stab at will again and again
Modi’s secret lies in that the demons he has so (B) Make the enemy a friend such that if can be
successfully harnessed to this heroic cause are not out deceived without suspicion
there but inside us, as all true demons are. It is not the (C) Call a true till well-equipped to defeat the
Islamic terrorist or fundamentalist which is the real enemy
bogeyman that Modi has exploited in his voters’ minds. It
is their fear of the fear of the Muslim other that he has (D) Periodically subduing the so called enemy but
addressed: You are afraid to be afraid of Them; don’t not wiping it out in order to keep the fear alive.
worry I’ll always be there to protect you from that subtle
Answers

ww
demon inside you. That’s the real monster that Modi has
created : the fear of the fear of the monster. Which
phoenix like will keep arising from its ashes, because
1. (B) 2. (C) 3. (C)
Passage–39
4. (C)

w
deliverance from it ashes, because deliverance from it can
be achieved not through the intercessionary office of a

.Ea
strongman hero-who actually must preserve the monster
in order repeatedly to kill it-but through a process of self- Do Away With Subsidies
Words–915

syE
exorcism. But that’s difficult to do. Not least because we
like to believe in heroes who, in the end, protect us from
no other monsters but those within ourselves.
“We spend far too much money funding subsidies in
the name of equity, with neither the equity nor efficiency
objective being met”. So said the prime minister during

ngi
1. The author is this passage is concerned with— the course of a recent speech in Delhi. He was repeating
what one of his predecessors, Rajiv Gandhi, had said
1. Megalomania at it diplomat best. several years ago - that only 17 paise out of every rupee
2. Heroes who are concerned with slaying monsters
within.
3. A hero who is exemplary.
nee
spent on subsidies actually reached the targeted
beneficiaries.

rin
Manmohan Singh's remark focuses attention on a
4. A hero hell bent on slaying all those who are malady that has affected our package of economic policies
intent on destroying our national structure. over the years. Successive governments have doled out
(A) 2 and 4
(C) 3 and 1
(B) only 1
(D) a and 4 g.n
subsidies to satisfy some vested interest or other. Perhaps,
some of these subsidies had some initial economic

(E) 3 and 1
2. The enigma of Modi’s stratagem and incredulous
victory will be according to the author ?
e
justification. Often, these subsidies lose all rationale after
some time. But the government of the day simply cannot
muscle up enough courage to remove them because that
would be politically unpopular. Such examples are easy t
1. Talked about decades henceforth in glowing to find. I have written about a couple of them in recent
termed. columns of this newspaper. Subsidised higher education
or the unrealistically low prices of petrol and LPG are a
2. A blunder India will remember. burden on government revenues, with the majority of the
3. Something that will confound experts and beneficiaries being relatively well-off people. And there
predictions. are many others. For instance, the subsidy on fertilisers
4. Analysed at least till power changes hands. has gradually assumed mammoth proportions over the
years. However, rich farmers benefit significantly more
(A) Only 1 (B) 3 and 1
than small ones from the fertilizer subsidy since they buy
(C) Only 4 (D) 3 and 4 substantially higher quantities. Until recently, the urban
(E) Only 3 middle classes were the biggest beneficiaries of the public
distribution system. Many of the really poor do not have
3. According to the passage Mr. Modi is slaying the
any fixed address, and so do find it difficult to obtain
monster within which implies—
ration cards. Even if they have ration cards, they do not
(A) Fear of the muslim other have money to purchase their weekly quota of food grains
(B) Fear of Terrorists on anyone day of the week.

564 | CAT Complete Course

Visit : www.EasyEngineering.net
Visit : www.EasyEngineering.net

Another major reason for the mounting subsidy bill Chronic under nourishment. As far as comparable
has been the support given to those public sector enter- achievements in the health sector are concerned, sector
prises that run at a loss. China surged far ahead of India in terms of infant and
These enterprises would have to fold up unless they child mortality as well as life expectancy.
receive financial support from the central or state govern- For instance, life expectancy at birth in China was68
ments. Of course, the closure of such companies mean year in 1981 compared to only 54 year was 68 years in
that the worker employed in these units would be unem- 1981, compared to only 54 years in India. The same
ployed. differential pattern of achievements was also witnessed in
While some form of safety net to mitigate their education, with literacy figures in China being comparable
suffering needs to be evolved, there is rid rationale of to the most advanced countries in the world, while India
continuing to operate chronically loss-making enterprises. lagged very far behind.
Walking on two legs is always better than limping
Unless the subsidy bill can be drastically pruned,
along on one leg. A balanced approach to development
there is no hope of restoring the fiscal health of the
must, incorporate policies that promote growth and
central and state governments. Perhaps, the Centre will be
prepare adequate safety nets. The latter must involve
in a slightly better position because the booming economy
some targeted subsidy schemes. But there can be no
and better tax compliance have ensured a healthy growth

ww
in central tax revenues. While this ensures some spin-off
benefits to the states through larger transfers from the
Centre, there has been a relatively small increase in the
p1ace for subsidy schemes where the major beneficiaries
can afford to stand on their own feet.
1. What according to the passage would be the most

w
poorer states' own tax revenues. Since these are precisely

.Ea
the states which need to accelerate development expen-
diture, the case for a drastic, reduction in subsidies is
stronger.
suitable and appropriate title—
(A) Subdue-subsidies
(B) Subsidies do not reach the subjects

syE
Of course, it is equally important to realise that all
types of subsidies should not he tarred by the same brush.
(C) Subtle subsidies
(D) Do away with subsidies
(E) Substitute sub subsidies

ngi
Consider, for instance, the recent National Rural Employ-
ment Guarantee Scheme. This is in principle a scheme 2. The author is mainly concerned with which of the
that is targeted towards the rural poor-the relatively well following—
off will not enrol in such schemes. The scheme has come
in for a lot of criticism because corruption has resulted in
some leakage. The correct conclusion to draw from this is nee (A) A majority of state subsidies fail to reach the
targeted beneficiaries
that efforts need to be made to ensure less corruption. If
schemes were to be scrapped simply because of the monitored
rin
(B) Neither equity nor objective are properly

attendant corruption, then practically all government


activity would have to be stopped.
g.n
(C) Bureaucratic corruption throws a spanner in the
efficiency and objective of the subsidies
(D) Some subsidies have some start- up economic
Critics of schemes such as the rural employment
guarantee scheme typically place all their bets on rapid
growth to alleviate poverty. The "growth- only” school
often cites the example of, China to bolster their argu-
ments. They point out that in the post-reform period in
logic
(E) All of the above e t
3. The reasons the author puts forth as to why govern-
ment of the day cannot totally eradicate subsidies is
China, rapid growth has resulted in huge reductions in
as follows ‘EXCEPT’—
poverty levels. They also emphasize that the Chinese
government did not adopt any particularly pro-poor stance (A) It would be a politically incorrect move
in their policies. If anything, there may have been a slight (B) To satisfy and placate ulterior motives
increase in inequality. The incidence of poverty came (C) According to the passage
down because of the so-called “trickle down” mechanism. (D) The government would became highly un-
People from all income classes enjoyed the benefits of popular
growth. (E) The targeted beneficiaries at least get some
What these figures hide is the vastly superior levels benefits
of achievement in the social sectors in China before 1979,
4. The reason why China has fared better than India is
when the current phase of reforms was initiated. In the
which one of the following:
yearly'50s, India and China were at more or less
comparable levels in terms of most social indicators. But, (A) A slight increase in inequality
by the late, 70s, China had pulled very far ahead. China's (B) Because of a so called trickle down mechanism
poor were much better fed, with a drastic reduction in (C) Superior levels of achievement in social sectors.

CAT Complete Course | 565

Visit : www.EasyEngineering.net
Visit : www.EasyEngineering.net

(D) The Chinese marched ahead in education. suggested restructuring the group to address the global
(E) The Chinese did not adopt any pro-poor stance. opportunity better.
"Tata identified the theme of going global very early
Answers on, but his initial judgment was that the group was not yet
1. (D) 2. (A) 3. (E) 4. (C) ready to move on to this agenda," says Alan Rosling, Ex-
15. Lengthy Type Reading Comprehension ecutive Director of Tata Sons. Rosling was hired person-
ally by Tata in 2003 to lead the group's drive to interna-
Passage 1 tionalize. "To begin with, Tata focused on competitive-
ness. We have to earn the right to survive, he would say.
Words–1672
Only when he judged that the group had moved to this
"IT is a statement I feel quite awkward in acknowl- position did he decide to stand up in 2003 to spell out the
edging”. That is Ratan Naval Tata's immediate reaction international agenda "Rosling adds.
when he is told that he has been voted India Inc's Best
Brand Icon in the Outlook Business-CNBC Universe Poll- A Man Of Ideas
the result marks him out as the flag bearer of India's Managers who have worked with Tata closely say
global aspirations. The response is not at all uncharac- that he has identified many such themes. Internation-
teristic of Tata. He is an extremely private person, and yet

ww
alisation was one. The push to hire young mangers at
one who has managed to transform the Tata’s from a Rs significant, decision-making levels across the group is
13,000-crore asynchronous group into a Rs 129,994-crore another. The entire focus on the bottom of the pyramid-be
cohesive global organisation in 16 years at the helm. "All

w
it the one-Iakh car, budget hotels or low-end watches-is
that I have been doing, in my own little way, is to make his idea. So is the focus on research and development.

.Ea
the group more competitive and bold," he says.
Quite true to his nature, that also is an under-
"No doubt, he has been a big influence on the group in
the last three-four years,” says R Gopalakrishnan, Execu-

syE
statement. In the last few years, the group has wrapped up tive Director of Tata Sons Rosling says, "he is a deep
35 cross border acquisitions, including same mind-blow- thinker, extremely strategic and long term. He is always
ing large deals that are now much too famous to require a two-three moves ahead."

ngi
mention here. International revenues now account far And once he has identified a theme, he often leads by
over 50 %of group revenues. (That figure was in single communication. He employs a very consultative style in
digits when he had taken charge.) And if there is one big seeding these ideas or themes into group companies. He
reason why corporate decision makers judged him to be
India's Best Brand Icon, it has gat to be his success in
taking the group global. "Both Indian and global stake- nee
encourages people to open their eyes to look at an
opportunity and gets them to think differently about
issues. but he will never tell them what to do. Often, he
holders are increasingly seeing us as a group that is
making big plays, taking more risks than it was known to
rin
communicates by asking questions. "Why can't you" or
"have you thought about this"- those are common phrases
do in the past and managing large global takeovers with
reasonable grace and finesse and, hopefully, success,"
Tata says. g.n
he employs. He will ask you questions that will lead to
the theme. Rosling calls this the Tata way of "socialising
ideas." Tata never imposes and never demands that
The mind of A leader
There are many facets to Ratan Tata's global game
plan his thought leadership-in identifying the need to go
e
idea, discusses and debates it and then allows managers
t
people fall in line with his beliefs. Rather, he floats an

to come up with what they would like to do about it. "He


has had significant personal impact on the way the group
global very early on; his wisdom, in waiting to make the has changed and internationalized, but he has done it
group more competitive before going in for the interna- through colleagues. That is what leadership is all about,"
tional push; his skill as a leader in making this theme says Rosling.
resonate all over the group; the aggression with which he
Be Bold
has won some of these cross-border deals; and his
unshakable resolve never to compromise on the ethics Admittedly, Tata has also been trying to increase the
and values that the group has cherished for over 100 years "dare quotient" of the group. He has been nudging his
now. managers to be bold in their planning. This relatively new
The earliest evidence of Tata's thought leadership facet is perhaps best summed by the mindset with which
could be found in a document unofficially called the Tata Tata walked into the Corus auction. "Tata went into the
Plan that he authored way back in 1983. Under the auction with the intention to win," says Gopalakrishnan.
leadership of JRD Tata, the group got Ratan Tata (then "You are now beginning to see that attitude being
Tata Industries' Chairman) to draw up a blueprint for the reflected in group companies," he adds.
future. In it Tata recommended that the group 'seek sub- Tata encourages aggression among group mangers in
stantial growth in international operations.' He also many ways. To begin with, he is always encouraging

566 | CAT Complete Course

Visit : www.EasyEngineering.net
Visit : www.EasyEngineering.net

companies to think big and be bold enough to attempt the announced that the holding company would pump in the
impossible. When such thinking leads a company to a required money to prevent Tata Finance deposit holders
cross-border deal, he makes himself available 24x7 to the or shareholders from suffering any loss. "By that one
CEO doing the acquisition. "When you come to him for a action, he gave a message that is far beyond all the
critical decision-which will always be in some negotia- speeches he could give in the next 10 years," he adds.
tions-he will give you a very quick answer,' says Rosling. Tata lives by these high standards in the international
His responses would be crisp, leaving no room for doubt. arena, too. Before a meeting with the Prime Minister of a
The answers would be something along these lines: "Yes, significant country, a senior group official suggested Tata
I agree that we should offer this price” or "yes I agree we lobby for a Specific proposal that could help the group in
should withdraw" (the group has, done that on occasions.) that country. “Tata declined,” recalls the official who
"His involvement in cross- border deals could be quite made the request. “Unlike global CEOs, who never
significant," says Rosling. And that's precisely what gives hesitate to lobby with governments, Tata seldom asks
the CEOs the confidence to move ahead without doubts. governments for specific favours,” he adds. This despite
Hands Off the fact that Tata is growing in global stature - among
other things, he is an advisor to South African President
Yet, in all this Tata never comes in the way of a Thabo Mbeki, the British government and Singapore's
manager functioning. Yes, he might step in to make a

ww
broad strategic adjustment, but he does not interfere in
operational issues. Only if his help or input is sought in
something specific does he come into the picture.
Economic Development Board on international invest-
ment related issues and he is on the board of Mitsubishi
Corporation, American International Group and J.P.

w
That can be said of his involvement in the global

.Ea
acquisitions as well. He is present as a member of the
leadership team; he is not there in managing the process.
Morgan Chase. "He has a very positive global stature,"
says Gopalakrishnan. "That does help group companies."
Many believe that Ratan Tata's global leadership is now
reaching iconic proportions, at least among his Indian

syE
He is available to CEOs as a sounding board, or to give peers. Like all icons in big business, his personality is
advice. "That was precisely his role in the Corus acquisi- beginning to reflect on the group's reputation.
tion," says Rosling. B Muthuraman, Tata Steel Managing Again, true to his nature, rather than bask in the

ngi
Director was running the process, Arun kumar Gandhi, glory, he chooses to deflect the credit and attention
Executive Director, Tata Sons was in the negotiation and
showered on him away from himself. "I have spent a lot
the bidding, Tata was there only to help with key deci-

nee
of time and energy trying to transform the group from a
sions based on Muthuraman's recommendations.
patriarchal concern into an institutionalised enterprise. It
Tata extends a similar philosophy into the way global would be a mark of failure on my part if the perception
acquisitions are managed and integrated into the group.
Cultural compatibility is one big area in which due
diligence is done before starting work on any cross- rin
gained ground that I epitomise the group's success," he
says. Very few global business leaden are as selfless.
border deal. "This ensures that they are, in a manner of
speaking, inclined to be in the Tata groove more readily,"
g.n
1. The author primary concern in the passage as for as
Ratan Tata is concerned is—
says Tata'. He prefers a non-prescriptive approach. "We
do not take a William the conqueror approach to cross-
border acquisitions," says Gopalakrishnan. "I have signed
the cheque. So I am here to tell you how to handle things
in the future -that's not what Tata believes in," he adds.
e
(A) Ratan Tata being named India Inc’s Best Brand
Icon in outlook Business CNBC universe poll
(B) Ratan Tata’s skill wisdom combined with his
thought leadership method which made his
t
company a global success
Man Of Integrity (C) Tata’s values and characteristic, chiefly
A key issue that ensures cultural competency is accounts for his success
ethics. This is where Tata has never diluted his value (D) Ratan Tata having acquired 50% of group
system. "I saw him stand by his principles, even though it revenues in International acquisitions
cost him entry into the lucrative airline business," recalls
(E) None of the above
a senior official of the group, referring to the jinxed Tata
Airlines Singapore Airlines proposal of the late '90s. 2. The title that would suit the passage beautiful and
"Tata has shown that there is no other way he will do aptly—
business other than do it ethically," says Gopalakrishnan. (A) Run Run Ratan Tata Internationally
He points to the Tata Finance episode (financial (B) Tata invites Globalisation
irregularities by senior company officials had led to the
(C) Global Leadership The Ratan Tata way
loss of few hundreds of crore) as an example. At that
time, when the loss was yet to be ascertained (estimates (D) Strike while the Iron is Hot
ranged from Rs 500 crore to Rs 1,000 crare), Tata (E) Ratan Tata, the man of steel

CAT Complete Course | 567

Visit : www.EasyEngineering.net
Visit : www.EasyEngineering.net

3. The various sides to Ratan Tata’s global game plan important one being the reconstitution and widening of
are all of the following ‘EXCEPT’— the scope of the Standing Committee of the Board, which
(A) His thought leadership which identifying the looks into matters concerning clearances for mining,
requirement to go International industry, roads and other development projects in
ecologically sensitive areas, including protected areas,
(B) The need to wait and make the group more and which had been reduced to a rubber stamp and a very
competitive prior to going global. diligent one at that. It was also decided to create a Tiger
(C) The skill to make his theme resonant thought Protection Force and urgently appoint a senior police
the group officer to head the dysfunctional Wildlife Crime Bureau.
(D) Unshakeable resolve once committed Many focused sub-committees were appointed–one
(E) Never to back down or compromise on ethics or for the affairs of the tiger, another for the conservation of
values except when millions are at shake other equally endangered species like the hangul and
snow leopard, and one dedicated to the conservation of
4. Ratna Tata is a team player who encourages his
marine species. A legal sub-committee to examine pro-
mangers perform at their best. The best examples are
posed amendments to the Wildlife (Protection) Act, and
as follows. All except one are false
any other legislation that may have an impact on forests
(A) The drive to his young manger

ww
(B) The been forces on research & development
(C) The employment of a very consultative style
and wildlife, was also proposed. The Prime Minister also
promised to make a special effort with State Chief
Ministers to ensure the implementation of vital wildlife

w
(D) He demands that people fall in line with his
beliefs

.Ea
(E) He allows his managers to debate, discuss and
come up with solution to an idea he has brought
initiatives.
This was good, good news. A dream run. A positive
meeting, with an outcome and decisions that actually held
the power to-if not halt-at least mitigate the absolutely
up
Answers syE catastrophic situation of wildlife in India. One should
have been glad. Or at least cautiously optimistic. I was,
brushing aside my innate cynicism and worrying that
1. (B) 2. (C) 3. (E)
Passage 2
4. (D)
ngi there was a huge gap between intent and implementation.
We celebrated too soon. First came the news that the
minutes of the meeting were being tweaked and hijacked
Words–1973
The Forest Rights Act will come into forces on nee
by the MoEF to suit their interests. Then came another
shock .
January 1, 2008. And on that sombre note the sun will
rise on the dawn of the New Year, unknowing that the
rin
To elaborate, one of the most vital decisions taken at
the meeting was the appointment of a committee that

g.n
earth it bathes in its life-giving rays has inched a little would look at the adverse impacts of the Forest Rights
closer to its death. The Act being yet another blow Act on forests and wildlife (which have been well-
delivered on an already-Fractured scarred planet. documented and established, but put in a nutshell, the Act
For the past three years , I have been writing an
overview of wildlife issues that have dominated the year
in this columm. Each year, I hope to strike an optimistic
note in this season of goodwill and cheer; yet, on the
e
will override all wildlife legislation and open up new
channels for destructive commercial exploitation, and
fragment wildlife habitats, including protected areas)
The committee was to suggested mitigating measures
t
contrary , the despondency only grows. This time I had before the rules are notified. The Prime Minister rooted
hoped that pessimism would take a back seat with the for this, even as the MoEF secretary, who in her previous
outcome of the National Board of Wildlife meeting that stint served in the Tribal Affaires Ministry, opposed any
took place in November 2007. such move.
After months of dilly-dallying, a board comprising How do we have the faith, Mr Prime Minister, when
members known for waging battles for wildlife and not more than a month after you promise that the impact
taking on the Government- including the current one-was of the Act would be studied and mitigating measures
formed. After much delay-and attempts by the Ministry considered, the Act will now come into force-post hast-
of Environment and Forests (MoEF) and the Prime from January 1, and tigers be damned?
Minister’s Office (PMO) to preempt discussion on any One can understand, of course. This was political
critical and sensitive subject-its meeting took place on double-speak at its best. It is the politics of votes that
November 1. holds sway, not conservation. Why would the UPA delay
Contrary to expectations, the outcome of the meeting the Act when its chairperson, Sonia Gandhi, uses it as her
was a pleasant surprise. The prime Minister took a keen trump card while canvassing for tribal votes? Why would
interest, and many positive decisions were arrived at, an they not bring it into force when the CPI(M), without

568 | CAT Complete Course

Visit : www.EasyEngineering.net
Visit : www.EasyEngineering.net

whose support they lose power, puts pressure to enforce In another macabre fallout of the man-animal con-
the Act ? They won’t. Not with elections around the flict, a bear and her two cubs were axed to death by the
corner. Most political parties are squabbling over “taking irate residents of Bedar village near Poonch in Jammu &
credit” for the Act. The UPA chairperson, in her capacity Kashmir on October 28. This was yet another bear
as Congress president, campaigned in Gujarat promising bearing the brunt of deforestation, and consequent man-
tribals that they would get land under the new Act. animal conflict in the State. Of leopards badgered, burnt
The Gujarat government also handed over pattas to and killed across the country, one has lost count.
30 tribal families, promising 2,204 tribals that they would The tiger, of course, has not been the only species
get their share soon till–fortunately–it was restrained by that was the target of the poacher’s gun. This year also
the Supreme Court. In the Gujarat election battle ground, saw a breach in the excellent protection according to
the Forest Rights Act became an instrument to woo the rhinos in Kaziranga over the past few years. Seventeen
vital tribal vote bank. Greater one-horned rhinos fell to the poacher’s gun in
It is worse in Andhra Pradesh. The Left parties, along 2007. Another victim was the beleaguered Asiatic lion.
with hundreds of locals armed with axes, entered and Barely 325 of them survive in the world today, and 17
vandalized Kawal Wildlife Sanctuary, indiscriminately were lost from January to April-half of them to poaching.
felling trees. Around 631 acres of thick forest were In earlier years, lions have been killed over man-animal

ww
destroyed and encroached upon. The rationale behind this
devastation was the anticipation of the Forest Rights Act.
This sets the tone for the coming year. The battle
conflict, and the odd one to feed the domestic market for
its claws. But the spate of killings this years was
different-the lion was now being slaughtered and sold in

w
ahead has only become tougher. And while this piece of

.Ea
news almost overwhelms other issues, let’s do a quick,
and by no means exhaustive, assessment of the issues of
the past year.
the animal markets of China and the Far East in the same
way the tiger is-for its derivatives to be used in traditional
medicine.
On December 13,65 skins were seized-one tiger skin,

syE
Let’s begin with tigers. Now we know–officially-that
there are fewer than 1300 tigers in India. And declining.
The tiger mortality for just this month is indication
21 leopard skins and 43 otter skins-in northern Karnataka.
All this only prompts a person to ask one question—
Where is the long promised Wildlife Crime Bureau ?
enough of the extent of damage.
A tiger was injured, and met an agonising death, after ngi And what does one do when the enemy is within ?
According to reports, the Chhattisgarh Forest Depart-
being hit by a speeding vehicle on the road that cuts
through Dudhwa Tiger Reserve in Uttar Pradesh on
December 4. nee
ment has come up with a plan to cut down over 20lakh
old-growth trees and replace them with monocultures of
teak and eucalyptus. This has already started in Sarguja
Only one day prior, three tiger skins and three
skeletons were seized in Allahabad. Going back a little, in
and Koria districts.
rin
Another disastrous decision that escaped public eye
the space of a month, two tigers were caught in iron traps
of the heart of India’s ‘best managed’ tiger reserve,
Kanha, while 20 such traps were recovered. g.n
was the denotification of the Saraswati Wildlife Sanc-
tuary, a rare haven in Haryana of about 200 hog deer and

India’s ‘Tiger State’, Madhya Pradesh, isn’t safe for


tigers either: their numbers have officially halved to a
mere 300. And Panna, we regret to announce, is almost
another Sariska, with no sign of a tigress in the park for
e
other species. They wanted to build a canal through it,
and when that was proving difficult because of the laws,
the sanctuary was denotified. In another part of the
country-Madhya Pradesh-the forest Department wants to t
start trophy hunting. How they propose to issue licenses a
the past six months. Sightings, pugmarks, scat and other country where the state of wildlife monitoring and the
clues that point to a tiger’s presence are getting thinner by enforcement of its laws is abysmal is not known.
the day. But the Forest Department valiantly assures that Another very distressing story was the proposed
30 tigers are in the reserve. thermal power plant in Chamalapura in south Karnataka.
Expectedly, man-animal conflict has been increasing. This borders Bandipur Tiger Reserve and Nagarhole
And as we encroach on, destroy, degrade and fragment National Park-prime tiger habitat, and home to other rare
habitats, the conflicts only escalate. Recently in Talodhi, flora and fauna. The plant is a disaster- it will require a
near the Todaba Andhari Tiger Reserve, there were eight million litres of fresh water from the Kabini daily, and
tiger attacks on humans-four of them fatal. So the tiger will discharge huge amounts of fly ash, sulphur dioxide
was declared a man-eater, and shot. Yet the killings nitrogen dioxide and carbon dioxide. The impact will also
continue. Another man lost his life after they slaughtered be felt in Mysore, 35km away. It will also displace over
the supposed man eater. Was this a tragic comedy of 20,000 people. The Karnataka Government seems to be
errors ? The stage is now set of course for another tiger to on a suicidal path-they have proposed a network of 73
be the target; and we will lose yet another big cat from mini hydel projects across the dense forests in the Western
the rapidly dwindling population. Ghats. The destruction is evident in the Kempholey

CAT Complete Course | 569

Visit : www.EasyEngineering.net
Visit : www.EasyEngineering.net

Reserved Forest, Where three major roads and a concrete (B) The P.M. took an eager interest, and several
weir dam has been constructed in the biodiversity-rich positive decisions were made
area. Unfortunately, there is no dearth of such destruction: (C) The reconstitution and broadening of the stand-
construction of ports and shipping channels threaten ing committee of the board
myriad marine species and roads, railway lines and canals
(D) The creation of a Tiger Protection Force
cut through protected areas. The list is endless.
(E) Sub-committees were formed for the affairs of
But there has been good news as well. This year saw
the tiger and the conservation of other equally
the discovery of some new wildlife species.
endangered species like the Han gal and show
A new bird Liocichla bugunorum, a kind of babbler, leopard
was discovered in May in the Eaglenest Wildlife Sanc-
tuary in Arunachal Pradesh. Two new caecilians-impor- 3. When the author mention that it was “political
tant indicators of healthy ecosystems and the least know double–speak at its best” he implies that:
and studied of the amphibians-were discovered in the (A) The MOEF tweaked and hijacked the meeting
Mahadeyi forests of Goa. These are the Gegeneophis minutes
goaensis and Gegeneophis mhadeiensis. Incidentally, the (B) The Wildlife Act was just another political lie
region it was found is now threatened by a proposed dam.
(C) The Forest Act was merely a pawn in the game

ww
A new species of spider, yet unnamed, was found in
Melghat. All of which only serves to underline the
importance of preserving our last remaining wild habitats.
of politics to be used as a deceptive trump card
to gain tribal votes and please their coalition
partners

w
Viable tiger populations and habitats have also been

.Ea
mapped-Corbettrajaji, Karanataka-Kerala-Tamil Nadu
which includes the protected areas of the Bhadra-
Bandipur-Bhimgrir-Nagarhole-Madumalai-Wayanad cor-
(D) The MOEF and the PMO transpired to destroy
all hopes saving Wildlife
(E) None of the above

Tadoba-Satpura-Pench-Kanha.
syE
ridor, and the Satpura belt whichb connects Melghat-

These need to be protected against all odds, and kept


4. The tone of the passage can best be described by
which of the following—
1. Cynical 2. Instructive
free from any biotic pressures and encroachment. There
are solution as has been demonstrated in Bhadra and
Melghat Tiger Reserves, and Rajaji National Park, where ngi 3. Disheartening
(A) 1 and 2
4. Approving
(B) 2 and 4
forest dwellers have been successfully rehabilitated in a
painstaking process. nee (C) 1 and 3
(E) 1, 3 and 4
(D) All of the above

Finally, it was Al Gore and the International Panel


for Climate Change jointly winning the Nobel Prize for
1. (A) 2. (A) rin
Answers
3. (C) 4. (C)

g.n
Peace, indicating that we had finally taken in the horrific
impacts of global warming, and recognized those who Passage 3
worked for its awareness and showed the path for

e
Words–1371
mitigation .
We cannot reverse the clock, cannot wipe away the
horrors on the earth. But surely we can start to heal.

day.
Else soon in the future, we do not live to see another
It’s 10 in the morning, and at the historic Azad
Maidan, where The Mahatma used to address swelling
crowds during the Civil Disobedience movement, another
movement is slowly gathering steam. It's Mumbai's first
t
major protest rally against corporatisation of the retail
1. Choose the most suitable and appropriate title from sector. 'Mike testing' is on as a sprinkling of protestors
the following— waits patiently under the sweltering sun. There are more
(A) Forest Act, Jungle Law television reporters and cops (two truckloads of the latter)
(B) Wildlife Sorry State than protestors waiting for the action' to begin. In sync
(C) Cat Act –Reaction less with the historic nature of the venue, there are banners
bearing the images of the founding fathers of the country-
(D) A Catty Affair
Gandhi, Jawaharlal Nehru, Bal Gangadhar Tilak, Subhash
(E) Cat and Mouse Politics Chandra Bose and even Bhagat Singh. The organisers of
2. Why is the author dismayed and saddened by the the event had 'promised' 50,000 protestors but less than
Forest Act which comes into force on 1 Jan., 08 ? 50 are around. Half an hour later, 50 more join exerting
(A) The Act will provide loopholes in all wildlife their vocal cords. Just as the camera crews were getting
legislation leaving it wide open for destructive restive, the 'action' begins.
commercial advantage in addition to scattering Almost out of the blue, they came-petty shopkeepers,
wildlife habitats protected and unprotected hawkers, pushcart vegetable vendors, chemists, onion

570 | CAT Complete Course

Visit : www.EasyEngineering.net
Visit : www.EasyEngineering.net

merchants and manual labourers. They came packed in and 12 states, since its launch in November 2006. The
buses and trucks, in motorcycle cavalcades from all over Bharti-Wal-Mart alliance is yet to start operations and the
Mumbai and nearby districts like Thane, Pune, Raigad. first store may be launched only in the latter half of 2008.
Passengers in Mumbai Other retailers like the future Group and Subhiksha
local trains complained of surging crowds and the that are rapidly expanding across the country have so far
occasional unruly behaviour. At the venue, it was 'byte managed to escape the wrath of the protestors. "We are
hunting' time. A local leader spews hyperbole and spittle, not into the fresh vegetables business in a big way. Also,
in equal measures. "East India Company came to India as as a policy we do not sell anything below the cost price,"
a trading company and then they took over the country. says Future Group CEO, Kishore Biyani. Chennai-head-
Today, we are an independent country and are inviting quartered Subhiksha Trading Services has faced protests
MNCs like Wal-Mart to take over the country," he in the past from chemists who claim that the retailer has
proclaims. been selling drugs at prices lower than the market rates.
But that was just the beginning. Nearly two dozen Subhiksha Managing Director R. Subramanian, however,
people mount the makeshift dais, including the anti-retail is confident about his expansion plans, including UP
movement "champion" from Uttar Pradesh and where the retailer has 60 outlets. "At the end of the day,
Samajwadi Party MP, Banwari Lal Kanchal. "Why do organised retail is a great thing for consumers. Of course,

ww
these companies that own
refineries and sell cloth, petrol and cellphones want
to sell onions and potatoes? thunders Shyam Bihari
we have to take care of all the stakeholders involved,"
says Subramanian.
It's a point that RIL Chairman Mukesh Ambani was

w
Mishra, a former BJP MP in a not-so veiled reference to

.Ea
Reliance Industries and Bharti Airtel. As the decibel
levels of the speakers soar, impassioned zindabads fill the
air. "People ask me why we use violent methods," asks
at pains to point out at the company's recent AGM. "Our
organised retail initiative is configured to increase income
in the hands of the farmers and serve consumers by
improving supply chain and distribution efficiency. We

syE
Kanchal who trashed- Reliance Fresh stores in Uttar
Pradesh. "Killing somebody is violence. Breaking the
door of a shopping mall is not violence. Until you break
want to achieve these twin goals by reducing wastage,"
he said. Considering that 30 to 35 per cent of 60 million
tonnes of fruits and vegetables in the country-in value

to you," rationalises Kanchal.


ngi
their malls and burn their goods, nobody is going to listen terms about Rs 58,000 crore and more than the fresh fruit
produce of the UK-are wasted due to a lack of storage

nee
By the time the speeches came to end, more than and other facilities, organised retail is not the evil
5,000 people had participated in the protest-a far cry from business it is being made out to be. "At the end of the
the 50,000 mark but still ominous enough. The protest day, India's retail market is so huge that there is place for
has been organised by Vyapar Rozgar Suraksha Kruti
Samiti, in association with a dozen other associations Subramanian.
rin
everybody, including smaller players," says Subhiksha's

g.n
such as Federation of Association of Maharashtra (FAM) The Indian retail sector is estimated to be around
and NGOs like India FDI watch. These bodies are $328 billion, with less than 4 per cent being accounted
protesting a list of things-from the entry of corporate in for by organized retail. Subramanian believes that the
retail to Foreign Direct Investment (FDI) in retail to
implementing a national policy on hawkers. Another
demand of the protestors is scrapping the Model APMC
Act. Like the President of Federation FAM, Mohan
e
present conflict is one largely caused by perception.
"When a retailer says - that they are going to make
investments to the tune of several thousand crores of
rupees, it raises everybody's eyebrows. That raises the
t
Gurnani, says: "We want the amendments to the fear levels of some of the smaller players," adds
Agricultural Produce Marketing Committee Act that al- Subramanian.
lows corporate to buy products from farmers and sell
them directly to the consumers." It's a problem that Ambani seems to be conscious of.
"We are sensitive to the interest of small shopkeepers.
Big is Bad Our retail initiative will in no way jeopardize their in-
Having said that, the protestors' ire seems to largely terests and that of small vendors who service customers,"
focus around the two corporate - Reliance's Fresh outlets said Ambani, who also cited the company's purchases of
and Bharti's tie-up with the international poster boy of banana crop as an example of how Reliance retail venture
Big Bad Retail Business, Wal-Mart. Reliance has had its could benefit farmers. "Reliance is bringing to farmers, to
stores vandalised in Orissa, Uttar Pradesh and Jharkhand. begin with, in Gujarat, Maharashtra and Andhra Pradesh,
In UP, the company was forced to down shutters on all its high quality tissue cultured banana plants that yield 35 to
23 Reliance Fresh outlets and let go of 235 or so em- 40 kg per bunch of fruits as against 20 with conventional
ployees. Protests have also been held against Reliance cultivation Reliance buys these banana from farmers at
Fresh in West Bengal, Madhya Pradesh, Jharkhand and prices that are 10 to 15 percent higher than what they get
Kerala. In all, Reliance has opened 300 stores in 30 cities through conventional channels headed.

CAT Complete Course | 571

Visit : www.EasyEngineering.net
Visit : www.EasyEngineering.net

But the problem for players like Reliance might lie 3. In the last paragraph of the passage the author
beyond just flawed communication or perceptions it’s suggest all of the following EXCEPT—
that ugly word called politics The problems that unlike 1. In the event of mid-term polls such a contentions
most of its other business the retail business opens up a and emotive issue will not be solved in quick
thousand pressure points for reliance as it involves a lot time.
of on-the–ground activity. And people (read politicians)
2. A nationwide stir involving violence & destruc-
sense an opportunity to pressurize the company and make
tion of malls and retail outlets will follow is
money” says a senior executive of a retailing group,
demands are not met.
requesting anonymity.
3. The consumers, farmers and small time shop-
Observe have been surprised by the lack of serious
keeper be the brunt of the agitation.
support for Reliance from other retailers. “Earlier, the
industry lacked consensus and bickered on key issues like 4. Reliance will eventually beat the odds.
FDI. Reliance took a tough stance against FDI in retail 5. The issue is only a storm in a tea cup
claming that it would harm the domestic retail, especially (A) 1 and 3 (B) 2 and 3
small traders. Now they cannot just turn around and ask
(C) 4 and 5 (D) 1, 4 and 5
for support from other players or the retail industry body.
(E) 3, 4 and 5

ww
As for the small traders he does not care who he is killed
by- the MNC or a large Indian corporate,” says the head
of another retailing major. However, industry association
4. What are the benefits of organised retail according to
Mukesh Ambani CMD of Reliance industries ?

poor. w
CII has condemned closure of Reliance Fresh stores in
UP and warned that such orders will adversely affect the

.Ea
For now the protesters have set a deadline January
1. The farmer will earn less that smaller players.
2. Farmer and consumers will benefit.
3. Improvement and efficiency of supply &

syE
distribution.
26, 2008.If by then the government does not heed to their
demands, they plan to take the stir nationwide and do not 4. High quality issue cultured plant to upgrade
yield double fold.
rule out trashing malls and retail outlet with mid-term

ngi
polls still likely, this is one emotive issue that will not die
silently. Unfortunately in the process, India–farmers,
5. Wastage can be reduced enormously through
proper storage & like facilities.
consumers and even small shopkeepers included–will lose
time and an opportunity to benefit.
1. The most appropriate and suitable title for the passage
nee (A) 1, 3 and 4
(C) 1, 2, 3, 4
(E) 1, 2, 4, 5
(B) all except 1
(D) 1, 3, 4, 5

would be—
(A) Retail rage rin
Answers

(B) Much ado about nothing


(C) A storm in a tea cup
1. (A) 2. (A) 3. (C)

g.n
Passage 4
4. (B)

(D) Politicians go vegetarian


(E) Small price-big scale
2. The author when he quotes an anonymous sr. execu-
e Words–778

t
Not until a year later, however, in March l974, did
the Pentagon finally admit to having deemed it necessary
-if not nice-to fool with Mother Nature over Laos, North
tive writes, “The problem for player like (Ambani)
Vietnam and South Vietnam, from 1966 through 1972.
Reliance might lie beyond communications or per-
Defence department officials made the admissions at a
ceptions. It’s that ugly word called politics” unquote,
briefing of the Senate Foreign Relations Committee.
it implies that
They said that cloud seeding project-in connection with
(A) The company will be exposed to pressure which the emplacement of electronic and chemical sensing
will open the gateways to bribery devices-had succeeded stanching North Vietnamese
(B) Politicians can use it as then U.S.P. for the infiltration down the Ho Chi Minh trail especially in the
forthcoming electrons summer of 1971. But they denied allegations that their
cloud seeding had been responsible for the devastating
(C) Small retailers will take advantage of the
flooding of North Vietnam in the fall of that year. The
situation and yell “Autocracy of Retail”
Pentagon people pointed out that cloud seeding had been
(D) Ultimately it is the protestors themselves who the object of civilian R and D for many years, and that the
will suffer military had simply found it to be compatible to be the
(E) Politicians are braying for reliance’s blood as cause of the war. What they did not emphasise was that
they are jealous of his power and wealth "technology" has now proceeded to the point that not just

572 | CAT Complete Course

Visit : www.EasyEngineering.net
Visit : www.EasyEngineering.net

rainshowers but torrents can be triggered : that entire prostitute the elements, they stood to gain. If they were
continents can be targeted for catastrophic cloudbursts. ahead, the enactment of prohibitive international law
During the senate debate of 1974 on the military would permit them to stay ahead. If they were behind-as
procurement bill for fiscal 1975, Senator Gaylord Nelson was probably the case-they could simply break the law
of Wisconsin introduced a floor amendment prohibiting and work to catch up. The latter hypothesis presupposed,
weather modification as a weapon of war. The senate of course, that the Pentagon would not break the law and
passed it, but it was deleted in conference with the House. would disband its R & D.
A short time later, the Senate Armed Services Committee So far, there is nothing at all to suggest that either the
asked Dr. Stepher J Lukasik, director of the Pentagon's US or the Russians or the Chinese have halted their
Advanced Research Projects Agency (ARPA), whether weather experiments. While it does not mean that the
the Defense Department was pursuing any R & D that longest drought that India faced at the turn of the decade
might be contrary to the intent of the Nelson amendment. was not because of the EI Nino effect but because of the
"There is no such work in ARPA", said Dr Lukasik, S-curve in a wave unleashed by a Chinese "Weatherman"
"but I am not familiar in detail with the totality of missile, it does mean that by the end of this decade it
activities in the defence department. To the best of my could really happen.
knowledge, there is nothing going on in the department 1. The term "ploy” in the passage specifically refers

ww
that is in conflict with that amendment." The committee
then asked Lukasik to look into the matter and provide a
statement for the' record. T' statement, when it came, was
to—
(A) The US undertaking.
(B) Prohibitive international law.

w
a masterpiece.

.Ea
“Department of defence research in weather modifi-
cation is entirely unclassified and does not involve any
unique techniques not known to the civilian community.
.
(C) A typical Russian strategy.
(D) Pentagon's defence R&D.
2. The author is least likely to agree with the statement

syE
Research is pursued to develop means for protecting
military personnel and resources and to prevent techno-
logy surprise, this research is thus not contrary to the
that—
(A) On the military procurement bill for fiscal 1975,

ngi
senator Gaylord Nelsons introduced an amend-
intent of the amendment referred to”. ment prohibiting weather modification as a
What it comes down to was that the Senate committee weapon of war.
actually had no power to force the Pentagon to account
fully for its actions with respect to an amendment which
had not become law. And the Pentagon did not. It is nee (B) Dr. Lukasik of ARPA was asked whether
Defence Department was pursuing any R&D
work contrary to Nelson amendment.
questionable in fact, that the Pentagon would have
responded fully even if the amendment had become law.
rin
(C) The Senate Committee had no power t o force

g.n
There is plenty of precede for the Pentagon not having the Pentagon to account for its actions in respect
done so. of an amendment which had become law
(D) It is questionable that the Pentagon would have

e
In mid-October 1974, the Soviet Union introduced a
responded fully even if the amendment had
little noticed draft convention at the United Nations
"banning the modification of the environment and the
climate for military and other purposes incompatible with
the interests of international security, the well-being and
health of people”. More specifically, the Soviets proposed
become law.
3. The term "engendering" in the passage specifically
refers to—
t
(A) Electromagnetic and acoustic fields
outlawing all modifications by man of "the surface of the
land, the floor of seas and oceans, the earth’s interior, (B) Inducing precipitation or redistributing water
water, the atmosphere or any other element of the natural resources
environment". This would include, they said, cloud (C) The act of inducing earthquakes
seeding or any other means of inducing precipitation or
(D) Gas exchanges between the hydrosphere and
redistributing water resources : artificial engendering of
atmosphere
seismic waves : creating electromagnetic and acoustic
fields in the ocean : disturbing the natural heat and gas 4. The authors personal opinion about the statement of
exchanges between the hydrosphere and the atmosphere, Dr Lukasik is—
or the heat al radiation balance among earth, atmosphere (A) Extremely sarcastic
and sun.
(B) Extremely positive and benevolent
To the Pentagon, this was nothing more than a
typical Russian ploy. Whether the Soviets were ahead or (C) Very cautious and circumlocutions
behind in the research and development of ways to (D) None of the above

CAT Complete Course | 573

Visit : www.EasyEngineering.net
Visit : www.EasyEngineering.net

Answers the most inspiring of days. Taking a cue from the


1. (C) 2. (C) 3. (B) 4. (A) Americans whom Jindal found to have a weakness for
raising their national flag, Jindal sought to do up his own
Passage 5 company building in Raigarh with the tricolour in 1992.
Words–1226 The result was anything but mild.
Naveen Jindal, 37, gives off a quiet aura. Yet, beneath The next thing he knew the government had slapped
the exterior lies the young politician's urge to transform a case on him, as the Flag Code of India didn't allow the
India into a wealth of opportunities. His Lok Sabha ticket flying of the flag on non-governmental institutions. Unde-
ensures that he is in the right profession to carry out the terred, Jindal fought to defend the rights of a citizen who
dreams of millions. To boot, Jindal, is the Executive Vice was only raising his national flag in respect. After all, he
Chairman and Managing and Director of Jindal Steel and was hoisting it in his own country. Jindal was to have the
Power (JSPL), part of the $ 6 billion Jindal Organisation. final laugh in the matter. After nearly 10 years of waiting
the Supreme Court handed out a verdict in Jindal’s favour.
With his credentials in place, his belief that the time
Jindal had scored a major and crucial victory. Lessons
is ripe for the nation's youth to have their tryst with des-
learnt ? Well, for one thing, Jindal found himself at the
tiny, will, no doubt, find many takers. Standing in the
receiving end of a groundswell of support. It convinced
way of Jindal are, however, only achievable milestones-

ww
no hurdles. The way Jindal sees it, there is no room for
any morale-busting shortcomings like corruption or poor
infrastructure, precisely because they ought to be the last
him more than ever before that if he held on to what he
believed in, then he might not be alone in his mission.
If there is another lesson that Jindal has learnt, it is

w
thing on the mind of the youth-a motto that is borne out

.Ea
by examples from his own political career.
After joining the Lok Sabha, representing Kuruk-
shetra constituency from Haryana, he played an active
from his father who had entered politics at the age of 60.
Unlike his father, he wasted no time in giving himself up
for public service. If politics, which Jindal has identified
as giving him the right platform to launch his reforms, is

syE
role in having smoking banned in the Central Hall of
Parliament in July 2004. He is also responsible for duty-
free shops at the country's international airports accepting
the way forward, then he believes in joining when one is
young. His desire to reach Parliament through the Lok
Sabha, rather than the Rajya Sabha, was a reflection of
Indian currency from September 2005. Jindal reckons
that efforts directed towards the people of the country
ngi his resolve and capacity for hard work.
Business As Usual

nee
should not be confined to the upper crust.
On the business front, Jindal sees rapid growth with
The country's booming growth, he believes, should
constantly changing paradigms. There is growth in the
reach all the way down to the underprivileged classes as
industry and the challenge is to keep on growing and
well. For the youth living on this side of wealth, "good
quality education," especially primary education, should
rin
streamlining all aspects of business, be it sourcing of raw
materials or logistics, having a really good team, adopting

g.n
be given. Says Jindal : "The need is not for more policy
the latest technology or an efficient production system.
decisions but for streamlining implementation. Our
People have to be ready to embrace changes. They have
delivery mechanism has failed us completely and we

e
to constantly innovate and adapt to the changing times.
have to focus on making it more efficient." The school-
level education is the most important, as this is where the
foundation is laid, and after school the youth should be in
a position to decide on what to do next. Therefore, there
should be a number of vocational and technical training
“But at the same time we must focus on keeping one's
core competence, core values and impeccable reputation
t
untouched," he warns. Incidentally, when Jindal had taken
over the reigns of Jindal Organisation's Raigarh opera-
tions, which had sponge iron as its backbone, it was in the
courses provided for the next stage, Jindal feels. "We
red. Soon, by adopting global economies of scale, Jindal
have to also focus on developing our infrastructure," he
was able to ring in changes and make it the largest coal-
adds. It is a question of becoming self-reliant at a young
based sponge iron manufacturing capacity in the world. It
age and standing up for what one represents. As a member
is to Jindal's credit that his steel company is a producer of
of the Congress youth brigade that includes Rahul Gandhi,
low-cost sponge iron. Taking the forward integration
Sachin Pilot and Jyothiraditya Scindia, Jindal is all too
path, he set up the Rail and Universal Beam mill.
aware of the responsibilities of a young politician in
carrying out his agenda to a picture-perfect finish. Right now, JSPL has a capacity of 3 million tonne
and Jindal wants it to go up to I5mt in the next 10years.
Initial Trigger Future plans include setting up 6 mt plants in Orissa and
Jindal's decision to enter politics may have had its Jharkhand. Besides having a number of other projects in
origins in an amusing incident that was played out over the pipeline, Jindal already has his hands full with his $2.
10 years-which Jindal, however, puts down to his father's I-billion iron and steel project in Bolivia. Jindal's power
influence. His student years in the USA, where Jindal did business has a capacity of producing 1,500 MW which he
his MEA from the University of Texas and Dallas, were is keen on increasing to 10,000 MW in the next decade.

574 | CAT Complete Course

Visit : www.EasyEngineering.net
Visit : www.EasyEngineering.net

Being part of a traditional family-run business, he 3. The most appropriate title for the passage would
says that his approach has been to change with the suitable be—
evolving competitive scenario. He understands that (A) Stand up for your rights
though in the early days family-run businesses ran a tight (B) Youth + opportunities = success
ship, now there has to be more delegation of authority if (C) Equip the youth-Conquer the World
systems are to work properly. More importantly, corporate
(D) Stand up for what you are
governance should be given more elbow room and there
(E) Charge of the youth Brigade
has to be a better team effort. As a polo player, Jindal
knows all too well what it takes to collectively work for a 4. Naveen Jindal has marked his presences as a serious
common goal. politician By fighting against all odds and succeeding
by a standing up for what he represent a few exam-
The Divide ples are—
Shuttling back and forth between politics and busi- 1. He played an active role in having smoking
ness is hard. But helping Jindal manage both ends of a banned in the Central Hall of Parliament.
difficult career is his devoted team. He concedes that 2. Winning a case against the government which
more than half of his time is taken up by politics and what 10 years later changed the flag code Allowing

ww
keeps him going is an undying passion for the people of
his constituency. One of his priorities has been on pro-
viding toilets to each household under the total sanitation
citizen of India to raise the National flag in non–
government institution

scheme.
w .Ea
"This, I believe, provides dignity to the people, espe-
cially women. Apart from this, I am focused on providing
health care. I have lately got involved in helping the
3. His desire to reach parliament through the lok
Sabha rather than the Rajya Sabha.
4. He took over the Raigarh operations which was
in the red and made it the largest coal-based

syE
Bharti Group's corporate social responsibility initiative of
opening 40 primary schools in my constituency. My role
has to be in facilitating their mission," he says. Besides
sponge iron manufacturing capacity in the world.
Choose the correct sequence—
(A) 1, 2, 3 (b) 1, 2, 3
the hard-nosed drive of a businessman, there is a humanist
in Jindal that is trying to break the mould of a conven- ngi (C) 2, 3, 4
(E) 2 and 3
(d) 1 and 4

tional do-gooder"
1. From the passage the reader can conclude that the
author is chiefly concerned about— nee1. (C) 2. (C)
Answers
3. (D) 4. (B)
(A) Corruption and pathetic infrastructure
rin
Passage 6

g.n
(B) The country’s flourishing growth should reach
the underprivileged classes too. Words–1810
In explaining Hume's critique of the belief in
(C) A young politician dream to transform India
into a treasure trove of opportunities
(D) Increasing vocational and technical training
course.
e
miracles, we must first understand the definition of a
miracle. The Webster Dictionary defines a miracle as: a

t
supernatural event regarded as to define action, one of the
acts worked by Christ which revealed his divinity an
(E) The nation’s youth trust with destiny extremely remarkable achievement or event, an unex-
pected piece of luck. Therefore, a miracle is based on
2. Naveen Jindal believes in all of the following
one's perception of past experiences, what everyone sees.
measures for the underprivileged to get the benefits
It is based on a individual's own reality, and the faith in
of India’s Booming growth ‘Except’—
which he/she believes in, it is based on interior events
(A) Our delivery system should be made more such as what we are taught, and exterior events, such as
efficient-productive what we hear or see first hand. Hence studying Hume's
(B) Good quality education must be provided view of a miracle, he interprets or defines a miracle as
such; a miracle is a violation of the laws of nature, an
(C) They should be groomed to carry out responsi-
event which is not normal to most of mankind. Hume
bilities to a picture perfect finished responsibi-
explains this point brilliantly when he states, "Nothing is
lities to a picture perfect finished
esteemed a miracle, if it has ever happened in the com-
(D) Vocational and technical training must be pro- mon course of nature. It is no miracle that a man seem-
vided after school level education ingly in good health should die on a sudden." Hume
(E) Laying emphasis on developing our infras- states that this death is quite unusual, however it seemed
tructure to happen naturally. He could only define it as a true

CAT Complete Course | 575

Visit : www.EasyEngineering.net
Visit : www.EasyEngineering.net

miracle if this dead man were to come back to life. This sanction and authority, which always attend perceived
would be a miraculous event because such an experience opinions." In any case many of the miraculous events
has not yet been commonly observed. In which case, his which happened in past history would not be considered a
philosophical view of a miracle would be true. Hume miracle in today's world, or at any other time in history.
critiques and discredits the belief in a miracle merely The reality most people believed at that period, as a result
because it goes against the laws of nature. Hume defines can be considered lies or exaggerations. Hume discredits
the laws of nature to be what has been "uniformly" the miracle as to the time period in which the miracle is
observed by mankind, such as the laws of identity and taking place, the mentality, or the reality of individuals at
gravity. He views society as being far too liberal in what that given time. Hume suggests that during certain times
they consider to be a miracle. He gives the reader four in history we are told of miraculous accounts of travellers.
ideas to support his philosophy in defining a true miracle, "Because we as individuals love to wonder, there is an
or the belief in a miracle. These points lead us to believe end to common sense, and human testimony, in these
that there has never been a miraculous event established. circumstances, loses all pretensions to authority."
Hume's-first reason in contradicting a miracle is, in all of
history there has not been a miraculous event with a The final point Hume gives to discredit the validity
sufficient number of witnesses. He questions the integrity of a miracle is that there must be a number of witnesses
of the men and the reputation which they hold in society. to validate the miracle. "So that not only the miracle

ww
If their reputation holds great integrity, then and only
then can we have full assurance in the testimony of men.
Hume is constantly asking questions to support proof for
destroys the credit of testimony, but the testimony
destroys itself". This basically means that the witnesses
must all give the exact same testimony of the facts of the

w
a miracle. He asks questions such as this; Who is

.Ea
qualified ? Who has the authority to say who qualifies ?
As he asks these questions we can see there are no real
answers, in which case, it tends to break the validity of
event. Hume finds difficulty in the belief or integrity of
any individual, and the difficulty of detecting falsehood
in any private or even public place in history. "Where it is
said to happen much more when the scene is removed to
the witnesses to the miracle.
syE
Hume's second reason in contradicting the validity of
ever so small a distance." A court of justice with accuracy
and judgment may find themselves often distinguishing
between and true and false. If it is trusted to society
a miracle is that he views all of our beliefs, or what we
choose to accept, or not accept through past experience
and what history dictates to us as discreditable. Further- ngi through debate, rumors, and man's passion it tends to be
difficult to trust the validity of the miracle. Throughout
the rest of the readings, Hume states a few events which
more, he tends to discredit an individual by playing on a
human beings' consciousness sense of reality. An
example is; using words such as, the individuals need for
nee
many believe, are miracles. He discredits many of these
miracles through his critiques. I have chosen to illustrate

rin
two "so-called” miracles from the New American Bible
"excitement" and "wonder" arising from miracles. Even and to show how Hume would view these miracles. The
the individual who cannot enjoy the pleasure immediately stories are of Noah's Ark and The Burning Bush. The
will still believe in a miracle, regardless of the possible
validity of the miracle. With this, it leads the individual to
feel a sense of belonging and a sense of pride. These g.n
story of Noah's Ark took place when the Lord began to
realize how great man's wickedness on earth had become.

individuals tend to be the followers within society. These


individuals will tend to be believers rather than be the
leaders in the society. With no regard to the miracles
validity, whether it is true or false, or second hand
e
He began to regret the fact that he had created man on
earth. The lord decided the only way to rid earth of the
wickedness would be to destroy all men, and all living
creatures living on the earth. The only men which he t
would not destroy were to be Noah, his sons, Noah's wife
information. Miracles lead to such strong temptations, and his son's wives. He also would save a pair of animals
that we as individuals tend to lose sense of our own belief of each species. The rest were to perish from the earth.
of fantasy and reality. As individuals we tend to believe He chose Noah to be the favored one to carry out the task.
to find attention, and to gossip of the unknown. Through The Lord requested Noah to build an ark and explained
emotions and behavior, Hume tends to believe there has exactly how it was to be made. Noah spent six hundred
been many forged miracles: regardless if the information years of his life building the ark which God insisted upon.
is somewhat valid or not. When the ark was finally complete The Lord told Noah it
His third reason in discrediting the belief in a miracle was time to gather the selected few as the floods were
is testimony versus reality. Hume states, "It forms a strong about to come. These floods lasted forty days and forty
presumption against all supernatural and miraculous nights. The floods wiped out all living creatures on earth,
events, that they are observed chiefly to abound among except all on the ark. In the six hundred and first year of
ignorant and barbarous ancestors; or if civilized people Noah's life the floods stopped and the earth began to dry.
has ever given admission to any of them, that people will Noah then built an altar to the Lord and choosing from
be found to have received them from these barbarous every clean animal he offered holocaust on the altar. As
ancestors, who transmitted them with that inviolable God states "Never again will I doom the earth because of

576 | CAT Complete Course

Visit : www.EasyEngineering.net
Visit : www.EasyEngineering.net

man, since the desires of man's heart are evil from the 2. What could possibly make 'The Dead Man Coming
start; nor will I ever strike down all living beings, as I Alive" incident unacceptable to Hume as a miracle ?
have done." (A) Doubts about the integrity of the witnesses.
In deciding upon whether this is a valid miracle in (B) Differences between testimony and reality.
Hume's opinion of miracles, I believe he would consider (C) Less number of witnesses to the event.
it to be a miracle but, would have a hard time validating
(D) All of the above.
the testimony of it. The reasons in which he would
criticize the validity together the testimony would be as 3. Miracles, according to Hume, are—
follows. The testimony versus the reality. To further (A) Supernatural occurrences requiring divine
support the theory he would argue the time period in intervention
which the miracle had taken place. And would find it (B) Any occurrences that defy the laws of nature.
difficult to believe without a reasonable doubt. There is a (C) Extra-ordinary occurrences that defy the laws of
question to whether it could be lies or exaggerations. nature
Furthermore, it could not possibly be a validated miracle
(D) Violations of Biblical teachings
considering the amount of men who witnessed the event,
as well as the integrity of the men. Although this mirade 4. Which of the following is false in context of Hume's

ww
was an act of God we can still question the validity of our definition of miracle ?
ancestors or God for that matter. Hume would not be (A) Plenty of miracles have occurred of late
satisfied not only with the integrity of the individuals but (B) Our beliefs influence our judgement of miracles.

w
the amount of witnesses at the given time. Therefore, we (C) The incident of Noah's Ark can't be established
can only view this as a miracle depending upon our own as a miracle.

.Ea
individual perceptions of what we believe to be true. This
leads to a non uniform event, since we as individuals hold
different beliefs of what we hold true, and false.
(D) Human testimony may be divorced from reality.
5. A suitable title for the passage is—

syE
The second miracle which I will discuss was that of
Moses and the burning bush. As Moses was working in
(A) The truth about the Story of the Ark
(B) What are Miracles ?

ngi
the fields an angel of the Lord appeared to him in a fire
(C) Hume on Miracles
flaming out of the holy bush. Almost amazing the bush
was full of flames but was not yet consumed. As he (D) None of these
walked closer he heard the voice, the voice of God telling
Moses he was the chosen one to take the Israelites out of
Egypt away from the cruel hands of the Egyptians. In nee
1. (D) 2. (D)
Answers
3. (C) 4. (A) 5. (C)
disbelief that he was the chosen one he set forth on his
journey to Egypt with God watching over him and
rin
Passage 7
Words–3416
leading the way. As Moses leads the Israelites out of
Egypt he comes to the Red Sea with the Egyptians close
behind. As the bible explains, the miracle takes place g.n
ALPANA Rai drummed on the glass top of her table
with her nails and keenly heard unassertive hollow
when the Red Sea splits leading the Israelites to freedom.
As the Egyptians were crossing the sea, it closed its gates
and let them drown within the waters of the sea. In
justifying whether Hume would discredit this miracle he
e
sounds. Soon she was unwittingly playing the parade beat
they used to at school. It is all about memory….. she
thought. Some how, mental actions reach out to old
grooves…… and resonate...… Kress Inc., where Alpana t
was the HR manager, was agog with a new dilemma. A
would definitely see how one may say it is a miracle, but moral one, which the CEO had firmly slapped down with
again would have a hard time validating the testimony of his verdict –sack the errant manager. And while he
the miracle. Again we seethe pattern of the fact that there thought he had easily accomplished it, Alpana, said
is no one to testify for the event. We can only view this as "What is the justification ?" This had thrown the whole
a truthful experience through our belief in God and the operations committee into a frightening spasm. Alpana
Bible. It is what we are taught to believe through was not ignorant of the fact that Amol, the 'errant'
religious texts, and our house of worship. It is the manager in question, was a social failure at Kress.
individual's perception of reality and what he or she
believes to be a valid event. A simple situation, but its genesis went far back in
time. This is what had happened. Until two years ago,
1. According to Hume, which of the following will
Kress, which was slow in delivering results, had just
qualify to be termed as a miracle ?
begun to pick tremendous energy under the new supply
(A) Dead man coming alive.
chain manager, Amol Dua. He verily whipped people into
(B) The incident of Noah's ark. action. Some called him a go getter, some said 'hard-as-
(C) The narrative about Moses and the burning bush. nails', slave driver; many epithets described him. The
(D) All of the above. bosses couldn't breathe a word, for this man was singu-

CAT Complete Course | 577

Visit : www.EasyEngineering.net
Visit : www.EasyEngineering.net

larly bringing home the sales and bottom line and rescued In the 360-degree appraisal that year, 9 out of 10
Kress from near disaster. called Samarth dispensable. He survived this using his
When Amol was hired, Kress had been in dire straits very renowned skill of challenging and ruffling. While
needed a strong supply chain focus. Amol was hired from Amol made it to the market on time, Samarth harbored a
a confectionery company at level 2, thus equating him grudge against Amol and held him singularly responsible
wiyh Samarth Soi, the finance manager. Where as until for the acrimonious 360.
Amol joined supply chain was a level-4 job. Thereafter, it became a pattern where Samarth would
So, Samarth, who felt his position weaken, resisted resist Amol, worse, not share relevant financial data with
Amol. As if justifying it, Amol was also tough. He had him or stall his work some more. During this period,
come from a blue chip MNC and was seen all round as Samarth chanced upon Amol talking to Seema Goyal, his
Kress's Saviour. Today, after 24 months, Kress Inc. had cost accountant, by the coffee machine twice and once
recycled its past weaknesses and Kress was a force to offering her a lift home. Samarth grew anxious. So, was
reckon with, evidenced by competitive attacks on it by Amol cutting the chain of command by simply dealing
even the market leader. Therefore, what command Amol with Seema for the data he needed? Samarth began to
possessed was seen by Samarth as arrogance. This monitor Seema even more closely. "I smell a rat," he said
perception was strengthened by the ringside view of to Dushyant Dhir, a senior engineer in IT. "Something

ww
Amol by managers in sales and marketing who at one strange is going on there. This chap is dealing with my
time were the propitiated Gods. Today, all devotion was staff directly…... wonder what is cooking!”
to the Lord of supply chain, and rightly so, for the foods Kress had a mature IT department with sophisticated

w
business was entirely supply-chain driven. monitoring tools. One of the tools allowed the IT
Amol's seniors respected him for his attitude and his managers to take control of any computer anywhere in

.Ea
ability to deliver. His peers were in awe of him, his
juniors on eggshells. He was himself non-political, work
driven and a bit rough at the edges, so that he said what
the network and actually see what the user was doing.
Smiling at Samarth's surprise' Dushyant said, "Imagine

syE
he meant, meant what he said. On the personal side,
Amol was unmarried, a conscious decision he took to
look after his paraplegic father. So, it was that he had less
this, suppose you are chatting with someone, and step out
for a meeting and I know you are not in your room. I can
take control of your PC and read your mailbox. So, if you

of the distractions that his married colleagues had and a


home that doubled up as an office, allowing him to ngi have serious doubts, I can take control of Seema's PC or
Arnol's too, in their absence, check all the mails, chat
sessions, you just tell me."
continue where he left off at work.
Samarth, on the other hand, was a manager who was
smart, politically correct and suave. He was someone nee Samarth agreed. He wanted to see if Seema had been
sharing classified financial data with Amol. As luck
would have it, Dushyant downloaded all mails between
who stirred up feelings and thoughts with his presence,
and relief with his departure. Thus, you could never say
rin
the two persons and discovered mails between Amol and
Seema which indicated a deeper, non professional rela-
he was wrong, but his being right never helped because
he was rarely part-of the solution. Better still, Samarth
forced his way into problems and assumed stances that g.n
tionship. The broad tone of these mails was affectionate,
periodically interspersed with anxiety over not having
were seemingly participative. But his presence ruffled
everything, put orderly perspective into disorder, raise
dust, where after he left leaving inane rhetorics for people
to deal with. This pattern was what many called
e
met. Some in between were terse with an undercurrent of
annoyance, sometimes frustration, and then there was

t
one, where Seema said, "Sorry, this is not going to work,
my dad's health is such and I am not pretending that he is
recovering or will recover."
'disruptive'.
Samarth's head went into an overdrive. Placing an
RECENTLY, when a key raw material fell short
this before the operations head, Ojas Dharker, he extrapo-
threatening sales, Amol had suggested they source from a
lated, "So, is this what we want to see happening in a
more expensive supplier-someone he had dealt with in his
clean company like Kress ? What about confidentiality of
old organization. This, he said, was to be a completely
data ? What all has he extracted by sexually harassing
temporary measure to tide the season and not lose the
her ?"
market. Samarth, of course, grew belligerent and threw
numerous spokes in the wheel. From 'bottom line cannot You don't know ?" Then leaning forward, Samarth
bear this' to 'we are losing profit focus owing to our said, "New developments. Seema called me this morning
egotistic need for market power'. Then, he delayed the to say she cannot come to work for a few days. Her
payment to the supplier and hinted at examining S.299 of pretext? 'Father unwell.' The plot thickens, Dharker, did
Companies' Act. Could Amol be deemed interested in the you read his last mail? This morning Seema asked to be
transaction ? Which itself caused a mild flutter. transferred to the Delhi office. Her friends say she has
Amol distanced himself from all this. As far as he been looking very harassed and disturbed. Who is to say
was concerned, the material was in, production was what he has done, what he has said to her ! The head of
racing and Samarth could go read up all Acts he wanted. our supply chain! I worry for the women here. Imagine

578 | CAT Complete Course

Visit : www.EasyEngineering.net
Visit : www.EasyEngineering.net

what he gained for us in the swing, we will lose in the Give me one week. Besides, how do we know there was
roundabout, thanks to this news getting out! Seema will sexual harassment?" TR did not like this, "I am the CEO
talk, I know that girl, she is a firebrand…" Alpana, don't make me speak unpleasant language. Well,
Dharker winced. Things looked bleak. Even a finan- the language I am told was flirtatious. Let's leave it at
cial scam was welcome, but not this, he thought. Without that," he said hurriedly, as if even dwelling on the words
a warning, Dharker ran into Dushyant in the men's room. would bring him impurity. Alpana did not understand,
And typical of men's room talk, Dushyant said in an "OK, I have read the mails, I thought they were tender
anxious whisper, "It is completely unaccept able that a exchanges. Where did harassment come from ?"
senior manager is using corporate resources to conduct an "How would 1 know ?" thundered TR. "Seema has
illicit affair ! What an example !" gone on abrupt leave, and the Dushyant, who probed the
Dharker hated this kind of banter. But he was PCs says…" Alpana gasped, "He did that ? Oh, dear
completely shaken by what he was hearing. Office gossip God… ok, give me the week I asked for.."
and speculation was a killer virus. Behind their smart Early next morning Alpana called Seema and asked
clothes, people were finally petty and bored. Amol was a to meet with her. On arriving, Alpana saw Seema sitting
tough nut, yes, but he was not one you classified as in the lawn reading to her old father while he sat there,
indecent. Sexual harassment? Indeed! head hanging to one side, drooling. "He has had a mild

wwBut the restlessness did not leave him. Almost carried


by the power of his anxiety, Dharker met the CEO,
Triumalai Rajan, (TR) and apprised him of the findings.
paralytic attack again," said Seema. "He keeps remem-
bering his sister in Delhi. If I can be transferred there for
a while or if I can take a sabbatical."

w
TR did not like any of this. He shook his head for a very
longtime, looking away. How was he to have known two

.Ea
years ago that he was hiring a maniac? Amol was a
stellar chap, but then he was not compromising Kress’
Alpana took the whole setting in and wondered if she
would be doing right asking the questions she planned on.
Then quickly setting aside her doubts, she said to Seema,
"I can see how hard you work at your life, Seema. Some

syE
dignity and purity, "For me, honesty and integrity are
tops," he said to Dharker. "If this is Amol, then he goes.
Simple."
choices we make or have to make will be tough, but if we
are doing them for all round good, the decision is not
tough at all, yes ?" Seema smiled and said, "You are
Picking up the phone, he called Alpana Rai, the HR
head, and said, "Sorry to bother you on your vacation.
There is a terrible situation that needs your help. Please ngi right."
Then Alpana said, "I am here on a mission that is not
very pleasant. But I have to do this to enable other
come and see me as early as you can." Alpana who had
taken a four-day break to attend to her per Frodo's
surgery, drove hurriedly to see TR. "Just four days and
nee
situations." Then choosing her words, she said, "There is
a belief among some people at work-and this has therefore
assumed grave proportions" No, this is inappropriate, just
the world has chosen to change in this short time!" she
mused. rin
come to the point it will be easier, she thought. Bracing
herself she said, "Seema, a situation has arisen which
In TR's office as she stood there arms crossed, head
cocked to her left (she did not hear too well on her right), g.n
compels me to request you to answer this question.
Specifically, I wish to know if you are under pressure or
TR spoke gravely, updating her on the episode. He said,
"Make sure he leaves the premises in the next hour and
effect his settlement swiftly in two days. I don't want to
see him again. And yes, a very strong message has to go
e
undue coercion from Amol." Seema was alarmed. "Won’t
you explain what this is all about ?" she asked.
Alpana hated her job now. She said, "Is there a
relationship between the two of you, that is..." Seema t
out that at Kress we value decency. Let this be an example looked hard at Alpana and said, "He is 35, I am 32... full
for everyone that we do not encourage such behaviour at grown adults by any lab test, both of us are unmarried. I
Kress. You know better than me that sexual harassment is am surprised I have to answer these questions."
a punishable offence in most countries. We are in a, ALPANA agreed and said, "Forgive me, Seema, it is
vulnerable situation especially since we hire women not easy for me either. As it happens, there is an im-
here." pression doing the rounds that you are under pressure."
Alpana, thoughtful and surprised, left TR's room, her And Alpana had no more words. The whole thing seemed
arms yet crossed, thinking. She sat in her room, drumming so wrong and unfair and juvenile to her.
on her glass table, thinking, until it was 8 p.m. Then she Sensing her discomfort, Seema said, "I don’t owe
called TR and said, "I am not falling for this. I am not you any of this, but let me make it easy for you. Yes, we
doing anything in two days. No. I am first going to meet have gone out a couple of times. But I can see there is no
the characters in this story." TR was agitated. "Alpana, future in this relationship. My father needs me now…, I
the whole organization is upset. Do we want to be seen as can not think of myself. As for Amol, no, he has not
someone shielding a sick man?" coerced me. He is a very, very decent guy. He under-
"How do we know he is sick ?" asked Alpana. "Let us stands, and if it helps you understand him better, he
not get theatric, TR, hang on to the faith that he is good. himself is nurturing his ailing father for 17 years now."

CAT Complete Course | 579

Visit : www.EasyEngineering.net
Visit : www.EasyEngineering.net

Returning to the office Alpana told TR, "There is is good for the goose is also good for the gander, isn't it?
nothing of the kind you suspect. I am not sacking Amol." You want objectivity? Then here it comes- Seema goes
TR looked concerned. "Alpana, you have gone by the too! But you want to make an example out of Amol, and
statements made by these two?" Now, she was really you Dushyant, you want to paint a halo around having
upset. "TR, how did they suddenly become 'these two' saved a woman, whereas Seema needs no saving! She has
people ? They are our employees, and among the finest said in very balanced terms that she was not being
we have. What is the concern here ? I have told you there harassed. If we have her word on this, what is the issue?"
was no sexual harassment." . TR made a sound that was a cross between exaspera-
Samarth who was called in as 'the aggrieved party', tion and disbelief and said, “Alpana, as a woman, I would
said, "I wouldn’t call them 'finest' ! Amol is setting a have expected some grace and sensitivity from you. This
wrong example for his subordinates-utilising office time situation is explosive. We owe our women a value-based
for personal relationships, then? And yes, maybe there workplace. You shock me with your blasé worldview!"
was no harassment, but there is certainly cause for "Do I ?" asked Alpana, now obviously angry. "How
concern over office decorum! Also, in my opinion, there did values become a feminine attribute and need? Values
is no place for emotions at work Period. People lose are for everyone and values include respect for colleagues,
objectivity which is a must in functions like finance, their personal lives and their private space! Yet here we

ww
legal, etc. How can we afford to have judgments getting are, gods of values, merrily trodding on their private
coloured by personal relationships ? lives!"
Alpana said in most bland tones, "In-office attractions TR, deeply confused and disturbed spoke, "I want

w
are a given and 1 do not want to pretend that this is
unholy. Were they betraying an existing marriage ? No.

.Ea
Was their conduct offensive to office decorum ? No. Now
where is the objection?" Then, facing TR in a manner so
Kress to be known for its purity, for its decency. You tell
me, was it 'ethical' to have indecent chats with someone
on the office machine during office hours? Whatever you
may say Alpana, I have serious objection to this.

syE
she would hear his protest better, she said, "We spend Tomorrow you will have others indulging in all this,
close to 10 hours a day at work. This verily becomes our then? Is there any sanctity to a workplace or not ?"
social environment. We bond and break here... what are Samarth nodded briskly, "Organizations need walls to

ngi
we complaining about?" protect sensitive information, budget numbers and the
Dushyant, present in this meeting in his capacity as like, lest they be misused say for insider trading, manipu-
the star detective said, "The moment a relationship lating sales performance, etc. So, such inside dalliances
becomes personal, there cannot be professionalism. You
don’t seem to notice that corporate assets and time was
being applied for a dalliance! Others will start using
nee
make this internal water- tight information 'protection'
system weak!"
Alpana nodded, then to TR she said, "I have seen the
office computers to surf the Net, write personal e-mails,
exchange jokes! Moreover, I would have thought that as a rin
chats, much as I hated doing it; six in all and not one
indecent expression or innuendo. Our over zealousness
woman, you would go out of your way to protect Seema
and women kind!"
g.n
can ruin career, two good careers, simply because you
think this is unclean."
Alpana wondered what it would take to enable
people to negotiate tradition a bit. She said to Dushyant,
"That time when you printed out whole portions on
"Trinidad & Tobago's neotropical invertebrate fauna' in
colour, on glossy paper for your daughter's project... that
e
Then Alpana said something that made them all turn

of Kress, it should be Dushyant and Samarth." TR was


now really annoyed. "Miss, you are crossing all limits!”
he said, his voice cracking in emotion. Alpana spoke her
t
pale, "If anyone has to be sacked for protecting the purity

also was office resources and time, no? The principle is mind, uninhibitedly. "You don't pry into your employee's
the same Dushyant - sometimes we do have to apply a private life, unless you think he is defrauding the
small bit of our office time to our personal care. Like I company. What these two have done is flagitious and
call my son six times during work. I think it is OK. As for diabolical. How dare anybody misuse their functional
personal and professional boundaries, how does the position and peep into another's computer? How dare!
positive effect love turn into professional transgression? "Tell me, TR, tomorrow if Dushyant thinks it fit that
Or are we all supposed to turn into zombies when we it is necessary for the company to know what takeovers
walk into our offices ? And then, professional transgres- you are planning, and so he simply seizes control of your
sion can also happen between two rivals who dislike each machine… see ? Nobody here has any business to misuse
other for mutual gain... are we noticing that ?” a technology. I cannot trust Dushyant anymore, nor can I
THEN when silence fell all round, Alpana said, “And trust Samarth. You don't need to have an affair to trade
if you must sack won't you also have to sack Seema? confidential information, Samarth, a good friendship like
Why only Arnol? How about also imagining that she too yours and Dushyant's did like-wise, no ? TR, this is abuse
was sexually harassing him? Possible ,no? I mean, both of power. That's what my HR manual tells me. It's like
persons have admitted to sharing a relationship. So, what someone having the master key to your house... I don't

580 | CAT Complete Course

Visit : www.EasyEngineering.net
Visit : www.EasyEngineering.net

feel safe working here anymore! What if they tamper feel its masses two square meals a day, the monster of
with my appraisals tomorrow ?" starvation and food insecurity continues to haunt the poor
TR was pained. He was losing control. Suddenly, the in our country.
headlines had changed, rendering the last few weeks' Increasing the purchasing power of the poor through
news fraudulent. Almost. providing productive employment leading to rising
“Amol and Seema are not the issue," said Alpana. income, and thus good standard of living is the ultimate
"The real issue is one of feeling safe in this organization. objective of public policy. However, till then, there is a
So, I ask how did this act of Dushyant's get approved ? need to provide assured supply of food through a
And how many such computers has he been peeking into? restructured, more efficient and decentralized public
And for the future, how do we prevent this? This is
distribution system (PDS).
serious!"
Although the PDS is extensive-it is one largest
1. What would be the best and most appropriate title for
the passage ? system in the world it has yet to reach the rural poor and
the far-off places. It remains an urban phenomenon, with
(A) Revenge is sweet
the majority of the rural poor still out of its reach due to
(B) Women appraised lack of economic and physical assess. The poorest in the
(C) Respect for privacy cities and the migrants are left out, for they generally do

ww
(D) Feminine values Vs. masculine ones not possess ration cards. The allocation of PDS supplies
(E) Jealously gone haywire in big cities is larger than in rural areas.
In view of such deficiencies in the system, the PDS
2. What arguments were put forward to extradite the

w
sacking of Amol by samarth and Dushyant ?

.Ea
All the following are false- EXCEPT—
(A) The extraction of classified material through
coercion
urgently need to be streamlined. Also considering the
large foodgrains production combined with food subsidy
on one hand and the continuing slow starvation and
dismal poverty of the rural population on the other, there

syE
is a strong case of making PDS target group oriented.
(B) Sexual harassment and improper use of office The growing salaried class is provided job security,
machinery & time regular income, and social security. It enjoys almost

ngi
(C) Misbehavior with a female worker and the use hundred percent insulation against inflation. The gains of
of expletives development have not percolated down to the vast
(D) Arrogance and disrespect for senior mangers majority of our working population if our compares only
(E) Failure to achieve deadlines and target sales
3. The attitude of the author in the passage can best be nee
dearness allowance to the employees in public and
private sector and looks at its growth in the past few
years, the rising food subsidy is insignificant to the point
deciphered as one of—
(A) Outrage and disbelief
rin
of inequity. The food subsidy is a kind of D.A. to the
poor the self-employed and those in the unorganized
(B) Approval and encouragement
(C) Didactic and Utilitarian
g.n
sector of the economy. However, what is most
unfortunate is that out of the large budget of the so called
food subsidy, the major part of it is administrative cost
(D) Sober and Practical
(E) Philosophical
4. Samarth’s angst towards Amol can be traced down to
which of the following reasons
e
and wastage. A small portion of the above budget goes to
the real consumer and an even lesser portion to the poor
who are in real need.
1. Which of the following, according to the passage, is
t
1. Jealousy 2. Inferiority complex true of public distribution system ?
3. Inadequacy 4. Under achieving (A) It is unique in the world because of its effec-
5. Ego tiveness
(A) 1, 2, 5 (B) 1, 2, 3, 5 (B) It has remained effective only in the cities
(C) 2, 3, 4, 5 (D) 1, 2, 3, 4, 5 (C) It has reached the remotest corner of the country
(E) 1 and 5 (D) It has improved its effectiveness over the years
Answers (E) It develops self-confidence among the people
1. (C) 2. (B) 3. (A) 4. (E) 2. Which of the following, according to the passage, is
the main reason for insufficient supply of enough
Passage 8 food to the poorest ?
Words–386
(A) Production of food is less than the demand
The public distribution system,. Which provides food
at low prices, is a subject of vital concern. There is a (B) Government’s apathy towards the poor
growing realization that though India has enough food to (C) Absence of proper public distribution system

CAT Complete Course | 581

Visit : www.EasyEngineering.net
Visit : www.EasyEngineering.net

(D) Mismanagement of food stocks cerebral cortex, the great top part of the brain, Highly
(E) None of these intelligent people also have good blood circulation to the
brain, bearing oxygen, glucose and certain other impor-
Answers tant chemicals. It is possible that a person with very
1. (B) 2. (C) special talent-a mathematical or musical genius, for
example, may have an unusually thick bundle of nerve
Passage 9 fibers in one particular place in the brain.
Words–924 But the physical endowment of your brain- is far less
There is no such thing as 'brain-fag'. Thinking that important than what you do with it. The number of brain
long concentrated mental effort produced tiredness in the cells in an individual with an I.Q. of 100 (which is
brain itself. Yet scientists believe that this state cannot average) is large enough so that, if used to the full, it
exist. Your brain is not like your muscles. Its operations could far exceed the record, so far as memory is con-
are not muscular but electrochemical in character. cerned, of the greatest genius who ever lived. A person of
When your brain appears to be tired after hours of average I.Q. who industrially stores up knowledge and
mental work, the fatigue is almost certainly located in skills year after year, is better than a person -with very
other parts of the body, your eyes or muscles of the neck high. I. Q. who refuses to study. Research has indicated
that some of the most important men in history had no

ww
and back. The brain itself can go on almost indefinitely.
A young woman undertook as an experiment to
multiply in her head a series of two four digit numbers
more than ordinary IQs.
Among them are statesmen such as Cromwell, John

w
one after the other as rapidly as possible. She went on Adams and Lincoln; military heroes like Drake, Napoleon
doing this for twelve hours. and Nelson, writers like Goldsmith, Thackeray and Emer-

.Ea
During that time there was only a slight decrease in
her efficiency, measured by speed and accuracy. At the
son. All these men, to be sure, were above the average in
intelligence; yet they ranked far below the most brilliant

syE
of the individuals studied. What they possessed in high
end of twelve hours she stopped only because of body
degree was character, and the ability to keep plodding
fatigue and hunger.
ahead until they achieved what they had set out to do.
What seems like mental fatigue is often merely

ngi
Age need not prevent your learning. One of the
boredom. In reading a difficult book, for example, you commonest misconceptions about the brain is that as you
are torn between the desire to go on and the impulse to grow older something happens to it causing the learning

nee
stop. According to an eminent psychologist, it is often not process to become more difficult. This is true only to
fatigue that you feel but inattention and the inability to such a minute extent that for most of us, if is of no
ignore distracting thoughts. practical importance.
The brain capacity is almost inexhaustible. That part
of your brain involved in thinking and memory, and all
rin
Learning is associated with ability to create new
reverberating electric circuits in the brain and as long as

g.n
your conscious activities, has at its most Important part that power remains you can continue to acquire new
ten or twelve thousand million minute cells. Each of these knowledge and skill - even at ninety.
has a set of tiny tendrils by means of which an elec- It is true that all old people suffer impairment of their
trochemical message can pass from one cell to another.
Thinking and memory are associated with the passage of
these electrical currents. The wisest man who ever lived
comes nowhere near using the full capacity of his won-
e
physical powers and that some experience a decline of

t
mental power. The best current medical opinion is that, in
both cases, what happens is a series of minor 'accidents'
to various parts of our marvelously complicated psycho-
derful mental storehouse. Quite possibly, people in logical mechanism. None of these may be serious by
general employ only ten to twelve percent of the capa- itself, but the total effect, can be severe. Impairment of
bilities of their brains. the brain in the aged is associated with decreased
Your I. Q. is less important than you probably think. circulation of the blood and the precious substances it
Many of us have unnecessary inferiority complex about carries, especially oxygen and glucose. This is probably
our I. Q's–the figure that represents native intelligence why old people remember happenings of their youth
as compared to that of the average individual. It is easy to more vividly than those of the recent past; the youthful
score lower in such a test than you deserve. This might memories were implanted when blood circulation was
result from temporarily ill. health or emotional distur- better.
bances. So, if you have ever seen your score on an I.Q.
Yet, severe mental impairment occurs only in some,
test, you can reasonable be sure that-your I.Q. is at least
elderly people. Everyone knows of men and women who
that high.
What is the physical basis of high intelligence? are vigorous and alert mentally into the ninth or even
Contrary to a common belief, it does not require an tenth decade of life. Their existence proves that impaired
unusually large skull and is likely to be associated with mental powers are not an inevitable accompaniment of
especially -large-number of surface convolutions in the passing years, but a result of disease processes.

582 | CAT Complete Course

Visit : www.EasyEngineering.net
Visit : www.EasyEngineering.net

1. Which of the following can said to be true? Our brain positioning and the value proposition. Imply put, there is
can function continuously for— a real and persistent danger that, caught up excitement
(A) twelve hour and after that it gets tired and hype of a new technology, marketers will once again
let attention to the short-term and tactical overwhelm
(B) an indefinite period of time
consideration of the long-term and strategic. In the new
(C) along period, provided the mental exercise is
world of interactive marketing, tactics often precede
interesting
strategy.
(D) None of the above
How many companies have created their Web site
2. The author is qf the opinion that a person with without thinking about how to maintain it or how to serve
average intelligence— the customer ? How do the capabilities of interactive
(A) cannot expect to achieve as high as a genius can marketing fit with the company's strategy of value
do delivery ? Have salespeople been brought on-line with
(B) usually has thin bundle of nerve fibres in a portable computers, customer databases, and Internet
particular place of the brain. connections before the company has improved any of its
(C) usually possesses high perseverance to achieve core capabilities or processes for value delivery ? Has any
what he sets to do. provision been made for screening and qualifying people

ww
(D) can exceed the record of the greatest genius if who request information ? As in the other national
he uses his intelligence in full. pastime, some hits are better than others.
3. Which one of the following can be concluded from Statistics that one hears cited about the growth of the

w
the passage ?

.Ea
(A) Highly intelligent persons usually have large
skull
(B) As man grows old, he loses the capacity of
Internet strains one's imagination and credulity: 2 million
Web sites; 22 million users in North America; new users
being added at a rate of 12,000 per day; a projection of 1
billion users the year 2000; a typical user spending 20
learning very rapidly
syE
(C) Even the wisest man of the world does not
utilise the full capacity of the brain
hours each week on the Internet. (Whatever be the num-
bers, they are obsolete, given the current rates of growth.)
Who are these people ? Where do they find the time?

ngi
What motivates them What will happen when they have
(D) The most intelligent men of the world are likely to pay for services that are now free? How many of these
to achieve little. Web surfers fit the definition of a potential customer-
4. The author is most likely to agree with which of the
following—
(A) success depends much upon the way we utilise nee
someone who is willing and able to buy the marketer's
products? Anecdotal evidence suggests that very few of
the thousands of marketers now on the Internet are
mental-capacity
(B) Impairment of physical powers due to old age rin
profitable. It has been estimated that only 10% to 15% of
Web surfers actually use the Web for shopping or to
does not hamper its apacity of learning
(C) only person with average intelligence have
g.n
obtain commercial services such as travel information.
But hope, in the form of hockey-stick-shaped profit
forecasts, springs eternally.

e
tenacity to overcome the difficulties of life and
that is
(D) why most of the great men are of average
intelligence
In order to achieve success in life, the main factor is
interactive marketplace should be wary. Old, familiar
paradigms of mass-marketing strategy can be dangerous
t
Marketers who are attempting to address the strategic
issues inherent in becoming a serious player, in the

in this new and unfamiliar setting. Concepts as simple as


a high level intelligence.
the four Ps of Product, Price, Promotion, and Place and
Answers the traditional distinction between strategy and the
1. (B) 2. (D) 3. (C) 4. (A) organizational structure used for implementing strategy
need to be seriously reconsidered and modified or
Passage 10 abandoned altogether.
Words–820 Consider the following : Every product can be
As managers become enchanted with the potential of thought of as information that comes in the form of
the Internet and the interactive marketplace, many Show promised benefits and a definition of value for the
evidence of forgetting some basic lessons of customer. Customers buy benefits and value. When I
Marketing strategy that they learned the hard way purchase Colgate toothpaste, I get a mint-flavored cream
over the past several decades. The most central of these containing sodium monofluorophosphate, but what I am
are the importance of defining and understanding the really buying is the promise of a clean taste, white teeth,
customer, the essential efficiency of market segmentation healthy gums, and fewer cavities. The brand is the
and targeting, and the life-or-death importance of product promise of value. The customer doesn't see product,

CAT Complete Course | 583

Visit : www.EasyEngineering.net
Visit : www.EasyEngineering.net

price, promotion, and place as separate variables. In the 3. We can infer from the passage that the author finds
interactive marketplace, this is even more true. All the the growth statistics of Internet as—
variables interact. They cannot be separated. What the (A) that which has no solid evidence.
customer sees is information that wraps product, price, (B) figments of imagination.
the selling message, and the store into one big value
(C) difficult to believe.
proposition. Equally important, customers in the elec-
tronic marketplace expect that value proposition to be (D) a gimmick and fraud.
tailored to them personally. In fact, they want to be part 4. As per the passage, to be a successful marketer, one
of the design process. The four Ps don't fit here as they do must:
in other contexts. 1. possess superb commun1cation skills and con-
The mass marketplace emphasized transactions built vincing capacity.
on mass communications, but the interactive marketplace 2. be a good listener and be sympathetic.
moves toward relationships based on tailored product 3. be innovative at all points of time.
offerings and messages. Marketers must stop thinking of
(A) Only 1 (B) Both 2 and 3
marketing as persuasion (one-sided communication).
They must develop a new set of conceptual skills that (C) Both 1 and 3 (D) All three
permit the design of customer-specific product offerings

ww
and messages. They must learn how to communicate with
customers, listening to them as well as sending messages 1. (B) 2. (D)
Answers
3. (C)
Passage 11
4. (D)

w
to them.
What does this mean in practical terms? It means Words–646

.Ea
thinking about strategy and organization as integral parts
of a business design that is capable of responding to the
Houston, Texas (CAP) California businessman
Dennis Tito paid about $ 20 million for an eight-day trip

syE
customer's ever changing definition of value. It means
seeing marketing as a set of skills, capabilities, and
processes that must both pervade the organization and
to space. Now, a Houston-based company can send you
to space for $ 50. Well, part of you anyway. Encounter
2001 is working to build an unmanned spacecraft, fill it

ngi
focus on the customer as the center of the business, with DNA samples and messages from up to 4.5 million
guiding the business's evolving strategic response to the people, then blast it beyond the solar system. The com-
customer's changing needs, wants, arid preferences. It pany hopes to launch its spacecraft in late 2003.
also means developing a flexible, responsive organiza-
tional structure that can evolve and move rapidly.
Marketing as a separate function must give way to a set nee "This is a chance for people to participate in a real
space mission," said Charles Chafer, Encounter 2001

rin
president. "Maybe one day it will be found." For $ 50,
of customer-driven processes, some of which may be people can have their digitized photos and messages as
performed by strategic partners in areas such as Web site well as hair samples placed on the spacecraft. Encounter
design and management, order fulfillment, database
management, and credit.
g.n
2001 is the Sister Company of Celestis Inc., which in
April 1997 began using commercial rockets to launch the
1. It can be deduced that the author will agree with the
statement that—
(A) internet site is the most imaginative and
marketed strategy
e
cremated remains of people into space. Encounter's
spacecraft will be made up of a solar sail the size of a
football field and a small container carrying the photos
and messages, plus dehydrated hair samples with the t
(B) qualitative delivery is being neglected in pursuit DNA codes of 4·5 million people. The solar sail the
advancement spacecraft's power source is a very thin sheet of reflective
(C) promotional price in commercial use needs to material that will use the sun's photons to propel it
considered forward, Chafer said. The concept is similar to a sailboat
(D) consumer value is sinking being pushed along the water by the force of the wind.
Like wind, sunlight exerts pressure and a large
2. Which of the following, according to the author, is a
enough sail in space could harness this force and travel
false statement ?
without using fuel. Although a solar sail is at first slower
(A) Integration of four Ps is profitable in interactive
than a conventional rocket, it continues to accelerate over
market
time and achieves a greater velocity. NASA and several
(B) Communication with the customer is of private groups are working on plans to use solar sail
paramount importance in the present scenario technology.
(C) Designing the product on consumer's specifica- The spacecraft, to be launched on an Ariane 5 rocket,
tion leads to success. will orbit Earth for three weeks so ground controllers can
(D) Implementing traditional marketing strategies conduct system checks. After the spacecraft leaves Earth
encourages success even in the new settings. is orbit, it will deploy its solar sail and begin its journey.

584 | CAT Complete Course

Visit : www.EasyEngineering.net
Visit : www.EasyEngineering.net

It will take about 15 years for the spacecraft to fly past 3. What does the author say about 'super civilization ?
Pluto, the solar system's outermost planet. When the (A) They may exist in some other time
sailcraft leaves the solar system, it will be traveling at 7.8 (B) The super civilization may get hold of the DNA
miles per second. That compares with the space shuttle's sample and may try to experiment on it
on-orbit speed of 5 miles per second. The spacecraft's
(C) They will be in a position to send his DNA
imaging component is scheduled to be tested during
samples into the space
space souttle Endeavor’s mission in late November. The
mission is expected to cost about $ 25 million. Most of (D) They may find his clone in the next century.
that is being paid by private investors. Some of the Answers
money, though, is coming from public participation in the
project. About 67,000 people so far have paid to take part 1. (D) 2. (B) 3. (B)
in the mission. Chafer said he expects the bulk of sales of Passage 12
participation kits to occur in the six months before the
launch. Words–517
Jim Glock, a teacher at Fairmont Junior High School Few entrepreneurs start out with both a well-defined
in suburban Deer Park, got 140 students in six of his strategy and a plan for developing an organization that
sixth, seventh and eighth grade classes signed up two can achieve that strategy. In fact, many start-ups, which

ww
years ago. "I wanted them to see how vast the universe is
and the time and distances there are to go from point A,
to point B," Glock said. another purchaser is famed
don't have formal control systems, decision-making
processes, or clear roles for employees, can hardly be
called organizations. The founders of such ventures

w
science fiction writer Arthur C. Clarke, who said he was

.Ea
delighted to be taking part, in some way, in technology he
wrote about. His 1963 short story "The Wind from the
Sun" envisioned space travel by using solar sails. "Fare
improvise. They perform most of the important functions
themselves and make decisions as they go along.
Informality is fine, as long as entrepreneurs aren't

syE
interested in building a large, sustainable business. Once
well my clone!" Clarke wrote on his message, referring to that becomes their goal, however, they must start
his DNA on board. developing formal systems and processes. Such organiza-

ngi
"One day, some super civilization may encounter this tional infrastructure allows a venture to grow, but at the
relic form the vanished species and I may exist in another same time, it increases overhead and may slow down
time, "Clarke told The Associated Press in an interview decision making. How much infrastructure is enough and
late last year. The work of Encounter and Celestis is a
natural progression of the exploration of space, Chafer
said. "Governments open the frontiers but without strong nee
how much is too much ? To match investments in
infrastructure to the requirements of a venture's strategy,
entrepreneurs must consider the degree to which their
commercial components, frontiers don't go anywhere," he
said. "It's sort of a natural evolution, combining real
rin
strategy depends on the following:
As a young venture grows, its founders will probably
missions with the mass market."
1. Which of the following is not a characteristic of the
solar sail ? g.n
need to delegate many of the tasks that they used to
perform. To get employees to perform those tasks

(A) It is the space craft's power source


(B) It is made up of very thin sheet of reflective
material that will use sun's photons to propel
e
competently and diligently, the founders may need to
establish mechanisms to monitor employees and standard
operating procedures and policies. Consider an extreme
example. Randy and Debbi Fields pass along their skills
and knowledge through software that tells employees in
t
forward
(C) It uses sun light and does not require fuel every Fields Cookies shop exactly how to make cookies
and operate the business. The software analyzes data such
(D) Its initial acceleration is very less and thus is
as loca.l weather conditions and the day of the week to
unable to achieve a great velocity
generate hourly instructions about such matters as which
2. Jim Glock got his students to participate in the cookies to bake, when to offer free samples, and when to
mission because— reorder chocolate chips.
(A) he was a science teacher and he wanted his Telling employees how to do their jobs, however,
students to know about this wonder of science. can stifle initiative. Companies that require frontline
(B) he wanted them to know about the distance and employees to act quickly and resourcefully might decide
time required in the universe to go from one to focus more on outcomes than on behaviour, using con-
particular point to another and about the trol systems that set performance targets for employees,
vastness of universe. compare results against objectives, and provide appro-
(C) he was amongst one of the investors. priate incentives.
(D) he wanted them to know about solar sail techno- In a small-scale start-up everyone does a little but of
logy used in space crafts. everything, but as a business grows and tries to achieve

CAT Complete Course | 585

Visit : www.EasyEngineering.net
Visit : www.EasyEngineering.net

economies of scale and scope, employees must be (C) gross operational integration is worthy of reward
assigned dearly defined roles and grouped into a (D) sound businesses invest in financial and organi-
propitiate organizational units. An all-purpose workshop zational structitring
employee for example, might become a machine tool
operator, who is part of a manufacturing unit. Specialized Answers
activities need to be integrated by, for example, creating 1. (C) 2. (A) 3. (D) 4. (B) 5. (D)
the, position of a general manager who coordinates the
Passage 13
manufacturing and marketing functions, or through sys-
tems that are designed to measure and reward employees Words–2473
for cross functional cooperation. Poor integrative mecha- The million dollar question which is top most on
nisms are why geographic expansion, vertical integration, everybody's mind is–what is the realistic external value of
broadening of product lines, and other strategies to the Indian Rupee ? And who is to decide this ? Well, all
achieve economies of scale and scope often fail. governments have meant well when they have said that
Cash-strapped businesses that are trying to grow the Rupee should ideally find its own value and be
need good systems to forecast and monitor the decided by the free forces of demand and supply. But it is
availability of funds. Outside sources of capital such as well said than done. Which government till today or cen-
banks often refuse to advance funds to companies with

ww
tral bank has been able to wash its hand off in deciding
weak controls and organizational infrastructure. Rupee value ?
If entrepreneurs hope to build a company that they RBI for years has lost several billions of foreign
can sell, they-must- start preparing early. Public markets

w
and potential acquirers like to see an extended history of

.Ea
well-kept financial records and controls to reassure them
of the soundness of the business.
1. The author would agree with the statement that—
exchange in arresting the Rupee fall though in vain. The
central bank has decided that the Indian industry does not
want a devaluation of the currency and a mere 7-10 per
cent slow depreciation in the currency would be satis-

syE
(A) Founders are only performance driven
(B) Segregated specialized activities are better
factory as this would lead to a more realistic level. But, is
this form of thinking correct, especially at a time when
dollar denominated exports have under performed and

ngi
always trade deficit today is at a high of around $ 9 billion ? A
(C) Early preparation of sound long-term strategy is notionally competitive Rupee is one aspect of good trade
desirable performance. In such situations RBI should be the sole
(D) Weak structural decisions can be improvised.
2. The author is least likely to agree with the fact that— nee
regulator of the exchange value and should not be roped
in by political lobbies.

rin
(A) Dictating jobs make it easier for the employees. But a look at today's events will show that RBI is
faced with situations of wars and the government falling
(B) The right-incentives ate derived from clear aims.
on which it has no control thus adding a hurdle to their
(C) Initiatives need to be encouraged.
(D) None of the above.
g.n
endeavour to ensure stable financial markets. RBI’s role
gets complicated when faced with complicated tasks of
3. As per the passage which of the following is true ?
(A) Clear cut positions for employees is a prereq-
uisite for any small business
(B) Establishing working procedures is a standard
e
maintaining a stable Rupee and simultaneously ensuring

affect the government’s borrowing programme. The only


option is foreign currency flows will reduce the pressuret
soft interest rates. Any hike in interest rates will adversely

rule for small businesses on the Rupee. This is provided the foreign fund managers
are comfortable with the political situation and judge the
(C) Prediction of results is beneficial
government's intentions in the right earnest.
(D) Weak organizational edifice needs to be eradi- Relaxation in guidelines on ADRs/GDRs and allow-
cated to grow into a bigger organization. ing Indian companies to use US $ 100 million from ADR/
4. Which of the following terms is not mentioned in the GDR funds for acquisition abroad is a stepping stone for
passage ? the Rupee to become convertible on the capital account.
(A) Protracted and steady business. Removing the criteria of a three year track record will
(B) Ease of telecommunication. make more companies tap this route. The government
(C) Funds monitoring. announced these relaxations with good intentions but they
haw; come at a time when global markets are fast moving
(D) Sound control.
and mega mergers and amalgamations worth billions of
5. We may draw a conclusion from the passage that: dollars are taking place the world over resulting in Indian
(A) with growth, delegation of performance should companies standing no chance with mere offerings of
take place $ 109 million. Thus, at this juncture,' the dream of Indian
(B) knowledge and skills should be shared companies becoming MNCs, is a far cry. The government

586 | CAT Complete Course

Visit : www.EasyEngineering.net
Visit : www.EasyEngineering.net

realising that the forex reserves were drying up announced preneur through the ADR/GDR route. This should have
the external commercial borrowings (ECB) programme been done way back in 1995-96, when interest rates were
which later started putting pressure as there was a drain high at 16-18 per cent. Today there is no danger of a run
on the forex reserves which was later replaced by the on forex reserves as they are quite comfortable at $40
GDR programme. Pre-conditions attached only saw core billion and even if there is a sudden panic on Indian paper
companies remain outside the scheme. It was only in floated overseas there is no major cause for concern.
1996-97 when the foreign exchange reserves started to Today the government is still hesitant about moving
show an improvement that the government took bold towards capital account convertibility as it has not
initiatives and relaxed the guidelines. But then they were completed it's homework despite the fact the Indian
faced with a bigger problem of demand recession which economy is fairly stable politically, growth is picking up
proved a dampener for raising equity abroad as excess in most sectors and there are no major volatile shocks
capacities existed in the economy. being experienced in the economy. Why, then, the delay ?
Why did this happen, especially at a time when Dilly dallying on the part of the government will once
growth was picking up and forex reserves were fairly again see India losing out on this golden opportunity of
comfortable ? Tight monetary policy, stringent pre- improving its world rating.
conditions attached to the ADR/GDR route, high interest But there are many complications involved in the

ww
rates and decline in both public and private investments
was a grave mistake. This all resulted in Indian companies
being denied to borrow cheap overseas funds. Public
process and unless and until the government is sure of the
possibilities, it should not take a hasty decision which
would prove to be expensive in the long run. Today

w
limited companies were worst affected as they were
forced to borrow from the domestic market at high

.Ea
interest rates burdening them with equal high interest
liabilities. The government also reduced plan outlays
foreign companies are allowed to be listed on the Indian
bourses. This could possibly mean that the Indian Rupee
is being allowed to be exported for the first time. A
foreign company raising money in India can raise only

syE
which forced public companies to heavily depend on
market borrowings to meet their capital investments.
What was the fallout of all this ? More public companies
Rupee funds. If they have a manufacturing unit then the
intention is quite clear what if they are a non-production
based company what do they do with the Rupee funds if

ngi
became red facing huge debt burdens only because of the they are not allowed to take them out of the country. Will
shortsighted policies of the government. all these are steps in the direction of Indian Rupee
Here again that the government gave its age-old moving towards Capital Account Convertibility (CAC).
justifications that a tight budgetary outlay was being
allocated to curb fiscal deficit and to bring about monetary
harmony in the economy which would 'help curtail its
nee When the report was framed under the chairmanship
of SS Tarpaper there were plenty of challenging tasks
ahead of them. But, today, India has liberalised which
high external debt burden of around $90 billion. But as
usual the government failed in its efforts and domestic rin
will make it easy for the Rupee to move towards CAC.
The gold regime has been liberalised, derivatives have
debt shot up and the country lost the opportunity of
raising cheap overseas funds.
g.n
been ushered and FDI and FIIs have been allowed in
more sectors. Reducing procedural hurdles and making

e
Today, the industrialists are euphoric about the investments hassle free is being given top most priority
liberalised guidelines as this provides the Indian com-
panies an opportunity to become global players. Today
the world over stock markets are volatile and Indian
companies have yet to penetrate listing on world stock
with respect to ADRs/GDRs. The report on CAC set a
three year time frame starting from 1997-98 to move
t
towards, full convertibility. But, the Asian crisis spiked
any discussion on the issue which resulted in the process
getting delayed. Today for foreign nationals CAC exists.
markets. World MNCs would like to guard the interests
of their own companies and would like to ensure But, the crux of the issue was to allow some flexibility
maximum penetration in the overseas market even in the for the Indian residents in effecting outward remittances.
field of Information Technology where India has a clear Currently, a lot of ground work is being done for
edge. If the government has to succeed it will have to foreign companies to get listed on the Indian bourses.
loosen their grip on the domestic money market and try India has liberalised its regime of inward flow of
and reduce the interest rates in which a beginning has currencies. But, the same cannot be said about outflows.
been made–a one per cent reduction was announced in But the encouragement to expand foreign listings on the
the PF rates which will cool off the tight monetary Indian stock markets is a clear signal that a move towards
conditions prevailing in the economy. But with further CAC is on the cards. Freeing FDI and encouraging Indian
cuts in interest rates likely it will no longer be attractive companies to raise equity abroad should provide some
for an Indian entrepreneur to tap overseas markets as cushion in case there is an outflow of funds.
world interest rates would work out to be the same as in But what is necessary at the moment is, for the
the domestic market. Thus, expanding the equity base government to keep a check on the fiscal deficit and have
would not be an attractive option for an Indian entre- a strong financial system in place so as to take a greater

CAT Complete Course | 587

Visit : www.EasyEngineering.net
Visit : www.EasyEngineering.net

strain of the larger currency inflows. Unless and until disruption in the growth process and social pain as assets
these conditions are in place, it will take a while till we were sold dirt cheap in distress sales. In the aftermath of
see Rupee fully convertible both on the current as well as the crisis, many economists have voiced very strongly
capital account. Looking at the Asian crisis, what is against CAC. It is also argued that many companies had
required is to have a proper sequencing of reforms and huge funds which they invested in these Asian countries
have overseas based financial sector wherein deficits are
only for speculative purposes with the sole intention of
firmly in check. The government needs to wake up to this
destabilising these economies which triggered the crisis.
reality and not delay the matter as lags affect the
credibility of a system. Indian markets are used to shocks To avoid such shocks in the future what should be
and an entry will put things in place. Banks and stock done is to allow only the amount of currency to be traded
markets underwent many shocks like entry of private that is actually used up to finance global trade and
players, FII role being enhanced and finally what services. Today, it is estimated that around US $2 trillion
happened the system absorbed the shocks and reacted is moving around in various currencies and only a small
positively. Fundamentals have to be in place and if the part is being used up which is where the entire problem
market requires a proof and although RBI is conservative lies as it is here that speculation creeps in. Volumes in the
at present, it will react favorably in the future and forex markets are so large that they say that currency
eventually put CAC in place. trading is around 20 times of world trade. The question is

ww
Today, with globalisation and integration as the as
buzzwords, dismantling economic barriers of course with
not of trading volumes but is it really required. True
currency trading is required for world trade but unneces-
sary currency trading is fraught with risk. A new code to

w
adequate safeguards CAC is considered to be a sine qua
non for effective economic management in the new

.Ea
millennium. What needs to be guarded is the activities of
the currency speculators and other unscrupulous players
monitor speculative funds is what is required which will
reduce the shocks in the future. There has to be fair
amount of transparency and disclosure practices which
will make the system more healthy. A better practice to

syE
who distort the smooth working of the markets with
speculation and rumors becoming major factors influ-
encing the market. Today, India is quite comfortable with
bring about stability is not to monitor currency stability
but to curb short-term capital inflows which could play an
important role in preventing a crisis in the future.

ngi
its forex reserves, there is a control on inflation, financial
sector reforms are in place, there exists a stringent fiscal Despite India today having a fairly high forex kitty
policy and exchange policy is quite flexible. There cannot and CPI at lows, there are many other indicators that are
be a flight of capital as all is not in liquid form and a
major part of it will be invested. But what Indian finance
ministers have to look into is how to manage the capital nee
flashing red for which India has to take a precaution note.
Looking at the above criteria, it is natural that at this
stage India is still not ready to fully take on CAC. Short-
inflows that will find their way into the developing
countries. Unless and until well utilised at the right time,
rin
term solutions are not the answer especially with the
financial system still groping with high levels of non-
they can create overheating in the economy which would
be a difficult situation if not handled in an organised
manner. g.n
performing assets, capital inflows practically drying up
and the IMF quota already stretched to the .maximum.

But the chief architect of the report, Tarapore himself


is not very confident about the financial sector of the
economy and does not trust the fiscal stability. He firmly
e
So the need of the hour is to have a change in the
conservative nature and thinking of our banking and
financial policies and restructure our macro economic
policies before we opt for CAC. It will take a while for t
believes that the Indian financial system will have to things to be put in place and once a domestic financial
cross many more hurdles before CAC can be ushered in. architecture and macro policies are put in place CAC can
Many argue that a wait and watch approach to put things flow in as a natural process. Volatility and speculation
in place will only result in India losing out. once again hit the India forex markets with the Rupee
At present, there is a dire need for funds to finance seeing an all time low of 44.70 against the US$. That's
several infrastructure projects which are a main foun- when the RBI moved in to check the Rupee fall and
played heavily with the forex reserves to get Rupee back
dation for future investments. Speculators and motivators
to 44.10/20 mark. A 50 per cent surcharge on import
will always exist and we cannot afford to waste our
finance and a 25 per cent surcharge on overdue export
precious time to keep them at bay. The Asian crisis was bills which would be phased out as early as possible to
triggered by unrestricted large inflow of foreign funds in some extent would help correct the distortions. However
largely unregulated and immature markets in Asia. These essentials and government related imports have been left
funds were inter alia utilised for unproductive activities, out from surcharge. RBI has assured the banking and
real estate boom and for creating other long-term assets. exporter community that it would meet any temporary
As a result, when the crisis really developed as a result of demand/supply dollar imbalances which arise due to
the sudden outflow of funds it led to devaluation, leads and lags in the system.

588 | CAT Complete Course

Visit : www.EasyEngineering.net
Visit : www.EasyEngineering.net

1. As per the passage, who should be the ultimate of the Answers


exchange value of the Rupee ? 1. (C) 2. (B) 3. (D) 4. (A) 5. (C) 6. (D)
(A) The Government of India 7. (A)
(B) Market Forces Passage 14
(C) The Reserve Bank of India
Words–645
(D) All of the above
Space exploration is awakening to the hunt for extra-
2. What is the remedy proposed by the author to terrestrial life–or at least the circumstances con-ducive to
stabilize the forex market ? such life. The first step in this quest has been to find
(A) Restrictions on FDI inflow planets outside the solar system. This is crucial as no other
(B) Curbing short-term capital inflows celestials body is even theoretically likely to harbour
(C) Monitoring currency stability organic life. For the longest time the only planets anyone
(D) Liberalizing FII inflows and outflows could be sure of were those circling the sun. not any
3 . What does the passage hold as the crux of the CAC more. A planet’s gravitational pull-tends to shake the star
issue ? that it circles. By measuring stellar wobbling, astronomers
calculate the size and distance of its planets. Over 30"
(A) CAC for foreign nationals
extrasolarplanets have been found circling sun-type stars

ww
(B) Freeing the FDI and FII regime
(C) Allowing ECT limits to be raised
(D) Allowing some flexibility for the Indian resi-
in the past five years. However, most of these planets
have been massive–Jupiter sized or bigger and too close
to their star's to life friendly

w dents in effecting outward remittances

.Ea
4. What according to the author should be learnt from
the Asian crisis in lndia's movement towards CAC ?
(A) To have a proper sequencing of reforms
Scientists recently confirmed the discovery of more
extra solar planets, but ones smaller in size and further
from their stars than any previously found. Several days

syE
ago, astronomers confirmed the discovery of a Jupiter
(B) Fundamental restructuring of the entire financial clone around the star Epsilon Eridani. But this is as far
system from its star as Mars and the asteroids are from the sun.
(C) Introducing derivatives

ngi
This would provide enough space for small, rocky planets
(D) All of the above like the earth also to orbit the star. In March, planet
5. As per the passage, what relationship exists between seekers found single planets around the stars 79 Ceti and
interest rates and maintenance of the stability of the
rupee ?
nee
HD46375. These planets were also remarkable because
they were less than a third of the mass of Jupiter. Still too
big, but a sign astronomers are closing in on their

rin
(A) No relationship maintained
ultimate goal of earth look a likes. What is striking is Ire
(B) Interest rates when low, make rupee value sheer plethora of planets. Dozens are being found within
unstable
(C) Interest rates when high, make rupee value
unstable g.n
the immediate galactic neighbourhood: Epsilon Eridani is
only 10 light years away. Far from being rare, planets
seem to be commonplace.
(D) None of the above
6. Which of the following is false in relation>to the
government reducing the plan outlay ?
e t
Planets are one piece of the jigsaw puzzle of alien
life. Another piece is determining if other planets have an
essential ingredient to life as we know it water. Hence the
excitement when it was announced in June that the space
(A) Reduction of the planned outlay forced public
companies to depend on market borrowings probe, Mars Global Surveyor, had produced two metre
resolution pictures indicated liquid water had flowed on
(B) Government preferred containing fiscal deficit
Mars's surface in the past one or two million years. There
as an explanation for the reduction
has long been evidence of watenon Mars some four or
(C) It was justified as a means of bringing about five billion years ago. Pictures from the National Aero-
monetary harmony in the economy nautical and Space Administration point to the existence
(D) None of the above of underground water which may still be finding its way
7. Why does the author want the government to loosen to the Martian surface. In 2003 NASA's Beagle 2 probe
its grip on the domestic money market ? will be off to the red planet to look for water and life. If
water, let alone life, is confirmed on two of the solar
(A) To allow Indian companies opportunities to system's planets, the likelihood of a universe filled with
become global players strange and exotic beings will take a quantum leap
(B) To improve forex reserves forward.
(C) To get over the problem of demand recessions Technology will soon give planet hunting a big
(D) To reduce the volatility of stock markets boost. It is still impossible to detect the imperceptible

CAT Complete Course | 589

Visit : www.EasyEngineering.net
Visit : www.EasyEngineering.net

wobbling stars experience because of smaller planets. Answers


Astronomers are now using large telescope arrays to catch 1. (D) 2. (D) 3. (D) 4. (B)
medium sized gas planets. More useful is transit photo-
metry, where even a small planet can be tracked by the Passage 15
degree to which it dims a stellar brightness as it passes Words–1831
across the star's face. Last year, this method was used for Spiders can be found in all environments throughout
the first time to detect a planet. NASA wants permission to the entire world, except in the air and sea. These
launch a probe, Kepler, in 2005. Floating in space, Kepler invertebrates of the order Aranea are one of the several
would watch 1,00,000 stars for such transit glimmers. groups of the Class Arachnida, with about thirty four
Space exploration lost its direction and excitement thousand species. They range in body size from only a
with the Cold War's end. Budget cuts have accompanied few millimetres in length to almost five inches. All are
this listlessness. The total lack of public interest in the $ carnivorous and have four pair of walking legs, one pair
20 billion International Space Station is telling. Searching of pedipalps, and one pair of chelicerae. Each chelicerae
for alien life is beginning to catch the popular imagina- consists of a base and a fang. The fang folds up inside of
tion. One example is last month's decision by Microsoft's a groove in the base until needed when attacking food,
cofounder, Mr. Paul Allen, to donate $ 11.5 million of his then moves out to bite and releases venom from a tiny

ww
own money to the Search for Extraterrestrial Intelligence opening at its end as it penetrates the prey. They are also
Institute. A quest brings out the exploratory best in used to "chew", getting, digestive juices inside the body
mankind. And the holy grail of extraterrestrial life is of the prey then squeezing out the liquid lunch. The

w
becoming discernible in the heavens. pedipalps are mainly used to catch and rotate the prey
1. What ruled out the existence of life forms on the 30 while the chelicerae inject it with poison to tear down the

.Ea
extrasolar planets discovered earlier ?
(A) The theory that organic life is not possible only
tissue. Later the bases of the pedipalps are used as
chewing parts. But in males, these palps are used to

syE
on Earth transfer sperm into the female. These twelve appendages
(B) The fact that these planets did not have a central are attached to a dorsal and a ventral plate, the carapace
sun-like star which could provide for the energy and sternum which cover the entire prosoma and provide

ngi
requirement necessary for the development of attachment points.
life The bodies of spiders consist of two parts, an anterior
(C) These planets were too small part called the prosoma and a posterior portion called the
(D) These planets were too close to their stars to
2. Which of the following is true about planets ? nee
opisthosoma. These two portions are held together by a
narrow stalk called the pedicel. This narrow junction
allows for the spider to be very-limber and acts somewhat
(A) The planets are visible through powerful tele-
scopes arrays
rin
as a hinge between the prosoma and opisthosoma. So as a
spider "moves forward creating a web, it can continue in
(B) The planets gravitational pull shakes the star
that it encircles
g.n
a straight line throwing its webbing in the direction it
chooses. This is how spiders create their zig-zag web

e
(C) The planets emit radiation that are detectable on formations.
advanced instruments.
(D) All of the above.
3. Which of the following is true according to the
passage ?
Covering both the prosoma and the opisthosoma is a

t
waxy covering that enables the spider to be a very effi-
cient water conserver. This is one of the characteristics
that spiders evolved to adapt to the harsh conditions of
(A) There are a large number of extra solar planets terrestrial life. There are eight eyes located in the head
(B) There is evidence of existence of water on Mars region usually in two rows, varying among families.
at some point in time Spiders that wait for and lunge at its prey will have a row
of very large eyes well adapted at detecting the precise
(C) It is difficult to detect the wobbling that stars
distance it is from its prey. Yet those spiders that make
experience of very small planets
webs. do not have as great a need for such advanced sight
(D) All of the above
and have smaller eyes. But not all spiders have eight
4. The central theme of the passage is— eyes. There are some spitting spiders that have only six,
(A) The recent scientific developments in the field and there are some with only two or four eyes. Some cave
of space exploration spiders have no eyes at all and rely only on vibration.
(B) The recent developments in the field of There are great differences in the ways which spiders
extraterrestrial research capture prey. Some may stalk their prey, while others
(C) The discovery of extra solar planets may lie in wait and ambush it. Other spiders may weave
(D) The growth of interest in space research over various types of webs used to capture passing prey, and
the years there are some smaller commensal spiders that live in

590 | CAT Complete Course

Visit : www.EasyEngineering.net
Visit : www.EasyEngineering.net

larger spiders' webs and feed on the smaller insects may blunder into its web, darting out and biting it. The
neglected by their host. insect will not be eaten where it is captured, but will be
All spiders spin silk, though not all of them weave taken back into the spider's retreat where the feeding
webs. Silk is most commonly seen used in forming webs, process will actually take place.
which may vary from a highly elaborated orb of spiraling Sheet web do not have any stickiness to them nor is
threads to a single sticky string. Most webs can be placed there a fixed pattern by which they are placed. Instead, an
into one of four different types: the orb webs, the funnel insect that may pass by will become entangled in the
webs, tangle webs, and the sheet webs. The main purpose vertical strands that act like a tripping line, connected to
of a web is for catching prey. With orb weavers the spider's sheet web underneath. Sheet web spiders
(Araneidae), the spider will first form a supporting (linyphiids) always hang beneath their dome web, and
structure of frame threads to which it will then add on when there is prey trapped in the vertical strands, they
radial threads. These tightly strung threads provide quick will shake the web so that it will fall onto the sheet. The
access to anywhere on the web, and also carry any spider will then pull its victim down through the web
vibrations from the outer perimeter to the center. After while biting and poisoning it.
the initial threads are placed, the spider will build on a The tangle web spiders are much like the linyphiids,
catching spiral made of sticky silk. These spirals will be but their sheet has a much more loose and irregular

ww
what capture and snare prey until the spider is able to pattern. Extending down from the sheet are vertical
reach it and inject it with its venom. Orb webs are very strands that are loosely connected to the ground, and are
delicate and lose their stickiness after a short period. So covered with sticky droplets a few millimeters from the

w
many orb weavers take down and replace their old webs ground. An insect passing by that touches one will stick
daily. They recycle the old silk by eating it as they layout

.Ea
to it and break it from the ground. While trying to pull
the new silk. Orb weavers must also consider orientation free it will tangle itself up in more similar strands while
with respect to where the wind is coming from, because the spider drops down to subdue it. Some of these spiders
they will also snag leaves and blowing debris. When the

syE
orb is completed, many orb weavers remain in the center
of the web called the hub. They will wait here for their
prey. When the web is hit by an insect, the spider turns in
build retreats that they cover with dirt and pieces of
leaves that they will hide in and carry prey into to eat. A
variation of this retreat is that of the purse web spider
the hub to face the direction from where the vibration
came. It will then jerk the web sharply to entangle the ngi (Atypus). This spider has a silken retreat that is mostly
buried underground but has a balloon like tube outside
that is covered with soil and bits of debris to appear like
victim by rapidly flexing one of its front legs. Eventually
after it is sure that the prey is stuck in the web, the spider
will follow down the strand. Once it is at a close enough nee
normal ground. When an insect walks across or lands on
it the spider will bite it from beneath and pull it through
the web.
distance to make contact, the spider will rush at and
quickly bite its victim, then retreat away until the venom
rin
Spiders do not only use threads to make webs and
bind prey. Non-web weavers use silk threads to climb up
has taken affect. After subduing the prey, the spider will
wrap it in silk before or after carrying it back to its hub or
the site it may choose to hide. g.n
and down with, as well as for draglines. These latter
threads are used to both help a spider slow-down and to

There are more than 2000 orb weaving species and


no two species build exactly the same web. But in most
cases, the differences are very minor and only concern
the symmetry of the web. But there are three dimensional
e
catch it in case it falls as it leaps from one place to the
next, such as from flower to flower. Jumping spiders,
most common to the class Salticidae, are known for using
t
draglines foranchoring and quick stops. These spiders use
their last pair of legs to propel them from the ground in
orb weavers that add extra threads from the center to an long or short leaps. Salticids use this jumping ability not
outside support, thus pulling out the web into a cone only to catch prey but also to escape danger. These
shape. This enables the spider to wait at the new attach- spiders can jump up to twenty five times their body
ment sometimes being the attaching bridge. When an length, which is very long for an insect with out any
insect flies into the web area the spider may cut or simply specialized jumping legs. As mentioned earlier, jumping
release the web so that it goes back and ensnares the spiders have larger eyes for being able to distinguish
flying victim. visible objects at greater distances. This makes good
Funnel web spiders (Agelenidae) are also common sense, because they have no other way to obtain prey but
spiders. They can be easily found outdoors in short with their own stealth and accuracy. They react very
grasses or small bushes, to large vegetation, and even acutely to any visual stimulus. First they will turn to face
between building edges. Their flat web narrows into a the stimulus and then walk closer towards it. They will
funnel like closure at one end where the spider hides and stalk their prey until within at least ten centimeters to be
waits for victims. This funnel is the spider’s retreat, and able to completely identify it then attack. Once the victim
is opened at both ends. With its legs feeling for any is captured, it is usually con5umed right where it is. Their
vibration, the spider can quickly ambush any insect that front legs are stronger so that they may seize prey, and

CAT Complete Course | 591

Visit : www.EasyEngineering.net
Visit : www.EasyEngineering.net

they have strong perpendicular fangs to penetrate and 10 Revision Test of Reading Comprehension
hold prey firm. Test–1
A similar spider to the jumping spiders is the wolf
Words–422
spider. These spiders lie in ambush and attack their prey.
They too have a large set of eyes on their upper posterior Real-time data is vital for most business. Fast moving
row, above a row of four generally small eyes. Although consumer goods (FMCG) companies have, in a study,
wolf spiders have well developed eyes, they react mainly found that over 5-10 % of their sales are lost from the
to vibrations received from beating wings or movement slow replenishment of fast moving items. Similarly, a
from insects on the ground. As with the jumping spiders, reduction in pilferage would add to the bottom line of an
there are a large pair of fangs that extend down to help enterprise. Technology is integral to logistics.
assist in seizing prey. The most well known wolf spider is We, at Blue Dart, realised this quite early. Since
the tarantula. These spiders can reach up to ten inches in consignment passes through several hands, modes of
their complete lengths. And although, lore has it that they transport, billings and labelling in the long chain from
are one of the most poisonous spiders, their bites are only pick-up to delivery, effective control over it was often not
painful to humans, not deadly. possible. So Blue Dart formed an in-house systems team
Though feeding habits vary with spiders their for developing information technology (IT) solutions, 17

ww
methods of reproduction are all relatively similar, though
each species has its own specific ritual. Because spiders
are cannibalistic, the much smaller male must be very
years ago. Till date, Blue Dart has spent over Rs 55 crore
on technology, with no regrets.
The extensive use of IT affords Blue Dart a premium

w
cautious in 'approaching a potential mate. If he simply

.Ea
rushes in towards the female, the chances are that he will
be seen only as food and consumed: So, spider courtship
has evolved into a special complex pattern that varies in
positioning. Over 79 % of its customers (in revenue
terms) avail of its technology offerings and derive value
out of them. Last year, 58 million shipments were carried
across India on Blue Dart's network, with a reliability

tion, so no gametes are wasted.


1.
syE
each species. This variation allows for species recogni-

What can be concluded from the web making habits


level of 99.96 %. It had introduced an online track and
trace system for international shipments as early as 1988.
Blue Dart's order-tracking tool, Track Dart, enables
of spiders ?
ngi
(A) It is essentially reflective of the spider's eating
clients to track online the status of each order. It helped
automate the pick-up process and eliminate the time--

nee
habits consuming, manual labelling of consignments; details fed
into the computer from the airway bill serve as the label
(B) It is in accordance with the spider's habitat on each carton.

rin
(C) It is triggered in response to different types of
Access to real-time information helps its customers
enemies
as much as the company. Leading pharma firms rely on

g.n
(D) None of the above the Blue Dart technology. A diagnostic centre uses Blue
2. What is false about the salticidaces ? Dart's tracking tool, Internet Dart, to monitor the daily
traffic of samples. A pharma major uses Track Dart to
(A) They have large eyes
(B) They are jumping spiders and can jump up to
(C) They are cannibalistic
(D) None of the above
e
replenish the stock of its distributors. TrackDart enables
the logistics personnel at the client's office and fill
demand gaps accordingly.
Blue Dart's another unique initiative is the backup
t
technology hub set up in Bangalore, a part of its Business
3. Which if the following is an accurate generalization
for all spiders ? Contingency Continuity Plan. Through this, its Bangalore
office serves as the hot standby site with real time
(A) They all have eyes, and use it to spot the food
switchover in the event of any failure at the Mumbai
(B) They all spin webs in order to catch the prey headquarters, which hosts the entire centralised applica-
(C) They all spin silk tions and the Bluedart.com website.
(D) They all jump over the prey and overpower ED! (electronic data interchange), e-commerce, ERP
them (enterprise resource planning) are buzzwords in the
4. An ideal title for the passage would be— business community and the trend looks set to continue
(A) Spiders (B) Arhanids and habits. attracting attention with the arrival of newer innovations
focusing on customer service. In the logistics business, IT
(C) Charolette's Web (D) Spiders and food.
makes all the difference, as it can deliver the advantage to
Answers the company and the client, in a field which is faced with
1. (D) 2. (D) 3. (C) 4. (A) ever increasing competition

592 | CAT Complete Course

Visit : www.EasyEngineering.net
Visit : www.EasyEngineering.net

1. The author is primarily concerned with which of the tracking network. After they confer, the DCGI will likely
following in the passage— call a meeting of the entire group.
(A) Real time data which is integral to logistics The group-National Pharmacovigilance Advisory
(B) Blue dart’s unique back up technology Committee-might request that the designated hospitals
(C) EDI (electronic data inter change) keenly track side-effects in patients prescribed rosigli-
(D) ERP (enterprise resource planning) tazone. "It has to be a focussed effort," says Gupta. The
(E) All of the above drug is not currently part of a list that the DCGI asked
these hospitals to take special note of about a year ago.
2. Blue Dart’s order tracking tool track dart enables its That list included medicines like sildenafil citrate
clients to do all of the following ‘EXCEPT’— (Viagra), where loss of vision is a suspected side-effect,
(A) To track online the status of every individual and painkiller nimesulide, which could harm the liver.
order But it is not that simple. For one, hospitals have to
(B) Automates the pick up process which eliminates prescribe a drug before they can report side-effects. This
the time consuming manual labelling of goods. may not always be the case. Some government hospitals
(C) Real time switch over in case of any failure at don't prescribe the newer medicines, like rosiglitazone,
blue dart’s headquarters. since they are more expensive than the older ones. "Our
(D) Details fed into the computer can serve as a

ww
patient population is not rich. So, our tendency is to
label on each carton prescribe standard drugs in the hospital list," says Meena
(E) None of the above Shrivastava, head, department of pharmacology, Indira

w
3. The most appropriate title for the passage would be Gandhi Government Medical College in Nagpur, Maha-
which one of the following— rashtra. Secondly, doctors have to be trained in the

.Ea
(A) Darting along ahead of the competition
(B) A dart that’s gone a long way
science of reporting. Hypothetically, a so-called accident-
like a car crash-could be the result of a person on a cold

syE
medicine falling asleep at the wheel because of its
(C) Blue Dart’s Bull’s eye sedating effect. In such a case, a medicine might have to
(D) Signed, sealed and delivered carry strong warnings on its label not to drive or operate
(E) Up, up and away machinery after taking it. On the flipside, a perceived
4. The Authors tone in the passage can best be described
as one of— ngi side-effect such as nausea after popping a pain killer
actually may have nothing to do with the drug.
(A) Dismay
(C) Annoyance
(b) Approval
(d) Panegyric nee There has to be sustained effort in training and raising
awareness among doctors in the network. "It is when the
network is active that (the committee) has things to do,"
(E) Amusement
Answers rin
says Ranjit Roy Chaudhury, another member and emeritus
scientist at National Institute of Immunology, New Delhi.
1. (A)
Test–2
2. (C) 3. (B) 4. (B)
g.n
The committee, he says, meets only twice a year on
average. "I would like us to meet more," Chaudhury says.
Clearly, there's miles to go.
Words–495
On 21 may, reports linked UK drug maker Glaxo-
SmithK1ine Pharmaceuticals' Avandia, a brand of dia-
the passage— e
1. The author is chiefly concerned with which issues in

(A) The discovery of a new drug for Indian t


betes drug rosiglitazone, to higher rates of heart ailments diabetics
and cardiac death in western countries. There could be (B) A no side effects drugs for diabetics for Indian
ethnic or other differences in the way Indians - large con- patients
sumers of medicines discovered in the US, Europe and (C) The side effects of foreign manufactured drugs
Japan - react to the same drug but some years ago there that have effected U.K. user adversely and
was no way of knowing. Now, there is a ray of hope. could affect Indians who use the same drugs
Under a two-year-old drug safety programme (D) Rosiglitazone a brand of medicine for diabetics
launched by the health ministry, a network of well-known should be banned in India
hospitals is tracking and reporting suspected drug side- (E) None of the above
effects in Indian patients that they treat. Also, an expert 2. The two year old drug safety programme launched
group evaluates the data and advises the Drugs Controller by the health ministry, where a network of well
General of India (DCGI) on the course of action. Some of known hospital are involved in—
these experts will meet in New Delhi soon, says Y.K. (A) The group tracks and records data of foreign
Gupta, head, department of clinical pharmacology, All manufactured drugs
India Institute of Medical Sciences. Gupta is a member of (B) The network tracks and records suspects side
this group and his hospital is part of the side-effect effects in Indian patients that they treat and

CAT Complete Course | 593

Visit : www.EasyEngineering.net
Visit : www.EasyEngineering.net

advise the DCDI on the course of action to be encashed about Rs. 1.38 crore at the fag end of October
taken especially in regard to drugs like Rosigli- while NM Munjee, an independent director in HDFC and
tazone currently chairman of Development Credit Bank,
(C) To monitor and ban the use of drugs which can encashed about Rs. 92 lakh through multiple transaction
cause heart ailments and cardiac death over the last eight weeks.
(D) Scan, research all foreign drugs in order to It’s not just service sector companies whose top exes
create a drug for diabetic to suit Indian patients have hit the bull’s eye. Take engineering and construction
(E) All of the above giant L and T, which has been facing succession issues.
3. The most appropriate title for the passage would be The company, which had to extend the retirement age of
its top management, including the chairman and managing
(A) Diabetic cure or death cause director has created quite a few millionaries.
(B) Death import Mr. Naik sold shares worth Rs. 8.6 crore between
(C) A challenging task end of October and early November. Among other
(D) Pill to hill Mr. Magapu encashed Rs. 8·5 crore while R.N. Mukhija
and K.V. Ramaswami pulled out approximately Rs. 87
(E) Flatter to deceive lakh and Rs. 84 lakh, respectively
4. The tone of the passage can best be defined as one of The top shots of L and T are still sitting on big

ww
(A) Approval
(C) Disapproval
(E) Panegyric
(B) Caution
(D) Alarm
money, given the value of their unsold shares. For
instance, Mr. Naik’s existing L and T shares are worth
Rs. 420·4 crore, or a whopping $ 105 million, While

w
Mr. Mukhija’s shares are worth Rs. 168·55 crore.
Answers Cement major Ambhuja Cements also proved to be a
1. (C)
Test–3
2. (B) 3. (C)
.Ea 4. (B) gold mine for its top executives. For instance, Mr.
Kulkaeni encashed close to Rs. 1.46 crore through multi-
ple transaction since mid-November. Among others,

syE Words–503
Forget year-end bonuses. Indian professionals have
Ambuja Cements managing director Al Kapur sold shares
worth Rs. 65 lakh and wholetime director BL Taparia
encased shares worth about Rs. 90 lakh as per recent

ngi
capitalized on the stock market boom to make some disclosures to the stock exchange.
serious money in the last quarter of 2007. With the Among other companies ITC director SSH Rehman
markets at record levels, a number of corporate big shots and Anup Singh sold shares worth Rs. 1·68 crore and
have made crores by enchased a part of their stock options
over the last three months. While there are numerous
executives who have pulled out a few lakhs, some nee
1.15 crore, respectively, over the last two months alone.
1. Who have been the top grosser in the run-up to the
new year (2008) ?
hotshots, including Tec Mahindra managing director
Vineet Nayyar, HDFC chairman Deepak Parekh and
rin
(A) Company executives
(B) Mid- level investors
executive director Renu Karnad, Larsen and Toubro
(L and T) chairman and managing director AM Naik and
senior executive vice-president VK Magapu, and Ambuja
Industry g.n
(C) Top-notch executives, V.P.’s and hotshots of

Cements whole time director PB Kulkatni, among others,


have grossed crores in the run-up the New Year.
The biggest gainer of them all was Mr. Nayyar, who
enchased Rs. 23 crore last week in two tranches. The head
risks
e
(D) The modest and careful investors who took big

(E) All blue-chip companies


2. Who is/was the maximum gainer and top compen-
t
sated CEO in the country ?
of the IT outsourcing firm, which went public in 2006, is
still sitting on a neat pile. Given the outstanding shares in (A) Exec Director Renn karnad (HDFC)
his name, Mr. Nayyar currently owns Tech Mahindra (B) HDFC chairman Deepak prarekh
shares worth a little over Rs. 140 crore, or roughly $ 35 (C) Executive director
million, which would place him among the best-com- (D) Larsen and Toubro Chairman-Managing Direc-
pensated chief executives in the country. tor A.N. Naik
Among other well-known names, Mr. Parekh and (E) Tech Mahindra managing director Vineet
Ms. Karnad sold shares worth Rs. 9.3 crore each in mid- Nayyar
December. They had earlier sold shares worth about Rs. 5 3. The most appropriate title for the passage would be
crore each in late September. And that’s not all. The (A) Top- gums blaze
current value of the shares that Ms. Karnad holds stands (B) Sensex boom, top cannons thunder
at Rs. 45·45 crore, or more than $ 11 million while that of (C) India Inc’s top gums make crores or sensex
Mr. Parekh stands at Rs. 60 crore, or about $ 15 million. boom
Some of the other board members of HDFC also (D) Boom, boom certain sex at sensex Boom
encased a part of their holding. For instance, SB Patel has (E) None of these

594 | CAT Complete Course

Visit : www.EasyEngineering.net
Visit : www.EasyEngineering.net

4. The author is most likely to agree to which of the Muslims (20%). Christians have the lowest proportion in
following statements— the bottom third with only 8% belonging to this category,
(A) The stock market is ripe for small time investors compared to 12% Hindus and 25% Muslims.
(B) The sensex will most likely drown those who However, education cannot by itself be the path for
are sailing on it economic progress this is also brought out by their
(C) The heavy weights will benefit greatly from the experience. Their participation in the workforce is
boom roughly the same as for other communities among men
and slightly higher among women. But compared to five
(D) The top guns will soon fall through the roof
years ago, this represents a slowing down of their
(E) The stock market is highly unpredictable
economic contribution, and hence implies lesser oppor-
Answers tunities. This is further confirmed by unemployment
rates. Among Christians, the extent of unemployed has
1. (C) 2. (E) 3. (E) 4. (C)
increased from 4 % to 4.4% in rural areas, and from 7%
Test–4 to 9% in urban areas between 1999-2000 and 2004-05.
Words–554 While there is cause for anger among Christians,
In some ways they are ahead of their compatriots in there is courage engendered by education and culture.
other religious communities, while in others they seem to And, as St. Augustine wrote one and a half millennia ago,

ww
grapple with the same shackles. Compared to other
communities, Christians are better educated, economically
better off and adopt a more equitable attitude towards
Hope has two beautiful daughters. Their names are anger
and courage; anger at the way things are; courage to see

w
they do not remain the way they are.
women. Yet, they have also adopted the caste hierarchy
1. It would be fairly accurate to say that the author’s

.Ea
though in a mellowed form. And, they are struggling with
growing unemployment. Whatever be the reasons, the
relatively small Christian community of India shares a
main concern is—
(A) Christians dwindling socio-economic status

wardness.
syE
complex struggle to shed social and economic back-

Christians have the highest literacy rate among all


(B) Though better off and comparatively more edu-
cated than other community Christians are not
getting their due

ngi
religious communities. For men, it is 80% in rural areas
and 96% in urban areas. For women, it is 69% in rural (C) Christians are not contributing economically
areas and 89 % in urban areas. This is way ahead of other (D) Christians do not endorse female foeticide.
communities, especially for women. Among Hindus and
Muslims, only about 41 % of the women are literate in
the rural areas. In urban areas, 73% of Hindu women and nee (E) Christians have anger and courage
2. The writer backs his statements with—
60% of Muslim women are literate.
This is not just a bureaucratic statistic. Detailed data
(A) Statistics
(B) Example rin
g.n
provided by a National Sample Survey report in 2004-05
shows that a larger proportion of Christian children start (C) National sample survey report
attending educational institutions earlier, and continue till (D) Assumption
later. Moreover, there are proportionately more graduates
among Christians than in any other community.
The Christian community has the highest proportion
of the elderly nearly 20% of the total. Among Hindus it is
14%, while among Muslims, it is 11 %. This may be
(E) St. Augustine view
e
3. The attitude of the passage can best be described
as— t
(A) Didactic (b) Cynical
because of better economic status and educational levels,
which would tend to lower birth rates and increase (C) Condescension (d) Encouraging
longevity, thereby skewing the age structure upwards (E) Informative
compared to other communities. 4. The writer infers that—
The survey also shows that the community treats its (A) The Christian community will overcome all the
women better its sex ratio is the highest among all
odds
communities in India. This can also be partly due to the
fact that a significant segment of the Christian population (B) It’s downhill all the way for the Christians
belongs to the tribal areas of the North East, and the (C) Christians are oppressed
tribals do not endorse the inhuman practice of female (D) Other communities are envious of their high
foeticide or discrimination against the girl child. literacy rate
Sustained educational levels have led the Christians (E) There are no grounds for inference in the entire
to a better economic status in India. According to the passage
NSS Report, 47% of Christians in urban areas and 38% in
the rural areas come within the top third of monthly Answers
earning categories, much ahead of Hindus (24%) and 1. (A) 2. (C) 3. (E) 4. (A)

CAT Complete Course | 595

Visit : www.EasyEngineering.net
Visit : www.EasyEngineering.net

Test–5 they keep, 28% tell a friend or a family member about


Words–502 their resolution, and 21% write it down.
Having a lithe and supple body followed a purse Among those who share their resolution with some-
throbbing with currency notes are the most popular new one, 66% share it with their close friend, 55% share it
year resolutions taken by Indians, an online survey says. with their spouses, and 32% tell their parents about their
The survey conducted global information and media resolution for the new year.
company Nielsen India showed that about 58% of Indians 1. The main idea the author is concerned about in the
surveyed online have improved their fitness levels in the passage can be best summed up by which of the
new year. following—
(A) On line Indians surveyed have resolved to
“This year, ‘remaining fit and healthy’ has all of a
manage their time better
sudden taken the lead in people’s resolution list compared
(B) Indians surveyed this year have resolved to save
to last year, taking over better time management, saving
more money
money, getting organised, and reading more books that
(C) The online survey on Indians showed that they
ranked the top last year,” The Nielsen company (India)
have made resolution to lose weight and improve
Directed of online Panel N.S. Muthukumaran said in a fitness
statement.
(D) That Indians surveyed online have resolved to

ww
According to the survey, 55% of India’s online
population have resolved to save money while 51% have
taken the resolution to manage their time better. Inter-
eat healthy
(E) This year Indians have decided to protect them-
selves from diseases

w
estingly, losing weight and improving fitness and health
were not ranking high in new years resolutions made by

.Ea
people in the past year, Nielsen said in the statement.
Remaining fit and healthy is a more popular resolution
2. Among the people surveyed lately the main reasons
for not keeping their resolution are :
1. Lack of planning
2. Weak commitment

syE
with 73% of people aged between 35 and 44 years.
Saving more money is popular among 62% of people
in the 25-34 years age group while only 26% in the age
3. No support nor energy
4. In sufficient time
5. All of the above
group of 45 years and above have resolved to save more
money in 2008.
Nielsen conducted the surveys using their online ngi (A) 1 and 4
(C) 1 and 2
(B) only 5
(D) 2 and 3
research panel your voice. About 292 people aged 15
years and above were surveyed understand what resolu-
tion people have made for 2008 and how they have fared
nee (E) 3 and 4
3. According to the writer last years survey conducted
by Nielsen India showed which of the following
on resolutions made in the past years.
“People are more health conscious today. They want rin
trends as compared to this years survey top priority—
(A) Better time management
to remain fit not only to look good physically, but also to
protect themselves from diseases. Eating healthy and
(B) Saving more money
(C) Reading more books g.n
regular exercise is the way to achieve optimum health and
fitness,” Muthukumaran added.
In the age of 15-24 years, 14% respondents have
taken the resolution to pursue higher education and 13%
(D) Being more organised
(E) All of the above
e
4. In regard to making resolutions which of the
following ranks highest on the resolutions scale— t
would like to manage their time in a better way. Mean- (A) Making new resolutions every year
while, among the people surveyed, 41% have been (B) Repeating the same resolutions for the past six
making resolutions for the past six to ten years, a quarter to ten years
have been making such resolution for over ten years and (C) Repeating the same resolutions for over ten years
27% respondents make a new years.
(D) Breaking new resolutions with in six months
Further, amongst those who have made new year’s
(E) Unable to sustain their resolution for even 2-3
resolutions in the past, 27% have fulfilled their resolutions months
successfully while 18% have been successful for more
than six months. About 23% have kept their resolutions Answers
for about three months. 1. (C) 2. (C) 3. (E) 4. (B)
Lack of planning (44%) and commitment (37%) are
the key reasons mentioned by respondents for not keep- Test–6
ing their new year resolutions while other factors include Words–891
lack of time, support, and energy. People also have After coming across a four year-old marathon runner,
different ways of making resolutions. While 79% of the all other records relating to age seem superfluous. Still, if
respondents make a mental note of the resolution that you were surprised to know that CEOs a young as 17 year

596 | CAT Complete Course

Visit : www.EasyEngineering.net
Visit : www.EasyEngineering.net

exist, Silicon Valley has some news for you. Anshul joined Bajaj Hindusthan in 2001, when its turnover was
Samar, a 13-year-old, runs a company called Elementeo about Rs 200 crore. It is now eight times that and he
there. The Samars of the world are extreme examples but wants the company to be among the global top three. His
they are symbolic of how age no longer matters for approach is to set targets and let professionals take care
business leadership. India Inc presents more moderate of the rest. In the process, he has largely done away with
examples of this trend. hierarchical boundaries and keeps everyone informed of
Even among the 30 Sensex companies, where the top his actions.
post is still the domain of those over 50 years (not count- Down South, a non-family Old Economy player is
ing a few such as Malvinder Singh, who, at 34, heads rooting for the young in a different way. Ashok Leyland
Ranbaxy), the next generation is already part of the doesn't have a youngster at the helm but its 59-year-old
leadership team. For instance, Tata Steel's VP (Finance) Managing Director R. Seshasayee reckons there's
Koushik Chatterjee, ICICI Bank's Executive Director immense value in tapping the skills of its young execu-
Vembu Vaidyanathan and Bharti Airtel's CFO Sarvjit tives. Last year, for instance, he got them to work on the
Dhillon are all aged 39. Aseem Dhru, a vertical head in exercise of management planning and budgeting, usually
HDFC Bank, is 36. the preserve of the top management. "They did a brilliant
job of challenging the management," says Seshasayee.
One feature that unites business leaders in their 30s
Ashok Leyland, in an ongoing programme, tries to spot
and early 40s is that they were groomed in the post-

ww
liberalisation era. "What is refreshing is that the young
guns do not carry the legacy of the license raj," says Prof
S Sriram, Executive Director of Chennai-based Great
"promising employees" and mentor them. The objective
of all these measures, Seshasayee says, "is that we become
more youthful and therefore more speedy and innovative
in responding to competition.”

w
Lakes Institute of Management. "So, they think free and
think big. They are confident, act fast and are ambitious,"

.Ea
says he. These are vital qualities in a marketplace that's
been turned on its head by the same force of liberalization.
"It was earlier easier to compete because of a closed
A company that has spotted a young leader is
Cognizant Technology Solutions. What does the $ 2-
billion company's 39-year-old CEO Francisco D'Souza
stand for ? "I don't think it's about age. Growth drives the

syE
environment," says Sulajja Firodia Motwani, the 37-year-
old Managing Director of Kinetic Motor Company.
"Today, the speed at which you do business is important.
company and is the fuel that keeps it going." But he isn't
smug. "The biggest issue is individuals or organisations
becoming complacent, falling into the trap of believing
Sulajja feels the customer has turned so savvy that a CEO
has to get the organisation to generate the necessary
energy to meet expectations. "The younger generation ngi success of the past guarantees success of the future. If
there's one thing we have learnt, it's that in today's world
things change too quickly." Amid speed, growth, aggres-
CEO is more pragmatic in decision making," says she.
Agrees India bulls CEO Gagan Banga: "We are younger
and more aggressive. The whole company works around nee
siveness and all those positives attributed to young
leaders, industry observers feel they might need to build
certain softer qualities. Sriram says, "young leaders
facts and data more than anything else."
Tech Check rin
generally think people contribute purely for monetary
rewards. They fail to understand the importance of non-
monetary things such as loyalty and commitment. And
A favourable business environment and the democra-
tizing role of technology have helped create young entre- g.n
tend to run businesses with their brains and not their
heart." Chella also reckons young leaders lack the aspect

e
preneurs who are ready to be unconventional. Take the of genuinely engaging with people. Those gaps could be

t
case of 42 year-old Shantanu Prakash, who decided plugged if leaders start to delegate better and there's a
against a corporate job after passing out of the IIM, support system to help them out, says he. Many believe
Ahmedabad. Instead, in 1994-95, he started setting up the biggest test for them is yet to happen, what with the
computer labs in schools, stepping into a realm where economy being so vibrant over the last decade .
NIIT and Aptech were ruling the roost. His Educomp is 1. The author if the passage is primarily concerned with
now a $ 26-million company. which of the following—
(A) Hierarchical boundaries are being done away
New values are coming up wherever the old guard with
has given way to the new, says Ganesh Chella, founder
(B) Young leaders try to be more inclusive and are
and CEO of HR consulting firm Totus. His list : there's a
against taking people for granted
far higher level of transparency, and the culture of ‘confi-
dentiality has gone; there's a process-based way of (C) The youth are the new leaders of India In
working, which is more structured; finally, young leaders (D) Vim-vigonus and stamina are needed to steer
try to be inclusive and don't take people for granted, as companies
they themselves have grown up in a competitive environ- (E) None of the above
ment. 2. The author cites many example to explain the success
Infusing Young Blood stories of young leaders of successful companies-
‘EXCEPT’—
Thirty-year-old Kushagra Nayan Bajaj provided a (A) The younger lot are more aggressive
good measure of those qualities while transforming his (B) They are more pragmatic in decision making
family sugar company into a formidable player. He and have high energy levels

CAT Complete Course | 597

Visit : www.EasyEngineering.net
Visit : www.EasyEngineering.net

(C) The youngset6 works around facts-data above Shah, who after passing out of Harvard Business School
all else joined Anderson Consulting and was part of the Jerry Rao
(D) They work mainly to achieve monetary gains team when MphasiS was sold. For Chandra Kopparapu,
(E) They work more with the head than the heart Vice President, Asia-Pacific of Foundry Networks who
relocated to India in 2004 after a 14-year stint in the US,
3. The following factors have helped create young it was a brighter career prospect that was attractive. "India
entrepreneurs— is a great learning experience because the way business is
1. Conducive business environment. done here is very different from what we are used to in
2. They were groomed in the post liberalization era. the US. You have to learn your way up," says Kopparapu.
3. Democratising role of technology. Not Just Money
4. New values are sprouting up whenever the old
It is not money alone that is luring Indians back to
guard has given way to new.
their roots. India offers an emotional pull. For the 47-
5. They do not carry the legacy of the license raj. year-old Chief of Jakarta-based Universal Success Enter-
(A) 1, 3, 5 (B) 1, 2, 3, 5 prises, Prasoon Mukherjee, who grew up in Kolkata,
(C) 1, 2, 3, 4, 5 (D) 1, 2, 5 nothing could be more rewarding than doing business in
(E) 2, 3, 4 his homeland. His career, which began with Great Eastern
Hotels, saw Mukherjee climb up the ladder with $25-
4. The attitude of the author can best be described as

ww
one of
(A) Approval (B) Disdain
billion Salim Group of Indonesia as a partner. "Emotional
bondages have grown over the years. Opportunities in
India are much more now than in the time I left. In fact, it

w
(C) Cynicism (D) Skepticism was only after I went abroad that the whole idea of doing
(E) Encouragement business came up," says Mukherjee.

1. (C) 2. (D) 3. (C).Ea


Answers
4. (E)
His nostalgic connect with India is nowhere as
keenly illustrated as in the investments he has lined up
over five years : $ 2 billion. Mukherjee belongs to the
Test–7
syE Words–961
new breed of non-resident Indian businessmen, led by
stalwarts like LN Mittal or Lord Swraj Paul who are on a
comeback trail. Even business houses that had kept a low
"It is the return season for the prodigal sons and
daughters who had left the country for better prospects.
ngi
And they are a legion. It is borne out by the fact that India
profile until now are plotting a return-the Hinduja Group
with stakes in Ashok Leyland, Ennore Foundries,
Induslnd Bank and Hinduja Ventures are giving India a
is among the largest remittance economies, topping
almost $ 24·5 billion last year. Promod Haque's is a case
in point. When his Indian engineering degree wasn't nee
shot again. Says Dheeraj Hinduja, "There will be a
definite change. We are looking at sector like insurance,
real estate, health care and energy.”
enough to land him a proper job, Haque, now Managing
Director of Norwest Venture Partners, decided to pack his
rin
For still others, who are venturing back to India, it is
a question of being helpful to the Indian poor who

g.n
bags for the US in the 1970s for studies. "I was looking represent the other side of the prosperous facade_,For
for an electrical engineering job in India but ended up Vikram Akula, Chief Executive Officer and Founder of
working as a salesperson selling electronics," remembers SKS Microfinance, his commitment is towards the
Haque. A similar compulsion saw Reddy Penumalli,
Managing Director of Analog Devices, book his berth on
a US-bound airline. The tide is turned. India's free
economy has put the country on the map of preferred
investment hotspots and as an incentive to boost its
e
millions who live in rural India. He feels that NRIs have
an obligation to the country, as many of them have
benefited from state-subsidised higher education. "There
needs to be an increase in initiatives by NRIs to bridget
the social divide-whether that be in education or health or
burgeoning growth, it has rolled out the welcome mat to economic development," says Akula.
the Indian diaspora. One upside of such a reverse brain drain is that even
Key Triggers multinationals like Microsoft and IBM, which have a
sizable presence in India, have hired Indians to head their
Driving this reverse migration back to India are operations here. These executives are moved back to
triggers professional and emotional. The IT and techno- India perhaps for their better understanding of the working
logy boom in India has made private equity, hedge funds culture and government policies. "Any multinational
and venture capitalists look towards India. Saurav would be governed by global policies and philosophy.
Srivastava, Kanwal Rekhi and Vinod Khosla are just a However, having a local team always helps in maneu-
few of them warming up to the India Story. After vering around the system," says Rohit Kochar, National
spending 30 years in the US, ex-McKinsey Chief Rajat Chairman and Managing Director of Kochar & Co, a
Gupta has set up a $ 1·5-billion fund under New Silk legal services company.
Route Ventures with his entire team in India. Even the
new graduates fresh from management institutes prefer Even though systems are more streamlined in
being part of the growth here and look for placements. developed countries like the US, one gets used to the
"Professionals are willing to take a salary cut to be in 'Indian way' of working over time. The influx of Indians
India and join the excitement," says 28-year-old Abhi with global skills into India will make their home-grown

598 | CAT Complete Course

Visit : www.EasyEngineering.net
Visit : www.EasyEngineering.net

Indian companies rethink and strive to be more com- As everyone knows, the nation has become a global
petitive. leader in information technology and other high- tech
"The Indian companies have to transform themselves fields such as pharmaceuticals telecommunication and
and compete in the changing dynamics or become telecom based business services. These sectors have
obsolete and exit the market," says Vikas Vasal, Executive contributed to the economy’s rapid growth since 2003
Director of KPMG. Now, with India's growth closing in which has lifted many millions of people out of poverty;
on double digits, it is a rosy scenario. But will the global continued growth could alleviate suffering and expand
Indian leave India if recession sets in ? Some feel that the opportunities for millions more. One day, we may look
Indian who followed opportunity outside and came back back on India’s progress during this decade as one of the
for a more vibrant India could do this again. Though, great humanitarian achievements of our time.
investments having sunk in the creation of assets and Equally exhilarating is how India’s rise may influence
infrastructure-from IT parks and SEZs to manufacturing the global community. The world will be a safer place if
plants-a. downturn might make it difficult to take away other nations can learn from the achievements of what is
these assets. Also, no one expects the boom phase to be not only the largest democracy, but also one of the most
temporary. Even if there are minor setbacks, the objective pluralistic cultures. The prime minister culture, Dr
is the long-term growth and there is unlikely to be any Manmohan singh, has said it well : “India’s success will
knee-jerk reactions. It looks like, for the global Indians, renew humanity’s faith in liberal democracy, in the rule
the good times will roll. of low, in free and open societies.” The entire world has a

ww
1. The most appropriate title for the passage would
most likely be—
(A) The Root cause
big stake in India’s future.
The power of Indian skills and Talent. It seems to me
that the India miracle, if you will, demonstrates the

w
(B) Return of the prodigals
(C) The tide has turned
.Ea
(D) India sons and daughter Return
wisdom of sustained investment in the primary asset of
any modern economy: people. During the nearly 60 years
since independence, India’s investments in human deve-
lopments have reduced hunger, increased literacy and

syE
(E) India shores show the way to wealth improved health conditions. Education investments have
2. The main reason/s why India are returning back to produced world-class scientists, engineers and techni-
India can best be described by— cians. They in turn have fuelled the growth of Indian
technology companies and attracted many global tech-

ngi
(A) Opportunities are more now
nology leaders, including Microsoft.
(B) India’s among the largest remit ten economics
People have been the key to Microsoft’s success in
(C) India’s free economy has turned the tide making

nee
India, and our experience may be illustrative. We entered
out bound Indians turn home word the country 17 years ago working closely with the govern-
(D) Professional emotional quotients ment, IT industry, academia and the local developer
(E) All of the above

rin
community. Over the years, the people of Microsoft India
3. The attitude of the author in the passage can best be have had end-to-end responsibility for the development of
described as one of many Microsoft technologies. They have made important
(A) Enthusiasm
(C) Optimism
(b) Cynicism
(d) Pessimism g.n
contributions to many other products, including windows
Vista and the 2007 Microsoft office system.
We currently employ more than 4,000 people across
(E) Anger
4. In the passage some have agreed that Indians could
do a turnaround the resons stated being—
(A) Lesser pay packets and perks
e
six business units in Delhi, Banglore, Mumbai,
Hyderabad, Kolkata, Pune and Chennai and we continue
t
to expend our presence. Outside the united state, Micro-
soft’s India Development centre is our second largest
(B) More opportunities and better environment product development facility. Two years ago, we opened
abroad Microsoft research India, where scientists and engineers
work to advance the frontiers of knowledge in computer
(C) If a recession sets in they will leave science and related fields, often in collaboration with
(D) Stiff government policies and high interest rates India's academic community. These teams have demon-
(E) All of the above strated India's great capacity for innovation by filing for
100 patents during the past two years. Other India units
Answers play major roles in our world wide customer support,
1. (A) 2. (E) 3. (C) 4. (C) consulting services and management of our internal
information systems.
Test–8 Beyond our direct presence, Microsoft also contri-
Words–1509 butes to India's growth through the thousands of local
partners, large firms and small, that develop and sell
For me(Bill Gates) and anyone else who is passionate products and services based on our software platform.
about using technology to help create opportunities for This year, 35 Indian companies qualified for the Forbes
people trends in India today are tremendously exciting 2000 list of the world's biggest corporate giants. Among
and encouraging. them were four valued Microsoft partners : Tata Consul-

CAT Complete Course | 599

Visit : www.EasyEngineering.net
Visit : www.EasyEngineering.net

tancy, Infosys Technologies Wipro and Satyam Computer In higher education, our efforts have included the
Services. Microsoft is extremely proud to be a part of the Developer Platform Evangelism Academy, which has
economic trans formation that these and other highly provided professional development to more than 1,000 IT
successful Indian companies have helped bring about. and engineering faculty members at 51 Indian colleges.
To help recent engineering graduates transition from
Sustaining Growth, Broadening The Opportunity school to careers, we recently began working with the
How can India best sustain its rapid growth and Indian government and industry on an online employ-
broaden opportunity for all its people? Much has been ability portal. It will enable graduates to assess their
written about the need for sharply increased investments skills, complete appropriate training and connect with
in highways, airports, power plants and other infra- prospective employers.
structure. Economists also point to a need for regulatory
reforms and better ,public services provided more trans- Technology And India's Future
parently. These are important challenges. Besides being an important tool in education and a
Also, from my perspective, investment in human growth sector of the Indian economy, information tech-
capital should continue to be a high priority, especially nology can aid social and economic development in many
efforts to further alleviate hunger, reduce illiteracy and ways. Wide deployment of computers, software and
improve public health. Threats to healths such as HIV/ telecommunications helps boost productivity and reduce
AIDS, for example, could upset much of India's recent transaction costs in many sectors, strengthening economic

ww
progress. The estimate is that less than one per cent of
adults are infected, but because of India's large popula-
tion, the number is among the highest in the world.
growth. Computers, mobile devices and software can help
expand the quality and availability of health care and
other public services, as well as education.

w
Education at every level remain crucial for continued A lack of access to technology, on the other hand,
growth. Output of college and university graduates is can hinder development. More than 30 years after the

.Ea
impressive in absolute terms, and has been a great source
of economic strength, but India cannot afford to become
complacent. The nation now faces an acute shortage of
invention of one of the most versatile and empowering
technologies of our time, the personal computer is readily
available to only 1 billion of the world's more than 6

syE
skilled workers, as Infosys and other employers have
warned recently. Education spending as a percentage of
GDP lags far behind that of countries such as South
billion people. Microsoft's founding vision of "a
computer on every desk and in every home" is a reality
for the roughly 1 billion people living near the top of the
Korea and Taiwan. Yet, one could argue that India needs
a skilled and educated workforce even more than the so-
called Asian Tigers do. They accelerated their develop- ngi global economic pyramid. But the digital revolution has
yet to spread very far in many rural areas, impoverished
communities and developing countries, including India.
ment through manufacturing, primarily, while India's
focus on services and technology makes its workforce
skills especially critical. nee Disparities in technology access are troubling, for as
the global economy is increasingly computerised and
moves online, social and economic development becomes
As many others have said and as the government has
recognised in its budget plans, India urgently needs to
build more primary and secondary schools, improve rin
even more difficult in the places and for the people left
behind, on the less fortunate side of the digital divide.
teaching and ensure that more children attend school, es-
pecially in rural areas. Higher education needs to be
g.n
This is a problem that Microsoft and others in the
information technology industry have been working to
address.

e
expanded and upgraded. Top-tier institutions are overrun
Microsoft's ultimate goal is to bring the benefits of

t
with applicants, while skill levels among graduates of
some other colleges do not meet world standards or the technology to every person. Toward that end, we have set
needs of employers. By one estimate, 25 % of all new our sights on an ambitious milestone: With governments
engineering graduates lack the skills to be employable in and other partners, we aim to deliver the power of
the IT industry, despite its dire need for workers. information technology to 1 billion more people
Microsoft is committed to helping improve Indian worldwide by the year 2015. We are expanding several
education. Over the past several years, we have been technology training and assistance programs.
engaged in many collaborative efforts, mainly focused on And we recently introduced the low-cost Microsoft
advancing the instructional uses of technology and Student Innovation Suite of software product including
expanding access to computers and computer skills. For versions of Windows, Microsoft Office, Learning Essen-
example, our Project Shiksha currently works with more tials and Microsoft Math. Although we invested many
than 10 state governments, bringing computer skills millions of dollars to develop these products, the suite will
training to more than 120,000 teachers so far. We have be available to students for about Rs 127, through govern-
helped enhance learning opportunities available to ment programmes in India and many other developing
students in slum and rural schools through support for countries, and in developed countries as part of targeted
Digital Study Hall, a project that records and distributes programmes that provide PCs to disadvantaged students.
DVDs of classes led by India's best grassroots teachers. We are taking these and other steps because, as
And to help overcome a scarcity of classroom computers, industry leaders and simply as human beings, we believe
Microsoft Research India has developed Windows Multi that all 6 billion people who share this planet deserve a
Point, a technology that enables several students to work chance to realise their full potential. We are especially
on a single PC. excited to be working toward realising this vision in

600 | CAT Complete Course

Visit : www.EasyEngineering.net
Visit : www.EasyEngineering.net

India, where progress on many fronts is already well cation is being processed by the employer - means that
underway . one can not switch jobs or be up for a promotion. Also,
1. The author’s chief concern in the passage is directly long-term decisions like buying a house or other
linked to— investments are normally put on hold until one is certain
(A) The highly exciting technological trends in of his status in the country. "It is like being in a state of
India contribute to the economy’s repaid growth limbo," says Nanda Kulkarni, who has been in the US for
(B) Pharmaceutical’s upsurge in India sends out more than seven years.
globally encouraging signals The bill hopes to transition the green card process
(C) India’s success renews humanity’s faith in into a merit-based points system, like the way it is done
liberal democracy in Canada and Australia. If rolled out, this system would
(D) India is the largest democracy and has the most remove the dependency on the employer and award green
pluralistic of cultures cards based on individual merit. While this would stand
(E) None of the above to benefit many employees, employers are not too pleased
as they lose control over who they want to retain in the
2. According to the author which of the following has
company. An Indian business owner in New Jersey,
contributed the most in reducing hunger and improv-
speaking on condition of anonymity, says, "The points
ing education and health in India—
system will judge paper resumes, not hands-on skills, that
(A) The sustained investment in the main asset of is not always an accurate estimate of a good employee."

ww India-people.
(B) Investment in rural development
(C) Investment in adult literacy
The details of the points system will be formalised
only after the bill is passed. But the tentative version that
is put forward is skewed toward illegal immigrants,

w
(D) Investment in health
(E) All of the above

.Ea
3. As the global economy moves online development
for any country lies in investing in the following:
according to some observers. “Undocumented workers
get points for owning property as that means they have
contributed to the economy, but legal, skilled worker
don’t get any points for that,” says Vikas Chowdhry, a

syE
(A) Literacy, health education and facilities and
repaid technological growth
(B) Rural development and infrastructure
software professional, who is also caught in the green
card quagmire.
More importantly, the proposed reform dose not

ngi
(C) Adult education factor in the backlogged applicants who have entered the
(D) Computer education country legally and are in line for their green cards.
(E) All of the above “India has the highest number of people in this backlog,”
4. The author’s tone the passage is—
(A) Encouraging
(C) Instructive
(B) Disapproving
(D) Cautious
nee
says Jwalant, who represents advocacy group
Immigration Voice.

rin
In a bid to move to the new points system and also to
(E) Condescending accommodate illegal immigrants into the fold, the number
of green cards available for backlogged applicants in the

1. (A) 2. (A)
Answers
3. (A) 4. (A)
g.n
old system has been set at 90,000. “This is a reduction
from the previous number of 140,000 available to us,”
says Aman Kapoor, president of Immigration Voice. He
Test–9
Words–1234
Rajesh Srinivasan, a software programmer in New
Jersey, regularly scans Naukry.com looking for possible
e
adds that when the cap was 140,000, applicants had to
wait for 6-10 years to get the green card. Now that the
numbers available to this backlogged group has further
decreased, the wait time could be anywhere 12-15 years.
t
opportunities in India. Srinivasan came to the US in 1999 The proposed points system, if passed, would go into
to pursue a master's degree in computer engineering. Eight effect only one financial year later. Moving to that system
years later, this lawabiding, regular taxpayer has still not would mean going back to the bottom of the queue.
received his permanent residency–the elusive green card. Besides, given that there are uncertainties involved with
His application lies somewhere in the backlog of 500,000 how the system will pay out, not many are willing to
applicants, and it could be another couple of years before make the shift.
he gets the green card. Meanwhile, a proposed Immigra- There is also a per-country limit on the number of
tion Bill by the Bush administration lays ,out an elaborate green cards available. A little ironical, considering that
plan to grant legal status to the 12 million illegal immi- there is little ironical, considering that there is no country
grants in the country, while not providing any clear time- cap on the number of H-1b visas issued. Typically, a
lines to legal, skilled workers like Srinivasan, who are in larger number of H-1B visas go to high skilled workers
the long queue for permanent residency. from India, China and Russia than other countries. In that
Basically, Indians who come to work in the US do so case, having a standard cap of 10 percent per year for
either on an H-lB visa or anL-1 visa. They can then apply each country will skew the distribution resulting in “some
for permanent residency through the employer, a process people being forced to go back," says a visibly frustrated
that can take anywhere between six and 10years. Kapoor, who feels the proposed Bill has implications that
Working on an H-lB visa - while the green card appli- are unfair to the H -1 B visa holders.

CAT Complete Course | 601

Visit : www.EasyEngineering.net
Visit : www.EasyEngineering.net

Several Indians share his sentiment. Having waited 1. The most beautiful headline for the passage would
patiently for immigration reform to make things easier for most likely be—
them in the country, the currently proposed Bill leaves (A) green card conundrum
many of them disappointed. "It would have been much (B) H- 1B or Bye
easier for me if I had entered the country illegally," says
Abhishek Katra, who faces the possibility of being sent (C) Green card is red hard
back to India because his green card application is not (D) Green stop
more than a year old, which is required for H-lB workers (E) Green card- stop or Go
who have been in the US for over six years.
2. The proposed immigration bill by the bush adminis-
Massachusetts Senator Edward M. Kennedy issued a tration according to the author liberates which con-
lengthy statement on his website supporting the proposed fining is best explanation by which of the following ?
Bill. Regarding the inflow of skilled labour, he said, "Our
plan recognises that our economy will continue to need (A) It plans to grant legal status to the 12 million
hardworking people who are willing to come here for a illegal immigrants in the country
few years... . We need computer programmers, scientists (B) Through it plane to grant legal status to illegal
and engineers. So our programme will allow them to immigrant it does not define a lucid time from
come as guest workers under a programme with strong to legal entrants
labour laws that protect American jobs and wages." (C) The underlying catch is that they have to apply

ww
The Bill does suggest increasing the number ofH-lB
visas to 115,000 annually from the present cap of 65,000,
with a possibility of increasing the cap to 180,000.
for legal status solely through their employees
(D) The problems that lies in the bill is that one con-
nect switch jobs or get promoted while the

w
However, it has raised the fee for filling an H-lB visa

.Ea
from $1,500 (61,500) to $5,000 (Rs 2,05,000) and added
some stringent guidelines on the use of the H-IB visa.
Employers with more than 50 employees may only hire
employer is processing the green card applica-
tion
(E) All of the above

syE
50 per cent of its employees on an H-lB visa. Also, a
company will be restricted from hiring H -lB visa holders
and placing them as consultants in other companies.
3. The main that legal immigrants applying for perma-
nent residency has—
(A) Undocumented workers get points for owing

ngi
Students, however, would be allowed to work for two property
years after graduation on optional practical training, (B) The proposed reforms do not factor backlogged
instead of one. applicants
"They are basically trying to make it difficult for
employers to hire people on H -lB visas," says New
York-based immigration attorney Kavitha Ramasami, nee (C) Reduction in the number of green card as
composed to previous years

rin
(D) The irony of the per country limit on the
who advises several Indian companies on their immi-
number of cards issued as there are no country
gration policies. The Bill is, in a way, setting the stage for
caps on the number of H –1 B visas issued

g.n
more off, shoring. As companies will be unable to afford
the expensive H-IB visas, they will star sending more (E) All of the above
work to off shore locations. "Capital is always going to Answers
follow skill," say : Kapoor, who is certain that many
Indians will leave the country if positive changes to
immigration don't happen soon. "Ten years ago, Indians
would probably have stuck it out rather than go back, but
today India has much more to offer," says Voice's
1. (A)
Test–10
2. (B) 3. (E)
e Words–1233
t
Pradhan. It is late in the evening your wife wants to eat
Chinese food, but you cannot seem to remember where
The American Immigration Lawyers Association
that restaurant your friend was talking about is located.
says, "Increased H-IB fees are nothing more than a tax on
So you go to Yahoo's Our City service, select your city
innovation that will end up driving US jobs; overseas by
and type in "Chinese food". Immediately a list of
making it more difficult to hire the highly educated talent
restaurants comes up, along with their locations on map.
America needs."
Result: great meal and a happy spouse. All this, thanks to
While the Bill has yet to pass in Congress, lots of global Internet giants, who are now localising their ap-
changes can be expected as the US tries to clean up its plications for Indian consumers.
immigration system. Some senators have proposed Two years ago, the research and development (R&D)
amendments based on the feedback from their consti- centres of global Internet giants in India worked almost
tuents. "There will be changes before the Bill finally entirely on developing products for other developed
comes out, hopefully they are positive," says attorney markets. This was done for the same reasons as the soft-
Allen Kaye, a New York attorney and ex-president of the ware outsourcing sector-cost efficiency and availability
American Immigration Lawyers Association. of talent. But the twist in the outsourcing tale for the
Meanwhile, those like Srinivasan, who fear being Internet space is India's growing importance as a market
lost in transition, are exploring their back-up options. . by itself.

602 | CAT Complete Course

Visit : www.EasyEngineering.net
Visit : www.EasyEngineering.net

Increasingly, global players are focussing on deve- Take languages. As the user base inches closer to the
loping products and applications specific to India. While 100- million mark, Internet companies will have to reach
the current Internet user base (about 50 million) is not out to the non-English speaking populace for further
very large relative to the population, India is now the growth. India's regional diversity requites offering the
fastest growing Internet market in the world. With the same vices in different languages. Rediff, which offers its
potential to go over 200 million users over the next mail service in 11 Indian languages, uses predictive text
several years (behind only China and the US), India is a to help users type in the local language. "Since the lack of
market worth pursuing early on for these players. local language hardware is a limitation, we have to work
Traditional markets such as the US and UK are getting around the software," says Manish Agarwal, V.P.
saturated in terms of growth in numbers of Internet users. (marketing), Rediff.
Today, 80 per cent of new Internet users come from What makes this difficult is that Indian languages are
outside the US with emerging markets adding numbers phonetic. The text is unlike Roman script, which is the
rapidly. And India is right in the middle of this growth, basis for most European languages. Indian languages also
making the country a valuable market for Internet do not have established transliteration rules, like Japanese
companies over the next five to ten years. or Chinese. That is, one Hindi name can be spelled
For companies such as Yahoo! and Google, which differently in English - Mansi, Manasi, Maanasi, and so
already have large R&D facilities here, there is no better on. Google's recently launched Blogger, in Hindi, was led
way to corner the India market than using local talent to by its India centre. Prasad Ram, head (R&D), Google

ww
work on local products. Almost all of the company's
India-specific applications are led by, if not entirely
developed in, their centres here. In the past two months
India, in Bangalore, says "The product is an example of
technology contribution that can be leveraged in other
non-Roman languages."

w
alone, the India R&D centres of Yahoo! and Google
developed at least two products each for the India market.

.Ea
Yahoo! launched Our Cities and Yahoo! Maps, while
Google launched Hindi Blogger and a Hindi news service
on Google News.
Secondly, the Internet will now become more local.
In developed countries, people use it to locate places- the
nearest bus stop, a convenient Wi-fi spot, and so on. This
requires accurate mapping. The problem in India is that

syE
there is no reliable geographical data available. In most
As India's Internet market expands, products from countries, such data comes from sources such as a
the India centres will focus on addressing the needs of government database. Here, companies have to resort to
these users. Why is this a big deal? India differs from satellite imagery to get their information, which is not an
other Internet markets due to its regional and linguistic
diversity, which poses some unique challenges from a
technology standpoint. If Internet companies can find a ngi easy task. These companies are now trying out different
ways to create an online presence for previously un-
mapped locations. Home-grown search company Guruji.
way to overcome the barriers here, it will help them
understand users in other countries where Internet is yet
to take off. Technology platforms developed to tackle
nee
com, for example, has ,tied up with Infomedia for access
to local data bases.
Thirdly, given the large scale adoption of mobile
challenges here could then be extended to other regions
with conditions similar to ours. This means that India
rin
telephony in the country, firms are devoting significant
resources towards making Internet access available across
could lead the development of technology to crack other
markets where growth is possible. India is also one of the
few large markets where the mobile revolution took off g.n
platforms. "The Internet is becoming device agnostic,"
says Jaspreet Bindra, country manager, MSN India in
Mumbai. MSN is looking at the mobile platform as the
before the Internet If that route to Internet access is
exploited well, it could become a model for regions such
as Africa.
Yahoo! recently made its Bangalore centre the R&D
key to cracking
e t
India's market. Microsoft's research lab in Hyderabad
is developing specific products for mobile devices. One
such product is an "on-deck" search application, which
hub for emerging markets. That Includes India, Latin searches for items in the mobile operator's domain. For
America and other Asian markets, like Vietnam and In- example, if you are a Hutch user, you could use its data-
donesia. Sharad Sharma, the newly appointed head (R&D) base for all available Kishore Kumar song downloads.
at Yahoo! India, says a key part of his mandate is to build While R&D for the Indian market has begun, it is
still only a small portion of the work that the Indian
more products specifically for emerging markets. "There
centres do. About 80 per cent of the product development
is definitely a greater emphasis on the Bangalore centre is still for foreign markets, But this will gradually
because of this," he says. decrease over the next few years.
Challenges Ahead Companies are ramping up R&D operations in India
like never before. Last year, Yahoo! doubled its R.&D
If the number of Internet connection in India have to workforce to 1,000. It had taken them four years to reach
increase substantially, there must be growth beyond the 400. And the recruitment is not just happening at the
metros. Internet services need to become more regional engineering level. Yahoo! also made several key appoint-
more local and easily accessible. They need to be in ments at the top, even pulling people from its global
regional languages and the options need to become more headquarters. Pranesh Anthapur moved from Yahoo's
localised. For example, a user should be able to locate the Sunnyvale office to take over as COO (R&D), Yahoo!
nearest auto showroom to his house, online. India. With the Bangalore centre, Yahoo's global R&D

CAT Complete Course | 603

Visit : www.EasyEngineering.net
Visit : www.EasyEngineering.net

capabilities have nearly doubled, claims Anthapur. like (D) WWW. Internet .IN
Yahoo!, search giant Google is also recruiting developers (E) Indian spiders crawling on the NET
in large numbers. For both companies, the Indian market
represents their largest R&D operations outside the US. 3. ‘Global Internet giants are localising their applica-
tions’ in India implies—
As India's Internet base becomes more active, there
(A) Indias’ growing importance as a market
will be more online services catering to the needs of
(B) India is presently the fastest growing internet
India's market. Dinners at bad restaurants will soon be a
market in the world
thing of the past.
(C) India has the potential to exceed 200 million
1. According to passage the author would most likely users over the next coming years
agree to which of the following suggestions. If the
(D) US and UK market are getting saturated in
number of internet connections have to increase
terms of numbers of internet user
substantially ?
(E) All of the above
1. Internet services need to become a lot more
regional. 4. Which of the following road blocks that are being
faced by Indian internet companies ‘EXCEPT’—
2. The internet services should be in regional
languages. (a) As Indian languages do not have established

ww
3. Options need to be more localised.
4. Indians languages are phonetic and do not have
transliteration rules which make it difficult for
transliteration rules. Hence, the use of predictive
text
(b) Accurate mapping is difficult as there is no

w internet users.
(A) 1, 2, 4
(C) 1 and 4
.Ea (B) 1, 2, 3
(D) 1, 2 and 4
reliable geographical data available
(c) There is a huge non English speaking populace
(d) Red tapism obstructs and hinders any such ven-
tures
(E) 1, 2, 3, 4

syE
2. The most catchy and apt title for the passage would
most likely be—
(A) 1, 2, 3
(C) Only 4
(E) 3 and 4
(B) 3, 4 only
(D) Only 1

(A) On Dot Indian


(B) Localising the Net ngi 1. (B) 2. (D)
Answers
3. (E) 4. (C)

nee
(C) Spread the Net inwards ●●●

rin
g.n
e t

604 | CAT Complete Course

Visit : www.EasyEngineering.net
Visit : www.EasyEngineering.net

2 Synonyms
Synonyms are words of the same grammatical class Average Medium, Ordinary, Fair
that have a similar, but not an identical meaning— Awkward Clumsy, Ungainly, Ungraceful, Embar-
Words Synonyms rassing
A B
Abandon Forsake, Give up, Abdicate, Relinquish Bane Curse, Mischief, Harm, Scourge
Abandoned Deserted, Vacant Banish Exile, Dismiss, Expel
Abdicate Give up, Vacate Bard Singer, Minstrel

ww
Ability Capacity, Aptitude, Capability, Expert-
ness
Base
Bashful
Vile, Low, Mean, Ignoble, Unworthy
Timid, Shy, Diffident, Constrained

w
Abnormal Unusual, Unnatural Bear Endure, Suffer, Tolerate, Sustain
Abound Plentiful, Overflow, Abundant Beautiful Pretty, Lovely, Graceful, Handsome,
Abrupt
Absorb .Ea
Sudden, Hasty
Assimilate, Merge, Take in Beg
Charming, Elegant
Implore, Beseech, Ask, Solicit
Absurd
Abundant
Accelerate
syE
Silly, Ridiculous
Ample, Plentiful
Quicken, Hasten, Speed up
Belief
Benevolent
Faith, Trust, Confidence, Credence
Generous, Philanthropic, Charitable

ngi
Blithe Joyous, Gay, Light-hearted, Merry
Active Alert, Agile, Alive Bogus Sham, Counterfeit, Spurious, False
Acute Keen, Piercing, Sharp Bold Courageous, Undaunted, Intrepid,
Adept
Adjust
Expert, Apt, Skilful, Dexterous
Regulate, Accommodate nee
Bounty
Impudent
Generosity, Gift
Admirable
Admit
Excellent, Commendable, Praiseworthy
Own up, Concede, Acknowledge,
Brave
Breathe rin
Courageous, Fearless, Valiant, Bold
Respire, Inhale, Exhale

Agree
Aid
Accept
Concur, Consent, Assent
Help , Relief, Support, Assistance
Bright
Build g.n
Lustrous, Luminous, Radiant, Glowing
Make, Construct, Fashion, Erect
Aimless
Allow
Amateur
Random, Wandering, Purposeless
Grant, Admit, Permit
Volunteer, Novice
Burning
Busy e
Flaming, Raging, Blazing
Engaged, Occupied, Engrossed, Em-
ployed t
Ambition Aspiration, Longing C
Anger Arouse, Inflame, Annoy, Provoke Callous Hardened, Insensitive, Stiff
Anger Resentment, Wrath, Rage, fury Calm Impassive, Placid, Serene, Cool-headed,
Apology Excuse, Pardon, Regret, Amends Tranquil
Appetite Want, Hunger, Longing, Passion Candid Frank, Blunt, Outspoken, Straight
forward
Apt Suitable, Qualified, Appropriate
Capture Arrest, Nab, Seize, Apprehend
Arrogance Pride, Insolence, Haughtiness
Artistic Beautiful, Graceful Champion Supporter, Defender, Protector, Victor
Ascent Rise, Lift, Elevation, Upswing Chaos Confusion, Disorder
Assist Help, Support Chaste Pure, Clean, Virtuous, Undefiled
Atrocity Outrage, Enormity, Brutality Cheat Defraud, Swindle, Dupe, Deceive
Attack Assault, Invasion, Aggression, Chronic Habitual, Confirmed
Onslaught Clear Transparent, Lucid, Distinct
Attain Win, Gain, Achieve, Succeed Clever Able, Skilful, Adroit, Intelligent

CAT Complete Course | 605

Visit : www.EasyEngineering.net
Visit : www.EasyEngineering.net

Cold Frigid, Cool, Chilly, Frosty, Indifferent, Deliver Relieve, Rescue, Redeem, Discharge
Reserved Demolish Destroy, Raze, Wreck, Wipe out
Commend Praise, Applaud, Approve, Compliment Denounce Decry, Censure, Accuse, Curse
Conceal Hide, Veil, Shroud, Disguise Depict Show, Exhibit, Portray, Delineate
Concern Anxiety, Solicitude, Worry Derisive Sarcastic, Contemptuous, Scornful,
Confess Admit, Acknowledge, Avow, Own Mocking
Confidence Reliance, Faith, Dependence Derogatory Defamatory, Humiliating, Discreditable
Conflict Fight, Clash, Strife, Collision Desolate Bleak, Barren, Lonely, Forlorn
Conjecture Guess, Speculation, Inference, Surmise Despise Scorn, Disdain, Dislike, Detest
Conquer Win, Overcome, Vanquish, Triumph Destination Goal, End, Objective, Target
Considerate Thoughtful, Kind, Sympathetic, Destroy Demolish, Ruin, Devastate, Raze
Humane Determination Resolution, Will, Firm, Decision,
Consolation Solace, Comfort, Sympathy, Hopeful- Judgement
ness Determined Resolute, Resolved, Decided, Firm
Contain Receive, Include, Hold, Comprise Detest Loathe, Abhor, Despise

ww
Contemptible Despicable, Despised, Vile, Mean, Low
Contribute Give, help, Assist, Subscribe
Devastate
Develop
Ravage, Destroy, Pillage, Ruin
Grow, Cultivate, Produce, Evolve

w
Control
Convict
Copious
Direct, Manage, Regular, Master

.Ea
Criminal, Prisoner, Captive
Plentiful, Ample, Abundant, Adequate
Devotee

Dexterous
Dignified
Votary, Disciple, Worshipper, Enthu-
siast
Skilful, Deft, Clever, Adept
Stately, Lofty, August, Noble, Majestic
Corrupt
Courage
Covet
syE
Dishonest, Demoralised, Debase
Boldness, Valour, Bravery, Audacity
Long for, Want, Desire, Crave
Dim
Diminish
Dusky, Dull, Dingy, Hazy, Vague
Lessen, Reduce, dwindle

Cruelty Brutality,
Torture
Barbarity, Persecution,
ngi
Disaster

Discard
Misfortune,
Catastrophe
Calamity, Tragedy,

Reject, Abandon, Eliminate, Repudiate,


Cultivate
Cunning
Grow, Farm, Develop
Craft, Shrewd, Sly, Wily nee
Discord
Remove
Strife, Dissension, Contention
Curtail Limit,
Diminish
Shorten, Abate, Reduce, Discover
Disfigure
rin
Find, Disclose, Reveal, Discern
Impair, Deface, Mar, Mutilate
Cut Hew, Chop, Carve, Slit, Slash, Divide,
Curtail, Reap
D
Disgrace

g.n
Insult, Dishonour, Humiliation
Disinterested Indifferent, Detached, Unconcerned

Damage
Dangerous
Dead
Loss, Harm, Injury, Hurt, Impair
Hazardous, Perilous, Unsafe
Deceased, Lifeless, Extinct, Obsolete
Display
Dispute
Distasteful e
Show, Exhibit, Disclose
Quarrel, Contention, Controversy
Unpalatable, Unsavoury, Bitter
Distinguished Celebrated, Famous, Eminent, Illus-
t
Dear Cherished, Precious, Costly, Valuable trious, Renowned, Noted
Decide Settle, Finalize, Fix, Determine Distress Discomfort, Trouble, Grief, Anxiety
Deduce Derive, Infer, Conclude, Reckon Distribute Divide, Classify, Scatter, Apportion
Deed Act, Action, Work Diversity Variety, Variation, Difference
Divine Holy, Godlike, Celestial, Superhuman
Defeat Conquer, Overcome, Foil, Frustrate
Docile Gentle, Submissive, Obedient, Teach-
Defend Protect, Shield, Guard, Safeguard,
able
Support
Dogmatic Bigoted, Pinioned
Defer Delay, Suspend, Postpone, Stay
Dreadful Fearful, Awful, Dire, Horrible
Dejection Depression, Grief, Sorrow, Despon-
dency Dynamic Forceful, Vigorous, Impelling
Delay Put off, Dally, Postpone, Procrastinate E
Deliberate Intentional, Studied, Thoughtful Eager Ardent, Zealous, Fervent, Earnest
Delightful Pleasing, Enjoyable, Charming, Alluring Earnest Serious, Solemn, Fervent, Determined

606 | CAT Complete Course

Visit : www.EasyEngineering.net
Visit : www.EasyEngineering.net

Economical Frugal, Careful, Thrifty, Saving G


Ecstasy Joy, Bliss, Rapture, Enthusiasm Gay Blithe, Lively, Merry, Jolly, Bright
Effort Attempt, Endeavour, Trial, Venture Generous Magnanimous, Unselfish, Liberal,
Emerge Issue, Appear, Arise, Come Forth Lavish
Eminence Distinction, Importance Genuine Sincere, Frank, Unaffected, Honest,
Empty Blank, Vacant, Void, Hollow Sterling, Authentic
Encourage Inspire, Animate, Strengthen, Embolden Gift Present, Donation, Bounty, Contribution
Endeavour Attempt, Struggle, seek, Strive Glad Pleased, Gratified, Happy, Joyful
Endure Continue, Last, Abide, Sustain Gloom Dejection, Shadow, Obscurity, Darkness
Enemy Foe, Opponent, Adversary, Antagonist
Greedy Grasping, Avaricious, Rapacious
Enormous Huge, Excessive, Tremendous, Colossal,
Grief Sorrow, Distress, Affliction, Tribulation,
Gigantic
Anguish
Enthusiasm Zeal, Spirit, Force, Fervour
Habitual Customary, Usual, Routine, Frequent,
Entreat Request, Implore, Plead, Solicit
Permanent

ww
Eradicate
Erect
Essential
Abolish, Eliminate, Exterminate
Raise, Build, Construct, Setup
Indispensable, Necessary
Hallucination Illusion, Delusion, Mirage
Hamper Obstruct, Impede, Restrict

Esteem
w
Eternal

Examine
Respect, Regard, Honour

.Ea
Endless, Everlasting, Immortal, Per-
petual
Test, Inspect, Scrutinize, Review
Hard
Hardship

Harmful
Difficult, Stern, Firm, Cruel
Difficulty, Adversity, Calamity, Afflic-
tion
Injurious, Detrimental, Damaging
Expedient
syE
Advisable, Desirable, Advantageous,
suitable, proper
Hasty Quick, Speedy, Expeditious, Swift,
Rapid, Hurried

ngi
Expensive Costly, Dear, High-priced, Prodigal Haven Refuge, Shelter, Protection, Sanctuary
Extravagant Lavish, Excessive, Absurd Helpful Useful, Cooperative, Assisting
F

nee
Holy Pious, Blessed, Saintly, Godly
Fabricate Build, Construct, Invent, Make-up Homage Respect, Tribute, Reverence
Fade Pale, Dim, Vanish Honest Open, Sincere, Frank
Faint
Fair
Swoon, Fade, Weaken, Dim
Beautiful, Comely, Spotless, Unblem-
Horrible
Horror rin
Dreadful, Appalling, Frightful
Dread, Terror, Disgust, Aversion

Fame
ished
Repute, Eminence, Renown, Distinction
Hostage
Humane g.n
Security, Pledge, Bond, Guarantee
Kind, Merciful, Tender, Sympathetic
Fatal
Fear

Fierce
Deadly, Lethal, Mortal, Fateful
Timidity, Diffidence, Apprehensiveness,
Dread, Fearfulness
Violent, Aggressive, Savage, Ferocious
Humble
e
Modest, Meek, Submissive
Hypocritical Insincere, Affected
I
t
Fit Appropriate, Expedient, Suitable, Proper Ideal Model, Perfect, Example, Paragon,
Flaw Defect, Fault, Mistake, Imperfection Epitome
Forbid Prohibit, Inhibit, Ban Idle Inactive, Useless, Futile, Unemployed
Formal Conventional, Ceremonious, Ritualistic
Ignoble Vile, Lease, Low, Detestable
Fortitude Courage, Patience, Endurance
Illegal Unlawful, Illegitimate, Illicit
Fragrant Aromatic, Balmy, Scented, Perfumed
Illustrious Famous, Distinguished, Eminent
Frail Fragile, Infirm, Feeble
Free Unconfined, Unchecked, Unhindered, Imaginative Original, Inventive, Creative, Idealistic,
Liberal Romantic
Frenzy Fury, rage, excitement, Mania, Agitation Immense Vast, Great, Tremendous, Huge
Friendly Amicable, Sociable, Cordial, Warm- Immoral Corrupt, Licentious, Characterless,
hearted debase
Furious Angry, Fuming, Raging, Turbulent, Impossible Unlikely, Unreasonable, Incredible,
Wild Unfeasible

CAT Complete Course | 607

Visit : www.EasyEngineering.net
Visit : www.EasyEngineering.net

Impure Foul, Dirty, Filthy, Debased, Mixed, Lurid Ghastly, Grim, Gruesome, Pale, Bright,
Lewd, Unchaste Brilliant
Incapable Incompetent, Unable, Inept Luxuriant Lush, Lavish, Profuse, Fertile
Incite Stir, urge, Provoke, Instigate M
Incredulous Sceptical, Distrustful, Doubtful, Merciless Ruthless, Unfeeling, Relentless, Pitiless
Unbelievable Mad Crazy, Insane, Frenzied, Wild
Independence Freedom, Liberty, Self-reliance Magnificent Grand, Splendid, Noble, Superb
Indifference Apathy, Unconcern, Coldness, Manifest Show, Display, Disclose, Bring forward
Neutrality
Marvel Wonder, Surprise, Astonishment
Industrious Diligent, Busy, Hard-working , Active,
Marvellous Wonderful, Incredible, Miraculous
Assiduous
Massacre Killing, Butchery, Slaughter, Carnage
Infinite Endless, Timeless, Limitless, Boundless
Meek Mild, Submissive, Humble, Modest
Ingenuous Frank, Open, Candid
Melancholy Sad, Dejected, Depressed, Dispirited
Insignificant Petty, Unimportant, Trivial, Small
Miserable Wretched, Forlorn, Worthless, Pathetic
Intellectual Mental, Scholarly, Learned, Well-

ww informed, intelligent
Intemperance Excess, Unrestricted, Dissipation
Mitigate
Monotonous
Mourn
Lessen, Alleviate, Soften, Abate
Humdrum, Tedious, Tiresome, Unvaried
Lament, Grieve, Bewail, Bemoan

w
Interesting Exciting, Alluring, Fascinating
Irritable Ill-humoured, Peevish, Petulant Mutiny Rebellion, Revolt, Uprising, Insurgency

Jealous .Ea
J
Envious, Distrustful, Suspicious
Mysterious Baffling,
Inscrutable
Obscure, Enigmatic,

Jolly
Junk syE
Merry, Jocose, Exuberant
Rubbish, Waste, Refuse, Trash
Mysterious Obscure, Occult Incomprehensible,
Hidden
N
Just
Justify
Juvenile
Upright, fair, Proper, Right
Defend, Prove, Vindicate, Explain
Youthful, Immature, Pubescent, Junior, ngi
Narrate
Naughty
Relate, Tell, Recite, Describe
Mischievous, Wayward, Defiant
Young
K nee
Neat
Nefarious
Tidy, Trim, Spick-and-span, Orderly
Evil, Wicked, Shameful, Unlawful

Keen Piercing, Penetrating, Cutting; Quick,


Eager
Nimble
Novel
rin
Spry, Agile, Alert, Brisk
New, Different, Unusual, Unique
Kind Benign, Gracious, Obliging, Type, Sort,
Character, Nature
Nuisance
g.n
Annoyance, Bother, Pest, Bore
O
Kindle
Knave
Knowledge
Ignite, Set on fire, Light, Stir, Arouse
Rogue, Rascal, Miscreant, Scamp
Information, learning, Understanding
L
Oath
Objectionable
Obligatory
Obscene
e
Pledge, Vow, Promise, Avowal
Obnoxious, Offensive, Undesirable
Compulsory, Imperative, Necessary
Immoral, Corrupt, Indecent, Unchaste
t
Laborious Toilsome, Hard-working, Industrious Obsolete Antiquated, Discarded, Outworn, Extinct
Lack Want, Need, Shortage, Deficiency Obstinate Tenacious, Stubborn, Obdurate, Dogged
Languish Weaken, Wither, Droop, Fade, Decline Obtain Get, Attain, Acquire, Procure
Lasting Enduring, Permanent, Continuing Obvious Manifest, Clear, Evident, Plain
Lazy Slow, Sluggish, Indolent, Slothful, Idle Offence Insult, Affront, Attack, Wrong
Liable Subject to, Open to, Exposed to, Old Aged, Elderly, Ancient, Antique,
Accountable Outdated
Liberty Freedom, Independence, Emancipation Oppressive Stifle, Tyrannical, Overbearing
Loathe Despise, Abhor, Detest, Hate Ostentatious Pretentious, Showy, Glittering
Loquacious Talkative, Chatter box, Garrulous, Outlaw Criminal, Bandit, Fugitive, Desperado
Chatty Outrage Insult, Wrong, Affront, Indignation,
Lucid Plain, Clear, Understandable, Cogent, Fury, Anger
Comprehensible Outstanding Prominent, Eminent, Remarkable

608 | CAT Complete Course

Visit : www.EasyEngineering.net
Visit : www.EasyEngineering.net

Overcome Conquer, Defeat, Overthrow, Surmount Q


Overlook Oversee, Skip, Miss, Forget Quack Impostor, Charlatan, Fraud
P Qualified Fit, Capable, Able, Accomplished,
Pact Treaty, Accord, Agreement, Bargain Eligible
Quarrel Dispute, Controversy, Wrangle
Paralyze Disable, Incapacitate, Cripple, Unnerve
Queer Quaint, Strange, Odd
Pardon Forgive, excuse, Absolution, Overlook
Quick Prompt, Alert, Ready, Rapid, Keen
Pathetic Touching, Sad, Pitiful, Distressing, Intelligent
Heartrending
Quiet Peaceful, Silent, Serene, Tranquil
Peak Top, Apex, Summit, Pinnacle
Quit Leave, Abandon, Resign, Stop
Pensive Melancholy, Meditative, Reflective R
Pensive Thoughtful, Sad, Dejected, Reflective
Radiant Bright, Brilliant
Perennial Lasting, Endless, Persistent
Rage Fury, Wrath, Frenzy, Craze
Peril Danger, Risk, Hazard, Chance Raise Lift, Stir up, Incite, Increase

ww
Pernicious Harmful, Detrimental, Ruinous, Sinister Rapid Swift, Speedy, Quick, Fleet, Fast
Perpetual Eternal, Everlasting, Ceaseless, Incessant Rash Incautious, Indiscreet, Hasty, Wild,
Persecute Oppress, Annoy, Trouble, Molest, Foolhardy

Pierce
w Maltreat
Philosophical Calm,

.Ea
Rational,
Deliberative, Speculative
Bore, Puncture, Penetrate
Imperturbable,
Raze
Reasonable
Demolish, Destroy, Level, Obliterate
Cheap, Low-Priced, Moderate, Fair,
Credible, Plausible

syE
Recover Regain, Redeem, Salvage, Recuperate
Pious Religious, Devout, Holy, Reverent Refined Cultivated, Cultured, Well-bred
Pithy Brief, Terse, Succinct, Forceful, Power- Regard Respect, Esteem, Consider

Placid
ful
Calm, Peaceful, Gentle, Serene
ngi Release
Reliable
Free, Liberate, Relinquish
Responsible, Trustworthy, Dependable
Plead
Pleasure
Pledge
Assert, State, Urge, Allege
Delight, Gladness, Joy, Enjoyment
Promise, Vow, Oath, Security
nee
Reliance

Remarkable
Assurance, Confidence, Credence,
Trust
Distinguished, Noteworthy, Rare,
Poisonous
Polite
Virulent, Deadly, Toxic, Venomous
Civil, Courteous, Polished, Refined Rescue
rin
Uncommon, Unusual
Liberate, Set free, Deliver, Save
Ponder
Postpone
Think, Muse, Reflect, Deliberate
Delay, Defer, Shelve
Rigid
Rival g.n
Stiff, Strict, Inflexible, Unyielding
Competitor, Opponent, Contender,
Praise
Pray
Precious
Applause, Approval, Acclaim
Implore, Beg, Request, Entreat
Priceless, Costly, Invaluable, Dear
Rival

Robust
Antagonist

Adversary e
Competitor, Contestant, Antagonist

Strong, Hardy, Seasoned, Tough, Sound


t
Prevent Hinder, Stop, Check, Restrain Romantic Idealistic, Fanciful, Poetic, Imaginary,
Prize Award, Medal, Trophy, Laurels Sentimental
Probe Search, Investigate, Explore, Verify Rude Barbaric, Uncivil, Insolent, Discour-
Proceed Advance, Progress, Move, Continue teous
Proficient Expert, Adept, Well-versed, skilful Ruin Destruction, Downfall, Wreck, Perdi-
Proficient Expert, Adept, Masterly, Dexterous, tion
Adroit Ruthless Cruel, Pitiless, Merciless, Relentless
Prominent Notable, Salient, Eminent, Distin- S
guished Sacred Holy, Divine, Hallowed, Blessed, Con-
Proper Right, Just, Equitable, Meet, Suitable secrated, Sanctified, Sacrosanct
Protect Defend, Guard, Save, Shield Sad Gloomy, Woeful, Melancholy, Mournful
Prudence Discretion, carefulness, caution Sarcastic Satirical, Cynical, Biting
Prudent Careful, Wary, Heedful, Cautious, Savage Wild, Rude, Barbarous, Uncivilized
Discreet, Circumspect Scanty Scarce, Insufficient, Meagre, Limited

CAT Complete Course | 609

Visit : www.EasyEngineering.net
Visit : www.EasyEngineering.net

Scorn Contempt, Despise, Derision, Disdain Uphold Approve, Support, Maintain, Champion
Scrupulous Strict, Careful, Conscientious Uproar Tumult, Hubbub, Bedlam
Secret Clandestine, Hidden, Confidential Urbane Polite, Courteous, Civil, Mannerly
Seize Grasp, Capture, Arrest, Confiscate Urge Exhort, Press, Plead, Solicit
Sensational Electrifying, Exciting, Stirring V
Sentimental Romantic, Emotional, Maudlin
Vague Unclear, Obscure, Indefinite, Indistinct,
Shake Tremble, Flutter, Quiver, Quake, Vibrate Ambiguous
Sham Imitation, Counterfeit, Make-Believe,
Valiant Brave, Valorous, Undaunted
Spurious
Vanish Disappear, Fade out, Dissolve
Shame Disgrace, Dishonour, Humiliation,
Ignominy Venture Dare, Risk, Hazard, Undertake
Shun Avoid, Evade, Elude, Eschew Vessel Utensil, Jar, Vase, Bowl, Sailing Vessel,
Solitary Lonely, Isolated, Sole, Lone Ship
Soothe Quiet, Assuage, Console, Comfort Vice Wickedness, Sin, Frailty
Sorry Grieved, Sorrowful, Remorseful, Victory Conquest, Triumph, Success, Winning

ww
Splendid
Apologetic
Gorgeous, Glorious, Magnificent
Vigilant
Vigilant
Alert, Wary, Watchful, Cautious
Cautious, Heedful, Chary, Prudent,

w
Spurious
Strange
Strenuous
Strife
False, Counterfeit, Sham, Specious

.Ea
Odd, Queer, Unusual, Unfamiliar
Energetic, Ardent, Zealous; Persevering
Contention, Rivalry, Discord, Dispute
Vigour
Vindictive
Alert, Attentive
Energy, Stamina, Force, Vitality
Revengeful, Resentful, Avenging

Struggle
Summit syE
Fight, Battle, Strive, Endeavour
Top, Apex, Peak, Zenith
Vivacious
Voluble
Sprightly, Animated; Lively, Gay, Brisk
Talkative, Fluent, Glib, Loquacious
W
Superfluous

Swift
Unnecessary, Needless, Useless, Extra,
Redundant
Rapid, Speedy, Quick, Fleet ngi
Wane Decrease, Decline, Ebb, Lessen,
diminish, fade

Tale
T
Story, Anecdote, Fable nee
Want
Wanton
Wish, Crave, Covet, Lack, Need
Capricious, Wayward, Perverse,

Target Aim, Goal, Object, Mark


Wedlock
rin
Unchaste, Immoral
Marriage, Wedding, Matrimony, Pledge

g.n
Teach Instruct, Educate, Train, Guide Wicked Evil, Bad, Cruel, Heartless
Tedious Dull, Dry, Wearisome, Uninteresting Wily Crafty, Artful, Sly, Subtle
Tepid
Tidy
Timid
Transient
Lukewarm, Warm, Mild
Neat, Spruce, Trim, Methodical
Cowardly, Shy, Diffident, Timorous
Momentary, Brief, Passing, Not lasting
Wisdom
Withhold
Witty
e
Learning, Prudence, Judgement
Restrain, Detain, Hold back, Suppress
Clever, Smart, Humorous t
Wordy Verbose, Prolix, Diffuse, Digressive
Tremendous Huge, Colossal, Stupendous
Worldly Earthly, Mundane, Carnal
Trifling Trivial, Frivolous
Wretched Mean, Miserable, Unfortunate,
Trust Faith, Belief, Reliance, Confidence
Deplorable
Tyranny Despotism, Absolutism, Autocracy,
Y
Cruelty
Yearn Pine, Long, Hanker
U
Yield Product, Produce, Give in, Succumb
Ugly Unlovely, Coarse, Plain, Shapeless, Ill- Young Youthful, Juvenile, Childlike, Immature
made, Surly
Z
Uncouth Rough, Rude, Unpolished
Zeal Enthusiasm, Eagerness, Earnestness,
Understanding Insight, Perception, Discernment Passion
Unique Singular, Matchless, Unprecedented Zenith Summit, Top, Apex, Climax
Unmistakable Certain, Sure, Positive, Clear Zest Enthusiasm, Zest, Devotion to a cause

610 | CAT Complete Course

Visit : www.EasyEngineering.net
Visit : www.EasyEngineering.net

3 Antonyms
An antonym is a word that has a meaning opposite to Acute Insensitive, Dull, Blind, Deaf
that of another word. English Language is very rich in Add Subtract, Remove, Withdraw
vocabulary i.e., in synonyms, antonyms, homophones, etc.
Addicted Averse, Disinclined
Below is given a list of antonyms. An exhaustive list of
antonyms of words is given. Antonyms of other important Adequate Inadequate, deficient
words have also been mentioned. Adhere Separate, Loosen
A Adherent Defector, Renegade, Dropout

ww
Abandon Join, Engage, Unite, Embrace, Retain Adjacent Apart, Separate, Distant
Abandoned Righteous, Virtuous Adjoining Separate, Distant, Remote
Abbreviate Lengthen, Extend, Augment Adjourn Assemble, Convene, Begin
Able
w
Abnormal
Abolish
Unable, Incompetent, Incapable

.Ea
Normal, Average, Usual
Establish, Keep, Retain
Admirable
Admiration
Admire
Contemptible, Despicable, Sorry
Contempt, Disdain, Disrespect
Loathe, Despise
Abominable
Above syE
Admirable, Fine, Noble
Below, Under, Beneath
Admit
Admonish
Ado
Deny, Obstruct, Reject
Praise, Commend, Extol
Quietness, Composure, Tranquillity

ngi
Abridge Expand, Extend, Increase
Abrupt Gradual Adore Hate, Despise, Loathe
Absence Presence, Existence Adorn Strip, Bare, Denude
Absent-minded
Absolute
Alert, Attentive, Observant, Aware
Partial, Limited, Fragmentary, nee
Adroit
Adult
Clumsy, Awkward, Graceless
Immature, Infantile

Absolutely
Incomplete
Doubtfully, Questionably
Advance
rin
Retreat, Withdraw, Flee, Retard,
Obstruct, Decline
Absolve
Absorb
Involve, Blame, Accuse, Charge
Leak, Drain
Advantage
Hindranceg.n
Disadvantage, Drawback, Obstacle,

Abstract
Abundance
Abundant
Concrete, Real, Substantial
Scarcity, Want, Dearth, Absence
Scarce, Scant, Absent, Rare, Uncom-
Adversary
Adverse
Adversity
Friend, Ally

e
Beneficial, Favourable
t
Prosperity, Felicity, Bliss, Fortune,
Happiness, Benefit
mon, Insufficient
Accelerate Slow down Advisable Inadvisable, Ill-considered, Imprudent
Accept Refuse, Reject, Ignore Advocate Oppose, Withstand, Resist, Opponent,
Foe, Adversary
Accompany Abandon, Leave, Forsake
Affection Dislike, Aversion, Antipathy
Accord Disagreement, Difference, Quarrel,
Discord, Dissent Affirm Deny, Contradict, Repudiate,
Disclaim
Accurate Inaccurate, Inexact, Mistaken, Wrong
Affliction Benefit, Gain, Relief, Comfort
Accuse Absolve, Clear, Discharge, Acquit
Afraid Unafraid, Bold, Cool, Confident
Acknowledge Deny, Refuse, Reject
Against For, In favour of, Pro, In support of,
Acquire Lose, Forfeit With
Acquit Condemn, Sentence Aggravate Ease, Relieve, Soothe
Active Lazy, Lethargic, Inactive, Apathetic Aggressive Peaceful, Friendly, Amicable, Passive,
Actual Unreal, Pretended, Fake, Bogus, False Shy, Timid

CAT Complete Course | 611

Visit : www.EasyEngineering.net
Visit : www.EasyEngineering.net

Agile Clumsy, Awkward, Inept Antagonism Friendliness, Geniality, Cordiality


Agitate Calm, Soothe Anxiety Peacefulness, Placidity, Calmness,
Agree Differ, Disagree, Argue, Refuse Tranquillity
Agreement Disagreement, Discord, Misunder- Anxious Calm, Composed
standing Apathetic Interested, Concerned
Aid Impede, Obstruct, Hinder Apologetic Unrepentant, Excited, Stubborn,
Ailing Well, Hale, Hearty Obstinate
Alarm Calm, Soothe, Comfort Appal Please, Edify, Comfort
Alert Listless, Dulled, Sluggish Apparent Real, Actual
Alien Familiar, Commonplace, Accustomed Appeal Repel, Repulse
Alight Board, Embark Appear Disappear, Vanish, Evaporate
Alike Different, Distinct, Separate Appetite Distaste, Surfeit
Alive Dead, Moribund, Inactive, Insensible Applaud Disapprove of, Criticize, Condemn,
Denounce
Allay Arouse, Worsen, Intensify

ww
Allege
Allegiance
Deny, Refute, Contradict
Disloyalty, Treachery, Infidelity
Appreciate Take for granted, Scorn, Depreciate,
Undervalue

w
Apprehension Composure, Confidence
Alleviate Aggravate, Embitter, Heighten
Apprehensive Calm, Composed
Allow
Inhibit
.Ea
Forbid, prohibit, Disallow, Deny,
Approach
Appropriate
Recede, Go away
Inappropriate, Unfit, Inapt
Ally
Alone
Aloof
syE
Enemy, Foe, Adversary
Accompanied, Together
Friendly, outgoing, Cordial, Warm
Approve
Approximate
Disapprove, Oppose
Exact, Precise, Perfect

Alter
Always
Keep, Preserve, Maintain
Never, Rarely ngi
Apt Unfit, Ill-becoming, Unsuitable, Inapt,
Unlikely

nee
Ardent Cold, Unemotional, Feeble
Amateur Professional
Arduous Easy
Amaze Bore, Tire, Weary
Argue Agree, Concur
Ambiguous
Ameliorate
Clear, Unmistakable,
Worsen
Argument
Arouse rin
Agreement, Harmony, Accord
Calm, Settle, Soothe, Dull
Amiable
Amiably
Disagreeable, Ill-tempered
Surly, Displeasing
Arrange
Array g.n
Disarrange, Disturb, Disorder
Disarray, Disorder
Amiss

Ample
Amplify
Properly, Correctly, Rightly, Right,
Correct
Insufficient, Inadequate
Abridge, Condense
Arrest
Arrive
Arrogance
e
Liberate, Release, Set free
Leave, Depart
Humility, Modesty
t
Arrogant Humble, Modest
Amuse Bore, Tire
Artful Artless, Simple, Naive
Amusement Boredom, Tedium
Artificial Real, Genuine, Authentic
Ancestor Descendant
Artistic Tasteless, Dull, Flat
Ancestry Posterity, Descendants
Ascend Descend, Go down
Ancient New, Fresh, Recent, Current
Angel Devil, Demon Ashamed Proud, Self-respecting
Anger Forbearance, Patience Asleep Awake, Alert
Angry Happy, Content, Peaceful, Tranquil Assemble Disassemble, Scatter, Disperse
Animate Inanimate, Dead Assent Refusal, Disapproval
Announce Suppress, Stifle, Censor Assert Deny, Contradict, Decline, Reject
Annoy Comfort, Soothe, Please Assertion Denial, Contradiction
Answer Question, Query, Inquiry Assist Hinder, Obstruct, Impede, Thwart

612 | CAT Complete Course

Visit : www.EasyEngineering.net
Visit : www.EasyEngineering.net

Assistance Obstruction, Interference C


Associate Separate, Divide, Avoid Calm Stormy, Boisterous
Assorted Same, Alike Candid Evasive, Foxy, Tricky
Assure Deny, Equivocate Care Neglect, Carelessness, Indifference,
Astonish Bore, Tire Heedlessness
Astute Slow, Duel Caution Negligent, Reckless
Attach Detach, Unfasten, Loosen Cavity Bulge, Protuberance
Attachment Detachment Cheerful Gloomy, Pensive
Attack Withdraw, Retreat Civilized Savage, Wild, Uncivilized
Attention Inattention, Absent-minded Comfortable Uncomfortable, Miserable, Cheerless
Attract Repel, Repulse Complicated Simple
Attraction Repulsion Compose Contrast
Attractive Unattractive, Plain, Ugly Conclude Begin

ww
Audible Inaudible, Indistinct Concord Discord
Austere Lenient, Permissive, Soft, Luxurious, Confess Deny, Renounce
Fancy, Opulent Confidence Diffidence

w
Austerity
Authentic
Authorize
Comfort, Luxury

.Ea
Fake, Bogus, Imitation, Counterfeit
Prohibit, Forbid
Conscious
Consent
Contempt
Unconscious, Unaware, Oblivious
Dissent
Respect, Esteem, Reverence
Autonomy
Available
Aversion
Dependence

syE
Unavailable, Unobtainable
Liking, Affinity, Attraction
Contrast
Converge
Cooperate
Comparison
Diverge
Counteract, Nullify
Avoid
Aware
Meet, Confront, Face, Encounter
Unaware, Unconscious ngi
Country
Courageous
Town
Timid
Awe
Awful
Scorn, Contempt
Wonderful, Delightful nee
Courtesy
Creation
Rudeness
Destruction
Awkward Graceful, Deft, Elegant, Skilled,
Skilful
Criminal
Cunning rin
Innocent, Moral
Artless, Naïve, Simple

Barbaric Civil
B
Dead Alive
D
g.n
Barren
Beautifu1
Begin
Fertile
Ugly, Repulsive
End, Conclude, Stop
Deep
Defeat
Defensive
Shallow
Victory
Offensive
e t
Belief Disbelief Deficit Surplus
Belief Disbelief Delay Haste
Beneficial Injurious, Harmful Delicious Bitter, Nauseous, Bland
Benevolent Malevolent Delightful Distressing, Horrid
Better Worse
Demand Supply
Bind Loosen, Set free
Demolish Construct, Restore
Bleak Cheerful, Bright, Sheltered
Deposit Withdraw
Blessing Curse
Desolate Inhabited
Bliss Agony, Sorrow
Blunt Sharp Despair Hope
Boon Bane Difference Resemblance, Likeness, Identity
Brave Cowardly Difficult Easy, Obliging
Bright Dim, Dark, Dull, Stupid Diligent Lazy

CAT Complete Course | 613

Visit : www.EasyEngineering.net
Visit : www.EasyEngineering.net

Diminish Increase G
Discourage Encourage, Hearten, Cheer Generous Stingy, Ignoble
Distress Comfort, Safe Genuine Spurious, Counterfeit
Docile Ungovernable, Headstrong Glory Humiliation, Shame
Domestic Wild, Untamed Gorgeous Plain, Simple, Ugly
Drunk Sober, Temperate Greatness Smallness
Dwarf Giant Grief Comfort, Solace, Joy, Exultation
Dynamic Static, Still Guest Host
E H
Eager Listless, Apathetic, Phlegmatic Happiness Sadness, Unhappiness, Adversity
Earn Spend Harmony Discord, Conflict
Easy Difficult Help Hinder, Hamper, Weaken
Ebb Flow Heredity Environment

ww
Economical
Economy
Egoism
Extravagant
Extravagance
Altruism
High
Honest
Honour
Low, Mean, Degraded
Dishonest, Corrupt
Shame, Dishonour

Endure
Enemy
w
Encourage Discourage

.Ea
Perish, Reject
Friend, Ally, Champion
Hope
Humble
Humility
Despair
Proud, Conceited
Insolence
Enjoyment
Entrance syE
Suffering, Anguish
Exit, Departure
Hurry
Hypocrisy
Delay
Sincerity, Frankness, Candour
Excess
Expedite
Deficiency, Dearth
Delay
F
ngi Idle Busy
I

Fact
Failure
Fiction
Success
nee
Ignorant
Impetuous
Wise, Learned, Aware, Informed
Careful,
Prudent
Cautious, Thoughtful,

Faithful Faithless, Untrue Import


Important rin
Export
Unimportant, Obscure

g.n
Falsehood Truth
Fame Infamy, Shame, Dishonour Include Exclude
Increase Decrease

e
Famous Notorious
Fickle
Fine
Firm
Flexible
Constant, Steady, Reliable
Coarse, Rough
Wavering, fickle
Rigid
Initial
Innocent

Join
Final
Guilty
J
Separate, Sever, Disconnect
t
Foreign Native Joy Sorrow
Formidable Weak, helpless Junior Senior
Fortunate Unfortunate, disastrous Justice Injustice, Inequity
Fortune Misfortune K
Frank Reserved Keen Unwilling
Freedom Slavery Kindle Extinguish
Fresh Stale Knowledge Ignorance
Friend Enemy, Foe L
Frown Smile Lad Lass
Full Empty Large Small, Little, Petty
Fuzzy Clear, lucid, Well-defined Lenient Strict

614 | CAT Complete Course

Visit : www.EasyEngineering.net
Visit : www.EasyEngineering.net

Liberty Restraint Progressive Orthodox, Reactionary


Liberty Bondage, Servitude, Slavery Prohibit Permit, Allow, Sanction
Light Darkness, Shade Prose Poetry
Like Unlike, Dislike Proud Humble
Literate Illiterate Prudence Imprudence, Folly, Indiscretion
Logical Illogical, Absurd Punishment Reward
Loyalty Disloyalty, Treachery, Treason Pure Impure, Adulterated
Luscious Sour Q
M Queer Common, Usual, Regular
Major Minor Quiet Noisy, Occupy
Make Mar, Destroy R
Meek Arrogant, Domineering, Proud, Rabid Rational, Logical
Blustering Rare Common, Ordinary

ww
Memory Forgetfulness, Oblivion Rational Irrational, Emotional
Merit Demerit Raw Ripe
Miser Spendthrift, Extravagant Real Unreal, Apparent, Imaginary
Modest

Mortal
w Immodest, Ambitious, Indecent,

.Ea
Conceited
Immortal
Rear
Rebellious
Front
Submissive, Acquiescent, Con-
forming, Docile, Manageable
Motion
Mourn
Rest
Rejoice
N
syE Recede
Recovery
Advance
Relapse

ngi
Regular Irregular
Native Alien, foreign Religious Secular, Irreligious
Natural Unnatural, Artificial Reluctant Eager, Avid, Desirous
Neat
New
Filthy, Disorderly, Slovenly
Old, Familiar, Common, Obsolete, nee
Remember
Renounce
Forget
Assert, Maintain

Noble
Antiquated
Ignoble, Mean
Rest
rin
Bustle, Motion, Commotion, Distur-
bance

Obstinate Pliable
O Restrain
Reveal Conceal g.n
Incite, Impel, Loose, Liberate

Optimistic
Oral
Overt
Pessimistic
Written
Covert, Secret
Rigorous
Ruthless
Simple, Easy
Merciful
S
e t
P Sacred Profane
Pain Pleasure Safety Danger
Peace War Savage Tame, Civilised
Plenty Scarce Sensibility Insensibility, Deadness, Numbness
Polite Impolite, Rude Severe Tolerant, Lenient, Lax
Positive Negative Shallow Deep
Poverty Affluence Skilful Unskilful, Clumsy, Inept
Praise Blame Slow Fast, Alert, Lively
Precarious Safe Slow Fast
Prefix Suffix Smile Frown
Pretentious Humble, Simple Solid Liquid
Pride Humility, Modesty Solid Liquid, Hollow
Progress Retrogress, Halt, Stop Steadfast Fickle

CAT Complete Course | 615

Visit : www.EasyEngineering.net
Visit : www.EasyEngineering.net

Steep Gradual, Flat 3. METICULOUS


Stop Start, Begin, Initiate (A) Slovenly (B) Meretricious
Stupidity Intelligence, Acuteness, Keenness (C) Shaggy (D) Mutual
Summit Base 4. AMICABLE
Superiority Inferiority (A) Cunning (B) Shy
Supernatural Natural, Usual, Common place (C) Hostile (D) Crazy
Sweet Sour 5. CLARITY
System Chaos, Disorder, Confusion (A) Exaggeration (B) Candour
T (C) Confusion (D) Reserve
Take Give 6. OPULENT
Talkative Silent (A) Wealthy (B) Poor
Tardy Prompt, punctual, timely (C) Sumptuous (D) Drooping
Terse Diffuse, Wordy
7. SANCTIFY

ww
Thrifty
Tragedy
Transient
Spendthrift, Prodigal
Comedy
Perpetual, Permanent, Lasting
(A) Dedicate
(C) Venerate
(B) Patronise
(D) Pollute

Unity
w
Transplant Opaque

.Ea
Diversity
U
8. CHALLENGE
(A) Admire
(C) Favour
(B) Accept
(D) Praise

Universal
Unruly
Regional
Orderly syE 9. TEDIOUS
(A) Pleasant
(C) Lively
(B) Lovely
(D) Gay

Vague
V
Definite, Precise
ngi10. COMMEND
(A) Censure (B) Condemn
Valiant
Valid
Victor
Cowardly, Afraid, Fearful
Invalid, Illogical, Fake
Vanquished
nee (C) Defy
11. SUPERFICIAL
(D) Defame

Violent Compassionate, Kind (A) Artificial


(C) Shallow rin (B) Deep
(D) Real
Virtue Vice
W 12. EFFETE
(A) Adamant g.n
(B) Strong
Win
Wisdom
Lose, Fail, Forfeit
Folly
Y
(C) Courageous y
13. COMMEND
(A) Suspend
e
(D) Bold

(B) Admonish t
Young Old, Mature, Grown (C) Hate (D) Dislike
Z 14. SERENE
Zealous Indifferent, Apathetic (A) Jovial (B) Moving
Test Paper–1 (C) Agitated (D) Nervous
15. ANTIPATHY
Below in each set the lead word is followed by four
options. Pick out the most suitable ANTONYM from the (A) Fondness (B) Obedience
options. (C) Agreement (D) Admiration
1. UNREALISTIC 16. DAUNTLESS
(A) Natural (B) Visionary (A) Cautious (B) Thoughtful
(C) Reasonable (D) Actual (C) Weak (D) Adventurous
2. DEPRAVED 17. CHAFFING
(A) Great (B) Enhanced (A) Expensive (B) Achieving
(C) Moral (D) Prosperous (C) Capitalistic (D) Serious

616 | CAT Complete Course

Visit : www.EasyEngineering.net
Visit : www.EasyEngineering.net

18. TRAGIC 11. JETTISON


(A) Funny (B) Comic (A) Rejoice (B) Surrender
(C) Light (D) Humorous (C) Accept (D) Defend
19. EXODUS 12. PERSISTENT
(A) Restoration (B) Return (A) Wavering (B) Obstinate
(C) Home-coming (D) Influx (C) Enduring (D) Steady
20. PREVENT 13. SCOLD
(A) Excite (B) Support (A) Enamour (B) Rebuke
(C) Invite (D) Incite (C) Criticise (D) Praise

Test Paper–2 14. PODGY


(A) Short (B) Thin
1. RUGGED
(C) Weak (D) Slim
(A) Delicate (B) Coarse
(C) Tough (D) Timid 15. SEGREGATION

2.
ww
EDIFICATION
(A) Lamentation (B) Annotation
(A) Appreciation
(C) Integration
(B) Cohesion
(D) Union

3. w
(C) Corruption
INNOCENT
(A) Sinful
(D) Segregation

.Ea
(B) Guilty
16. JITTERY
(A) Profuse
(C) Bold
(B) Tense
(D) Shaky

4.
(C) Deadly
PACIFY syE
(D) Corruption 17. VIRTUOUS
(A) Scandalous
(C) Wicked
(B) Vicious
(D) Corrupt
(A) Quarrel
(C) Threaten
(B) Challenge
(D) Darken
ngi18. EXASPERATE
(A) Belittle (B) Annoy
5. FURTIVE
(A) Straight (B) Obvious
nee (C) Please
19. SORDID
(D) Tarnish

rin
(C) Unambiguous (D) Open
(A) Steady (B) Enthusiasti
6. SECULAR

g.n
(C) Generous (D) Splendid
(A) Righteous (B) Religious
(C) Spiritual (D) Moral 20. GRIM

7. DEAR
(A) Cheap
(C) Free
(B) Worthless
(D) Priceless
(A) Serious
(C) Delightful

Answers
e
(B) Satisfying
(D) Painful

t
8. ASCETICISM Test Paper 1
(A) Bliss (B) Pleasure 1. (C) 2. (C) 3. (A) 4. (C) 5. (C) 6. (B)
(C) Joy (D) Trance 7. (D) 8. (B) 9. (A) 10. (A) 11. (B) 12. (B)
13. (B) 14. (C) 15 (A) 16. (D) 17. (B) 18. (B)
9. PREDILECTION
19. (D) 20. (B)
(A) Denial (B) Concealment Test Paper 2
(C) Aversion (D) Attraction
1. (A) 2. (C) 3. (B) 4. (A) 5 (D) 6. (B)
10. APPOSITE 7. (A) 8. (B) 9. (C) 10. (A) 11. (C) 12. (a)
(A) Inappropriate (B) Intemperate 13. (D) 14. (B) 15 (C) 16. (C) 17. (B) 18. (C)
(C) Inconsistent (D) Irregular 19. (C) 20. (C)
●●●

CAT Complete Course | 617

Visit : www.EasyEngineering.net
Visit : www.EasyEngineering.net

4 One Word Substitution


Abdicate — Renounce formally or by default Apiculture — Bee-keeping.
power, right, office. Apostle — A messenger to preach gospel.
Aborigines — Original or first inhabitants. Aquatic — Animals living in water.
Accelerate — To increase the speed. Arbiter — One who is appointed by two parties
Acoustics — Relating to the sense of hearing. to settle a dispute.
Affidavit — A statement on oath. Archaeology — Study of antiquities (historic and
Agnostic — One who holds the view that nothing prehistoric times).

ww can be known about the existence of


God.
Arsonist — A person guilty of maliciously setting
on fire of property etc.

w
Alimony — Allowances due to a wife from her Ascetic — One who practices severe self-
hus-band on separation. discipline.
Allegation —
person..Ea
A charge or statement against a Atheist
Auction


One who does not believe in God
A bargain Where things are sold to
Allegorical

Allergy


syE
A narrative describing one subject
under the guise of another
Sensitiveness to the action of a
Audible
Audience

the highest bidder..
A thing which can be heard.
An assembly of listeners.

ngi

particular food or other things.
Authentic — A reliable piece of information.
Alleviate — To mitigate or lesson the pain or
Autocracy — Absolute government.

Altruist —
suffering.
A person who has regard and concern
for others, unselfish.
nee
Aviary
Bankrupt


A place where birds are kept.
One who cannot pay his debts.
Amateur — A person who engages in a pursuit as
a pastime rather than a profession, a
Bathos —
rin
A fall from the sublime to the
ridiculous.
person who does something unskil-
fully.
Bellicose
Belligerent

— g.n
Given to fighting or fond of fighting.
Aggressive, eager to fight, engaged
Ambassador — A person who represents the interest
of his country's Government abroad
and acts in pursuance of the policy of
the Government.
Bibliophile

Biennial


in war.

e
A person who is a great lover of
books.
Happening or appearing once in two
t
Ambiguous — Having more than one interpretation. years.
Anachronous — Anything out of harmony with its Bigamist — One who has two wives or husbands
period, an old-fashioned or out-of- at one time.
date person or thing.
Bigamy — The state of having two wives.
Anarchist — A person who believes that all
Bigot — One blindly devoted to a particular
government and authority should be
creed or party.
abolished.
Bilingual — Containing or speaking two
Annihilate — To destroy completely. languages.
Anonymous — That which does not bear a name of Biography — A written account of the life of a
the writer/creator. person.
Antidote — Anything which destroys the effect Bizarre — Strange in appearance or effect,
of poison. eccentric, grotesque.
Antonym — A word opposites in meaning to Blasphemy — The act of speaking disrespectfully
another about holy and sacred things.

618 | CAT Complete Course

Visit : www.EasyEngineering.net
Visit : www.EasyEngineering.net

Blonde — A person having a fair complexion Confiscate — To seize something by authority.


and light golden hair. Congenital — Acquired at the time of or before
Bohemian — A man of free and easy habits- birth. (esp. a disease, etc.)
socially unconventional. An expert — a person who understands profoundly
Boor — Clumsy or ill-bred fellow. the value of art, antiques, etc.
Botany — Science of the structure of plants. Connoisseur — One who is will versed in any subject;
Bourgeoisie — A conventionally middle-class, a critical judge of any art, particularly
selfishly materialistic and upholding of fine arts.
the interests of the capitalist class. Conservative — Disposed to maintain existing
Bovine — Pertaining to cows or cattle. institutions.
Brittle — Liable to be easily broken. Constellation — A group of stars.
Coche — Gold or silver in bulk or bars Contagious — Spreading through actual contact,
before coining or manufacture. disease, etc.
Cadaverous — Looking like a dead body. Contemporary — One who is living or lived at the
same time.

ww
Calligraphy
Cannibal


Beautiful handwriting.
One who eats human flesh or animals,
that eat their own species.
Contiguous
Convalescent
— Touching along a line; in contact.
— (one) gradually recovering health.
Cant
w
Cardiologist
Carnivorous



Affected manner of speech.

.Ea
Specialist in heart diseases.
Flesh-eating.
Cosmo

Credulous
— One who is at home in any country
of the world, a citizen of the world
very broad minded.
— On who easily believes everyone and
Carnivorous
Catalogue
Cosmopolitan



syE
One who lives on human flesh.
A list of names, books, etc.
Of interest or use of all; universal,
Culmination
everything.
— Reaching the highest or the final
point.
all-embracing, of wide sympathies
or interests. ngi Cynic — One who is a sneering critic of every-
thing.
Caucus — A small group within a political party,
influencing party decision and policy
etc. nee
Cynosure
Declamation
— Centre of attraction or interest.
— Act of speaking oratorically or force-

Celibate — The state of being without a wife,


abstaining from marital relationship.
Defector
fully.

rin
— One who leaves or breaks his

Centenarian
Centenary


A person who is hundred years old.
Hundredth anniversary.
Defensible
g.n
allegiance to a party.
— Capable of annulment, liable to
forfeiture.
Charlatan

Chauvinism —
Chronic —
— Empty or shallow pretender of
knowledge or skill.
Bellicose patriotism or nationalism.
extending over a long time.
Deist

Delegate
e
— One who believes in God but not in
the Revelation.
— one to whom full authority to act hast
been given.
Circumlocution— A round about way of expression. Delinquent — A person who fails in the perfor-
Cliche — Hackneyed phrase or opinion. mance of his duty or a person who
Colleague — one associated with others in some commits an offence.
employment. Deluge — A large flood, a heavy fall of rain.
Colloquial — Belonging to or proper to ordinary or Demagogue — A political orator - agitator appealing
familiar conversation; not formal or to the basest instincts of a mob.
literary. Democracy — Government in which supreme power
Combustible — liable to take fire and bum. is vested in the people collectively.
Compensation — To give something to someone to Dermatologist — Specialist in skin diseases.
make good for loss, injury or damage. Dessert — Fruit or sweet dish usually eaten after
Competence — Sufficient income to live on in main meals
comfort. Destitution — The state of being miserable, bereft
Compromise — (to settle a dispute by) mutual give of all material possessions, the state
and take. of extreme poverty.

CAT Complete Course | 619

Visit : www.EasyEngineering.net
Visit : www.EasyEngineering.net

Detest — Hate, loathe Epidemic — A disease which attacks a large num-


Despot — A ruler who uses force in order to ber of people simultaneously,
subjugate the people to obey him. Epigram — A short but pithy and weighty saying,
Digress — To wander away from the main point. a short witty phrase
Dilettante : an amateur, lover of fine Epitaph — Words inscribed on a tomb.
arts. Epitome — an abridgment or short summary.
Dilettante — A dabbler in art and literature, a Equestrian — pertaining to horses or horsemanship.
person who studies a subject or area Equivocal — Ambiguous, of uncertain nature.
of knowledge superficially. Eradicate — To get rid of something and remove
Diplomacy — The art of negotiation, specially all traces of it.
between states. Erratic — Without a fixed or regular course,
Diplomat — One who is engaged in the inconsistent, unconventional, eccen-
diplomatic service of one's country. tric.
Domestic — belonging to the house. Escapist — One who is always seeking to escape
from the harsh and -bitter realities of
Drawn — A contest in which neither party

ww
Duplicity
wins.
— The nature of a double-dealer or
Ethics
life. established conventions, beliefs.
customs, etc.
— Science of morals or rules of conduct.

w
impostor.
Etymology — The study of the origin of words
Eccentric — not conforming to common rules.

.Ea
Eccentricity — A personal peculiarity of
temperament, an unusual trait.
Eugenics — Science of production of healthier
and finer children.
Euphemism — Substitution of a mild for a very blunt
Edible
Effeminate syE
— that may be eaten.
— One who is like a woman, unmanly. Excavate
expression or pointed phrase.
— to unearth by digging.

ngi
Efficacious — able to produce quick and intended Exchange — give or receive one thing in place of
result. another.
Egotism — talking too much about oneself. Encyclopaedic — That which is all-inclusive or deals
Elegy
Elite
— a song of mourning.
— the best or choice part of a larger nee comprehensively with a subject.
Exhibitionism — Perverted mental condition charac-

rin
body or group, a select group or class terized by indecent exposure of the-
elect. person

g.n
Exonerate — to free from the burden of blame or
Emancipate — To free from the bondage of some-
obligation.
thing, to liberate from.
Expiate — To atone for one's sins.
Emeritus

Emigrant
— A professor who has honourably
retired from service.
— one who leaves his own country and
settles down in another.
Explicit

Export
Expurgate
stated. e
— not implied merely ,but distinctly

— send out goods to another country.


— to remove objectionable matter from
t
Empiric — One who relies on experience and
observation, a book;
Endemic — Something regularly found in a Extempore — A speech made on the spur of the
particular area or among a particular moment, without proper preparation.
people or community. Fanatic — One filled with excessive and mis-
Ennui — mental weariness from want of taken enthusiasm in religions.
employment, A state of mental Factitious — Artificial, false.
weariness from lack of occupation of Fallible — One who is subject to failure, or
interest . committing mistakes.
Entomologist — scientist who studies insects. . Fastidious — Careful in all details, exacting,
Entomology — The study of insects. meticulous, difficult to please, easily
Ephemeral — lasting only for a day disgusted.
Epicurean — one devoted to luxuries or sensuous Fatalist — A person who believes in fate having
enjoyment or one who believes in the firm belief that all events are predeter-
philosophy of pleasure mined by fate.

620 | CAT Complete Course

Visit : www.EasyEngineering.net
Visit : www.EasyEngineering.net

Fauna — The animals of a particular region or Heterodox — holding an unconventional opinion.


epoch. Heterogeneous — Things which have diverse elements.
Feasible — That which is practicable or possible. Homicide — murder or murderer of man
Feminist — A man who is genuinely interested in Homogenous — composed of similar elements.
the welfare, betterment and emanci-
Honorarium — Voluntary fees paid for professional
pation of women.
services which carry no salary.
Fictitious — As opposed to realistic; imagined,
Honorary — (an office) without performing any
not real.
service or receiving any reward.
Flamboyant — Extremely showy and colourful
Hygienist — One who is very careful about one’s
personality.
health.
Foster — Child a child brought up by one who
Hymeneal — pertaining to marriage;
is not the real parent.
Hyperbole — Rhetorical exaggeration in speaking
Fratricide — One who kills one's brother or sister;
or in a piece of writing.
killing one's brother or sister.
Hypochondriac— One who suffers from mental depres-
Frugal — Avoiding unnecessary expenditure

ww
Garage
of money, thrifty.
— A building where motor vehicles are
Iconoclast —
sion caused by imaginary fear of
some disease or aliment.
One who breaks images or idols a

w
housed.
man who assails cherished beliefs
Garrulous — given to excessive talking, talkative.

.Ea
and ideas a breaker of idols or images;
Genealogy — Plant or animals line of development one woo is opposed to well
from earlier forms. Idiosyncrasy — A mental constitution, view, or feel-
Genocide

Genocide
syE
— Act of killing one's clan, family or
community.
— Extermination of a race or community Idolater —
ing or mode of behaviour peculiar
to a person, anything highly eccentric.
a worshipper of idols.

Geologist
by mass murders.
— Scientist who studies the composition ngi Ignominy
Illegible


Dishonour , infamy.
that cannot be read or deciphered.

Geology
of the earth.
— The science of the study of the
origin, history and structure of
nee
Illegitimate —
Illicit: not allowed by law.
Born of parents not married to each

Germicide — medicine that kills germs.


rin
other, not authorised by law,
abnormal, improper.
Gerentology — Scientific study of old age and its

Glutton
disease.
— one who eats excessively.
Illiterate
Illusion


g.n
One who can neither read nor write.
A deceptive appearance, statement or
belief.
Gourmet

Gregarious
— A connoisseur of table delicacies and
wine.
— Animals which live in groups.
Immutable

Implacable


e
that which is not subject to, any
change or alteration.
One who cannot be soothed or t
Gullible — one who can be fooled easily. calmed or pacified.
Gymnasium — a large room or hall with apparatus Implicit — not distinctly stated but implied.
for physical training. Import — bring goods from a foreign country.
Gynaecology — science of the diseases of women. Impostor — one who pretends to be what he is not
Hedonism — Doctrine that the pleasure is the chief or assumes some body else's name or
good. title in order to deceive others.
Herbivorous — (animals) that feed on herbs. Impracticable — That which cannot be put into
practice. not able to be done.
Hereditary — Traits, physical or mental-received
from forefathers by birth. Impregnable — That cannot be seized. or taken by
force; able to resist all attacks.
Heretic — One who expresses ideas which are,
not in conformity with conventional Imprudent — who lacks foresight; indiscreet
reli-gious teachings. Imputation — Attributing or ascribing to something
Hermit — One who lives in seclusion with or someone.
thoughts of God. Inaccessible — That cannot be easily approached.

CAT Complete Course | 621

Visit : www.EasyEngineering.net
Visit : www.EasyEngineering.net

Inaudible — That which cannot be heard. Ludicrous — Laughable, absurd, ridiculous,


Incendiaries — Wilfully setting fire to buildings etc. preposterous.
Incentive — Something which provides a person Lunar — pertaining to the moon.
attraction or interest. Macafee — A scene or situation which is grue-
Incognito — concealed under a disguised somely imaginative full of gruesome
character and assumed name. details.
Incompatible — incapable of existing together in Machiavellism — Philosophy of practising duplicity in
harmony . statecraft.
Incorrigible — One who cannot be corrected or Majority — The greater number.
reformed. Malady — Feeling of bodily discomfort without
Indescribable — that cannot be described. clear signs of a particular disease.
Inimitable — That which cannot be easily imitated. Malapropism — A ridiculous confusion and misuse of
words.
Innocuous — that is harmless or without effect.
Malleable — Any metal that can be spread out in
Innuendo — An implied (generally deprecatory)
sheets. Animals which feed theyoung

ww
Insomnia
remark.
— A disease in which one suffer from
sleeplessness. inability to sleep.
Manuscript
with milk from their breasts.
— Hand-written matter.

w
Instigate
Insurgent
— The act of provoking a person.

.Ea
— Rising in active revolt, a rebel, a
revolutionary.
Materialistic — Concerned solely with material

Matriarchy
objects.
— A society in which mother is head of
family.
Insurmountable—

syE
That cannot be overcome.
Intermittent — ceasing at intervals.
Intestate — (one who dies) without making a
Matricide
Matrimony
— The murder or murderer of mother.
— State of being married.

ngi
Mediator — One who plays the role of bringing
will.
two antagonistic part together
Intimidation — Language or gesture which implies
Melodrama — A drama which is marked by very

Invincible
threat to the other.
— that cannot be overcome.
Invulnerable — that cannot be wounded or hurt.
nee
Mercenary
crude appeal to feelings and emotions.
— A person working or a soldier fighting

Irrational — Not rational, opposed to reason.


rin
merely for money.
Metamorphosis— The state of being changed or trans-
Irreconcilable — Incapable of being reconciled.
Irrefutable — That cannot be proved false. means.
g.n
formed by natural supernatural

Metaphysics — philosophy dealing with the ultimate


Irrelevant
Irrevocable
Irritable
Itinerant
— Not to the point.
— That cannot be recalled.
— Easily provoked or irritated.
— Travelling from one place to another.
Meticulous
truth.
e t
— Giving great or excessive attention to
details, very careful and precise .
Migratory — Birds moving from o n e place to
Jingoism — Blustering or blind patriotism and another.
nationalism. Millennium — A golden age, a period of 1,000 years.
Jurist — A person who is well-versed in law. Minority — The smaller number.
Kleptomania — An excessively morbid desire to Misanthrope — Hater of mankind
steal.
Misanthropist — One who is a hater of mankind.
Lexicographer — One who makes or compiles
dictionary. Misogynist — hater of Women.
Libertine — A free thinker on religion, a dissolute Monarchy — government by a king.
or licentious person. Monogamist — A person who believes in being
Licentious — Extremely extravagant in manners married to one woman or one man at
and morals. a time
Linguist — Proficient in many languages. Monogamy — practice of being' married to only one
Loquacious — One who talk continuously, talkative, person at a time.
chattering, babbling. Monotheism — Doctrine that there is only one God.

622 | CAT Complete Course

Visit : www.EasyEngineering.net
Visit : www.EasyEngineering.net

Monotony — The state of being monotonous, dull Ostracise — To banish or turnout of society and
or tedious routine fellowship.
Morphology — Study of the forms of animals, plants Pacifist — one who believes that war should be
or words. abolished.
Mortuary — A place where dead bodies are kept Palaeography — Study of ancient writings and docu-
until burial. ments.
Mundane — Belonging to the earthly world. Panacea — A universal remedy for all ailments
Narcotic — Drug that in produces sleep or or diseases.
insensible condition. Pandemonium — A disorderly assembly or tumultuous
Naturalization — adapted to different conditions. noise.
granted the rights of natural born Pantheism — Doctrine that God is everything and
citizens. everything is god
Navigable — that may be passed by ships. Parasite — one who lives upon another
Nepotism — undue favour to one's relations by Parasol — A lady's umbrella.
appointing them to high and lucrative

ww
Neurosis
posts.
— nervous disorder.
Pathology
Patricide
— science of diseases.
— the murder or murderer of father.

w
Neurotic

Nihilism
— (a person) suffering .from disordered
nerves.

.Ea
— The rejection of all religious and
moral principles, an extreme form of
Patrimony

Patriot
Pawn
— right or estate inherited from one’s
father.
— A lover of one's country.
— A person used by others for their own

syE
skepticism characterized by the
assertion that nothing really exists.
Numismatics — Science and study that treats coins
Pedagogue
purposes.
— A school master who is very strict

ngi
and teaches in a pedantic manner.
and medals.
Pedant — One who makes a show of one’s
Oasis — Fertile spot in a desert, where water
knowledge.

Obituary
is found.
— Notice of death especially in
newspaper. nee
Paediatrician — Specialist in children's diseases.
Penology — The study of the punishment of crime
Obsolete
Octagon
— That which is no longer in use.
— A figure having eight sides. Perception rin
and of prison management
— Understanding, comprehension
Octogenarian: one who is eighty
years old.
Peroration
g.n
— A passage marking the close of a
speech.
Oligarchy — government by a small exclusive
class.
Omnipotent — all powerful.
Omnipresent — Present everywhere.
Pessimist
e
— One who takes a gloomy view of life.
Phil1uithropist — A man who generously donates and
gives help to welfare projects. t
Philanderer — One who makes love in a light and
Omniscient — having knowledge of all things. non-serious manner.
Omnivorous — Feeding on anything Philanthropy — A lover of mankind.
Opaque — Not reflecting or transmitting light. Philatelist — One who collects stamps.
Ophthalmologist Philately — Science dealing with stamp collec-
— specialist in eye diseases. tion.
Opportunist — An unscrupulous person who puts Philistine — A person who is hostile or different
expediency before principle. to art and culture.
Optimist — One who habitually looks at the Philologist — One who studies the history and
brighter side or as things. growth of language. .
Ornithologist — scientist who studies birds. Plagiarism — Act of stealing from the writings or
Orthography — The art of spelling words correctly. ideas of others.
Ostentatious — Extremely loud and showy as Plagiarist — One who steals from the writings or
opposed to reserved and modest. ideas of others.

CAT Complete Course | 623

Visit : www.EasyEngineering.net
Visit : www.EasyEngineering.net

Platitude — A commonplace or an oft-repeated Rebel — one who takes up arms against a


statement or remark. Plutocracy: government.
government of the rich and powerful Recalcitrant — Obstinately disobedient.
people.
Red-tapism — Strict adherence to all forms of
Plutocracy — Government by the rich. official formalities
Polyandrous — Woman who has more than one hus- Regicide — the murder or murderer of king.
band at a time.
Reminiscent — That which reminds something,
Polyandry — The custom of having several tending to recall or talk of the past
husbands at a time. '
Remit — to send money by post. '
Polygamy — The practice of having more than
Remuneration — compensation rendered as equivalent
one husband or wife.
of a service.
Polyglot — One who knows many languages.
Renegade — A deserter of party, principles or an
Polytheism — Belief in or worship of more than one apostate.
god.
Renounce — To give up entirely.
Post mortem — Operation of the body after death.

ww
Posthumous — born after the father's death; published
after the author's death.
Republic — A state where the Head of the State is
elected and sovereignty resides in the
people.

w
Post-mortem — that which is held after death.

.Ea
Pragmatism — Philosophy of judging the truth or
validity of one's actions solely on
their practical success.
Reticent — reserved in speech.
Retrospective — which has reference to what is past.
Sacrilege — The act of violating the sanctity of a
holy place.
Precedent

Precedent
past.
syE
— Some previous example from the

— A former holder of an office or


Sadist — One who takes pleasure in cruel,
inhuman acts.

Prevaricate ngi
position with respect to a later holder.
— To make evasive or misleading
Samaritan — one who helps the poor and helpless
in trouble.

nee
Sanatorium — A health resort, an institution for the
statements.
treatment of chronic diseases.
Primogeniture — right of succession belonging to the
Sartorial — pertaining to tailors or clothes.
first born son.
Prophecy — Statement showing remarkable
degree of prediction
Savvy
rin
— Know, knowingness, shrewdness,
understanding.

Pseudonym — A fictitious name assumed by a


writer
Scapegoat
fixed.
g.n
— Someone on whom blame of others is

Pseudonym

Psychiatrist
— an imaginary name assumed by
author.
— specialist in mental and emotional
disorders.
Sinecure

Sociologist
without work.
e
Simultaneously— at one and the same time.
— an office or post with salary but

— one who studies human society;


t
Psychologist — one who studies the human mind. Solar — pertaining to the sun.
Soliloquy — Talking loudly when alone, a speech
Pugnacity — readiness or inclination to fight.
addressed to self on a dramatic stage.
Purist — one who pays great attention to the
Somnambulist — One who walks in sleep.
correct use of words language. etc.
Sophism — False arguments intended to deceive
Purl tan — Person affecting extreme strictness in
religion or morals. Sophisticated — Extremely refined in dress, conduct
and speech.
Quack — A person who pretends to have
Sordid — Dirty or squalid, ignoble, mean,
knowledge or skill especially in
mercenary.
medicine
Spinster — An elderly unmarried women.
Quinquennial — occurring once in five years.
Sterilize — to free anything of germs.
Radical — Far-reaching, thorough, advocating
thorough reforms, holding extreme Stoic — A person not easily moved by
views. pleasure or plain.

624 | CAT Complete Course

Visit : www.EasyEngineering.net
Visit : www.EasyEngineering.net

Suicide — murder of self. Unsociable — indisposed to society.


Superannuation Usrer — The practice of charging exorbitant
— to al1ow to retire from service on a or excessive. interest on money
pension on account of o1d age or Utilitarianism — the doctrine that actions. should be
infirmity. judged as right or wrong solely by
Supercilious — Overbearing, haughty, arrogant. their utility or promoting the happi-
ness and good of the greatest number.
Sycophant — A person who flatters other for
personal motive. Utopian — admirable but impracticable.
Synchronize — To take place at the same time as Vacillation — wavering between different courses.
another. Valetudinarian — one who always thinks that he is ill.
Syndicate — an association of persons formed with Vamp — an adventurer who extracts money
a view to from men by means of sex appeal.
Synopsis — a summary giving a general view of Vandal — One who destroys all work of art.
some subject. Vandalism — wilful destruction of works of art,
Taxidermist — one who skins animals.

ww
Teetotaller — Art of preparing and mounting skins
of animals in life like manner.
Vegetarian
Venial
— one who lives on vegetables only.
— a fault or sin which may be pardoned.
Ventriloquist — one who can produce sounds and

w
Teetotaler

Telepathy
— A person advocating or practising

.Ea
abstinence from alcoholic drinks.
— Communication between minds. -A
person advocating or practising
Venue
words without any motion of the
mouth.
— the place where an action is laid or

syE
the trial of a case takes place.
abstinence
Verbatim — Repetition of a writing or conversa-
Tempest — A violent storm.
tion word by word.

ngi
Termagant — a woman of over bearing nature.
Verbose — Using more words than necessary,
Terminus — the extreme station at either end of a wordy, prolix.
railway or railroad.
Theist
Theocracy
— One who believes in God .
— government based on religion. nee
Verisimilitude — likeness to truth.
Versatile — having ability of many kinds.

Tirade
Titanic
— A long vehement speech or reproof.
— enormous in size and strength.
Vesper
Veteran
rin
— Evening prayers in the church.
— One who has long experience or
Toddler — one who walks with short steps in a
tottering .way as a child or an old Veteran
expertise.

g.n
— one having a long experience of

e
man does. something.
Transmigration— the passing of soul after death from
one body to another.
Transparent — that can be seen through.
Voluntary
Vulnerable

Wardrobe
— offered of one's own accord.
— capable of being wounded or
criticised.
— a place where clothes are kept.
t
Treason — disloyalty or treachery to the state.
Triennial — that which happens once in three Whirligig — anything that revolves rapidly (like
years. fortune).
Truant — a student who runs away from the Widow — a woman whose husband is dead.
class or school without permission. Widower — a man whose wife is dead.
Truism — an oft-repeated truth. Zeal — Extreme enthusiasm for a cause.
Turncoat — One who easily gives up his party or Zealous — Ardent, eager, enthusiastic.
principles Zenith — The highest point.
Ubiquitous — present everywhere at the same time. Zest — Spice, relish, tang; gusto
Unanimous — With everyone agreeing Zone — Area, region, district, section.
Underhand — in a clandestine manner and often Zoo — a place where animals are kept.
with a bad design. Zoology — science of animal life.
Unintelligible — that which cannot be understood. ●●●

CAT Complete Course | 625

Visit : www.EasyEngineering.net
Visit : www.EasyEngineering.net

5 How To Use Foreign Expression


A fortiori—with strong reason. Ad valorem—according to value. For example,
A posteriori—form the effect to the cause. stamp fee on sale of land is charged is charged according
Abandon—to leave, to give up, to surrender. to the value of the land, or value added price or tax
Abate—to put end. included in price.
Ab antique—form ancient time. Addenda—list of additions.
Ab antique—form olden times. Advers possession—a person not being the owner of
Ab initio—form the very beginning. the property is in possession of the property.
Abduction—this is an offence committed against a Advocare—to defend, to call to one’s aid also to

ww
person of any age. Section 352 of the India Penal Code vouch to warranty.
deals with this offence. Advocate general—a person appointed by the state
Abide—to act in accordance with rule of law. to represent state in cases.

w
Abiure—to renounce on oath or affirmation. Advocatus diabolin—the devil`s advocate, an officer
of the sacred congregation of rites at Rome, whose duty is
Abrogate—to repeal.

dafander. .Ea
Absolvitor—an acquittal; a decree in favour of the
to prepare all possible argument against the admission of
any one to the posthumous honouree of beatification and
canonization.

principal thing. syE


Accession cedit principali—an accessory thing
when annexed to a principal thing becomes part of the

Acta exoriora indicant interiora secreta—an


Aequitas—equity, i.e., fair of just according to
natural law.
Affidavit—a written statement under an oath, which is
indicate the intention.
Action personalis mortar cum personal—a per- ngi sworn to and signed by person making it, as true.
Agent—a person employed to act on behalf of
another; an act of an agent done within the scope of his
sonal actiondies with the promisor.
Actus dei nemini facit injuriam—law holds no man
responsible for the Act of God. nee
authority, binds his principal.
Agreement—every promise and every set of promise,
either written or oral for the purpose of contract forming
man.
Actus legis nemini facit injurium—law wrongs no

Actus non facit reum, nisi mens sit rea—an act rin
the consideration for each other.
Allegans contraria non set audiendus—a person
does not constitute guilt unless it is done with a guilty
intent.
Alias—otherwise calls.
g.n
alleging contradictory facts should not be heard.

Alibi—a plea taken by the accused to prove that the


Actus rea—physical involvement in crime.
Actus reus—the physical acts involving or constitu-
ting occurrence of crime.
Ad arbitrium—at will.
Ad ea Quae Frequentius Accident Jura Adap-
he has not committed the crime.
e
Alientio Rei Preferature Juri Accrecendi—alience
is favoured by the law itself.
Alimony—a maintenance given by a husband to his
t
divorced wife.
tatur—law are adapted to those cases which more fre- Amicus cuiae—literally ‘friend pf Court’. The name
quently occur. is given to a lawyer appointed by a Court to represent a
Ad hoc—for this purpose. poor litigant.
Ad idem—of the same mind; agrees. Amnesty—a general pardon for political offences.
Ad infinitum—for ever. Amnesty international—an organization which
Ad interim—in the meanwhile. fights for human rights.
Ad largum—at large; used in the following and Animus atestandi—the intention of attesting.
other expression: title at large, commom at large, assize at Animus deserendi—the intention of deserting.
large, verdict at large, to vouch at large. Annuity—an amount that is payable yearly.
Ad naseam—to a disgusting extent. Annul—to deprive a judicial proceeding of its opera-
Ad quem—to whom. tion, either retrospectively or only as to future transac-
Ad referendum—for further consideration. tions.
Adjourn—to postpone hearing of a case for some Antenuptial—before marriage.
future date. Appeal convict—an application made to a superior
Ad rem—to the point. court/higher court against the decision of a lower court.

626 | CAT Complete Course

Visit : www.EasyEngineering.net
Visit : www.EasyEngineering.net

Apriori—form the cause to the effect Causa celebre—a celebrated case.


Arbitration—setting disputes by referring them to Causa sine qua non—factor essential to the
independent third party, as an alternative to the court occurring of event.
Arbitrator—he is a person appointed by parties to Caveat—an order which says “lethim beware”.
decide any difference between them. Warning [lat. Let him take heed].
Argumentum As Inconvenient Plurimum Valet in Cvceat emptor—let the buyer beware.
lega—an argument drawn form inconvenience is forcible Caveat venditor—let the seller beware.
in law. Caveat viator—let the traveller beware.
Arrest—this is a deprivation of personal liberty of a Censuer—a reprimand form a superior.
person so that he become available during the trial of any Certiorari—a writ of a superior court calling forth
offence to which he is involved. the records and entire proceeding of an inferior court or a
Assault—striking or attempting to strike another writ by which causes are removed form an inferior court
person. into a superior court.
Assignatus Utitur Jure Auctoris—an assignee is Certum est quod certum redid potest—that is
clothed with the right of his principal. certain which can be rendered certain.
Assumpsit—he promised or undertook. Cessante ratione legis cessat ipsa lex—reason is the
Attorney—one duly appointed or constituted to act soul of law, and when the reason of any particular law

ww
for another in business or legal matters. A properly quali-
fied legal agent who conducts litigation.
Au fait—conversant with.
ceases, so does the law itself.
Cestui que trus—a person for whom another is
trustee : a beneficiary.

w
Audi alteram partem—hear the other side. Chattel—movable property.
Authority—a judicial decision cite d as a statement Cognate—blood relation other than agnate and

.Ea
of law [also called precedent].
Autrefois—nobody cannot be tried twice for the
same offence.
includes a female relation.
Cognizance—judicial knowledge. Thus to take
cognization of an offence is to bring to the knowledge of

syE
Autrefois acquit—formerly acquitted.
Autrefois convict—formerly convicted.
Bail—reease of arrested person on furnishing surety
law enforcers or to proceed.
Compos mentis—of sound mind.
Consensus ad idem—agreement as to the same
bonds.
Bailiff—a subordinate officer of the court who
ngi thing.
Consensus, non concubitus, facit matri-monium—

nee
executes writ and other court orders, such as serving it is the consent of the parties, not their cohabitation,
summons. which constitutes a valid marriage.
Ballot—a system of secret voting. Consummation—voluntary sexual intercourse.

rin
Bar—it is a collective trem used for lawyers as a Contea bonos mores—against good morals.
body. Contemporanea ezpositio est optima et fortissimo
Bench—term is used to explain judges physical in lege—the best way to construe a document is to read it
force on a person.
Bigamy—marrying again in the life time of another
as it would have read made.
g.n
Contra pacem—against the peace.

e
spouse without getting divorce. Coram—in the presence of.
Bona fide—in good faith, honestly, without fraud,
collusion or participation in wrongdoing.
Bona gestura—good behaviour.
Bona mobilia—movable effects and goods.
Bona vacantia—goods that do not have an owner.
Corner—an officer who in enquires into any un-
natural death.
Coup d`etat—violent or illegal change.
Coup d`grace—finishing stroke.
t
Criminology—this refer to a science that deals with
Generally they goto the finder. crimes as well as criminals.
Boni judici est ansliare jurisdictioness—it is the Culpa—wrongful default.
part of a good judge to enraged his jurisdiction. Curia Regis—the King`s Court.
Breach—breaking of law, contract. Damage—a sum of money which the court orders
Bye-law—regulation made by the local authority or the defendant to pay to the plaintiff as compensation for
corporation or company or society for its members for breach of contract or tort [i.e., civil wrong].
their day to day operation. They are provided in their Damnum absque injuria—loss or damage for
principal acts. which there is no legal remedy.
Caeteris paribus—other things being equal. Damnum sine injuria—damage without a legal
Carte blanche—full discretionary power. wrong.
Capital punishment—death sentence. De die in diem—form day to day.
Casus ommissus—a matter which should have been De facto—in fact.
has not been, provided for in a statute or in statutory rules. De hors—outside the scope of.
Causa causans—the immediate cause, the last link Deed—an instrument in writing which is signed,
in the chain of causation. sealed and delivered.

CAT Complete Course | 627

Visit : www.EasyEngineering.net
Visit : www.EasyEngineering.net

De jure—by virtue of law. Esprit de corps—regard for honour of body one


De minimias non curat lex—law dose not concern belongs to.
itself with trifles. Estoppel—rule of evidence in which a person is
Demise—the grant of lrase, fallen. stopped form denying what he has said earlier.
De novo—a new. Et incumdit probation qui dicit, non qui negat—
De nisi—a conditional decree, not absolute. the burden of proof is on him who alleges, and not on him
Defeasible right—right with can be defeated. who denies.
Dei gratia—by god`s geace. Evidence—a document, a preventing a person either
Delegates non potest delegare—a delegate cannot oral or written or any other thing which the court bylaw is
further delegate. permitted to take into consideration for making clear or
ascertaining the truth of the fact or point in issue.
Demission Regis vel coronoe—transfer of property.
Ex aequo et bono—in justice and good faith.
Deo volente—if nothing prevents.
Ex contractu—arising out of contract.
Doctrine of harmonious construction—a law is so
interpreted to give effect to all its parts and the Ex curia—out of court.
presumption is taken for various provisions of statutes. Ex debito justitive—prayer is grantable.
Doctrine of pith and substance—true subject Ex debito—arising out of wrong.
matter of legislation. Ex Dolo Malo Non Actio—right of action cannot

wwDoli capax—capable of crime.


Doli incapax—incapable of crime.
Domicile—a place where a person has his permanent
arise out of fraud.
Ex done—as a gift.
Executor—a representative appointed in a will to

w
home with intention to return, or a place where a person
is ordinarily resident.

.Ea
Domus sua cuique est tutissinum refugium—to
everyone his own house in the safest refuge.
execute after the testator’s death.
Ex gratia—as an act of grace or favour.
Ex nudo pacto—non oiur action. An action dose not
arise form a bare promise.
Ex nudo pacto Non Oritur Actio—no cause of

syE
Donation mortis causa—a gift personal property
made in contemplation of death.
Donee—one to whom a gift is made.
action arise form a bare promise.
Ex officio—by virtue of pervious office.
Ex parte—an order granted after hearing one party

ngi
Donor—one who makes a gift.
Double jeopardy—a second prosecution after a first only.
trial for the same offence. Ex post facto—by a subsequent act.
Droit administrative—it is an ordinance or process
where la court are deprived of their jurisdiction in
administrative matters. nee Ex turpi causa non oritur catio—no action arises
form a base cause.
Expression Unius Est Exclusio Alierius—express
Durante absentia—during absence.
Durante vita—during life.
rin
mention of one thing implies the exclusion of another.
Expression unius personae vel rei, est exclusion—
special or express mention of one thing or person implies

g.n
Ejusdem generic—of the same kind or nature; the
rule that where particular words are followed by general exclusion of another.
words, the general words are limited to the same kind as Extra vires—beyond powers.
the particular words.
Emeritus—retired after long service.
Endorsement—a writing on the back of a document
and includes endorsement in negotiable instrument and
also writing in evidence of payment of any amount or
all the marks of the original.
e
Facsimile—make it like. An exacta copy preserving

Factum probanda—facts which are required to be


proved.
Fait accompli—a thing already done.
t
portion of amount due on a document. Faux pas—tactless mistake.
Eminent domain—power of the state to acquire Feme sole—an married women.
private for public use. Ferae naturae—of a wild or ferocious nature.
Fiat—a command.
En bloc—all at the same time
Fiduciary—relationship based on good faith or trust.
En masse—in a body.
FIR—first information report of grievance which
En route—on the way. given to police.
End of justice—objective of justice. Flagrante delicto—in the commission of the
Eo nomine—by that very name. offence.
Equali Jure Melior Conditio Possidentis—if right Force majeure—irresistible compulsion.
is equal, the claim of party is actual possession shall Freehold—the absolute ownership of land.
prevail. Functus offcio—having discharged his duty. Thus
Easaped convict—a convict who was run away once an arbitrator has given his award, he is functus
form lawful custody. offcio, and cannot revoke the award and re-try the case.
Escheat—the lapsing of property to the sovereign or Furiosis nulla voluntas ets—Having discharged his
state on the death of the owner intestate and without heir. duty.

628 | CAT Complete Course

Visit : www.EasyEngineering.net
Visit : www.EasyEngineering.net

Generalia specialibus non derogant—general Judgment—a decision of the court with legal
things do not derogate form special things. reasoning for the same.
Generalibus specialia derogant—special things Judicial review—the process of review of executive
derogate form general things. action by court of law.
Gestis pro haerede—behaviour as heir. Jurisdiction—the extent or territorial area over
Gratis dictum—mrer assertion. which authority of court runs.
Gratuitous—without valuable or legal consideration. Jurisprudence—the philosophy of law, which
Habeas corpus—a prerogative writ to a person who relates to fundamental questions, criticisms, etc.
detains another in custody and with and which commands Jus—law.
him to produce or have the body of that person before Juvenile delinquency—an offence committed by a
him. person who is under 16 years of age.
Hiba—agift in mohammaden law accompanied by Kin—bool relatives.
delivery and acceptance. Lex—statute.
Holograph—a document written in the maker`s own Lexfori—the law of the forum or court in which a
handwritten. case is tried. More particularly the law relating to
Hypothecation—a pledge in which the pledges procedure or the formalities in force [adjective law] in a
retains the possession of the thing pledged. given place.

ww
Ibid—in same place.
Impasse—an insoluble difficulty.
In curia—in open court.
Lex loci—the law of a place.
Lex mercatoria—the law merchant.
Lis pendens or lite pendente—a pending suit,

w
In fra—below.
In futuro—in the future.
Injuria—a legal wrong.
.Ea
In lieu of—in place of.
action, petition or matter, particularly one relating to land.
Litera legis—letter of the law.
LL.B.—[legume baccalaureus] bachelor of law.
Locus standi—right to be heard in court.

syE
In limine—on the threshold. Mala fide—bad faith, bad intention.
In loco parentis—in the place of a parent. Malfeasance—the doing of an unawtulact e.g. a
In memoriam—in memory of. trespass.
Mandamus—[we command] a prerogative writ or

ngi
In pais—in the country.
In pari delicto—where both parties are equally in command issued by a higher court to a lower court.
fault. Mansuetae naturae—harmless or tame by nature.
In pari material—where two enactments have a
common purpose.
In personam—an act, proceeding or right done or nee Mc Naghten rule—classic rule which determine
whether a person is legally insane.
Mens rea—guilty mind, an evil intention, or a
directed against or with reference to a specific person, as
opposed to in rem.
rin
knowledge of the wrognfulness of an act.
Mesne—middle, intervening or intermediate.
Modus operandi—mode of operating.
In re—in the matter of.
In rem—an act, proceeding or right available against
the world at large, as opposed to in personam. g.n
Mortgage—a loan of money taken after giving
security of immovable property.
In situ—in its original situation.
In toto—totally, wholly.
Infra—below.
Ignoratia juris non excusat—ignorance of law is no
made.
e
Mutatis mutandis—whit necessary changes being

Nisi—ineffective unless person affected fail to show


cause. t
Non assumpsit—he did not promise.
excuse.
Non compos mentis—not of sound mind.
Injuria sine damnum—legal wrong without a
damage. Non-cognizable offence—this is an offence where
the police has no legal authority to arrest an offender
Insolvent—a person whose assets are insufficient to without a warrant form court.
pay his debts.
Nocumentum—an annoying, unpleasant or
International law—the which regulates relations obnoxious things or pratice.
between countries.
Non obstante—nothwithstanding.
Inter alia—among other things.
Noscitur a sociis—a word is known by its associates,
Inter se—amongst themselves. i.e., the meaning of a word can be gathered form the
Inter vivos—during life, between living persons. context.
Intra vires—within power [as opposed to ‘ultra Nota bene (NB)—take notice.
vires’ which is beyond power]. Nudum pactum—a nude contract i.e. unenforceable.
Intestate—dying with leaving a will. Obiter dictum—[a saying by the way] an incidential
Ipso facto—by reason of the fact. and collateral opinion uttered a by a judge while
Issue—matter or fact that raises the cause of action. delivering a judgment and which is not binding.

CAT Complete Course | 629

Visit : www.EasyEngineering.net
Visit : www.EasyEngineering.net

Obligee—one to whom a bond is made. Remand—the committal of an accused to prison.


Obligor—one who binds himself by bond. Res communes—common things; things common to
Ombudsman—a parliamentary commission or a al by the law of nature.
grievance man of administration. Res extra commercium—things thrown out of
Onus—burden of proof. commerce.
Op. cit—the book previously cited. Res gestae—all facts so connected with a fact in
Pacta sunt servanda—states are found to fulfill in good issue as to introduce it, explain its nature, or form in
faith the obligations assumed by them under agreements. connection with it one continuous transaction.
Par excellence—eminently. Res ipsa loquitur—the thing speaks for itself.
Parens Patrice—protector of rights. Res judicata—a case or suit already decided.
Pari material—on the same subject. Res sub judice—a matter under judicial considera-
Pari passu—equally, without preference. tion.
Passim—in various places. Scienter—knowledge; an allegation in a pleading
Patent—exclusive privilege granted by the sovereign that the thing has been done knowingly.
authority to an inventor with respect of his invention. Scienti non fit injuria—an injury is not done to one
Per—as stated by. who knows.
Per annum—by the year. Sic utere tuo ut alienum non laedas—live and let

wwPer capital—individually.
Per curiam—by the court.
Per mensem—by the month.
live and hbe reasonable as to your acts in regard to your
neighbours.
Spes successionts—a mere hope of succeeding to

w
property.
Per se—by itself. Stare decisis—adherence to earlier precedents as

.Ea
Penal—relating to punishment.
Pendente lite—during the process of litigation.
Perjury—an offence of giving a false statement
authoritative and binding.
Status quo—the state in which thing are, or were.
Status quo ante—the state in which thing were

syE
before the court which a person knows to be false.
Prima facie—on the face of it.
Pro bono publico—for the good of the public.
Sub silentio—under silence.
Suit—a process instituted in a court of justice for
protection recovery or reinstatement of a right.
Pro et contra—for that exent.
Pro rata—in proportion.
Pro tanto—to that exent. ngi Summons—an order by which a person is called to
appear before a court, judicial officer etc.

nee
Supra—above.
Pro tempore—for the time being. Testimony—the statement made by a witness under
Procedure—mode or form of conducting judicial oath.
proceedings.
Proviso—a condition or a clause.
Puberty—earliest age of being capable of bearing rin
Tort—wrong conduct.
Ubi jus ibi remedium—where there is a right, there
is a remedy.
child.
Qua—with respect to.
g.n
Ut lite pendente nihil innovateur—nothing new
should be introduced during the pndency of a suit.

e
Quasi—as if, as it were analogous to. Ut res margis valeat quam pereat—it is better for a
Qui facit per alium facit per se—he who acts
through another is deemed to act in person.
Qui prior est tempore potior est jure—he who is
first in time has the strongest claim.
Qui sentit commodum debet et onus—he who
meaning to them.
Void ab intio—not valid form the very beginning.
t
thing to have effect than to be made void, i.e., the words
of a statute must be constructed soas to give a sensible

Void—not valid.
accepts the benefit of a transaction must also accept the Vigilantibus non dormientibus leges subveniunt—
burden of the same. the law aids the diligent and not the indolent.
Quia timet—because he fears.
Volenti non fit injuria—where there is consent,
Quorum—minimum number of person necessary for there is no injury.
conduct of proceedings.
Volkogeist—general awareness of the people.
Quo warranto—[by what authority] a prerpgative
writ requiring a person to show by what authority he Vs—versus or aganst or prefix symbol of opposite
exercise a public office. party. Also written as ,v,. for example, Maneka Gandhi v.
Raison d’etre—reason for the existence of a thing. Union of India.
Ratio legis—according to spirit of law. Wager—a bet.
Ratio decidendi—the reason or groumd of a judicial Writ—a judicial process of written command or
decision. It is the ratio deciendi of a case which make the order by court, by which any one is summoned or
decision a precedent for the future. directed; a legal instrument to enforce obedience to the
Re—in the matter of. order for restraining to do some act.
●●●

630 | CAT Complete Course

Visit : www.EasyEngineering.net
Visit : www.EasyEngineering.net

6 Drills
Direction—In the following DRILLS you are asked 11. avowal
to select the definition closest in meaning. The correct (A) sacred oath (B) open declaration
answer is not necessarily, an exact equivalent or even a
(C) harsh sound (D) sterndenial
very good definition. But it is the one choice that comes
closest in meaning to the word . 12. berate
Drill—One (A) deny (B) downgrade
(C) scold (D) judge
1. abash

ww
(A) squash
(C) amaze
(B) embarrass
(D) refuse
13. bicameral
(A) meeting twice a year
(B) having two legislative branches

w
2. abate
(A) aid
(C) lessen
.Ea
(B) remove
(D) howl
(C) having twin lenses
(D) published every two years',
14. blatant
3. abominable
(A) unfortunate
(C) cheap
syE
(B) loathsome
(D) stormy
(A) tardy
(C) depressed
(B) futile
(D) noisy

4. acclaim
(A) demand (B) applaud ngi 15. capitulate
(A) summarize (B) execute

(C) surpass
5. addicted
(D) elect
nee (C) withdraw
16. Careen
(D) surrender

(A) strongly disposed to


(B) mad
(A) secure
(C) decay
rin (B) sway
(D) fondle

(C) increased
(D) sentenced
17. cauterize
(A) sear g.n
(B) warn

6. affront
(A) insult
(C) invasion
(B) projection
(D) success
(C) cut away
18. cherubic
(A) mischievous
e
(D) bind

(B) expensive t
(C) rustic (D) angelic
7. altercation
19. compliance
(A) drastic change (B) angry dispute
(A) flexibility (B) spite
(C) noisy dialogue (D) loud explosion
(C) obedience (D) weakness
8. anomaly
20. compunction
(A) abnormality (B) ignorance
(A) remorse (B) conscience
(C) accident (D) rarity
(C) piercing blow (D) satisfaction
9. arboreal
(A) holiday (B) bower Answers
(C) treelike (D) shady 1. (B) 2. (C) 3. (B) 4. (B) 5. (A) 6. (A)
10. askew 7. (B) 8. (A) 9. (C) 10. (A) 11. (B) 12. (C)
(A) turned to one side (B) direct 13. (B) 14. (D) 15. (D) 16. (B) 17. (A) 18. (D)
(C) doubtful (D) wide open 19. (C) 20. (A)

CAT Complete Course | 631

Visit : www.EasyEngineering.net
Visit : www.EasyEngineering.net

Drill—Two 34. diurnal


(A) news account (B) solar
21. consternation
(C) daily (D) everlasting
(A) group of stars (B) humble service
35. dolorous
(C) large display (D) great amazement
(A) sorrowful (B) financial
22. corrosive
(C) sacred (D) parsimonious
(A) polishing (B) acid-forming
36. dowdy
(C) hiding (D) eating away
(A) corpulent (B) rakish
23. covert (C) elegant (D) unstylish
(A) patent (B) secret 37. dulcet
(C) ditch (D) greedy (A) melodious (B) zither
24. covetous (C) pastry (D) twofold
(A) sheltered (B) hidden 38. echelon

ww
(C) grasping
25. cumbersome
(D) thrifty (A) level of command (B) squadron leader
(C) summit (D) battleground

w
(A) heavy
(C) unwieldy
26. debility
(B) sorrowful

.Ea
(D) laborious
39. edify
(A) amuse
(C) consume
(B) satisfy
(D) instruct
(A) debit
(C) pain
syE
(B) instability
(D) weakness
40. engender
(A) maneuver (B) cause

ngi
27. decor (C) fertilize (D) incite
(A) dramatic presentation
(B) showpiece Answers
(C) ornamental setting
(D) rich furniture nee
21.
27.
(D) 22. (D)
(C) 28. (A)
23. (B)
29. (D)
24. (C)
30. (C)
25. (C) 26.(D)
31. (C) 32. (B)
28. derisive 33.
39.
(D) 34. (C)
(D) 40. (B) rin
35. (A) 36. (D) 37. (A) 38. (A)

g.n
(A) mocking (B) copied
(C) limited (D) borrowed Drill—Three

e
29. derogatory 41. epithet
(A) questionable
(C) humble
30. devious
(B) inquisitive
(D) depreciating
(A) inscription
(C) descriptive term
42. expedient
(B) shoulder piece
(D) honorary award t
(A) multitudinous (B) guessing (A) advantageous (B) free
(C) circuitous (D) premature (C) fatigued (D) rapid

31. dilatory 43. expiate


(A) expanded (B) casual (A) expire (B) sanctify
(C) slow (D) amateurish (C) demolish (D) atone

32. discursive 44. exude


(A) profane (B) rambling (A) evaporate (B) overflow
(C) detailed (D) extraneous (C) wither away (D) ooze out

33. disparage 45. facet


(A) separate (B) discourage (A) gem (B) aspect
(C) compare (D) belittle (C) spout (D) trait pertaining to a

632 | CAT Complete Course

Visit : www.EasyEngineering.net
Visit : www.EasyEngineering.net

46. filial 60. insipid


(A) parent (B) son (A) tasteless (B) animated
(C) duty (D) wise man (C) interminable (D) unplanned
47. fillip Answers
(A) beverage (B) acrobatic trick
41. (C) 42. (A) 43. (D) 44. (D) 45. (B) 46. (B)
(C) large dose (D) stimulus
47. (D) 48. (A) 49. (C) 50. (D) 51. (D) 52. (B)
48. flippancy 53. (D) 54. (D) 55. (A) 56. (B) 57. (A) 58. (C)
(A) levity (B) dexterity 59. (B) 60. (A)
(C) heaviness (D) clumsiness
Drill—Four
49. germane
61. interloper
(A) bacterial (B) Teutonic
(A) acrobat (B) intruder
(C) relevant (D) microscopic
(C) slanderer (D) malingerer
50. gratuitous

ww
(A) thankful
(C) satisfactory
(B) reproachful
(D) uncal1ed for
62. jocose
(A) trite
(C) useless
(B) playful
(D) illusory

w
51. guise
(A) deceit
(B) malice
.Ea
63. malign
(A) disapprove
(C) curse
(B) mistreat
(D) slander ..
(C) protection

syE
(D) appearance composed of
52. heterogeneous
64. manifesto
(A) cargo list (B) secret treaty

(A) similar parts


(C) smooth surfaces
(B) unlike elements
(D) complex problems ngi (C) revolutionary plot (D) public declaration
65. maudlin

53. idiosyncrasy
(A) personality (B) lack of intelligence nee (A) overwrought
(C) exceedingly sad
(B) weakly sentimental
(D) dispirited

(C) absolute rule (D) distinctive characteristic


66. morose
(A) quick-tempered
rin (B) miserly

g.n
54. impinge
(C) illhumored (D) despondent
(A) paint (B) constrict
67. mutation
(C) steal (D) encroach
55. incisive
(A) penetrating
(C) compendious
(B) short
(D) assured
(A) silence
(C) display
68. obviate
e
(B) severance
(D) variation to make:
t
(A) unnecessary (B) clear
56. incongruous (C) sure (D) difficult
(A) unofficial (B) incompatible 69. ostentatious
(C) poorly timed (D) uneven (A) modest (B) flagrant
57. incumbent (C) showy (D) diligent
(A) obligatory (B) dutiful 70. perfunctory
(C) weak (D) slanting (A) lazy (B) official
58. ineptitude (C) mechanical (D) impromptu
(A) dullness (B) vacillation 71. plaudit
(C) awkwardness (D) inexperience (A) expression of approval
59. insinuate (B) consent
(A) spy upon (B) suggest slyly (C) detonation
(C) set free (D) cause injury (D) pleasure,

CAT Complete Course | 633

Visit : www.EasyEngineering.net
Visit : www.EasyEngineering.net

72. prevaricate 84. ruminate


(A) authenticate (B) delay (A) slander (B) digest
(C) lie (D) anticipate (C) meditate (D) remove
73. pristine 85. salacious
(A) meritorious (B) original (A) briny (B) purchasable
(C) expensive (D) traditional (C) obscene (D) flavored
74. privation 86. savant
(A) seclusion (B) sloop (A) cleansing agent (B) learned person
(C) security (D) hardship (C) young student (D) French courtier
75. proton 87. scrutinize
(A) tribal leader (B) meat substitute (A) erase completely (B) turn aside
(C) food element (D) positive particle (C) examine closely (D) read aloud
76. protrude 88. silo

ww
(A) stick out (B) insult
(C) act discourteously (D) emigrate
(A)
(B)
sandy surface
water tower

w
77. raffish
(A) made of straw
(C) disreputable .Ea
(B) ludicrous
(D) due to chance
(C)
(D)
89. subsidy
structure for storage
musical notes

78. rampant
(A) forbidding syE
(B) lion like
(A) replacement
(C) public funds
90. torpid
(B) financial aid
(D) depth charge

(C) protective
79. reiterate
(D) raging unchecked
ngi (A) stormy
(C) warm
(B) hibernating
(D) inactive
(A) stutter
(C) rewrite
(B) repeat
(D) reassess nee
91. travesty
(A) garment (B) long journey
80. replica
(A) mythical creature (B) answer
(C) parody
rin (D) deterioration

g.n
92. tussock
(C) copy (D) public building (A) soft cushion (B) low hammock
(C) bunch of grass (D) small hill

61.
67.
(B)
(D)
62. (B)
68. (A)
Answers
63. (D)
69. (B)
64. (D)
70. (C)
65. (B) 66. (C)
71. (A) 72. (C)
93. tycoon
(A) labor leader
(C) mystic prophet
e
(B) autocratic ruler
(D) industrial magnate'
t
73. (B) 74. (D) 75. (D) 76. (A) 77. (C) 78. (D)
79. (B) 80. (C) 94. upbraid
(A) plait (B) reproach
Drill—Five
(C) elevate (D) foster
81. retrospect
95. vapid
(A) brief summary (B) survey of the past
(A) spiritless (B) foggy
(C) close examination (D) full payment
(C) accelerated (D) shapeless
82. rhapsodic 96. venerable
(A) ecstatic (B) bombastic (A) antique (B) retired
(C) tightly knit (D) fervent (C) inimitable (D) worthy of respect
83. roster 97. vernacular
(A) nesting place (B) professional team (A) native speech (B) slang
(C) speaker's platform (D) list of persons (C) local custom (D) uneducated group

634 | CAT Complete Course

Visit : www.EasyEngineering.net
Visit : www.EasyEngineering.net

98. vituperation 8. A phlegmatic person—


(A) wordy abuse (B) poisonous liquid (A) sheds tears at an emotional play
(C) bombast (D) violent action (B) becomes hysterical in a crisis
99. winnow (C) does not become easily emotional
9. Erudite men are most interested in—
(A) blow (B) fish
(A) scholarly books (B) light fiction
(C) separate (D) minimize
(C) the comics
100. wry
10. People who are complacent about their jobs will—
(A) sad (B) smiling
(A) take it easy
(C) U1ldeserved (D) twisted (B) worry about their future
Answers (C) keep an eye on the help-wanted ads
80. (B) 82. (A) 83. (D) 84. (C) 85. (C) 86. (B) 11. A punctilious person is a stickler for—
87. (C) 88. (C) 89. (B) 90. (D) 91. (C) 92. (C) (A) originality (B) courage

ww
93. (D) 94. (B) 95. (A) 96. (D) 97. (A) 98. (A) (C) proper etiquette
99. (C) 100. (D) 12. To be indefatigable, one usually needs a great amount
of—
Drill—Six

w .Ea
1. Loquacity is an inordinate amount of—
(A) singing (B) attention to details
(A) money
(C) education
13. Vapid people are—
(B) energy

syE
(C) talking (A) boring (B) successful
2. Gullible people fall easy prey to— (C) quarrelsome
(A) doctors (B) used-car salesmen 14. Iconoclasts are opposed to—
(C) teachers
3. Suave men are experts at— ngi (A) change
(C) reform
(B) tradition

(A) home repair (B) surfing


(C) getting along with women nee
15. A misanthrope dislikes—
(A) people (B) good food

4. Pomposity probably comes from—


(C) literature

rin
16. Men are most likely to be puerile when—

g.n
(A) fear (B) obesity (A) they don't get their own way
(C) vanity (B) they are reading
5. Most likely to be esthetic is an—
(A) electrician
(C) artist
(B) aviator

6. Taciturnity would likely be found in—


(C) they are eating
17. Most ascetics prefer to—
(A) drink excessively
(B) eat sparingly
e t
(A) salesmen (B) public speakers (C) participate in orgies
(C) hermits Answers
7. Opinionated assertions may likely lead to— 1. (C) 2. (B) 3. (C) 4. (C) 5. (C) 6. (C)
(A) marriage (B) arguments 7. (B) 8. (C) 9. (A) 10. (A) 11. (C) 12. (B)
(C) truth 13. (A) 14. (B) 15. (A) 16. (A) 17. (B)
●●●

CAT Complete Course | 635

Visit : www.EasyEngineering.net
Visit : www.EasyEngineering.net

7 Test Papers
12. Colgate has also set an ambitious aim of an eight per
Test Paper 1 cent value share of the tooth paste market by the end
1. The manager tried hard to his men to return to work of the first year—
before declaring a lockout— (A) keeping (B) distributing
(A) motivate (B) persuade (C) cornering (D) soliciting
(C) encourage (D) permit 13. Marie Curie was excited when she knew that she
2. There was so much material in the speech that it was was on the …………, of a new discovery.
difficult to know what the peaker wanted to stay— (A) outskirts (B) frontier

ww
(A) banal
(C) superficial
(B) extraneous
(D) variegated
3. Our flight wasfrom Jaipur to Agra air port—
(C) threshold (D) gateway
14. Many women in developing countries experience a

w
(A) deflected
(C) diverted
.Ea
(B) shifted
(D) reverted
4. His handling resulted in all that destruction and
cycle of poor health that ………… before they are
born and persists through adulthood passing from
generation to generation.
(A) derives (B) establishes

syE
damage— (C) begins (D) originates
(A) inept (B) skilful 15. I have for one month's leave—
(C) sophisticated (D) uncouth
(A) demanded (B) requested
5. Once I forgot the piece of paper on which the Dame
of the hotel was written, I was as ………… as lost.
(A) much (B) sure ngi (C) wanted (D) asked
16. A man remains narrow-minded, self complacent and
(C) good (D) bad
6. Infant mortality rate in China has from 200 per nee ignorant unless he visits other people and from
them—
(A) hears (B) earns
thousand to 14 per thousand—
(A) retarded (B) declined (C) learns
rin
(D) borrows

g.n
(C) contracted (D) minimised 17. His companions prevailed upon him not to …………
to violence.
7. Time once lost cannot be …………
(A) refer (B) resort
(A) gained (B) recalled
(C) renumerated (D) recovered
8. He is very on meeting foreigner and befriending
them—
(C) prone
(E) pertain e
(D) provoke

t
18. No country can to practice a constant, rigid foreign
(A) anxious (B) fond police in view of the world power dynamics—
(C) insistent (D) keen (A) obligate (B) anticipate
9. Laser is a (an) of Light Amplification by stimulated (C) afford (D) envisage
Emission of Radiation. (E) visualise
(A) homonym (B) acronym 19. If the President does not grant in his case, he will be
(C) malapropos (D) collocation hanged shortly—
10. The exhibition was a curious of the ok and the new— (A) parole (B) release
(A) melange (B) fervour (C) freedom (D) gratitude
(C) mixture (D) blend (E) clemency
11. The Hubble Space Telescope will search for planets 20. The cancer patients are into their mode of life for
around other stars, a key to the ………… for extra research purposes—
terrestrial life. (A) diagnosed (B) checked
(A) discovery (B) quest (C) probed (D) examined
(C) perception (D) enquiry (E) investigated

636 | CAT Complete Course

Visit : www.EasyEngineering.net
Visit : www.EasyEngineering.net

Answers 10. The current ……… in global negotiations over the ban
on unclear test does not augur well for the future—
1. (B) 2. (C) 3. (C) 4. (A) 5 (A) 6. (B)
(A) moratorium (B) controversy
7. (B) 8. (D) 9. (B) 10. (A) 11. (B) 12. (C) (C) insight (D) stalemate
13. (C) 14. (D) 15 (D) 16. (C) 17. (B) 18. (C) (E) rapprochement
19. (C) 20. (E) 11. Improvement in efficiency and productivity has to be
the key ……… of policy in respect of both the public
Test Paper 2 and the private sectors in the Five Years Plans—
1. I request you to your crime— (A) ingredient (B) purpose
(A) apologise (B) agree (C) role (D) platform
(E) criterion
(C) confess (D) pardon
(E) submit 12. The skill and ease with which he repaired the
machine proved that he is a/an ………… mechanic.
2. The payment was delayed this time because some of (A) able (B) handy
the supporting documents for the claim were (C) nimble (D) maladroit
missing— (E) competent
(A) unduly (B) unjustifiably 13. The chemical warfare capability of Iraq is a ………

ww
(C) ul1foreseenly
(E) unquestionably
(D) undeservedly

3. The prisoner was released on for good behaviour—


to aggression even by the Super powers.
(A) pretext
(C) constraint
(B) deterrent
(D) detriment

w
(A) probation
(C) parole
(E) guarantee
(B) bail

.Ea
(D) grounds

4. Although religion does not inhibit acquisition of


(E) precaution
14. Though the issue of bonus provision was not on the
agenda of the meeting, the Chairman was sure that
this would up—

syE
wealth, the tenor of its teaching is to induce an
attitude ofto worldly things—
(A) indifference (B) hostility
(A) bring
(C) trickle
(E) heat
(B) shoot
(D) crop

(C) affinity
(E) attachment
(D) immunity
ngi 15. They have decided to meet the Prime Minister in
order to have their heard—
(A) agony (B) apathy
5. ………… eyewitnesses, the news reporter gave a
graphic description of how the fire broke.
(A) Reporting (B) Observing nee (C) woes
(E) sufferings
(D) sorrow

16. The foundation of all civilizations and societies is


(C) Seeing
(E) Examining
(D) Quoting
rin
the ability of humans to with each other—
(A) bear (B) dispens1
6. The library expects you to return each and every
book that you have …………….
(C) unite
(E) exchange
g.n (D) collaborate

17. The childrencrackers to celebrate the victory of their


(A) demanded
(C) lent
(E) given
(B) taken
(D) handed

7. …………… between labour and management is


team—
(A) burst
(C) shot
(E) released
e(B) fired
(D) broke
t
inevitable hi any industrial society. 18. The ……… council has passed against the magazine
(A) Controversy (B) Friction for their irresponsibility in reporting are justified—
(C) Association (D) Competition (A) laws (B) ban
(E) Coordination (C) penalty (D) codes
(E) strictures
8. This article tries to ………… us with problems of
poor nations so that we help them more effectively. 19. After a recent mild paralytic attack, his movements
(A) allow (B) enable are …… restricted, otherwise he is still very active—
(A) entirely (B) nowhere
(C) convince (D) project
(C) not (D) slightly
(E) acquaint
(E) frequently
9. One should develop a ……… habit of going through
a newspaper, some selected magazines and general 20. His of the topic was so good that students had few
literature. doubts to raise at the end.
(A) continuous (B) constant (A) handling (B) clarity
(C) regular (D) persistent (C) exposure (D) exposition
(E) recurring (E) picturisation

CAT Complete Course | 637

Visit : www.EasyEngineering.net
Visit : www.EasyEngineering.net

Answers 10. The current …… in global negotiations over the ban


1. (C) 2. (A) 3. (C) 4. (A) 5. (D) 6. (B) on unclear test does not augur well for the future.
7. (B) 8. (E) 9. (C) 10. (D) 11. (C) 12. (E) (A) moratorium (B) controversy
(C) insight (D) stalemate
13. (B) 14. (D) 15 (C) 16. (D) 17. (A) 18. (E)
(E) rapprochement
19. (D) 20. (D)
11. Improvement in efficiency and productivity has to be
Test Paper 3 the key ……… of policy in respect of both the public
1. I request you to your crime— and the private sectors in the Five Years Plans.
(A) ingredient (B) purpose
(A) apologise (B) agree
(C) role (D) platform
(C) confess (D) pardon (E) criterion
(E) submit
12. The skill and ease with which he repaired the
2. The payment was delayed this time because some of machine proved that he is a/an ………… mechanic.
the supporting documents for the claim were (A) able (B) handy
missing— (C) nimble (D) maladroit
(A) unduly (B) unjustifiably (E) competent
(C) ul1foreseenly (D) undeservedly

ww
(E) unquestionably
3. The prisoner was released on for good behaviour—
13. The chemical warfare capability of Iraq is a ………
to aggression even by the Super powers.
(A) pretext (B) deterrent

w
(A) probation (B) bail (C) constraint (D) detriment
(C) parole (D) grounds (E) precaution
(E) guarantee
.Ea
4. Although religion does not inhibit acquisition of

You might also like